Mahesh M. Rathore - Thermal Engineering-Tata McGraw-Hill Education (2010)

You might also like

Download as pdf or txt
Download as pdf or txt
You are on page 1of 1170

Contents i

Thermal Engineering
ii Contents

About the Author

Mahesh M Rathore is working as Professor and Head of the Mechanical Engineering


Department of Shri Neminath Jain Brahmacharyashram College of Engineering
(Jain Gurukul), Neminagar, Chandwad, (Nashik) Maharashtra, India. He is also an
Energy Auditor certified by the Bureau of Energy Efficiency, India, and a Chartered
Engineer. With a teaching experience of more than twenty-five years, he has
developed several teaching aids and learning materials for students and has guided
several credential projects. He has presented several research papers in national and
international conferences and has six books to his credit, besides the present one.
Professor Rathore is a life member of the Indian Society for Technical Education,
India, and the Institution of Engineers (India). He is also a faculty of PRINCE
(Promoters and Researchers In Non-Conventional Energy), Dhule, Maharashtra.
Contents iii

Thermal Engineering

Mahesh M Rathore
Energy Auditor and Chartered Engineer
Professor and Head
Mechanical Engineering Department
S N J B’s KBJ College of Engineering, Chandwad
Nashik, Maharashtra

Tata McGraw Hill Education Private Limited


NEW DELHI

New Delhi New York St Louis San Francisco Auckland Bogotá Caracas
Kuala Lumpur Lisbon London Madrid Mexico City Milan Montreal
San Juan Santiago Singapore Sydney Tokyo Toronto
iv Contents

Tata McGraw-Hill

Published by the Tata McGraw Hill Education Private Limited,


7 West Patel Nagar, New Delhi 110 008.

Copyright © 2010, by Tata McGraw Hill Education Private Limited.

No part of this publication may be reproduced or distributed in any form or by any means, electronic, mechanical,
photocopying, recording, or otherwise or stored in a database or retrieval system without the prior written permission
of the publishers. The program listings (if any) may be entered, stored and executed in a computer system, but they
may not be reproduced for publication.

This edition can be exported from India only by the publishers,


Tata McGraw Hill Education Private Limited.

ISBN (13): 978-0-07-068113-2


ISBN (10): 0-07-068113-9

Managing Director: Ajay Shukla


Head—Higher Education Publishing: Vibha Mahajan
Manager—Sponsoring: SEM & Tech. Ed.: Shalini Jha
Editorial Executive: Surabhi Shukla
Asst Development Editor: Devshree Lohchab
Executive—Editorial Services: Sohini Mukherjee
Sr Production Manager: P L Pandita
General Manager: Marketing—Higher Education: Michael J. Cruz
Dy Marketing Manager: SEM & Tech Ed.: Biju Ganesan
Asst Product Manager: SEM & Tech Ed.: Amit Paranjpe
General Manager—Production: Rajender P Ghansela
Asst General Manager—Production: B L Dogra

Information contained in this work has been obtained by Tata McGraw Hill, from sources believed to be reliable.
However, neither Tata McGraw Hill nor its authors guarantee the accuracy or completeness of any information
published herein, and neither Tata McGraw Hill nor its authors shall be responsible for any errors, omissions, or
damages arising out of use of this information. This work is published with the understanding that Tata McGraw
Hill and its authors are supplying information but are not attempting to render engineering or other professional
services. If such services are required, the assistance of an appropriate professional should be sought.

Typeset at Tej Composers, WZ-391, Madipur, New Delhi 110063, and printed at S P Printers, 30-A, Patpar Ganj
Village, Delhi - 110 091.
Cover Printer: S P Printers
RACYCRXZDQXQC
The McGraw-Hill Companies
Contents v

Dedicated to my Beloved Parents


vi Contents
Contents vii
Contents vii

Contents
Preface xxiii
Nomenclature xxviii
Visual Walkthrough xxxii
1. Basic Concepts 1
1.1 Thermodynamics 1
1.2 Thermodynamic System 1
1.3 Macroscopic V/S Microscopic Views 7
1.4 Working Fluid 8
1.5 Continuum 8
1.6 Thermodynamic Properties of a System 8
1.7 State, Path, Process and Cycle 10
1.8 Point Function and Path Function 12
1.9 Quasi-Static Process 12
1.10 Equilibrium 13
1.11 Dimensions and Units 14
1.12 Pressure 15
1.13 Temperature and Zeroth Law of Thermodynamics 16
1.14 Measurement of Temperature 16
1.15 Temperature Scale 18
1.16 The International Practical Temperatures Scale 19
1.17 Entropy 20
Summary 24
Glossary 24
Review Questions 25
Problems 26
Objective Questions 26
2. Energy and Work Transfer 28
2.1 Energy 28
2.2 Sources of Energy 28
2.3 Classification of Energy Sources 29
2.4 Forms of Energy 30
viii Contents

2.5 Enthalpy 32
2.6 Heat 32
2.7 Specific Heat 33
2.8 Work 35
2.9 Forms of Work Transfer 37
2.10 First Law of Thermodynamics 43
2.11 First Law of Thermodynamics for a Cyclic Process—Joule’s Experiment 45
2.12 Energy—A Property of the System 46
2.13 Perpetual Motion Machine of the First Kind 46
Summary 50
Glossary 51
Review Questions 51
Problems 51
Objective Questions 54
3. Working Substances 55
3.1 Pure Substance 55
3.2 Phases of a Pure Substance 55
3.3 Phase-Change Phenomenon of a Pure Substance 56
3.4 Terminology of Pure Substances 58
3.5 Property Diagrams 59
3.6 The p–v–t Surface 62
3.7 Critical Point and Triple Point 62
3.8 T–s and h–s Diagrams 63
3.9 Enthalpy Changes during Formation of Steam 65
3.10 Wet Steam 66
3.11 Superheated Steam 67
3.12 Specific Volume of Steam 67
3.13 Entropy of a Pure Substance 68
3.14 External Work Done during Evaporation 69
3.15 Internal Latent Heat 69
3.16 Internal Energy of Steam 70
3.17 Steam Tables 70
3.18 Points to Consider before Solving the Problems 71
3.19 Advantages and Applications of Use of Steam 71
3.20 Measurement of Dryness Fraction of Steam 77
3.21 Ideal Gas Model 82
3.22 Equation of State 83
3.23 Characteristic Gas Equation 83
3.24 Internal Energy and Enthalpy of an Ideal Gas 87
3.25 Specific Heats of Ideal Gases 88
3.26 Relation Between Specific Heats for an Ideal Gas 89
3.27 Real Gases 90
3.28 Other Equations of State 92
Contents ix

3.29 Analytical Test for Equation of State 94


Summary 98
Glossary 99
Review Questions 99
Problems 100
Objective Questions 101
4. First Law Applied to Non-Flow Systems 103
4.1 Ideal-Gas Processes 103
4.2 Vapour Process 131
Summary 139
Glossary 140
Review Questions 140
Problems 141
Objective Questions 144
5. First Law Applied to Flow Processes 145
5.1 Flow Process and Flow Energy 145
5.2 Mass and Energy Analysis of an Open System 146
5.3 Energy Balance in Steady Flow 147
5.4 Some Steady-Flow Devices 148

Ú
5.5 Significance of – vdp 152

5.6 Relation Between Non-Flow Work Ú pd v and Flow Work – Ú vdp 152
5.7 Transient Flow Processes 176
Summary 178
Glossary 179
Review Questions 179
Problems 180
Objective Questions 184
6. Second Law of Thermodynamics 185
6.1 Limitations of the First Law of Thermodynamics 185
6.2 Thermal Reservoir 186
6.3 Heat Engine 187
6.4 Refrigerator 188
6.5 Heat Pump 189
6.6 Statements of the Second Law of Thermodynamics 189
6.7 Perpetual Motion Machine of the Second Kind 190
6.8 Reversible Process: Ideal Process 191
6.9 Irreversible Processes: Actual Processes 191
6.10 Carnot Cycle, or Carnot Engine 192
6.11 Reversed Carnot Cycle 194
6.12 Carnot Theorem 194
x Contents

6.13 Thermodynamic Temperature Scale 196


Summary 217
Glossary 218
Review Questions 219
Problems 219
Objective Questions 222
7. Entropy 223
7.1 Definition 223
7.2 Two Isentropic Lines Cannot Intersect Each Other 223
7.3 Clausius’ Theorem 224
7.4 Clausius, Inequality 225
7.5 Entropy: A Property of the System 227
7.6 Change of Entropy in a Reversible Process 228
7.7 Temperature–Entropy Diagram 228
7.8 The Increase of Entropy Principle 229
7.9 Entropy Transfer 230
7.10 Entropy Generation 230
7.11 Entropy Balance 231
7.12 Physical Concept of Entropy 231
7.13 Tds Relations 232
7.14 Third Law of Thermodynamics 233
Summary 243
Glossary 243
Review Questions 244
Problems 244
Objective Questions 245
8. Availability and Irreversibility 247
8.1 Sources of Energy 247
8.2 Available and Unavailable Energy 248
8.3 Availability of Energy Entering a System 250
8.4 Availability of a Closed System 254
8.5 Availability in a Steady Flow Process 256
8.6 The Second-Law Efficiency 266
Summary 270
Glossary 271
Review Questions 271
Problems 272
Objective Questions 275
9. Thermodynamic Relations 276
9.1 Helmholtz and Gibbs Function: Gibbsian Equations 276
9.2 Principal Exact Differentials 276
Contents xi

9.3 Partial Derivative Relations 277


9.4 Maxwell Relations 277
9.5 Thermodynamic Square 278
9.6 Volumetric Expansivity, Isothermal and Isentropic Compressibility 279
9.7 Internal Energy, Enthalpy, and Entropy 280
9.8 Specific Heats 283
9.9 Joule–Thompson Coefficient—The Porous Plug Experiment 285
9.10 Clausius–Clapeyron Equation 286
Summary 294
Review Questions 295
Problems 295
Objective Questions 296
10. Compressible Fluid Flow 298
10.1 Static Properties 298
10.2 Stagnation Properties 298
10.3 Velocity of Sound: Sonic Velocity 301
10.4 Mach Number 303
10.5 Property Relations for Isentropic Flow Through a Duct 308
10.6 Property Relations at Stagnation Conditions 309
10.7 One -Dimensional Isentropic Flow 312
10.8 Effect of Back Pressure on Mass Flow Rate 315
10.9 Mass Flow Rate Through an Isentropic Nozzle 317
10.10 Shock Wave 320
10.11 Flow Through Actual Nozzles and Diffusers 323
10.12 Effect of Irreversibilities on Nozzle Efficiency 325
Summary 326
Glossary 327
Review Questions 328
Problems 329
Objective Questions 330
11. Gas Power Cycles 332
11.1 Definitions 332
11.2 Air Standard Analysis 334
11.3 Carnot Cycle 334
11.4 Stirling Cycle 335
11.5 Ericsson Cycle 336
11.6 Otto Cycle 337
11.7 Diesel Cycle 338
11.8 Dual Cycle 339
11.9 Comparison of Otto and Diesel Cycles 340
11.10 Lenoir Cycle 361
11.11 Atkinson Cycle 362
xii Contents

11.12 Brayton Cycle 364


Summary 373
Glossary 373
Review Questions 374
Problems 374
Objective Questions 377
12. Vapour Power Cycles 380
12.1 Modelling a Steam Power Plant 380
12.2 Performance Parameters of Vapour Power Cycle 381
12.3 Carnot Vapour Power Cycle 382
12.4 Rankine Cycle 385
12.5 Comparison Between Carnot and Rankine Cycles 395
12.6 Difference Between Carnot and Rankine Cycles 396
12.7 Irreversibilities and Losses in Vapour Power Cycle 398
12.8 Effect of Operating Variables on Rankine Cycle 401
12.9 Reheating of Steam 405
12.10 Super Critical Rankine Cycle 407
12.11 Mean Temperature of Heat Addition 415
12.12 Regenerative Rankine Cycle 415
12.13 Modified Rankine Cycle 433
12.14 Characteristics of the Working Fluid in Vapour Power Cycle 434
12.15 Cogeneration 435
12.16 Binary Vapour Cycle 438
12.17 Combined Gas-Vapour Power Cycle 438
Summary 441
Glossary 442
Review Questions 442
Problems 443
Objective Questions 445
13. Refrigeration 447
13.1 Refrigeration 447
13.2 Refrigerators and Heat Pumps 448
13.3 Refrigeration Terminology 449
13.4 Types of Refrigeration Systems 450
13.5 Gas Refrigeration Systems 450
13.6 Brayton Refrigeration Cycle: Bell Coleman Cycle 452
13.7 Ideal Vapour Compression Refrigeration Cycle 456
13.8 Vapour Absorption Refrigeration Cycle 469
13.9 Comparison of Vapour Absorption System with Vapour Compression System 471
13.10 Steam Jet Refrigeration 472
13.11 Heat Pump 472
Contents xiii

13.12 Refrigerant 473


Summary 476
Glossary 477
Review Questions 477
Problems 478
Objective Questions 479
14. Ideal Gas Mixtures 481
14.1 Mass and Mole Fractions 481
14.2 Dalton’s Law of Partial Pressures 482
14.3 Amagat–Leduce Law of Additive Volumes 482
14.4 Density and Gas Constant of Gas Mixture 483
14.5 Properties of Gas Mixture—Gibbs Theorem 487
14.6 Mixing of Ideal Gases 491
14.7 Mixture of Real Gases 496
14.8 Compressibility Factor 497
Summary 497
Glossary 498
Review Questions 498
Problems 498
Objective Questions 500
15. Psychrometry 502
15.1 Psychrometer 502
15.2 Dry, Moist and Saturated Air 502
15.3 Properties of Moist Air 503
15.4 Partial Pressure of Air and Vapour 505
15.5 Adiabatic Saturation Temperature 508
15.6 Psychrometric Chart 510
15.7 Air-Conditioning Process 512
15.8 Adiabatic Mixing of Two Moist Air Streams 522
15.9 Air Washer 525
Summary 526
Glossary 526
Review Questions 527
Problems 527
Objective Questions 528
16. Fuels and Combustion 530
16.1 Fuels 530
16.2 Characteristic of an Ideal Fuel 530
16.3 Coal 531
16.4 Liquid Fuels 533
16.5 Gaseous Fuel 533
xiv Contents

16.6 Conversion of Volumetric Analysis to Gravimetric Analysis 534


16.7 Conversion of Gravimetric Analysis to Volumetric Analysis 535
16.8 Combustion 535
16.9 Composition of Dry Air 538
16.10 Amount of Air Required for Combustion 539
16.11 Air–Fuel Ratio 540
16.12 Air–Fuel Ratio from Analysis of Flue Gases 540
16.13 Flue Gas Analysis—Orsat Apparatus 553
16.14 Heat Generated by Combustion 557
16.15 Calorific Value, or Heating Value of Fuel 561
16.16 Bomb Calorimeter 563
16.17 Junker’s Gas Calorimeter 566
Summary 568
Glossary 568
Review Questions 569
Problems 569
Objective Questions 572
17. Steam Generators 574
17.1 Indian Boiler Regulation (IBR) 574
17.2 Boiler Systems 575
17.3 Comparison Between Fire Tube and Water Tube Boilers 577
17.4 Fire-Tube Boilers 577
17.5 Water Tube Boilers 583
17.6 Some Industrial Boilers 584
17.7 High-Pressure Boilers 587
Summary 592
Glossary 592
Review Questions 592
Objective Questions 593
18. Boiler Mountings and Accessories 594
18.1 Boiler Mountings 594
18.2 Boiler Accessories 599
Summary 604
Review Questions 605
Objective Questions 605
19. Boiler Draught and Performance 607
19.1 Boiler Draught (Draft) 607
19.2 Natural, or Chimney, Draught 607
19.3 Artificial Draught 616
19.4 Performance Evaluation of Boilers 619
19.5 Energy Balance in a Boiler 630
Contents xv

19.6 Energy-Conservation Opportunities 637


Summary 639
Glossary 639
Review Questions 639
Problems 640
Objective Questions 642
20. Steam Engines 644
20.1 Classification of Steam Engines 644
20.2 Construction of Steam Engine 645
20.3 Working of a Double-Acting Steam Engine 646
20.4 Hypothetical Indicator Diagram of Steam Engine 646
20.5 Actual Indicator Diagram 650
20.6 Cylinder Condensation 651
20.7 Mass of Steam in Cylinder 651
20.8 Missing Quantity 651
20.9 Steam Consumption 652
20.10 Steam Compression in the Cylinder 653
20.11 Governing of Steam Engines 653
20.12 Power Output of Steam Engine 655
20.13 Efficiencies of a Steam Engine 656
20.14 Compound Steam Engines 667
Summary 668
Glossary 669
Review Questions 669
Problems 669
Objective Questions 670
21. Steam Nozzles 673
21.1 Types of Steam Nozzles 673
21.2 Steam Flow Through a Nozzle 674
21.3 Flow Through Actual Nozzles 678
21.4 Supersaturated Expansion of Steam 691
Summary 695
Glossary 696
Review Questions 696
Problems 696
Objective Questions 698
22. Steam Turbines 699
22.1 History of Steam Turbines 699
22.2 Working Principle of a Steam Turbine 700
22.3 Classification of Steam Turbines 700
22.4 The Simple Impulse Turbine 702
xvi Contents

22.5 Optimum Operating Conditions from Blade-Velocity Diagram 705


22.6 Effect of Blade Friction on Velocity Diagram 710
22.7 Condition for Axial Discharge 711
22.8 Compounding of Impulse Turbine 719
22.9 Reaction Turbine (Impulse Reaction Turbine) 728
22.10 Comparison Between Impulse and Reaction Turbines 737
22.11 Losses in Steam Turbines 738
12.12 Governing of Steam Turbine 739
22.13 Special Forms of Turbines 743
Summary 744
Glossary 744
Review Questions 745
Problems 745
Objective Questions 748
23. Steam Condensers 750
23.1 Condenser 750
23.2 Functions of a Condenser 750
23.3 Elements of a Condensing Plant 751
23.4 Types of Condensers 751
23.5 Jet Condenser 751
23.6 Surface Condenser 754
23.7 Estimation of Cooling Water Required 757
23.8 Condenser Efficiency 757
23.9 Analysis of Condenser Operation 760
23.10 Air Extraction 769
23.11 Cooling Towers 772
23.12 Cooling Pond 773
Summary 774
Glossary 775
Review Questions 775
Problems 775
Objective Questions 777
24. Internal Combustion Engines 779
24.1 Classification of IC Engines 779
24.2 Components of Engines 780
24.3 Otto Cycle Engines: Petrol Engines 781
24.4 Diesel Engines 786
24.5 Comparison Between Petrol and Diesel Engines 791
24.6 Comparison Between Two-Stroke and Four-Stroke Engines 791
24.7 Advantages and Disadvantages of Two-Stroke Cycle Engines 792
24.8 Air–Fuel Mixture 792
24.9 Carburation 794
Contents xvii

24.10 Fuel-Injection System 796


24.11 Combustion 800
24.12 Governing of IC Engines 803
24.13 Ignition System 804
24.14 Firing Order 808
24.15 Engine-Cooling System 808
24.16 Engine Lubrication 811
24.17 Lubrication Systems 812
24.18 Performance of Internal Combustion Engines 814
24.19 Efficiencies of IC Engines 816
24.20 Supercharging 832
Summary 833
Glossary 833
Review Questions 834
Problems 835
Objective Questions 836
25. Reciprocating Air Compressor 838
25.1 Uses of Compressed Air 838
25.2 Classification 839
25.3 Reciprocating Compressor Terminology 839
25.4 Compressed Air Systems 840
25.5 Reciprocating Air Compressor 840
25.6 Minimizing Compression Work 846
25.7 Clearance Volume in a Compressor 849
25.8 Actual Indicator Diagram 852
25.9 Volumetric Efficiency 852
25.10 Free Air Delivery (FAD) 854
25.11 Limitations of Single-Stage Compression 864
25.12 Multistage Compression 864
25.13 Cylinder Dimensions of a Multistage Compressor 874
Summary 883
Glossary 885
Review Questions 885
Problems 886
Objective Questions 888
26. Rotary Compressor 890
26.1 Classification of Rotary Compressors 891
26.2 Roots Blower Compressor 891
26.3 Vane-Type Compressor 894
26.4 Lysholm Compressor—A Screw Compressor 897
26.5 Centrifugal Compressor 898
26.6 Axial Compressor 910
xviii Contents

26.7 Difference Between Centrifugal and Axial Flow Compressors 916


Summary 920
Glossary 921
Review Questions 921
Problems 921
Objective Questions 922
27. Gas Turbine Plant 924
27.1 Applications of Gas Turbines 925
27.2 Classification of Gas Turbines 926
27.3 Comparison Between Close-Cycle Gas Turbine and Open-Cycle Gas Turbine 927
27.4 Modelling a Gas Turbine Plant 928
27.5 Deviation of Actual Gas Turbine Cycle from Brayton Cycle 928
27.6 Methods for Improvement of Thermal Efficiency of Gas Turbine Plant 935
Summary 959
Glossary 960
Review Questions 960
Problems 960
Objective Questions 961
28. Jet and Rocket Propulsions 963
28.1 Jet Propulsion 963
28.2 IC Engine-Driven Propulsive System 964
28.3 Ramming Effect Propulsion Systems 964
28.4 Gas Turbine Propulsion Systems 966
28.5 Turbojet Engine 966
28.6 Turboprop Engine 967
28.7 Turbofan 968
28.8 Terminology Used with Turbojet Engine 969
28.9 Analysis of Turbojet Cycle 970
28.10 Thrust Augmentation in Turbojet Engines 972
28.11 Rocket Propulsion 986
28.12 Solid Propellant Rocket 988
28.13 Liquid Propellant Rockets 988
28.14 Hybrid Propellant Rocket 989
28.15 Nuclear Prolellant Rocket 989
28.16 Propellants 990
28.17 Analysis of Rocket Propulsion 993
Summary 996
Glossary 997
Review Questions 997
Problems 998
Objective Questions 999
Contents xix

29. Air-Conditioning 1001


29.1 Applications of Air-Conditioning 1001
29.2 Comfort Air-Conditioning 1002
29.3 Effective Temperature 1003
29.4 Air-Conditioning Systems 1005
29.5 Air-Conditioning Cycle 1005
29.6 Summer Air-Conditioning System 1006
29.7 Winter Air-Conditioning System 1006
29.8 Year-Round Air Conditioning System 1007
29.9 Unitary System 1007
29.10 Central Air-Conditioning System 1009
29.11 Classifications of Central Air-Conditioning System 1010
29.12 Rating of Air-Conditioning 1010
29.13 Cooling and Heating Load Calculations 1010
29.14 Sensible Heat Factor 1012
29.15 Water Coolers 1017
29.16 Ice Plant 1017
29.17 Air Coolers 1018
29.18 Difference Between Air Cooler and Air-Conditioner 1021
Summary 1022
Glossary 1022
Review Questions 1023
Problems 1023
Objective Questions 1024
30. Elements of Heat Transfer 1025
30.1 Modes of Heat Transfer 1025
30.2 Fourier Law of Heat Conduction 1025
30.3 Thermal Conductivity 1026
30.4 Convection Heat Transfer: Newton’s Law of Cooling 1026
30.5 Radiation Heat Transfer: Stefan–Boltzmann Law 1027
30.6 Steady-State Heat Conduction in Solids 1027
30.7 Combined Modes of Heat Transfer 1029
30.8 Overall Heat-Transfer Coefficient 1032
30.9 Log Mean Area 1033
30.10 Principle of Heat Convection 1037
30.11 Convection Boundary Layers 1038
30.12 Physical Significance of the Convection Dimensionless Parameters 1039
30.13 Dimensional Analysis 1040
30.14 Summary of Dimensionless Parameters and their Correlations 1042
30.15 Flow Through Ducts 1043
30.16 Free Convection 1045
30.17 Empirical Relations for Free Convection 1045
xx Contents

30.18 Radiation Heat Transfer 1047


30.19 Theories of Radiation 1047
30.20 Black-Body Radiation 1047
30.21 Surface Absorption, Reflection and Transmission 1048
30.22 Kirchhoff’s Law 1050
30.23 Heat Exchangers 1053
30.24 Heat Exchanger Analysis 1055
Summary 1057
Glossary 1058
Problems 1058
Objective Questions 1060
Appendix A Thermophysical Properties of Matter 1062
A-1 Atomic mass and critical constants
A-2 Properties of selected solids at 25ºC
A-3 Properties of selected liquids at 25ºC
A-4 Thermo physical properties of selected substances at 25ºC and 1atm
A-5 Constants for van der Waals, Redlich-Kwong and Benedict-Webb-Rubin Equations of state
A-6 Variation of Cp with temperature of selected ideal gases
A-7 Ideal gas specific heats for some common gases
A-8 Ideal gas Properties of air
A-9 Ideal gas Properties of selected gases
A-10 One dimensional isentropic compressible flow function for an ideal with constant specific heat
and molecular mass and g = 1.4
A-11 One dimensional normal shock function for an ideal with constant specific heat and molecular
mass and g = 1.4
A-12 Properties of saturated water: Temperature entry
A-13 Properties of saturated water: Pressure entry
A-14 Properties of superheated water vapor.
Appendix B Properties of Refrigerants 1088
B-1 Properties of Saturated refrigerant 22 (Liquid-vapor): Temperature entry
B-2 Properties of Saturated refrigerant 22 (Liquid-vapor): Pressure entry
B-3 Properties of Superheated refrigerant 22 vapor
B-4 Properties of Saturated refrigerant 12 (Liquid-vapor): Temperature entry
B-5 Properties of Superheated refrigerant 12
B-6 Properties of Saturated refrigerant 134a (Liquid-Vapor): Temperature entry
B-7 Properties of Saturated refrigerant 134a (liquid-Vapor): Pressure entry
B-8 Properties of Super heated refrigerant 134a vapor
B-9 Properties of Saturated Ammonia (Liquid-Vapor): Temperature entry
B-10 Properties of Saturated Ammonia (liquid-Vapor): Pressure entry
B-11 Properties of Super heated Ammonia vapor
B-12 Properties of Saturated Propane (Liquid-Vapor): Temperature entry
B-13 Properties of Saturated Propane (liquid-Vapor): Pressure entry
B-14 Properties of Super heated Propane vapor
Contents xxi

B-15 Properties of Saturated Nitrogen (Liquid-Vapor): Temperature entry


B-16 Properties of Super heated Nitrogen vapor
Appendix C Figures 1118
C-1 Temperature entropy diagram for water
C-2 Mollier diagram for water
C-3 Pressure enthalpy diagram for Refrigerant 134a
C-4 Psychrometric chart
References and Suggested Readings 1122
Index 1124
xxii Contents
Contents xxiii

Preface
It gives me immense pleasure to present this book on ‘Thermal Engineering’. This text is intended for
undergraduate students of Mechanical, Automobile and Aeronautical engineering as well as AMIE courses and
competitive examinations. It integrates thermodynamics, applied thermodynamics and thermal engineering
and hence covers the syllabi of almost all subjects pertaining to thermal engineering taught from the first year
to the final year of engineering curriculum.

Aim
During my teaching span of more than two decades, I felt that the subjects based on thermal engineering are
often perceived as difficult by students. I observed that customarily, major problems are faced by students
in understanding the text and illustrations. They need a text written in a simple and interesting way which
exposes the subject systematically along with a variety of illustrative examples supporting the theoretical
concepts.
Through this book I am making an attempt to overcome the problems of students as well as to impart
sound knowledge. The presentation is simple, lucid and easy to understand. The topics are explained right
from the fundamentals with the help of illustrative figures, enabling even a beginner to understand the subject
very easily. Solutions for the problems also are explained with the help of illustrative figures so that the logic
behind them is easily understood.
This book discusses the basic concepts first and then supports the theory with applications and solved
numerical problems. This approach will help the students in developing an analytical mind. An engineer with
an analytical mind and approach would be able to face any problems encountered in the actual engineering
field. Moreover, it is my earnest hope that this book will provide a unique combination of features that will
make it inviting and effective for both faculty and students.

Salient Features
The salient features of the book are the following:
Complete coverage of both courses (a) Engineering Thermodynamics (b) Applied Thermodynamics
Tutorial Approach of problem solving
Solved Examples based on questions from numerous universities all across India as well as competitive
examinations like GATE, IES etc
Diverse and useful pedagogical features like Summary, Glossary, Review Questions, Problems and
Objective Questions
Well-labeled and apt schematic diagrams supporting theoretical and mathematical explanations
xxiv Preface

Organisation
Principally, the book is divided into three parts.
The First Part of the book deals with the subject of thermodynamics, which is a core course taught to the
first-year students of all disciplines in almost all the engineering colleges and universities. It includes the first
eight chapters—chapters 1 to 8.
The Second Part of the book is designed for an applied thermodynamics course. This part includes the
next eight chapters—chapters 9 to 16.
The Third Part provides the foundation learning material on thermal engineering. This part covers steam
engineering, internal combustion engines, air compressors, gas turbines, jet and rocket propulsions, air
conditioning and an introduction to heat transfer it includes remaining chapters—chapters 17 to 30.
Chapter 1 provides an overview of the basic concepts of thermodynamics. Concepts are an essential part
of any science and in the case of thermodynamics, experience has shown that this is an area students find
difficult. Several illustrations and day to day examples are provided in support of definitions and concepts to
help the students have a thorough understanding of the topics.
Energy is a basic requirement for work transfer. Chapter 2 gives a detailed treatment of energy and its
forms, work and heat transfer. Chapter 3 deals with the properties of pure substances. The properties of
common working substances, steam and ideal gases are worked out in this chapter.
The first and second laws of thermodynamics are regarded as pillars of thermodynamics. The first law
speaks of energy and its conservation (quantity), while the second law deals with the quality aspect of
energy. Applications of the first law of thermodynamics to non-flow processes (for closed systems) and flow
processes (open systems or control volume) are explained with support of several examples in Chapters 4
and 5, respectively. The second law of thermodynamics is treated in a comprehensive manner in Chapter 6
and prolonged in Chapter 7 with the concept of entropy. Entropy is an abstract property of the second law
and can be thought of as a measure of disorder in the system responsible for energy degradation taking place
in real processes. Other second-law concepts, availability and irreversibility, are introduced in Chapter 8
along with the development of a procedure for performance evaluation of a system.
Chapter 9 deals with thermodynamic relations. Maxwell’s relations, volumetric expansivity, isothermal
and isentropic compressibility, Joule—Thomson coefficient and Clausius—Clapeyron equations are
explained in this chapter. Chapter 10 deals with compressible fluid flow. It uses the concept of the first and
second laws for a moving fluid. The comprehensive thermodynamic analysis of gas power cycles (Chapter
11), vapour power cycles (Chapter 12), refrigeration cycles (Chapter 13), ideal gas mixtures (Chapter
14) and psychrometry (Chapter 15) are carefully considered. Chapter 16 deals with fuels and combustion.
Various types of fuels and determination of their calorific value are explained in this chapter.
Emphasis is given to steam engineering from Chapter 17 to Chapter 23. Chapter 17 gives an overview
of different types of boilers. Chapter 18 incorporates various important boiler mountings and accessories.
The boiler draught and performance are discussed in Chapter 19. Even though steam engines are obsolete
nowadays, still the concept and preliminary analysis of steam engines are taken up in Chapter 20. The
relevance of compressible fluid flow to steam nozzles are explained in Chapter 21. The concept and analysis
of impulse and reaction steam turbines are incorporated in Chapter 22. Chapter 23 gives an elementary
treatment to steam condensers. The internal combustion engines are discussed in Chapter 24. Chapters
25 and 26 take up the analytical treatment to reciprocating and rotary air compressors. Chapter 27 gives
an elementary treatment to gas turbines. Chapter 28 provides an outlook to jet and rocket propulsions.
Chapter 29 takes up theoretical and analytical treatment to air conditioning and lastly, Chapter 30 provides
an elementary introduction of heat transfer.
Preface xxv

Dependency Chart

Chapter 1

Basic Concepts

Chapter 2
First Law
Energy & Work

Chapter 3, 14 Chapter 4, 5

Working Applications of
Substances First Law

Chapter 6 Chapter 30

Elements of Heat
Second Law
Transfer

Chapter 7, 8 Chapter 9, 15
Thermodynamic
Entropy & Exergy
Relations &
Chapter 11–13 Psychrometry
Chapter 29
Power &
Refrigeration Chapter 10 Air Conditioning
Cycles
Compressible
Chapter 24 Fluid Flow

I.C. Engines

Chapter 16

Fuel &
Combustion
Chapter 25, 26

Air Compressors
Chapter 17-19 Chapter 21

Steam Systems Steam Nozzles

Chapter 27, 28 Chapter 20


Gas Turbines &
Jet Propulsions Steam Engines

Chapter 23 Chapter 22
Steam
Steam Turbines
Condenser
xxvi Preface

Web Supplements
The web supplements for this book can be accessed at http://www.mhhe.com/rathore/te/1e and contains the
following material:
For Instructors

For Students

Acknowledgements
On completion of this version of the book Thermal Engineering, I express my heartiest gratitude to my
students, past and present, whose inquisitive queries and feedback motivated me to write this book. I am
indebted to all authors who shaped my thoughts on the subject and whose work has been freely consulted in
preparation of this text. I also owe an enormous debt of gratitude to my colleagues and students who helped
me directly or indirectly in preparation of this treatise.
I would also like to mention the names of the reviewers whose valuable inputs have gone a long way in
shaping this text.
Akhilesh Gupta Indian Institute of Technology (IIT)
Roorkee, Uttar Pradesh
Pradyumna Ghosh Institute of Technology, Banaras Hindu University (ITBHU)
Varanasi, Uttar Pradesh
M K Das Motilal Nehru National Institute of Technology (MNNIT)
Allahabad, Uttar Pradesh
P Srinivasan Birla Institute of Technolgy and Science (BITS)
Pilani, Rajasthan
S K Soni Punjab Engineering College (PEC), University of Technology,
Chandigarh, Punjab
Sunil Punjabi Government Engineering College
Ujjain, Madhya Pradesh
K P Tyagi Krishna Institute of Engineering and Technology
Ghaziabad, Uttar Pradesh
Sudarshan Singh National Institute of Technology (NIT)
Patna, Bihar
Ranjan Basak Sikkim Manipal Institute of Technology (SMIT)
Rangpo, East Sikkim
Kanchan Chatterjee Dr B C Roy Engineering College
Durgapur, West Bengal
Santanu Banerjee Birbhum Institute of Engineering and Technology
Suri, West Bengal
V Venkat Raj Bharati Vidyapeeth College of Engineering and Technology
Navi Mumbai, Maharashtra
Preface xxvii

D A Warke J T Mahajan College of Engineering,


Jalgaon, Maharashtra
R G Kapadia Shri Sad Vidya Mandal Institute of Technology (SVMIT)
Bharuch, Gujarat
K Senthil kumar School of Mechanical and Building Sciences
VIT University, Vellore, Tamil Nadu
M Ganapathy Dr MGR University
Periyar, Tamil Nadu
P Chitambarnathan Dr Sivanthi Adithanar College of Engineering
Tuticorin, Tamil Nadu
Appukuttan National Institute of Technology, Karnataka (NITK)
Surathkal
B Srinivas Reddy G Pulla Reddy Engineering College
Kurnod, Andhra Pradesh
P I Urgan K L E Society’s College of Engineering and Technology
Belgaum, Karnataka
I take the opportunity to express my sincere thanks to the TMH editorial team: Ms Surabhi Shukla, Ms
Devshree Lohchab, and Ms Sohini Mukherjee who put their consistent and pre-eminent efforts for making
the text as best as it could be. My special thanks to Mr Sagar Divekar, whose repeated persuasion made it
possible to prepare the text for TMH. He has provided required support from time to time. I am pleased to
acknowledge the contribution of Mr P L Pandita and the whole production staff.
I would like to extend my gratitude to administration and Executive Management of SNJB’s K B J
College of Engineering, Chandwad, (Nashik) who extended all the facilities and full cooperation during the
preparation of this manuscript.
Above all, I wish to place on record my earnest gratitude to my parents, caring wife—Mrs Meera; sons—
Dr Ankit and Prateek, for their wholehearted support and patience, which helped me indirectly in completing
the project.
A human creation can never be perfect. Some mistakes might have crept in the text. My efforts in writing
this book will be rewarded if readers send their constructive suggestions and objective criticism with a view
to improve the usefulness of the book. For any suggestion, query or difficulty, you are most welcome to write
to me at mmrathore@gmail.com.

M M Rathore

Publisher’s Note
Tata McGraw Hill Education looks forward to receiving views, comments and suggestions from
readers, all of which may be sent to tmh.cefeedback@gmail.com mentioning the title and author’s
name in the subject line. Piracy related issues may also be reported.
xxviii Contents

Nomenclature
A area,
Ac cross-sectional area
a linear acceleration, specific Helmhotz function, accoustic velocity
b constant
BP brake power
Bsfc brake specific fuel consumption
bwr back work ratio
C specific heat, constant
CV calorific value
Cp specific heat at constant pressure
CT constant temperature coefficient
Cv specific heat at constant volume
c clearance ratio
D dimeter, constant
d bore; diameter
E total energy
e energy per unit mass
F force
FP friction power
G Gibbs function, H-TS
g gravitational acceleration, specific gibbs function, h-Ts,
H enthalpy, chimney height
HCV higher calorific value
h specific enthalpy, heat transfer coefficient, draught in water column
I irreversibility
i specific irreversibility
IP indicated power
k thermal conductivity, spring constant, number of cylinders, blade velocity coefficient
KE total kinetic energy
ke kinetic energy per unit mass
L stroke, length, thickness, length dimension
LCV lower calorific value
M mass dimension, Mach number
Nomenclature xxix

M molecular weight
m mass
m rate of mass flow
N rotational speed
n polytropic index, number of moles
P power
p absolute pressure
PE total potential energy
pe potential energy per unit mass
pm mean effective pressure
Q heat transfer
Q rate of heat transfer
q heat transfer per unit mass, heat flux
R gas constant, radius
Ru universal gas constant
r radius, compression ratio
rw work ratio
S entropy
s specific entropy, displacement
T absolute temperature; torque, temperature dimension
t time, time dimension
U internal energy
u specific internal energy
V volume
Vc clearance volume
Vs swept volume
V velocity
v specific volume
v– molar volume
W. weight, work transfer
W rate of work transfer, power
Wsh shaft power
w specific weight, workdone per kg
wsh shaft work/kg
x dryness fraction; length
xi mass fraction
yi mole fraction
Z compressibilty factor
z elevation.

Greek Symbols
a absorptivity
b coefficient of volumetric expansion
D finite change in quantity
xxx Nomenclature

e emissivity, effectiveness
g ratio of specific heats, Cp/Cv,
F avaialability of closed system
f specific avaialability of closed system, relative humidity
y specific availability of flow system
v kinematic viscosity
m Joule–Thomption coefficient
h efficiency
q temperature difference, angle, total energy of flow system
r density, cutoff ratio, reflectivity
s Stefan’s–Boltzmann constant
t transmissivity,
w specific humidity, angular speed.

act actual
atm atmosphere
c cross section, clearance
cr critical point
g, gauge gauge
d diameter
db dry bulb
dp dew point
f liquid state, of formation
fg liquid vapour mixture
i initial state, ice point, intermediate state in multistage, ith component in a mixture
g gaseous state
H high (temperature, TH)
HP heat pump
L low (temperature, TL)
O exit state
p constant pressure
prop propulsive
R reaction
r reduced coordinates
REF refrigerator
ref referenece temperatures
s, sat saturated state
sup superheated state
t total
th throat, theoretical, thrust
tp triple point
v constant volume
Nomenclature xxxi

w water
wb wet bulb
wet wet state
0 dead state, stagnation state
1 initial state
2 final state

UNITS AND DIMENSIONS

Base Units
Quantity Units Dimensions
Length metre m
Mass kilogram kg
Time second s
Electric current ampere A
Temperature kelvin K

Derived Units
Acceleration a metre per second squared m/s2
Angular acceleration w radiation per second squared rad/s2
Area A square metre m2
Electric current I ampere A
Electric potential difference V volt W/A
Electric resistance Re ohm
Energy E joule J or N.m
Entropy s joule per kelvin J/K
Force F newton kg.m/s2
Frequency v hertz Hz or 1/s
Heat energy Q joule J or N.m
Power P watt W or J/s
Radiation Intensity I watt per steradian W/sr
Specific heat Cp joule per kilogram kelvin J/kg-K
Stress s pascal N/m2
Thermal conductivity k watt per metre-kelvin W/m-k
Velocity U metre per second m/s
Volume V cubic metre m3
Work W Joul J or N./m
xxxii Contents
VISUAL WALKTHROUGH

Basic Concepts 1 Each chapter begins with an

1 chapter importance.

Basic Concepts

Introduction
The definition of the basic concepts forms a sound foundation for understanding of any subject. We start
this chapter with an overview of thermodynamics, and a discussion of some basic concepts such as closed
and open systems, isolated and adiabatic systems, working substance, continuum, property, state, path,
process, cycle, and equilibrium. Then we discuss the pressure, pressure measurement, temperature and its
measurement in this chapter. A careful study of these topics is necessary for understanding the following
chapters.

Ú
- 5.6 RELATION BETWEEN NON

ment as well as graphical proof for con- -FLOW WORK Ú pdv AND FLOW
Ú vdp

The steady-flow energy equation for unit mass-flow


rate
V2 V2
q – w = h2 - h1 + 2 - 1 + ( z2 - z1) g
2 2
It can be written in differential form as
dq – dw = dh + VdV + gdz
Using h = u + pv and
dq = du + pdv
Inserting it above, we get
du + pdv – dw = d(u + pv) + VdV + gdz
or du + pdv – dw = du + d(pv) + VdV + gdz
\ pdv – dw = d(pv) + VdV + gdz
or pdv = dw + d(pv) + VdV + gdz
p

1
c

Process 1-2

d 2

0 v
e f

Fig. 5.18
Ú pdv
Ú vdp
Visual Walkthrough xxxiii

For easy understanding of the topics, a

In some chapters, the whole concept is


xxxiv Visual Walkthrough

The boiler produces dry and satu-


rated steam at 30 bar. The steam expands in the turbine
sg = 6.1869 kJ/kg
At p2 = 20 kPa;
Numerous Worked examples are pro-
to a condenser pressure of 20 kPa. Compare the cyclic vf = 0.001017 m3/kg
work done and thermal efficiency of Carnot and Rankine TL = 60.06°C
cycles for these conditions. hf = 251.38 kJ/kg
hfg = 2358.33 kJ/kg
Solution
sf = 0.8319 kJ/kg ◊ K text material.
Given Dry saturated steam pressure sfg = 7.0766 kJ/kg ◊ K
Boiler pressure p1 = 30 bar The specific enthalpies
Condenser pressure; p2 = 20 kPa h1 = 2804.2 kJ/kg
h4 = 1008.42 kJ/kg
To find
(i) Work done by Carnot cycle,
Carnot Cycle
methodology.
(ii) Work done by Rankine cycle, Analysis
(iii) Thermal efficiency of Carnot cycle, The efficiency of Carnol cycle is given by
(iv) Thermal efficiency of Rankine cycle. TL (60.06 + 273)
hCarnot = 1 - =1-
Properties of steam TH ( 233.9 + 273)
At p1 = 30 bar; = 0.343 or 34.3%
hf = 1008.42 kJ/kg The heat supplied per kg of steam
hg = 2804.2 kJ/kg qin = h1 – h4 = 2804.2 – 1008.42
TH = 233.9°C = 1795.78 kJ/kg

summary
Summary
system is used on two wheelers, racing cars and
that converts chemical energy of fuel into aircrafts.
mechanical energy. internal combustion engines are subjected
to very high temperature during combustion of
cycle in two strokes of the piston. The three charge. Due to overheating of engine, there may
ports; inlet, transfer, and exhaust ports are used be uneven expansion in some parts, burning of
for suction, transfer and discharge of charge, lubricant, valve seats, etc. Therefore, the engine
respectively. A deflector-shapped piston is used should be provided with adequate cooling
to direct the charge inside the cylinder. arrangement.

cycle in four strokes of the piston as suction, maintenance-free and is widely used on two
compression, expansion and exhaust stroke. They wheelers and light-duty engines.
are widely used on motor cycles, cars, buses,
trucks and aeroplanes. Due to good thermal of heat during combustion, and therefore, they
efficiency of four-stroke engines, the specific fuel are water cooled. The water-cooling arrangement
consumption is less. consists of a pump, a fan, a water jacket around
Petrol engines use low compression ratio in the engine and a radiator.
the range of 4 to 10, while Diesel engines use
high compression ratio usually in the range of moving parts. The mist lubrication system is
14 to 21. The petrol engines induct carburetted used in two-stroke engines, while all four-stroke
homogeneous air–fuel mixture as charge into engines use wet or dry sump lubrication system.
the cylinder while Diesel engines induct only air
during suction, and diesel is injected at the end supplies the charge to the cylinder above
of the compression stroke. The fuel burns in the atmospheric pressure. Thus, the volumetric
presence of hot air. Therefore, the Diesel engines efficiency of the engine improves and the engine
are also called compression ignition (CI) engines. produces more power.
-
gines, while Diesel engines are quality-governing cylinder of the engine where the admitted charge
engines. The hit-and-miss governing is used for pushes the combustion products out of the
high speed gas engines. cylinder. Scavenging takes place in two-stroke
Visual Walkthrough xxxv

A list of commonly used terms in the chap-


Glossary
Triple point The point where the solid, liquid and va- Critical state The peak of the saturation curve form of a glossary
pour phases coexist in thermal equilibrium Quality (x) The ratio of mass of vapour present to the
Vaporization The transformation of liquid into vapour total mass of steam
by supplying heat Wetness fraction The ratio of mass of liquid present to
Evaporation The transformation of liquid into vapour the total mass of steam (= 1 – quality = 1 – x)
when liquid itself exacts its latent heat Wet steam A mixture of moisture and vapour, it is a
Condensation The transformation of a vapour into a substances that exists under the saturation curve.
liquid Phase The physical state of matter
Melting The transformation of a solid into a liquid, also Dry steam Steam at saturation temperature, which is
known as fusion free from moisture
Solidification The transformation of a liquid into a Superheated steam Dry steam at a temperature greater
solid, also called freezing than the saturation temperature
Sublimation The transformation of solid into vapour Ideal gas An ideal working substance for thermody-
Saturation The condition that exists when two or more namic cyles
phases coexist in equilibrium

Each chapter contains a set of Review


to recall the essence of the con-
1. What are the limitations of the first law of 15. List suitable examples of reversible and irrevers-
cepts discussed in the chapter. thermodynamics? ible processes.
2. State the importance of the second law of thermo- 16. What are the conditions to be a reversible process?
dynamics. 17. What is a thermodynamic temperature scale?
3. Define thermal reservoir, heat engine, refrigerator 18. Explain the establishment of a thermodynamic
and heat pump. temperature scale. Why is the thermodynamic
4. What is a thermal energy reservoir? Define in temperature scale called absolute temperature
terms of heat source and heat sink. scale?
5. What are the characteristic features of a heat 19. Sketch the Carnot cycle on a p–v diagram.
engine? 20. Explain Carnot cycle and prove that
6. What is a heat pump. How does it differ from a T
h carnot = 1 - L .
refrigerator? TH
7. What do you mean by coefficient of performance? 21. Is it possible to operate a heat engine on a Carnot
Show that (COP)HP = (COP)R + 1. cycle? What purpose does a Carnot cycle serve?
8. What do mean by thermal efficiency? Derive an 22. State and prove the Carnot theorem.
expression for thermal efficiency of a heat engine. 23. Prove that a reversible engine is more efficient
9. State the Kelvin–Planck and Clausius statements than an irreversible engine operating between the
of the second law of thermodynamics. same temperature limits.

problems with answer(s)


Problems
1. A rigid tank of volume 2 m3 is filled with saturated reduced by using helium instead of air in the
steam at 2 bar. It contains 0.2 m3 of moisture and tyres? What other considerations are involved in -
the rest is vapour. Calculate the mass of moisture, deciding whether to use helium?
mass of vapour and the quality of steam. 10. A tank with a volume of 50 m3 is being filled with
[(i)188.5 kg (ii) 2.032 kg (iii) 0.01066] air. At a particular instant, the air in the tank has a
2. Steam at 0.75 bar and 150°C is condensed revers- temperature of 400 K and a pressure of 1380 kPa.
ibly. Find the heat removed and change in entro- For this system, the pressure is increasing at the
py. Sketch the process on a T–s diagram. rate of 138 kPa/s and temperature is increasing at
[2393.8 kJ/kg, 6.536 kJ/kg ◊ K] a rate of 25 K/s. Calculate the air flow rate into
3. Determine the increase in entropy of 1 kg of the tank in kg/s. [21.25 kg/s]
water when it is heated at atmospheric pressure
11. A spherical vessel of 3-m diameter contains steam
from –23°C to 152°C. Take sp. heat of fusion and
at a pressure of 7 bar (gauge) and a temperature of
superheating as 2.093 kJ/kg ◊ K and for water as
4.187 kJ/kg ◊ K. [9.0403 kJ/kg] 200° C, Find
4. Obtain specific volume, specific enthalpy, entropy (a) Total enthalpy of steam
of 2 kg of water at 1 bar and 60°C. (b) Total internal energy of steam
5. A perfect gas of 0.01 –kg mass occupies a volume (c) Total mass of steam
of 0.02 m3 at a pressure of 286.4 kPa and 30°C re- Take atmospheric pressure as 1 bar.
spectively. The gas is allowed to expand until the [(a) 156.16 MJ (b) 144.85 MJ (c) 55.16 kg]
xxxvi Visual Walkthrough

enable the user to


Objective Questions
Choose the correct answer: 9. At the critical point, the temperature of water is
- 1. The specific volume of water during freezing equal to
(a) increases (b) remains constant (a) 0°C (b) 100°C
(c) decreases (d) none of the above (c) 374°C (d) –100°C
2. The latent heat of vaporization with increase in 10. The total enthalpy of steam at 10 bar is
pressure of water 2000 kJ/ kg. The condition of steam is
(a) increases (b) remains constant (a) wet (b) dry and saturated
(c) decreases (d) none of the above (c) superheated (d) none of the above
3. With increase in pressure, the saturation tempera- 11. Dryness fraction of steam is given by
ture of water Mass of dry steam in a sample
(a)
(a) increases (b) remains constant Mass of water particles in thee sample
(c) decreases (d) none of the above
Mass of water particles in the sample
4. The specific volume of wet steam is given by (b)
vg Mass of dry steam in the sample
(a) (b) xvf Mass of wet steam in the sample
x (c)
(c) xvg (d) x 2vg Mass of dry steam in the sam
mple

A and C.
Appendices—A, B,

Appendix
Table A.1

Chemical M R r Tc pc pc vc
Formula (kg./kmol) kJ/kg-K kg/m3 (K) (bar) Zc =
Substance RTc
Acetylene C2H2 26.04 0.3193 –1.05 309 62.8 0.274
Air (equivalent) – 28.97 0.287 1.169 133 37.7 0.284
Ammonia NH3 17.03 0.4882 0.694 406 112.8 0.242
Argon Ar 39.94 0.2081 –1.613 151 48.6 0.290
Benzene C6H6 78.11 0.1064 — 563 49.3 0.274
Butane C4H10 58.12 0.1430 2.407 425 38.0 0.274
Carbon C 12.01 — — — — —
Carbon dioxide CO2 44.01 0.1889 1.775 304 73.9 0.276
Carbon monoxide CO 28.01 0.2968 1.13 133 35.0 0.294

0 1 2 3 4 5 6 7 8 9 10
1200 1200
h = 5000 kJ/kg
00
5500

50

5000
4500
1100 4900 1100
4800
h = 4200 kJ/ 4700
kg
1000 4600
1000
/m 3

4000
= kg

4500
3800
sity

/m 3

900 4400 900


Den

300 kg

3600 4300
15 000 ar

3400 4200
20 00 b

800 800
/m 3
4 00 6000
100 000

4100
300

80000
0

100 kg

3200
P=

30 000

4000
20 0
50

0
15 00

700 3000 700


/m 3
00

3900
Temperature, °C

30 kg

2800
00

3800
10

/m 3

0
600 2600 80 3700 600
10 kg

0
60
0
2400 50 3600
3 kg/m 3

0
40 0
500 kJ/
2200 kg 35 00 3500 500
1 kg/m 3

h= 3 0
25 0 3400
2000
20
kg/m 3

g/m 3

0
1800 15 3300
400 400
3k

0
10
0.3

kg/m 3

0.0

3200
1600 80
60
kg/m 3
40

0
0.1

3100
140
30

300 Sa 300
20

.01

id tu
15

qu
10

0 ra
120 d li
=0

te d 3000
ate va
tur
sity

0 Sa p
100 or
8
6

1.5
4

1.0

2900
3
2

Den

200 200
1 20

14

h=
1 00 0

20
0.4
0.8
0.6

0.2

0.1
16

0.3

0%
0.1

22 2800
00

26
18

00 00 24
=1
00
0

00 00 00
k J/k
80 0

ty
15

%
08

ali 20 g
0.02
0.08
6

3
0.04

Qu
0.0
0.0
%

0.0
0.0
40%

100 100
30

Qu
h = 40

2650
50%
60 0

a lit
60%

70 80% y= h = 2600 kJ/kg


0 06

% 90 %
0.0.004
0.0

0.0 03
0k

02
20 0

2550
J/ k

0 0
g

0 1 2 3 4 5 6 7 8 9 10
Entropy, kJ/kg · K

Fig. C.1
Basic Concepts 1

1
Basic Concepts

Introduction
The definition of the basic concepts forms a sound foundation for understanding of any subject. We start this
chapter with an overview of thermodynamics, and a discussion of some basic concepts such as closed and
open systems, isolated and adiabatic systems, working substance, continuum, property, state, path, process,
cycle, and equilibrium. Then we discuss the pressure, temperature and its measurement in this chapter. A
careful study of these topics is necessary for understanding the following chapters.

nuclear power plants. An energy-efficient home is


designed for minimum heat loss in winter and mini-
Thermodynamics can be defined as the ‘science mum heat gain in summer. The size, location and
of energy’. In fact, the name ‘thermodynamics’ power input to an air-conditioner is decided after
originates from two Greek words, threme (heat thermodynamic analysis of a room.
energy) and dynamics (motion or power). Thus,
the subject of thermodynamics deals with energy
and its transformation, including heat, work and
physical properties of substances. It also deals A thermodynamic system, or simply a system, is
with thermodynamic equilibrium and feasibility of defined as a certain quantity of matter or a pre-
processes. scribed region in space considered for thermody-
namic study.
Every engineering activity involves an interac-
tion between energy and matter, and it is hard to The region outside the system is called the
find an area which does not relate to thermodynam- surroundings or environment. The real or
ics in some respect. An ordinary house has a gas imaginary surface that separates the system from its
stove, an electric iron, fans, a cooler, a refrigerator, surroundings is called the boundary. The boundary
pressure cooker and televisions. The design of each of the system may be fixed or movable. The system
item requires the knowledge of thermodynamics. and its surroundings constitute the universe. These
terms are illustrated in Fig 1.1.
In the engineering field, thermodynamics plays
an important role in the design of automobile The thermodynamic systems
engines, compressors, turbines, refrigerators, rock- can be classified as
ets, jet engines, solar collectors, conventional and
2 Thermal Engineering

Fig. 1.1

1. Closed and open systems


2. Homogeneous and heterogeneous systems
Fig. 1.3

2. Food Items in a Pressure Cooker, Fig. 1.4


A closed system (also known as control mass) has
the following characteristics: (i) The inside surface of the pressure cooker
and its cover forms the boundary.
(i) It consists of a fixed amount of mass, and (ii) The boundary of the system is fixed.
no mass can cross its boundary, i.e., no mass
(iii) No mass of the food can cross the boundary,
can enter or leave a closed system.
unless by the process of whistling.
(ii) The volume of a closed system may vary and
(iv) Energy as a heat can leave or enter the
hence its boundary is movable.
boundary of the system.
(iii) The energy in the form of heat or work can
cross the boundary.
Figure 1.2 shows the representation of a close
a system. Some examples of closed system are
discussed below.

Heat
Heat
Mass (No)
Control Fig. 1.4
Energy (Yes)
mass
3. Refrigerator and Ice-cream F Figure 1.5
(b) shows the basic components of a refrigerator
as a system along with its boundary. The working
Fig. 1.2
substance is the refrigerant.
1. Gas Trapped within a Piston–cylinder Device,
Fig. 1.3
(i) The compressor, condenser, capillary tube
and condenser together constitute a system.
(i) The inside surface of the cylinder and piston
(ii) No mass of the working substance can leave
forms the boundary.
the system.
(ii) With the movement of the piston, a part of
(iii) The boundary of the system is fixed.
the boundary can move.
(iv) The energy as electrical work enters the
(iii) The movement of the piston is restricted by
compressor, and energy as heat leaves the
a stopper, so no mass of gas can leave the
condenser and enters the evaporator. Thus,
system.
energy crosses the boundary.
(iv) The energy, as heat and work, can cross the
Hence, the refrigerator is a closed system.
boundary.
Basic Concepts 3

Evaporator Capillary
coils tube

Kitchen air
Heat in
-10°C Heat out
Evaporator

Condenser Compressor
coils

5°C

Condenser
Expansion
Compressor device

(a) (b)

Fig. 1.5

4. Steam (Thermal) Power Plant Figure 1.6 shows 5. Electrolytic B


the basic components of a steam power plant as a (i) The acid and lead plates in a plastic box
closed system along with its boundary. The working make up the system.
substance is water vapour. (ii) The boundary of the system is fixed.
(i) The boiler, turbine, condenser, and feed (iii) No mass of the acid can leave the boundary.
pump together constitute a system. (iv) Energy in electrical form can leave or enter
(ii) No mass of the working substance can leave the boundary of the system.
the system.
(iii) The boundary of the system is fixed. +
I (current)
(iv) The energy as heat and work can cross the
– V2 – V1
boundary of the system.
Thus, the steam power plant is a closed system.
Electrolytic System
Heat supply Battery Boundary

Feed water
Boiler
Superheated Fig. 1.7
steam

Pump Wout 6. Bulbs and Lamps, Fig. 1.8


Win Turbine
(i) The mass of the inert gas remains fixed
inside bulbs and lamps.
Condenser
(ii) The boundary of the system is fixed.
Condensate Moist steam
(iii) Energy as electricity, heat and light can cross
Heat
the boundary of the system.
Rejection
An isolated system is a special
Fig. 1.6 case of a closed system, in which energy can also
4 Thermal Engineering

An open system (or a control volume) is a properly


selected region in space. It usually encloses a device
which involves mass flow, such as a compressor,
turbine or nozzle. Flow through these devices is
best studied by selecting a region within the devices
as control volume.
The boundary of the control volume is called the
control surface. Both mass and energy can cross the
Fig. 1.8 control surface. Thus, for an open system:
(i) The system has a selected region (fixed
not cross the boundary of the system. This system volume), called control volume.
is not in communication with its surroundings in (ii) The boundary of an open system is fixed.
any way. (iii) Mass can cross the control surface.
Thus, an isolated system neither exchanges (iv) The energy, in the form of heat and work,
energy in any form nor any mass with the can cross the control surface.
surroundings. Hence, by definition, the universe An open system can be represented by Fig. 1.11.
can be considered as an isolated system. The Some examples of open systems are discussed
representation of an isolated system is shown in below.
Fig. 1.9. Other examples are thermos flask
(Fig. 1.10) and ice box. Control surface
(boundary)
Mass in
Boundary
Control volume
Mass (Yes) or Energy (Yes)
Open system
Mass (No) Energy (No)
Mass out

Fig. 1.11
Fig. 1.9
1. Flow through Tubes and Nozzles, Fig. 1.12
(i) The interior surface of the tube or nozzle
forms the real boundary, and left and right
openings form the imaginary boundary.
(ii) The mass can enter and leave the imaginary
boundary of the control volume.
(iii) Energy as well as mass across the boundary
can enter or leave the system.
2. Water Boiler, Fig. 1.13
(i) The interior surface of the boiler shell forms
the real boundary, and the left and right
Fig. 1.10 openings form the imaginary boundary.
Basic Concepts 5

(ii) Mass of air can enter and leave the control


Fluid in Flow through a tube Fluid out volume through the valves.
(a)
(iii) Energy as work and heat can cross the
control surface.
4. Internal combustion engines, gas turbine,
Fluid in
Convergent
Fluid Out
Fig. 1.15
nozzle
(i) The interior surface and openings form the
control surface.
(b)
(ii) Mass as air and fuel mixture enters and
Fig. 1.12 leaves as combustion products.
(iii) Some part of the combustion heat is
Steam converted into work and the remaining is
out
Steam discharged as waste heat from the system.

Control Energy as heat


Water in surface
Fuel
Heat Combustion
products
Air
Fig. 1.13

Energy as work
(ii) The mass of the water enters, and the mass (a) Internal combustion engine
of steam comes out the control surface. Heat
(iii) Energy as heat enters the control surface.
(iv) The energy in the form of heat or work can
Fuel Combustion
cross the boundary.
products
Figure 1.14
shows the reciprocating air compressor as an open Work
system. Combustion
Air products
(i) The interior surface of cylinder and piston
forms the control surface.
Stationary blades Rotating blades
Compressed
air out (b) Gas turbine engine
Fig. 1.15
Piston

An adiabatic system is a special


case of an open system, in which mass can cross the
Control Air Being
Compressed Work control surface, but energy in the form of heat is not
values
allowed to cross the control surface of the system.
However, energy in other forms can enter and leave
Cylinder
Fresh the system. Insulated turbines, throttle valves, water
air in
pumps, water turbines, insulated heat exchangers,
etc., are some examples of adiabatic systems.
6 Thermal Engineering

Some common features of these devices are the Fluid B

following: No heat transfer

(i) The interior surface of the device forms the Heat


control surface.
Fluid A Fluid A
(ii) Fluid enters and leaves the control surface.
(iii) The energy other than heat energy can enter Insulation

or leave the control surface.


(iv) Heat transfer is negligible at its outer control
Insulation
surface. Fluid B

Insulation Fig. 1.19

Hot fluid in
Q=0
Adiabatic
system Mass (Yes)
Heat
Cold Cold
fluid out fluid in

Fig. 1.16

Hot fluid out

Fig. 1.20

as shown in Fig.1.20 and it will not be treated as an


adiabatic system.
The thermodynamic relations that are applicable
to closed and open systems are different. Therefore,
Fig. 1.17 it is important that we must recognise the type of
system before analysing it.
Steam in

Control
Shaft work
volume
Closed system Open system
1. It is also called a non- It is also called a flow
flow system. system.
Steam out 2. A certain quantity of A certain region is
Fig. 1.18 matter is considered considered for study.
for study. Thus, a This region is called
If the heat exchanger considered above in closed system has a control volume.
Fig. 1.19 is not insulated at its outer boundary then control mass.
the heat transfer will take place across its boundary Contd.
Basic Concepts 7

3. The system is An open system liquid or gas phase only. It is treated as one
surrounded by a real is surrounded by a constituent for its analysis. Thus, analysis becomes
boundary, which may control surface, which simple, for example, ice, water and steam (three
be fixed or movable. is a combination of distinct phases of water), sugar or salt dissolved in
real and imaginary water, air, oxygen gas and nitrogen gas.
boundaries.
4. No mass can cross the Mass as well as energy
boundary, while can enter or leave the When a system is a mixture of two or more than
energy can enter or control surface of the
two phases of matter, it is called a heterogeneous
leave the boundary of system.
system.
the system.
Since each constituent present in the system has
5. If energy transfer does If heat transfer does
its own properties independent of each other, the
not take place across not take place across
system cannot be analysed as a single constituent,
the boundary then the the control surface
closed system is called then an open system for examples, mixture of ice and water; mixture of
an isolated system. is called an adiabatic water and steam; dal, rice and water in a pressure
system. cooker, etc.
6. Examples of a closed Examples of an open
system are pressure system are scooter
cooker and refrigerator. engine, air compressor
and gas turbine.
It is well known that every substance is composed
of a large number of molecules. The properties
of the substance depend on the behavior of these
molecules. In the macroscopic approach, a certain
Isolated system Adiabatic system quantity of matter is considered without the events
1. It is a special type of a It is a special type of an occurring at the molecular level. The macroscopic
closed system. open system. approach in the study of thermodynamics is also
2. Mass and energy do Mass and energy, called classical thermodynamics. It provides a
not cross the boundary except heat energy, can direct and easy way to the solution of engineering
of the system. cross the boundary of problems. In the macroscopic approach,
the system.
1. The structure of the matter is not considered,
3. It is a closed system, It is an open system, 2. Only a few variables are needed to describe
which is insulated at which is insulated at its the state of the system,
its boundaries, thus it real boundaries, thus
3. The values of these variables can be mea-
becomes isolated from heat cannot cross it.
its surroundings. sured.
4. Examples of an Examples of an The microscopic approach is more elaborate.
isolated system are ice adiabatic system are We know that every system is composed of a large
box and thermos flask, water pump, throttle number of molecules. All have the same mass but
etc. valve and insulated each moves with a velocity independent of others.
steam turbine etc. Similarly, each molecule has its own position,
temperature, etc. The microscopic approach of such
a system will involve a large number of equations,
specifying three location coordinates and three
A system is called a homogeneous system, if it
velocity components for each molecule. It is
consists of a single physical phase, either solid,
8 Thermal Engineering

very difficult to adopt in practice even with high- r

speed computers. The properties of the system are Region of Region of


based on the average behavior of a large group of molecular effect continuum

molecules under consideration. Thus, this approach Dm P


is also called statistical thermodynamics. In the P
DV
microscopic approach,
1. The knowledge of the structure of the matter DV¢ DV
is necessary
Fig. 1.21
2. A large number of variables are needed to
describe the state of the system smaller than this volume will lead to discontinuity
3. The values of the variables cannot be in the particles, atoms and electrons, etc., and the
measured easily. density becomes unpredictable.
The variation in density is tentatively shown in
Fig. 1.21. When the volume approaches zero, the
density becomes uncertain.
The matter contained within the system boundaries
is called working fluid. It is used in thermodynamic Ê Dm ˆ
r = lim Á ˜ ...(1.1)
devices as a medium for energy transport between DV Æ DV ' Ë DV ¯
the system and surroundings, while undergoing a
thermodynamic process or cycle. A working fluid Similarly, the definition of pressure is also
may be gas, vapour, liquid or any non-reactive required for a minimum area DA¢ at the which force
mixture of these constituents. The working fluids DFn acts.
frequently absorb, store or release energy. For Ê D Fn ˆ
r = lim Á
DA ˜¯
examples, water vapour is the working fluid in a ...(1.2)
DAÆ DA ' Ë
steam power plant, and a refrigerant is the working
fluid in a refrigerator. The continuum is not applicable when the
number of molecules in a system becomes
negligible, e.g., a system under high vacuum.

Matter is made up of discrete particles, called


atoms. These atoms are widely spaced (free path)
in a gaseous phase and matter may also have some
voids. The macroscopic approach is applicable Any characteristic of a system is called a property.
when the smallest unit of the matter is large enough Every system has certain characteristics by which
compared to the mean free path of the atoms. its physical condition may be described, e.g.,
Under such circumstances, the matter in a system volume, temperature, pressure, etc. The list can
is considered as continuous and homogeneous be extended to include velocity, viscosity, thermal
without any hole. This is called the concept of conductivity, modulus of elasticity, coefficient of
continuum. thermal expansion, resistivity and elevation, etc.
Let us consider the mass Dm in a volume DV The salient features of a thermodynamic
surrounded the point P as shown in Fig. 1.21. The property are the following:
ratio of Dm/DV is the average mass density of the (a) A property is a measurable characteristic,
system within the volume DV. The volume DV ¢ is describing the state of a system.
the smallest volume about the point P, for which the (b) It has a definite value when the system is in
mass can be considered continuous. Any volume a particular state.
Basic Concepts 9

(c) It also helps to distinguish one system from properties are independent of the mass also.
another. Properties such as pressure, temperature, density,
(d) The magnitude of a property depends on the velocity, etc., are examples of intensive properties.
state of the system, and it is independent These are properties that
of the path or route followed by a system vary directly with the extent of the system. These
during a process. properties depend on the mass of the system. The
(e) A property is an exact differential. properties such as mass, area, volume, total energy,
The differential quantity of a property P is etc., are examples of extensive properties.
designated as dP, and its integral between states 1 An easy way to distinguish whether a property
and 2 of the system is is intensive or extensive is to divide the system
2 into two equal parts with a partition as shown in
Ú1
dP = P2 – P1 …(1.3) Fig. 1.22.
For a given expression of a property,
dP = Mdx + Ndy E (kJ) ½E ½E
m (kg) ½m ½m
A simple check can be a useful tool to recognize V(m3/s) ½V ½V
whether a quantity is a property or not. r (kg/m3) r r
V (m/s) V V
Ê ∂M ˆ Ê ∂N ˆ o

ÁË ∂y ˜¯ = ÁË ∂x ˜¯ ….(1.4) T ( C) T T
p (kPa) p p
x y

Properties may be classified Fig. 1.22


as
1. Intrinsic and extrinsic properties according
to their origin, and After partition, each part will have the same
value for the intensive properties as of the original
2. Intensive and extensive properties according
system, but half the value for extensive properties.
to their dependability.

An extensive property expressed


These are the basic properties,
per unit mass of the system is called a specific
and cannot be defined in terms of other properties.
property. Examples include specific volume,
Their values can be assigned independently for
specific energy, specific enthalpy, specific internal
example, length, mass, time, area volume, pressure,
energy, etc. and thus are intensive properties.
temperature, electric current, etc.
Mass density or simply density is a measure
These are those properties of the amount of working substance contained in
whose values cannot be assigned independently. a given volume and is defined as mass per unit
These are characteristic of the motion or position volume.
of a system and are measured in reference to certain
Mass of substance m
datum such as velocity, acceleration, potential r= = (kg/m3) ...(1.5)
energy, kinetic energy, enthalpy, entropy, etc. Volume occupied V
Similarly, , or relative density, is
defined as the ratio of density of a substance (rs) to
These the properties that the density of water at 4°C (rwater = 1000 kg/m3). It
do not depend on the extent of the system. These is designated by SG.
10 Thermal Engineering

rs Solution
SG = (A dimensionless quantity)
rwater v p
or rs = SG ¥ rwater = SG ¥ 1000 (kg/m3) ...(1.6)
Given The quantity dp + d v.
T T
The ( ) is the reciprocal of To find Whether the given quantity is a property or not.
mass-density and is defined as the volume per unit
Analysis Comparing the given quantity with M dx +
mass of a system. N dy, we get
V 1 v
= = (m3/kg) ...(1.7) M= and x=p
m r T
The (g) is defined as the weight p
and N= and y=
of a substance per unit volume, or T
mg Obtaining the partial differentials, we get
g = ...(1.8)
V Ê ∂M ˆ ∂ Ê vˆ 1
The relation between specific weight and mass ÁË ∂v ˜¯ = ∂v ÁË T ˜¯ = T
p
density is given as
g = rg ...(1.9) Ê ∂N ˆ ∂ Ê pˆ 1
and ÁË ∂p ˜¯ = ∂p ËÁ T ¯˜ = T
v
Example 1.1 Recognise whether the following quan-
tities are properties or non-properties: Ê ∂M ˆ Ê ∂N ˆ
Since Á = , thus the given quantity is a
(a) pd + dp (b) pd Ë ∂v ˜¯ p ÁË ∂p ˜¯
v
(c) dp
property.
where p is the pressure and is the specific volume.

Solution
(a) The quantity is pd + dp.
Its differential is d(p ).
It is an exact differential, and thus the quantity pd +
dp, is a property. The thermodynamic state is the condition of the
(b) The quantity is pd .
system as characterised by certain thermodynamic
Here, p is functionally related with and its
properties like pressure, temperature, specific
intergration cannot be evaluated unless the relationship volume, etc.
between p and is known. Thus, the quantity pd is not If any system is not undergoing any change then
a property. all of its properties can be measured or calculated,
(c) The quantity is dp. which gives us a set of properties that completely
Here, is functionally related with p, and thus by the describe the condition or state of the system. At this
same reasoning as given in (ii), the quantity dp is not a state, all thermodynamic properties of the system
property. have the same value throughout the system. If the
value of even one property changes, the system will
If the pressure p, specific volume and change its state to a different one.
absolute temperature T are funtionally related as Consider a certain quantity of a gas as a system
v p in a piston–cylinder device as shown in Fig.1.23
dp + d v , examine whether the quantity is a
T T (a). At the position 1 of the piston at any instant,
property. the condition of the system can be described by
pressure p1, temperature T1 and volume V1. The
Basic Concepts 11

system is said to exist at the state 1. After expansion The system may reach from state 1 to state 2
of the gas, the system will reach a new position 2 by a number of paths depending on the type of
(state 2) as shown in Fig.1.23 (b). expansion.

Cylinder Piston

The transformation of a thermodynamic system


from one thermodynamic state to another is called
Gas
a process.
A series of states through which a system passes
Position 1 Position 2 during a process is called the path of the process.
(a) Piston at two positions within a cylinder The processes are classified as follows:
p 1. A process undergone by a fluid in a closed
system, is referred as a non-flow process.
p1 State 1 2. A process undergone by a fluid in an open
system is referred as a flow process.
3. When a process proceeds in such a
p2 State 2 manner that the system remains almost
infinitesimally close to equilibrium, such a
process is called a quasi-static process.
V 4. A process is called a reversible process if
V1 V2
it once has been taken place between two
(b) Representation of states on a plot
states, can be reversed to restore the system
Fig. 1.23 to initial conditions without leaving any
effect on the surroundings. It passes through
a series of equilibrium states.
5. A process which cannot be reversed by the
When a gas expands in the cylinder, the piston
same path, and follows in one direction only
moves outward, the properties of a system change
is called an irreversible process. It passes
and the system reaches to the new state 2. It is
through a series of non equilibrium states.
called a change of state. A locus of series of states
through which a system passes between initial and 6. When a system undergoes a process, while
final states is called a path as shown in Fig.1.24. enclosed by an adiabatic wall (ideal insu-
lator), the system does not experience any
p heat exchange between the system and its
1
p1 surroundings. Such a process is called an
Pa
th
2
adiabatic process.
Pa The prefix iso- is often used to designate a type
th
1
p2 2 of process for which a particular property remains
constant.
() P The temperature remains
V constant during the process.
V1 V2
( ) P The pressure remains
Fig. 1.24 constant during the process.
12 Thermal Engineering

( ) P The volume rection to represent the quantity on any plot, e.g.,


remains constant during the process. heat, work, etc. The path functions have inexact
differentials represented by the symbol d.
( ) P The entropy remains
constant during the process. Therefore, a differential amount of work or heat
is written as dW or dQ. A typical representation of
( ) P The enthalpy remains expansion work (a path function quantity) is shown
constant during the process. in Fig. 1.26.

If a system undergoes a series of processes in such


a way that its initial and final states are identical
then the system is said to have undergone a cyclic
process or simply a cycle. A thermodynamic cycle is
a sequence of processes that begins and ends at the
same state as shown in Fig. 1.25. At the conclusion
of a cycle, all properties of the fluid have the same
values as they had at the initial state.
Fig. 1.26
p 3

2 When a process proceeds in such a manner that


4
the system remains infinitesimally close to an
equilibrium state at all times, it is called a quasi-
1 static, or quasi-equilibrium process. A quasi-static
V
process is viewed as a sufficiently slow process in
Fig. 1.25 which system changes its state very slowly under
the influence of an infinitesimally small driving
force. The system adjusts itself internally, so that
the properties in one part of the system do not
change any faster than those in the other part.
When a system undergoes a change from one state Figure 1.27 illustrates a quasi-static process.
to another, the properties of the system also change, The system consists of a gas initially at equilibrium
which depend only on end states and not on the under the piston in a cylinder. The piston is loaded
path followed between these two states. Therefore, by a number of small masses. As one mass is
these properties are called state functions or point removed, the gas expands slightly, allowing the
functions. Point functions can be represented by a piston to move slowly upward. During such
point on any plot, e.g., temperature, pressure, vol- expansion, the molecules get sufficient time to
ume, etc. These properties have exact differentials redistribute, and thus the gas would depart only
designated by the symbol d. Therefore, change slightly from equilibrium and as the pressure and
in volume or pressure is represented by dV or dp, other intensive properties become uniform, it will
respectively. attain a new equilibrium state. Moreover, if the
A quantity, whose value depends on the par- same mass is put on the piston, the gas would be
ticular path followed during the process is called restored to its initial state.
a path function. It requires a particular path and di-
Basic Concepts 13

A quasi-static process is an ideal process and it


is not a true representation of actual processes. But
Incremental masses
removed during an many actual processes closely approximate it and
expansion of a gas they can be modeled as quasi-equilibrium-process
with negligible error. Engineers are interested
in quasi-equilibrium processes for two reasons.
Boundary First, these are easy to analyse; and second, work-
Gas
producing devices deliver the maximum work, and
work-absorbing devices require minimum work
(a) Illustration of quasi-equibrium expansion when they operate on quasi-equilibrium processes.
p Therefore, quasi-static processes serve as standard
1 for other processes.
Process
path

2 Thermodynamics deals with equilibrium states or


state of balance. A system is said to be in equilibrium
V
V1 V2 state when there is no unbalance potential (driving
force) within the system. It means that the intensive
p
properties are same throughout the entire system
and there is no tendency for a change of state. If
two systems are in the same intensive state, they are
(b) Work of quasi-equilibrium process
in equilibrium with each other. If the system and its
Fig. 1.27 surroundings are in the same intensive state, they
are also in equilibrium state.
If several masses were removed from piston
one after an other, the gas would pass through a Thermodynamic equilibrium consists of many
series of equilibrium states without ever being far types of equilibrium, like thermal, mechanical,
away from equilibrium. If the increments of mass chemical, etc.
are made negligibly small, the gas would undergo A system is said to be in thermal equilibrium,
a quasi-equilibrium expansion process. A quasi- if its temperature is the same throughout the entire
equilibrium compression can also be visualized in system, because temperature difference is potential
the similar way. for heat flow.
However, if in actual process, a gas in a A system is said to be in mechanical equilibrium,
piston–cylinder device is compressed suddenly, in the absence of any unbalance force. The force is
the molecules near the face of the piston do not directly related with pressure. Thus, if there is no
have enough time to escape and they pile up in a pressure difference (driving force for mechanical
small region in front of the piston, thus creating work) throughout the system then the system would
a high pressure region. Because of this pressure be in mechanical equilibrium.
variation, the system can no longer be said to be If a system involves two phases, it is in phase
in equilibrium and this makes the entire process equilibrium, when the mass of each phase reaches
go in non quasi-static equilibrium. For an actual an equilibrium level and stays there.
process, the system does not pass through a series Finally, a system is in chemical equilibrium
of equilibrium intermediate states, and thus its path when a system does not undergo any chemical
cannot be recognized. reaction or its chemical composition does not
change with time.
14 Thermal Engineering

An isolated system has no interaction with its


surroundings and is always in internal equilibrium.
When any one of the above conditions of equilibrium Qunatity Unit Symbol
are not satisfied, the system is not considered to be Acceleration metre per m/s2
in thermodynamic equilibrium. second
Angular radian per rad/s2
acceleration second square
Angular velocity radian per rad/s
Any physical quantity may be characterized second
by dimensions. The reference standard used to Area square metre m2
measure the dimensions of a physical quantity is Electric resistance ohm W
called a unit. In the tenth and eleventh General Frequency hertz Hz or 1/s
Conference of Weight and Measures, it was decided Specific volume kilogramme per kg/m3
to use the single universally accepted system cubic metres
of units throughout the world and a system of Velocity metre per
measurement called the International System of Volume second m/s
Units was introduced. This system of units is called cubic metre m3
Système International d ‘Unités and is abbreviated
as SI units.
The seven units used for the seven fundamental
quantities, considered as basic units, are given in
Table 1.1. Qunatity Unit Symbol Expression in terms
of basic units
Force newton N kg ◊ m/s2
Pressure pascal Pa kg/m ◊ s2
Energy, joule J kg ◊ m2/s2
Qunatity Unit Symbol work, heat
Length metre m Power watt W kg ◊ m2/s3
Mass kilogram kg
Time second s capital letter. For example, the unit of force
Electric current ampere A named after Sir Isaac Newton is written as
Temperature kelvin K ‘newton’ (not Newton). Similarly, the unit
Amount of light candela cd of temperature named after Lord Kelvin is
Amount of matter mole mol written as ‘kelvin’ (not Kelvin).
2. Symbols of unit names derived after
A list of some derived physical quantities, their
scientist’s names are always written with an
symbols, units and dimensions are given in Table 1.2.
initial capital letter. For example, the symbol
Table 1.3 shows a list of some secondary units ‘N’ is used for newton and ‘K’ for kelvin.
and their expressions in terms of basic units.
3. If the symbol of a unit is not derived from
Guidelines for Writing Units and their Symbols a scientist’s name, it is written with a small
The following guidelines should strictly be followed letter. For example, ‘m’ for metre and ‘kg’
for writing correct units and their dimensions: for kilogram.
1. All unit names derived from scientist’s 4. The symbols of units do not take a plural
names are not to be written with an initial form. However, the full name of the unit
Basic Concepts 15

may be pluralized. For example, the length


of an object can be written as 5 m or 5 metres
but not 5 ms. Similarly, the mass of an object (i) Atmospheric It is the pressure exerted
can be written as ‘5 kg’ or ‘5 kilograms’ but by the envelope of air surrounding the earth’s
not ‘5 kgs’ . surface. The standard atmospheric pressure is equal
5. No full stop or other punctuation mark is to the pressure produced by a 760 mm high column
placed after symbols, unless they appear at of mercury, the density of mercury being 13,595
the end of a sentence. For example, metre is kg/m3 and the acceleration due to gravity being
written as ‘m’ not ‘m.’. 9.80665 m/s2 at sea level.
6. For better appearance, a single space must patm = 101325 N/m2 =1.01325 bar
always be provided between a numerical
value and a symbol of a unit. (ii) Absolute The actual pressure at a
given position is called absolute pressure. It is
designated as pabs, or simply p, and it is measured
by a barometer above the absolute zero pressure.
Pressure is defined as the normal force exerted by a
fluid per unit area. We speak of pressure only when (iii) Gauge All pressure-measuring in-
we deal with a gas or liquid. The counterpart of struments and gauges are calibrated to read zero at
pressure in a solid is stress. If F is the force normal atmospheric pressure, and so they indicate the dif-
to the area A, then ference between the actual (absolute) pressure and
local atmospheric pressure (patm ). This difference
F is called gauge pressure. It is denoted by pgauge and
p= (N/m2) ...(1.10)
A is expressed as
where force, F = mass (m) ¥ acceleration due to pgauge = pabs – patm ...(1.12)
gravity (g)
mass, m = volume ¥ density = area (A) ¥ depth The pressure measured below atmospheric
(h) ¥ density (r) pressure is called vacuum (gauge) pressure and

or F = Ahrg p > 1 atm


Positive
Ah r g gauge
Absolute pressure above atmosphere

and pressure, p = = r g h (N/m2) pressure


A
...(1.11) p = 1 atm
Negative
gauge
Local atmospheric pressure

pressure
Pressure is measured in newtons per square metre, p < 1 atm
which is called pascal (Pa). The pressure unit
below atmosphere
Absolute pressure

pascal being too small, very often kilopascal (kPa),


megapascal (MPa), bar and standard atmosphere
(atm) are used. These are related as
1 kPa = 103 Pa
1 MPa = 106 Pa = 103 kPa
p = 0 (Perfect
1 bar = 100 kPa = 105 Pa Absolute vacuum)
zero pressure
1 atm = 101, 325 Pa = 101.325 kPa
1 Torr = 133.32 Pa Fig. 1.28
16 Thermal Engineering

is measured by vacuum gauges, which indicate the Adiabatic wall

difference between the atmospheric pressure and


actual (absolute) pressure. It is expressed as System S1 System S2
Diathermal wall
pvacuum = patm – pabs = – pgauge ...(1.13)

System S3 Isolation from


surroundings

Fig. 1.29

Temperature can be defined as a measure of hotness S1 and S2 will also be in thermal equilibrium with
or coldness, but it is not the exact definition. each other, even though they are not in contact.
Temperature is a basic property, such as mass, length The zeroth law serves a basis for the validity of
and time thus cannot be defined precisely.Based on temperature measurement, by replacing the third
our sensation, we express the level of temperature system by a thermometer. The zeroth law can be
qualitatively with words like ‘cold’, ‘freezing cold’, restated as two systems are in thermal equilibrium
‘warm’, ‘hot’ and ‘red hot’. However, a numerical if both have the same temperature reading even if
value cannot be assigned to these feelings, because they are not in contact.
our sensations may be misleading. For example, a
metal chair will feel much colder than a wooden
one, even when both are at the same temperature.
The measurement of temperature depends upon
the establishment of thermal equilibrium between
a system and the device used to measure the
If a hot system and cold system are brought into temperature. The sensing device should have at
contact with each other, isolated from their sur- least one measurable property that changes with
roundings, the hot system gives its heat energy to change in temperature. Such a property is called a
the cold system till they reach a common tempera- thermometric property. The substance which shows
ture, which is the requirement for thermal equilib- the changes in the thermometric property is called
rium. Two systems attain equal temperature if no thermometric substance.
changes occur in any property when they remain
Fortunately, several properties of materials
in contact.
change with temperature and this forms the basis
for temperature measurement. A list of some
1.13.3
properties is given below.
It states that when two systems are in thermal
(i) Change in dimension Expansion or
equilibrium with a third system, they in turn have
contraction of material, such as mercury in
thermal equilibrium with each other.
glass thermometer.
Consider two systems S1 and S2 which are
(ii) Change in electrical resistance of metals
separated by an adiabatic wall, and a third system
and semiconductors, such as resistance
S3 is in communication with both the systems as
thermometers and thermistors.
shown in Fig. 1.29.
(iii) Thermoelectric emf between cold and hot
If systems S1 and S2 are individually in thermal
junctions, such as thermocouples.
equilibrium with a third system S3, then the systems
(iv) Change in intensity and colour of emitted
radiations, such as pyrometers
Basic Concepts 17

The above properties are calibrated into


corresponding temperatures through comparison The schematic diagram of a constant-volume gas
with established standards. The commonly used thermometer is shown in Fig. 1.31. The volume of
thermometers are an ideal gas in the sensing bulb D is kept constant
(a) Liquid-in-glass thermometer, and by adjusting the level of mercury in the arm B
(b) Constant volume gas thermometer. of the manometer. The arm B and the arm A are
connected by a flexible tube to form a U-tube
manometer. The arm B is also connected to the gas
bulb D via a capillary tube C, while the other arm A
The most commonly used thermometer is the
of the manometer is open to atmosphere and can be
liquid-in-glass type as shown in Fig. 1.30. It works
moved vertically to adjust the mercury level, so the
on the expansion or contraction of a thermometric
mercury just touches the mark L of the capillary.
substance with temperature. It consists of a uniform-
The pressure in the bulb B is used as a thermometric
diameter glass capillary tube connected to a bulb
property and can be given by
filled with a liquid at one end. The assembly is
sealed to preserve a partial vacuum in the capillary. p = patm + r g h ...(1.14)
As the temperature increases, the liquid expands in where, patm = atmospheric pressure
volume and rises in the capillary. r = density of the mercury
h = mercury column in manometer
The gas bulb D is first placed in a constant-
temperature bath at the triple point temeprature
Ttp of water and the level of mercury is adjusted to
touch the mark L by moving the manometer arm A
up and down. As the volume of the bulb becomes

Fluid in

Fig. 1.30

The height of the liquid column is calibrated


into a temperature scale, which may then be read.
Mercury is widely used for measuring ordinary
temperatures; alcohol, ether, and other liquids are
also employed for this purpose. Fig. 1.31
18 Thermal Engineering

constant and the height difference of the mercury applications to monitor temperature of liquids and
in the two arms is recorded as htp, the pressure, ptp gases in storages and flowing pipes and ducts.
corresponding to the mercury column at the triple The commonly used thermocouple materials are
point is calculated by Eq. (1.14). tabulated below.
Now the bulb is brought in contact with a system
whose temperature T, is to be measured. Again, in a
similar manner, by keeping the volume of gas in the
bulb constant, the height difference of the mercury
in the two arms is recorded and the corresponding Materials Type Range Range
new pressure p is calculated by Eq.(1.14). 1 Cooper constantan T −220°C to ± 0.75%
From the ideal gas equation, the new temperature 371°C
is given by 2 Iron constantan J −190°C to ± 1.0%
760°C ± 0.75%
p 3 Chromel–Alumel K −190°C to
T = 273.15 ¥ ...(1.15)
ptp 1260°C ± 0.75%
where 273.15 K is the triple point temperature of 4 Chromel– E − 100°C to
constantan 1260°C
water.
5 Platinum (90% ) 0°C to
+ rhodium (10%,) S 1482°C ± 0.5%

The thermocouple works on the principles of


Seebeck effect, Peltier effect, and Thomson effect. Inexpensive and has high output.
When two dissimilar metals are joined together as Inexpensive; iron oxidizes rapidly
shown in Fig. 1.32, an emf will exist between the above 760°C.
two points A and B. The Seebeck emf is caused by
Good resistant to oxidation
the junction of dissimilar metals, the Peltier emf
within specified temperature limit. It should not be
is caused by flow of current in the circuit and the
used in reducing atmosphere.
Thomson emf is caused by the temperature gradient
existing in the material. The emf generated at the Highest output with good
junction is the function of temperature of two stability.
dissimilar metals. This emf generated is calibrated
Thermocouple is expensive,
in temperature scale for temperatre measurement.
low output, but most accurate. It is very stable
The temperature is measured by a thermocouple
and resistant to oxidation. It is used only for high
by inserting the probe into a stream of hot gas,
temperature application.
hot liquid or attaching it to a hot solid. The
thermocouples are widely used in industrial
External
circuit Several temperature scales have been introduced
Material 1
A so far. All temperature scales are based on easily
i
reproducible states, such as freezing and boiling
Junction
point temperatures of water, which are also called
B ice point and steam point, respectively.
Material 2
The temperature scale used in SI units is Celsius
Fig. 1.32 scale (formerly, Centigrade scale). On the Celsius
Basic Concepts 19

373.15

671.67
Further, it should be noted that the temperature

100.0
K °C °R °F

212
Steam magnitude of each division of 1 K and 1°C are
point
identical. Therefore, when we are dealing with
temperature difference, the temperature scale on
Ice point 0.00
both the scales is same.

32.0
491.67
273.15

DT (K) = DT (°C) ...(1.20)


Similarly, DT (R) = DT (°F) ...(1.21)

Fahrenheit
Rankine
Celsius

If a relation involves temperature difference


Kelvin

(such as q = CpDT), it makes no difference and


either scale can be used. But when the relation is in

–459.67
–273.15

temperature only (as p = RT ), then the temperature


0.00

00.0

Absolute in kelvin scale (K) must be used.


zero

Fig. 1.33

scale, ice point and steam point are assigned the In the seventh General Conference on Weights and
numerical values of 0 and 100°C, respectively. The Measures held in 1927, a more convenient scale,
English system today uses the Fahrenheit scale known as International Practical Temperature
which assigns 32 and 212°F as the ice point and Scale was formulated to be used for calibration of
steam point, respectively. temperature-measuring instruments. It was revised
A more useful temperature scale in thermody- in the Thirteenth General Conference in 1968. It
namics is the absolute temperature scale (no nega- consists of reproducible reference temperatures,
tive temperature is possible on this scale). This defined by triple point, boiling point and melting
scale is also called the Kelvin scale. The temper- point of pure substances.
ature unit on this scale is kelvin designated as K, It is a state of equilibrium, where all
without the degree symbol. three phases (solid, liquid and gas) of a substance
The Kelvin scale is related to the Celsius scale coexist simultaneously. The triple point of water is
by 0.01°C (271.16 K).
T (K) = T (°C) + 273.15 ...(1.16)
It is a state of equilibrium, where
In the English system, the absolute temperature liquid and gaseous phases of a substance coexist
scale is the Rankine scale and it is related to simultaneously. For water, it is 100°C (373.15) at
Farhenheit scale by 1 standard atm.
T (R) = T (°F) + 459.67 ...(1.17)
It is a state of equilibrium, where
The two temperature scales and corresponding
solid and liquid phases of a substance coexist
absolute temperature scales are shown in Fig. 1.33.
simultaneously. More often, all metals have their
The values 273.15 and 459.67 are often replaced by
specific melting point.
approximate values of 273 and 460, respectively.
The temperature scales on two unit systems are The whole temperature scale is divided into four
related by ranges:
T (R) = 1.8 T (K) ...(1.18) (i) From −259.34°C (triple point of hydrogen)
T (°F) = 1.8 T (°C) + 32 ...(1.19) to 0°C
20 Thermal Engineering

Solution

Given Absolute pressure at triple point


Equilibrium state Assigned value of
temperature ptp = 752 + 32 = 784 mm of Hg

K °C Absolute pressure at given pressure,


1. Triple point of 13.81 –259.34 p = 752 + 76 = 828 mm of Hg
hydrogen Triple point temperature ,
2. Normal boiling point 20.28 –252.87
Ttp = 273.15 K
of hydrogen
3. Normal boiling point 27.10 –246.05 To find Temperature T at pressure p.
of neon
4. Triple point of 54.36 –218.79 Analysis For constant volume gas thermometer:
oxygen ptp p
Normal boiling point 90.19 –182.96 =
5. Ttp T
of oxygen
Triple point of water 273.16 0.01 p 828
6. or T = ¥ Ttp = ¥ 273.15
7. Normal boiling point 373.15 100.00 ptp 784
of water = 288.48 K
8. Normal freezing 630.74 357.59
Unknown temperature,
point of antimony
Normal freezing 692.73 419.58 T = 288.48 − 273.15 = 15.33°C
9.
point of zinc
Calculate the temperature at which the
10. Normal freezing 1235.08 961.93
Celsius and Fahrenheit scales agree.
point of silver
11. Normal freezing 1337.58 1064.43 Solution
point of gold
Given Condition for two temperature as T (°F) = T (°C)
(ii) From 0°C to 630.74°C (antimony point), To find Temperature at which the Celsius and Fahrenhiet
(iii) From 630.74°C to 1064.43°C (gold point) scales becomes equal.
(iv) Above 1064.43°C Analysis The temperature at the two scales is related as
T (°F) = 1.8 T (°C) + 32
Using the given condition T (°F) = 1.8 T (°C) = T say
It is an abstract property of the second law of T = 1.8 T + 32
thermodynamics. It is considered as a measure of or 0.8 T = −32
degree of the molecular disorder in the matter. The
or T = − 40°C
change in value is calculated as
2 dQ 2 dT Example 1.5 The resistance of the windings in a cer-
DS = Ú1 T
= Ú
1
mC
T
...(1.22) tain motor is found to be 100 ohms at a room temperature
of 27°C. When operating at full load under steady condi-
tions, the motor is switched off and the resistance of the
Example 1.3 The pressure in a constant gas ther-
winding is found to be 120 ohms. The windings are made
mometer is measured as 32 mm of Hg above atmospheric
of copper wire, whose resistance at t°C is given by
pressure at triple point. Determine the temperature in °C,
when the pressure is 76 mm of Hg above atmospheric Rt = R0 (1 + 0.004 t)
pressure. The barometer reads 752 mm of Hg.
Basic Concepts 21

where R0 is the resistance at 0°C. Find the temperature Using A and B for thermometric property at 3.5,
attained by the coil during full load. t = 62.13 ¥ ln (3.5) − 25.2 = 54.64°C

Solution Two Celsius thermometers A and B


Given Resistance, R1 = 100 W, with temperature readings TA and TB agree at ice point
Temperature, t1 = 27°C and steam point, but else where they are related by
Resistance, R2 = 120 W TA = p + qTB + rTB2, where p, q and r are constants.
To find The temperature at full load When the thermometers are immersed in an oil bath,
A shows a temperature of 51°C, while B shows 50°C.
Analysis The relation between resistance at any
Determine the temperature TA, when TB is 25°C.
temperature is given by
Rt = R0(1 + 0.004t) Solution
Using values at the state 1
Analysis The relation for temperature is given as
100 = R0 (1 + 0.004 ¥ 27)
TA = p + qTB + rTB2
or R0 = 90.25 W At ice point, 0 =p+q¥0+r¥0
The temperature corresponds to 120 W; or p =0
120 = 90.25 (1 + 0.004t2) At steam point, 100 = p + 100 q + (100)2 r
or t2 = 82.4°C or 100 = 100q + 10000 r ...(i)
Further, 51 = p + 50 q + (50)2 r
Example 1.6 The temperature scale of a certain ther- or 51 = 50q + 2500 r ...(ii)
mometer is given by the relation t = Aln p + B, where A Solving these equations, we get,
and B are constants and p is the thermometric property
q = 1.04 and r = − 4 ¥ 10 − 4
of the fluid in thermometer. At ice point and steam point,
if the thermometric property is found to be 1.5 and 7.5 Using p, q and r at TB = 25°C,
respectively, what will be the temperature corresponding TA = 0 + 1.04 ¥ 25 − 4 ¥ 10 −4 ¥ (25)2 = 25.75°C
to the thermometric property of 3.5 on celsius scale?
A thermocouple with a test junction at
Solution t°C on a gas thermometer scale gives the emf as
Given Initial property, p1 = 1.5, e = 0.22 t − 5.5 ¥ 10−4 t2 mV
Final property, p2 = 7.5, The milivoltmeter is calibrated at ice point and steam
Ice-point temperature, T1 = 0°C, points. What will be the reading on this thermometer,
Steam-point temperature, T2 = 100°C, when the gas thermometer reads 60°C?
Thermometric property, p3 = 3.5
To find The temperature corresponding to the thermo- Solution
metric property of 3.5 Given The relation for emf at t°C
Analysis The relation for temperature is given as e = 0.22t − 5.5 ¥ 10−4 t 2 mV
t = Aln p + B, with ice point, t1 = 0°C
At ice point, 0 = A ln (1.5) + B Steam point, t2 = 100°C
or 0 = 0.405 A + B ...(i) To find The reading of temperature on the scale cor-
At steam point, 100 = A ln (7.5) + B responding to 60°C
or 100 = 2.015 A + B ...(ii)
Analysis The emf is generated according to
Solving these equations, we get,
e = 0.22t − 5.5 ¥ 10 −4 t2 mV
A = 62.13 and B = − 25.2
22 Thermal Engineering

at ice point e1 = 0.22 ¥ 0 − 5.5 ¥ 10 −4 ¥ (0)2 = 0 mV Substituting the value of constants a and b in
at steam point e2 = 0.22 ¥ 100 − 5.5 ¥ 10−4 ¥ (100)2 Eq. (i), we get
= 16.5 mV 100 100
at 60°C e3 = 0.22 ¥ 60 − 5.5 ¥ 10−4 ¥ (60)2 T(°C) = p- pi
ps - pi ps - pi
= 11.22 mV
The reading of the temperature on the gas thermometer Ê p - pi ˆ
= 100 Á
scale corresponds to 60°C. Ë ps - pi ˜¯
(100∞C)
t= ¥ (11.2 mV) = 68°C T (°C) Ê p - pi ˆ
(16.5 mV) or = Á ...(iv)
100 Ë ps - pi ˜¯
Example 1.9 It is proposed to construct a new scale
Similarly, on the new scale °N at steam and ice
with the value 5°N assigned to ice point and 20°N to
points, with constants c and d;
steam point. The pressure of an ideal gas at constant
volume is considered as a thermometric property. 20 = c ps + d ...(v)
5= cpi + d ....(vi)
(a) Set up a linear relationship between pressure and
temperature in °N on a new scale. What is the Substracting Eq. (vi) from Eq. (v), we get
kelvin absolute zero on this scale? 15
(b) Derive an expression between °N and K. c =
ps - pi
Solution 15
and constant d = 5- ¥ pi
Given The construction of a new scale with °N. ps - pi
Ice-point temperature, Ti = 5°N and 0°C, Substituting the value of constants c and d in
Steam-point temperature, Ts = 20°N and 100°C, Eq. (i), we get
To find (a) (i) Linear relationship between pressure 15 15
T (°N) = p + 5- pi
and temperature on a new scale, °N, ps - pi ps - pi
(ii) Absolute zero on new scale.
Ê p - pi ˆ
(b) Relationship between new scale in °N and = 5 + 15 Á
kelvin scale. Ë ps - pi ˜¯
T (°N) - 5 p - pi
Analysis or = ...(vii)
15 ps - pi
(a) (i) Linear relationship between pressure and tem-
Equating Eq. (iv) and (vii), we get
perature with two constants a and b as
T = ap + b ...(i) T (°N) - 5 T (°C)
=
Assuming pressure ps at steam point and pi at 15 100
ice point, then on Celsius scale; 15
or T(°N) = T (°C) + 5 ...(viii)
100 = aps + b ...(ii) 100
0= a pi + b ....(iii)
It is the linear relationship between celsius and
Subtracting Eq. (iii) from Eq. (ii), we get the new temperature scale on °N.
100 = a(ps − pi) (ii) At absolute zero on Celsius scale
100 0 K = −273°C
or a =
ps - pi 15
Therefore, T (°N) = (- 273) + 5
100 100
and constant b = - pi
ps - pi = −35.95°N
Basic Concepts 23

(b) Relationship between new scale and Kelvin scale Subtracting (ii) from (i), we get
T (K) = T(°C) + 273 100
a =
or T(°C) = T (K) − 273 pi - ps
100
Using in Eq. (viii); and b = 100 - pi
pi - ps
15
T (°N) =
100
[T (K) - 273] + 5 Using this, we get
100 100
t¢ = p + 100 - pi
100 pi - ps pi - ps
or T (K) = [T (°N) - 5] + 273
15 Ê p - pi ˆ
= 100 ¥ Á + 100
20 Ë pi - ps ˜¯
or T (K) = 273 + [T(°N) - 5] p - pi
3 t ¢ - 100
or = ...(A)
pi - ps 100
Example 1.10 Consider a particular Celsius scale
Similarly, using 0°C for ti and 100°C for ts on
assigned the value of 0°C to steam point and 100°C to
normal temperature scale with constants c and d;
ice point.
(a) Using ideal gas as the thermometer medium, 0 = cpi + d ...(iii)
set up a relationship between 0°C and pressure 100 = cps + d ...(iv)
for a constant volume thermometer. Proceed to Subtracting (iii) from (iv),
derive the correlation between the two celsius 100 100
scales. At what temperature are the two scales c= and d=− pi
ps - pi ps - pi
are numerically equal?
(b) What is the numerical value of obsolute zero for Then normal temperature scale
the particular scale? What is 200 K in °C? 100 100
t = ¥ p- ¥ pi
ps - pi ps - pi
Solution Ê p - pi ˆ
= 100 ¥ Á
Given Two Celsius scales. Ë ps - pi ˜¯
t p - pi
To find (a) (i) Correlation between the two celsius or = ...(B)
scales, 100 ps - pi
(ii) Temperature at which two scales are Equating (A) and (B), we get
numerically equal, t t ¢ - 100
=
(b) (iii) Numerical value of absolute zero, 100 100
(iv) Temperature in °C corresponding to or two Celsius scales are related by
200 K. t = t¢ − 100
(ii) If t = t¢
Analysis then t = 100 − t
(a) (i) Let the temperature scale relate the pressure or 2t = 100
linearly: or t = 50°C
t ¢ = ap + b (b) (iii) At absolute zero
where p is thermometric property, and a and b t = −273°C
are two constants. −273 = 100 − t¢
Then for a constant-volume gas thermometer or t¢ = 373°C
100 = api + b ...(i) (iv) At 200 K, t = 200 − 273 = −73°C
0 = aps + b ...(ii) Therefore t¢ = 100 − t = 100 − (−73) = 173°C
24 Thermal Engineering

Summary
from one state to another is called a process. A
with energy and its transformation. process with identical states at the end and start
is called a cycle. During a quasi-equilibrium, the
certain fixed region in space. A system of fixed system remains practically in equilibrium at all
mass is called a closed system or control mass, times.
and a system that involves the mass transfer Pressure is defined as force per unit normal area.
across its boundary is called an open system. It is measured in Pascal, kPa, bar. The absolute,
property is called an gauge and vacuum pressure are related as
extensive property, while a property independent pgauge = pabs − patm
of mass is called an intensive property. and pvacuum = patm − pabs = −pgauge

called total energy, which consists of internal, two bodies are in thermal equilibrium, if both
kinetic and potential energy. Internal energy have a common temperature even is they are not
represents the molecular energy of the system in contact.
and may exist in sensible, latent, chemical and unit system are
nuclear form. Celsius and Kelvin scales. These are related as
T(K) = T(°C) + 273.15
equilibrium, if it maintains thermal, mechanical, DT(K) = DT(°C)
phase and chemical equilibrium. Any change

Glossary
Thermodynamics Science of energy that deals with Homogeneous system A system containing only a
heat energy and power single physical phase of a substance
System Certain quantity of matter or prescribed region Heterogeneous system A system of mixture of two or
in space considered for analysis more than two phases of matter
Surroundings Every thing external to a system Macroscopic Gross or overall behavior
Boundary Real or imaginary surface that separates a Microscopic Average behavior of molecules making up
system from its surroundings a system
Universe Combination of a system and its surroundings Thermodynamic State Condition of system described
Closed system Particular quantity of matter under study by independent thermodynamic properties
Isolated system A closed system that does not interact Properties A characteristic of a system
with its surroundings Intensive property Any property, which is independent
Open system Prescribed region in space under study of mass and size of the system
Adiabatic system An open system in which heat is not Extensive property Any property, which depends on
allowed to cross the boundary mass and size of the system
Physical phase A molecular configuration of matter Equilibrium State of balance
categorized as either solid, liquid or gas Thermal equilibrium A situation in which the system
Working substance A medium for energy transport does not have variation in temperature throughout
between a system and its surroundings, while undergoing Mechanical equilibrium A situation in which all
a thermodynamic process within a device mechanical forces within the system are balanced
Basic Concepts 25

Phase equilibrium A system without phase change Quasi-static process A process which is always close
Chemical equilibrium A situation in which a system to thermodynamic equilibrium
does not interact chemically Pressure Force acting per unit area of a fluid
Process Transformation of a system from one Internal energy Sum of all microscopic forms of
equilibrium state to another energies of a system
Cycle Sequences of processes that begins and ends at
the same state

Review Questions
1. Define thermodynamics and write its importance 12. State and differentiate between extensive,
and applications. intensive and specific properties.
2. How does classical thermodynamics differ from 13. What do you mean by thermodynamic
statistical thermodynamics? equilibrium? How does it differ from thermal
3. Explain the concept of macroscopic and equilibrium?
microscopic view points applied to the study of 14. Define thermodynamic equilibrium. What are the
thermodynamics. conditions necessary to establish thermodynamic
4. What is meant by classical and statistical equilibrium to a system?
thermodynamics ? Explain. 15. Define quasi-static process. State its salient
5. Define thermodynamic systems. Classify them. features.
6. Differentiate between closed system and open 16. State different types of processes according to
system. their natures.
7. Define isolated and adiabatic systems and 17. Define state function and path function.
differentiate them. 18. What is the concept of continuum? How are
8. Recognize whether the system is open or closed: density and pressure defined using this concept?
(i) A tube of a bicycle filled with air, 19. Define zeroth law of thermodynamics. Write its
importance in thermodynamics.
(ii) A jet engine in flight,
(iii) A household refrigerator in operation, 20. Define temperature. How is it measured?
(iv) Water pump, 21. State and explain zeroth law of thermodynamics
(v) Pressure cooker without whistling, used for temperature measurement.
(vi) Steam turbine, 22. What do you mean by thermodynamic property
(vii) Car battery, and thermodynamic substance?
(viii) An electric geyser. 23. Establish a relationship between Celsius scale
9. Define control volume and control surface. and Fahrenheit scale.
10. Differentiate between homogeneous and hetero- 24. What do you mean by absolute scale of
geneous systems. temperature? How does the Celsius scale differ
from the absolute Kelvin scale?
11. Define thermodynamic property, state, path,
process and cycle. 25. State the operating principle of gas thermometer.
Explain its working.
26 Thermal Engineering

Problems
1. Convert the following temperatures from °C to (c) Find a relation between the new scale and
°F: Kelvin scale.
(a) 21°C (b) − 17.78°C (d) If the pressure at the steam point is 10 atm,
(c) − 50°C (d) 300°C what is the pressuire at 0 degree at new ?
(e) 100°C
T ( °C) T( ° N) – 4 p – pi
2. Convert the following temperatures from °F to [(a) = =
100 12 ps – pi
°C:
(b) – 28.8°N (c) T(K) = 273
(a) 212°F (b) 68°F
(c) 32°C (d) − 40°F + 25
3 (T ( °C) – 4 ) (d) – 24.3°N]
(e) − 460°F
3. The resistance of a platinum wire is found to be 5. A new absolute temperature scale is proposed
11.00 W at ice point, 15.247 W at steam point and with ice point as 150°S and steam point as 300°S.
27.949 W at the zinc point (692.73 K). Find the Determine the temperature in °C that corresponds
constants in the equation to 100°S and 400°S, respectively.
R = R0(1 + AT + BT 2) where T is in °C 6. A tank which is 4 m long, 3 m wide and 2 m deep
Find the resistance at a temperature of 250°C. is half full of water. How much work is required
[A = 3.915 ¥ 10–3, B = 5.959 ¥ 10–7, 21.36 W] to raise all the water over the top edge of the
4. It is proposed to develop a temperature scale with tank? [117.72 kJ]
values 4°N and 16°N assigned as ice and steam 7. A rectangular tank measuring 0.6 m ×1 m at the
points, respectively. The pressure of an ideal gas base is filled half to a depth of 15 cm with water.
at constant volume is used as a thermometric Calculate the following:
property. (a) Total gravitational force exerted on the base
(a) Obtain a linear relationship between the of the tank
pressure and temperature. (b) The pressure exerted by water at the base of
(b) What is Kelvin absolute zero on this scale? the tank

Objective Questions
1. The unit of force in SI units is 4. 1 pascal in SI units is
(a) joule (b) newton (a) 1 N/m² (b) 100 kPa
(c) watt (d) calorie (c) 1 ¥ 105 N/m² (d) 1 ¥ 103 N/m²
2. The unit as kN/m2 is called 5. A closed system is one in which
(a) kW (b) kJ (a) both energy and mass cross the boundary of
(c) kPa (d) kcal the system
3. 1 bar in SI units is (b) the mass does not cross the boundary, but
(a) 1 ¥ 105 Pa (b) 100 kPa energy interaction takes place
(c) 0.987 atm (d) All of above. (c) neither mass nor energy cross the boundary
of the system
Basic Concepts 27

(d) the mass crosses the boundary but energy 9. Which of the following is an intensive property?
does not (a) Volume (b) Temperature
6. An isolated system is one in which (c) Energy (d) Work ransfer
t
(a) both energy and mass cross the boundary of 10. Which of the following is not a property?
the system (a) Volume (b) Temperature
(b) the mass does not cross the boundary, but (c) Energy (d) Work ransfer
t
energy interaction takes place 11. Which of the following is an extensive property?
(c) neither mass nor energy cross the boundary (a) Volume (b) Temperature
of the system (c) Pressure (d) Density
(d) the mass crosses the boundary but energy 12. Mercury in glass thermometer works on the
does not principle of
7. An open system is one in which (a) fusion
(a) both energy and mass cross the boundary of (b) thermo-electric effect
the system (c) expansion of fluid
(b) the mass does not cross the boundary, but (d) change in radiation intensity.
energy interaction takes place 13. A 70-kg woman walks on snow with a total foot
(c) neither mass nor energy cross the boundary imprint area of 500 cm². What pressure does she
of the system exert on the snow?
(d) the mass crosses the boundary but energy
(a) 0.5 kPa (b) 12.5 kPa
does not
(c) 13.73 kN/m² (d) 25.46 kN/m²
8. An adiabatic system is one in which
14. When two bodies are in thermal equilibrium with
(a) both energy and mass cross the boundary of a third body, then they are in thermal equilibrium
the system with each other. This statement is called
(b) the mass does not cross the boundary, but
(a) first law of thermodynamics
energy interaction takes place
(b) second law of thermodynamics
(c) neither mass nor energy cross the boundary
(c) third law of thermodynamics
of the system
(d) zeroth law of thermodynamics.
(d) mass crosses the boundary, heat energy
15. A sequence of processes, in which initial and
does not cross the boundary of the system
final states of a system are identical, is called a
(a) path function (b) point function
(c) cycle (d) none of the above

15. (c) 14. (d) 13. (c) 12. (c) 11. (a) 10. (d) 9. (b)
8. (d) 7. (a) 6. (c) 5. (b) 4. (a) 3. (d) 2. (c) 1. (b)
Answers
28 Thermal Engineering

2
Energy and Work Transfer

Introduction
Energy is one of the major inputs for the economic development of any country. In the case of developing
countries, the energy sector assumes a critical importance in view of the ever-increasing energy needs
requiring huge investments to meet them.
Work is a high-grade energy and of prime interest as it is the output from a system, when energy is the
input. The rate of work transfer is referred as power. High power generation is the need of any developing
country.
Sources and forms of energy, enthalpy, forms of work transfer, concept of thermodynamic work transfer,
heat, specific heat, sign convention for work and heat transfer are discussed in this chapter as a foundation
to the following chapters. The first law of thermodynamics is explained with the help of Joule’s experiment
and other examples.

ENERGY SOURCES OF ENERGY

Energy is defined as the capacity to do work. It is a The sources of energy can be divided into four
scalar quantity. It is measured in kJ in SI units, and categories according to their availabilty:
kcal in MKS units. Energy can have many forms as
(i) Transitional Energy The energy in motion, i.e.,
shown in Fig. 2.1.
wind energy, hydel energy, etc.

Transitional Capital Celestial Stored


energy energy energy energy

Energy

Electrical Mechanical Chemical Nuclear Thermal Electromagnetic


energy energy energy energy energy energy

Fig. 2.1
Energy and Work Transfer 29

(ii) Capital Energy The energy derived from fuels Non-commercial forms of energy are not avail-
existing in the earth, i.e., fossile fuels, nuclear able in the commercial market for a definite price.
fuels, etc. Non-commercial energy sources include fuels such
as firewood, cattle dung and agricultural wastes,
(iii) Celestial Energy The energy coming from which are traditionally gathered, and not bought
outer atmosphere, i.e., sun, moon, etc. at a price, and used especially in rural house-
(iv) Stored Energy The energy existing in various holds. These are also called traditional fuels. Non-
masses, i.e., flywheel, tides, geothermal, hydraulic commercial energy is often ignored in energy ac-
energy, etc. counting.

2.3 CLASSIFICATION OF ENERGY Energy


SOURCES
Renewable energy is obtained from sources that are
The sources and forms of energy can be classified not exhaustible. These are freely available in nature
into several types based on the following criteria: and can be continuously used.
Examples of renewable resources include wind
power, solar power, geothermal energy, tidal power,
ocean thermal energy, fuel cells, energy from bio-
mass and hydraulic energy. The most important
Energy feature of renewable energy is that it can be har-
nessed without the release of harmful pollutants.
Primary forms of energy are those that are either
Non-renewable energy is obtained from conven-
tional fossil fuels such as coal, oil and gas, nuclear
sources are coal, oil, natural gas, and biomass (such
fuels, and heat traps which are accumulated in the
as wood). Other primary energy sources available
earth crust. These have been in use for several de-
include nuclear energy from radioactive substanc-
cades. The sources of non-renewable energy are
es, thermal energy stored in the earth’s interior
depleting at a fast rate and may not be sufficient
(geothermal energy), and potential energy due to
to meet the inceasing energy demand in future.
the earth’s gravity.
Therefore, these sources are also called exhaustible
Secondary forms of energy are those forms sources of energy and they cannot be replenished
which are derived from the primary forms of en- immediately.
ergy; for example, coke, oil or gas converted into
steam and electricity. R
Non-R Energy S
Energy and Non-
Energy Renewable sorces of energy
These are inexhaustible.
Commercial forms of energy are available in the
market for a definite price. The most important These are freely available.
forms of commercial energy are electricity, coal These are environment-friendly.
- Energy concentration varies from region to
gy serves the basis of industrial, agricultural, trans- region.
port and commercial development in the modern
world. high, but maintenance and operational costs are
minimum.
30 Thermal Engineering

elevation in a gravitational field is called potential


be designed. energy (PE ) and is expressed as
Non-renewable sources of energy PE = mgz ( joules) ...(2.1)
These are exhaustible. on unit mass basis pe = gz (J/kg) ...(2.2)
These are available at definite prices. where g is the acceleration due to gravity and z is
the elevation of the system relative to some outside
Energy concentration does not vary. reference.

Energy
are high.
- The energy that a system possesses, as a result of
rily. motion relative to some reference is called kinetic
energy (KE ). When all parts of a system move with
ENERGY the same velocity, the kinetic energy is expressed as
1
Energy can exist in numerous forms, such as inter- KE = mV2 ( joule) ...(2.3)
2
nal, thermal, electrical, mechanical, kinetic, poten- 1
tial, wind, and nuclear energy, on unit mass basis ke = V2 (J/kg) ...(2.4)
2
In thermodynamic analysis, all forms of energy where V is the velocity of the system with respect
can be put into two groups: to some reference.
(a) Stored energy, and
(b) Transit energy. Energy

(a) Stored The stored form of energy can The sum of all the microscopic forms of energy
further be classified as is called internal energy. The internal energy
of a system is the energy stored within the body
(i) Macroscopic forms of energy: potential en- resulting from the kinetic and potential energy
ergy and kinetic energy, and of its molecules. Thus, it is related to molecular
(ii) Microscopic forms of energy: internal en- structure and degree of molecular activities. The
ergy. molecules of any system may possess both kinetic
The macroscopic forms of energy are defined as and potential energy. Thus, the internal energy of
energy with respect to some outside reference. any system may be viewed as the sum of kinetic
The microscopic forms of energy are those and potential energy of molecules. It is denoted by
which are related to the molecular structure of a U and is measured in joules.
system and degree of molecular activities and are U = K + P ( joules) …(2.5)
independent of the outside reference. where K = internal kinetic energy of molecules,
and
(b) Transit Transit energy means energy in
P = internal potential energy of molecules.
transition. It is the energy possessed by a system,
which is capable of crossing the boundaries. Heat (a) Internal Kinetic Energy All molecules in a
energy and work transfer are transit forms of energy. system move around with some velocity, vibrate
about each other, and rotate about an axis during
Energy their random motion as shown in Fig. 2.2. The
internal kinetic energy is the sum of all these
The energy that a system possesses as a result of its
motions of molecules. When energy passes into a
Energy and Work Transfer 31

Energy
Mechanical energy can be defined as a form of en-
ergy that can be converted directly and completely
into mechnical work by an ideal mechanical device
such as an ideal turbine or pump. The kinetic and
potential energies are the common forms of me-
Molecular translation Molecular rotation chanical energy. Thermal (heat) energy is not a
form of mechanical energy since it cannot be con-
verted to work directly and completely.
An ideal turbine extracts mechanical energy
from a flowing fluid by reducing its pressure, while
a pump transfers mechanical energy to a fluid by
raising its pressure. The pressure force acting on
a fluid through a distance produces flow work ( pv
per unit mass). It is the energy of a flowing fluid,
Molecular vibrations Molecular spins and is thus called flow energy. Therefore, the me-
chanical energy of a flowing fluid on a unit mass
basis is viewed as
system, it increases the motion of molecules, thus V2
emech = pv + + gz ...(2.6)
the internal kinetic energy of system is increased, 2
and this change is reflected by an increase in V2
temperature of the system. Sometimes, kinetic where pv is the flow energy, is the kinetic en-
2
internal energy of molecules is referred as sensible ergy and gz is the potential energy of unit mass of
energy. The average velocity and the degree of a fluid. It can also be expressed in the rate form as
activities of molecules are proportional to the
Ê V2 ˆ
temperature of a gas. Thus, at higher temperature, a Emech = m Á pv + + gz ˜ ...(2.7)
system will possess higher internal energy. Ë 2 ¯
where m is the mass flow rate of fluid. The me-
(b) Internal Potential Internal potential
chanical energy change of a fluid is
energy of a system is the energy of molecular sepa-
ration. It is the energy that the molecules have as Ê V22 - V12 ˆ
Demech = ( p 2v2 – p1v1) + Á ˜
a result of their position in relation to one another. Ë 2 ¯
The greater the degree of molecular separation, the
greater is the internal potential energy. + g(z2 – z1) ...(2.8)
When a system expands or changes its physical and D Emech
state with addition of energy, a rearrangement of È Ê V 2 - V12 ˆ ˘
molecules takes place that increases the mean dis- = m Í( p2 v2 - p1v1 ) + Á 2 ˜ + g ( z2 - z1 ) ˙
ÍÎ Ë 2 ¯ ˙˚
tance between them. An internal work is required
to pull the molecules against the forces of attraction ...(2.9)
between them. An amount of internal potential en- In the absence of any losses, the mechanical en-
ergy equal to the amount of internal work done for ergy change represents the mechanical work sup-
rearrangement of molecules is called latent energy plied to the fluid (if Demech > 0); or extracted from
or latent heat. the fluid (if Demech < 0). The shaft work, spring
work, aceleration work, gravitational work, work
32 Thermal Engineering

done on a solid elastic bar are also some mechani- Q


cal forms of work. q= (kJ/kg) ...(2.13)
m
In electrical work, the force is voltage (potential
difference) and displacement is electrical current;
in magnetic work, the force is the magnetic field The heat or heat energy is generally referred as
strength and displacement is magnetic dipole mo- heat transfer. The transfer of heat into a system is
ment and are not mechanical forms of work. called heat addition or heat supply and the transfer
of heat from the system is called heat rejection. It

example, a hot potato rejects its internal energy as


The sum of the internal energy U and the product heat at its boundary to its surroundings as shown in
of pressure p and volume V appears frequently in Fig. 2.3(b).
many thermodynamic analyses. Therefore, it is
Heat
convenient to give a name to this combination, en-
thalpy. It is also called total enthalpy and is desig-
nated by H. By definition, Baked potato
at 100°C Surroundings
H = U + pV ...(2.10) at 30°C

Since U, p and V all are properties, the enthalpy


is also a property of the system. It is measured in (a) Heat transfer by virtue of temperature difference
units of internal energy, i.e., kJ in SI units. The
enthalpy for unit mass system is referred as specific 2 kJ
enthalpy and is denoted by h (kJ/kg),
2 kJ
h = u + pv ...(2.11) Baked
potato
The enthalpy per mole basis is expressed as 2 kJ
at 100°C

h = u + pv ...(2.12)
It should be noted that the enthalpy is a com-
bination of other properties and it is not a form of (b) Energy as heat crossing the boundary
energy. Fig. 2.3

A process during which there is no heat transfer


is called an adiabatic process as shown in Fig. 2.4.
It is a transfer form of energy that flows between In an adiabatic process, energy content and the tem-
two systems (or a system and its surroundings) by perature of a system can be changed by other pro-
virtue of the temperature difference between them.
cesses, such as work.
The temperature difference is the potential for heat
transfer. There would be no heat transfer between Control surface
two systems if they are at the same temperature.
The amount of heat transferred from the state 1
System Q=0
to the state 2 is designated Q1–2 or Q and it is mea-
sured in joules (J) or kilojoules (kJ) in SI units and
calories (cal) or kilocalories (kcal) in MKS units.
Heat transfer per unit mass of a system is de- Fig. 2.4 -
noted by q and is expressed as
Energy and Work Transfer 33

Q = 30 kJ 1. Heat transferred to a system (heat supply) is


m = 2 kg considered positive.
t=5s
2. Heat transferred from a system (heat rejec-
tion) is considered negative.
In other words, the quantity of heat which in-
.
Q = 6 kW creases the energy content of a system is positive
q = 15 kJ/kg and any heat transfer that decreases the energy con-
tent of a system is negative.

The quantity of heat transferred in unit time is It is defined as heat energy required to change the
called heat-transfer rate. It is designate as Q and temperature of the unit mass of a substance by
is measured in kJ/s or kW. It is given as one degree. It is designated as C and is measured
Q in kJ/kg ◊ K or kJ/kg ◊ ◊K
Q= ... (2.14) in MKS units. In general, the specific heat can be
Dt
calculated as
Energy 1 Ê dQˆ dq
C = Á ˜ = …(2.15)
m Ë dT ¯ dT
In thermodynamics, heat and internal energy are
two different forms of energy. Internal energy is Since the heat transfer is a path function, the
a property, while heat is not. A body may contain specific heat also becomes path function and it de-
energy (in stored form) but not heat. The internal pends on how the process is executed. The value of
energy is associated with a state, while heat is as- energy storage capacity of the substance depends
sociated with a process. Therefore, heat or heat en- upon specifc heat. The value of specific heat de-
ergy is defined as a form of energy in transit. Heat pends upon
is a path function. It requires a specific direction in (i) molecular arrangement of the system,
its representation on a plot. -
curs,
(iii) How the system executes the process
Heat is a directional quantity, and its specification
requires magnitude and direction. Universally ac- S Liquids
cepted sign conventions for heat energy are shown Essentially, gases have two specific heats, Cp and
in Fig. 2.6: Cv. But for liquids and solids, the specific volume
is very small and its change with pressure and
Surroundings

d ( pv) is neglected from the differential form of


Qout (–)
Eq. (2.11), and thus
System dh ª du
Qin (+)
It indicates that for solids and liquids, the en-
thalpy is equal to internal energy. Thus they have
Boundary only one specific heat designated as C. Therefore,
for any process of solids and liquids,
Fig. 2.6
dq = dh = du ∫ C dT ...(2.16)
34 Thermal Engineering

Q 981m
or DT = = = 0.233°C
The product of mass and specific heat is defined as mC m ¥ 4200
heat capacity of the system. It is measured in kJ/K
Example 2.3 An automobile vehicle of 1500 kg is
running at a speed of 60 km/h. The brakes are suddenly
Example 2.1 4 kg of solid material is heated from applied and the vehicle is brought to rest. Calculate
15°C to 115°C with addition of 750 kJ of heat in a fur- the rise in temperature of brake shoes, if their mass is
nace. Calculate its specific heat. 15 kg. Take the specific heat of brake shoe material as
0.46 k J/ kg ◊ K.
Solution
Solution
Given Mass of system, m = 5 kg
Initial temperature, T1 Given A running automobile vehicle is brought to rest.
Final temperature, T2 Initial vehicle speed, V1 = 60 km/h = 16.67 m/s,
Heat added Q = 750 kJ Final vehicle speed V2 = 0
Mass of vehicle, mvehicle = 1500 kg,
To find The specific heat of the solid system Mass of brake shoes, mbrake = 5 kg,
Analysis The heat supplied to a system is expressed as Specific heat C = 0.46 kJ/kg ◊ K
Q = mC (T2 – T1) To find The temperature rise of brake shoes
Q
\ C = All the heat generated is absorbed by
m (T2 - T1 )
Assumption
brake shoes.
750 kJ
= Analysis The deceleration work of vehicle
(1 kg) ¥ (115° C - 15° C)
1
= 7.5 kJ/kg ◊ °C Wdeceleration = DKE = mvehicle ( V22 - V12)
2
1
Example 2.2 Estimate the rise in temperature of wa- = ¥ (1500 kg) ¥ ((0 m) 2 - (16.67 m) 2 )
2
ter when it falls through a height of 50 m. Assume that
= –208417 J = –208.417 kJ
all the heat generated stays in water. The specific heat of
water may be taken as 4.2 kJ/kg ◊ °C. When brakes are applied, this deceleration work con-
verts into heat energy and is transferred to brake shoes.
Solution That is,
Q = 208.417 kJ
Given A running automobile vehicle is brought to rest
and Q = mbrake CDT
Height of water h = 100 m,
(208.417 kJ )
Specific heat of water C = 4.2 kJ/kg ◊ \ DT =
= 4200 J/kg ◊
(15 kg) ¥ (0.46 kJ / kg ◊ K )
= 30.2°C
To find The temperature rise of water
Assumption Acceleration due to gravity is 9.81 m/s2. Example 2.4 During a certain process, the specific
heat capacity of a system is given by C = (0.4 + 0.004 T)
Analysis The potential energy change of water when it kJ/kg°C. Find the heat transferred and mean specific
falls through 100 m heat of gas, when the temperature changes from 25°C to
DPE = mgh = m ¥ (9.81 m/s2) ¥ (100 m) = 981m J 125°C. The mass of the system is 5 kg.
Decrease in potential energy will convert into heat
energy, thus Solution
Q = DPE = 981 m J Given Initial temperature, T1
and Q = m C DT Final temperature, T2
Energy and Work Transfer 35

C = (0.4 + 0.004T) kJ/kg ◊


Mass of system, m = 5 kg
To find The heat-transfer rate
Analysis The relation for specific heat is given as
C = (0.4 + 0.004T
The heat transfer can be calculated as Fig. 2.7
T2
Q = m
ÚT1
C dT

125
= 5¥
Ú 25
(0.4 + 0.004T ) dT
125
È T2 ˘
or Q = 5 ¥ Í0.4T + 0.004 ¥ ˙
ÍÎ 2 ˙˚
25
125
È (125) 2 - ( 25) 2 ˘
= 5 ¥ Í0.4 ¥ (125 - 25) + 0.004 ¥ ˙
ÍÎ 2 ˙˚ 25
= 5 ¥ (40 + 30) = 350 kJ
The heat transfer can also be expressed in terms of
mean specific heat as the system boundary as the insulated wall and the
Q = mC (T2 – T1) moving blades add internal energy to the system.
350
Thus C = = 0.7 kJ/kg°C
5 ¥ (125 - 25) D
In thermodynamics, force and distance are not easily

to the thermodynamic definition of work, an energy


Work like heat is also a form of energy in transit. interaction between a system and its surroundings
It is defined as the energy transfer associated with during a process can be considered as work trans-
force acting through a distance. It is an interaction fer, if its sole effect on every thing external to the
between a system and its surroundings. system could have been to raise a weight.
Energy can cross the boundary of the system as Thermodynamic work refers to transfer of en-
heat or work. Therefore, if the energy crossing the ergy due to potential difference other than tempera-
boundary is not heat, then it must be work. A mov- ture difference, without transfer of mass across the
ing piston, a rotating shaft, a rising weight are all system boundary. It is an extension of the concept
associated with work interaction. of work in mechanics.
Work is also measured in kJ. Work done during
a process from the state 1 to the state 2 is denoted battery, switch, and a resistance coil outside the
by W1–2 or W. The work done per unit mass (w) is system as shown in Fig. 2.9(a). When the switch
defined as is closed, the current flows through the resistance
W coil. Thus, electrical energy crossing the boundary
w= (kJ/kg) ...(2.17)
m of the system is converted into heat energy. At the
An insulated chamber containing a system of boundary of the system, force and motion are not
gas, and exchanging energy as work transfer only evident; and thus according to mechanics, this ener-
is shown in Fig. 2.7. The heat energy does not cross gy interaction cannot be regarded as work transfer.
36 Thermal Engineering

Boundary Sometimes the power is also expressed in horse


I power (hp). The relation between hp and kW is
Switch Resistance 1 hp = 0.746 kW
heater

V2 – V 1 +

Work Transfer
The complete specification of work transfer also
requires magnitude and direction. The universally
Electrolytic accepted sign conventions for work transfer are
battery
shown in Fig. 2.10.
1. The production of work is desirable, there-
(a) Battery work through a resistance fore, the work done by a system is consid-
ered positive.
Pulley
Switch I
Wpw
2. The consumption of work is always unde-
Motor sirable, therefore, work done on a system is
V2 – V 1 + considered as negative.

Raising
weight
Electrolytic
battery

Boundary

(b) Battery works through an electric motor-pulley


and weight
Fi
Fig. 2.9
By the sign convention, work produced by a car
However, an electric current is driven by an engine, hydraulic motor, steam, and gas turbines
electrical potential difference external to the system. is positive and work consumed in operation of a
Thus, the work is done by the battery according to compressor, a pump, a refrigerator, a fan, etc., is
thermodynamic definition. This concept can be negative.
illustrated by replacing the external resistance by
an imaginary motor-pulley and weight as shown in Work Transfer
Fig. 2.9(b). When the switch is closed, the battery The features of work transfer are given below:
drives the motor. The pulley rotates and in turn
raises the suspended weight. Thus, the sole effect
thus it is a boundary phenomenon.
external to the system becomes to raise a weight.
(ii) Work is transferred in specific direction in a
process; thus it is a path function.
(iii) If the nature of the process changes between
The quantity of work transfer per unit time is called two given states, the magnitude of work may
power. It is actually the rate of work transfer. It is change.
denoted by P and measured in kW (= kJ/s).
(iv) Magnitude of work transfer can also be ob-
dW tained by calculating the area under the path
P= (kW) …(2.18)
dt of the process.
Energy and Work Transfer 37

Work
Transfer
Heat and work both are the interaction of energy 1. Electrical work 2. Mechanical work
between a system and its surroundings and they 3. Moving boundary work 4. Flow work
have some similarities between them. 5. Gravitational work 6. Acceleration
work
system as they cross it, thus, both heat and 7. Shaft work 8. Spring work
work are boundary phenomena.
2. A system may have energy, but not heat or
work, because, heat and work are transient Electrical work is the energy interaction due to
phenomena. crossing of electrons at the system boundary. In an
3. Both are associated with a process, not a electric field, the electrons in a wire move under
state. Therefore, unlike properties heat or the effect of electromotive forces for doing work
work has no meaning at a state. (driving a motor, fan, etc.). The resistance heating
4. Both are path functions. They are represented as an electrical work is shown in Fig. 2.11.
by a path followed during the process.
5. The equations for heat and work transfer
cannot be differentiated exactly. The differ-
ential quantities of heat and work are repre- Fig. 2.11
sented as dQ and dW, respectively.
electrical work transfer can be expressed
Work as
Transfer
WE = V I (watts) ...(2.19)
1. Heat is a low-grade energy, whereas the The work done WE in time Dt is
work is a high-grade energy. WE = V I Dt ( joules) ...(2.20)
2. Heat transfer takes place due to temperature
difference only, while work transfer may
take place due to any potential difference in
In mechanics, the work done by a system is expressed
pressure, voltage, height, velocity, tempera-
as a product of force (F ) and displacement (s)
ture, etc.
3. A stationary system cannot do work, while W = Fs ...(2.21)
such a restriction is not imposed on heat If the force is not constant, the work done is ob-
transfer. tained by adding the differential amounts of work,
2
Ú
4. The entire quantity of work can be converted W = F ds ...(2.22)
into heat or any other form of energy, while 1
conversion of the entire quantity of heat into
work is not possible. Boundary Work
work into heat or another In many thermodynamic problems, mechanical
form of energy is possible with a single work is the form of moving boundary work. The
process, while conversion of heat into work moving boundary work is associated with real en-
requires a complete cyclic process, like a gines and compressors. The pressure difference is
steam power plant. the driving force for mechanical work.
38 Thermal Engineering

- energy. The gravitation force Fg acting on a body


ton cylinder arrangement as shown in Fig. 2.12. Let of mass is
the gas pressure is p, volume V and piston cross- Fg = mg …(2.26)
sectional area is A. If the piston is allowed to move where g is the acceleration due to gravity. The
through a distance ds in a quasi-equilibrium man- gravitational work done to raise the body from the
ner, the force applied on piston is elevation z1 to z2 is
F = pressure ¥ cross-sectional area of piston 2 z2
= pA Wg = Ú 1
Fg dz = mg Ú z1
dz
= mg (z2 – z1) ( joules) ...(2.27)
It is an increase in potential energy of the body
due to gravitation work done on it.

The work associated with change in velocity of


a system is defined as the acceleration work.
Acceleration force, Fa according to Newton’s sec-
ond law of motion is
Fa = ma ...(2.28)
where a is the acceleration, which is defined in
Fig. 2.12 terms of the velocity V as
dV
a = ...(2.29)
dt
Then differential work transfer through a dis- and the velocity V in terms of the displacement s
placement of ds during this process is defined as
dW = pAds = pdV ...(2.23) ds
Thus, the moving boundary work of a system in V = ...(2.30)
dt
differential form is equal to the product of absolute the ds = Vdt
pressure and differential change in its volume dV
Thus, the acceleration work
(= A ds). 2 2 Ê dV ˆ V2
The total boundary work can be obtained by Wa = Ú1
Fa ds = Ú
1

Ë dt ˜¯
( Vdt ) = m Ú V1
Vd V
adding all differential works from the initial state 1
to the final state 2 as = (1/2) m(V22 – V12) ( joules) ...(2.31)
2 where V1 and V2 are the initial and final velocities
W= Ú1
pdV (kJ) ...(2.24) of the moving mass m. The acceleration work is
This work transfer during a process is equal to recognised as the change in kinetic energy.
the area under the curve on a p–V diagram and the
work done by each kg of system is
2 The shaft work is the work associated with energy
w=
Ú 1
pdv (kJ/kg) ...(2.25)
transmission with a rotating shaft. It is the product
of torque (product of force and radius of shaft) and
angular displacement.
The work done against the gravitational force is r, rotating with N
called gravitational work or change in potential revolutions per minute as shown in Fig. 2.13. If the
Energy and Work Transfer 39

Then the spring work


dWspring = k x dx
If the spring length changes from x1 (undisturbed
position) to x2 under the action of force, the spring
work is
x2
Wspring = k Ú x1
x dx

force F is acting through an arm radius r then the = (1/2) k (x 22 – x 12) ...(2.33)
torque is
Example 2.5 A gas is compressed from an initial
T volume of 0.38 m3 to a final volume of 0.1 m3. During
T = Fr or F =
r the quasi-equilibrium process, the pressure changes with
This force acts through a displacement per unit volume according to the relation, p = aV + b, where, a =
–1200 kPa/m3 and b = 600 kPa. Calculate the work done
time,
N during this process.
s = ( 2p r ) ¥
60 Solution
Then the shaft work per unit time (shaft power)
can be expressed as Given
V1 = 0.38 m3, V2 = 0.1 m3,
ÊNˆ T
Wsh = Fs = 2p r ¥ Á ˜ ¥ a 3
, b
Ë 60 ¯ r
and relation p = aV + b
2p N T
= (watts) ...(2.32) To find The work done by the system.
60
Analysis The work done by a system can be calculated
as
2 0.1
When the force is applied on a spring, its length W =
Ú
1
pdV =
Ú
0.38
(aV + b) dV
changes as shown in Fig. 2.14. If dx is the change in 0.1
È1 ˘
the length of a spring under the influence of a force = Í aV 2 + bV ˙
F then the work done by the spring is Î2 ˚ 0.38

dWspring = Fdx Using the numerical values


where the force F exerted can be defined in terms of 0.12 - 0.382
W = –1200 ¥ + 600 ¥ (0.1 – 0.38)
the spring constant k (N/m) as 2
F = k x (N) W = 80.64 – 168 = – 87.36 kJ

Example 2.6 In a reversible non-flow process,


Position at the work is done by a substance in accordance with
rest
2.80 3
V= m , where p is the pressure in bar. Find the
p
x1
work done on or by system as pressure increases from
x2
0.7 bar to 7 bar.

Solution
Given A reversible non-flow process with
F p1 p2
Fig. 2.14 2.80 3
and relation V = m
p
40 Thermal Engineering

To find The work interaction by the system K 10


or V2 = = = 0.223 m3
Analysis The initial and final volumes of the working p2 200
substance Now, the work done during the process
2.80 2.8
V1 = = = 4 m3 2 2 K È1 1˘
p1 0.7 W =
Ú1
pdV =
Ú
1 V 2
dV = K Í - ˙
V
Î 1 V2˚
2.80 2.8
V2 = = = 0.4 m3 È 1 1 ˘
p2 7 or W = 10 ¥ Í - ˙
From the given relation, the pressure p can be ex- Î 0.1 0.233 ˚
pressed as = 10 ¥ (10 – 4.29) = 57.1 kJ
2.80 2.80
p = bar = 100 ¥ kPa Example 2.8 A spherical balloon contains 5 kg of air
V V
at 200 kPa and 500 K. The balloon material is such that
The work done by a system can be calculated as
2 0.4
the pressure inside is always proportional to the square
1
W =
Ú1
pdV = 100 ¥ 2.8 ¥
Ú 4 V
dV of the diameter. Determine the work done when the vol-
ume of the balloon doubles as a result of heat transfer.
Ê 0.4 ˆ
= 280 ¥ ÈÎln (V )˘˚
0.4
= 280 ¥ ln Á
4 Ë 4 ˜¯ Solution
= – 644.72 kJ Given m = 5 kg, p1
T1 = 500 K, V2 = 2V1.
Example 2.7 In a piston–cylinder arrangement, the
pressure is inversely proportional to the square of the To find The work done by gas, when volume of balloon
volume. The initial pressure is 10 bar in the cylinder and doubles.
the initial volume is 0.1 m3. The volume is now changed
Assumption The gas constant for air is 0.287 kJ/kg ◊ K
so that the final pressure is 2 bar. Find the work done in
kJ. Analysis D is the diameter of the balloon.
According to the given condition,
Solution p μ D2 or p = KD2
1 It is the equation of state with a constant of propor-
Given The relation p μ ,
V2 tionality K.
p1 Further, from the relation for a perfect gas
V1 = 0.1 m3, p1v1 = RT1
p2 R T1 0.287 ¥ 500
v1 = =
To find Work done during the process p1 200
= 0.7175 m3/kg
Analysis The given relation
Thus, the volume of the balloon at initial state;
1 K V1 = m v1 = 5 ¥ 0.7175 = 3.5875 m3
pμ 2
or p=
V V2 Thus, the diameter of the balloon can be calculated as
1 K V1 = (1/6) ¥ p ¥ D13
At the state 1, p1 μ or p1 = 2
V12 V1 or D13 = 6.851
where K is the constant of proportionality and it is or D1 = 1.899 m
calculated by initial condition. Therefore, When the volume of the balloon doubles, the diameter
K = p1 V12 ¥ (0.1 m3)2 of balloon
6
V2 = (1/6) ¥ p ¥ D23
Now at the state 2, but V2 = 2 V1 = 7.175
K or D 23 = 13.703
p2 =
V2 2 or D2 = 2.393 m
Energy and Work Transfer 41

Now from the given relation at the state 1 of volume (m3) is given by
p1 = K D12 0.86110 8.60673 ¥ 10 -5
p = -
or
p
K = 12 =
200
= 55.44 V V2
D1 (1.8993) 2 (a) Find the amount of work done in kJ.
The work done by a system; (b) If the atmospheric pressure, i.e., 1 bar acting on
2 2 2 Êp ˆ the other side of piston is considered, find the net
W =
Ú
1
pdV =
Ú1
KD 2 dV = K Ú1
D 2 d Á D3 ˜
Ë6 ¯ work done in kJ.

K 2
=
6

Ú 1
2
D ¥ 3D dD 2 Solution
Given V1 = 0.8611 m3, V2 = 0.17212 m3,
1 2
= p ¥ 55.44
2 Ú 1
4
D dD patm
0.86110 8.60673 ¥ 10 -5
and p = - (bar)
È D5 D5 ˘ V V2
= 87.08 ¥ Í 2 - 1 ˙
ÍÎ 5 5 ˙˚ To find
(i) The work done by the gas,
È ( 2.393)5 (1.8993)5 ˘ (ii) The work done by the gas when atmospheric
= 87.08 ¥ Í - ˙
ÍÎ 5 5 ˙˚ pressure on other side of piston is considered.
= 936.22 kJ Analysis
(i) Work done by the gas without considering
Example 2.9 The van der Waals equation is given by atmospheric pressure;
Ê aˆ V2
ÁË p + v ˜¯ (v - b) = RT W =
ÚV1
pdV
0.17212 Ê 0.86110
where a and b are constants and other terms have usual
meanings. Determine the work done in a reversible
= (100 kPa / bar ) Ú 0.8611
ÁË V

isothermal expansion. 8.60673 ¥ 10 -5 ˆ


- ˜ dV
V2 ¯
Solution The van der Waals equation is È
= 100 ¥ Í0.86110 ln (V ) - 8.60673
Ê aˆ Î Ê 1 ˆ˘
0.17212
ÁË p + v ˜¯ (v - b) = RT ¥ 10 -5 ¥ Á - ˜ ˙
Ë V ¯˚
0.8611
It can be rearranged as
È Ê 0.17212 ˆ
RT a = 100 ¥ Í0.86110 ¥ ln Á - 8.60673
p = - Î Ë 0.8611 ˜¯
v-b v
Ê 1 1 ˆ˘
The work done by the gas can be calculated as ¥ 10 - 5 ¥ Á - ˙
v2 Ë 0.8611 0.17212 ˜¯ ˚
w=
Ú v1
p dv = –138.6 kJ
v2 1 v2 a (ii) Work done on the atmosphere by the piston for its
or w = RT
Ú
v1 v-b
dv -
Ú
v1 v
dv volume change from V1 to V2
V2

or
Ê v - bˆ
w = RT ln Á 2 ˜
È1 1˘
+ a Í - ˙ (kJ/kg)
Watm =
ÚV1
pdV = p (V2 – V1)
Ë v1 - b ¯ Î v2 v1 ˚ = 100 ¥ (0.17212 – 0.8611)
= – 68.9 kJ
Example 2.10 A quantity of gas is compressed in a Net work done by the gas
piston–cylinder from a volume of 0.8611 m3 to a final Wnet = W – Watm = –138.6 + 68.9
volume of 0.17212 m3. The pressure (in bar) as a function = – 69.7 kJ
42 Thermal Engineering

Example 2.11 A system of 1 kg of gas expands from pressure is found to be 500 kPa. Determine the work done
an initial state at pressure of p1 bar and a volume of v1 m3/ by the gas. Take the atmospheric pressure as 1 bar.
kg to a volume of v2 m3/kg. Calculate the work done by
the gas, when expansion is (a) isobaric, (b) isothermal,
and (c) polytropic with the law pv n = constant.

Solution
Analysis
(i) When the process of expansion is isobaric
(constant pressure):
p1 (bar) = 100p1 (kPa)
v2 v2 Gas under spring force
w =
Úv1
pdv = 100 p1
Ú v1
dv
Solution
= 100 p1 (v2 – v1)
(ii) When the process of expansion is isothermal Given A cylinder with a piston connected to coil
(constant temperature): spring.
The law of process, V1 = 0.1 m3, p1 = 200 kPa,
100 C V2 = 2V1, p2 = 500 kPa.
pv = C or p =
v and Fspring μ x or Fspring = kx
v2 v2
Ú
1
Ú
The work done w = pdv = 100 C To find Work done by the gas.
dv
v1 v1 v
Analysis The force balance at any position of the piston
Êv ˆ
= 100 C ln Á 2 ˜ pA = patm A + kx
Ëv ¯ 1
V
Êv ˆ using di splacement x =
or w = 100 p1v1 ln Á 2 ˜ ( kJ /kg ) A
Ëv ¯ 1 kV
Then pA = patm A +
(iii) When the process of expansion is polytropic: A
The law of process, kV
or p = patm + ...(i)
100C A2
pvn = C or p =
vn
2 v2 The work done
1
The work done, w =
Ú pdv = 100 C Ú dv 2 2Ê kV ˆ
1

È v 1- n - v11- n ˘
v1 vn W =
Ú1
pdV =
Ú 1
ÁË patm + 2 ˜¯ dV
A
= 100 C Í 2 ˙ k
ÍÎ n -1 ˙˚ = patm(V2 – V1) + (V22 - V12 )
2 A2
n n
Now using C = p1 v1 = p2 v2
k
Ê p v - p1v1 ˆ
= patm(V2 – V1) + (V2 - V1 ) (V2 + V1 )
We get, w = 100 ¥ Á 2 2 2 A2
Ë n-1 ˜¯ kJ/kg
È k ˘
= (V2 – V1) Í patm + 2
(V2 + V1 )˙ ...(ii)
Î 2A ˚
Example 2.12 A cylinder with a frictionless piston
From Eq. (i), we get
contains 0.1 m3 of gas at 200 kPa. The piston is connected
to a coil spring which exerts a force proportional to the kV = A2 ( p – patm)
displacement from its equilibrium position. The gas È p - patm p1 + patm ˘
\ W = (V2 – V1) Í patm + 2 + ˙
is heated until the volume is doubled, at this state; the Î 2 2 ˚
Energy and Work Transfer 43

È p + p2 ˘ In the absence of any work interaction between


On solving W = (V2 – V1) Í 1 ˙ a system and its surroundings, the amount of net
Î 2 ˚
Using numerical values; heat transfer is equal to the change in the energy
Ê 200 + 500 ˆ of a system.
W = (0.2 – 0.1) ¥ Á ˜¯ = 35 kJ
Ë 2 Q = DE when W = 0 ...(2.34)
Now consider a well-insulated room, heated by
an electric heater as shown in Fig. 2.17. As a result
of electrical work done, the energy of the room will
The first law of thermodynamics, also known increase. Since the room is adiabatic and cannot
as the conservation of energy principle. It states have any heat interaction with its surroundings,
that during any process, if the energy disappears the conservation of energy principle dictates that
in one form, it appears in other form, but its total electrical work done on the room must be equal to
quantity remains always constant. That is, the increase in energy of the room. That is,
energy can be neither created nor destroyed, it can –W = DE
only change its form. For example, for the energy
interaction between a system and its surroundings,
DE = +10 kJ
the energy lost by a system must be exactly equal to
the amount of energy gained by the surroundings. Room as
a system W = –10 kJ
The first law can be proved mathematically, but no
process in nature is known to have violated the first
law of thermodynamics.
It is the relation of energy balance and is appli-
cable to any kind of system (open or closed) under-
going any kind of process.
For an adiabatic processes, the amount of work
Let us consider a process that involves only heat done is equal to the change in energy of the system.
transfer but no work interaction. A hot potato taken That is,
from an oven is exposed to room air as shown in
W = –DE when Q = 0 ...(2.35)
Fig. 2.16. As a result of heat transfer from the hot
potato, its energy will decrease. In absence of other Now consider work and heat transfer simultane-
effects, the decrease of total energy of the potato
becomes equal to the amount of heat transferred to gas in a piston cylinder device. The gas is heated
its surroundings. Therefore, the principle of conser- inside the cylinder as a result of heat supply. As the
vation of energy can be expressed as energy of the gas increases, its pressure and tem-
perature also increase. Then the gas will expand
–DE = –Q
and work will be done at the boundary of the sys-
where D E = E2 – E1
tem. The conservation of energy principle reveals
that
Q = W + DE
Potato Surroundings
at 100°C at 30°C or Q – W = DE (kJ) ...(2.36)
DE = –20 kJ where Q = net heat transfer across system
boundaries
Heat
Q = – 20 kJ W = net work transfer in all forms
DE = net change in total energy of the
Fig. 2.16
system (E2 – E1)
44 Thermal Engineering

W The energy of a system on unit mass basis is


denoted as e and is defined as
E
e = (J / kg) ...(2.43)
m
System
Q The magnetic, wind, thermal, electrical and me-
DE
chanical energy are transit forms of energy. Most
often, in thermodynamic analysis, the total energy
Fig. 2.18 of a system is considered as the sum of kinetic en-
ergy, potential energy and internal energy and is
The total energy of the system is the sum of expressed as
internal energy U, potential energy PE and kinetic E = U + KE + PE ...(2.44)
energy KE, i.e., On unit mass basis e = u + ke + pe ...(2.45)
DE = DU + DPE + DKE (kJ) ...(2.37)
Most closed systems in practice are stationary, Energy Transfer
i.e., they do not involve kinetic energy and potential The energy can be transferred to or from a system
energy during a process. Thus, the stationary sys- in three forms: heat, work and mass flow. The ener-
tems are called non-flow systems and the first law gy interactions are recognised at the system bound-
of thermodynamics is reduced to ary as they cross it, and direction of energy transfer
Q – W = DU (kJ) ...(2.38) represents the energy gain or loss by a system dur-
ing a process.
1. Heat transfer to a system
increases the energy of molecules and thus an in-
crease in the internal energy of the system, and heat
transfer from a system decreases the energy of mol-
Fig. 2.19 ecules and thus results in a decrease in the internal
energy of the system.
2. The work refers to transfer
For unit mass, of energy due to potential difference other than
q – w = Du ...(2.39) temperature difference, between a system and its
In rate form of energy, dividing Eq. (2.36), on boundary. A rising piston, rotating shaft, and an
both sides by the time interval dt electric wire carrying current crossing the system
dE boundary—all these energy transfers are associated
Q -W = ...(2.40)
dt with work interactions. The work transfer to a sys-
In differential form, tem (called work done on a system) increases the
dQ – dW = dE ...(2.41) energy of the system and work transfer from a sys-
tem (called work done by the system) decreases it.
Energy 3. When mass enters a system, the
The sum of all the forms of energy is called total energy of the system increases, because mass car-
energy or energy E of a system. Thus ries energy with it. Similarly, when some mass
E = PE + KE + IE leaves the system, the energy of the system de-
+ other forms of energy ...(2.42) creases, because the leaving mass takes some en-
ergy with it.
Energy and Work Transfer 45

The net change in energy of a system is the In his experimental arrangement as shown in
difference between the amounts of heat, work and Fig. 2.20, he took a known quantity of water in
mass transferred in and out and the energy balance a rigid insulated tank fitted with a paddle wheel.
can be written more clearly as Water was agitated by a stirrer connected with the
DEsystem = (Qin – Qout) + (Win – Wout) paddle wheel (process A). The amount of work done
+ (Emass,in – Emass,out) ...(2.46) on the water by the stirrer was accurately measured
where quantity ‘in’ and ‘out’ denote qunatities that as a product of weight and its displacement. The
enter and leave the system, respectively. temperature rise of water during the process was
The heat transfer Q also recorded.
system, the work transfer W
that does not involve work interaction and the
energy transport with mass Emass
systems.
The net change in total energy of a system during
a process is equal to the difference between the
total energy entering and the total energy leaving
the system during that process.
Fig. 2.20 -
Ê Total energy ˆ - Ê Total energy ˆ
ÁË entering the system ˜¯ ÁË leaving the system˜¯

= ÊÁ
Net change in total ˆ
Ë energy of the system˜¯ Further, the insulation from the tank was
or Ein - Eout = D Esystem (kJ) removed and the whole system was placed in a
Net energy transfer Change in internal, kinetic, water bath. The heat was transferred from the
by heat, work and mass potential, etc., en
nergies
system (process B) in order to restore the system to
…(2.47)
the same initial state. Thus process A and process
In rate form B form a cycle. During process A, there was work
dEsystem
Ein - Eout = (KW) done on the system but no heat transfer; during the
dt
Rate of net energy transfer process B, there was heat transferred but no work
by heat, work and masss Rate of change in internal,
kinetic, potentiall, etc., energies done. During a complete cycle, there was net work
…(2.48) input and net heat output from the system.
For constant rates, the total quantities during a Joule repeated the experiment for different
time interval Dt are related to the quanties per unit systems, and for different amounts of work
time as interactions, and measured the corresponding
Q = Q D t , and amount of heat interaction in each case for restoring
W = W Dt , ...(2.49) the system back to its initial state. Joule found in
his experimental observations that,
“Whenever a closed system undergoes a cycle,
the work input to the system is proportional to the
net heat output”. It is expressed as

Joule conducted several experiments which led to Ú dW μ


Ú dQ
the formulation of the first law of thermodynamics. or
Ú dW Ú
= J dQ ...(2.50)
46 Thermal Engineering

where J is the constant of proportionality, called


mechanical equivalent of heat, whose value de- 2 2 2 2
pends on the units selected for W and Q. In SI units Ú 1
d QA - Ú 1
dW A = Ú 1
d QB - Ú 1
dWB ...(2.53)
its value is unity, therefore;
The quantity dQ – dW is dE according to
Ú dW = Ú d Q ...(2.51) Eq. (2.36). The integration of each side of Eq. (2.53)
leads to the energy transfer during the paths (pro-
When a system undergoing a cyclic process cesses) A and B, thus
comprises of more than one process, then DEA = DEB ...(2.54)
SW = SQ
Now consider another cycle 1–A–2–C–1. The
or SW – SQ = 0 ...(2.52) processes A and C together constitute a cycle, for
For a cyclic process, the net heat transfer is al- which,
ways equal to the net work transfer. 2 1 2 1
Ú
1
d QA + Ú 2
d QC = Ú
1
dW A + Ú dW
2
C

2 2 2 2
or
Ú 1
d QA - Ú
1
dW A = Ú
1
d QC - Ú
1
dWC

or DEA = DEC ...(2.55)


For any cyclic process, the first law of thermody-
From Eq. (2.54) and (2.55), we get
namics is given by Eq. (2.51). Many times we are
concerned with heat and work transfer during a DEB = DEC
single thermodynamic process. Thus, we formulate It indicates that the change in energy between
the first law of thermodynamics for a process. two states of the system does not depend upon the
path followed by the process, but depends on the
from 1 to 2, along a path A and returning from states between which the process proceeds. Thus,
state 2 to 1, along the path B, thus completing a the energy is a state function and property.
cycle 1–A–2–B–1 as shown in Fig. 2.21.
p

1 A device that violates the first law of thermodynam-


A
C ics by producing work from nothing is called a per-

2 Fig. 2.22. It is proposed to produce steam by a re-


v sistance heater placed inside the boiler, instead of

Applying the first law for the cyclic process


1–A–2–B–1;
2 1 2 1
Ú
1
d QA + Ú 2
d QB = Ú
1
dW A + Ú dW
2
B

2 2 2 2
or
Ú
1
d QA - Ú1
d QB = Ú
1
dW A - Ú 1
d WB

Fig. 2.22
Energy and Work Transfer 47

by combustion of fuel. The part of electricity gen- Solution For any process;
erated by the plant is to be used for the resistance Q = W + DU
heating and pump work. The rest of the electricity Q1–2 = W1–2 + DU1–2
is the net output of the plant. The inventor claims, Using the values;
once the system is started, the power plant produces 400.0 kJ/min = 150.0 kJ/min + DU1–2
the electricity for an infinite time without requiring or DU1–2 = 250.0 kJ/min
any energy input from the outside. Q2–3 = W2–3 + DU2–3
Here, the inventor tries to solve the world energy Using the values;
crisis, if it works. Unfortunately, such a proposal 200.0 kJ/min = W2–3 + 300.0 kJ/min
is not possible in practice, because the device only or W2–3 = –100.0 kJ/min
produces energy output as electrical work, WE and Since there are two unknowns in the process 3–4, and
the heat rejection Qout from the condenser without they cannot be determined with one equation, hence we
any energy input, which is a violation of the first consider process 4–1;
law of thermodynamics. Therefore, such a device Q4–1 = W4–1 + DU4–1
is called the perpetual motion machine of the first Using values;
0.0 kJ/min = 75 kJ/min + DU4–1
or DU4–1 = –75.0 kJ/min
Example 2.13 A system is composed of a gas con- All processes together constitute a cycle, thus for a
tained in a cylinder fitted with a piston. The gas expands cyclic process:
from the state 1 for which E1 = 75 kJ to a state 2 for SQ = SW
which E2 = –25 kJ. During the expansion, the gas does Q1–2 + Q2–3 + Q3–4 + Q4–1
60 kJ of work on the surroundings. Determine the heat
= W1–2 + W2–3 + W3–4 + W4–1
transferred to or from the system during the process.
Using the values
Solution 400.0 + 200.0 + (–200.0) + 0
= 150.0 + (–100.0) + W3–4 + 75.0
Given E1 = 75 kJ, E2 = –25 kJ and W = 60 kJ or W3–4 = 275.0 kJ/min
To find The amount of heat transferred. Now considering the process 3–4
Q3–4 = W3–4 + DU3–4
Analysis According to the first law of thermodynamics
Using values;
for a process
–200.0 kJ/min = 275.0 kJ/min + DU3–4
Q – W = DE
or DU3 – 4 = – 475.0 kJ/min
or Q = W + DE
For a cyclic process:
where DE = E2 – E1 = –25 – 75 = –100 kJ,
Net heat transfer = net work transfer = 400 kJ/min
Using the above Q = (60 kJ) + (–100 kJ)
The power output
= – 40 kJ (heat is rejected)
Net work transfer 400
P = =
Example 2.14 A system undergoes a cyclic process time in seconds 60
composed of four processes 1–2, 2–3, 3–4, and 4–1. The = 6.67 kW
energy transfer is tabulated below: The complete table is

Process Q kJ/min W kJ/min DU kJ/min Process Q kJ/min W kJ/min DU kJ/min


1–2 400.0 150.0 — 1–2 400.0 150.0 250.0
2–3 200.0 — 300.0 2–3 200.0 –100.0 300.0
3–4 –200 — — 3–4 –200 275.0 – 475.0
4–1 0 75 —
4–1 0 75.0 –75.0
Complete the table and determine the power output.
48 Thermal Engineering

Example 2.15 A non-flow system undergoes a fric- For a 10-kg fluid system,
tionless process according to a law p = (4.5/v) + 2, where Q = (10 ¥ 415.94) = 41594 kJ
p is in bar and the volume v is in m3/kg. During the pro- (ii) The change in specific enthalpy can be calculated
cess, the volume changes from 0.12 m3/kg to 0.04 m3/ kg as
and the temperature increases by 133°C. The change in Dh = Du + D(pv) = Du + ( p2 v2 – p1v1)
internal energy of the fluid is given as du = Cv dT, where The pressure p1 and p2 can be calculated from the
Cv = 0.71 kJ/kg ◊ K and dT is temperature change. Find given expression:
out (a) heat transferred, and (b) change in enthalpy.
È 4.5 ˘
Assume a fluid quantity of 10 kg. p1 = 100 ¥ Í + 2˙ kPa
Î v1 ˚
Solution È 4.5 ˘
= 100 ¥ Í + 2˙
È 4.5 ˘ Î 0.12 ˚
Given The process law p = Í + 2˙ bar
Î v ˚ = 3950 kPa
È 4.5 ˘ È 4.5 ˘
= 100 ¥ Í + 2˙ kPa and p2 = 100 ¥ Í + 2˙ kPa
Î v ˚ v
Î 2 ˚
Initial volume, v1 = 0.12 m3/kg, È 4.5 ˘
= 100 ¥ Í + 2˙
Final volume, v2 = 0.04 m3/kg Î 0 .04 ˚
Temperature rise, DT = 133°C, = 11450 kPa
Specific heat, Cv = 0.71 kJ/kg ◊ K Thus flow energy,
du = Cv dT, D(pv) = 11450 ¥ 0.04 – 3950 ¥ 0.12
Mass of fluid, m = 10 kg = –16.0 kJ/kg
To find Change in enthalpy, Dh = 94.43 + (–16)
= 78.43 kJ/kg
(i) Heat transferred, and
For 10 kg fluid, DH = m Dh = 10 ¥ 78.43
(ii) change in enthalpy.
= 784.3 kJ
Analysis
(i) The change in specific internal energy of the fluid Example 2.16 The power developed by a turbine in a
system can be calculated by integrating the given certain steam power plant is 1200 kW. The heat supplied
expression to the boiler is 3360 kJ/kg. The heat rejected by the system
2 to the cooling water is 2520 kJ/kg and feed pump work
Du =
Ú
1
Cv dT = Cv DT required to pump the condensate back into the boiler is
= (0.71 kJ/kg ◊ K) ¥ (133°C or K) 6 kW. Calculate the steam flow through the cycle in kg/s.
= 94.43 kJ/kg
Solution
The work done by each kg of fluid
2 Given A steam power plant (Fig. 2.23)
È 4.5 ˘
p = 100
Ú
1
Í v + 2˙ d v
Î ˚ q1 = 3360 KJ/kg

È Êv ˆ ˘ Boiler
= 100 ¥ Í4.5 ¥ ln Á 2 ˜ + 2 ( v2 - v1 )˙
ÍÎ Ë v1 ¯ ˙˚ 4 1

È Ê 0.04 ˆ ˘ Pump turbine


= 100 ¥ Í4.5 ¥ ln Á ˜¯ + 2 ¥ (0.04 - 0.12)˙
Î Ë 0. 12 ˚ Wp = –6 kW Ws = 1200 kW

= 100 × (– 4.943 – 0.16) = –510.37 kJ/kg 2

The heat transfer for one kg of fluid system: Condenser


3
q = w + Du = –510.37 + 94.43 q2 = –2520 kJ/kg
= – 415.94 kJ/kg
Fig. 2.23
Energy and Work Transfer 49

To find Mass flow of steam


therefore, work transfer;
Analysis Since the working substance (steam/water)
W2–3 = p1(V3 – V2) = 100 ¥ (0.06 – 0.3)
undergoes a cyclic process, thus
= –24 kJ

Ú dQ = Ú dW Applying the first law of thermodynamics


Q2–3 = W2–3 + E3 – E2
or Q1 + Q2 = WT + W p Using the values;
or m ( q1 + q2 ) = WT + W p –105 = –24 + E3 – (–29.7)
1200 - 6 or E3 = –110.7 kJ
or m = = 1.421 kg/s
3360 - 2520 Example 2.18 The energy (J) of a closed system can
be expressed as E = 100 + 50T + 0.04T 2. The heat (J)
Example 2.17 A slow chemical reaction takes place
absorbed is given by Q = 5000 + 20 T. The temperature
in a fluid at a constant pressure of 0.1 MPa. The fluid
in these relations is expressed in K (Kelvin). Calculate
is surrounded by a perfect heat insulator during reac-
the work done during the process, when the temperature
tion, which begins at the state 1 and ends at the state 2.
rises from 500 K to 1000 K.
The insulation is then removed and 105 kJ of heat flows
to surroundings as the fluid goes to the state 3. The fol- Solution
lowing data are observed for fluid at states 1, 2 and 3
respectively: Given A closed system:
State 1: T1 = 500 K,
State V (m3) T (°C) State 2: T2 = 1000 K
1 0.003 20 To find Work transfer during the process
2 0.3 370 Analysis For any process the work transfer is expressed
3 0.06 20 as
W = Q – DE
For the fluid, calculate E2 and E3, if E1 = 0.

Solution
=
Ú dQ - Ú dE
T2 T2
Given A closed system at constant process:
State 1: V1 = 0.003 m3,
=
Ú (5000 + 20 T ) dT - Ú
T1 T1
(100

+ 50 T + 0.04 T 2 ) dT
T1
p1 (T22 - T12 )
= 5000 (T2 – T1) + 20 ¥
State 2: V2 = 0.3 m3, 2
È (T 2
- T 2
) (T 3 - T12 ) ˘
T2 - Í100 (T2 - T1 ) + 50 2 1
+ 0.04 2 ˙
Q1–2 = 0 Î 2 3 ˚
State 3: V3 = 0.06 m3, Using the values of T1 and T2
T3 Ê 1000 2 - 500 2 ˆ
W = 5000 ¥ (1000 – 500) + 20 ¥ Á
Q2–3 = –105 kJ Ë 2 ¯˜

To find Magnitude of energy at states 2 and 3 È Ê 1000 2 - 500 2 ˆ


- Í100 ¥ (1000 - 500) + 50 ¥ Á ˜
Analysis For constant pressure process 1–2; ÍÎ Ë 2 ¯
W1–2 = p1(V2 – V1) Ê 10003 - 5003 ˆ ˘
= 100 ¥ (0.3 – 0.003) = 29.7 kJ + 0.04 ¥ Á ˜˙
Applying the first law of thermodynamics Ë 3 ¯ ˙˚
Q1–2 = W1–2 + E2 – E1 = 2.5 ¥ 106 + 7.5 ¥ 106 – [50 ¥ 103
Using the values; 0 = 29.7 + E2 – 0 + 18.75 ¥ 106 + 11.667 ¥ 106]
6
or E2 = – 29.7 kJ = –20.467 ¥ 10 J = – 20462 kJ
50 Thermal Engineering

Example 2.19 On a warm summer day, a housewife We consider the room with the refrigerator as an iso-
decides to beat the heat by closing the windows and lated system. The refrigerator is operated with electrical
doors in the kitchen and opening the refrigerator door. work input and then applying first law of thermodynam-
At first she feels cool and refreshed but after a while, the ics for an isolated system
effect begins to wear off. Q = DU + W
Evaluate the situation as it relates to the first law of O = DU + (–We)
thermodynamics, considering the room including the or DU = We
refrigerator as a system. The internal energy (temperature) of the room will
increase on the work input. Hence the cooling effect will
Solution Initially, the refrigerator has cold air and
wear off.
when its door is opened, the cold air mixes with room air
and makes the room air colder for a while. This makes
the lady feel cool and refreshed.

Summary
Energy is defined as the capacity to do work. pletely into mechnical work by an ideal mechani-
Primary forms of energy are those that are either cal device such as an ideal turbine or pump.
found or stored in nature. Secondary forms of is a transfer form of energy that
energy are those forms which are derived from flows between two systems (or system and its
the primary form of energy. Commercial forms of surroundings) by virtue of the temperature differ-
energy are available in the market for a definite
price. Non-commercial forms of energy are not the system boundary.
available in the commercial market for a definite Work transfer like heat energy is also a form of
price. Renewable energy is obtained from sources energy in transit
that are not exhaustible. Non-renewable energy system boundary. The quantity of work transfer
is obtained from conventional fossil fuels such per unit time is called power.
as coal, oil and gas, nuclear fuels, and heat traps
The various forms of work are expressed as
which are accumulated in the earth crust.
1. Electrical work, WE = V I Dt
of its elevation in a gravitational field is called
potential energy PE, expressed as
2. Mechanical work, W = Ú pdv
3. Gravitation work, DPE = mg (z2 – z1)
PE = mgz ( joules) 4. Acceleration work,
DKE = (1/2) m(V22 – V12)
motion relative to some reference is called kinetic 5. Shaft work, Ws = 2p nT
energy KE. 6. Spring work,
1 Wspring = (1/2) k (x22 – x12)
KE = mV 2 ( joule)
2 The first law of thermodynamics states ‘energy
scopic forms of energy can be neither created nor destroyed, it can
is called the internal energy of the system. The change only its form’.
enthalpy or total enthalpy (H ) is the sum of the work interaction between a
internal energy U and the product of pressure and system and its surroundings, the amount of net
volume pV and is expressed as heat transfer is equal to the change in the energy
H = U + pV of a system.
Mechanical energy can be defined as a form of Q = DE when W = 0
energy that can be converted directly and com-
Energy and Work Transfer 51

work done where the term E is called total energy and it is


is equal to the change in energy of the system. the sum of all the forms of energy of a system.
W = – DE when Q = 0 Thus
E = PE + KE + IE + other forms of energy
Q – W = DE (kJ)

Glossary
Internal energy Sum of all microscopic forms of ener- Thermodynamic work A form of energy in transit due
gies of a system to potential difference other than temperature difference
Enthalpy Sum of internal energy and product of pres- Power work transfer per unit time
sure and volume Specific heat The quantity of heat required to change
Heat A form of energy in transit due to temperature dif- the temperature of unit mass of substance by one degree
ference
Work Energy transfer associated with force acting Latent energy Heat energy associated with phase
through a distance change of a substance

1. Define energy and its various forms. of heat transfer and work transfer for each of the
2. Define internal energy. following processes:
3. Define sensible heat and latent heat with reference (a) Gas in an insulated cylinder expands as the
to molecular activities. piston is slowly moved outward.
4. Why is it incorrect to say that a system contains (b) A closed rigid vessel containing steam at
heat? -
5. Explain the concept of thermodynamic work.
6. Differentiate between mechanical and thermody- (c) The air in a tyre and connecting pump; the
namic work. pump plunger is pushed down forcing air
into the tyre. The tyre, pump and connecting
work is a function of a process.
tube are non-conducting.
8. What are the salient features of work transfer?
(d) The water and steam in a rigid metallic con-
9. Define moving boundary work, gravitational tainer is heated.
work and acceleration work. (e) A viscous fluid is stirred by a paddle wheel
10. Define heat energy and specific heat. in an insulated closed tank.
11. Write the universally accepted sign convention 12. What are the similarities and dissimilarities be-
for heat and work transfer and state the direction tween heat and work interactions?

Problems
1. A tank which is 4 m long, 3 m wide and 2 m deep 2. A rectangular tank measuring 0.6 m ¥ 1 m at the
is half full of water. How much work is required base is filled half to a depth of 15 cm with water.
to raise all the water over the top edge of the
tank? [117.72 kJ] (a) Total gravitational force exerted on the base
of the tank
52 Thermal Engineering

(b) The pressure exerted by water at the base of 9. In a quasi-static process for a closed system, the
the tank [(a) 882 N, (b) 1.47 kN/m2] gas expands from a volume of 0.15 m3 and a pres-
3. An engine cylinder has a piston area of 0.1 m2, sure of 120 kPa to a volume of 0.25 m3 in such a
and contains gas at a pressure of 1.5 MPa. The manner that p(V + 0.03) = constant, where V is in
gas expands according to a process, which is rep- m3. Calculate the work done. [9.5 kJ]
resented by a straight line on a p–v plane. The 10. Air expands in a cylinder according to the law
final pressure is 0.15 MPa. Calculate the work pV1.4 = constant, from an initial volume of 3 m3
done on the gas, if the stroke is 0.3 m. [24.75 kJ] and a pressure of 450 kPa to a final volume of
4. A fluid system undergoes a non-flow frictionless 4 m3. Compute the work done by the gas.
process. The pressure and volume are related as [368 kJ]
5 11. A spherical balloon has a diameter D1, when the
p= + 1.5
V pressure of the gas inside it is p1 and atmospheric
where p is in bar and V is in m3. During the pressure p0. The pressure of the gas inside the
process, the volume changes from 0.15 m3 to balloon is proportional to the diameter. The gas
0.05 m3. Calculate the work done by the fluid. is heated till the volume of the balloon doubles.
[– 564 kJ] Calculate the work done by the gas. How much
5. A fluid at a pressure of 3 bar and with a specific work is done on the atmosphere?
volume of 0.18 m3/kg is contained in a cylinder [1.14 p1V1, 2 poV1]
behind a piston. It expands reversibly to a pressure 12. An electric potential of 100 V is impressed on
of 0.6 bar according to a law a certain resistor such that a current of 12 A is
A drawn. Calculate the energy dissipated in 3 min.
p=
v [216 kJ]
where A is a constant. Calculate the work done on
13. A certain fluid at 10 bar is contained in a cylinder
the fluid. [29.84 kJ/kg]
behind a piston. The initial volume is 0.05 m3.
6. A gas is trapped by a frictionless piston in a Calculate the work done by the fluid, when it
vertical cylinder having an inner diameter of expands reversibly
80 cm. The piston is solely supported by gas
(a) At constant pressure to a final volume of
pressure. The initial gas pressure is 250 kPa. The
0.2 m3
gas is then heated so that the piston is raised by
(b) According to a linear law to a final volume
a distance of 10 cm. Calculate the work done
of 0.2 m3 and a final pressure of 2 bar
by the gas. How much of this work is done to
(c) According to the law pV = const. to a final
lift the piston and how much to push back the
volume of 0.1 m3
atmospheric air ? What is the mass of the piston ?
Take atmospheric pressure as 100 kPa. (d) According to the law pV3 = const. to a final
volume of 0.56 m3
[12.5 kJ, 5kJ, 7.5 kJ, 7706 kg] A B
7. In a closed system, the gas is compressed fric- (e) According to the law p = 2 - to a final
V V
tionlessly from a volume of 0.01 m3 and a pres- 3
volume of 0.1 m and a final pressure of
sure of 0.70 kPa to a volume of 0.025 m3 in such 1 bar, where A and B are constants. Sketch
a manner that p(v + 0.030) = constant, where v is all processes on p–V diagram.
in m3. Calculate the work done by the gas.
[(a) 150 kJ, (b) 90 kJ (c) 34.7 kJ
[8.91 J]
(d) 24.8 kJ (e) 19.2 kJ]
8. A gas trapped in a cylinder behind a piston has
14. 0.05 m3 of a gas at 6.9 bar expands reversibly
an initial volume of 0.40 m3 and its pressure
according to the law pV 2 = const. until the volume
is 159 kPa. It is made to undergo a process,
is 0.08 m3. Calculate the work done by the gas
which follows the relationship (V – 0.2) ¥ 105 =
and sketch the p–V diagram. [15.48]
( p – 300)2, until the pressure is 400 kPa. Sketch
15. A non-flow reversible process occurs for which
the p –V diagram of this process and calculate the
pressure and volume are correlated by an expres-
work done. [9.5 kJ]
sion
Energy and Work Transfer 53

2 6 through the walls and windows is estimated to be


p= V + 12000 kJ/h. If the room air is to be maintained
V
where p is in bar and V is in m3. What amount of at a constant temperature of 21°C, determine the
work will be done when the volume changes from number of window air-conditioner units required
2 to 4 m3? [22.83 ¥ 102 kJ] for the classroom. [2 units]
16. The properties of a system during a reversible 21. Fill in the missing data for each of the following
constant pressure non-flow process at 1.8 bar processes of a closed system between states 1 and
changes from 2.
v1 = 0.35 m3/kg, T1= 30°C to v2 = 0.6 m3/kg, (Each quantity is expressed in kJ.)
T2 = 270°C
Process Q W E1 E2 DE
È 75 ˘
If Cp = Í1.5 + ˙ kJ/kg ◊ °C, where T is in (a) 18 –6 35
Î T + 45 ˚
(b) –10 4 –15
°C, calculate Q, W, Du and Dh per kg of mass of
system. (c) 12 3 32
[467.63 kJ/kg, 45 kJ/kg, 422.67 kJ/kg, (d) 25 14
10
467.63 kJ/kg]
17. A domestic refrigerator is loaded with fresh food 22. A gas in a cylinder fitted with a frictionless piston
and the door is closed. During a certain period, undergoes a cycle comprising of three processes.
the machine consumes 1.25 kWh of electrical First, the gas expands at constant pressure with
energy and the internal energy of the food items a heat addition of 400.0 kJ and a work output of
decreases by 4500 kJ. Calculate the magnitude 100.0 kJ. Then, it is cooled at a constant volume
and direction of heat transfer for the system. by removal of 500 kJ of heat. Finally, an adia-
[–9000 kJ] batic process brings the system to its initial state.
18. Water is being heated in a closed pan, while being Determine (a) the work of adiabatic process, and
stirred by a paddle wheel. During the process, (b) the internal energy of the gas at each of two
40 kJ of heat is added to the water and 8 kJ of other states, if initial internal energy of the system
heat is lost to the surrounding air. The paddle is 150.0 kJ. [(a) –200 kJ, (b) 450 kJ, –50 kJ]
wheel work amounts to be 500 Nm. Determine 23. A stone of 25-kg mass is dropped in a tank con-
the final energy of the system, if its initial energy taining 225 kg of water. Initially, the stone is 20 m
is 10 kJ. above the water level and stone and water are in
[42.5 kJ] thermal equilibrium. Determine change in inter-
19. A closed system undergoes a cycle consisting of nal energy, change in potential energy, change in
two processes. During the first process, 40 kJ of kinetic energy, heat transfer and work transfer;
heat is transferred to the system, while the system when
does 60 kJ of work. During the second process, (a) the stone is about to enter the water,
45 kJ of work is done on the system. (b) the stone has come to rest in the tank, and
(a) Calculate the heat transfer during the sec- (c) the heat is transferred to surroundings in
ond process. order to restore the stone and water to their
(b) Determine the net work and net heat trans- initial temperature.
fer for the cycle. [(a) DU = 0, W = 0, Q = 0, DPE = – 4905 J,
[(a) – 25 kJ, (b) 15 kJ, 15 kJ] DKE = 4905 J, (b) DKE = 0, W = 0, Q = 0,
20. A classroom that normally contains 60 students is DPE = – 4905 J, DU = 4905 J, (c) DKE = 0,
to be air-conditioned with window air conditioner W = 0, Q = – 4905 J, DPE = 0, DU = 4905 J]
units of 5 kW rating. A person at rest may be as- 24. The internal energy of system is given by U = 100
sumed to dissipate heat at a rate of 360 kJ/h. There + 50T + 0.04T 2 joules and the heat transfer Q =
are 10 tube lights in the room, each of rating 40 4000 + 16 T joules, where T is the temperature in
W. The rate of heat transferred to the classroom kelvin. If the temperature of the system changes
54 Thermal Engineering

from 300 K to 500 K, calculate the work transfer being 2 bar. The initial and final volumes are
and its direction. 0.2 m3 and 1.2 m3, respectively. The specific
[–13.2 kJ, Work is done on the system] internal energy varies according to u = (1.5p
25. 1.5 kg of a gas at 10 bar pressure undergoes a – 85) kJ/kg, where p in m3/kg.
quasi-static expansion according to p = a + b , work and heat transfer.
where a and b are constants, the final pressure [900 kJ, 90 J]

Objective Questions
1. The unit of work in SI units is (c) balance of qunatity of energy
(a) joule (b) newton (d) measurement of energy transfer
(c) watt (d) calorie 9. Joule’s experiment states that for a cycle
2. The unit of heat in SI units is (a) change of pressure is proportional to tem-
(a) joule (b) newton perature change
(c) watt (d) calorie (b) change of volume is proportional to tem-
3. The unit of power in SI units is perature change
(a) joule (b) newton (c) change of internal energy is proportional to
(c) watt (d) calorie temperature change
4. The unit of energy in SI unit is (d) sum of all heat transfers is proportional to
(a) joule (b) newton sum of all work transfers
(c) watt (d) calorie 10. The law of thermodynamics, which states that
heat and work are mutually convertible is known
5. During a heating process, the temperature of an
as
equivalent to
(a) 20°F (b) 52°F (b) first law of thermodynamics
(c) 36°F (d) 68°F (c) second law of thermodynamics
6. During a cooling process, the decrease of tem- (d) third law of thermodynamics
This temperature 11. The perpetual motion machine of the first kind is
change on the Kelvin scale is impossible according to the
(a) 15 K (b) 298 K
(c) 258 K (d) 59 K (b) first law of thermodynamics
7. The first law of thermodynamics was given by (c) second law of thermodynamic
(d) third law of thermodynamics
(c) Wilson (d) James Watt 12. The heat supplied to a system is considered
8. The first law of thermodynamics deals with (a) negative (b) positive
(a) heat and work
(b) quality of energy

12. (b) 11. (b) 10. (b) 9. (d)


8. (c) 7. (a) 6. (a) 5. (b) 4. (a) 3. (c) 2. (a) 1. (a)
Answers
Working Substances 55

3
Working Substances

Introduction
Thermodynamic analyses require knowledge of properties of the working substance. Steam and ideal gas
are two important working substances discussed in this chapter. In the first part of the chapter, the concept
of pure substance and physics of phase-change phenomenon are discussed with the help of various property
diagrams and p-v-T surface of pure substance. The formats and use of steam tables are demonstrated. In
the second part of the chapter, ideal gas, ideal gas laws, equation of state, characteristic gas equation and
specific gas constant are incorporated. Finally, specific heats are defined and relations are obtained between
two specific heats and for a change in internal energy and enthalpy in terms of specific heat and temperature.
Real gases, compressibility factor and other equations of state are discussed at the end of the chapter.

will contain a higher fraction of nitrogen than in the


original mixture in the gaseous phase. Similarly, if
A pure substance may be defined as a substance, air contains moisture, and it is evaporated on heating
which is chemically homogeneous and has a fixed or condensed on cooling, its chemical composition
chemical composition. In general, any substance will change, it will be no longer a pure substance. A
that appears with invariable chemical composition mixture of carbon monoxide and oxygen is a pure
in either phase or a combination of phases may be substance as long as they do not react to form CO2.
treated as a pure substance. Water, nitrogen, helium,
carbon dioxide, etc., are pure substances. A system
composed of liquid and vapour phases of water
(H2O) is a pure substance, because in either state, A phase is the homogenous part of a system that
the system will always be chemically homogeneous is physically distinct. There are three principle
without any change in its chemical composition. phases of a substance: solid, liquid and gaseous.
A mixture may also be a pure substance. For A substance may have several phases. Within
example, dry air is mainly a mixture of oxygen and the principle phase, each phase has a different
nitrogen. In its gaseous phase, it may be heated, molecular structure. For example, carbon may exist
cooled, compressed or expanded with no change in as graphite or diamond in solid phase. Pure sulphur
its chemical composition. However, if it is liquefied, and iron, each has three solid phases. Helium has
it is no longer a pure substance, because liquid air two liquid phases.
56 Thermal Engineering

A substance in each phase has a definite


molecular structure different from an other phase.
It will be helpful to have some understanding of the
molecular arrangement involved in each phase.
In a solid phase, the molecules are arranged
in a three-dimensional lattice,which is repeated
throughout its volume. The attraction forces of (a) Molecular pattern in a (b) Molecular pattern in a
molecules on each other are very strong and thus, liquids gases
keep the molecules in a fixed position within the
solid as shown in Fig. 3.1.
In the gaseous phase, the molecules are far apart
from each other without any molecular pattern and
force of attraction. The molecules of a gas are in
continuous motion, colliding with each other and
rebounding to move in a new direction.
Molecules in the gaseous phase possess a
considerable higher energy than they do in solid
or liquid phase. Therefore, a gas releases a large
amount of heat before it condenses or freezes.

The molecules in a solid phase cannot move


relative to each other, they can only oscillate Consider a cylinder fitted with a frictionless piston,
about their equilibrium position. The velocity of which may be loaded to any desired pressure. The
molecules during these oscillations depends on cylinder contains 1 kg of ice at –10°C under 1 atm
temperature. When a solid is subjected to a very pressure at the state A as shown in Fig. 3.3. The
high temperature, the velocity of molecules may stages of heat addition are illustrated in Fig. 3.4.
reach a critical point, where the intermolecular
forces become partially weak and a group of Process A B When any amount of heat is added
molecules break away. It is the beginning of the to ice, it gets warmer and its temperature rises till
melting process. it approaches 0.01°C (generally referred as 0°C) as
In the liquid phase, the molecular spacing is shown by the line AB in Fig. 3.4.
similar to that of the solid phase with relatively Process B C The following facts are observed
weaker molecular forces of attraction. In liquid during this process:
phase, the molecules do not confine a fixed
(i) Ice begins to melt at 0°C and a two-phase
position relating to each other, therefore, groups of
mixture is formed.
molecules can float easily.
(ii) The temperature of the two-phase mixture
The distances between molecules in the liquid
(M1) of ice and water does not change on
phase are slightly greater than the distances
heat addition as it is shown by the line BC.
between molecules in the solid phase of the same
substance. Water being the only exceptional case, (iii) There is slight decrease in volume because
which expands on freezing, and its molecular the liquid water at 0°C is heavier than ice.
spacing becomes slightly greater in the solid phase (iv) At the point C, all ice melts to water without
than in the liquid phase. change in pressure (1 atm) and temperature
(0°C).
Working Substances 57

This phase transformation from solid to liquid is Process C D When heat is added to liquid water,
called the melting or fusion of ice. the following facts are observed:
58 Thermal Engineering

(i) The temperature of water rises with heat condensation will take place at higher temperatures;
supply and keeps on rising until it reaches and at lower pressures, these will take place at
boiling point temperature, the point D. lower temperatures. The temperature at which
(ii) There is decrease in the specific volume of the boiling and condensation of a fluid take place
water, when its temperature rises from 0°C is known as saturation temperature, denoted as
to 4°C and thereafter, the specific volume Tsat. The corresponding pressure of fluid is called
increases with temperature rise till it reaches the saturation pressure, denoted as psat. For a
the saturation temperature. The piston moves given pressure, there is a fixed value of saturation
up slightly during this process as shown in temperature. Figure 3.5 shows the variation of
Fig. 3.3. (v). saturation temperature with pressure of fluid.
(iii) The pressure in the cylinder remains constant
at 1 atm.

Process D–E After water reaches saturation tem-


perature (i.e., 100°C at 1 atm), any addition of
heat will cause some liquid to vaporise at the same
temperature. This is again a phase-change process
from saturated water to saturated vapour. During
this phase-change process, the following facts are
observed:
(i) There exists a two-phase mixture M2 of
water and vapour, called the wet steam.
(ii) The temperature of the mixture remains
constant until all water does not convert into
vapour (steam).
(iii) The process of phase change takes place at
constant temperature and constant pressure. Liquid existing at a temperature lower than
(iv) The specific volume of vapour is considerable saturation temperature is called compressed liquid
larger than that of saturated water as shown or sub cooled liquid, because pressure of liquid is
in Fig. 3.3 (vii). higher than its saturation pressure and this liquid is
At the state E, all the water has been vaporised not about to vaporise. The temperature by which it
and this state of steam is called dry and saturated is lower than the saturation temperature is known as
steam. The phase change from liquid to vapour is degree of subcooling, that is
called vaporisation. Degree of subcooling = Tsat – Ti ...(3.1)
Process E F Once the steam becomes dry and sat- where Ti is the initial temperature of fluid. The
urated, it bahaves as an ideal gas and its tempera- liquid at saturation temperature at a given pressure
ture and volume start increasing with further supply is about to vaporise, and is thus called saturated
of heat. This steam is called superheated steam. liquid.

3.3.1 Saturation

At normal atmospheric pressure, the boiling of


water and condensation of steam begin at 100°C. 1. It is the process that involves
Similarly, at higher pressures, the boiling and change of phase from liquid to vapour, when the
Working Substances 59

latent heat of phase change is supplied to saturated


water.
The property diagrams are useful in the study of
2. It is the process of vapour gen-
variation of properties during phase change process.
eration only at the free surface of the liquid. The
We will discuss T–v, p–v and p–T diagrams for
molecules at the free surface extract their latent
pure substances.
heat of phase transformation from the surrounding
medium and break away as vapour from the liquid
T–v
surface and escape to the surrounding atmosphere.
The phase-change phenomenon of water (a pure
3. It is the phenomenon of vapour
substance) at 1 atm pressure is discussed in detail
formation in the whole mass of liquid, when heat
in the last section and plotted on the T–q diagram
is supplied.
in Fig. 3.4. The T–v diagram is very similar to this
4. It is the temperature diagram from the state C to F for liquid and vapour
at which a pure substance starts to evaporate at a states as shown below in Fig. 3.6.
given pressure. We repeat the process of phase transformation
5. p It is the pressure at from liquid water to vapour at different pressures
which a pure substance starts to evaporate at a by adding different weights on the top of the piston.
given temperature. At new pressures, the paths followed by liquid to
vapour phase change, (shown in Fig. 3.7), are very
It is the gases phase of water. similar to the path at 1 atm pressure with following
7. The steam is about to condense. observations:

8. It is the mixture of dry steam and (i) as pressure increases, the saturation temper-
water particles as moisture. ature Tsat for boiling and condensation also
increases.
9. It is a measure of quality of
(ii) As pressure increases, the specific volume
wet steam.
of saturated liquid increases slightly, while
10. Saturated vapour, which the specific volume for saturated vapour
is free from moisture. decreases considerably, thus the saturation
line on the T–v diagram will continue to get
11. The steam existing at higher
shorter.
temperature than its saturation temperature.
(iii) As pressure increases, the temperature for
12. It is the temperature rise of melting and freezing is slightly decreased.
superheated steam above its saturation temperature.
The T–v diagram of Fig. 3.8 is constructed from
13. A locus on the saturation curve, Fig. 3.7 with following considerations:
where saturation liquid line and saturated vapour
line meet. (i) All saturated liquid states are connected by a
solid line, called the saturated liquid line.
14. A locus on the p–T diagram, where (ii) All saturated vapour states are connected
all three phases of water coexist. by another solid line, called the saturated
15. Transformation of solid phase vapour line.
directly into vapour phase is called sublimation. (iii) The saturated liquid line and saturated
vapour line meet at the critical point and
16. It is the solid vapour saturation
form a dome as shown in Fig. 3.8.
line on the p–T diagram.
60 Thermal Engineering

(iv) The region located left to the saturated liquid (vi) The substance can exist in the single phase
line is called the compressed liquid region. only in the compressed liquid or superheated
(v) The region located right to the saturated vapour region as a liquid or a vapour.
vapour line is called the superheated vapour (vii) The region under the dome involves equilib-
region. rium between saturated liquid and saturated
vapour, and is called the wet vapour region.
Working Substances 61

T Critical The T–v and p–v diagrams discussed above


point
p1 represent the equilibrium states involving the liquid
>
t.
ns and vapour phases only. These diagrams can also be
Sa
co

tur
=
line

p2 extended to include the solid phase as well as solid–


ate t.
d ns
Compressed liquid and solid–vapour saturation regions as shown
co
liquid

=
liquid va p1 in Fig.3.10. All the three phases of a pure substance
region po
ur Superheated exist along a line, called the triple line. Along the
vapour
Saturated triple line a substance has the same pressure and
ated

region
liquid + vapour lin
temperature, but different specific volumes. Since
Satur

e
region
the water expands on freezing, therefore, a portion
of the triple line is extended towards the left to the
saturated liquid line.
v
p
v Critical
point

p–v

Solid + Liquid
Superheated

Liquid
The overall shape of a p–v diagram of a pure vapour

substance is very similar to a T–v diagram, except


Saturated
that the constant temperature lines on this diagram liquid + vapour
Solid

have a downward trend. A pressure-specific volume


(p–v) diagram for water is shown in Fig. 3.9. Triple line

Solid + Vapour
v

p–T
The p–T diagram of a pure substance is generally
called the phase diagram, since it shows solid, liq-
uid and vapour regions of a pure substance simulta-
neously. Its salient features are the following:
(i) Each single phase of a pure substance is
separated by saturation lines. The sublimation
line separates the solid and vapour regions,
the vaporisation line separates the liquid and
vapour regions, and the fusion line separates
v
the solid and liquid regions.
It is evident from a p–v diagram, as pressure of (ii) The slope of the fusion line is negative.
a pure substance decreases at constant temperature, It indicates that the melting point of ice
the specific volume of liquid increases marginally, decreases with increasing pressure.
but the specific volume of vapour increases (iii) The sublimation, fusion and vaporisation
considerably. lines meet at point, called triple point . Thus
the triple point can be defined as a locus
where all three phases of a substance coexist.
62 Thermal Engineering

p
Critical point

e
lin p
n
ne

io
at
ion li

Liquid is
region por
Va
Fus

Liquid

Solid

Cr
region Vapour

itic
region

al
e Triple point
lin

iso
on
ati

the
blim Li p=

rm
Su qu
i
co
ns
d
+ tan

T
T Va

=
t
p

co
ou

ns
r
Va

ta
So

nt
lid pou
+ r
The phase (p–T ) diagram of water for some Va T
po
ur
typical pressure and temperature values is shown
in Fig. 3.12. v
v
Critical
point
225 atm

1 atm

Water (liquid)
Pressure

Ice If the pressure of liquid water is continuously


(solid)
increased to such a state where the saturated liquid
Water vapor (gas)
state and the saturated vapour state become identical
0.006 atm Triple point then the saturation line takes the form of a point
as shown in Figs. 3.7–3.10. This point is called the
0.01°C 100°C 374°C
Temperature critical point. Thus, the critical point represents the
maximum boiling point temperature and maximum
saturation pressure for a single-phase system and
above this temperature, the pure substance cannot
be liquefied. At the critical point, the latent heat
p–v–T
of vaporisation of the liquid becomes zero and the
The relationship among pressure, specific volume liquid directly changes to vapour.
and temperature of a pure substance can be The pressure, temperature and specific volume
better understood by the three-dimensional p–v–T of a pure substance at the critical point are called
diagram. critical pressures pcr , critical temperature Tcr
Figure 3.13 shows a surface p–v–T plot for wa- and critical specific volume vcr , respectively. The
ter, which expands on freezing. It shows p–v, p–T critical parameters for water are
and T–v diagrams simultaneously on a three-di- pressure, pcr = 22.09 MPa
mensional plot. A constant temperature line drawn
temperature, Tcr = 374.14 °C
in the figure passing through the critical point is
called critical isotherm. specific volume, vcr = 0.003155 m3/kg
Working Substances 63

The intersection point of the sublimation line, Triple point Critical point
fusion line and vaporisation line on the phase
Substance Pressure Tempera- Pressure Tempera-
diagram (p–T ) represents a point, where all three ptp, kPa ture Ttp, K pcr, MPa ture Tcr, K
phases coexist in equilibrium as shown in Fig. 3.11.
This point is called triple point. On other property Hydrogen 7.000 14.00 5.30 60
diagrams (p–v and T–v diagrams), such a condition Nitrogen 12.500 63.00 3.40 126
is represented by a line called triple line as shown Oxygen 0.150 54.00 5.00 154
in Fig. 3.10. Water 0.611 273.16 22.09 647.29
(0.01°C) (374.14 °C)

T–s h–s

The temperature–entropy (T–s) diagram of a pure


substance is shown in Fig. 3.15 with the following
observations:
1. The absolute temperature data is plotted
along the ordinate, and the specific entropy
The triple point and critical point temperatures data is plotted along the abscissa.
and pressures for various substances are given in 2. The value of specific entropy at triple point
Table 3.1 below: is zero, and thus the saturated liquid line
originates at a temperature of 273.16 K.
64 Thermal Engineering

3. The saturated liquid line and saturated properties of steam with reasonable accuracy, while
vapour line divide the diagram into three analysing the steady flow devices such as a steam
regions, i.e., compressed liquid region left to turbines, nozzles, etc. The use of a Mollier chart
the saturated liquid line, superheated vapour eliminates the complex calculation work and it is
region right to the saturated vapour line and also convenient to use.
the wet vapour region between these two In the enthalpy–entropy chart, the enthalpy
lines. The two saturation lines meet at the is plotted against entropy. The h–s chart covers
critical point. a pressure range from 0.01 bar to 1000 bar and
4. In the compressed liquid region, the con- a temperature up to 800°C. The lines of constant
stant-pressure lines almost coincide with the dryness fraction are drawn in the wet region to
saturated liquid line. the value less than 0.5. The lines of constant
5. In the saturated liquid–vapour mixture re- temperature are drawn in the superheated region.
gion, the constant pressure lines and con- The h–s chart does not show the value of specific
stant temperature lines are horizontal and enthalpy, specific entropy and specific volume for
parallel to each other. saturated water at pressures which are generally
6. In the superheated vapour region, the con- associated with a steam condenser. Hence, this
stant volume lines are steeper than the con- chart is only useful during the expansion process
stant pressure lines. of a steam cycle.
A schematic for h–s diagram is shown in
Fig. 3.16. The constant pressure lines are indicated
The enthalpy–entropy diagram is referred as Mollier by p1, p2, p3 …, etc., the constant temperature lines
diagram. It is most commonly used to obtain the by T1, T2, T3 …, etc., the constant dryness fraction
Working Substances 65

lines by x1, x2, x3 …, etc., and the constant volume 2.


lines are drawn dotted lines as indicated by v1, v2, ( )
v3 …, etc. hfusion = Latent heat of fusion of ice = 334.5 kJ/kg.
Any two independent properties which appear on
the chart are sufficient to define the state of steam. 3.
In the wet region, the pressure p1 and the dryness ( )
fraction x1 define the state A. In the superheated DhC–D = hf = Cpw (Tsat – 0) (kJ/kg) ...(3.3)
region, the pressure p3 and the temperature T4
where Cpw = 4.187 kJ/kg ◊ K, as the specific heat of
define the state B. A vertcal line BC of constant
water. This enthalpy hf for heating of water from
entropy can easily be drawn between the pressure
0°C to its saturation temperature at a given pressure
p3 and the pressure p2 to obtain state C. It consists
or temperature can directly be obtained from steam
of the following features:
tables.
1. The constant temperature lines are straight If water is at the initial temperature of Ti other
and almost horizontal in the superheated than 0°C, then the enthalpy of water is calculated as
vapour region specially at low pressures.
hf = Cpw (Tsat – Ti ) (kJ/kg) ...(3.4)
2. The lines of constant dryness fraction (x)
are also shown in the diagram, which are 4.
parallel to the saturation line. ( )
3. The constant-pressure lines do not change DhD–E = hfg
their shape in either region because for a = Latent heat of vaporisation of water
dh d q in kJ/kg can be obtained from steam
constant-pressure process; = = T.
ds ds tables.
4. The constant-volume lines are steeper than
At normal atmopsheric pressure,
constant-pressure lines.
Tsat = 100°C; hfg = 2257 kJ/kg
5. The compressed-liquid region is not shown
in the diagram. 5.

( )
DhE–F = Cps (Tsup – Tsat) (kJ/kg) ...(3.5)

Figure 3.4 illustrates the formation of steam where Cps = the specific heat of superheated steam,
from –10°C ice to superheated steam. The heat is taken as 2.0 to 2.3 kJ/kg ◊ K,
supplied at constant pressure; thus it can be treated Tsup – Tsat = degree of superheat, i.e., the tem-
as change in enthalpy. During the three stages of perature rise above saturation temperature in K
phase change, the enthalpy changes are as follows:
(or °C).
1. ( It is observed that the phase change of water
) from ice to superheated steam involves two types
DhA–B = hice = Cp,ice (0 – Tice) (kJ/kg) ...(3.2) of pattern of changes, when heat is supplied at
constant pressure.
where
Cp, ice = the specific heat of ice taken as 2.1 kJ/kg ◊ K, 1. Temperature changes when phase does not
change.
Tice = initial temperature of ice.
2. Temperature remains constant, when phase
\ hice = 2.1 × [0 – (–10)] = 21 kJ/kg changes.
66 Thermal Engineering

Further, as a result of heat input, the molecular amount of energy. No temperature rise is recorded
arrangement of the working substance (water) also during this process. Therefore, the specific heat
changes from strong forces of attraction between of the water during vapourisation is also assumed
molecules in solid phase to free molecules in infinity.
gaseous phase. These inputs can also be defined as
follows: hg
It is the sum of enthalpy of saturated water and
enthalpy of vaporisation. It may be defined as the
The latent heat of fusion is defined as the quantity amount of heat required to convert 1 kg of water
of heat, required to convert one kg of ice into water at 0°C into dry and saturated steam at a given
at constant temperature (0°C). Its value is taken as pressure. It is designated as hg and
334.5 kJ/kg. hg = hf + hfg (kJ/kg) ...(3.6)
The amount of heat added during the fusion
process is used to break up strong molecular bonds hsup
in the solid phase into relatively weaker bonds in the
It can be defined as the amount of heat required
liquid phase and giving them a considerable amount
to convert 1 kg of water at 0°C into superheated
of energy for their movement. No temperature rise
steam at constant pressure. Normally, it is the sum
is recorded during this process. The specific heat
of enthalpy of dry and saturated steam and heat
during fusion is treated as infinity.
supplied during superheating of steam. Thus
hsup = hg + Cps (Tsup – Tsat) (kJ/kg) ...(3.7)
hf
Sensible heat or enthalpy of saturated water is
defined as an amount of heat energy absorbed by When saturated steam contains saturated water
1 kg of water during its heating from 0°C to the particles evenly distributed in saturated vapour, it is
saturation temperature (Tsat ) at a given pressure. called the wet steam. The wet steam is charecterised
It is designated as hf in the steam tables and can by its dryness fraction.
directly be obtained from steam tables.

hfg The vapour fraction in the wet steam is considered


dryness fraction and designated as x. The dryness
It is defined as an amount of heat energy requied fraction of steam is defined as ratio of mass of
to convert 1 kg of saturated water into dry and actual dry and saturated vapour to total mass of
saturated steam keeping the temperature and steam considered.
pressure constant. The magnitude of enthalpy of
If mg is the mass of dry and saturated steam and
vaporisation or latent heat decreases as the pressure
mw is the mass of saturated water in suspension
of water increases and becomes zero at the critical
in steam considered then dryness fraction x is
point. It is denoted by hfg and is directly obtained
expressed as
from steam tables. mg
The heat added during vaporisation is also used dryness fraction, x = ...(3.8)
mg + mw
to break up molecular bonds in liquid phase, make
the molecules free and give them a considerable Wetness fraction is another term associated
with wet steam. It is defined as the ratio of mass
Working Substances 67

of moisture in suspension with total mass of steam, (Tsup– Tsat) = temperature rise is called degree of
which contains it. It is denoted by y and expressed superheat.
as Then total heat of superheated steam
mw mg
Wetness fraction, y = =1– hsup = hf + hfg + Cps (Tsup – Tsat) kJ/kg
mg + mw mg + mw
= hg + Cps (Tsup – Tsat) (kJ/kg) ...(3.12)
=1–x ...(3.9a)
The superheated steam behaves like a perfect
gas and therefore, it follows the gas law pvn =
constant. The value of n (index) for superheated
It is the representation of wetness fraction in per- steam is generally assumed as 1.3.
centage.
Priming = (1– x) ¥ 100 ...(3.9b)

All power plants as well as industries use super-


When steam is not completely dry then its enthalpy heated steam. The advantages of the use of super-
of vaporisation is less than that of dry and saturated heated steam are the following:
steam and it cannot be obtained from steam tables.
1. The heat content of superheated steam
Enthalpy of wet steam is calculated as
is large and it does more work without
Enthalpy of wet steam = Enthalpy of saturated water increasing pressure drop.
+ dryness fraction ¥ Enthalpy of vaporisation 2. Due to large heat content of superheated
or hwet = hf + x hfg (kJ/kg) ...(3.10) steam, the steam consumption for a given
output is less. It results into reduction in size
and weight of the system.
3. The high temperature of superheated steam
always results in improved thermal efficiency.
When water is completely converted into steam, 4. Since superheated steam is free from mois-
the resulting steam is called dry and saturated ture, its use in an apparatus for curing, dry-
steam. When the dry and saturated steam is ing, etc., avoids corrosion and erosion of the
further heated, its temperature increases with surfaces.
corresponding increase in its specific volume. This 5. When superheated steam is used in a steam
steam is called superheated steam and this process engine or steam turbine, it can be expanded
is called superheating. The superheating is carried considerably to do work before it becomes
out at constant pressure. The additional amount wet.
of heat supplied to steam during superheating is
called superheat, and the temperature rise of steam
above saturation temperature is called degree of
superheat. The volume occupied by unit mass of a fluid is
Heat added during superheating called specific volume of fluid. It is expressed in
qsuperheating = Cps (Tsup – Tsat) (kJ/kg) ...(3.11) cubic metre per kg of the fluid and it is denoted
by v.
where For dry and saturated steam, the specific volume
Cps = specific heat of superheated steam, is designated as vg and the specific volume during
which varies from 2.0 to 2.3 kJ/kg ◊ K.
68 Thermal Engineering

evaporation is designated as vfg. The specific


volume of steam decreases with increase in pressure
dq
ÚT
and it can directly be obtained from steam tables It is calculated as between two states. The
corresponding to saturation pressure or saturation
temperature. temperature– entropy diagram in Fig. 3.17 shows
the changes in entropy of a pure substance.

The wet steam is a mixture of dry vapour and


moisture. Its specific volume is the sum of moisture
volume and change of volume during evaporation.
If 1 kg of wet steam has a dryness fraction x then
vwet = vf + x vfg
= vf + x (vg – vf )
where
vg =specific volume of dry and saturated
steam
vf = specific volume of moisture (water)
vfg = specific volume change of steam during
evaporation = vg – vf
\ vwet = x vg + (1 – x) vf ...(3.13)
It is noticed that the volume of moisture at low
pressures is very small, and is generally dropped
from the expression. Thus Let 1 kg of water be heated from temperature T1
to T2 at constant pressure. The change in entropy
vwet ª x vg ...(3.14) is given by
2 dq ÊT ˆ
Ds = Ú1 T
= C pw ln Á 2 ˜ (kJ/kg ◊ K) ...(3.16)
Ë T1 ¯
Since superheated steam behaves like a perfect gas
and superheating is carried at constant pressure, The entropy of saturated water, sf from 273 K
thus it follows the property relation to Tsat can be obtained directly from the steam
tables or it can be calculated as
vg vsup
= ÊT ˆ
Tsat Tsup sf = C pw ln Á sat ˜ (kJ/kg ◊ K) ...(3.17)
Ë 273 ¯
Tsup
or vsup = v g ¥ (m3/kg) ...(3.15)
Tsat
where The specific entropy change during evaporation
Tsup = Temperature of superheated steam in K, is denoted by sfg for dry and saturated steam and
obtained from steam tables. Mathematically, for
Tsat = Temperature of dry and saturated steam in K.
dry and saturated steam,
h fg
sfg = (kJ/kg ◊ K) ...(3.18)
Tsat
Working Substances 69

For wet steam, the heat supplied during evapora- piston is displaced due to change in its volume as
tion is x hfg at saturation temperature Tsat (K). Then shown in Fig. 3.18 (b) . Thus, external work is done
the change of entropy during evaporation, by steam due to increase in specific volume. This
x h fg work produced is called the external work done
swet = x sfg = ...(3.19) during evaporation
Tsat

sg
It is the total entropy of steam.

a Entropy Change of Dry and Saturated Steam It


is the sum of entropy of saturated water sf , and the
entropy of evaporation sfg. In other words,
sg = sf + sfg (kJ/kg ◊ K) ...(3.20)
It can be directly obtained from steam tables.

b Entropy Change of Wet Steam It is the sum


of entropy of saturated water sf and the entropy of
partial evaporation. In other words,
swet = sf + x sfg (kJ/kg ◊ K) ...(3.21)

c Entropy Change of Superheated Steam The


entropy change during superheating of 1 kg of dry
steam from Tsat to Tsup at constant pressure
TSup dq Tsup dT
ssuperheating =
Ú
TSat T
= C ps Ú
Tsat T
The external work done during evaporation
ÊT ˆ
= C ps ln Á sup ˜ ...(3.22) wex = p (vg – vf ) = p vfg ...(3.24)
Ë Tsat ¯
Then the total entropy of superheated steam Since, vf < < vg at low pressure, it is neglected.
above the freezing point temperature of water Then for dry and saturated steam
wex ª p vg (kJ/kg) ...(3.25)
Ê Tsup ˆ
ssup = sf + sfg + C ps ln Á
Ë Tsat ˜¯ For wet steam with a dryness fraction x, the
work of evaporation is
Ê Tsup ˆ
= sg + C ps ln Á wwet ª p (x vg) (kJ/kg) ...(3.26)
Ë Tsat ˜¯
...(3.23)
Note. The pressure p should be expressed in kPa in
Note. The temperatures must be used in Kelvin (K). the above relations.

Actually, the latent heat or enthalpy of evaporation


The phase transformation of saturated water to is always more than the external work done during
saturated steam takes place at constant pressure, evaporation. This excess amount of latent heat
its specific volume increases from vf to vg and the is called internal latent heat. It is required to
70 Thermal Engineering

overcome the internal molecular forces to change The uf, ufg, ug and usup can also be obtained from
the phase of substance. the steam tables.
Mathematically, the internal latent heat
= latent heat – external work of evaporation
For dry and saturated steam, The laws of perfect gases are not applicable to
internal latent heat = hfg – pvg ...(3.27) vapours, hence the variation between their properties
are obtained from charts and tables. Experimentally
For wet steam determined thermodynamic properties of water
internal latent heat = x hfg – p (xvg) ...(3.28) are presented in three different forms of tables as
follows:
In this table,
temperature is chosen as an independent variable
The internal energy of steam (u) is actually heat
and properties of coexisting liquid and vapour
energy stored in steam above the freezing point
phases are presented for saturated water/steam.
temperature of water, and it is the sum of sensible
Table 3.2 shows a portion of such a steam table.
heat and internal latent heat at a given pressure p in
kPa. In other words, In this table, the
Internal energy of steam pressure is chosen as an independent variable
and other properties are presented against it for
= Sensible heat + Internal latent heat
saturated water/steam as shown in Table 3.3.
Total heat of steam
= Sensible heat + Internal Latent 3. In this table, the
heat + External work done pressure and temperature are two independent
h = Internal energy (u) + pv variables and properties of superheated steam can
Hence u = h – pv be obtained against these variables as shown in
Table 3.4.
For dry steam, ug = hg – p vg ...(3.29)
For wet steam uwet = (hf + x hfg) – p (x vg)
= x ufg ...(3.30) 1. The triple point of water (0.01°C, 0.611 kPa)
For superheated steam, is chosen as reference state for presenting
usup = hg + Cps (Tsup – Tsat) data in the steam tables. At triple point,
the internal energy uf and entropy sf for
– pvg ¥ Tsup ...(3.31) saturated liquid is arbitrarily assigned a zero
Tsat value.

Sat. Specific volume Specific enthalpy Specific entropy


Temp.
pressure (m3/kg) (kJ/kg) (kJ/kg ◊ K)
T (°C) psat (kPa) vf vg hf hfg hg sf sg
0.01 0.6113 0.001000 206.14 0.01 2505.3 2505.4 0.000 9.1562
100 101.325 0.001044 1.6729 419.02 2257.03 2676.05 1.3068 7.3548
200 1553.8 0.001157 0.12736 852.45 1938.48 2790.93 2.3309 6.4323
374.14 22090 0.003155 0.003155 2099.3 000.00 2099.3 4.4298 4.4298
Working Substances 71

Abs. Sat Specific volume Specific enthalpy Specific entropy


pressure temp. m3/kg kJ/kg kJ/kg ◊ K
p, kPa Tsat °C
vf vg hf hfg hg sf sg
0.6113 0.01 0.001000 206.14 0.01 2501.3 2501.4 0.000 9.1562
100 99.63 0.001043 1.6940 417.46 2258.0 2675.46 1.3026 7.3594
1000 179.91 0.001127 0.19444 762.81 2014.29 2777.10 2.1386 6.5864
22090 374.14 0.003155 0.003155 2029.3 0 2029.3 4.4298 4.4298

Sp. volume Sp. Enthalpy Sp. Entropy


Temp. v h s
T °C m3/kg kJ/kg kJ/kg ◊ K

p = 10 kPa (45.81 °C)


Sat 14.674 2584.63 8.1502
50 14.869 2592.56 8.1749
100 17.196 2687.46 8.4479
150 19.512 2782.99 8.6881
200 25.825 2879.52 8.9037
1300 72.602 5409.70 15.5810

2. The choice of steam table for saturated steam 5. The steam tables give the values of properties
is immaterial, because every table gives the above 0°C. Hence if the initial temperature
same data of saturated liquid and saturated of water is other than 0°C then the initial
vapour. enthalpy of water can be obtained from the
temperature entry steam table at a given
temperature. The enthalpy of fluid is then
obtained by deducting the initial enthalpy
from the total enthalpy of steam.
1. The steam table gives the properties on
per kg basis. For the different masses they
should be multiplied by the given mass.
2. If the value of the independent property of
the column 1, i.e., temperature or pressure is Steam, as a working substance, offers the following
not included in steam tables, the properties advantages and applications.
should be obtained by linear interpolation.
Advantages
3. A meagre negligible variation in answers is
inevitable due to usage of different steam 1. It is capable to supply process heat at
tables. constant temperature while condensing.
4. In case the nature of pressure is not specified, 2. It is cheap, and can be produced everywhere.
it should be assumed absolute pressure. 3. It is a clean, odourless and tasteless source
of heat energy.
72 Thermal Engineering

4. It can be used repeately again and again as Calculate volume, density, enthalpy,
well as first used for power generation and and entropy of 2 kg of steam at 80°C and having a
then for process heating. dryness fraction of 0.85.
5. Its flow rate can easily be controlled and
readily distributed. Solution

Main Applications Given Wet steam


1. The heat content of steam is large and thus m = 2 kg T = 80°C x = 0.85
it is suitable for process heating (for curing, To find
drying, etc.) in many industries like sugar (i) Volume, (ii) Density,
mills, textile mills, chemical industries, etc. (iii) Enthalpy, and (iv) Entropy.
2. It is also used for power generation in
Properties of water At 80°C;
thermal power plants.
3. Due to its high heat content, steam is used psat = 47.39 kPa vf = 0.001029 m3/kg
for cooking many items like steam rice, vg = 3.40715 m3/kg hf = 334.88 kJ/kg
idlies, etc. hfg = 2308.77 kJ/kg ◊ K sf = 1.0752 kJ/kg ◊ K
4. Steam can also be used for heating buildings sfg = 6.5369 kJ/kg ◊ K
and producing hot water in winter. Analysis
5. Steam is also used for creation of vacuum,
(i) At 80°C, the specific volume of steam;
ejection of gases and sterlisation.
v = (1 – x) vf + x vg ª x vg
Example 3.1 Calculate specific enthalpy, specific = 0.85 ¥ 3.40715 = 2.896 m3/kg,
volume and density of 1 kg of steam at a pressure of 1.9 For m = 2 kg;
MPa, having a dryness fraction 0.85. V = m v = 2 ¥ 2.896 = 5.792 m3
Solution (ii) The density of steam;
1 1
Given 1 kg of wet steam r= = = 0.345 kg/m3
v 2.896
p = 1.9 MPa, x = 0.85
(iii) Specific enthalpy of steam;
To find hwet = hf + x hfg = 334.88 + 0.85 ¥ 2308.77
(i) Total he
at, (ii) Volume, (iii) eDnsity. = 2297.33 kJ/kg
Properties of steam At p = 1.9 MPa = 19 bar; Total enthalpy
H = m hwet = 2 ¥ 2297.33 = 4594.66 kJ
Tsat = 209.80°C vf = 0.001172 m3/kg,
vg = 0.10467 m3/kg hf = 896.8 kJ/kg, (iv) Specific entropy of steam;
hfg = 1899.3 kJ/kg swet = sf + x sfg = 1.0752 + 0.85 ¥ 6.5369
Analysis = 6.631kJ/kg ◊ K
Total entropy
(i) The specific enthalpy (total heat) of steam
S = m swet = 2 ¥ 6.631 =13.26 kJ/K
h = hf + x hfg
= 896.8 + 0.85 ¥ 1899.3 = 2511.20 kJ/kg Example 3.3 Calculate volume, density, enthalpy,
(ii) The specific volume of 1 kg of steam and entropy of 2 kg of water at 2 bar and 80°C.
v = (1 – x) vf + x vg ª x vg
= 0.85 ¥ 0.10467 = 0.0889 m3 Solution
(iii) The density of steam Given Water
1 1 T = 80°C p = 2 bar m = 2 kg
r= = = 11.248 kg/m3
v 0.0889
Working Substances 73

To find hsup2 = hsup1 + Cps (Tsup2 – Tsup1)


(i) Volume, (ii) Density, or 2875.27 = 2776.38 + Cps (200 – 150)
(iii) Enthalpy, and (iv) Entropy.
2875.27 - 2776.38
or Cps = =1.978 kJ/kg ◊ K
Assumption The specific heat of water as 4.187 kJ/kg ◊ K. 50
Properties of water At T = 80°C Identify the type of steam in the
psat = 47.39 kPa following three cases, using the steam tables and giving
vf = 0.001029 m3/kg necessary calculations supporting your claim.
Analysis At 80°C, water pressure is 2 bar, thus it is (a) 2 kg of steam at 8 bar with an enthalpy of 5538.0
compressed liquid. kJ at a temperature of 170.4°C
(i) At 80°C, the specific volume of water (b) 1 kg of steam at 2550 kPa occupies a volume of
0.2742 m3. Also, find the steam temperature.
vf = 0.001029 m3/kg
(c) 1 kg of steam at 60 bar with an enthalpy of
For m = 2 kg; 2470.73 kJ/kg.
V = m vf = 2 ¥ 0.001029 = 0.002058 m3
Solution
(ii) The density of water
1 1 Case (i): Given The steam
r= = = 971.82 kg/m3
v f 0.001029 m = 2 kg p = 8 bar
(iii) The enthalpy of water H = 5538 kJ Tsat = 170.4°C
H = m Cpw (Twater – 0) To find Quality of steam.
= 2 ¥ 4.187 ¥ (80°C – 0) = 669.92 kJ
Analysis The properties of steam at 8 bar;
(iv) Entropy of water
Tsat = 170.4 °C hf = 721.1, kJ/kg
+ 273 ˆ
S = mCpw ln ÊÁ water
T
hfg = 2048.04 kJ/kg hg = 2769.13 kJ/kg
Ë 0 + 273 ˜¯
The specific enthalpy of a given mass of steam
Ê 80 + 273 ˆ H 5538
= 2 ¥ 4.187 ¥ ln Á
Ë 0 + 273 ˜¯
= 2.152 kJ/K = = 2769.0 kJ/kg
h=
m 2
Example 3.4 Using steam tables, determine the mean The specific enthalpy of given steam is approximately
specific heat for superheated steam at 1 bar between equal to the total enthalpy of vapour at saturation
temperatures of 150°C to 200°C . temperature, and therefore, the given sample of steam is
dry and saturated.
Solution Case (ii): Given The steam

Given Superheated steam m = 1 kg p = 2550 kPa v = 0.2742 m3

p = 1 bar = 100 kPa, Tsup1 = 150°C, Tsup2 = 200°C To find Quality of steam.

To find Mean specific heat of superheated steam Analysis The properties of steam at 2550 kPa

Properties From superheated steam table at 1 bar Tsat = 225 °C vg = 0.07835 m3/kg

At Tsup1: hsup1 = 2776.38 kJ/kg The given volume of steam is more than the volume
At Tsup2: hsup2 = 2875.27 kJ/kg. of vapour at saturation temperature. Therefore,
Tsup
Analysis The enthalpy of superheated steam can be v = vg ¥
Tsat
expressed as
74 Thermal Engineering

Tsup The specific internal energy of wet steam


or 0.2742 = 0.07835 ¥
(225 + 273) uwet = hwet – p (x vg)
or Tsup = 1742.84 K = 1469°C = 2475.8 – (10 ¥ 100 kPa)
¥ 0.85 ¥ (0.19444 m3/kg)
The given sample of steam is superheated.
= 2310.52 kJ/kg
Case (iii): Given The steam (ii) For dry steam;
m = 1 kg p = 60 bar h = 2470.73 kJ/kg u g = h g – p vg
= 2778.08 – (10 ¥ 100) ¥ 0.19444
To find Quality of steam. = 2583.64 kJ/kg
Analysis The properties of steam at 60 bar (iii) When steam is superheated,
Tsup – Tsat = 50°C
Tsat = 275.64 °C hf = 1213.32 kJ/kg
Tsup = 179.9 + 50 = 229.9°C
hfg = 1571.0 kJ/kg hg = 2784.33 kJ/kg
The specific enthalpy of superheated steam
The enthalpy of the given steam is less than the total hsup = hg + Cps (Tsup – Tsat)
enthapy of steam at saturation temperature. Therefore,
= 2778.08 + 2.01 × 50 = 2878.58 kJ/kg
h = hf + x hfg Specific volume of superheated steam
or 2470.73 = 1213.32 + x ¥ 1571.0 229.9 + 273
vsup = 0.19444 ¥ = 0.216 m3/kg
or x = 0.8 179.9 + 273
The given sample of steam is wet. The internal energy of superheated steam
usup = hsup – p vsup
Find the internal energy of 1 kg of steam = 2878.58 – (10 ¥ 100) ¥ 0.216
at a pressure of 10 bar, when the condition of steam is = 2662. 67 kJ/kg
(a) wet with a dryness fraction of 0.85,
Example 3.7 Using the data given below, calculate
(b) dry and saturated, and
the volume, enthalpy and internal energy of 2 kg of steam
(c) superheated, the degree of superheat being 50°C. at 10.3 bar pressure in each of the following states:
The specific heat of superheated steam at constant (a) Dryness fraction of 0.85, (b) dry and saturated steam,
pressure is 2.01 kJ/kg ◊ K. and (c) at a temperature of 220° C.

Sat. Temp Sp. volume Enthalpy Enthalpy


Solution
Pressure, Tsat °C of dry steam of liquid of vapour
Given 1 kg of steam at 10 bar with three conditions p, bar
vg cc/gm hf kJ/kg hg kJ/kg
(i) x = 0.85 10.02 180 194.10 763.22 2278.2
(ii) x = 1 10.50 182 186.00 772.20 2779.9
(iii) Tsup – Tsat = 50°C, and Cps = 2.01 kJ/kg ◊ K
Solution
To find The internal energy of steam in each case
Analysis Using steam table at 10 bar pressure Given Three types of steam with

Tsat = 179.9°C vg = 0.19444 m3/kg m = 2 kg p = 10.3 bar


hf = 762.79 kJ/kg hfg = 2015.3 kJ/kg Case 1. Wet steam with x = 0.85,
hg = 2778.08 kJ/kg 2. Dry and saturated steam, and
(i) When steam is wet and the dryness fraction is 0.85 3. Superheated steam at 220°C.
The specific enthalpy of steam Tabulated data as
hwet = hf + x hfg = 762.79 + 0.85 ¥ 2015.3 p1 = 10.02 bar p2 = 10.5 bar
= 2475.8 kJ/kg Tsat1 = 180° C Tsat2 = 182° C
Working Substances 75

vg1 = 194.10 cc/gm vg2 = 186.00 cc/gm Now the data in tabulated form:
hf1 = 763.22 kJ/kg hf2 = 772.20 m3/kg
p bar Tsat °C vg cc/gm hf kJ/kg hg kJ/kg
hg1 = 2278.2 kJ/kg hg2 = 2779.9 kJ/kg
10.02 180.00 194.10 763.22 2278.2
To find In each case of steam 10.3 181.1667 189.375 768.458 2570.85
(i) Volume of steam in m3, 10.5 182.0 186.00 772.20 2779.9
(ii) Enthalpy of steam in kJ, and
Case 1: Wet steam with dryness fraction x = 0.85
(iii) Internal energy of steam in kJ.
(i) Volume of steam in m3
Analysis In tabulated data, the data against a pressure V = m (x vg)
of 10.3 bar is not given. Hence, we have to obtain the data
against 10.3 bar pressure by interpolating the available 10- 6
= 2 ¥ (0.85 ¥ 189.375) ¥ = 0.322 m3
data. Using linear interpolation as 10- 3
y2 - y1
yp = y1 +
p2 - p1
( p - p1 ) (ii) Enthalpy of steam
H = m [hf + x (hg – hf)]
where yp is the unknown data to calculate against = 2 ¥ [768.458 + 0.85 ¥ (2570.85 – 768.458)]
p = 10.3 bar = 2 ¥ [768.458 + 1709.13] = 4601 kJ
(a) Saturation temperature, Tsat (iii) Internal energy U = H – p V
Tsat2 - Tsat1 = 4601 – (10.3 ¥ 100) ¥ 0.322
Tsat = Tsat1 +
p2 - p1
( p - p1 ) = 4269.33 kJ
182 - 180 Case 2: Dry and saturated steam
= 180 + ¥ (10.3 - 10.02)
10.5 - 10.02 10- 6
(i) V = m vg = 2 ¥ 189.375 ¥ = 0.378 m3
= 180 + 1.1667 = 181.1667° C 10- 3
(b) Specific volume vg (ii) H = m hg = 2 ¥ 2570.85 = 5141.7 kJ
v g 2 - v g1 (iii) U = H – p V
vg = v g1 +
p2 - p1
( p - p1 ) = 5141.7 – (10.3 ¥ 100) ¥ 0.378 = 4752.36 kJ

186.00 - 194.10 Case 3: Superheated steam at 220° C


= 194.10 + ¥ (10.3 - 10.02)
10.5 - 10.02 Tsup
= 194.10 – 4.725 = 189.375 cc/g (i) V = m vg ¥
Tsat
(c) Specific enthalpy of liquid hf
Ê 10-6 ˆ Ï 220 + 273 ¸
h f 2 - h f1 = 2 ¥ Á189.375 ¥ ˜ ¥Ì ˝
hf = h f1 + ( p - p1 ) Ë 10-3 ¯ Ó181.1667 + 273 ˛
p2 - p1
= 0.411 m3
772.20 - 763.22
= 763.22 + ¥ (10.3 - 10.02) (ii) H = m [hg + Cps (Tsup – Tsat)]
10.5 - 10.02
= 2 ¥ [2570.85 + 2.2 ¥ (220 – 181.1667)]
= 763.22 + 5.238 = 768.458 kJ/kg = 5312.66 kJ
(d) Specific enthalpy of vapour, hg (iii) U = H – pV =5312.66 – (10.3 ¥ 100) ¥ 0.411
hg 2 - hg1 = 4889.2 kJ
hg = hg1 +
p2 - p1
( p - p1 )
Calculate the amount of heat to be
2779.9 - 2278.2
= 2278.2 + ¥ (10.3 - 10.02) supplied to produce 5 kg of steam at a pressure of 8 bar
10.5 - 10.02 and a temperature of 320°C from water at 30°C. Take
= 2278.2 + 0.9916 = 2570.85 kJ/kg specific heat for superheated steam as 2.2 kJ/kg ◊ K.
76 Thermal Engineering

Solution (iii) Internal energy of superheated steam, and


(iv) Change in internal energy.
Given Generation of steam from water
Assumptions
m = 5 kg p = 8 bar Tsup = 320°C
T1 = 30°C Cp s = 2.2 kJ/kg ◊ K (i) Specific heat of water as 4.187 kJ/kg K.
(ii) Specific heat of superheated steam as 2.2 kJ/kg ◊ K.
To find The amount of heat supplied to make the steam
superheated. Analysis The properties of steam at 10 bar;

Assumptions Tsat = 179.91°C hf = 762.81, kJ/kg


hfg = 2014.3 kJ/kg vg = 0.19444 m3/kg
(i) The specific heat of water is 4.187 kJ/kg ◊ K.
vf = 0.001127 m3/kg hg = 2777.08 kJ/kg
(ii) The heat is supply at constant pressure.
(i) Heat supplied per hour in the boiler
Analysis At 8 bar, the properties of steam The heat is already available with 1 kg of water at
Tsat = 170.43 °C hg = 2768.4 kJ/kg 30°C;
Amount of heat (enthalpy) required by 1 kg of water from h1 = Cpw (T1 – 0) = 4.187 ¥ (30 – 0)
0°C for its superheating to 320 °C; = 125.61 kJ/kg
hsup = hg + Cps (Tsup – Tsat) Amount of heat required by 1 kg of water to
= 2768.4 + 2.2 ¥ (320 – 170.43) generate steam 0.97 dry from 0°C.
= 3097.52 kJ/kg h2 = hf + x hfg = 762.81 + 0.97 ¥ 2014.3
Water is initially at 30°C, the amount of heat = 2716.65 kJ/kg
(enthalpy) already with water at 30°C; Net amount of heat required by 1 kg of water in
hi = Cpw (Ti – 0) = 4.187 ¥ (30 – 0) = 125.61 kJ/kg the boiler;
h3 = h2 – h1
The net amount of heat/kg (enthalpy) required
= 2716.65 – 125.61 = 2591.04 kJ/kg
= hsup – hi = 3097.52 – 125.61 = 2971.9 kJ/kg
Total amount of heat required in the boiler per hour
The total amount of heat required
= m h3 = 1000 ¥ 2591.04
= 5 ¥ 2971.9 = 14859.55 kJ
= 2591.04 ¥ 103 kJ/h = 2591.04 MJ/h
A certain boiler generates 1000 kg of (ii) Heat supplied in the superheater per kg of steam
steam per hour at 10 bar and 0.97 dry. This steam is then h4 = Heat supplied to make steam dry and saturated
taken to a superheater and is heated to a temperature of + Heat supplied to make steam superheated.
280°C keeping the pressure constant. The feed water is
Alternatively:
available at 30°C. Find
h4 = Enthalpy of superheated steam/kg
(a) Heat supplied per hour in the boiler, – Enthalpy of steam/kg generated in boiler.
(b) Heat supplied per hour in the superheater,
Choosing alternate formula:
(c) Internal energy of superheated steam,
Enthalpy of superheated steam per kg
(d) Change in internal energy of steam.
hsup = hg + Cps (Tsup – Tsat)
Solution = 2777.08 + 2.2 ¥ (280 – 179.91)
Given Generation of steam in a boiler = 2997.30 kJ/kg
m = 1000 kg/h p = 10 bar x = 0.97 Then h4 = hsup – h3 = 2997.30 – 2591.04
Tsup = 280 °C T1 = 30°C p = constant = 406.26 kJ/kg
Total amount of heat supplied in superheater per
To find
hour
(i) Heat supplied per hour in the boiler,
= m ¥ h4 = 1000 ¥ 406.268
(ii) Heat supplied per hour in superheater, = 406268 kJ/hr = 406.26 MJ/h
Working Substances 77

(iii) Internal energy of superheated steam (b) As a lower limit, if the steam is completely
usup = hsup – p vsup wet, the mass of vapour will be zero. Then
Tsup the above relation yields to zero dryness.
where vsup = vg ¥ (iii) Superheated steam behaves as a perfect gas, and
Tsat
thus follows the property relation
Ê 280 + 273 ˆ vg vsup
= 0.19444 ¥ ÁË ˜ =
179.91 + 273 ¯ Tsat Tsup
= 0.2374 m3/kg Tsup
or vsup = v g ¥ (m3/kg)
Then usup = (2997.30 kJ/kg) Tsat
– (10 ¥ 100 kPa) ¥ (0.2374 m3/kg) where
= 2759.89 kJ/kg Tsup = temperature of superheated steam in K,
(iv) Change in internal energy Tsat = temperature of dry and saturated steam in K,
Initial internal energy of water,
and Tsup > Tsat. Therefore, the specific volume
u1 = hf – p vf of superheated steam will be greater than the
or u1 = 125.61(kJ/kg) – (10 ¥ 100 kPa)
specific volume of saturated steam.
¥ 0.001127 m3/kg
= 124.483 kJ/kg.
The change in internal energy
usup – u1 = 2759.89 – 124.483 = 2635.4 kJ/kg

Prove the following statements: There are many calorimeters used to determine the
dryness fraction of wet steam. We shall discuss the
(a) Temperature of wet steam equals that of dry and
following calorimeters in this text:
saturated steam at same pressure.
(b) Dryness fraction of steam does not go below zero 1. Barrel Calorimeter
or above unity. 2. Separating Calorimeter
(c) Specific volume of superheated steam is greater 3. Throttling Calorimeter
than that of dry and saturated steam at the same 4. Combined separating and throttling calori-
pressure. meter
Solution
(i) Wet steam is produced during the phase change
of water and only latent heat of vaporisation is The barrel calorimeter is shown in Fig. 3.19. A
supplied during this process. This latent heat known mass of steam at a pressure, p is condensed
is used to overcome the molecular forces of in the presence of a known quantity of water filled in
attraction and the specific heat of water becomes a barrel calorimeter. As steam condenses, the mass
infinity during phase change. Therefore, the and temperature of water increase. The amount of
temperature of wet steam equals that of dry and heat lost by wet steam and amount of heat received
saturated steam at the same pressure. by water in the calorimeter are equated to obtain the
(ii) The dryness fraction is defined as dryness fraction of wet steam as given below:
mass of dry vapour Heat lost by wet steam
x=
mass of dry vapour + mass of moisture = msteam [(x hfg) + Cpw (Tsat – T2)] ... (i)
(a) As an upper limit, if the steam is completely Heat gained by water in the calorimeter
dry, the mass of moisture will be zero. Then
the above relation yields to unity dryness. = (mc Cpc + mw Cpw) (T2 – T1) ... (ii)
78 Thermal Engineering

Solution

Given Wet steam with


p1 = 5.5 bar mw = 80 kg
T1 = 25 °C mc = 10 kg
T2 = 40 °C Cpc = 0.406 kJ/kg ◊ K
mw + msteam = 82.5 \ msteam = 2.5 kg
To find Dryness fraction of wet steam.
Assumption The specific heat of water as 4.187 kJ/kg ◊ K
Analysis The enthalpy of vaporisation and saturation
temperature at 5.5 bar
hfg = 2097 kJ/kg, Tsat = 155.48 °C
The dryness fraction of steam
Equating (i) and (ii), we get the dryness fraction. (mcC pc + mwC pw )(T2 - T1 ) - msteam C pw (Tsat - T2 )
x=
(mc C pc + mwC pw )(T2 - T1 ) - msteam C pw (Tsat - T2 ) msteam h fg
x=
msteam h fg (10 ¥ 0.406 + 80 ¥ 4.187) ¥ (40 – 25)
x=
...(3.32) 2.5 ¥ 2097.0
2.5 ¥ 4.187 ¥ (155.48 - 40)
where mc = mass of the barrel calorimeter, kg –
2.5 ¥ 2097.0
Cpc = specific heat of the barrel calorimeter,
= 0.749
kJ/kg ◊ K
mw = mass of water filled in the calorimeter, kg
Cpw = specific heat of water, kJ/kg ◊ K
msteam = mass of steam condensed, kg The separating calorimeter separates water particles
from wet steam mechanically by centrifugal action.
hfg = latent heat of steam at given pressure p,
The steam passing into the calorimeter is forced
kJ/kg
to change its direction of motion by means of a
T1 = initial temperature of water, °C perforated cup. The water particles due to their
T2 = final temperature of water, °C greater moment of energy (being heavier) tend to
Tsat = saturation temperature of steam in °C move away and get separated from the mixture
at given pressure p as shown in Fig. 3.20. Some quantity of moisture
The value of the dryness fraction obtained by drains through the perforated cup, some quantity
this method involves considerable error, since the falls as large droplets and some quantity sticks
heat losses due to convection and radiation are not with the wall of the separator. The condensate
taken into account. is collected in the separator and its quantity is
measured by a glass tube. The dry steam is passed
Example 3.11 The steam at a pressure of 5.5 bar is through a small condenser, where it condenses and
passed into a tank filled with 80 kg of water at 25°C. The its mass is measured.
tank is made of copper, is 10 kg in mass and has a specific
The dryness fraction x of wet steam is determined
heat of 0.406 kJ/kg ◊ K. After steam has condensed and
mixed with water, its temperature reaches 40°C and the as
mg
mass of the mixture becomes 82.5 kg. Determine the x= ...(3.33)
dryness fraction of steam. mg + mw
Working Substances 79

where
mg = mass of dry and saturated steam, kg
mw = mass of water particles in suspension, kg
sampling tube. The wet steam is then throttled
It is noticed that the dryness fraction obtained
through a partially opened or restricted valve to a
from such experimentation is not accurate, but it is
pressure p2 and temperature T2 in such a way that
just close approximation.
the steam reaches the superheated region. During
ln an experiment, a sample of wet the throttling process, the enthalpy of steam
steam is passed through a separating calorimeter. At remains constant.
some instant, the mass of water collected in the separator The throttling process is an irreversible process
was 0.2 kg, while the amount of steam condensed was and hence, it is shown by a dotted line on the h–s
found to be 2.5 kg. Determine the dryness fraction of diagram in Fig. 3.22.
steam entering the calorimeter. The state 1 is defined by pressure p1 and dryness
fraction x1, while the state 2, in superheated region
Solution after throttling can be defined by the pressure p2
Given mw = 0.2 kg mg = 2.5 kg and temperature T2. Using properties of wet steam
at p1 and superheated steam at p2 and T2, we get
To find Dryness fraction of wet steam
h 1 = h2
Analysis The dryness fraction of wet steam is
h
calculated as
mg
x= State before
mg + mw throttling p1 State after
throttling
Using numerical values, we get p2
2.5 T2
x= = 0.926 1
2.5 + 0.2 p1 2
Saturation
p2 curve

x1
The schematic of a throttling calorimeter is shown
s
in Fig. 3.21. A sample of wet steam at a pressure p1
is taken from the steam main through a perforated
80 Thermal Engineering

or hf1 + x1 hfg1 = h2
h2 - h f1
or x1 = ...(3.34)
h fg1
where
hf1 = sensible heat of wet steam at p1, kJ/kg
hfg1 = the enthalpy of vaporisation at p1, kJ/kg
h2 = enthalpy of superheated steam at p2 and T2
= hg2 + Cps (T2 – Tsat2)
To obtain good approximation from a throttling
calorimeter, the steam after throttling should be
superheated at least by 5°C. If the steam is very wet
then the resultant dryness fraction obtained by this
technique is not very accurate. When the steam is very wet then the dryness
In a throttling calorimeter, the steam fraction of steam is determined by using a separating
is admitted at 10 bar, it is throttled to atmospheric and throttling calorimeter. The wet steam taken
pressure and 110°C. Determine the dryness fraction of from a steam main via a steam stop valve is first
steam. Assume the specific heat of superheated steam as passed through a separating calorimeter.
2.2 kJ/kg ◊ K. Some part of the moisture is removed from
steam, due to sudden change in its direction. The
Solution resulting semi-dry steam is then throttled into a
Given The wet steam at state p1 = 10 bar throttling calorimeter. The steam coming out the
The steam, after throttling p2 = 1.01325 bar, throttling calorimeter is superheated at a pressure
T2 = 110 °C p2 and temperature T2. The throttling calorimeter
Cps = 2.2 kJ/kg ◊ K is well insulated to prevent any heat loss. The
steam coming out of the throttling calorimeter is
To find The dryness fraction of steam.
condensed and its mass is measured.
Analysis From steam table A – 13;
If mw = mass moisture collected in separator, kg
State 1, 10 bar hf1 = 762.81 kJ/kg mg = mass of steam condensed after throttling, kg
hfg1 = 2014.3 kJ/kg
State 2, 1.01325 bar, hg2 = 2676 kJ/kg then the dryness fraction of steam from separating
Tsat2 = 100°C calorimeter
mg
As shown in Fig. 3.22, the enthalpy of steam remains x1 =
constant during the throttling process. Thus mg + mw
h1 = h2 Similarly, the dryness fraction x2, from the
or hf1 + x hfg1 = hg2 + Cps(T2 – Tsat2) throttling calorimeter is obtained as discussed
or 762.81 + x ¥ 2014.3 = 2676 + 2.2 (110 – 100) earlier.
2676 + 22 - 762.81 h2 - h f1
or x= = 0.961 x2 =
2014.3 h fg1
The final dryness fraction of steam in steam
main
The schematic of combined separating and thrott- x = x1 x2 ...(3.35)
ling calorimeter is shown in Fig. 3.23.
Working Substances 81

Example 3.14 A combined separating and throttling extracts moisture at a rate of 0.2 kg/min. The steam is
calorimeter was used to determine the dryness fraction then throttled to a pressure of 3.2 bar and passed through
of steam flowing through a steam main at a pressure coils which extract 650 kJ/min of heat without loss of
of 9 bar. The pressure and temperature of steam after pressure. The steam leaves the coil with a dryness fraction
of 0.93. Calculate the temperature of steam immediately
throttling were 1.25 bar and 115°C, respectively. The
after throttling and the dryness fraction of steam before
mass of steam condensed after throttling was 2.2 kg and
the mass of water collected in the separator was 0.20 kg. and after the separator. Take Cps = 2.1 kJ/kg ◊ K.
Estimate the dryness fraction of steam in the main.
Solution
Take specific heat for superheated steam as
2.1 kJ/kg ◊ K. Given A sample of wet steam
p1 = 1 MPa m = 4 kg/min
Solution mw = 0.2 kg/min p2 = 3.2 bar
Given For separating calorimeter x3 = 0.93 Q = 650 kJ/min
Cps = 2.1 kJ/kg ◊ K
mw = 0.2 kg mg = 2.2 kg
To find
For throttling calorimeter
(i) Temperature of steam after throttling,
p1 = 9 bar p2 = 1.25 bar (ii) Dryness fraction before and after separator.
T2 = 115 °C Cps = 2.1 kJ/kg
Properties of Steam
To find Final dryness fraction of steam At p1 = 1 MPa: hf1 = 762.61 kJ/kg
Analysis The dryness fraction x1 of wet steam from hfg1 = 2013.6 kJ/kg
separating calorimeter At p3 = 3.2 bar: hf 3 = 570.90 kJ/kg
mg 2.2 hfg3 = 2156.7 kJ/kg
x1 = = = 0.9167 Tsat = 135.75 C
mg + mw 2.2 + 0.2
hg3 = 2727.6 kJ/kg
For throttling calorimeter, the properties of steam.
Analysis Mass of dry steam,
Before throttling at p1 = 9 bar,
mg = m – mw = 4 – 0.2 = 3.8 kg/min
hf1 = 742.83 kJ/kg, hfg1 = 2031.1 kJ/kg Then dryness fraction of steam before separator
After throttling at p2 = 1.25 bar, mg 3.8
x1 = = = 0.95
m 4
Tsat2 = 106 °C, hg2 = 2685.4 kJ/kg
(i) The heat extracted at the coil
Enthalpy before throttling, Q = 650 kJ/min
h1 = hf1 + x2 hfg1 = 742.83 + x2 ¥ 2031.1 650
or q= = 171.02 kJ/kg
3.8
Enthalpy after throttling,
The heat content in steam at 3.2 bar when it enters
h2 = hg2 + Cps (T2 – Tsat2) the coil
= 2685.4 + 2.1 ¥ (115 – 106) = 2704.3 kJ/kg = heat loss on coil
Equating two enthalpies, we get + enthalpy of steam on leaving the coil
h3 = q + hf 2 + x3 hfg3
2704.3 - 742.83
x2 = = 0.9657 = 171.02 + 570.90 + 0.93 ¥ 2156.7
2031.1
= 2747.65 kJ/kg
Final dryness fraction of steam
Since h3 > 2727.6 kJ/kg (total enthalpy at
x = x1 . x2 = 0.9167 ¥ 0.9657 = 0.885 3.2 bar), therefore, steam leaving the throttling
calorimeter is superheated.
Example 3.15 Steam at a pressure of 1 MPa, and a
flow rate of 4 kg/min is passed through a separator which h3 = hg1 + Cps (Tsup – Tsat)
82 Thermal Engineering

2747.65 = 2727.6 + 2.1 (Tsup – 135.75)


2747.65 - 2727.6 State 1
or Tsup = + 135.75º C = 145.3ºC
2.1

Pressure, p
pv = C
Degree of superheat = 145.3 – 135.75 = 9.55°C T1

(ii) Dryness fraction of steam after separator : steam T2 State 2


pressure, 1 MPa
h2 = hf + x2 hfg1 = h3
2747.65 = 762.61 + x2 (2013.6) Volume, v
2747.65 - 762.61
or x2 = = 0.986
2013.6
Therefore, the dryness fraction of steam inversely proportional to absolute pressure, i.e.,
x = x1 x2 = 0.95 ¥ 0.986 = 0.936
1

p
or pv = C ...(3.36)
where C is a constant of proportionality.
A gas can be modeled as an ideal gas when it has If a gas changes its state from p1, v1 to p2, v2
the following features: without any change in its temperature, then

1. It has no intermolecular forces of attraction p1 v1 = C


or repulsion. and p2 v2 = C
2. It does not change its phase during a Hence p1 v1 = p2 v2 ...(3.37)
thermodynamic process. A plot on the p–v diagram is shown in Fig. 3.24.
3. It obeys Boyle’s law, Charles’s law and the Boyle’s law represents a rectangular hyperbola
characteristic gas equation. curve. This curve is also called an isotherm and the
The internal energy of gases decreases rapidly process ocurring at constant temperature is known
with decreasing pressure and disappears when as an isothermal process.
absolute pressure approaches zero. At zero
pressure, all real gases behave in a similar manner
and this state of identical behaviour is referred as It states that if a gas undergoes a process at constant
ideal state. The behaviour of real gases at an ideal pressure, the change in its specific volume is
state suggests the concept of an ideal gas that will directly proportional to its absolute temperature
behave in as ideal manner at all pressures. change, i.e.,
In actual, no gas is completely ideal gas, but
many gases such as air, nitrogen, oxygen, hydrogen
helium, argon, neon, krypton, carbon dioxide, etc.,
can be treated as ideal gases with negligible error.
Dense gases, such as water vapour, refrigerant
vapour, however, should not be treated as ideal
gases.

It states that when a gas undergoes a process


at constant temperature, its specific volume is v
Working Substances 83

vμT simultaneously. On the basis of this characteristic,


v an equation is derived with the help of Boyle’s and
or =C
T Charles’s laws as
where C is the constant of proportionality at 1
constant pressure. Boyle’s law vμ when T is consant
p
For an ideal gas undergoing a constant pressure
process from v1, T1 to v2, T2 ; By Charles’s law v μ T when p is constant
v1 v If we combine both relations, we get
= 2 ...(3.38)
T1 T2 T
vμ when both p and T vary
Charles’s law has also been stated as the p
absolute pressure of an ideal gas varies directly
RT
with absolute temperature if the volume of a gas or v=
is maintained constant during the process. That is, p
pμT or pv = RT ...(3.41)
p where R is the constant of proportionality and is
or = C (constant of proportionality at con-
T called specific gas constant,
stant volume)
p is the absolute pressure,
For an ideal gas undergoing a constant volume T is the absolute temperature, and
process from p1, T1 to p2, T2 ; v is the specific volume.
p1 p
= 2 ...(3.39) This is known as the characteristic gas equation
T1 T2 for an ideal gas or ideal-gas equation of state.
The molecular or molar mass M can be defined
as the mass of one kmol (also known as kg-mol) of
It is observed that thermodynamic properties are a substance in kilograms.
interrelated. Any equation that relates the pressure, The mass of a system is the product of its
temperature and specific volume of a substance is molecular mass M in kg/kmol and number of
known as an equation of state. kmols, n :
For any homogenous system, if the pressure and m = n M (kg) ...(3.42)
temperature are specified, its specific volume is
automatically fixed by some functional relation. In The ideal-gas equation of state can be written in
other words, the thermodynamic properties exist in several different forms as follows:
a certain relationship, such as For a given mass system, the total volume
F ( p, v, T) =0 V = m v (m3) ...(3.43)
or p = f (v, T) then p V = mRT ...(3.44)
and v = g (p, T) 3
For a given molar specific volume v (m /kmol),
T = h (p, v) ...(3.40) the total volume
where F, f, g and h are some functional relationship. V = n v (m3) ....(3.45)
then p V = n Ru T ...(3.46)
where Ru = MR is known as the universal gas
In any thermodynamic system of an ideal gas, the constant. Its value is found to be constant for all
pressure, temperature and specific volume vary gases.
84 Thermal Engineering

Ru
When the molecular mass of any gas (M) is
multiplied by its specific gas constant (R), it is Molar mass, Gas constant,
observed that the product M R is always same Gas Ru
M, kg/kmol R = , kJ/kg ◊ K
for all gases. This product is called universal gas M
constant and it is denoted as Ru. Air 28.97 0.287
Ru = MR ...(3.47) Carbon dioxide 44.01 0.189
Carbon monooxide 28.01 0.297
For SI system, the value of the universal gas
Hydrogen 2.01 4.124
constant is 8.31447 kJ/kmol ◊ K.
Nitrogen 28.01 0.297
Oxygen 32.00 0.259
R
Water vapour 18.01 0.462
The constant of proportionality used in Eq. (3.41) is
called the specific gas constant. Its value is different
for different gases.

Unit of R If the absolute pressure is expressed It states that the molecular mass of all perfect
in kN/m , specific volume in m3/kg, and absolute
2 gases occupies the same volume under identical
temperature in K (kelvin) then the specific gas conditions of pressure and temperature. Avogadro’s
constant R has the unit of kJ/kg ◊ K. experiment shows that the average volume for one
kmol (kg-mole) for any perfect gas is 22.413 m3
Calculation of R The value of the gas constant is at standard atmospheric pressure (1.01325 bar) and
determined from dividing the universal gas constant 0°C (NTP condition).
Ru by its molecular mass (M) as M RT Ru T
or V= =
Ru p p
R= ...(3.48)
M (8.31447 kJ/kmol ◊ K) ¥ (273.15)
=
Table 3.5 presents the values of molar masses (101.325 kPa)
and gas constants for the most commonly used = 22.413 m3/kmol
gases.
The gauge pressure of air in an
automobile tyre at a temperature of 25°C is 1.75 bar.
Due to running condition, the temperature of air in the
From Eq.(3.44), for an ideal gas undergoing change tyre rises to 70°C. Determine the gauge pressure after
of state from p1, V1, T1 to p2, V2, T2 we get running. Assume atmospheric pressure to be 1.0132 bar
p1V1 and volume of air in the tyre to be constant.
at the state 1, = mR
T1
Solution
p2 V2
at the state 2, = mR Given Constant volume of air in an automobile tyre:
T2
Equating the left terms of the two equations, we State 1: T1 = 25°C + 273 = 298 K and pg1 = 1.75 bar
get State 2: T2 = 70°C + 273 = 343 K, patm = 1.0132 bar
p1V1 p V To find The gauge pressure of air in tyre after running.
= 2 2 =C ...(3.49)
T1 T2 Assumptions
It is a property relation for an ideal gas. (i) Air is an ideal gas.
(ii) Process of heating is reversible.
Working Substances 85

Analysis Absolute pressure of air in the tyre at intial Solution


state.
Given An oxygen cylinder with constant volume
p1 = Atmospheric pressure + gauge pressure
V = 50 lit = 50 ¥ 10–3 m3,
or p1= patm + pg1 = 1.0132 + 1.75 = 2.7632 bar
T = 18°C + 273 = 291 K
In the tyre, the volume remains constant (V1 = V2), p = 10 MPa = 10 ¥ 103 kN/m²
Eq. (3.49) reduces to M = 32 kg/kmol
p1 p2
= To find
T1 T2
Hence after running, i.e., at the state 2, the pressure p2 (i) Mass of oxygen in the cylinder,
(ii) Molar volume,
p1 (2.7632 bar) (iii) Density of oxygen.
p2 = × T2 = × (343 K) = 3.18 bar
T1 (298 K)
Analysis The universal gas constant Ru = 8.31447
The gauge pressure at the state 2; kJ/kmol ◊ K and the specific gas constant R for oxygen
pg2 = p2 – patm = 3.18 – 1.0132 = 2.167 bar can be calculated as
R 8.31447 kJ/kmol ◊ K
R = u =
Example 3.17 The compression ratio of an engine is M 32 kg/kmol
15 to 1. The pressure of the gas at the beginning of the = 0.259 kJ/kg ◊ K.
stroke is 100 kPa and the temperature is 30°C. Calculate
(i) Mass of oxygen in cylinder It can be calculated
the absolute pressure at the end of compression stroke, if
from the characteristic gas equation.
the temperature used to be 980°C.
pV = m R T
pV
Solution or m=
RT
Given Compression of a gas in an engine with (10 ¥ 103 kN/m 2 ) ¥ (50 ¥ 10-3 m3 )
=
State 1: V1 = 15V2 p1 = 100 kPa (0.259 kJ/kg ◊ K) ¥ (291 K)
= 6.63 kg
and T1 = 30°C = 303 K
(ii) Molar volume or molecular volume
State 2: T2 = 980°C + 273 = 1253 K
p V = Ru T
To find The pressure of gas after compression
Ru T (8.31447 kJ/kmol ◊ K) ¥ (291 K)
or V = =
Analysis Using properties relationship, Eq. (3.49) p (10 ¥ 103 kN/m 2 )
3
p1V1 p V = 0.24 m /kmol
= 2 2
T1 T2 (iii) Density of oxygen From the characteristic gas
equation for unit mass,
p1V1 T2
or p2 = ¥ p v = RT
T1 V2
We have r =
1
(100 kPa) ¥ (15 V2 ) ¥ (1253 K) v
=
(303 K) ¥ V2 p (10 ¥ 103 kN/m 2 )
Therefore,r = =
= 6203 kPa = 62.03 bar RT (0.259 kJ/kg ◊ K) ¥ ( 291 K)
= 132.68 kg/m3
Example 3.18 A cylinder of 50-litre capacity contains
oxygen at 18°C and at a pressure of 10 MPa. Calculate Example 3.19 10 kg of carbon monoxide at 40°C
occupies 3 m3. Determine the gas pressure in bar. An
(a) the mass of oxygen in the cylinder,
additional mass of carbon monoxide is then very slowly
(b) the molar volume,
added to raise the tank pressure to 10 bar. Assuming that
(c) the density of oxygen. the gas temperature remains constant, how much extra
The molecular mass of oxygen is 32 kg/kmol. mass has been added?
Assume R = 0.297 kJ/kg ◊ K.
86 Thermal Engineering

Solution To find In each case,

Given A constant volume and constant temperature (i) Mass of gas, and
process; (ii) Number of moles.
m1 = 10 kg T1 = 40°C + 273 = 313 K V = 3 m3 Analysis The specific gas constants:
T1 = T2, p2 = 10 bar R = 0.297 kJ/kg ◊ K
Ru 8.31447
For air Rair = = = 0.287 kJ/kg ◊ K
To find Mair 28.97
(i) Initial gas pressure in bar, Ru 8.31447
(ii) Mass of CO added to system. For hydrogen, RH2 = =
MH 2 2
Assumptions = 4.157 kJ/kg ◊ K
(i) Carbon monoxide is a perfect gas. 4
Volume of balloon V = p ro3
(ii) Process is reversible and nonflow. 3
4
Analysis = ¥ p ¥ (8)3 = 2144.66 m3
3
(i) The characeristic gas equation,
(i) When air is filled in the balloon
p1V = m1 RT1
Initial gas pressure (a) Mass of air filled
m1 R T1 pV 100 ¥ 2144.66
p1 = mair = = = 2491 kg
V RairT 0.287 ¥ 300
(10 kg) ¥ (0.297 kJ/kg ◊ K) ¥ (313 K) (b) Number of moles of air
=
(3 m3 ) mair 2491
n= = = 85.98
= 309.87 kPa. = 3.098 bar Mair 28.97
(ii) Since additional CO is added to the tank, as V = C (ii) When hydrogen is filled in the balloon under
and T = C indentical conditions
m μ pressure pV 100 ¥ 2144.66
(a) mH2 = = = 172 kg
m2 p2 10 R H2 T 4.157 ¥ 300
Therefore, = =
m1 p1 3.0987
(b) Number of moles of H2,
10 ¥ 10 172
and m2 = = 32.2768 kg n= = 86
3.0987 2
Additional mass m = m2 – m1
= 32.2768 – 10 = 22.2768 kg An aerostat balloon is filled with
hydrogen, it has a volume of 1000 m3 at a temperature of
A spherical balloon of 8 m radius is 300 K and pressure of 100 kPa. Determine the payload
floating in the atmosphere at 100 kPa pressure and 300 K that can be lifted with the aid of the aerostat.
temperature. Determine the mass and number of moles of
air displaced by the balloon. Solution
If hydrogen gas is filled in the balloon under the
same conditions of temperature and pressure, calculate Given An aeorstat filled with hydrogen
the mass and number of moles of hydrogen. Molecular V = 1000 m3 p = 100 kPa T = 300 K
masses of air and hydrogen are taken as 28.97 and 2,
To find The payload that can be lifted by the aerostat.
respectively.
Assumptions
Solution
(i) Hydrogen as an ideal gas.
Given A spherical balloon filled with air/hydrogen (ii) Molecular weight of H2 as 2 kg/kmol,
ro = 8 m p = 100 kPa T = 300 K RH2 = 4.157 kJ/kg ◊ K.
Mair = 28.97 MH2 = 2 (iii) Gas constant of air Rair = 0.287 kJ/kg ◊ K.
Working Substances 87

Analysis The mass of hydrogen in the balloon (i) Mass of air originally in the room minus volume
pV 100 ¥ 1000 of balloon
mH2 = = = 80.18 kg p1 V1 (100 kPa) ¥ (950 m3 )
R H 2 T 4.157 ¥ 300 m initial = =
The balloon displaces the volume of air equivalent to RT (0.287 kJ/kg ◊ K) ¥ (300 K)
its volume at the same pressure and temperature. = 1103.37 kg
The volume displaced by the balloon (ii) Final mass of air after mixing of air of balloon in
= volume of air equivalent to volume of the the room:
balloon (1000 m3) p2 Vroom (130 kPa) ¥ (1000 m3 )
mfinal = =
Thus, the mass of air displaced RT (0.287 kJ/kg ◊ K) ¥ (300 K)
pV 100 ¥ 1000 = 1509.9 kg
m2 = = = 1161.44 kg
RairT 0.287 ¥ 300 (iii) Initial pressure of air in the balloon
The load that can be attached with the hydrogen balloon The mass of air in balloon
mballoon = mfinal – minitial
= m2 – m1 = 1161.44 – 80.18 = 1081.26 kg
= 1509.9 kg – 1103.37 kg = 406.5 kg
Pressure
Example 3.22 A room of 1000 m3 volume, contains
air at 1 bar and 300 K. In the room, there is a balloon mballoon RT
pballon =
which contains air at 300 K. Suddenly, the balloon Vballoon
bursts. Before any air can escape through the doors or
(406.5 kg) ¥ (0.287 kJ/kg ◊ K) ¥ (300 K)
windows, the pressure in the room becomes 1.3 bar, while =
its temperature is still 300 K. If the volume of balloon was (50 m3 )
50 m3, find: = 700 kPa or 7 bar
(a) mass of air originally in the room (outside the
balloon),
(b) initial pressure in bar of air in the balloon, and
(c) final mass of air in the room.
Joule submerged two tanks A and B connected by
Solution
a valve in an insulated tank of water as shown in
Given A room with a balloon Fig. 3.26.
Vroom = 1000 m3 Vballoon = 50 m3
T = 300 K p1 = 1 bar = 100 kPa, Thermometer
p2 = 1.3 bar 130 kPa
To find
(i) Mass of air originally in the room minus balloon A B
Gas Vaccum
contents, Water bath
(ii) Final mass of air in the room, and
(iii) Initial pressure of air in the balloon.
Assumption The specific gas constant for air,
R = 0.287 kJ/kg ◊ K
One tank A was filled with air and the other tank
Analysis The volume of air in the room excluding the B was evacuated. The air, tanks and surrounding
balloon’s volume water were allowed to come to thermal equilibrium.
V1 = Vroom – Vballoon = 1000 – 50 = 950 m3 The temperature of the water bath was measured.
88 Thermal Engineering

Then the valve between the two tanks was opened


to let the air pass as a result of free expansion from
one tank to other. Of course, no work transfer was All the gases have two kinds of specific heats:
involved. Joule observed no change in temperature specific heat for constant volume Cv and specific
of the water bath during free expansion of air. From heat at constant pressure, Cp.
this experiment, he concluded that in the absence
of any heat and work interactions to or from the Cv
system of air; the internal energy of air did not The specific heat at constant volume is defined as
change. The temperature of air remained constant the amount of heat energy transferred to change
even though pressure and volume changed. He the temperature of the unit mass of a substance
established that the internal energy of an ideal gas by one degree, when the volume of substance is
does not depend on pressure and volume, it depends maintained constant.
only on temperature. Consider a closed system of an ideal gas
Hence, the conclusion of Joule’s experiment was undergoing a change of state at constant volume.
that the internal energy of an ideal gas is a function According to the first law of thermodynamics for
of temperature only. It is known as Joule’s law and unit mass system,
can be stated as dq = dw + du = pdv + du
u = u (T ) ...(3.50) But for constant volume, dv = 0; thus dw = 0,
In differential form, du = Cv dT ...(3.51)) therefore,
The change in specific internal energy for an dq = du = Cv dT
ideal gas during a process from the state 1 to state 2 Ê du ˆ
Cv = Á
Ë dT ˜¯ v
or ...(3.57)
is obtained by integrating Eq. (3.51);
2 Further, the specific heat at constant volume
Du = Ú1
Cv dT Cv can also be defined as change of specific
internal energy of a substance per unit change
or Du = u2 – u1 = Cv (T2 – T1) ...(3.52)
in temperature at constant volume. It relates the
By definition, the enthalpy per unit mass, change in properties, and thus it is also a property.
h = u + pv
Cp
For an ideal gas, pv = RT
Thus h = u + RT ...(3.53) It is defined as the amount of heat energy
transferred to change the temperature of unit mass
Both terms on the right side of the equation
of a substance by one degree when the pressure is
depend on temperature only, so enthalpy h is
maintained constant during the process.
function of temperature only. Thus
Consider an ideal gas undergoing a process at
h = h(T) ...(3.54) constant pressure. The enthalpy for unit mass, in
In differential form, dh = Cp dT ...(3.55) differential form can be expressed as
The change in specific enthalpy for an ideal dh = du + d(pv)
gas during a process from the state 1 to state 2 is
As the pressure is constant, therefore,
obtained by integrating Eq. (3.55);
2 dh = du + pdv = dq
Dh = Ú1
C p dT or, dh = Cp dT
or Dh = h2 – h1 = Cp (T2 – T1) ...(3.56) Ê dh ˆ
Cp = Á
Ë dT ˜¯ p
or ...(3.58)
Working Substances 89

Further, the specific heat at constant pressure Analysis


Cp can also be defined as change in the specific (i) The specific heat at constant volume is given as
enthalpy of a substance per unit change in
R (0.287 kJ/kg ◊ K)
temperature at a constant pressure. It relates the Cv = =
g -1 1.4 - 1
change in properties, and thus it is also a property.
= 0.7175 kJ/kg ◊ K
These specific heats are properties of the gas and
(ii) The specific heat at constant pressure is given as
both vary with pressure and temperature. However,
for an ideal gas they are assumed constant. Cp = Cv + R = 0.7175 + 0.287 =1.0045 kJ/kg ◊ K

For air Cp = 1.005 kJ/kg ◊ K, and (iii) The change in internal energy
Cv = 0.718 kJ/kg ◊ K DU = m Cv (T2 – T1)
= 10 ¥ 0.7175 ¥ (373 – 293)
= 574 kJ
(iv) The change in enthalpy
DH = m Cp (T2 – T1)
A special relationship between Cp and Cv , for an
= 10 ¥ 1.0045 ¥ (373 – 293)
ideal gas can be obtained by differentiating the
= 803.6 kJ
enthalpy relation Eq. (3.53);
dh = du + R dT Example 3.24 A gas initially at a pressure of 510 kPa
and a volume of 142 litres undergoes a process and has
Replacing dh by Cp dT and du by Cv dT, then
a final pressure of 170 kPa and a volume of 275 litres.
Cp dT = Cv dT + R dT During the process, the enthalpy decreases by 65 kJ. Take
Dividing both sides by dT, we get Cv = 0.718 kJ/kg ◊ K.
Determine (a) change in internal energy, (b) specific
Cp = Cv + R (kJ/kg ◊ K) ...(3.59) heat at constant pressure, and (c) specific gas constant.
It is a very important relationship for ideal gases.
The ratio of the two specific heats is defined as Solution
Cp Given A gas process
g= ...(3.60)
Cv State 1: p1 = 510 kPa V1 = 142 litres = 142 ¥ 10-3 m3
State 2: p2 = 170 kPa V2 = 275 litres = 275 ¥ 10–3 m3
10 kg of air is heated in a rigid vessel DH = – 65 kJ (since it is decreased)
from 20°C to 100°C. If the ratio of specific heat is 1.4,
Cv = 0.718 kJ/kg ◊ K
estimate the values of Cp and Cv, change in internal
energy, and enthalpy. To find
(i) Change in internal energy,
Solution (ii) Specific heat at constant pressure,
Given Air in a rigid vessel (iii) Specific gas constant.
Analysis
V = Const. m = 10 kg g = 1.4
State 1: T1 = 20°C = 293 K, (i) The change in enthalpy can be expressed as
State 2: T2 = 100°C = 373 K, DH = DU + p2V2 – p1V1
Using numerical values;
To find
– 65 = DU + (170) ¥ (275 ¥ 10–3) – (510)
(i) Cp (ii) Cv
¥ (142 ¥ 10–3 )
(iii) DU and (iv) DH
or – 65 = DU + (46.75 – 72.42)
Assumption The specific gas constant for air as R = or DU = –65 + 72.42 – 46.75 = –39.33 kJ
0.287 kJ/kg ◊ K. (decrease in internal energy)
90 Thermal Engineering

(ii) The change in internal energy can also be T2 È 40 ˘


expressed as = m ÚT1
Í1.1 + T + 30 ˙ dT
Î ˚
DU = m Cv (DT)
= m È1.1 T + 40 ¥ ln (T + 30 )˘ 2
T
or –39.33 = 0.718 (mDT) Î ˚T1
or m DT = – 54.777
È Ê T + 30 ˆ ˘
Further the change in enthalpy can also be = m Í1.1 (T2 - T1 ) + 40 ¥ ln Á 2 ˙
ÍÎ Ë T1 + 30 ˜¯ ˙˚
expressed as
DH = m Cp DT = Cp (m DT) Using numerical values, we get
or –65 = Cp (–54.777) È
Q1–2 = 1 ¥ Í1.1 ¥ (370 - 50 ) +
65 Î
or Cp = = 1.1866 kJ/kg ◊ K
54.777 Ê 370 + 30 ˆ ˘
40 ¥ ln Á ˙
Ë 50 + 30 ˜¯ ˚
(iii) Specific gas constant,
R = Cp – Cv
= 352 + 64.37 = 416.37 kJ
= 1.1866 – 0.718 = 0.468 kJ/kgK
(ii) Work done at constant pressure
Example 3.25 1 kg of a certain gas undergoes a 2
reversible constant pressure process at 1.2 bar, during
W1–2 =
Ú 1
pdV = p1(V2 – V1)
which its volume changes from 1 m3 to 1.8 m3 and the = (120 kPa) ¥ (1.8 – 1) (m3)
temperature changes from 50°C to 370°C. The specific = 96 kPa
heat for a substance at constant pressure is given by
(iii) Change in internal energy can be obtained as
È 40 ˘ DU = Q – W = 416.37 – 96
Cp = Í1.1 + kJ/kg ◊ K, where T is in °C.
Î T + 30 ˙˚ = 320.37 kJ
Find (a) Heat supplied (b) Work done (iv) Change in enthalpy
(c) Change in internal (d) Change in enthalpy DH1–2 = Q1–2 = 416.37 kJ
energy,

Solution
REAL GASES
Given A constant pressure process with
m = 1 kg p1 = 1.2 bar = 120 kPa The ideal gas equation of state, pv = RT can be used
with the assumption of very little or no attraction
State 1: V1 = 1 m3 T1 = 50°C
force of molecules within the gas and the volume
3
State 2: V2 = 1.8 m T2 = 370°C of molecules is negligibly small in comparison to
È 40 ˘ volume of gas. For many gases, at very low pres-
and Cp = Í1.1 + kJ/kg K
Î T + 30 ˙˚ sure and very high temperature, the forces of inter-
molecular attraction and volume of molecules com-
To find pared to the volume of gas are significantly small
(i) Heat supplied, and the real gases obey very closely the ideal gas
(ii) Work done, equation.
(iii) Change in internal energy, and
But at higher pressure, the forces of intermo-
(iv) Change in enthalpy.
lecular attraction and repulsion are significant and
Analysis the volume of molecules are also appreciable com-
(i) Heat supplied at constant pressure pared to total gas volume. Then the real-gas behav-
2 iour deviates from ideal-gas behaviour. The slight
Q1–2 =
Ú1
mC p dT modification in the ideal-gas equation of state to fit
Working Substances 91

accurate within few percentage. The deviation of


as real-gas behaviour from that of ideal gas is greatest
Real pÆ0 Ideal
gas gas in vicinity of the critical point.
Better results can be obtained for some gases
by using pseudo-reduced coordinates in place of
reduced properties. These are
p T
p'R = , T R¢ = …(3.66)
pc + A Tc + B
real-gas behaviour is to introduce a correction fac- where A and B are some constants and presudo-
tor called compressibility factor Z. It is defined as reduced specific volume v R¢ is defined as
pv v
Z= …(3.61) vR = …(3.67)
RT RTc / pc
For an ideal gas, obviously, Z = 1
Thus, v¢R is related to pc and Tc not to vc .
For a real gas, Z is a function of pressure
and temperature and it is usually determined
1 mole of CO2 at 88.7 bar and 61°C
empirically.
is compressed in reversible isothermal manner till the
The real gases behave differently at a given volume halves. Calculate the work transfer during the
temperature and pressure, but they behave very process.
likely at their reduced pressure and reduced
temperature. These are Solution
p p
Reduced pressure pR = actual = …(3.62) Given Isothermal compression of CO2
pcritical pc p1 = 88.7 bar = 8870 kPa T1 = 61°C = 334 K
Tactual T 1
Reduced temperature TR = = …(3.63) v2 = v1, n = 1 mole
Tcritical Tc 2
v v To find Work transfer during the process.
Reduced sp. volume vR = actual = …(3.64)
vcritical vc Analysis The CO2 is real gas, the gas compressibility
is to include.
where the subscript c represents critical state.
The isothermal work
The disadvantage of the use of compressibility
nRuT
Ú pdv = Ú
factor chart is that a separate chart is needed for W= dv
each gas. But at the same reduced coordinates, the v
compressibility factor Z is approximately same for For a real gas
all gases. It is called law of corresponding states, v2 dv Êv ˆ
thus
vR = f (pR, TR) …(3.65)
W = nRuT Z
Ú v1 v
= nRuT Z ln Á 2 ˜
Ë v1 ¯
where Z = f (pR, TR)
A diagram of pR versus TR as a parameter plotted
p T
from data of any gas would apply to all other gases. and pR = and TR =
The Fig. 3.28 shows a generalised compressibility pc Tc
chart, where Z values are plotted against pR, and From Table A–1; For CO2, pC = 73.9 bar, Tc = 304 K
TR for several gases. The use of compressibility 88.7 334
chart requires the knowledge of critical parameter Then pR = = 1.2, TR = = 1.1
73.9 304
given in factor and TR for several gases. The Z = 0.62
results obtained by using compressibility chart are
92 Thermal Engineering

1.1
TR = 2.00
1.0

0.9 TR = 1.50

0.8

Z 0.7 TR = 1.30

0.6
TR = 1.20

0.5 Legend
Methane Iso-pentance
0.4 TR = 1.10 Ethylene n-hentane
Ethane Nitrogen
0.3 Propane Carbon dioxide
TR = 1.00
n-butance Water
0.2
Average curve based on data on hydrocarbons
0.1

0 0.5 1.0 1.5 2.0 2.5 3.0 3.5 4.0 4.5 5.0 5.5 6.0 6.5 7.0
PR

The work transfer Percentage error = 3%


W = 1 ¥ 8.314 ¥ 334 ¥ 0.62 ln (1/2) Steam at 500 bar and 1200°C
= –1229 kJ vsup = 0.013561 m3/kg

Prove that the superheated steam can For superheated steam as an ideal gas
be modelled as an ideal gas. Ê 8.314 ˆ Ê 1200 + 273 ˆ
vg = Á ¥
Ë 18 ˜¯ ÁË 500 ¥ 100 ˜¯
Solution Let vsup is the specific volume of
superheated steam and vg is the volume of an ideal gas. = 0.01360 m3/kg
Consider p = 10 kPa and Tsup = 500°C Percentage error = 0.34%
vsup = 35.6789 m3/kg (from steam tables) The above calculations show that superheated steam
approximates an ideal gas reasonably well at low and
For superheated steam to be ideal gas
high pressures with high degree of superheat.
Ê R ˆ T Ê 8.314 ˆ (500 + 373)( K )
vg = Á u ˜ =Á ˜¯ ¥
Ë M¯ Ë p 18 (10 kPa )
3
= 35.7040 m /kg
v g - vsup Many extensive p, v, T data are available, but
Percentage error = ¥ 100 = 0.07% no equation of state that represents the p, v, T
vg
behaviour of the gas could be used accurately over
Now consider steam at 100 bar and 700°C
a larger region.
vsup = 0.04358 m3/kg
For superheated steam as an ideal gas 1. Van der Waals Equation In 1873, J.D. van der
Waals presented an equation of state which was
Ê 8.314 ˆ Ê 700 + 273 ˆ of interest on physical reasoning, introduced two
vg = Á
Ë 18 ˜¯ ÁË 100 ¥ 100 kPa ˜¯ correction constants in the equation of ideal gas
= 0.04494 m3/kg
Working Substances 93

equation of state. Ru T a
Ê aˆ p= - …(3.70)
ÁË p + 2 ˜¯ ( v - b ) = RuT
v - b v ( v + b) T 1 / 2
…(3.68)
v where Ru = universal gas constant = 8.314 kJ/kg mol.K
The constant a was introduced to account for the and v is molal volume. The values of these two
existence of intermolecular attraction, the constant constants are also determined from critical state
b was introduced to account for volumes of mol- parameter and these are
ecules and Ru is the universal gas constant. These R 2T 2.5
constants are evaluated from the behaviour of the a = 0.427 u c
pc
gas at critical point. It was assumed that critical
RT
temperature line on a p–v diagram is horizontal at and b = 0.0866 u c …(3.71)
pc
critical point, thus
Ê ∂ pˆ Ru Tc 2a 3. In 1928, Beattie-
ÁË ∂T ˜¯ = 2
+ 3 =0 Bridgeman proposed an equation of state. The mea-
T c
(vc - b) vc sured values of pressure, temperature and specific
Ê ∂2 p ˆ 2 Ru Tc 6a volume with good accuracy. It is in the form of
and Á 2 ˜ = 3
+ =0
Ë ∂ v ¯T (vc - b) v c4 Ru T Ï C ¸ A
c = Const p=
2 Ì
1- 3 ˝ ( v + B ) - 2 …(3.72)
Solving these equations, we get v Ó vT ˛ v
27 Ru2 Tc2 RT Ê aˆ
a= ,b= u c Where A = A0 Á1- ˜
64 pc 8 pc Ë v¯
Ru Tc 8 Ê bˆ
and = …(3.69)
and B = B0 Á1- ˜ …(3.73)
pc vc 3 Ë v¯
Except at higher pressures, the real gases do not Here v = molal volume in m3/kg mol
obey van der Waals equation in all ranges of pres- Ru = Universal gas constant
sures and temperatures. Despite of its limitations,
= 8.314 kJ/kg mol. K
the van der Waal equation has a historical impor-
The constants appearing in above equation are
tance because it was the first attempt to model the
given in Table 3.6 below:
behaviour of real gases.
4. Benedict-Webb-Rubin Equation of State In
2. Redlich-Kwong-Equation In 1949, Redlich
1940, Benedict-Webb and Rubin extended the work
and kwong presented an equation of state with two
of Beattle Bridgeman by increasing the number of
constants, which is more accurate than the van der
constants to eight. It is expressed as
Waals equation over a wide range.

Table 3.6 Constants that appear in the Beattie-Bridgeman equations of state

Gas A0 a B0 b C ¥ 10–4
Air 131.8441 0.019 31 0.046 11 – 0.001 101 4.34
Argon 130.7802 0.023 28 0.039 31 0.0 5.99
Carbon dioxide 507.2836 0.071 32 0.104 76 0.072 35 66.00
Helium 2.1886 0.059 84 0.014 00 0.0 0.0040
Hydorgen 20.0117 –0.005 06 0.020 96 – 0.043 59 0.0504
Nitrogen 136.2315 0.026 17 0.050 46 –0.006 91 4.20
Oxygen 151.0857 0.025 62 0.046 24 0.004 208 4.80
94 Thermal Engineering

Gas a A0 b B0 c C0 a g
n-Butane, C4H10 190.68 1021.6 0.039 998 0.124 36 3.205 ¥ 107 1.006 ¥ 108 1.101 ¥ 10–3 0.0340
Carbon dioxide, CO2 13.86 277.30 0.007 210 0.049 91 1.511 ¥ 106 1.404 ¥ 107 8.470 ¥ 10–5 0.00539
Carbon monoxide, CO 3.71 135.87 0.002 632 0.054 54 1.054 ¥ 105 8.673 ¥ 105 1.350 ¥ 10–4 0.0060
Methane, CH4 5.00 187.91 0.003 380 0.042 60 2.578 ¥ 105 2.286 ¥ 106 1.244 ¥ 10–4 0.0060
Nitrogen, N2 2.54 106.73 0.002 328 0.040 74 7.379 ¥ 104 8.164 ¥ 105 1.272 ¥ 10–4 0.0053

RuT Ï C ¸ 1 bR T - a ing effect on pv/RT. The effect may be either posi-


p= + Ì B0 RuT - A0 - 02 ˝ 2 + u 3 tive or negative. The positive value means replusion
v Ó T ˛v v
of molecules and negative value indicates attraction
aa c È g ˘ -g / v 2 between molecules.
+ + Í1 + ˙e …(3.74)
v 6 vT 3 Î v 2 ˚ It is assumed that when second virial coefficient
where p = pressure in kPa, is negative then third and succeeding virial coef-
3 ficients must be positive. The number of terms in-
v = molal volume, m /kg mol,
T = Absolute temperature, K cluded in virial equation depends upon the region
Ru = Universal gas constant under study.
= 8.314 kJ/k mol. K The virial equation can be converted into power
A0, B0, C0, a, b, c, a, g are Eight constants. series in the form
The values of these constants appearing in the pv
= 1 + A1 p + A2p2 + A3 p3 …(3.75)
equation are given in Table 3.7 below. RT
5. Virial Equation of State The equation of state Where A1, A2, A3 … are virial coefficients, func-
for a real gas can be expressed in the form of power tion of temperature alone, and virial coefficients are
series related as
pv a b c d
= 1+ + 2 + 3 + 4 a b - a2
RT v v v v A1 = , A2 = …(3.76)
where a, b, c, d, etc., are second, third, fourth, etc., RT R2 T 2
virial coefficients and these are function of tem- (a) For an ideal gas, an isometric line would be
perature only. These coefficients can be determined a straight line, starting from origin and with
experimentally or theoretically from statistical slope proportional to density,
mechanics. The above equation and several other (b) For real gas, the isometrics are displaced
equations of such form are known as virial equa- from origin and either straight line or a
tions. curve.
The accuracy of virial equation of state depends Note: Eqn (3.68) to (3.76) can also be used for real
on number of coefficients considered. gases by replacing molar volume v by specific
The all equations of state discussed above can volume v and universal gas constant Ru by specific
only be applied to gaseous phase of the substance. gas constant R.
The region in the vicinity of p = 0 corresponds
to concept of ideal gas with negligible force of at-
traction between molecules (pv/RT = 1). As density
increases at constant temperature, the binary col-
The equation of state and physical picture of an ideal
lision between molecules becomes more frequent
gas can be obtained from property relationships.
and second virial coefficient exerts with an increas-
Working Substances 95

Consider a gas that obeys Joule’s law by equation point of inflection at critical point
È Ê ∂ pˆ ˘ Ê ∂ pˆ Ê ∂2 p ˆ
du = Cv dT + ÍT Á ˜ - p ˙ dv ÁË ∂ v ˜¯ = 0 and Á 2˜ =0
ÍÎ Ë ∂T ¯ v ˙˚ T Ë ∂v ¯
and at constants temperature, dT = 0, then Also at critical point the third derivative
should be either zero or negligible, but
Ê ∂u ˆ Ê ∂ pˆ fourth derivative must be zero.
ÁË ∂v ˜¯ = T ÁË ∂T ˜¯ – p = 0 …(3.77)
T v
Ê ∂3 p ˆ Ê ∂4 p ˆ
Ê ∂ pˆ p f (v, T ) Á 3 ˜ = (0 or –ve) and Á 4˜ =0
Hence ÁË ∂T ˜¯ = T = …(3.78) Ë ∂v ¯ T Ë ∂v ¯
v T
Eq. (3.78) shows that the partial derivative with (iii) The curvature of isometrics of equation of
respect to temperature must be equal to original state on p–T diagram should be negative,
function of v, T divided by temperature and hence, positive or zero in the various region. In
the solution of Eq. (3.78) is particular, the critical isometric should be
straight, while the curvature of all other iso-
p = f (v, T) …(3.79)
metric should approach zero with either de-
It is the Joule’s law of equation of state for a gas creasing density or increasing temperature;
with attraction or repulsion forces.
The typical behaviour of internal energy for a Ê ∂2 p ˆ > Ê ∂2 p ˆ ¸Ô v = c
real gas is shown in Fig. 3.29. Á 2˜ < 0 and Á 2˜ = 0 ˝ pÆ0
Ë ∂T ¯ v Ë ∂T ¯ v Ô˛ T
0
(iv) The slope of isotherm of equation of state
–200 on the Z-p plot or compressibility factor, Z
–400 –150°C should be negative, positive or zero in the
–100°C
various regions.
–600

–800 –50°C Ê ∂Z ˆ 1 È ∂ ( p v) ˘
ÁË ∂p ˜¯ = R T Í ∂ p ˙
–1000
0°C
T u Î ˚T
1 È Ê ∂v ˆ ˘ >
50°C
–1200
= Ív + pÁ ˜˙ = 0
Ë ∂p ¯ ˚T <
u(kJ/kg)
Ru Î
Reduced density pr

∂2 Z ∂2 Z
lim = 0, lim =0
p Æ 0 ∂p ∂T p Æ 0 ∂r ∂T

There are certain conditions that should be (v) The equation of state should predict the
obeyed by an equation of state. Joule–Thomson coefficient
(i) The equation of state should be reduced to an 1 È Ê ∂v ˆ ˘ Ru T 2 Ê ∂Z ˆ
ideal gas equation of state, when its pressure m= ÍT Á ˜ - v ˙ = ÁË ∂T ˜¯
isothermally approaches to zero.
Cp ÍÎ Ë ∂T ¯ p ˙˚ rC p p

lim pv = RT at inversion state m = 0


pÆ0
∂Z ˆ Ê ∂Z ˆ = 0
T ÊÁ = v or
(ii) The critical isotherm of equation of state Ë ∂T ˜¯ p ÁË ∂T ˜¯
p
should have a horizontal tangent and at a
96 Thermal Engineering

Example 3.28 Consider nitrogen gas at 145 K with The value of p obtained by Redlich Kwong equation
specific volume of 0.0022 m3/kg. Calculate its pressure is 44% less than ideal gas equation calculation.
using Redlich Kwong equation and compare with pres-
sure obtained by ideal gas equation. Use pc = 3390 kPa, Example 3.29 Determine the pressure exerted by
Tc = 126.2 K. CO2 in a container of 1.5 m3 capacity when it contains
5 kg at 27°C.
Solution (a) Using ideal gas equation
Given Nitrogen gas with specific volume (b) Using Van der Waal’s equation

v = 0.0022 m3/kg, T = 145 K use a = 3.6285 ¥ 105 J 4/kg mol2, b = 0.0422 m3/kg mol
constants of van der Waal’s equation
pc = 3390 kPa Tc = 126.2 K
To find Pressure of gas by Redlich Kwong equation Solution
and then by ideal gas equation and compare it.
Given CO2 gas in a a container
Analysis Since specific volume of nitrogen is given, m = 5 kg, V = 1.5 m3 T = 27°C = 300 K
thus calculating specific gas constant for nitrogen
To find Pressure of gas
Ru 8.3144
RN2 = =
MN 2 28 Analysis The molecular mass of CO2 gas
= 0.297 kJ/kg ◊ K MCO2 = 12 + 32 = 44 kg/kg mol.
Calculating the constants of Redlich Kwong equation Ru 8.314
2
Sp. gas constant, R = =
RN Tc2.5 MCO2 44
2
a = 0.427
pc = 0.1889 kJ/kg ◊ K
2 2.5 (i) Using perfect gas equation
(0.297) ¥ (126.2)
= 0.427 ¥
3390 mRT 5 ¥ 0.1889 ¥ 300
p= = = 188.95 kPa
m3 K 0.5 V 1.5
= 1.988 kg The molar volume v of CO2
RN2 Tc V V MCO2 1.5 ¥ 44
b = 0.0866 v = = =
pc n m 5
0.297 ¥ 126.2 = 13.2 m3/kg mol.
= 0.0866 ¥
3390 (ii) Using Van der Waal’s equation
–4 3
= 9.575 ¥ 10 m /kg Ê aˆ
using Redlich Kwong equation in terms of v and RN2 ÁË p + 2 ˜¯ ( v – b) = RuT
v
R N2 T a
p = – RT a
v–b v ( v + b) T 1 / 2 or p = u –
v-b v2
0.297 ¥ 145
= 4 (8.314 ¥ 100) ¥ 300 3.6285 ¥ 105
0.0022 9.575 ¥ 10 = -
1.988 13.2 - 0.0422 (13.2) 2

0.0022 ¥ (0.0022 + 9.575 ¥ 10 – 4 ) ¥ (145)1/2 = 189560.56 – 2082.477
= 34659.66 – 23766.65 = 187478 pa or 1.875 bar
= 10893 kPa or 108.93 bar
Example 3.30 Determine the critical pressure of
Using ideal gas equation
oxygen using perfect gas equation and Van der Waal’s
R N2 T 0.297 ¥ 145 equation. Take v = 0.0745 m3/kg-mol, Tc = –118.10°C.
p = = c
v 0.0022
= 19575 kPa or 195.75 bar
Working Substances 97

Solution 1 1
Density r = =
Given Oxygen gas with v 0.03837
3
= 26.06 kg/m3
vc = 0.0745 m /kg ◊ mol. Tc = –118.10°C = 154.9 K (ii) Using perfect gas equation
To find Critical pressure of oxygen by using Ê Ru ˆ
(i) Perfect gas equation, and pv = RT = Á ˜T
Ë M H2O ¯
(ii) Van der Waals equation.
Ê M H 2O ˆ 18 ¥ (100 ¥ 105 )
Analysis Using perfect gas equation or r=Á ˜ p=
Ë RuT ¯ 8314 ¥ 873
pc vc = RuTc
= 24.8 kg/m3
RuTc 8.314 ¥ 154.9 (iii) Using van der Waals equation with constants
or pc = =
vc 0.0745 a = 5.5113 ¥ 105 J4/kg mol2
= 17,286.42 kPa and b = 0.03 m3/kg mol.
using Van der Waals equation at critical point
Ê aˆ
Ê aˆ ÁË p + 2 ˜¯ (v - b ) = RuT
v
Á pc + 2 ˜ ( vc - b) = RuTc
Ë vc ¯ Ê 5 ˆ ¥ (v - 0.030 )
5 5.5113 ¥ 10
or Á100 ¥ 10 + ˜
27 Ru2 Tc2 Ru Tc Ë v 2
¯
with constants a = and b = the Van
64 pc 8 pc = 8314 ¥ 873
der Waals equation at critical point yields to 5.5113 0.16534
100 v + –3– – 72.58 = 0
pc vc 3 v v2
=
Ru Tc 8 It is a non-linear equation, its numerical solution
leads to v = 0.68 m3/kg mol.
3 8.314 ¥ 154.9
\ pc = ¥ = 6482.4 kPa The specific volume
8 0.0745
nv v 0.68
Example 3.31 Determine the density of the steam v = = =
m MH 2 O 18
at 100 bar and 600°C by using (a) steam table, (b) the
perfect gas equation, (c) Van der Waals equation with = 0.0378 m3/kg
constants 1 1
a = 5.5113 J 4/kg mol2 Density, r = =
v 0.0378
b = 0.030 m3/kg mol
= 26.455 kg/m3
and (d) compressibility chart.
(iv) Using compressibility chart:
Solution
For water, pc = 221-2 bar,
Given Steam at p = 100 bar = 100 ¥ 105 N/m2 Tc = 374.15°C
and T = 600°C = 873 K = 647.15 K
Reduced parameter
To find Density of steam
p 100
Analysis pr = = = 0.4522
pc 221.2
(i) Using steam table (superheated steam)
T 873
at p = 100 bar Tsup = 600°C Tr = = = 1.349 ª 1.35
Tc 647.15
v = 0.03837 m3/kg
98 Thermal Engineering

From generalized compressibility chart com- = 0.0383 m3/kg


pressibility factor, Z = 0.95 1 1
Density r = =
Then using generalized equation v 0.0383
ZRuT = 26.10 kg/m3
pv =
MH 2O
0.95 ¥ 8.314 ¥ 873
or v =
100 ¥ 100 ¥ 18

Summary
- Enthalpy hsup = hg + Cps (Tsup – Tsat) (kJ/kg)
tion throughout. Water has a constant chemical
composition in all of its phases, and therefore, it Entropy ssup = sg + C ln Ê Tsup ˆ (kJ/kg ◊ K)
ps Á
is considered as a pure substance. Ë Tsat ˜¯
Tsup 3
Specific volume vsup = v g ¥ (m /kg)
from 0°C to saturation temperature is called Tsat
sensible heat. It is denoted by hf in steam tables. 1
The density of steam, r = (kg/m3)
v
saturation temperature is called latent heat or phase change is
heat of evaporation or vaporization. It is denoted calculated as
by hfg in steam tables. we = pv
total u = h – pv
heat or total enthalpy of water. It is denoted by hg Internal latent heat = latent heat
in steam tables. – external work of evaporation
ideal gas is an ideal working substance for
wet steam. Its quality is defined by dryness all thermodynamic cycles. The characteristic gas
fraction ‘x’, which is calculated as equation for an ideal gas is
mass of dry steam mg pV = mRT
x= =
mass of total steam mw + mg where R is the specific gas constant and is
where mw = mass of moisture present obtained as
in steam sample. R universal gas constant 8.31447
R= u = =
mg = mass of dry steam in M molecular weight M
the sample. p, V and T are
Quality of steam = 100 x% related as
p1V1 pV
wet steam are calculated as = 2 2
Enthalpy hwet = hf + x hfg T1 T2
Entropy swet = sf + x sfg
Specific Volume vwet = xvg energy and change in enthalpy can be calculated
as
saturation temperature is
called superheated steam and the temperature rise Change in internal energy
above the saturation temperature is called degree DU = mCv (T2 – T1)
of superheat. Its properties are calculated as Change in enthalpy
Degree of superheat = Tsup – Tsat DH = mCp (T2 – T1)
Working Substances 99

Glossary
Triple point The point where the solid, liquid and va- Critical state The peak of the saturation curve
pour phases coexist in thermal equilibrium Quality (x) The ratio of mass of vapour present to the
Vaporization The transformation of liquid into vapour total mass of steam
by supplying heat Wetness fraction The ratio of mass of liquid present to
Evaporation The transformation of liquid into vapour the total mass of steam (= 1 – quality = 1 – x)
when liquid itself exacts its latent heat Wet steam A mixture of moisture and vapour, it is a
Condensation The transformation of a vapour into a substances that exists under the saturation curve.
liquid Phase The physical state of matter
Melting The transformation of a solid into a liquid, also Dry steam Steam at saturation temperature, which is
known as fusion free from moisture
Solidification The transformation of a liquid into a Superheated steam Dry steam at a temperature greater
solid, also called freezing than the saturation temperature
Sublimation The transformation of solid into vapour Ideal gas An ideal working substance for thermody-
Saturation The condition that exists when two or more namic cyles
phases coexist in equilibrium

Review Questions
1. Define pure substance. 8. Draw a saturation curve on a T–s diagram and
2. Explain the phase transformation of water from mention the different states of water on it.
freezing state to superheated state. 9. Define (a) triple point, (b) sublimation, and (c)
3. Describe the phase-change process of water using critical point.
a T–v diagram. 10. Explain external work of evaporation.
4. Define (a) dry and saturated steam (b) wet steam, 11. Define (a) sensible heat, (b) latent heat, and (c)
and (c) superheated steam. super heat.
5. Define (a) dryness fraction, and (b) degree of 12. Define dryness fraction and state various methods
superheat. of determination of dryness fraction.
6. Explain the following terms: 13. Explain the procedure of obtaining dryness
fraction of steam by a throttling calorimeter.
(a) Latent heat of fusion
14. Discuss (a) entropy of water, (b) entropy of
(b) Latent heat of vaporisation
evaporation, and (c) entropy of steam.
(c) Sensible heat of water 15. Draw a format of pressure entry steam table.
(d) Enthalpy 16. Define enthalpy of wet steam and superheated
(e) Dryness fraction steam with necessary equations.
(f) Superheated steam 17. Prove that for a constant pressure process,
7. What is an internal latent heat and how it relates dq = dh.
with enthalpy of the vaporization? 18. Explain why food cooks faster in a pressure
cooker than in conventional cooking?
100 Thermal Engineering

Problems
1. A rigid tank of volume 2 m3 is filled with saturated reduced by using helium instead of air in the
steam at 2 bar. It contains 0.2 m3 of moisture and tyres? What other considerations are involved in
the rest is vapour. Calculate the mass of moisture, deciding whether to use helium?
mass of vapour and the quality of steam. 10. A tank with a volume of 50 m3 is being filled with
[(a)188.5 kg (b) 2.032 kg (c) 0.01066] air. At a particular instant, the air in the tank has a
2. Steam at 0.75 bar and 150°C is condensed revers- temperature of 400 K and a pressure of 1380 kPa.
ibly. Find the heat removed and change in entro- For this system, the pressure is increasing at the
py. Sketch the process on a T–s diagram. rate of 138 kPa/s and temperature is increasing at
[2393.8 kJ/kg, 6.536 kJ/kg ◊ K] a rate of 25 K/s. Calculate the air flow rate into
3. Determine the increase in entropy of 1 kg of the tank in kg/s. [21.25 kg/s]
water when it is heated at atmospheric pressure
11. A spherical vessel of 3-m diameter contains steam
from –23°C to 152°C. Take sp. heat of fusion and
at a pressure of 7 bar (gauge) and a temperature of
superheating as 2.093 kJ/kg ◊ K and for water as
4.187 kJ/kg ◊ K. [9.0403 kJ/kg] 200° C, Find
4. Obtain specific volume, specific enthalpy, entropy (a) Total enthalpy of steam
of 2 kg of water at 1 bar and 60°C. (b) Total internal energy of steam
5. A perfect gas of 0.01 –kg mass occupies a volume (c) Total mass of steam
of 0.02 m3 at a pressure of 286.4 kPa and 30°C re- Take atmospheric pressure as 1 bar.
spectively. The gas is allowed to expand until the [(a) 156.16 MJ (b) 144.85 MJ (c) 55.16 kg]
pressure is 101 kPa and final volume is 4 times 12. Steam at 15 bar and 0.9 dry is supplied to a super
the initial volume. Calculate (a) the molal mass heater at the rate of 5,000 kg/h, the temperature
of the gas, and (b) final temperature. of steam coming out of the super heater is 300°C.
[(a) 44 kg/kmol (b) 154.5°C] The temperature of feed water supplied to the
6. A quantity of air at a temperature of 80°C is boiler is 45°C. Calculate
heated at constant pressure until the temperature (a) Heat supplied in the boiler
is 400°C, when the volume was found to be (b) Heat supplied in the super heater
1.7 m3. What was initial volume of air ? [(i) 12.045 ¥ 106 kJ/h, (ii) 2.102 ¥ 106 kJ/h]
[0.892 m3] 13. Wet steam at a pressure of 8.5 bar enters a sepa-
7. The air in a car tyre was at a pressure of 3 bar rator at the rate of 2 kg/s in which it loses 0.05
and at 20°C. After running, the pressure rises to kg/s of moisture. Then it is throttled to a pres-
3.5 bar. Estimate the temperature of air inside the sure of 5.4 bar before it is used for process work,
tyre. [64.8°C] during which it loses the heat at the rate of 920
8. A vessel having a volume of 0.32 m3 containing kW at constant pressure until the dryness fraction
air at 120 kPa, 12°C was sealed 4200 years ago becomes 0.8. Calculate the temperature of steam
and placed in a tomb of Egypt. If since that time immediately after throttling and dryness fraction
air has leaked from the vessel to cause the pres- of steam before and after the separator.
sure to drop to 119.9 kPa with no change in tem- [125.39°C, 0.975, 0.9477]
perature, determine the average rate (molecules 14. The following data was obtained in a test on a
per second) at which the air has left the vessel. combined separating and throttling calorimeter.
The pressure of the steam sample is 15 bar, the
9. An airplane has two tyres that are inflated to
pressure of steam at exit is 1 bar, and the tem-
1500 kPa. Each tyre has a volume of 0.021 m3.
perature is 150°C, discharge from the separating
How much would the weight of the airplane be
calorimeter is 0.5 kg/min, and the discharge from
Working Substances 101

the throttling calorimeter is 10 kg/min. Calculate Water separated = 2 kg


the dryness fraction of steam. The steam supplied to throttling
[x = 0.9477] calorimeter = 20.5 kg
15. The following observations were obtained with a Temperature of steam after throttling = 120°C
separating and throttling calorimeter arranged in Barometer reading 760 mm of Hg
series: Final pressure = 65 mm of Hg
Pressure of steam = 12 bar Estimate the quality of steam. [0.88]

Objective Questions
1. The specific volume of water during freezing 9. At the critical point, the temperature of water is
(a) increases (b) remains constant equal to
(c) decreases (d) none of the above (a) 0°C (b) 100°C
2. The latent heat of vaporization with increase in (c) 374°C (d) –100°C
pressure of water 10. The total enthalpy of steam at 10 bar is
(a) increases (b) remains constant 2000 kJ/ kg. The condition of steam is
(c) decreases (d) none of the above (a) wet (b) dry and saturated
3. With increase in pressure, the saturation tempera- (c) superheated (d) none of the above
ture of water 11. Dryness fraction of steam is given by
(a) increases (b) remains constant Mass of dry steam in a sample
(c) decreases (d) none of the above (a)
Mass of water particles in thee sample
4. The specific volume of wet steam is given by
Mass of water particles in the sample
vg (b)
(a) (b) xvf Mass of dry steam in the sample
x Mass of wet steam in the sample
(c) xvg (d) x 2vg (c)
Mass of dry steam in the sammple
5. Internal latent heat of steam is given by
Mass of dry steam in the sample
(a) hfg – pvg (b) hg – pvg (d)
Total mass of steam in the sample
(c) hsup – pvg (d) hfg + pvg
12. To obtain the dryness fraction of steam from a
6. Specific internal energy of steam is given by
throttling calorimeter, the steam after throttling
(a) Cv (T2 – T1) (b) h – pv
should be
(c) h + pv (d) hfg + pvg
(a) wet
7. During throttling process
(b) dry and saturated
(a) h2 = h12 (b) h2 = h1
(c) superheated by atleast 5°C
h fg h fg
(c) h2 = h1 + (d) h1 = h2 + (d) completely condensed
Tsat Tsat
13. Throttling calorimeter is used to measure the
8. Critical point is the point at which dryness fraction up to
(a) melting point and boiling temperature be- (a) 0.7 (b) 0.98
come equal (c) 1.0 (d) none of the above
(b) change in volume during evaporation be- 14. Entropy of wet steam is given by
come zero h fg
(c) latent heat of evapoartion becomes zero. (a) s fg + (b) sf + x(sg – sf)
Tsat
(d) none of the above ÊT ˆ
(c) sf + sfg (d) s g + C ps ln Á sup ˜
ËT ¯
sat
102 Thermal Engineering

15. The perfect gas 17. The specific gas constant of any perfect gas
(a) has no intermolecular forces of attraction or (a) increases with increase in temperature
repulsion (b) increase with increase in pressure
(b) does not change its phase during a process (c) is a function of pressure and temperature
(c) obeys Boyle’s and Charles’s laws (d) is a constant
(d) all of the above 18. The ratio of two specific heats of a perfect gas
16. Universal gas constant of a perfect gas (a) always remains constant
(a) increases with increase in temperature (b) always varies with pressure
(b) decrease with temperature (c) always varies with temperature
(c) increases with increase in molar mass (d) none of the above
(d) always remains constant

18. (c). 17. (d)


16. (d) 15. (d) 14. (b) 13. (b) 12. (c) 11. (d) 10. (a) 9. (c)
8. (c) 7. (b) 6. (b) 5. (a) 4. (c) 3. (a) 2. (c) 1. (a)
Answers
First Law Applied to Non-Flow Systems 103

4
First Law Applied to
Non-Flow Systems

Introduction
In Chapter 2, we have considered various forms of energy and energy transfer, and we have developed a
general relation for energy balance. In Chapter 3, we have discussed the properties of working substances:
steam, ideal gases and real gases. In this chapter, we will apply the energy balance relation to those systems
that do not involve mass flow across their boundary; that is, closed systems.

volume of the system remains constant, the process


IDEAL-GAS PROCESSES
is called constant volume process or isochoric
There are an infinite variety of processes. The five process, for example, processes taking place in a
important non-flow processes commonly dealt in pressure cooker, gas cylinder, etc.
engineering applications are (b) Properties Relationship The properties rela-
1. Constant-volume process tionship for any system undergoing a process be-
2. Constant-pressure process tween two states is expressed as
3. Constant-temperature process p1V1 pV
= 2 2
4. Constant-entropy process T1 T2
5. Polytropic process For a constant-volume process,
The processes considered in this chapter are re- V1 = V2
versible non-flow and can be represented by a solid
Then the above relation reduces to
curve on thermodynamic plots. A thermodynamic
system can undergo either one or a combination of p1 p
= 2 ...(4.1)
processes. T1 T2
(c) p–V and T–S Diagrams A constant-volume
process is represented by a vertical straight line on
(a) If any system of gas undergoes a a p–V diagram and a curved line on a T–S diagram
process in such a way that during the process, the as shown in Fig. 4.1.
104 Thermal Engineering

p T Thus, for a constant-volume process, the change


1
of internal energy is always equal to heat trans-
p1 1 p1
ferred during the process.
Cooling
p2
p2 2 (g) Change of Enthalpy The change of specific
2
enthalpy of an ideal gas during a process is calcu-
v s lated as
(a) (b)
Dh = Cp (T2 – T1) (kJ/kg)
Fig. 4.1 Constant volume cooling process on p-v and
T–s diagrams
where Cp = Specific heat at constant pressure,
kJ/kg ◊ K
(d) Work Done during the Process For a non-flow (T2 – T1) = Temperature difference in °C or K.
system, the work done can be obtained either from For a given mass system, the change in enthalpy
the area under the curve or mathematically as
2
DH = m Dh = mCp (T2 – T1) (kJ) ...(4.6)
W1–2 = Ú
1
pdV
(h) Change in Entropy The change in entropy of
For a constant-volume process, an ideal gas can be calculated as
dV = 0 2 dQ 2 dT
Hence W1–2 = 0 ...(4.2)
DS = Ú
1 T
= Ú
1
m Cv
T
It is also evident from the p–V diagram, as there ÊT ˆ
= mCv ln Á 2 ˜ (kJ/K) ...(4.7)
is no area under the vertical line. Ë T1 ¯
(e) Change of Internal Energy For an ideal gas, the where T is the absolute temperature in kelvin (K).
change of specific internal energy is calculated as
Example 4.1 A certain gas occupies a volume of
Du = Cv (T2 – T1) (kJ/kg) 0.3 m3 at a pressure of 2 bar. The temperature of the gas at
where, Cv = specific heat at constant volume in this state is 350 K. The gas undergoes a thermodynamic
kJ/kg ◊ K and (T2 – T1) is temperature difference in constant-volume process until the pressure raises to 7 bar.
°C or K. For a given mass system, the change of Determine the temperature at the end of the process,
internal energy change in internal energy, change in enthalpy and change
in entropy during this process. Take Cv = 0.712 kJ/kg ◊ K
DU = m Du = mCv (T2 – T1) (kJ) …(4.3) and R = 0.287 kJ/kg ◊ K.
where, m = mass of the system in kg.
Solution
(f ) Heat Transferred In general, the heat transferred
Given A constant volume process
during a process is expressed as
2 State 1: V = 0.3 m3 p1 = 2 bar = 200 kPa T1 = 350 K
Q= Ú 1
m C dT
State 2: V = 0.3 m 3
p2 = 7 bar = 700 kPa
or Q = mass ¥ specific heat ¥ change of Cv = 0.712 kJ/kg ◊ K, R = 0.287 kJ/kg ◊ K
temperature
To find
For a constant-volume process (i) Final temperature of the gas,
Q = mCv (T2 – T1) = DU ...(4.4) (ii) Change in internal energy,
and for unit mass, (iii) Change in enthalpy, and
2 (iv) Change in entropy.
q= Ú1
Cv dT = Cv (T2 - T1 ) (kJ/kg) ...(4.5)
Analysis
We have R = Cp – Cv
First Law Applied to Non-Flow Systems 105

It gives Cp = R + Cv State 1: V = 0.2 m3


= 0.287 + 0.712 @ 1.0 kJ/kg ◊ K p1 = 350 kPa
Using characteristic gas equation to calculate mass of the T1 = 15°C + 273 = 288 K
gas in the rigid tank mass State 2: T2 = 170°C + 273 = 443 K
p1V V = 0.2 m3
m = Q = 50 kJ
RT1
Cv = 743 J/kg ◊ K = 0.743 kJ/kg ◊ K
(200 kPa) ¥ (0.3 m3 )
=
(0.287 kJ/kg ◊ K) ¥ (350 K) To find
= 0.5973 kg (i) Final state of the gas,
(ii) Change in internal energy,
(i) The temperature of gas after constant-volume
(iii) Change in entropy.
process
p1 p2 Assumptions
We have =
T1 T2 (i) Nitrogen as a perfect gas.
p 7 (ii) The ratio of two specific heats, g = 1.4 for perfect
It gives T2 = 2 T1 = ¥ (350 K)
p1 2 gas.
= 1225 K or 952°C Analysis
(ii) Change in internal energy Cp
DU = m Cv (T2 – T1) We have R = Cp – Cv and g =
Cv
= (0.5973 kg) ¥ (0.712 kJ/kg ◊ K) Hence R = Cv (g – 1)
(1225 – 350)(K) = 743 ¥ (1.4 – 1) = 297 J/kg ◊ K
or DU = 372.12 kJ = 0.297 kJ/kg ◊ K.
(iii) Change in enthalpy Using characteristic gas equation to calculate mass of the
DH = m Cp (T2 – T1) gas in the rigid tank.
= (0.5973 kg) ¥ (1.0 kJ/kg ◊ K) p1V
m=
(1225 – 350)(K) RT
or DH = 522.63 kJ (350 KPa) ¥ (0.2 m3 )
(iv) Change in entropy = = 0.818 kg
(0.297 kJ/kg ◊ K) ¥ (288 K)
ÊT ˆ
DS = m Cv ln Á 2 ˜ The temperature of gas after 50 kJ of heat supply
ËT ¯1 Q = m Cv (T2 – T1)
Ê 1225 ˆ 50 kJ = (0.818 kg) ¥ (0.743 kJ/kg ◊ K) (T2 – 288 K)
DS = 0.5973 ¥ 0.712 ¥ ln Á
Ë 350 ˜¯ or T2 = 370.26 K = 97.26°C
= 0.5328 kJ/K Change in internal energy
DU = Q = 50 kJ (For constant volume process)
Example 4.2 A rigid tank of 0.2 m3 capacity contains
Change in entropy
nitrogen at 350 kPa and 15°C. The tank is exposed to a
constant temperature source at 170°C, which results in ÊT ˆ
DS = m Cv ln Á 2 ˜
50 kJ of heat being tranferred to nitrogen. Determine ËT ¯ 1
(a) Final state of the gas, Ê 370.28 ˆ
DS = 0.818 ¥ 0.743 ¥ ln Á = 0.153 kJ/K
(b) Change in internal energy, Ë 288 ˜¯
(c) Change in entropy, Take Cv = 743 J/kg ◊ K
for Nitrogen. Example 4.3 The usual cooking gas (mostly methane)
cylinder is about 25 cm in diameter and 80 cm in height.
Solution It is charged to 12 MPa at a room temperature of 27°C.

Given A constant temperature and constant volume (a) Assuming the ideal gas law, find the mass of gas
processes. filled in the cylinder.
106 Thermal Engineering

(b) If the cylinder is protected against the excessive (b) Properties Relationship In general, the prop-
pressure by means of a fusible plug, at what erties are related betwen two states as
temperature should the plug melt to limit the
maximum pressure of 15 MPa? p1V1 pV
= 2 2
T1 T2
Solution
But for a constant pressure process,
Given A constant-volume process in a cooking cylinder p1 = p2,
d = 25 cm = 0.25 m, L = 80 cm = 0.8 m V1 V
then = 2 ...(4.8)
State 1: p1 = 12 MPa = 12000 kPa, T1 T2
T1 = 27°C + 273 = 300 K
State 2: p2 = 15 MPa = 15000 kPa Ú
(c) Work Done The pdV work done during the
process can be evaluated from an area under the
To find curve on a p–V diagram or can be calculated as
(i) Initial mass of gas in the cylinder, and
2
(ii) Final temperature. W= Ú
1
pdV = p (V2 - V1 ) (kJ) ...(4.9)
Analysis The molecular mass of methane
where p is expressed in kPa (kN/m2), and V is ex-
MCH4 = MC + 2 MH2
pressed in m3.
= 12 + 2 ¥ 2 = 16 kg/kmol
Specific gas constant of methane (d) p-V and T-S Diagrams A constant–pressure
Ru 8.314 process is a horizontal straight line on a p–V plane
R = = = 0.5196 kJ/kg ◊ K and an inclined curve on a T–S plane as shown in
MCH 4 16
Fig 4.2.
The volume of gas cylinder
p 2 p
d L = ¥ (0.25) ¥ 0.8 = 0.0393 m3
2
V =
4 4
Using characteristic gas equation to calculate mass of gas
p1V
m =
RT
(12000 KPa) ¥ (0.0393 m3 )
= = 3.02 kg
(0.5196 kJ/kg ◊ K) ¥ (300 K)
The final temperature of gas, when the pressure
approaches 15000 kPa,
Êp ˆ Ê 15000 ˆ
T2 = T1 Á 2 ˜ = 300 ¥ Á
Ë p ¯
1 Ë 12000 ˜¯
= 375 K or 102°C

(a) If any system of gas undergoes a


process in such a way that during the process, the
pressure of the gas remains constant then such pro-
Fig. 4.2
cess is called a constant pressure or isobaric pro-
diagrams
cess.
First Law Applied to Non-Flow Systems 107

(e) Change of Internal Energy For any process, it P = 10.04 W g = 1.4


is calculated as p = 0.1 MPa = 100 kPa To = O°C = 273 K
DU = m Cv (T2 – T1) (kJ) To find
(i) Specific heat at constant pressure Cp,
(f ) Heat Transferred For a constant-pressure
(ii) Specific gas constant R, and
process, the heat transferred can be obtained as
(ii) Density of air at 0°C and 0.1 MPa.
2
Q= m Ú1
C p dT = m C p (T2 - T1 ) (kJ) ...(4.10) Assumptions
and for a unit mass system (i) Air as an ideal gas, and
(ii) Process is reversible.
q = Cp (T2 – T1) (kJ/kg) ...(4.11)
Analysis
where Cp must be expressed in kJ/kg ◊ K or
(i) The amount of heat supplied during the process
kJ/kg ◊ °C and (T2 – T1) as temperature difference
in °C or K. Q = P Dt = 10.04 ¥ 300 = 3012 J
Further the heat transferred during a constant
(g) Change in Enthalpy For any process, it is cal- pressure process is also expressed as
culated as Q = m Cp (T2 – T1)
DH = m Cp (T2 – T1) (kJ) Q
and Dh = Cp (T2 – T1) (kJ/kg) Cp =
m(T2 - T1 )
Thus for a non-flow constant pressure process, (3012 J)
the change in enthalpy is always equal to the heat =
(0.05 kg) ¥ (347 – 287)(K)
transferred during the process. = 1004 J/kg ◊ K
(h) Change in Entropy It is calculated as (ii) The specific gas constant R
Cp
2 dQ 2 dT We have g=
DS = Ú1 T
= Ú1
mCp
T
Cv
Cp 1004
ÊT ˆ or Cv = = = 717 J/kg ◊ K
= mC p ln Á 2 ˜ (kJ/K) ...(4.12) g 1.4
Ë T1 ¯ The gas constant
The temperature T must be expressed in R = Cp – Cv
K (kelvin). = 1004 – 717 = 287 J/kg ◊ K
or R = 0.287 kJ/kg ◊ K
Example 4.4 In an experimental determination of (iii) Density of air at 0°C and 100 kPa
specific heat of air at constant pressure, 50 g of air at From the characteristic equation for an ideal gas
constant pressure was heated from 14°C to 74°C during pv = RT
a period of 300 seconds by a continuous electrical p
or = RT
heating method, The power consumed was 10.04 watts. r
Determine p (100 ¥ 103 N/m2 )
(a) Specific heat of air at constant pressure, or r= =
RT (287 J/kg ◊ K) ¥ (273 K)
(b) Characteristic gas constant R for air,
= 1.276 kg/m3
(c) The density of air at 0°C and 0.1 MPa.
Take Y = 1.4 for air. Example 4.5 1 kg of nitrogen at a temperature of
Given A constant-pressure process 150°C occupies a volume of 0.2 m3. The gas undergoes
a fully restricted constant-pressure expansion without
m = 50 g = 0.05 kg T1 = 14°C + 273 = 287 K friction to a final volume of 0.36 m3. Calculate the final
Dt = 300 s T2 = 74°C + 273 = 347 K temperature, work done, heat transferred and change in
entropy.Take Cv = 743 J/kg K, R = 0.297 kJ/kg ◊ K.
108 Thermal Engineering

Solution (ii) Work done


For a constant-pressure process, the work done
Given A constant-pressure expansion process of nitro-
can easily be calculated as area under the curve
gen is shown in Fig. 4.3.
1–2 and it is
m = 1 kg Cv = 743 J/kg K R = 0.297 kJ/kg K
W1–2 = p(V2 – V1)
State 1: T1 = 150°C + 273 = 423 K, V1 = 0.2 m3 Using numerical values
State 2: V2 = 0.36 m3 W1–2 = (624.15 kJ) ¥ (0.36 – 0.2) (m3)
= 100.5 kJ
To find (iii) Heat transferred Q1–2
(i) Final temperature T2, It is calculated as
(ii) Work done,
Q1–2 = m Cp (T2 – T1)
(ii) Heat transferred,
Then Q1–2 = (1 kg) ¥ (1.04 kJ/kg K) ¥ (761.4 K
(iv) Change in entropy.
– 423 K)
p
= 352 kJ
T (iv) Change of entropy
2
1 p=C ÊT ˆ
p 2 DS1–2 = mCp ln Á 2 ˜
ËT ¯1
Ê 761.4 ˆ
= (1 kg) ¥ (1.04 kJ/kg K) ¥ ln Á
1 Ë 423 ˜¯
= 0.611 kJ/K
V
V1 V2 S

(a) p–V diagram (b) T–S diagram 4.1.3


Fig. 4.3
(a) If a system undergoes a process in
Analysis From the characteristic gas equation such a way that during the process, the temperature
m RT1 of the system remains constant, such a process is
p1 =
V1 called an isothermal process or constant tempera-
(1 kg) ¥ (0.297 kJ/kg ◊ K) ¥ (423 K) ture process.
=
(0.2 m3 ) (b) Relation between p, V and T The property re-
= 624.15 kPa
lation
The specific heat at constant pressure can be calcu- p1V1 pV
= 2 2
lated as T1 T2
C p = R + Cv for a constant temperature process (T1 = T2) the
= (0.297 kJ/kg K) + (0.743 kJ/kg K) relation is reduced to
= 1.04 kJ/kg K
p1V1 = p2V2
(i) Final temperature T2
The properties relationship for constant pressure or pV = C (constant) ...(4.13)
process, It is the equation of a hyperbola, and hence
V1 V an isothermal process is also called a hyperbolic
= 2
T1 T2 process.
V 0.36 (c) p–V Diagram An isothermal process is rep-
or T2 = 2 ¥ T1 = ¥ ( 423 K)
V1 0.2 resented on a p–V and T–S diagrams as shown in
= 761.4 K = 484.4°C Fig. 4.4.
First Law Applied to Non-Flow Systems 109

(e) Change of Internal Energy For any system,


DU1–2 = m Cv (T2 – T1)
But T1 = T2 (for isothermal process)
Hence DU = 0
According to the first law of
( f) Heat Transferred
thermodynamics for any process,
2
Q= Ú
1
TdS = T (S2 – S1) ...(4.17)
or Q = DU + W
Here DU = 0
Ê V2 ˆ
Hence Q1–2 = W1–2 = p1V1 ln Á ˜
Ë V1 ¯
Ê p1 ˆ
= m RT ln Á ˜ ...(4.18)
Ë p2 ¯
During an isothermal process, the heat transfer
is equal to work transfer.
(g) Change in Enthalpy For any process, it is
Fig. 4.4
process. calculated as
Dh = Cp (T2 – T1)
(d) Work Done The work done during a process
But T1 = T2
is expressed as
2 Thus Dh = DH = 0
W1–2 = Ú1
pdV
(h) Change in Entropy It is calculated as
For an isothermal process, 2 dQ 2 dW 2 pdV
pV = C DS = Ú
1 T 1 T
= Ú
1
= Ú T
C p mR
\ p= But = from p V = m RT
V T V
2C V2 dV
Then W1–2 =
1 VÚ dV = C ÚV1 V Then DS =
1
2
Ú mR
dV
V
Ê V2 ˆ
= m R ln Á ˜ ...(4.19)
Ë V1 ¯
ÊV ˆ Ê p1 ˆ
= C ln Á 2 ˜ or = m R ln Á ˜ ...(4.20)
Ë V1 ¯ Ë p2 ¯
Using C = p1V1
Ê V2 ˆ A mass of 0.8 kg of air at 1 bar and 25°C
we get W1–2 = p1V1 ln Á ˜ ...(4.14)
ËV ¯ is contained in a gas-tight frictionless piston–cylinder
Further, p1V1 = m R T1 1
device. The air is now compressed to a final pressure of
ÊV ˆ
Then W1–2 = m R T1 ln Á 2 ˜ ...(4.15) 5 bar. During the process, the heat is transferred from
Ë V1 ¯ air such that the temperature inside the cylinder remains
From properties relation, we have constant. Calculate the heat transferred and work done
V2 v2 p1 during the process and direction of each in the process.
= =
V1 v1 p2
Solution
Ê p ˆ
Then W1–2 = m R T1 ln Á 1 ˜ ...(4.16) Given Constant temperature process
Ë p2 ¯ m = 0.8 kg,
110 Thermal Engineering

State 1: T1 = 25°C + 273 = 298 K, Solution


p1 = 1 bar = 100 kPa,
Given Constant temperature and constant pressure
State 2: p2 = 5 bar = 500 kPa; T2 = T1
processses.
To find Heat transfer and work transfer
State 1: p1 = 60 kPa V1 = 0.1 m3, T1 = 800 K,
State 2: T2 = T1 V2 = 0.5 V1, V2 = 0.05 m3
State 3: p2 = p3 V3 = 0.5 V2, V3 = 0.025 m3
Cp = 1.005 kJ/kg ◊ K.
To find
(i) Total work transfer, and
(ii) Total heat transfer.
p– Assumption
Gas constant for air R = 0.287 kJ/kg ◊ K.
Assumptions
Analysis The isothermal and isobaric processes on p–V
(i) Air as perfect gas,
plane are shown in Fig.4.6.
(ii) Gas constant for air
R = 0.287 kJ/kg ◊ K, and
(iii) The specific heat at constant volume
Cv = 0.7165 kJ/kg ◊ K

Analysis For isothermal compression process, the


work done is given by Eq.(4.16).
Ê p ˆ
W1–2 = m RT1 ln Á 1 ˜
Ë p2 ¯

= 0.8 ¥ 0.287 ¥ 298 ¥ ln ÊÁ


100 ˆ
Ë 500 ¯˜
= – 110.12 kJ
Fig. 4.6
It is the work supplied to the system during compression processes
process.
For isothermal process, the heat transfer From the characteristic gas equation,

Q1–2 = W1–2 = – 110.12 k J p1V1 (60 kPa) ¥ (0.1 m3 )


m= = = 0.0261 kg
The heat is rejected from the system during compression RT1 (0.287 kJ/kg ◊ K) ¥ (800 K)
process. (i) For isothermal compression process 1-2,
V1 0.1
Example 4.7 Air initially at 60 kPa pressure, 800 K p2 = p1 = (60 kPa) ¥ = 120 kPa
V2 0.05
temperature and 0.1 m3 volume is compressed isother-
Work transfer is given by Eq. (4.14)
mally until the volume is halved, and subsequently the
air is cooled at contant pressure till the volume is halved Ê V2 ˆ
W1–2 = p1V1 ln
ÁË V ˜¯
again. Sketch the process on a p–V plane and determine 1
Ê 0.05 ˆ
(a) total work interaction, and = (60 kPa) ¥ (0.1 m3) ¥ ln Á
Ë 0.1 ˜¯
(b) total heat interaction.
= –4.158 kJ
Assume ideal-gas behaviour for air and take Cp = For isothermal process, the heat transfer
1.005 kJ/kg ◊ K.
Q1–2 = W1–2 = – 4.158 kJ
First Law Applied to Non-Flow Systems 111

(ii) For constant-pressure process 2–3,


V 0.025
T3 = T2 3 = (800 K) ¥ = 400 K
V2 0.05
Work transferred during constant-pressure pro-
cess is given by Eq. (4.9)
W2–3 = p2 (V3 – V2) = (120 kPa)
¥ (0.025 – 0.05) = – 3 kJ
Heat transferred during constant-pressure process
2–3,
Q2–3 = m Cp (T3 – T2)
= (0.0261kg) ¥ (1.005 kJ/kg ◊ K)
¥ (400 K – 800 K) = – 10.5 kJ
(a) Total work transfer,
WTotal = W1–2 + W2–3 = – 4.153 kJ + (– 3 kJ)
= – 7.153 kJ
(b) Total heat transfer,
QTotal = Q1–2 + Q2–3 = – 4.153 kJ + (– 10.5 kJ) Fig. 4.7
= – 14.643 kJ
(iii) During constant pressure cooling process, 3–1:
Example 4.8 An ideal gas is heated at constant p3 = p1
volume until its temperature is doubled. Then it is
w3–1 = p3 (v1 – v3) = m R (T1 – T3)
expanded isothermally till it reaches its original pressure.
Finally, the gas is cooled at constant pressure till it is = R (T1 – 2T1) = – R T1
restored to the original state. Determine the net work Net work done = w1–2 + w2–3 + w3–1
done per kg of gas. Ê p2 ˆ
= 0 + 2RT1 ln Á ˜ – R T1
Solution Ë p1 ¯

Given An ideal-gas cycle È Ê ˆ ˘


= R T1 Í2 ln p2 - 1˙
v1 = v2 T2 = 2T1 T2 = T3 p3 = p1 ÁË p ˜¯
ÍÎ 1 ˙˚
To find Net work done per kg of gas
Assumption All processes in the cycle are reversible.
Analysis
If any process of expansion or compression is
(i) During constant-volume heating process 1–2, carried out without exchange of heat between
(v1 = v2) system and surroundings, i.e., operation in an
w1–2 = 0 insulated cylinder, the process is called adiabatic
(ii) During isothermal expansion process 2–3: process. During this process, the system neither
T2 = T3 = 2T, receives nor rejects any amount of heat.
Ê v3 ˆ When an adiabatic process is reversible,
w2–3 = p2 v2 ln Á ˜
Ë v2 ¯ i.e., frictionless, etc., then this process is called
Ê p2 ˆ Ê p2 ˆ isentropic or reversible adiabatic or constant
= p2 v2 ln Á ˜ = RT2 ln Á ˜ entropy process.
Ë p3 ¯ Ë p1 ¯

Ê p2 ˆ (a) Law of an Isentropic Process For unit mass


= 2R T1 ln Á ˜ of a system, the energy equation for a non-flow
Ë p1 ¯
112 Thermal Engineering

process is expressed as T2 p2 V2
or = ...(4.24)
dq = du + dw T1 p1V1
For adiabatic process, dq = 0 For an isentropic process,
\ du + dw = 0 g
pV = C or
g
pv = C
or Cv dT + pdv = 0 ...(4.21) g g
or p1V1 = p2V2
For unit mass of an ideal gas, the characteristic g -g -g
equation p2 Ê V1 ˆ ÊV ˆ Êv ˆ
or = Á ˜ = Á 2˜ =Á 2˜ ...(4.25)
p v = RT p1 Ë V2 ¯ Ë V1 ¯ Ë v1 ¯
1
Differentiating both sides, we get -
v2 V2 Êp ˆ g
or = = Á 2˜ ...(4.26)
p dv + vdp = RdT ...(4.22) v1 V1 Ë p1 ¯
pd v + vdp Substituting Eq. (4.25) in Eq. (4.24), we get
or dT =
R -g 1-g 1-g
T2 Ê V2 ˆ Ê V2 ˆ Ê V2 ˆ Êv ˆ
Substituting for dT in Eq. (4.21), we have ÁË V ˜¯ = ÁË V ˜¯ =Á 2˜
T1 ÁË V1 ˜¯
= ...(4.27)
1 1 Ë v1 ¯
pd v + vdp
Cv + pd v = 0 Using Eq. (4.26) in Eq. (4.24), we get
R
1 g -1
or Cv pdv + Cv vdp + R pdv = 0 -
T2 Ê p ˆ Ê p ˆ g Ê p2 ˆ g
= Á 2˜ Á 2˜ =Á ˜ ...(4.28)
Using R = Cp – Cv , a relation between two specific T1 Ë p1 ¯ Ë p1 ¯ Ë p1 ¯
heats, we have
(c) p–V Diagram The p–V diagram for an isentropic
Cv p dv + Cv vdp + (Cp – Cv ) p dv = 0 process is shown in Fig 4.8.
or Cv vdp + Cp p dv = 0
Dividing both sides by p vCv , we get
dp C p d v
+ =0
p Cv v
Cp
Assuming ratio of two specific heats =g
Cv
and integrating the above equation, we get
loge p + g loge v = loge C
where loge C is the constant of integration
Taking antilog on both sides, we get
pvg = C (a constant)
It is the law of isentropic expansion or compres-
sion. For a given mass system, it is
pVg = C (constant) ...(4.23)
(b) Relation between p, V and T The thermody-
namic properties for a perfect gas are related as
p1 V1 p V
= 2 2
T1 T2 Fig. 4.8 or an isentropic
expansion process
First Law Applied to Non-Flow Systems 113

(d) Work done During the Process The work (h) Change of Entropy For isentropic process,
done during a non-flow process is expressed as S1 = S2 ;
2
W1–2 = Ú1
pdV Thus DS = Ú
dQ
T
=0 ...(4.31)
For an isentropic process
C Example 4.9 A quantity of gas occupying 0.14 m3
pVg = C, or p = g
V at a pressure of 1400 kPa and 300°C is expanded
2 C V2 isentropically to 280 kPa. Calculate (a) mass of gas,
Then W1–2 = Ú1 V g
dV = C Ú V1
V - g dV (b) final temperature , (c) work transfer , and (d) change
in entropy.

or W1–2
È V - g +1 ˘
= CÍ ˙
V2

=
(
C V2 - g +1 - V1- g +1 ) Take Cp= 1.04 kJ/kg ◊ K and Cv = 0.74 kJ/kg ◊ K

ÍÎ -g + 1 ˙˚ V1
1- g
Solution
g g
Substituting for C as p1V1 and p2V2 Given Isentropic expansion of a gas
- g +1 - g +1 = 0.14 m3
p2 V2 V2 - p1 V1 V1 State 1: V1
W1–2 =
1- g p1 = 1400 kPa
T1 = 300°C = 573 K
We get State 2: p2 = 280 kPa,
p2V2 - p1V1 mR (T2 - T1 ) Cp = 1.04 kJ/kg ◊ K
W1–2 = = ...(4.29)
1- g 1- g Cv = 0.74 kJ/kg ◊ K
Cp
If we put R = Cp – Cv and g = , we get
Cv
R
Cv =
g -1
Using in Eq. (4.29), we get
W1–2 = – m Cv (T2 – T1) = – DU ...(4.30)
This expression explains that during an isen-
tropic expansion process, the work done is always
equal to decrease in internal energy during the pro-
cess. Fig. 4.9
(e) The Change of Internal Energy As explained
by Eq. (4.30), the change of internal energy during To find
isentropic process (i) Mass of the gas,
(ii) Final temperature,
DU1–2 = –W1–2 (adiabatic work)
(iii) Work transfer, and
or DU1–2 = m Cv (T2 – T1)
(iv) Change in entropy.
(f ) Heat Transferred During an adiabatic opera-
Assumption The given gas is a perfect gas.
tion, the heat transferred is zero, i.e., no area under
the curve on a T–S diagram. Analysis

Q1– 2 = 0 (i) For an ideal gas


p1V1 = m RT1
(g) Change in Enthalpy For any process where R = Cp – Cv = 1.04 – 0.74
D H = m Cp (T2 – T1) = 0.3 kJ/kg ◊ K
114 Thermal Engineering

Using values, we get 1


Multiplying both sides by , we get
p1V1 1400 ¥ 0.14 bv p
m= = =1.14 kg
RT1 0.3 ¥ 573 1 + b d v dp
+ =0
(ii) Final temperature b v p
For an isentropic process Integrating both sides
g -1
T2 Ê p2 ˆ g Ê1+ bˆ
= Á ˜ ÁË b ˜¯ ln (v) + ln (p) = ln (C)
T1 Ë p1 ¯
Cp Taking antilog
1.04
where g= = = 1.405 Ê 1+ b ˆ
Cv 0.74 ÁË b ˜¯ =C
1.405 -1 pv
Ê 280 ˆ 1.405 1+ b
then T2 = 573 ¥ Á
Ë 1400 ˜¯ Using g=
b
= 360 K = 87 °C then pvg = C Proved
(iii) Work transfer
For an isentropic process Example 4.11 1 kg of gas occupying 0.15 m3 at a
pressure of 12 bar is heated at constant pressure until its
p2V2 - p1V1 mR (T2 - T1 ) volume is 0.28 m3. The gas is then expanded adiabatically
W= =
1- g 1- g until its volume is 1.5 m3. Calculate
1.14 ¥ 0.3 ¥ (360 - 573) (a) Temperature at the end of constant-pressure
= = 180 kJ
1 - 1.405 heating and at the end of adiabatic expansion.
(b) Total work done
(iv) Change in entropy
Take Cp = 1.068 kJ/kg ◊ K and Cv= 0.775 kJ/kg ◊ K
For an isentropic process S1 = S2
Thus DS = 0 Solution

Example 4.10 The following equation, which con- Given A gas undergoes constant pressure and adiabatic
nects u, p and v for several gases is processes.
u = a + bpv m = 1 kg, Cp = 1.068 kJ/kg ◊ K
where a and b are constants. Prove that for a reverseble Cv = 0.775 kJ/kg ◊ K
adiabatic process State 1: p1 = 12 bar = 1200 kPa V1 = 0.15 m3
b+1 State 2: V2 = 0.28 m3 p2 = p1
p vg = constant, where g =
b State 3: V3 = 1.5 m3 s1 = s2
Solution For a reversible adiabatic process
Q =0
For a process of unit mass system
dq = du + pd v
or 0 = du + p d v ...(i)
We have u = a + bpv
Differentiating both sides,
du = b vdp + b p d v
Using in Eq. (i);
0 = b vdp + b p dv + p d v
or p (1 + b) d v + b v dp = 0
First Law Applied to Non-Flow Systems 115

To find g = 1.4 p2 = 0.4 MPa = 400 kPa


s1 = s2 p2 = p3
(i) Temperature after constant pressure heating,
(ii) Temperature afer adiabatic expansion, and To find
(iii) Total work done. (i) Work done,
Analysis For an ideal gas (ii) Heat transfer, and
(iii) Change in entropy.
R = Cp – Cv = 1.068 – 0.775 = 0.293 kJ/kg ◊ K
and p1V1 = m RT1 Assumptions
p1V1 1200 ¥ 0.15 (i) The system is non-flow, and
Then T1 = = = 614 K = 341°C (ii) Air is a perfect gas.
mR 1 ¥ 0.293
(i) Temperature after constant pressure heating p
p=C
V 614 ¥ 0.28 0.4 MPa
2 3
T2 = T1 2 =
V1 0.15 g
pV = C
= 1146 K = 873°C
Work done W1–2 = p1(V2 – V1)
= 1200 ¥ (0.28 – 0.15) = 156 kJ 80 kPa 1

(ii) During adiabatic expansion V


V2 V1
Cp 1.068
g= = = 1.38 p–V diagram
Cv 0.775
T
Temperatures and volumes are related as
1- g 1-1.38 3
T3 Ê V3 ˆ Ê 1.5 ˆ
=Á p=C
T2
= Á ˜
Ë V2 ¯ Ë 0.28 ¯˜ = 0.525
2

or T3 = 1146 ¥ 0.525 = 601.65 K = 328.65°C V=C

m R (T3 - T2 ) 1
Work done W2–3 = S
1- g T–s diagram
1 ¥ 0.293 ¥ (601.65 - 1146) Fig. 4.11 – –
W =
1 - 1.38
= 419.72 kJ Analysis
(iii) Total work transfer (i) Work transfer
Wtotal = W1–2 + W2–3 = 156 + 419.72 = 575.72 kJ The temperature after isentropic compression1–2;
g –1 1.4 – 1
Example 4.12 0.5 kg of air is compressed reversibly Ê p2 ˆ g Ê 400 ˆ 1.4
= 333 ¥ Á
Ë 80 ˜¯
T2 = T1 Á ˜
and adiabatically from 80 kPa and 60°C to 0.4 MPa and Ë p1 ¯
is then expanded at constant pressure to the original = 527.41 K
volume. Sketch the process on p-V and T-S diagrams, For constant-pressure process 2–3;
compute work transfer, heat transfer and change in
T3 V3
entropy for whole path. =
T2 V2
Take R = 0.287 kJ/kg ◊ K and g = 1.4
But V3 = V 1
Solution 1 1
T3 V1 Ê p2 ˆ g Ê 400 ˆ 1.4
= = Á ˜ = ÁË
80 ˜¯
Given Reversible adiabatic and constant pressure Hence, = 3.157
T2 V2 Ë p1 ¯
processes;
m = 0.5 kg p1 = 80 kPa. or T3 = T2 ¥ 3.157 = 527.41 ¥ 3.157
R = 0.287 kJ/kg ◊ K T1 = 60°C + 273 = 333 K = 1665.0 K
116 Thermal Engineering

The work done in reversible adiabatic process Polytropic Process


( p2 V2 - p1V1 ) = m R (T2 - T1 ) The most commonly used functional relationship
W1–2 =
(1 - g ) (1 - g ) for many processes can be approximated by an
0.5 ¥ 0.287 ¥ (527.41 - 333)
equation as
or W1–2 =
1 - 1.4 p1V1n = p2V2n = pV n = C ...(4.32)
27.89 where the exponent n is constant and it is referred
= = – 69.744 kJ as the polytropic index. The process in the form of
- 0.4
p V n = C is designated as polytropic process.
The work done during constant-pressure expan-
sion (a) Determination of Polytropic Index n If a
W2–3 = p2 (V3 – V2) = m R (T3 – T2) polytropic process is taking place from an initial
= 0.5 ¥ 0.287 ¥ (1665.0 – 527.41) state p1, V1 to a final state p2, V2, then
= 163.24 kJ p1V1n = p2V2n
Total work transfer, Taking logarithms on both sides, we get
Wtotal = W1–2 + W2–3
ln p1 + n ln V1 = ln p2 + n ln V2
= – 69.744 + 163.24 = 93.5 kJ
or n (ln V1 – ln V2) = ln p2 – ln p1

( )
(ii) Heat transfer
(a) During isentropic compression process ln p2 p
1

( )
Q1–2 = 0 or n= ...(4.33)
ln V1
(b) During constant-pressure expansion V2
Q2–3 = m Cp (T3 – T2)
(b) p, V, T Relationship If the adiabatic index g is
= (0.5 ¥ 1.005) ¥ (1665 – 527.41)
replaced by polytropic index n in equations (4.25),
= 571.44 kJ
(4.26), (4.27) and (4.28), we get
Total heat transfer,
n -n -n
QTotal = Q1–2 + Q2–3 p2 ÊV ˆ ÊV ˆ Êv ˆ
= Á 1˜ = Á 2˜ =Á 2˜ ...(4.34)
= 0 + 571.44 = 571.44 kJ p1 Ë V2 ¯ Ë V1 ¯ Ë v1 ¯
(iii) Change in entropy -
1
v2 V2 Êp ˆ n
(a) During isentropic expansion, = = Á 2˜ ...(4.35)
v1 V1 Ë p1 ¯
DS1–2 = 0
-n
(b) During constant-pressure expansion, T2 ÊV ˆ Ê V2 ˆ
\ = Á 2˜ ÁË V ˜¯
dQ ÊT ˆ T1 Ë V1 ¯
DS2–3 = ÚT = mC p ln Á 3 ˜
Ë T2 ¯ 1- n
1
1- n
ÊV ˆ Êv ˆ
= Á 2˜ =Á 2˜ ...(4.36)
= 0.5 ¥ 1.005 ¥ ln ÊÁ
1665 ˆ
Ë V1 ¯ Ë v1 ¯
Ë 527.41¯˜
1 n -1
= 0.578 kJ/K -
T2 Ê p2 ˆ Ê p2 ˆ n Êp ˆ n
The total entropy change, and = Á ˜ Á ˜ =Á 2˜ ...(4.37)
T1 Ë p1 ¯ Ë p1 ¯ Ë p1 ¯
DSTotal = DS1–2 + DS2–3 = 0 + 0.578
= 0.578 kJ/K The polytropic relation p V n = C represents the
most convenient curve fitting to actual experimen-
tal data between p and V with the value of the index
First Law Applied to Non-Flow Systems 117

‘n’ for the process. Each of the four processes dis- help of Eq. (4.37),
cussed earlier can also be represented by polytropic n -1 1
1-
process as T2 Êp ˆ n Êp ˆ
= Á 2˜ =Á 2˜
T1 Ë p1 ¯ Ë p1 ¯
1
1 -
Ê p2 ˆ Ê p2 ˆ
= ÁË p ˜¯ ¥ ÁË p ˜¯
1 1
1
-
1 Ê p2 ˆ
∵ =0 and ÁË p ˜¯ =1
1
T2 p2
Thus =
T1 p1
It is a relation for the constant volume
process.
All processes discussed above are illustrated
graphically on p-V and T-S diagrams in Fig. 4.12.

(c) Work Done For a non-flow process, the poly-


tropic work transfer is calculated as
2
W1–2 = Ú1
pdV
The pressure p can be expressed as
C
p = n = CV –n
V
2 V2
Hence W1–2 = Ú1
CV - n dV = C ÚV1
V - n dV
Polytropic processes on p-V and T-s diagrams
È V - n +1 - V1- n +1 ˘
(i) When n = 0, p1V1 = p2V20 0 = CÍ 2 ˙
ÍÎ -n +1 ˙˚
or p1 = p2
It is a constant-pressure process. Using C = p1V 1n = p2V2n
(ii) When n = 1, the process takes the form of p2V2 - p1V1
we get W1–2 =
isothermal or hyperbolic process; pV = C 1- n
(iii) When n = g (adiabatic index), the law of m R (T2 - T1 )
process takes the form = ...(4.38)
1- n
g
pV = C where, p2V2 = m RT2 and p1V1 = m RT1
It is an isentropic or adiabatic process. -
1
(iv) When n = then V2 Ê p2 ˆ n
Using = Á ˜
V1 Ë p1 ¯
p1 V1 = p2 V2
Ï 1-
1 ¸
where V indicates a very large volume p1V1 ÔÊ p2 ˆ n Ô
then W1–2 = Ì - 1˝ ...(4.39)
and thus can be treated as constant,volume 1 - n ÔÓÁË p1 ˜¯ Ô˛
process. It can also be illustrated with the
118 Thermal Engineering

(d) Change in Internal Energy For any process, (g) Change of Entropy For unit mass, the heat
DU = mCv (T2 – T1) transfer
dq = Cn dT
(e) Change in Enthalpy It is calculated as
Then change of entropy between two states
DH = mCp (T2 – T1)
2 dq T2 dT
(f ) According to the first law of Ds1–2 = Ú1 T
= Ú
T1
Cn
T
thermodynamics,
ÊT ˆ
Q = DU + W = Cn ln Á 2 ˜ (kJ/kg ◊ K) ...(4.44)
Ë T1 ¯
m R (T2 - T1 )
= mCv (T2 – T1) + For a given mass system
1- n
Ï R ¸ Ê T2 ˆ
= m (T2 – T1) ÌCv + ˝ DS = m Cn ln Á ˜ (kJ/K) ...(4.45)
Ó 1 - n˛ Ë T1 ¯
Using R = Cp – Cv where T is expressed in kelvin (K)
we get Q = m(T2 – T1)
(
Cv - nCv + C p - Cv ) Example 4.13 The internal energy of a certain sub-
1- n stance is given by the following relation:
Ê C p - nCv ˆ
= mÁ (T2 - T1 ) u = 3.56 pv + 84
Ë 1 - n ˜¯ where u is given in kJ/kg, p in kPa, and v in m3/kg.
or Q = mCn (T2 – T1) ...(4.40) A system composed of 3 kg of this substance expands
from an initial pressure of 500 kPa and a volume of
C p - nCv 0.22 m3 to a final pressure of 100 kPa in a process in
where Cn = ...(4.41)
1- n which pressure and volume are related by p v 1.2 = con-
Cn is called polytropic specific heat. stant.
(a) If the expansion is quasi-static, find Q, DU and W
Again rearranging the above equation, we have
for the process.
Ê Cp ˆ (b) In another process, the same system expands
ÁË C - n˜¯ according to the same pressure – volume
¥ (T2 - T1 )
v
Q = m Cv relationship as in part (i), and from the same
1- n initial state to the same final state as in part (i),
Cp R but the heat transfer in this case is 30 kJ. Find the
Using = g and Cv = , we get
Cv g -1 work transfer for this process.

Q=
(g - n) ¥ m R (T2 - T1 )
Solution
(1 - n) (g - 1)
A substance with
g - n mR (T2 - T1 )
Given
= ¥ m = 3 kg,
g -1 1- n u = 3.56 pv + 84;
g -n State 1: v1 = 0.22 m3/kg
= ¥ Polytropic work transfer
g -1 ...(4.42)
p1 = 500 kPa
State 2: p2 = 100 kPa;
Rearranging, we get
Case (i) executes a polytropic process with pv1.2 =
Q=
(g - n) ¥ m R (T2 - T1 ) constant,
(n - 1) (1 - g ) Case (ii) heat transfer, Q = 30 kJ
g -n
= ¥ Adiabatic work transfer ...(4.43) To find Case (i) (a) Total change in internal energy,
n -1
First Law Applied to Non-Flow Systems 119

p Case (ii) For a system from the same initial state to


1
500 kPa the same final state, and same pressure–volume relation-
ship, the change in internal energy will remain same,
pV
1.2
=C thus
DU = – 277.5 kJ
The total work transferred between the two states
100 kPa 2 W = Q – DU
= 30 kJ – (– 277.5 ) = 307.5 kJ
0.22 m3/kg V

Example 4.14 An internal combustion engine has a


Fig. 4.13
cylinder diameter of 15 cm and a stroke of 20 cm. The
pressure and temperature of gas at the beginning of the
(b) Total work transferred, and
compression stroke are 1 bar and 30°C, respectively.
(c) Total heat transferred.
If the clearance volume is 1.147 litres and the law of
Case (ii) Total work transferred. compression is pV1.2 = constant, determine pressure and
temperature at the end of compression. If the working
Analysis substance is air, determine the change in internal energy
Case (i) For a polytropic process, between two states of air during compression. Take Cv = 0.718 kJ/kgK and
R = 0.287 kJ/kgK.
p1 v11.2 = p2 v21.2
Specific volume at the state 2; Solution
1
Ê p1 ˆ 1.2 Given Compression of air
v2 = v 1 Á ˜
Ë p2 ¯ d = 15 cm = 0.15 m, L = 20 cm = 0.2 m
1 State 1: p1 = 1 bar = 100 kN/m2;
Ê 500 ˆ
3 1.2
= (0.22 m /kg) ¥ Á T1 = 30°C = 303 K
Ë 100 ˜¯
3 State 2: pV1.2 =C
= 0.841 m /kg
V2 = Vc = 1.147 lit = 1.147 ¥ 10–3 m3
Change in specific internal energy
Cv = 0.718 kJ/kg ◊ K,
Du = (3.56p2 v2 + 84) – (3.56 p1 v1 + 84)
R = 0.287 kJ/kg ◊ K
= 3.56 (p2 v2 – p2 v2)
= 3.56 ¥ (100 ¥ 0.841 – 500 ¥ 0.22) = – 92.5 kJ/kg To find
(a) Total internal energy change (i) Pressure p2, and temperature T2 after compression,
and
DU = m Du = 3 ¥ (– 92.5 kJ/kg) = – 277.5 kJ
(ii) Change in internal energy of air after compression.
(b) The work transfer/kg
( p2 v2 - p1 v1 ) Assumption The compression process is quasi-static.
w=
1- n
2

=
(100 ¥ 0.841 - 500 ¥ 0.22) = 129.5 kJ/kg
p2

1 - 1.2 1.2
pV =C
Total work transferred;
W = mw
= (3 kg) ¥ (129.5 kJ/kg) = 388.5 kJ 1 bar 1
(c) Heat tranfrer Q = DU + W
= –277.5 + 388.5 = 111 kJ V
Vc V1
120 Thermal Engineering

Analysis The volume of air in the cylinder before p


2
compression. 6 bar

V1 = clearance volume + swept volume pV n = C


= Vc + (p/4) d2L
= 1.147 ¥ 10–3 + (p/4) ¥ (0.15)2 ¥ 0.2
= 4.68 ¥ 10–3 m3
1 bar 1
(i) The pressure after compression can be calculated
as
n V
Ê V1 ˆ 0.03 m3 0.12 m3
p2 = p1 Á ˜
Ë V2 ¯
Fig. 4.15
1.2
Ê 4.68 ¥ 10- 3 ˆ
= (100 kN/m2) ¥ Á
-3 ˜ Process: Polytropic process
Ë 1.147 ¥ 10 ¯
Constants: R = 0.287 kJ/kg,
= 540 kN/m2 = 5.4 bar
Cv = 0.717 kJ/kg ◊ K
And the final temperature
1- n To find
Ê V2 ˆ
T2 = T 1 ¥ Á ˜ (i) Index of compression,
Ë V1 ¯
(ii) Increase in internal energy, and
1-1.2
Ê 4.68 ¥ 10- 3 ˆ (iii) Heat transferred.
= 303 ¥ Á -3 ˜
Ë 1.147 ¥ 10 ¯ Analysis
= 401.4 K = 124.4°C (i) Considering the compression process is a quasi-
(ii) The change in internal energy static polytropic process, thus
DU = m Cv (T2 – T1) p1V1n = p2 V2n
where m = mass of air can be calculated from Index of compression,
p1V1 = m RT1
Ê p ˆ Ê 1ˆ
p1V1 (100 kPa) ¥ (4.68 ¥ 10- 3 m3 ) ln Á 1 ˜ ln Á ˜
or m= = Ë p2 ¯ Ë 6¯ -1.7917
RT1 (0.287 kJ/kg ◊ K) ¥ (303 K) n= = =
Ê V2 ˆ Ê 0.03 ˆ -1.3863
ln Á ˜ ln Á
= 5.38 ¥ 10–3 kg Ë V1 ¯ Ë 0.12 ˜¯
Then DU = 5.38 ¥ 10–3 ¥ 0.718 ¥ (401.4 – 303) = 1.292
= 0.38 kJ (ii) Temperature after compression
1- n 1-1.292
Example 4.15 A cylinder contains 0.12 m3 of air at ÊV ˆ Ê 0.03 ˆ
T2 = T1 Á 2 ˜ = 363 ¥ Á
1 bar and 90°C. It is compressed to 0.03 m3, the final Ë V1 ¯ Ë 0.12 ¯˜
pressure being 6 bar. Find the index of compression, = 544.5 K
increase in internal energy and heat transfered. Mass of air in the cylinder
Take R = 0.287 kJ/kg ◊ K and Cv = 0.717 kJ/kg ◊ K
p1 V1 (100 kPa) ¥ (0.12 m3 )
m= =
Solution RT1 (0.287 kJ/kg ◊ K) ¥ (363 K)
= 0.115 kg
Given Compression of gas in a cylinder
Change of internal energy of air during compres-
State 1: V1 = 0.12 m3
sion;
p1 = 1 bar = 100 kPa
DU = m Cv (T2 – T1)
T1 = 90°C = 363 K
= 0.115 ¥ 0.717 ¥ (544.5 – 363)
State 2: V2 = 0.03 m3
= 14.99 kJ
p2 = 6 bar = 600 kPa
First Law Applied to Non-Flow Systems 121

Work transfer
p2V2 - p1V1 600 ¥ 0.03 - 100 ¥ 0.12
W= =
1- n 1 - 1.292
= –20.548 kJ
(iii) Heat transfer during compression
Q = DU + W
Heat transferred Q = 14.99 – 20.548
= – 5.55 kJ (Rejected)

Example 4.16 In a vertical cylinder, air below a


frictionless piston occupies a volume of 0.09 m3 at a
pressure of 10 bar. The diameter of the cylinder is 0.6 m.
The mass of the piston is 90 kg. A stopper which keeps
the piston is suddenly removed. The piston moves upward
through a distance of 1.5 m.
The pressure of air above the piston is 1 bar.
The air expands according to pV1.3 = C. Neglect the
velocity of air and calculate the velocity of the piston
when it has risen 1.5 m.

Solution
Given
V1 = 0.09 m3 p1 = 10 bar = 1000 kPa
d = 0.6 m m = 90 kg
h = 1.5 m p0 = 100 kPa Fig. 4.16
n = 1.3
This work is used to increase the potential and kinetic
To find Velocity of piston as it releases.
energy of the piston and to displace atmospheric air by
Analysis The change in volume during piston displace-
1.5 m.
ment of 1.5 m
Thus W = m gh + (1/2) mV 2 + p0 (DV)
p 2 p
DV = d h = ¥ (0.6 m ) ¥ (1.5 m ) = 0.4241 m3
2
or 122.146 = 90 ¥ [(9.81) ¥ (1.5) + (1/2) ¥ (V 2 )]
4 4
+ (1 ¥ 106 Pa) ¥ 0.4241 m3
The volume after expansion
V2 = V1 + DV = 0.09 + 0.4241 = 0.5141 m3
(122.146 - 42410 - 1324.35) ¥ 2
or V2 =
90
The pressure after expansion = 1742.5 m²
n
Ê V1 ˆ 1.3
p2 = p1 ¥ Á ˜ = 1000 ¥ ÊÁ
0.09 ˆ
˜
or V= (1742.45)
Ë 2¯
V Ë 0.5141¯
= 41.74 m/s
= 103.78 kPa.
The work done by the gas during expansion Example 4.17 0.2 m3 of a mixture of fuel and air at
1.2 bar and 60°C is compressed till its pressure becomes
( p2V2 - p1V1 ) 12 bar and temperature 270°C. Then it is ignited sudden-
W1–2 =
(1 - n) ly at constant volume and its pressure becomes twice the
103.78 ¥ 0.5141 - 1000 ¥ 0.09 pressure at the end of compression. Find the maximum
= temperature reached and change in internal energy. Also,
1 - 1.3
find the heat transfer during the compression process.
= 122.146 kJ
122 Thermal Engineering

Consider the mixture as perfect gas. p

Take Cp = 1.05 kJ/kg ◊ K and g = 1.4. p3 3

Solution
12 bar 2
Given A mixture of fuel and air as a perfect gas pV n = C

State 1: p1 = 1.2 bar = 120 kPa V1 = 0.2 m3


T1 = 60°C + 273 = 333 K 1.2 bar 1
State 2: p2 = 12 bar = 1200 kPa T2 = 270°C + 273
= 543 K V
V2 = V 3 0.2 m3
p3 = 2 p2 V2 = V3
Cp = 1.05 kJ/kg ◊ K g = 1.4 Fig. 4.17
To find The maximum temperature
(i) T3, the maximum temperature, p
T3 = T2 3 = 543 ¥ 2 = 1086 = 813°C
(ii) Net change in Internal energy, DU, T2
(iii) Heat transfer during compression Q1–2. Net change in internal energy,
DU1–3 = m Cv (T3 – T1)
Analysis Since the states before and after compression
= 0.24 ¥ 0.75 ¥ (1086 – 333)
are known and the type of compression is not mentioned,
therefore, calculation of polytropic index: = 135.67 kJ/kg ◊ K
n -1 The polytropic specific heat
T2 Ê p2 ˆ n C p - nCv
= Á ˜ Cn =
T1 Ë p1 ¯ 1- n
Taking logarithms, we get
=
(1.05 - 1.27 ¥ 0.75)
= –0.36 kJ/kgK
ÊT ˆ Êp ˆ 1 - 1.27
ln Á 2 ˜ = ÊÁ1 - ˆ˜ ln 2
1
Ë T1 ¯ Ë n ¯ ÁË p1 ˜¯ Heat transfer during compression process
Q1–2 = m Cn (T2 – T1)
ln Ê p2 ˆ = (0.24 kg) ¥ (– 0.36 kJ/kg ◊ K) (543 – 333)(K)
Ë p1 ¯

( )
n= = –14.2 kJ (The heat is rejected)
ln Ê p2 ˆ - ln T2
Ë p1 ¯ T1
Example 4.18 A certain mass of air initally at a
Substituting the values in the equation, we get
pressure of 480 kPa and a temperature 190°C is expand-

n=
(
ln 1200120 ) = 1.27
ed adiabatically to a pressure of 94 kPa. It is then heated
(
ln 1200
120 ) (
- ln 543
333 ) at constant volume until it attains its initial temperature
when the pressure is found to be 150 kPa. State the type
The specific heat at constant volume of compression necessary to bring the system back to its
C p 1.05 original pressure and volume. Determine
Cv = = = 0.75 kJ/kg ◊ K
g 1.4 (a) the index of adiabatic expansion,
The characteristic gas equation (b) the work done per kg of air, and
R = Cp – Cv = 1.05 – 0.75 = 0.3 kJ/kg ◊ K (c) the change in specific entropy of air.
The mass of the gas Take R = 0.29 kJ/kg ◊ K.
pV (120 kPa) ¥ (0.2 m3 )
m= 1 1 = = 0.24 kg Solution
RT1 (0.3 kJ/kg ◊ K) ¥ (333 K)
The property relationship for constant volume process Given Three processes constitute a cycle
2-3; State 1: p1 = 480 kPa T1 = 190°C + 273 = 463 K,
p2 p3
= s1 = s2 R = 0.29 kJ/kg ◊ K
T2 T3
First Law Applied to Non-Flow Systems 123

State 2: p2 = 94 kPa V 2 = V3 Therefore,


State 3: T3 = T1 p3 = 150 kPa Ê p2 ˆ Ê 94 ˆ
T2 = Á ˜ T1 = Á ¥ 463 = 290.14 K
Ë p3 ¯ Ë 150 ¯˜
To find
(i) Index of adiabatic expansion, Now the adiabatic index g can easily be calculated
(ii) The work done per kg of air, and as,
g -1
(iii) Change in specific entropy. T2 Ê p2 ˆ g
= Á ˜
Assumptions T1 Ë p1 ¯
(i) The air is perfect gas, and Taking logarithm on both sides, we get
(ii) Reversible process. ÊT ˆ ÔÏÊ g - 1ˆ Ô¸ Ê p2 ˆ
ln Á 2 ˜ = ÌÁ ˜ ˝ . ln Á ˜
Ë 1¯
T ÓÔË g ¯ ˛Ô Ë p1 ¯
p
ÔÏ Ê p ˆ Ê T ˆ Ô¸
or g Ìln Á 2 ˜ - ln Á 2 ˜ ˝ = ln Ê p2 ˆ
1 Ë ¯ Ë T1 ¯ ˛Ô ÁË p ˜¯
480 kPa ÓÔ p1 1
ln ( p2 p1 )
or g =
T=C ln ( p2 p1 ) - ln (T2 T1 )
ln (94 480)
g =
150 kPa 3 ln (94 480) - ln ( 290.14 463)
g
PV = C V=C = 1.401
94 kPa 2 (ii) (a) Work done per kg of air during adiabatic
process 1–2;
V R (T2 - T1 )
p–V diagram w1-2 =
1- g
T1 0.29 ¥ ( 290.14 - 463)
w1-2 =
1 - 1.4
1 T=C
3 = 125.32 kJ/kg
(b) Work done during constant-volume process
2–3;
S=C w2–3 = 0
V=C
(c) Work done during isothermal process 3–1;
Êp ˆ
T2
2 w3-1 = R T1 ln Á 3 ˜
Ë p1 ¯
= 0.29¥ 463 ¥ ln ÊÁ
S 150 ˆ
T–s diagram Ë 480 ¯˜
= –156.17 kJ/kg
Fig. 4.18
Net work done per kg of air
Analysis wnet = 125.32 + 0 – 156.176
(i) To determine adiabatic index, the state before = –30.856 kJ
and after expansion must be defined. Therefore, (iii) (a) Change in specific entropy for isentropic
we have to consider constant-volume heating expansion process, Ds1–2 = 0
process 2-3, in order to calculate the temperature (b) Change in specific entropy during constant
at state 2. volume heating process 2–3
p3 p2
= ÊT ˆ
T3 T2 s3 – s2 = Cv ln Á 3 ˜
Ë T2 ¯
where T3 = T1
124 Thermal Engineering

R 0.29 p
where Cv = =
g - 1 1.401 - 1 750 kPa
3 2
= 0.723 kJ/kg ◊ K

Ds2–3 = 0.723 ¥ ln ÊÁ
463 ˆ
\
Ë 290.14 ˜¯ pV = C pV
1.2
=C
= 0.337 kJ/kg ◊ K
(c) Change in entropy during isothermal process
3–1 150 kPa 1

s3 – s1 = R ln Ê 3 ˆ
p
V
ÁË p ˜¯
1
T
= 0.29¥ ln ÊÁ
150 ˆ
Ë 480 ¯˜ 2

= – 0.337 kJ/kg ◊ K
p=C 1.2
Net change of entropy during cyclic process pV =C
1-2-3-1; 3
360 K 1
D scycle = 0 – 0.337 + 0.337
= 0 (since entropy is a property)
S
3 kg of air at a pressure of 150 kPa
p-V and T-s diagram
and temperature 360 K is compressed polytropically to
750 kPa according to law pV1.2=C. The gas is then cooled (i) (a) The work done during polytropic compres-
to initial temperature at constant pressure. The air is then sion process 1–2;
expanded at constant temperature till it reaches original
pressure of 150 kPa. Draw the cycle on p–V diagram and ( p2V2 - p1V1 ) = m R (T2 - T1)
W1-2 =
determine the net work and heat transfer. (1 - n) (1 - n)
Solution 3 ¥ 0.287 ¥ ( 470.75 - 360)
=
Given Three processes constitute a cycle
(1 - 1.2)
m = 3 kg air = – 476.8 kJ
State 1: p1 = 150 kPa, T1 = 360 K (b) The work done during constant pressure
cooling process 2–3;
State 2: p2 = 750 kPa pV1.2 = C
W2–3 = m R (T3 – T2)
or p1V11.2 = p2V21.2
= 3 ¥ 0.287 ¥ (360 – 470.74)
State 3: p2 = p3 = 750 kPa, T3 = T1 = 360 K
= – 95.355 kJ
To find (c) The work done during isothermal process
(i) Work transfer, and 3–1;
(ii) Heat transfer. Êp ˆ
W3–1 = m R T1 ln Á 3 ˜
Assumptions Ë p1 ¯
(i) The specific gas constant R = 0.287 kJ/kg ◊ K Ê 750 ˆ
= 3 ¥ 0.287 ¥ 360 ¥ ln Á
(ii) The specific heat Cp = 1.005 kJ/kg ◊ K Ë 150 ˜¯
(iii) The specific heat Cv = 0.7165 kJ/kg ◊ K
= 494.86 kJ
Analysis The temperature after polytropic compression Net work transfer;
n -1 1.2 -1 Wnet = W1-2 + W2-3 + W3-1
Ê p2 ˆ n Ê 750 ˆ 1.2
= – 476.8 + (–95.355) + 494.86
= 360 ¥ Á
Ë 150 ˜¯
T 2 = T1 Á ˜
Ë p1 ¯ = –73.5 kJ
= 470.75 K
First Law Applied to Non-Flow Systems 125

(ii) (a) Heat transfer during polytropic compression


g -n
Q1-2 = ¥ Polytropic work done
g -1

=
(1.4 - 1.2)¥ (– 476.8)
1.4 - 1
= – 234.4 kJ
(b) Heat transfer during constant pressure
cooling
Q2–3 = m Cp (T3 – T2)
= 3 ¥ 1.005 ¥ (360 – 470.75)
= –334 kJ Using p1, p2, V1 and V2 to obtain constants a and C,
(c) Heat transfer during isothermal expansion Assuming pi = aVi + C
= work done during the process then 420 = a ¥ 0.004 + C ...(i)
Q3–1 = W3–1 = 494.86 kJ and 140 = a ¥ 0.02 + C ...(ii)
Net heat transfer Solving these equations, we get
Q net = Q1–2 + Q2–3 + Q3–1 a = –17500 kPa/m3
= –234.4 – 334 + 494.86 and C = 490 kPa
= –73.5 kJ Therefore, for linear law, the pressure is expressed as
for a cycle Q net = Wnet = – 73.5 kJ p = (–17500 V + 490) kPa
The work transfer during the process
Example 4.20 1 kg of gas expands reversibly 2 2
according to linear law from 4.2 bar to 1.4 bar. The
initial and final volumes are 0.004 m3 and 0.02 m3,
W1–2 =
Ú1
pdV =
Ú
1
(-17500V + 490) dV
0.02
respectively. The gas is then cooled at constant pressure È V2 ˘
= Í-17500 ¥ + 490V ˙
and finally compressed isothermally back to its initial ÍÎ 2 ˙˚0.004
state of 4.2 bar and 0.004 m3. Calculate the work done
in each process stating its direction. Sketch the cycle on 17500
= - ¥ [0.022 – 0.0042]
a p–V diagram. 2
+ 490 ¥ [0.02 – 0.004]
= –3.36 + 7.84 = 4.48 kJ
Solution
The quantity W1–2 is positive and hence, it is the work
Given Expansion, cooling and compression of gas done by the system.
m = 1 kg Volume after constant-pressure cooling process 3–1
State 1: p1 = 4.2 bar = 420 kPa, V1 = 0.004 m3 p1V1 p1V1 420 ¥ 0.004
State 2: p2 = 1.4 bar = 140 kPa, V2 = 0.02 m3 V3 = = =
p3 p2 140
State 3: p3 = p2 = 140 kPa, and p3V3 or p2V3 = p1V1
= 0.012 m3
To find The work transfer during constant pressure cooling
(i) Sketch of processes on a p–V diagram, process, p2 = p3,
(ii) Work transfer during each process. W2–3 = p2(V3 – V2)
Then W2–3 = (140 kPa) ¥ (0.012 – 0.02) m3
Assumptions
= – 1.12 kJ
(i) The gas is an ideal gas, and
It is negative, thus work is done on the system.
(ii) Each process is a reversible process.
Work transfer during an isothermal process,
Analysis The initial process is proceeded linearly and ÊV ˆ
W3–1 = p1V1 ln 1
its law may be ÁË V ˜¯
3
p = aV + C
126 Thermal Engineering

Work done per kg in isothermal compression


= 420 ¥ 0.004 ¥ ln ÊÁ
0.004 ˆ
Ë 0.012 ˜¯ Êv ˆ Êv ˆ
w1-2 = p1v1 ln Á 2 ˜ = RT1 ln Á 2 ˜
= – 1.845 kJ Ë v1 ¯ Ë v1 ¯
It is also negative thus; work done on the system is w1-2 = 0.5 ¥ 300 ¥ ln(1/8) = –312 kJ/kg
isothermal compression work. pv
Now p2 = 1 1 = 1 ¥ 8 = 8 bar
v2
Example 4.21 A perfect gas undergoes a cycle The maximum pressure in the cycle,
comprises of three processes. It is first compressed p2 = p3 = 8 bar
isothermally from 1 bar and 27°C to one-eighth of its Now considering process 3-1
initial volume. The energy is then added at constant g -1
pressure, increasing the temperature of gas and the T3 Ê p3 ˆ g
cycle is completed by isentropic expansion to original = Á ˜
T1 Ë p1 ¯
conditions. Take Cp = 1.25 kJ/kg ◊ K and R = 0.5 kJ/kg ◊ K.
g -1 g -1
Calculate the maximum cycle temperature and pressure. Ê p2 ˆ
Êp ˆ g g
Also, find the net work transfer per kg. and T3 = T1 Á 3 ˜ = T1 Á ˜
Ë p1 ¯ Ë p1 ¯
Solution The maximum temperature
1.667 -1
Given A perfect gas undergoes a cyclic process ; Ê 8 ˆ 1.667
T3 = 300 ¥ Á ˜
Ë 1¯
State 1: p1 = 1 bar T1 = 27°C + 273
= 100 kPa, = 300 K = 689.4 K = 416.4°C
State 2: T2 = T1, v2 = (1/8) v1 Work done per kg during constant pressure process
State 3: p 3 = p2 s3 = s1 w2–3 = p2(v3 – v2) = R(T3 – T2) = R(T3 – T1)
= 0.5 ¥ (689.4 – 300) = 194.6 kJ/kg
R = 0.5 kJ/kg ◊ K Cp = 1.25 kJ/kg ◊ K
Work done per kg during isentropic expansion
To find
R (T1 - T3 ) 0.5 ¥ (300 - 689.4)
(i) Maximum pressure and temperature in the cycle, w3-1 = =
1- g 1 - 1.667
and
(ii) Net work transfer in the cycle. = 291.75 kJ/kg
Net work transfer in the cycle
Analysis The specific heat at constant volume wnet = –312 + 194.6 + 291.75 = 174.35 kJ/kg
Cv = Cp – R
= 1.25 – 0.5 = 0.75 kJ/kg ◊ K Example 4.22 A system expands from the state 1 (p1,
Ratio of two specific heats V1, T1) to the state 2 (p2, V2, T2) in a reversible non-flow
process as defined below:
Cp 1.25
g = = = 1.667 (a) pV n = C
Cv 0.75
(b) A constant-pressure process followed by a
P
constant-volume process.
2 (c) A constant-volume process followed by a
3
constant-pressure process.
Show these processes on a p–V diagram and find the
pv = C g ratio of work done in these processes, if p1 : p2 = 5 : 2
pv = C
and V2 : V1 = 2 : 1 and n = 1.4.

1 Solution Processes are shown on a p–V diagram


v
Path 1. Process 1–a –2 process pVn = C
Fig. 4.21
First Law Applied to Non-Flow Systems 127

Path 2. Process 1– b –2 constant-pressure process Process 2: 940 kJ/kg of heat is added to air to constant
followed by constant-volume process. volume.
Path 3. Process 1– c – 2 constant-volume process followd Process 3: Process 2 is followed by a reversible adia-
by constant-pressure process. batic expansion up to initial volume.
Process 4: Finally, heat is rejected at constant volume
p so as to reach the initial condition.
Path-2
1 Draw the four processes on a p V diagram.
b

Path-1
Determine the maximum temperature, net work done and
Path 3 a heat rejected per kg of air. Assume adiabatic index of
compression and expansion of 1.4 and constant-volume
specific heat as 0.717 kJ/kg ◊ K.
2
c
V Solution
Fig. 4.22 Given Air undergoes four processes in a cyclic process.
Process 1–2: Reversible adiabatic
Work done during path 1, polytropic process
p1 = 100 kPa
( p2V2 - p1V1 ) T1 = 50°C = 323 K
W1 =
(1 - n) v2 = 0.2v1
Work done during path 2, Process 2–3: Constant volume heat addition
W2 = p1(V2 – V1) + 0 q2 3 = 940 kJ/kg
Work done during path 3, Process 3–4: Reversible adiabatic expansion and
W3 = 0 + p2 (V2 – V1) v4 = v1
p2 2 V2 2 Process 4–1: Constant volume heat rejection to
Put = and =
p1 5 V1 1 approach initial state:
Constants: g = 1.4, Cv = 0.717 kJ/kg ◊ K
W1 È { p V - p V } (1 - n) ˘
2 2 1 1
= Í ˙ To find
W2 ÍÎ ÈÎ p1 (V2 - V1 )˘˚ ˙˚
(i) To draw a p–V diagram,
È p1V1 {1 - p2V2 / p1V1}˘˚
= Î
{
1 - ( 2 / 5) ¥ 2 } (ii) Maximum temperature T3 in the cycle,

{( ) }
= (iii) Net work done per kg of the cycle, and
0.4 ¥ ( 2 - 1)
(n - 1) p1V1 V2 V1 - 1 (iv) Heat rejected per kg of air.
Analysis
5
= 0.5 = (i) The p–V diagram is shown in Fig. 4.23.
10

Now
W2
=
{
p1 (V2 - V1 ) 5 10
= =
}
W3 {
p2 (V2 - V1 ) 2 4 }
Therefore W1 : W2 : W3 = 5 : 10 : 4

Example 4.23
Process 1: Air initially at 100 kPa and 50°C undergoes
reversible adiabatic comprerssion such that
its volume is reduced to one-fifth of its initial
volume.
Fig. 4.23
128 Thermal Engineering

The specific gas constant for air 0.287 ¥ (1011.68 - 1925.9)


=
R = Cv (g 1) = 0.717 ¥ (1.4 1) 1 - 1.4
= 0.287 kJ/kg . K = 655.95 kJ/kg
(ii) Maximum temperature in the cycle. Net work done per kg in the cycle
(a) Temperature after adiabatic compression wnet = 209.42 kJ/kg + 655.95 kJ/kg
1-g 1 -1.4
Ê v2 ˆ Ê 1ˆ = 446.53 kJ/kg
T2 = T1 Á ˜ = 323 ¥ Á ˜
Ë v1 ¯ Ë 5¯ (iv) Heat rejection per kg of air during the process
4 1
= 614.88 K
q4–1 = Cv (T1 T4)
(b) Temperature after constant volume heat
= 0.717 ¥ (323 1011.68)
addition
q2 3 = Cv (T3 T2) = 493.78 kJ/kg
or 940 = 0.717 ¥ (T3 614.88) ( ∵ q1–2 = 0 and q3–4 = 0 adiabatic processes)
940 Heat rejection can also be obtained.
or T3 = + 614.88 Sw = Sq
0.717
fi q4 –1 + 940 = 446.53
= 1925.9 K
or q4 –1 = 493 kJ/kg
Max. temperature, T3 = 1925.9 K or 1652.9°C
Temperature T4 can be obtained as Example 4.24 A non-flow system executes four dif-
1- g 1- g ferent thermodynamics processes in sequence continu-
T4 Êv ˆ Êv ˆ
= Á 4˜ =Á 1˜ ously as followed:
T3 Ë v3 ¯ Ë v2 ¯
1–2: Adiabatic compression,
1- g
Ê v1 ˆ 1 -1.4 2–3: Isobaric heat addition,
or T4 = T3 Á ˜ = 1925.9 ¥ (5)
Ëv ¯ 2 3–4: Adiabatic expansion, and
= 1011.68 K 4–1: Constant volume heat rejection
(iii) Net work done per kg of air in the cycle; The temperature at four silient points 1, 2, 3 and 4
are respectivley 300 K, 700 K, 1500 K, and 600 K. The
wnet = Sw
working substance is 1 kg mass of air, for which
= w1–2 + w2–3 + w3–4 + w4–1
Cp = 1.005 kJ/kg ◊ K, Cv = 0.718 kJ/kg ◊ K
But for constant-volume process
Determine the following for each process:
w2–3 = 0 and w4 –1 = 0
\ wnet = w1–2 + w3–4 (a) Heat transfer,
(b) Work transfer,
Calculating w1 2 and w3 4 separately;
(c) Change in internal energy Also, find
(a) Compression work per kg during the process
(d) Net work and heat transfer for the cycle, and
1 2:
(e) Change in internal energy for the cycle.
R (T2 - T1 )
w1–2 = Comment on results (d) and (e).
1- g
0.287 ¥ (614.88 - 323) Solution
=
1 - 1.4 Given Four different processes are in sequence, ex-
= 209.42 kJ/kg ecuted by a non-flow system of air.
(b) Expansion work per kg, duirng the process m = 1 kg, Cp = 1.005 kJ/kg ◊ K,
3–4: Cv = 0.718 kJ/kg ◊ K T1 = 300 K,
R(T4 - T3 )
w3–4 = T2 = 700 K, T3 = 1500 K,
1- g
and T4 = 600 K
First Law Applied to Non-Flow Systems 129

To find 1 ¥ 0.287 ¥ (700 - 300)


=
(i) For each process, 1 - 1.4
(a) Heat transfer, = 287 kJ
(b) Work transfer, and (c) Change in internal energy:
(c) Change in internal energy. DU = mCv (T2 T1)
(ii) For cyclic process with comments: = 1¥ 0.718 ¥ (700 300) = 287 kJ
(a) Net work transfer, and ∵ Q1 2 = 0 \ DU = W1–2
(b) Net heat transfer. 2. Process 2–3: Isobaric heat addition
T2 = 700 K, and T3 = 1500 K, p = Const.
(a) Heat transferred,
Q2–3 = m Cp (T3 T2)
= 1 ¥ 1.005 (1500 700) = 804 kJ
(b) Work transferred,
W2–3 = p2 (V3 V2) = m R (T3 T2)
= 1 ¥ 0.287 ¥ (1500 700)
= 229.6 kJ
(c) Change in internal energy
DU2–3 = Q2 3 W2 3
= 804 229.6 = 574.4 kJ
3. Process 3–4: Adiabatic expansion process
T3 = 1500 K, T4 = 600 K
(a) Heat transferred, Q3–4 = 0
(b) Work transferred,
m R (T4 - T3 )
W3–4 =
1- g
1 ¥ 0.287(600 - 1500)
p–V and T–s diagram =
1 - 1.4
Assumption All processes are reversible processes. = 645.75 kJ
(c) Change in internal energy,
Analysis The specific gas constant DU3– 4 = W3–4 = 645.75 kJ
R = Cp Cv = 1.005 0.718 = 0.287 kJ/kg ◊ K 4. Process 4–1: Constant-volume heat-rejection
Ratio of two specific heats process
Cp 1.005 T4 = 600 K, T1 = 300 K v = Const.
g= = = 1.4
Cv 0.718 (a) Heat rejection,
Q4–1 = m Cv (T1 T4)
(i) Considering each process separately
= 1 ¥ 0.718 ¥ (300 600) = 215.4 kJ
1. Process 1–2: Adiabatic compression process, pV g
=C (b) Work transferred,
(a) Heat transferred: W4 –1 = 0 ( ∵ V = Const. dV = 0)
Q1–2 = 0 for adiabatic process (c) Change in internal energy,
(b) Work transfer; DU4 –1 = Q4–1 = 215.4 kJ
m R(T2 - T1 )
W1–2 =
1- g
130 Thermal Engineering

(ii) For cyclic process 1 cm = 300 kPa on p scale


(a) Net work transfer, 1 cm = 0.1 m3 on V scale.
Wnet =
Ú dW Qrej = 1000 kJ/kg
To find
= W1 2 + W2 3 + W3 4 + W4 1
(i) Work transfer during a cycle, and
= 287 + 229.6 + 645.75 + 0
(ii) Thermal efficiency.
= 584.35 kJ
(b) Net heat transfer Analysis
Qnet =
Ú dQ = Q 1– 2 + Q2–3 + Q3– 4 + Q4–1 (i) The net work transfer is the area within the curve
= Area of the circle on p–V diagram.
= 0 + 804 + 0 215.4 = 584.6 kJ p 2 p
Area = D = ¥ (Dia. on p scale)
Comment: For cyclic process 4 4
¥ (Dia. on V scale)
Ú dQ = Ú dW =
p
4
¥ (10 ¥ 300) ¥ (10 ¥ 0.1)
and
Ú dU = 0 = 2356.2 kJ/kg.
(ii) The thermal efficiency is expressed as
i.e. 287 + 574.4 645.75 215.4 = 0
Net work transfer
hth =
Example 4.25 An imaginary engine receives heat and Heat supplied
does work on a slowly moving piston at such rates that
Net work transfer
the cycle of operation of 1 kg of fluid can be represented =
Net work transfer + Heat rejected
as a circle of 10 cm diameter on a p–V diagram on which
1 cm = 300 kPa and 1 cm = 0.1 m3/kg. =
2356.2
= 0.7020
2356.2 + 1000
(a) How much work is done by each kg of fluid for
each cycle of operation? = 70.20%
(b) Calculate the thermal efficiency of the engine,
if heat rejected by the engine is 1000 kJ/kg of Example 4.26 The working substance in a Carnot
working fluid. engine is 0.04 kg of air. The maximum cycle temperature
is 930 K and the maximum pressure is 4.4 ¥ 103 kPa. The
p
heat added per cycle is 4 kJ. Determine the maximum
cylinder volume, if minimum temperature during the
cycle is 300 K.
3000 kPa

Solution
Given A Carnot cycle with
m = 0.04 kg T1 = 930 K
T3 = 300 K p1 = 4.4 ¥ 103 kPa
a V Q1– 2 = 4 kJ
b
1 m3
To find Maximum cylinder volume, V3
Fig. 4.25
Assumptions
Solution (i) Specific gas constant for air, R = 0.287 kJ/kg ◊ K
Given An imaginary engine (ii) Ratio of two specific heats g = 1.4
m = 1 kg
Analysis Using perfect-gas equation:
Dia. of circle = 10 cm
p1V1 = m RT1
First Law Applied to Non-Flow Systems 131

p sic energy equations derived from the first law are


1 applicable. Equations related with work done, heat
44 bar
transfer, enthalpy, internal energy do apply to va-
2 pour under specified conditions. The following
basic-energy equation can be applied to vapour by
4 taking care of units. For a unit mass system;
3
Heat transfer; dq = Tds
d q = du + pdv
and d q = dh vdp
V
V3 Internal energy; u = h pv
2
Work transfer; w= Ú
1
pdv
mRT1 The unknown state property can be obtained by
or V1 =
p1 equating constant property between two states of a
(0.04 kg) ¥ (0.287 kJ/kg ◊ K) ¥ (930 K) process, as given:
=
(4.4 ¥ 103 kPa) For constant-volume process; v1 = v2
3 3
= 2.426 ¥ 10 m For isentropic process; s1 = s2
After heat addition during the process 1 2; For isothermal process; p1v1 = p2 v2
For polytropic process; p1v1n = p2 v2n
Q1–2 = p1V1 ln Ê 2 ˆ
V
ÁË V ˜¯
1 Steam is stored in a container of
or 4 = 4.4 ¥ 103 ¥ 2.426 ¥ 10 3
0.45 m3 capacity. The pressure of steam is 8 bar and tem-
Ê V2 ˆ perature is 200°C. The container is cooled and pressure
¥ ln Á -3 ˜ drops to 2.5 bar. Find (a) quality of steam after cooling,
Ë 2.426 ¥ 10 ¯ and (b) heat lost to atmosphere due to cooling. Neglect
Ê V2 ˆ the volume of water.
or ln Á = 0.3746
Ë 2.426 ¥ 10- 3 ˜¯
Solution
or V2 = 2.426 ¥ 10 3 ¥ exp (0.3746)
= 3.53 ¥ 10 3 m3 Given The steam is in the container, thus
During isothermal process; V1 = V2 = 0.45 m3 = constant
T2 = T1 = 930 K p1 = 8 bar, T1 = 200°C
During isentropric expansions 2 3 p2 = 2.5 bar
1-g To find
T3 ÊV ˆ
= Á 3˜ (i) The quality of steam after cooling, and
T2 Ë V2 ¯
1 1 (ii) Heat lost to atmosphere during cooling.
Ê T3 ˆ 1-g 3 Ê 300 ˆ 1-1.4
= 3.53 ¥ 10 ¥ Á
Ë 930 ˜¯
or V3 = V 2 Á ˜ Properties of steam
Ë T2 ¯
At 8 bar and 200°C (from superheated steam table)
= 3.53 ¥ 10 3 ¥ 16.920 = 0.06 m3
v1 = 0.2608 m3/kg h1 = 2839.25 kJ/kg
Maximum volume in the cycle in 0.06 m3 or 60 litres
At 2.5 bar (from pressure entry, saturated steam table)
Tsat = 127.43°C, vg2 = 0.71871 m3/kg,
hf2 = 535.34 kJ/kg, hfg2 = 2185.55 kJ/kg
Analysis
Equations for ideal gases as p v = RT, Joule’s law,
etc., are not applicable to vapour, though the ba- (i) Quality of steam
132 Thermal Engineering

Hence Q = 1.725 ¥ (1263.06 2630.61)


= 2359.0 kJ (Rejected)

Example 4.28 A pressure cooker contains 2 kg of dry


and saturated steam at 5 bar. Find the quantity of heat
that must be rejected so as to reduce quality upto 60%
dry. Determine the pressure and temperature at the new
state.

Solution
Given A constant volume cooling process with
p1 = 5 bar x1 = 1.0
m = 2 kg x2 = 0.6
To find After cooling,
(i) Pressure and temperature at new state,
(ii) Heat transferred.

Analysis At 5 bar, the properties of steam


vg = 0.3749 m3/kg, hg = 2748.7 kJ/kg
Fig. 4.27 Constant-volume process on p-V and T-s Since the volume remains constant during cooling
diagrams process, thus
vg1 = x2 vg2
Since the volume in the container is constant, thus 0.3749
V1 = V2 or vg2 = = 0.6248 m3/kg
0.6
or v1 = v2 From steam tables, the pressure and saturation tem-
But specific volume vg2 at 2.5 bar is greater than perature can directly be obtained corresponding to va-
specific volume v1, thus steam is wet at 2.5 bar, pour specific volume of 0.6248 m3/kg , if available in the
therefore, steam table.
v1 = x vg2 But this volume is not available in the steam table.
0.2608 = x ¥ 0.7187 The specific volume of 0.6573 m3/kg at 2.75 bar and
or x = 0.3628 0.6058 at 3 bar are given.
(ii) Heat transferred during cooling Approximating the new pressure, by using linear
Mass of steam in container, interpolation formula as
V 0.45 m3 Ê y2 - y1 ˆ
= = 1.725 kg y = y1 Á ( x - x1)
m=
v1 0.2608 m3/kg Ë x2 - x1 ˜¯

The heat transferred at constant volume Here y1 = p1 = 2.75 bar y2 = p2 = 3.0 bar
Q = DU = m(Du) = m(u2 u1) x1 = vg1 = 0.6573 m3/kg x2 = vg2 = 0.6058 m3/kg
where u1 = h1 p1v1 = 2839.25 (8 ¥ 100) Using value x = 0.6248 m3/kg for calculation of pres-
¥ 0.2608 sure as
= 2630.61 kJ/kg Ê 3 - 2.75 ˆ
p = 2.75 + Á ¥ (0.62483 - 0.657)
u2 = h2 p2v2 = (hf2 + xhfg2 ) p2 (xvg2 ) Ë 0.6058 - 0.6573 ˜¯
= (535.34 + 0.3628 ¥ 2185.55) = 2.75 + ( 4.854) ¥ ( 0.0325) = 2.907 bar
(2.5 ¥ 100) ¥ (0.3628 ¥ 0.7187) = 1.0161 + 1.891 = 2.907 bar
= 1263.06 kJ/kg Similarly, other properties can be obtained at this
state.
First Law Applied to Non-Flow Systems 133

The saturation temperature corresponding to the new = (1 0.85) ¥ 2015.3 + 2.1 ¥ (350 179.91)
state = 302.3 + 357.2 = 659.5 kJ/kg
Tsat = 133.1°C Total heat supplied to steam,
The new state is 2.907 bar and 0.6 dry. Q = m ¥ h1 = 0.9076 ¥ 659.5 = 598.5 kJ
hf = 555 kJ/kg, hfg = 2166.5 kJ/kg
The work done during superheating,
(ii) Heat transferred
Q = m (u2 u1) È Tsup ˘
w = p(v2 v1) = p Ív g ¥ - x vg ˙
where u2 = h2 p2 v2 Î Tsat ˚
= (h f 2 + x2 h fg 2 ) - p2 ( x2 v g 2 ) È 350 + 273 ˆ
w = 1000 ¥ Í0.19444 ¥ ÊÁ ˜
= (555 + 0.6 ¥ 2166.5) 290.7 ¥ 0.6 ¥ 0.6248 Î Ë 179 .91 + 273 ¯
= 1745.91 kJ/kg ˘
and u1 = h1 p1v1 - 0.85 ¥ 0.19444 ˙
˚
= hg1 - p1 v g1 = 2748.7 500 ¥ 0.3749 = 1000 ¥ (0.2675 0.1653) = 102.18 kJ/kg
= 2561.25 kJ/kg Total external work done
Hence, heat transferred We = mw = 0.9075 ¥ 102.18 = 92.73 kJ
Q = 2 kg ¥ (1745.91 2561.25) (kJ/kg) The percentage of total heat supplied as external work
= 1630.68 kJ (rejected) 92.73
= ¥ 100 = 15.5%
Example 4.29 A quantity of steam at 10 bar and 0.85 598.5
dry occupies a volume of 0.15 m3. Determine the heat
Example 4.30 A closed vessel of 0.2 m3 capacity
supplied to raise the temperature of steam to 350°C at
contains steam at 8 bar and 200°C.
constant pressure and percentage of this heat supplied,
which appears as external work. (a) What is the weight of steam contained in the
vessel ?
Solution (b) The vessel is cooled and steam becomes just dry
and saturated. What would be the pressure at this
Given p1 = 10 bar = 1000 kPa Tsup = 350°C
state ? Estimate specific entropy.
x = 0.85 V1 = 0.15 m3
(c) The vessel is further cooled till the temperature
To find drops to 160.8°C. Find the pressure and condition
(i) Heat supplied, of steam at this state.
(ii) External work, and
(iii) Percentage of heat supplied as external work Solution
done Given The steam in a closed vessel
Assumption Specific heat of superheated steam, V = 0.2 m3 p = 8 bar Tsup = 200°C
Cps = 2.1 kJ/kg ◊ K x2 = 1.0 T3 = 160.8°C

Properties From steam table at 10 bar To find


Tsat = 179.91°C (i) Weight of steam
hf = 762.81 kJ/kg,
(ii) (a) Pressure p2 of steam in saturation state
hfg = 2015.3 kJ/kg, vg = 0.19444 m3/kg
(b) Specific entropy of saturated steam
Analysis Mass of steam (iii) Pressure p3 and quality x3 of steam at the state 3.
V (0.15 m3 ) Assumption Specific heat of superheated steam as
m= = = 0.9076 kg
x vg 0.85 ¥ (0.19444 m3/kg) 2.1 kJ/kg ◊ K
Heat supplied per kg of steam during superheating Analysis
h1 = (1- x) h fg + C ps (Tsup - Tsat ) (i) At 8 bar, the properties of steam
134 Thermal Engineering

Therefore, the pressure at 160.8°C, using linear


interpolation formula
Ê y2 - y1 ˆ
y = y1 + Á ( x - x1 )
Ë x2 - x1 ˜¯
Ê p2 - p1 ˆ
or p3 = p1 + Á ˜ (T - Tsat1 )
Ë Tsat 2 - Tsat1 ¯
Ê 7.005 - 6.178 ˆ
p3 = 6.178 + Á ¥ (160.8 - 160)
Ë 165 - 160 ˜¯
= 6.31 bar
Fig. 4.28 Similarly, specific volume of steam

v3 = 0.3071 + Ê 0.2727 - 0.3071ˆ ¥ (160.8 – 160)


3
Tsat = 170.43° C vg = 0.2404 m /kg
ÁË 165 - 160 ˜¯
hg = 2769.13 kJ/kg
The specific volume of superheated steam = 0.3016 m3/kg
Tsup The condition of steam can be calculated by
v 1 = vg ¥
Tsat equating the volume at two states;
200 + 273 ˆ
= 0.2404 ¥ Ê
v1 = v 3
ÁË 170.43 + 273 ˜¯ or 0.25643 = x3 ¥ 0.3016
0.255
= 0.25643 m3/kg or x3 = = 0.85
0.3016
Note It can also be obtained directly from
superheated steam table.
Example 4.31 Dry saturated steam at 1.4 MPa un-
The mass of steam in the vessel dergoes a reversible hyperbolic expansion in a non-flow
V system until its pressure is 0.80 MPa. Calculate the work
m=
v1 done during the process.
0.2 m3
= = 0.78 kg
0.25643 m3 / kg
Weight of steam = mg = 0.78 ¥ 9.81 = 7.65
(ii) When steam is cooled to just become saturated
(state 2), its specific volume remains constant, but
pressure reduces, i.e., v g 2 = 0.25643 m3/kg.
(a) From steam tables, saturation pressure cor-
responding to 0.25643 m3/kg is 7.5 bar (ap-
prox.)
(b) The specific entropy at this pressure is Fig. 4.29
sg = 6.6847 kJ/kg ◊ K
(iii) When the vessel is cooled to T3 = 160.8°C (state Solution
3), taking this as saturation temperature, the Given Reversible hyperbolic expansion of steam:
pressure can be calculated by linear interpolation.
State 1: p1 = 1.4 MPa (Dry)
At Tsat1 = 160°C vg1 = 0.3071 m3/kg
x1 = 1
p1 = 6.178 bar
State 2: p2 = 0.80 MPa, after hyperbolic expansion.
At Tsat2 = 165°C vg2 = 0.2727 m3/kg
p2 = 7.005 bar To find Work done during the process.
First Law Applied to Non-Flow Systems 135

Properties of steam at pressure


p1 = 1.4 MPa Tsat1 = 195.07°C,
vg1 = 0.14084 m3/kg, hg1 = 2790.00 kJ/kg
At a pressure of 0.80 MPa, after hyperbolic expansion
Tsat2 = 170.43°C, vg2 = 0.2404 m3/kg
hf2 = 721.1 kJ/kg hfg2 = 2048 kJ/kg
Analysis For hyperbolic process:
p1v1 = p2 v2
1.4 ¥ 0.14084 = 0.8 ¥ v2
and v2 = 0.2464 m3/kg
The work done per kg of steam during isothermal
process,
Ê v2 ˆ
w = p1 v1 ln Á ˜
Ë v1 ¯
Ê 0.2464 ˆ
= (1.4 ¥ 103) ¥ 0.14084 ¥ ln Á
Ë 0.14084 ˜¯
= 110.34 kJ/kg
Fig. 4.30

Example 4.32 Dry saturated steam at 1550 kPa is Analysis The steam is superheated during the process,
isothermally expanded to 100 kPa pressure. Calculate the thus it behaves as an ideal gas, i.e.,
change in internal energy, work done, and heat transfer p1v1 = p2v2
during the process. 1550 ¥ 0.1275 = 100 ¥ v2
or v2 = 1.976 m3/kg
Solution The temperature at the state 2;
Given Reversible isothermal expansion of dry saturated T2 = T1 = 199.86°C
steam: The enthalpy of steam at the state 2
State 1: p1 = 1550 kPa (Dry) x1 = 1 h2 = hg 2 + Cps (T2 Tsat 2)
State 2: p2 = 100 kPa, after isothermal expansion. = 2675.46 + 2.1 ¥ (199.86 – 99.63)
= 2885.94 kJ/kg
To find
(i) Heat transfer, (i) The change in specific internal energy:
(ii) Work done, and Du = u2 u1 = (h2 p2v2) (h1 p1v1)
(iii) Change in internal energy during the process. = h2 h1 (∵ p2 v2 = p1v1)
= 2885.94 2790.8 = 95.14 kJ/kg
Assumptions
(ii) The work done per kg of steam during isothermal
(i) The steam in superheated region as an ideal gas. process,
(ii) The specific heat for superheated steam as 2.1 kJ/ Ê v2 ˆ
kg ◊ K. w = p1 v1 ln Á ˜
Ë v1 ¯
= (1550) ¥ 0.1275 ¥ ln ÊÁ
Properties of steam at pressure p1 = 1550 kPa 1.976 ˆ
Ë 0.1275 ¯˜
Tsat1 = 199.86°C, vg1 = 0.1275 m3/kg = 541.65 kJ/kg
hg1 = 2790.8 kJ/kg, sg1 = 6.4289 kJ/kg ◊ K (iii) The heat transferred by steam during isothermal
At pressure 100 kPa, after isothermal expansion process,
Tsat2 = 99.63°C, vg2 = 1.6937 m3/kg q = w + Du = 541.65 + 95.14
hg2 = 2675.46 kJ/kg, = 636.79 kJ/kg
136 Thermal Engineering

Example 4.33 A piston–cylinder arrangement con- v1 0.5


or x1 = = = 0.295
tains 1 kg of water at 100 kPa. The initial volume is v g1 1.694
0.5 m3. The heat transferred to the water is an amount Enthalpy of water h1 = hf1 + x1hfg1
which is just necessary to cause a slow expansion at
= 417.46 + 0.295 ¥ 2258
constant temperature. The process is terminated when
= 1083.57 kJ/kg
the volume is doubled. Determine the magnitude of heat
transfer. Solve the problem if the process of expansion is Since the steam is wet, therefore, during further
hyperbolic. heat addition, its temperature and pressure remain
constant, thus,
Solution v2 = x2 vg = 1 m3
1
Given Water in a piston–cylinder arrangement x2 = = 0.59
1.694
m1 = 1 kg, v1 = 0.5 m3
3 Enthalpy h2 = hf2 + x2hfg2 = 417.46 + 0.59 ¥ 2258
v2 = 1 m p1 = 100 kPa
= 1749.68 kJ/kg
To find
Heat added to water,
(i) Heat supplied to water at constant temperature,
Q = m (u2 u1)
and
where u1 = h1 p1v1 = 1083.57 100 ¥ 0.5
(ii) Heat supplied during the hyperbolic process.
= 1033.57 kJ/kg
Properties of water at 100 kPa
u2 = h2 p2 v2 = 1749.68 – 100 ¥ 1
Tsat = 99.63°C vg = 5.694 m3/kg
hf = 417.46 kJ/kg hfg = 2258 kJ/kg = 1649.68 kJ/kg
\ Q = 1 ¥ (1649.68 – 1033.57)
Analysis
= 616.11 kJ
(i) Heat supplied at constant temperature
(ii) If expansion is hyperbolic
The initial volume,
p1v1 = p2 v2
v1 = x1 vg1 = 0.5 (since m = 1 kg) p1 v1 100 ¥ 0.5
or p2 = = = 50 kPa
p v2 1
From steam table at 50 kPa,
1 2
vg2 = 3.24 m3/kg, hf2 = 340.47 kJ/kg,
1 bar
hfg2 = 2305.4 kJ/kg,
Specific volume; v2 = x2 vg2 = 1
v 1
x2 = 2 = = 0.3086
v g 2 3.24
0.5 m3 1 m3 V h2 = h f 2 + x2 h fg 2
(a) p–V diagram for constant-temperature heating = 340.47 + 0.3086 ¥ 2305.4
p = 1051.92 kJ/kg
Work done during the hyperbolic process
Ê v2 ˆ
1 bar 1 W = m p1 v1 ln Á ˜
Ë v1 ¯
Ê 1 ˆ
= 1 ¥ 100 ¥ 0.5 ¥ ln Á = 34.65 kJ
0·5 bar 2 Ë 0.5 ˜¯
Internal energy change for m = 1 kg
0.5 m3 1m 3 V DU = m(u2 u1)
(b) p–V diagram for hyperbolic expansion = m[(h2 – p2v2) – (h1 – p1v1)]
Fig. 4.31 = m (h2 – h1)
First Law Applied to Non-Flow Systems 137

= 1 ¥ (1051.92 1083.57) sf3 = 5.146 kJ/kg ◊ K sfg3 = 6.387 kJ/kg ◊ K,


= 1083.57 kJ/kg sg3 = 7.533 kJ/kg ◊ K
or DU = 31.63 kJ (decrease)
Analysis
The heat transfer Q = DU + W
(i) During isentropic expansion 1 2
= 31.65 + 34.65 = 3.00 kJ
s1 = s2 = s f 2 + x2 s fg 2
Example 4.34 Steam at 2 MPa and 250°C is expand- 6.5453 = 1.727 + x2 ¥ 5.213
ed isentropically to 0.35 MN/m2 and it is then expanded 6.5453 - 1.727
or x2 = = 0.924
hyperbolically to 0.06 MN/m2. Using steam tables, de- 5.213
termine Condition of steam after hyperbolic expansion 1 2;
(a) final condition of steam, p2 v2 = p3v3
Tsup3
(b) change in specific entropy during hyperbolic pro- or p2 (x2 vg2) = p3 v g3 ¥
cess. Tsat3
0.35 ¥ (0.924 ¥ 0.5243)
Solution È Tsup ˘
= 0.06 ¥ Í2.73 ¥ ˙
Î (86 + 273) ˚
Given Isentropic and hyperbolic expansion of steam;
0.16955 ¥ 359
State 1: p1 = 2 MPa, T1 = 250°C or Tsup 3 =
0.1638
State 2: p2 = 0.35 MPa, s1 = s2 = 371.6 K = 96.6°C
State 3: p3 = 0.06 MPa, p2 v2 = p3 v3 Degree of superheat = 96.6 86 = 10.6°C
To find Hence, steam is superheated by 10.6°C.
(i) Final condition of steam, and (ii) Change in specific entropy during the process 2–3
(ii) Change in entropy during hyperbolic expansion. Ds = s3 s2
where s2 = sf2 + x2 sfg2
Properties of steam
= 1.727 + 0.924 ¥ 5.213
From steam tables = 6.5453 kJ/kg ◊ K
At 2 MPa (2000 kPa) and 250°C: Ê Tsup 3 ˆ
h1 = 2902.5 kJ/kg s1 = 6.5453 kJ/kg ◊ K s3 = s g3 + C ps ln Á ˜
Ë Tsat3 ¯
v1 = 0.111 m3/kg
Ê 371.6 ˆ
= 7.533 + 2.1 ¥ ln Á
Ë 86 + 273 ˜¯
At 0.35 MPa or at 350 kPa:
Tsat = 138.9°C, hf2 = 584.33 kJ/kg = 7.605 kJ/kg ◊ K
hfg2 = 2148.1 kJ/kg, sf2 = 1.727 kJ/kg ◊ K Now change in entropy,
sfg2 = 5.213 kJ/kg ◊ K vg2 = 0.5243 m3/kg D s = 7.605 6.5453 = 1.061 kJ/kg ◊ K
At 0.06 MPa or 60 kPa:
Tsat = 86°C = 359 K vg 3 = 2.73 m3/kg Example 4.35 1 kg of steam at 8 bar, 250°C expands
hf3 = 359.9 kJ/kg hfg3 = 2293.6 kJ/kg polytropically to 2 bar according to p v1.3 = constant.
Determine the quality of steam at final state, heat transfer
1
T 250°C and change in entropy during expansion process.

Pa Solution
5M
2 0.3 Pa
6M Given Polytropic expansion of steam:
0.0
3 m = 1 kg
p1 = 8 bar = 800 kPa
T1 = 250°C = 523 K
p2 = 2 bar = 200 kPa
s 1.3
p v = constant
Fig. 4.32
138 Thermal Engineering

The change in internal energy


Du = u2 u1
where u1 = h1 p1v1
= 2948 800 ¥ 0.2931
= 2713.52 kJ/kg
u2 = h2 p2v2 = (hf 2 + x2 hfg2) p2v2
= (504.7 + 0.961 ¥ 2202)
200 ¥ 0.8514
Fig. 4.33 = 2450.54 kJ/kg
Thus Du = 2450.54 2713.52
To find = 262.98 kJ/kg.
(i) The final quality of steam, The heat transfer per kg of steam
(ii) Heat transfer, and
q = Du + w = 262.98 + 214
(iii) Change in entropy.
= 48.98 kJ/kg
Assumptions (iii) The entropy change during expansion
(i) The reversible process, and Ds = s2 s1
(ii) Superheated steam as perfect gas. where s2 = s f2 + x2 s f g 2
Properties of steam The properties of superheated = s f2 + x2 ( s g 2 - s f 2 )
steam at 8 bar, 250°C: = 1.5301 + 0.961 ¥ (7.1271 1.5301)
v1 = 0.2931 m3/kg, = 6.9 kJ/kg ◊ K
h1 = 2948 kJ/kg
Thus Ds = 6.9 7.034 = 0.125 kJ/kg ◊ K
s1 = 7.034 kJ/kg ◊ K
The properties of steam at 2 bar: Example 4.36 A vessel with a partition in it, contains
vg2 = 0.8857 m3/kg hf2 = 504.7 kJ/kg initially 2 kg of dry and saturated steam at 7.0 bar in one
hfg2 = 2202 kJ/kg sf2 = 1.5301 kJ/kg ◊ K compartment and 1 kg of steam with dryness fraction of
sg2 = 7.127 kJ/kg ◊ K 0.8 at 3.5 bar in the other compartment. After the parti-
Analysis tion is removed, the pressure of the mixture is found to be
5 bar. Neglecting the volume of water, find
(i) The steam expands polytropically as
(a) Dryness fraction of the mixture
p1 v11.3 = p2 v21.3
1 1
(b) The heat interaction between the vessel and the
Ê p1 ˆ 1.3 Ê 8 ˆ 1.3 surroundings
or v2 = Á ˜ v1 = ÁË ˜¯ ¥ 0.2931
Ë p2 ¯ 2
A B
= 0.8514 m3/kg
2 kg 1 kg
Specific volume v2 is less than vg2, hence the
7 bar 3.5 bar
steam is wet after expansion, thus,
x=1 x = 0.8
v2 = x2 vg2
v2 0.8514
or x2 = = = 0.961 Solution
v g 2 0.8857
Given p1 = 7 bar m1 = 2 kg x1 = 1
(ii) The polytropic work done
p2 = 3.5 bar m2 = 1 kg x2 = 0.8
p2 v2 - p1v1
w= p3 = 5 bar of mixture
1- n
200 ¥ 0.8514 - 800 ¥ 0.2931 To find
= (i) Dryness fraction of steam after mixing,
1 - 1.3
= 214 kJ/kg (ii) Heat interactions between vessel and surroundings.
First Law Applied to Non-Flow Systems 139

Properties of Steam Properties of steam from steam v3 0.32174


table: or dryness fraction x3 = = = 0.858
v g3 0.3749
At 7 bar vg1 = 0.2729 m3/kg hg1 = 2763.5 kJ/kg From the first law of thermodynamics,
At 3.5 bar vg 2 = 0.5243 m3/kg hf2 = 584.33 kJ/kg Q = W + DU
hfg2 = 2148.5 kJ/kg For a constant–volume process,
At 5.0 bar vg 3 = 0.3749 m3/kg hf 3 = 640.23 kJ/kg W =0
hfg 3 = 2108.5 kJ/kg and DU = Internal energy after mixing
Internal energy before mixing.
Analysis The volume of the 1st compartment
\ Q = U3 (U2 + U1)
V1 = m1vg1 = 2 ¥ 0.2729 = 0.5458 m3
where U3 = m [h3 p3 v3] = m[(hf3 + xhfg 3) p3 v3]
The volume of the 2nd compartment, = 3 ¥ [(640.23 + 0.858 ¥ 2108.5) (5 ¥ 100
V2 = m2 (x2 vg 2) = 1 ¥ (0.8 ¥ 0.5243) = 0.41944 m3 ¥ 0.32174)]
After the removal of partition, the total volume of the = 6865.36 kJ
compartment U1 = m1 [h1 p1v1]
V3 = V1 + V2 = 0.5458 + 0.41944 = 0.96524 m3 = 2 ¥ [(2763.5) (7 ¥ 100 ¥ 0.2729)]
= 5144.94 kJ
The mass of steam after removal of partition;
U2 = m2 [(hf2 + x hfg2) (p2 ◊ x vg2)]
m = m1 + m2 = 2 + 1 = 3 kg
= 1 ¥ [(584.33 + 0.8 ¥ 2148.1) (3.5 ¥ 100
Specific volume of steam; ¥ 0.8 ¥ 0.5243)]
V 0.96524 = 2156.0 kJ
v3 = = = 0.32174 m3/kg
m 3 Thus Q = 6865.36 (5144.94 + 2156.0)
But specific volume of dry saturated steam at 5 bar; = 435.56 kJ
vg 3 = 0.3749 m3/kg, is greater than v3 Heat is rejected from vessel.
Thus v3 = x 3 vg 3

Summary
The characteristic gas equation for an ideal gas is The summary of equations for ideal processes are
pV = m RT given below.
where R is the specific gas constant and it is 1. Constant-volume Process
obtained as p1 p2
Law V = C, =
Ru Universal gas constant 8.31447 T1 T2
R= = = Work transfer W =0
M Molecular weight M
Heat transfer Q = m Cv (T2 T1)
The thermodynamic properties p, V and T are
related as = DU
p1V1 p V Change in entropy DS = m Cv ln(T2/T1)
= 2 2 2. Constant-Pressure Process
T1 T2
For all type of processes, the change in internal V1 V2
Law p = C, and =
energy and change in enthalpy can be calculated T1 T2
as follows: Work transfer W = p (V2 V1)
Change in internal energy Heat transfer Q = m Cp(T2 T1)
DU = m Cv (T2 T1) = DH
Change in enthalpy DH = m Cp(T2 T1) Change in entropy DS = m Cp ln(T2/T1)
140 Thermal Engineering

3. Isothermal Process Change in entropy DS = 0


Law T = C, or pV = C, 5. Polytropic Process
p1V1 = p2V2 Law pV n = C
Work transfer W = p1V1 ln(V2/V1) 1- n
T2 ÊV ˆ
= m RT1 ln (p1/p2) Properties relation = Á 2˜
Heat transfer Q =W T1 Ë V1 ¯
Change in entropy DS = m R ln(V2/V1) n -1
Êp ˆ n
= m R ln(p1/p2) = Á 2˜
Ë p1 ¯
4. Isentropic Process
Work transfer
Law pV g = C,
g -1
ÊV ˆ
1-g
Ê p2 ˆ g p2V2 - p1V1 m R (T2 - T1 )
T2
= Á 2˜ = Á ˜ W= =
T1 Ë V1 ¯ Ë p1 ¯ 1- n 1- n

Work transfer Heat transfer Q = mCn (T2 T1)


where polytropic specific heat
W1 =
( p2V2 - p1V1 ) = m R (T2 - T1 ) C p - nCv
2
1- g 1- g Cn =
1- n
= DU
Ê T2 ˆ
Heat transfer Q =0 Change in entropy DS = m Cn ln Á ˜
Ë T1 ¯

Glossary
Non-flow process A process in which mass of system Isothermal process Constant-temperature process
does not change Isentropic process Constant-entropy process
Isochoric process Constant-volume process Polytropic process A general representation of all
Isobaric process Constant-pressure process processes by a relation pV n = constant.
Isenthalpic process Constant-enthalpy process

Review Questions
1. Define an ideal gas. given by
2. What is an equation of state? R (T1 - T2 )
3. Show that for an ideal gas, Cp Cv = R. W1- 2 =
g -1
4. State the difference between standard symbols E
where T1 and T2 are initial and final temperatures
and U.
and R is the charecteristic gas equation.
5. Define an adiabatic process. Show that a
8. How is polytropic index determined and within
reversible adiabatic process for a given mass of a
what limits can it change?
perfect gas is pV g = constant.
9. Define polytropic specific heat and prove that
6. How are the state parameters p, V and T related
in a polytropic process? What are values of the C p - Cv
Cn =
polytropic index for isobaric, isochoric, isother- 1- n
mal and adiabatic processes? What is the significance of its negative sign?
7. Show that the work transfer per kg for a perfect 10. For a polytropic process, derive the following
gas during an adiabatic expansion process is relations:
First Law Applied to Non-Flow Systems 141

g -n (g – n): (n – 1): (g – 1)
Q1- 2 = ¥ polytropic work transfer
g -1 12. The internal energy of a certain closed system is
g -n given by U = M + N pV. Show that if it undergoes
and Q1- 2 = ¥ adiabatic work transfer
n -1 a reversible non-flow process with Q = 0, the
11. Show that when an ideal gas is compressed poly- relation between p and V is pV g = C, where C is
tropically with index n, the heat rejection, in- N +1
some constant and g = .
crease in internal energy and work done is in the N
ratio

Problems
1. 1 kg of gas expands isentropically through a mally. It is finally cooled at a constant volume to
volume ratio of 5. The initial pressure and tem- bring the system to initial state.
perature are 28 bar and 220°C, respectively. Find (a) Show the process on p–V and T–S diagrams.
(a) final pressure and temperature, (b) work done (b) Calculate the heat transfer and work transfer
during expansion, and (c) change in entropy. For in each case.
a gas take [W = 16 kJ, 18.46 kJ and Q = − 4.28 kJ,
Cp = 1.024 kJ/kg ◊ K and Cv = 0.713 kJ/kg ◊ K. 18.46 kJ, − 11.7 kJ]
[(a) 2.94 bar, − 28.6°C (b) 178 kJ (c) 0] 6. Air initially at 4 bar, 60°C, expands isothermally
2. 0.1 kg of gas at a pressure of 1 bar and 15°C is to 200 kPa and then it expands isentropically to
compressed adiabatically to 28 bar. Assuming 100 kPa. Sketch the processes on p–V and T–S
that pv = 287T for one kg of gas and the index of diagrams and calculate for entire expansion
adiabatic compression is 1.404, calculate (a) the process
final volume and temperature, (b) ratio of com- (a) heat transfer,
pression, and (c) work done during the compres- (b) change in specific internal energy, and
sion. [(a) 0.0785 m3, (b) 487°C (c) 324.7 kJ]
(c) change in specific entropy.
3. A volume of 2.5 litres of a gas at 14 bar and
[(a) 66.24 kJ/kg (b) – 43 kJ/kg
1100°C expands to a pressure of 280 kPa
according to pV1.28 = C. Determine (a) final (c) 0.198 kJ/kg ◊ K.]
volume, (b) final temperature and (c) work done 7. 0.4 kg of air at 6 bar receives an amount of
by gas during expansion. heat at constant volume, so that its temperature
[(a) 8.8 litre, (b) 692.5°C, (c) 3.7 kJ] rises from 383 K to 923 K. It is then expanded
polytropically according to pV1.32 = constant to
4. The air initially at 75 kPa, 727°C, 0.12 m3 is com-
initial temperature and finally, it is compressed
pressed isothermally until the volume is one half
isothermally to its original volume. Calculate (a)
of the initial volume. Further, it is cooled at a con-
pressure at end states, and (b) work transfer and
stant pressure till its volume is again halved.
heat transfer during each process. Take
(a) Sketch the process on p–V and T–S planes.
Cv = 0.718 and R = 0.287 kJ/kg ◊ K.
(b) Determine the total work transfer for the
two processes. [(a) p2 =14.45 bar, p3 = 0.3845 bar, (b) W1–2 =
0, Q1–2 = 155 kJ, W2–3 = 193 kJ, W2–3 = 38.64,
(c) Calculate the total heat transferred.
W3–1 = Q3–1 = −120.886 kJ]
[(b) − 10.74 kJ (c) − 22 kJ]
8. 0.5 kg air undergoes a Carnot cycle with 50%
5. Air initially at 1.00 bar has a specific volume
efficiency. The initial pressure and volume of
0.12 m3/kg. It is compressed polytropically ac-
air are 7 bar and 0.12 m3, respectively. The heat
cording to the law pv1.3 = constant to a pressure
supplied to air during isothermal expansion is 40
of 4.2 bar. It is then allowed to expand isother-
kJ. Calculate:
142 Thermal Engineering

(a) maximum and minimum temperature of the pressure of 5 bar and then expanded adiabatically
cycle, to its initial volume. Determine for this quantity of
(b) volume at the end of isothermal expansion, gas: (a) Heat transfer during the compression (b)
(c) the work and heat transfer for each of four Change in internal energy during the expansion
processes. (c) the mass of the gas.
[(a) Tmax = 312.4°C, Tmin = 19.7°C, (b) V2 = Take g = 1.4 and Cp = 1 kJ/kg ◊ K.
0.193 m3 (c) W1–2 = 40 kJ, W2–3 = 105 kJ, W3–4 [(a) − 44.3 kJ (b) − 35.5 kJ (c) 0.35 8 kg]
= −20 kJ, W4–1 = −105 kJ, Q1–2 = 40 kJ, Q2–3 = 15. A volume of 0.36 m3 of a gas at 288 K and 1.03
Q4–1 = 0, Q3–4 = 20 kJ]. bar is compressed reversibly and adiabatically to
9. 2 kg of air at 150°C and 3 bar expands according 10 bar. It is then cooled at constant pressure to its
to pV1.2 = constant, to a final pressure of 1 bar. original temperature after which it expands iso-
Find heat transfer, work transfer, change in inter- thermally. Find the heat transfer at constant pres-
nal energy and change in entropy. sure, heat transfer during isothermal expansion
Take R = 0.287 kJ/kg ◊ K and g = 1.4 and work transfer during the cycle.
[101 kJ, 203.12 kJ, –99.1 kJ and 0.26 kJ/K.] Take g = 1.4 and R = 0.287 kJ/kg ◊ K.
10. One kilogram of oxygen is compressed isother- [−119.12 kJ, 84.28 kJ, −35.2 kJ]
mally in a closed system from 100 kPa and 25°C 16. A volume of 115 litres of air at 1.05 bar and 90°C
to 300 kPa. Calculate (a) work transfer, (b) heat is compressed adiabatically until the volume
transfer, and (c) change in internal energy. reaches 11.5 litres. Find the change in internal
11. 1 kg of air initially at 1 bar and 40°C is com- energy and change in enthalpy.
pressed adiabatically until its pressure is 50 bar. [45.69 kJ, 63.96 kJ]
The air then receives 125.6 kJ of heat at constant 17. In a closed system, 0.070 kg of air is compressed
pressure. Find for each process, the change in in- polytropically from 100 kPa and a volume of
ternal energy. 0.060 m3 to a volume of 0.030 m3 with a poly-
[ U1–2 = 464.6 kJ, U2–3 = 89.73 kJ,] tropic exponent of 1.4. Compute (a) final tem-
12. 1 kg of air at 20 bar, 200°C is expanded reversibly perature, (b) the work done, (c) the change in
to 12 bar, 125°C in such a way that the process is internal energy, and (d) the heat transfer.
represented by a straight line on a p–V diagram. 18. 0.028 m3 of air initially at 260°C and 700 kPa
Determine (a) law of pressure relating volume, is expanded at constant pressure to 0.084 m3. A
(b) work and heat transfer, and (c) change in polytropic process is carried according to law
1.5
internal energy. pV = constant. The system is brought to initial
[(a) p = 40 −294 V (bar); (b) 43.82 kJ, − 9.9 kJ; state by a constant temperature process.
(c) − 53.9 kJ] (a) Sketch the processes on p–V and T–S
13. A gas at a pressure of 14 bar and a temperature diagrams,
of 343°C is expanded adiabatically to a pressure (b) Determine the net heat transfer and work
of 1.05 bar. The gas is heated at constant volume transfer,
until it again attains 343°C temperature, when (c) Find the thermal efficiency of the cycle.
its pressure is found to be 2.222 bar and finally [(b) 52.9 kJ, 52.9 kJ, (c) 38.57%]
it is compressed isothermally until the original 19. A closed system contains an ideal gas R =
pressure of 14 bar is attained. Sketch the p–V 0.27 kJ/kg ◊ K and Cv = 0.5 kJ/kg ◊ K. The system
diagram for these operations and for 0.5 kg mass undergoes the following process in a cycle. At
of the gas, calculate the change in internal energy the state 1, the temperature is 450 K and pressure
during the adiabatic expansion. is 4.5 bar. The gas is then heated at constant
Take Cp = 1.005 kJ/kg ◊ K. [–115.8 kJ] pressure until the temperature becomes 900 K at
14. A quantity of perfect gas occupies a volume of the state 2. The gas is then compressed at constant
0.3 m3 at a pressure of 1 bar and a temperature temperature unitl the value of entropy equals that
of 20°C. This is compressed isothermally to a at states 1 and 3 and finally, there is an isentropic
First Law Applied to Non-Flow Systems 143

expansion from the state 3 to the state 1. Sketch reached to 11.5 litres. Find the change in internal
the p–V and T–S diagrams. Calculate (a) pressure energy and change in enthalpy.
at the state 3, (b) heat transfer from the state 1 to [45.69 kJ, 63.96 kJ]
state 3, and (c) cyclic work transfer. 25. Air trapped in a cylinder expands frictionlessly
[(a) 32.48 bar, (b) −133.8 kJ/kg, against a piston so that pV = constant. Initially the
(c) −133.8 kJ/kg] air is at 400 kN/m2, 4°C, and occupies a volume
20. 50 litres of air at 1.013 bar and 100°C temperature of 0.02 m3. The local value of g is 9.51 m/s2.
is compressed to 28 bar. Volume of air at the end (a) To what pressure must the air expand in
of polytropic compression is found to be 4 litres. order to perform 8100 J of work?
Air is now heated at constant volume till pressure (b) What is the mass of air in the system?
rises to 56 bar. Assuming Cp = 1.00 kJ/kg ◊ K and [(a) 145.3kN/m2 (b) 0.1 kg]
Cv = 0.71 kJ/kg ◊ K over complete range of work- 26. 100 kg of air at 300 kPa and 10°C is trapped in-
ing. Determine (a) polytropic index of compres- side a vertical cylinder, which is fittted at the top
sion, and (b) entropy change in each process. with the weighted piston so that the pressure of
[(a)1.314 (b) −7.93 × 10−3 kJ/K, 0.023 kJ/K] the air is held constant, there is no heat transfer.
21. A cylinder of an engine at the beginning of suc- A paddle wheel in the cylinder is turned until
tion stroke contains 400 cm3 of gas at a pressure the volume of air has increased by 20 percent.
of 100 kPa, and temperature of 110°C. If com- Determine : (a) the work done on the air and the
pression takes place according to law pV1.3 = con- amount of the work done on the air by paddle
stant, determine the temperature and pressure of wheel. [(a) – 0.409 kJ (b) 0.571 kJ]
the gas when the volume is 80 cm3. Also calculate 27. 0.05 kg of steam at 15 bar is contained in a rigid
the work done on the gas during the compression vessel of volume 0.0076 m3, what is the tempera-
and change of internal energy of 1 kg of gas if Cv ture of steam? If the vessel is cooled, at what tem-
= 0.750 kJ/kg ◊ K. perature will the steam be just dry and saturated?
If the presure is increased to 40 bar during The cooling is continued until the pressure in
combustion at constant volume, determine the the vessel is 11.0 bar. Calculate the final dryness
temperature of gas. fraction of steam and heat rejected between initial
[− 0.0826 kJ, 177.5 kJ/kg, 2788°C] and final states.
22. A certain quantity of air has a volume of [(a) 72.36°C, (b) 192.5°C,
0.028 m3 at a pressure of 1.25 bar and 25°C. It is (c) 0.856, (d) 19.48 kJ]
compressed to a volume of 0.0042 m3 according 28. A cylinder of volume 0.1 m3 contains nitrogen
to the law pV 1.3 = constant. Calculate the final gas at 101 kPa and 20°C. If 0.5 kg of nitrogen is
temperature and work done during compression. pumped into the cylinder, calculate the new pres-
Also detemine the reduction in pressure at a sure, when the cylinder has returned to its initial
constant volume process required to bring the air temperature. Assume that the specific gas con-
back to its original temperature. stant for nitrogen is 0.297 kJ/kg ◊ K. [536.1 kPa]
[T2 = 253.48°C, W1–2 = − 8.945 kJ, 29. A vessel of 1 m3 capacity contains steam of
p2–3 = 6.393 bar] dryness fraction of 0.80. The pressure gauge on
23. If air in a cylinder is kept at constant pressure the vessel shows 9.8 bar and local barometer
by a gas tight dead weight piston. The cylinder reads 720 mm of Hg. How many kg of steam is
is 600 mm in diameter, contains a quantity of air contained in the steam vesslel? [6.89 kg]
at 27°C in such a way that the piston is 150 mm 30. A vessel having a capacity of 0.6 m3 contains
from the base of cylinder. If heat is supplied to the steam at 15 bar and 250°C. The steam is blown
air, increasing the temperature to 55°C, how far off until the pressure drops to 4 bar. Then the
the piston will move? [14 mm] valve is closed and vessel is cooled until the
24. A volume of 115 litres of air at 1.05 bar and 90°C pressure falls to 3 bar. Assuming that during the
is compressed adiabatically until the volume blowing-off period, the entropy remains constant,
144 Thermal Engineering

determine Calculate the initial and final pressures of the gas.


(i) the mass of steam blown off, The universal constant is 8314 J/kgmol.K. The
(ii) the quality of steam in the vessel after cool- molecular mass of nitrogen is 28.
ing, and [1.35 bar, 2.22 bar]
(iii) the heat transferred during the cooling pro- 34. A high altitude chamber, whose volume is 30 m3
cess is put into operation by reducing the pressure
[(a) 0.963, (b) 2.601 kg (c) –620 kJ] from 1 bar to 0.35 bar and the temperature from
31. A rigid vessel of 1 m3 volume contains steam 27°C to 5°C.
at 19 bar and 300°C. The vessel is cooled until (i) How many kg of air must be removed from
the steam is just dry and saturated. Calculate the the chamber during the process?
mass of steam in the vessel, the final pressure, (ii) Express this mass as a volume measured at
and the heat removed during the process. 1 bar and 27°C.
[7.55 kg, 15 bar and –1361 kJ] [21.68 kg, 18.67 m3]
32. 1 kg of steam at 120 bar and 400°C expands 35. 0.44 kg of gas having a volume 0.28 m3 and a
reversibly in a perfect insulated cylinder behind pressure of 1.4 bar is compressed to a pressure of
a piston until the pressure falls to 38 bar and the
14 bar according to pV1.3 = C. Find the change of
steam becomes just dry and saturated. Calculate
internal energy.
the work done by the steam. [196.34 kJ/kg]
Take Cp = 1.041 kJ/kg ◊ K
33. A mass of 3 kg of nitrogen occupying 2 m3 is
heated from 30°C to 225°C at a constant volume. and Cv = 0.743 kJ/kg ◊ K.
(68.54 kJ)

Objective Questions
1. The polytropic index n in equation pV n = constant
for a isochoric process is (c) change in internal energy is zero
(a) 1 (b) 1.4 (d) enthalpy change is maximum
(c) 0 (d) 4. In a reversible adiabatic process, the work transfer
2. A process that does not involve heat transfer is is equal to
called (a) decrease in enthalpy
(a) isothermal process (b) isolated process (b) decrease in internal energy
(c) Steady process (d) adiabatic process (c) heat ransfer
t
3. In an isothermal process (d) the product of pressure and change in
(a) temperature increases gradually volume
(b) volume remains constant

4. (b) 3. (c) 2. (d) 1. (d)


Answers
First Law Applied to Flow Processes 145

5
First Law Applied to
Flow Processes

Introduction
Most engineering problems involve flow of mass across a system boundary. The mass and energy analyses
for an open system are discussed with the help of continuity equation and the first law of thermodynamics
in this chapter. For analysis of an open system, the control-volume concept is used. The rate of work and
heat interactions with their sign convention is also considered across the control surface. The steady flow
work, relation between Ú pdv and - Ú vdp work are also carried out in the chapter. The steady-flow energy
equation is applied to the nozzle, diffuser, turbine compressor, pump, boiler, condenser and throttling
processes.

5.1 FLOW PROCESS AND FLOW


ENERGY

Most engineering problems involve mass flow in


and out of a system, such as turbines, compressors,
nozzles and pumps, etc. Such systems are called
open systems. Therefore, their analysis is carried
out with the concept of control volume, control sur-
Flow process and control volume
face and flow energy.
A flow process constitutes an open system as Consider a fluid entering the control volume
shown in Fig. 5.1, in which the working substance through a cross-sectional area A as shown in
enters and leaves the control surface of a system. Fig. 5.2. If p is the fluid pressure acting uniformly
Energy interaction in the form of heat and work at the imaginary piston at the entrance of the con-
may also take place within the system. trol volume, the force applied on the fluid element
Flow energy or flow work refers to work re- by imaginary piston is
quired to push a certain mass of fluid into and out of F = pA(kN)
the control volume. It is necessary for maintaining If the fluid is pushed by a distance L then
continuous flow through a control volume. the flow work Wf = p AL = pV ...(5.1)
146 Thermal Engineering

L Imaginary where m = mass flow rate in kg/s,


piston
Ac = cross sectional area of flow in m2,
p1, V1 A Control
Surface
v = sp. volume of fluid in m3/kg,
Control V = fluid velocity in m/s.
volume 1
Further, v =
p2, V2
r
Therefore, m = r AcV ...(5.3)
Fig. 5.2 The volume flow rate through a cross-sectional
area per unit time is called fluid discharge rate and
the flow work at entrance Wf1 = p1V1 (kJ) it is designated as V , and is expressed as
the flow work at exit Wf 2 = p2V2 (kJ) V = AcV (m3/s) ...(5.4)
Steady-Flow Process
5.2 MASS AND ENERGY ANALYSIS A steady-flow process can be defined as a process
OF AN OPEN SYSTEM during which all properties of fluid at each location
within the system remain constant with respect to
5.2.1 Conservation of Mass
time. That is, the fluid properties can change from
Principle—Continuity of Mass
point to point within the control volume but at any
The conservation of mass is one of the most fun- fixed location they remain the same during the en-
damental principle for flow systems. It states that tire process. Steady means no change with time.
the mass of a system can neither be created nor The steady-flow process requires the following par-
destroyed but its amount remains constant during ticular conditions:
any process. It only changes its form (phase). For 1. The fluid properties (intensive or extensive)
example, when 16 kg oxygen reacts with 2 kg of within the control volume remain constant at
hydrogen, 18 kg of water is formed. The conserva- each location with respect to time.
tion of mass principle for a control volume (CV)
2. The properties of the fluid crossing the
can be expressed as
boundary (inlet and outlet) remain constant
Total mass entering CV – Total mass leaving CV at each point of the boundary.
= Net change in mass within CV 3. The mass-flow rate into the system is always
The amount of mass flowing through a cross- equal to the mass flow rate out the system.
section per unit time is called the mass flow rate 4. Heat and work interactions within the
and it is calculated as surroundings occur at a steady rate.
m = Ac V ...(5.2)
v 5.2.3 Total Energy of Closed and
Open Systems
As explained earlier, the total energy of a non-
flow (closed) system comprises of internal energy,
potential energy and kinetic energy as shown in
Fig. 5.4(a). For unit mass
V2
e = u + ke + pe = u + + gz (J/kg)
2
Fig. 5.3 where V is the velocity and z is the elevation of
volume the system relative to some reference. In an open
First Law Applied to Flow Processes 147

Non-flow system q = total energy of the fluid, including


Potential flow work per unit mass, J/kg
Kinetic
2
S m1 = total mass flow rate into the control
V
e=u+
2
+ gz volume in kg/s
Internal
and S m2 = total mass flow rate out of the
control volume in kg/s.
(a) Total energy of a non-flow system
.
Q
Flow system 2
Flow
. out
Potential Sm2, q2
Kinetic Control Volume
V2 2
1 of
q = u + pv + + gz Flow
2 . in Steady-flow Device .
Sm1, q1 W z2
Flow Internal
energy z1 1

(b) Total energy of a flow system


Fig. 5.4 Fig. 5.5

or flow system as shown in Fig. 5.4(b), the fluid Energy balance on the control volume reveals:
entering and leaving possesses an additional form Total energy Total energy Total energy
of energy—flow energy (or flow work), pv which rate crossing rate leaving rate entering
pushes the fluid in and out. Then total energy of the boundary as = control – the control
flowing fluid on a unit-mass basis (denoted by q), heat and work surface 2 surface 1
becomes
or Q – W = S m2 q 2 – S m1 q1 ...(5.7)
q = pv + e = u + pv + ke + pe
V2 For a single-stream (one inlet and one exit)
= u + pv + + gz ...(5.5) system, the mass-flow rate at the inlet and exit
2
Using h = u + pv can be expressed as m1 and m2 . The inlet and exit
V2 states of the fluid for control volume are designated
Then q =h+ + gz ...(5.6)
2 by subscripts 1 and 2, respectively.
5.3 ENERGY BALANCE IN Q – W = m2 q2 – m1 q1 ...(5.8)
STEADY FLOW Using q from Eq. (5.5);

Consider a steady-flow device within a control vol- È V2 ˘


Q – W = m2 Íu2 + p2v2 + 2 + z2 g ˙
ume as shown in Fig. 5.5. The working substance ÍÎ 2 ˙˚
enters the device at the section 1 and leaves at the
È V2 ˘
section 2. - m1 Íu1 + p1v1 + 1 + z1 g ˙
Let Q = rate of heat transfer to control volume ÍÎ 2 ˙˚
in W For steady flow, the mass-flow rate through the
W = rate of shaft work done by the fluid in entire control volume remains constant, thus m1 =
control volume in W m2 = m , then
p = pressure in N/m2 È
Q - W = m Í(u2 - u1 ) + ( p2 v2 - p1v1)
u = specific internal energy in J/kg Î
h = specific enthalpy in J/kg Ê V2 V2 ˆ ˘
V = velocity of fluid in m/s + Á 2 - 1 ˜ + (z2 - z1) g ˙ ...(5.9)
z = elevation above datum in m Ë 2 2 ¯ ˙˚
148 Thermal Engineering

Using h = u + pv W = 0, since they do not involve any


È Ê shaft, thus no work interaction,
V22 V12 ˆ
then Q - W = m Í( h2 - h1) + Á - Dpe = 0, fluid experiences usually
ÍÎ Ë 2 2 ˜¯
very little or no change in its
˘ elevation between inlet and
+ (z2 - z1) g ˙ (J/s)...(5.10) outlet.
˚
Dividing both sides by m , the steady-flow en-
ergy equation on a unit-mass basis as
Ê V2 V2 ˆ
q – w = ( h2 - h1) + Á 2 - 1 ˜ + ( z2 - z1) g
Ë 2 2 ¯
...(5.11)
or q – w = Dh + Dke + Dpe (J/kg)
Q
where q= (heat transfer per unit mass, J/kg)
m
W
w= (work transfer per unit mass, J/kg)
m
Dh = Du + D(pv) (specific enthalpy change,
J/kg)
The above equation can be arranged as
q – w = Du + D(pv) + Dke + Dpe (J/kg) ...(5.12)
If the fluid has negligible change in kinetic ener- Fig. 5.6
gy and potential energy then the steady flow equa-
tion reduces to For nozzles and diffusers the steady-flow energy
q – w = Dh = Du + D(pv) (J/kg) ...(5.13) equation,
V 2 - V12
h2 – h1 + 2 =0 ...(5.14)
2
Some common steady-flow devices are discussed
below: Water turbine is a device which takes in water from
a height. The water enters into the turbine, a part of

Nozzles and diffusers as shown in Fig. 5.6 are used


in a number of engineering devices. A nozzle is a
device that increases the velocity of a fluid at the
expense of pressure drop. A diffuser is a device that
increases the pressure of a fluid by slowing it down.
Nozzles and diffusers are the devices in which
cross sectional area changes along the flow. The
following point should be noted for flow through
these devices.
Q = 0, the heat transfer rate is
negligible, Fig. 5.7
First Law Applied to Flow Processes 149

Water from. a
reservoir m, V1

Turbine z1
wheel

Generator
z2 = 0

Boundary
Stationary blades Rotating blades

Fig. 5.9
Water. flow
out m, V2
Gas or steam in .
Q (if any)

its potential energy is converted into useful work


(shaft work), which is used to generate electric Turbine
.
W
power in a generator.
The following facts should be kept in mind,
while analysing a water turbine;
Rate of work done by turbine, Gas or steam out
W = + ve
Heat transfer rate from turbine,
Q = 0, Change in potential energy,
Change in specific internal energy, Dz = 0 (if elevations are not mentioned)
Du = u2 – u1 = 0, Heat-transfer rate,
(no change in temperature of water) Q ª 0 (if turbine is insulated)
Specific volume of water, v1 = v2 = v (remains Q = – ve (if turbine is not insulated)
constant) The steady-flow energy equation
The steady flow energy equation reduces
È V22 - V12 ˘
È Ê V2 V2 ˆ Q - W = m Í( h2 - h1) + + ( z2 - z1 ) g ˙
m Í( p2 v2 - p1 v1) + Á 2 - 1 ˜ ÍÎ 2 ˙˚
ÍÎ Ë 2 2 ¯
...(5.16)
˘
+ (z2 - z1) g ˙ + W = 0 ...(5.15)
˙˚
A compressor is a device, which takes in low
pressure gas and compresses it to higher pressure.
These are the devices (Fig. 5.9 and 5.10) in which
(i) Reciprocating Compressor The reciprocating
the fluid (steam or gas) expands, work is produced
compressor is shown in Fig. 5.11. It has inlet and
on the shaft of the turbine. The work produced by a
outlet gas-flow streams and work and heat inter-
turbine is given to the generator to produce electric
actions at its control volume. For a reciprocating
power. According to sign convention, the work
compressor:
done by the turbine is considered positive.
Rate of work input W = –ve,
Rate of work done W = + ve
150 Thermal Engineering

Inlet

The water pump (Fig. 5.13) increases the pressure


of incompressible fluid like water, therefore, these
are power consuming devices. The particular con-
ditions for water pumps;
Tip

Impeller
eye
Deliver

compressor Air in take


Shaft

Fluid in
.
h1, V1 Q

Electric
Compressor
(a) Impeller of centrifugal compressor
motor
. Discharge
W
scroll
Air
flow
Fluid out
h2, V2

Fig. 5.12 compressor

Heat rejection (if any) Q = –ve,


Change in potential energy Dpe = 0, Air
Impeller flow
Change in kinetic energy Dke = 0,
Diffuser passages Vanes
The steady-flow energy equation for reciprocat-
(b) Centrifugal compressor
ing compressor becomes
Q – W = m ( h2 - h1 ) (J/s) ...(5.17) Fig. 5.13

(ii) Centrifugal Compressor/Root Blower Generally, Rate of work input to pump,


the change in potential energy is taken negligible, W = –ve,
or No heat-transfer rate from the pump,
Dpe = 0, Q = 0,
Rate of work input is W = –ve Change in specific internal energy,
Heat rejection (if any) Q = –ve Du = u2 – u1 = 0,
Then steady-flow energy equation becomes Specific volume of water,
È v1 = v2 = v = constant
Ê V2 V2 ˆ ˘
Q – W = m Í( h2 - h1 ) + Á 2 - 1 ˜ ˙ (J/s) The steady-flow energy equation reduces to
ÍÎ Ë 2 2 ¯ ˙˚
...(5.18) È Ê V2 V2 ˆ ˘
– W = m Í( p2 v2 - p1 v1) + Á 2 - 1 ˜ + ( z2 - z1) g ˙
ÍÎ Ë 2 2 ¯ ˙˚
...(5.19)
First Law Applied to Flow Processes 151

q – w = Dh + Dke + pe
Here q = 0, w = 0, Dpe = 0, then
Dh + Dke = 0
V2 V2
or h1 + 1 = h2 + 2
2 2
When V1 = V2 then h1 = h2
For a perfect gas Cp T1 = Cp T2
T1 = T2
Thus for a perfect gas, the temperature before
and after throttling is always same.

Fig. 5.14 These are the devices where the objective is to


transfer heat energy between hot and cold fluids,
therefore, the heat transfer rate cannot be taken as
zero.
It is an irreversible process. Whenever a fluid ex-
pands from a region of high pressure to low pres-
sure through a porous plug, partly open valve, an
orifice, a capillary tube or through any other ob-
struction without exchanging any energy as heat
and work transfer with surroundings and with neg-
ligible change in kinetic and potential energies, the
enthalpy of the fluid remains constant and fluid is Cold fluid in
said to undergo a throttling process.
Figure 5.14 shows steady flowing fluid through
an insulated duct, passes through an orifice at Hot fluid in Hot fluid out
section X – X. Applying steady flow energy equation
between sections 1–1 and 2–2,
1 Duct Orice 2 Cold fluid out
X
Fig. 5.16

They do not involve any shaft work or work in-


teraction. Therefore,
1
X
2 W = 0 and Q π 0 (compulsory)
If change in potential energy is negligible then,
Fluid in Fluid out Dpe = 0, (generally)
The steady-flow energy equation for these de-
(a) Adjustable valve
vices may be expressed as
È V 2 - V12 ˘
(b) Capillary tube Q = m Íh2 - h1 + 2 ˙ (J/s) ...(5.20)
ÍÎ 2 ˙˚
152 Thermal Engineering

Note The remaining terms for all devices have It should be noted that the area behind curve is
usual meanings, if specific conditions are not men-
tioned.
equal to steady flow work - vdp . Ú
5.6 RELATION BETWEEN NON-FLOW
Ú vdp WORK Ú pdv AND FLOW
The steady-flow energy equation (5.11) for unit
mass-flow rate is expressed as Ú vdp
V2 V2 The steady-flow energy equation for unit mass-flow
q – w = h2 - h1 + 2 - 1 + ( z2 - z1) g
2 2 rate
It can be written in differential form as V2 V2
q – w = h2 - h1 + 2 - 1 + ( z2 - z1) g
dq – dw = dh + VdV + gdz 2 2
Using h = u + pv It can be written in differential form as
Therefore dh = du + pdv + vdp dq – dw = dh + VdV + gdz
Inserting it above, we get Using h = u + pv and
dq – dw = du + pdv + vdp + VdV + gdz dq = du + pdv
p
Inserting it above, we get
du + pdv – dw = d(u + pv) + VdV + gdz
1
p1
or du + pdv – dw = du + d(pv) + VdV + gdz

Process 1-2
\ pdv – dw = d(pv) + VdV + gdz
or pdv = dw + d(pv) + VdV + gdz
p
p2 2
2

0

z
1
vdp

v
c
1

v1 v2
Process 1-2
Fig. 5.17
Ú vdp

d 2
According to the first law of thermodynamics
for the moving system, if an observer moving with 0 e
v
f
the fluid,
dq = du + pdv
Fig. 5.18
Ú pdv Ú vdp
Then du + pdv – dw = du + pdv + vdp + VdV + gdz Integrating both sides, we get
or – dw = vdp + VdV + gdz
V22 - V12
Integrating both sides and rearranging, we get
Ú Ú
pd v = d w + D ( p v) +
2
+ ( z2 - z1) g
V 2 - V12
Ú
w = - vdp - 2
2
- ( z2 - z1) g ...(5.21) Inserting w from Eq. (5.21), we get
If the kinetic and potential energy changes are
negligible, the above equation is reduced to
Ú pdv = - Ú vdp + D( pv)
or – vdp = p v + Ú pdv – p v
Ú
w = - vdp ...(5.22) Ú 1 1 2 2 ...(5.23)
First Law Applied to Flow Processes 153

p p p
1 1
p1 c p1

Area 1-2-f - e -1
Co
m
Area o-1-e-0-c pr
es
sio
n

Area d -2 -f -o
p2 d 2 p2 d 2

e e f f
0 v1 0 v1 v2 0 v2

Fig. 5.19

The three areas of Fig. 5.18 are shown separately

Ú
Reservoir u2
in Fig. 5.19. Thus the - vdp work can also be
p1 , h 1
related with non-flow work Ú pdv graphically as
Ú pdv = area e-1-2-f Fig. 5.20
- Ú vdp = area c-1-2-d negligible. Applying energy equation to flow process
= area c-1-e-o + area e-1-2-f V22 - V12
q – w = (u2 – u1) + (p2 v2 – p1v1) +
– area d-2-f-o 2
+ (z2 – z1)g
Ú
- vdp = p1v1 + Ú pd v - p v 2 2 The above equation reduces to
0 = (u2 – u1) – p1v1
It proves that the steady flow work - vdp is the Ú or
or
u2 = u1 + p1v1
u2 = h1
sum of net flow work and non-flow work Ú pdv . The internal energy of the gas in the cylinder is equal
to the enthalpy of air in the reservoir.
Example 5.1 A small rigid evacuated cylinder filled
from a large high-pressure reservoir. Assuming adiabatic Example 5.2 In a steady-flow system, a substance
conditions, determine the internal energy of the gas in flows at a rate of 5 kg/s. It enters the system at a pressure
the cylinder after charging. of 6 bar, velocity 300 m/s, internal energy 2000 kJ/kg
and specific volume of 0.38 m3/kg. It leaves the system
Solution
at a pressure of 1.5 bar, velocity 150 m/s, internal energy
Given Adiabatic conditions, q = 0 1600 kJ/kg and specific volume of 1.26 m3/kg. During
its passage through the system, the substance losses
To find Internal energy of the gas in the cylinder.
80 kJ/kg of heat to the surroundings. Determine the
Assumptions power of the system, stating whether it is from or to the
(i) No shaft work done during the process, w = 0. system. Neglect any changes in the potential energy.
(ii) Since there is no back pressure, therefore,
Solution
p2 v2 = 0.
(iii) Change in potential energy is negligible, z1 = z2. Given A steady-flow system, with negligible potential
(iv) Change in kinetic energy is negligible, V1 = V2. energy change.
m = 5 kg/s, q = – 80 kJ/kg
Analysis As the cylinder is very small as compared to
reservoir, therefore, the changes in the reservoir are also V1 = 300 m/s V2 = 150 m/s
p1 = 6 bar = 600 kPa p2 = 1.5 bar = 150 kPa
154 Thermal Engineering

u1 = 2000 kJ/kg u2 = 1600 kJ/kg To find Specific internal energy change device.
v1 = 0.38 m3/kg v2 = 1.26 m3/kg
Assumptions
Dz = 0
(i) Steady-state conditions, and
To find Power of the system and its direction with
(ii) Acceleration due to gravity g = 9.81 m/s2.
reference to system.
Analysis The steady-flow energy equation for 1 kg of Analysis The steady-flow energy equation is given by
mass of the substance. V22 - V12
q – w = Dh + Dke + Dpe q – w = (u2 – u1) + (p2 v2 – p1v1) +
2
= Du + D(pv) + Dke + 0 + (z2 – z1) g
Calculating each quantity separately Substituting the values of properties with proper care
Du = u2 – u1 = 1600 – 2000 = – 400 kJ/kg of units
D(pv) = p2v2 – p1v1 (– 9000) – 135 ¥ 103
= 150 ¥ 1.26 – 600 ¥ 0.38 = – 39 kJ/kg = u2 – u1 + (100 ¥ 103 ¥ 0.62)
2 2
V22 - V12 150 - 300 – (600 ¥ 103 ¥ 0.37)
Dke = =
2 2 270 2 - 16 2
= –33.75 ¥ 103J/kg = – 33.75 kJ/kg + + (0 – 32) ¥ 9.81
Now – 80 – w = – 400 – 39 – 33.75 = – 472.75 2
– 144,000 = u2 – u1 + 62000 – 222000 + 36322
or w = 472.75 – 80 = 392.75 kJ/kg
– 313.92
The power output of the system
P = m w = 5 ¥ 392.75 = 1963.75 kW u2 – u1 = – 20008 J/kg ª –20.0 kJ/kg (decrease)
Power developed by system.
Example 5.4 75 kg/min air enters the control volume
Example 5.3 In a steady-flow apparatus 135 kJ work of a steady-flow system at 2 bar and 100°C, at an eleva-
is done by each kg of fluid. The specific volume of the tion of 100 m above the datum. The same mass leaves
fluid, pressure and velocity at the inlet are 0.37 m3/kg, the control volume at 150 m elevation from datum with
600 kPa and 16 m/s, respectively. The inlet is 32 m above pressure of 10 bar and at a temperature of 300°C. The
floor level. The discharge is at floor level. The discharge entrance velocity is 40 m/s and exit velocity is 20 m/s.
conditions are 0.62 m3/kg, 100 kPa and 270 m/s. The total During the process 54,000 kJ/h of heat is transferred to
heat loss between inlet and outlet is 9 kJ/kg of the fluid. control volume and rise in enthalpy is 8 kJ/kg. Calculate
In flowing apparatus, does the specific internal energy the power developed.
increase or decrease and by how much?
Solution
Solution Given m = 75 kg/min = 1.25 kg/s
Given A steady-flow apparatus as shown in Fig. 5.21.
q = –9 kJ/kg = –9000 J/kg
.
m = 1 kg/s 1
V1 = 16 m/s
p1 = 600 kPa
= 600 ¥ 105 N/m2
w = 135 kJ/kg = 135 ¥ 103 J/kg
v1 = 0.37 m3/kg 1
Control
z1 = 32 m
volume .
2 m = 1 kg/s
V2 = 270 m/s
3 2
Floor level p2 = 100 kPa = 100 ¥ 10 N/m
v2 = 0·62 m3/kg
2 z2 = 0

Fig. 5.21
First Law Applied to Flow Processes 155

Property Inlet Outlet


Pressure (bar) 10 8.93
Specific enthalpy (kJ/kg) 2827 2341
Velocity (m/s) 20 120
Elevation (m) 3.2 0.5

Fluid flow rate through the device is 2.1 kg/s. The


work output of the device is 750 kW.

Fig. 5.22 Solution

Q = 54,000 kJ/h Given Flow through a steady-flow device as shown in


= 15 ¥ 103 W (heat supply to system) Fig. 5.23.
p1 = 2 bar p2 = 10 bar To find Heat-transfer rate from the device.
V1 = 40 m/s V2 = 20 m/s
Assumption Acceleration due to gravity g = 9.81m/s2.
z1 = 100 m z2 = 150 m
T1 = 100°C T2 = 300°C Analysis Applying steady-flow energy equation
Dh = 8 kJ/kg = 8 ¥ 103 J/kg ÔÏ V 2 - V12 Ô¸
Q - W = m Ì( h2 - h1) + 2 + ( z2 - z1 ) g ˝
To find The power developed by the system. ÓÔ 2 ˛Ô
Analysis The steady-flow energy equation is given by Calculating each item separately
h2 – h1 = 2341 – 2827 = – 486 kJ/kg
È Ê V22 V12 ˆ ˘
Q - W = m Í( h2 - h1 ) + Á - ˜ + ( z2 - z1 ) g ˙ V22 - V12 120 2 - 20 2
ÍÎ Ë 2 2 ¯ ˙˚ =
2 2
Substituting the values of properties with proper care = 7000 J/kg = 7 kJ/kg
of units (z2 – z1)g = (0.5 – 3.2) ¥ 9.81
20 2 - 40 2 = – 26.487 = – 0.0264 kJ/kg
(15 ¥ 103) – W = (1.25) ¥ [(8 ¥ 103) +
2 Substituting the value in steady-flow energy equation
+ (150 – 100) ¥ 9.81]
Q – 750 kW = (2.1 kg/s) ¥ (– 486 kJ/kg
or W = (15 ¥ 103 ) – (1.25 )
¥ (8 ¥ 103 – 600 + 490.5 ) + 7 kJ/kg – 0.0264 kJ/kg)
= (15 ¥ 103) – 9863.125 or Q = 750 – 1006 = – 256 kW
= 5136.87 W ª 5.13 kW (Heat is rejected)

Example 5.5 In a steady-flow device, the inlet and Nozzle


outlet conditions are given below. Determine the heat
loss/gain by the system. Example 5.6 Air enters a convergent nozzle with
a velocity of 40 m/s. The enthalpy of air decreases by

Fig. 5.23
156 Thermal Engineering

180 kJ/kg. Determine the exit velocity. Assume adiabatic


conditions in the nozzle.
Solution
Given V1 = 40 m/s,
Dh = – 180 kJ/kg = – 180 ¥ 103 J/kg, Schematic diagram of convergent-divergent
Q = 0 ( for adiabatic conditions) nozzle
To find Exit velocity of air at nozzle exit
To find
Assumptions (i) Exit velocity of the fluid,
(i) No change in potential energy, or z1 = z2. (ii) Mass flow rate, and
(ii) Properties of the system remain constant at (iii) Exit area of the nozzle.
their locations.
Assumption Properties of the system remains constant
Insulation at their locations.
.
Q=0
Analysis
(i) Since the nozzle does not have any work and
V1 = 40 m/s V2 = ? heat interaction, hence the steady-flow energy
equation reduces to
V22 - V12
0 = h2 – h1 +
2
Fig. 5.24 nozzle 0 = (3000 ¥ 103 J/kg – 2757
V 2 - (60 m) 2
¥ 103 J/kg) + 2
Analysis Since the nozzle does not have any work and 2
heat interaction, hence the steady-flow energy equation or V22 = 2 ¥ 243 ¥ 103 J/kg + 3600 m2
reduces to = 489600
V 2 - V12 or V2 = 699.71 m/s
0 = h2 - h1 + 2
2 (ii) The mass-flow rate can be calculated by using the
V 2 - (40 m)2 continuity equation;
0 = -180 ¥ 103 + 2
2 A1 V1 A2 V2
or V22 = 2 ¥ 180 ¥ 103 + 1600 = 361600 m = =
v1 v2
or V2 = 601.33 m/s
(0.1 m2 ) ¥ (60 m/s)
or m =
Example 5.7 At the inlet to a convergent–divergent (0.187 m3/s )
nozzle the enthalpy of the fluid passing is 3000 kJ/kg and = 32.08 kg/s
the velocity is 60 m/s. At the discharge end, the enthalpy (iii) The exit area of the nozzle can also be calculated
is 2757 kJ/kg. The nozzle is horizontal and the heat loss by continuity equation;
during flow is negligible. Find (a) velocity of the fluid at
m v2 (32.08 kg/s) ¥ (0.498 m3/kg)
the exit of the nozzle, (b) if the inlet area is 0.1 m2 and A2 = =
V2 (699.71 m/s)
the specific volume at the inlet is 0.187 m3/kg, find the
mass flow rate of the fluid, and (c) if the specific volume = 0.0228 m2
at the outlet is 0.498 m3/kg, find the area at the exit of
Example 5.8 Steam enters the nozzle, operating at
the nozzle.
steady state at 30 bar and 350°C with negligible velocity
Solution and exits at 15 bar and a velocity of 500 m/s. The mass-
flow rate is 2.22 kg/s. Determine (a) the exit temperature
Given A convergent–divergent nozzle shown in of steam, and (b) exit area of nozzle.
Fig. 5.24 with data
First Law Applied to Flow Processes 157

Solution Tsup2
and v2 = v g2
Given Flow of steam through a nozzle; Tsat 2
p1 = 30 bar T1 = 350°C (292.65 + 273)
= 0.13177 ¥
V1 = 0 p2 = 15 bar (198.32 + 273)
V2 = 500 m/s m = 2.22 kg/s = 0.15814 m³/kg
To Find (ii) The exit area can be calculated from the continuity
(i) Exit temperature of steam, and equation
A V
(ii) Exit area of the nozzle. m = 2 2
v2
Assumptions m v2 2.22 ¥ 0.15814
A2 = =
(i) No heat loss at the boundary of the nozzle, q = 0. V2 500
(ii) No change in potential energy, Dpe = 0 = 7.02 ¥ 10 – 4 m2
(iii) Specific heat of superheated steam as 2.1 kJ/kg ◊ K = 7.02 cm2
Properties of steam
Example 5.9 Helium at 300 kPa, 60°C, enters a noz-
At 30 bar and 350°C: h1 = 3115.3 kJ/kg
zle with negligible velocity and expands steadily without
At 15 bar, Tsat 2 = 198.32 C vg 2 = 0.13177, heat transfer in a quasi-equilibrium manner to 120 kPa.
hg2 = 2792.2 kJ/kg The process is such that pv1.67 = constant. Calculate the
exit velocity.

Solution
Given A nozzle with data as shown in Fig. 5.27.

Analysis
(i) The steady-flow energy equation for unit mass
q – w = Dh + Dke + Dpe
For nozzle; w = 0 and q = 0
\ 0 = h2 – h1 + Dke
( V22 - V12) (500 2 - 0 2)
Dke = =
2 2 To find Exit velocity of the fluid.
3
= 125 ¥ 10 J/kg
And h2 = h1 – Dke = 3115.3 – 125 Assumptions
= 2990.3 kJ/kg (i) No change in potential energy, or z1 = z2
Further, the h2 at 15 bar > 2792.2 kJ/kg for dry (ii) Properties of the system remain constant at their
and saturated steam. locations.
Therefore, steam is superheated at this (iii) The gas constant for helium as 2078.5 J/kg ◊ K. as
pressure. 8314
R= J/kg ◊ K
h2 = hg 2 + Cps (Tsup2 – Tsat2) 4
2990.3 = 2792.2 + 2.1 ¥ (Tsup – 198.32) Analysis The steady-flow energy equation for nozzle
2990.3 - 2792.2 V22 - V12
Tsup 2 = + 198.32 q – w = ( h2 - h1 ) + + ( z2 - z1 ) g
2.1 2
= 94.33 + 198.32 = 292.65 C where w = 0 ( no work done in a nozzle), inlet velocity
V1 is negligible and change in potential energy is also
158 Thermal Engineering

negligible and the equation is reduced to than that of air entering. Cooling water in the compressor
jackets absorbs heat from the air at the rate of 58 kW. (a)
V22
0 = ( h2 - h1 ) + Compute the power input to the compressor, and (ii) ratio
2
of inlet pipe diameter to outlet pipe diameter.
This equation cannot be used for calculation of exit
velocity V2, because h2 – h1 is also unknown. Thus, using Solution
the equation
V22 - V12 Given An air compressor with m = 0.5 kg/s of air
Ú
w = - vdp -
2
– (z2 – z1)g
.
Q = –58 kW V2 = 5 m/s
Using the assumptions p2 = 700 kPa
V22
Ú
v2 = 0.19 m3/kg
0 = - vdp -
2 u2 = u1 + 90 kJ/kg
2 Air
or V22 = -2
Ú 1
vdp
.
W=?
compressor

.
but p and v are related as pvn = C m = 0.5 kg/s
V1 = 7 m/s
C1 / n p1 = 100 kPa
or v =
p1/ n v1 = 0.95 m3/kg
2
Then V22 = - 2C1/ n
Ú1
p -1/ n dp

Ê p -1/ n + 1 - p -1/ n + 1 ˆ
= - 2C1/ n Á 2 1
˜ To find Shaft power input to compressor
Ë ( - 1 / n) + 1 ¯
Assumptions
using C1/n = p11/n v1 = p21/n v2
(i) No change in potential energy, or z1 = z2
Êp v -pvˆ (ii) Properties of the system remain constant at their
we get V2 2
= - 2n Á 2 2 1 1 ˜
Ë n -1 ¯ locations.
Ê n ˆ Analysis
= -2 Á R (T2 - T1 )
Ë n - 1˜¯ (i) The steady-flow energy equation with the above
n -1 1.67 -1
assumptions is given by
Êp ˆ n Ê 120 ˆ 1.67
where T2 = T1 Á 2 ˜ = 333 ¥ Á
Ë p1 ¯ Ë 300 ˜¯ Q -W

= 230.56 K È Ê V22 V12 ˆ ˘


= m Í(u2 - u1 ) + ( p2 v2 - p1 v1) + Á - ˜˙
Using the values ÍÎ Ë 2 2 ¯˙
˚
Ê 1.67 ˆ
V22 = –2 ¥ Á ¥ 2078.5 Calculating each term separately;
Ë 1.67 - 1˜¯
¥ [(230.56 – 333)] Change in flow work;
p2 v2 – p1v1
or V22 = – 2 ¥ 2.4925 ¥ 2078.5 ¥ (–102.44)
= (700 kPa) ¥ (0.19 m3/kg) – (100 kPa)
= 1061414 m2/s2
¥ (0.95 m3/kg) = 38 kJ/kg
or V2 = 1030.25 m/s
Change in kinetic energy;
2 2
Compressor, Pumps V22 V12 5 -7
DKe = - = = –12 J/kg
2 2 2
Example 5.10 Air flows steadily at the rate of 0.5 kg/s = –0.012 kJ/kg
through an air compressor, entering at 7 m/s velocity, Using in the above equation
100 kPa pressure and 0.95 m3/kg specific volume and –58 kW – W = (0.5 kg/s) ¥ [90 kJ/kg + 38 kJ/
leaving at 5 m/s, 700 kPa and 0.19 m3/kg, respectively. kg – 0.012 kJ/kg]
The internal energy of the air leaving is 90 kJ/kg greater
First Law Applied to Flow Processes 159

or W = –58 – 0.5 ¥ 127.988 of air leaving is 70 kJ/kg greater than that of air enter-
= –121.994 kW ing, and increase in enthalpy of circulating water is
93 kJ/kg. Compute the amount of heat transferred to the
ª –122 kW (input)
atmosphere from the compressor per kg of air.
(ii) The mass-flow rate can be expressed as
AV AV
m = 1 1= 2 2 Solution
v1 v2
A1 p d12 v1 V2 0.95 ¥ 5 Given Work input, w = –175 kJ/kg,
or = = = = 3.5714
A2 p d22 v2 V1 0.19 ¥ 7 h2 – h1 = 70 kJ/kg,
d1 qwater = – 93 kJ/kg
= 3.5714 = 1.89 (Enthalpy increase of water = heat rejected by system)
d2
To find
Example 5.11 Air at 100 kPa and 280 K is compressed
(i) The amount of heat transferred to atmosphere
steadily to 600 kPa and 400 K. The mass-flow rate of air
is 0.02 kg/s and heat losses of 16 kJ/kg occurs during the Assumptions
process. Assuming the changes in potential and kinetic (i) The heat transfer, q = qwater + qatmosphere ,
energies are negligible, determine the necessary power (ii) z1 = z2, since no information is provided regarding
input to the compressor. the potential energy change,
(iii) Neither inlet velocity nor outlet velocity is given,
Solution
therefore, we assume change in kinetic energy is
Given Steady compression of air negligible, i.e., V1 = V2.
p1 = 100 kPa p2 = 600 kPa T1 = 280 K
Analysis The steady-flow energy equation for unit-
T2 = 400 K m = 0.02 kg/s q = – 16 kJ/kg mass flow rate
Dke = 0 Dpe = 0 q – w = Dh + Dke + Dpe
To find Power input to the compressor. Using the values
q – (–175 kJ/kg) = (70 kJ/kg) + 0 + 0
Assumptions
or q = –175 kJ/kg + 70 kJ/kg
(i) Steady flow process, and = –105 kJ/kg
(ii) The specific heat at constant pressure is Since heat transfer q = qwater + qatmosphere
1.005 kJ/kg ◊ K.
(– 105 kJ/kg) = (– 93 kJ/kg) + qatmosphere
Analysis Applying steady-flow energy equation for or qatmosphere = – 105 + 93 = –12 kJ/kg
1 kg of air;
q – w = Dh + Dke + Dpe Example 5.13 A centrifugal air compressor com-
For air, an ideal gas presses 5.7 m3/min of air from 85 kPa, 0.35 m3/kg to
Dh = Cp (T2 – T1) 650 kPa, 0.1 m3/kg. If the suction-line diameter is 10 cm
= 1.005 ¥ (400 – 280) = 120.6 kJ/kg and the discharge line diameter is 6.25 cm, determine
Then – 16 – w = 120.6 + 0 + 0 (i) the mass flow rate of fluid in kg/min, (ii) the change in
or w = – 136.6 kJ/kg
It is the work input to the compressor.
The power input to compressor
p = m w = 0.02 ¥ (–136.6)
= –2.732 kW

Example 5.12 In a test of water-cooled air compres-


sor, it is found that the shaft work required to drive the
compressor is 175 kJ/kg of air delivered and the enthalpy Fig. 5.29
160 Thermal Engineering

flow work between the boundaries, and (iii) the inlet and
m v2 (0.271 kg/s) ¥ (0.1 m3/kg)
outlet velocities. and V2 = =
(p /4) d22 (p /4) ¥ (0.0625 m) 2
Solution = 8.83 m/s
Given A centrifugal compressor with
Example 5.14 Air is compressed steadily at a rate
V = 5.7 m3/ min, of 0.46 kg/s from 100 kPa, 20°C to a final pressure of
v1 = 0.35 m3/kg, 320 kPa. The compression is polytropic with a polytropic
p1 = 85 kPa = 85 ¥ 103 N/m2, index of 1.32. The volume of air changes from 3 m3/kg to
v2 = 0.1 m3/kg, 0.8 m3/kg. The inlet velocity is 25 m/s, while exit velocity
p2 = 650 kPa = 650 ¥ 103 N/m2, is 130 m/s. The delivery connection is 12 m above the
d1 = 10 cm = 0.1 m inlet. What is the shaft power of the compressor? Is it a
d2 = 6.25 cm = 0.0625 m, power-absorbing or power-producing device?

To find Solution
(i) Mass-flow rate of fluid in kg/min, Given An air compressor with mass flow rate, m =
(ii) Change in flow work between the boundaries, 0.46 kg/s, of air
(iii) Inlet and exit velocities.
Assumptions
(i) No change in potential energy, or z1 = z2 ,
(ii) No heat transfer at system boundary,
(iii) Properties of the system remains constant at their
locations.
Analysis
(i) The mass-flow rate of air
V 5.7 m3/min
m = =
v1 0.35 m3/kg
= 16.3 kg/min = 0.271 kg/s
To find Shaft power input to compressor.
(ii) The change in flow work
Assumptions
D(pV) = m ( p2 v2 – p1 v2)
(i) No heat transfer at system boundary.
= 16.3 ¥ (650 ¥ 0.1 – 85 ¥ 0.35)
(ii) Properties of the system remain constant at their
= 574 kJ/min = 9.57 kW locations.
(iii) Inlet and exit velocities (iii) Acceleration due to gravity is 9.81 m/s².
These velocities can be calculated from the continuity
equation as Analysis The relation for steady work is expressed as
AV A V V22 - V12
m = 1 1= 2 2
v1 v2 Ú
w = - vdp -
2
- ( z2 - z1) g

m v1 m v1 The p and v are related as pvn = C


or V1 = =
A1 (p / 4) d12 C1/n
3
or v =
(0.271 kg/s) ¥ (0.35 m /kg) p1/n
= = 12 m/s
(p /4) ¥ (0.1 m)2 Ê p2 v2 - p1v1 ˆ V22 - V12
we get w = - n Á - - ( z2 - z1) g
Ë n - 1 ˜¯ 2
First Law Applied to Flow Processes 161

Using the values with proper care of units using assumption, then

È 320 ¥ 0.8 - 100 ¥ 3 ˘


w = - 1.32 ¥ Í
1.32 - 1
˙ Ú Ú
w = - v dp = - v dp
Î ˚ During compression of water, the specific volume re-
130 2 - 252 12 ¥ 9.81 mains constant, therefore, total power input
- -
2 ¥ 1000 1000 p2

The kinetic energy and potential energy changes are


W = mv
Ú p1
dp = – m v( p2 – p1)

divided by 1000 to get each quantity in kJ/kg. = – (10 kg/s) ¥ (0.001 m3/kg) ¥ (5000 – 500) (kPa)
w = 181.5 – 8.1375 – 0.117 = – 45 kW (input power)
= 173.25 kJ/kg
Example 5.16 In a water-cooled compressor, 0.6 kg
The power input to the compressor of air is compressed per second. The power required to
W = mw run the compressor is 40 kW. The heat lost to the cooling
= (0.46 kg/s) ¥ (173.25 kJ/kg) water is 30% of input, and 10% of input is lost in bearing
= 79.7 kW and other frictional effects. The air enters the compressor
at 1 bar and 30°C. If the changes in potential energy and
Example 5.15 Water at the rate of 10 kg/s is com- kinetic energy are neglected, estimate the exit tempera-
pressed adiabatically from 5 bar to 50 bar in a steady ture of air. Take Cp for air as 1.005 kJ/kg ◊ K.
flow process. Calculate the power required, assuming
that the specific volume of water to be 0.001 m3/kg, which Solution
remains almost constant.
Given A water-cooled compressor;
m = 0.6 kg/s W = – 40 kW (work input)
Solution
p1 = 1 bar = 100 kPa T1 = 30°C = 303 K
Given Compression of water The total losses of input may be considered as
m = 10 kg/s p1 = 5 bar = 500 kPa
Q = (0.3 + 0.1) of input = 0.4 ¥ 40
p2 = 50 bar = 5000 kPa Q =0
= –16 kW (losses),
v = 0.001 m3/kg
Cp = 1.005 kJ/kg ◊ K
To find Power input to pump
To find Exit temperature of air
Assumptions
(i) No change in potential energy, and
(ii) No change in kinetic energy.
Analysis The steady-flow energy equation;
Q - W = m (Dh + Dke + Dpe)
Fig. 5.31 Dropping out, Dke and Dpe, and using the values
(– 16 kW) – (– 40 kW) = (0.6 kg/s) ¥ (Dh)
Assumption
we get Dh = 40 kJ/kg
(i) Change in potential energy, Dpe = 0,
(ii) Change in kinetic energy Dke = 0, The change in specific enthalpy can be expressed as
Analysis In a steady flow process, the work transfer Dh = Cp (T2 – T1)
can be obtained as or 40 = 1.005 ¥ (T2 – 303)

Ú
w = - vdp - D ke - D pe or T2 = 342.8 K = 69.8°C
162 Thermal Engineering

Example 5.17 A centrifugal pump delivers 50 kg of The flow rate of water can be expressed as
3 3
water per second. The inlet and outlet pressures are 1 bar V = m v = (50 kg/s) ¥ (0.001 m /kg) = 0.05 m /s
and 4.2 bar respectively. The suction is 2.2 m below the and V = V1 A1 = V2 A2
centre of the pump and delivery is 8.5 m above the centre
V V (0.05 m3/s) ¥ 4
of the pump. The suction and delivery pipe diameters are V1 = = 2
=
20 cm and 10 cm, respectively. Determine the capacity of
A1 (p /4) d1 p ¥ (0.2 m)2
electric motor to run the pump. = 1.59 m/s
V V (0.05 m3/s) ¥ 4
Solution or V2 = = =
A2 (p /4) d22 p ¥ (0.1 m) 2
Given A centrifugal water pump with flow rate = 6.37 m/s
m = 50 kg/s Calculating each term of the steady-flow energy
d1 = 20 cm = 0.2 m equation separately:
d2 = 10 cm = 0.1 m (i) Since the temperature remains constant during
p1 = 1 bar = 1 ¥ 105 N/m2, pumping, therefore, the specific internal energy
p2 = 4.2 bar = 4.2 ¥ 105 N/m2 change,
z1 = 8.5 m Du = 0
z2 = – 2.2 m (ii) The change in kinetic energy,
To find The power input to pump V22 - V12 6.372 - 1.592
Dke = =
2 2
= 19.02 J/kg = 0.019 kJ/kg
(iii) Change in potential energy,
Dpe = g(z2 – z1)
= (9.81m/s²) ¥ [8.5 m – (–2.2 m)]
= 105 J/kg = 0.105 kJ/kg
(iv) Change in flow work,
D(pv) = p2v2 – p1v1 = v( p2 – p1)
= (0.001 m3/kg) ¥ (420 kPa – 100 kPa)
= 0.32 kJ/kg
Substituting the values of each term in the
equation we get,
0 – W = 50 ¥ (0 + 0.32 + 0.019 + 0.105)
or W = –22.2 kW
Fig. 5.32
Turbine
Assumptions
(i) No temperature change of water during pumping, Example 5.18 Steam enters a turbine with a velocity
thus Du ª 0. of 40 m/s and specific enthalpy of 2500 kJ/kg; and
(ii) No heat transfer during the pumping. leaves with a velocity of 90 m/s and specific enthalpy of
(iii) Specific volume of water is constant and is 2030 kJ/ kg. Heat losses from the turbine to surroundings
0.001 m³/kg. are 240 kJ/min and the steam-flow rate is 5040 kg/h.
(iv) Acceleration due to gravity, g = 9.81 m/s. Neglect the change of potential energy. Find the power
developed by the turbine.
Analysis The steady-flow energy equation is given by
Q - W = m (Du + D(pv) + Dke + Dpe) Solution
Given A steam turbine as shown in Fig. 5.33.
First Law Applied to Flow Processes 163
.
1 Q = – 240 kJ/min = – 4 kW p1 = 7.2 bar 1
h1 = 2500 kJ/kg T1 = 850 oC
V1 = 40 m/s V1 = 160 m/s
Steam
turbine . Gas turbine
Dpe = 0 W=? W¢ = ?
.
m = 5040 kg/h = 1.4 kg/s
Q=0
2
h2 = 2030 kJ/kg 2
p2 = 1.15 bar
V2 = 90 m/s
cp = 1.04 kJ/kg.K T2 = 450oC
Fig. 5.33 V2 = 250 m/s

Fig. 5.34
To find Power developed by the turbine.
Assumptions
Analysis The steady-flow energy equation is
Q – W = m (Dh + Dke + Dpe) (i) No change in potential energy, or Dpe = 0
Calculating each term of the steady-flow energy (ii) Properties of the system remains constant at their
equation separately locations
(i) Change in specific enthalpy, Analysis The steady-flow energy equation is
Dh = h2 – h1 = 2030 – 2500 q – w = Dh + Dke + Dpe
= – 470 kJ/kg Calculating each term of the steady-flow energy
(ii) Change in kinetic energy, equation separately
V22 - V12 Change in specific enthalpy,
Dke =
2 Dh = Cp (T2 – T1)
90 2 - 40 2 = (1.04) ¥ (450 – 850) = – 416 kJ/kg
= = 3250 J/kg
2 Change in kinetic energy,
= 3.25 kJ/kg V22 - V12 (250)2 - (160)2
Dke = =
Substituting the values of each term in the equation, 2 2
we get, = 18450 J/kg = 18.45 kJ/kg
(– 4 kW) – W = (1.4 kg/s) ¥ (– 470 kJ/kg Substituting the values of these terms in steady-flow
+ 3.25 kJ/kg + 0) energy equation, we get
or W = 653.45 – 4 = 649.45 kW 0 – w = (– 416 kJ/kg) + (18.45 kJ/kg) + 0
or w = 397.55 kJ/kg
Example 5.19 The gas turbine of a turbojet engine
receives a steady flow of gases at a pressure of 7.2 bar, Example 5.20 A hydraulic turbine is supplied with
a temperature of 850°C and a velocity of 160 m/s. It dis- 25 m3/s of water. At the turbine inlet, the water is at 5 bar,
charges the gases at a pressure of 1.15 bar, a temperature and 25°C with an elevation above datum of 100 metres
of 450°C, and a velocity of 250 m/s. Determine the ex- and a flow velocity of 1 m/s. At the turbine exit, the water
ternal work output of the turbine in kJ/kg of the gas flow. is at 1.2 bar and 25.1°C with zero elevation and a flow
The process may be assumed to be adiabatic and Cp for velocity of 11 m/s. The turbine loses 5 J of heat per kg of
combustion gases may be taken as 1.04 kJ/kg ◊ K. water flowing through it. Assuming that the water is an
incompressible fluid with specific heat of 4.178 kJ/kg ◊ K,
Solution determine
(a) the change in potential energy,
Given A gas turbine of a turbojet engine as shown in
Fig. 5.34. (b) the change in internal energy, and
(c) the power output in MW.
To find The work developed by the turbine in kJ/kg.
164 Thermal Engineering

Solution (Water has a single specific heat, which is given


as 4.178 kJ/kg ◊ K.
Given A hydraulic turbine is shown in Fig. 5.35.
(iii) Change in kinetic energy,
V = 25 m3/s p1 = 5 bar
p2 = 1.2 bar T1 = 25°C Ê V 2 - V12 ˆ
DKE = m ¥ Á 2 ˜
V1 = 1 m/s T2 = 25.1°C Ë 2 ¯
z1 = 100 m z2 = 0 (11m) 2 - (1 m) 2
V2 = 11 m/s Cp = 4.178 kJ/kg ◊ K = (25000 kg/s) ¥
2
q = –5 J/kg = 15 ¥ 105 J/s = 1500 kW
The total heat-loss rate
Water from a
reservoir Q = m q = (25000 kg/s) ¥ (–5 J/kg)
.
V = 25 m3/s, = –125 ¥ 103 J/s = – 125 kW
V1 = 1 m/s, The steady-flow energy equation is given by
p1 = 5 bar,
T1 = 25°C Q - W = m (Dh + Dke + Dpe)
Turbine Substituting the values of each term in the
wheel equation, we get,
z1 = 100 m –125 kW – W = 10445 kW + 1500 kW
.
– 24.525 ¥ 103 kW
W or W = 12445 kW or 12.445 MW
Boundary
Example 5.21 Steam initially at 1.5 MPa and 300°C
expands reversibly and adiabatically in a steam turbine
q = –5 J/kg
Water flow out
z2 = 0 to 40°C. Determine the
V2 = 11 m/s, (a) condition of steam after expansion,
p2 = 1.2 bar, (b) work done by the turbine per kg of steam.
T2 = 25.1oC

Fig. 5.35 Solution


Given Isentropic expansion of steam;
To find
p1 = 1.5 MPa T1 = 300°C
(i) The change in potential energy, Q =0 T2 = 40°C
(ii) The change in internal energy, and
(iii) Power developed by the turbine in MW. To find
(i) Condition of steam after expansion,
Analysis The mass-flow rate of water
m = Volume ¥ density T

= 25 m3/s ¥ 1000 kg/m3 1 1.5 MPa, 300°C


= 25000 kg/s
(i) The change in potential energy,
DPE = m g (z2 – z1)
= (25000) ¥ (9.81) ¥ (0 – 100)
= –24.525 ¥ 106 J/s 2
40°C
= – 24.525 ¥ 103 kW
(ii) The change in internal energy
DU = DH = m Cp DT
= (25000) ¥ (4.178) ¥ (25.1 – 25) s
= 10445 kW Fig. 5.36
First Law Applied to Flow Processes 165

(ii) Turbine work output. ditions are 1.25 bar and 40 m/s. The mass-flow rate of air
is 1000 kg/h. The flow of air is assumed to be reversible
Assumptions
adiabatic. Calculate
(i) The kinetic energy change is negligible. (a) the temperature of air at exit, and
(ii) The potential energy change is negligible. (b) the power output of the turbine.
(iii) No heat transfer occurs during expansion. Assume Cp = 1.053 kJ/kg ◊ K and adiabatic index =
Properties of steam At 1.5 MPa (1500 kPa) and 300°C: 1.375.
h1 = 3038.9 kJ/kg s1 = 6.92 kJ/kg ◊ K
Solution
At 40°C p2 = 7.3 kPa sf2 = 0.572 kJ/kg ◊ K
sg2 = 8.257 kJ/kg ◊ K hf2 = 167.57 kJ/kg Given A turbine with operating data as shown in
hfg2 = 2406.7 kJ/kg Fig. 5.37.
m = 1000 kg/h = 0.278 kg/s
Analysis
Cp = 1.053 kJ/kg ◊ K,
(i) From observation of the magnitude of entropy g = 1.375
values at the two states, we find that the steam To find
should be wet at exhaust. For isentropic (i) The temperature of air at exit,
expansion, (ii) The power output of the turbine.
s1 = s2 = sf2 + x(sg2 – sf2)
Assumptions
6.92 = 0.572 + x(8.257 – 0.572)
(6.92 - 0.572) (i) No change in potential energy, or Dpe = 0,
or x = = 0.826 (ii) Properties of the system remains constant at their
7.685
(ii) Steady-flow work per kg of steam, locations.
q – w = Dh + Dke + Dpe Analysis
Substituting the values in the above equation, we
get (i) The exit temperature can be calculated by
the using property relationship for reversible
w = h1 – h2
adiabatic process;
Where h2 = hf2 + xhf g2
(g -1)
= 167.57 + 0.826 ¥ 2406.7
T2 Êp ˆ g
= 2155.5 kJ/kg = Á 2˜
T1 Ë p1 ¯
and w = h1 – h2 = 3038.9 – 2155.5
(g -1)
= 883.4 kJ/kg Êp ˆ g
or T2 = T1 Á 2 ˜
Ë p1 ¯
Example 5.22 A turbine operating on air has inlet
(1.375 - 1)
conditions of 10 bar, 750 K, and 200 m/s, while exit con-
Ê 1.25 ˆ 1.375
= 750 ¥ Á
Ë 10 ˜¯
p1 = 10 bar = 1 ¥ 106 N/m2
T1 = 750 K
1 = 750 ¥ 0.567 = 425.3 K
V1 = 200 m/s

.
m = 1000 kg/h = 0.278 kg/s Air turbine
Cp = 1.053 kJ/kg.K, .
W=?
g = 1.375
.
Q=0
2
p2 = 1.25 bar = 125 ¥ 103 N/m2
V2 = 40 m/s

Fig. 5.37
166 Thermal Engineering

(ii) The steady-flow energy equation is Analysis The mass rate of steam leaving the turbine
Q – W = m (Dh + Dke + Dpe) m3 = m1 + m2 = 5 + 15 = 20 kg/s
Calculating each term of steady-flow energy equation Total enthalpy of steam coming from the boiler 1;
separately: H1 = m1 h1 = (5 kg/s) ¥ (3911.72 kJ/kg)
Change in specific enthalpy, = 19558.6 kJ/s
Dh = Cp (T2 – T1) Total enthalpy of steam coming out of the boiler 2;
= (1.053) ¥ (425.3 – 750) H2 = m2 h2 = (15 kg/s) ¥ (3480.6 kJ/kg)
= –341.83 kJ/kg = 52209 kJ/s
Change in kinetic energy, Total enthalpy of steam leaving the turbine
V22 - V12 40 2 - 200 2 H3 = m3 h3 = m3 (hf 3 + x3 hfg3)
Dke = = = (20 kg/s) ¥ (191.81 + 0.96
2 2
= –19200 J/kg ¥ 2392.82) (kJ/kg)
= –19.2 kJ/kg = 49778.3 kJ/s
Substituting the values of these terms in the steady- Applying steady-flow energy equation
flow energy equation, we get, Q - W = DH + DKE + DPE
0 – W = (0.278 kg/s) ¥ [(– 341.83 kJ/kg) Calculating each item separately
+ (–19.2 kJ/kg) + 0] Given Q = 0; DKE = 0 and DPE = 0
or W = 100.3 kW Then DH = Exit enthalpy – Inlet enthalpy
= H3 – (H1 + H2)
Example 5.23 A steam turbine receives steam from = 49778.3 – (19558.6 + 52209)
two boilers One flow is 5 kg/s at 3 MPa, 700°C and the = – 21989.25 kJ/s
other flow is 15 kg/s at 800 kPa, 500°C. The exit state Then – W = DH = – 21989.25 kW
is 10 kPa, with a quality of 96%. Find the total power or W = 21989.25 kW = 21.99 MW
output of the adiabatic turbine.
Example 5.24 The mass-flow rate of steam into a
Solution steam turbine is 1.5 kg/s and the heat transfer from the
Given A steam turbine receiving steam from two boil- turbine is 8.5 kW. The steam is entering in the turbine at
ers: the pressure of 2 MPa, temperature of 350°C, velocity
Boiler 1: m1 = 5 kg/s p1 = 3 MPa T1 = 700°C of 50 m/s, elevation of 6 m and is leaving the turbine
Boiler 2: m2 = 15 kg/s p2 = 800 kPa T2 = 500°C at a pressure of 0.1 MPa, quality of 100 %, velocity of
200 m/s, elevation of 3 m. Determine the power output
Exit : Q =0 p3 = 10 kPa x3 = 0.96
of the turbine.
To find Power output of the turbine
Solution
Assumptions
Given Flow through a turbine
(i) Steady flow through the turbine,
m = 1.5 kg/s, Q = –8.5 kW
(ii) Changes in kinetic and potential energies are
negligible. p1 = 2 MPa, p2 = 0.1 MPa
T1 = 350°C x2 = 1.0
Properties of steam V1 = 50 m/s V2 = 200 m/s
At 3 MPa or 3000 kPa and 700°C; z1 = 6 m z2 = 3m
v1 = 0.14838 m3/kg, h1 = 3911.72 kJ/kg
To find Power output of the turbine.
At 800 kPa and 500°C;
v2 = 0.44331 m3/kg, h2 = 3480.6 kJ/kg Assumptions
At 10 kPa; (i) Steady flow condition,
vg3 = 14.673 m3/kg, hf 3 = 191.81 kJ/kg, (ii) Acceleration due to gravity as.
hfg3 = 2392.82 kJ/kg, hg 3 = 2584.63 kJ/kg g = 9.81 m/s2
First Law Applied to Flow Processes 167
.
2 MPa Q = –8.5 kW To Find
50 m/s, (i) Dryness fraction of steam after isentropic expan-
350°C
1.5 kg/s sion and adiabatic heat drop, and
6m (ii) Pressure after throttling and increase in entropy
Steam during throttling.
turbine .
W Analysis Properties of steam
At pressure 25 bar, hf1 = 962 kJ/kg
0.1 MPa hfg1 = 1839 kJ/kg
200 m/s sf 1 = 2.5543 kJ/kg ◊ K
x =1,
3m sg1 = 6.2536 kJ/kg ◊ K
Fig. 5.38 At pressure 15 bar, hf 2 = 844.75 kJ/kg
hfg2 = 1947.2 kJ/kg
Analysis Properties of steam: sf 2 = 2.3145 kJ/kg ◊ K
State 1: 2MPa, 350°C, (From superheated steam table) sg2 = 6.4406 kJ/kg ◊ K
h1 = 3136.96 kJ/kg (i) The dryness fraction of steam after isentropic
State 2: 0.1 MPa, (From pressure entry steam table) expansion,
x = 1.0 h2 = 2675.45 s1 = s 2
Using steady-flow energy equation. sf1 + x(sg1 – sf1) = sf2 + x2 (sg2 – sf2)
Q - W = m (Dh + Dke + Dpe) 2.5543 + 0.9 ¥ (6.2536 – 2.5543)
Calculating each quantity separately = 2.3145 + x2 (6.4406 – 2.3145)
Dh = h2 – h1 = 2675.45 – 3136.96 5.8848 - 2.3145
= – 461.51 kJ/kg or x2 = = 0.865
4.1256
V22 - V12 ( 200) 2 - (50) 2 Enthalpy drop during isentropic expansion
Dke = =
2 2 Dh = h1 – h2 = (hf + x1hfg ) – (hf + x2hfg )
1 1 2 2
= 18750 J/kg = 18.75 kJ/kg = (962 + 0.9 ¥ 1839) – (844.75 + 0.865
Dpe = (z2 – z1) g = (3 – 6) ¥ 9.81 ¥ 1947.2)
= – 29.43 J/kg = – 0.0294 kJ/kg = 88.0 kJ/kg
Using these values in the above equation; (ii) The pressure after throttling
– 8.5 – W = 1.5 ¥ [– 461.51 + 18.75 – 0.0294] Since h2 = h3
or – W = 8.5 + 1.5 ¥ (– 442.78) The enthalpy of steam after throttling
or W = 655.68 kW h3 = h2 = 844.75 + 0.865 ¥ 1947.2
= 2529.08 kJ/kg
Example 5.25 Steam initially at a pressure of 25 bar
Pressure corresponds to this enthalpy when steam
and 0.9 dry expands isentropically to 15 bar and it is then
is just dry.
throttled until it is just dry. Find
= 0.016 bar
(a) the quality of steam after isentropic expansion
Entropy at this pressure
and adiabatic heat drop,
s3 = 8.834 kJ/kg ◊ K
(b) the pressure after throttling, increase in entropy
Change in entropy
during throttling,
= s3 – s2 = s3 – [sf2 + x2 (sg2 – sf2)]
Solution = 8.834 – [2.3145 + 0.865 (6.4406 – 2.3145]
= 2.945 kJ/kg K
Given Isentropic expansion and throttling of steam
p1 = 25 bar x1 = 0.9 dry Example 5.26 The power output of a steam turbine is
p2 = 15 bar, x3 = 1 5 MW. The inlet conditions are 2 MPa of pressure, 400°C
168 Thermal Engineering

of temperature, 50 m/s of velocity and 10 m of elevation.


V22 - V12
The exit conditions are 15 kPa, 0.9 dry quality, 180 m/s, Dke =
2
and 6 m elevation.
(180 m/s) 2 - (50 m/s) 2
(a) Compute the magnitude of Dh, Dke, and Dpe. =
2
(b) Determine the work done per kg mass of steam. = 14950 J/kg = 14.95 kJ/kg
(c) Calculate the mass flow rate of steam. and Dpe = g (z2 – z1)
= (9.81 m/s2) ¥ (6 m – 10 m)
Solution = –39.24 J/kg = – 0.0392 kJ/kg
Given Steam turbine W = 5 MW (ii) Work done by the turbine per kg of steam
p1 = 2 MPa = 200 kPa V1 = 50 m/s Applying steady-flow energy equation for unit
T1 = 400°C + 273 = 673 K, z1 = 10 m mass
p2 = 15 kPa x2 = 0.9 q – w = Dh + Dke + Dpe
V2 = 180 m/s z2 = 6 m 0 – w = –885. 87 + 14.95 – 0.0392
or w = 870.95 kJ/kg
2 Mpa q=0
o
(iii) The required mass-flow rate for a 5-MW power
400 C
50 m/s 1 output
10 m
Steam m = W = 5 ¥ 1000 = 5.74 kg/s
turbine .
W = 5 kW w 870.95

15 kPa Example 5.27 A turbine operating on a steady flow


0.9 dry 2
of air is to produce 1 kW of power by expanding air from
180 m/s
6m 300 kPa, 350 K, 0.346 m3/kg to 120 kPa. For preliminary
Fig. 5.39 design, the inlet velocity is assumed to be 30 m/s, the
exit velocity is assumed to be 50 m/s and the expansion
To Find follows the law pv1.4 = constant. Determine the required
mass-flow rate of air.
(i) The magnitude of Dh, Dke and Dpe,
(ii) Work done per kg mass of steam, Solution
(iii) The mass flow rate of steam.
Given An air turbine as shown in Fig. 5.40.
Assumptions
(i) Steady-state conditions through the turbine, To find Mass-flow rate of air.
(ii) No heat loss from the boundary of turbine, (q = 0). Assumptions
Analysis (i) No change in potential energy, or z1 = z2,
(i) At the inlet conditions, 2 MPa and 400°C: (ii) No heat transfer from the turbine,
h1 = 3247.6 kJ/kg (iii) Properties of the system remains constant at their
At the turbine exit, at 15 kPa: locations.
hf = 225.94 kJ/kg p1 = 300 kPa
hfg = 2373.1 kJ/kg T1 = 350 K
h2 = hf2 + xhfg2 V1 = 30 m/s
v1 = 0.346 m3/kg Turbine .
W = 1 kW = 1000 W
= 225.94 + 0.9 ¥ 2373.1 pv1.4 = C

= 2361.73 kJ/kg
Now, Dh = h2 – h1 p2 = 120 kPa
V2 = 50 m/s
= 2361.73 – 3247.6
= – 885.87 kJ/kg
First Law Applied to Flow Processes 169

Analysis The steady-flow energy equation for analysis w = 82953.18 J/kg @ 83 kJ/kg
is given by The mass-flow rate can be obtained as
V22 - V12 W 1 kW
q – w = h2 – h1 + + g (z2 – z1) m= = = 0.012 kg/s = 43.3 kg/h
2 w 83 kJ/kg
where q = 0 and change in potential energy is also negli-
gible and then equation reduces to Example 5.28 Consider a gas turbine power plant
V 2 - V12 with air as the working fluid. Air enters at 100 kPa, 20°C,
– w = h2 – h1 + 2 r1 = 1.19 kg/m3 with a velocity of 130 m/s through an
2
If the work per unit mass is determined, the mass flow opening of 0.112 m2 cross-sectional area. After being
rate can be calculated as compressed, heated and expanded through the turbine,
W = mw the air leaves at 180 kPa, 150°C, r2 = 1.48 kg/m3,
But with this equation also, we cannot proceed for through an opening of the same size. The power output
calculation of exit velocity V2, because h2 – h1 is also of the plant is 375 kW. The internal energy and enthalpy
unknown and cannot be directly calculated. Thus using of air are given in kJ/kg by u = 0.717 T and h = 1.004 T,
the equation where T is the temperature on kelvin scale. Determine the
net amount of heat transferred to air in kJ/kg.
Ú
w = - vdp - D ke - D pe
Solution
Dropping potential energy change
Given A gas turbine of a power plant
Ú
w = - vdp - D ke
A = 0.112 m2
p and v are related as pvn = C u = 0.717 T (kJ/kg)
C1 / n and h = 1.004 T ( kJ/kg)
or v = 1/ n
p To find Heat transferred per kg.
2
Then w = - C1 / n
Ú1
p -1/ n dp - D ke

È p ( -1/ n) + 1 - p1( -1/ n) + 1 ˘


= - C1 / n Í 2 ˙ - D ke
ÍÎ ( -1 / n ) + 1 ˙˚
Using C1/n = p11/n v1 = p21/n v2
Ê p v - p1v1 ˆ
We get w = - n Á 2 2 - D ke
Ë n - 1 ¯˜ Fig. 5.41
1
-
v2 Ê p2 ˆ n Assumptions
Using = Á ˜
v1 Ë p1 ¯ (i) No change in potential energy, or z1 = z2.
È ( n -1) ˘ (ii) Properties of the system remains constant at their
Ê n ˆ ÍÊ p2 ˆ n ˙
We get w = - Á pv - 1˙ - D ke locations.
Ë n - 1˜¯ 1 1 ÍÍÁË p1 ˜¯ ˙
Î ˚ Analysis The steady-flow energy equation for unit
Using the values mass flow rate;
Ê 1.4 ˆ V22 - V12
w = –Á ¥ 300 ¥ 103 q – w = h2 – h1 + + g (z2 – z1)
Ë 1.4 - 1˜¯
2
The mass-flow rate
ÈÊ 120 ˆ 0.4 /1.4 ˘ 50 2 - 30 2 m = r1 V1 A1
¥ 0.346 ÍÁË ˜ - 1˙ -
Í 300 ¯ ˙ 2 = (1.19 kg/m3) ¥ (130 m/s) ¥ (0.112 m2)
Î ˚
= 17.326 kg/s
170 Thermal Engineering

The exit velocity of fluid Solution


m (17.326 kg/s)
V2 = = Given A steam power plant operates as a steady flow
r2 A2 (1.48 kg/m3 ) ¥ (0.112 m2 )
system as shown in Fig. 5.42.
= 104.5 m/s
To find Power output from the power plant.
The work done per unit mass;
Assumptions
W 375 kW
w= = = 21.64 kJ/kg (i) Boiler and turbine together as a system,
m 17.326 kg/s
(ii) Properties of the system remain constant at their
The specific change in enthalpy;
locations.
h2 – h1 = Cp (T2 – T1)
= (1.004 kJ/kg ◊ K) ¥ (423 K – 293 K) Analysis The steady-flow energy equation for a com-
= 130.52 kJ/kg bined system is given by
The change in kinetic energy; Q – W = m [Dh + Dke + D pe]
V22 - V12 Calculating each term separately
Dke =
2
(i) The net heat-transfer rate,
(104.52 - 130 2 ) ( m2 / s 2 ) Q = Heat supplied to the boiler
=
2 – heat rejected from the system
= – 2990 J/kg = – 2.99 kJ/kg = m (h2 – h1) – Qrej
Substituting these values in the steady-flow energy = (1 kg/s) ¥ (3140 – 840)(kJ/kg)
equation – 20 kW = 2280 kJ/kg
q – 21.64 = 130.52 – 2.99 + 0 (ii) The change in specific enthalpy;
or q = 148.55 kJ/kg Dh = h3 – h1
= 2640 kJ/kg – 840 kJ/kg
In a steam power plant, the steady
= 1800 kJ/kg
flow conditions prevail, 3600 kg of water per hour enters
(iii) The change in kinetic energy;
the boiler at a specific enthalpy of 840 kJ/kg and a speed
of 300 m per minute at 5-m elevation. The water receives V32 - V12 (50 2 - 52 ) (m 2 /s 2 )
Dke = =
heat at constant pressure in the boiler and increases 2 2
specific enthalpy to 3140 kJ/kg and then steam formed = 1237.5 J/kg = 1.237 kJ/kg
enters the turbine. The steam leaves the turbine at the (iv) The change in potential energy
speed of 3000 m/min at an elevation of 1 m and a specific D pe = g (z3 – z1)
enthalpy of 2640 kJ/kg. Heat losses from the turbine and
= (9.81 m/s²) ¥ (1 m – 5 m)
boiler are 72000 kJ/h. Determine the power output of the
power plant. = – 39.24 J/kg = – 0.039 kJ/kg

. .
QS Qrej = –72000 kJ/h = –20 kW

.
m = 3600 kg/h = 1 kg/s h2 = 3140
h1 = 840 kJ/kg 1 KJ/kg
V1 = 300 m/min = 5 m/s Boiler
2 .
z1 = 5 m Turbine W
Control volume
3
V3 = 3000 m/min = 50 m/s
h3 = 2640 kJ/kg
z3 = 1 m

Fig. 5.42
First Law Applied to Flow Processes 171

Substituting the values of these terms in steady flow The temperature T2, after polytropic expansion
energy equation 1- n
T2 Ê V2 ˆ
2280 kW – W = (1 kg/s) ¥ (1800 + 1.237 – 0.039) = Á ˜
T1 Ë V1 ¯
(kJ/kg) = 180.197 kW
1- n
or W = 478.8 kW Ê V2 ˆ
or T2 = T1 Á ˜
Ë V1 ¯
Example 5.30 Helium gas is expanded polytropically
= 573 ¥ (2.5)1 – 1.5135 = 358.1 K
in a turbine from 4 bar, 300°C to 1 bar such that the final
The gas constant R for helium,
volume is 2.5 times the initial volume. The velocity of gas
Ru 8.314
at the exit is 50 m/s. What is the mass-flow rate of gas R = = = 2.0785 kJ/kg K
required to produce 1 MW turbine output? How much M 4
The steady flow work/kg during expansion is ex-
is the heat transfer during the process? Also determine
pressed as
exit area of the turbine. Assume specific heat of helium =
5.193 kJ/kg at constant pressure. Ú
w = - vdp - D ke - D pe
where for polytropic expansion process
Solution
2 Ê n ˆ
Given Polytropic expansion of helium gas through a (a) - Ú
1
vdp = - Á
Ë n - 1˜¯
(p2 v2 – p1 v1)
turbine n
= - R (T2 – T1)
Helium gas
. n -1
Q =?
p1 = 4 bar 1.513
= - ¥ 2.0785 ¥ (358.1
T1 = 300°C
. (1.513 - 1)
Gas turbine W = 1 MW – 573)
= 1317.37 kJ/kg
Cp = 5.193 kJ/kg.K p2 = 1 bar V22 - V12 (50 m)2 - (0) 2
(b) Dke = =
V2 = 2.5 V1 2 2
V2 = 50 m/s = 1250 J/kg = 1.25 kJ/kg
(c) Dpe = 0
Therefore, the specific work done during
To find expansion
(i) Mass-flow rate of helium through the turbine, w = 1317.37 + 1.25 + 0
(ii) Heat transferred during expansion process, and = 1318.62 kJ/kg
(iii) Exit area of the turbine. Further, turbine output
W = mw
Assumptions
or 1000 kW = m ¥ 1318.62
(i)The molecular weight of the helium = 4 kg/kmol. or m = 0.7583 kg/s or 2730 kg/h
(ii)Steady flow conditions through the turbine. (ii) Heat transferred during the steady flow process
(iii)Universal gas constant Ru = 8.314 kJ/kmol ◊ K.
Q - W = m (Dh + Dke + Dpe)
(iv) Inlet velocity of helium gas is negligible i.e.,
V1 = 0. where Dh = Cp (T2 – T1)
(v) Change in potential energy is negligible. = 5.193 ¥ (358.1 – 573)
= – 1115.97 kJ/kg
Analysis Therefore, the heat transferred,
(i) The index of expansion of helium gas Q – 1000 = 0.7583 ¥ (– 1115.97 + 1.25 + 0)
p1V1n = p2V2n or Q = 1000 – 845.3 = 154.7 kW
Ê 4ˆ (iii) Exit area of the turbine can be obtained by using
ln Á ˜
ln ( p1 / p2) Ë 1¯ continuity equation;
or n = = = 1.513
ln (V2 /V1) ln ( 2.5) V2 = A2 V2
172 Thermal Engineering

where p1 V1 = m R T1 To find
0.7583 ¥ 2.0785 ¥ 573 (i) Rate of heat transfer in heat exchanger,
V1 =
400 (ii) Power output from turbine, and
3 (iii) Exit velocity of air from nozzle.
= 2.257 m /s
and V2 = 2.5 V1 = 5.644 m3/s Assumptions
V 5.664
Therefore, A2 = 2 = = 0.113 m2 (i) Steady flow condition,
V1 50 (ii) Since no information regarding the elevation is
= 11.3 ¥ 104 mm2 provided, thus assuming no change in potential
energy, i.e., z1 = z2 = z3 = z4
Example 5.31 Air-enters at 15°C through a heat
exchanger at a velocity of 30 m/s where its temperature Analysis Considering steady flow process through
is raised to 800°C. It then enters a turbine with the same each device separately.
velocity of 30 m/s and expands until the temperature (i) Control volume: Air heat exchanger
falls to 650°C. On leaving the turbine, air is taken at a Initial state: T1 = 15°C, V1 = 30 m/s
velocity of 60 m/s to a nozzle where it expands until the Final state: T2 = 800°C, V2 = 30 m/s
temperature has fallen to 500°C. If the air-flow rate is Applying steady flow energy equation to heat
2 kg/s, calculate. exchanger,
(a) Rate of heat transfer to the air in the heat
ÔÏ V22 - V12 Ô¸
exchanger, Q - W = m Ì( h2 - h1) + + ( z2 - z1) g ˝
ÔÓ 2 Ô˛
(b) Power output of the turbine, assuming no heat
loss, and
1 2
(c) Velocity of air at the exit from nozzle, assuming
no heat loss. Take specific enthalpy of air as h
Heat
= CpT, where Cp = 1.005 kJ/kg ◊ K and T is the exchanger
temperature in °C.
Fig. 5.45
Solution
For a heat exchanger
Given Air undergoes a steady-flow process through a W = 0, V1 = V2 and z1 = z2 (assumed)
heat exchanger, turbine and nozzle as shown below. Then Q = m (h2 – h1) = m Cp (T2 – T1)
Air m = 2 kg/s = ( 2 kg/s) ¥ (1.005 kJ/kg ◊ K)
h = CpT Cp = 1.005 kJ/kg ◊ K ¥ (800 – 15) (K)
State 1: T1 = 15°C V1 = 30 m/s = 1577.85 kJ/s
State 2: V2 =30 m/s, T2 = 800°C (ii) Control volume: Air turbine
State 3: T3 = 650°C, V3 = 60 m/s
Initial state: T2 = 800°C, V2 = 30 m/s
Q2–3 = 0
Final state: T3 = 650°C, V3 = 60 m/s
State 4: T4 = 500°C, Q3–4 = 0
and Q2 – 3 = 0
.
Q Applying steady-flow energy equation to turbine
1 2 ÔÏ V32 - V22 Ô¸
Q - W = m Ì( h3 - h2) + + ( z3 - z2) g ˝
Heat ÓÔ 2 ˛Ô
Turbine .
exchanger W Using the data it reduces to
3
ÏÔ V32 - V22 ¸Ô
Nozzle 4
– W = m ÌC p (T3 - T2) + ˝
ÔÓ 2 Ô˛
Fig. 5.44
First Law Applied to Flow Processes 173

2 or V42 = 2 ¥ 1005 ¥ (650 – 500)


Turbine .
+ (60)2 = 305100
W or V4 = 552.36 m/s
Q=0
z2 = z3 3
Example 5.32 A heat exchanger receives air at a
velocity of 25 m/s and 20°C. The temperature of air in-
Fig. 5.46
creases to 780°C in a heat exchanger.
Ï The air then passes through the turbine. The velocity
60 2 - 30 2 ¸
or – W = 2 ¥ Ì1005 ¥ (650 - 800) + ˝ of air while entering the turbine is 25 m/s. The air
Ó 2 ˛ expands in the turbine, where its temperature falls to
= 2 ¥ (– 150750 + 1350)
630°C. On leaving the turbine, air passes through the
= – 298800 W nozzle. The velocity of air entering the nozzle is 60 m/s.
or W = 298800 W = 298.80 kW The air expands in the nozzle till its temperature falls to
(iii) Control volume: Nozzle 500°C. The air-flow rate is 2.5 kg/s. Determine
Initial state: T3 = 650°C, V3 = 60 m/s (a) Rate of heat transfer to air in the heat exchanger
Final state: T4 = 500°C (b) Power output from the turbine, assuming no heat
For nozzle W = 0, Q = 0, z3 = z4 loss
(c) The velocity at the exit from nozzle, assuming no
heat loss

Solution
Given Air undergoes a steady flow process through a
heat exchanger, turbine and nozzle as shown below:
Air m = 2.5 kg/s
Fig. 5.47 State 1: T1 = 20°C V1 = 25 m/s
State 2: V2 = 25 m/s, T2 = 780°C
Then steady-flow energy equation reduces to
State 3: T3 = 630°C, V3 = 60 m/s
ÔÏ V42 - V32 Ô¸ Q2–3 = 0
0 = m Ì( h4 - h3 ) + ˝
ÓÔ 2 ˛Ô State 4: T4 = 500°C, Q3–4 = 0
2 (h3 – h4) + V32 = V42 To find
or 2
V4 = 2 Cp (T3 – T4) + V3 2 (i) Rate of heat transfer in the heat exchanger,
(ii) Power output from turbine, and
(iii) Exit velocity of air from nozzle.

Air 1 2
V1 = 25 m/s
o
T1 = 20 C
.
m = 2.5 kg/s V2 = 25 m/s
Heat T2 = 780 C
o

exchanger
Turbine
.
W

3
V3 = 60 m/s V4 = ?
T3 = 630oC o
T4 = 500 C
Nozzle 4

Fig. 5.48
174 Thermal Engineering

Assumptions V42 = 2 (h3 – h4) + V32


(i) Steady flow condition, or V42 = 2 Cp (T3 – T4) + V32
(ii) Since no information regarding the elevation is or V42 = 2 ¥ 1005 ¥ (630 – 500) + (60)2
provided, thus assuming no change in potential = 264900 m2/s2
energy, i.e., z1 = z2 = z3 = z4, or V4 = 514.68 m/s
(iii) Specific heat of air at constant pressure is
1.005 kJ/kg ◊ K.
Analysis Applying steady-flow process through each
Example 5.33 The steam at 2 MPa and 300°C
device separately.
is throttled to an atmospheric pressure of 100 kPa.
(i) Control volume: heat exchanger Determine the final temperature of air. Take the specific
W = 0, Dpe = 0 Dke = 0 heat of superheated steam as 2.2 kJ/kg ◊ °C.
Initial state: T1 = 20°C, V1 = 25 m/s
Final state: T2 = 780°C, V2 = 25 m/s Solution

The steady flow energy equation reduces to Given The throttling of steam
Q = m (h2 – h1) = m Cp (T2 – T1)
= (2.5) ¥ (1.005) ¥ (780 – 20) p1 = 2 MPa p2 = 100 kPa
T1 = 300°C T2 = ?
= 1909.5 kJ/s = 1909.5 kW
(ii) Control volume: Air turbine Fig. 5.49
Initial state: T2 = 780°C, V2 = 25 m/s To find The temperature of air after throttling.
Final state: T3 = 630°C, V3 = 60 m/s
Assumptions
Q2 – 3 = 0
(i) Flow through the throttle valve is assumed
Steady flow energy equation reduces to adiabatic, Q = 0.
ÔÏ V 2 - V22 Ô¸ (ii) Throttling valves do not involve any type of work
– W = m Ì( h3 - h2 ) + 3 ˝ interaction, therefore, W = 0.
ÓÔ 2 ˛Ô
(iii) No information regarding elevations and inlet
ÏÔ V 2 - V22 ¸Ô and exit velocity is provided, thus assuming,
or – W = m ÌC p (T3 - T2 ) + 3 ˝
ÔÓ 2 Ô˛ Dpe = 0, Dke = 0.

ÏÔ Analysis For a throttling device, the enthalpy of the


or – W = (2.5) ¥ Ì(1.005) ¥ (630 - 780) system before and after throttling remains constant. Thus,
ÓÔ h1 = h2
(60) 2 - ( 25) 2 ¸Ô The enthalpy of steam at 2 MPa and 300°C can be
+ ¥ (10 -3) ˝ obtained from the steam table for superheated steam as
2 Ô˛
h1 = 3023.5 kJ/kg
or W = 373.156 kW This enthalpy equals the exit enthalpy, thus
(iii) Control volume: Nozzle h2 = 3023.5 kJ/kg
W = 0, Dpe = 0 Q =0 But the exit pressure is 100 kPa, the properties of
Initial state: T3 = 630°C, V3 = 60 m/s saturated steam at 100 kPa;
Final state: T4 = 500°C hg,100 kPa = 2675.5 kJ/kg,
Then Steady flow energy equation reduces to Tsat = 99.63°C
ÏÔ V42 - V32 ¸Ô Since the enthalpy after throttling is greater than
0 = m Ì( h4 - T3 ) + ˝ enthalpy of saturated steam at 100 kPa, thus the quality
ÓÔ 2 ˛Ô of steam will be superheated. It can be obtained as
First Law Applied to Flow Processes 175

h2 = hg, 100 kPa + Cps (T2 – Tsat) During the throttling process enthalpy remains con-
stant;
or 3023.5 kJ/kg = 2675.5 kJ/kg + (2.2 kJ/kg ◊ °C)
h1 = h2
¥ (T2 – 99.63)(°C)
or Cp T1 = Cp T2 or T1 = T2
3023.5 - 2675.5
or T2 = + 99.63 Then specific volume at exit
2.2
RT2 0.287 ¥ 368
= 258.81°C v2 = =
p2 3.5 ¥ 100
Example 5.34 Air flows at the rate of 2.3 kg/s in a = 0.301 m3/kg
15-cm diameter pipe. It has a pressure of 7 bar and a The exit velcoity, from continuity equation
temperature of 95°C before it is throttled by a valve to m v2 2.3 ¥ 0.301
3.5 bar. Find the velocity of air demonstration of the V2 = =
A2 (p / 4)(0.15) 2
restrictions.
= 39.2 m/s
Solution
Mixing of Streams
Given Throttling of air as shown in Fig. 5.50
Example 5.35 A certain water heater operates under
1 2
d1 = 0.15 m steady flow conditions, receiving 4.2 kg/s of water at
p1 = 7 bar p2 = 3.5 bar
.
75°C temperature and an enthalpy of 313.93 kJ/kg. The
m = 2.3 kg/s d2 = 15 cm water is heated by mixing with steam, which is supplied
T1 = 95°C
to a heater at a temperature of 100.2°C and an enthalpy
Fig. 5.50 of 2676 kJ/kg. The mixture leaves the heater as liquid
water at a temperature of 100°C and an enthalpy of
To find The velocity of air after throttling 419 kJ/kg. How much steam must be supplied to the
heater per hour?
Assumptions
(i) Adiabatic conditions ( Q = 0), Solution
(ii) Steady flow process,
Given Heating of water with steam mixing as shown
(iii) Change in potential energy is negligible, i.e.,
in Fig. 5.51
(z1 = z2)
(iv) Air properties, R = 0.287 kJ/kg ◊ K, Cp = 1.005 Steam
kJ/kg ◊ K T2 = 100.2°C
h2 = 2676 kJ/kg
Analysis For the given data, initial specific volume
RT1 0.287 ¥ 368 Water
v1 = =
p1 700 T3 = 100°C
= 0.151 m3/kg h3 = 419 kJ/kg

From continuity equation the initial velocity Water


.
AV m = 4.2 kg/s
m = 1 1 T1 = 75°C
v1 h1 = 313.93 kJ/kg
m v1 m v1
or V1 = =
A1 (p /4) d12
2.3 ¥ 0.151 To find Mass-flow rate of steam to heater.
or V1 = = 19.63 m/s
p
¥ (0.15) 2 Assumptions
4
(i) No heat loss to surroundings,
176 Thermal Engineering

(ii) Steady flow process, The initial energy of fluid in cylinder


(iii) Change in kinetic and potential energies is E1 = m1u1 + (m2 – m1)ui ...(5.26)
negligible. Energy of system at the end of charging
Analysis For adiabatic mixing streams, the steady flow E2 = m2u2 ...(5.27)
energy equation can be expressed as When the mass (Dmsystem) enters the cylinder,
m1h1 + m2 h2 = ( m1 + m2) h3 the cylinder volume becomes (m2 – m1)vi. The
or 4.2 ¥ 313.93 + m2 ¥ 2676 = (4.2 + m2 ) ¥ 419 work is done on the system to force the gas into the
cylinder at pressure pi acting in the supply mains.
or 1318.5 + 2676 m2 = 1759.8 + 419 m2
W = – (m2 – m1) p i vi.
or 2257 m2 = 441.3
where suffix i indicates the condition in the tank
or m2 = 0.1955 kg/s
after charging of cylinder.
Applying the first law of thermodynamics to this
system
Q -W = E2 - E1
Many practical processes involve unsteady flow,
Net energy transfer Change in energy
which cannot be analysed with the steady-flow by heat, work and mass
energy equation. While charging and discharging a or Q + (m2 – m1) p ivi
gas cylinder, the properties of gas inside the cylinder = m2u2 – [m1u1 + (m2 – m1)ui]
vary with time. The mass flow rates into and out of
or Q + (m2 – m1)(pivi + ui)
the cylinder are not same and there is accumulation
= m2u2 – m1u1
of mass inside the cylinder. Such flow processes are
transient and analysis of such processes is made by or Q + (m2 – m1)hi = m2u2 – m1u1
control mass analysis and then by control volume If the cylinder is insulated (Q = 0) and initially
analysis. evacuated (m1 = 0), then during adiabatic charging,
the specific enthalpy of gas in the supply main is
equal to the specific internal energy energy of the
Supply line
gas in the cylinder at the end of charging. That is
hi = u2 ...(5.28)
For an ideal gas h = Cp T and u = Cv T, then
T2 = g Ti ...(5.29)

Control volume Example 5.36 A spherical vessel of 1.5-m diameter


containing air at 315 K is evacuated until the vaccum
inside the vessel is 730 mm of Hg ; the evacuation process
is carried out at constant temperature. Determine the
mass of air pumped out. Also, calculate the pressure in
the tank, if remaining air is cooled to 275 K.
Mass of the gas, which enters the cylinder during
For air R = 0.287 kJ/kg ◊ K. Take atmospheric pres-
charging process can be expressed as sure as 760 mm of Hg.
m2 – m1 = Dmsystem ...(5.24)
Energy balance: Solution

E2 – E1 = DEsystem ...(5.25) Given Spherical vessel of constant capacity


V = Constant patm = 760 mm of Hg,
where suffix 1 = initial state, and 2 = final state of
R = 0.287 kJ/ kg ◊ K
control volume.
First Law Applied to Flow Processes 177

State 1: D = 1.5 m or ro = 0.75 m Example 5.37 An oxygen cylinder has a capacity of


T1 = 315 K 300 litres and contains oxygen at a pressure of 3.10 MPa
State2: pvacuum = 730 mm of Hg T2 = 315 K and a temperature of 18°C. The stop valve is opened and
State 3: T3 = 275 K some gas is used. If the pressure and temperature of the
oxygen left in the cylinder falls to 1.7 MPa and 15°C re-
To find
spectively, determine the mass of oxygen used.
(i) Mass of air pumped out, and If the stop valve is closed, the oxygen remaining in the
(ii) Pressure of air in the tank after cooling. cylinder gradually attains its initial temperature of 18°C.
Determine the amount of heat transferred through the
Assumption Density of mercury, r = 13.59 ¥ 103
cylinder wall from the atmosphere. The density of oxygen
kg/m3. at 0°C and 0.101325 MPa is 1.429 kg/m3 and g = 1.4 for
Analysis oxygen.
(i) The volume of spherical tank
Solution
4 3 4 3
V = p r o = p ¥ (0.75)
3 3 Given An oxygen cylinder with
= 1.767 m3 V = 300 lit = 300 ¥ 10–3 m3 = 0.3 m3
p1V State 1: p1 = 3.1 MPa= 3100 kPa
The initial mass of air ; m1 =
RT1 T1 = 18°C + 273 = 291 K
p2V
Final mass of air, m2 = (T1 and V are State 2: p2 = 1.7 MPa = 1700 kPa
RT1
constant) T2 = 15°C + 273 = 288 K
The mass of air pumped out State 3: T3 = 18°C + 273 = 291 K
(p1 - p2 )V r = 1.429 kg/m3
fi m1 – m2 = ...(i) T0 = 0°C + 273 = 273 K
RT1
Absolute pressure of air in the tank after evacution p0 = 0.101 325 MPa g = 1.4
p2 = pam – pvaccum = 760 – 730 To find
= 30 mm of Hg (i) Mass of oxygen used,
Then p1 – p2 = 760 – 30 = 730 mm of Hg (ii) Heat transfer Q2–3.
or p1 – p2 = rgh
Ê 730 ˆ Assumptions
= 13.59 ¥ 103 ¥ 9.81¥ Á m
Ë 1000 ˜¯ (i) Oxygen as a perfect gas,
= 97322 Pa = 97.32 kPa (ii) Reversible process.
Using in Eq (i) with other numerical values
Analysis
97.32 ¥ 1.767
m1 – m2 = = 1.90 kg (i) The density of oxygen at 0°C (or 273 K) and
0.287 ¥ 315
0.101325 MPa (101.325 kPa) is given as 1.428
(ii) Pressure of air in the tank after cooling;
p2 p3 kg/m3. Using perfect gas equation for unit mass;
= p0
T2 T3 = RT0
r
T3 (101.325 KPa)
or p3 = p 2 ¥ or R =
T2 (1.429 kg/m3 ) ¥ (273 K)
275 = 0.259 kJ/kg ◊ K
= 30 mm of Hg ¥
315
= 26.2 mm of Hg From the characteristic gas equation, the initial
This pressure can be expressed in kPa as mass of the oxygen in the cylinder,
p3 = r g h p1V1 (3100 KPa) ¥ (0.3 m3 )
m1 = =
26.2 RT1 (0.259 kJ/kg ◊ K) ¥ (291 K)
= 13.59 ¥ 103 ¥ 9.81 ¥ ¥ 10 -3
1000
= 12.3 kg
= 3.49 kPa
178 Thermal Engineering

The mass of the oxygen remaining in the cylinder To find


after its use. (i) Mass of steam admitted into cylinder,
p2 V2 (ii) Temperature of steam.
m2 =
RT2
Properties of steam
(1700 KPa) ¥ (0.3 m3 )
= At 10 bar (Saturated)
(0.259 kJ/kg ◊ K) ¥ (288 K)
v1 = 0.1944 m3/kg
= 6.83 kg
h1 = 2778.08 kJ/kg
Therefore, the mass of oxygen used
u1 = 2582.64 kJ/kg
Dm = m1 – m2 = 12.3 – 6.83 = 5.46 kg
At 40 bar and 400°C (Superheated)
(ii) The heat transferred during temperature change
v2 = 0.07341 m3/kg
of remaining oxygen in the cylinder
h2 = 3213.51 kJ/kg
Q2–3 = m2 Cv (T3 – T2)
R 0.259 Analysis The initial mass of steam in the tank
where Cv = =
g - 1 1.4 - 1 V 2 m3
= 0.6475 kJ/kg ◊ K m1 = = = 10.288 kg
v1 0.1944 m3/kg
Then Q2–3 = (6.83 kg) ¥ (0.6475 kJ/kg ◊ K) The mass of steam after steam admission at 4.0 MPa
¥ (291 K – 288 K)
V 2 m3
= 13.26 kJ m2 = = = 27.244 kg
v2 0.07341 m3/kg
A 2 m3 tank with perfectly insulated The mass of steam admitted into tank
walls contains saturated steam at a pressure of 1.0 MPa. madmitted = m2 – m1 = 27.244 – 10.288
This tank is connected to a steam line through a valve. = 16.956 kg
The valve is opened and the superheated steam at 4 MPa
For transient filling,
and 400°C flows into the tank until the pressure in the
tank is 4 MPa. Calculate the mass and temperature of Q + (m2 – m1)hi = m2u2 – m1u1
steam that enters the tank. For insulated tank Q = 0; then
16.956 ¥ 3213.51
Solution
= 27.244 u2 – 10.288 ¥ 2582.64
Given A tank of fixed capacity V = 2 m3 It gives u2 = 2975.27 kJ/kg
State 1: p1 = 1 MPa = 10 bar (Saturated steam) Interpolating at 40 bar between u(450°C) = 3010.13
State 2: p2 = 4 MPa = 40 bar kJ/kg and u(400°C) = 2919.88 kJ/kg, the final temperature
T2 = 400°C (Superheated steam) to be 430°C.

Summary
The mass-flow rate through a cross-section is mass-flow rate of the system is
expressed as
È
m = rAV (kg/s) Q - W = m Íu2 - u1 + p2 v2 - p1 v1
The volume-flow rate through a cross-section is ÍÎ
V 2 - V12 ˘
given by + 2 + (z2 - z1 ) g ˙
V = AV = m v (m3/s) 2 ˙˚
Using h = u + pv
The steady-flow energy equation for a given
First Law Applied to Flow Processes 179

then Q - W and duct, the shaft work w or W is always


zero.
È V 2 - V12 ˘
= m Íh2 - h1 + 2 + (z2 - z1 ) g ˙ (J/s) (ii) For throttle valve h1 = h2.
ÍÎ 2 ˙˚ (iii) For work consuming devices like water
Dividing both sides by m , the steady flow energy pumps, blowers, and compressors, the shaft
equation on a unit mass basis as work (input) W (or w) is always negative.
V22 - V12 (iv) For a water pump, v1 = v2, u1 = u2 and q = 0.
q – w = h2 – h1 + + g (z2 – z1) (v) For a turbine engine, the shaft work W (or
2
The steady flow process can be modeled for the w) is always positive.
following devices as follows: (vi) The other terms in the steady-flow energy
(i) For flow through a nozzle, diffuser, boiler, equation have their usual meaning if not
condenser, heat exchanger, throttle valve specified in the problem.

Glossary
Internal energy Total energy minus kinetic and poten- Diffuser Converts kinetic energy into pressure energy
tial energy Turbine Rotary engine used to generate power
Enthalpy Sum of internal energy and product of pres- Compressor A device which increases the pressure of a
sure and volume fluid
Steady No change with time Water pump Lifts water through an elevation at the
Steady flow process A process of fluid flow with con- cost of power input
stant properties at each location with respect to time Heat exchanger A device that transfers heat energy
Flow energy Work associated with fluid push in and between two fluids at different temperatures
out of a control volume Throttling Device A flow-control valve which reduces
Nozzle Converts pressure energy into kinetic energy pressure irreversibly

Review Questions
1. State the continuity equation with its nomencla- 8. Draw a control volume and write the energy and
ture. mass transfer and derive an expression for steady-
2. What is the total energy of a flow system? Identify flow energy equation.
different forms of energy that constitute the total 9. Write the steady-flow energy equation and state
energy of a system. the significance of various terms involved.

Ú
3. Define and explain how following properties are
10. State the significance of - vdp .
related with heat energy
(i) intrenal energy , and (ii) enthalpy.
4. Define and explain the flow work.
Ú
11. What is the relation between - vdp and Ú pdv ?
5. Distinguish between energy of a non-flow system 12. Differentiate between pv work and
Ú pdv work.
and a flow system.
6. What is flow energy? Do fluids at rest possess any
13. Explain the significance of
Ú pdv in case of
flow energy?. (a) flow process, and (b) non-flow process.
7. What is a steady flow process? What are the 14. Deduce the steady-flow energy equation for
conditions for a steady flow process? (a) reciprocating compressor, and (b) water pump.
180 Thermal Engineering

15. State the consequences of the first law of thermo- 19. Why are throttling devices commonly used in
dynamics. refrigeration and air-conditioning appliances?
16. When air enters a diffuser and decelerates, does 20. Would you expect the temperature of a liquid to
its pressure increase or decrease? Deduce the change as it is throttled? Explain.
steady-flow energy equation for a diffuser. 21. Apply steady-flow energy equation to each of the
17. Define the mass and volume flow rates. How are following :
they related to each other? (a) Boiler (b) Nozzle (c) Centrifugal pump
18. How is a steady flow system characterised? (d) Steam turbine (e) Water turbine

Problems
1. A stream of gases at 7.5 bar, 750°C and 140 m/s transfers to surroundings and engine foundation
is passed through a turbine of a jet engine. The is 25 MJ/h. The fuel–air mixture enters the engine
stream comes out of the turbine at 2.0 bar, 550°C at the rate of 150 kg/h. Determine the change in
and 280 m/s. The process may be assumed enthalpy of the air–fuel mixture stream in kJ/kg.
adiabatic. The enthalpies of gas at the entry and Neglect changes in kinetic energy and potential
exit of the turbine are 950 kJ/kg and 650 kJ/kg energy. [–1864 kJ/kg]
of gas respectively.Determine the capacity of the 5. 12 kg of a fluid per minute goes through a
turbine if the gas flow is 5 kg/s. [1353 kW] reversible steady flow process. The properties of
2. A steam turbime has the following data: fluid at the inlet are p1 = 1.4 bar, r1 = 25 kg/m3,
V1 = 120 m/s and u1 = 920 kJ/kg and at the exit
Parameter Inlet Outlet are p2 = 5.6 bar, r2 = 5 kg/m3, V2 = 180 m/s and u2
Steam pressure (bar) 14 0.07 = 720 kJ/kg. During the passage, the fluid rejects
Internal energy (kJ/kg) 2730 2340 60 kJ/s and rises through 60 metres. Determine
Specific volume (m³/kg) 0.166 18.6 (a) the change in enthalpy (Dh), and (b) work
Velocity of steam (m/s) 120 330 done during the process (W).
[Dh = – 93.6 kJ/kg ; W = – 44.2 kW]
The heat rejected from the turbine is 20 kJ/kg and
the power output is 20 kW. Neglecting changes in 6. A gas flows steadily through a rotary compressor.
potential energy, determine the mass flow rate of The gas enters the compressor at a temperature
steam. [0.0471 kg/s] of 16°C, a pressure of 100 kPa, and an enthalpy
of 391.2 kJ/kg. The gas leaves the compressor at
3. In one of the section of the heating plant in which a temperature of 245°C, a pressure of 0.6 MPa
there are no pumps, a steady flow of water enters and an enthalpy of 534.5 kJ/kg. There is no heat
at a temperature of 50°C and a pressure of 3 bar transfer to or from the gas as it flows through the
(h = 240 kJ/kg). The water leaves the section at a compressor.
temperature of 35°C and at a pressure of 2.5 bar
(h = 192 kJ/kg). The exit pipe is 20 m above the (a) Evaluate the external work done per unit
entry pipe. Assuming change in kinetic energy to mass of gas, assuming the gas velocities at
be negligible, evaluate the heat transfer from the entry and exit to be negligible.
water per kg of water flowing. [47.8 kJ/kg] (b) Evaluate the external work done per unit
4. A petrol engine operates steadily with a work mass of gas when the gas velocity at entry
output of 36 kW. The heat transfer rate to cooling is 80 m/s and that at the exit is 160 m/s.
water is 125 MJ/h. The sum of all other heat [143.3 kJ/kg, 152.9 kJ/kg]
First Law Applied to Flow Processes 181

7. In a steady flow process, the working fluid flows of the gas flow. The process may be assumed to be
at a rate of 240 kg/min. The fluid rejects 120 kJ/s adiabatic and Cp of gas as 1.04 kJ/kg ◊ K.
passing through the system. The conditions of [397.55 kJ/kg]
fluid at inlet and outlet are given as V1 = 300 m/s,
12. The centrifugal air compressor of a gas turbine
p1 = 6.2 bar, u1 = 2100 kJ/kg, v1 = 0.37 m3/kg
receives atmospheric air at 1 bar and 300 K.
and V2 = 150 m/s, p2 = 1.3 bar, u2 = 1500 kJ/kg,
It discharges the air at 4 bar and 480 K with
v2 = 1.2 m3/kg. The suffix 1 indicates the conditions
a velocity of 100 m/s. The mass-flow rate
at inlet and 2 indicates at outlet of the system.
compressor is 15 kg/s. Determine the power input
Neglecting the change in potential energy,
to compressor. [2810 kW]
determine the power capacity of the system in
MW. [2.7086 MW] 13. A blower handles 1 kg/s of air at 20°C and
consumes a power of 15 kW. The inlet and outlet
8. 15 kg of air per min. is delivered by a centrifugal
velocities are 100 m/s and 150 m/s respectively.
compressor. The inlet and outlet conditions of
Find the exit air temperature, assuming adiabatic
air are V1 = 10 m/s, p1 = 1 bar, v1 = 0.5 m3/kg
condition. Take Cp of air as 1.005 kJ/kg ◊ K.
and V2 = 80 m/s, p2 = 7 bar, v2 = 0.15 m3/kg.
The increase in enthalpy of air passing through [28.38°C]
the compressor is 160 kJ/kg, and heat loss to the 14. A gas flows steadily through a rotary compressor.
surroundings is 720 kJ/min. Assuming that inlet The gas enters the compressor at a temperature
and discharge lines are at the same level, find of 16°C a pressure of 100 kPa and an enthalpy of
(a) Motor power required to drive the compres- 391.2 kJ/kg. The gas leaves the compressor at a
sor, temperature of 245°C, a pressure of 0.6 MPa and
an enthalpy of 534.5 kJ/kg. If the flow through the
(b) Ratio of inlet to outlet pipe diameter.
compressor is adiabatic, calculate
[(a) 52.78 kW (b) d1/d2 = 5.16]
9. A compressor takes air at 100 kN/m2 and (a) Work input to the compressor per unit
delivers the same at 550 kN/m2. The compressor mass of gas, assuming no change in kinetic
discharges 16 m3 of free air per minute. The energy of the gas,
density of air at inlet and exit are 1.25 kg/m2 (b) Work input per kg of gas, if its inlet velocity
and 5 kg/m3. The power of the motor driving is 80 m/s and exit velocity is 160 m/s
the compressor is 40 kW. The heat lost to the [(a) 143.3 kJ/kg, (b) 152.9 kJ/kg]
cooling water circulated around the compressor 15. A centrifugal compressor supplies air at 0.5 MPa,
is 30 kJ/kg of air passing through the compressor. 200°C and 250 m/s velocity to a stationary gas
Neglecting changes in PE and KE, determine the turbine plant. The air enters the compressor from
change in specific internal energy. [60 kJ/kg] an atmosphere at 1 bar, 35°C and with negligible
10. 60 kg of water is delivered by a centrifugal pump velocity. Determine the work input per unit
per second. The inlet and outlet pressures are 1 mass of the air. Assuming adiabatic flow, will an
bar and 4 bar respectively. The suction is 2 m electric motor of 1200 kW rating be adequate for
below the centre of the pump and delivery is driving this compressor, if the mass-flow rate of
8 m above the centre of the pump. Determine the the air is 8 kg/s? [197.07 kJ/kg, No]
capacity of the electric motor to run the pump. 16. Calculate the work input per kg to pump water
The suction and delivery pipe diameters are insentropically from 100 kPa, 30°C to 5 MPa.
20 cm and 10 cm and respectively. [27.15 kW] [4.92 kJ/kg]
11. A gas turbine of a turbojet engine receives 17. Determine the power required to derive a pump
a steady flow of gases at a pressure of 7.2 bar, which raises the water pressure from 1 bar at
temperature of 850°C and a velocity of 160 m/s. entry to 25 bar at exit and delivers 2000 kg/h of
It discharges the gases at a pressure of 1.15 bar, a water. Neglect the changes in volume, elevation,
temperature of 450°C and a velocity of 250 m/s. velocity and take specific volume of water as
Determine the work output of the turbine in kJ/kg 0.001045 m³/kg. [–1.393 kW]
182 Thermal Engineering

18. Exhaust gases from a diesel engine enter a turbine 23. Air at 50 bar and 300 K is flowing through a
of a turbocharger at 2 bar, 600°C and 50 m/s. pipeline. An evacuated and insulated cylinder
The gases leave the turbine at 1 bar, 270°C and of 0.1-m3 volume is connected to the pipeline
220 m/s. The inlet area is 7.1 cm2. The heat loss through a valve. The valve is opened and the
from the turbine may be assumed 15 kJ/kg of cylinder is filled with air till the pressure in the
gases. Determine cylinder reaches 50 bar and then the valve is
(a) Mass flow rate of exhaust gases, closed. Determine the temperature of air in the
(b) Power output from the turbine. cylinder at the end of the filling operation and
mass of air that is filled in the cylinder. Take g =
Assume properties of gases are same as that of
1.4. [420 K and 4.15 kg]
air. [(a) 0.02833 kg/s (b) 8.32 kW]
19. Air enters a control volume at 10 bar, 400 K and 24. Steam at a pressure of 20 bar and 500°C is
20 m/s through a flow area of 20 cm2. At the exit, flowing in a pipe. An evacuated tank is connected
the pressure is 6 bar, the temperature is 345.7 K, to this pipe through a valve. The valve is opened
and the velocity is 330.2 m/s. The air behaves as and the tank is filled with steam until the pressure
an ideal gas. For steady-state operation, determine is 20 bar and then the valve is closed. The process
takes place adiabatically and changes in kinetic
(a) the mass flow rate, in kg/s
and potential energy can be assumed negligible.
(b) the exit flow area, in cm2
Determine the temperature of steam in the tank at
[(a) 0.3484 kg/s, (b) 1.744 cm2] the end of the filling operation. [698.4°C]
20. Air enters a control volume operating at steady
25. An insulated cylinder with a volume of 0.1 m3
state at 1.2 bar, 300 K, and leaves at 12 bar,
contains air at 50 bar and 300 K. The valve of the
440 K with a volumetric flow rate of 1.3 m³/min.
cylinder is opened allowing some of air to escape
The work input to the control volume is 240 kJ
till the air pressure in the cylinder reduces to 30
per kg of air flowing. Neglecting potential and
bar. Calculate the temperature of air left in the
kinetic energy effects, determine the heat transfer
cylinder and mass of air that escaped from the
rate, in kW. [– 3 kW]
cylinder. [29.2 K, 1.775 kg]
21. Air expands through a turbine from 10 bar, 900 K
26. A rigid and insulated tank of 0.5-m3 volume con-
to 1 bar, 500 K. The inlet velocity is very small
tains nitrogen at 20 bar and 400 K. The valve
compared to exit velocity of 100 m/s. The turbine
is opened, allowing some of the nitrogen to es-
operates at steady state and devlops a power of
cape till pressure in the tank reduces to 4 bar.
3200 kW. Heat transfer between the turbine and
Assuming nitrogen as an ideal gas, calculate the
its surroundings and potential energy effects are
mass of nitrogen discharged from the tank.
negligible. Calculate the mass flow rate of air in
kg/s and the exit area in m². [5.75 kg]

[8.06 kg/s, 0.115m2] 27. A rigid vessel of 1-m3 capacity contains steam at
20 bar and 300°C. A valve on the tank is opened,
22. In a conference hall, comfort temperature condi-
allowing some steam to escape until the pressure
tions are maintained in winter by circulating hot
reduces to 2 bar, and the temperature of the tank
water through a piping system. The water enters
is maintained constant while steam discharges
the system at 3-bar pressure and 50°C tempera-
from the tank. Calculate the energy transferred as
ture with an enthalpy of 240 kJ/kg and leaves at
heat. [41.942 MJ]
2.5 bar, and 30°C with an enthalpy of 195 kJ/kg.
The exit of water is 15 m above its entry. The hot 28. A container has a capacity of 3-m3. It contains
water supplies 30 MJ/h heat to the conference air at 1.5 bar and 298 K. Additional air is now
hall. Calculate the quantity of water circulated pumped until its pressure rises to 30 bar and tem-
through the pipe per minute. Assume there is no perature to 333 K. Calculate the mass of air added
pump in the system and change in kinetic energy to the container. If the container is allowed to cool
is negligible. [11.15 kg/mim] to its initial temperature, determine the pressure
of air in the container. Assuming air as an ideal
First Law Applied to Flow Processes 183

gas, calculate the heat transfer and change in en- 35. Dry saturated steam at 800 kPa enters a steady
tropy during the cooling process. flow system and is expanded reversibly and iso-
[88.9 kg, 26.85 bar, – 2346.74 kJ, – 7.44 kJ/K] thermally to 500 kPa. There is no change of ki-
netic energy. The flow rate is 5.0 kg/s. Calculate
29. A vessel of 3-m3 capacity contains air at a pressure
the work done during expansion. [95.5 kJ/kg]
of 0.5 bar and a temperature of 25°C. Additional
air is now pumped into the system until the pres- 36. A turbine in a steam power plant operating under
sure rises to 30 bar and the temperature rises to steady state receives 1 kg/s superheated steam at
60°C. Determine the mass of air pumped in and 4 MPa and 300°C. The steam enters the turbine
express the quantity as a volume at a pressure of with the velocity of 10 m/s at an elevation of
1.02 bar and a temperature of 20°C. The gas is 5 m from the ground level. The turbine discharges
allowed to attain its temperature again at 25°C. the wet steam of 0.85 quality at 50 kPa pressure
Calculate the pressure in the vessel. Determine with a velocity of 50 m/s at an elevation of
the heat transferred and change in entropy of the 10 m above the ground level. The energy losses as
gas during the cooling process. heat from the turbine casing are estimated to be
10 kW. Estimate the power output of the turbine.
[Add. Mass = 92.41 kg, Equivalent volume
If the changes in kinetic energy and potential
= 76.2 m3, 26.84 bar, 2363 kJ, − 7.509 kJ/K]
energies are neglected, how much error will be
30. Steam at 0.6 MPa, 200°C enters an insulated introduced? [682.17 kW, 0.183%]
nozzle with a velocity of 50 m/s. It leaves at a
37. The power developed by a turbine in a certain
pressure of 5.5 bar and a velocity of 600 m/s.
steam power plant is 1200 kW. The heat supplied
Determine the final temperature, if the steam is
to the steam boiler is 3360 kJ/kg, the heat rejected
superheated in the final state and the quality, if it
by the system to cooling water in the condenser
is saturated. [x = 0.99]
is 2520 kJ/kg and the feed pump work required
31. Steam at 800 kPa, 300°C is throttled to 200 kPa. to pump the condensate back into the boiler is
Change in kinetic energy is negligible. Determine 6 kW. Calculate the steam flow through the cycle
the final temperature of the steam. [292.4°C] in kg/s. [5125.7 kg/h]
32. Steam enters a steam turbine at a pressure of 1 38. Steam at 5 MPa, 500°C enters a nozzle steadily
MPa, 300°C and a velocity of 50 m/s. The steam with a velocity of 80 m/s, and it leaves at 2 MPa
leaves the turbine at a pressure of 150 kPa and a and 400°C. The inlet area of the nozzle is 38 cm2,
velocity of 200 m/s. Determine the turbine output and heat is being lost at a rate of 8 kJ/s. Determine
per kg of steam flowing, assuming the process to (a) the mass flow rate of the steam, (b) the exit
be reversible adiabatic. [377.5 kJ/kg] velocity of the steam, and (c) the exit area of the
33. Consider the reversible adiabatic flow of steam nozzle.
through a nozzle. Steam enters a nozzle at 1 MPa, [(a) 4.43 kg/s, (b) 612.5 m/s, (c) 5.09 cm2]
300°C with a velocity of 30 m/s. The pressure of
39. Steam at 3 MPa and 400°C enters an adiabatic
steam at the nozzle exit is 0.3 MPa. Determine the
nozzle steadily with a velocity of 40 m/s and
exit velocity of steam leaving the nozzle.
leaves at 2.5 MPa and 300 m/s. Determine (a) the
[737 m/s] exit temperature and (b) the ratio of the inlet to
34. Steam is supplied to a turbine at 5.4 MPa, 300°C exit area A1/A2. [(a) 370°C, (b) 0.165]
expanded isentropically to an exhaust pressure at 40. Steam at 0.1 MPa and 150°C enters a diffuser
105.3 kPa. To what pressure must the incoming with a velocity of 180 m/s and leaves as saturated
steam be throttled in order to reduce the work vapour at 120°C with a velocity of 50 m/s. The
done per kilogram to two third of the work exit area of the diffuser is 0.08 m2. Determine
obtained without throttling? Assume the flow (i) the mass rate of steam (ii) the rate of heat
through the turbine is still reversible adiabatic transfer and (iii) the inlet area of the diffuser.
and exhaust pressure is also same. [510 kPa]
[(a) 4.484 kg/s, (b) 352 kW, (c) 0.054 m2]
184 Thermal Engineering

41. Steam flows steadily through adiabatic turbine rate of the steam is 25 kg/s. Determine (a) the
where the inlet conditions of the turbines are change in kinetic energy; (b) the power output
12.5 MPa, 500°C and 80 m/s and exit conditions and (c) the turbine inlet area.
are 10 kPa, 0.92 dry and 40 m/s. The mass-flow [(a) – 2.4 kJ/kg (b) 23.8 MW (c) 0.0080 m2]

Objective Questions
1. An adiabatic heat exchanger is used to heat cold (b) the system is in equilibrium with its sur-
water from 15°C entering at a rate of 5 kg/s by roundings
hot air at 90°C entering also at 5 kg/s. If the exit (c) properties change with time
temperature of air is 20°C, the exit temperature of Ê ∂v ˆ
water is (d) when Á ˜ is constant
Ë ∂t ¯
(a) 27°C (b) 85°C 6. A heat exchanger is used to heat cold water
(c) 32°C (d) 52°C from 15°C entering at a rate of 2 kg/s by hot air
2. A control volume refers to at 100°C entering at 3 kg/s. The heat exchanger
(a) a fixed region in space is not insulated and is losing heat at a rate of 40
(b) a fixed quantity of matter kJ/s. If the exit temperature of air is 20°C, the exit
(c) an isolated system temperature of water is
(d) a closed system (a) 44°C (b) 72°C
3. If all the variables of a flow system are indepen- (c) 49°C (d) 39°C
dent of time, the system is said to be 7. A nozzle
(a) closed flow (b) steady flow (a) accelerates the fluid flow
(c) constant flow (d) unsteady flow (b) decelerates the fluid low
4. Internal energy of a perfect gas depends on (c) does not affect the velocity of fluid
(a) temperature, specific heat and enthalpy (d) none of the above
(b) temperature, specific heat and entropy 8. During a throttling process
(c) temperature, specific heat and pressure (a) internal energy remains constant
(d) temperature only (b) enthalpy of fluid remains constant
5. Steady flow occurs when (c) pressure remains constant
(a) properties do not change with time (d) temperature remains constant

8. (b) 7. (a) 6. (a) 5. (a) 4. (d) 3. (b) 2. (a) 1. (c)


Answers
Second Law of Thermodynamics 185

6
Second Law of
Thermodynamics

Introduction
Energy has a quantity as well as quality. The first law of thermodynamics deals with energy as a property
and states that the quantity of energy must be conserved during a process. The second law of thermody-
namics deals with the quality of energy and asserts that the processes occur in a certain direction only.
The second law also recognizes that the mutual conversion of heat into work during a process is impossible,
while its opposite is possible. A process cannot take place unless it satisfies both the first and second laws
of thermodynamics.
Thermal reservoir, operation of heat engine, refrigerator, and heat pump are discussed first in this chapter
to explain the two statements of the second law. Reversible and irreversible processes are explained to give
insight to the performance of the devices. Then the Carnot cycle and absolute temperature are introduced.

Further, it is our experience that if a glass of hot


6.1 LIMITATIONS OF THE FIRST
LAW OF THERMODYNAMICS milk is left in the room, it eventually cools off after
a certain period. The amount of heat rejected by the
The first law of thermodynamics states that during milk is gained by the room air and the quantity of
any cyclic process, the net work transfer is always heat is balanced. Now let us consider the reverse
directly proportional to heat transfer and therefore, of the process, i.e., getting cold milk hotter as a
work and heat are mutually convertible one into result of heat transfer from the room air. But such a
another. A certain energy balance must be held process is impossible in nature.
when a system undergoes a process. Another common example is that an electrical
But the first law of thermodynamics does not work can be converted into heat energy through an
impose any restriction on the direction in which the electric heating element, but its reverse process—
process is feasible. As far as the first law is con- supplying an amount of heat to this heating
cerned, all heat transferred to a heat engine could element—will not generate any amount of electric
be converted to useful work. But we all know it is work.
not possible to convert net heat into work. Other common example is when a cycle rider
186 Thermal Engineering

One way

Hot Heat Actual process


milk
Surroundings

Fig. 6.3 Process occurs in a certain direction and not


(a) Hot milk losing heat to surroundings in reverse direction
Heat
It is clear from the above discussion that the
I=0 processes proceed in a certain direction only and
not in the reverse direction. The first law of ther-
modynamics does not place any restriction on the
(b) Transferring heat to heating element will not direction of process. Satisfying the first law does
generate electricity not ensure that the process will actually occur, be-
Fig. 6.1 cause it only keeps the account of energy in quan-
tity during any process. The second law of thermo-
stops his cycle by applying friction brakes. The dynamics is introduced to overcome the remedies
kinetic energy of the moving wheel is absorbed by of the first law of thermodynamics. The second law
brake blocks, whose temperature rises, and hence of thermodynamics takes into account the direction
the kinetic energy lost by the moving wheel is of process as well as quality of energy, and it states
converted into heat energy. The first law should be that in any process, high-grade energy can only be
equally satisfied, if the brake blocks are to cool off converted to low-grade energy. The energy cannot
and they give their energy back to the cycle wheel, be upgraded by its own. According to the second
causing it to rotate. However, it is never seen in law of thermodynamics, heat and work can be clas-
practice. sified as low and high-grade energy, respectively.
Other common experiences are the following: Therefore, net quantity of work can be converted in
(i) Heat always flow in the direction of decreas- any form of energy, but net quantity of heat cannot.
ing temperature.
(ii) Water always flows downwards.
(iii) A paddle wheel is operated by falling weight.
As it rotates, it stirs a fluid within an insu- It is a hypothetical body with an infinite heat capac-
lated container. The decrease in potential ity. A thermal reservoir can supply or absorb any
energy of weight is equal to increase of in- amount of heat without affecting its temperature.
ternal energy of the fluid mass. However, the For example, large bodies of water such as oceans,
reverse of the process—raising the weight rivers as well as the atmosphere can be considered
by decreasing internal energy of the fluid— as thermal reservoirs.
does not occur in nature. A two-phase system of fluids can also be treated
as a thermal reservoir as long as it does not return
to single phase, since the two-phase system can
absorb or release any amount of heat while main-
taining a constant temperature. Similarly, large
industrial furnaces can also be treated as thermal
reservoirs. They are maintained almost at constant
temperatures and are capable of transferring a large
Fig. 6.2 Supplying heat to a paddle wheel will not
quantity of heat energy in an isothermal manner.
cause it to rotate
Second Law of Thermodynamics 187

A reservoir that supplies heat energy is called


Energy source at TH
heat source and one that absorbs heat energy is
called heat sink.
Feed water
Boiler
HEAT ENGINE Superheated
steam

We know that the net amount of heat cannot be con- Pump Wout
Win Turbine
verted into work directly but conversion of heat into
work requires some special devices. Such devices
are called heat engines.
Condenser
In a broader sense, work-producing devices may Condensate Moist steam

be included in heat engines such as internal com-


bustion engines, steam and gas turbines. But these Energy Sink at TL

devices are operated in mechanical cycles, not in (a) Schematic diagram of a steam power plant
thermodynamic cycles, since the working fluid
does not undergo a complete cycle. High temperature
resevoir at TH
A heat engine is a device which operates in a
cycle. It receives the heat energy from a high-
temperature reservoir, converts some of the heat QH

energy into work and rejects the remaining heat to


a low-temperature reservoir. A steam power plant
best fits into the definition of a heat engine. The HE
schematic diagram of a basic steam power plant and Wnet
a heat engine are shown in Fig. 6.4. The working of
the heat engine can be illustrated as follows: QL

1. It receives heat QH from the high-temperature


reservoir (source) at TH.
Low temperature
2. It converts the part of heat supplied into resevoir TL
useful work as Wnet.
(b) Schematic diagram of a heat engine
3. It rejects the remaining heat QL to a low-
temperature reservoir (sink) at TL. Fig. 6.4

The net work output of a heat engine is the desired output is the net work Wnet, and the energy
difference between the total work done by the input is the heat supplied QH. Thermal efficiency is
turbine and work input to feed pump, etc. denoted by the symbol ‘hth’.
Wnet = Wout Win Thermal efficiency,
For any cyclic heat engine, the net work can Net work output W
hth = = net
also be obtained from the difference between heat Energy input as heat QH
supplied and heat rejection. Q - QL Q
or hth = H =1- L ...(6.2)
Wnet = QH QL ...(6.1) QH QH
Thermal efficiency is the per- Since QH and QL both are positive quantities,
formance-measuring parameter of a heat engine. thermal efficiency of the work producing devices is
In general, it is defined as the ratio of the desired always less than unity. The ordinary spark ignition
output to the energy input. For a heat engine, the internal combustion engine is about 30% efficient,
188 Thermal Engineering

while a diesel engine is about 32% efficient. Large Cold refrigerated space at TL
gas and steam power plants have a thermal efficien-
QL
cy of about 45%.
Low Evaporator Low pressure
pressure vapors
REFRIGERATOR low temp.
Win
refrigerant
Expansion
A refrigerator is a device, operating in a cycle, that Valve
High High
maintains a body at lower temperature than its pressure pressure
surroundings. A refrigerator extracts heat continu- liquid Condenser superheated
vapors
ously from a controlled space, and thus it is main- QH
tained at a lower temperature than its surroundings.
Surroundings at TH
The working fluid in the refrigerator is called the
refrigerant.
(a) Basic components of a refrigerator
The most frequently used refrigeration cycle is the
vapour compression cycle. The vapour compression
cycle is used in household refrigerators. Its basic Surroundings at TH
components are: a compressor, a condenser, an
expansion device and an evaporator as shown in
Fig. 6.5(a). QH

The refrigerant enters the compressor as low


pressure vapour and is compressed to high pressure
and temperature. In the condenser, the refrigerant Win
vapour passes through the long condenser coils,
where it rejects its latent heat to the surroundings.
The liquid refrigerant then enters a capillary tube
(an expansion device for small refrigerators), QL

where its pressure reduces drastically due to


Cold refrigerated
throttling effect and its temperature also reduces. space at TL
This low-temperature and low pressure refrigerant
then passes through the evaporator coils ( freezer), (b) Schematic diagram of a refrigerator
where it evaporates by absorbing heat from the Fig. 6.5
refrigerated space to keep it at low temperature.
The cycle completes as the refrigerant re-enters the Refrigerating Effect QL
(COP)R = = ...(6.3)
compressor. Work input Win
( ) From the heat energy balance for a cyclic device,
The performance of a refrigerator is measured in the work input Win can be expressed as
terms of the coefficient of performance (COP)R. It Win = QH QL
is defined as the ratio of the desired output (refrig- QL
Hence, (COP)R = ...(6.4)
erating effect) to the energy (work) input. QH - QL
Consider an amount of heat QL is removed from The value of (COP)R of a refrigerator may be
the refrigerated space at temperature TL. The work greater than unity. That is, the amount of heat
input to the compressor is Win and the heat rejected removed from the refrigerated space may be greater
at the condenser is QH. than the work input.
Second Law of Thermodynamics 189

QH
(COP)HP = ...(6.5)
Win
A heat pump is a device, operating in a cycle, that But QH = QL + Win
maintains a space at a higher temperature than the
Q + Win
surroundings. The heat pump supplies heat contin- (COP) HP = L
uously to the controlled space, and thus maintained Win
at a higher temperature than its surroundings. The QL
= 1+ = 1 + (COP)R ...(6.6)
refrigerator and heat pump are operated on the same Win
thermodynamic cycle, but they differ in their objec- For given values of QH and QL, the coefficient of
tives. The discharge of heat at higher temperature performance of a heat pump is always greater than
to a space is the objective for a heat pump. The heat the coefficient of performance of a refrigerator by
pump absorbs the heat from low-temperature sur- unity.
roundings and supplies it to a higher temperature
space at the cost of work input to the compressor. 6.6 STATEMENTS OF THE SECOND
If an ordinary refrigerator is placed in a window LAW OF THERMODYNAMICS
of a house with its evaporator open to the outside,
and its condenser located in the room. Then the
refrigerator acts as a heat pump and it supplies heat It relates the working of heat engines and is stated
to the room. as
Let an amount of heat QL be absorbed from the “It is impossible to construct an engine which
low temperature region and Win be the work input. while operating in a cycle, produces no other effect
Then the heat supply, QH to the room is the desired except to extract the heat from a single temperature
effect for a heat pump. reservoir and do equivalent amount of work”.
The coefficient of performance of heat pump is According to the Kelvin–Planck statement, a
expressed as heat engine cannot be 100 per cent efficient. In
Heat supplied practice, no heat engine can convert all heat sup-
(COP)HP =
Work input plied to useful work. The engine receives heat from
a high-temperature reservoir and it must reject some
amount of heat to a low-temperature reservoir. The
work done by a cyclic heat engine is the difference
between heat supplied and heat rejection.

It relates the working of the refrigerators and heat


pumps and it can be stated as,
“It is impossible for any device that operates in a
cycle and produce no effect other than the transfer
of heat energy from a lower temperature body to
higher temperature body”.
It is well-known tendency of heat energy to
flow from a hot body to a cold body, and it can-
not flow by itself from a low-temperature body to
a high-temperature body without addition of work.
Fig. 6.6
190 Thermal Engineering

Refrigerators and heat pumps transfer energy from a high temperature TH and a low temperature TL.
a low-temperature region to higher temperature re- The heat engine is receiving heat QH from the high-
gion at the cost of work input to their compressors. temperature reservoir at TH and it converts all heat
into net work Wnet. It does not reject any amount
of heat to the low-temperature reservoir, thus
At first sight, the two statements appear to be unre- violating Kelvin–Planck statement of the second
lated but it can easily be proved that these are two law.
parallel statements of the second law. Any device Let us assume the work produced by the engine
that violates one statement also leads to violation is supplied to a cyclic refrigerator, that removes
of the other statement. heat QL from a low-temperature reservoir at TL
Consider a heat engine (PMM2) and an actual and discharges heat in amount QH + QL to a high-
refrigerator as shown in Fig. 6.7, operating between temperature reservoir at TH. Now, if the refrigerator
and heat engine are grouped together, they
High-temperature reservoir
at TH
constitute a device, whose sole effect is to transfer
heat energy QL from the low temperature reservoir
to the high-temperature reservoir without any work
QH QH + QL
input from outside.
This device clearly violates the Clausius state-
W = QH
ment. Therefore, the violation of Kelvin–Planck
Rev.
heat
Actual statement leads to violation of Clausius statement.
refrigerator
engine
It can also be proved that violation of Clausius
statement leads to the violation of the Kelvin–
QL Planck statement.

Low-temperature
reservoir at TL
OF THE SECOND KIND
(a) Refrigerator powered by 100% efficient engine
According to the first law of thermodynamics, the
High-temperature reservoir heat and work are mutually convertible one into
at TH
another. If the total heat supplied is converted into
net work by any machine then its efficiency would
QL be 100 percent and it is called a perpetual motion
machine of the second kind (PMM2). A heat engine
W=0
(of Fig. 6.7a) is PMM2 which absorbs heat from a
Heat engine
+ single temperature reservoir and converts it com-
Refrigerator
pletely into work, leading to the violation of the
second law.
Consider a steam power plant without a
QL condenser as shown in Fig. 6.8. It is thought that
the condenser rejects more than one half of heat
Low temperature
reservoir at TL supplied as waste. Let such a steam power plant be
where all the heat transferred into the boiler will be
(b) The device (heat engine + refrigerator) which violates converted to work by turbine, and thus the power
Clausius’s statement plant will theoretically have 100 per cent efficiency.
Fig. 6.7 It is a perpetual motion machine of the second kind
Second Law of Thermodynamics 191

as engines and turbines develop maximum work


and work consuming devices, such as compressors,
blowers and pumps, etc., consume least work when
they are operated reversibly.
Let the initial state of the system be represented
by A and let the system be taken to the final state B
by the path APB as shown in Fig. 6.9. If the system
and surroundings both are restored to their initial
Fig. 6.8 states without any change in the universe, i.e., the
system returns by the same path then the process
which works in a cycle satisfying the first law, but APB is called a reversible process.
violates the second law. Therefore, it will not work,
p
because heat is a low-grade energy and cannot be A
completely converted into work.

A reversible process is defined as a process that B


once having taken place in a direction, can be re- v
versed without leaving any trace on either system Reversible process
or surroundings.
A reversible process is one which is proceeded
in such a way that at the conclusion of the process,
both system and surroundings may be restored to
their initial states without producing any change All processes occuring in nature are irreversible.
in the universe. This is possible only if the net When these processes are reversed, they cannot
transfer of heat and work between the system and return to their initial state of the system without
surroundings for original and reverse process is changing the surroundings. And when such pro-
zero. cesses are reversed, they always leave some mark
Once a cup of hot tea cools off due to heat on the surroundings or system. The irreversibility
transfer to its surroundings, it cannot heat up again of the processes may be caused due to
by recovering the lost heat from the surroundings (i) Mechanical or thermal irreversibility
without any external effect. If it could be possible (ii) Internal and external irreversibility
then the system and surroundings can be restored
Mechanical irreversibility is associated with fric-
to their original states and it would be a reversible
tion. When two bodies have relative movement, a
process.
frictional force opposes the motion at the interface
Actually reversible processes do not occur in
of these two bodies and some work is lost to over-
nature. They are idealization of actual processes.
come this frictional force. When the direction of
Reversible processes can be approximated by
motion is reversed, some work is further required
actual devices, but they can never be achieved.
to overcome the friction. Friction is also involved
Reversible processes are easy to analyse, since
between fluid and solid surfaces and between the
the system passes through a series of equilibrium
layers of fluid moving at different velocities during
states. Engineers are always interested in reversible
any process.
processes, because work-producing devices such
192 Thermal Engineering

Thermal irreversibility is associated with heat trans- Internal irreversibility is also caused due to mixing
fer due to finite temperature difference between a of different layers of fluid at different temperatures.
system and its surroundings. An amount of heat lost It is also due to free or unrestrained expansion of
from a system during a compression process cannot fluid as shown in Fig. 6.10(c).
be regained during an expansion process, thus caus- External irreversibility is associated with friction at
ing irreversibility. bearings and friction between atmospheric air and
All actual processes are internally irreversible. rotating members. The system absorbs some work
During an internally irreversible process, a system to overcome these frictional forces.
does not proceed through a series of equilibrium
A reversible
states. Consider an amount of gas in the piston–cyl-
process should possess the following properties:
inder device. If the piston is pushed rapidly, the gas
molecules near the piston face will not have suffi- 1. The process should not involve friction of
cient time to escape and they will pile up in front of any kind.
the piston face as shown in Fig. 6.10(a). This will 2. Heat transfer should not take place due to
raise the pressure near the piston face, while the gas finite temperature difference between the
in the other part will have lower pressure. Because system and surroundings.
of this higher value of pressure near the piston face, 3. There should not be a mixing of fluid layers
the compression process will require a large work at different temperatures.
input than required for a quasi-static process. The 4. There should not be free and unrestrained
non-uniformity of pressure will cause the process expansion.
irreversible. 5. The process must proceed through a series
of equilibria.

6.10 CARNOT CYCLE, OR CARNOT


(a) Fast compression (b) Fast expansion ENGINE

A french engineer, Sadi Carnot, in 1824 proposed


50 kPa 50 kPa
an engine which works on a reversible cycle. It is
theoretically a heat engine that converts the maxi-
mum amount of energy into mechanical work.
(c) Unstrained expansion Carnot showed that the efficiency of any engine
Fig. 6.10 - depends on the difference between the highest and
cesses lowest temperatures reached during one cycle. The
greater the temperature difference, higher the effi-
When the process is reversed and gas expands ciency.
rapidly, the gas molecules in the cylinder will not The Carnot cycle, also called reversible cycle,
be able to follow the piston as fast, thus creating a comprises of four reversible processes as given:
low-pressure region before the piston face as shown
1. Reversible isothermal expansion,
in Fig. 6.10(b). Because of this low-pressure value
at the piston face, the process delivers less work 2. Reversible adiabatic (isentropic) expansion,
than a corresponding reversible one. Thus, the work 3. Reversible isothermal compression, and
done by the gas during expansion is less than the 4. Reversible adiabatic (isentropic) compres-
work done by the surroundings on the gas during sion.
compression. Thus, surroundings have a net deficit The working of the engine is idealized with the
of work. following assumptions:
Second Law of Thermodynamics 193

p
1. The working substance for the Carnot engine
1
is a perfect gas. Q
H
2. The piston movement in the cylinder is 2
frictionless.
3. The walls of the cylinder and piston are
considered perfectly insulated. 4
3
4. The cylinder head is so arranged that it is Q
L
partially a very good conductor of heat and V

partially a perfect insulator. T

5. The heat supply and heat rejection are at 1 2


TH
constant temperatures.

TL
4 3

The cylinder head is a good conductor of heat,


and the heat source is brought in its contact. The 0 S1 S2 S
heat is added to the working medium at constant
Fig. 6.11
temperature TH. The gas expands slowly in an
isothermal manner from the state 1 to state 2. temperature (heat sink). The piston is pushed in-
Heat addition during the process, ward by an external force and the gas is compressed.
As the temperature of gas tends to rise, its heat is
ÊV ˆ
QH = p1 V1 ln Á 2 ˜ absorbed by the heat sink, keeping the temperature
Ë V1 ¯ of gas constant at TL. Therefore, heat is rejected in
The work done during the process, an isothermal manner from the state 3 to 4.
ÊV ˆ Heat rejection during the process,
W1–2 = QH = p1 V1 ln Á 2 ˜
Ë V1 ¯ ÊV ˆ ÊV ˆ
QL = p3 V3 ln Á 3 ˜ = m RTL ln Á 3 ˜ ...(6.9)
Ê V2 ˆ Ë V4 ¯ Ë V4 ¯
= m RTH ln Á ˜ ...(6.7)
Ë V1 ¯ The work done on the gas,
ÊV ˆ
The cylinder head is now made the insulator. The W3 – 4 = QL = p3 V3 ln Á 3 ˜
Ë V4 ¯
gas continues to expand slowly doing work on the
surroundings until the temperature drops to TL. The
piston movement is assumed frictionless, therefore, The contact of the heat sink with the cylinder head
the process is adiabatic and reversible (isentropic). is removed and the cylinder head is again made
Heat transfer, Q2–3 = 0 an insulator, and the gas is allowed to compress
isentropically from the state 4 to 1.
The work done on the gas,
p V - p2V2 m R (TH - TL )
Heat transfer during the process,
W2–3 = 3 3 = ...(6.8) Q4–1 = 0
g -1 g -1
The work done on the gas,
p1V1 - p4V4
The insulation from the cylinder head is removed W4–1 =
g -1
mR (TH - TL )
and it is made a good conductor of heat and
brought in contact of the thermal reservoir at low = ...(6.10)
g -1
194 Thermal Engineering

Engine
The thermal efficiency of any heat engine can be Since the Carnot cycle is reversible, therefore, its
calculated as direction can be reversed. In such a case, it can be
operated as a reversible heat pump or a reversible
Net work done Wnet
hth = = refrigerator. The cycle undergoes the same path
Heat suplied QH and processes, only their directions are reversed.
For a cyclic heat engine, Now the cycle requires the work input Wnet and it
Wnet = QH – QL transfers heat QL from the low temperature reser-
Q - QL Q voir to the high-temperature reservoir. Its p–V and
Therefore, hth = H =1- L T–S diagrams are shown in Fig. 6.12.
QH QH
Using Eqs. (6.7) and (6.9), we get p

m R TL ln (V3 /V4 )
1
QH
hth = 1 ...(6.11)
m R TH ln (V2 /V1 ) 4

Considering the isentropic process 2–3;


p2 V2g = p3V3g 2
g -1
TH ÊV ˆ QL 3
and = Á 3˜ ...(6.12)
TL Ë V2 ¯ V
0
Similarly, for the isentropic process 4 –1, (a) p-V diagram for a reversed Carnot cycle
p1 V1g = p4 V4g
T
g -1
TH ÊV ˆ
and = Á 4˜ ...(6.13) 1 QH 4
TL Ë V1 ¯ TH

Equating the above two equations , we get


V3 V4 TL
= 2 QL 3
V2 V1
V3 V2
or = ...(6.14) S
V4 V1 0

Substituting in Eq. (6.11), we get; (b) T-S diagram for a reversed Carnot cycle
TL Fig. 6.12
hrev = 1 ...(6.15)
TH
Equation (6.15) reveals that the Carnot cycle ef- CARNOT THEOREM
ficiency is independent of the working substance. It
only depends on source and sink temperatures. The The Carnot theorem is also called Carnot principle.
efficiency becomes maximum when the sink tem- It states that
perature TL approaches zero, but it is not possible (1) No engine can be more efficient than a re-
because it violates the Kelvin–Planck’s statement versible engine operating between the same
of the second law. Hence, any value of the efficien- two temperature reservoirs, or a Carnot
cy can be obtained when the temperature difference (reversible) engine among all engines is the
(TH TL) will have a finite value. most efficient.
Second Law of Thermodynamics 195

(2) The efficiency of all reversible heat engines High temperature Reservoir
operating between the same two temperature at TR
reservoirs are the same.
To prove the first principle, consider two heat QH QH
engines operating between the same two thermal
reservoirs as shown in Fig. 6.13(a). The engine
Wirrev Wrev Reversed
A is an irreversible engine and the engine B is a Carnot engine
Irreversible as
reversible engine. Each engine is supplied with an HE(A) refrigerator
equal amount of heat QH. Let the amount of work B
Wirrev – Wrev
Wrev produced by the reversible engine B be less
than the work Wirrev produced by an irreversible QL, irrev QL, rev
engine A, thus irrevesible engine will be more
efficient than the reversible engine.
Low-temperature reservoir
hA > hB or Wirrev > Wrev at TL

High-temperature reservoir
(c) Arrangement with irreversible heat engine drives
at TH reversed Carnot engine

QH QH
Wirrev – Wrev
Combined
HE +
refrigerator
Wirrev Wrev
Irrevesible Reversible
HE(A) HE(B)
QL, rev – QL, irrev

QL, irrev < Q L, rev Q L, rev


Low temperature
reservoir at TL
Low-temperature
reservoir at TL
(d) Equivalent combined arrangement
(a) A reversible and irreversible heat engine operating Fig. 6.13
between the same two reservoirs
Now let the reversible heat engine B be reversed
High-temperature reservoir at TH as shown in Fig. 6.13(b). The magnitude of heat
and work transfer will remain same, but their
direction will be reversed and it will operate as
QH QH
a refrigerator. The reversible refrigerator B will
Reversed require the work input of Wrev , a part of Wirrev and it
Wirrev Wrev
Irrevesible Carnot engine
HE(A) as refrigerator
will supply heat QH to a high-temperature reservoir
B equal to heat input to the irreversible heat engine A.
Thus, the net heat exchange from high temperature
QL, irrev QL, rev reservoir becomes zero and it can be eliminated
as shown in Fig. 6.13(c). The heat discharged by
Low-temperature reservoir at TL the refrigerator B may directly be supplied to the
irreversible engine A.
(b) Arrangement with an irreversible engine and
The irreversible engine A and the reversible re-
reversed cannot engine operating as a refrigerator
frigerator B together constitute a machine as shown
196 Thermal Engineering

in Fig. 6.13(d) that produces the net work Wirrev QH


Wrev, while exchanging the heat (QL, rev QL, irrev) = f (TH, TL) ...(6.17)
QL
with the low-temperature reservoir only. Thus, this
If some relationship is assigned between TH,
machine violates the Kelvin Planck’s statement of
TL, QH and QL, the resulting expression will be the
the second law. Therefore, our initial assumption
definition of the temperature scale.
that hirrev > hrev is wrong. Thus we can conclude
that no heat engine is more efficient than a revers- Let us consider three Carnot heat engines as
ible heat engine operating between the same two shown in Fig. 6.14. The engines A and C are sup-
thermal reservoirs. plied with the same amount of heat Q1 from the
source at the temperature T1. The engine C rejects
The second Carnot principle can also be proved
heat Q3 to the sink at the temperature T3. The en-
in a similar manner by replacing the irreversible en-
gine A rejects heat Q2, which is supplied to the en-
gine A by a reversible engine that is more efficient
gine B. The engine B rejects heat Q3 to the sink at
and thus delivers more work than the reversible en-
T3. The amount of heat rejected by engines B and
gine B. By the same reasoning, we can conclude
C is same. Since the engines A and B can be com-
that a machine which operates on a single-temper-
bined into one reversible heat engine, operating be-
ature reservoir and produces an equivalent amount
tween same temperature limits as the engine C, the
of work, thus violates the Kelvin Planck’s state-
combined engine will have the same efficiency as
ment of the second law. Therefore, we can state
the engine C has.
that no reversible heat engine can be more efficient
than another reversible engine operating between Applying Eq. (6.17) to all three engines sepa-
the same two temperature limits. rately;
Q1
= f (T1, T2),
6.13 THERMODYNAMIC Q2
Q2 Q1
= f(T2, T3), and = f(T1, T3)
Q3 Q3
A temperature scale that is independent of the prop-
erties of the substance and is used to measure the
temperature is called a thermodynamic temperature Thermal energy reservoir (source) at T1

scale. Q1
The thermal efficiency of any heat engine cycle Q1

receiving heat QH from a source at TH and rejecting Rev. HE WA


heat QL to a sink at TL is given by A
Rev. HE WC
QL Q2 C
hth = 1 = f (TH, TL) ...(6.16) T2
QH
Q2
It is necessary to maintain a temperature
difference in order to obtain the work output Rev. HE
B WB Q3
from an engine. Further, the efficiency of all
reversible heat engines operating between the same
Q3
temperature limits is same, because the efficiency
of any reversible heat engine is only a function
Thermal energy reservoir (sink) at T3
of source and sink temperatures. A functional
relationship between heat supply and heat rejection Fig. 6.14
can be expressed as
Second Law of Thermodynamics 197

The identity of the reversible heat engines gives identical (1 K ∫ 1°C) and the temperature on these
Q1 Q Q two scales is related as
= 1 ¥ 2
Q3 Q2 Q3 T(K) = T (°C) + 273.15
= f(T1, T2) ¥ f(T2, T3) ...(6.18)
But the ratio Q1/Q3 depends only on temperatures
T1 and T3, and is independent of the temperature T2.
This condition will be satisfied only, if the function A refrigerator that operates on reversed Carnot
f has the following form; Cycle is called a Carnot refrigerator.
QL 1
j (T1 ) j (T2 ) (COP)R = =
f(T1, T2) = and f(T2, T3) = QH - QL (QH / QL ) - 1
j (T2 ) j (T3 )
Since the performance of all reversible devices
In this form, j (T2) will be cancelled from the
operating between the same two temperature limits
product of f(T1, T2) and f(T2, T3) and the right-
is same, the COP of all reversible refrigerators will
hand side functional relation yields to
be equal. The ratio QH/QL is replaced by TH /TL.
Q1 j (T1 ) Thus, for a reversible refrigerator
= f (T1, T3) =
Q3 j (T3 )
1 TL
This relation is more specific for functional form (COP)R, rev = = ...(6.21)
(TH /TL ) - 1 TH - TL
of Q1/Q3 in terms of T1 and T3. Thus, for a reversible
heat engine operating between two temperatures TH Similarly, the COP of a heat pump is expressed
and TL; earlier by Eq. (6.6)
QH j (TH ) QH 1
= f (TH, TL) = ...(6.19) (COP)HP = =
QL j (TL ) QH - QL 1 - (QL / QH )
Several functions of j (T ) will satisfy this equa- For a reversible heat engine, replacing QL/QH by
tion and the choice is completely arbitrary. Lord TL/TH, we get
1 TH
Kelvin first proposed by taking j (T ) = T to a ther- (COP)HP, rev = = ...(6.22)
modynamic scale as 1 - (TL /TH ) TH - TL
j (TH ) T It is the inverse of efficiency of a reversible heat
= H ...(6.20) engine.
j (TL ) TL
Futher, Eq.(6.21) and (6.22) represent the highest
This temperature scale is called the Kelvin scale
coefficient of performance that a refrigerator or a
and temperature on this scale are called absolute
heat pump can attain, when operating between the
temperature. Thus, on the Kelvin scale, for any
same two temperature limits of TH and TL , because
reversible heat engine, the ratio of temperatures
they require least work input for a given output
associated with the source and sink is equal to the
effect.
ratio of energy supplied by the source and energy
The coefficient of performance of an actual and
rejected to the sink.
a reversible refrigerator can be compared as
At the International Conference on Weights and
Measures held in 1954, the triple point of water was < (COP)R,rev for an irreversible
assigned a value of 273.15 K. The magnitude of refrigerator
one kelvin is defined as 1/273.15 of the temperature (COP)R = (COP)R,rev for a reversible
interval between absolute zero and the triple- refrigerator ...(6.23)
point temperature of water. The magnitudes of > (COP)R,rev for an impossible
temperature units on Kelvin and Celsius scales are refrigerator.
198 Thermal Engineering

A similar relation can be established between


an actual and a reversible heat pump by replacing 1.0
(COP)R by (COP)HP in Eq. (6.23). TL
Slope = 2
TH
TH and TL

The efficiency of the Carnot engine is expressed by


Eq. (6.15) TL TH
TL (a) Effect of source temperature on efficiency
hrev = 1
TH
The efficiency of a reversible engine can be
improved by 1.0

(i) by increasing the temperature TH of the


high-temperature reservoir, or 1
Slope = –
TH
(ii) by decreasing the temperature TL of the low-
temperature reservoir.
Both possibilities are analysed below: TH TL

1. Keeping the temperature TL of sink constant, (b) Effect of sink temperature on efficiency
and differentiating above relation with re- Fig. 6.15
spect to TH, for maximizing the efficiency,
Ê ∂hrev ˆ TL Ê ∂hrev ˆ Ê ∂h ˆ
ÁË ∂T ˜¯ = ...(6.24) > Á rev ˜ ...(6.26)
H T =C TH 2 ÁË ∂T ˜¯ Ë ∂TH ¯ T
L L T H =C L =C
As TH increases, the efficiency hrev increas-
Since TH > TL, therefore, the efficiency
es and slope (∂hrev/∂TH) TL =C decreases as
can be increased effectively by decreasing
shown in Fig. 6.15(a).
TL than increasing TH.
2. Keeping the temperature TH of the source
constant, and differentiating the above rela- A heat engine operates on a Carnot
tion with respect to TL, for maximizing the cycle between source and sink temperatures of 337°C
efficiency and 57°C, respectively. If the heat engine receives 400 kJ
of heat from the source, find the efficiency, net work done
Ê ∂hrev ˆ 1 and heat rejected to the sink.
ÁË ∂T ˜¯ = - ...(6.25)
L T =C
TH
H
Solution
Thus, as TL decreases, the efficiency increas-
es linearly and (∂hrev/∂TL) TH= C remains con- Given A Carnot heat engine with
stant as shown in Fig. 6.15(b). TH = 337°C + 273 = 610 K,
TL = 57°C + 273 = 330 K,
Further, the rate of efficiency increase de-
QH = 400 kJ.
pends on the slope. The slope of the curve of
Fig. 6.15(b) remains constant with decrease To find
of TL. Thus for any reversible engine, com- (i) The efficiency of reversible heat engine,
paring two slopes
Second Law of Thermodynamics 199

(ii) The net workdone by Carnot engine, Example 6.3 An engine manufacturer claims that he
(iii) The heat rejected to sink. has developed an engine which will produce 210 kW of
power, while taking in 0.5 kg/min of fuel of calorific value
Analysis
of 42000 kJ/kg. Further, he states that the engine receives
(i) The thermal efficiency of a reversible heat engine heat at 527°C and rejects heat at a temperature of 77°C.
TL 330 Find if the claim of manufacturer is true or false.
hrev = 1 - =1- = 0.459
TH 610
Solution
(ii) Further, the thermal efficiency of a reversible
engine can also be given as Given Claim of a manufacturer to develop an engine
Work done W with
hrev = = TH = 527°C + 273 = 800 K
Heat supplied QH
TL = 77°C + 273 = 350 K
The work done, W = hrev ¥ QH = 0.459 ¥ 400 kJ
CV = 42000 kJ/kg
= 183.6 kJ Wnet = 210 kW
(iii) The heat rejected to sink, m = 0.5 kg/min
QL = QH W = 400 183.6 = 216.4 kJ
To find To justify the claim of the engine manufacturer
Example 6.2 A cyclic heat engine operates between Hint According to the Carnot principle, the reversible
a source temperature of 800°C and a sink temperature of engine has maximum efficiency.
30°C. What is the least rate of heat rejection per kW net
output of the engine? Analysis The efficiency of a reversible heat engine,
TL 350
Solution hrev = 1 - =1- = 0.5625
TH 800
Given A cyclic heat engine with Further, the efficiency of an actual heat engine can be
TH = 800°C + 273 = 1073 K expressed as
TL = 30°C + 273 = 303 K, Rate of work done Wnet
hAct = =
Wnet = 1 kW Rate of heat supply QH
The heat-supply rate,
To find The least rate of heat rejected from the engine.
QH = m ¥ CV = (0.5 kg/min) ¥ (42000 kJ/kg)
Hint The reversible engine rejects minimum heat. = 21000 kJ/min = 350 kW
Analysis The efficiency of a reversible heat engine, 210 kW
hAct = = 0.6
350 kW
TL 303 The effciency hAct of the actual engine is greater than
hrev = 1 - =1- = 0.7176
TH 1073 that of reversible heat engine. The claimed heat engine is
Further, the efficiency of a reversible heat engine can impossible to operate under given conditions. Hence the
be expressed as claim is baseless.

Rate of work done Wnet Example 6.4 An inventor claims to have invented
hrev = = a refrigeration machine operating between 23°C and
Rate of heat supply QH
27°C. It consumes 1 kW electrical power and gives
Heat-supplyra te, 21600 kJ of refrigeration effect in one hour. Comment on
Wnet 1 kW his claim.
QH = = = 1.393 kW
hrev 0.7176
Solution
The heat-rejection rate from the engine
Given Claim of a manufacturer to develop a refrigerat-
QL = QH - W = 1.393 − 1 = 0.393 kW ing machine with
200 Thermal Engineering

TH = 27°C + 273 = 300 K


TL = 23°C + 273 = 250 K
QL = 21600 kJ/h = 6 kW
Win = 1 kW
To find To compare the COP of the claimed refrigerator
with the reversible one.
Hint According to the Carnot principle, the reversible
refrigerator has maximum COP.
Analysis The COP of a reversible refrigerator,
TL 250
(COP)R, rev = = = 5.0
TH - TL 300 - 250
Further, the COP of an actual refrigerator can also be
expressed as
Heat removal rate Q 6 kW
(COP)R, Act = = L = = 6.0
Rate of work input Win 1 kW QL
or Win =
The (COP)R, Act of the actual refrigerator is greater (COP )R
than that of reversible one. The claimed refrigerator is
impossible to operate. 1.75 kW
= = 0.305 kW
5.73
A domestic food freezer maintains a
temperature of –15°C. The ambient temperature is 30°C. A Carnot cycle receives heat at 527°C,
The heat leaks into the freezer at 1.75 kJ/s. What is the causing an increase in entropy equal to 5 kJ/kg ◊ K. The
minimum power necessary to pump this heat out? engine delivers 2000 kJ/kg of work. Determine cycle
efficiency and lowest temperature in the cycle.
Solution
Solution
Given A domestic food freezer with
Given A Carnot cycle with
TH = 30°C + 273 = 303 K
TH = 527°C + 273 = 800 K,
TL = 15°C + 273 = 258 K
Ds = 5 kJ/kg ◊ K
QL = 1.75 kJ/s or kW.
w = 2000 kJ/kg
To find The minimum power required by food freezer.
To find
Hint Reversible refrigerator requires minimum power. (i) Carnot cycle efficiency, and
Analysis The COP of reversible refrigerator, (ii) Lowest temperature in the cycle.
TL Analysis The heat supply qH to heat engine can be
(COP)R, rev =
TH - TL obtained
258 qH = TH (Ds) = 800 ¥ 5
= = 5.73
303 - 258 = 4000 kJ/kg
Further, the COP of the refrigerator can also be Thermal efficiency of the Carnot cycle
expressed as w 2000
hth = = = 0.5
Heat removal rate Q qH 4000
(COP)R = = L
Rate of work input Win T
Further, hrev = 1 - L
TH
Second Law of Thermodynamics 201

To find The power required by the heat pump.


Analysis The COP of the Carnot heat pump,
TH 298
(COP)HP, rev = = = 19.86
TH - TL 298 - 283
The COP of the actual heat pump is
(COP)HP, act = 0.3 ¥ 19.86 = 5.96
The COP of an actual heat pump can also be expressed
as
Heat supply rate QH
(COP)HP, act = =
Fig. 6.17 Work input rate Win
QH 25 kW
TL or Win = = = 4.19 kW
or 0.5 = 1 -
800 (COP )HP , act 5.96
or TL = 800 ¥ 0.5 = 400 K = 123°C
A heat pump maintains a space at 22°C
A heat pump is used to maintain an au- on a day, when the outdoor air temperature is 0°C. The
ditorium hall at 25°C, when the atmospheric tempera- heating requirement of the space is 100,000 kJ/h and
ture is 10°C.The heat load of the hall is 1500 kJ/min. power consumed by the pump is 5 kW. Calculate the rate
Calculate the power required to run the heat pump, if its at which heat is extracted from the outside air and COP
COP is 30% of COP of the Carnot heat pump, working of the heat pump. Also, calculate the maximum COP.
between the same temperatures. Does the heat pump violate the second law of thermody-
namics?
Solution
Solution
Given A heat pump with
TH = 25°C + 273 = 298 K Given For a heat pump;
TL = 10°C + 273 = 283 K QH = 100,000 kJ/h = 27.78 kW,
QH = 1500 kJ/min = 25 kW TH = 22°C + 273 = 295 K
(COP)HP = 0.3 ¥ (COP)HP, rev TL = 0°C + 273 = 273 K,
Win = 5 kW

Fig. 6.19
202 Thermal Engineering

To find A reversed Carnot cycle operating as


(i) Heat extraction rate QL , a refrigerator has a refrigerating capacity of 100 kJ/s
(ii) Actual COP of heat pump, and while operating between temperature limits of 20°C
(iii) Maximum COP of heat pump. and 35°C. Determine
(a) Power input, and (b) COP.
Analysis
If the system is used for heating purpose only, find
(i) Heat extraction rate from outdoor air; its COP.
QL = QH – Win What would be its efficiency if it runs as an engine?
= 27.78 – 5 = 22.78 kW
(ii) The actual COP of heat pump,
QH 27.78 kW
(COP)HP = = = 5.56
Win 5 kW
(iii) Maximum COP of heat pump,
TH 295 K
(COP)HP, rev = =
TH - TL 295 K - 273 K
= 13.4
The (COP)HP, rev of the reversible heat pump is greater
than the (COP)HP of the actual heat pump. Thus it does
not violate the second law of thermodynamics.
Fig. 6.20
A heat pump delivers 2 kW of heat to a
room maintained at 25°C and receives heat from a res- Solution
ervoir at –10°C. If the actual coefficient of performance
is 50% of that of an ideal heat pump operating between Given A reversed Carnot cycle
the same temperature limits, what is the actual power re- QL = 100 kJ/s
quired in kW to run the heat pump? TH = 35°C + 273 = 308 K
TL = 20°C + 273 = 253 K
Solution
To find For refrigerator
Given A heat pump:
(i) Power input Win,
QH = 2 kW
(ii) (COP)R,
TH = 25°C = 298 K
(iii) For heat pump: (COP)HP , and
(COP)HP, act = 0.5(COP)HP, rev
(iv) For reversible engine: thermal efficiency.
TL = –10°C = 263 K
Analysis
To find Power input to heat pump.
(i) When the reversed Carnot cycle operates as a
Analysis The COP of the Carnot heat pump reversible refrigerator
TH 298 TL
(COP)HP, rev = = = 8.514 (COP)R, rev =
TH - TL 298 - 263 TH - TL
Actual COP of heat pump 253
(COP)HP, act = 0.5 ¥ 8.514 = 4.257 = = 4.6
308 - 253
Further, the COP of heat pump is given as
Refrigerating effect QL
Heat supply rate QH Further, (COP)R = =
(COP)HP, act = = Work input rate Win
Work input rate Win
QL 100 kJ/s
2 kW or Win = = = 21.74 kJ/s
Power input, Win =
4.257
= 0.47 kW ( )R
COP 4.6
= 21.74 kW
Second Law of Thermodynamics 203

(ii) When the reversed Carnot cycle operates as a Solution


reversible heat pump
Given Preservation of fish in a freezer
TH 308
(COP)HP, rev = = = 5.6 m = 300 kg,
TH - TL 308 - 253
Ti = 5°C
(iii) When the cycle operates as a reversible heat TH = 40°C + 273 = 313 K,
engine
TL = 2°C + 273 = 271 K,
TL 253 Cp = 4.182 kJ/kg ◊ K,
hrev = 1 - =1- = 0.178 or 17.8%
TH 308 hfg = 234.5 kJ/kg,
(COP)R = 0.6 ¥ (COP)R, rev ,
A substance executes a reversed
Dt = 10 h = 36000 s
Carnot cycle during which it receives 105.5 kJ/min of
heat. Determine the work required in kW, if the adiabatic To find The power required to remove heat from the
compression process triples the initial absolute tempera- fish freezer in 10 hours.
ture.
Analysis The (COP)R, rev of a reversible refrigerator,
Solution TL 271
(COP)R, rev = = = 6.452
TH - TL 313 - 271
Given A reversed Carnot cycle
The (COP)R of an actual refrigerator;
QL = 105.5 kJ/min,
(COP)R = 0.6 ¥ (COP)R, rev
TH = 3TL
= 0.6 ¥ 6.452 = 3.87
To find Work input in kW. The (COP)R of an actual refrigerator is expressed as
Assumption The reversed Carnot cycle as a reversible Refrigerating effect QL
(COP)R = =
refrigerator. Work input Win
The refrigeration effect from fish
Analysis The COP of a reversible refrigerator can be
QL = m ¥ [sensible heat + latent heat of fusion]
obtained as
= m ¥ [Cp (Ti TL ) + hfg ]
TL TL 1
(COP)R, rev = = = = 0.5 = 300 ¥ [4.182 ¥ {5 ( 2)} + 234.5]
TH - TL 3 TL - TL 2
= 300 ¥ 263.774 = 79132.2 kJ
Refrigerating effect QL
Further (COP)R = = Thus work required,
Power input Win QL 79132.2 kJ
Win = =
or
QL 105.5 (COP )R 3.87
Win = = = 211 kJ/min
( )R 0.5
COP = 20447.6 kJ
This heat is to be removed in a period of 10 hours,
= 3.51 kW
thus rate of work (power input);
(It can also be solved by assuming reversed Carnot
Win 20447.6 kJ
cycle as a reversible heat engine) Win = = = 0.567 kW
Dt 3600 s
300 kg of fish at 5°C is to be frozen A one-tonne (3.51 kW) reversible re-
at –2°C. The specific heat of fish above freezing point frigerator maintains a cold space at –13°C, while the
is 4.182 kJ/kg ◊ K and the latent heat of fusion is 234.5 surroundings are at 40°C. Determine the power con-
kJ/ kg. Freezing point is –2°C. A refrigerator is used sumed by the refrigerator.
for this purpose which rejects heat in the ambient at
If the same refrigerator is used as a freezer maintain-
40°C. The COP of the refrigerator is 60% of the COP
ing a space at a temperature of –23°C, while its sur-
of a Carnot refrigerator operating between the same
roundings remain at 40°C, how much refrigeration will
temperatures limits. How much power must be required
be produced? Assume same power consumption in both
to remove the heat in 10 hours?
cases.
204 Thermal Engineering

(ii) For a freezer operating between 250 K and 313 K,


TL2 250
(COP)R, rev = = = 3.968
TH - TL2 313 - 250
Refrigeration effect,
QL2 = (COP)R, rev ¥ Win
= 3.968 ¥ 0.7155 = 2.84 kW
Rating of refrigerator
2.84 kW
= = 0.808 TR
3.51 kW/TR

A reversible heat engine operates


between two reservoirs at 600°C and 40°C. The engine
drives a reversible refrigerator which operates between
the same 40°C reservoir and a reservoir at –18°C. The
heat transfer to the heat engine is 2100 kJ and there is
Fig. 6.21 Schematic of refrigerator or freezer a net work output of 370 kJ from the combined plant.
Evaluate the heat transfer to the refrigerator and the net
Solution heat transfer to the 40°C reservoir.
Given A 1-tonne refrigerator
Solution
RE = QL1 = 1 TR = 3.51 kW,
TL1 = 13°C = 260 K, Given A heat engine drives a refrigerator:
TH = 40°C = 313 K, T1 = 600 + 273 = 873 K
TL2 = 23°C = 250 K, T2 = 40 + 273 = 313 K
W1 = W2 = W (say). T3 = –18 + 273 = 255 K
Q1 = 2100 kJ
To find Wnet = 370 kJ
(i) The power consuption by a reversible refrigerator
when it operates between 260 K and 313 K, and To find
(ii) The refrigerartion effect when the refrigerator (i) Refrigeration effect, and
operates between 250 K and 313 K. (ii) Net heat transfer to reservoir at 40°C.
Analysis Analysis The efficiency of a reversible heat engine
(i) The (COP)R, rev of a reversible refrigerator, T2 313
hrev = 1 =1- = 0.642 = 64.2%
TL1 T1 873
(COP)R, rev = Further, thermal efficiency for any engine can also be
TH - TL1
expressed as
260
= = 4.905 Work done W
313 - 260 hth = = 1
The (COP)R of a refrigerator can also be expressed Heat supplied Q1
as Work done by engine,
Refrigerating effect QL W1 = h th Q1 = 0.642 ¥ 2100
(COP)R = = = 1348.0 kJ
Power input Win
But net work,
The power consumption,
Wnet = W1 W2 = 370
3.51 kW
Win = = 0.7155 kW or W2 = W1 370 = 1348 370 = 978 kJ
4.905
Second Law of Thermodynamics 205

Fig. 6.23 Heat engine and


Heat engine and

Heat rejected by engine, Q2 = 2100 1348 = 754 kJ W = 0.27Q1


COP of a reversible refrigerator, and Q2 = Q1 W = (1 0.27)Q1 = 0.73Q1
T3 255 Further, the COP of the heat pump is expressed as
(COP)R, rev = = = 4.4
T2 - T3 313 - 255 Heat supplied Q3
Referigeration effect Q3 (COP)HP = = =4
Further, (COP)R = = Work input W
Work input W2
Heat supplied,
Refrigeration effect,
Q3 = (COP)HP ¥ W = 4 ¥ 0.27Q1
Q3 = (COP)R ¥ W2 = 4.4 ¥ 978
= 1.08Q1
= 4300 kJ
Since the circulating water receives heat from the
Heat rejected by the refrigerator to the 40°C reservoir, engine as rejected heat Q2 and heat pump as Q3, thus the
Q4 = Q3 + W2 = 4300 + 978 = 5278 kJ total heat supplied to the circulating water
Hence total heat supplied to the reservoir at 40°C
QTotal = Q2 + Q3 = 0.73Q1 + 1.08Q1 = 1.81Q1
Qtotal = Q2 + Q4 = 754 + 5278 = 6032 kJ
Hence, the ratio of heat supplied to the circulating
A heat engine is used to drive a heat water and heat supplied to engine
pump. The heat transfers from heat pump and heat engine QTotal 1.81Q1
is used to heat the water circulating through a building. = = 1.81
Q1 Q1
The effciency of the heat engine is 27% and the COP of
the heat pump is 4. Evaluate the ratio of heat transfer to A reversible refrigerator is used in an
the circulating water to heat transfer to the engine. ice-making plant. It absorbs heat energy from water at
0°C and rejects heat energy to the ambient at 27°C. If
Solution a reversible heat engine operates between a source at
Given A heat engine drives a heat pump: 500°C and the same ambient at 27°C is used to drive the
heat pump, calculate
hth = 0.27, (COP)HP = 4.
(a) Energy removed as a heat from the water by the
To find The ratio of heat supplied to circulating water
refrigerator for each kilojoule of energy taken in
to heat supplied to engine.
by the engine
Analysis The efficiency of a reversible heat engine (b) Energy rejected to the ambient at 27°C for each
W kilojoule of energy absorbed by the engine from
hth = = 0.27 the source.
Q1
206 Thermal Engineering

Solution COP of a reversible refrigerator,


TR 273
Given A reversible refrigerator driven by a reversible (COP)R, rev = = = 10.11
TL - TR 300 - 273
heat engine;
For a reversible heat engine: Since the engine directly supplies work to the revers-
ible refrigerator, thus
TH = 500°C + 273 = 773 K
TL = 27°C + 273 = 300 K Win = WEngine = 0.612 kJ
QH = 1 kJ The cooling effect of the reversible refrigerator
For a reversible refrigerator: RE = (COP)R, rev ¥ Win = 10.11 ¥ 0.612
TR = 0°C + 273 = 273 K, = 6.188 kJ
TL = 27°C + 273 = 300 K Heat rejected by the refrigerator to the 27°C reservoir,
Win = WEngine QR = RE + Win = 6.188 + 0.612 = 6.8 kJ
The total heat rejected to the ambient by the reversible
To find refrigerator and the reversible heat engine for each kJ
(i) Refrigerating effect in a reversible refrigerator for input to engine;
each kJ of heat input in reversible heat engine. QTotal = QR + QL = 6.8 + 0.388 = 7.188 kJ
(ii) Heat energy rejected to the ambient by a reversible
refrigerator and a reversible heat engine for each Example 6.17 A reversible refrigerator is used to
kJ of heat input to engine. maintain a space at the temperature of 0°C, when it re-
jects heat to the surroundings at 27°C. If the heat remov-
Analysis The efficiency of a reversible heat engine al rate from the refrigerator is 90 MJ/h, determine the
TL 300 COP of the system.
hrev = 1 - = 1- = 0.612 = 61.2%
TH 773 If the required input to run the refrigerator is supplied
For each kJ of heat input to the reversible heat engine by a reversible engine which receives heat at 400°C and
Work done WEngine rejects heat to the surroundings, determine the overall
hrev = = COP of the system.
Heat supplied QH
Work done by engine, Solution
WEngine = hrev ¥ QH = 0.612 ¥ 1 = 0.612 kJ
Given For a reversible refrigerator,
Heat rejected by the engine
TL = 0°C = 273 K,
QL = QH WEngine = 1 0.612 = 0.388 kJ
TH = 27°C = 300 K
and QL = 90 MJ/h = 25 kW
For a reversible engine,
T1 = 400°C = 673 K
To find
(i) (COP)R, and
(ii) Overall COP of combined system.
Analysis For a reversible refrigerator,
TL 273
(COP)R, rev = = = 10.11
TH - TL 300 - 273
Heat removal rate QL
Further; (COP)R = =
Power input Win
25
or Win = = 2.473 kW
Fig. 6.24 Schematic of reversible heat engine and re- 10.11
versible refrigerator together
Second Law of Thermodynamics 207

T1 = 637 K, source Space TL = 273 K High-temperature High-temperature


reservoir at reservoir at
. TH = 673 K TH = 373 K
QH . 1 2
QL = 90 MJ

QH = 12000 kW QH = 25000 kW
1 2

.
W
HE R
HE HE
W1 W2

QL QL
1 2

Surroundings at TH = 300 K
Low-temperature Low-temperature
Fig. 6.25 reservoir reservoir
TL = 293 K TL = 293 K
The power input Win of 2.473 kW is produced by (a) Operation 1 (b) Operation 2
a reversible heat engine operating between 673 K and
300 K. The efficiency of a reversible engine Fig. 6.26
T 300 Operation 2
hrev = 1 - H = 1 - = 0.5542
T1 673 TH2 = 100°C + 273 = 373 K
Power produced W TL = 20°C + 273 = 293 K
Further, efficiency, hrev = =
Heat supply rate QH QH2 = 25000 kW
2.473
Therefore, QH = = 4.462 kW To find Operation in which the engine will develop
0.5542
more power.
The overall COP of the system
Sole effect from the system Analysis
(COP)Overall =
Energy input to the system Operation 1
Refrigeration effect QL 25 The efficiency of a reversible heat engine
= = = T 293
Heat input rate QH 4.462 h1 = 1 - L = 1 - = 0.5646
= 5.60 TH1 673
Power developed by the engine in the operation 1,
Example 6.18 A reversible heat engine operates P 1 = h 1 ¥ QH
in two environments. In the first operation, it draws = 0.5646 ¥ 12000 = 6775.63 kW
12000 kW from a source at 400 °C and in the second
Operation 2
operation, it draws 25000 kW from a source at 100 °C.
In both operations, the engine rejects heat to a thermal The efficiency of a reversible heat engine
sink at 20 °C. Detemine the operation in which the engine TL 293
h2 = 1 - =1- = 0.2144
delivers more power. TH 2 373
Power developed by the engine in the operation 2,
Solution
P1 = h2 ¥ QH
Given A reversible heat engine operates with two = 0.2144 ¥ 25000 = 5361.93 kW
sources separately: The engine will develop more power in the operation
Operation 1 All engines develop more power when heat is supplied
TH1 = 400°C + 273 = 673 K at higher temperature. Heat energy at higher temperature
TL = 20°C + 273 = 293 K has more work capability.
QH = 12000 kW
1
208 Thermal Engineering

A household refrigerator maintains a QL 300 000


or Win = =
space at a temperature of 0°C. Every time the door is (COP ) Act 2.275
opened, warm material is placed inside, introducing an = 131868.13 kJ
average 400 kJ of heat, but making only a small change
1 electric unit = 1 kWh = 3600 kJ
in temperature of the refrigerator. The door is opened
Thus, the power consumed by the refrigerator
25 times a day and the refrigerator operates at 25% of
ideal COP. The cost of work is Rs.3.50 per kWh. What 131868.13
= = 36.63 kWh
is the monthly bill of this refrigerator? The atmospheric 3600
temperature is at 30°C. The monthly bill of the refrigerator
= Power consumption ¥ Unit cost
Solution = 36.63 ¥ Rs. 3.50 = Rs. 128.20
Given Working of a household refrigerator
Two Carnot engines are working in
TH = 30°C + 273 = 303 K series between a source and a sink. The first engine re-
TL = 0°C + 273 = 273 K ceives heat from a reservoir at a temperature of 1000 K
(COP)Act = 0.25(COP)R,rev and rejects the waste heat to another reservoir at the
Unit cost = Rs 3.50/kWh temperature T2. The second heat engine receives the heat
Refrigeration effect, energy rejected by the first engine. It converts some of
Q2 = 400 (kJ/time) ¥ 25 (times/day) ¥ 30 (days) energy into useful work and rejects the rest to a reservoir
= 300 000 kJ at temperature of 300 K.
(a) If both engines deliver equal power, determine
To find The monthly bill of the refrigerator
the efficiency of each engine.
Analysis The COP of the reversible refrigerator (b) If thermal efficiency of both engines are same,
TL 273 determine the intermediate temperature.
(COP)R,rev = = = 9.1
TH - TL 303 - 273
Solution
Actual COP of the refrigerator,
(COP)Act = 0.25 ¥ 9.1 = 2.275 Given Two Carnot heat engines in series
Heat removed QL Source temperature, T1 = 1000 K
Further, (COP)Act = =
Work input Win Sink temperature, T3 = 300K
Reservoir at 1000 K

Q1

Rev. W1
HE 1

Q2
T2

Q2

Rev. W2
HE 2

Q3

Reservoir at 300 K
Fig. 6.27
Second Law of Thermodynamics 209

To find Equating (i) and (ii), we get


(i) If W1 = W2 then efficiency is h1 and h2. T1 - T2 T - T3
= 2
(ii) If h1 = h2 then intermediate temperature is T2. T1 T2
After cross multiplication, we get
Analysis The efficiencies of the reversible engines HE1
and HE2 are given by T1T2 T22 = T1T2 T1T3
T - T2 W1 W1 or T2 = T1 T3 = 1000 ¥ 300
h1 = 1 = = ...(i)
T1 Q1 Q2 + W1
= 547.72 K
T2 - T3 W2
and h2 = = ...(ii) If both reversible engines are in series and have
T2 Q2 equal efficiency then the intermediate tempera-
(i) If both engines deliver equal work ture is the geometric mean of source and sink
From Eq. (i), we get temperature.
T1 - T2
W1 = (Q2 + W1) ¥ Example 6.21
T1
(a) Two reversible heat engines operate in series be-
È T1 - T2 ˘ È T - T2 ˘ tween a high-temperature and low-temperatutre
W1 Í1 - ˙ = Q2 Í 1 ˙
Î T1 ˚ Î T1 ˚ reservoirs, T1 and T2. Engine A receives 500 kJ of
heat and it rejects heat to the engine B, which in
T2 È T1 - T2 ˘
or W1 ¥ = Q2 Í ˙ turn rejects heat to a low-temperature reservoir.
T1 Î T1 ˚ The high-temperature reservoir supplies heat to
È T - T2 ˘ the engine A at 1000 K, while the low-temper-
or W1 = Q2 Í 1 ˙ ...(iii) ature reservoir receives heat at 400 K and both
Î T2 ˚
engines are equally efficient. Determine (i) tem-
and from Eq. (ii), we get
perature of heat rejection by the engine A, (ii) the
È T - T3 ˘ work done by engines A and B, and (iii) the heat
W2 = Q2 Í 2 ˙ ...(iv)
Î T2 ˚ rejected by the engine B.
(b) Use the above arrangements of equal efficient
Equating Eq. (iii) and Eq. (iv), we get
engine and consider the work done by the en-
T1 – T2 = T2 – T3
gine B is 527.7 kJ, the low-temperature reservoir
or 2T2 = T1 + T3
is at 305 K and heat received by the engine A is
T + T2
or T2 = 1 2110 kJ. Determine (i) the thermal efficiency of
2 each engine, (ii) the temperature of heat supply to
1000 + 300 the engine A, (iii) the work done by the engine A,
or T2 = = 650 K
2 (iv) the heat rejected by the engine B, and (v) the
Hence when both reversible engines are in series temperature of heat addition to the engine B.
and have equal work output, the intermediate
temperature is the arithmetic mean of source and Solution
sink temperature. Given Two equal efficient engines are in series ar-
Efficiency of the first engine rangement as shown in Fig. 6.29.
T1 - T2 1000 - 650 Source temperature T1 = 1000 K,
h1 = = = 0.35
T1 1000 Sink temperature T2 = 400 K,
Efficiency of the second engine Heat supply to the engine A Q1 = 500 kJ,
T2 - T3 650 - 300 hA = h B
h2 = = = 0.538
T2 650
To find
(ii) If both reversible engines have equal thermal
(i) When both engines are equally efficient then
efficiency
210 Thermal Engineering

Reservoir at T1 (ii) The work done by the reversible engine A can be


calculated with the help of effciency as
Q1 T - TB 1000 - 632.45
hA = 1 = = 0.3675
T1 1000
Further, the effciency of the engine A is also
HEA WA
expressed as
W
hA = A
TB
QB Q1
or WA = hA ¥ Q1
QB
= 0.3675 ¥ 500 kJ = 183.775 kJ
HEB WB The heat rejected by the engine A:
QB = Q1 WA = 500 183.775
= 316.225 kJ
Since both engines have equal effiencies, thus
Q2
hB = hA = 0.3675
Reservoir at T2 Further, the effciency of the engine B can also be
expressed as
Schematic of reversible heat engines in series
W
hB = B
(ii) Temperature TB of heat rejection by engine A, QB
(iii) The work done by engine A and engine B, Thus, the work delivered by the engine B:
(iv) The heat rejected by engine B. WB = hB ¥ QB = 0.3675 ¥ 316.225 kJ
= 116.2 kJ
Assumptions
(iii) The heat rejected by the engine B
(i) Intermediate temperature between two reversible Q2 = QB WB = 316.225 116.2
engines is TB. = 200 kJ
(ii) The heat rejected by the engine A and received by (b) With arrangement of engines as in part (a),
the engine B is QB. Work done by the engine B
Analysis WB = 527.5 kJ
Sink temperature, T2 = 305 K
(i) The efficiency of the reversible engines HEA and
HEB are given by: Heat supplied to the engine A
Q1 = 2110 kJ
T1 - TB W A WA
hA = = = ...(i) h A = hB
T1 Q1 QB + W A
TB - T2 WB To find
and hB = = ...(ii) (i) thermal efficiency of each engine,
TB QB
If both reversible engines have equal thermal (ii) temperature of heat supply to the engine A,
efficiencies: (iii) work done by the engine A,
Equating (i) and (ii) in terms of temperature only, (iv) heat rejected by the engine B, and
T1 - TB T - T2 (v) temperature of heat addition to the engine B.
= B
T1 TB Analysis
After cross multiplication, we get (i) Since both engines have equal efficiencies, thus
T1TB – TB2 = T1TB – T1T2 Q - QB WB
hA = 1 =
or TB = T1 T2 = 1000 ¥ 400 Q1 QB
2110 - QB 527.5
= 632.45 K or =
2110 QB
Second Law of Thermodynamics 211

or 2110QB – Q B2 = 1113025
2
Q B – 2110QB + 1113025 = 0
It is a quadratic equation and its solution is
QB = 1055 kJ
The thermal efficiency of each engine is
2110 - 1055
hA =
2110
= 0.5 or 50%
(ii) The temperature of heat supply to the engine A
The effciency of either reversible engine is
expressed as
T - T2
hB = B
TB
TB - 305
or 0.5 =
TB
The intermediate temperature, TB = 710 K
Further, the efficiency for the engine A is Fig. 6.30
T - TB
hA = 1 or Q3 = (COP)A Q2 – (COP)A Q3
T1
1 + (COP ) A
T1 - 710 It gives Q2 = ¥ Q3 ...(i)
or 0.5 = (COP ) A
T1
The temperature of heat supply, 1 + (COP ) B
Similarily, Q1 = ¥ Q2 ...(ii)
T1 = 1420 K (COP ) B
(iii) Now the work done by the engine A; Using Q2 from Eq. (i) into Eq. (ii), we get
WA = Q1 – QB = 2110 – 1055 1 + (COP ) B 1 + (COP ) A
Q1 = ¥ ¥ Q3
= 1055 kJ (COP ) B (COP ) A
(iv) The heat rejected by the engine B:
Q1 [1 + (COP ) B ] [1 + (COP ) A ]
Q2 = QB – WB = 1055 – 527.5 \ = ...(iii)
Q3 (COP ) A ¥ (COP ) B
= 527.5 kJ
(v) Intermediate temperature For a Carnot refrgerator working between low-and
TB = 710 K high-temperature reservoirs
Q3 Q3
(COP)R, overall = =
Two Carnot refrigerators A and B are W A + WB (Q2 - Q3 ) + (Q1 - Q2 )
arranged in series. Prove that the overall COP of the
combined system is given by Q3 1
= =
(COP ) A ¥ (COP ) B (Q1 - Q3 ) Q 1
(COP)R, Overall = -1
1 + (COP ) A + (COP ) B Q3
Q
where (COP)A and (COP)B are the coefficient of Substituting the value of 1 from Eq. (iii), we get
Q3
performance of Carnot refrigerator A and B, respectively.
1
(COP)R, overall =
Solution Two Carnot refrigerators A and B are in [1 + (COP ) B ][1 + (COP ) A ] 1
-
series as shown in Fig. 6.30. (COP ) A ¥ (COP ) B
(COP)A = 3 =
Q Q3 (COP ) A ¥ (COP ) B
=
W A Q2 - Q3 [1 + (COP ) B ][1 + (COP ) A ] - (COP ) A ¥ (COP ) B
212 Thermal Engineering

(COP ) A ¥ (COP ) B (1000 – T2) ¥ (Q2 + W1) = 1000 W1


or (COP)R, overall = or (1000 – T2) Q2 = W1 T2
1 + (COP ) A + (COP ) B
W1 T2
which is the required expression. or Q2 = ...(ii)
1000 - T2
Three Carnot engines C1, C2 and C3 For the Carnot engine C2;
operate in series between two heat reservoirs, which T2 - T3 W2
h2 = = ...(iii)
are at temperatures of 1000 K and 400 K. Calculate the T2 Q2
temperature of the intermediate resrvoirs if the amount Q2 (T2 - T3 )
of work produced by these engines in the proportion of or W2 =
T2
5:4:3.
Using the value of Q2 from Eqn. (ii), we get
W1 T2 (T2 - T3 )
W2 =
(1000 - T2 ) T2
W2 T2 - T3
or = ...(iv)
W1 1000 - T2
Given that
W1 5 W2 4
= or =
W2 4 W1 5
Using in Eq. (iv), we get
4 ¥ (1000 T2) = 5 ¥ (T2 T3)
4000 – 4T2 = 5T2 – 5T3
4000 + 5T3
or T2 = ...(v)
9
Again Q2 = Q3 + W2
Substituting in Eq. (iii), we get
W2 T2 - T3
=
W2 + Q3 T2
or W2 T2 = W2 (T2 T3) + Q3 (T2 T3)
Q3 (T2 - T3 )
or W2 = ...(vi)
Fig. 6.31 T3
Further, the efficiency of the third engine C3;
Solution T - T4 W3
h3 = 3 =
Given Three Carnot engines C1, C2 and C3; T3 Q3
Source temperature T1 = 1000 K Q3 (T3 - T4 )
Sink temperature T4 = 400 K or W3 =
T3
W1 : W2 : W3 = 5 : 4 : 3 W2 4
Given that = or 3 W2 = 4 W3
To find Intermediate temperatures T2, T3. W3 3
Using the values of W2 and W3 in the above equation;
Analysis The efficiency of the Carnot engine C1:
Q3 (T2 - T3 ) Q3 (T3 - T4 )
T - T2 W1 3¥ =4¥
h1 = 1 = T3 T3
T1 Q1
or 3 ¥ (T2 T3) = 4 ¥ (T3 T4)
But Q1 = Q2 + W1
Using the value of T2 from Eq. (v) and T4 = 400 K;
1000 - T2 W1
Using the above h1 = = ...(i) 4000 + 5T3
1000 Q2 + W1 3¥ - 3T3 = 4 ¥ (T3 400)
9
Second Law of Thermodynamics 213

or 4000 + 5T3 9T3 = 12T3 4800 To find Minimum power input to heat pump.
or 16T3 = 8800
Analysis The COP of the reversible heat pump
or T3 = 550 K TH 293
The intermediate temperature T2 between engine C1 (COP)HP, rev = = = 9.766
TH - TL 293 - 263
and C2 from Eq. (v); The COP of a heat pump is also expressed as
4000 + 5 ¥ 550 Q
T2 = = 750 K (COP)HP = H
9 Win
A house is maintained at a tempera- QH
or Win =
ture of 20°C by means of a heat pump in the winter by (COP )HP
pumping heat from atmosphere. Heat losses through the where QH = 0.65 kJ/K ¥ DT
walls of the house are estimated at 0.65 kJ/K tempera- = 0.65 ¥ (293 263)
ture difference between inside of the house and outside = 19.5 kJ
atmosphere. Using the values of COP and QH ;
(a) If the atmospheric temperature is –10°C, what 19.5 kJ
Win = = 1.996 kW
is the minimum power required to drive the 9.766
heat pump? (ii) For summer:
(b) It is proposed to use the same heat pump to Given House temperature, TL¢ = 293 K
cool the house in summer. If the same power
Power input to heat pump, Win = 1.996 kW
is supplied to heat pump, what is the maximum
permissible atmospheric temperature? To find Surrounding temperature TH.
Analysis The COP of a reversible refrigerator:
Solution
TL¢ 293
(COP)R, rev = =
Given TH¢ - TL¢ TH¢ - 293
(i) For winter The refrigeration effect, QL¢ = 0.65 ¥ (TH¢ – 293)
House temperature TH = 20°C = 293 K, Q¢ 0.65 ¥ (TH¢ - 293)
(COP)R = L =
Atmospheric temperature TL = –10°C = 263 K, Win 1.996
Heat load of the house QH = 0.65 kJ/K Equating the two equations for COP of a refrigerator
293 0.65 ¥ (TH¢ - 293)
=
House at House at (TH¢ - 293) 1.996
TH = 20°C TL = 20°C
or (TH¢ – 293)2 = 900
or T H¢ = 293 + 30 = 323 K = 50°C
QH Q¢L
A working fluid undergoes a Carnot
cycle of operation. The upper absolute temperature of the
fluid is q1 and the lower absolute temperature is q2. The
HP W HP W
amount of heat taken in and rejected by the working fluids
are H1 and H2 , respectively. On account of losses of heat
due to conduction etc. the heat source temperature T1 is
higher than q1 and the heat sink temperature T2 is lower
Q¢H
QL than q2.
If T1 = q1 + KH1, and T2 = q2 KH2
Atmosphere, TL = – 10°C Atmosphere, TH
where K is some constant for both the equations, show
(a) Schematic of heat pump (b) Schematic of heat pump that the efficiency of the plant is given by
supplying heat to a house extracting heat from a
T2
house h = 1-
T1 - 2 KH1
Fig. 6.32
214 Thermal Engineering

Solution The schematic for the given arrangement A reversed Carnot cycle operates as
is shown in Fig. 6.33. either a refrigerator or a heat pump. In either case, the
power input is 20.8 kW. Calculate the quantity of heat
extracted from the cold body for either type of machine. In
both cases, 3500 kJ/min heat is delivered by the machine.
In case of the refrigerator, the heat is transferred to the
surroundings while in case of the heat pump, the space
is to be heated. What are their respective coefficient of
performances?
If the temperature of the cold body is 0°C for the re-
frigerator and 5°C for the heat pump, what are the re-
spective temperatures of the surrounding for refrigerator
and heated space for heat pump? What reduction in heat-
rejection temperatures would be achieved by doubling
the COP for the same cold body temperature?

Solution
Fig. 6.33 reversible heat engine
Given A reversed Carnot cycle with
The efficiency of a reversible engine is given by Win = 20.8 kW
H q Q1 = 3500 kJ/min = 58.33 kW
hrev = 1 - 2 = 1 - 2 ...(i)
H1 q1 T2 = 0°C + 273 = 273 K
By absolute temperature scale T3 = 5°C + 273 = 278 K
q1 H
= 1 ...(ii) To find
q2 H2
From the given relation, (i) (COP)R, rev ,
T -q (ii) (COP)HP, rev,
H1 = 1 1 (iii) Temperature at which reversible refrigerator
K
q 2 - T2 rejects heat,
and H2 = (iv) Temperature at which reversible heat pump
K
Using values of H1 and H2 in Eq. (ii), we get supply heat,
q1 T -q (v) Temperature at which refrigerator rejects heat, if
= 1 1 its (COP)R, rev is doubled.
q2 q 2 - T2
or q1 q2 q1 T2 = q2 T1 q1 q2 Analysis For a reversed Carnot cycle, the heat extracted
or 2q1q2 q2 T1 = q1 T2 from low temperature is
or q2 (2q1 T1) = q1 T2 Q2 = Q1 - Win = 58.33 20.8
q1 T2 = 37.53 kW
or q2 =
2q1 - T1 Q2 37.53 kW
(i) (COP)R, rev = = = 1.804
Using q1 = T1 KH1 in the denominator above, we get Win 20
W.8 k
q1 T2 q1 T2 Q1 58.53 kW
q2 = = (ii) (COP)HP, rev = = = 2.804
2 T1 - 2 K H1 - T1 T1 - 2 K H1 Win 20.8 kW
Substituting in Eq. (i) for efficiency (iii) The (COP)R, rev of a reversible refrigerator can
q1 T2 also be expressed in terms of the surrounding
hrev = 1 - temperature T1 and the cold space temperature
q1 (T1 - 2 K H1 )
T2 T2( = 273 K) as
or hrev = 1- T2
(T1 - 2KH1 ) (COP)R, rev =
T1 - T2
Second Law of Thermodynamics 215

or 1.804 ¥ (T1 273) = 273 to the engine be QH and heat rejection from the engine be
or 1.804T1 = 273(1 + 1.804) QL as shown in Fig. 6.34.
= 765.92 K The thermal efficiency of the engine,
or T1 = 424.33 K = 151.3°C Q
hA = 1 - L ...(i)
(iv) The temperature T1 at which heat is supplied to QH
heated space by a reversible heat pump, when it Since the reversible heat engine receives equal
absorbs heat from surroundings at 5°C (= 278 K): amount of heat from reservoirs A and B, therefore, it
T1 will absorb QH /2 from each reservoir and rejects heat
(COP)HP, rev =
T1 - T2 QL to the thermal reservoir C. According to absolute
or 2.804 ¥ (T1 278) = T1 temperature scale;
QH Q Q
or (2.804 1)T1 = 278 ¥ 2.804 = 779.51 K + H = L
2TA 2TB TC
or T1 = 432.13 K = 159.1°C
(v) When the (COP)R, rev of the reversible refrigerator È T + TB ˘ QL
or QH Í A ˙ =
is doubled then Î 2.TA TB ˚ TC
(COP)R1, rev = 2(COP)R, rev QH (TATC + TB TC )
= 2 ¥ 1.804 = 3.608 or QL = ...(ii)
2 TA TB
T2
Further, (COP)R1, rev = Substituting the value of QL in Eq. (i), then
T1¢ - T2 QH (TATC + TB TC )
or 3.608 ¥ (T1¢ – 273) = 273 hA = 1
QH ¥ 2 TA TB
or 3.608T1¢ = 273(1 + 3.608)
TA TC + TB TC
= 1258 K hA = 1 – ...(iii)
2TA TB
or T1¢ = 348.66 K = 75.66°C
Now considering the engine operating between the
A reversible heat engine works between thermal reservoirs A and C, the thermal efficiency of such
three reservoirs A, B and C. The engine absorbs an equal engine is expressed as
amount of heat from the thermal reservoir A and B kept at T - TC
hC = A ...(iv)
temperatures TA and TB , respectively , and rejects heat to TA
the thermal reservoir C at TC. The efficiency of the engine But the efficiencies in two arrangements are related as
is a times the efficiency of the reversible engine operating
hA = a hC
between the thermal reservoir A and C. Prove that
Therefore,
TA TA
= ( 2a - 1) + 2 (1 - a ) TA TC + TB TC È TA - TC ˘
TB TC 1 =aÍ ˙
2TA TB Î TA ˚
Solution Consider the reversible heat engine operat- or 2TA TB TA TC TB TC = 2aTA TB – 2a TB TC
ing between reservoirs A, B and C. Let the heat supplied Dividing both sides by TB TC, we get
2TA TA T
Reservoir A, TA Reservoir B, TB - 1 = 2a A - 2a
TC TB TC
QH QH
2 2
Rearranging we get,
QH TA TA
= (2a 1) + 2(1 a)
HE Wnet TB TC

QL A reversible engine supplied heat


from two constant temperature sources at 800 K and
Reservoir C at TC
400 K and rejects heat to a constant temperature sink at
200 K. If the engine executes a number of cycles, while
developing 100 kW and rejecting 3500 kJ of heat per
216 Thermal Engineering

mintue. Determine the heat supplied by each source per A reversible engine is used to operate
minute, and efficiency of the engine. a satellite between a hot reservoir at TH and a radiating
pannel at TL.The radiation from the pannel is proportional
to its area and the fourth power to temperature TL. For
a given work output and fixed temperature TH , show that
the area will be minimum when TL/TH = 0.75.

Solution Let the reversible engine receives heat


QH from reservoir at TH and rejects heat QL to radiating
pannel at TL.
The radiation heat QL from pannel
QL μ ATL4
Fig. 6.35 or QL = sA TL4 ...(i)
where s is constant of proportionality.
Solution The efficiency of a reversible heat engine
Given A reversible engine between three reservoirs T - TL W
hrev = H =
TA = 800 K TH QH
TB = 400 K Equating 2nd and 3rd terms in the above equation, we
TC = 200 K get
Wnet = 100 kW = 6000 kJ/min QH W
= ...(ii)
QL = 3500 kJ/min TH TH - TL
To find According to the absolute temperature scale;
QH Q
(i) Heat supplied by each source, and = L ...(iii)
(ii) Efficiency of the engine. TH TL
Thus, Eq. (ii) can be written as
Analysis
QL W
(i) The heat supply rate to engine by two sources =
TL TH - TL
QH = Wnet + QL
Using QL from Eq. (i)
= 6000 + 3500 = 9500 kJ/min
The quantity QH is supplied by two resevoir A s ATL4 W
=
and B, thus TL TH - TL
Q A + QB = QH = 9500 kJ/min ...(i) W
and according to the absolute temperature scale or A =
s (TH TL3 - TL4 )
Q A QB Q
+ = L For minimum area, differentiating the above equation
TA TB TC with respect to TL, by treating W, TH constant.
QA Q 3500 W (3TH TL2 - 4TL3 )
or + B = dA
= =0
800 400 200 dTL [s (TH TL3 - TL4 )]2
or Q A + 2 QB = 14000 ...(ii) or 3TH TL2 = 4TL3
Solving Eq. (i) and (ii), we get TL 3
or = = 0.75
Q A = 5000 kJ/min and QB = 4500 kJ/min TH 4
(ii) Thermal efficiency of the reversible heat engine,
Wnet 6000 kJ/min 0.5 kg of air (ideal gas) executes a
hth = = Carnot cycle with thermal efficiency of 50%. The heat
QH 9500 kJ/min
transfer to air during isothermal expansion is 40 kJ. At
= 0.631 or 63.1%
the beginning of isothermal expansion, the pressure and
Second Law of Thermodynamics 217

volume of air are 7 bar and 0.12 m3, respectively. Find For Carnot engine, the efficiency is given by
the maximum and minimum temperature of the cycle. T
hrev = 1 - L
Take Cp = 1.008 kJ/kg ◊ K, Cv = 0.721 kJ/kg ◊ K. TH
TL
or 0.5 = 1 -
585.36
Minimum Temperature
TL = 292.68 K or 19.68°C

The efficiency of Carnot engine is


20%. The efficiency gets doubled, when the sink tempera-
ture is reduced by 60°C. Estimate the source and sink
temperatures.

Solution
Given A Carnot engine with its efficiency
hrev1 = 0.2 hrev2 = 0.4
TL2 = (TL1 60) K
Solution
To find Source and sink temperatures.
Given A Carnot engine with its efficiency
Analysis Carnot engine efficiency is given by
m = 0.5 kg hrev = 0.5
TL
QH = 40 kJ p1 = 7 bar = 700 kPa hrev1 = 1 - 1 ...(i)
TH
V1 = 0.12 m3 Cp = 1.008 kJ/kg ◊ K
TL
Cv = 0.721 kJ/kg ◊ K or 0.2 = 1 - 1
TH
To find Source and sink temperatures. or TL1 = 0.8TH
Analysis The specific gas constant for air When the sink temperature is lowered by 60°C, then
R = Cp Cv hrev2 = 0.4,
= 1.008 0.721 = 0.287 kJ/kg ◊ K T - 60
or 0.4 = 1 - L1
Using characteristic gas equation at state 1; TH
p1V1 = mR TH or TL1 60 = 0.6TH ...(ii)
Sustituting TL from Eq. (i) into Eq. (ii), we get
Maximum temperature
0.2TH = 60
p1V1 700 ¥ 0.12
TH = = Source temperature TH = 300 K
m R 0.5 ¥ 0.287
and sink temperature TL1 = 240 K
= 585.36 K or 312.36°C

second law of thermodynamics states that second law are the following:
the processes occur in certain direction only. 1. According to Clausius’s statement, the heat
A process is not possible, if it does not satisfy energy cannot be transferred from a low-
both statements of the second law and first law temperature body to a high-temperature
of thermodynamics. The two statements of the body without addition of external work.
218 Thermal Engineering

2. According to the Kelvin–Planck statement, For a reversible refrigerator, COP can also be
a cyclic heat engine cannot convert all heat expressed as
into work, while exchanging heat from a TL
(COP)R, rev =
single-temperature reservoir. TH - TL
These two statements are equivalent in their Similarly, the COP of a heat pump is defined as
consequences. If any of these is taken as a start- Heat supplied QH
(COP)HP = =
ing point, the second law can be deduced. Any Work input Win
machine that violates the first or second law of QH
=
thermodynamics is called the perpetual motion QH - QL
machine. For a reversible heat pump, COP can also be
high-grade energy and can be expressed as
converted directly in any form of energy like heat, TH
etc. But the heat is low-grade energy and cannot (COP) HP, rev =
TH - TL
be converted into work without a heat engine. The
reversible process is one, after it has occurred,
thermal efficiency of heat engines is defined as
both system and surroundings can be restored to
W Q
hth = net = 1 - L their original states. A process may be reversible
QH QH
in absence of friction, non quasi-equilibrium
For a reversible heat engine, the efficiency can expansion or compression and heat transfer
also be expressed as through a finite temperature difference. All actual
T
hrev = 1 - L processes are irreversible.
TH Carnot cycle is a reversible cycle, consisting
of two reversible isothermal and two isentropic
that absorb heat from low-temperature reservoir at processes.
TL and rejects heat to high temperature reservoir According to Carnot principle, all reversible
at TH. The performance of a refrigerator and a heat engines operating between same tempera-
heat pump is expressed in term of coefficient of ture limits are equally efficient and no heat engine
performance, which is defined as can be more efficient than a reversible heat engine
Desired effect operating between same two temperature limits.
COP =
Energy input as work -
The COP of a refrigerator is given as cal properties of any substance is called absolute
temperature scale.According to this scale, the
Refrigeration effect QL
(COP)R = = heat transfers by a reversible device between high
Work input Win
and low-temperature reservoirs can be related as
QL
= QH T
Q H - QL = H
QL TL

Thermal reservoir A hypothetical body of infinite heat Heat pump A machine which maintains a body at a
capacity higher temperature than the atmospheric temperature
Heat engine A cyclic work-producing device, which COP Ratio of desired output to energy input for
converts the heat energy into useful work refrigerator or heat pump
Thermal efficiency Ratio of net work done and energy Reversibility Ability of a process to operate in reverse
input as heat direction without leaving any change on atmosphere
Refrigerator A machine which maintains a body at a Irreversibility Characteristics of actual process, which
lower temperature than the atmospheric temperature make it irreversible
Second Law of Thermodynamics 219

Carnot engine A hypothetical engine which gives Absolute temp. scale A temperature scale which is
highest efficiency among all engines operating between independent of properties of working medium
same two temperature limits

1. What are the limitations of the first law of 15. List suitable examples of reversible and irrevers-
thermodynamics? ible processes.
2. State the importance of the second law of thermo- 16. What are the conditions to be a reversible process?
dynamics. 17. What is a thermodynamic temperature scale?
3. Define thermal reservoir, heat engine, refrigerator 18. Explain the establishment of a thermodynamic
and heat pump. temperature scale. Why is the thermodynamic
4. What is a thermal energy reservoir? Define in temperature scale called absolute temperature
terms of heat source and heat sink. scale?
5. What are the characteristic features of a heat 19. Sketch the Carnot cycle on a p–v diagram.
engine? 20. Explain Carnot cycle and prove that
6. What is a heat pump. How does it differ from a T
h carnot = 1 - L .
refrigerator? TH
7. What do you mean by coefficient of performance? 21. Is it possible to operate a heat engine on a Carnot
Show that (COP)HP = (COP)R + 1. cycle? What purpose does a Carnot cycle serve?
8. What do mean by thermal efficiency? Derive an 22. State and prove the Carnot theorem.
expression for thermal efficiency of a heat engine. 23. Prove that a reversible engine is more efficient
9. State the Kelvin–Planck and Clausius statements than an irreversible engine operating between the
of the second law of thermodynamics. same temperature limits.
10. Prove that the Kelvin Plank and Clausius’s 24. Prove that all reversible engines operating between
statements of the second law of thermodynamics same temperature limits are equally efficient.
are equivalent to each other. 25. A reversible heat engine absorbs an equal amount
11. Is it possible for a heat engine to operate without of heat from two thermal reservoirs kept at tem-
rejecting any amount of heat. Explain with proper perature T1 and T2, respectively, and rejects heat
sketch. to the thermal reservoir at T2. The efficiency of
12. Prove that violation of Kelvin–Planck statement this engine is b times the efficiency of the revers-
leads to violation of Clausius statement. ible engine operating between thermal reservoir
13. Prove that violation of the Clausius statement at T1 and T3. Prove that
leads to violation of the Kelvin–Planck statement.
1 È T2 - T3 T2 ˘ T1
14. Explain reversibility and irreversibility. State the b= Í + ˙¥
2 Î T1 - T3 T1 ˚ T2
factors responsible for irreversibility of a process.

1. A steam power plant with a power output of 2. An engine manufacturer claims to have developed
100 MW, consumes coal at a rate of 40 tonnes/h. an engine that takes in 500 kJ of heat at a tem-
If the calorific value of coal 30 MJ/kg, determine perature of 720 K and rejects 200 kJ of heat at a
the thermal efficiency of this plant. temperature of 360 K, while producing 260 kJ of
[30.0 percent] work. Is his claim true? Explain. [Claim is false]
220 Thermal Engineering

3. Determine the heat input to a Carnot engine that forms 75% of the Carnot refrigerator, estimate the
operates between 400°C and 47°C and produces refrigerating effect per kW of power consump-
110 kJ of work. [209.7 kJ] tion. [5.53, 4.149 kW]
4. An experimentalist claims to have developed a 12. The cooling load on an air-conditioner is 25 kW,
heat engine that takes in 300 kJ of heat from a when the outdoor temperature is 42°C and the
source at 500 K, produces 160 kJ of work and indoor air is maintained at 22°C. Determine the
rejects the rest of the heat as waste to a sink at minimum power requirement to operate the air-
300 K. Are these measurements reasonable? conditioner. [1.695 kW]
Why? [False claim] 13. A Carnot refrigerator removes heat at a rate of
5. A Carnot engine receives 220 MJ/h of heat from 40.0 kW from a cold storage space at –20°C. The
a source at 350°C and rejects 130 MJ/h to another refrigerator is driven by a Carnot engine that takes
cold reservoir. What is the temperature of the cold heat from a reservoir at 400°C and discharges,
reservoir? [95.1°C] heat to the atmosphere at 25°C. Determine (a)
6. Determine the minimum heat input to a heat the power required to drive the refrigerator, and
engine that operates between 350°C and 25°C (b) the total rate of heat rejected to atmosphere by
and produces 100 kJ of work. [191.7 kJ] the combined devices.
7. A Carnot heat engine receives 500 kJ of heat [(a) 11 kW (b) 52.76 kW]
from a source of unknown temperature and 14. An air-conditioning system is used to maintain
rejects 200 kJ of it to a sink at 17°C. Determine a house at a constant temperature of 20°C. The
(a) the temperature of source, and (b) the thermal house is gaining heat from outdoors at a rate of
efficiency of the heat engine. [725 K, 60%] 20,000 kJ/h and the heat generated in the house
8. In a Carnot engine using air as working medium, from the people, lights and appliances amounts
the air enters the engine at 500 kPa, 95°C and to 8000 kJ/h. For a COP of 2.5, determine the
occupies volume of a 0.2 m3 at the beginning of required power input for this air-conditioning
the isothermal expansion. The volume doubles system. [3.11 kW]
during isothermal expansion. The temperature at 15. A household refrigerator, having a power input
the end of adiabatic expansion is 0°C. Determine of 450 W and a COP of 2.5, is used to cool
(a) the heat added, and (b) the heat rejected. five watermelons of 10 kg each to 8°C. If the
[(a) 69.13 kJ (b) 51.2 kJ] watermelons are initially at 20°C, determine how
9. A Carnot heat engine receives heat from a long it will take for the refrigerator to cool them.
reservoir at 900°C at a rate of 700 kJ/min and The properties of watermelon can be taken as that
rejects waste heat to the ambient at 27°C. The of water, i.e., specific heat as 4.2 kJ/kg ◊ K and
entire work output of the heat engine is used latent heat of freezing as 335 kJ/kg. [2240 s]
to drive a refrigerator that removes heat from 16. A 2-kg ice-cream mix at 27°C is placed in a
a space at a temperature of –5°C and transfers refrigerator for freezing. The specific heat of the
it to the same ambient at 27°C. Determine (a) ice-cream mix above its freezing temperature is
the maximum rate of heat removal from the 4.2 kJ/kg ◊ K and its freezing temperature is 0°C.
refrigerated space, and (b) the total heat rejection The latent heat of freezing is 330 kJ/kg. The
to ambient air. refrigerator operates at 60% of COP of the Carnot
[(a) 4338.25 kJ/min; (b) 5038.25 kJ/min] refrigerator when placed in an ambient of 37°C.
10. An inventor claims to have developed a refrig- How much work in kWh is required for removing
eration system that removes heat from a cooled this heat? [0.056 kWh]
region at –5°C and transfers it to the surrounding 17. A heat pump used to heat a house runs about one-
air at 22°C, while maintaining its COP of 7.5. Is third of the time. The house is losing heat at an
this claim reasonable? Why? [Reasonable] average of 15,000 kJ/h. If the COP of the heat
11. A Carnot refrigerator is working between reser- pump is 4.5, determine the power input to the heat
voirs of –13°C and 34°C. What is the coefficient pump. [13.33 kW]
of performance? If the actual refrigerator per-
Second Law of Thermodynamics 221

18. A heat pump is used to maintain a house at a 23. A reversible heat engine receives heat from
constant temperature of 22°C. The house is two thermal reservoirs maintained at constant
losing heat to the outside air through the walls temperatures of 750 K and 500 K. The engine
and windows at a rate of 40 MJ/h, while energy develops 100 kW of power and rejects 3600
generated within the house from the people, kJ/min of heat to a heat sink at 250 K. Determine
lights and appliances amount to 6000 kJ/h. For a the thermal efficiency of the engine and heat
COP of 2.4, determine the required power input supplied by each thermal reservoir.
to the heat pump. [3.93 kW] [62.5%, 7200 kJ/min and 2400 kJ/min]
19. A heat pump with a COP of 3.2 is used to heat a 24. A Carnot heat engine absorbs heat from three
house. When it is operated, it consumes a power thermal reservoirs at 1000 K, 800 K and 600 K.
at a rate of 5 kW. If the temperature in the house The engine develops 10 kW of power and rejects
is maintained at 7°C, when heat pump is turned 400 kJ/min of heat to a sink at 300 K. If the heat
on, how long will it take to raise the temperature supplied by reservoir at 1000 K is 60% of the
of the house to 22°C? Is this answer realistic heat supplied by a reservoir at 600 K, calculate
or optimistic? Explain. Assume the entire mass the quantity of heat absorbed by each reservoir.
within the house ( air, furniture, etc. ) is equivalent [Q1 = 312.5 kJ/min, Q2 = 500 kJ/min,
to 1500 kg of air for which Cv = 0.72 kJ/kg ◊ K and and Q3 = 187.5 kJ/min]
Cp = 1.0 kJ/kg ◊ K. [23.44 min.] 25. A reversed Carnot cycle operating as a refrigerator
20. A Carnot heat pump is used to heat and maintain has a refrigerating capacity of 100 kW, while
a residential building at 22°C. An energy analysis operating between temperature limits of –20°C
of the house reveals that it loses heat at a rate and 35°C. Determine,
of 2500 kJ/h°C temperature difference between
(a) power nput,
i and (b) COP.
indoors and outdoors. For an outdoor temperature
If the system is used for heating purposes only,
of 4°C, determine (a) the COP, and (b) the power
find its COP. What would be its efficiency if it
required to run the heat pump.
runs as an engine?
[(a) 16.4, (b) 0.762 kW]
26. A heat pump working on a reversed Carnot cycle
21. A Carnot refrigerator removes heat at a rate of
takes in heat energy from a reservoir maintained
40.0 kJ/s from a cold storage room at –2°C. The
at 5°C and delivers it to another reservoir at 77°C.
refrigerator is driven by a Carnot engine that takes
The heat pump derives the power from a revers-
heat from a reservoir at 400°C and discharges heat
ible heat engine operating between temperatures
to the atmosphere at 25°C. Determine (a) power
of 1077°C and 77°C. For 100 kJ/kg of energy
required to drive the refrigerator, and (b) the rate
supplied to the reservoir at 77°C, calculate the
at which heat is supplied to the Carnot engine
heat energy supplied to reversible heat engine.
from the high-temperature reservoir.
[25.9 kJ/kg]
[(a) 3.985 kW, (b) 7.152 kW]
27. A heat engine receives half of its heat at a
22. An ice plant is working on a reversible heat pump
temperature of 1000 K and the rest at 500 K,
and produces 15 tonnes of ice per day. The ice is
while rejecting heat to a sink at 300 K. What is
formed from water at 0°C. The heat is rejected
the maximum possible thermal eficiency of this
to the atmosphere at 25°C. The heat pump used
heat engine?
to run the ice plant, is coupled to a Carnot heat
È
engine, which absorbs heat from a reservoir at Í Hint : Refer example 6.27
220°C, burning liquid fuel of 44500 kJ/kg heating Î
value and the engine rejects heat to atmosphere. T T + TB TC ˘
hrev = A C = 55% ˙
Determine (a) power developed by heat engine, 2TA TB ˚
and (b) fuel consumed per hour. Take the enthalpy 28. A refrigerator transfers 120 kJ of heat from a cold
of fusion of ice as 334.5 kJ/kg. space, and needs 40 kJ of work input. Calculate
[(a) 5.3 kW, (b) 1.082 kg/h] its coefficient of performance. [3]
222 Thermal Engineering

uestions
1. The second law of thermodynamics deals with 6. A Carnot cycle operates between temperatures of
(a) direction of process and quality of energy 727°C and 227°C, the efficiency of the engine is
(b) energy balance (a) 40% (b) 50%
(c) balance of internal energy (c) 60% (d) 45%
(d) system efficiency 7. A heat engine receives heat from a source at
2. It is impossible to construct an engine which 1000°C and rejects the waste heat to a sink at
while operating in a cycle, produces no other 50°C. If the heat is supplied to the engine at the
effect except to extract the heat from a single- rate of 100 kW. The maximum power output of
temperature reservoir and do equivalent amount this engine is
of work. (a) 25.48 (b) 55.44
(a) It refers to Clasius statement. (c) 74.62 (d) 79.85
(b) It refers to Kelvine–Planck’s statement. 8. A heat pump is absorbing heat from a cold
(c) It refers to Carnot’s theorem. outdoor at 5°C and supplying heat to a house at
(d) It refers to Clasius’s theorem. 22°C at a rate of 18000 kJ/h. The power consumed
3. A Carnot cycle comprises of by the heat pump is 2.5 kW, the coefficient of
performance of heat pump is
(a) two isothermal and two isentropic processes
(a) 0.5 (b) 1.0
(b) two constant volume and two isentropic
processes (c) 2.0 (d) 5.0
(c) two constant pressure and two isentropic 9. The efficiency of the Carnot engine using an ideal
processes gas as working substance is
T - TL TH - TL
(d) one constant volume, one constant pressure (a) H (b)
and two isentropic processes TH TH TL
4. In a thermal power plant, turbine does 10,000 kJ TH TL
(c) (d)
of work, pump consumes 10 kJ of work. The TH - TL TH - TL
boiler receives 30,000 kJ of heat. Thermal 10. An air-conditioning system operating on the
efficiency of the plant is reversed Carnot cycle is required to remove heat
(a) 27% (b) 33.3% from the house at a rate of 32 kW to maintain
(c) 35% (d) 40% the temperature constant at 20°C. If the outdoor
5. A refrigerator and heat pump operates between temperature is 35°C, the power required to
same temperature limits. If the COP of the refrig- operate the air-conditioning system is
erator is 4, what is the COP of heat pump? (a) 1.638 kW (b) 3.2 kW
(a) 3 (b) 5 (c) 1.56 kW (d) 2.26 kW
(c) 4 (d) 3.4

10. (a) 9. (a)


8. (c) 7. (c) 6. (b) 5. (b) 4. (b) 3. (a) 2. (b) 1. (a)
Answers
Entropy 223

7
Entropy

Introduction
The second law of thermodynamics gives a precise definition of a property called entropy. Entropy can be
thought of as a measure of the disorder in the system. The second law states that the entropy—that is, the
disorder—of an isolated system can never decrease.
This chapter starts with the concept of entropy, Clausius’ theorem, Clausius inequality, increase of
entropy principle, entropy transfer and entropy generation. The temperature–entropy diagram and the third
law of thermodynamics are also discussed in this chapter.

of heat into work and for minimum entropy, there is


maximum possibility of conversion.
The second law of thermodynamics deals with
reversible and irreversible processes. Entropy is a
property of reversible processes and is an abstract
property, which cannot be defined precisely. It may
be understood by studying its uses in engineering Let us assume two reversible adiabatic (isentropic)
processes. lines AC and BC intersecting each other at the point
The amount of heat supplied to an engine is not C. Let a reversible isothermal line AB join the two
important but the temperature at which heat sup- isentropic lines at A and B as shown in Fig. 7.1.
plied is important, because the heat supplied at
high temperatures has a greater possibility of con-
version into work than the heat supplied at lower
temperatures.
Entropy is a function of quality of heat (i.e.,
temperature), which leads the conversion of heat
into work. Thus, the increase in entropy is small
when heat is transferred at higher temperature and
change in entropy is large when heat is transferred
at lower temperatures. Therefore, for maximum Two isentropic lines and one isothermal line
entropy there is minimum possibility of conversion constitute an impossible cycle
224 Thermal Engineering

The reversible processes AB, BC and CA with is equal to the area under the curve 1–2 as shown
suitable direction, together constitute a reversible in Fig. 7.2.
cycle. The area enclosed by the three processes on Applying the first law of thermodynamics to the
a p–V plot is work output. reversible process 1–2;
But the operation of any device in such a cycle Q1– 2 = W1–2 + U2 – U1 ... (i)
is impossible because it exchanges heat during a For processes 1–A, A–B and B–2
single isothermal process AB and produces the net
Q1–A–B–2 = W1–A– B–2 + U2 – U1 ...(ii)
work. Thus this cycle violates the Kelvin–Planck
Since W1–2 = W1– A–B–2
statement of the second law. Thus we can conclude
that the two reversible adiabatic lines cannot inter- Therefore, Q1–2 = Q1–A–B–2
sect each other. = Q1–A + QA–B + QB–2
Two constant-volume lines, two constant-pres- But isentropic (reversible adiabatic) lines have
sure lines or two constant-temperature lines cannot no heat transfer, i.e.,
intersect each other, because each line contains a Q1–A = 0 and QB–2 = 0
property whose magnitude differs from the other Thus Q1–2 = QA–B
line. Similarly, we may imagine that each revers- The heat transfer during the reversible process
ible adiabatic line must have some property, whose 1–2 is equal to the heat transfer during the
magnitude differs in two lines, thus they are not isothermal process A–B.
intersecting. This property of reversible adiabatic Consider a smooth closed curve representing a
lines is referred as entropy. Thus, a reversible adia- reversible cycle. Let the large number of reversible
batic process is called an isentropic process. adiabatic and reversible isothermal lines be drawn
in such a way that the original reversible cycle
is approximated by a number of small Carnot
cycles as shown in Fig. 7.3. The sum of the heat
It states “a reversible line can be replaced by transferred during all small Carnot cycles should be
two reversible adiabatic lines and a reversible equal to the heat transferred during the reversible
isothermal line.” closed cycles.

A reversible line replaced by two isentropic


and one isothermal line

Consider a reversible process 1–2. Let two


reversible adiabatic lines 1–A and B–2 and a A reversible cycle approximated by a number
reversible isothermal line A–B replace the reversible of small Carnot cycles
line 1–2 in such a way that the area under 1–A–B–2
Entropy 225

For a small Carnot cycle 1–2–3–4, d Q1 is the


heat supplied at the constant temperature T1 and
dQ2 is the heat rejection at constant temperature T2.
Thus according to absolute temperature scale for a
reversible cycle.
d Q1 d Q2
=
T1 T2
Considering sign convention for heat transfer,
i.e., –ve sign for heat rejection.
d Q1 d Q2
Then + =0 ...(i) A reversible and an irreversible heat engines
T1 T2 operating between the same two-temperature
Similarly, for the small Carnot cycle 5– 6 – 7– 8, resevoir
transfser of heat dQ3 at T3 and dQ4 at T4,
dQ
d Q3 d Q4 The cyclic integral or summation of for
+ =0 ...(ii) T
T3 T4 a cycle is always less than or equal to zero. This
and so on. inequality is true for all types of cycles, reversible
For all small Carnot cycles approximating a or irreversible.
reversible cycle. Consider the reversible and irreversible heat
engines, both operating between a high temperature
Ê d Q1 d Q2 ˆ Ê d Q3 d Q4 ˆ
ÁË T + T ˜¯ + ÁË T + T ˜¯ + … = 0 reservoir at TH and a low-temperature reservoir at TL
1 2 3 4 as shown in Fig. 7.4. The amount of heat supplied,
dQ QH, to both heat engines is same. The reversible
or Ú T
= 0 for a reversible cycle ...(7.1)
heat engine rejects heat QL1 and does work Wrev.
This is Clausius’ theorem and it states that the First, considering the reversible heat engine
dQ ÊdQˆ d QH d QL1
Ú Ú Ú
algebraic summation of all is always zero for -
T ÁË ˜¯ =
a reversible cycle. T rev TH TL
1 1
=
TH Ú dQH -
TL ÚdQ
L1

The second law of thermodynamics often leads to QH QL1


= -
inequalities. According to Carnot’s theorem, an TH TL
actual (irreversible) heat engine is always less ef- According to the Kelvin temperature (absolute)
ficient than a reversible one, operating between the scale,
same two temperature limits. Similarly, the COP of QH Q
an actual heat pump or refrigerator is always less = L1
TH TL
than a reversible one operating between the same
two thermal reservoirs. Another inequality of ther- ÊdQˆ Q Q
modynamics is the Clausius inequality, that is ex-
Thus
Ú ÁË T
˜¯
rev
= H - L1 = 0
TH TL
...(7.3)
pressed as
Equation (7.3) is valid for a totally reversible
dQ
Ú T
£0 ...(7.2) heat engine as well as an internally reversible heat
226 Thermal Engineering

engine. For an internally reversible heat engine, TH Heat supplied, QH = 600 kJ


and TL are temperatures of the working fluid at lo- Source temperature TH = 1000 K
cations where heat transfer take place, not the tem- Work done W = 150 kJ
Heat rejected QL = 450 kJ
perature of thermal reservoir.
Sink Temperature TL = 300 K
Now consider an irreversible heat engine deliv-
ering less work Wirrev than the reversible engine. To find
Thus it rejects more heat QL2, Ê dQˆ
Therefore, QL2 > QL1
(i) Cyclic
Ú ÁË T ˜¯ .
Expressing as QL2 = QL1 + Qdiff (ii) To compare the reversible and actual efficiency of
Then cyclic integral for irreversible heat engine the engine.
dQ d QH d QL2 Analysis
Ú T
= Ú TH
- Ú TL ÊdQˆ Q H QL
dQ QH QL1 Qdiff
(i)
Ú ÁË T ˜¯ = -
Ú
TH TL
or = - -
T TH TL TL Ê 600 ˆ Ê 450 ˆ
= Á - = – 0.9
QH QL1 Ë 1000 ˜¯ ÁË 350 ˜¯
But - = 0 (refer Eq. 7.3)
TH TL
ÊdQˆ
Thus Ú
dQ
= -
Qdiff
π0
Since the cyclic
Ú ÁË T ˜¯ is less than zero,
T TL therefore, the cyclic heat engine satisfies
The quantity of heat Qdiff is positive, therefore,
the Clausius inequality of the second law of
dQ
Ú
thermodynamics.
<0 ...(7.4)
T (ii) The reversible efficiency of the heat engine
The Clausius inequality is obtained by combin- T 300
hrev = 1 – L = 1 – = 0.7 or 70%
ing Eqs. (7.3) and (7.4), TH 1000
dQ
Ú
Actual efficiency of the heat engine
£0
T Q 450
hact = 1 – L = 1 – = 0.25 or 25%
which is valid for all cycles. Similarly, the Clausius QH 600
inequality can also be obtained for refrigeators and The actual efficiency of the heat engine is less
heat pumps. But for any machine than the Carnot efficiency, therefore, the Carnot
dQ principle is satisfied.
Ú T
>0 ...(7.5)
Example 7.2 A system undergoes a reversible cycle
is impossible. by exchanging heat Q1, Q2, and Q3 with three thermal
reservoirs at T1 T2, and T3 , respectively. Show that
Example 7.1 A heat engine receives 600 kJ of heat
from a high-temperature reservoir at 1000 K during a Q1 Q2 Q3
= =
cycle. It converts 150 kJ of this heat to net work and 1 1 1 1 1 1
- - -
rejects the remaining 450 kJ to a low-temperature sink at T2 T3 T3 T1 T1 T2
300 K. Determine if this heat engine violates the second
law of thermodynamics on the basis of (a) the Clausius Solution According to the first law of thermodynam-
inequality, and (b) the Carnot principle. ics
Q1 + Q2 + Q3 = 0 ...(i)
Solution According to Clausius’ theorem for a reversible cycle
Q1 Q2 Q3
Given A heat engine with different operating condi- + + =0 ...(ii)
tions: T1 T2 T3
Entropy 227

Solution
Given A heat engine operates with a different quantity
of heat rejection
Q1 = 1130 kW
T1 = 292°C + 273 = 565 K
T2 = 5°C + 273 = 278 K
To find Whether the result indicates a reversible, irre-
versible or an impossible cycle.

From Eq. (i), we getQ 3 = – (Q1 + Q2) Analysis According to Clausius inequality to the cyclic
Substituting the value of Q3 in Eq. (ii), we get process.
dQ
Q1 Q2 Ê Q1 + Q2 ˆ
+ -
T1 T2 ÁË T3 ˜¯
=0 Ú T
£0

(i) Heat rejection is 834 kW.


Ê1 1ˆ Ê 1 1ˆ
Q1 Á - ˜ = Q2 Á - ˜ dQ Ê 1130 ˆ Ê 834 ˆ
or
Ë T1 T3 ¯ Ë T3 T2 ¯ Ú T
= Á -
Ë 565 ˜¯ ÁË 278 ˜¯
= 2 – 3 = –1

Q1 Q2 Therefore, the cycle is an irreversible cycle.


or = ...(iii)
Ê 1 1ˆ Ê 1 1ˆ (ii) Heat rejection is 556 kW.
ÁË T - T ˜¯ ÁË T - T ˜¯
dQ Ê 1130 ˆ Ê 556 ˆ
Ú
2 3 3 1
= Á -
Ë 565 ˜¯ ÁË 278 ˜¯
=2–2=0
Again from Eq. (i), we get Q2 = – (Q1 + Q3) T
Substituting the value of Q2 in Eq. (ii), we get Therefore, the cycle is a reversible cycle.
Q1 Ê Q1 + Q3 ˆ Q3 (iii) The engine rejects 278 kW of heat. Then
- + =0
T1 ÁË T2 ˜¯ T3 dQ Ê 1130 ˆ Ê 278 ˆ
Ê1 1ˆ Ê 1 1ˆ
Ú T
= Á -
Ë 565 ˜¯ ÁË 278 ˜¯
=2–1=1

or Q1 Á - ˜ = Q3 Á - ˜ Therefore, the cycle is an impossible cycle.


Ë T1 T2 ¯ Ë T2 T3¯
Q1 Q3
or = ...(iv)
Ê 1 1ˆ Ê1 1ˆ
ÁË T - T ˜¯ ÁË T - T ˜¯
2 3 1 2

From the identity of Eqs. (iii) and (iv), we get dQ


Q1
=
Q2
=
Q3
The mathematical quantity
ÚT
discussed above
Ê 1 1 ˆ Ê 1 1ˆ Ê 1 1ˆ is the property of a reversible line, and is thus a
ÁË T - T ˜¯ Á - ˜ Á - ˜
2 3 Ë T3 T1 ¯ Ë T1 T2 ¯ point function. The cyclic integral of any property is
which is the required expression. always zero. To demonstrate entropy as a property,
we consider a cycle of two reversible processes
Example 7.3 A heat engine is supplied with 1130 kW 1–A–2 and 2–B–1 as shown in Fig. 7.6.
of heat at a constant temperature of 292°C and it rejects The cyclic integral for the reversible process
heat at 5°C. The following results were recorded: 1–A–2 and 2–B–1,
(a) 834 kW heat is rejected, (b) 556 kW heat is 2 ÊdQˆ
dQ 1 ÊdQˆ
rejected, and (c) 278 kW heat is rejected.
Determine whether results report a reversible cycle,
Ú T
= Ú Á ˜ +
1 Ë T ¯A Ú2
ÁË ˜
T ¯B
irreversible or impossible cycle. For a reversible cycle,
dQ
Ú T
=0
228 Thermal Engineering

Engineers are usually concerned with the change


in entropy, not in the absolute value of entropy. The
absolute value of entropy can be determined on the
basis of the third law of thermodynamics.
dQ
The integration gives us the value of
T
entropy change for an internally reversible process
Entropy change between two states is same, dQ
when the process is reversible only. The integral along an irreversible process
T
is not a property and different values are obtained
Thus when the integration is carried out along different
2 ÊdQˆ 1 ÊdQˆ irreversible paths. Therefore, for an irreversible
Ú1
ÁË ˜ +
T ¯A Ú2
ÁË ˜ =0
T ¯B process, the entropy change should be calculated by
Changing the limit of the second integral and carrying out the integration along some convenient
rearranging, we get imaginary internally reversible path between the
two states.
2 ÊdQˆ 2 ÊdQˆ
Ú1
ÁË ˜ =
T ¯A Ú
1
ÁË ˜
T ¯B
...(7.6)

Since processes 1–A–2 and 2–B–1 are internally


reversible processes between states 1 and 2 and this
integral depends on end states, between which the If entropy is taken along the x-axis and absolute
process takes place not on the path followed by temperature along the y-axis, the diagram so ob-
ÊdQˆ tained is called a temperature–entropy (T–S) dia-
the process. Thus, the quantity Á is a
Ë T ˜¯ int, rev
gram. Such a diagram for a process 1–2 is shown in
Fig. 7.7. Consider the equation of entropy
property. Clausius stated that he had discovered a
new thermodynamic property and he named this ÊdQˆ
dS = Á
property as entropy. It is designated as s and is Ë T ˜¯ Int, rev
defined as or (dQ)int,rev = TdS ...(7.9)
ÊdQˆ The differential area dA under the process on
dS = Á (kJ/K) ...(7.7)
Ë T ˜¯ int, rev a T–S diagram represents the differential amount
The entropy is an extensive property of the
system, and therefore, sometimes, it is also referred
as total entropy. The entropy per unit mass, called
specific entropy and designated as s, is an intensive
property, measured in kJ/kg ◊ K.

The entropy change of a system during a process


can be obtained by integrating Eq. (7.7) between
initial and final states.
represents the heat transfer for an internally
2 ÊdQˆ
DS = S2 – S1 =
Ú1
ÁË T ˜¯
int, rev
(kJ/K) ...(7.8) reversible process
Entropy 229

of heat transfer (dQ) for an internally reversible 1 ÊdQˆ


process. S2 – S1 + Ú
2
ÁË ˜
T ¯ irrev
<0
The total heat transferred during an internally
It can be rearranged as
reversible process can be obtained on integration of
Eq. (7.9). 2 ÊdQˆ
2
S2 – S1 > Ú ÁË ˜
T ¯ irrev
...(7.12)
Ú T dS ( kJ )
1
Qint, rev = ...(7.10)
1
Therefore, we can conclude that, the area under In the differential from
the process curve on a T–S diagram represents the Êd Q ˆ
dS > Á
Ë T ˜¯ irrev
internally reversible heat transfer. It is analogous to ...(7.13)
a reversible boundary work being represented by
the area under the process curve on a p–V diagram. We can conclude that the entropy change of a
On a unit-mass basis closed system during an irreversible process is
d qint, rev = Tds (kJ/kg) dQ
greater than the integral of , evaluated for that
2 T
Ú
and qint,rev = T ds (kJ/kg) ...(7.11) process.
1 If the process 1–A–2 is considered reversible
then
dQ
dS =
T
2 dQ
Consider an irreversible cycle, consisting of two and S2 – S1 =
1 T Ú
process, one internally reversible and the other ir- Thus, the general form of Eq. (7.14), becomes
reversible as shown in Fig. 7.8. 2 dQ
From the Clausius inequality, S2 – S1 ≥
1 T Ú ...(7.14)
dQ
T
£0 Ú The quantity S2 – S1 represents the entropy
change of the system during a reversible process
For an irreversible cycle 1–A–2–B–1, 2 dQ
2 ÊdQˆ 1 ÊdQˆ
and is equal to
Ú , which represents entropy
Ú1
ÁË ˜ +
T ¯A Ú2
ÁË ˜ <0
T ¯B
1 T
transfer with heat. That is, for a reversible process,
The first integral in the above relation is for an DS = S2 – S1 or S1 = S2
internally reversible process and can be written as or S = Constant ...(7.15)
the entropy change S2 – S1. Therefore, The inequality sign in Eq. (7.13) represents that
the entropy change of a closed system during an
irreversible process is always greater than the en-
tropy transfer for a reversible process. That is, some
entropy is generated or created during an irrevers-
ible process due to presence of irreversibilies. The
entropy generated during a process is called entro-
py generation, and is designated as Sgen. Thus for
an irreversible process;
2 dQ
A cycle composed of a reversible and an
irreversible process
S2 – S1 =
1 T Ú + S gen ...(7.16)
230 Thermal Engineering

The entropy generation Sgen is always a positive namics makes no difference between heat and work
quantity for an irreversible process and zero for a transfer, two forms of energy. But the second law
reversible process, or explains that an energy transfer with entropy trans-
> 0 irreversible process fer is heat transfer and an energy transfer without
Sgen = 0 reversible process ...(7.17) entropy transfer is work transfer.
< 0 impossible process The total entropy is a product of mass and spe-
Further, the increase of entropy principle states cific entropy. When the mass transfers within an
that the entropy of an isolated system, (the heat open system, entropy transfer also takes place.
transfer is zero) either increases or remains con- Both energy and entropy transport takes place into
stant. or out of the system with a stream of mass flow. The
DSisolated ≥ 0 ...(7.18) closed system does not involve any mass transfer,
and thus no entropy transport.
Thus, the entropy change of an isolated system
can never decrease, if a system and surroundings
together (universe) constitute an isolated system.
Thus, for all possible processes in the universe, All actual processes involve irreversibilities like
DSuniverse ≥ 0 ...(7.19) friction, mixing of fluid streams, chemical reac-
or DSsystem + DSsurroundings ≥ 0 ...(7.20) tions, heat transfer through finite temperature dif-
Since no actual process in the universe is truly ference, unrestrained expansion, non quasi-static
reversible, some entropy is generated during each compression or expansion. These irreversibilities
process and therefore, the entropy of the universe always cause entropy of a system to increase. This
(an isolated system) is continuously increasing.
The more irreversibilities involved in a process, the T
larger the entropy generation during that process.
TH
A B
Reversible
heat
transfer
from
The entropy can be transferred to or from a system thermal
by heat transfer and mass transfer. The entropy reservoir
transfer is recognised at the boundary of the system SA SB S
and entropy is either gained or lost during a process. (a) The entropy decrease of the
Heat is a disorganised form of energy and some reservoir is SB – SA
disorganisation (entropy) always flows with heat T
transfer. The heat transfer to a system increases
the entropy of that system and thus the level of T2

molecular disorder and randomness also increase.


The heat rejection from a system decreases the T1
entropy of the system. Hence, the entropy transfer Irreversible
heat transfer
takes place in the direction of heat flow. to block
The entropy does not transfer with work trans-
S1 S2 S
fer, because work is an organised form of energy,
(b) The entropy increase of the block
and thus can perform tasks such as raising a weight
S2 – S1 is greater than SB – SA
or generating electricity. The first law of thermody-
Entropy 231

increase in entropy of a system is called entropy


Âm s -Âm
dS
generation and is designated Sgen. = i i e se
dt
 T +S
For a reversible process, the entropy generation Q
+ gen ...(7.23)
is always zero, and thus entropy change of a system
is always equal to entropy transfer into the system. dS
For a steady flow device, =0
Consider the heating of a steel block from dt
temperature T1 to T2. The heat is supplied from a
Â
Q
constant-temperature reservoir at the temperature S gen = m (se - si ) - ...(7.24)
T
TH (TH > T2). The block and reservoir together con-
stitute an isolated system. The transfer of heat from
the reservoir to the block through the finite temper-
Entropy is considered as a measure of molecular
ature difference is irreversible, and thus entropy of
disorder or molecular randomness in the matter.
an isolated system increases. The entropy increase
When the entropy of a system increases, the posi-
of the block is greater than the entropy decrease
tions of molecules become less predictable, or the
of the reservoir. The difference of two quantities
molecules become highly disordered.
of entropy is entropy generation. It is illustrated in
All the solids have fixed positions for their mol-
Fig. 7.9.
ecules. Thus molecules can only oscillate about
their mean positions and the entropy change for
solids is lowest. It is highest in the gaseous phase.
The molecules in the gaseous phase possess a
considerable amount of kinetic energy. Thus they
A closed system does not involve mass transfer
randomly collide with each other, vibrate and can
across its boundaries thus its entropy change during
move in any direction. When heat is added to a gas-
a process is the sum of entropy transfer due to heat
eous substance, the uncertainty of molecules fur-
transfer and entropy generation within a system.
ther increase with increase in their energy level. But
Thus,
these disorganised molecules as shown in Fig. 7.10
dQ
S2 – S1 = Ú T
+ Sgen are unable to do any work. They cannot rotate a
paddle wheel. Probably the different directions of
Â
Q
or S2 – S1 = + S gen ...(7.21) molecules nullify their net effect causing the wheel
T to remain motionless.
For an adiabatic process (Q = 0), the above
expression reduces to Load
S2 – S1 = Sgen Molecules Molecules
(DS)adiabatic = Sgen ...(7.22)
Unorganised energy of molecules cannot

An open system involves mass transfer and heat


transfer across its boundaries. Therefore, the rate of Now consider a paddle wheel working on an
entropy change of an open system is the sum of net amount of gas in a closed container as shown in
rate of entropy transport due to mass flow, entropy- Fig. 7.11. The work done by the stirrer on the gas
transfer rate due to heat transfer at its boundaries is converted into internal energy of the gas causing
and the net rate of entropy generation within the rise in temperature and entropy, creating higher
system due to irreversibilities involved in the level of molecular disorder. The organised paddle-
process. Thus, for one-dimensional flow. wheel energy is converted to a highly disorganized
232 Thermal Engineering

For a unit-mass system


Tds = pdv + du ...(7.26)
Equation (7.26) is called the first Tds relation or
Gibb’s relation.
Further, the specific enthalpy is expressed as
The energy is degraded during the process,
h = u + pv
and entropy increases
In differential form,
form of energy, which cannot be converted back dh = du + pdv + vdp
to the paddle wheel by lowering the temperature or du + pdv = dh – vdp
of the gas, although a part of this energy can be Using in Eq. (7.26), we get
converted into useful work by using a heat-engine T ds = dh – vdp ...(7.27)
cycle. Therefore, the energy is degraded, entropy This equation is the second Tds relation.
increases during this process and the ability to do Thus, the entropy change during a quasi-equilib-
work is reduced. rium process can be obtained by integrating either
The quantity of energy is always preserved in relation given below:
either form during an actual process (the first law),
du pd v
but the quality of energy decreases (the second ds = + ...(7.28)
T T
law). This degradation of energy always causes an
dh vdp
increase in entropy. and ds = - ...(7.29)
The concept of entropy as a measure of dis- T T
organisation can be applied to all areas. Efficient
people have low-entropy (highly organised) lives. pv = RT and du = Cv dT
Therefore, they perform their jobs with minimum p R
delay. On the other hand, inefficient people are or =
T v
disorganised and lead high entropy lives. They per- Using in Eq. (7.28) and integrating, we get
form their jobs in disorganised manner, and hardly
ÊT ˆ Êv ˆ
meet their goals in the stipulated time. Ds = Cv ln Á 2 ˜ + R ln Á 2 ˜ ...(7.30)
We can conclude that all natural disorganised Ë T1 ¯ Ë v1 ¯
processes are in the direction of increase in mo- Similarly, for an ideal gas
lecular disorder. Further, all processes in the uni- v R
= , and dh = Cp dT
verse are irreversible, molecular disorder increases T p
day by day, and thus increases of the entropy of the Using in Eq. (7.29) and integrating, we get
universe.
Ê T2 ˆ Ê p2 ˆ
Ds = C p ln Á ˜ - R ln Á ˜ ...(7.31)
Ë T1 ¯ Ë p1 ¯
Tds

The differential form of the conservation-of-energy


equation is
dQ = dW + dU Solids and most of the liquids are considered as in-
For an internally reversible process, Eq. (7.9) compressible since their volume remains constant.
dQ = T dS Thus, for solids and liquids, dv = 0 and Eq. (7.26)
and dW = pdV reduces to
Using, we get Tds = du = Cv dT ...(7.32)
T dS = pdV + dU ...(7.25)
Entropy 233

Since Cv = Cp = C for solids and incompressible Solution


fluids, therefore du = C dT. The entropy for a process (i) 1–a–2; Reversible constant volume followed by
whose specific heat depends on temperature is constant-pressure process.
2 dT (ii) 1–b–2; Reversible adiabatic process followed by
s2 – s1 =
Ú
1 T
(kJ/kg ◊ K) ...(7.33)
C (T ) constant-pressure process.
(iii) 1–c–2; Isothermal process followed by isobaric
For constant specific heat,
process.
ÊT ˆ (iv) 1–2; Reversible process.
s2 – s1 = Cav ln Á 2 ˜ ...(7.34)
Ë T1 ¯

As discussed above, entropy is a measure of the un-


certainty of molecular position in matter in either
phase. This uncertainty of molecular position in
matter is a function of temperature and it decreases
as temperature decreases. It is due to different ener-
gy levels of molecules in the matter. The molecules
of a substance become motionless at absolute zero
temperature, thus having zero entropy. Therefore,
the entropy of a pure substance at absolute zero
temperature is zero. This statement is known as the
third law of thermodynamics.
The third law of thermodynamics is an indepen-
dent principle and it cannot be deduced from the
first and second laws of thermodynamics or any
other principle of nature. But it is important that the
third law of thermodynamics provides an absolute
reference point (S = 0 at T(K ) = 0) for the determi-
nation of entropy.
Given Reversible expansion of air
Air expands in a non-flow system revers-
ibily from 200 kPa, 167°C to 100 kPa, 112°C. Calculate State 2: p1 = 200 kPa
the change in entropy for the following processes: T1 = 167°C + 273 = 440 K
State 2: p2 = 100 kPa
(a) A reversible constant-volume process followed by
a reversible constant-pressure process T2 = 112°C + 273 = 385 K
(b) A reversible adiabatic process followed by a Cp = 1.015 kJ/kg ◊ K
reversible constant-pressure process Cv = 0.728 kJ/kg ◊ K
(c) A reversible isothermal process followed by a To find Entropy change in each combination of pro-
reversible constant-pressure process cesses.
(d) If air expands reversibily between two states
Analysis
assuming Cp and Cv are constants and are 1.015
and 0.728 kJ/kg ◊ K respectively. (i) The reversible constant-volume process followed
by reversible constant-pressure process, i.e., path
1–a–2
234 Thermal Engineering

2 dq a dq 2 dq During isothermal process (T1 = Tc ) and


Ds =
Ú
1 T
=
Ú 1 T
+
Ú a T Vc p
= 1 = 1 =
p 200
=2
a 2 V1 pc p2 100
dT dT
= Cv
Ú
1 T
+ Cp
Úa T and Tc = T1 = 440 K
The entropy change during the path 1–c–2,
Ê Ta ˆ Ê T2 ˆ
= Cv ln Á ˜ + C p ln Á ˜ ...(A) ÊV ˆ ÊT ˆ
Ë T1 ¯ Ë Ta ¯ Ds = R ln Á c ˜ + Cp ln Á 2 ˜ ...(C)
Ë V1 ¯ Ë Tc ¯
During constant-volume process, V1 = V2 and
p1 pa p Ê 385 ˆ
= = 2 = 0.287 ¥ ln(2) + 1.015 ¥ ln Á
T1 Ta Ta Ë 440 ˜¯
Êp ˆ = 0.0634 kJ/kg ◊ K
Ê 100 ˆ
or Ta = T1 Á 2 ˜ = 440 ¥ Á
Ë p1 ¯ Ë 200 ¯˜ (iv) When air expands reversibly between two states 1
and 2
= 220 K For an ideal gas, the change in specific entropy is
Using numerical values, expressed as
Ê 220 ˆ Ê 385 ˆ ÊT ˆ Êp ˆ
Using Ds = 0.728 ¥ ln Á + 1.015 ¥ ln Á Ds = Cp ln Á 2 ˜ - R ln Á 2 ˜
Ë 440 ˜¯ Ë 220 ˜¯ Ë T1 ¯ Ë p1 ¯
= – 0.5046 + 0.568 = 0.0634 kJ/kg ◊ K Ê 385 ˆ Ê 100 ˆ
= 1.015 ¥ ln Á - 0.287 ¥ ln Á
(ii) Isentropic process followed by isobaric process Ë 440 ˜¯ Ë 200 ˜¯
during path 1–b–2
= 0.0634 kJ/kg ◊ K
Ratio of specific heats, It proves that entropy is a property, and therefore,
Cp 1.015 change in its value depends on end states not on
g = = = 1.394
Cv 0.728 path followed during the process.
Temperature after isentropic expansion
g -1 g -1 Example 7.5 A thermal energy source at 800 K loses
Tb Êp ˆ g Êp ˆ g 2000 kJ of heat to a sink at (a) 500 K, and (b) 750 K.
= Á b˜ =Á 2˜ Determine which heat transfer process is more irrevers-
T1 Ë p1 ¯ Ë p1 ¯
ible.
1.394 -1
Ê 100 ˆ 1.394
Tb = 440 ¥ Á = 361.7 K Solution
Ë 200 ˜¯
The entropy change during path 1–b–2, Given Tsource = 800 K, Qsource = 2000 kJ
2 dQ b dQ 2 dQ To find More irreversible process.
Ds =
Ú
1 T
=
Ú
1 T
+
Ú
b T (i) Tsink = 500 K,
ÊT ˆ (ii) Tsink = 750 K.
= 0 + Cp ln Á 2 ˜ ...(B)
ËT ¯ b Analysis The entropy change for each reservoir can be
Ê 385 ˆ Q
= 0 + 1.015 ¥ ln Á determined as DS =
Ë 361.7 ˜¯ T
= 0.0634 kJ/kg ◊ K (i) When heat is transferred to a sink at 500 K
Qsource - 2000 kJ
(iii) When isothermal process is followed by isobaric DSsource = =
process, i.e., path 1–c–2 Tsource 800
Specific gas constant, = –2.5 kJ/kg
R = Cp – Cv = 1.015 – 0.728 Qsink + 2000 kJ
= 0.287 kJ/kg ◊ K DSsink = = = + 4 kJ/K
Tsink 500
Entropy 235

and DSTotal = DSsourse + DSsink (ii) Further, for constant volume process
= –2.5 + 4.0 = + 1.5 kJ/K p2 T
= 2
(ii) When heat is transferred to a sink at 750 K p1 T1
DSsource = –2.5 kJ/K 316.76
or p2 = ¥ (1 bar ) = 1.0588 bar
2000 kJ 300
DSsink = = 2.7 kJ/K
750 K (iii) Change in entropy: It can be calculated as
and DSTotal = –2.5 + 2.7 = + 0.2 kJ/K ÊT ˆ ÊV ˆ
The total entropy change for the process (a) is large DS = m Cv ln Á 2 ˜ + m R ln Á 2 ˜
Ë T1 ¯ Ë V1 ¯
and, therefore, more irreversible.
Since volume remains constant, thus
Example 7.6 1.5 kg of air at 1 bar, 300 K is contained ÊT ˆ
DS = mCv ln Á 2 ˜
in a rigid insulated tank. During the process, 18 kJ of work Ë T1 ¯
is done on the gas through a paddle-wheel mechanism.
Ê 316.76 ˆ
= 1.5 ¥ 0.716 ¥ ln Á
Ë 300 ˜¯
Determine the final temperature, final pressure of air in
the tank and change in entropy. Assume specific heats of
air to be constant. = 0.0584 kJ/K

Solution Example 7.7 Two kg of a certain gas with


Cp = 0.85 kJ/kg ◊ K and Cv = 0.70 kJ/kg ◊ K is allowed to
Given m = 1.5 kg expand adiabatically through a partly open valve, where-
p1 = 1 bar = 100 kPa by its volume changes from 0.0113 m3 to some higher
T1 = 300 K volume and during the process the entropy increases by
W = –18 kJ (Work done on the system) 0.8 kJ/K. Find the volume after expansion of gas.
V = constant (rigid tank)
Q = 0 (insulated tank) Solution

To find Given m = 2 kg,


(i) Final temperature of air in the tank, Cp = 0.85 kJ/kg ◊ K
(ii) Final pressure of air in the tank, and Cv = 0.70 kJ/kg ◊ K
(iii) Change in entropy of air. V1 = 0.0113 m3
Q = 0 (Since adiabatic)
Assumptions W = 0 (Since throttling process)
(i) Air is an ideal gas. DS = 0.8 kJ/K
(ii) Specific heats of air as
To find Final volume of gas.
Cp = 1.005 kJ /kg · K, Cv = 0.716 kJ/kg · K.
Analysis The entropy change can be calculated by
Analysis For an insulated and rigid tank, heat transfer
and boundary work are zero. However, the paddle-wheel ÊT ˆ ÊV ˆ
DS = m Cv ln Á 2 ˜ + m R ln Á 2 ˜
work is added to a system of air. Hence, according to the Ë T1 ¯ Ë V1 ¯
first law of thermodynamics, for a closed system, According to the first law of themodynamics,
(i) Q = Wpaddle + DU Q = W + DU
or 0 = –18 kJ + DU For free expansion Q = 0, W = 0
or DU = 18 kJ Thus, DU = 0
But DU = mCv (T2 – T1) Therefore, T1 = T2
\ 18 = 1.5 ¥ 0.716 ¥ (T2 – 300) The gas constant R = Cp – Cv = 0.85 – 0.7
or T2 = 316.76 K = 43.76°C = 0.15 kJ/kg ◊ K
236 Thermal Engineering

ÊV ˆ The total entropy change for this process is the sum


Then, DS = mR ln Á 2 ˜ of these two processes. Since the block and lake together
Ë V1 ¯
form an isolated system,
Ê V2 ˆ
0.8 = 2 ¥ 0.15 ¥ ln Á
Ë 0.0113 ˜¯ DSTotal = DSiron + DSlake = –12.65 + 16.97
or
Ê V2 ˆ = 4.32 kJ/K
ln Á
Ë 0.0113 ˜¯
or = 2.667
Example 7.9 One kg of ice at –20°C is exposed to the
or V2 = 0.0113 ¥ exp (2.667)
atmosphere which is at 20°C. The ice melts and comes
= 0.1627 m3
into thermal equilibrium with the atmosphere. Calculate
the entropy increase of the universe. Take Cp of ice as
Example 7.8 A 50-kg block of iron casting at 500 K
2.093 kJ/kg ◊ K and latent heat of the fusion of ice as
is thrown into a large lake which is at a temperature of
334.5 kJ/kg.
285 K. After the iron block reaches thermal equilibrium
with the lake water, determine
Solution
(a) Entropy change of the iron block,
Given m = 1 kg of ice
(b) Entropy change of the lake water,
T1 = –20°C + 273 = 253 K
(c) The total entropy change during this process.
TA = 20 + 273 = 293 K
Assume average specific heat for the iron block as
Latent heat of fusion = 334.5 kJ/kg
0.45 kJ/kg ◊ K.
Sp. heat of ice, Cp, ice = 2.093 kJ/kg ◊ K
Solution To find Entropy change of the universe.
Given m = 50 kg Ts = 500 K
Tlake = 285 K
To find
(i) Entropy change of the iron block,
(ii) Entropy change of the lake, and
(iii) Total entropy change during the process.
Analysis
(i) The entropy change of the iron block
dQ dT
D Siron =
Ú
T Ú
= m Cav
T
Ê T2 ˆ
= m Cav ln Á ˜
Ë T1 ¯ Assumption Sp. heat of water, Cpw = 4.187 kJ/kg ◊ K
Ê 285 ˆ
= 50 ¥ 0.45 ¥ ln Á ˜¯
Analysis Entropy change during heating of ice from
Ë
= –12.65 kJ/K 500 –20°C to 0°C
(ii) The entropy change of the lake water Process 1-2;
The heat receive a by the lake water at constant 2 ÊdQˆ 2 dT
temperature
= The heat rejected by the iron block
DS1–2 =
Ú1
ÁË ˜ = m
T ¯ Ú
1
Cp, ice
T

Qlake = mCav (Ts – Tlake) ÊT ˆ Ê 273 ˆ


= m C p, ice ln Á 2 ˜ = 1 ¥ 2.093 ¥ ln Á
= 50 ¥ 0.45 ¥ (500 – 285) Ë T1 ¯ Ë 253 ˜¯
= 4837.5 kJ = 0.1592 kJ/K
Q 4837.5
and DSlake = lake = = 16.97 kJ/K
Tlake 285
Entropy 237

Entropy change during fusion of ice (Process 2-3) m2 = 1 kg of ice


(Q = m hfg) T2 = 0°C
3 ÊdQˆ Q 1 ¥ 334.5
DS2–3 =
Ú2
ÁË ˜ =
T ¯ T2
=
273
= 1.2253 kJ/K
To find
Latent heat of fusion = 335 kJ/kg
The net change of entropy.
Entropy change during sensible heating of water from
0°C to 20°C, Assumption Specific heat of water, Cpw = 4.187
4 ÊdQˆ
kJ/kg ◊ K
DS3–4 =
Ú Á
3 Ë T ¯
˜
Analysis On mixing of ice and water, say final tem-
4 Ê dT ˆ Ê T4 ˆ
= m
Ú
3
Cpw Á
Ë T ˜¯
= m Cpw ln Á ˜
Ë T3 ¯
perature is Tf .
Heat loss by water
Ê 293 ˆ
= 1 ¥ 4.187 ¥ ln Á = 0.296 kJ/K = Heat gain by the ice–water system
Ë 273 ˜¯ = Heat gain in fusion of ice + heating of water
Total entropy change of the ice-and-water system m1 Cpw (T1 – Tf )
DS1–4 = D S1–2 + DS2–3 + DS3–4 = m2 [latent heat of fusion + Cpw (Tf – 0)]
= 0.1592 + 1.2253 + 0.296 = 1.680 kJ/K Since temperature difference is involved in the rela-
Heat absorbed by the ice–water system from the tion, therefore, either temperature scale can be used.
atmosphere to reach 20°C Using Celsius scale for calculation of Tf of mixture,
= Heat absorbed in warming of ice 4 ¥ 4.187 ¥ (27 – Tf ) = 1 ¥ (335 + 4.187Tf )
+ Latent heat of fusion 452.196 – 16.742 Tf = 335 + 4.187Tf
+ Heat absorbed during sensible heating 20.929Tf = 117.196
of water to raise its temperature from Tf = 5.5999°C ª 5.6°C = 278.6 K
0°C to 20°C
Q = m[Cp, ice DTwarming + hfg + Cpw DTheating] Entropy change using temperature in K
= (1 kg) ¥ [(2.093 kJ/kg ◊ K) ¥ (0 – (– 20)) (°C) (i) Entropy decrease of 4 kg of water from 27°C to
+ 334.5 kJ/kg] 5.6°C
Ê Tf ˆ
+ (4.187 kJ/kg ◊ K) ¥ (20 – 0) (°C) DSwater1 = m1 Cpw ln Á ˜
= 460 kJ ËT ¯1

Entropy decrease of the atmosphere = 4 ¥ 4.187 ¥ ln (278.6/300)


Q 460 = –1.239 kJ/K
DSatm = = = 1.570 kJ/K
T 293 (ii) Entropy change of 1 kg of ice = Entropy increase
Entropy change of the universe during fusion of ice
= Entropy increase of the ice–water system Latent heat È 335 ˘
– Entropy decrease of atmosphere DSice = m2 = 1¥ Í ˙
T0 Î 273 ˚
= 1.680 – 1.570 = 0.109 kJ/K = 1.227 kJ/K
(iii) Entropy increase of 1 kg of water from 0°C to
Example 7.10 4 kg of water at 27°C is mixed with
5.6°C
1 kg of ice at 0°C. Assuming adiabatic mixing, determine Ê Tf ˆ
the final temperature of the mixture of water and ice. D Swater 2 = Cpw ln Á ˜
Ë T0 ¯
Calculate the net change of entropy. Assume enthalpy of
È Ê 278.6 ˆ ˘
fusion of ice as 335 kJ/kg. = 1 ¥ Í4.187 ¥ ln Á
Î Ë 273 ˜¯ ˙˚
Solution = 0.0850 kJ/K
Total entropy increase of 1 kg of ice–water system
Given Water mixing with ice
= DSice + DSwater 2
m1 = 4 kg of water
DStotal = 1.227 + 0.0850 = 1.312 kJ/K
T1 = 27°C or 300 K
238 Thermal Engineering

Net entropy change due to mixing of 1 kg of ice (iii) The process 3–4, a polytropic cooling process
and 4 kg of water
ÔÏ ÊT ˆ Ê p ˆ Ô¸
DSnet = DStotal + DSwater = 1.312 – 1.239 DS3–4 = m ÌC p ln Á 4 ˜ - R ln Á 4 ˜ ˝
ÓÔ Ë T3 ¯ Ë p3 ¯ Ô˛
= 0.730 kJ/K

Example 7.11 4 kg of air is compressed from 40°C = 4 ¥ ÏÌ1.005 ¥ ln Ê 453 ˆ - 0.288 ¥ ln Ê 125 ˆ ¸˝
Á ˜
Ë 523 ¯ Á ˜ Ë 257 ¯ ˛Ô
and 125 kPa to 250°C and 875 kPa. It is then throttled to ÓÔ
257 kPa. Finally, it is cooled to a pressure of 125 kPa and = 4 ¥ [– 0.144 + 0.2075] = 0.2527 kJ/K
180°C. Calculate the overall change in entropy and also Overall change in entropy,
for each process. D STotal = DS1–2 + DS2–3 + DS3–4
Take Cp = 1.005 kJ/kg ◊ K and Cv = 0.717 kJ/kg ◊ K = – 0.178 + 1.411 + 0.2527
= 1.485 kJ/K
Solution
Example 7.12 Show that for an ideal gas, the slope
Given Compression, throttling and cooling of air.
of the constant-pressure line is less than that of the con-
m = 4 kg
stant-volume line.
Cp = 1.005 kJ/kg K
Cv = 0.717 kJ/kg K Solution For 1 kg of ideal gas, first Tds relation.
State 1: p1 = 125 kPa Tds = du + pdv = Cv dT + pdv
T1 = 40°C + 273 = 313 K For a constant-volume process, dv = 0
State 2: p2 = 875 kPa Tds = Cv dT
T2 = 250°C + 273 = 523 K T Ê dT ˆ
= Á
Ë ds ˜¯ v = c
State 3: p3 = 257 kPa or ... (i)
Cv
h2 = h3
Again from the second T ds relation
State 4: p4 = 125 kPa
Tds = dh – vdp
T4 = 180°C + 273 = 453 K
For a constant-pressure process, dp = 0
To find Change in entropy for each process and net Tds = dh = Cp dT
change. T Ê dT ˆ
= Á
Ë ds ˜¯ p = c
or ...(ii)
Analysis The gas constant for air Cp
R = C p – Cv
Ê dT ˆ
The quantity Á represents the slope of the line
= 1.005 – 0.717 = 0.288 kJ/kg K Ë ds ˜¯
(i) The process 1–2, a polytropic process on T–s diagram and Cp > Cv for ideal gases, therefore
ÔÏ Ê T2 ˆ Ê p ˆ Ô¸ Ê dT ˆ Ê dT ˆ
DS1– 2 = m ÌC p ln Á - R ln Á 2 ˜ ˝
ÓÔ Ë T1 ˜¯ Ë p1 ¯ Ô˛ ÁË ˜
ds ¯ p = c
< ÁË ˜
ds ¯ v = c
= 4 ¥ ÏÌ1.005 ¥ ln Ê 523 ˆ - 0.288 ¥ ln Ê 875 ˆ ¸˝ Therefore, as shown in Fig. 7.14, the slope of constant
Á ˜
Ë 313 ¯ Á ˜
Ë 125 ¯ ˛Ô
ÓÔ pressure line is less than the constant volume line.
= 4 ¥ [0.516 – 0.5604] = – 0.178 kJ/K
T
(ii) The process 2–3, throttling process
v=C
h 2 = h3
or T2 = T3 C
=
p
ÏÔ Ê p ˆ ¸Ô
\ DS2–3 = m Ì- R ln Á 3 ˜ ˝
ÔÓ Ë p2 ¯ Ô˛
s
Ï Ê 257 ˆ ¸
= 4 ¥ Ì- 0.288 ¥ ln Á ˝
ÓÔ Ë 875 ˜¯ ˛Ô
= 1.411 kJ/K
Entropy 239

Example 7.13 Two identical bodies of same heat Ê T2 T ˆ


or ÁË T 2 ˜¯ = 1
capacity are at the same initial temperature T1 . A re-
1
frigerator operates between these two bodies, until one
body is cooled to the temperature T2 . If the bodies are at T12
or T =
constant temperatures and do not undergo any change of T2
phase, prove that the minimum amount of work needed by Substituting in Eq. (i)
the refrigerator is ÈÊ T 2 ˆ ˘
ÈT 2 ˘ w = C p ÍÁ 1 ˜ + T2 - 2 T1 ˙ (Proved)
Cp Í 1 + T2 - 2 T1 ˙ ÍÎË T2 ¯ ˙˚
ÍÎ T2 ˙˚
Example 7.14 A mass m of fluid at a temperature T1
Solution Consider two bodies A and B. Heat is re- is mixed with an equal mass of the same fluid at a tem-
moved from the body A to bring it to T2. perature T2. Prove that the resultant change in entropy of
Heat removed per kg from the body A the universe is
qL = Cp (T1 – T2) 2 mCp ln (T1 + T2 )
Heat discharged per kg by refrigerator to the body B 2 T1 T2
qH = q + w = Cp (T – T1)
where T becomes the temperature of the body B after Solution If equal masses of two fluids are mixed then
heat addition. the equilibrium temperature T (say) can be obtained as
Work input to the refrigerator, m1Cp T1 + m2 Cp T2 = mCp T
w = Cp (T – T1) – Cp (T1 – T2)
( m1 T1 + m2 T2 ) (T + T )
= Cp (T – 2T1 + T2) ...(i) or T = = 1 2
m 2
The decrease of entropy of the body A ∵ m1 = m2 and m = 2m1
2 Ê dT ˆ ÊT ˆ The entropy change can be expressed as
DsA =
Ú
1
Cp Á
Ë T ˜¯
= Cp ln Á 2 ˜
Ë T1 ¯ T dT T2 dT
The entropy increase of the body B
DS = m C p
Ú
T1 T
- m Cp
Ú
T T
2 Ê dT ˆ ÊTˆ ÊTˆ ÊT ˆ
D sB =
Ú
1
Cp Á
Ë T ˜¯
= C p ln Á ˜
Ë T1 ¯
= m Cp ln Á ˜ – mCp ln Á 2 ˜
Ë T1 ¯ ËT¯
Net entropy change of the reversible refrigeration Ê T2 ˆ
cycle = 0 = mCp ln Á ˜
dq Ë T1 T2 ¯
Thus, cyclic
Ú
T
= DsA + D sB = 0
Substituting the value of T in the above relation, we
ÊT ˆ ÊTˆ get
Therefore, Cp ln Á 2 ˜ + Cp ln Á ˜ = 0 DS = mCp ln [(T1 + T2)2/(4T1 T2)]
Ë T1 ¯ Ë T1 ¯
ÏÔÊ T + T ˆ 2 ¸Ô
ÊT T ˆ 1 2
or ln Á 2 2 ˜ = 0 = mCp ln ÌÁ ˜ ˝
Ë T1 ¯ ÓÔË 2 T1 2¯ ˛
T Ô
Ê T +T ˆ
Body B = 2mCp ln Á 1 2 ˜ Proved
Ë 2 T1 T2 ¯
(T1 + T2 )
w R The arithmetic mean is always greater than
2
ÏÊ T + T ˆ 2 ¸Ô
geometric mean (T1 T2 ) . Therefore, ln ÔÌ 1 2
Á ˜ ˝
Body A
is always positive. ÓÔË 2 T1 T2 ¯ ˛Ô
240 Thermal Engineering

Example 7.15 A heat engine having a working sub-


or Wmin = m Cp ( T1 - T2 ) 2 (Proved)
stance of mass m and specific heat Cp works between a
source at a temperature T1 and a sink at a temperature
T2. Prove that the maximum obtainable work from such Example 7.16 Show that the entropy change between
an engine is given by states1 and 2 in a polytropic process pvn = constant is
given by the following relation
{ }
2
Wmax = mCp T1 - T2 n-g ÊT ˆ
(a) s2 – s1 = R ln Á 2 ˜ , and
(g - 1)( n - 1) Ë T1 ¯
Solution The reversible engine gives maximum Êp ˆ
n-g
work. Since it works between the source temperature (b) s2 – s1 = R ln Á 2 ˜
n (g - 1) Ë p1 ¯
T1 and the sink temperature T2. Let the engine produces
work at the temperature T.
Solution For a polytropic process,
p1 v n1= p2 v2n
n -1
1- n
T2 Êv ˆ Êp ˆ n
and = Á 2˜ = Á 2˜
T1 Ë v1 ¯ Ë p1 ¯
Taking natural log on both sides
ÊT ˆ Êv ˆ n - 1 Ê p2 ˆ
ln Á 2 ˜ = (1 - n) ln Á 2 ˜ = ln Á ˜ ...(i)
Ë T1 ¯ Ë v1 ¯ n Ë p1 ¯
The general relations for change in entropy of a
Fig. 7.15
system
ÊT ˆ Êv ˆ
Net entropy change at the engine = Entropy decrease s2 – s1 = Cv ln Á 2 ˜ + R ln Á 2 ˜ ...(ii)
Ë T1 ¯ Ë v1 ¯
of source – Entropy increase of sink
ÊT ˆ Êp ˆ
T dQ T2 dQ and s2 – s1 = C p ln Á 2 ˜ - R ln Á 2 ˜ ...(iii)
DS =
ÚT1 T
-
ÚT T
Ë T1 ¯ Ë p1 ¯
Êv ˆ Ê 1 ˆ Ê T2 ˆ
ÊTˆ ÊT ˆ Using ln Á 2 ˜ = - Á ln from Eq. (i) in
= m C p ln Á ˜ - mCp ln Á 2 ˜
ËT¯ Ë v1 ¯ Ë n - 1˜¯ ÁË T1 ˜¯
Ë T1 ¯
Eq. (ii), we get
For a reversible engine operating in a cycle ÊT ˆ ÊT ˆ
R
DS = 0 s2 – s1 = Cv ln Á 2 ˜ - ln 2
Ë T1 ¯ ( n - 1) ÁË T1 ˜¯
Therefore,
R
ÊTˆ ÊT ˆ we have Cv = , then
mCp ln Á ˜ – mCp ln Á 2 ˜ = 0 g -1
Ë T1 ¯ ËT¯
R ÊT ˆ R ÊT ˆ
ÊTˆ ÊT ˆ s2 – s1 = ln 2 - ln 2
or ln Á ˜ = ln Á 2 ˜ g - 1 ÁË T1 ˜¯ ( n - 1) ÁË T1 ˜¯
Ë T1 ¯ ËT¯
Ê 1 1 ˆ ÊT ˆ
T T or = Á - ˜ R ln Á 2 ˜
or = 2 or T 2 = T1 T2 Ë g - 1 n - 1¯ Ë T1 ¯
T1 T
n-g ÊT ˆ
or T = T1 T2 or s2 – s1 = R ln Á 2 ˜ Proved (a)
(g - 1) ( n - 1) Ë T1 ¯
Now the work done by the engine
= Heat supplied – heat rejected ÊT ˆ Ê n - 1ˆ Ê p2 ˆ
Using ln Á 2 ˜ = Á ln from Eq. (i) in
Wmin = mCp (T1 – T) – mCp (T – T2) Ë T1 ¯ Ë n ˜¯ ÁË p1 ˜¯
= mCp (T1 – 2T + T2) Eq. (iii), we get
= mCp (T1 - 2 T1 T2 + T2 )
Entropy 241

n -1 Êp ˆ Êp ˆ È ÊT ˆ ˘
s2 – s1 = Cp ln Á 2 ˜ - R ln Á 2 ˜ Win = mCp ÍTH ln Á H ˜ - (TL - TL )˙
n Ë p1 ¯ Ë p1 ¯ ÍÎ Ë TL ¯ ˙˚
È Ê n - 1ˆ Ê g R ˆ ˘ Ê p2 ˆ Using numerical values
= ÍÁ
Ë ˜¯ ÁË g - 1˜¯ - R ˙ ln Á p ˜ È Ê 295 ˆ ˘
ÍÎ n ˙˚ Ë 1 ¯ Win = 2 ¥ 4.184 Í298 ¥ ln Á
Ë ˜¯ - ( 298 - 275)˙
È gR ˘ Î 275 ˚
Í∵ Cp = ˙ = 7.833 kJ
Î g - 1˚
È ng - g - ng + n ˘ Ê p2 ˆ Example 7.18 A novel reversible heat engine plot on
= Í ˙ R ln Á ˜
Î n (g - 1) ˚ Ë p1 ¯ a T–s diagram is as circle. The maximum and minimum
temperatures are 1100 K and 200 K, respectively and the
n-g Êp ˆ
= R ln Á 2 ˜ Proved (b) maximum entropy change in the cycle is 2 kJ/K. Calculate
n (g - 1) Ë p1 ¯ the heat added to the cycle, heat rejected, net work output
and the thermal efficiency of the cycle.
Example 7.17 Calculate the minimum work required
to chill 2 kg of drinking water from a temperature of 25°C Solution
to 2°C. The Cp of water as 4.184 kJ/kg ◊ K.
Given T–s diagram is a circle as shown in Fig. 7.16.
Solution T1 = 1100 K
T2 = 200 K
Given m = 2 kg of water
DS = 2 kJ/K
Tatm = TH = 25°C = 298 K
TL = 2°C = 275 K, To find
Cp = 4.184 kJ/kg ◊ K (i) Heat addition to engine,
(ii) Heat rejection by engine,
To find The minimum (or reversible) work required.
(iii) Efficiency of the engine.
Assumption Assuming constant pressure during cool-
Analysis Mean temperature
ing process.
(T1 + T2 )
Analysis During cooling process of drinking water T3 =
2
Entropy decrease of water, (1100 + 200)
T3 = = 650 K
Ê TH ˆ 2
DS1 = mCp ln Á
Ë TL ˜¯ From Fig. 7.16,
And heat removed from the water, Area 1231 = area 1341
QL = m Cp (TH – TL) = area of semicircle
Heat supplied to the atmosphere,
T
QH = Win + QL
or QH = Win + m Cp (TH – TL) 4
1100 K
The entropy increase of atmosphere
1 3
Win + m Cp (TH - TL )
= 200 K
TH 2

For a refrigerator operating on the reversible cycle the


net entropy change is zero. Therefore, 2 KJ/K
Win + mC p (TH - TL ) ÊT ˆ 5 6
- mCp ln Á H ˜ = 0
TH Ë TL ¯ s

Ê ˆ Fig. 7.16
or Win = mCp ln Á TH ˜ TH – mCp (TH – TL)
Ë L¯
T
242 Thermal Engineering

Êpˆ Assumption The specific heat of water as 4.187 kJ/kg ◊ K.


= 0.5 ¥ Á ˜ ¥ D 2
Ë 4¯ Analysis
Êpˆ (i) Heat transferred to water from reservoir,
= 0.5 ¥ Á ˜ ¥ (Ds ¥ DT )
Ë 4¯ Q = mCp (DT )
where Ds = 2 kJ/K or Q = 1 ¥ 4.187 ¥ (90 – 0) = 376.83 kJ
and DT = 1100 – 200 = 900 K (a) Entropy change of water,
Êpˆ ÊT ˆ
or Area 1231 = 0.5 ¥ Á ˜ ¥ 2 ¥ 900 = 706.85 kJ DSw = mCp ln Á 2 ˜
Ë 4¯
Ë T1 ¯
Heat added = Upper semicircle area
+ lower rectangle area Ê 363 ˆ
= 1 ¥ 4.187 ¥ ln Á
Ë 273 ˜¯
= 1.193 kJ/K
Thus, heat added = area (1341) + area (1365)
= 706.85 + 2 ¥ 650 = 2006.85 kJ (b) Entropy change of reservoir,
Heat rejection = Area of rectangle 1365 Q 376.83
(DS)reservoir = = = 1.038 kJ/K
– area of semicircle 1321 T2 363
= 2 ¥ 650 – 706.85 = 593.15 kJ (c) Entropy change of universe,
The work done in the cycle DSuniverse = DS2 – DSreservoir
= Head added – heat rejection = 1.193 – 1.038 = 0.155 kJ/K
= 2006.85 – 593.15 = 1413.7 kJ (ii) Given
Thermal efficiency of cycle m = 1 kg
Work done
= T1 = 273,
Heat supplied
T2 = 40 + 273 = 313 K,
1413.7 T3 = 90 + 273 = 363 K
hth = = 0.7044 or 70.44%
2006.85 Heat transferred to water from reservoir at 40°C
Example 7.19 (a) 1 kg water at 0°C is brought into Q1 = m Cp (T2 – T1)
contact with a heat reservoir at 90°C. When water has = 1 ¥ 4.187 ¥ (40 – 0) = 167.48 kJ
reached 90°C, find Heat transferred to water from reservoir at 90°C
(i) Entropy change of water, Q2 = m Cp (T3 – T1)
(ii) Entropy change of the reservoir, = 1 ¥ 4.187 ¥ (90 – 40) = 209.35 kJ
(iii) Entropy change of the universe. Now entropy change of the two reservoirs
(b) If water is heated from 0°C to 90°C by first bringing Q1 Q2
(DS)res = +
it in contact with the reservoir at 40°C and then with a T2 T1
reservoir at 90°C, what will be the entropy change of the
167.48 209.35
universe? (DS)res = +
313 363
(c) Explain how water might be heated from 0°C to 90°C
= 0.535 + 0.576
with almost no change in entropy of the universe?
= 1.111 kJ/kg
Solution The entropy of water will remain same for the
same temperature difference. Thus entropy
Given m = 1 kg change of the universe,
T1 = 0°C = 273 K T2 = 90°C = 363 K DSuniverse = (DS)w – (DS)res
To find = 1.193 – 1.111
(i) Entropy change of water, = 0.0842 kJ/K
(ii) Entropy change of reservoir, (iii) If water is heated reversibly, then net entropy
change of universe will be zero.
(iii) Entropy change of universe.
Entropy 243

Summary
Entropy is an abstract property. It is the quantita- entropy generated during an irreversible
tive measure of molecular disorder within a sys- process is called entropy generation, Sgen.
tem. The entropy change dS is defined as 2 dQ
ÊdQˆ
dS = Á (kJ/K)
S2 – S1 =
Ú
1 T
+ S gen

Ë T ˜¯ int, rev entropy change of a system or its surround-


total entropy (extensive property) change ings can be negative during a process, but entropy
during a process is obtained by integrating the generation cannot. The increase of entropy prin-
above relation. ciple can be summarised as
> Ï 0 irreversible process
2 ÊdQˆ
DS =
Ú 1 ËÁ T ¯˜
(kJ/K) Sgen = ÔÌ 0 reversible process
int, rev
< ÔÓ 0 impossible process
entropy per unit mass s is called specific
entropy and it is an intensive property. The Tds relations are
change in its value during a process is Tds = du + pdv
Tds = dh – vdp
2 Êd qˆ
Ds =
Ú 1
ÁË T ˜¯
int, rev
(kJ/Kg ◊ K) entropy change during a process
can be obtained by integrating these relations,
entropy change for a cyclic process is and we get
explained by Clausius inequality as ÊT ˆ Êv ˆ
Ds = Cv ln Á 2 ˜ + R ln Á 2 ˜
dQ Ë T1 ¯ Ë v1 ¯
Ú T
£0

where equality sign holds for an internally or ÊT ˆ Êp ˆ


and Ds = Cp ln Á 2 ˜ – R ln Á 2 ˜
totally reversible process and the inequality for Ë T1 ¯ Ë p1 ¯
irreversible process. The entropy for all actual Cp = Cv =
processes always increases and its increase of Cav and v1 = v2, thus
entropy principle
ÊT ˆ
dQ DS = Cav ln Á 2 ˜
dS ≥ Ë T1 ¯
T
or DSisolated ≥ 0 entropy remains con-
stant is called an isentropic process.

Glossary
Entropy A measure of molecular disorder or molecular Entropy generation Entropy generated during a pro-
randomness in matter cess
Specific entropy Entropy per unit mass Third law of thermodynamics Entropy of a pure sub-
stance at absolute zero temperature is zero
244 Thermal Engineering

Review Questions
1. Define entropy and prove that entropy is a show that the entropy is a property of a system.
property. 9. Derive a general expression for the change in
2. Prove that two isentropic lines cannot intersect entropy of a system during a process.
each other. 10. Derive an expression for change in the entropy of
3. State and prove Clausius’ theorem. a system during a constant-volume process.
4. What is the Clausius inquality? Discuss. 11. Derive an expression for change in the entropy of
5. What is entropy generation? Explain. a system during a constant-pressure process.
6. State and prove Clausius inequality. 12. Derive an expression for change in the entropy of
7. What is the concept of entropy? Explain. a system during an isothermal process.
dQ 13. Derive an expression for change in the entropy of
8. Prove that dS = for a reversible process and a system during a polytropic process.
T

Problems
1. 300 kW of heat is supplied at a constant 4.00 kJ of work. Calculate the heat transferred to
temperature of 290°C to a heat engine. The air and the entropy change of the universe.
engine rejects heat at 8.5°C. The following results [2.2 kJ, 0.016 kJ/kg]
were recorded: 5. 1 kg of air expands from 400 kPa, 550°C to
(a) 215 kW is rejected. 100 kPa. During the process, the entropy change
(b) 150 kW is rejected. of the surroundings is 0.135 kJ/K. Determine
the minimum possible temperature of air in the
(c) 75 kW is rejected.
system.
Classify which of the results reports a reversible,
6. A 10-kg metal piece with constant specific heat of
irreversible or impossible cycle.
0.9 kJ/kg K at 200°C is dropped into an insulated
[(a) irreversible cycle, (b) reversible cycle,
tank which contains 100 kg of water at 20°C.
(c) impossible cycle]
Determine the final equilibrium temperature and
2. Air expands steadily through a turbine from total change in entropy for this process.
200 kPa, 60°C to 90 kPa, 15°C. The entropy
[23.8°C, 1.183 kJ/K]
of the surroundings decreases by 0.04 kJ/kg K.
7. 1 MJ heat is supplied from a thermal reservoir at
Would such a process be irreversible, reversible
800 K to a thermal reservoir at 400 K. Calculate
or impossible? Why? [impossible process]
the entropy change of the universe resulting from
3. A refrigerator removes heat from a cold space at
this heat-exchange process. [1.25 kJ/K]
2°C at a rate of 300 kJ/min and rejects heat to
8. During isothermal heat addition process of a
the kitchen air at 26°C at a rate of 345 kJ/min.
Carnot cycle, 800 kJ heat is added to the working
Determine whether the refrigerator violates the
fluid from a source of 527°C. Determine
second law on the basis of (a) Clausius inequality,
(a) change in entropy of the working fluid,
and (b) Carnot principle.
[Voilates (a), Satisfy (b)] (b) change in entropy of the source,
4. Heat is transferred from a very large mass of water (c) total entropy change during the process.
at 90°C to 0.1 kg of air that expands irreversibly [(a) 1 kJ/K, (b) –1 kJ/K, (c) 0]
from 400 kPa, 60°C to 150 kPa, 35°C, doing 9. A rigid tank contains air at 35°C, which is being
stirred by a paddle wheel. The paddle wheel does
Entropy 245

500 kJ of work on the air. During the stirring 14. Air expands irreversibly in a cylinder from
process, the air temperature remains constant, 280 kPa, 60°C to 140 kPa. The air does
owing to transfer of heat between the system and 30 kJ/kg of work and 14 kJ/kg of heat is removed
surroundings, which is at 15°C. Calculate from the air during the expansion. The initial
(a) change in entropy of air in the tank, and volume of air is 0.00878 m3. Calculate the
(b) change in entropy of the surroundings. specific entropy change of air. [0.0095 kJ/k]
Does this process satisfy the increase-of-entropy 15. 3 kg of an ideal gas is expanded from a pressure
principle? of 7 bar and a volume of 1.5 m3 to a pressure
10. Air expands irreversibly from 3 bar, 200°C to of 1.4 bar and a volume of 4.5 m3. Determine:
1.5 bar, 105°C. Calculate the specific entropy (a) Specific gas constant, (b) initial and final
change of air. [– 4.242 kJ/kg K] temperature, and (c) change in entropy during
11. Air flows through an insulated horizontal duct. heat exchange.
The pressure and temperatures are measured at Take Cp = 1.05 kJ/kg ◊ K
two different stations A and B. They are pA = [(a) 0.3 kJ/kg ◊ K, (b) 893.67°C, 427°C,
0.2 MPa; TA = 150°C; pB = 0.18 MPa, and TB = (c) –0.1606 kJ/K]
130°C. Determine the specific entropy change of 16. Two boilers discharge an equal amount of steam
air. [– 0.533 kJ/kg ◊ K] into a steam main. The steam from one boiler is at
12. A rigid cylinder containing 0.006 m3 of nitrogen 18 bar and 380°C and from the other boiler, it is
(R = 0.296 kJ/kg ◊ K) at 1.04 bar, 150°C is 0.95 dry at 18 bar. Determine
heated reversibly until the temperature reaches (a) equilibrium condition after mixing,
90°C. Calculate the change of entropy and heat (b) the loss of entropy by high temperature
supplied. Take g = 1.4. [0.00125 kJ/K, 0.406 kJ] steam,
13. Heat is transferred from a very large mass of (c) gain of entropy by low temperature steam,
water at 90°C to 0.1 kg of oxygen that expands (d) net increase or decrease of entropy.
irreversibly in a cylinder fitted with a piston from [(a) 63.7°C (b) 0.383 kJ/kg ◊ K (c) 0.470 kJ/kg ◊ K
400 kPa, 60°C to 150 kPa, 350°C. The oxygen (d) 0.854 kJ/kg ◊ K]
does 4.0 kJ of work. Calculate (a) the heat
transfer to the oxygen, and (b) entropy change of
the universe. [(a) 2.34 kJ (b) 0.0118 kJ/K]

Objective Questions
1. Entropy is
(a) an extensive property (c) pressure volume product
(b) an abstract property (d) none of the above
(c) a function of quality of heat 4. When heat is absorbed by a gas, the change in
(d) all of the above entropy of gas is considered to be
2. Entropy is a function of (a) zero (b) positive
(a) work transfer (b) volume (c) negative (d) none of the above
(c) temperature (d) pressure 5. The condition of reversibility of a cycle is
dQ dQ
3. For any thermodynamic process, the area under
the T–s curve represents
(a)
Ú T
<0 (b)
Ú T
>0

dQ
Ú
(a) work transfer
(c) =0 (d) none of the above
(b) heat transfer T
246 Thermal Engineering

6. The condition of irreversibility of a cycle is (a) more (b) less


dQ dQ (c) equal (d) none of the above
(a)
Ú T <0 (b)
Ú T
>0
11. For an irreversible process, the entropy of the
system is
dQ
(c)
Ú T =0 (d) none of the above (a) zero (b) increased
(c) decreased (d) none of the above
dQ
7. If
Ú T
> 0 then the cycle is 12. Entropy of the universe always tends to
(a) reversible (b) irreversible (a) zero (b) increase
(c) impossible (d) none of the above (c) decrease (d) none of the above
8. During reversible adiabatic process, the entropy 13. Entropy of an isolated system increases when a
(a) increases (b) decreases process is
(c) remains constant (d) none of the above (a) reversible (b) irreversible
9. During a throttling process, the entropy (c) ideal (d) none of the above
(a) increases (b) decreases 14. Entropy of water at 0°C is assumed to be
(c) remains constant (d) none of the above (a) 0 (b) 1
10. On a T–s diagram, the slope of constant-pressure (c) –1 (d) none of the above
lines in comparison with constant-volume lines is

14. (a) 13. (b) 12. (b) 11. (b) 10. (b) 9. (a)
8. (c) 7. (c) 6. (a) 5. (c) 4. (b) 3. (b) 2. (c) 1. (d)
Answers
Availability and Irreversibility 247

8
Availability and Irreversibility

Introduction
The second law of thermodynamics asserts that energy is degraded during a process; entropy is generated
which reduces capacity to do work. The availability (exergy) concept is also derived from the second law
of thermodynamics and it is closely related with reversibility and entropy. The availability is the maximum
useful work which can be obtained from the system when it operates reversibly at a given state in a given
environment. The irreversibility (also called exergy destruction) is the waste work potential during a process
as a result of irreversibilities. Here, we have developed the availability function and availability balance
relation for closed and open systems. The second law efficiency for various system is also taken up in this
chapter.

(c) Heat obtained from combustion of fossil


fuels
It becomes clear from the first and second laws of (d) Geothermal energy
thermodynamics that energy has quantity as well
High-grade energy can be converted directly in
as quality. Therefore, the sources of energy can be
any from of energy with its equivalent quantity, but
divided into two categories as high-grade energy
low-grade energy such as heat cannot be converted
and low-grade energy.
directly into high-grade energy. The conversion of
High-Grade Energy low-grade energy into high-grade energy requires
(a) Mechanical work a cyclic process and its complete conversion is
(b) Electrical energy impossible.
(c) Water power or hydraulic energy According to the second law of thermodynamics,
(d) Wind energy the part of low-grade energy that can be converted
(e) Tidal energy into work is known as available energy and the part
(f ) Kinetic energy of a jet which is rejected as waste is known as unavailable
energy.
Low Grade Energy Further, the quality of energy is always degraded
(a) Thermal energy during a process, hence entropy is generated, and
(b) Heat obtained from nuclear fission or fusion the opportunities to produce work are reduced.
248 Thermal Engineering

Reservoir at TH

QH

As discussed above, the quality of energy is a


measure of the work potential of a given quantity of Reversible
energy. A reversible heat engine operating between heat Wrev, net
engine
a source temperature TH and a sink temperature TL
has a work potential as
È T ˘ Q0
Wrev, net = hrev ¥ QH = Í1 - L ˙ QH ...(8.1)
Î TH ˚ Atmosphere at T0

where QH is the amount of heat supplied to the (a) A reversible heat engine converts a constant energy
reversible heat engine by the source at TH. source QH into network Wrev,net.
We can increase the work potential either by in- T
creasing the source temperature TH or by lowering
the sink temperature TL. However, the low tempera- TH

ture TL has a large impact on the work potential, Available


energy
but the lowest temperature cannot be below the at-
T0
mospheric temperature T0. Then Eq. (8.1) becomes
Unavailable
Ê T ˆ energy
Wrev, net = QH Á1 - 0 ˜ ...(8.2)
Ë TH ¯ 0
s
The quantity given at the right side of Eq. (8.2) (b) T–s diagram for constant energy source as available and
is the available portion of the total energy quantity unavailable energy portions
QH. Therefore, the available and unavailable energy
can be defined as follows:
The available energy (A) is that portion of the unavailable energy to overcome the irrerversibili-
amount of heat energy supplied to a reversible ties associated with processes, thus increasing the
engine, which could be converted into useful work. unavailable energy of universe.
The maximum work can only be obtained from
a reversible heat engine, because no engine is
more efficient than a reversible engine, operating If the state of a system is brought close to the at-
between the same temperature limits. mosphere, the opportunity for developing work
The unavailable energy is that portion of energy diminishes and ceases completely, when the sys-
which cannot be converted into useful work by any tem reaches thermodynamic equilibrium with the
means. atmosphere. This state of the system is called the
The unavailable energy is always present with dead state. At the dead state, both the system and
the total energy; it is low-quality energy and its up- atmosphere possess energy, but the capacity to do
gradation is not possible. For example, the energy work is zero, because there is no possibility of any
in the atmosphere is huge in quantity but no part of interaction between the system and the atmosphere.
it can be converted into work. Further, all the man- The properties at the dead state are designated with
made and natural process are irreversible, and thus subscript 0, such as pressure p0, temperature T0,
some portion of the available energy is converted to volume V0, etc.
Availability and Irreversibility 249

Thus, the availability of a system is defined as


The useful work is defined as the network done the maximum useful work that can be obtained from
by the system. It does not include work done on a system, when it undergoes a reversible process
the atmosphere. When a gas expands in a piston from an initial state to dead state. The availability
cylinder device from an initial volume Vi to final of a system is sometimes referred as exergy.
volume Vf, the gas does work W. But all the work
Irreversibility
W would not be available for delivery, a part of this
work is used to push the atmospheric air at pressure When an actual process occurs, it produces certain
p0 through a volume change of Vf – Vi as shown effects, therefore, the process cannot be reversed
in Fig. 8.2 and the work done on the atmosphere and the system and its surroundings cannot be re-
equals to stored to their initial states. During an irreversible
Wsurr = p0 (Vf – Vi) process, the total energy remains constant but ca-
and the difference between gas work W and pacity to do work is lost due to degradation of some
surroundings work Wsurr is called the useful work portion of available energy.
given as This degradation of energy is responsible for
Wuseful = W – Wsurr = W – p0 (Vf – Vi) ...(8.3) entropy generation within the system during a pro-
cess. The entropy generation is always equal to ir-
reversibilities involved in the process.
Atmosphere Atmosphere Simply the irreversibility is defined as the differ-
po
ence between the reversible work Wmax and actual
work Wuseful during a process. It is designated as I
po
and expressed as
I = Wmax – Wuseful (kJ) ...(8.4)
or i = wmax – wuseful (kJ/kg) ...(8.5)
System Vi System Vf The irreversibility generated during a process
per unit time is called the irreversibility rate I and
is given by
I = Wmax - Wuseful (kW) ...(8.6)
Availablility For a reversible process Wmax is always equal
The work is only produced by a system; if it is ini- to the useful work Wuseful, thus the irreversibility is
tially not in dead state. Further, the maximum useful zero.
work can be produced, if a system approaches the
Example 8.1 A heat engine receives heat from a
dead state through a reversible process. Therefore, source at 1200 K at a rate of 500 kW and rejects the
we conclude that a system can produce maximum waste heat to a medium at 300 K. The power output of the
possible work, when it undergoes a change of state heat engine is 180 kW. Determine the reversible power
in a reversible manner from an initial state to the and the irreversibility rate from this process.
state of its surroundings, i.e., the dead state. This
useful maximum work is referred as availability of Solution
the system. In simple words, the availability is the Given A heat engine operates between
useful work potential of the energy contained in a TH = 1200 K
system of a specified state. TL = 300 K
250 Thermal Engineering

QH = 500 kW
Wuseful = 180 kW
To find
(i) Reversible work, and
(ii) Irreversibility rate in the process.
Analysis
(i) Reversible work: Carnot engine work.
TH Wrev
hCarnot = 1 - =
TH QH
Ê 300 ˆ
or Wrev = Á1 - ¥ 500 = 375 kW
Ë 1200 ˜¯
(ii) Irreversibility rate
I = Reversible power
– Useful (actual) power
Ê T - T0 ˆ
Unavailable portion = dQH – d QH Á
Ë T ˜¯
I = 375 kW – 180 kW = 195 kW
Here, QH - Wrev = 500 – 375
T
= 125 kW is unavailable energy. = d QH 0 ...(8.8)
T
The hatched protion in Fig. 8.3 represents the
unavailable portion of energy added.
2dQ
Unavailable energy added = T0 Ú
1 T
Energy may enter a system either as heat or work dQ
across its boundary. In actual practice, as heat is We have = dS
T
added to a system, its temperature rises from T1 to Thus
T2 as shown in Fig. 8.3. If this amount of heat is unavailable energy = T0 (S2 – S1)
directly given to a reversible heat engine instead
An available portion of energy added
to a system, that would produce maximum amount
of work. This work denotes the amount of heat as A = QH – T0 (S2 – S1) ...(8.9)
available energy. Since a finite temperature difference must
The available portion of the heat energy that be needed to cause a heat flow from a body to
crosses the boundary of a system may be determined another, but as heat flows from a high-temperature
by assuming that the reversible engine receives the source to a relatively lower temperature body, the
heat in the same manner as a system receives. temperature of heat energy decreases from source
Consider a differential amount of heat dQH is to destination, thus the capacity to producing work
added at a temperature T to a reversible heat en-
gine. Thus, the reversible engine operates between is lost. Figure 8.4 shows the loss of available energy
a temperature level T and the temperature of the during heat transfer.
ambient T0. The maximum work produced by the The source transfers heat energy irreversibly,
engine becomes the available portion of dQH and its temperature decreases from T1 to T2. As a
is given by result of heat addition to a system, its temperature
Ê T - T0 ˆ increases from T3 to T4. The average temperature
dWmax = d QH Á
Ë T ˜¯
...(8.7) of heat transfer to a system is less than the average
temperature of heat addition from the source,
Availability and Irreversibility 251

Example 8.2 Air at 1 bar and 27°C is heated in a


non-flow system at constant pressure to 177°C. Heat is
supplied from a constant temperature reservoir at 577°C.
The atmospheric temperature is 20°C. What percentage
of heat added per kg of air is the available energy?

Solution
Given p = 1 bar
T1 = 27°C,
T2 = 177°C
TH = 577°C = 850 K
T0 = 20° = 293 K
To find Percentage available energy of heat supplied.
Assumption The specific heat of air at constant
pressure is 1.005 kJ/kg ◊ K.
thus available energy is reduced during such heat Analysis The heat transfer to air at constant pressure
transfer process. QH = Cp (T2 – T1)
= 1.005 ¥ (177 – 27) = 150.75 kJ/kg
T
Unavailable energy Qunav = QH ¥ 0
1. Availability is a measure of work potential TH
293
of a system that a system can deliver when it = 150.75 ¥ = 51.96 kJ/kg
850
is brought to atmospheric conditions. Therefore, the available energy
2. The value of availability cannot be negative. A = QH – Qunav = 150.75 – 51.96
If a system is at any state other than the dead = 98.8 kJ/kg
state, the system would be able to change its Percentage of heat added as available energy
condition spontaneously towards the dead A 98.8 ¥ 100
state and this tendency is completely ceased = ¥ 100 =
QH 150.75
when the system reaches the dead state. = 65.53%
Accordingly, at least zero work is being de-
veloped, and thus the availability (maximum Example 8.3 The exhaust gas leaves an internal
work) cannot be negative. combustion engine at 800°C and 1 atm, after having
3. Availability is not conserved, but is destroyed done 1050 kJ of work per kg of gas in the engine. Take Cp
by irreversibilities. As a limiting case, the of gas as 1.1 kJ/kg ◊ K and temperature of the surround-
ings as 30°C.
availability of a system may be destroyed
completely if there is no provision to obtain (a) How much available energy per kg of gas is lost
by throwing away exhaust gases?
the work.
(b) What is the ratio of lost available energy to the
4. The availability is thus viewed as the maxi-
engine work?
mum theoretical work obtainable, when a
system passes a given state to dead state, Solution
while interacting with the atmosphere only.
Given Exhaust coming out of an IC engine
5. The availability is an extensive property.
p = 1 atm
252 Thermal Engineering

TH = 800°C = 1073 K
T0 = 30° = 303 K
wnet = 1050 kJ/kg
Cpg = 1.1 kJ/kg ◊ K
To find
(i) Loss of available energy in exhaust, and
(ii) Ratio of lost available energy to work of engine.
Assumption Exhaust process at constant pressure.
Analysis
(i) Loss of available energy
Total available energy in exhaust gases above the
surrounding temperature
A = Cpg (TH – T0)
= 1.1 ¥ (1073 – 303) = 847 kJ/kg
Change of entropy of exhaust gases at constant mw = 1890 kg/min
pressure Cpw = 4.27 kJ/kg ◊ K
ÊT ˆ Ê 1073 ˆ
Ds = C pg ln Á H ˜ = 1.1 ¥ ln Á T3 = 35°C = 308 K
Ë T0 ¯ Ë 303 ˜¯
T0 = 27°C = 300 K
= 1.39 kJ/kg ◊ K T4 =?
Unavailable energy due to change in entropy To find Loss of available energy due to irreversible
qunav = T0 Ds = 303 ¥ 1.39 heat transfer.
= 421.45 kJ/kg
Therefore, the loss in available energy Assumption The heat exchanger at constant pressure
in heat exchanger.
q2 = A – qunav = 847 – 421.45
= 425.55 kJ/kg Analysis For flow of fluids across a heat exchanger,
(ii) Ratio of lost energy to engine work applying energy balance on heat exchanger
q 425.55 mg Cpg (T1 – T2) = mwCpw (T4 – T3)
= 2 = = 0.4014
wnet 1050 using numerical values
1510 ¥ 1.088 ¥ (650 – 145) = 1890 ¥ 4.27 ¥ (T4 – 35)
Example 8.4 The exhaust gas at 650°C from a boiler
or T4 = 137.8°C = 410.8 K
is used to heat water. The rate of gas flow is 1510 kg/ min
The unavailable evergy removed from the gases
and the rate of water flow is 1890 kg/min. The water en-
ters the heat exchanger at 35°C, and the gases leave the ÊT ˆ
Qunav1 = T0 (S1 – S2) = T0 mg Cpg ln Á 1 ˜
exchanger at 145°C. Assume that the mean specific heat Ë T2 ¯
of gases and water are 1.088 kJ/kg ◊ K and 4.27 kJ/ kg ◊ K Ê 923 ˆ
= 300 ¥ 1510 ¥ 1.088 ¥ ln Á
Ë 418 ˜¯
respectively. The atmospheric temperature is 27°C.
Determine the loss of available energy resulting from = 390, 421 kJ/min
heat transfer. The unavailable energy added to water
Given The flow across a heat exchanger ÊT ˆ
Qunav2 = T0 (S4 – S3) = T0 mw Cpw ln Á 4 ˜
T1 = 650°C = 923 K ËT ¯ 3
T2 = 145°C = 418 K Ê 410.8 ˆ
= 300 ¥ 1890 ¥ 4.27 ¥ ln Á
Cpg = 1.088 kJ/kg ◊ K Ë 308 ˜¯
= 697, 290 kJ/min.
mg = 1510 kg/min
Availability and Irreversibility 253

Loss in available energy = Qunav2 – Qunav1 (ii) As energy enters the system
= 697, 290 – 390, 421 Available energy A = QH – T0 (DS2)
= 306869 kJ/min = 1000 – 293 ¥ (1.912)
= 439.78 kJ
Example 8.5 1000 kJ of heat is supplied by hot gases Unavailable portion of energy
at 1400°C from a fire box. This heat is used to generate = T0 (DS2)
the steam at 250°C. The atmospheric temperature is = 293 ¥ 1.912
20°C. Calculate the energy as available and unavailable = 560.2 kJ
portion To prove DI = T0 (DS)universe
(a) as it leaves the hot gases, The change in irreversibility
(b) as it enters the system. DI = –175.214 + 560.21
Prove that DI = T0 (DS)universe holds good. = 385 kJ
and T0 (DS)universe = 293 ¥ 1.314
Solution = 385 kJ
Given QH = 1000 kJ The two quantities are equal, thus
TH = 1400°C = 1673 K DI = T0 (DS)universe Proved
TL = 250°C = 523 K
T0 = 20°C = 293 K
To find Available and unavailable portion of energy Consider a thermodynamic system which under-
(i) as it leaves the hot gases, goes a process from a state 1 to a state of ambient
(ii) as it enters the system, p0, T0. The system is exchanging heat with its ambi-
To prove DI = T0(DS)universe . ent only. The first law and second law of thermody-
namics for such a process can be expressed as the
Analysis The decrease of entropy of source during heat
transfer
first law
QH 1000 kJ Q – W = DE = E2 – E1
DS1 = =- = – 0.598 kJ/K Q
TH 1673 K Second law Sgen = S2 – S1 + surr
Change of entropy of steam during heat absorption; T0
QH 1000 kJ where
DS2 = = = 1.912 kJ/K
TL 523 K Qsurr = – Q (heat transferred to surroundings)
Net change in entropy Sgen = entropy generations during the process.
(DS )universe = DS1 + DS2 Rearranging the above equation as
= – 0.598 + 1.912 = 1.314 kJ/K W = E1 – E2 + Q
(i) As energy leaves the hot gases and T0 Sgen = T0 (S2 – S1) – Q
Available energy of hot gases or Q = T0 (S2 – S1) – T0 Sgen
A = QH – T0 DS1
= –T0 Sgen – T0 (S1 – S2)
= 1000 – 293 ¥ (0.598)
Combining these equations, we get
= 824.786 kJ
W = E1 – E2 – T0 Sgen – T0 (S1 – S2) ...(8.10)
Unavailable portion of energy in hot gases
= T0 (DS1)
It is the actual work done during a process
= 293 ¥ (0.598)
undergoing a change of state from states 1 to 2. If
= 175.214 kJ
the volume of the system is also changing during
the process, then useful work
254 Thermal Engineering

Wuseful = W – Wsurr then maximum work done during a process from


= W – p0 (V2 – V1) the state 1 to the state 2 can be expressed as
or Wuseful = E1 – E2 – T0 Sgen Wrev = F1 – F2 + Heat transfer to surroundings
– T0 (S1 – S2) + p0 (V1 – V2) Ê T ˆ
= F1 – F2 + QR Á1 - 0 ˜ ...(8.17)
For a reversible process Sgen = 0, then Ë TR ¯
Wrev = Wmax = E1 – E2 – T0 (S1 – S2) where
+ p0 (V1 – V2) ...(8.11) T0 = Absolute temperature of surroundings,
It is the maximum useful work that can be TR = Absolute temperature of reservoir,
produced during a reversible process, thus becomes
QR = Heat transfer between system and
the definition availability.
reservoir: – ve to reservoir and + ve from
reservoir
The net entropy generation during a process
Q Q
Sgen = (S2 – S1)sys + surr + R ...(8.18)
The maximum useful work potential of a closed T0 TR
system is designated as F and is obtained from
Eq. (8.11) by replacing the state 1 by parameters Example 8.6 Prove that the availability of a closed
without a subscript and the state 2 by the dead state system always decreases in an irrevesible process.
with the subscript ‘0’. Further for a closed system, Analysis The availability of a closed system is ex-
the changes in kinetic and potential energies are pressed as
negligible, and (E = U) thus the availability of a f = (u – u0) – T0 (s – s0) + p0 (v – v0) ...(i)
closed system becomes For any isolated system, due to internal irreversibilities
F = (U – U0) – T0 (S – S0) Ds > 0
+ p0 (V – V0) kJ ...(8.12) The first law of thermodynamics applied to an isolated
system
For unit mass
q = w + Du
f = (u – u0) – T0 (s – s0)
But q = 0 for isolated system
+ p0 (v – v0) kJ/kg ...(8.13)
and w = – Du ...(ii)
Since the availability of a closed system at the
where w is the total work transfer and it is expressed as
dead state is zero then Eq. (8.13) yields to
w = wuseful + p0 (v – v0)
a = u – T0 s + p0 v ...(8.14)
wuseful = – Du – p0 (v – v0)
This expression represents the availability func-
= –(u – u0) – p0 (v – v0) ...(iii)
tion ‘a’of the closed system. The availability func-
tion of the closed system is the composite proper- It is the gain in availability from the surroundings.
Therefore, the change in availability of a closed system
ties of the system and surroundings.
(DA)closed = Availability of the system
If the system undergoes a change of state from 1
to 2, then change in availability is referred as work + gain in availability
transfer of the system. = (u – u0) – T0 (s – s0) + p0 (v – v0)
Wrev = (F1 – F0) – (F2 – F0) + [– (u – u0) – p0 (v – v0)]
= –T0 (s – s0)
= F1 – F2 (kJ) ...(8.15)
But (s – s0) is a positive quantity, therefore for an
or wrev = f1 – f2 = a1 – a2 ...(8.16)
irreversible process
If a closed system exchanges heat with a (D A)closed = –ve or < 0
constant-temperature reservoir during a process,
Availability and Irreversibility 255

Example 8.7 Air in a piston–cylinder arrangement The change in availability can be calculated as
is heated at constant pressure by addition of 100 kJ/kg f2 – f1 = (u2 – u1) – T0 (s2 – s1) + p0 (v2 – v1)
of air. The air is initially at 28°C while the surroundings = Cv (T2 – T1) – T0 (Ds) + p0 (v2 – v1)
is at 21°C. Calculate the change in availability per kg = 0.718 ¥ (400.5 – 301) – 294 ¥ 0.287
of air. + 100 ¥ (1.15 – 0.864)
Take Cp = 1.005 kJ/kg ◊ K = 15.66 kJ/kg
and atmospheric air pressure is 1 bar.
Example 8.8 A rigid cylinder with a volume of 2.5 m3
Solution contains air at 150 kPa and 27°C. The heat is transferred
to air from a constant-temperature heat source at
Given A piston–cylinder arrangement
1500 K and air in the cylinder is heated to 700 K. The
q = 100 kJ/kg atmosphere is at 1 bar and 17°C. Calculate the initial
T1 = 28°C = 301 K and final availability of air, maximum useful work and
T0 = 21°C = 294 K irreversibility.
Cp = 1.005 kJ/kg ◊ K
Solution
p0 = 1 bar = 100 kPa
Given A rigid cylinder with
To find The change in availability per kg of air.
V = 2.5 m3
Assumptions p1 = 150 kPa
T1 = 27°C = 300 K
(i) The initial pressure of air in the cylinder at the TH = 1500 K
atmospheric pressure of 100 kPa, T2 = 700 K
(ii) Air as an ideal gas, T0 = 17°C = 290 K
(iii) The specific gas constant for air, p0 = 1 bar = 100 kPa
R = 0.287 kJ/kg ◊ K.
To find
Analysis Heat addition at constant pressure is given by (i) Initial availability of air,
q = Cp (T2 – T1) (ii) Final availability of air,
or 100 = 1.005 ¥ (T2 – 301) (iii) Maximum useful work, and
or T2 = 400.5 K. (iv) Irreversibility.
The initial specific volume
RT1 0.287 ¥ 301 Assumptions
v1 = = = 0.864 m3/kg
p0 100 (i) A non-flow process,
The final specific volume, (ii) Specific heat at constant volume is 0.717 kJ/kg ◊ K,
RT2 0.287 ¥ 400.5 (iii) The gas constant of air as 0.287 kJ/kg ◊ K,
v2 = = = 1.15 m3/kg (iv) Dke = 0 and Dpe = 0.
p0 100

The change in specific entropy Analysis


The initial availability of air in a closed system.
ÊT ˆ
Ds = C p ln Á 2 ˜ F1 = m[(u1 – u0) + p0 (v1 – v0)
Ë T1 ¯
– T0 (s1 – s0)]
Ê 400.5 ˆ 150 ¥ 2.5
= 1.005 ¥ ln Á = 0.287 kJ/kg ◊ K. p1V
Ë 301 ˜¯ Here m = =
RT1 0.287 ¥ 300
= 4.35 kg

The specific heat at constant volume RT0 0.287 ¥ 290


v0 = =
Cv = Cp – R p0 100
= 1.005 – 0.287 = 0.718 kJ/kg ◊ K = 0.832 m3/kg
256 Thermal Engineering

V 2.5 Here, QR = m (u2 – u1) = mCv (T2 – T1)


v1 = = = 0.574 m3/kg
m 4.35 = 4.35 ¥ 0.717 ¥ (700 – 300)
u1 – u0 = Cv (T1 – T0) = 0.717 ¥ (300 – 290) = 1247.58 kJ
= 7.17 kJ/kg Therefore,
Ê 290 ˆ
Calculating each item separately Wmax = (22.83 – 504.68) + 1247.58 ¥ Á1 -
Ë 1500 ˜¯
p0 (v1 – v0) = 100 ¥ (0.574 – 0.832)
= – 481.85 + 1006.38 = 524.53 kJ
= – 25.7 kJ/kg
The irreversibility of the closed system
ÊT ˆ Êv ˆ
s1 – s0 = Cv ln Á 1 ˜ + R ln Á 1 ˜ I = Wmax – Wact
Ë T0 ¯ Ë v0 ¯
For a constant-volume process Wact = 0
Ê 300 ˆ Thus I = Wmax = 524.53 kJ
= 0.717 ¥ ln Á +
Ë 290 ˜¯
Ê 0.574 ˆ
0.287 ¥ ln Á
Ë 0.832 ˜¯
= – 0.082 kJ/kg ◊ K
Therefore, the initial availability
F1 = 4.35 ¥ [(7.17) + (– 25.7) The availability in a steady flow process can be
– 290 ¥ (– 0.082)] obtained from Eq. (8.10) by using
= 22.83 kJ Ê V2 ˆ
E = mÁu + + z g˜
(ii) The final availability of air Ë 2 ¯
F2 = m[(u2 – u0) + p0 (v2 – v0) and writing the equation on a time-rate bases.
– T0 (s2 – s0)]
For a constant-volume cylinder È V2 ˘ È V2 ˘
Wrev = m1 Íu1 + 1 + z1 g ˙ - m2 Íu2 + 2 + z2 g ˙
v1 = v2 = 0.574 m3/ kg Î 2 ˚ Î 2 ˚
T2 700 – T0 (S1 – S2) – p0 (V2 – V1)
p2 = p1 = 150 ¥ = 350 kPa
T1 300 For steady flow m1 = m2, thus
u2 – u0 = Cv (T2 – T0)
È V2 ˘
= 0.717 ¥ (700 – 290) Wrev = m Íu1 + p0 v1 + 1 + z1 g - T0 s1 ˙
= 293.97 kJ/kg Î 2 ˚
ÊT ˆ Êv ˆ È V 2 ˘
s2 – s0 = Cv ln Á 2 ˜ + R ln Á 2 ˜ - m Íu2 + p0 v2 + 2 + z2 g - T0 s2 ˙
ËT ¯
0 Ëv ¯ 0 Î 2 ˚
Ê 700 ˆ
= 0.717 ¥ ln Á È V 2 - V22
Ë 290 ˜¯ = m Íh1 - h2 + 1
ÍÎ 2
˘
Ê 0.574 ˆ
+ 0.287 ¥ ln Á + ( z1 - z2 ) g - T0 ( s1 - s2 ) ˙ (kJ) ...(8.19)
Ë 0.832 ˜¯
˚
= 0.525 kJ/kg ◊ K For a unit-mass flow rate
p0 (v2 – v0) = – 25.7 kJ/kg V12 - V22
\ F2 = 4.35 ¥ [293.97 – 25.7 – 290 ¥ 0.525] wrev = (h1 – h2) +
2
= 504.68 kJ
+ (z1 – z2)g – T0 (s1 – s2) (kJ/kg) ...(8.20)
(iii) The air receives heat from a constant-temperature
reservoir at 1500 K, thus the maximum useful It can be interpreted as
work wrev = T0 Ds – Dh – Dke – Dpe (kJ/kg) ...(8.21)
Ê
The availability of the fluid stream in an open
T ˆ
Wmax = F1 – F2 + QR Á1 - 0 ˜ system is called the stream availability and is
Ë TR ¯
Availability and Irreversibility 257

denoted by y. It is obtained from Eq. (8.20) by Properties of steam At 3 MPa and 450°C
replacing inlet parameters without subscript and h1 = 3344 kJ/kg
the exit parameter by dead state with subscript ‘0’ s1 = 7.0833 kJ/kg ◊ K
with V0 = 0, z0 = 0: At 0.1 MPa, dry steam
V2 h2 = 2675.52 kJ/kg
y = (h – h0) – T0 (s – s0) + + zg (kJ/kg)
2 ...(8.22) s2 = 7.3593 kJ/kg ◊ K
The reversible work for a process between states Ds = s2 – s1 = 7.3593 – 7.0833 = 0.276 kJ/kg ◊ K
1 and 2 can be expressed as Dh = h2 – h1 = 2675.52 – 3344 = – 668.48 kJ/kg.
Wrev = m (y1 – y2) kW ...(8.23) Then wmax = 298 ¥ 0.276 – (– 668.48) = 750.72 kJ/kg
or wrev = y1 – y2 (kJ/kg) ...(8.24) Example 8.10 The steam at 1000 kPa, 275°C enters
The rate of irreversibility I or irreversibility per a steady flow system with negligible velocity and leaves
unit mass flow rate i associated with a steady flow at 100 kPa, 120°C with a velocity of 160 m/s. The steam
process is flow rate is 9500 kg/h. Heat is exchanged with only
I = Wrev – Wuseful = T0 S gen ...(8.25) surroundings at 15°C. Determine the maximum possible
or i = wrev – wuseful = T0 sgen ...(8.26) power output.
When a steady flow device exchanges heat Solution
with a thermal reservoir at TR at a rate of QR, then
reversible work. Given Steam in a steady flow system.
Ê p1 = 1000 kPa
T0 ˆ
Wrev = m (y1 – y2) + QR Á1 - (kW) ...(8.27)
TR ˜¯
p2 = 100 kPa
Ë
T1 = 275°C
where the sign of QR is taken with respect to T2 = 120°C
reservoir. V1 = 0
V2 = 160 m/s
Example 8.9 Determine the maximum work that can
ms = 9500 kg/h
be produced by a steam turbine which has an inlet state
of 3 MPa and 450°C and an outlet state as a dry saturated T0 = 15°C
steam at 0.1 MPa. The heat transfer with surroundings is To find Maximum power output from the system.
at 25°C.
Schematic
Given A steam turbine with
Inlet state: p1 = 3 MPa = 3000 kPa
T1 = 450°C = 723 K
Exit state: p2 = 0.1 MPa = 100 kPa
T0 = 25°C = 298 K
To find Maximum work.
Assumptions
(i) Negligible change in kinetic energy. Assumptions
(ii) Negligible change in potential energy. (i) No information about inlet and exit elevations
(iii) Steadily expansion of steam in the turbine. is provided, thus we neglect the potential energy
change.
Analysis Maximum reversible work of turbine,
(ii) Constant properties.
Eq. (8.21)
wmax = T0 Ds – Dh Analysis The availability of a steady flow system for
unit mass flow rate, Eq (8.21) is
258 Thermal Engineering

Wrev = Wmax = T0 Ds – Dh – Dke (kJ/kg) To find


Properties of steam (i) Availability at the entrance to the gas turbine.
At 1000 kPa, 275°C; (ii) Availability at the exit from the gas turbine.
h1 = 2996.6 kJ/kg, Assumptions
s1 = 7.0257 kJ/kg ◊ K (i) No heat transfer from the turbine during
At 100 kPa, 120°C; expansion of gas.
h2 = 2716.3 kJ/kg, (ii) No change in potential and kinetic energies.
s2 = 7.4656 kJ/kg ◊ K (ii) Air as ideal gas with Cp = 1.005 kJ/kg ◊ K and R =
Then, wrev = wmax 0.287 kJ/kg ◊ K.
= 288 ¥ (7.4656 – 7.0257)
Analysis
160 2
– (2716.3 – 2996.6) – (i) Availability at the turbine inlet
2000
= 394 kJ/kg Enthalpy of gas above atmospheric temperature at
For ms = 9500 kg/h, the maximum possible power the turbine inlet
output h1 – h0 = Cp (T1 – T0)
9500 = 1.005 ¥ (1300 – 298)
wmax = ms wmax = ¥ 394 = 1040 kW
3600 = 1007 kJ/kg
Change in entropy;
Example 8.11 1.5 kg of gas flows through a gas turbine
unit from its initial pressure and temperature of 600 kPa ÊT ˆ Ê p ˆ
s1 – s0 = Cp ln Á 1 ˜ - R ln Á 1 ˜
and 1300 K, respectively and exhausts at a pressure of Ë T0 ¯ Ë p0 ¯
102 kPa and a temperature of 600 K to the atmosphere.
Ê 1300 ˆ Ê 600 ˆ
The atmospheric pressure and temperature are 100 kPa = 1.005 ¥ ln Á - 0.287 ¥ ln Á
Ë 298 ˜¯ Ë 100 ˜¯
and 298 K. Calculate availability at the entrance to the
gas turbine and exhaust of the gas turbine. Take necessary = 0.966 kJ/kg
assumptions. The availability of gas at the turbine inlet
yinlet = (h1 – h0) – T0 (s1 – s0)
Solution = 1007 – 298 ¥ 0.966
Given An air compression process with = 719.08 kJ/kg of gas
p1 = 600 kPa (ii) Availability at the turbine exit
T1 = 1300 K Enthalpy of gas above atmospheric temperature at
p2 = 102 kPa turbine exit
T2 = 600 K h2 – h0 = Cp (T2 – T0)
T0 = 298 K = 1.005 ¥ (600 – 298)
p0 = 100 kPa = 303.51 kJ/kg
Change in entropy;
ÊT ˆ Êp ˆ
s2 – s0 = Cp ln Á 2 ˜ - R ln Á 2 ˜
Ë T0 ¯ Ë p0 ¯
Ê 600 ˆ Ê 102 ˆ
= 1.005 ¥ ln Á - 0.287 ¥ ln Á
Ë 298 ˜¯ Ë 100 ˜¯
= 0.6976 kJ/kg
The availability of gas at the turbine inlet
yexit = (h2 – h0) – T0 (s2 – s0)
= 303.51 – 298 ¥ 0.6976
= 95.61 kJ/kg of gas
Availability and Irreversibility 259

Example 8.12 Air enters a compressor in steady Change in entropy;


flow manner at 140 kPa, 17°C and 70 m/s and leaves it ÊT ˆ Ê p ˆ
s1 – s2 = Cp ln Á 1 ˜ - R ln Á 1 ˜
at 350 kPa, 127°C and 110 m/s. The environment is at Ë T2 ¯ Ë p2 ¯
100 kPa and 7°C. Calculate per kg of air:
Ê 290 ˆ Ê 140 ˆ
(a) Actual amount of work required, = 1.005 ¥ ln Á - 0.287 ¥ ln Á
Ë 400 ˜¯ Ë 350 ˜¯
(b) The minimum work required, and
(c) The irreversibility of the process. = – 0.0602 kJ/kg ◊ K
(i) Actual work input
Solution For a unit mass flow rate through the compressor
Given An air-compression process with in absence of change in potential energy, the
steady flow energy equation flow yields to
p1 = 140 kPa
T1 = 17°C = 290 K V12 - V22
wact = (h1 – h2) +
V1 = 70 m/s 2
p2 = 350 kPa = –110.55 – 3.6 = – 114.15 kJ/kg
T2 = 127°C = 400 K (ii) Minimum (reversible) work of compression
V2 = 110 m/s V12 - V22
wrev = (h1 – h2) + - T0 ( s1 - s2 )
p0 = 100 kPa 2
T0 = 7°C = 280 K = –110.55 – 3.6 – 280 ¥ (– 0.0602)
= – 97.29 (kJ/kg)
To find
(iii) Irreversibility of the process
(i) Actual work input to compressor,
I = wrev – wrev
(ii) Minimum work input, and
= – 97.29 – (–114.15)
(iii) Irreversibility in the compression process.
= 16.86 kJ/kg
Assumptions
Example 8.13 An air compressor receives air at 1 bar
(i) Adiabatic compression process,
and 27°C. It compresses the air to a pressure of 10 bar,
(ii) No change in potential energies,
while its temperature reaches 267°C. The compressor
(iii) Air as ideal gas with Cp = 1.005 kJ/kg ◊ K and R =
rejects 50 kJ/kg of heat from its body during compression
0.287 kJ/kg ◊ K.
process. Find the actual and reversible work transfer, and
Analysis Change in enthalpy irreversibility in the process.
h1 – h2 = Cp (T1 – T2) = 1005 ¥ (17 – 127)
Solution
= –110.55 kJ/kg
Change in kinetic energy Given An air compression process with
V12 - V22 2
70 - 110 2 p1 = 1 bar = 100 kPa
= = – 3600 J/kg T1 = 27°C = 300 K
2 2
p2 = 10 bar = 1000 kPa
= –3.6 kJ/kg
T2 = 267°C = 540 K
q = – 50 kJ/kg
To find
(i) Actual work input to compressor,
(ii) Reversible work input, and
(iii) Irreversibility in the compression process.
Assumptions
(i) Non-adiabatic compression process.
260 Thermal Engineering

(ii) No change in kinetic and potential energies.


(iii) Air as ideal gas with Cp = 1.005 kJ/kg ◊ K and R =
0.287 kJ/kg ◊ K.
Analysis The schematic of air compressor is shown in
Fig. 8.9.

of irreversibility. Take atmospheric temperature as 27°C.

Solution
Given A feed water heated with two heat sources.
m = 5 kg/s pi = 5 MPa
Ti = 40°C pe = 5 MPa
Te = 180°C Q1 = + 900 kw
T1 = 100°C T2 = 200°C
(i) For a unit-mass flow rate through the compressor T0 = 27°C = 300 K
in the absence of change in kinetic and potential
To find
energies, steady flow energy equation flow
(i) Reversible work, and
wact = (h1 – h2) + q
(ii) Irreversibility rate.
= Cp (T1 – T2) + q
= 1.005 ¥ (27 – 267) – 50 Assumptions
= – 291.2 kJ/kg (i) Constant-pressure heat addition,
(ii) The reversible work transfer during compression (ii) No change in kinetic and potential energies,
wrev = (h1 – h2) – T0 (s1 – s2) (iii) Mean specific heat of water as 4.18 kJ/kg ◊ K.
For a non-adiabatic process.
Analysis The energy equation for feedwater heater
ÊT ˆ Êp ˆ
s2 – s1 = Cp ln Á 2 ˜ - R ln Á 2 ˜ h1 + q1 + q2 = he
Ë T1 ¯ Ë p1 ¯
or Q2 = m (he – hi) – Q1
Ê 540 ˆ Ê 10 ˆ = m Cpw (Te – Ti) – Q1
= 1.005 ¥ ln Á - 0.287 ¥ ln Á ˜
Ë 300 ˜¯ Ë 1¯ = 5 ¥ 4.18 ¥ (180 – 40) – 900
= 0.5907 – 0.6608 = – 0.070 kJ/kg ◊ K. = 2026 kW
and h1 – h2 = 1.005 ¥ (300 – 540) = – 241.2 kJ The reversible work with heat transfer from two
Then wrev = – 241.2 – 300 ¥ (– 0.070) constant temperature reservoirs.
= – 220.2 kJ/kg Ê T0 ˆ Ê T0 ˆ
Wrev = m (y1 – y2) + Q1 Á1 - ˜ + Q2 Á1 - ˜
(iii) Rate of irreversibility Ë T1 ¯ Ë T2 ¯
i = wrev – wact = – 220.2 – (– 291.2) where,
= 71 kJ/kg y1 – y2 = (hi – he) – T0 (si – se)

Example 8.14 A feedwater heater has 5 kg/s water ÊT ˆ


= Cpw (Ti – Te) – T0 Cpw ln Á i ˜
at 5 MPa, 40°C flowing through it. The water is heated Ë Te ¯
from two constant temperature sources. One source adds È Ê 40 + 273 ˆ ˘
= 4.18 ¥ Í( 40 - 180) - 300 ¥ ln Á ˙
Ë 180 + 273 ˜¯ ˙˚
900 kW from a 100°C reservoir and the other source adds
ÍÎ
heat from a 200°C reservoir in such a way that water is
heated to 180°C. Calculate the reversible work and rate = – 121.6 kJ/kg
Availability and Irreversibility 261

Therefore, The steady flow mixing of two streams of water leads to


m3 h3 = m1 h1 + m2 h2
Wrev = (5 kg/s) ¥ (–121.6 kJ/kg)
or m3 Cpw Tm = m1Cpw T1 + m2Cpw T2
Ê 300 ˆ Ê 300 ˆ
+ (900 kW) Á1 - + 2026 Á1 - 65 ¥ 4.18Tm = 25 ¥ 4.18 ¥ 90 + 40 ¥ 4.18 ¥ 40
Ë 100 + 273 ˜¯ Ë 200 + 273 ˜¯
Temperature Tm of water after mixing
= – 608.05 kW + 176.14 + 741.0
= 309.1 kW 25 ¥ 90 + 40 ¥ 40
Tm = = 59.2°C
The rate of irreversibility 65
The availability of 65 kg water at 59.2°C
I = Wrev = 309.1 kW
È Ê 59.2 + 273 ˆ ˘
y3 = 65 ¥ 4.187 ¥ Í(59.2 - 20) - 293 ln Á ˙
Example 8.15 25 kg of water at 90°C is mixed
Î Ë 293 ˜¯ ˚
with 40 kg of water at 40°C at constant pressure. The
= 655.77 kJ
atmospheric pressure is 1 bar and 20°C. Calculate the
decrease in available anergy. The loss in availability done to mixing.
= y m – y3
Solution = 866.17 – 655.77 = 210.39 kJ
Given The mixing of water, Example 8.16 2 kg of water at 50°C is mixed with 3 kg
m1 = 25 kg m2 = 40 kg of water at 100°C in a steady flow process. Calculate the
T1 = 90°C T2 = 40°C temperature of resulting mixture, state whether the mixing
T0 = 20°C p = 1 bar is isentropic?
To find Loss in availability. If not, what is the entropy change and unavailable
energy with respect to surroundings at 50°C?
Assumptions
(i) No heat loss from mixture to surroundings, Solution
(ii) The specific heat of water as 4.187 kJ/kg ◊ K, Given The mixing of two streams of water,
(iii) The change in potential and kinetic energies is m1 = 2 kg m2 = 3 kg
negligible. T1 = 50°C T2 = 100°C
Analysis The availability of energy can be calculated as T0 = 50°C
È Ê Ti ˆ ˘ To find
y = (hi – h0) – T0 (si – s0) = mCpw Í(Ti - T0) - T0 ln Á ˜ ˙
ÍÎ Ë T0 ¯ ˙˚ (i) State of water after mixing,
The availability of 25 kg water at 90°C (ii) Entropy change of mixture,
(iii) Unavailble energy.
È Ê 363 ˆ ˘
y1 = 25 ¥ 4.187 ¥ Í(90 - 20) - 293 ¥ ln Á ˙
Î Ë 293 ˜¯ ˚ Assumptions
= 104.67 ¥ (70 – 62.77) = 756.8 kJ (i) No heat loss to surroundings during mixing of
two streams of water.
The availability of 40 kg water at 40°C
(ii) The specific heat of water as 4.187 kJ/kg ◊ K.
È Ê 313 ˆ ˘
y2 = 40 ¥ 4.187 ¥ Í( 40 - 20) - 293 ¥ ln Á ˙ (iii) The change in potential and kinetic energies is
Î Ë 293 ˜¯ ˚
negligible.
= 109.37 kJ
Analysis
Total available energy
ym = y1 + y2 = 756.8 + 109.37 = 866.17 kJ (i) Mixing of two streams of water
Total mass of water after mixing Total mass of water after mixing
m3 = m1 + m2 = 25 + 40 = 65 kg m3 = m1 + m2 = 2 + 3 = 5 kg
262 Thermal Engineering

The steady flow mixing of two streams of water Assumption The specific heat of water as 4.187 kJ/kg ◊ K.
leads to
Analysis
m3 h3 = m1 h1 + m2 h2
(i) The initial angular velocity of the flywheel
or m3Cpw Tm = m1Cpw T1 + m2 Cpw T2
5 ¥ 4.18 Tm = 2 ¥ 4.18 ¥ 50 + 3 ¥ 4.18 ¥ 100 2p N1 2p ¥ 3000
w1 = = = 314.2 rad/s
Temperature Tm of water after mixing 60 60
Initial available energy of the flywheel = initial
2 ¥ 50 + 3 ¥ 100
Tm = kinetic energy
5
1
= 80°C or 353 K A1 = Iw 21
2
Thus the mixing is not isentropic. 1
= ¥ (0.54 kg m2) ¥ (314.2 rad/s)2
2
(ii) Entropy change
= 2.66 ¥ 104 Nm = 26.6 kJ
Entropy change during mixing process
When this kinetic energy is dissipated as frictional
ÊT ˆ ÊT ˆ heat then rise in temperature of shaft and bearings
D s = m1 C pw ln Á m ˜ + m2 C pw ln Á m ˜
Ë T1 ¯ Ë T2 ¯ K.E. = m Cpw (DT )
Ê 353 ˆ Ê 353 ˆ 26.6 = (2 kg) ¥ (4.187 kJ/kg ◊ K.) (DT °C)
= 2 ¥ 4.187 ¥ ln Á + 3 ¥ 4.187 ¥ ln Á
Ë 323 ˜¯ Ë 373 ˜¯ 26.2
or DT = = 3.12°C
= 0.7437 – 0.692 = 0.0517 kJ/kg ◊ K 2 ¥ 4.187
(iii) Unavailable energy Thus the final temperature of bearings
qunavail = T0 Ds = 323 ¥ (0.517) = 16.71 kJ/kg T2 = T0 + DT = 15°C + 3.12°C = 18.12°C
(ii) The maximum energy returned to flywheel as
Example 8.17 The moment of inertia of a flywheel is available energy
0.54 kg ◊ m2 and it rotates at a speed of 3000 rpm in a large a2 = Dh – T0 Ds
insulated system at 15°C. The kinetic energy of the flywheel
ÏÔ Ê T ˆ ¸Ô
is distributed as frictional heat at the shaft bearings. The and A2 = mCpw Ì(T2 - T0 ) - T0 ln Á 2 ˜ ˝
water equivalent of the shaft bearings is 2 kg. Find the ÔÓ Ë T0 ¯ Ô˛
rise in temperature of the shaft bearing when flywheel has È Ê 18.12 + 273 ˆ ˘
come to rest. Determine the maximum possible heat, which =2 ¥ 4.187 ¥ Í(3.12) - 288 ln Á ˜¯ ˙
Î Ë 288 ˚
may return to the flywheel as high-grade energy. Calculate
= 1.80 kJ
how much amount of kinetic energy becomes unavailable.
(iii) The amount of kinetic energy becoming unavailable
What would be the final rpm of the flywheel if it is set in
UE = A1 – A2 = 26.6 – 1.80 = 24.8 kJ
motion with this available energy?
(iv) The final rpm of fly wheel
Solution 1
Available energy retained by flywheel = I w 22
2
Given A flywheel with 3 1
= 1.80 ¥ 10 Nm = ¥ 0.54 ¥ w 22
I = 0.54 kg ◊ m2 2
N1 = 3000 rpm or w 2 = 81.6 rad/s
m = 2 kg 2p N 2
Further w 2 =
T0 = 15°C = 288 K 60
w 2 ¥ 60 81.6 ¥ 60
To find or N2 = = = 779.2 rpm
2p 2p
(i) Temperature rise of shaft bearings,
(ii) Maximum possible heat returned to flywheel as Refrigerant 134 a, initially a saturated
available energy, vapour at –30°C, is contained in a rigid insulated vessel.
(iii) Unavailable kinetic energy The vessel is fitted with a paddle wheel connected to
(iv) Final rpm of flywheel a pulley and suspended mass. As the mass descends a
Availability and Irreversibility 263

certain distance, the refrigerant is stirred until it attains Assumptions


a state where the pressure is 1.4 bar. The only significant (i) The refrigerant in rigid insulated tank, suspended
changes of state are experienced by the falling mass and mass and isolated system are under consideration.
the refrigerant. If the mass of the refrigerant is 1.11 kg, For an isolated system, Q = W = 0.
determine (ii) Due to paddle work addition, only significant
(a) initial availability, final availability and change change experienced by refrigerant and suspended
in availability of the refrigerant in kJ, mass.
(b) change in availability of suspended mass in kJ, Analysis
(c) change in availability in isolated system of the (i) The initial and final availability of the refrigerant
vessel and pulley–mass assembly, in kJ. From Table B-6
Take atmospheric temperature and pressure as 293 K Properties of Refrigerant 134a, saturated vapour
and 1 bar, respectively. at initial state of –30°C
u1 = ug = 360.63 kJ/kg,
Solution v1 = vg = 0.22402 m3/kg
Given Refrigerant 134a in a rigid, insulated container s1 = sg = 1.7493 kJ/kg ◊ K
is stirred by a paddle wheel connected by a pulley–mass From Table B-8 at 1 bar and 20°C,
assembly. v0 = 0.23392 m3/kg, h0 = 420.05,
T0 = 293 K = 20°C s0 = 1.8869 kJ/kg ◊ K
p0 = 1 bar, p1 = 1.4 bar u0 = h0 – p0 = 420.05 – 100 kPa ¥ 0.23392
T1 = –30°C = 247 K = 396.658 kJ/kg
m = 1.11 kg Initial availability of refrigerant
To find F1 = m [(u1 – u0) – T0 (s1 – s0) + p(v1 – v0)] kJ
(i) Initial, final and change in availability of = (1.11 kg) ¥ [(360.63 – 396.658)
refrigerant. – (293 K) ¥ (1.7493 – 1.8869) + (100 kPa)
(ii) Change in availability of suspended mass. ¥ (0.22402 – 0.23392)]
(iii) Change in availability of an isolated system of the = 1.11 ¥ [–36.028 + 40.3168 – 0.99]
vessel and pulley–mass assembly. = 3.66 kJ/kg
264 Thermal Engineering

Final state of refrigerant, at p2 = 1.4 bar and v2 = Heat transfer between outer surface of heat exchanges
v1 by interpolation. and surroundings can be ignored. Kinetic and potential
u2 = 450.148 kJ/kg, energy effects are negligible. Assuming that the combus-
s2 = 2.041 kJ/kg ◊ K tion gas stream has the properties of air, and using ideal
Final availability of refrigerant, gas model for both streams, determine for the heat ex-
changer:
F2 = 1.11 ¥ [(450.148 – 396.658) – 293
¥ (2.0410 – 1.8669) + 100 ¥ (0.22402 (a) The exit temperature of combustion gas in °C,
– 0.23392)] (b) Net change in flow availability rate from inlet to
exit of each stream is MW,
= 1.11 ¥ [53.49 – 45.15 – 0.99]
(c) Rate of availability loss in MW.
= 8.1585 kJ
Take atmospheric pressure and temperature as 1 bar
Change in availability of refrigerant 134a
and 27°C respectively.
F2 – F1 = 8.1585 – 3.66 = 4.4985 kJ
(ii) Change in availability of suspended mass Solution
Initial availability of mass
Given The steady flow operation in a heat exchanger
m y1 = Wrev
Cold fluid: air Hot fluid: combustion gases
= m [Dh + Dke + Dpe – T0 Ds] kJ
T1 = 340°C = 613 K
For a suspended mass system
T3 = 750°C = 1023 K
Dh = 0,
T2 = 590°C = 863 K
Dke = 0,
T4 = ? p1 = 10 bar, p2 = 9.7 bar
Ds = 0, thus
mc = 90 kg/s p3 = 1.1 bar, p4 = 1 bar
\ DF = m(Dpe) = DPE
mh = 90 kg/s
The change in potential energy of the suspended
mass can be obtained from an energy balance p0 = 1 bar, T0 = 27°C = 300 K
for the isolated system which is the sum of the To find
energy changes of the refrigerant and suspended (i) Exit temperature of combustion gases,
mass. Thus,
(ii) Net change in flow availability rate from inlet to
0 0
(DkE + DPE + DU)refrigerant exit each stream,
0 0 0 0 (iii) Rate of availability loss in MW.
+ (DkE + DPE + DU)mass = Q – W
Schematic
D PEmass = – DUrefrigerant
= –1.11 ¥ [450.148 – 360.63]
= – 99.365 kJ
The availability of mass decreases, because mass
descends.
(iii) Change in availability of an isolated system of
vessel and pulley mass assembly
F = (DF)refrigerant + (DF)mass
= 3.66 kJ + –99.365 kJ = –95.7 kJ
The availability of the isolated system decreases.

Example 8.19 Compressed air enters a counter-


flow heat exchanger operating at steady state at 340°C,
10 bar and exits at 590°C, 9.7 bar. The hot combustion
gases enters as a separate stream at 750°C, 1.1 bar and
exit at 1 bar. Each stream has a mass-flow rate of 90 kg/s.
Availability and Irreversibility 265

Assumptions Ê 773 ˆ Ê 1ˆ
= 1.005 ¥ ln Á – 0.287 ¥ ln Á ˜
(i) The specific heat of air as Cp = 1.005 kJ/kg ◊ K Ë 1023 ˜¯ Ë 1.1¯
(ii) For control volume Q = 0, W = 0
= – 0.2542 kJ/kg ◊ K
Analysis Change in availability
(i) The exit temperature of hot combustion gases can y3 – y4 = 300 ¥ (– 0.2542) + 251.25
be evaluated by making energy balancy on the = 174.99 kJ/kg
heat exchanger: Total change in availability rate
Heat given by hot fluid = Heat gain by cold fluid = m (y3 – y4) = 90 ¥ 173.99
mh Cp,h (T3 – T4) = mc Cp, c (T2 – T1) = 15749.1 kW
Here mh = mc = 15.75 MW
and Cp,h = Cp,c = Cp of air It is decrease in availability rate of combustion
Therefore, T4 = T3 + T1 – T2 gases.
= 750 + 340 – 590 (iii) Rate of availability loss within control volume
= 500°C (773 K) = 15.75 MW – 13.095 MW
(ii) The net change in flow availability rate from inlet = 2.655 MW
to exit. The change in availability kg of a steady
flow system is given by Example 8.20 Steam enters a turbine with a pressure
y1 – y2 = T0 Ds – Dh – D ke – Dpe (kJ/kg) of 30 bar, a temperature of 400°C, a velocity of 160 m/s.
Here Dpe = Dke = 0, The steam exists as saturated vapour at 100°C with a
velocity of 100 m/s. At steady state, the turbine develops
Thus y1 – y2 = T0 Ds – Dh
work at the rate of 540 kJ/kg of steam flowing through the
For compressed air stream
turbine. The heat transfer between turbine and surround-
Dh = h2 – h1 = Cp (T2 – T1)
ings occurs at an average outer surface temperature of
= ¥ (863 – 613) = 251.25 kJ/kg
1.005 350 K. Develop an accounting of net availability carried
Ê ˆ
D s = C p ln T2 - R ln Ê p2 ˆ
in by steam, per unit mass of steam. Neglect the change
ÁË T ˜¯ ÁË p ˜¯ in potential energy between inlet and exit. Let T0 = 25°C,
1 1
p0 = 1 atm.
Ê 863 ˆ Ê 9.7 ˆ
= ¥ ln Á
1.005 – 0.287 ¥ ln Á
Ë 613 ˜¯ Ë 10 ˜¯ Solution
= 0.3525 kJ/kg ◊ K
Given The steam expands through a turbine in steady
Change in availability per kg state manner.
y1 – y2 = 300 ¥ 0.3525 – 251.25 Inlet: p1 = 30 bar
= – 145.5 kJ/kg T1 = 400°C = 673 K
The total change in availability of compressed air V1 = 160 m/s
= m (y1 – y2) = 90 ¥ (– 145.5) Exit: T2 = 100°C = 373 K
= – 13.095 MW V2 = 100 m/s
–ve sign indicates increase in availability rate. w1–2 = 540 kJ/kg
Change in availability rate of hot combustion Ts = 350 K
gases
T0 = 25°C = 298, p0 = 1 atm
Dh = h4 – h3 = Cp (T4 – T3)
D pe = 0
= 1.005 ¥ (773 – 1023)
= – 251.25 kJ/kg To find Net change of availability enters, flow and exit
the system.
ÊT ˆ Êp ˆ
Ds = C p ln Á 4 ˜ - R ln 4
Ë T3 ¯ ÁË p ˜¯ Analysis From steam tables, A-14,
3
At 30 bar, 400°C,
266 Thermal Engineering

h1 = 3230.9 kJ/kg,
s1 = 6.9212 kJ/kg ◊ K
At 100°C The reversible work or maximum work done by an
h2 = 2676.1 kJ/kg, engine is exactly equal to decrease in availability
s2 = 7.3549 kJ/kg ◊ K during a process. But the irreversibility is always
The net available heat carried per unit mass of steam involved in actual process. Therefore, the actual
flowing through the turbine. work done by an engine is always less than the
y1 – y2 = T0 Ds – Dh – Dke – Dpe reversible work. The second-law efficiency is relat-
where ed with actual and reversible work transfer by the
Dh = h2 – h1 = 2676.1 – 3230.9 = – 554.8 kJ/kg devices such as engines, turbines, compressors, etc.
T0 Ds = 298 ¥ (7.3549 – 6.9212) = 129.034 kJ/kg ◊ K The second-law efficiency is also called effective-
V22 - V12 100 2 - 160 2 ness and it is denoted as h II.
Dke = = = –7800 J/kg
2 2
= –7.8 kJ/kg
Dpe = 0 (given)
Then y1 – y2 = 129.034 – (–554.8) – (–7.8) – 0 It is defined as the ratio of actual thermal efficiency
= 691.634 kJ/kg to reversible thermal efficiency under the same
During a steady-state process, the heat transfer rate working conditions.
to surroundings
hth, act
q – w = Dh + Dke + Dpe h II = (8.28)
or q = 540 + (–554.8) + (–7.8) + 0
hth,rev
= –22.6 kJ/kg It is sometimes also referred as turbine effective-
The loss of availability with this heat rejection is the ness.
maximum work capacity of this heat quantity It can also be expressed as the ratio of the actual
Ê T ˆ Ê 298 ˆ work output and maximum possible work output
= Á1 - 0 ˜ q = Á1 - ˜ ( - 22.6)
Ë Ts ¯ Ë 350 ¯
Wact Wuseful
= – 3.36 kJ/kg hII = = (8.29)
Wrev Wrev
It is loss of availability with heat transfer to surround-
ings.
Availability and Irreversibility 267

where mc and mh are the mass flow rates for cold


and hot fluids, respectively.
The second-law efficiency of a power-consuming For direct contact, a heat exchanger as shown
device such as a compressor, pump or a blower can in Fig. 8.15 operates at steady state with no heat
be expressed as the ratio of minimum reversible transfer with its surroundings. The energy balance
work input to actual work input for such a heat exchanger is expressed as
Wrev m1h2 + m2 h2 = m3 h3
h II = (8.30)
Wact where m3 = m1 + m2 for a mass rate balance. The
second-law efficiency or effectiveness is given by
Heat transferred by hot fluid
e =
Heat absorbed by cold fluid
The performance of a refrigerator or a heat pump m3 y 3
is measured in terms of coefficient of performance. = ...(8.32)
m1y 1 + m2 y 2
Therefore, the second-law efficiency for such Hot
. stream
1 m1
devices is expressed as
(COP ) act
h II = ...(8.31) Mixed 3 2 Cold
(COP ) rev stream,
. stream
.
m3 m2

The heat exchanger shown in Fig. 8.14 operates at Example 8.21 1 kg water initially at 25°C is heated
steady state with no heat transfer with its surround- to 90°C using an electric heating coil. Assuming that the
ings and both streams are separately flowing and at heat losses to the surroundings at 300 K are negligible.
a temperature above T0. For such a heat exchanger, Calculate the first law and second law efficiencies of the
the efficiency is expressed as process.

Gain in availability of system Solution


hII = e =
Loss in availability of surroundings Given Heating of water
Actual heat transferred for use m = 1 kg
=
Total heat rejected by hot fluid T1 = 25°C, T2 = 90°C
T0 = 300 K Q=0
mc (y 4 - y 3 )
=
mh (y 1 - y 2 )

To find
(i) First law efficiency,
(ii) Second law efficiency.
268 Thermal Engineering

Assumption Specific heat of water, Cpw = 4.187 kJ/kg ◊ K.


Analysis The system is illustrated in Fig. 8.16.
(i) First law efficiency
Applying first law of thermodynamics
–We = U2 – U1 = m (u2 – u1)
= m Cpw (T2 – T1)
= 1 ¥ 4.187 ¥ (90 – 25) = 272.15 kJ
First law efficiency can be expressed as
Desired energy output
hI =
Required energy input
U 2 - U1 272.15 T0 = 25°C
= = ¥ 100 = 100%
-We 272.15 p0 = 100 kPa
(ii) Second law efficiency
To find
Increase in availability of water
(i) Actual power output,
f 2 – f 1 = (u2 – u1) – T0 (s2 – s1) + p0 (v2 – v1)
(ii) Reversible work,
= (h2 – h1) – T0 (s2 – s1)
(iii) Second-law efficiency,
From steam table A-12
(iv) Irreversibility, and
h1 = 105 kJ/kg, s1 = 0.367 kJ/kg ◊ K
(v) Availability of steam at inlet conditions.
h2 = 377 kJ/kg s2 = 1.193 kJ/kg ◊ K
Then f2 – f1 Analysis The properties of steam
= (377 – 105) – 300 ¥ (1.193 – 0.367) Inlet condition p1 = 3 MPa,
= 24.2 kJ/kg T1 = 450°C
Total availability increase of water s1 = 7.0834 kJ/kg ◊ K
f2 – f1 = m(f2 – f1) = 24.2 kJ h1 = 3344.0 kJ/kg.
Exit condition p2 = 0.2 MPa
Second law efficiency of the process
T2 = 150°C
Increase of availability of water
hII = h2 = 2768.8 kJ/kg,
Energy input
s2 = 7.2795 kJ/kg ◊ K
24.2
= = 0.089 or 8.9% Dead state p0 = 100 kPa
272.15
T0 = 25°C,
h0 = hf = 104.89 kJ/kg
Example 8.22 The steam enters a steam turbine
s0 = sf = 0.3674 kJ/kg ◊ K
steadily at 3 MPa and 450°C at a rate of 8 kg/s and
exits at 0.2 MPa and 150°C. The steam is losing heat (i) The actual power output from steam turbine
to the surroundings at 100 kPa and 25°C at a rate or Q - W = m (Dh + Dke + Dpe) = m (Dh)
300 kW. The changes in kinetic and potential energies are Using numerical value
negligible.Calculate – 300 kW – Wact
= (8 kg/s) ¥ (2768.8 – 3344.0) (kJ/kg)
(a) actual power output,
or Wact = 4301.6 kW
(b) the reversible work,
(ii) Reversible work done by the turbine
(c) second law efficiency,
Wrev = m [(h1 – h2) – T0 (s1 – s2)]
(d) the irreversibility, and = 8 ¥ [(3344.0 – 2768.8)
(e) the availability of steam at inlet conditions. – 298 ¥ (7.0834 – 7.2795)]
= 5069 kW
Solution (iii) The second-law efficiency
W 4301.6
Given A steam turbine with hII = act = = 84.86%
Wrev 5067
Availability and Irreversibility 269

(iv) Irreversibility (i) Maximum work possible in air motor


I = Wrev - Wact = Change in availability of compressed air
= 5069 – 4301.6 = 767.4 kW f1 – f0= u1 – u0 + p0 (v1 – v0) – T0 (s1 – s0)
(v) Availability of steam at the inlet condition where u1 – u0 = 0
y1 = (h1 – h0) – T0 (s1 – s0)
È RT RT ˘
= (3344.0 – 104.89) p0 (v1 – v0) = p0 Í - ˙
– 298 ¥ (7.0834 – 0.3674) Î p1 p0 ˚
= 1238 kJ/kg È 1 1 ˘
= 101.3 ¥ 0.287 ¥ 300 ¥ Í - ˙
Î 2000 101.3 ˚
Example 8.23 In a mine, the used air motor = – 81.749 kJ/kg
requires minimum 7 bar air pressure for its operation È T Ê p ˆ˘
and delivering 105 kJ/kg of work from air supplied. A T0 (s1 – s0) = T0 ÍCv ln 1 - R ln Á 1 ˜ ˙
ÍÎ T0 Ë p0 ¯ ˙˚
compressed air bottle of 1.66 m3 volume containing air at
20 bar and 300 K is available for operation of air motor. È Ê 2000 ˆ ˘
= 300 ¥ Í0 - 0.287 ¥ ln Á
If the ambient conditions are 1 atm and 300 K, Î Ë 101.3 ˜¯ ˙˚
Calculate: = – 256.82 kJ/kg
(a) Maximum work output of air motor, operated \ f1 – f 0 = – 81.749 – (– 256.82)
from one bottle of compressed air, and = 175 kJ/kg
(b) Second law efficiency of air motor. Total availability of compressed air
Wmax = m(f1 – f0)
Solution
= 38.5 ¥ 175 = 6740.6 kJ
Given Operation of air motor. (ii) Second law efficiency
V = 1.66 m3,p1 = 20 bar = 2000 kPa, T = 300 K Actual work delivered
hII =
Air motor: p2 = 700 kPa, wout = 105 kJ/kg, Maximum possible work
Dead state p0 = 1 atm = 101.3 kPa, T0 = 300 K 2625 kJ
= = 0.389 or 38.9%
To find 6740.6 kJ
(i) Maximum work delivered by air motor using one Example 8.24 A steam power plant is designed to
bottle of compressed air, operate on an ideal Rankine cycle. The boiler pressure
(ii) Second law efficiency of the system. is 40 bar and condenser pressure is 7.5 kPa. The
Assumption The specific gas constant of air, R = mass flow rate of water is 38 kg/s and enthalpy rise of
0.287 kJ/kg ◊ K. generated steam is 2630 kJ/kg. While change in entropy
is 5.5 kJ/ kg ◊ K.
Analysis The initial mass of compressed air in the The heating for steam generation in the boiler unit
bottle is provided by a steady stream of hot gases initially at
p1V
m1 = 2000 K and 1 atm pressure. The hot gas exhausts at 450 K
RT1
to the ambient at 300 K and 1 atm. The heating rate in the
2000 ¥ 1.66 boiler is 100 MW. Calculate
= = 38.5 kg
0.287 ¥ 300 (a) the mass flow rate of hot flue gases,
The mass of air remaining in bottle, when its pressure (b) Availability of hot flue gases at inlet conditions,
reaches to 7 bar (c) Availability of hot flue gases at boiler exit,
p2 V 700 ¥ 1.66 (d) Change in availability of hot flue gases in steam
m2 = = = 13.5 kg
RT2 0.287 ¥ 300 generation process, and
The mass of air supplied to air motor (e) second law efficiency in the entire process.
m = m1 – m2 = 38.5 – 13.5 = 25 kg Take Cp g = 1.1 kJ/kg ◊ K for hot flue gases.
Total work delivered by air motor
W = mw = 25 ¥ 105 = 2625 kJ
270 Thermal Engineering

Solution where hi – h0 = Cpg (Ti – T0)


= 1.1 ¥ (2000 – 300) = 1870 kJ/kg
Given Generation of steam with the help of hot flue
ÊT ˆ Ê 2000 ˆ
gases. si – s0 = Cp g ln Á i ˜ = 1.1 ¥ ln Á
Boiler pressure p1 = 40 bar ms = 38 kg/s Ë T0 ¯ Ë 300 ˜¯
Condenser pressure, p2 = 7.5 kPa Dhw = 2630 kJ/kg = 2.087 kJ/kg ◊ K
Heating rate, \ y1 = 1870 – 300 ¥ 2.087
Q = 100 MW = 1,00,000 kW
Hot flue gases, Ti = 2000 K, Te = 450 K, = 1243.95 kJ/kg
Cp g = 1.1 kPa Total availability at inlet conditions
pi = 1 atm, pe = 1 atm. = mg y1 = 58.65 ¥ 1243.95
Dead state: p0 = 1 atm = 101.3 KPa = 72,957.67 or 72.95 MW
T0 = 300 K. (iii) Availability of flue gases at boiler exit
DSw = 5.5 kJ/kg ◊ K mg y2 = mg [(he – h0) – T0 (se – s0)]
= 58.65
To find
È Ê 450 ˆ ˘
(i) Mass flow rate of flue gases, ¥ Í1.1 ¥ ( 450 - 300) - 300 ¥ 1.1 ¥ ln Á ˙
Î Ë 300 ˜¯ ˚
(ii) Availability of hot flue gases at boiler inlet
condition, = 1829.6 kW or 1.83 MW
(iii) Availability of hot flue gases at boiler at boiler (iv) Decrease in availability of hot flue gases
exit conditions, = mg (y1 – y2)
(iv) Decrease of availability of hot flue gases in the = 72.95 – 1.83 = 71.12 MW
boiler, and (v) Second law efficiency
(v) Second law efficiency in entire process. Availability increase of water during steam
generation
Analysis
ms ys = ms [Dhw – T0 Dsw]
(i) The mass flow rate of hot flue gases for 100 MW
= 38 ¥ [2630 – 300 ¥ 5.5]
heat supply to boiler
= 37,240 or 37.24 MW
Q = mg Cpg (Ti – Te)
1,00,000 = mg ¥ 1.1 ¥ (2000 – 450) Availability rise of water
\ hII = ¥ 100
or mg = 58.65 kg/s or 2.11 ¥ 105 kg/h Availability of flue gases
(ii) Availability of hot flue gases at boiler inlet 37.24
= ¥ 100 = 51.05%
y1 = (hi – h0) – T0 (s1 – s0) 72.95

Summary
Availability is a composite property of system
The quality of energy is always degraded during and surroundings.
a process, hence the entropy is generated, and the
opportunities to produce work are lost. the availability becomes zero.
available energy is that portion of heat energy
applied to a reversible engine, which could be than the work produced by the system because a
converted into useful work. portion of work produced by the system is lost
- to displace the atmosphere through a change of
tem at the specified state is called availability. volume. Thus
Wuseful = Wsys – Wsurr = W – p0 (v2 – v1)
Availability and Irreversibility 271

Wsurr is zero for cyclic


devices, steady flow devices and systems with to 2, the change in availability is referred as work
fixed boundaries. transfer during the process and it is expressed as
- Wrev = m(f1 – f2) (kJ)
ible work Wrev and actual useful work Wuseful for a or Wrev = m (y1 – y2) kW
process. For any system, QR with a constant
I = Wrev – Wuseful (kJ) = T0 Sgen (kJ) temperatrue reservoir at TR during a process then
or i = wrev – wuseful (kJ/kg) the maximum work done from states 1 to 2 is
= T0 sgen (kJ/kg) Ê T ˆ
Wrev = F1 – F2 + QR Á1 - 0 ˜ (kJ)
or I = Wrev - Wuseful (kW) Ë TR ¯
= T0 S gen (kW)
Ê T ˆ
entropy or Wrev = m (y1 – y2) + QR Á1 - 0 ˜ kW
Sgen is generated. For a totally reversible process, Ë TR ¯
entropy generation and irreversibilities are zero.
f) and a turbines, the second-law efficiency is defined as
steady flow system y are given as h Wuseful
hII = th =
f = (u – u0) – T0 (s – s0) + p0 (v – v0) (kJ/kg) hrev Wrev
V22 - V12
y = (h – h0) – T0 (s – s0) +
2 pump, compressors etc., the second law efficiency
+ (z2 – z1) g (kJ/kg) is
COP W
hII = = rev
(COP ) rev Wact

Glossary
High-grade energy Energy that can be converted equilibrium with the atmosphere
directly into useful work and other forms of energy with Useful work The net work done by the system; System
its equivalent quantity work–work done against atmosphere
Low-grade energy Energy that cannot be converted Irreversibility The difference between the reversible
into useful work and other forms of energy with its equiv- work and actual work during a process
alent quantity Availability function Availability of a closed system:
Available energy The part of energy that can be con- composite properties of the system and surroundings
verted into useful work Stream availability Availability of the fluid stream in
Unavailable energy The part of energy which cannot an open system
be converted into useful work and is rejected as waste Second law efficiency Ratio of reversible output to ac-
Exergy Work potential of energy, also called availabilty tual output for engines, turbines, etc., and ratio of revers-
Dead state State of system when it is in thermodynamic ible input to actual input for compressors, pumps, etc.

Review Questions
1. Define availabilty and unavailability. 3. Define dead state and useful work, maximum
2. What is the available energy? work.
272 Thermal Engineering

4. What is irreversibility? What are the effects of 14. How are the concept of entropy and unavailable
irreversibility on work output of a system? energy related to each other?
5. Discuss the availability of energy entering a sys- 15. An ideal gas is stored in a closed vessel at pressure
tem. p and temperature T.
6. Discuss the availability of a closed system. (a) If T = T0, derive an expression for the
7. Discuss the availability of a steady flow system. specific availability in terms of p, p0, T0 and
8. Discuss the steady flow availability function. the gas constant R.
9. Explain the loss in availabilty. (b) If p = p0, derive an expression for the
10. Derive an expression for available energy from specific availability in terms of T, T0, and
finite energy source at temperature T, when the the specific heat Cp, which can be taken as
environment temperature is T0. constant. Neglect the effect of motion and
11. What is meant by quality of energy? gravity.
12. What do you understand by energy and exergy? ÈT Ê T ˆ˘
(b) C p T0 Í - 1 - ln Á ˜ ˙
13. Define second-law efficiency of turbine, pump ÍÎ T0 Ë T0 ¯ ˙˚
and heat exchanger.

Problems
1. How much is the available energy of 1 kg of air 5. In a boiler, the saturated water at 250°C is
increased by heating it reversibily at constant evaporated into dry saturated steam at the rate of
pressure of 1.5 bar from 40°C to 250°C with a 5 kg/s by heat transfer from hot flue gases. The
lowest available temeprature of 20°C? gases are cooled during the process from 1127°C
[60 kJ/kg] to 527°C. The surrounding temperature is at
2. A constant temperature source is maintained 27°C. Find the following:
at 727°C, while the surrounding temperature (a) Mass flow rate of hot gases,
is 17°C. If the heat transferred from the source (b) Entropy change of hot gases, water and net
is 4000 kJ in a reversible manner; calculate the change in entropy,
availability of heat energy and unavailable heat. (c) Availability of hot gases and steam,
[2840 kJ, 1160 kJ]
(d) Unavailable energy of hot gases and steam,
3. A heat engine receives heat from a source at
(e) Increase in unavailable energy due to
1200 K at a rate of 500 kW and rejects the waste
heat to its surroundings at 300 K. The power irreversible heat transfer.
output of the heat engine is 180 kW. Determine Assume latent heat of steam at 250°C, hfg =
maximum power output and irreversibility rate 1716.2 kJ/kg, specific heat of hot gases Cpg =
for the process. [375 kW, 195 kW] 1.00 kJ/kg.
4. The heat is supplied to a reversible engine from [(a) 14.3 kg/s, (b) –8.002 kW/K, 16.4 kW/K,
a finite heat reservoir in a reversible manner. 8.40 kW/K (c) 6180.25 kW, 3658.81 kW,
During the heat supply process, the temperature (d) 2400.75 kW, 4922.2 kW, (e) 2521.44 kW]
of the working fluid increases from 523 K to 6. A crater lake has a base area of 10,000 m2 and
873 K. The water equivalent is 100 kJ/K. The is 12 m deep. The ground of the crater is nearly
heat rejection during the cycle takes place at a flat and is 140 m below the base of the lake.
surrounding temperature of 288 K. All processes Determine the maximum amount of electric work
are reversible. Determine the total heat abstracted, that can be generated by feeding this water to a
availability and the loss of availability. hydroelectric power plant. [45,780 kWh]
[35000 kJ, 20244.3 kH, 14755.7 kJ]
Availability and Irreversibility 273

7. A steam turbine receives steam at 6 MPa, 800°C. 13. A 10-kg iron disk brake on a car is initially at
It has a heat loss of 49.7 kJ/kg. For an exit 10°C. Suddenly the brake is applied and the brake
pressure of 15 kPa and surroundings at 20°C, find temperature rises by friction to 110°C. Determine
the actual work and reversible work, if the turbine the availability of the disk and energy depletion
has an isentropic efficiency of 90%. of the car gas, if the car engine has a thermal
[1483.9 kJ/kg, 1636.8 kJ/kg] efficiency of 35%. [64.6 kJ, 1285 kJ]
8. 2 kg of air at 500 kPa, 80°C expands isentropical- 14. Air flows at 1500 K, 100 kPa through a constant-
ly in a non-flow system until its volume doubles, pressure heat exchanger, giving energy to a heat
the temperature and pressure approach to am- engine and comes out at 500 K. At what constant
bient condition at 5°C and 100 kPa. Determine temperature should the same heat be delivered to
(a) maximum work, (b) the change in availability, provide same availability? [924 K]
and (c) irreversibility. 15. The exhaust gases from a gas turbine are used to
For air, take heat water. The gases leave the turbine at 650°C
Cv = 0.718 kJ/kg ◊ K, R = 0.287 kJ/kg ◊ K. and may be cooled to 145°C. The rate of the gas
[(a) 122.72 kJ, (b) 82.2 kJ, (c) 15.2 kJ] flow is 1510 kg/min. and the rate of water flow is
9. A 500-kg iron block initially at 200°C is allowed 1890 kJ/min. The water enters at 35°C. Assume
to cool to 27°C by transferring heat to its the mean specific heat of the gases and water as
surrounding air at 27° C. Determine the reversible 1.088 kJ/kg ◊ K and 4.187 kJ/kg ◊ K, respectively.
work and irreversibility for the process. The atmospheric temperature is 32°C. Determine
the loss of available energy resulting from the
[8191 kJ, 8191 kJ]
heat transfer. [310164.5 kJ/min]
10. A 5-kg iron block initially at 350°C is quenched
16. In a steady flow system, air enters at 10 bar,
in an insulated tank which contains 100 kg of
200°C and a velocity of 200 m/s and the exit is
water at 30°C. Assume the water vapourised
at 1.5 bar, 30°C and a velocity of 100 m/s. The
during the process condenses back in the tank.
ambient conditions are 1 bar, 30°C. For unit mass
The ambient conditions are 20°C and 100 kPa.
flow rate, determine the irreversibility and the
Determine (a) final equilibrium temperature,
effectiveness. Assume Cp = 1 kJ/kg ◊ K and R =
(b) the availability of the combined system at
0.287 kJ/kg ◊ K. [30 kJ/kg, 0.86]
initial and final states, and (c) the wasted work
potential during this process. 17. Air enters in an air turbine at a pressure of 6 bar,
327°C with a velocity of 100 m/s and leaves at
[(a) 31.7°C (b) 95.7 kJ (c) 219.3 kJ]
1 bar, 177°C and at 60 m/s. The flow is adiabatic
11. An insulated steam turbine receives 30 kg of
and surrounding air temperature is 300 K.
steam per second at 3 MPa and 350°C. The steam
Calculate
expands in the turbine to 0.5 MPa, where steam
at the rate of 5 kg/s is bled off. The temperature (a) Actual work done per kg of air,
of this steam is 200°C. The remaining steam is (b) Reversible work done per kg of air,
further expanded to 15 kPa and 90% quality, and (c) Availability of air entering and leaving the
then it is exhausted. Calculate the availability per turbine.
kg of steam entering and at both points at which Assume Cp = 1.0 kJ/kg ◊ K, Cv = 0.71 kJ/kg ◊ K
steam leaves the turbine and the second-law and steady flow conditions.
efficiency. [(a) 154.2 kJ/kg, (b) 222.78 kJ/kg,
[1109.6 kJ/kg, 755.3 kJ/kg, (c) 252.94 kJ/kg, 30.16 kJ/kg]
195 kJ/kg, 0.817%] 18. Calculate the availability of following closed
12. A refrigerator has a second-law efficiency of 45% systems.
and it removes heat at the rate of 3 kW. If the (a) 5 kg water at 1 bar and 90°C,
refrigerated space is maintained at 3°C, while the (b) 2 kg ice at 1 bar and –10°C,
surrounding air temperature is at 27°C, determine (c) 0.1 kg of steam at 40 bar and 500°C, and
the power input to the refrigerator. [0.58 kW] (d) 0.5 kg of wet steam at 0.1 bar, 0.85 dry
274 Thermal Engineering

Consider atmospheric pressure is 1 bar and tem- 23. A thermal energy storage system (A) composed
perature is 27°C. of water receives 1000 kJ of heat and heats up
[(a) 122 kJ, (b) 81.2 kJ, (c) 98.7 kJ (d) 623 kJ] water from 27°C to 100°C. The system remains
19. A flywheel with a moment of intertia of in liquid state throughout the heating process.
6.74 kg ◊ m2, rotates at 3000 rpm. As the flywheel An alternative storage system (B) uses water
is braked to rest, its kinetic energy is converted at 27°C that becomes dry saturated steam at
entirely to internal energy of the brake lining. 100°C and 1 atm, while receiving 1000 kJ of
The brake lining has a mass of 2.27 kg and has heat. The atmospheric conditions are 1 atm and
a specific heat of 4.19 kJ/kg ◊ K. Heat transfer to 300 K. Calculate the availability change in two
surroundings is negligible. processes. [105 kJ, 183.5 kJ]
(a) Determine final temperature of brake lining 24. A counterflow heat exchanger operating in
in °C, if its initial temperature is 16°C. steady state has oil and liquid water flowing in
(b) Maximum possible rotational speed in separate steams. The oil is cooled from 440 K to
rpm, that could be attained by the flywheel 320 K, while the water temperature increases from
using energy stored in the brake lining after 290 K to 305 K. No stream experiences pressure
flywheel has been braked to rest. Assume drop. The mass flow of oil is 500 kg/h. The oil
T0 = 16°C. [51°C, 711 rpm] and water can be treated as incompressible fluids
with constant specific heats of 2 and 4 kJ/kg ◊ K,
20. Water at 25°C, 1 bar is drawn from a mountain
respectively. Assume no heat is transferred to the
lake 1 km above a valley and allowed to flow
surroundings. Determine
through a hydraulic turbine generator to a pond
on the valley floor for steady-state operation. (a) mass-flow rate of water in kg/h
Calculate minimum mass-flow rate required to (b) second-law efficiency of heat exchanges
generate electricity at a rate of 1 MW. Let T0 = (c) hourly cost of loss of available energy if
25°C and p0 = 1 bar. [101.9 kJ/s] available energy is valued at Rs 10 per kWh
21. Saturated steam at 0.008 MPa and a mass flow Take T0 = 17°C, p0 = 1 atm.
rate of 2.6 ¥ 105 kg/h enters the condenser of a [2000 kg/h, 89%, Rs. 69.6]
100 MW power plant and exits a saturated liquid 25. A geothermal energy source is the underground
at 0.008 MPa. The cooling water enters at 15°C high temperature water or steam which is heated
and leaves at 35°C with negligible pressure drop. by the earth’s interior. Calculate the steady flow
For a steady state operation, determine availability per unit mass of water at 150°C and
(a) the rate of energy leaving the plant with the 0.6 MPa. [83.7 kJ/kg]
cooling water stream in MW, 26. Calculate irreversibilty per unit mass for a steady
(b) the rate of availability leaving the plant with flow steam turbine operating between 3 MPa,
the cooling water stream in MW. 450°C and 0.1 MPa. The work produced by the
[173.3 MW, 2.8 MW] turbine is 650 kJ/kg. [100.6 kJ/kg]
22. Air at 7 bar, 1000°C enters a turbine and expands 27. Air enters a compressor of a gas turbine at 0.9 bar,
to 1.5 bar, 665°C with a mass flow rate of 5 kg/s. 15°C with a velocity of 120 m/s. The air leaves
The turbine operates at steady state with negligible the compressor at a pressure of 4 bar, 200°C with
heat transfer with its surroundings. Assume the a velocity of 60 m/s. The process is adiabatic.
ideal gas model with 1.35 and neglecting kinetic Calculate the reversible work and irreversibility
and potential energy changes, determine per kg of gas during the process. Take atmospher-
ic temperature as 15°C, Cp = 1.004 kJ/ kg ◊ K and
(a) isentropic turbine efficiency, and
R = 0.287 kJ/kg ◊ K. [–159.48 kJ/kg, 20.86 kJ/k]
(b) second-law efficiency.
Take T0 = 25°C, p0 = 1 atm. [80%, 92.3%]
Availability and Irreversibility 275

Objective Questions
1. The available energy is
(a) high-grade energy (a) TdS – dQ (b) TdS0 – dQ
(b) portion of energy as useful work (c) T0 dS0 – dQ (d) T0 dS – dQ
(c) theoretical maximum amount of work 7. To improve the work capacity of energy transfer
(d) none of the above as heat from high temperature to low temperature,
2. Availability function of a closed system is (a) lower temperature should be lowered, keep-
(a) f = u + p0 v – T0s ing the temperature difference same
(b) f = u + p0 dv – T0 ds, (b) higher temperature should be increased
(c) f = du + p0 dv – T0 ds, keeping the temperature difference same
(d) f = u + p0 v + T0 s. (c) temperature difference should be increased
3. Availability function of a steady flow system (d) temperature difference should be decreased
(a) y = h – T0 s (b) y = h – T0 ds 8. If heat is transferred to a system, which of the
following statement is not correct?
(c) y = h + p0 dv – T0 ds (d) y = h + T0 s
(a) D ssystem decreases
4. Which one of the follwing represents availability?
(b) Dssystem increases
(a) DQ –T0 (DS0) (b) DQ – T (DS)
(c) (Dssystem + Dssystem) decreases
(c) DQ –T0 (DS) (d) DQ – T (DS0)
(d) (Dssystem + Dssystem) increases
5. Which one of the follwing represents unavailabil-
ity? 9. The degradation of energy is responsible for
(a) T0 (DS0) (b) T (DS) (a) entropy generation within the system
(c) T0 (DS) (d) T(DS0) (b) decrease of entropy within the system
6. Which one of the follwing represents irreversibil- (c) maximum work done by the system
ity of a closed system? (d) none of the above

9. (a)
8. (b) 7. (c) 6. (d) 5. (c) 4. (c) 3. (a) 2. (a) 1. (b)
Answers
276 Thermal Engineering

9
Thermodynamic Relations

Introduction
In the previous chapters, we have studied a number of properties like pressure, specific volume, temperature,
mass, internal energy, enthalpy, entropy and specific heats. Some of these are measured experimentally
and the remaining are calculated from experimental data. In order to calculate the properties that cannot
be measured directly, certain thermodynamic relations are required. These relations are developed and
discussed in this chapter.

9.1 HELMHOLTZ AND GIBBS Using Eq. (9.5)


FUNCTION: GIBBSIAN EQUATIONS dH = TdS – pdV + pdV + Vdp
= TdS + Vdp ...(9.6)
The Helmholtz and Gibbs functions are new func- Further differentiating Eq. (9.1) and Eq. (9.2)
tions used in problems related to chemical equilib- dF = dU – TdS – SdT
rium and combustion. The Helmholtz and Gibbs = – pdV – SdT ...(9.7)
functions are extensive properties. These are ex-
and dG = dH – TdS – SdT = Vdp – SdT ...(9.8)
pressed as
Rewritting the above derived equations, we get
Helmholtz Function F = U – TS ...(9.1)
the following set of Gibbsian relations:
Gibbs Function G = H – TS ...(9.2)
dU = TdS – pdV ...(9.5)
The corresponding specific values
dH = TdS + Vdp ...(9.6)
f = u – Ts ...(9.3)
dF = – pdV – SdT ...(9.7)
g = h – Ts ...(9.4)
dG = Vdp – SdT ...(9.8)
In order to derive Gibbsian equation using first
and second law of thermodynamics,
dQ – dW = dU
where dQ = TdS If z is the property expressed in the form
and dW = pdV z = f (x, y)
Thus dU = TdS – pdV ...(9.5) Its partial derivative is
Enthalpy H = U + pV Ê ∂z ˆ Ê ∂z ˆ
dH = dU + pdV + Vdp dz = Á ˜ dx + Á ˜ dy ...(9.9)
Ë ∂x ¯ y Ë ∂ y¯ x
Thermodynamic Relations 277

It can be expressed as Rearranging,


dz = Mdx + Ndy ...(9.10) ÈÊ ∂z ˆ Ê ∂x ˆ Ê ∂z ˆ ˘
ÍÁË ˜¯ ÁË ∂y ˜¯ + ÁË ∂y ˜¯ ˙ dy
where
Ê ∂z ˆ
M= Á ˜
Ê ∂z ˆ
and N = Á ˜ ÎÍ ∂x y z x˙
˚
Ë ∂x ¯ y Ë ∂ y¯ x
È Ê ∂z ˆ Ê ∂x ˆ ˘
The coefficient M is the partial derivative of z = Í1 - Á ˜ Á ˜ ˙ dz
Ë ∂x ¯ y Ë ∂z ¯ y
with respect to x, when the variable y is kept con- Î ˚
stant. Similarly, N is the partial derivative of z with But y and z are independent of each other. If z is
respect to y, when the variable x is being constant. constant, dz = 0. Then
Taking the partial derivative of M with respect to Ê ∂z ˆ Ê ∂x ˆ Ê ∂z ˆ
ÁË ˜¯ Á ˜ + Á ˜ = 0
y and N with respect to x ∂x y Ë ∂y ¯ z Ë ∂y ¯ x
Ê ∂M ˆ È ∂ Ê ∂z ˆ ˘ ∂2 z Ê ∂z ˆ Ê ∂x ˆ Ê ∂z ˆ
ÁË ∂ y ˜¯ = Í Á
Ë ¯ ˜ ˙ = ...(9.11) or ÁË ˜¯ Á ˜ = – Á ˜
x Î ∂ y ∂x y ˚ x ∂x ∂y ∂x y Ë ∂ y ¯ z Ë ∂ y¯ x

È ∂ Ê ∂z ˆ ˘ ∂2 z Ê ∂x ˆ Ê ∂ y ˆ Ê ∂z ˆ
Ê ∂N ˆ or ÁË ∂ y ˜¯ ÁË ∂z ˜¯ ÁË ∂x ˜¯ = –1 ...(9.15)
and Á
Ë ∂x ˜¯ y
= Í Á ˜ ˙ = ...(9.12)
ÍÎ ∂x Ë ∂y ¯ x ˙˚ y ∂x ∂y z x y

It is called the cyclic relation.


Since z is a property, thus it is an exact differen-
tial. Therefore, the two relations above are identi-
cal.
Ê ∂M ˆ Ê ∂N ˆ The equations that relate the partial derivative of
ÁË ∂ y ˜¯ = ÁË ∂x ˜¯ ...(9.13) properties of p, v, T and s of a compressible fluid
x y
are called Maxwell relations.
It is an important relation for partial derivatives
The four Gibbsian relations for a unit mass are
and is used to test a function whether it is an exact
du = Tds – pdv ...(i)
or inexact differentiation. An exact differentiable
dh = Tds + vdp ...(ii)
function is always a property or point function.
df = – pdv – sdT ...(iii)
dg = –sdT + vdp
Since u, h, f and g are the properties thus point
The function z = f (x, y) can also be expressed as functions and the above relations can be expressed
x = f(z, y), then in the form
dz = Mdx + Ndy
Ê ∂x ˆ Ê ∂ xˆ
dx = Á ˜ dz + Á ˜ dy ...(9.14)
Ë ∂z ¯ y Ë ∂ y¯ z Ê ∂M ˆ Ê ∂N ˆ
with ÁË ∂ y ˜¯ = ÁË ∂x ˜¯
x y
Using Eq. (9.14) in Eq. (9.9), we get
Applying the cyclic relation as
Ê ∂z ˆ ÏÔ Ê ∂x ˆ Ê ∂x ˆ ¸Ô Ê ∂z ˆ
dz = Á ˜
Ë ∂x ¯ y Ì ÁË ˜¯ dz + Á ˜ dy ˝ + Á ˜ dy Ê ∂M ˆ Ê ∂N ˆ
ÔÓ ∂z y Ë ∂y ¯ z Ô˛ Ë ∂y ¯ x Mdx + Ndy Æ Á = Á
Ë ∂ y ˜¯ x Ë ∂x ˜¯ y
ÈÊ ∂z ˆ Ê ∂x ˆ Ê ∂z ˆ ˘ Thus replacing M, N, y and x by T, p, v, s of each
dz = ÍÁ ˜ Á ˜ + Á ˜ ˙ dy
ÍÎË ∂ x ¯ y Ë ∂ y ¯ Ë ∂y ¯ x ˙˚ of Gibbsian equations in cyclic order, we get the
z
following:
Ê ∂z ˆ Ê ∂x ˆ
+ Á ˜ Á ˜ dz
Ë ∂x ¯ y Ë ∂z ¯ y
278 Thermal Engineering

The first relation leads to


Ê ∂T ˆ Ê ∂p ˆ The Gibbsian relations can be obtained by using the
ÁË ˜¯ = – Á ˜ ...(9.16)
∂v s Ë ∂s ¯ v following rule:
The second relation leads to (i) Each potential is flanked with its natural
Ê ∂T ˆ Ê ∂v ˆ variable as u = f (v, s), h = f (p, s), etc. The
ÁË ∂p ˜¯ = ÁË ∂s ˜¯ ...(9.17)
differential of these variables are multiplied
s p
The third relation leads to by its properties lying on their diagonally
opposite corners. For example,
Ê ∂p ˆ Ê ∂s ˆ
ÁË ˜ = ÁË ˜¯ ...(9.18) du = Tds – p dv
∂T ¯ v ∂v T
(ii) If the arrow is pointing away from the natu-
And the fourth relation leads to ral variable then a positive sign is used be-
Ê ∂s ˆ Ê ∂v ˆ fore the quantity and if the arrow is point-
ÁË ∂p ˜¯ = – ÁË ∂T ˜¯ ...(9.19) ing towards the quantity then a negative sign
T p
should be used before the quantity. For ex-
These derived relations are called the first, sec-
ample, if the arrow is pointing away from s,
ond, third and fourth Maxwell relations, respec-
a positive sign is considered for Tds, and the
tively. These are extremely important in thermody-
arrow is pointing towards v, a negative sign
namics and provide means of determining change
for pdv.
in entropy.

THERMODYNAMIC SQUARE
The Maxwell relations can also be written by using
Figure 9.1 shows a thermodynamic square, a mne- thermodynamic square or mnemonic diagram as
monic diagram which is very useful to obtain the follows:
Maxwell and Gibbsian relations. The corners are
(i) The partial derivative of the properties lying
levelled with properties T, p, s and v, respectively.
on a corner of a side with respect to the other
The sequence of properties T, p, s, v can be kept in
corner property is equated with partial de-
mind by remembering a single word riLoh (Tpsv),
rivative of the property on a corner opposite
which starts from left top corner of the square and
with respect to the other corner property of
runs clockwise.
the opposite side. For example,
g
T p Ê ∂T ˆ Ê ∂v ˆ
ÁË ∂p ˜¯ = ÁË ∂s ˜¯
s p

f h (ii) If the partial derivative of properties lying on


the horizontal line are equated then a positive
sign is used in the relation. If the partial de-
v u s
rivatives of properties lying on vertical lines
Fig. 9.1 are equated then a negative sign appears on
either side of equations. For example, partial
The sides of the square are levelled by potential derivative lying on vertical lines
u, f, g and h, starting from the bottom side and run
clockwise. The square is also stamped with two Ê ∂T ˆ Ê ∂p ˆ
ÁË ˜ = – ÁË ˜¯
diagonal arrows heading towards the left corners. ∂v ¯ s ∂s v
Thermodynamic Relations 279

Sr. No Gibbsian Relations Maxwell Relations The coefficient of isothermal compressibility, k is


defined as the ratio of fractional change in volume
Ê ∂T ˆ Ê ∂p ˆ to change in pressure when temperature remains
1. du = Tds – pdv ÁË ˜ = - ÁË ˜¯
∂v ¯ s ∂s v constant or in other words, the isothermal com-
Ê ∂T ˆ Ê ∂v ˆ pressibility is an indication of the change in volume
2. dh = Tds + vdp ÁË ∂p ˜¯ = ÁË ∂s ˜¯ with change in pressure, when the temperature is
s p
kept constant. It is expressed as
Ê ∂s ˆ Ê ∂p ˆ 1 Ê ∂v ˆ
ËÁ ∂v ¯˜ T
= Á
Ë ∂T ¯˜ v
3. df = – sdT – pdv k =– ...(9.22)
v ÁË ∂p ˜¯ T
Ê ∂s ˆ Ê ∂v ˆ
4. dg = –sdT + vdp
ÁË ∂p ˜¯ = - ÁË ∂T ˜¯ Ê ∂v ˆ
p where the quantity Á ˜ is the slope of the curve
T
Ë ∂p ¯ T
on a v–p diagram. Since volume decreases with an
increase in pressure, therefore, negative sign is used
in the above relation, which makes k positive for all
substances in all phases.
The equation of state for any thermodynamic sys- The coefficient of volumetric expansion and iso-
tem is expressed as thermal compressibility are thermodynamic prop-
erties like specific volume. the value of b and k for
f ( p, v, T ) = 0
liquid water at 1 atm for different temperatures are
Any two properties of the system are independent
given in Table 9.2.
and the value of the third property is fixed by
the values of the other two properties. The cyclic
b
relation for their partial derivative can be written as
k
Ê ∂p ˆ Ê ∂v ˆ Ê ∂T ˆ
ÁË ˜¯ ÁË ˜ = –1 ...(9.20)
∂v T ∂T ¯ p ÁË ∂p ˜¯ v T(°C) Density b ¥ 106 k ¥ 106
r(kg/m3) K –1 (bar) –1
0 999.84 –68.14 50.89
4 1000 0 50.01
The coefficient of volumetric expansion b is defined 10 999.70 87.90 47.81
as the ratio of fractional change in volume to change 20 998.21 206.6 45.90
in temperature when pressure is kept constant. It is 30 995.65 303.1 44.77
expressed as 40 992.22 385.4 44.24
1 Ê ∂v ˆ 50 988.04 457.8 44.18
b= Á ˜ ...(9.21)
v Ë ∂T ¯ p
In other words the coefficient of volumetric ex-
pansion is an indication of change in volume with
The isentropic compressibility a is the ratio of
respect to change in temperature, while pressure re-
fractional change in volume to change in pressure,
mains constant. It is also called coefficient of ther-
when entropy remains constant. It is an indication
mal expansion.
Ê ∂v ˆ of change in volume with change in pressure for an
The quantity Á ˜¯ is the slope of the curve isentropic process. It is given as
Ë
on v-T diagram. ∂T v
280 Thermal Engineering

1 Ê ∂v ˆ Ê ∂p ˆ Ê ∂p ˆ Ê ∂v ˆ
a =– ...(9.23) or ÁË ˜¯ = – ÁË ˜¯ ÁË ˜
v ÁË ∂p ˜¯ s ∂T v ∂v T ∂T ¯ p
The isentropic compressibility is measured in ( ∂v / ∂T ) p b
kPa–1, a reciprocal of pressure unit. =– = ...(9.26)
(∂v / ∂p)T k

b and k
For a perfect gas
pv = RT
Its differential form The internal energy, enthalpy and entropy are prop-
pdv + vdp = RdT ...(i) erties of the substance and they cannot be measured
When pressure p is kept constant, dp = 0 directly.
pdv = RdT But these properties are calculated from mea-
sured properties considering a simple compressible
Ê ∂vˆ R
ÁË ˜¯ = ...(ii) system.
∂T p p
and from ideal gas relation
1 p
= ...(iii) We assume that the internal energy is function of
v RT temperature and volume, i.e., u = u (T, v) and
The product of Eq. (ii) and (iii) yields coefficient
of volumetric expansion b Ê ∂u ˆ Ê ∂u ˆ
du = Á dT + Á ˜ dv ...(9.27)
Ë ∂T ˜¯ v Ë ∂v ¯ T
1 Ê ∂v ˆ p Ê Rˆ 1
b= Á ˜ = = ...(9.24)
v Ë ∂T ¯ p RT ÁË p ˜¯ T Ê ∂u ˆ
using the definition of Cv = Á
Thus, for an ideal gas, the coefficient of Ë ∂T ˜¯ v
volumetric expansion b is the inverse of absolute Ê ∂u ˆ
temperature of gas. Similarly, when temperature T Thus du = Cv dT + Á ˜ d v ...(9.28)
Ë ∂v ¯ T
is kept constant, dT = 0, then Eq. (i) gives
We also have du = Tds – pdv
Ê ∂v ˆ v
Now we assume entropy is function of tempera-
ÁË ∂p ˜¯ = – p
T ture and volume
The coefficient of isothermal expansion becomes s = f (T, v) and
1 Ê ∂v ˆ Ê ∂s ˆ Ê ∂s ˆ
k =– Á ˜ ds = Á dT + Á ˜ dv
v Ë ∂p ¯ T Ë ∂T ˜¯ v Ë ∂v ¯ T
1 Ê vˆ 1 Substituting ds above, we get
=– ÁË - p ˜¯ = p ...(9.25)
v Ê ∂s ˆ ÏÊ ∂s ˆ ¸
du = T Á
Ë ∂T ˜¯ v ÌÁË ˜¯ T - p ˝ dv ...(9.29)
dT +
Thus for an ideal gas, the isothermal compress- Ó ∂v T ˛
ibility k is inverse of absolute pressure of gas. Equating the coefficient of dT and dv in
Eq. (9.28) and (9.29) we get
b and k
Ê ∂s ˆ
Cv = T Á and
The cyclic relation for properties p, v, T Ë ∂T ˜¯ v
Ê ∂p ˆ Ê ∂T ˆ Ê ∂v ˆ
ÁË ˜ Á ˜ = –1 Ê ∂u ˆ Ê ∂s ˆ
∂T ¯ v Ë ∂v ¯ p ÁË ∂p ˜¯ ÁË ˜¯ = T ÁË ˜¯ – p ...(9.30)
T ∂v T ∂v T
Thermodynamic Relations 281

Using the third Maxwell relation Using the fourth Maxwell relation
Ê ∂s ˆ Ê ∂p ˆ Ê ∂s ˆ Ê ∂v ˆ
ÁË ˜¯ = ÁË ˜ ÁË ∂p ˜¯ = – ÁË ∂T ˜¯
∂v T ∂T ¯ v p
T
Ê ∂u ˆ Ê ∂p ˆ Using in Eq. (9.34), we get
We get ÁË ˜¯ = T ÁË ˜ –p ...(9.31)
∂v T ∂T ¯ v
È Ê ∂v ˆ ˘
dh = Cp dT + Ív - T Á
Ë ∂T ˜¯ p ˙
Then Eq. (9.28) becomes dp ...(9.37)
È Ê ∂p ˆ ˘ Î ˚
du = Cv dT + Í T Á
Ë ˜¯ - p ˙ dv ...(9.32) The change in enthalpy of a simple compressible
Î ∂ T v ˚ fluid can be calculated as
The change in internal energy of a simple T2 p2 È Ê ∂p ˆ ˘
compressible fluid can be calculated as h2 – h1 = Ú
T1
Cp dT + Ú
p1
Ív - T ÁË
Î
˜ ˙ dp
∂T ¯ p ˚
T2 v2 È Ê ∂p ˆ ˘
u2 – u1 = ÚT1
Cv dT + Ú
v1
ÍT ÁË
Î ∂T v
˜¯ - p ˙ dv
˚
...(9.38)

...(9.33)

The entropy is a property of the system, it cannot


Assuming the enthalpy as a function of temperature be measured directly. but it is calculated in terms
and pressure, i.e., h = h(T, p) and of other easily measurable properties, the pressure,
temperature and specific volume. The entropy can
Ê ∂h ˆ Ê ∂h ˆ
dh = Á dT + Á ˜ dp be expressed in the functional form as
Ë ∂T ˜¯ p Ë ∂p ¯ T
s = f (T, v) ...(i)
Using the definition of Cp
s = f(p, T ) ...(ii)
Ê ∂h ˆ s = j(p, v) ...(iii)
Cp = Á
Ë ∂T ˜¯ p
Using Eq. (i) and taking its partial derivative
Ê ∂h ˆ Ê ∂s ˆ
Thus dh = Cp dT + Á ˜ dp ...(9.34) Ê ∂s ˆ
ds = Á dT + Á ˜ dv
Ë ∂p ¯ T Ë ∂T ˜¯ v Ë ∂v ¯ T
We have dh = Tds + vdp
Ê ∂s ˆ Ê ∂s ˆ
Further, choosing the entropy as function of and Tds = T Á ˜ dT + T Á ˜ dv
Ë ∂T ¯ v Ë ∂v ¯ T
temperature and pressure,
s = f (T, p) For a constant volume process.
dT
Ê ∂s ˆ Ê ∂s ˆ ds = Cv
and ds = Á dT + Á ˜ dp
Ë ∂T ˜¯ p Ë ∂p ¯ T
T
Using above, we get Ê ∂s ˆ
or Cv = T Á ...(9.39)
Ë ∂T ˜¯ v
Ê ∂s ˆ È Ê ∂s ˆ ˘
dh = T Á dT + ÍT Á ˜ + v ˙ dp ...(9.35)
Ë ∂T ˜¯ p ÍÎ Ë ∂p ¯ T ˙˚
From Maxwell’s third relation
Ê ∂s ˆ Ê ∂p ˆ
Comparing Eq. (9.34) with Eq. (9.35), we have ÁË ˜¯ = ÁË ˜
∂v T ∂T ¯ v
Ê ∂s ˆ Ê ∂p ˆ
Cp = T Á
Ë ∂T ˜¯ p
and Therefore, Tds = Cv dT + T Á dv ...(9.40)
Ë ∂T ˜¯ v
Ê ∂h ˆ Ê ∂s ˆ Equation (9.40) is known as the first Tds relation,
ÁË ∂p ˜¯ = T ÁË ∂p ˜¯ + v ...(9.36) Re-writing (Eq. (9.40)) in the form
T T
282 Thermal Engineering

∂T Ê ∂p ˆ The Eq. (9.46) is known as the second Tds


ds = Cv + Á dv ...(9.41)
T Ë ∂T ˜¯ v relation.
The change in entropy
For an ideal gas pv = RT
R p dT Ê ∂v ˆ
– Á
Ë ∂T ˜¯ p
or = ds = Cp dp ...(9.47)
v T T
For a constant-volume process,
For an ideal gas
R Ê ∂p ˆ R v
= Á
v Ë ∂T ˜¯ v =
p T
∂T R
or ds = Cv + dv ...(9.42) For constant-pressure process
T v
Ê ∂p ˆ
R Ê ∂v ˆ
= Á
Further, the term Á
Ë ∂T ˜¯ v
of Eq. (9.41) can be p Ë ∂T ˜¯ p
expressed as
dT R
Ê ∂p ˆ Ê ∂v ˆ Ê ∂v ˆ b Thus ds = Cp – dp ...(9.48)
ÁË ˜ = ÁË ˜ ÁË ∂p ˜¯ = k T p
∂T ¯ v ∂T ¯ p T
1 Ê ∂v ˆ
Therefore, Eq. (9.41) can be written as Further, b = Á ˜
v Ë ∂T ¯ p
dT b
ds = Cv + dv ...(9.43) Ê ∂v ˆ
k bv = Á
Ë ∂T ˜¯ p
T or
Using functional relation for entropy as in
Eq. (ii) and taking its partial derivative Therefore, Eq. (9.47) can also be expressed in
the form
Ê ∂s ˆ Ê ∂s ˆ dT
ds = Á ˜ dT + Á ˜ dp – b vdp
Ë ∂T ¯ p Ë ∂p ¯ T ds = Cp ...(9.49)
T
Ê ∂s ˆ Ê ∂s ˆ Now using s = j(p, v)
and Tds = T Á dT + T Á ˜ dp ...(9.44)
Ë ∂T ˜¯ p Ë ∂p ¯ Its partial derivative
T
For a constant-pressure process Ê ds ˆ Ê ∂s ˆ
ds = Á ˜ dp + Á ˜ dv
dT Ë dp ¯ v Ë ∂v ¯v
ds = Cp
T Ê ∂s ˆ Ê ∂s ˆ
and Tds = T Á ˜ dp + Á ˜ Tdv ...(9.50)
Ê ∂s ˆ Ë ∂p ¯ v Ë ∂v ¯ p
Cp = T Á
Ë ∂T ˜¯ p
or ...(9.45)
Using chain relation of derivatives of properties
Further, from Maxwell’s fourth relation
Ê ∂s ˆ Ê ∂p ˆ Ê ∂T ˆ
Ê ∂s ˆ Ê ∂v ˆ ÁË ∂p ˜¯ ÁË ∂T ˜¯ ÁË ∂s ˜¯ = 1
ÁË ∂p ˜¯ = – ÁË ∂T ˜¯ v v v
p
T
Ê ∂s ˆ Ê ∂s ˆ Ê ∂T ˆ Cv Ê ∂T ˆ
Using in Eq. (9.44), we get or ÁË ∂p ˜¯ = ÁË ∂T ˜¯ ÁË ∂p ˜¯ = T ÁË ∂p ˜¯
v v v v
Ê ∂v ˆ We have
Tds = Cp dT – T Á dp ...(9.46)
Ë ∂T ˜¯ p
1 Ê ∂v ˆ
b = Á ˜
v Ë ∂T ¯ p
Thermodynamic Relations 283

1 Ê ∂v ˆ Ê ∂v ˆ Ê ∂p ˆ
k= Cp dT – T Á dp = Cv dT + T Á dv
Ë ∂T ˜¯ p Ë ∂T ˜¯ v
and
v ÁË ∂p ˜¯ T
k Ê ∂T ˆ Ê ∂p ˆ Ê ∂v ˆ
or (Cp – Cv ) dT = T Á dv + T Á dp
or
b
= Á
Ë ∂p ˜¯ v Ë ∂T ˜¯ v Ë ∂T ˜¯ p
Therefore, Ê ∂p ˆ Ê ∂v ˆ

Ë ∂T ˜¯ p
TÁ dp
Ë ∂T ˜¯ v
dv
Ê ∂s ˆ Cv k
ÁË ∂p ˜¯ = T b ...(9.51) or dT = + ...(9.56)
v C p - Cv Cp - Cv
Similarly, from chain relation at constant pres- For an equation of state T = f ( p, v) and
sure
Ê ∂T ˆ Ê ∂T ˆ
Ê ∂ v ˆ Ê ∂s ˆ Ê ∂T ˆ dT = Á ˜ dv + Á dp ...(9.57)
ÁË ˜¯ ÁË ˜ Á ˜ =1 Ë ∂v ¯ p Ë ∂p ˜¯ v
∂ s p ∂T ¯ p Ë ∂v ¯ p
Comparing Eq. (9.56) and Eq. (9.57), we get
Ê ∂s ˆ Ê ∂s ˆ Ê ∂T ˆ C p Ê ∂T ˆ
and ÁË ∂ v ˜¯ = ÁË ∂T ˜¯ ÁË ∂ v ˜¯ = T ÁË ∂ v ˜¯ Ê ∂p ˆ Ê ∂v ˆ

p p p p Ê ∂T ˆ

Ë ∂T ˜¯ v Ê ∂T ˆ Ë ∂T ˜¯ p
ÁË ∂ v ˜¯ = and ÁË ∂p ˜¯ =
Cp 1 C p - Cv C p - Cv
= ...(9.52) p v
T bv Both these equations yield to
Therefore, we get third Tds relation in the form
k Ê ∂p ˆ Ê ∂v ˆ
1 Cp – Cv = T Á Á ˜ ...(9.58)
Tds = Cv dp + Cp dv ...(9.53) Ë ∂T ˜¯ v Ë ∂T ¯ p
b bv
From cyclic relation for these properties
Ê ∂p ˆ Ê ∂T ˆ Ê ∂ v ˆ
ÁË ˜ = –1
∂T ¯ v ÁË ∂ v ˜¯ p ÁË ∂p ˜¯ T
Recalling the definitions of Cp and Cv,
Ê ∂h ˆ Ê ∂p ˆ Ê ∂v ˆ Ê ∂p ˆ
Cp = Á ÁË ˜¯ =– ÁË ˜ Á ˜
Ë ∂T ˜¯ p ∂T v ∂T ¯ p Ë ∂v ¯ T
2
Ê ∂u ˆ Ê ∂ v ˆ Ê ∂p ˆ
Thus, Cp – Cv= –T Á ...(9.59)
and Cv = Á
Ë ∂T ˜¯ v Ë ∂T ˜¯ p ÁË ∂ v ˜¯
T
We have also obtained for a simple compressible It is very important equation in thermodynamics.
fluid, Eq. (9.36) The following facts can be concluded:
Ê ∂h ˆ Ê ∂s ˆ Ê ∂v ˆ
(i) The slope Á may be positive or nega-
Cp = Á
Ë ∂T ˜¯ p
=T Á
Ë ∂T ˜¯ p
...(9.54) Ë ∂T ˜¯
tive, but its square is always positive. But the
Similarly, from Eq. (9.30)
Ê ∂p ˆ
slope Á ˜ for any substance is always
Ê ∂u ˆ Ê ∂s ˆ Ë ∂v ¯ T
Cv = Á =TÁ ...(9.55)
Ë ∂T ˜¯ v Ë ∂T ˜¯ v negative, therefore, Cp – Cv is always posi-
tive. The temperature T is the absolute tem-
Cp – Cv perature, which is always positive.
An expression for the difference between Cp – Cv (ii) As the absolute temperature approaches
can be obtained by equating the two differential zero, then
equations for entropy given by Eqs. (9.40) and Cp ª Cv
(9.46).
284 Thermal Engineering

Ê ∂v ˆ Similarly, for s = f(p, T )


(iii) When Á
Ë ∂T ˜¯ p
= 0 for an incompressible flu-
Ê ∂s ˆ 1
ÁË ˜¯ = – ∂T ...(iv)
ids, then the two specific heats are identical ∂T v Ê ˆ Ê ∂p ˆ
i.e. Cp = Cv = C ÁË ∂p ˜¯ ÁË ∂s ˜¯
s T

(iv) For an ideal gas Using Eq. (iii) in Eq. (i) and Eq. (iv) in Eq. (ii);
Ê ∂v ˆ R v we get
ÁË ˜¯ = =
∂T p p T Cv 1
=– ...(v)
Ê ∂p ˆ RT T Ê ∂T ˆ Ê ∂ v ˆ
ÁË ˜ Á ˜
and ÁË ∂ v ˜¯ = – 2 ∂v ¯s Ë ∂s ¯ T
T v
Using in Eq. (9.59), we get Cp 1
and =– ...(vi)
2 T Ê ∂T ˆ Ê ∂p ˆ
Ê vˆ Ê T ˆ ÁË ∂p ˜¯ ÁË ∂s ˜¯
Cp – Cv = –T Á ˜ ¥ Á - R 2 ˜ = R ...(9.60)
ËT ¯ Ë v ¯ s T

Equation (9.59) can also be expressed in Dividing (vi) by (v),


terms of coefficient of volumetric expansion Ê ∂T ˆ Ê ∂v ˆ
and isothermal compressibility Cp ÁË ∂ v ˜¯ ÁË ∂s ˜¯
s T
1 Ê ∂v ˆ = ...(9.62)
b = Á ˜ Cv Ê ∂T ˆ Ê ∂p ˆ
v Ë ∂T ¯ p ÁË ∂p ˜¯ ÁË ∂s ˜¯
s T
1 Ê ∂v ˆ
and k =– Rearranging Eq. (9.62), we get
v ÁË ∂p ˜¯ T
Equation (9.59) takes the form
Cp ÈÊ ∂p ˆ Ê ∂T ˆ ˘ ÈÊ ∂ v ˆ Ê ∂s ˆ ˘
= ÍÁ ˜ Á ˜ ˙ ÍÁË ˜¯ Á ˜ ˙ ...(9.63)
vTb 2 Cv ÍÎË ∂T ¯ s Ë ∂v ¯ s ˙˚ ÎÍ ∂s T Ë ∂p ¯ T ˙˚
Cp – Cv = ...(9.61)
k The chain rule allows us to write
Cp /Cv Ê ∂v ˆ Ê ∂v ˆ Ê ∂s ˆ
ÁË ∂p ˜¯ = ÁË ∂s ˜¯ ÁË ∂p ˜¯
An expression for ratio of two specific heats of T T T
an ideal gas can be obtained from Eqs. (9.30) and
Ê ∂p ˆ Ê ∂p ˆ Ê ∂T ˆ
(9.36) and rearranging to obtain and ÁË ∂v ˜¯ = ÁË ∂T ˜¯ ÁË ∂v ˜¯
s
Cv Ê ∂s ˆ s s
= Á
Ë ∂T ˜¯ v
...(i) Then Eq. (9.63) takes the form
T
Cp Ê ∂s ˆ Cp Ê ∂v ˆ Ê ∂p ˆ
= Á
Ë ∂T ˜¯ p
...(ii) = Á ˜ Á ˜ ...(9.64)
T Cv Ë ∂p ¯ T Ë ∂v ¯ s
Using cyclic relation for s = f (v, T )
Where ratio Cp /Cv = g (gamma) for ideal gases.
Ê ∂s ˆ Ê ∂T ˆ Ê ∂ v ˆ From Eqs. (9.22) and (9.23), the above expres-
ÁË ˜ = –1
∂T ¯ v ÁË ∂ v ˜¯ s ÁË ∂ s ˜¯ T sion can be written in terms of the isothermal and
isentropic compressibilities as
Ê ∂s ˆ 1
Thus ÁË ˜¯ = – ∂T ...(iii) Cp k
∂T v Ê ˆ Ê ∂v ˆ g = = ...(9.65)
ÁË ∂ v ˜¯ ÁË ∂s ˜¯ Cv a
s T
Thermodynamic Relations 285

Joule–Thompson conducted experiments on


porous plug for the same gas with given initial
pressure p1 and temperature T1 but with different
The porous plug experiment was designed to mea- downstream pressures p¢2 p¢2 , p≤2, etc. The different
sure the resulting temperature change, when a fluid exit temperatures were obtained after expansion
flows steadily through a porous plug, which is in- as T2, T 2¢ , T 2≤, respectively. The downstream pres-
serted in a thermally insulated horizontal pipe as sure was altered by controlling the opening of the
shown in Fig. 9.2(a). The steady flow through such throttle valve. The plot of exit pressures and tem-
a restricted passage is called throttling process or peratures for a gas yields an isenthalpic curve as
isenthalpic process. It is an irreversible process. shown in Fig. 9.2(b). During the throttling process,
A gas at initial pressure p1, temperature T1 and the temperature may decrease, increase or remains
enthalpy h1 flows steadily through a porous plug constant. The Joule–Thompson coefficient is the
located in an insulated, horizontal tube and emerges slope of isenthalpic curve. It is designated as m and
into a space, which is maintained at pressure p2. defined as
Consider the region enclosed by dotted lines as
Ê ∂T ˆ
control volume. Since the pipe is insulated and it m = Á ...(9.66)
Ë ∂p ˜¯ h
does not experience any work transfer; Q = 0, and
W = 0. If the kinetic and potential energy changes Joule and Thomson conducted a series of experi-
are negligible, then the steady-flow energy equation ments for real gases on a porous plug with constant
reduces to inlet pressure and temperature. The gas is forced to
h1 = h2 flow through different sizes of porous plugs, each
Thermometer giving a different set of T2 and p2. When these ex-
perimental data are plotted on a T –p diagram, it
1 2 gives a family of curves as shown in Fig. 9.3. It
is observed that each curve passes through a maxi-
mum point referred as inversion point, A curve
p1, T1, h1 p2, T2, h2 passing through the inversion point is known as an
inversion curve.
The temperature on the point is known as inver-
Porous plug Control volume sion temperature. The slope of isenthalpic curve
(a) Porus plug experiment of Joule–Thompson Ê ∂T ˆ
m= Á is positive. If the state after throttling
T Ë ∂p ˜¯ h
falls within the inversion curve then the throttling
p2¢ produces a drop in temperature with decrease in
p2¢¢
T2¢ pressure. Therefore, the region within the inversion
T2¢¢ Isenthalpic curve
T2 curve is known as the cooling zone. At room tem-
p2 State before
T1 throttling perature and at low moderate pressures, the state
of most gases after throttling falls inversion curve.
Similarly, if the state after throttling falls out
of the inversion curve where the slope isenthalpic
Ê ∂T ˆ
curve m = Á is negative, the temperature
Ë ∂p ˜¯ h
p1 p
(b) Throttling process on T–p diagram
increases with decrease in pressure and there is a
Fig. 9.2
heating effect after throttling. Therefore, the region
286 Thermal Engineering

For an ideal gas


v R
=
T p
Ê ∂v ˆ R v
\ ÁË ˜¯ = = ... (9.69)
∂T p p T
Using in Eq. (9.68), for an ideal gas, we get
1 Ï v¸
m =– Ìv - T ˝ = 0
Cp Ó T˛
[Thomson later became Lord Kelvin. Therefore,
the coefficient is also referred as Joule–Kelvin
Fig. 9.3 coefficient].
outside the inversion curve is known as heating 9.9.2 The
zone. The Joule–Thomson coefficient m decides the
heating or cooling effect with decrease in pressure.
When the data obtained from porous plug experi-
Ï < 0 T increases ments are plotted on a an h – p diagram. The slope
Ô
m Ì= 0 T remains constant ...(9.67) of curve gives the constant temperature coefficient,
Ô > 0 T decreases it is denoted by CT and is expresed as
Ó
The state of any ideal gas after throttling always Ê ∂h ˆ
CT = Á ˜ ...(9.70)
falls at the point on the curve, where slope m = Ë ∂p ¯ T
Ê ∂T ˆ Using the cyclic relationship to properties h, p
ÁË ∂p ˜¯ = 0. Therefore, the temperature remains and T
h
constant after throttling for ideal gases. (∵ h = Ê ∂h ˆ Ê ∂ p ˆ Ê ∂T ˆ
const., \ T = const.) ÁË ∂ p ˜¯ ÁË ∂T ˜¯ ÁË ∂h ˜¯ = –1
T h p

1 1
CT = –1
p v T and Cp data m Cp
We have Eq. (9.37) CT
or m =– ...(9.71)
Cp
È Ê ∂v ˆ ˘
dh = Cp dT + Ív - T ÁË ˜ ˙ dp For a perfect gas, m = 0, and at constant tempera-
Î ∂T ¯ p ˚
ture dh = 0, thus CT = 0.
For constant enthalpy process. dh = 0, then
Ê dT ˆ Ê ∂v ˆ
=v–TÁ EQUATION
Ë ∂T ˜¯ p
– Cp Á ˜
Ë dp ¯ h
The Clausius–Clapeyron equation relates satura-
Ê ∂T ˆ tion pressure, saturation temperature, enthalpy of
or m= Á
Ë ∂p ˜¯ h vaporisation and specific volume of two phases of
1 È Ê ∂v ˆ ˘ saturated fluid during phase change.
=– Ív - T ÁË ˜ ˙ ...(9.68)
Cp Î ∂T ¯ p ˚ Considering entropy as a function of temperature
and specific volume s = f (T, v) and
Thermodynamic Relations 287

Ê ∂s ˆ The above equation becomes


Ê ∂s ˆ
ds = Á ˜ dT + Á ˜ dv ...(9.72) phf g
Ë ∂T ¯ v Ë ∂v ¯ T Ê dp ˆ
ÁË ˜¯ = ...(9.78)
During the phase change of a pure substance, dT sat RT 2
the saturated temperature remains constant, thus This equation is referred as Clausius Clapeyron
dT = 0 and the above equation reduces to equation. Rearranging the above equation,
dp h fg dT
Ê ∂sˆ =
ds = Á ˜ dv R T2
Ë ∂ v¯ T p
On integration,
ds Ê ∂sˆ
or = Á ˜ ...(9.73) h fg È 1 1 ˘
dv Ë ∂ v¯ T Êp ˆ
In Á 2 ˜ = Í - 2˙ ...(9.79)
Using the third Maxwell relation, we get Ë p1 ¯ R Î T1 T ˚
This equation can be used more frequently.
Ê ∂sˆ Ê ∂p ˆ ds
ÁË ˜¯ = ÁË ˜¯ = dv ...(9.74) The change in specific entropy during a phase
∂v T ∂T v
change from saturated liquid to saturated vapour
For two phases of a pure substance in equilib- can be obtained from Eq. (9.66).
rium, the pressure and temperature are independent
The change in specific internal energy can be
of specific volume. Thus the derivative in Eq, (9.74)
determined by using
may be written as
dh = du + pdv
Ê ∂p ˆ Ê dp ˆ Integration over a phase change leads to
ÁË ˜ = ÁË ˜¯
∂T ¯ v dT sat hg – hf = ug – uf + p(vg – vf )
Ê dp ˆ ds sg - sf sf g hfg = ufg + pvfg
and ÁË ˜¯ = = = ...(9.75)
dT sat dv vg - vf vf g or ufg = hfg– pvfg ... (9.80)
the pressure and temperature during phase change, Thus change in specific internal energy during a
remain constant, thus phase change at constant temperature and pressure
q = h 2 – h1 = hg – hf = hfg can be determined from Eq. (9.80).
and q = Tsfg for a constant temperature process
The students of a thermodynamics class
hf g went to a hill station on a trip. When they reached the top
\ sfg = ...(9.76)
T of a hill they interested to know the approximate height
Using Eq. (9.75), we get of the hill from sea level. Since they had not carried
any instrument with them, they decided to use their
Ê dp ˆ hf g
ÁË ˜¯ = ...(9.77) thermodynamics knowledge to calculate the height of
dT sat T vf g the hill. In this connection, they boiled the water on the
This equation is known as Clapeyron equation. hill and found it boiled at 90°C. Assume the atmospheric
The derivative dp/dT represents the slope on the temperature is 27°C and the pressure variation is
governed by pv = p0 v0 where p0 is sea level pressure,
pressure–temperature curve. Equation (9.76) can
101kPa. Estimate the height of the hill.
also be applied to solid–gas and solid–liquid phase
changes. Solution
At temperatures relatively high but below
Given Tb = 90°C = 363 K, p0 = 101 kPa,
critical point, vg >> vf and pure substance follows
T0 = 27°C = 300K
the ideal gas behaviour
RT To find The height of hill.
vf g = vg =
p
288 Thermal Engineering

Assumptions Calculate the enthalpy of vaporisation


(i) The normal boiling temperature of water is as of water at 300°C. Use the steam table for required data.
100°C = 373 K. Compare the value obtained with the Clapeyron equa-
(ii) The latent of vapourisation is 2257 kJ/kg. tion.
(iii) Acceleration due to gravity g = 9.81 m/s2. Solution
Analysis The Clausius–Clapeyron equation Given T = Ts = 300°C = 573 K
Ê p2 ˆ h fg È 1 1˘
ln Á ˜ = Í - ˙ To find The enthalpy of vaporisation.
Ë p1 ¯ R Î T1 T2 ˚
Analysis From steam table at 300°C,
R 8.314 p = 85.81 bar,
R for water = u = = 0.461 (kJ/kg ◊ K)
M 18 hf g = 1404.9 kJ/kg
Using the values, we get vg = 0.02167 m3/kg,
vf = 0.001404 m3/kg
Êp ˆ ( 2257 kJ/kg) È 1 1 ˘
ln Á 2 ˜ = ¥ - Ê dp ˆ
Ë p1 ¯ (0.461 kJ/kg.K) ÍÎ 373 363 ˙˚ For calculating the slope Á , we take
Ë dT ˜¯ 300∞C
= – 0.361 ...(i)
Ts1 = 295°C p1 = 79.93 bar
Further the pressure difference across a vertical fluid
Ts2 = 305°C p2 = 92.02 bar
column, measured from top is given as
vfg = vg – vf = 0.02167 – 0.001404 = 0.02027 m3/kg
dh
or dp = –rgdh = – g ...(ii) Ê ∂p ˆ p - p1
v Then slope Á = 2
Ë ∂T ˜¯ 300∞C Ts2 - Ts1
Using relation pv = p0 v0 p + dp
92.02 - 79.93
p v =
or v = 0 0 305 - 295
p A = 1.209 bar/°C or 120.9 kPa/K
Using in Eq. (ii), we get mg
From Clapeyron equation
g dh dh Ê dp ˆ h fg
or dp = – p ÁË dT ˜¯ =
p0 v0 sat Tv fg
dp g dh Therefore,
or =–
p p0 v0 hf g = (120.9 kPa/K) ¥ (573K) ¥ (0.02027 m3/Kg)
Integrating on both sides = 1404.6 kJ/kg
p The approximated value is slightly different from tab-
Êp ˆ gh ulated value, this difference may be due to approximation
ln Á 2 ˜ = – Fig. 9.4
Ë p1 ¯ p0 v0 dp
used for calculation of slope .
dT
For air p0 v0 = RT0
Example 9.3 Using p-v-T data for saturated water,
Êp ˆ gh
Thus ln Á 2 ˜ = – ...(ii) calculate at 100°C (a) hfg, (b) ufg, and (c) sfg. Compare
Ë p1 ¯ RT0 the results with respective steam table values.
Using the value
Solution
- (9.81 m/s 2) h
–0.361 =
( 287 J/kg K) ¥ (300 K) Given Saturated water at 100°C.
or h = 3168.4 m To find Using saturation data at 100°C,
(i) Specific enthalpy of evaporation,
Thermodynamic Relations 289

(ii) Specific internal energy, and To find Per-cent error in Cv if Cp = Cv is assumed.


(iii) Specific entropy. Analysis Using Eq. (9.61) for obtaining difference
Assumption between Cp and Cv
(i) Unit mass of water. vTb 2 b 2T
Cp – Cv = =
(ii) Saturation data at 100°C (= 373 K) k rk
ps = 1.014 bar From Table 9.2, by interpolation at 27°C (300 K)
vf = 1.0435 ¥ 10–3 m3/kg, r = 996.42, b = 274.15 ¥ 10–6 K–1,
vg = 1.673 m3/kg. k = 45.11 ¥ 10–6 (bar)–1
Analysis The Clapeyron equation can be expressed as Using numerical values.
274.15 ¥ 10 - 6 ¥ 300 ¥ 100
(i) hfg = T (vg – vf ) ÊÁ dp ˆ˜
Cp – Cv =
996.42 ¥ 45.11 ¥ 10 - 6
Ë dt ¯ sat
Ê dp ˆ = 0.0501 kJ/kg ◊ K
This equation requires slope Á ˜ , we take The specific heat of water at 27°C, (from standard
Ë dt ¯ sat
from steam tables tables)
T1 = 95°C p1 = 0.8455 bar Cp = 4.179 kJ/kg ◊ K)
T2 = 110°C p2 = 1.433 bar Then Cv = 4.179 – 0.0501 = 4.129 kJ/kg ◊ K
Ê dp ˆ p - p1 1.433 - 0.8455 Per-cent error in approximation
ÁË dt ˜¯ = 2 =
T2 - T1 110 - 95
sat Ê C p - Cv ˆ 0.0501
= 0.03916 bar/°C = 3.9167 kPa/°C =Á ˜ ¥ 100 = ¥ 100 = 1.21%
Ë C v ¯ 4.129
Using numerical values
hfg = 373 ¥ (1.673 – 1.0435 ¥ 10–3)
Example 9.5 Prove that
¥ 3.9167
= 2442.6 kJ/kg Ê ∂u ˆ
ÁË ∂T ˜¯ = Cp – pvb
which is 8% higher, due to poor estimation of p
Ê dp ˆ
ÁË dt ˜¯ . Solution Using equation
(ii) The change in specific internal energy ufg du = Tds – pdv
ufg = hfg – p(vg – vf) At constant pressure, it can be written as
= 2442.6 – 1.014 ¥ 100 Ê ∂u ˆ Ê ∂s ˆ Ê ∂v ˆ
=TÁ –pÁ
¥ (1.673 – 1.0435 ¥ 10–3) ËÁ ∂T ¯˜ p
Ë ∂T ¯˜ p Ë ∂T ¯˜ p
= 2273.06 kJ/kg From Eq. (9.45)
which is also higher than the tabulated value. Ê ∂s ˆ
TÁ = Cp
(iii) The change in specific entropy during evaporation. Ë ∂T ˜¯ p
h fg 2442.6
sfg = = and Eq. (9.21) gives
Ts 373
= 6.5485 kJ/kg ◊ K Ê ∂v ˆ
= vb
which is again higher than the tabulated value. ËÁ ∂T ¯˜ p
Thus
Example 9.4 Calculate the per-cent error in Cv that Ê ∂u ˆ
would result if Cp = Cv is assumed for liquid water at ÁË ∂T ˜¯ = Cp – pvb Proved
p
1 atm and 27°C.

Solution Example 9.6 Prove that


Ê ∂u ˆ Cp
Given Liquid water at ÁË ∂ v ˜¯ = – p.
p vb
p = 1 atm T = 27°C = 300 K
290 Thermal Engineering

Solution Using equation for internal energy change Using the above in Eq. (iii), we get
du = Tds – pdv È Ê ∂p ˆ Ê ∂p ˆ ˘
dh = Ív Á ˜ + T Á ˙ dv
Partial differentiation with respect to specific volume ÍÎ Ë ∂v ¯ T Ë ∂T ˜¯ v ˙˚
at constant pressure
The equation of state of the gas is
Ê ∂u ˆ Ê ∂s ˆ Ê ∂v ˆ
ÁË ∂v ˜¯ = T ÁË ∂v ˜¯ – p ÁË ∂v ˜¯ (i)
p =
RT
– 2
a
p p p v v
Ê ∂s ˆ
The quantity Á ˜ can be written as Ê ∂p ˆ RT a
Ë ∂v ¯ p and ÁË ∂v ˜¯ =– 2 + 3
T v v
Ê ∂s ˆ Ê ∂s ˆ Ê ∂T ˆ Cp
ÁË ∂v ˜¯ = ÁË ∂T ˜¯ ÁË ∂v ˜¯ = b v (ii) Ê ∂p ˆ R
p p p ÁË ∂T ˜¯ = v – 0
Using Eq. (ii) in Eq. (i), we get v

Ê ∂u ˆ Cp Using in Eq. (iv)


ÁË ∂v ˜¯ = vb – p Proved
È v RT a v RT ˘ a
p dh = Í- 2 + 3 + dv = 2 dv
Î v v v ˙˚ v
Example 9.7 If a gas obeys the following equation
of state Integrating both sides,
RT a 2 v2 v
p = – 2 a È v -2 +1 ˘ 2
v v
calculate the change in enthalpy at constant temperature.
Ú1
dh = h2 – h1 =
Ú
v1
v 2
dv = a Í ˙
ÍÎ -2 + 1 ˙˚ v
1

È1 1˘ Èv - v ˘
Solution The change in enthalpy is expressed by =aÍ - ˙ =aÍ 2 1˙
Eq. (9.37) as Î v1 v2 ˚ Î v1 v2 ˚
È Ê ∂v ˆ ˘
dh = Cp dT + Ív - T Á ˙ dp
ÍÎ Ë ∂T ˜¯ p ˙ Example 9.8 Prove that for a real gas, whose equa-
˚
tion of state is
At constant temperature, dT = 0, then a
R Tv
È Ê ∂v ˆ ˘ p (v – b) e u = RuT
dh = Ív - T Á ˙ dp ...(i)
ÍÎ Ë ∂T ˜¯ p ˙ the critical pressure is given by
˚ a a
Let p = f (v, T), then pc = 2 2 c
v = 2b, Tc =
4e b 4 Rub
Ê ∂p ˆ Ê ∂p ˆ
dp = Á ˜ dv + Á dT Ê ∂p ˆ Ê ∂2 p ˆ
Ë ∂v ¯ T Ë ∂T ˜¯ v Take ÁË ˜¯ = 0 and Á 2 ˜ = 0
∂v T Ë ∂v ¯ T
\ dT = 0
Ê ∂p ˆ
\ dp = Á ˜ dv ...(ii) Solution The equation of state
Ë ∂v ¯ T a
Substituting Eq. (ii), in Eq. (i). we get p(v – b) e RuT v = RuT
È Ê ∂ v ˆ ˘ Ê ∂p ˆ a
dh = Ív - T Á ˙ dv RT -
ÍÎ Ë ∂T ˜¯ p ˙ ÁË ∂v ˜¯ T or p = u e RuT v ...(i)
˚ v-b
È Ê ∂p ˆ Ê ∂ v ˆ Ê ∂p ˆ ˘ Its partial derivative at constant temperature
= Ív ¥ Á ˜ - T Á ˙ dv ...(iii)
ÍÎ Ë ∂v ¯ T Ë ∂T ˜¯ p ÁË ∂v ˜¯ T ˙ Ï a ¸
˚ Ê ∂p ˆ RuT Ô - RuT v Ê a ˆÊ 1 ˆÔ
The cyclic relation for properties p, v and T ÁË ∂ v ˜¯ = Ì e - -
ÁË R T ˜¯ ÁË 2 ˜¯ ˝
T
v-b Ô u v Ô
Ê ∂p ˆ Ê ∂ v ˆ Ê ∂ T ˆ Ó ˛
ÁË ∂ v ˜¯ ÁË ∂T ˜¯ Á ∂ p ˜ = –1 -
a
T p Ë ¯v RuT v RuT
-e
Ê ∂p ˆ Ê ∂v ˆ Ê ∂p ˆ ( v - b) 2
or ÁË ∂v ˜¯ ÁË ∂T ˜¯ = – ÁË ∂T ˜¯
T p v
Thermodynamic Relations 291

RuT –a/RuTv a RuT a 3a 2ab


= e – e–a/RuTv 0 = – +
( v - b) RuT v 2
( v - b) 2 v 2 ( v - b) v 2 ( v - b) v3 ( v - b)
a 2a
a RuT – +
2 2
= e – a/Ru Tv
– –a/Ru Tv
e v ( v - b) v ( v - b)
v 2 ( v - b) ( v - b 2) 2ab a
or =
For maximum pressure v3 ( v - b) v 2 ( v - b)
Ê ∂p ˆ or b = v/2 or vc = 2b ...(iii)
ËÁ ∂v ¯˜ T
=0
Using the value of vc in Eq. (ii)
a RuT a( 2b - b) ab a
then = RuTc = = =
v 2 ( v - b) 4b 2 4b 2 4b
a
a ( v - b) \ Tc =
or RuT = ...(ii) 4 Ru b
v2
Taking second-order partial derivative and equating it Using the value of Tc and vc in Eq. (i)
a 4 Ru b
to zero -
Ru a
pc = e Ru a 2b
Ê ∂2 p ˆ 4 Ru b ( 2b - b)
Á 2˜ =0 a
Ë ∂v ¯ T = Proved
4b 2e 2
ÏÔ Ê a ˆ ¸Ô
a v 2 ( v - b) Ìe - a / RuT v a (3v 2 - 2vb)e - a / RuT v
ÁË R T v 2 ˜¯ ˝ - Example 9.9 A gas obeys the equation of state as
ÔÓ u Ô˛
= p(v – b) = RT
v 4 ( v - b) 2
(a) Does (Cp – Cv) change or remain constant?
RuT Ê a ˆ Ê 1ˆ 2 RuT e - a / RuT v
– ◊ e– a/Ru Tv ◊ Á - ˜ Á - 2˜ + (b) Will the temperature of this gas change during the
(v - b )2 Ë RuT ¯ Ë v ¯ ( v - b )3 throttling process? If it does, does it increase or
decrease?
Dividing both sides by e(–a/R u Tv), we get
(c) Does the internal energy change depend upon the
a[( v - b) ◊ ( a / RuT ) - 3v 2 + 2vb] a change in pressure?
0 = –
4
v ( v - b) 2
v ( v - b) 2
2 Prove all the conclusions.
2 RuT
+ Solution
( v - b )3 An equation of state p(v – b) = RT or p=
RT
...(i)
v-b
a2 a(3v - 2b) a RT
or 0 = – – and v = +b ...(ii)
4
RuT v ( v - b) 3
v ( v - b) 2
v 2 ( v - b) 2 p
2 RuT (i) The change in specific heats is given by Eq. (9.59)
+
( v - b )3 2
Ê ∂v ˆ Ê ∂p ˆ
Cp – Cv = – T Á
Ë dT ¯˜ p ËÁ ∂v ¯˜ T
Dividing both sides by (v – b), we get ...(iii)

a2 a (3v - 2b) a 2 RuT Now from Eq. (i)


0= 4
– – +
RuT v 3
v ( v - b)
v ( v - b) 2 ( v - b) 2
Ê ∂p ˆ RT
ËÁ ∂v ¯˜ T
Using the value of RuT from Eq. (ii) =– ...(iv)
( v - b) 2
a2 v2 3a v - 2ab a
0 = – – and from Eq. (ii)
a ( v - b) v 4 v3 ( v - b) v 2 ( v - b)
2 a( v - b ) Ê ∂v ˆ R
+ ÁË ˜¯ = ...(v)
( v - b) 2 v 2 dT p p
292 Thermal Engineering

Using the values in relation Eq. (iii), the Partial derivative of Eq. (i) with respect to T
Ê Rˆ Ï
2
Ô RT ¸Ô Ê ∂p ˆ R
ËÁ ∂T ˜¯ v
=
ÁË p ˜¯ Ì- (v - b) 2 ˝
Cp – Cv = –T
v-b
ÓÔ ˛Ô
È RT ˘
T 2 R3 Then du = Cv dT + Í - p ˙ dv
= Îv - b ˚
p 2 ( v - b) 2
Using the value of RT from equation of state as
R 2T 2 R p(v – b)
=
p 2 ( v - b) 2 We get du = Cv dT + [p – p]dv = Cv dT
Using the value of RT from equation of state as Thus, change in internal energy is function of
p(v – b), then temperature only, it does not depend on pressure
p 2 ( v - b )2 variation.
Cp – Cv = 2 R=R
p ( v - b )2
Example 9.10 Van der waals equation is given as
Thus the gas constant R does not change
(ii) During throttling process, the change in enthalpy Ê aˆ
ÁË p + 2 ˜¯ (v – b) = RT
dh = 0 v
Using in Eq. (9.37); R
Prove that Cp – Cv =
È Ê ∂v ˆ ˘ 2a ( v - b) 2
dh = 0 = Cp dT + Ív - T Á 1-
Ë ∂T ˜¯ p ˙˙
dp
ÍÎ ˚ RT v3

Ê ∂v ˆ Solution The change in specific heats is given by


Using the value of Á
Ë dT ˜¯ p
from Eq. (v) 2
Ê ∂v ˆ Ê ∂p ˆ T b2v
Cp – Cv = –T Á ˜ Á ˜ = ...(i)
È R˘ Ë dT ¯ p Ë ∂v ¯ T k
0 = Cp dT + Ív - T ˙ dp
Î p˚ The van der waals equation
Using the value of v from Eq. (ii) Ê aˆ
ÁË p + 2 ˜¯ (v – b) = RT
È RT RT ˘ v
0 = Cp dT + Í +b- dp
Î p p ˙˚ RT a
or p = – 2 ...(ii)
= Cp dT + bdp v-b v
Ê ∂p ˆ RT 2a
or Cp dT = – bdp and ÁË ˜¯ = – + 3 ...(iii)
∂v T ( v - b) 2
v
Rearranging, we get
From cyclic relationship for properties p, v, T
Ê ∂p ˆ Cp
ÁË ˜¯ = – Ê ∂p ˆ Ê ∂v ˆ Ê ∂ T ˆ
∂T h ÁË ˜¯ ÁË ˜ = –1
∂v T dT ¯ p ÁË ∂p ˜¯v
b
Cp
or m =– Ê ∂v ˆ
b Ê ∂p ˆ Ê ∂v ˆ
and ÁË ˜ =–Á
The Joule–Thomson coefficient is negative, thus dT ¯ p Ë ∂T ˜¯ v ÁË dp ˜¯
T
temperature will increase with decrease in pres- For given equation of state
sure.
Ê ∂p ˆ R
(iii) The change in internal energy of a real gas is ÁË ˜ =
∂T ¯v v-b
given by Eq. (9.32)
Using the value in Eq. (i)
È Ê ∂p ˆ ˘ 2 2
du = Cv dT + ÍT Á
Ë ˜¯ - p ˙ dv Ê ∂p ˆ Ê ∂v ˆ Ê ∂ p ˆ
Î ∂T v ˚ Cp – Cv = –T Á
Ë ∂T ˜¯ v ÁË dp ˜¯ ÁË ∂ v ˜¯ T
T
Thermodynamic Relations 293

2 where u0 is constant of integration and can be evaluated


Ê ∂p ˆ Ê ∂v ˆ
= –T Á
Ë ∂T ˜¯ v ÁË dp ˜¯
from boundary conditions.
T The change in entropy is given by Eq. (9.40)
R2 1 dT Ê ∂p ˆ
= –T ¥ + Á
Ë ∂T ˜¯ v
2 RT 2a ds = Cv dv
( v - b) - + T
( v - b) 2 v3
2
RT 1 dT Ê R ˆ
¥ = Cv + Á dv
=–
( v - b) 2 3
RT v + 2a ( v - b) 2 T Ë v - b ˜¯
-
( v3 )( v - b) 2 On integration
R Tv 2 3 ÊT ˆ Ê v - bˆ
=– Ds = s2 – s1= Cv ln Á 2 ˜ + R ln Á 2 + s0
3
- RT v + 2a ( v - b) 2 Ë T1 ¯ Ë v1- b ˜¯
R where s0 is constant of integration.
= 2
2a ( v - b )
1- Prove that
RT v3
Ê ∂u ˆ
Example 9.11 Derive an expression for change in ÁË ∂p ˜¯ = pvk – Tvb
T
internal energy and change in entropy for van der waal’s
equation. Using the method of partial differential.

Solution The van der waals Equation is Solution We have


du = Tds – pdv ...(i)
Ê aˆ
ÁË p + 2 ˜¯ (v – b) = RT Differentiating it partially w.r.t. p, treating T as con-
v stant:
The change in internal energy, Eq. (9.32)
Ê ∂u ˆ Ê ∂ sˆ Ê ∂ vˆ
È Ê ∂p ˆ ˘ ÁË ∂p ˜¯ = T ÁË ∂ p ˜¯ – p ÁË ∂ p ˜¯
du = Cv dT + ÍT Á
Ë ˜ - p ˙ dv
¯
T T T
Î ∂T v ˚ Introducing the fourth Maxwell’s relation
The van der waals equation can be written as Ê ∂s ˆ Ê ∂v ˆ
p =
RT
– 2
a ÁË ∂ p ˜¯ = – ÁË ∂T ˜¯
T p
v-b v
Ê ∂u ˆ Ê ∂v ˆ Ê ∂ vˆ
Ê ∂p ˆ R ÁË ∂ p ˜¯ = –T ÁË ∂T ˜¯ – p ÁË ∂ p ˜¯ ...(ii)
and ÁË ˜ =
∂T ¯ v v-b T p T
Using the definition of volumetric expansion coeffi-
È RT ˘
Thus du = Cv dT + Í - p ˙ dv cient
Î v - b ˚
b =
1 Ê ∂v ˆ
RT Á ˜
Using the value of from equation of state as p v Ë ∂T ¯ p
v-b
+ a/v2, we get and coefficient of isothermal compressibility
Èa˘ 1 Ê ∂ vˆ
du = Cv dT + Í 2 ˙ dv k =–
Îv ˚ Á ˜
v Ë ∂ p¯ T
Integrating both sides
Equation (ii) leads to
2 T2 v2 a
Ú1
du = u2 – u1 =
Ú T1
Cv dT +
Úv1 v2
dv Ê ∂u ˆ
ÁË ∂ p ˜¯ = – Tvb + pvk Proved
T
È1 1˘
= Cv (T2 – T1) + a Í - ˙ + u0
v
Î 1 v 2˚
294 Thermal Engineering

Summary
È Ê ∂v ˆ ˘
dh = Cp dT + Í v - T Á
Ë ∂T ˜¯ p ˙˙
properties which cannot be measured directly. dp
ÍÎ ˚
du = Tds – pdv dT Ê ∂p ˆ
+ Á
Ë ∂T ˜¯ v
ds = Cv dv
dh = Tds + vdp T
dh = – pdv – sdT
dT Ê ∂v ˆ
– Á
Ë ∂T ˜¯ p
dg = vdp – sdT = Cp dp
T
Maxwell relations are equations that relate
the partial derivative of properties p, v, T and s of Cp and Cv
a simple compressible fluid to each other. These Ê ∂ sˆ
Cp = T Á
Maxwell relations derived from Gibbsian rela- Ë ∂T ˜¯ p
tions are listed below:
Ê ∂ sˆ
Ê ∂T ˆ Ê ∂p ˆ Cv = T Á
ÁË ˜ = – ÁË ˜¯ Ë ∂T ˜¯v
∂ v ¯s ∂s v
Ê ∂ T ˆ Ê ∂ vˆ Ê ∂p ˆ Ê ∂ v ˆ
Cp – Cv = T Á
ÁË ∂p ˜¯ = ÁË ∂ s ˜¯ Ë ∂T ˜¯ v ÁË ∂T ˜¯ p
s p
2
Ê ∂p ˆ Ê ∂ sˆ Ê ∂ v ˆ Ê ∂p ˆ
Cp – Cv = – T Á
ËÁ ∂T ˜¯ v
= Á ˜
Ë ∂ v¯ T Ë ∂T ˜¯ p ÁË ∂ v ˜¯ T

Ê ∂ sˆ Ê ∂v ˆ vTb 2
=
ÁË ∂ p ˜¯ = – ÁË ∂T ˜¯ k
T p
coefficient of volumetric expansion b is de-
Cp Ê ∂ v ˆ Ê ∂p ˆ
= Á ˜ Á ˜
fined as Cv Ë ∂p ¯ T Ë ∂ v ¯ s
1 Ê ∂v ˆ
b = ÁË ˜
v ∂T ¯ p Ê ∂T ˆ
m =
ÁË ∂p ˜¯
coefficient of isothermal compressibility is h
defined as The Joule–Thomson coefficient m decides the
1 Ê ∂ vˆ
k =– Á ˜
heating or cooling effect after throttling
v Ë ∂p ¯ T Ï< 0 T increases
Ô
m = Ì= 0 T remains constat
defined as Ô> 0 T decreases
Ó
1 Ê ∂ vˆ
a =– Á ˜
v Ë ∂p ¯ s h = C, thus T = Constant
Ê ∂p ˆ b
and Á
Ë ∂T ˜¯ v
= as
k
- Ê ∂ hˆ
CT = Á ˜
thalpy and entropy in terms of p, v, T and specific Ë ∂p ¯ T
heat data Clausius–Clapeyron equation yields
È Ê ∂p ˆ ˘ dp hf g
du = Cv dT + ÍT Á ˜ - p ˙ dv =
Î Ë ∂T ¯ v ˚ dT T vf g
Thermodynamic Relations 295

Review Questions
1. What are Maxwell relations and why they are (b) Coefficient of isothermal compressibility.
important in thermodynamics? (c) Isentropic compressibility.
2. Sketch the thermodynamic mnemonic diagram 10. Establish the relationship between b and k for an
and explain its use to obtain Gibbsian equations ideal gas.
and Maxwell relations.
11. Considering specific entropy as function of
3. Define Helmholtz and Gibbs functions. pressure, sp. volume and temperature, derive
4. Show that the work done by the system during an three Tds relations.
adiabatic process is equal to decrease in internal Using Tds relations, prove that
energy of the system. 2
5. What is Clapeyron equation? Ê ∂v ˆ Ê ∂p ˆ
Cp – Cv = T Á
Ë ∂T ˜¯ p ÁË ∂v ˜¯ T
6. What are assumptions made in obtaining Clausius
equation from Clapeyron equation? 12. What is Joule Thompson coefficient. Discuss the
7. State Helmholtz and Gibbs function and then zone of heating and cooling with help of inversion
derive Gibbsian relations. curve.
8. State Gibbsian relations and then derive Maxwell 13. Considering u = f (T, v), prove that
relations.
È Ê ∂p ˆ ˘
9. Define du = CvdT + ÍT ÁË ˜¯ - p ˙ d v
Î ∂ T v ˚
(a) Coefficient of volumetric expansion,

Problems
1. Develop an expression for the change in entropy 6. A rigid vessel of 0.3 m3 volume contains 10 kg of
of a gas for which the equation of state is air at 300 K. Using (a) the perfect gas equation,
RT a (b) the van der Waals equation of state, and (c)
p = – 2 the generalised compressibility chart, determine
v- b v
pressure extracted by air on the vessel.
2. Show that the latent heat of vaporisation may be
expressed as [Ans. (a) 28.67 bar (b) 28.13 bar
(c) 28.32 bar]
vg Ê ∂p ˆ
hfg = T
Ú vf
ÁË ˜ dv
∂T ¯ v
7. At pressures above 22 MPa, an increase in pres-
sure at constant temperature of 10°C produces a
3. A certain gas follows the equation of state p (v decrease in entropy of water. What must happen
– b) = RT. Show that for a reversible adiabatic to the specific volume under these conditions?
process of this gas 8. For a pure substance s = f (p, v). Prove that
T (v – b) R/Cv = constant
Ê kC ˆ Ê Cpˆ
4. Determine the Joule–Thomson coefficient of wa- Tds = Á v ˜ dp + Á
Ë vb ˜¯
dv
ter at Ë b ¯
(a) 70 kPa, 150°C and (b) 20.5 MPa. 400°C. 9. Prove that
5. Prove that for a van der Waals gas the inversion
temperature is given by Ê ∂f ˆ pk Ê ∂gˆ 1
ÁË ∂ p ˜¯ = – b and ÁË ˜¯ = –
2 s
∂v T k
2a Ê bˆ
T = Á1 - ˜¯
bR Ë v
296 Thermal Engineering

10. Prove that the slope of an isentropic process on a Take b = 2.07 ¥ 10 –4 K–1, and k = 4.85 ¥ 10–4
T–p diagram is given by (MPa)–1 for this liquid.
Ê ∂T ˆ b vT 12. For the van der Waals equation, prove that
ÁË ∂ p ˜¯ = C v È 2a ( v - b) 2 - T Ru b v2 ˘
s p
m = Í ˙
11. Calculate the rise in pressure of a liquid, when it C p Í R T v 2 - 2a ( v - b) 2 ˙
Î u ˚
is heated at constant volume from 25°C to 75°C.

Objective Questions
1. Maxwell’s Thermodynamic relations are valid for 6. The cyclic relation of three variable is given by
(a) all processes Ê ∂ xˆ Ê ∂ yˆ Ê ∂ z ˆ
(a) Á ˜ Á ˜ Á ˜ = 1
(b) a closed system Ë ∂ y¯ z Ë ∂ z ¯ x Ë ∂ x¯ y
(c) a thermodynamic system in equilibrium.
(d) an open system. Ê ∂ xˆ Ê ∂ yˆ Ê ∂ z ˆ
(b) Á ˜ Á ˜ Á ˜ = –1
2. The coefficient of volumetric expansion b is Ë ∂ y¯ z Ë ∂ z ¯ x Ë ∂ x¯ y
given by Ê ∂zˆ Ê ∂zˆ
1 Ê ∂v ˆ (c) Á ˜ Á ˜ = 1
1 Ê ∂v ˆ Ë ∂ x¯ y Ë ∂ y¯
(a) Á ˜ (b) Á ˜ x
v Ë ∂T ¯ p p Ë ∂T ¯ p
Ê ∂zˆ Ê ∂zˆ
1 Ê ∂v ˆ (d) Á ˜ Á ˜ = – 1
1 Ê ∂p ˆ Ë ∂ x¯ y Ë ∂ y¯
(c) Á ˜ (d) Á ˜
p Ë ∂T ¯ T
x
T Ë ∂T ¯ v 7. The Tds equation is
3. Isothermal compressibility k of a substance is
Tb
1 Ê ∂ pˆ 1 Ê ∂ vˆ (a) Tds = Cv dT + dv
(a) – Á ˜ (b) – k
p Ë ∂ v¯ T p ÁË ∂p ˜¯ T
Tb
(b) Tds = Cv dT – dv
1 Ê ∂ vˆ k
(c) – (d) none of the above
v ÁË ∂p ˜¯ T Tk
(c) Tds = Cv dT + dv
4. The specific heat relation is b
vT b 2 vTk Tb
(a) Cp – Cv = (b) Cp – Cv = (d) Tds = Cv dT + dp
k b k
8. The Tds relation is also given by
vT k v 2T b
(c) Cp – Cv = (d) Cp – Cv = kCv Cp
b 2 k (a) Tds = dv + dv
b vb
5. When z is the function of two independent
variables x and y, then kCv Cp
(b) Tds = dp + dv
b vb
∂y ∂x
(a) dz = dz + dz k Cp Cp
∂x ∂y (c) Tds = dv + dv
b vb
∂y ∂x
(b) dz = dz – dz
∂x ∂y k Cv Cv
(d) Tds = dv + dv
∂z ∂z b vb
(c) dz = dx + dz
∂x ∂y 9. The specific heat at constant pressure is given by
∂z ∂z Ê ∂s ˆ Ê ∂T ˆ
(d) dz = dx – dy (a) T ÁË ˜ (b) T ÁË ˜
∂x ∂y ∂T ¯p ∂s ¯p
Thermodynamic Relations 297

Ê ∂v ˆ Ê ∂p ˆ 12. When m > 0, the temperature of gas with decrease


(c) T Á
Ë ∂T ˜¯ p
(d) T Á
Ë ∂ T ˜¯ v in pressure
(a) decreases (b) increases
10. A relation of vapour pressure to enthalpy of va-
(c) remains constant (d) none of the above
porisation is expressed in
13. The coefficient of constant temperature CT is
(a) van der waals equation
given by
(b) Maxwell’s erlations
Ê ∂p ˆ Ê ∂ hˆ
(c) Carrier’s equation (a) Á (b) Á ˜
Ë ∂T ˜¯ h Ë ∂ p¯ T
(d) Clausius–Claypeyron equation
11. Joule–Thompson coefficient m is given as Ê ∂T ˆ Ê ∂v ˆ
(d) Á
Ë ∂ T ˜¯ p
(c) Á
Ë ∂ h ˜¯ p
k Ê ∂T ˆ
(a) (b) Á
a Ë ∂ p ˜¯ h 14. m, Cp and CT are related as
CT
(a) m = CT Cp (b) m =
Ê ∂v ˆ Ê ∂s ˆ Cp
(c) (d) Á ˜
ÁË ∂p ˜¯ Ë ∂ p¯ CT m
s T (c) m = – (d) CT = –
Cp Cp

14. (c) 13. (b) 12 (a) 11. (b) 10. (d) 9. (a)
8. (b) 7. (a) 6. (b) 5. (c) 4. (a) 3. (c) 2. (a) 1. (c)
Answers
298 Thermal Engineering

10
Compressible Fluid Flow

Introduction
When the density of fluid is the function of pressure, then the fluid is called compressible fluid. The
compressible fluid flow involves motion of fluid with Mach number greater than 0.3. It also includes
dynamic, thermal and viscous effects. The fluid dynamics of compressible flows is generally referred as gas
dynamics. The knowledge of gas dynamics is essential in the design of turbo-machines.

The stagnation enthalpy is the total energy of a


Whenever the kinetic and potential energies of a
steadily flowing fluid. It is designated as h0 and ex-
fluid are negligible, the properties of the fluid are
pressed as
referred as static properties. For example, the en-
V2
ergy of a fluid system can be expressed by enthalpy hstagnation = h0 = h + (J/kg) ...(10.2)
as combination of internal energy and flow energy 2
of the fluid as The stagnation enthalpy h0 is made of a static
h = u + pv ...(10.1) V2
part h and a dynamic part . In the absence of
If the kinetic energy of the fluid system becomes 2
negligible then this enthalpy is referred as static
enthalpy.

For high speed fluid flow, the potential energy


of the fluid is negligible but kinetic energy is
significant. In such cases, each property of the fluid
is influenced by kinetic energy, thus it is convenient
to choose a property with kinetic energy in a single
term to form a stagnation property. Hence, a stag-
nation state is defined as a state in a fluid flow field,
when the fluid is brought to rest isentropically.
Compressible Fluid Flow 299

potential energy change, the steady-flow energy


equation reduces to
For an ideal gas, the enthalpy is a function of tem-
V2 Ê V2 ˆ
q – w = h2 + 2 – Á h1 + 1 ˜ perature only. The stagnation enthalpy can be ex-
2 Ë 2 ¯ pressed as
= h02 – h01 ...(10.3) V2
It represents the total energy of the flowing fluid Cp T0 = Cp T +
2
system and if the fluid velocity is brought to zero
isentropically; (q = 0, w = 0), then V2
or T0 = T + (°C) ...(10.6)
h02 = h01 ...(10.4) 2C p
V12 where T0 is the stagnation or total temperature of
or h1 + = h2 + 0 = h02 ...(10.5)
2 the flowing fluid and it represents the temperature
Thus, the stagnation enthalpy or total enthal- that a fluid attains when it is brought to rest isen-
py represents the enthalpy of the fluid when it is tropically. It should be expressed in °C. The term
brought to rest isentropically. V12
represents temperature rise during such
2C p
isentropic process and thus it is called dynamic
temperature or velocity temperature.

Total pressure or stagnation pressure of a flowing


fluid is the pressure that it attains when the fluid is
brought to rest isentropically. The stagnation pres-
sure exceeds the static pressure by the pressure
equivalent of the velocity. For an ideal gas with
constant specific heat, the stagnation pressure p0 is
related to static pressure p as
g -1
p0 ÊT ˆ g
= Á 0˜ ...(10.7)
p ËT¯

The stagnation density or total desnity of a flowing


fluid at any location is the density corresponding
to stagnation pressure at that location. From the
equation of state for an ideal gas
p
r0 = 0
RT0
For isentropic deceleration process, the stagna-
tion density r0 is related to the actual density r of
the flowing fluid as
1
r0 ÊT ˆg -1
= Á 0˜ ...(10.8)
r ËT¯
300 Thermal Engineering

Example 10.1 Air is flowing isentropically through 1


a nozzle at 27°C and 0.8 bar with a velocity of 120 m/s. 80 Ê 307.16 ˆ 1.4 - 1
= ¥
Calculate the stagnation enthalpy, stagnation tempera- 0.287 ¥ 300 ÁË 300 ˜¯
ture, stagnation pressure and stagnation density of air. = 0.985 kg/m3

Solution Example 10.2 Atmospheric air at 101 kPa and 27°C


is accelerated isentropically in a nozzle from 13.5 m/s
Given The isentropic flow of air through a nozzle with
to 223.8 m/s. Find the changes in (a) temperature,
T = 27°C + 273 = 300 K
(b) pressure, (c) density, (d) stagnation temperature and
p = 0.8 bar = 80 kPa stagnation pressure.
V = 120 m/s
To find Solution
(i) Stagnation enthalpy, Given Isentopic expansion of air through a nozzle
(ii) Stagnation temperature, p1 = 101 kPa T1 = 27°C = 300 K
(iii) Stagnation pressure, and V1 = 13.5 m/s V2 = 223.8 m/s
(iv) Stagnation density.
To find Change in
Assumptions For air: (i) Temperature,
(i) Specific gas constant R = 0.287 kJ/kg ◊ K. (ii) Pressure,
(ii) Specific heat at constant pressure, Cp = 1005 (iii) Density, and
J/kg ◊ K. (iv) Stagnation temperature and stagnation pressure.
(iii) Ratio of specific heat g = 1.4.
Assumptions
Analysis (i) Air is an ideal gas with constant specific heats,
(i) Stagnation Enthalpy thus
V2 V2 R = 0.287 kJ/kg ◊ K
h0 = h + = Cp T +
2 2 Cp = 1.005 kJ/kg ◊ K,
(120) 2 g = 1.4
= 1005 ¥ 300 + (ii) Changes in potential energy is negligible.
2
= 308.87 ¥ 103 J/kg Analysis
(ii) Stagnation temperature (i) Change in static temperature
V12 Applying steady-flow energy equation between
T0 = T +
2C p two states
(120) 2 V12 V2
= 300 + = 300 + 7.16 h1 + = h2 + 2
2 ¥ 1005 2 2
= 307.16 K V12 V2
or C p T1 + = C p T2 + 2
(iii) Stagnation pressure 2 2
g -1 V12 - V2 2
ÊT ˆ g or T2 = T1 +
p0 = p Á 0 ˜ 2C p
ËT¯
1.4 -1 (13.5) 2 - ( 223.8) 2
Ê 307.16 ˆ 1.4 = 300 +
= 0.8 ¥ Á = 0.868 bar 2 ¥ 1005
Ë 300 ˜¯ = 275.18 K
(iv) Stagnation density Change in temperature,
1 1
DT = T2 – T1 = 275.18 – 300
Ê T ˆ g -1 p Ê T0 ˆ g -1
r0 = r Á 0 ˜ =
RT ÁË T ˜¯
= – 24.82 K (or °C)
ËT¯
Compressible Fluid Flow 301

(ii) Change in static pressure 1.4


Ê 300.09 ˆ 0.4
For isentropic expansion = 101 ¥ Á
g Ë 300 ˜¯
p2 ÊT ˆg -1 = 101.10 kPa
= Á 2˜
p1 Ë T1 ¯ Stagnation pressure after expansion
1.4 g
Ê 275.18 ˆ 1.4 - 1 Ê T ˆ g -1
= Á = 0.739 = p2 Á 02 ˜
Ë 300 ˜¯
p02
Ë T2 ¯
or p2 = 0.739 ¥ (101 kPa) 1.4
Ê 300.09 ˆ 0.4
= 74.65 kPa = 74.65 ¥ Á
Ë 275.18 ˜¯
Change in pressure
= 101.1 kPa
D p = p2 – p1 = 74.65 – 101
There is no change in stagnation pressure. The
= – 26.5 kPa
stagnation pressure remains constant throghout
(iii) Change in density of air
the nozzle for isentropic flow.
p 101
Initial de nsity, r1 = 1 =
RT1 0.287 ¥ 300
= 1.173 kg/m3
Final density of air,
p2 74.65
r2 = = It is the velocity with which the sound travels
RT2 0.287 ¥ 275.18
through a medium. The velocity at which a small
= 0.945 kg/m3
pressure wave propagates in a fluid is called veloc-
Change in density,
ity of sound or sonic velocity. It is designated as a.
Dr = r2 – r1 = 0.945 – 1.173
Consider a gas at the rest within a constant area
= 0.228 kg/m3
duct as shown in Fig. 10.4. The piston moves to the
(iv) Change in stagnation temperature of air
right with a small and constant incremental velocity
Staganation temperature at the state 1
V12
T01 = T1 +
2C p
(13.5) 2
= 300 + = 300.09 K
2 ¥ 1005
Stagnation temperature after expansion
V22
T02 = T2 +
2
( 223.8) 2
= ( 275.18 K ) +
2 ¥ 1005
= 300.09 K
No change in stagnation temperature. The stagna-
tion temperature remains constant throghout the
nozzle for isentropic flow.
(v) Change in stagnation pressure
Initial stagnation pressure of air,
g
Ê T ˆ g -1
p01 = p1 Á 02 ˜
Ë T1 ¯
302 Thermal Engineering

dV, creating a sonic wave. The wave front travels to Substituting dV in Eq. (10.10),
the right through the gas at the sonic velocity a and dp
r = adr
it separates the moving gas adjacent to the piston ra
from the gas which is still at rest. dp
or a2 = ...(10.12)
Consider a control volume that encloses the dr
wave front and moves with it as shown in Fig. 10.5. The intensity of the sonic wave is very small
The flow pattern can be better studied, when an ob- and does not cause any significant change in the
server moves with the wave front. The stagnation pressure and temperature of the gas. Hence, the
gas to the right, will appear to be moving towards propagation of the wave can be considered as
the wave front with a velocity a and the gas to the isentropic. Then
left will appear to be moving away from the wave Ê ∂p ˆ
front with a velocity a – dV. Thus, the wave can a = Á ˜ ...(10.13)
be considered stationary within the control volume Ë ∂r ¯ s = C
and its flow from right to left. For an isentropic process,
p
= C (a constant)
rg
Differentiating with respect to r, we get
dp Ê ∂p ˆ
= Á ˜
dr Ë ∂r ¯ s = C
gp
= g Cr g -1 = ...(10.14)
r
The continuity equation p
m = r Aa ...(10.9) Using = RT
r
For a steady flow system, it can be expressed as
where R is the gas constant. Combining Eqs (10.13)
m left = m right
and (10.14)), we get
(r + dr) (a – dV ) A = rAa
At the left of the wave front, the gas experiences a = g RT ...(10.15)
an incremental change in its properties, while on In Eq. (10.15), the quantities g and R are con-
the right side of the wave front, the gas maintains stants. Thus the sonic velocity is the function of
its original properties as shown in Fig. 10.5. temperature only. For a given gas, the speed of the
(r + dr) (a – dV ) = ra sound depends only on the square root of the abso-
or ra – rd V + adr – d Vdr = ra lute temperature. Equation (10.15) can be written
Neglecting the higher order term dVdr , we get as
r d V = adr ...(10.10) g RuT
a = ...(10.16)
Further, the rate of change of momentum of gas M
will be equal to applied forces. Hence where M is the molecular weight of the gas and Ru
A[ p – (p + dp)] = m [(a – dV ) – a] is the universal gas constant (= 83144 J/kmol ◊ K).
or A dp = m d V The g is almost constant for all gases, and hence,
Using the continuity equation (10.9), we get speed of sound at a given temperature is inversely
dp = radV proportional to its molecular weight.
dp Some typical values for the speed of sound at
or dV = ...(10.11) 0°C are given in Table 10.1.
ra
Compressible Fluid Flow 303

If the fluid flows with a Mach


number less than unity (that is, V < a) then the flow
Gas M g a, Speed is called a subsonic flow. Such a flow is character-
Molecular, of sound
ised by smooth streamlines and continuously vary-
weight at 0°C.
(kg/kmol) (m/s) ing properties. Mach number ranges from 0.3 to 1.
Air 28.96 1.404 331 If the Mach number is equal to 1 (that
Argon 39.94 1.667 308 is, V = a) then the flow is regarded as a sonic flow.
Carbon dioxide (CO2) 44.01 1.300 258 If fhe flow is with a Mach number around unity (0.8
Freon 12 (CCI2F2) 120.90 1.139 146 < M < 1.2), it is said to be a transonic flow.
Helium (He) 4.00 1.667 970
If the fluid flows with Mach
Hydrogen (H2) 2.01 1.407 1270
number greater than unity (that is, V > a) then the
flow is called a supersonic flow. Such a flow is ob-
served with an oblique shock in the flow field and a
significant change in the properties of the fluid dur-
The Mach number at a point in a flow field is de- ing flow. Mach number ranges from 1 to 5.
fined as the ratio of local fluid velocity to the sonic
or acoustic velocity in the medium. It is a non-di- If the fluid flows with a very
mensional number and it is designated as M and high Mach number (that is, M > 5) then the flow is
expressed as called a hypersonic flow. Such a flow is observed
with a severe shock in the flow field and abrupt
Fluid velocity V
M= = ...(10.17) change in properties of fluid during flow.
Speed of sound a
The definition of Mach number can also be
interpreted as the square root of of the ratio of the
inertia force due to flow to the elastic force of fluid.
When the pressure field is created by a point dis-
Inertia force of flow
M= turbance in a stagnant gas, the pressure pulses gen-
Elastic force of fluid erated from the point of disturbance move in all
The Mach number may be regarded as a measure directions as spherical sound waves with a sonic
of the ratio of kinetic energy of the flowing fluid to velocity in the gas medium.
the kinetic energy of random molecular motion in In order to illustrate the effect of a velocity of
the fluid. body relative to the speed of sound in the flow field,
consider a small object moving at a uniform linear
velocity V in a stationary compressible gas. This
Fluid Flow
small object, as it moves, tends to divert the sta-
The Mach number is a very important parameter in tionary gas. The diverted fluid pushes the surround-
the analysis of compressible fluid flow. The com- ing gas, and this local disturbance creates pressure
pressible regions have been classified according to pulses which propagate further into the surround-
Mach number of flow as ing gas.
Mach number Classification Figures 10.6, 10.7 and 10.8 show the pressure-
pulse pattern for different speeds of the moving ob-
M<1 Subsonic flow
ject in a gas. In each pattern, the point A (at t = 0)
M=1 Sonic flow
represents the present position of the object. The
M>1 Supersonic flow
points B, C, D are respective positions of the object
M>5 Hypersonic flow
before time intervals t, 2t, 3t, respectively.
304 Thermal Engineering

The spherical sound wave originates from the


point source and grows in all directions. The dis-
tance travelled by the object is represented by Vt
in time t. While the distance travelled by the wave
front generated at that time is marked with respec-
tive radii (sonic velocity ¥ time) at, 2at, 3at, re-
spectively, where, at represents the distance trav-
elled by the wave front in time t, 2at in time 2t and
3at in time 3t drawn from locations B, C and D
respectively.

Since the wave front cannot overtake the body,


the fluid ahead of the body does not receive any
information about the body. The region ahead of
the the point source is called a zone of silence and
the sound cannot be heard in this zone. The zone
behind the point source, where sound waves are tra-
versed, is called the zone of action. The two zones
are separated by a line perpendicular to the direc-
tion of the motion of the body.
When the point source travels with supersonic
velocity (M > 1), the wave front circles will not ex-
ist at the point source as shown in Fig. 10.8. The
wave front originating before time interval t, 2t and
When the point source velocity V is less than the 3t are drawn with their originating locations. Since
sonic velocity a, (subsonic M < 1), the sound waves the object is moving with a speed greater than the
are generated and travel in all directions around sonic velocity, the wave front of disturbance creat-
the originating point. Since the sonic velocity a is ed by the body lags behind the body and the wave-
always greater than the velocity of the object V, front cannot overtake the moving body. The zone of
the wavefront generated by the moving object is action takes the form of a cone.
always ahead of the object as shown in Fig. 10.6.
Consequently, if the body moves in a compressible
fluid with a subsonic speed, the fluid ahead of the
body becomes aware of the presence of the body.
For example, all the automobiles travel with sub-
sonic velocities, thus the horn is heard before the
vehicle reaches a person standing on the road.
When the point source travels with sonic veloc-
ity (M = 1), the front of the pressure disturbance
moves the same distance as a small object as shown
in Fig. 10.7. Consequently, all the disturbances are
created by the object at the present position A of the -
object, and all wave fronts are contained in the half
plane to the right of the object.
Compressible Fluid Flow 305

(ii) Mach number is given by Eq. (10.17);


V 450
When the speed of an object is greater than the M= = = 1.816
a 247.8
speed of sound, the object is always ahead of the (iii) Mach angle is given by Eq. (10.18);
spherical wavefronts generated by the object. The
Ê 1ˆ Ê 1 ˆ
circles of wavefronts are formed along a straight a = sin -1 Á ˜ = sin -1 Á = 33.5°
ËM¯ Ë 1.816 ˜¯
line, known as Mach line. The angle between the
Mach line and the direction of flow is known as the Example 10.4 An air plane travels at a Mach number
Mach angle. All the wavefronts are confined to the of 1.5 at an elevation where the temperature is –37°C.
region within a cone, called the Mach cone. It is Determine the velocity of the plane in kmph. Assume
the zone of action as shown in Fig. 10.8. The at- g = 1.4.
mosphere outside the Mach cone is called the zone Solution
of silence. Hence, the sound of a jet plane is heard
after it passes forward. Given Flight of an air plane
It is observed that for supersonic flow, all the M = 1.5
waves lie within the Mach cone, which has its ver- T = –37°C = 236 K
tex at the point source, A. The semivertex angle of g = 1.4
the Mach cone is the Mach angle, denoted by a and To find Velocity of the air plane in kmph
given as
Assumption
a 1
sin a = = (i) Air as a perfect gas,
V M
(ii) The specific gas constant R = 287 J/kg ◊ K,
-1 Ê 1 ˆ (iii) Steady state operation.
or a = sin Á ˜ ...(10.18)
ËM¯
Analysis The sonic velocity,
Example 10.3 A gas has a molecular weight of 44 a = g RT
and a specific heat ratio of 1.3. Calculate the speed of the
sound in this gas if the temperature is –23°C. If this gas = 1.4 ¥ 287 ¥ 236 = 307.94 m/s
is flowing at a velocity of 450 m/s, calculate the Mach Actual velocity of the plane, using Eq. (10.17)
number and Mach angle. V = Ma
= 1.5 ¥ 307.94 = 461.9 m/s
Solution = 461.9 ¥ 10–3 ¥ 3600
Given = 1662.8 kmph
M = 44 g = 1.3
Example 10.5 A supersonic aircraft is flying at an al-
T = – 23°C = 250 K V = 450 m/s titude of 3 km with a constant flight speed of 2000 km/h.
To find The aircraft passes over a ground observation post. Find
(i) Speed of sound, the time taken to hear the sound waves from the aircraft
at the observation post after it has past directly over it.
(ii) Mach number, and
Assume the average temperature of atmospheric air at
(iii) Mach angle.
the obervation post to be 27°C.
Analysis The specific gas constant for the given gas;
Solution
Ru 8314
R = = = 188.95 J/kg ◊ K
M 44 Given Supersonic aircraft
(i) The speed of sound is given by Eq. (10.15); H = 3 km = 3000 m
V = 2000 km/h
a= g RT = 1.3 ¥ 188.95 ¥ 250 = 247.8 m/s
T1 = 27°C = 300 K
306 Thermal Engineering

To find Time taken by sound waves to reach the ground. To find


Assumptions (i) The initial Mach number.
(ii) Final temperature of air.
(i) The gas constant R = 287 J/kg ◊ K.
(ii) The ratio of specific heats, g = 1.4. Assumptions
Analysis The flight velocity; (i) The gas constant R = 287 J/kg ◊ K.
(ii) The specific heat Cp = 1005 J/kg ◊ K.
2000 ¥ 1000
V = = 555.56 m/s (iii) The ratio of specific heats, g = 1.4.
3600
Velocity of sound in the atmospheric air; Analysis The velocity of sound at the initial state
a = g RT = 1.4 ¥ 287 ¥ 300 a = g RT1 = 1.4 ¥ 287 ¥ 400
= 347.2 m/s = 400.9 m/s
V 555.56 (i) Initial mach number
Flight mach number; M = = = 1.6
a 347.2 V1 100
M1 = = = 0.249
Mach angle is given by Eq. (10.18); a1 400.9
Ê 1ˆ Ê 1 ˆ (ii) The final temperature of air
a = sin -1 Á ˜ = sin -1 Á
ËM¯ Ë 1.6 ˜¯ The initial stagnation temperature
= 38.68° V12
T01 = T1 +
From Fig. 10.9, distance 2C p

AB = Vt =
Height BC
=
3000 m (100) 2
= 400 + = 404.97 K
tan a tan (38.68∞) 2 ¥ 1005
= 3747.3 m For isentropic flow; T01 = T02
Distance AB 3747.3 m Sonic velocity of air at exit
Time t = =
V 555.56 m/s
a2 = g RT2
= 6 .75 s
= 1.4 ¥ 287 ¥ T2 = 20.04 T2
Exit velocity of air;
V2 = M2 a2 = 2.0 ¥ 20.04 T2
= 40.08 T2
Finalt emperature;
V22
T2 = T02 –
Fig. 10.9 2Cp
( 40.08 T2 ) 2
Example 10.6 The air at 100 m/s initially is acceler- = 404.97 –
2 ¥ 1005
ated isentropically in a nozzle. If the temperature at the
or T2 = 404.97 – 0.8T2
initial state is 400 K and the Mach number at the final
state is 2.0, determine (a) initial Mach number, and (b) fi- 404.97
or T2 = = 224.98 K
nal temperature. 1.8
Example 10.7 Air at 110 kPa, 90°C, with a velocity
Solution of 180 m/s is to be expanded isentropically through a con-
Given Acceleration of air in a nozzle; vergent–divergent nozzle, until its Mach number becomes
1.5. The mass flow rate of air is 0.15 kg/s. Determine the
V1 = 100 m/s, T1 = 400 K
final pressure and cross-sectional area at the nozzle exit.
M2 = 2.0, s l = s2
Compressible Fluid Flow 307

Solution Using continuity equation,


m = r2 A2 V2
Given Expansion of air in a convergent–divergent
0.15 = 0.465 ¥ A2 ¥ 486.1
nozzle
or A2 = 6.636 ¥ 10– 4 m2 = 6.636 cm2
p1 = 110 kPa T1 = 90°C = 363 K
V1 = 180 m/s s1 = s2 Example 10.8 Static air at 10 bar and 800°C flows
M2 = 1.5 m = 0.15 kg/s through a duct. Calculate the temperature, velocity and
Mach number at points where pressures are 5 bar and
To find
1 bar, respectively. Assume flow to be isentropic and Cp =
(i) Final pressure, and
1.005 kJ/kg ◊ K and g = 1.4 for air.
(ii) Cross-sectional area at the exit.
Assumption Solution
(i) The gas constant R = 0.287 kJ/kg ◊ K. Given Isentropic flow through a duct
(ii) The specific heat of air, Cp = 1.005 kJ/kg ◊ K. p1 = 10 bar T1 = 800°C = 1073 K
(iii) g = 1.4. p2 = 5 bar Cp = 1.005 kJ/kg ◊ K
p3 = 1 bar g = 1.4
Analysis Using steady-flow energy equation in the ab-
sence of potential energy change. V1 = 0

V22 - V12 To find


h1 – h2 =
2 (i) Temperature at two stations,
V22 - V12 (ii) Velocity at two stations, and
or Cp (T1 – T2) =
2 (iii) Mach number at two stations, where pressures are
V22 - 180 2 5 bar and 1 bar, respectively.
1005 ¥ (363 – T2) =
2 Analysis Considering the station where pressure is
5 bar
or V2 = 32400 + 2010 ¥ (363 - T2 ) ...(i)
(i) Static temperature after isentropic flow,
V2 V2
Now using M2 = = g -1
a2 g RT2 Êp ˆ g
T2 = T1 Á 2 ˜
or V2 = 1.5 ¥ 1.4 ¥ 287 T2 = 30.06 T2 Ë p1 ¯
1.4 -1
...(ii)
Ê 5ˆ 1.4
Equating the two equations, we get = 1073 ¥ Á ˜ = 880.22 K
Ë 10 ¯
30.06 T2 = 32400 + 2010 ¥ (363 - T2 )
= 607.22°C
903.6T2 = 32400 + 2010 ¥ (363 – T2)
(ii) For static air, V1 = 0
or 903.6T2 + 2010T2 = 32400 + 729630
Using steady-flow energy equation in the absence
or 2913.6T2 = 762030 of potential energy change (q = w = 0)
or T2 = 261.5 K
V22 - V12
and velocity; V2 = 30.06 ¥ 261.5 = 486.1 m/s 0 = Dh + Dke = h2 – h1 +
g 2
p2 Ê T ˆ g -1 or 2Cp (T1 – T2) = V 22
Using = Á 2˜
p1 Ë T1 ¯ Velocity V2 = 2 ¥ 1005 ¥ (1073 - 880.22)
1.4
Ê 261.5 ˆ 1.4 -1 = 622.5 m/s
or p2 = 110 ¥ Á = 34.9 kPa
Ë 363 ˜¯ (iii) The speed of sound at the station, where
p2 34.9 p2 = 5 bar
Further, r2 = =
RT2 0.287 ¥ 261.5 a2 = g RT2 = 1.4 ¥ 287 ¥ 880.22
= 0.465 kg/m3 = 594.7 m/s
308 Thermal Engineering

V2 622.5 gR
Mach number, M2 = = = 1.046 But Cp = and a2 = g RT
a2 594.7 g -1
Similarly at the station where the pressure is
1 bar, V2 V 2 (g - 1)
\ =
T3 = 55.75 K = 282.75°C, 2C p T 2g RT
V3 = 1019.63 m/s,
M3 = 2.157 V2 g -1 g -1 2
= 2
¥ = M
a 2 2
g -1 2
1+ M1
T2 2
Therefore = ...(10.19)
T1 g -1 2
1+ M2
2
Consider isentropic flow through a duct as shown The relation between pressures and densities are
in Fig. 10.10. For isentropic flow between states 1 g
and 2, the properties are related as È g - 1 2 ˘ g -1
g -1
g -1 p2 Í1 + 2 M1 ˙
T2 Êp ˆ g Êr ˆ = Í ˙ ...(10.20)
= Á 2˜ =Á 2˜ p1 Í1 + g - 1 M 22 ˙
T1 Ë p1 ¯ Ër ¯
1 ÍÎ 2 ˚˙
The steady-flow energy equation applied to is- 1
entropic flow through duct (q = 0, w = 0, D pe = 0), È g - 1 2 ˘ g -1
gives r2 Í1 + 2 M1 ˙
and = Í ˙ ...(10.21)
V2 V2 r1 Í1 + g - 1 M 22 ˙
h1 + 1 = h2 + 2
2 2 ÍÎ 2 ˙˚
V12 V2
or Cp T1 + = C pT2 + 2
2 2 Example 10.9 Air at 28°C and 700 kPa enters a
Ê nozzle with a velocity of 80 m/s. The nozzle has inlet
V ˆ
2 Ê V22 ˆ
or T1 Á1 + 1 ˜ = T2 Á1 + ˜ area of 10 cm2. The air leaves the nozzle of a pressure of
Ë 2C p T1 ¯ Ë 2C p T2 ¯ 250 kPa. Determine
Ê V2 ˆ (a) mass flow rate of air through the nozzle, and
1+ Á 1 ˜ (b) velocity at the exit of nozzle, assuming one-
T2 Ë 2C p T1 ¯ dimensional isentropic flow.
or =
T1 Ê V2 ˆ
1+ Á 2 ˜ Solution
Ë 2C p T2 ¯
Given Air flow through nozzle as shown in Fig. 10.11.
To Find
(i) Mass flow rate of air,
Compressible Fluid Flow 309

(ii) Velocity of air at nozzle exit. Actual exit velocity of air from the nozzle
V2 = M2 a2 = 1.335 ¥ 300.17
Assumptions
= 400.72 m/s
(i) Air as an ideal gas.
(ii) For air, R = 287 J/kg ◊ K, and g = 1.4.
Analysis
(i) Mass flow rate of air through nozzle
At the section 1, the density of air The equations derived above for isentropic flow
p1 700 ¥ 10 3 through a duct are applicable at stagnation condi-
r1 = = = 8.1 kg/m3 tions. At stagnation state, the Mach number is zero
RT1 287 ¥ 301
The continuity equation gives and properties are denoted by the subscript 0, and
m = r1 A1V1 = 8.1 ¥ (10 ¥ 10 – 4) ¥ 80 the static properties are represented without sub-
= 0.648 kg/s script. Then the stagnation temperature of an ideal
(ii) Exit velocity of air gas is related to static temperature
Sonic velocity at the section 1 T0 Ê g - 1ˆ 2
=1+ Á M ...(10.22)
a1 = g RT T Ë 2 ˜¯
= 1.4 ¥ 287 ¥ 301 = 347.77 m/s The relation between stagnation pressure p0 and
V1 80 m/s
the static pressure p is
g
Mach number M1 = = = 0.23
a1 347.77 m/s p0 È Ê g - 1ˆ 2 ˘ g -1
= Í1 + Á ˜M ˙ ...(10.23)
For isentropic flow through a duct using p Î Ë 2 ¯ ˚
Eq. (10.21) 1
g r0 È Ê g - 1ˆ 2 ˘ g -1
È g - 1 2 ˘ g -1 and = Í1 + Á ˜M ˙ ...(10.24)
p2 Í1 + 2 M1 ˙ r Î Ë 2 ¯ ˚
= Í ˙
p1 Í1 + g - 1 M 22 ˙ The properties of fluid at a location, where Mach
ÍÎ 2 ˚˙ number becomes unity (throat), are called critical
È 1.4 - 1 2˘
properties and the relations are called critical ratios.
Í1 + 2 ¥ (0.23) ˙ Assuming the superscript (*) represents the critical
or Í ˙
Í 1 + 1.4 - 1 M 22 ˙ values and setting M = 1, the critical equations are
ÍÎ 2 ˙˚
1.4 -1
T0 Ê g - 1ˆ g + 1
=1+ Á
Ë 2 ˜¯
= ...(10.25)
Ê 250 ˆ 1.4 T* 2
= Á = 0.74.5
Ë 700 ˜¯ g g
p0 È (g - 1) ˘ g -1 Ê g + 1ˆ g -1
= Í1 + =Á
Ë 2 ˜¯
0.2 M22 ...(10.26)
2 ˙˚
or 1 + = 1.356
p* Î
or M2 = 1.335 1 1
Further, for isentropic flow r0 È (g - 1) ˘ g -1 Ê g + 1ˆ g -1
= Í1 + =Á
Ë 2 ˜¯
...(10.27)
2 ˙˚
g -1 1.4 -1
Êp ˆ g Ê 250 ˆ 1.4 r* Î
T2 = T1 Á 2 ˜ = 301 ¥ Á
Ë p1 ¯ Ë 700 ˜¯
Example 10.10 Air at 500 kPa and 330 K, enters a
= 224.24 K
convergent nozzle with negligible velocity. The nozzle
Sonic velocity at nozzle exit discharges into a receiver, where a pressure of 120 kPa is
a2 = g RT = 1.4 ¥ 287 ¥ 224.24 maintained. Assuming isentropic flow, calculate the ve-
locity at the nozzle exit.
= 300.17 m/s
310 Thermal Engineering

Solution V1 = 150 m/s A = 50 cm2


pb = 0.7 MPa
Given Flow of air in a convergent nozzle
p1 = 500 kPa V1 = 0 m/s To find The mass flow rate through the nozzle.
\ T0 = 330 K pb = 120 MPa Assumptions
To find The velocity at the exit of the nozzle. (i) Isentropic flow,
(ii) Air as ideal gas with specific heat Cp = 1005
Assumptions
J/kg ◊ K and g = 1.4.
(i) Isentropic flow,
(ii) Air as ideal gas with specific heat Cp = 1005 Analysis The stagnation pressure and temperature can
J/kg ◊ K, g = 1.4 and R = 0.287 kJ/kg ◊ K. be calculated as
V12 (150 m/s) 2
Analysis Since the inlet velocity of the fluid is negli- T01 = T1 + = 773 K +
gible, thus 2C p 2 ¥ (1005 J / kg ◊ K )
T1 = T01 and p1 = p01 = 784.2 K
g 1.4
For isentropic flow through a nozzle, the velocity
Ê T01 ˆ g -1 Ê 784.2 ˆ 1.4 -1
reaches sonic velocity at the nozzle exit and temperature p01 = p1 Á = (1 MPa ) ¥ ÁË ˜
Ë T ¯˜
1 773 ¯
and pressure become critical.
The critical temperature, = 1.05 MPa
and the critical temperature,
Ê 2 ˆ
T * = T01 Á
Ë g + 1˜¯ Ê 2 ˆ
T * = T01 Á = 784.2 ¥
2
= 653.5 K
˜
Ë g + 1¯ (1.4 + 1)
2
= 330 ¥ = 275 K
(1.4 + 1) The critical pressure ratio at the throat
g 1.4
The critical pressure at the throat p* Ê T * ˆ g -1 Ê 653.5 K ˆ 1.4 -1
g = Á =Á = 0.528
Ê T * ˆ g -1 p01 Ë T01 ˜¯ Ë 784.2 K ˜¯
p* = p01 Á
Ë T01 ˜¯ Since flow through the nozzle is assumed isentropic,
1.4 the stagnation pressure and temperature remain constant.
Ê 275 K ˆ 1.4 -1 The pressure at the throat
= (500 kPa ) ¥ Á
Ë 330 K ˜¯ p* = 0.528 ¥ 1 MPa = 0.528 MPa = 528 bar
= 264 kPa p 0.7 MPa
Back-pressure ratio b = = 0.667
Since the receiver pressure is lower than the critical p01 1.05 MPa
pressure p*, the pressure at the throat of the convergent Since the back pressure ratio is greater than the criti-
nozzle will be p*. The velocity of the flow at the throat cal pressure ratio, the exit pressure will be equal to the
will be sonic velocity; back pressure.
V2 = a = g RT * = 1.4 ¥ ( 287 J/kg ◊ K ) ¥ ( 275 K ) From Table 10.2, at a back pressure ratio of 0.667,
= 332 m/s M* = 0.778, and temperature ratio at the throat
T*/T0 = 0.892
Example 10.11 Air at 1 MPa and 500°C enters a Thus, temperature at the throat,
convergent section with a velocity of 150 m/s. Determine T* = 0.892 ¥ 784.2 = 699.5 K
the mass flow rate through the nozzle, for its throat area Density of air at the throat,
of 50 cm2 and back pressure of 0.7 MPa. p* 700 kPa
r* = =
RT * (0.287 kJ/kg ◊ K) ¥ (699.5 K)
Solution = 3.484 kg/m3
Given Flow of air in a convergent nozzle The velocity of air at the throat,
p1 = 1 MPa T1 = 500°C = 773 K V* = M*a = M* g RT *
Compressible Fluid Flow 311

= (0.778) ¥ 1.4 ¥ ( 287 J/kg ◊ K) ¥ (699.5 K) The critical temperature


= 412.5 m/s Ê 2 ˆ 2
T * = T0 Á = 463 ¥
The mass flow rate at the throat, Ë g + 1¯˜ 1.4 + 1
m* = r* A* V* = 385.8 K
= (3.484 kg/m3) ¥ (50 ¥ 10– 4 m2) The specific volume of air at the throat
¥ (412.5 m/s) RT * 287 ¥ 385.8
v* = =
= 7.185 kg/s p* ( 4.543 ¥ 105 )
Air at 8.6 bar, and 190°C expands at = 0.244 m3/kg
a rate of 4.5 kg/s through a convergent–divergent nozzle The velocity of air at the throat
to an atmospheric pressure of 1.013 bar. Assuming that V* = g RT * = 1.4 ¥ 287 ¥ 385.8
the inlet velocity is negligble, calculate the throat and
exit cross-section areas of the nozzle. = 393.7 m/s
Using continuity equation to find the throat area
Solution m v* 4.5 ¥ 0.244
A* = =
Given Isentropic flow of air through a nozzle with V* 393.7
m = 4.5 kg/s V1 = 0 = 0.002788 m2 = 2788.8 mm2
p0 = 8.6 bar T0 = 190°C = 463 K (ii) For isentropic expansion process, the temperature
p2 = 1.013 bar at the nozzle exit
g -1 1.4 -1
To find T2 Ê p2 ˆ g Ê 1.013 ˆ 1.4
= =Á
(i) The throat area of the nozzle, and T0 ÁË p ˜¯
0 Ë 8.6 ˜¯
(ii) The exit area of the nozzle. = 0.5424
Schematic or T2 = 0.5424 ¥ 463 = 252.14 K
The specific volume at the exit
287 ¥ 251.14
v2 = = 0.711 m3/kg
1.013 ¥ 105
Using steady-flow energy equation
V22
h0 = h2 +
2
or V2 = 2 ( h0 - h2 ) = 2Cp (T0 - T2 )

= 2 ¥ 1005 ¥ ( 463 - 251.14)


Assumptions
= 652.56 m/s
(i) Air as a perfect gas,
The exit area
(ii) The specific gas constant of air as 0.287 kJ/kg ◊ K,
(iii) Change in potential energy is negligible. mv2 4.5 ¥ 0.711
A2 = =
V2 652.56
Analysis
= 0.00490 m2 or 4903 mm2
(i) The critical pressure ratio for flow through a
nozzle is given by Example 10.13 A gas which has a molar mass of
g 1.4
p* Ê 2 ˆ g -1 Ê 2 ˆ 1.4 -1 35.9 and the specific heat ratio of 1.60 is discharged from
= Á =Á
p0 Ë g + 1˜¯ Ë 1.4 + 1˜¯ a large chamber in which the pressure is 470 kPa, the
= 0.528 temperature is 27°C and the velocity is effectively zero
through a nozzle. Assuming one-dimensional isentropic
or p* = 0.528 ¥ 8.6 = 4.543 bar
flow, find
312 Thermal Engineering

(a) if the pressure at a certain section in the nozzle Temperature at exit


is 75 kPa, and calculate the Mach number, -1
Ê g - 1 2ˆ
temperature and velocity at this section T = T0 Á1 + M ˜
Ë 2 ¯
(b) if the nozzle has circular cross-section and its -1
diameter is 12 mm at the section discussed in (a) Ê 0.6 ˆ
= (300 K ) ¥ Á1 + ¥ (1.816) 2 ˜
above, and calculate the mass flow rate through Ë 2 ¯
the nozzle. = 150.8 K = –122.2°C
The specific gas constant for air
Solution R 8314
R = u = = 231.58 J/kg ◊ K
Given The steady one-dimensional isentropic flow M 53 .9
through a nozzle with The sonic velocity corresponds to this temperature
M = 35.9 kg/kmol g = 1.60 a = g RT = 1.6 ¥ 231.58 ¥ 150.8
p0 = 470 kPa T0 = 27°C = 300 K = 236.38 m/s
V0 = 0 Hence fluid velocity,
\ M =0 V = a M = 236.38 ¥ 1.816
p = 75 kPa = 429.28 m/s
d = 12 mm = 12 ¥ 10–3 m (ii) Mass flow rate;
p 75 ¥ 103
The density r = =
RT 231.58 ¥ 150.8
= 2.15 kg/m3
Hence
p
m = r AV = 2.15 ¥ ¥ (12 ¥ 10 -3 ) 2 ¥ 429.28
4
= 0.104 kg/s

Fig. 10.13 10.7 ONE -DIMENSIONAL


ISENTROPIC FLOW
To find
(i) Mach number, temperature and velocity, when The flow through the ducts, where the flow cross-
pressure is 75 kPa at some point in the nozzle. sectional area varies gradually is of practical inter-
(ii) When the nozzle has diameter of 12 mm, the est. The nozzles and diffusers are most common ex-
mass flow rate through the nozzle. amples of flow devices in which passage of cross-
Analysis sectional area changes in the flow direction. In
(i) For isentropic flow, we have the relation such devices, the heat transfer may be considered
g negligible. If the frictional effects in such devices
p0 È g - 1 2 ˘ g -1
= Í1 + M ˙ are negligibly small then the flow through such de-
p Î 2 ˚ vices can be treated as one-dimensional isentropic
It follows the Mach number flow with good accuracy. In addition, when a fluid
flows through a varying cross-section area duct, the
È g -1 ˘
2 ÍÊ p0 ˆ g ˙ pressure, temperature, and velocity of fluid vary
M = Í - 1˙
g - 1 ÍÁË p ˜¯ ˙
contiuously. These variations are analyzed in this
Î ˚ section.

È 1.6 - 1 ˘
2 ÍÊ 470 ˆ 1.6 ˙
Hence, M = ¥ ÍÁ - 1˙ = 1.816
0.6 Ë 75 ˜¯ Consider a control volume of a varying cross-
Í ˙
Î ˚ sectional area duct in which the fluid flows steadily
Compressible Fluid Flow 313

Substituting the definition of velocity of sound


in an isentropic flow, Eq. (10.12), we get
dA dp Ê 1 1ˆ
= Á - 2˜
A r VË 2
a ¯
dp Ê V2 ˆ
= 2 Á
1- 2 ˜
rV Ë a ¯
Using the definition of Mach number,
Eq. (10.17),
dA dp
= 2 (1 – M 2) ...(10.33)
and isentropically. A V r
The continuity equation for a steady one-dimen- Further, using Eq. (10.31) for the value of dp/r
sional flow process: in Eq. (10.33), we get
m = rAV = constant
dA dV
Differentiating and dividing the resultant equa- or =– (1 – M 2) ...(10.34)
tion by continuity equation, we get, A V
2
Further, using dp = a dr in Eq. (10.33), to get
dr dA d V
+ + =0 ...(10.28) area variation in terms of density change
r A V
The steady flow energy equation with negligible dA a 2 dr
= (1 - M 2 )
potential energy change ( w = 0, q = 0, Dpe = 0 can A r V2
be expressed as
dr Ê 1 - M 2 ˆ
V2 V2 = ...(10.35)
h1 + 1 = h2 + 2 r ÁË M 2 ˜¯
2 2
In differential form, Equations (10.33), (10.34) and (10.35) are im-
dh + Vd V = 0 ...(10.29) portant relations for isentropic flow through a duct,
Using Tds relation, since these equations describe the variation of pres-
Tds = dh – v dp sure, velocity and density with change of duct area.
For isentropic flow (Tds = 0),
dp
dh = v dp = ...(10.30)
r A nozzle is a device, used to increase the velocity of
Substituting dh in Eq. (10.29), we get
fluid passing through it at the expense of pressure
dp dV
VdV + =0 ...(10.31) drop. Hence for a nozzle, the quantity must be
r V
dp
This relation is the differential form of positive and the quantity must be negative in
Bernoulli’s equation. p
the direction of flow.
Substituting dV from Eq. (10.31) into
For subsonic flow, M < 1, M 2 <
Eq. (10.28), we get
1; dV > 0 fi dA < 0, the term (1 – M2) is always
dr dA dp positive. It indicates that when a duct converges
+ - 2 =0
r A V r in the direction of flow, i.e., flow area decreases
(dA < 0), the fluid pressure decreases, Eq. (10.33),
dA dp Ê 1 dr ˆ
or = Á - ...(10.32) while fluid velocity increases, Eq. (10.34) as shown
A r ËV 2 dp ˜¯
in Fig. 10.15(a).
314 Thermal Engineering

a diffuser, d V must be negative and dp must be


positive.
For subsonic flow, M < 1, M 2 <
1; d V < 0 fi dA > 0: the term (1 – M2) is positive.
It indicates that when duct diverges in the direction
of flow, i.e., flow area increases (dA > 0), the fluid
pressure increases, Eq. (10.33), and fluid velocity
decreases, Eq. (10.34), as shown in Fig. 10.17(b).

For supersonic flow, M > 1, M 2


> 1, d V > 0 fi dA > 0. The term (1 – M 2) is always
negative. It indicates that when the duct diverges
in the direction of flow, i.e., flow area increases
(dA > 0), the fluid pressure decreases, Eq. (10.33),
while fluid velocity increases Eq. (10.34) as shown
in Fig. 10.15(b). Hence to obtain nozzle action in
supersonic stream, the passage must be diverging.
Whenever highest velocity is desired, then a
converging nozzle is used at subsonic velocities For supersonic flow, M >
and a diverging nozzle is used at supersonic ve- 1, M 2 > 1; d V < 0 fi dA < 0: the term (1 – M 2)
locities. Hence, the nozzles are constructed with is negative. It indicates that when duct converges
three parts, converging duct, throat and diverging in the direction of flow, i.e., flow area decreases
duct as shown in Fig. 10.16. In the converging duct, (dA < 0), the fluid pressure increases, Eq. (10.33),
the velocity of fluid increases up to sonic velocity while fluid velocity decreases, Eq. (10.34) as shown
(highest attainable in convergent portion) and then in Fig. 10.17(b).
fluid flows in the diverging section via the throat, to
Whenever a compressible fluid is desired to
further accelerate the velocity.
decelerate from supersonic speed to subsonic
speed, the fluid must pass a converging–diverging
duct of shape as shown in Fig. 10.18. To get the
diffuser action in the duct, the fluid flow at the
throat must be sonic.

A diffuser is a device, used to increase the pressure


of the fluid at the expense of velocity drop. For
Compressible Fluid Flow 315

mass flow rate of gases through the nozzle.


3. Further, decrease in back pressure results
in greater mass flow rate. In this case, the
The back pressure is the pressure in the exhaust re- velocity is subsonic through the nozzle and
gion outside the nozzle. The case of a converging the exit pressure pe equals the back pressure
nozzles is taken up first and then converging–di- pb. The Mach number increases as pb
verging nozzle is considered. decreases.
4. If the back pressure pb is further reduced to
p3 = p*, the mass flow rate reaches its maxi-
mum possible value. This case is represented
Consider a convergent nozzle is attached to a large
by 3 on Fig. 10.19. The Mach number reach-
reservoir as shown in Fig. 10.19. Since the gas ve-
es unity and it cannot be increased further in
locity at the inlet to nozzle will be relatively small
conversing section. p* is the pressure corre-
and flow through the nozzle is isentropic, hence the
sponding to sonic velocity, thus called criti-
stagnation pressure and the stagnation temperature
cal pressure.
at any section of the nozzle will be equal to gas
pressure and temperature in the reservoir, and thus, 5. If the back pressure is further reduced to a
may be designated as p0 and T0, respectively. The value p4 less than p* as in the case 4, the
pressure at the exit plane of the nozzle is pe and the velocity at the nozzle exit remains sonic
back pressure is pb. velocity. Neither the mass flow rate nor
pressure variation in the nozzle is affected
1. When the back pressure pb = p0, there is no and the nozzle is said to be chocked. For a
mass flow m = 0 through the nozzle. This chocked nozzle, the mass-flow rate reaches
corresponds to the case 1 of Fig. 10.19. the maximum possible value for given stag-
2. If the back pressure pb is reduced to p2 as in nation conditions.
the case 2, it causes the pressure to decrease 6. For pb less than p* the flow expands outside
in flow direction which results in a certain the nozzle to match the lower back pressure.
316 Thermal Engineering

2. When the back pressure pb is just slightly


below inlet pressure p0 ( pB in the case B),
the fluid velocity increases in the converg-
The highest velocity to which a fluid can be sub-
ing section and reaches maximum at the
jected in a convergent nozzle is limited to sonic
throat (M < 1), but decreases in the diverg-
velocity (M = 1), which occurs at the exit plane of
ing section. There is a decrease in the pres-
the nozzle. However, the fluid flow can be accel-
sure in converging section, which becomes
erated from subsonic to supersonic by attaching a
minimum at the throat and increases in the
diverging duct to the subsonic nozzle at the throat.
diverging section at the expense of the veloc-
For given inlet conditions, the flow through a con-
ity. The flow becomes subsonic throughout
vergent–divergent nozzle is governed by the back
the nozzle.
pressure as explained below:
3. As the back pressure pb is further reduced
Consider an isentropic fluid flow from an infinite
( pC in the case C ), the pressure at the throat
reservoir, through a convergent–divergent nozzle to
becomes the critical pressure p*, the fluid
an exhaust chamber as shown in Fig. 10.20. Since
velocity becomes sonic at the throat. The
the inlet velocity V to the nozzle is very small,
fluid mass flow reaches maximum and the
hence, the conditions at the inlet to the nozzle are at
flow is said to be choked. But the diverging
the stagnation state. Let the inlet conditions be p0,
section acts as diffuser and hence, the fluid
T0, and r0. The nozzle exit pressure be pe, while pb
velocity decreases and pressure increases in
is the back pressure (pressure in the chamber).
this section as shown by the curve C.
1. When the back pressure pb is equal to p0 (pA 4. When the back pressure pb is further reduced
in the case A) then the pressure in the nozzle ( pD in the case D), the fluid flow rate at the
will be uniform throughout and there will no throat remains constant. The fluid velocity
fluid flow through the nozzle. continues to increase and accelerate in the
Compressible Fluid Flow 317

diverging section of the nozzle and its ve- 2g R


locity becomes supersonic as shown by the V = (T0 - T )
g -1
curve D.
However, at a section in downstream, a 2g RT0 Ê Tˆ
discontinuity occurs in the flow and abrupt
or V =
g -1 ÁË 1 - T ˜¯ ...(10.36)
0
increase in pressure is noticed and fluid flow
For isentropic process, using
decelerates from supersonic to subsonic. g -1
This discontinuity in the flow is called nor- T Ê pˆ g
mal shock. The flow through the shock is = Á ˜
T0 Ë p0 ¯
steady, irreversible and adiabatic.
5. If the back pressure pb is further reduced to Ï g -1 ¸
pE, the shock shifts towards downstream, ap- 2g RT0 Ô Ê pˆ g Ô
then V= Ì1 - Á ˜ ˝ ...(10.37)
proaching the nozzle exit as shown by the g -1 Ô Ë p0 ¯ Ô
curve E. Ó ˛
6. When the back pressure pb is at the value The mass flow rate
represented by the point F, the normal shock m = r AV
stands in the exit plane of the nozzle. The g -1 ¸
Ï
flow through the duct is isentropic; subson- 2g RT0 Ô Ê pˆ g Ô
ic in convergent nozzle, sonic at the throat or m = rA Ì1 - Á ˜ ˝ ...(10.38)
g -1 Ô Ë p0 ¯ Ô
and supersonic in the diverging portion. Ó ˛
However, the fluid passes through the shock p
Using r =
as it leaves the nozzle and hence, jet leaving RT g -1
the nozzle becomes subsonic.
Ê pˆ g
7. When the back pressure pb is slightly below and T = T0 Á ˜
Ë p0 ¯
pF, the fluid expands isentropically up to the
g -1 1
nozzle exit plane and no shock forms within p Ê p0 ˆ g p Ê p ˆg
Thus r = = 0 Á ˜
or outside the nozzle and one-dimensional RT0 ÁË p ˜¯ RT0 Ë p0 ¯
supersonic jet leaves the nozzle.
Substituting in to Eq. (10.38)
For any further reduction in back pressure
pb, the flow within and outside, the nozzle Ï 2 g +1 ¸
remains the same. Ap0 2g RT0 ÔÊ p ˆ g Ê p ˆ g Ô
m = Ì -Á ˜ ˝
RT0 g -1 ÔÓÁË p0 ˜¯ Ë p0 ¯ Ô˛

Ï 2 g +1 ¸
Ap0 2g ÔÌÊ p ˆ g Ê p ˆ g Ô˝
or m = -Á ˜
Since the fluid flow through the nozzle is isentropic, RT0 g - 1 ÔÓÁË p0 ˜¯ Ë p0 ¯ Ô˛
therefore, the stagnation enthalpy remains constant.
...(10.39)
The velocity at any cross section in the nozzle; 1/ 2
Ï 2 g +1 ¸
2( h0 - h) Ê p ˆg Ê p ˆ g Ô
m = C ÔÌÁ ˜ - Á
V=
or ˝ ...(10.40)
Using h = Cp T; V = 2C p (T0 - T ) ÔË p0 ¯ Ë p0 ˜¯ Ô
Ó ˛
gR
Using Cp = where C is constant and an equivalent of constant
g -1
quantities g, R, T0, p0 and A in the above equation.
318 Thermal Engineering

For maximum discharge rate, differentiating the AMp0


above equation with respect to throat pressure p and m =
g
equating it to zero; Ê g - 1 2 ˆ g -1
ÁË1 + 2 M ˜¯
dm
=0 1
dp g
g È g - 1 2˘2
p* Ê 2 ˆ g -1 ¥ ¥ 1+ M ˙
we get, = Á ...(10.41) RT0 ÍÎ 2 ˚
p0 Ë g + 1˜¯
The properties of a fluid at throat (M = 1) are AMp0 g
or m = g +1
...(10.46)
called the critical properties and denoted by the RT0
Ê g -1 2 ˆ 2 (g -1)
subscript asterisk (*). The pressure p* corresponds ÁË1 + 2 M ˜¯
to maximum discharge, thus it is called critical
pressure. For all back pressures lower than p*, the For choked flow, M = 1 and A = A*, and the
pressure of the exit plane of the convergent nozzle flowing fluid is air (g = 1.4), the maximum flow
is equal to p*. rate through the nozzle is obtained as
The velocity at the throat can be obtained by 0.6847 p0 A*
mmax = ...(10.47)
substituting the above Eq. (10.41) in Eq. (10.37) as RT0
2g RT0 ÏÔ Ê 2 ˆ ¸Ô Thus, for any ideal gas, mmax through the nozzle
V* = Ì1 - Á ˜ ˝ ...(10.42) is controlled by p0 and T0, hence, a convergent
g -1 ÓÔ Ë g + 1¯ Ô˛ nozzle can be used as a flowmeter.
2 g RT0
or V* = ...(10.43) Example 10.14 Helium gas at 6.9 bar and 93°C
g +1 enters a convergent nozzle with negligible velocity and
expands isentropically to a pressure of 3.6 bar. Calculate
or V* = g RT * = a* ...(10.44)
the mass flow rate per square metre of the exit area.
2 Take Cp for helium gas as 5.19 kJ/kg ◊ K and molar
Where T* = T0 ...(10.45)
g +1 mass as 4 kg/kmol.
The temperature T* is the critical temperature at
the throat. The fluid has sonic velocity at the throat
when the mass flow rate is maximum.
The convergent–divergent nozzle has minimum
area at the throat, thus the fluid mass flow rate is
maximum at the throat. Under steady flow condi-
tion, the mass flow rate through the nozzle is con-
stant and can be expressed as
p
m = r AV = A ( M g RT )
RT Solution
g
= AMp Given Isentropic flow of helium gas through a conver-
RT
gent nozzle
Using terms of T0 and p0 from Eqs. (10.22) and p1 = 6.9 bar T1 = 93°C = 366 K
(10.23), the above equation can be expressed as p2 = 3.6 bar V1 = 0
C p = 5.19 kJ/kg ◊ K M = 4 kg/kmol
Compressible Fluid Flow 319

To find The mass flow rate of helium per square metre ture, exit Mach number and exit velocity for the following
of the nozzle exit area. conditions:
Assumptions (a) Sonic velocity at the throat, diverging section
acting as a nozzle,
(i) Helium as a perfect gas.
(b) Sonic velocity at the throat, diverging section
(ii) Change in potential energy is zero.
acting as a diffuser.
Analysis The specific gas constant for helium
Solution
Ru 8314
R = = = 2078.5 J/kg ◊ K.
M 4 Given A convergent–divergent nozzle
gR p0 = 1 MPa T0 = 360 K
and Cp =
g -1 A/A* = 2 A* = 500 mm2
g -1 R 2078.5
or = = = 0.4 To find
g C p 5.19 ¥ 103 (i) Mass flow rate,
1 (ii) Exit temperature,
Thus g = = 1.667
1 - 0.4 (iii) Exit pressure,
The critical pressure ratio for helium gas flow through (iv) Exit Mach number, and
the nozzle (v) Exit velocity.
g 1.667
p* Ê 2 ˆ g -1 Ê 2 ˆ 1.667 -1 = 0.487 Assumptions
= Á =Á
p1 Ë g + 1˜¯ Ë 1.667 + 1˜¯ (i) Isentropic flow, and
or p* = 0.487 ¥ 6.9 = 3.360 bar (ii) Air as ideal gas with Cp = 1005 J/kg ◊ K, g = 1.4
which is less than the exit pressure and hence the fluid and R = 0.287 kJ/kg ◊ K.
does not reach the critical conditions and the nozzle is Analysis
not chocked.
(i) For A/A* = 2, we get two Mach numbers, greater
Temperature T2 at the nozzle exit than unity and less than unity. When diverging
g -1 1.667 - 1
section acts as supersonic nozzle, then M > 1.
Êp ˆ g Ê 3.6 ˆ 1.667
T2 = T1 Á 2 ˜ = 366 ¥ Á From Table A-10;
Ë p1 ¯ Ë 6.9 ˜¯
A pe
= 282.14 K = 2 Me = 2.197 = 0.0938
A* p0
The velocity at the exit of the nozzle Te
= 0.5089
V2 = 2 Cp (T1 - T2 ) T0
Therefore, pe = 0.0938 ¥ (1 MPa) = 0.0938 MPa,
= 2 ¥ 5.19 ¥ 103 ¥ (366 - 282.14) Te = 0.5089 ¥ (360 K) = 183.2 K
= 933 m/s
ae = g RTe
The specific volume at the nozzle exit
= 1.4 ¥ ( 287 J/kg ◊ K ) ¥ (183.2 K )
RT2 2.0785 ¥ 282.14
v2 = = = 1.63 m3/kg = 271.3 m/s
p2 3.6 ¥ 10 2
Ve = Me ae = 2.197 ¥ (271.3 m/s)
The mass flow rate of helium for 1 m2 exit area
= 596.1 m/s
AV2 1 ¥ 933
m = = = 572.75 kg/s The mass flow rate is determined at throat
v2 1.63
condition. Here, the velocity at the throat is sonic
velocity (M = 1). Thus, the density of air at the
Example 10.15 A convergent–divergent nozzle has
throat,
an exit area to throat area ratio of 2. Air enters this noz-
zle with a stagnation pressure of 1 MPa and a stagna- pe (0.0938 ¥ 1000 kPa )
re = =
tion temperature of 360 K. The throat area is 500 mm². RTe (0.287 kJ/kg ◊ K ) ¥ (183.2 K )
Determine the mass flow rate, exit pressure, exit tempera- = 1.784 kg/m3
320 Thermal Engineering

The mass flow rate at the throat, (700 kPa )


m = r e A e Ve m =
(0.287 kJ/kg ◊ K ) ¥ (301 K )
= (1.784 kg/m3) ¥ (500 ¥ 10– 6 m2)
¥ (10 ¥ 10–4 m2) ¥ (80 m/s)
¥ (271.3 m/s)
= 0.648 kg/s
= 0.242 kg/s
(ii) The sonic velocity at the inlet can be calculated as
(ii) When the diverging section acts as a diffuser
nozzle then M < 1. From Table A-10; ai = g RTi = 1.4 ¥ ( 287 J/kg ◊ K) ¥ (301 K)
A = 347.7 m/s
= 2, Me = 0.308,
A* The inlet Mach number is
pe Te Vi 80 m/s
= 0.936, = 0.9812 Mi = = = 0.23
p0 T0 ai 347.7 m/s
Therefore, pe = 0.308 ¥ (1 MPa) = 0.308 MPa, For isentropic expansion through the nozzle
Te = 0.9812 ¥ (360 K) = 353.3 K g
È g - 1 2 ˘g -1
ae = g RTe = 1.4 ¥ ( 287 J/kg ◊ K) ¥ (353.3 K) pe Í1 + 2 M i ˙
= Í ˙
= 376.8 m/s pi Í1 + g - 1 M e2 ˙
Ve = Me ae = 0.308 ¥ (376.8 m/s) = 116 m/s ÍÎ 2 ˙˚
The mass flow rate remains the same, since
M = 1.
1.4
Example 10.16 Air flows through a nozzle which È 1.4 - 1 2 ˘ 1.4 -1
250 Í1 + 2 (0.23) ˙
has an inlet area of 10 cm2. If air has a velocity of or = Í ˙
80 m/s, a temperature of 28°C, and a pressure of 700 kPa 700 Í 1 + 1.4 - 1 M e2 ˙
ÍÎ 2 ˙˚
at the inlet section and a pressure of 250 kPa at the exit,
find the mass flow rate through the nozzle and the velocity or Me = 1.335
at the exit, when flow conditions are isentropic. The exit temperature
g -1
Solution Êp ˆ g
Te = Ti Á e ˜
Ë pi ¯
Given 1.4 -1
pi = 700 kPa Ti = 28°C = 301 K Ê 250 ˆ 1.4
Te = 301 ¥ Á = 224.3 K
Vi = 80 m/s Ai = 10 cm2 Ë 700 ¯˜
pe = 250 kPa si = se The exit velocity,
To find Ve = Me ¥ ae = Me g RTe
(i) The mass flow rate, and = 1.335 ¥ 1.4 ¥ 287 ¥ 224.3
(ii) The exit velocity. = 400.8 m/s
Assumptions
(i) Isentropic flow
(ii) Air as an ideal gas with Cp = 1005 J/kg ◊ K, g = 1.4
and R = 0.287 kJ/kg ◊ K. The shock wave is an extremely thin region in the
Analysis flow field, in which transition from supersonic ve-
(i) The mass flow rate is given by
locity to relatively low velocity occurs. The normal
p waves exist due to abrupt changes in fluid prop-
m = ri Ai Vi = i ¥ Ai ¥ Vi
RTi erties. A shock wave may be viewed similar to a
hydraulic jump which occurs in free surface liq-
Compressible Fluid Flow 321

uid flows. The shock waves are generally curved. The momentum equation;
However, many shock waves that occur in practical A( px – py) = m (Vx – Vy) ...(10.50)
are straight, being at right angles to flow direction The second law of thermodynamics;
(normal shock wave), at an angle to upstream flow sy – sx ≥ 0 ...(10.51)
(oblique shock wave). If we combine energy and continuity equation
Figure 10.22 shows a control surface that into a single equation and plot it on an h–s dia-
includes normal shock. The changes in fluid gram using property relations, the resultant curve
properties across a normal shock are illustrated in is called Fanno line. The momentum and continu-
Fig. 10.22. The fluid velocity decreases, while the ity equations in combination give Rayleigh line.
pressure, temperature, density, etc., increase after Both these lines are shown in Fig. 10.23 on an h–s
normal shock occurs. diagram, simultaneously. The points a and b cor-
respond to maximum entropy for M = 1. The lower
part of each curve corresponds to supersonic ve-
locities, and the upper part corresponds to subsonic
velocities.
The two points, where all three equations are
satisfied are the points x and y, the point x being in
a supersonic region and the point y in subsonic re-
gion. For any adiabatic process, sy – sx ≥ 0 accord-
ing to the second law of thermodynamics. Hence,
the normal shock can proceed from x to y. The su-
personic velocity of flow becomes subsonic after
normal shock.
The fluid is assumed in thermal equilibrium be- Since the stagnation enthalpy remains constant
fore and after the shock wave. The property chang- across the shock, thus for an ideal gas
es across a normal shock can be obtained from T0x = T0y ...(10.52)
continuity equation rV = G (a constant) and the That is, the stagnation temperature of an ideal
momentum equation p + rV 2 = F (a constant). gas also remains constant across a normal shock.

Assuming steady flow with no work and heat in-


teractions and no potential energy changes, let the
subscripts x and y denote the conditions upstream
(inlet) and downstream (exit) of shock, respectively
as shown in Fig. 10.22.
The steady-state energy equation;
Vx2 V2
hx + = hy + y
2 2
h0x = h0y ...(10.48)
The continuity equation for constant area duct;
m
= rxVx = ry Vy ...(10.49)
A
322 Thermal Engineering

The momentum and continuity equations can be


combined to give an equation of Rayleigh line.
The actual flow process inside the shock wave is
m
non-isentropic, but the flow process ahead of and px – py = (Vx – Vy )
after the shock can be considered isentropic. Using A
Eq. (10.22) across the shock = ry Vy2 – rx Vx2
T0 x (g - 1) 2 or px + rx Vx2 = py + ry Vy2
=1+ Mx p py 2
Tx 2 or px + x Vx2 = py + Vy
T0 y (g - 1) 2 RTx RTy
and =1+ My Using a2 = g RT
Ty 2
We get px (1 + g Mx2) = py (1 + g My2)
Dividing the first equation by the second equa-
tion; py 1 + g M x2
(g - 1) 2 or = ...(10.56)
1+ Mx px 1 + g M y2
Ty 2
= ...(10.53) It is the equation of Rayleigh line.
Tx (g - 1) 2
1+ My The Eqs. (10.55) and (10.56) in combination
2
For an ideal gas, give
2
p py M x2 +
rx = x and ry = g -1
RTx RTy My2 = ....(10.57)
2g
Substituting it into the continuity equation, M x2 - 1
g -1
r x Vx = ry Vy
It represents the intersection of Fanno and
px Vx p y Vy Rayleigh lines and relates the Mach number up-
=
RTx RTy stream of the shock to downstream of the shock.
Rearranging the above equation and using These shocks occur in hypersonic nozzles as
M = V/a and a = g RT well as ahead of the supersonic air craft or bullets
as shown in Fig. 10.24. If we consider a station-
Ty p y Vy p y My ay
= = ary aircraft and movement of air with supersonic
Tx px Vx px Mx ax velocity towards the aircraft, the air follows the
stagnation process and it stops when it reaches the
p y M y g RTy p y M y Ty
= = nose of the aircraft. The normal shock occurs where
px Mx g RTx px Mx Tx velocity of air suddenly decelerates to subsonic. It
2 creates a sonic boom.
Ê py ˆ Ê Mx ˆ
2
Ty
Therefore, Á ˜ = ¥ ...(10.54) The entropy change across the shock can be
Ë px ¯ ÁË M y ˜¯ Tx calculated as
Combining Eqs. (10.53) and (10.54), i.e., T p
sy – sx = Cp ln y – R ln y ...(10.58)
combining energy and continuity equation, gives Tx px
pressure ratio across the shock;
Shock Shock
wave wave
g 1 Bullet
py Mx 1 + M x2
2
= ...(10.55)
px g 1
My 1+ M y2
2
It is also called the equation of Fanno line.
Compressible Fluid Flow 323

V2
= ...(10.61)
V2s
It follows that the velocity coefficient is equal to
The flow through the nozzles and diffusers are not the square root of the nozzle efficiency;
reversible adiabatic (isentropic) but irreversible
adiabatic. Thus, three important parameters are CV = h N ...(10.62)
used by which the actual flow can be compared The mass flow rate through the nozzle is the
with isentropic flow. These are nozzle efficiency, design consideration, because it is affected by
velocity coefficient and discharge coefficient. irreversibilities. An important parameter, coefficient
The nozzle efficiency (hN) can be expressed as of discharge (CD) relates the actual mass flow with
Actual kinetic energy at nozzle exit mass flow under the isentropic conditions for the
hN = same nozzle.
Kinetic energy at nozzle exit foor
isentropic flow to the same exist pressure Actual mass flow rate
CD =
Mass flow rate with isentropic flow
V22 m
= ...(10.59) = ...(10.63)
V22s ms
The nozzle efficiency can also be defined in Diffusers are designed to increase the pressure
terms of actual enthalpy drop and isentropic en- of a fluid by decelerating it. The diffuser efficiency
thalpy drop. It can be expressed as hD is based on its ability to convert the kinetic
h - h2 energy of the fluid into a useful pressure rise.
hN = 01 ...(10.60)
h01 - h2s Actual kinetic energy available
where h01 is stagnation enthalpy of the fluid at the for pressure rise
hD =
nozzle inlet and h2 is the enthalpy of fluid at the Maximum kineetic energy available
exit for actual nozzle, while h2s is the exit enthalpy
Refer Fig. 10.26, the h–s diagram states 1 and 01
for a nozzle under isentropic conditions. The noz-
are actual and stagnation states at the diffuser inlet,
zle efficiency varies from 90% to 99%. The larger
states 2 and 02 are actual and stagnation states at
nozzle has more efficiency than smaller nozzle.
the diffuser exit. The state 02s is a fictitious diffuser
Figure 10.25 shows the effect of irreversibility.
exit state, which would be attained for isentropic
The nozzle velocity coefficient (CV) is also an process in diffuser. The states 2 and 02 become
important parameter and can be expressed as identical for zero exit velocity of diffuser.
Actual velocity at the nozzle exit
CV =
Velocity at the nozzle exit with isentropic
flow and same exit pressure
h

01

2 2
½Ve ½V2s

2
2s

s
324 Thermal Engineering

The maximum kinetic energy at the diffuser inlet To Find


V12 (i) Ratio of exit area to inlet area.
= h01 – h1 (ii) Pressure rise coefficient.
2
Actual enthalpy drop, which can be converted to Assumptions
pressure rise (i) The fluid is an ideal gas.
D hs = h02s – h1 (ii) The specific heat Cp = 1005 J/kg ◊ K.
Then the diffuser efficiency can be expressed as (iii) Adiabatic diffuser.
h -h Analysis For adiabatic diffuser
hD = 02s 1 ...(10.63)
h01 - h1 T01 = T02
The diffuser efficiency varies from 90 to The stagnation temperature at the diffuser inlet
100 per cent for subsonic diffusers and it deceases V12
T01 = T1 +
with increasing Mach numbers. 2C p
Another parameter to measure the diffuser abil- (150) 2
= 300 K + = 311.2 K
ity to increase the pressure of the fluid is the pres- 2 ¥ 1005
sure recovery factor, FP, which is defined as the ra- V22 V2
tio of actual stagnation pressure at the diffuser exit and T2 = T02 – = T01 – 2
2Cp 2Cp
to isentropic stagnation pressure.
60 2
p = 311.2 – = 309.4 K
FP = 02 ...(10.64) 2 ¥ 1005
p01 Then the stagnation pressure at the diffuser inlet
A third parameter to evaluate diffuser’s perfor- g 1.4
mance is the pressure-rise coefficient CPR . It is de- Ê T ˆ g -1 Ê 311.2 ˆ 0.4
p01 = p1 Á 01 ˜ = (100 kPa ) ¥ Á
fined as the ratio of actual pressure rise in the diffus- Ë T1 ¯ Ë 300 ˜¯
er to that corresponding to isentropic process. Thus, = 113.68 kPa
Actual pressure rise For air as an ideal gas, the diffuser efficiency can be
CPR = expressed as
Isentropic pressure rise
h - h T -T
p - p1 hD = 02 s 1 = 02s 1
= 2 ...(10.65) h01 - h1 T01 - T1
p01 - p1
The pressure rise coefficient depends on flow
characteristics and shape of the diffuser and its
value is usually less than 0.8.

Example 10.17 A certain diffuser having an efficien-


cy of 90% is used to reduce the velocity of an air stream
initially at 150 m/s, 300 K, and 100 kPa down to 60 m/s.
Calculate the ratio of exit area to inlet area to accom-
plish this reduction. What is the value of pressure rise
coefficient for this diffuser?

Solution
Given A diffuser with
hD = 0.9 V1 = 150 m/s
T1 = 300 K, p1 = 100 kPa
V2 = 60 m/s
Compressible Fluid Flow 325

T02 s - 300 divergent portion. The fluid friction is also small


or 0.9 = fi T02s = 310.08 K
311.2 - 300 in the convergent portion than in the divergent por-
1.4
ÊT ˆ Ê 310.08 ˆ 0.4 tion, since the fluid velocity in the convergent por-
and p02s = p1 Á 02 s ˜ = (100 kPa) ¥ Á
Ë T ¯
1 Ë 300 ˜¯ tion is also small. Thus, most of the friction occurs
= 112.26 kPa in the divergent portion of the nozzle and the h–s
Refering Fig. 10.27, p02s = p02, then diagram takes the shape as shown in Fig. 10.28.
g 1.4 The nozzle efficiency for such a nozzle is given as
Ê T ˆ g -1 Ê 309.4 ˆ 0.4
p2 = p02 Á 2 ˜ = 112.26 ¥ Á h* - he
Ë T02 ¯ Ë 311.2 ˜¯ h N,divergent = * ...(10.66)
h - hes
= 110.01 kPa
The pressure rise coefficient
p - p1 110.01 - 100
CPR = 2 = = 0.732
p01 - p1 113.68 - 100
Exit-area-to-inlet-area ratio is determined by use of
continuity equation
m = r1V1 A1 = r2 V2 A2
p
Using r =
RT
A2 p T V 100 ¥ 309.4 ¥ 150
we get = 1 2 1= = 2.343
A1 T1 p2 V2 300 ¥ 110.01 ¥ 60

Another cause of irreversibility in the nozzles


and diffusers is flow separation, which induces
strong turbulence near the nozzle wall. Flow sep-
In the preceeding section of this chapter, the flow
aration occurs when the angle of divergence in a
through the nozzles and diffusers is approximated
convergent–divergent nozzle is made too large.
isentropic. Actual expansion through nozzles is
Consequently, the flow area increases faster than
non-isentropic flow. It is due to presence of irre-
the fluid expands. Thus the included (cone) angle
versibilies at the surface of flow and within the fluid
of the divergent duct is usually kept less than 20°C.
itself. The primary cause of irreversibly in nozzles
(and diffusers) is the presense of frictional effects, The frictional losses in the nozzle depends upon
which are due to the material of construction, size, shape and surface
because the wall surface in the large nozzles occu-
(i) friction between fluid and wall surface of pies a smaller potion of total flow volume.
nozzle, and
The effect of friction in a nozzle can be summa-
(ii) friction within the fluid itself. rized as
The friction between the wall surface and fluid
(i) Reduction in enthalpy drop,
molecules makes the expansion adiabatic but not
(ii) Reheating of fluid,
isentropic. The energy lost in overcoming the fric-
(iii) Reduction in exit velocity,
tion is used to reheat the fluid and the enthalpy and
entropy of fluid increase during the process. (iv) Increase in specific volume, and
The convergent portion of the nozzle is smaller (v) Decrease in mass flow rate.
than the divergent portion. Thus the wall friction
is small in the convergent portion as compared to
326 Thermal Engineering

Summary
T0 Ê g - 1ˆ 2
= 1+ Á M
stagnation enthalpy is the combination of enthal- T Ë 2 ˜¯
py and kinetic energy of fluid g
V 2 p0 È Ê g - 1 ˆ 2 ˘ g -1
h0 = h + (kJ/kg) = Í1 + Á ˜M ˙
2 p Î Ë 2 ¯ ˚
stagnation state are called the 1 /(g -1)
stagnation properties and are designated by the r0 È Ê g - 1ˆ 2 ˘
= Í1 + Á ˜M ˙
subscript 0. The stagnation temperature of an r Î Ë 2 ¯ ˚
ideal gas with constant specific heat is M = 1, the above relations are termed as
V2 critical ratio, and critical properties are denoted
T0 = T +
2C p by a superscript * (asterisk):
which is the temperature when an ideal gas is T* 2
brought to rest isentropically. =
T0 g +1
g
related as p* Ê 2 ˆ g -1
= Á
Ë g + 1˜¯
g
p0
p0 Ê T ˆ g -1
= Á 0˜ 1
p ËT¯ r* Ê 2 ˆ (g -1)
= Á
Ë g + 1˜¯
1
r0 Ê T ˆ (g -1) r0
= Á 0˜
r ËT¯ nozzle is
called the back pressure. For all back pressures
velocity of sound in a gas medium is given by
lower than the critical pressure p*, the pressure at
Ê ∂p ˆ the exit plane of the convergent nozzle is equal to
a = ÁË ∂r ˜¯ = g RT
p* and Mach number at the exit plane is unity and
s=c
mass flow rate is maximum (or choked)
Mach number is defined as
V
M = through a nozzle is given by
a
M = 1; subsonic g
AM p0
when M < 1; supersonic when M > 1; and hyper- RT0
m =
sonic when M > 5. g +1
- Ê g - 1 2 ˆ 2(g -1)
ÁË1 + 2 M ˜¯
rection are called convergent nozzles. The nozzles
whose flow area first decreases and then increases back pressure, the gas attains
in flow direction are called convergent–divergent the sonic velocity at the throat of a convergent–
nozzles. The location of the smallest flow area of divergent nozzle and is accelerated to supersonic
a nozzle is called throat. velocity. In the divergent section-experiences a
nozzle, the fluid can only be ac- normal shock, which causes a sudden rise in pres-
celerated to sonic velocity. Acceleration of fluid sure and temperature and a sudden drop in veloc-
to supersonic velocity is only possible in a con- ity. The flow through shock is irreversible. The
vergent–divergent nozzle. properties of an ideal gas with constant specific
- heats before (subscript x) and after (subseript y) a
ties for ideal gases with constant specific heats shock are related as
are
Compressible Fluid Flow 327

T0x = T0y defined as


h01 - h2 V2
g -1 2 hN = = 22
py Mx 1 + Mx h01 - h2 s V2 s
= 2
px g -1 2 Actual enthalpy drop
My 1 + My =
2 Isentropic enthalpy drop
2 nozzle velocity coefficient;
M x2 +
g -1 V2
My2 = CV =
2g 2 V2s
Mx -1
g -1 Actual velocity at nozzle exit
=
Velocity at nozzle exit for isentropic flow
Ê Ty ˆ Ê py ˆ
sy – sx = C p ln Á ˜ - R ln Á ˜
Ë Tx ¯ Ë px ¯ m
CD =
ms
nozzle wall surface
and within the fluid makes the fluid flow non- Actual mass flow rate
=
isentropic but adiabatic. The nozzle efficiency is mass flow rate for isentropic expansio
n
o

Glossary
Incompressible Fluid A fluid whose density does not Supersonic flow The fluid flows with Mach number
change during a flow process greater than unity (that is, V > a).
Compressible Fluid A fluid whose density changes Hypersonic flow The fluid flows with a very high Mach
during a flow process number (that is, M > 5)
Static properties Properties of fluid with negligible Mach line A straight line along which circles of wave
kinetic energy front are formed
Stagnation Properties Properties of fluid with signifi- Mach angle The angle between the Mach line and the
cant kinetic energy direction of fluid flow
Sonic velocity The velocity at which a small pressure Mach cone A cone, which confines all the wave front
wave propagates in a fluid Zone of silence Region outside the mach cone
Mach number Ratio of local fluid velocity to the sonic Critical properties The properties of fluid at a location
velocity in the medium where the Mach number becomes unity (throat)
Subsonic flow The fluid flows with Mach number less Chocked Flow Flow of fluid condition when the mass
than unity (that is, V < a) flow rate reaches the maximum possible value
Sonic flow The fluid flow with Mach number is equal Fanno line A curve on h–s plot which represents en-
to 1 (that is, V = a) ergy and continuity equations both
Transonic flow The fluid flow with Mach number Rayleigh line A curve which represents momentum
around unity (0.8 < M < 1.2) and continuity equations in combination
328 Thermal Engineering

Review Questions
1. Discuss the shapes of nozzles required for the 12. Discuss the effect of back pressure on convergent
subsonic and supersonic flow. nozzle.
2. Derive the required condition for maximum mass 13. Prove that the velocity at the throat is sonic under
flow rate in case of nozzles. the conditions of maximum mass flow rate in case
3. Define sonic velocity and prove that of nozzles.
a = g RT 14. What do you understand by the term ‘critical
where a = sonic velocity pressure’ as applied to steam nozzles.
g = ratio of specific heats 15. In case of steam nozzles, show that the pressure
ratio of steam at throat to inlet is given by the
R = Specific gas constant
equation
T = absolute temperature of gas.
k
4. Define Mach number and classify various types p* Ê 2 ˆ k -1
= Á
of flow on the basis of Mach number. p1 Ë n + 1˜¯
5. Explain why a horn is heard before an automabile
reaches a person? under maximum mass flow rate conditions
6. Why is the sound of a jet plane heard after it has Hence, derive the equations for critical velocity
passed forward? and maximum mass flow rate.
7. Define (a) stagnation enthalpy, (b) stagnation 16. Prove that for an isentropic flow of an ideal gas
temperature, and (c) stagnation pressure. with constant specific heat ratio g, the ratio of
8. Prove that the stagnation temperature and stag- temperature T * to the stagnation temperature T0
nation pressure remain constant for an isentropic is
flow through the nozzle. T* 2
=
9. Prove that for an isentropic flow through a duct, T0 g +1
g -1 2
T2 1+ M1 17. Discuss the effect of friction in case of nozzles
= 2
g -1 2 with the help of Mollier’s diagram.
T1 1+ M2
2 18. Discuss the effect of variation of back pressure in
10. Prove that for sonic flow at throat case of convergent–divergent nozzles.
T* 2 19. What is shock wave? Define normal shock and
= oblique shock.
T0 g +1 1
p* Ê 2 ˆ g -1 20. Define
and = Á
p0 Ë g + 1˜¯ (a) Nozzle efficiency,
(b) Nozzle velocity coefficient,
11. Discuss the effects of area changes on fluid veloc-
(c) Coefficient of discharge, and pressure re-
ity, pressure and Mach number for
covery factor.
(a) subsonic nozzle,
(b) subsonic diffuser,
(c) supersonic nozzle, and
(d) supersonic diffuser,
Compressible Fluid Flow 329

Problems
1. Air leaves a compressor in a pipe with a area of the nozzle, the exit velocity, and increase
stagnation temperature and pressure of 150°C, of entropy per kg of air.
300 K, and a velocity of 125 m/s. The cross- 7. Calculate the throat and exit area of the nozzle
sectional area is 0.02 m2. Determine the static to expand air at the rate of 4.5 kg/s from 8.3 bar,
pressure, temperature and mass flow rate. 327°C into a space at 1.38 bar. Neglect the inlet
[281 kPa, 425.4 K, 5.9 kg/s] velocity and assume isentropic flow.
2. A convergent nozzle has an exit area of 500 mm2. [3290 mm2, 4840 mm2]
Air enters the nozzle with stagnation pressure of 8. It is required to produce a stream of helium at
10 bar and a stagnation temperature of 360 K. a rate of 0.1 kg/s, travelling at sonic velocity at
Determine the mass-flow rate for the back pres- a temperature of 15°C. Assume negligible inlet
sure of 800 kPa, 528 kPa and 300 kPa, assuming velocity, isentropic flow, and back pressure of
isentropic flow. 1.013 bar. Determine:
[0.8712 kg/s, 1.0646 kg/s, 1.0646 kg/s] (a) required inlet pressure and temperature,
3. A convergent–divergent nozzle has a throat diam- (b) exit area of the nozzle.
eter of 0.05 m and an exit diameter of 0.1 m. The For helium, take molar mass as 4 kg/k mol and
inlet stagnation state is 500 kPa, 500 K. Find the g = 1.66. [(a) 2.077 bar, 110°C, (b) 593 mm2]
back pressure that will lead to maximum possible 9. A gas expands isentropically through a converg-
flow rate and the mass flow rate for air. ing nozzle from a large tank at 10 bar, 600 K.
4. A gas with a molar mass of 4 and a specific heat Assuming ideal gas behaviour, calculate the criti-
ratio of 1.3 flows through a variable area duct. At cal pressure p* in bar and the corresponding tem-
some location in the flow, the velocity is 150 m/s, perature in kelvins, if the gas is
the pressure is 100 kPa and the temperature is (a) air,
15°C. Find the Mach number at this location of (b) oxygen (O2),
flow. At some other location of flow, the tempera-
(c) water vapour.
ture is found to be –10°C. Find the Mach num-
10. Air enters a nozzle operating at steady state at
ber, pressure and velocity at this location in the
3 bar, 400 K, with a velocity of 145 m/s, and
flow, assuming the flow to be isentropic and one-
expands isentropically to an exit velocity of
dimensional.
460 m/s. Determine
[M1 = 0.17, M2 = 0.8157, p2 = 67.5 kPa,
(a) exit pressure in a bar,
V2 = 687.6 m/s]
(b) ratio of exit area to inlet area,
5. A nozzle is designed for an isentropic flow with
(c) whether the nozzle is diverging only, con-
a Mach number of 2.6. The air flow is with stag-
verging only, or converging and diverging
nation pressure and temperature of 2 MPa and
in cross-section.
150°C. The mass-flow rate is 5 kg/s. Take the
ratio of specific heats as 1.4. Determine the exit 11. Air as an ideal gas with g = 1.4 undergoes a
pressure, temperature, area and the throat area. normal shock. The upstream conditions are px =
Suppose that the back pressure at the nozzle exit is 0.5 bar, Tx = 280 K, and Mx = 1.8. Determine
raised to 1.4 MPa and that the flow remains isen- (a) the pressure py in bar,
tropic except for a normal shock wave. Determine (b) the stagnation pressure p0x in bar,
the exit Mach number and the temperature, and (c) the stagnation temperature T0x in K,
the mass-flow rate through the nozzle. (d) the change in specific entropy across the
6. Air expanded in a nozzle from 700 kPa, 200°C, to shock, in kJ/kg ◊ K.
150 kPa in a nozzle having an efficiency of 90%. [(a) 1.81 bar, (b) 2.87 bar, (c) 461 K,
The mass flow rate is 4 kg/s. Determine the exit (d) 5.94 ¥ 10–3 kJ/kg ◊ K]
330 Thermal Engineering

12. Air expands in a convergent divergent nozzle (a) Mass flow rate through the nozzle stating
from 6.89 bar and 427°C into a space at 1 bar. whether nozzle is expanding or over-ex-
The throat area of the nozzle is 650 mm2 and the panding,
exit area is 975 mm2. The exit velocity is found to (b) Nozzle efficiency and coefficient of velocity.
be 680 m/s, when the inlet velocity is negligible. [(a) 0.684 kg/s, under expanding,
Assuming that friction in the convergent portion (b) p2 = 1.39 bar; 89.5%, 0.946]
is negligible, determine

Objective Questions
1. A fluid is a compressible fluid when its density 8. Whole friction loss in a convergent-divergent
(a) decreases with pressure nozzle occurs in
(b) increases with pressure (a) divergent portion
(c) increases with temperature (b) convergent portion
(d) both (a) and (b) (c) at throat
2. The Mach number is defined as (d) none of the above
Distance Sonic velicty 9. The exit pressure from a nozzle is equal to or
(a) (b) more than the critical pressure when it is
Flight velocity Flight distance
(a) in a divergent portion,
Local velocity
(c) (d) None of the above (b) convergent
Sonic velocity
(c) convergent–divergent
3. Flow of fluid is called transonic when (d) none of the above
(a) M > 1 (b) M = 1 10. If the exit pressure from a nozzle is less than
(c) M < 1 (d) 0.8 > M > 1.2 critical pressure, it is
4. The sound of an automobile is heard (a) divergent
(a) before it passes (b) after it passes (b) convergent
(c) when it reaches (d) none of the above (c) convergent–divergent
5. A nozzle is designed for (d) none of the above
(a) maximum pressure at outlet 11. Critical pressure in a convergent–divergent noz-
(b) minimum pressure at outlet, zle is defined as the ratio of
(c) maximum discharge at outlet (a) exit pressure and inlet pressure
(d) maximum discharge and maximum pres- (b) inlet pressure and outlet pressure
sure at outlet (c) throat pressure and inlet pressure
6. When a fluid is coming out of a duct at a higher (d) none of the above
pressure than it enters, the duct is called a/an 12. If the exit pressure from a nozzle is less than the
(a) orifice (b) nozzle critical pressure, the mass flow rate will be
(c) diffuser (d) venturi (a) decreasing (b) increasing
7. Friction loss in a nozzle (c) constant (d) none of the above
(a) increases the exit velocity 13. At critical pressure ratio for a nozzle, the velocity
(b) decrease the exit velocity at the outlet will be
(c) has no effect on exit velocity (a) more than the sonic velocity
(d) none of above (b) less than the sonic velocity
Compressible Fluid Flow 331

(c) equal to the sonic velocity 16. By increasing the length of the nozzle, the nozzle
(d) none of the above efficiency
14. Nozzle efficiency is defined as the ratio of (a) decreases
(a) actual enthalpy drop to isentropic enthalpy (b) remains constant
drop (c) increases
(b) isentroic enthalpy drop to actual enthalpy (d) is affected by frictional resistance
drop 17. Coefficient of discharge is designed as
(c) product of isentropic enthalpy drop and Actual kinetic energy change
(a)
actual enthalpy drop Isentropic kinetic energy change
(d) square root of isentropic enthalpy drop to Actual enthalpy drop
actual enthalpy drop (b)
Isentropic enthalpy drop
15. Frictional loss in a nozzle Actual nozzle exit velocity
(a) increases the enthalpy drop (c)
Nozzle exit velocity in isentropic flow
(b) decreases the enthapy drop Actual mass flow rate
(c) has no effect on the enthalpy drop (d)
Mass flow rate with isentropic flow
(d) none of the above

17. (d)
16. (c) 15. (b) 14. (a) 13. (c) 12. (c) 11. (c) 10. (c) 9. (b)
8. (a) 7. (b) 6. (c) 5. (c) 4. (a) 3. (d) 2. (c) 1. (d)
Answers
332 Thermal Engineering

11
Gas Power Cycles

Introduction
The devices producing net power output are called engines, and the thermodynamic cycles on which they
operate are called power cycles. The thermodynamic power cycles can be categorized as gas power cycles
and vapour power cycles.
Two principal types of engines are used: rotary and reciprocating engines. The reciprocating internal-
combustion engines operate on the Otto-cycle, Diesel cycle and dual cycle. The Otto-cycle engine, named
after its inventor, the German technician Nikolaus August Otto, is the familiar gasoline engine used in
automobiles and airplanes. The Diesel engine, named after the French-born German engineer Rudolf
Christian Karl Diesel, operates on a different principle and usually uses oil as a fuel. It is employed in
electric-generating and marine-power plants, in trucks and buses, and in some automobiles. Both Otto-cycle
and Diesel-cycle engines are manufactured in two-stroke and four-stroke cycle models.

3. It is also called piston displace-


ment volume or stroke volume. It is the volume cre-
Figures 11.1 and 11.2 show sketches of an inter- ated or displaced by the piston during its one stroke
nal combustion engine, a piston that reciprocates travel, i.e., travel of the piston from one dead centre
within a cylinder fitted with two valves and a spark to other dead centre. It is denoted by Vs and calcu-
plug or fuel injector. The sketches are lebelled with lated for one cycle as
some special terms, defined below: Vs = (p /4) d 2 L ...(11.1)
1. It is the internal diameter of the cylinder 4. Cubic Capacity or It is the
of the reciprocating engine. It is denoted by d and product of swept volume of a cylinder and the
expressed in mm. number of cylinders in an engine, i.e., total volume
displaced in each cycle of an engine. For k number
2. It is the linear distance through which
of cylinders,
the piston travels between the top dead centre
(TDC) and the bottom dead centre (BDC) in the Cubic Capacity = k Vs
cylinder. It is also called stroke length. It is denoted 5. It is the volume left in the
by L and expressed in mm. cylinder when the piston reaches the top dead cen-
tre. It is denoted by Vc.
Gas Power Cycles 333

Spark plug or as
fuel injector
Vc
Valve c = ...(11.4)
Vs
Top
dead center Clearance 9. R It is the ratio of cut-off volume to
Bore volume swept volume. It is denoted by r. This term is used
in Diesel-cycle and dual-cycle-engines.
Stroke Cylinder
wall 10. Expansion Ratio It is the ratio of maximum
volume to minimum volume during expansion in
Bottom the cylinder. It is denoted by re.
dead center Piston

Reciprocating 11. There are two fixed positions in


motion
the cylinder, between which the piston reciprocates.
The uppermost position is called top dead centre
Crank mechanism
(TDC), while the bottom-most position is called the
Rotary bottom dead centre (BDC).
motion

12. It is air-petrol mixture for petrol en-


Nomenclature for reciprocating volume en- gines and only air for Diesel engines, which is in-
gine
troduced during suction stroke of the engine.

13. It is a hypotheti-
Clearance
TDC Vc volume
cal average pressure, which if acted on the piston
Swept Piston during the entire power stroke, will produce the
L Vs
volume same power output as produced during the actual
BDC
cycle.
Net work done in a cycle,
Wnet = pm ¥ (piston area) ¥ (stroke)
(a) Swept volume (b) Clearance volume
= pm ¥ (displacement volume)
Swept volume and clearance of a reciprocat- = pm ¥ Vs
ing engine
W
Thus pm = net ...(11.5)
6. It is the maximum volume in the Vs
cylinder and is the sum of swept volume and clear- The actual indicator diagram of an engine and
ance volume. It is denoted by V1. its corresponding mean effective pressure for the
V1 = Vs + Vc ...(11.2) same power output is shown in Fig. 11.3. For any
particular engine, under specific operating con-
7. It is the ratio of maximum ditions, there will be an indicated mean effective
possible volume to clearance (minimum) volume in pressure (imep or pmi) and a corresponding brake
the cylinder. It is denoted by r and expressed as mean effective pressure (bmep or pmb). From a giv-
V V V + Vc en indicator diagram, the indicated mean effective
r = max = 1 = s ...(11.3)
Vmax V2 Vc pressure can be obtained as
Area of indicator diagram (mm 2 )
8. It is the ratio of clearance vol- pmi =
ume to swept volume. It is usually kept 3–5% of Length of the indicator diagram (mm)
the swept volume. It is denoted by c and expressed ¥ Spring constant (kPa/mm) ...(11.6)
334 Thermal Engineering

Air
Combustion
Actual engine
Fuel Products

Work and Heat


(a) Actual engine
Heat

Air Air-standard cycle Air


engine

Work and Heat


(b) Air-standard cycle engine

6. In addition, in cold air standard analysis, the


The network of a cycle is a product of the
specific heats of air are assumed to be con-
stant at their ambient temperature value.
The mean effective pressure is used to compare Cp = 1.005 kJ/kg ◊ K
the output of similar engines of different sizes. Cv = 0.717 kJ/kg ◊ K
R = 0.287 kJ/kg ◊ K
g = 1.4
The simplest models for both spark ignition and
compression ignition engines are air standard
cycles shown in Fig. 11.4. The processes of these
Basic operational principle of Carnot cycle is
ideal cycles are selected such that they should
discussed in Chapter 6. It consists of four reversible
be similar to actual cycles, as much as possible,
processes: isothermal heat addition, isentropic
because the analysis of actual cycles is very much
expansion, isothermal heat rejection and isentropic
complex. The assumptions made in the analysis of
compression. The p–v and T–s diagrams of Carnot
air standard cycles are the following:
cycle are replotted in Fig. 11.5.
1. The system is closed, thus the cycle is a From Fig. 11(b),
complete thermodynamic cycle and the Heat supplied per kg of air during isothermal
same fluid is used repeatedly. process 3–4
2. Air as an ideal gas is used as working fluid in qin = TH (s4 – s3)
the cycle.
Heat rejected per kg of air during isothermal
3. The compression and expansion processes
process 1–2;
are reversible, adiabatic.
qout = TL (s1 – s2)
4. The combustion process is replaced by a re-
versible heat-addition process from an exter- Process 2–3 and 4–1 are isentropic processes
nal source. \ q2–3 = 0 and q4–1 = 0
5. The exhaust process is replaced by a revers- and s1 = s4 and s2 = s3
ible constant-volume, heat-rejection process The net work done per kg of air in the cycle
to the surroundings to restore the system wnet = Sq = qin – qout
back to its initial state. = TH (s1 – s2) – TL (s1 – s2)
= (TH – TL) (s1 – s2)
Gas Power Cycles 335

Moreover, the thermal efficiency of Carnot cycle


conveys an important message that is applicable to
all types of engines. Thermal efficiency increases
with an increase in the average temperature at which
heat is added to the system or with a decrease in
average temperature at which heat is rejected from
the system.

This is an air standard cycle developed by Er. James


Stirling and Dr. Robert Stirling. It consists of two
isothermal processes and two reversible constant
volume processes.
Similar to Carnot cycle, the heat is added at
constant temperature TH and heat is rejected at
constant temperature TL. The p–v and T–s diagrams
of an ideal Stirling cycle are shown in Fig. 11.6
If two constant volume processes of Stirling
cycle are made regenerative i.e. heat rejected by

Carnot cycle

Thermal efficiency of Carnot cycle


w (T - TL ) ( s1 - s2 )
hCarnot = net = H
q in TH ( s1 - s2 )
TL
=1– ...(11.7)
TH
In actual practice, the transfer of heat energy at
constant temperature is very difficult to achieve,
because it would require very large heat exchangers
and the heat would be transfer with extremely slow
rate to be isothermal.
Further, the isentropic processes are very fast
processes, therefore, it is impractical to design an
engine which would operate with very fast and
extremely slow processes in succession thus no
thermodynamic cycle closely approximate Carnot
cycle.
Thermal efficiency of Carnot engine is the
function of source and sink temperatures only and
it is more than that of all actual and ideal cycles
operating between same two temperature limits.
336 Thermal Engineering

process 2–3 is given to process 4–1. Then net heat pressure lines. Thus the Ericsson cycle consists
transfer with these two constant volume processes of two isothermal and two reversible constant
becomes zero, pressure processes. Heat addition and heat rejection
Heat supplied per kg of air in the cycle take place at constant pressure as well at constant
Êv ˆ Êv ˆ temperature processes. Fig 11.7 shows p–v and T–s
qin = q3–4 = p3 v3 ln Á 4 ˜ = RTH ln Á 4 ˜ ...(i) diagrams of an Ericsson cycle.
Ë v3 ¯ Ë v3 ¯
It is evident from Fig. 11.7(b), that process 2–3
Heat rejected per kg of air in the cycle and 4–1 are parallel to each other on T–s diagram,
Êv ˆ Êv ˆ therefore, the heat rejected during process 4–1 can
qout = q1–2 = p1 v1 ln Á 1 ˜ = RTL ln Á 1 ˜ be supplied regenerative to process 2–3. External
Ë v2 ¯ Ë v2 ¯
heat is supplied during process 3–4 and heat is
Net work done in the cycle with ideal
rejected to ambient during process 1–2. Then the
regeneration
efficiency of an Ericsson cycle is equal to Stirling
wnet = S q = qin – qout
or Carnot cycle engine i.e.
Êv ˆ Êv ˆ T
= RTH ln Á 4 ˜ – RTL ln Á 1 ˜ hEricsson = hStirling = h Carnot = 1 – L ...(11.9)
Ë v3 ¯ Ë v2 ¯ TH
The Ericsson cycle uses smaller pressure ratio
Since processes 2–3 and 4–1 are constant
than Stirling and Carnot cycle. Ericsson cycle does
volume processes
not find any practical application in piston cylinder
v2 = v3 and v1 = v4 engines but it is approached by a gas turbine using
Êv ˆ a large number of stages with heat exchangers, and
\ wnet = R(TH – TL) ln Á 1 ˜
Ë v2 ¯ reheaters.
Thermal efficiency of Stirling cycle
Êv ˆ
R (TH - TL ) ln Á 1 ˜
wnet Ë v2 ¯
hStirling = =
qin Êv ˆ
RTH ln Á 1 ˜
Ë v2 ¯
TH - TL T
= =1– L ...(11.8)
TH TH
same as Carnot engine efficiency.
The Stirling engine was used earlier for hot air
engines. But as Otto and Diesel cycles are developed
and came into use, Stirling engine became obsolete.
The design of Stirling engine involves practical
difficulties of isothermal heat transfer. Still there is
an interest in this cycle because it has potential of
high thermal efficiency.

This cycle was also designed for hot air engine by


John Ericsson by replacing two constant volume
lines of Stirling cycle by two reversible constant
Gas Power Cycles 337

p
p3 3

qin
The Otto cycle is an ideal air standard cycle for Ise
the gasolene (petrol) engine, gas engines and high- ntr
p2 o pic
speed engines. The operations of the Otto cyle is 2

shown in Fig. 11.8 and the p–v and T–s diagrams Ise
ntro 4
pic
are shown in Fig. 11.9. The Otto cycle consists of qout
four processes: 1
0 v2 v1 v
Process 1–2 Isentropic compression,
Clearance Swept volume
Process 2–3 Reversible constant volume heat volume, vc vs = v1 – v2
addition, (a) p–v diagram
Process 3–4 Isentropic expansion, and T

Process 4–1 Reversible constant-volume heat


T3 3
rejection.
In an actual engine, the working substance t.
ns
co
becomes the air–fuel mixture and heat transfer v
=
processes are replaced by combustion process and T4 4
exhaust process.
st.
2 con
Analysis T2 v=

For Otto cycle, per unit mass of air T1


1
0 s
Heat supplied, qin = q2–3 = Cv (T3 – T2)
(a positive quantity) (b) T–s diagram for air standard Otto Cycle
Heat rejected qout = q4 –1 = Cv (T4 – T1)
(a negative quantity)
Net work done = Sq for a cycle
wnet = qin – qout
= Cv (T3 – T2) – Cv (T4 – T1)
The thermal efficiency of the cycle
wnet qin - qout q
hth = = = 1 - out
qin qin qin

Cv (T4 - T1 ) T - T1
= 1- = 1- 4
Cv (T3 - T2 ) T3 - T2

ÊT ˆ
T1 Á 4 - 1˜
Ë T1 ¯
= 1- ...(i)
ÊT ˆ
T2 Á 3 - 1˜
Ë T2 ¯
Considering the isentropic process 1–2,
g -1
T2 Êv ˆ
= Á 1˜ = r g -1 ...(ii)
T1 Ë v2 ¯
338 Thermal Engineering

v1
where r= , is called compression ratio.
v2
For the isentropic process 3– 4,
g -1 g -1
T3 Êv ˆ Êv ˆ
= Á 4˜ =Á 1˜ = r g -1 ...(iii)
T4 Ë v3 ¯ Ë v2 ¯

Equating Eqs. (ii) and (iii), we get


T2 T T4 T
= 3 or = 3
T1 T4 T1 T2
Using in Eq. (i), we get
T 1 1
hOtto = 1 - 1 = 1 - = 1 – g –1 ...(11.10)
T2 T 2 (r)
T1
For air, the ratio of specific heats g (gamma) is
assumed to be constant, and thus the efficiency of
the Otto cycle given by Eq. (11.10) is the function
of compression ratio only. The efficiency of the
Otto cycle increases with increase in compression Diesel cycle
ratio.
Process 3–4 Isentropic expansion, and
Process 4–1 Reversible constant volume heat re-
jection.
The Diesel cycle is an ideal cycle for reciprocating
Analysis
compression ignition (CI) engines. It was proposed
by Rudolph Diesel in 1892. For 1 kg of air undergoing a Diesel cycle,
In gasolene engines, a mixture of air and fuel Heat supplied qin = q2–3 = Cp (T3 – T2)
is compressed during the compression stroke, and Heat rejected qout = q4 –1 = Cv (T4 – T1)
thus the compression ratio is limited by engine The efficiency of the cycle
knock (auto-ignition of charge). In a Diesel en-
gine, only air is compressed during the compres- h = 1 - qout = 1 - Cv (T4 - T1 )
qin Cp (T3 - T2 )
sion stroke, thus eliminating the possibility of auto-
ignition. Therefore, Diesel engines are designed to Ê T - T1 ˆ
hDiesel = 1 - Á 4 ...(11.11)
operate at a very high compression ratio, typically Ë g (T3 - T2 ) ˜¯
between 12 and 24. The efficiency of the Diesel cycle may be ex-
In the air-standard Diesel cycle (shown in pressed in the following terms:
Fig. 11.10) the combustion process is approximated
as a constant-pressure, heat-addition process. The Compression ratio
remaining three processes are same for both Otto V1 v1
and Diesel cycles. r = =
V2 v2
Process 1–2 Isentropic compression, Volume before compression
=
Process 2–3 Reversible constant pressure heat Volume after compression
addition,
Gas Power Cycles 339

V3 v3
r= =
V2 v2 Since the cut-off ratio r is always greater than unity
Volume after heat supply then the quantity
=
Volume after compression
1 Ê r g - 1ˆ
is also greater than unity. Therefore,
g ÁË r - 1 ˜¯
V4 v4 the efficiency of the Diesel cycle is less than that
re = =
V3 v3 of the Otto cycle for the same compression ratio.
Volume after expansion However, since Diesel engines employ much higher
=
Volume before expansion compression ratio, thus their thermal efficiency is
higher.
It can be proved that
As cut-off ratio decreases, (Fig. 11.13) the
Êv v v ˆ efficiency of the Diesel cycle increases. For a
r = re r as Á 4 ¥ 3 = 1 = r ˜ ...(11.12)
Ë v3 v2 v2 ¯ cut-off ratio r ª 1, the quantity in the bracket of
Now for the isentropic process 1–2, Eq. (11.13) approaches unity and the efficiency of
g -1 Otto and Diesel cycles becomes identical.
T2 Êv ˆ
= Á 1˜ = r g –1
T1 Ë v2 ¯
or T2 = T1 rg –1 ...(i)
For the constant-pressure heat-addition process
2–3,
T3 v
= 3 =r
T2 v2
or T3 = rT2 = r r g –1 T1 ...(ii)
For the isentropic process 3–4
g -1
T4 Êv ˆ
= Á 3˜
T3 Ë v4 ¯
g -1 g -1
Êv ˆ Êv v ˆ
= Á 3˜ =Á 3 ¥ 2˜ Diesel cycle
Ë v1 ¯ Ë v2 v1 ¯
g -1 g -1
Ê rˆ Ê rˆ
or T4 = Á ˜ T3 = Á ˜ r rg -1 T1
Ë r¯ Ë r¯
In the Otto cycle, the combustion is assumed at
= r g T1 ...(iii) constant volume, while in the Diesel cycle, the
Using the values of T2, T3 and T4 from Eqs. (i), combustion is at constant pressure. In actual Diesel
(ii) and (iii), respectively in Eq. (11.11), we get engine, the fuel injection starts before the end of
compression stroke and thus a part of heat is added
(r g - 1)T1
hDiesel = 1 - at constant volume and rest at constant pressure as
g ( r r g -1 - r g -1 )T1 shown in Fig. 11.12. Such cycle is referred as air
standard dual cycle or mixed cycle or limited pres-
1 È rg - 1 ˘
= 1- Í
g -1 ˙ ...(11.13) sure cycle.
r ÍÎ g ( r - 1) ˙˚
340 Thermal Engineering

p For constant pressure process 3– 4,


3 4
pvg = C v4 v
2
qin = 3
T4 T3
v
5 or T4 = 4 T3 = rT3
v3
qout
1
v
= rrp r g –1 T1 …(vii)
v2 = v3 v4 v1 = v5
(a) p–v diagram
For isentropic expansion process 4 –5
g -1 g -1 g -1
T T5 Ê v4 ˆ Êv ˆ Êv v v ˆ
4 = =Á 4˜ =Á 4 ¥ 3 ¥ 2˜
T4 ÁË v5 ˜¯
T4
p=
C Ë v1 ¯ Ë v3 v2 v1 ¯
T3 3 g -1
C Ê rˆ
T5 v= = Á ˜ (\ v3 = v2)
T2 C 5 Ë r¯
2 v=
g -1
T1 Ê rˆ
1 or T5 = Á ˜ rrp r g –1 T1 = r g rp T1 …(viii)
s Ë r¯
(b) T–s diagram
Using the values for T2, T3, T4 and T5 in Eq.
(11.14)
T1 ( r g rp - 1)
Heat supplied, qin = Cv (T3 – T2) + Cp(T4 – T3) h Dual = 1 –
Heat rejected, qout = Cv (T5 – T1) T1 ( rp r g -1 - r g -1 ) + g T1 ( r rp r g -1 - rp r g -1)
q È rp r g - 1 ˘
Thermal efficiency h= 1- out = 1- 1 Í ˙ …(11.15)
qin g -1
r ÍÎ ( rp - 1) + g rp ( r - 1) ˙˚
Cv (T5 - T1 )
= 1- Therefore, the efficiency of Dual cycle given by
Cv (T3 - T2 ) + C p (T4 - T3 )
Eq. (11.15) is function of r, r, rp and g. When
(T5 - T1 )
= 1- …(11.14) (i) rp = 1, then hDual = rDiesel
(T3 - T2 ) + g (T4 - T3 )
(ii) r = 1, rp = 1, hDual = hOtto
Considering the following ratio to express the (iii) For same compression ratio and cut-off ratio
efficiency of Dual cycle as pressure rp increases, thermal efficiency
v of Dual cycle increases.
Compression ratio r= 1 …(i)
v2
v Hence, modern Diesel engines are designed to
Cut-off ratio, r= 4 …(ii) operate more closely to Dual cycle.
v3
p
Pressure ratio, rp = 3 …(iii)
p2
v
Expansion ratio, re = 5 …(iv)
v4 The three cycles can be compared on the basis of
For isentropic compression process 1–2; either the same compression ratio or the same max-
T2 = T1 r g –1 …(v) imum pressure and temperature.
For constant volume process 2–3,
T3 p
= 3 = rp
T2 p2 Figure 11.13 shows the comparison of three cycles
or T3 = rp r g –1 T1 …(vi) on p–v and T–s diagrams.
Gas Power Cycles 341

p
3

6 7
g
pv = C
2 5

1
v
(a) p–v diagram

T 3
C

7
=
v

6 5
C
p=
2
4
C
v=

1
The peak pressure (p3 = p5), peak temperature T5
s and heat rejection (area under the curve 4–1 on the
(b) T–s diagram T–s diagram) are same for the three cycles.
The heat input for the Diesel cycle is the area un-
same compression ratio der the curve 3–5, for Dual cycle, area under 6–7–5
Here, and for the Otto cycle, it is the area under the curve
Cycle 1–2–3– 4 –1 is Otto c ycle 2–5 on the T–s diagram. It is evident that the heat
supply in a Diesel cycle is more, and hence, it is
Cycle 1–2–5– 4 –1 is Diesel c ycle
the most efficient. The Otto cycle receives the least
Cycle 1–2–6–7–5– 4–1 is Dual cycle
heat supply, and thus is the least efficient for same
It is evident from p–v and T–s diagrams, that
maximum pressure and temperature. Therefore,
the heat-rejection process 4–1 for Otto, Diesel and
hDiesel > hDual > hOtto
Dual cycles is same, but heat supply (area under
2–3 on the T–s diagram) and work done (area un-
der 3–4 on the p–v diagram) are highest for Otto
Example 11.1 Calculate the ideal air-standard cycle
cycle. Thus, it is the most efficient cycle. Heat sup-
efficiency of a petrol engine operating on Otto cycle. The
ply (process 2–5) and work done (process 2–5– 4)
cylinder bore is 50 mm, a stroke is of 75 mm and the
for Diesel cycle are least, and thus it is the least clearance volume is of 21.3 cm3.
efficient among the three cycles. Therefore,
hOtto > hDual > hDiesel Solution
Given An air-standard Otto cycle engine
d = 50 mm = 5 cm
L = 75 mm 7.5 cm
Figure 11.14 shows the Otto cycle and Diesel cycle Vc = 21.3 cm3
on p–v and T–s diagrams. To find The thermal efficiency of the cycle.
Cycle 1–2–5– 4 –1 is Otto c ycle Assumptions
Cycle 1–2–3–5– 4 –1 is Diesel c ycle (i) The working substance is air, an ideal gas.
Cycle 1–2–6–7– 5 – 4–1 is Dual c ycle (ii) Cold air standard assumptions with g = 1.4.
342 Thermal Engineering

Analysis The swept volume of the cylinder


p
Vs = d 2L
4
p
= ¥ (5 cm)2 ¥ (7.5 cm)
4
= 147.26 cm3
Total volume of the cylinder
V1 = Vs + Vc
= 147.26 + 21.3 = 168.56 cm3
The compression ratio of the engine
V1 168.56 cm3
r = = = 7.913
Vc 21.3 cm3 Example 11.3 The pressure and temperature of air at
the beginning of compression in an Otto cycle is 103 kPa
The thermal efficiency of the air-standard Otto cycle
and 27°C, respectively. The heat added per kg of air is
1
hOtto = 1 - g -1 1850 kJ. The compression ratio is 8. Determine maxi-
r mum temperature, maximum pressure, thermal efficiency.
1 1
= 1- =1-
(7.913)1.4 -1 2.287 Solution
= 0.56.28 or 56.28% Given An engine operates on Otto cycle with
p1 = 103 kPa,
Example 11.2 In an engine working on an ideal Otto T1 = 27°C = 300 K
cycle, the temperatures at the beginning and at the end of
Heat supplied, qin = 1850 kJ/kg
compression are 27°C and 327°C. Find the compression
Compression ratio r =8
ratio and air-standard efficiency of the engine.
To find
Solution (i) Maximum temperature in the cycle.
Given An engine working on ideal cycle with (ii) Maximum pressure in the cycle.
T1 = 27°C = 300 K (iii) Thermal efficiency of the cycle.
T2 = 327°C = 600 K Assumptions
To find (i) Constant specific heat of air, Cv = 0.718 kJ/kg K.
(i) Compression ratio of engine. (ii) Ratio of two specific heats, g = 1.4 and R = 0.287
(ii) Thermal efficiency of the engine. kJ/kg ◊ K.
(iii) 1 kg air in the cycle.
Assumptions
(i) Working substance is air. Analysis
(ii) Constant specific heats and its ratio as g = 1.4. (i) The temperature after isentropic compression
T2 = T1 r g –1 = 300 ¥ 81.4 –1 = 689.2 K
Analysis
Temperature of air after heat addition at constant
(i) The compression ratio is expressed as
1 1
volume
V Ê T ˆ g -1 Ê 600 ˆ 1.4 -1 qin = Cv (T3 – T2)
r = 1 =Á 2˜ = Á = ( 2) 2.5
V2 Ë T1 ¯ Ë 300 ˜¯ 1850 = 0.718 ¥ (T3 – 689.2)
or T3 = 3265.8 K = 2992.8°C
= 5.65
The maximum temperature in the cycle is
(ii) Thermal efficiency of Otto cycle
2992.8°C
1 1
hOtto = 1 – g -1 = 1 - = 0.5
r (5.65)1.4 -1
= 50%
Gas Power Cycles 343

(ii) Cold air assumptions, i.e.,


Cp = 1.005 kJ/kg ◊ K
Cv = 0.718 kJ/kg ◊ K
g = 1.4
R = 0.287 kJ/kg ◊ K.
Analysis The swept volume of all cylinders
Vs = (p /4) d 2Lk
= (p/4) ¥ (0.25 m)2 ¥ (0.375 m) ¥ 4
= 0.0736 m3
The total volume of the cylinder
(ii) The pressure after isentropic compression
g V1 = Vs + Vc = 0.0736 + 0.01052
Êv ˆ
p2 = p1 Á 1 ˜ = 103 ¥ (8)1.4
= 1893 kPa = 0.08412 m3
Ëv ¯
2
The compression ratio
The pressure after constant-volume heat addition
V1 0.08412
p 1893 r = = =8
p3 = 2 ¥ T3 = ¥ 3265.8 Vc 0.01052
T2 689.2
= 8970.3 kPa » 89.7 bar (i) Air standard efficiency
The maximum pressure in the cycle is 89.7 bar. 1 1
hOtto = 1 - g -1
=1- 1.4 -1
(iii) Thermal efficiency of the Otto cycle r (8)
1 1 = 0.565 or 56.5%
hOtto = 1 - g -1 = 1 - 1.4 -1 = 0.564
r 8 (ii) Temperature after isentropic compression
= 56.4% T2 = T1 r g –1 = 320 ¥ (8)1.4 –1
= 735.17 K.
Example 11.4 A four-stroke, four-cylinder petrol en-
Pressure after isentropic compression
gine of 250-mm bore and 375-mm stroke works on Otto
cycle. The clearance volume is 0.01052 m3. The initial p2 = p1rg = (1 bar) ¥ (8)1.4
pressure and temperature are 1 bar and 47°C. If the = 18.34 bar
maximum pressure is limited to 25 bar, find the following: Temperature after heat addition at constant
(a) The air standard efficiency of the cycle, volume
(b) The mean effective pressure. p3 Ê 25 bar ˆ
T3 = ¥ T2 = Á ¥ (735.17 K)
p2 Ë 18.34 bar ˜¯
Solution = 1000 K
Given Four-stroke petrol engine on Otto cycle The mass of air in the cylinders
d = 250 mm = 0.25 m p1V1 (100 kPa) ¥ (0.08412 m3)
L = 375 mm = 0.375 m m = =
RT1 (0.287 kJ/kg ◊ K) ¥ (320 K)
Vc = 0.01052 m3
= 0.0922 kg
p1 = 1 bar T1 = 47°C = 320 K
The amount of heat added at constant volume
p3 = 25 bar k =4
Qin = m Cv (T3 – T2)
To find = 0.0922 ¥ 0.718 ¥ (1000 – 735.17)
(i) Air standard efficiency, and = 17.53 kJ
(ii) The mean effective pressure. The work done per cycle
Wnet = hOtto Qin = 0.565 ¥ 17.53
Assumptions
= 9.9 kJ.
(i) Air as working fluid in the engine.
344 Thermal Engineering

The mean effective pressure of the cycle Process 2–3: Constant-volume heat addition
Wnet 9.9 kJ p 30
pm = = T3 = 3 T2 = ¥ 593.28
Vs 0.0736 m2 p2 10.877
= 134.6 kN/m2 = 1.346 bar = 1636.33 K or 1363.33°C
p3 = pmax = 30 bar
Example 11.5 In an SI engine working on the ideal Process 3–4: Isentropic expansion
Otto cycle, the compression ratio is 5.5. The pressure T3 1636.33
T4 = =
and temperature at the beginning of compression are 1 ( r )g -1 (5.5)1.4 -1
bar and 27°C , respectively. The peak pressure is 30 bar.
= 827.42 K or 554.42°C
Determine the pressure, temperature at the salient points,
g 1.4
the air standard efficiency, and mean effective pressure. Ê 1ˆ Ê 1 ˆ
p4 = p3 Á ˜ = 30 ¥ Á = 2.758 bar
Assume ratio of specific heat to be 1.4 for air. Ë r¯ Ë 5.5 ˜¯
Solution (ii) The air-standard efficiency of an Otto Cycle
1 1
Given An air-standard Otto cycle hOtto = 1 - g -1 = 1 -
r = 5.5 p1 = 1 bar = 100 kPa r (5.5)1.4 -1
T1 = 27°C = 300 K p2 = 30 bar = 0.494
or 49.4%
g = 1.4 The heat supplied per kg of air
qin = Cv (T3 – T2)
To find
= ¥ (1636.33 – 593.28)
0.717
(i) The pressure, temperature and volume at the = 747.86 kJ/kg
salient points in the cycle.
The net work done per kg of air
(ii) Air standard efficiency.
wnet = hOtto qin = 0.494 ¥ 747.86
(iii) Mean effective pressure.
= 369.44 kJ/kg
Assumptions Cp = 1.005 kJ/kg ◊ K, Cv = 0.717 kJ/kg ◊ K The total volume of the cylinder at beginning of
and R = 0.287 kJ/kg ◊ K compression
RT1 0.287 ¥ 300
v1 = = = 0.861 m3/kg
p1 100
wnet wnet
pm = =
v1 - v2 Ê 1ˆ
v1 Á1 - ˜
Ë r¯
369.44
=
Ê 1 ˆ
0.861 ¥ Á1 -
Ë 5.5 ¯˜
= 524.44 kPa or 5.24 bar

Example 11.6 An engine working on Otto cycle has a


Analysis total volume of 0.45 m3, pressure 1 bar and temperature
27°C at the beginning of the compression stroke. At the
(i) Pressure and temperature at salient points
end of the compression stroke, the pressure is 11 bar, and
Isentropic compression process 1–2
210 kJ of heat is added at constant volume. Calculate
Temperature T2 = T1(r)g –1 = 300 ¥ (5.5)1.4–1
(a) the pressure, temperature and volume at the sa-
= 593.28 K or 320.28°C lient points in the cycle,
Pressure p2 = p1 r g = 1 ¥ (5.5)1.4 (b) percentage clearance volume,
= 10.877 bar (c) net work done per cycle,
Gas Power Cycles 345

(d) the ideal power developed by the engine if the (i) Pressure, temperature and volume at salient
number of working cycles per minute is 210 points
Assume Cp = 1.005 kJ/kg ◊ K Isentropic compression process 1–2
and Cv = 0.717 kJ/kg ◊ K. 1 1
Ê p ˆg Ê 1 ˆ 1.4
V2 = V1 Á 1 ˜ = 0.45 ¥ Á ˜ = 0.081 m3
Solution Ë p2 ¯ Ë 11¯
Given An air-standard Otto cycle The compression ratio
V1 = 0.45 m3 p1 = 1 bar = 100 kPa V 0.45
r = 1 = = 5.55
T1 = 27°C = 300 K, Qin = 210 kJ V2 0.081
p2 = 11 bar T2 = T1 (r)g –1 = 300 ¥ (5.55)1.4 –1
No. of working cycles = 210 per min. = 595.68 K or 322.68°C
Cp = 1.005 kJ/kg ◊ K Process 2–3: Constant volume heat addition
Cv = 0.717 kJ/kg ◊ K Qin = m Cv (T3 – T2)
To find Rearranging and using numerical value
210
(i) The pressure, temperature and volume at the T3 = 595.68 +
salient points in the cycle, 0.52 ¥ 0.717
(ii) Percentage clearance volume, = 1159 K or 886°C
(iii) Net work done per cycle, For constant-volume process
(iv) The ideal power developed by the engine if the V3 = V2 = 0.081 m3
p3 p
number of working cycles per minute is 210. and = 2
T3 T2
Analysis The ratio of specific heats 11
C p 1.005 or p3 = ¥ 1159 = 21.4 bar
g = = = 1.4 595.68
Cv 0.717 Process 3–4: Isentropic expansion
Specific gas constant T3 1159
T4 = = = 584 K
R = Cp – Cv = 1.005 – 0.717 ( r )g -1 (5.55)1.4 -1
= 0.288 kJ/kg ◊ K g g 1.4
ÊV ˆ Ê 1ˆ Ê 1 ˆ
The mass of the air present in the cycle p4 = p3 Á 3 ˜ = p3 Á ˜ = 21.4 ¥ Á
p1V1 (100 kPa ) ¥ (0.45 m3 ) Ë V4 ¯ Ë r¯ Ë 5.55 ˜¯
m = =
RT1 (0.288 kJ/kg ◊ K) ¥ (300 K ) = 1.94 bar
V4 = V1 = 0.45 m3
= 0.52 kg/cycle.
(ii) Percentage clearance volume
Swept volume, Vs = V1 – V2 = 0.45 – 0.081
= 0.369 m3
Clearance volume, Vc = V2 = 0.081 m3

Percentage clearance volume,


Vc 0.081
c= = = 0.219 or 2.19%
Vs 0.369
(iii) Net work done per cycle
The thermal efficiency of an Otto Cycle
1 1
hOtto = 1 - g -1 = 1 - = 0.496
r (5.55)1.4 - 1
346 Thermal Engineering

The network done per cycle (g – 1) T1 rg –2 = (g – 1) T3 r–g


Wnet = hOtto Qin = 0.496 ¥ 210 = 104.2 kJ T
or r 2g –2 = 3
(iv) Ideal power developed T1
Power = Workdone per cycle 1

¥ No. of working cycles per second. Ê T ˆ 2(g -1)


or r = Á 3˜ ...(ii)
ËT ¯ 1
Ê 210 ˆ
= 104.2 ¥ Á = 364.68 kW
Ë 60 ˜¯ Substituting the value of r in Eq. (i)
g -1
Ê T ˆ 2(g -1)
Example 11.7 (a) An engine working on an air T2 = T1 Á 3 ˜
standard Otto cycle operates between upper and lower ËT ¯ 1
temperature limits T3 and T1. Prove that for maximum 1 1
output from the cycle, the intermediate temperature is = T1 T3 2 = T1 T3
2
given by
Similarly,
T2 = T 4 = T1 T3
T3
(b) If the above engine has an upper temperature T4 = g -1
= T1 T3 Proved.
of 1450 K and a lower temperature of 310 K, calculate Ê T3 ˆ 2(g -1)
the maximum power developed by the engine, if air ÁË T ˜¯
1
circulation per minute is 0.38 kg.
(b) Given
Solution T3 = 1450 K,
(a) Given An ideal Otto cycle with T1 = 310 K, m = 0.38 kg/min
Lower temperature = T1 To find Maximum power of the cycle
Upper temperature = T3
Analysis The net work output per kg of the cycle
To prove For maximum work output from the cycle wnet = Cv (T3 – T2) – Cv (T4 – T1)
T2 = T4 = T1 T3 using T2 = T4 = T1 T3 = 310 ¥ 1450 = 670.44 K
Analysis The heat supplied per kg of air during the Cv = 0.717 kJ/kg ◊ K
process 2–3 is wnet = 0.717 ¥ (1450 – 670.44)
qin = Cv (T3 – T2) – 0.717 ¥ (670.44 – 310)
Heat rejected per kg of air during the process 4 –1 is = 300.5 kJ/kg
qout = Cv (T4 – T1) The power developed by the cycle
Net work done in the cycle P = mwnet
wnet = Sq = qin – qout
Ê 0.38 ˆ
= Cv (T3 – T2) – Cv (T4 – T1) = Á kg/s˜ ¥ (300.5 kJ/kg) = 1.90 kW
Ë 60 ¯
T3
where T2 = T1 r g –1 and T4 = ...(i) Example 11.8 In an air-standard Otto cycle the pres-
( r )g -1
Using, we get sure ratio during the compression is 15. The tempera-
ture of air at the beginning of compression is 37°C and
È T ˘
wnet = Cv ÍT3 - T1 r g -1 - g 3-1 + T1 ˙ maximum temperature attained in the cycle is 1950°C.
Î r ˚ Determine
Differentiating with respect to r and equating to zero (a) Compression ratio,
dwnet (b) Thermal efficiency of the cycle.
= Cv [0 – (g – 1)T1 r g –2
dr (c) Work done.
– T3 (1 – g) r1–g –1 + 0] Take g = 1.4, Cv = 0.717 kJ/kg ◊ K.
=0
Gas Power Cycles 347

Solution Solution
Given An air-standard Otto cycle with (a) Given An air standard Otto cycle operating for
p2 = 15p1 T1 = 37°C = 310 K maximum work output between minimum temperature
T3 = 1950°C = 2223 K g = 1.4 T1 and maximum temperature T3.

To find
(i) Compression ratio,
(ii) Thermal efficiency of the cycle
(iii) Work done.
Analysis
(i) Compression ratio 1
V1 Ê p2 ˆ g
r = = Á ˜
V2 Ë p1 ¯
1
= (15)1.4 = 6.919
(ii) Thermal efficiency of the cycle To find To prove compression ratio as
1
1 1 Ê T ˆ 2(g -1)
hOtto = 1 – = 1- = 0.5387 r = Á 3˜
(6.919)1.4 -1
g -1
r
ËT ¯1
= 53.8%
(iii) Work done Analysis The work done per kg of air in an Otto cycle
T2 = T1 r g –1 = 310 ¥ (6.919)0.4 = 672 K Wnet = qin – qout
Heat supplied per kg of air = Cv (T3 – T2) – Cv (T4 – T1)
qs = Cv (T3 – T2) = Cv (T3 – T2 – T4 + T1)
= 0.717 ¥ (2223 – 672) = 1112 kJ/kg ÊT T T ˆ
= Cv T1 Á 3 - 2 - 4 + 1˜
Work done = hOtto ¥ qs Ë T1 T1 T1 ¯
= 0.538 ¥ 1112 = 598.3 kJ/kg T2
Here, = r g –1
T1
Example 11.9 (a) Prove that the compression ratio T
corresponding to the maximum work in the Otto cycle T4 = g 3-1
r
between upper and lower limits of absolute temperature
For given T1 and T3, the work done will be maximum,
T3 and T1, respectively is given by
1 when
Ê T ˆ 2(g -1) dwnet
r = Á 3˜ =0
ËT ¯ 1 dr
(b) Calculate the air standard efficiency of cycle, when d È Ï T3 g -1 T3 1 ¸˘
it develops maximum work and operates between 300 K 0 = ÍCv T1 Ì - r - ◊ g -1 + 1˝˙
dr ÍÎ Ó T1 T1 r ˛˙˚
and 1200 K as minimum and maximum temperatures,
respectively with working fluid as air. Ï T ¸
= Cv T1 Ì0 - (g - 1) r g - 2 - 3 (1 - g ) r -g + 0 ˝
(c) Calculate the change in efficiency, if helium is Ó T1 ˛
used in Otto cycle? The cycle operates between same
T
temperature limits for maximum work output. or r g –2 = 3 r –g
T1
For air g = 1.4
T3
For helium take Cp = 5.22 kJ/kg ◊ K, Cv = 3.13 kJ/kg ◊ K or r 2(g –1) =
T1
348 Thermal Engineering

1 250 mm, a stroke of 375 mm and a clearance volume of


Ê T ˆ 2 ( g - 1) 1500 cc, with fuel cut off occurring at 5% of the stroke.
or r = Á 3˜ Proved
ËT ¯ 1 Assume g = 1.4.

(b) Given For air T1 = 300 K, Solution


T3 = 1200 K,
Given Diesel engine with
g = 1.4
Bore d = 250 mm = 25 cm
To find Air standard efficiency of Otto cycle. Stroke L = 375 mm = 37.5 cm
Clearance volume Vc = V2 = 1500 cc
Analysis The compression ratio for maximum work
1 g = 1.4
Ê T ˆ 2 (g -1) Fuel cut–off = 5% of stroke
r = Á 3˜
ËT ¯ 1 To find Air-standard efficiency of Diesel cycle.
1
Ê 1200 ˆ 2 (1.4 -1)
= Á = 5.656
Ë 300 ˜¯
The air standard efficiency of Otto cycle
1 1
hOtto = 1 – g -1
=1–
r (5.656)0.4
= 0.5 = 50%
(c) Given For helium
T1 = 300 K, T3 = 1200 K,
Cp = 5.22 kJ/kg ◊ K Cv = 3.13 kJ/kg ◊ K
To find Air standard efficiency of Otto cycle.
Analysis The ratio of specific heats Analysis The piston displacement (swept volume)
Vs = (p /4) d2L
Cp 5.22
g = =
= 1.667 = (p/4) ¥ (25)2 ¥ (37.5) = 18408 cc
Cv 3.13
Total volume V1 = Vs + Vc = 18408 + 1500
The compression ratio for maximum work
= 19908 cc
1
Ê T ˆ 2 (g -1) V1 19908
r = Á 3˜ Compression ratio,r = = = 13.27
ËT ¯ 1 V2 1500
1 Since V3 = V2 + 0.05Vs
Ê 1200 ˆ 2 (1.667 -1) = 1500 + 0.05 ¥ 18408
= Á = 2.823
Ë 300 ˜¯ Cut-off volume V3 = 2420.4
The air standard efficiency of Otto cycle V 2420.4
Cut-off ratio r = 3 = = 1.613
1 1 V2 1500
hOtto = 1 - g -1 = 1 –
r ( 2.823)0.667 The air standard efficiency of the cycle
= 0.5 = 50%
1 È rg -1 ˘
There is no change in air standard efficiency of Otto hDiesel = 1 - Í ˙
cycle by changing the fluid. r g -1 ÍÎ g ( r -1) ˙˚

Diesel Cycle 1 È (1.613)1.4 - 1 ˘


= 1- ¥ Í ˙
Example 11.10 Determine the air-standard efficien- (13.27)1.4 - 1 ÍÎ1.4 ¥ (1.613 - 1) ˙˚
cy of the Diesel engine having a cylinder with a bore of = 0.6052 = 60.52%
Gas Power Cycles 349

Example 11.11 A Diesel engine has a compression


ratio of 18 and cut off takes place at 5% of the stroke.
Calculate the air-standard efficiency. Take g = 1.4.

Solution
Given An air-standard Diesel cycle
r = 18
Cut off = 0.05 of stroke (Vs)
g = 1.4
To find Air-standard efficiency of the Diesel cycle.
Analysis The compression ratio is given as Analysis The temperature after isentropic compression
V1 T2 = T1 (r)g –1
r = or V1 = rV2 = 300 ¥ (14)1.4–1 = 862.13 K.
V2
\ V1 = 18V2 (i) The cut-off ratio after constant-pressure heat
The swept volume addition
V T 2500
Vs = V1 – V2 = 18V2 – V2 = 17V2 r = 3 = 3 =
The cut-off volume V2 T2 862.13
= 2.9
V3 = V2 + 0.05Vs
(ii) The thermal efficiency of an air-standard Diesel
= V2 + 0.05 ¥ 17V2 = 1.85V2
Cycle.
V1
The cut-off ratio, r = = 1.85 È rg - 1 ˘
V2 1
hDiesel = 1 - -1
Í ˙
The air standard Diesel cycle efficiency, Eq. (11.13) ( r )g ÍÎ g ( r - 1) ˙˚
1 È rg - 1 ˘ 1 È 2.91.4 - 1 ˘
hDiesel = 1 - Í ˙ = 1- ¥Í ˙
r g -1 ÍÎ g ( r -1) ˙˚ (14) 1.4 - 1
ÍÎ1.4 ¥ ( 2.9 - 1) ˙˚
1 È (1.85)1.4 - 1 ˘ = 0.55 or 55%
= 1- 1.4 - 1
¥Í ˙
(18) ÍÎ1.4 ¥ (1.8 5- 1) ˙˚
Example 11.13 An engine works on a Diesel cycle
= 0.6387 or 63.877% with an inlet pressure and temperature of 1 bar and
17°C. The pressure at the end of the adiabatic compres-
Example 11.12 An air-standard Diesel cycle has a sion is 35 bar. The ratio of expansion, i.e., after constant-
compression ratio of 14. The pressure at the beginning pressure heat addition is 5. Calculate the heat addition,
of the compression stroke is 1 bar and the temperature heat rejection and efficiency of the cycle.
is 300 K. The maximum cycle temperature is 2500 K. Assume g = 1.4
Determine the cut-off ratio and thermal efficiency. Cp = 1.004 kJ/kg ◊ K
Cv = 0.717 kJ/kg ◊ K
Solution
Given An air-standard Diesel Cycle Solution
T1 = 300 K, p1 = 1 bar Given An engine works on the Diesel cycle
r = 14 T3 = 2500 K p1 = 1 bar T1 = 17°C = 290 K
To find p3 = p2 = 35 bar re = 5
(i) Cut-off ratio g = 1.4 Cp = 1.004 kJ/kg ◊ K.
(ii) Air-standard efficiency of the Diesel cycle Cv = 0.717 kJ/kg ◊ K
350 Thermal Engineering

To find Example 11.14 Calculate the percentage loss in air


(i) Heat addition in the cycle, standard efficiency of a Diesel engine with compression
(ii) Heat rejection in the cycle, and ratio 14 and if fuel cut-off is delayed from 5% to 8%.
(iii) Efficiency of the cycle.
Solution
Given A Diesel engine working on an air standard
Diesel cycle.
v3 - v2
r = 14 and = 0.05 and 0.08
vs
To find Percentage loss in air standard Diesel efficiency.
Assumption
(i) The working fluid in piston cylinder assembly as
a closed system.
(ii) Compression and expansion are isentropic.
(iii) Working substance is air as an ideal gas with
Analysis The temperature after adiabatic compression g = 1.4.
g -1 1.4 -1 (iv) Heat addition at constant pressure.
Êp ˆ g Ê 35 ˆ 1.4
T2 = T1 Á 2 ˜ = 290 ¥ Á ˜ = 801 K Analysis When fuel cut-off takes place at 5% of the
Ë p1 ¯ Ë 1¯
swept volume
The pressure after heat addition
v3 - v2 v - v2
p3 = p2 = 35 bar = 3 = 0.05
vs v1 - v2
The pressure after adiabatic expansion
p 35 bar v3
p4 = g3 = = 3.677 bar -1
v2 r -1
re (5)1.4 or = = 0.05
For the constant-volume process 4 – 1
v1
-1 r -1
v2
p4 3.677
T4 = T1 = ¥ 290 = 1066.37 K or r = 1 + 0.05 ¥ (14 – 1) = 1.65
p1 1
For the isentropic expansion process 3–4 The air standard Diesel cycle efficiency
T3 = T4 (re) g –1 = 1066.37 ¥ (5)1.4 –1 = 2030 K
1 Ê rg - 1 ˆ
(i) The heat addition per kg of air in the cycle hDiesel = 1 - ¥Á ˜
g -1
qin = Cp (T3 – T2) r Ë g ( r - 1) ¯
= 1.004 ¥ (2030 – 801) Ê 1.651.4 - 1 ˆ
1
= 1233.9 kJ/kg = 1- ¥Á ˜
(ii) Heat rejection per kg of air in the cycle
140.4 Ë 1.4 ¥ (1.6 5- 1) ¯
qout = Cv (T4 – T1) = 0.611 or 61.1%
= 0.717 ¥ (1066.37 – 290) When fuel cut-off takes place at 8% of the swept
= 556.67 kJ/kg volume
(iii) Efficiency of the cycle r = 1 + 0.08(14 – 1) = 2.04
Cycle and air-standard Diesel cycle efficiency is
Net work done qin - qout
hCycle = =
Heat sypplied qin 1 Ê 2.041.4 - 1 ˆ
hDiesel = 1 - ¥Á ˜
q
= 1 - out = 1 -
556.67 140.4 Ë 1.4 ¥ ( 2.04 - 1) ¯
qin 1233.9 = 0.590 = 59%
= 0.588 or 54.88% Percentage change = 61.1% – 59% = 2.1%
Gas Power Cycles 351

Example 11.15 An engine operates on air standard


Diesel cycle. The pressure and temperatures at the begin-
ning of compression are 100 kPa and 27°C. The compres-
sion ratio is 18. The heat added per kg of air is 1850 kJ.
Determine maximum pressure, maximum temperature,
thermal efficiency, network done and mean effective pres-
sure of the cycle. Assume g = 1.4 and Cp = 1.005 kJ/
kg ◊ K.

Solution
Given An engine operates on air standard Diesel cycle
with
p1 = 100 kPa, T1 = 27°C = 300 K,
r = 18, qin = 1850 kJ/kg,
g = 1.4 Cp = 1.005 kJ/kg ◊ K
To find
(i) Maximum pressure in the cycle,
(ii) Maximum temperature in the cycle,
(iii) Thermal efficiency,
(iv) The net work done of the cycle, and
(v) The mean effective pressure.
Schematic p–v and T–s diagrams are shown in Fig.
v3 T 2794
11.23. or r = = 3 = = 2.93
v2 T2 953.5
Analysis
Using Eq. (11.13)
(i) Maximum pressure in the cycle as p2 = p3 = pmax
g 1 È rg - 1 ˘
Êv ˆ hDiesel = 1 - Í ˙
p2 = p1 Á 1 ˜ = p1 r g rg -1
ÍÎ g ( r - 1) ˙˚
Ëv ¯
2
= (100 kPa) ¥ (18)1.4 = 5720 kPa. 1 È ( 2.93)1.4 - 1 ˘
= 1- ¥Í ˙
= 57.2 bar (18)1.4 -1 ÍÎ1.4 ¥ ( 2.9 3- 1) ˙˚
(ii) Maximum temperature in the cycle, T3
= 0.592 or 58.2%
Temperature after compression
(iv) The net work done of the Diesel cycle
T2 = T1 r g –1 = 300 ¥ (18)1.4–1
wnet = hDiesel ¥ qin = 0.592 ¥ 1850
= 953.3 K.
Since 1850 kJ heat is added per kg of air at = 1095 kJ/kg
constant pressure, thus, (v) The mean effective pressure
qin = Cp (T3 – T2) The specific heat at constant volume
or 1850 = 1.005 ¥ (T3 – 953.3) Cp 1.005
Cv = = = 0.718 kJ/kg ◊ K
1850 g 1.4
or T3 = + 953.3
1.005 The gas constant for air,
= 2794 K = 2511°C R = Cp – Cv = 1.005 – 0.718
Thus, the maximum temperature is 2521°C = 0.287 kJ/kg ◊ K
(iii) Thermal efficiency of Diesel cycle Initial specific volume of air,
For constant pressure process. RT1 0.287 ¥ 300
v2 v v1 = =
= 3 p1 100
T2 T3 3
= 0.861 m /kg
352 Thermal Engineering

The volume after compression, The cut-off ratio after constant-pressure heat addition
v1 0.861 V3 T3 2775
v2 = = = 0.478 m3/kg r = = = = 2.638
18 18 V2 T2 862.13
Swept volume, (i) The thermal efficiency of an air-standard Diesel
vs = v1 – v2 = 0.861 – 0.0478 = 0.813 m3/kg cycle
Using Eq. (11.5); È rg - 1 ˘
1
w 1095 hDiesel = 1 - Í ˙
g -1
pm = net = (r) ÍÎ g ( r - 1) ˙˚
vs 0.813
= 1346.5 kPa ª 13.46 bar 1 È ( 2.638)1.4 - 1 ˘
= 1- 1.4 -1
¥Í ˙
(14) ÍÎ1.4 ¥ ( 2.638 - 1) ˙˚
Example 11.16 In an air-standard Diesel engine cy-
cle with a compression ratio of 14, the condition of air at = 0.561 or 56.1%
the start of the compression stroke are 1 bar and 300 K. The heat addition per kg of air in the cycle
After addition of heat at constant pressure, the tempera- qin = Cp (T3 – T2)
ture rises to 2775 K. Determine the thermal efficiency of = 1.005 ¥ (2775 – 862.13)
the cycle, net work done per kg of air and the mean effec- = 1922.43 kJ/kg
tive pressure. (ii) Work done per kg of air
Given An air-standard Diesel cycle wnet = hDiesel qin = 0.561 ¥ 1922.43
T1 = 300 K, p1 = 1 bar = 1078.48 kJ/kg
r = 14 T3 = 2775 K (iii) Mean effective pressure
The total volume of the cylinder at the beginning
To find of compression
(i) Air-standard efficiency of the Diesel cycle, RT1 0.287 ¥ 300
(ii) Work done per kg of air, and v1 = =
p1 100
(iii) Mean effective pressure.
= 0.861 m3/kg
Assumptions wnet wnet
pm = =
(i) Air as working fluid in the engine. v1 - v2 Ê 1ˆ
v1 Á1 - ˜
(ii) Cold air assumptions, i.e., Ë r¯
Cp = 1.005 kJ/kg ◊ K 1078.48
=
Cv = 0.718 kJ/kg ◊ K Ê 1ˆ
0.861Á1 - ˜
g = 1.4 Ë 14 ¯
R = 0.287 kJ/kg ◊ K = 1348.94 kPa or 13.49 bar
Analysis The temperature after isentropic compression
Example 11.17 Consider an ideal Diesel cycle. At
T2 = T1 (r) g – 1 the beginning of the compression process, the cylinder
= 300 ¥ (14)1.4 –1 = 862.13 K. volume is 1500 cm3 and at the end of the heat addition
p process, it is 150 cm3. The compression ratio is 15. Air is
2 at 101 kPa and 20°C at the beginning of the compression
3 T3 = 2775 K
process. Calculate
g
pv = const. (a) Pressure at the beginning of heat rejection pro-
cess,
(b) Net work per cycle in kJ,
r = 14 4 (c) The mean effective pressure of the cycle.
T1 = 300 K
1 bar 1
v
Gas Power Cycles 353

Solution = (101 kPa ) ¥ (15)1.4


Given An ideal Diesel cycle with = 4475.58 kPa or 44.75 bar
p1 = 101 kPa T1 = 20°C = 293 K Pressure at the end of heat addition;
r = 15 V1 = 1500 cm3 p3 = p2 = pmax = 44.75 bar
V3 = 150 cm3 Temperature T3 at the end of constant-volume heat
addition 2–3;
To find
T3 = rT2 = 1.5 ¥ 865.57 = 1298.37 K.
(i) The pressure at end of heat rejection,
Temperature T4 at the end of isentropic expansion;
(ii) The net work done in kJ, and
(iii) Mean effective pressure. T4 = r g T1 = (1.5)1.4 ¥ 293 = 516.88 K.

Assumptions (i) The pressure at the end of isentropic expansion


(i) Air as working fluid in the engine. 3–4
g g g
(ii) Cold air assumptions, i.e., Êv ˆ Êv v ˆ Ê rˆ
p4 = p3 Á 3 ˜ = p2 Á 3 ¥ 2 ˜ = p2 Á ˜
Cp = 1.005 kJ/kg ◊ K Ë v4 ¯ Ë v2 v1 ¯ Ë r¯
Cv = 0.718 kJ/kg ◊ K
1.4
g = 1.4 Ê 1.5 ˆ
= 44.75 ¥ Á ˜ = 1.78 bar
R = 0.287 kJ/kg ◊ K Ë 15 ¯

Schematic p–V diagram is shown in Fig. 11.25. (ii) The net work done of the cycle
Heat supplied per kg of air
qin = Cp (T3 – T2)
= 1.005 ¥ (1298.37 – 865.57)
= 434.96 kJ/kg
Heat rejected per kg of air
qout = Cp (T4 – T1)
= 1.005 ¥ (516.88 – 293)
= 225 kJ/kg
Net work done per cycle
Wnet = m(qin – qout)
= 1.8 ¥ 10–3 ¥ (434.95 – 225)
Analysis The mass of the air inducted per cycle = 0.38 kJ
p1V1 (101 kPa ) ¥ (1500 ¥ 10 - 6 m3 ) (iii) The mean effective pressure
m = =
RT1 (0.287 kJ/kg ◊ K) ¥ (293 K ) Wnet Wnet
pm = =
= 1.80 ¥ 10–3 kg/cycle. Vs V1 - V2
The volume at the end of compression 0.38 kJ
=
V 1500 cc
V2 = 1 = = 100 cc (1500 - 100) ¥ 10 -6
r 15 = 269.95 ª 2.7 bar
V 150 cc
The cut-off ratio r = 3 = = 1.5
V2 100 cc Example 11.18 An engine working on an air stan-
Temperature after isentropic compression 1–2; dard Diesel cycle operates between minimum and maxi-
T2 = T1 r g –1 = 293 ¥ (15)1.4–1 = 865.57 K. mum temperatures of the cycle as T1 and T3, respectively.
If this engine operate for its maximum power, find the
The pressure at the end of compression;
g
expression for its compression ratio in terms of the given
Êv ˆ parameters.
p2 = p1 Á 1 ˜ = p1 r g
Ë v2 ¯
354 Thermal Engineering

Solution The net work done in the Diesel cycle is Solution


given as
Given An air standard dual cycle with
wnet = Cp (T3 – T2) – Cv (T4 – T1) ...(i)
p1 = 0.1 MPa = 100 kPa T1 = 27°C = 300 K
where T2 = T1 r g –1 v p
g -1 r = 1 = 18 rp = 3 = 1.5
T3 Ê rˆ v2 p2
and T4 = = T3 Á ˜
( re )g - 1 Ë r¯ v
r = 4 = 1.2
Substituting in Eq. (i) v3
È Ê r ˆ g -1 ˘
wnet= Cp [T3 – T1 rg –1] – Cv ÍT3 Á ˜ - T1 ˙ ...(ii) To find
Í Ë r¯ ˙ (i) Thermal efficiency, and
Î ˚
(ii) Mean effective pressure.
T
3 Assumptions
T3
C (i) Working, fluid air is modeled as an ideal gas.
p=
T4 2 (ii) Air in piston cylinder arrangement is the closed
T2 4 system.
C
v= (iii) All processes are internally reversible.
(iv) The specific heat ratio for air, g = 1.4, R = 0.287
T1
1 kJ/kg ◊ K, Cp = 1.005 kJ/kg ◊ K, Cv = 0.718 kJ/
s kg ◊ K
Schematic
p
For maximum work output of the engine, differentiat- 3 4
p3 = p 4 pvg = const.
ing Eq. (ii) with respect to r, and equating it to zero,
dwnet
= –Cp T1(g – 1) r g – 2 p2
2
dr
5
– Cv T3 (r)g – 1 (1 – g)r1–g –1 = 0
Cp -2 0.1 MPa 1
or T1(g - 1) r g = T3 (r) g –1 (g – 1)r –g v
Cv 0 v3 = v2 v4 v1

T3 rg - 1 È Cp ˘ (a) p–v diagram for Dual cycle


or r g –2+ g = Í∵ =g˙
T1 g Î C v ˚ T
T4 4
1
È T3 r g - 1 ˘ 2( g - 1)
or r = Í ˙ T3
3
ÍÎ T1 g ˙˚
T2 5
It is the required relation for compression ratio, when 2
cycle produces maximum power. 300 K
1
s
Dual Cycle
(b) T–s diagram for Dual cycle
Example 11.19 In an air standard dual cycle, the
pressure and temperature are 0.1 MPa and 27°C. The
compression ratio is 18. The pressure ratio for the con- Analysis
stant volume part of heating process is 1.5 and the vol- (i) The air-standard efficiency of a dual cycle is
ume ratio for the constant pressure part of heating is 1.2, given by
determine, (a) thermal efficiency, (b) mean effective pres- 1 È rp rg - 1 ˘
sure in MPa. hdual = 1 – g -1 Í ˙
r ÍÎ ( rp - 1) + g rp ( r - 1) ˙˚
Gas Power Cycles 355

Using numerical values, Schematic


1 È 1.4
1.5 ¥ (1.2) - 1 ˘ p
hdual = 1 - Í ˙ 3 4
1.4 -1
(18) ÍÎ (1. 5 - 1) + 1 .4 ¥ 1 .5 ¥ (1 .2 - 1) ˙˚
qin pvg = C
1 È 0.936 ˘ 2
= 1- ¥
3.178 ÍÎ 0.5 + 0.4 2˙˚
5
= 0.68 ª 68% qout
1 bar 1
(ii) Mean effective pressure r = 16
v
Net work done w h ¥ qin 0
pm = = net = dual (a) p–v diagram
Swept volume vs v1 - v2
RT1 0.287 ¥ 300 T
4
Here, v1 = = = 0.861 m3/kg C
p1 100 p=
3
v1
v2 = = 0.0478 m3/kg C qin
18 v= 5
g –1 1.4–1 qout
T2 = T 1 r = 300 ¥ (18) C
v=
= 953.3 K 330 K
1 s
T3 = T2 rp = 953.3 ¥ 1.5 = 1430 K 0
T4 = T3 r = 1430 ¥ 1.2 = 1716 K (b) T–s diagram
The heat supplied per kg of air:
qin = Cv (T3 – T2) + Cp (T4 – T3)
= 0.718 ¥ (1430 – 953.3) + (ii) Maximum temperature in the cycle,
1.005 ¥ (1716 – 1430) (iii) Thermal efficiency, and
= 630.6 kJ/kg (iv) Mean effective pressure.
0.68 ¥ 630.6 Assumption
Then pm =
0.861 - 0.0478 (i) Working substance is air.
= 5.22 kPa ª 0.527 MPa (ii) Compression and expansion are isentropic.
(iii) Heat addition and rejection are reversible.
Example 11.20 In an air standard dual cycle, the (iv) Constant specific heats, Cp = 1.005 kJ/kg ◊ K,
pressure and temperature at the beginning of compres-
Cv = 0.718 kJ/kg ◊ K, R = 0.287 kJ/kg ◊ K, and
sion are 1 bar and 57°C, respectively. The heat supplied
g = 1.4.
in the cycle is 1250 kJ/kg, two third of this being added at
constant volume and rest at constant pressure. Analysis
If the compression ratio is 16. Determine the maximum (i) The pressure and temperature after isentropic
pressure, temperature in the cycle, thermal efficiency and compression
mean effective pressure. p2 = p1 r g = (1 bar) ¥ (16)1.4 = 48.5 bar
T2 = T1 r g –1 = (330 K) ¥ (16)1.4 –1
Solution
= 1000.3 K
Given An air standard dual cycle with Heat added at constant volume,
p1 = 1 bar = 100 kPa, T1 = 57°C = 330 K 2 2
q2–3 =
qin = ¥ 1250
2 1 3 3
qin = 1250 kJ/kg, q2–3 = q in, q3– 4 = q in
3 3 = 833.33 kJ/kg.
Heat added at constant pressure,
To find
1 1250
(i) Maximum pressure in the cycle, q3–4 = qin = = 416.67 kJ/kg
3 3
356 Thermal Engineering

During process 2–3 The piston displacement volume, per kg of air


q2–3 = Cv (T3 – T2) vs = v1 – v2 = 0.9471 – 0.0592
or 833.33 = 0.718 ¥ (T3 – 1000.3) = 0.888 m3/kg
or T3 = 2161 K The mean effective pressure,
p3 p w 832.8
and = 2 pm = net = = 938 kPa
T3 T2 vs 0.888
or pressure ratio = 9.38 bar
p3 T 2161
rp = = 3 = = 2.16 Example 11.21 The pressure and temperature at the
p2 T2 1000.3 beginning of compression in an air-standard dual cycle
and p3 = 2.16 p2 = 2.16 ¥ 48.5 are 1 bar and 30°C, respectively. The compression ratio
= 104.77 bar is 9. The maximum pressure in the cylinder is limited to
It is the maximum pressure attained in the cycle. 60 bar. The heat is added during constant pressure pro-
(ii) During process 3 – 4, cess upto to 4% of the stroke. Assuming cylinder bore and
stroke as 250 mm and 300 mm, respectively, determine:
Heata ddition q3 – 4 = Cp (T4 – T3)
(a) Air standard efficiency of the dual cycle,
or 416.67 = 1.005 ¥ (T4 – 2161)
(b) Power developed, if the number of working cycles
or T4 = 2575.6 K = 2302.6°C
are 3 per second.
It is the maximum temperature reached in the
For air take, Cv = 0.71 kJ/kg ◊ K and Cp = 1.0 kJ/kg ◊ K
cycle.
v4 T 2575.6 Solution
cut-off ratio r = = 4 = = 1.191
v3 T3 2161
Given An air standard dual cycle with
g -1 g -1
T5 Êv ˆ Êv v ˆ p1 = 1 bar, pmax = 60 bar
Therefore, = Á 4˜ = Á 4 ¥ 2˜ T1 = 30°C = 303 K, r =9
T4 Ëv ¯
5 Ë v3 v1 ¯
d = 250 mm L = 300 mm
g -1 0.4
Ê rˆ Ê 1.191ˆ Cv = 0.71 kJ/kg ◊ K Cp = 1.0 kJ/kg ◊ K
= Á ˜ = Á = 0.353
Ë r¯ Ë 16 ˜¯ V4 – V3 = 0.04 vs
rps = 3(No. of cycle per second)
T5 = 2575.6 ¥ 0.353 = 911.16 K
Heat rejected Schematic With given data
q5–1 = Cv (T5 – T1)
= 0.718 ¥ (911.16 – 330)
= 417.2 kJ/kg
(iii) Thermal efficiency
q 417.2
hdual = 1 – out = 1 –
qin 1250
= 0.6667 = 66.67%
(iv) Net work done in the cycle,
wnet = qin – qout = 1250 – 417.2
= 832.8 kJ/kg
The initial specific volume of air,
RT1 0.287 ¥ 330
v1 = =
p1 100
= 0.9471 m3/kg
The clearance volume,
v 0.9471
v2 = 1 = = 0.0592 m3/kg
r 16
Gas Power Cycles 357

To find Temperature after isentropic expansion 4–5


(i) Air standard efficiency of dual cycle. g -1 g -1 g -1
T4 ÊV ˆ ÊV ˆ ÊV V ˆ
(ii) Power developed. = Á 5˜ = Á 1˜ = Á 1 ¥ 3˜
T5 Ë V4 ¯ Ë V4 ¯ Ë V2 V4 ¯
Assumptions
g -1
(i) The air in piston-cylinder assembly as a closed Ê rˆ
= Á ˜
system. Ë r¯
(ii) Air as an ideal gas. 2646.3
or T5 = 0.4
= 1228.1 K
Analysis Ê 9 ˆ
ÁË 1.32 ˜¯
(i) Air standard efficiency:
p Heat supplied per kg of air
Sweptv olume Vs = d 2L
4 qin = q2–3 + q3– 4 = Cv (T3 – T2) + Cp (T4 – T3)
p = ¥0.71
(2005.1 – 735.2) +
= ¥ (0.25)2 ¥ 0.3 = 0.0147 m3
4 1.0 ¥ (2646.73 – 2005.1)
The compression ratio = 901.63 + 641.63 = 1543.2 kJ/kg
V V + Vc V
r = 1 = s =1+ s Heat rejected per kg of air during process 4 –5
V2 Vc Vc q4 –5 = qout = Cv (T5 – T1) = 0.71 ¥ (1228.1 – 303)
V 0.0147
or Vc = V2 = V3 = s = = 656.82 kJ/kg
r -1 8 q 656.82
hair-standard = 1 – out = 1 –
= 0.0184 m3 qin 1543.2
and V1 = Vs + Vc = 0.0147 + 0.00184 = 0.574 = 57.4%
= 0.0165 m3 (ii) Power developed by the engine:
The ratio of specific heats Specific gas constant for air,
Cp 1 R = Cp – Cv = 1 – 0.71 = 0.29 kJ/kg ◊ K
g = = = 1.408
Cv 0.71
Mass of air
Pressure after isentropic compression 1–2:
p1V1 100 ¥ 0.0165
p2 = p1, rg = (1 bar) ¥ (9)1.408 m = =
RT1 0.29 ¥ 303
= 22.0 bar
= 0.0187 kg
The temperature after isentropic compression
Work done per cycle,
T2 = T1 r g –1 = 303 ¥ (9)0.408 = 735.2 K W = m(qin – qout)
After constant volume heat addition 2–3: = ¥ (1543.2 – 656.82)
0.0187
p 60 = 16.64 kJ
rp = 3 = = 2.727
p2 22 Power developed
T3 = T2 rp = 735.2 ¥ 2.727 = 2005.1 K = Work done per cycle ¥ No. of cycles/
Constant pressure heat addition upto 4% of stroke second
volume i.e., = ¥ 3 cycle/s = 49.93 kW
16.64
V4 – V3 = 0.04 (V1 – V2)
Example 11.22 The compression and expansion ra-
V4 Ê V1 ˆ tios of an oil engine working on air standard dual cycle
or = 1 + 0.04 Á - 1˜ (∵ V2 = V3) are 9 and 5, respectively. The initial pressure and tem-
V3 Ë V2 ¯
perature are 1 bar and 30°C, respectively. The heat liber-
or r = 1 + 0.04 (9 – 1) = 1.32 ated at constant pressure is twice the heat liberated at
For constant pressure process constant volume. The expansion and compression follow
T4 = rT3 = 1.32 ¥ 2005.1 the law pV1.25 = const. Determine:
= 2646.73 K
358 Thermal Engineering

(a) Pressure and temperature at all salient points. p2 = p1(r) n = (1 bar) ¥ (9)1.25
(b) The mean effective pressure of the cycle. = 15.58 bar
(c) Thermal efficiency of the cycle. Givent hat q3–4 = 2q2–3
(d) Power developed of the engine, if eight cycles Cp (T4 – T3) = 2Cv (T3 – T2) …(i)
complete in a second. r
We have re =
Take cylinder bore = 250 mm and stroke = 400 mm r
r 9
Thus, r = = = 1.8
Solution re 5
V T
Given An oil engine works on air standard dual cycle and r = 4 = 4
r = 9, re = 5 V3 T3
p1 = 1 bar T1 = 30°C = 303 K or T4 = rT3 = 1.8 T3 …(ii)
q3–4 = 2q2–3 Using in Eq. (i), we get
1.005 ¥ (1.8T3 – T3)
pV1.25 = const. n = 1.25 N = 8 cycles/s
= 2 ¥ 0.717 ¥ (T3 – 524.81)
d = 250 mm L = 400 mm
or 0.804T3 = 1.434T3 – 752.57
To find Temperature at state 3:
(i) Pressure and temperature at all salient points, T3 = 1194.56 K
(ii) The mean effective pressure of the cycle, Pressure at state 3:
(iii) Thermal efficiency of the cycle, and T
p3 = p2 ¥ 3
(iv) Power developed of the engine, if eight cycles T2
complete in a second. 1194.56
= 15.58 ¥ = 35.46 bar
Schematic 524.81
At state 4:p4 = p3 = 35.46 bar
T4 = 1.8 T3 = 2150.22 K
After polytropic expansion 4 –5:
n n
ÊV ˆ Ê 1ˆ
p5 = p4 Á 4 ˜ = p4 Á ˜
ËV ¯ 5 Ër ¯ e
1.25
Ê 1ˆ
= 35.46 ¥ Á ˜ = 4.74 bar
Ë 5¯
T4 2150.22
T5 = n -1
= = 1438 K
( re ) (5)0.25
(ii) Thermal efficiency of the cycle
Assumptions Heat supplied per kg of air
(i) The working substance in the piston cylinder qin = Cv (T3 – T2) + Cp (T4 – T3)
assembly as a closed system.
= 0.717 ¥ (1194.56 – 524.81)
(ii) The working substance is air, modeled as an
+ 1.005 ¥ (2150.22 – 1194.56)
ideal gas, with g = 1.4, Cp = 1.005 kJ/kg ◊ K, Cv
= 480.21 + 960.44 = 1440.65 kJ/kg
= 0.717 kJ/kg ◊ K
Heat rejected per kg of air
Analysis qout = Cv (T5 – T1)
(i) Pressure and temperature at all salient points: = 0.717 ¥ (1438 – 303)
After polytropic compression 1–2; = 813.75 kJ/kg
T2 = T1 (r)n–1 = (303 K) ¥ (9)1.25–1 Net work done per kg of air
= 524.81 K wnet = qin – qout = 1440.65 – 813.75
= 626.9 kJ/kg
Gas Power Cycles 359

Thermal efficiency Schematic p–v and T–s diagrams


w 626.9
hth = net =
q in 1440.65
= 0.435 or 43.5%
The specific volume of air inducted
p 100
v1 = 1 =
RT1 0.287 ¥ 303
= 1.15 m3/kg
v 1.15
v2 = 1 = = 0.1277 m3/kg
r 9
v3 = v1 – v2 = 1.15 – 0.1277
= 1.0222 m3/kg
(iii) The mean effective pressure
w 626.9
pm = net = Analysis The specific volume after compression;
vs 1.0222
1
= 613.27 kPa or 6.13 bar v2 = v1
Swept volume 15th
p 2 p Temperature after the isentropic compression 1–2;
Vs = d L= ¥ (0.25)2 ¥ 0.4
4 4 T2 = T1 r g –1 = 323 ¥ (15)1.4 –1
= 0.0196 m3 = 954.2 K or 681.2°C
Mass of air The pressure at the end of compression;
Vs 0.0196 p2 = p1rg
m = = = 0.0192 kg
vs 1.0222 = (1 bar ) ¥ (15)1.4 = 44.312 bar,
(iv) Power developed Temperature T3 at the end of constant-volume heat
P = m wnet ¥ No. of cycle/s addition 2–3;
= 0.0192 ¥ 626.9 ¥ 8 = 96.33 kW
Ê p3 ˆ
T3 = Á ˜ T2 = 2 ¥ 954.2
Example 11.23 A high-speed oil engine operating Ë p2 ¯
on a dual combustion cycle has a pressure of 1 bar and = 1908.4 K or 1635.4°C
a temperature of 50°C before compression. Air is then Pressure p3 at the end of constant-volume heat
compressed isentropically to 1/15th of its original vol- addition;
ume. The maximum pressure is twice the pressure at the
p3 = pmax = 2p2 = 2 ¥ 44.312 = 88.625 bar
end of isentropic compression. If the cut-off ratio is 2,
determine the temperature at the end of each process and Temperature T4 at end of constant-pressure heat
ideal efficiency of the cycle. Take g = 1.4. addition 3 – 4;
T4 = rT3 = (2) ¥ 1908.4
Solution
= 3816.8 K or 3543.8°C
Given A high-speed engine operates on air-standard The temperature at the end of isentropic expansion
dual cycle with 4–5;
p1 = 1 bar, T1 = 50°C = 323 K, g -1
Ê rˆ
r = 15, r = 2, T5 = T4 Á ˜
Ë r¯
g = 1.4 p3 = 2 p2 1.4 -1
Ê 2ˆ
To find = 3816.8 ¥ Á ˜
Ë 15 ¯
(i) Temperature at the end of each process, and
= 1704.8 K or 1431.8°C
(ii) Air standard efficiency of cycle.
360 Thermal Engineering

(ii) Air standard efficiency of the cycle The efficiency of the Diesel cycle is given as
Heat supplied per kg of air
1 È rg - 1 ˘
qin = Cv (T3 – T2) + Cp (T4 – T3) hDiesel = 1 - Í ˙
( r )g -1 ÍÎ g ( r - 1) ˙˚
Heat rejected per kg of air
V1
qout = Cp (T5 – T1) For both cycles, r = (same)
V2
qout Cv (T5 - T1 )
hth = 1 - = 1- V5
qin Cv (T3 - T2 ) + C p (T4 - T3 ) For Diesel cycle r =
V2
(T5 - T1 ) Since V5, volume after heat addition is always greater
= 1-
(T3 - T2 ) + g (T4 - T3 ) than V2 thus r > 1 g
r -1
1794.8 - 323 Thus quantity will always be greater than 1
= 1- g ( r - 1)
(1908.4 - 954.2) + 1.4 ¥ (3816.8 - 1908.4) 1
and when it is multiplied to g -1 , the negative quantity
= 0.6189 or 61.9% r
will increase, thus efficiency of Diesel cycle will be less
Example 11.24 For the same compression ratio, than Otto cycle.
prove that the efficiency of the Otto cycle is greater than
that of the Diesel cycle. Example 11.25 A four-cylinder, four-stroke engine
has a displacement volume of 300 cc per cylinder. The
Solution The Fig. 11.32 shows Otto and Diesel compression ratio of the engine is 10 and operates at a
cycles on p–v and T–s diagrams. speed of 3000 rpm. The engine is required to develop an
Cycle 1–2–3– 4–1 Otto cycle output power of 40 kW at this speed. Calculate thermal
Cycle 1–2–5– 6–1 Diesel cycle efficiency of the cycle, assuming that the engine operates
The efficiency of the Otto cycle is given as on the Otto cycle and that the pressure and temperature
at the inlet condition are 1 bar and 27°C, respectively.
1
hOtto = 1 - If the above engine is operating on the Diesel cycle
( r )g -1
and receiving heat at the same rate, calculate thermal ef-
p ficiency and maximum temperature of the cycle. Compare
the efficiency of Otto and Diesel cycles.
3

Solution
2 5
Given A four-stroke, four-cylinder engine operates on
6
4
Otto and Diesel cycles.
N
1 n = , Effective suctions k = 4
0 v2 vs
v 2 ¥ 60
(a) Otto and Diesel cycles on p–v diagrams Vs = 300 cc r = 10
N = 3000 rpm Wnet = 40 kW
T 3
p1 = 1 bar T1 = 27°C = 300 K
5
C To find
v= =C
p = C
6 v (i) The efficiency of the engine when it operates on
4
2 Otto cycle.
(ii) Thermal efficiency and maximum temperature in
1 cycle, when it operates on Diesel cycle.
s
0
(b) Otto and Diesel cycles on T–s diagrams Assumptions
(i) Air standard cold air assumptions.
Gas Power Cycles 361

Total volume,
V1 = Vs + V2 = 300 + 33.33
= 333.33 cc
= 333.33 ¥ 10–6 m3
The mass of air in a cylinder/cycle
p1V1 (100 kPa ) ¥ (333.33 ¥ 10 -6 m3)
m = =
RT1 (0.287 kJ/kg ◊ K ) ¥ (300 K )
= ¥ 10–4 kg
3.871
The mass flow rate to four-cylinders of the four-
stroke engine
3000
m = m k n = 3.87 ¥ 10–4 ¥ 4 ¥
2 ¥ 60
(ii) Ratio of specific heat g = 1.4. = 0.0387 kg/s
(iii) Specific heats Cp = 1.005 kJ/kg/K and Cv = 0.72
kJ/kg ◊ K. The heat supplied per second to the engine is
given by
Analysis Qin = m Cp (T3 – T2)
(i) When the engine operates on the Otto cycle Using numerical values
Thermal efficiency of the cycle. 66.67 = 0.0387 ¥ 1.005 (T3 – 753.56)
1 It gives T3 = 2468 K = 2195 °C
hOtto = 1 - g -1
r The cut off ratio
1
= 1- = 0.60 or 60% V3 T 2468
(10)1.4 -1 = 3 = = 3.27
V2 T2 753.56
The efficiency is also given as
Wnet The efficiency of the Diesel cycle
hth =
Qin 1 È rg - 1 ˘
hDiesel = 1 - Í
g -1
˙
Heat supply rate, r ÍÎ g ( r - 1) ˙˚
Wnet 40 kW È (3.27)1.4 - 1 ˘
Qin = = = 66.67 kW 1
hth 0.6 = 1- Í ˙
(10)1.4 - 1 ÍÎ1.4 ¥ (3.27 - 1) ˙˚
(ii) When the engine operates on the Diesel cycle
= 0.467 or 46.7%
r = 10
p1 = 1 bar Comment The efficiency of the Otto cycle is more than
T1 = 300 K that of the Diesel cycle for same compression ratio and
N = 3000 rpm same heat input.
Temperature after isentropic compression
T2 = T1 r g – 1
= 300 ¥ (10)1.4 –1 = 753.56 K
The compression ratio is given by
The Lenoir cycle is used for pulse jet engines. It
V V + V2 V
r = 1 = s =1+ s consists of three reversible processes:
V2 V2 V2
Vs Process 1–2 Constant-volume heat addition,
or = 10 – 1 = 9
V2 Process 2–3 Isentropic expansion, and
V 300
or V2 = s = = 33.33 cc Process 3–1 Constant-pressure cooling.
9 9
362 Thermal Engineering

The p–v and T–s representations of the Lenoir Ê g - 1ˆ


Ê 1ˆ
cycle are shown in Fig.11.34. Ê 1 ˆ ËÁ g ¯˜ ÁË g ˜¯
= T1 rp Á ˜ = T1 rp ...(iii)
p
2
Ë rp ¯
p2
pv g = C Substituting Eqs. (ii) and (iii) in Eq. (i),
Ê Ê 1ˆ ˆ
Á ˜
Á T1 rpË g ¯ - T1 ˜
hLenoir = 1- g Á ˜
p1
1
3 Á T1 rp - T1 ˜
v
ÁË ˜¯
0

T Ê Ê 1ˆ ˆ
2 Á ˜
Á rpË g ¯ - 1˜
= 1- g Á ˜ ...(11.16)
v=
C Á rp - 1 ˜
3 ÁË ˜¯
p=C The net work done per kg of air during Lenoir
1
0 s
cycle
Net work done = Sq for a cycle
wnet = qin – qout = Cv (T2 – T1) – Cp (T3 – T1)
For unit mass of air È Ê Ê 1ˆ ˆ˘
= Cv T1 ÍÍ( rp - 1) - g Á rpË g ¯ - 1˜ ˙
Heat Supplied, Á ˜

qin = q1–2 = Cv (T2 – T1) Á ˜˙


ÍÎ Ë ¯ ˙˚
(a positive quantity)
Heat rejected È Ê Ê 1ˆ ˆ˘
R Í Á rpË g ¯ - 1˜ ˙ (kJ/kg)
Á ˜
qout = q3–1 = Cp (T3 – T1) = T1 Í( rp - 1) - g
g -1 Á ˜˙
(a negative quantity) ÍÎ Ë ¯ ˙˚
The thermal efficiency of the Lenoir cycle can ...(11.17)
be expressed as
qout
hLenoir = 1 -
qin
The Atkinson cycle is an ideal cycle for Otto engine
C p (T3 - T1 ) Ê T3 - T1 ˆ exhausting to a gas turbine. The constant-volume
1- = 1- g Á ...(i)
Cv (T2 - T1 ) Ë T2 - T1 ˜¯ combustion gas turbine plants operate on Atkinson
cycle.
p2
Expressing rp = In Atkinson cycle, the heat addition takes place
p1 at constant volume, the remaining three processes
For the constant-volume process 1–2; are same as in air standard Brayton cycle. The p–v
T2 = T1 rp ...(ii) and T –s diagrams are shown in Fig. 11.35. The
For the isentropic process 2–3; nature of processes are
Ê g - 1ˆ Process 1–2 Isentropic compression,
Ê p ˆ ÁË g ˜¯
T3 = T2 Á 3 ˜ Process 2–3 Reversible constant-volume heat ad-
Ë p2 ¯ dition,
Gas Power Cycles 363

p3 3
v4 Volume after expansion
qin
re = =
Ise v3 Volume before expansion
ntr
p2 op
2 ic Expressing each temperature in terms of T1 with
Ise the use of above terms,
ntro
pic For the isentropic process 1–2;
4
1 qout
g -1
0 v Êv ˆ
= T1 r g -1
v2 v1 v4
T2 = T1 Á 1 ˜ ...(ii)
(a) p–v diagram Ë v2 ¯
T
For the constant-pressure process 4 –1;
T3 3
Êv ˆ Êv v ˆ r
ns
t. T4 = T1 Á 4 ˜ = T1 Á 4 ¥ 2 ˜ = T1 e ...(iii)
=
co Ë v1 ¯ Ë v3 v1 ¯ r
v
For the isentropic process 3– 4;
T4 4
g -1
2 Êv ˆ
= T4 reg -1
T2
nst. T3 = T4 Á 4 ˜
= co
T1
1
p
Ë v3 ¯
0 s
(b) T–s diagram Using T4 from Eq. (iii), we get

re g -1 rg
T3 = T1 re = T1 e ...(iv)
Process 3–4 Isentropic expansion, and r r
Process 4–1 Reversible constant-pressure heat Using Eq. (ii), (iii) and (iv) in Eq. (i);
rejection.
For unit mass of air Ê re ˆ
Á r -1 ˜
Heat supplied, hAtkinson = 1 - g Á g ˜
qin = q2–3 = Cv (T3 – T2) Á re g -1 ˜
ÁË -r ˜
(a positive quantity) r ¯
Heat rejected È r -r ˘
qout = q4 –1 = Cp (T4 – T1) = 1 - g Í ge g ˙ ...(11.18)
ÍÎ re - r ˙˚
(a negative quantity)
The thermal efficiency of the cycle The net work done per kg of air during the
q Atkinson cycle
hth = 1 - out
qin Net work done = Sq for a cycle
C p (T4 - T1 ) Ê T - T1 ˆ wnet = qin – qout = Cv (T3 – T2) – Cp (T4 – T1)
= 1- =1- g Á 4
C (T - T ) Ë T - T ˜¯
v 3 2 3 2 ÈÊ r g ˆ Êr ˆ˘
...(i) = Cv T1 ÍÁ e - r g -1 ˜ - g Á e - 1˜ ˙
ÍÎË r ¯ Ër ¯˙
The efficiency of the Atkinson cycle may be ˚
expressed in the following terms: R 1
= T1 [( reg - r g ) - g ( re - r )]
Compression ratio g -1 r
v1 Volume before compression ...(11.19)
r= =
v2 Volume after compression
364 Thermal Engineering

Example 11.26 In an ideal Atkinson cycle, the gas The compression ratio;
1 1
is compressed isentropically from 1 bar, 27 °C to 4 bar.
v Ê p ˆ g Ê 4 ˆ 1.328
The maximum pressure of the cycle is limited to 16 bar. r = 1 =Á 2˜ =Á ˜ = 2.84
v2 Ë p1 ¯ Ë 1¯
Calculate:
(a) the work done per kg of cycle, Temperature after isentropic compression,
-1
(b) the thermal efficiency of the cycle, T2 = T1 r g = 300 ¥ ( 2.84)1.328 - 1 = 422.5 K
(c) mean effective pressure of the cycle. Temperature after constant-volume heat addition;
Take Cp = 0.761 kJ/kg ◊ K, p3 16
and Cv = 0.573 kJ/kg ◊ K. T3 = T2 = ¥ 422.5 = 1690 K
p2 4
Solution Temperature after isentropic expansion
g -1 1.328 -1
Given An ideal Atkinson cycle with Êp ˆ g Ê 1ˆ 1.328
T4 = T3 Á 4 ˜ = 1690 ¥ Á ˜ = 852 K
p1 = 1 bar, p2 = 4 bar Ë p3 ¯ Ë 16 ¯
T1 = T 4 = 27°C = 300 K
(i) Work done per kg of gas
p3 = 16 bar
Heat supplied,
Cp = 0.761 kJ/kg ◊ K
qin = q2–3 = Cv (T3 – T2)
Cv = 0.573 kJ/kg ◊ K
= 0.573 ¥ (1690 – 422.5) = 726.3 kJ/kg
To find Heatr ejected
(i) The net work done, qout = q4 –1 = Cp (T4 – T1)
(ii) The thermal efficiency of the cycle, and = 0.761 ¥ (852 – 300) = 420.1 kJ/kg
(iii) Mean effective pressure. wnet = qin – qout
= 726.3 – 420.1 = 306.2 kJ/kg
Assumptions
(ii) Thermal efficiency of the cycle
(i) Air standard assumption.
W 306.2
(ii) Constant specific heat of gas. hth = 1 - net =
qin 726.3
(iii) Kinetic and potential energy effects are negligible.
= 0.422 or 42.2%
Analysis An ideal Atkinson cycle is shown on the p–v (iii) The mean effective pressure of the cycle
diagram in Fig. 11.36. The swept volume per kg of gas
The ratio of two specific heats Ê 1ˆ RT1 Ê 1ˆ
vs = v1 – v2 = v1 Á1 - ˜ = 1- ˜
g =
Cp 0.761 Ë r¯ p1 ÁË r¯
= = 1.328
Cv 0.573 0.188 ¥ 300 Ê 1 ˆ
= ¥ Á1 -
The specific gas constant; 100 Ë 2.84 ˜¯
R = C p – Cv
= 0.3654 m3/kg
= 0.761 – 0.573 = 0.188 kJ/kg ◊ K
w 306.2
pm = net =
vs 0.3654
= 838 kPa or 8.38 bar

The Brayton Cycle is also called Joule Cycle. It


is an ideal cycle for gas turbine plants. The sche-
matic arrangement for a Brayton cycle is shown
in Fig. 11.37 and its cooresponding p –v and T –s
Gas Power Cycles 365

diagrams are shown in Fig. 11.38. Air as the work-


ing fluid is compressed isentropically in the com-
pressor, heated at constant pressure in the heat ex-
changer, expanded isentropically in the turbine and
cooled again to the initial state before re-entering
the compressor.
The assumptions made in the Brayton cycle are
the: processes. In absence of any changes in kinetic and
potential energies; the steady-flow energy equation
1. Working fluid is air as an ideal gas.
on a unit mass basis, as
2. Actual combustion process in the combustion
q – w = Dh = hexit – hinlet
chamber is replaced by constant-pressure
heat addition from an external source. Assuming constant specific heat of air, the heat
3. Exhaust process is replaced by a constant- supplied to and rejected from air are
pressure heat rejection in a heat exchanger. qin = h3 – h2 = Cp (T3 – T2) ...(11.20)
The Brayton cycle consists of four internally re- qout = h4 – h1 = Cp (T4 – T1)
versible processes. Work developed per kg of air by turbine in
isentropic manner
Process 1–2 Isentropic compression of air in the
compressor, wT = h3 – h4 = Cp (T3 – T4) ...(11.21)
Work input to compressor
Process 2–3 Constant-pressure heat addition to
wC = h2 – h1 = Cp (T2 – T1) ...(11.22)
air,
Net work of the gas turbine plant
Process 3–4 Isentropic expansion of air in the
wnet = Sq or Sw
turbine,
= qin – qout or wT – wC
Process 4–1 Constant pressure heat rejection
Thermal efficiency of the ideal Brayton cycle is
from air.
wnet qout C p (T4 - T1 )
The p–v and T–s diagrams for an ideal Brayton hBrayton = = 1- =1-
qin qin C p (T3 - T2 )
cycle are shown in Fig. 11.38. All the four processes
of the Brayton cycle are executed in a steady flow T1 (T4 / T1 - 1)
= 1-
manner. Thus they are analysed as steady flow T2 (T3 / T2 - 1)
366 Thermal Engineering

For isentropic processes 1–2 and 3– 4;


g -1
g -1
Êp ˆ
( )g
T2 g
= Á 2˜ = rp (11.23)
T1 Ë p1 ¯
g -1 g -1
g -1
Êp ˆ Êp ˆ
( )g
T3 g g
and = Á 3˜ =Á 2˜ = rp
T4 Ë p4 ¯ Ë p1 ¯
(11.24)
Since p2 = p3 and p4 = p1, we get
T2 T T4 T
= 3 or = 3
T1 T4 T1 T2
Substituting in Eq. (i), we get
1
hBrayton = 1 - (g -1) /g
...(11.25)
rp
p2
where, rp = , pressure ratio and g is ratio of two
p1
specific heats.

Equation (11.25) indicates that the thermal effi-


ciency of an ideal Brayton cycle depends on pres-
sure ratio and specific heats ratio.
Figure 11.39(a) illustrates the effect of pressure
ratio on thermal efficiency of the Brayton cycle.
When pressure, after compression increases from function of pressure ratio
p2 to p 2¢ , both heat supply qin and net work wnet process, also increases. The turbine blades cannot
increase by an amount equal to the area 2–2¢–3¢– withstand very high temperature, (not more than
3–2, whereas the heat rejection qout from the cycle 1700 K, limit imposed by metallurgical consid-
remains unchanged. Hence, the thermal efficiency erations); thus temperature T3 is restricted. This
of the cycle increases with increase in pressure also limits the pressure ratio that can be used in
ratio as shown in Fig. 11.39(b). the cycle. For fixed T3, the net work output per
cycle increases with pressure ratio, reaches a maxi-
mum value and then starts to decrease as shown in
Fig. 11.40.
Figure 11.40 shows the effect of pressure ratio
From Fig. 11.39, it is evident that as pressure ratio for fixed maximum temperature on work output of
increases from p2/p1 to p 2¢ /p1, the amount of heat Brayton cycle. The pressure ratio rp2 > rp1 on the
supplied and work done are increased by an area effective area of cycle 1–2¢–3¢– 4¢–1 is more than on
2–2¢–3¢–3–2. Hence, the thermal efficiency of the the area 1–2–3–4, thus more work output is obtained
cycle increases. from the cycle. But as pressure ratio increases to
But as pressure p2 increases, the maximum rp3, the area of 1–2≤–3≤– 4≤–1 decreases, hence
temperature T3 reaching at the end of combustion work output of the cycle also decreases, and thus
Gas Power Cycles 367

È g -1 ˘
g -1
( ) g - 1˙˙
Í rp
T
= 1 rp
T3
( ) g Í g -1
Í
( ) g - 1˙˙˚
ÍÎ rp
g -1
=
T1
T3
( )g
rp

g -1
Back work ratio rbw =
Tmin
Tmax
rp ( ) g ...(11.28)

Usually, more than one half of the turbine work


output is used to drive the compressor. A gas power
plant with a high back work ratio requires a larger
the cyclic efficiency decreases. Therefore, in a gas
turbine to provide additional power requirement of
turbine power plant, there should be a compromise
the compressor. Therefore, the turbines used in gas
between the pressure ratio and network output.
power plants are larger than those used in steam
power plants.
Back
The back work ratio is an important characteristic
of a gas turbine plant. It is defined as the ratio of
The work ratio is another important characteristic
work input to the compressor to work produced by
of a gas turbine cycle and it is defined as the ratio of
the turbine. It is denoted as rbw and expressed as
the net work of the plant to the turbine work.
Compressor work
rbw = ...(11.26) Net work output
Turbine work rw = ...(11.29)
During steady-state operation, the back work Turbine work
ratio is
C p (T2 - T1) T2 - T1 Turbine work - Compressor work
h -h =
rbw = 2 1 = = Turbine work
h3 - h4 C p (T3 - T4 ) T3 - T4
T1 [T2 /T1 - 1] = 1-
Compressor work
= 1 - rbw
= ...(11.27)
T3 [1 - T4 /T3 ]
Turbine work
g -1
From Eq. (11.23) and (11.24)
g -1
= 1-
Tmin
Tmax
rp ( )g ...(11.30)
T2
T1
= rp ( ) g
In expressions (11.28) and (11.30), the tempera-
g -1 ture T3 represents the maximum temperature Tmax
and
T3
T4
= rp ( ) g or
T4
T3
=
1
g -1 reached in the cycle after heat addition (combus-
(rp ) g tion). The temperature T1 represents minimum tem-
Using we get perature Tmin, i.e., ambient temperature generally
remains constant. Therefore, the work ratio rw de-
È g -1 ˘
rbw = Í
( )
T1 Í p g - 1 ˙
r
˙
pends on temperature ratio Tmin/Tmax and pressure
ratio rp. The size of the gas turbine plant depends
T3 Í 1 ˙ on the magnitude of work ratio.
1- g -1 ˙
Í
ÍÎ rp g ˙( ) ˚
368 Thermal Engineering

g
Ê T ˆ 2 (g -1)
The air rate is defined as the amount of air required or rp = Á 3 ˜ ...(11.33)
to generate 1 kWh power output of the turbine. It is Ë T1 ¯
designated as AR and expressed as For fixed temperature limits of cycle-T1 and T3,
Mass of air required the maximum pressure ratio possible for maximum
AR =
1 kWh output
( kg/kWh ) specific work output
g -1
3600
=
Wnet (kW)
( kg/kWh ) ...(11.31) rp, max
ÊT ˆ
= Á 3˜
g

Ë T1 ¯
Inserting in Eq. (11.33), we get
For an ideal Brayton cycle, the net work output per rp = rp, max ...(11.34)
kg is given by Thus, the pressure ratio for maximum work is
wnet = wT – wC the function of limiting temperature ratio.
= Cp (T3 – T4) – Cp (T2 – T1)
Example 11.27 In an ideal Brayton cycle, air is com-
ÔÏ Ê T ˆ ÊT ˆ Ô¸ pressed from 1 bar to a pressure ratio of 6. Calculate the
= C p ÌT3 Á1 - 4 ˜ - T1 Á 2 - 1˜ ˝ ...(11.32)
ÓÔ Ë T3 ¯ Ë T1 ¯ Ô˛ cyclic efficiency. If the ratio of lower to upper tempera-
ture is 0.3 then calculate the work ratio.
For a gas turbine plant, the atmospheric tem-
perature T1 and turbine inlet temperature T3 are Solution
limiting temperatures. The efficiency and net work
output are greatly influenced by these temperatures. Given An ideal Brayton cycle with
Therefore, expressing T2 and T4 in terms of T1 and p1 = 1 bar
T3. rp = 6
T1/T3 = 0.3
g -1
T2
T1
T
= 3 = rp
T4
( ) g To find
(i) Brayton cycle efficiency,
g -1 (ii) Work ratio.
Using k =
g Assumptions
ÏÔ Ê 1ˆ ¸Ô (i) Cold air standard assumptions,
then wnet = C p ÌT3 Á1 - k ˜ - T1 ( rpk - 1) ˝ (ii) Ratio of specific heats g = 1.4, Cp = 1.005 kJ/kg ◊ K,
ÔÓ Ë rp ¯ Ô˛ (iii) The effect of kinetic and potential energy is
negligible.
Differentiating with respect to rp and equating it
to zero for maximization. Analysis
(i) The Brayton cycle efficiency is given by Eq.
ÏÔ ¸
dwnet k k -1 Ô (11.21)
= C p ÌT3 k + 1 - T1 k rp ˝ = 0
drp ÔÓ rp ˛Ô 1
hBrayton = 1 - g -1
or
T3
k +1
= T1 rpk –1 ( )
rp g
rp
1
Ê g -1ˆ = 1- 1.4 -1
= 0.40 or 40%

or
T3
= rp2k =

rp Ë g ¯
˜
(6 ) 1.4
T1
Gas Power Cycles 369

(ii) The work ratio is given by Eq. (11.24) Process 3–4: Isentropic expansion of gas in turbine.
g -1 T3 923
rw =
T
( )
1 - 1 rp g T4 = g -1
= 1.4 -1
= 582.76 K
T3
1.4 -1
(rp ) g (5) 1.4
= ()
1 - 0.3 ¥ 6 1.4 = 0.5 (i) Compressor work per kg of air
wC = h2 – h1 = Cp (T2 – T1)
= 1.005 ¥ (472 – 298)
Example 11.28 The pressure ratio and maximum
temperature of a Brayton cycle are 5:1 and 923 K, = 174.87 kJ/kg
respectively. Air enters the compressed at 1 bar and (ii) Turbine work per kg of air
298 K. Calculate for 1 kg of air flow, the compressor wT = h3 – h4 = Cp (T3 – T4)
work, turbine work and the efficiency of the cycle. = 1.005 ¥ (923 – 582.76)
= 341.94 kJ/kg
Solution (iii) Thermal efficiency of cycle
Given An ideal Brayton cycle with The net work output per kg of air
rp = 5, p1 = 1 bar, wnet = wT – wC
T1 = 298 K, T3 = 923 K = 341.94 – 174.87 = 167.07 kJ/kg
m = 1 kg Heat supplied per kg of air
qin = h3 – h2 = Cp (T3 – T2)
To find
= 1.005 ¥ (923 – 472)
(i) Compressor work,
= 453.25 kJ/kg
(ii) Turbine work, and Thermal efficiency;
(iii) The thermal efficiency of the cycle. w 167.07
hth = net =
Assumptions qin 453.25
(i) Air standard assumption. = 0.3686 or 36.86%
(ii) Kinetic and potential energy effects are negligible.
(iii) For air Cp = 1.005 kJ/kg ◊ K and g = 1.4. Example 11.29 Air enters the compressor of an
air-standard Brayton cycle at 100 kPa, 300 K with a
Analysis An ideal Brayton cycle is shown on the T–s volumetric flow ratio of 5 m3/s. The compressor pressure
diagram in Fig. 11.41. ratio is 10. The turbine inlet temperature is 1400 K.
Determine (a) thermal efficiency of the cycle, (b) back
work ratio. (c) net power developed in kW.

Solution
Given An air-standard Brayton cycle operates with
p1 = 100 kPa T1 = 300 K
V = 5 m3/s rp = 10
T3 = 1400 K
To find
(i) Thermal efficiency,
(ii) Back work ratio, and
Process 1–2: Isentropic compression of gas in com- (iii) Net power developed.
pressor Assumptions
g -1 1.4 -1
T2 = ( )
rp g T1 = (5) 1.4 ¥ 298 = 472 K (i) Each component in the cycle is assumed in steady
state.
370 Thermal Engineering

Mass flow rate of air


= r V = 1.161 ¥ 5 = 5.80 kg/s
Net work developed
= Mass flow rate ¥ Net work done/kg
= 5.80 ¥ 397.67 = 2309.34 kW

Example 11.30 The pressure and temperature at the


beginning of compression in an air standard Brayton
cycle are 100 kPa and 27°C. The heat added per kg of
air is 1850 kJ. The compression ratio is 4. Determine the
(ii) Turbine and compressor processes are isentropic. maximum pressure and temperature, thermal efficiency
(iii) No pressure drop in heat exchangers. and mean effective pressure. Assume g = 1.4, Cp =
(iv) The working fluid is air with Cp = 1.005 kJ/kg ◊ K, 1.005 kJ/kg ◊ K
g = 1.4 and R = 0.287 kJ/kg ◊ K
Solution
(v) Kinetic and potential energy effects are negligible.
Given
Analysis
p1 = 100 kPa, T1 = 27°C = 300 K
(i) The thermal efficiency of an air-standard Brayton
qin = 1850 kJ/kg r = 4,
cycle is given by
1 1 g = 1.4, Cp = 1.005 kJ/kg ◊ K
hBrayton = 1 - (g -1) / g = 1 - 1.4 -1
rp To find
(10) 1.4 (i) Maximum pressure in the cycle,
= 0.482 or 48.2%
(ii) Maximum temperature in the cycle,
(ii) The back work ratio
Compressor work T -T (iii) Thermal efficiency, and
rbw = = 2 1 (iv) Mean effective pressure.
Turbine work T3 - T4
g -1
1.4 -1
Here T2 = T1 rp g = 300 ¥ (10 ) 1.4

= 579.2 K
T3 1400
T4 = g -1
= 1.4 -1
= 721.1 K
(rp ) g (10) 1.4

Then back work ratio


579.2 - 300
rbw = = 0.417 = 41.7%
1400 - 725.1
(iii) Net power developed
Net work done per kg of air
= Turbine work – Compressor work.
= Cp (T3 – T4) – Cp (T2 – T1)
= 1.005 ¥ (1400 – 725.1)
– 1.005 ¥ (579.2 – 300)
= 397.67 kJ/kg
Density of fresh air
p 100
r = 1 =
RT1 0.287 ¥ 300
= 1.161 kg/m3
Gas Power Cycles 371

Assumptions Example 11.31 A gas turbine working on an air


(i) Working substance is air, standard Brayton cycle operates between the temperature
(ii) Compression and expansion are isentropic, limits of 300 K and 1200 K and pressure limits of 101 kPa
(iii) Heat addition and rejection are reversible. and 505 kPa. Calculate (a) thermal efficiency of the cycle,
(b) compressor work in kJ/kg, (c) turbine work in kJ/kg,
Analysis and (d) air flow rate for 2.0 kW of net power output.
(i) The pressure and temperature after isentropic
compression Solution
pmax = p2 = p1r g = 1 bar ¥ (4)1.4
Given A gas turbine operating on air-standard Brayton
= 6.96 bar cycle
The maximum pressure in the cycle is 6.96 bar T1 = 300 K T3 = 1200 K
T2 = T1 r g –1 = 300 ¥ (4)1.4–1 = 522.23 K p1 = 101 kPa p2 = 505 kPa
(ii) Temperature after heat addition P = 2.0 kW
q2–3 = Cp (T3 – T2)
or 1850 = 1.005 ¥ (T3 – 522.23) To find
or T3 = 2363 K = 2090°C (i) Thermal efficiency of the cycle,
The maximum temperature in the cycle is 2090°C (ii) Compressor work in kJ/kg,
(iii) Thermal Efficiency (iii) Turbine work in kJ/kg, and
Pressure ratio in the cycle (iv) Mass flow rate of air for power output of 2 kW.
p 6.96
rp = 2 = = 6.96 Assumptions
p1 1
(i) Constant specific heat of air as Cp = 1.005 kJ/kg ◊ K.
1 1
then hBrayton = 1 - g -1
= 1- 1.4 -1 (ii) Ratio of specific heat as g = 1.4.
( rp ) g (6.96) 1.4 (iii) Each device in the cycle as steady flow device.
= 0.4255 or 42.55% Analysis The air standard Brayton cycle for the given
(iv) Mean Effective pressure pm: data is shown in Fig. 11.44.
wnet = hBrayton ¥ qin = 0.4255 ¥ 1850
= 787.26 kJ/kg
The specific gas constant for air
g -1 1.4 – 1
R = Cp = ¥ 1.005
g 1.4
= 0.287 kJ/kg ◊ K
The initial specific volume of air
RT1 0.287 ¥ 300
v1 = =
p1 100
= 0.861 m3/kg
The clearance specific volume
v 0.861
v2 = 1 = = 4 m3/kg
r 4
The swept volume per kg of air The pressure ratio
p 505 kPa
vs = v1 – v2 = 0.861 – 4 rp = 2 = =5
= 4 m3/kg p1 101 kPa
The mean effective pressure (i) The efficiency of the Brayton cycle is given as
w 787.26 1
pm = net = = 4 kPa hBrayton = 1 - g -1
vs
= 4 bar
4
(rp ) g
372 Thermal Engineering

1 Tmin = 27°C = 300 K


= 1- 1.4 - 1
= 0.3686 = 36.86%
Tmax = 800°C = 1073 K
( 5) 1.4
To find
Process 1–2: Isentropic compression of air in the
compressor; (i) Pressure ratio for hCarnot = hBrayton .
g -1 1.4 -1 (ii) Pressure ratio for maximum work done.
( )
T2 = T1 rp g = 300 ¥ (5) 1.4 (iii) Efficiency comparison between hCarnot and
hBrayton .
= 475.14 K
Analysis
Process 3–4: Isentropic expansion of air in the (i) The pressure ratio at which the cycle efficiency
turbine; approaches the Carnot cycle efficiency:
T3 1200 The air-standard Carnot cycle efficiency
T4 = g -1
= 1.4 -1
= 757.66 K
( )
Tmin 300
rp g (5 ) 1.4 hCarnot = 1 -
Tmax
= 1-
1073
= 0.72

(ii) The compressor work per kg of air The air standard Brayton cycle efficiency is given
wC = h2 – h1 = Cp (T2 – T1) as
= 1.005 ¥ (475.14 – 300) 1
hBrayton = 1 - g -1
= 176.0 kJ/kg
(iii) Turbine work per kg of air (rp ) g
wT = h3 – h4 = Cp (T3 – T4) When hCarnot approaches hBrayton then
= 1.005 ¥ (1200 – 757.66)
= 444.5 kJ/kg 1
0.72 = 1 - 1.4 -1
The net work output per kg of air
wnet = wT – wC
(rp ) 1.4
1.4 -1
= 444.5 – 176 = 268.5 kJ/kg
(iv) The mass flow rate of air ( ) 1.4
or rp =
1
0.28
P 2.0 kW 1.4
mair = = Ê 1 ˆ 0.4
wnet 268.5 kJ/kg Pressure ratio; rp = Á = 86.08
Ë 0.28 ˜¯
= 7.45 ¥ 10–3 kg/s = 26.81 kg/h
(ii) Pressure ratio for maximum work done
Example 11.32 A gas turbine plant operates on an g
air-standard Brayton cycle between the temperature Ê Tmax ˆ 2(g -1)
(rp)opt = Á
limits of 27°C K and 800°C. Ë T ˜¯ min
(a) Find the pressure ratio at which the cycle efficien- 1.4
cy approaches the Carnot cycle efficiency. Ê 1073 ˆ 2 ¥ (1.4 -1)
= Á = 9.3
(b) Find the pressure ratio at which the work done Ë 300 ˜¯
per kg of air would be maximum.
(c) Compare the efficiency at this pressure ratio with (iii) Comparison efficiency for rp = 9.3
Carnot efficiency for the given temperature. 1 1
hBrayton = 1 - g -1
= 1- 1.4 -1
= 0.471
Solution ( )
rp g (9.3) 1.4

Given A gas turbine operating on air standard Brayton hBrayton 0.471


Ratio = = 0.654
cycle hCarnot 0.72
Gas Power Cycles 373

Summary
Otto cycle except that the heat is added at con-
converts chemical energy of fuel into mechanical stant pressure. Its thermal efficiency is given by
energy.
r of a reciprocating engine 1 È rg - 1 ˘
hDiesel = 1 - Í ˙
is the ratio of maximum possible volume to the r g -1 ÍÎ g ( r - 1) ˙˚
clearance volume in the cylinder.
V V V hOtto > hDual > hDiesel
r = max = BDC = 1
Vmin VTDC V2
pm is that pressure, hDiesel > hDual > hOtto
which if it acts on the piston during the entire is an ideal cycle for the gas
power stroke, would produce an amount of work turbine plants and its thermal efficiency is given
equal to that actually produced by the engine. as
Wnet W 1
pm = = net (kPa) hBrayton = 1 - (g -1) / g
Vmax - Vc Vs rp
p2
where, rp = , pressure ratio and g is ratio of
substance is air as an ideal gas. The combustion p1
process of an actual cycle is replaced by heat- two specific heats. The thermal efficiency of an
addition process and exhaust process by heat- ideal Brayton cycle depends on pressure ratio and
rejection process. specific heats ratio.
- back work ratio is an important characteristic
tion reciprocating engines. It consists of four of a gas turbine cycle. It is defined as
reversible processes—isentropic compression, Compressor work
rbw =
constant-volume heat adddition, isentropic ex- Turbine work
pansion and constant-volume heat rejection. The Atkinson cycle is an ideal cycle for Otto en-
thermal efficiency of an Otto cycle is gine exhausting to a gas turbine. Its efficiency is
1 given by
hOtto = 1 - g -1
r 1 È re - r ˘
Diesel cycle is an ideal cycle for the com- hAtkinson = 1 - Í ˙
g ÍÎ reg - r g ˙˚
pression ignition engines. It is very similar to

Glossary
BDC Bottom dead centre Compression ratio Ratio of maximum volume to mini-
Bore Internal diameter of cylinder mum volume in the cylinder
Brake power Power available from the engine for ex- Diesel cycle Theoretical cycle for Diesel and oil en-
ternal use gines
CI engine Compression ignition (Diesel) engine Friction power Difference between indicated power
Clearance volume Volume left in the centre, when pis- and brake power
ton is at TDC IC engines Internal combustion reciprocating engines
374 Thermal Engineering

Indicated power Power developed on the piston by SI engine Spark ignition (petrol) engine
combustion gases inside the cylinder Stroke Linear distance between TDC and BDC
Mean effective pressure Ratio of net work done to Swept volume Piston displacement volume in cylinder
swept volume in the cycle TDC Top dead centre
Mechanical efficiency Ratio of brake power to the in- Thermal efficiency Ratio of brake power to the heat
dicated power supply rate
Otto cycle Theoretical cycle for petrol engines

Review Questions
1. Define compression ratio. 8. Discuss the deviation of Stirling and Ericsson
2. What is mean effective pressure? and what is its cycles from Carnot cycle.
significance. 9. Compare Otto, Diesel and Dual cycles for given
3. Define (a) work ratio (b) swept volume, (c) compression ratio.
charge, (d) thermal efficiency. 10. Prove that the compression ratio corresponds to
4. Derive an expression for thermal efficiency of maximum work in the Otto cycle, is given by
Otto cycle. 1
Ê T ˆ 2 (g -1)
5. What are cold air assumptions? r = Á max ˜
ËT ¯
min
6. Why are engines not operated on Carnot cycle?
Explain 11. Discuss the effect of pressure variation on thermal
7. Write the drawback of Carnot cycle. efficiency of Brayton cycle.

Problems
1. The engine of a car has four cylinders of 70-mm 4. A four cylinder spark ignition engine has a
bore, and 75-mm stroke. The compression ratio is compression ratio of 8, each cylinder has a
8. Determine the cubic capacity of the engine and maximum volume of 0.6 litre. At the beginning of
the clearance volume of each cylinder. the compression process, the air is at 98 kPa and
[1154 cc, 41.21 cc] 17°C and the maximum temperature in the cycle
2. A four-cylinder, four-stroke petrol engine has a is 1800 K. Assume engine operates on ideal Otto
bore of 80 mm and a stroke of 80 mm. The com- cycle, determine: (a) amount of heat supplied
pression ratio is 8. Calculate the cubic capacity of per cylinder (b) thermal efficiency.
the engine and clearance volume of each cylinder. [2.32 kJ, 56.4%]
What type of engine is this? 5. An ideal Otto cycle has a compression ratio of 9.2
[(a) 1608.4 cc, (b) 57.4 cc, (c) Square engine] and uses air as a working fluid. At the beginning
3. The compression ratio in an air standard Otto of the compression process, the air is at 98 kPa
cycle is 10. At the beginning of compression and 27°C. The pressure is doubled during heat
strokes, the pressure is 100 kPa and temperature addition process. Determine: (a) Amount of heat
is 15°C. The heat transferred to the air per cycle added to air, (b) net work output, (c) thermal
is 1800 kJ/kg of air. Determine: efficiency. [523.3 kJ/kg, 307.9 kJ/kg, 58.8%]
(a) The pressure and temperatures at the salient 6. An engine is working on Otto cycle. Air enters
points. the cylinder at 1 bar and 27°C, it is compressed
(b) The mean effective pressure. isentropically with a compression ratio of 7 and
then heated at constant volume till the temperature
Gas Power Cycles 375

rises to 2000 K. Find the air standard efficiency, 10. The compression ratio in an air-standard Otto
pressure of air at the end of compression and cycle is 8. At the beginning of compression
after heat addition process, and the mean process, the pressure is 1 bar and the temperature
effective pressure of the cycle. Assume Cv = is 300 K. The heat transfer to the air per cycle is
0.718 kJ / kg ◊ K, g = 1.4, and R = 0.287 kJ/ kg ◊ K. 1900 kJ/kg of air. Calculate:
[54.08%, 15.245 bar, 46.66 bar, 708.5 bar] (a) Thermal efficiency,
7. A four stroke engine working on Otto cycle has a (b) The mean effective pressure.
swept volume of 0.1 m3. The compression ratio is [(a) 56.47%, (b) 14.24 bar]
7. The condition at the start of the cycle is at 1 bar, 11. The temperature at the beginning of the
90°C. The heat addition at the constant volume is compression process of an air standard Otto
100 kJ/cycle. Find the thermal efficiency, mean cycle with a compression ratio of 8 is 300 K,
effective pressure, the pressure and temperature the pressure is 1 atm and the cylinder volume is
at salient points in the cycle. Assume air as 560 cc. The maximum temperature in the cycle is
working substance with Cv = 0.718 kJ/ kg ◊ K and 2000 K. Calculate
g = 1.4 (a) Temperature and pressure at the end of each
[46%, 4.592 bar, T2 = 790.6 K, p2 = 15.25 bar, process of cycle,
p3 = 39.24 bar, T3 = 2034.1 K, (b) Thermal efficiency of the cycle.
p4 = 2.57 bar, T4 = 934.0 K] [p2 = 18.3 atm, p3 = 53.33 atm, p4 = 2.9 atm,
8. An engine of 250 mm bore and 375 mm stroke T2 = 689.2 K, T4 = 870.5 K, hOtto = 56.4%]
works on Otto cycle. The clearance volume is 12. A four-stroke engine having a swept volume of
0.00263 m3. The initial pressure and temperature 0.13 m3 operates on Otto cycle. The compression
are 1 bar and 50°C. If the maximum pressure is ratio is 6 and the conditions at the beginning
limited to 25 bar, find the following: of compression are 1 bar and 60°C. The heat
(a) The air standard efficiency of the cycle. supplied is 150 kJ per cycle. Calculate the work
(b) The mean effective pressure for the cycle. done, efficiency and mean effective pressure.
Assume the ideal conditions. Take Cp = 1.005 kJ/kg ◊ K.
[(a) 56.5%, (b) 1.334 bar] and Cv = 0.718 kJ/kg ◊ K.
9. An engine working on Otto cycle has a volume of [76.75 kJ/cycle, 51.2%, 5.9 bar]
0.45 m3, pressure 1 bar and temperature 30°C at 13. An engine working on a constant volume cycle
the beginning of compression stroke. At the end of has a clearance volume of 1 litre and a stroke
compression stroke, the pressure is 11 bar, 210 kJ volume of 6 litres. The suction pressure and
of heat is added at constant volume. Determine: temperature are 1 bar and 20°C, respectively.
(a) Pressures, temperature and volumes at The pressure at the end of heat addition is 25 bar.
salient points in the cycle, Determine
(b) Percentage clearance, (a) Thermal efficiency of cycle,
(c) Efficiency, (b) Work done per cycle.
(d) Net work per cycle, Take Cv during heat addition = 0.837 kJ/kg ◊ K.
(e) Mean effective pressure, Cv during heat rejection = 0.737 kJ/kg ◊ K and
(f) Ideal power developed by the engine if the g = 1.4 [59.5%, 1.693 kJ]
number of working cycles per minute is 14. A four-cylinder, four-stroke Diesel engine is to
210. develop 30 kW at 1000 rpm. The stroke is 1.4
Assume the cycle is reversible. times the bore and the indicated mean effective
[(a) 600 K, 0.081 m3, 1172 K, 21.48 bar, 0.081 m3, pressure is 6.0 bar. Determine the bore and stroke
1.97 bar, 591.8 K, 0.45 m3 (b) 21.95%, (c) 0.495, of the engine. [176 mm, 246 mm]
(d) 2.818 bar, (e) 364 kW]
376 Thermal Engineering

15. A Diesel engine has an inlet temperature and Assume Cp = 1.005 kJ/kg ◊ K, Cv = 0.718 kJ/ kg ◊ K.
pressure of 17°C and 1 bar, respectively. The [Otto cycle (a) 6.8 (b) 53.54%, (c) 11.31 KW
compression ratio is 15 and the maximum cycle Diesel cycle (a) 12.41, (b) 58.21%, (c) 13.67 KW]
temperature is 1400 K. Calculate the air-standard 21. An engine with 200 mm cylinder diameter and
efficiency of the Diesel cycle. Take g = 1.4. 300 mm stroke works on theoretical Diesel cycle.
[1.634, 62.3%] The initial pressure and temperature of air used
16. An engine working on the Otto cycle has a clear- are 1 bar and 27°C. The cut off is 8% of the
ance of 17% of stroke volume and the initial pres- stroke. Determine:
sure of 0.95 bar, and a temperature of 30°C. If (a) Pressures and temperatures at all salient
the pressure at the end of constant volume heat points.
addition is 28 bar, find (b) Theoretical air standard efficiency.
(a) Air standard efficiency (c) Mean effective pressure.
(b) The maximum temperature in the cycle (d) Power of the engine if the working cycles
(c) Ideal mean effective pressure per minute are 380.
Assume working fluid to be air. Assume that compression ratio is 15 and working
[(a) 53.7%, (b) 1024.8°C, (c) 3.167 bar] fluid is air. Consider all conditions to be ideal.
17. A four-stroke Diesel engine with four cylinders [(a) 44.31 bar, 886.2 K, 0.0006728 m3,
develops an indicated power of 125 kW and 44.31 bar, 0.001426 m3, 1878.3 K, 2.866 bar,
delivers brake power of 100 kW. Calculate 858.38 K, 0.0101 m3 (b) 59.8% (c) 7.424 bar,
(a) friction power, and (b) mechanical efficiency (d) 44.27 kW]
of the engine. [(a) 25 kW, (b) 80%] 22. An air-standard Diesel cycle has a compression
18. An air standard Diesel cycle has a compression ratio of 18, and the heat transferred to the working
ratio of 16 and a cut-of ratio of 2. At the beginning fluid per cycle is 1800 kJ/kg. At the beginning of
of the compression process the air is at 95 kPa the compression stroke, the pressure is 1 bar and
and 27°C. Determine: the temperature is 300 K. Calculate: (a) Thermal
(a) temperature after heat addition, efficiency, (b) The mean effective pressure.
(b) the thermal efficiency, and [(a) 61%, (b) 13.58 bar]
(c) mean effective pressure. 23. A compression ignition engine has a stroke
19. An ideal Diesel engine has a compression ratio of 270 mm, and a cylinder diameter of 165 mm.
20 and uses air as working fluid. At the beginning The clearance volume is 0.000434 m3 and the
of compression stroke, the air is at 100 kPa and fuel ignition takes place at constant pressure for
20°C. If the maximum temperature in the cycle 4.5 per cent of the stroke. Find the efficiency
is not to exceed 2200 K, determine: (a) thermal of the engine assuming it works on the Diesel
efficiency, and (b) mean effective pressure. cycle. [61.7%]
[(c) 63.5%, (b) 933 kPa] 24. The compression ratio of dual combustion cycle
20. Two engines are operated on ideal Otto and diesel is 14 and the maximum temperature is limited to
cycles for which the following information is 1850°C. If the working substance receives equal
available: amount of heat during both constant volume
Maximum temperature = 1227°C and constant pressure process, determine the
Exhaust temperature = 447°C air standard efficiency of the cycle. Assume
temperature at the start of the cycle is 27°C.
Ambient condition = 1 bar, 37°C
[64.12%]
Air consumption = 2 kg/min
Estimate: (a) Compression ratio, (b) Air standard
efficiency, and (c) power output.
Gas Power Cycles 377

uestions
1. Thermal efficiency of a heat engine is defined as
Work done Indicated Power 6. Which statement is true for Diesel cycle?
(a) (b)
Heat supplied Brake Power (a) Heat addition at constant volume and heat
Brake Power rejection at constant volume
(c) (d) none of the above (b) Heat addition at constant volume and heat
Indicated Power
rejection at constant pressure
2. Air-standard efficiency of Otto cycle is given by
(c) Heat addition at constant pressure and heat
Ê g -1 ˆ
rejection at constant volume
Ê 1 ˆ ÁË g ˜¯ (g -1)
(a) 1 - Á ˜ (b) 1 - ( r ) (d) Heat addition at constant pressure and heat
Ë r¯
(g -1) rejection at constant pressure
Ê 1ˆ
(c) r (g -1) - 1 (d) 1 - Á ˜ 7. Air standard Diesel cycle consists of
Ë r¯
(a) two isothermal and two constant-volume
3. The mean effective pressure of an engine is processes
defined as (b) two isentropic and two constant-pressure
Work done processes
(a)
Stroke volume (c) two isentropic and two constant-volume
Work done/cycle processes
(b) (d) none of the above
Cylinder volume
Work done/cycle 8. The efficiency of air standard Otto cycle depends
(c) on
Stroke volume
Work done/kg (a) pressure ratio in the cycle
(d) (b) temperature ratio in the cycle
Stroke volume
(c) Compression ratio in the cycle
4. Which statement is true for Otto cycle?
(d) mean effective pressure
(a) Heat addition at constant volume and heat
9. The efficiency of air standard Diesel cycle de-
rejection at constant volume
pends on
(b) Heat addition at constant volume and heat
(a) cut-off ratio in the cycle
rejection at constant pressure
(b) compression ratio in the cycle
(c) Heat addition at constant pressure and heat
(c) both (a) and (b)
rejection at constant volume
(d) none of above
(d) Heat addition at constant pressure and heat
10. Air standard efficiency of Diesel cycle is given by
rejection at constant pressure
5. Air standard Otto cycle consists of 1 È r g -1 ˘
(a) 1 - Í ˙
(a) two isothermal processes and two constant- ( r )g -1 ÍÎ g ( r - 1) ˙˚
volume processes
(b) two isothermal processes and two constant- 1 È rg - 1 ˘
(b) 1 - Í ˙
pressure processes ( r )g -1 ÍÎ g ( r - 1) ˙˚
(c) two isentropic processes and two constant-
1 È r g -1 ˘
volume processes (c) 1 - Í ˙
(d) two isothermal processes and two isentropic ( r )g -1 ÍÎ ( r - 1) ˙˚
processes
1 È r -1 ˘
(d) 1 - Í ˙
( r )g -1 ÍÎ ( rg - 1) ˙˚
378 Thermal Engineering

11. Cut-off ratio is defined as 17. Which one of the following is part of air standard
Stirling cycle?
Volume before compression
(a) (a) Reversible isothermal compression
Volume after compression
(b) Polytropic expansion
Volume after heat supply (c) Isobaric heat addition
(b)
Volume after compression (d) Reversible isentropic compression
Volume after expansion 18. Which one of the following is part of air-standard
(c)
Volume after compression Ericsson cycle?
(a) Reversible isothermal compression
Volume after expansion
(d) (b) Polytropic expansion
Volume before exp ansion
(c) Isochoric heat rejection
12. Which process is included in air standard Diesel (d) Reversible isentropic compression
cycle?
19. Which one of the following is part of air standard
(a) Polytropic compression Atkinson cycle?
(b) Isochoric heat addition (a) Isothermal heat addition
(c) Isobaric heat addition (b) Constant-volume heat rejection
(d) Isochoric and isobaric heat addition (c) Constant-volume heat addition
13. Which one of the following is part of air standard (d) Constant-pressure heat addition
dual cycle? 20. Which one of the following is part of air standard
(a) Polytropic compression Lenoir cycle?
(b) Isochoric heat addition (a) Constant volume addition
(c) Isobaric heat addition (b) Isentropic expansion
(d) Isochoric and isobaric heat addition (c) Constant pressure heat rejection
14. Which one of the following is part of air standard (d) All of the above
Brayton cycle?
21. Air standard efficiency of Brayton cycle is given
(a) Polytropic compression by
(b) Isochoric heat addition
1 È rg -1 ˘
(c) Isobaric heat addition (a) 1 - Í ˙
(d) Isochoric and isobaric heat addition ( r )g -1 ÍÎ g ( r - 1) ˙˚
15. Air standard Stirling cycle consists of È rg - 1 ˘
1
(a) two isothermal and two constant-volume (b) 1 - g -1
Í ˙
processes
(r) ÍÎ g ( r - 1) ˙˚
(b) two isothermal and two constant-pressure 1
(c) 1 -
processes rp(g -1) / g
(c) two isentropic and two constant-volume 1
(d) 1 -
processes ( r )g -1
(d) two isothermal and two isentropic processes
22. Air standard efficiency of Lenoir cycle is given
16. Air standard Ericsson cycle consists of
by
(a) two isothermal and two constant-volume
processes 1 È rg -1 ˘
(a) 1 - g -1
Í ˙
(b) two isothermal and two constant-pressure (r) ÍÎ g ( r - 1) ˙˚
processes
(c) two isentropic and two constant-volume 1 È rg -1 - 1 ˘
(b) 1 - g -1
Í ˙
processes (r) ÍÎ g ( r - 1) ˙˚
(d) two isothermal and two isentropic processes
Gas Power Cycles 379

1 24. For given compression ratio, among Otto, Diesel


(c) 1 - (g - 1) / g and dual cycles
rp
(a) Diesel cycle is most efficient
Ê Ê 1ˆ ˆ (b) Otto cycle is most efficient
Á r ÁË g ˜¯ - 1˜ (c) dual cycle is most efficient
(d) 1 - g Á ˜
p
Á rp - 1 ˜ (d) None of the above
Á ˜ 25. For same maximum pressure and temperature
Ë ¯
among Otto, Diesel and dual cycles
23. Air standard efficiency of Atkinson cycle is given (a) Diesel cycle is most efficient
by (b) Otto cycle is most efficient
1 È rg -1 ˘ (c) dual cycle is most efficient
(a) 1 - g - 1 Í ˙
(r) ÍÎ g ( r - 1) ˙˚ (d) none of the above
26. The efficiency of a cycle is maximum when it is
1 È rg - 1 ˘
(b) 1 - g - 1 Í ˙ (a) Diesel cycle engine
(r) ÍÎ g ( r - 1) ˙˚ (b) Otto cycle engine
È r -r ˘ (c) dual cycle engine
(c) 1 - g Í ge g ˙ (d) reversible engine
ÍÎ re - r ˙˚ 27. Gas power cycle is not used for
Ê Ê 1ˆ ˆ (a) Diesel cycle engine
Á r ÁË g ˜¯ - 1˜ (b) Otto cycle engine
(d) 1 - g Á ˜
p
ÁË rp - 1 ˜¯ (c) Locomotive engine
(d) Aircraft engine

(c) 27. (d) 26. 25. (a)


24. (b) 23. (c) 22. (d) 21. (c) 20. (d) (c) 19. (a) 18. 17. (a)
16. (b) 15. (a) 14. (c) 13. (d) 12. (c) (b) 11. (b) 10. 9. (c)
8. (c) 7. (d) 6. (c) 5. (c) 4. (a) (c) 3. (d) 2. 1. (a)
Answers
380 Thermal Engineering

12
Vapour Power Cycles

Introduction
In the vapour power cycle, the phase of working substance changes alternatively. The change of phase
allows more energy to be stored in the working substance that can be stored by only sensible heating. The
working substance expands as a vapour but it is compressed as a liquid with much smaller specific volume,
thus a very little of expansion work is used for compression process. The most common working substance
is water and thus the cycle and plant are called steam power cycle and steam power plant, respectively, even
though water is used in the liquid phase during a part of the cycle. Steam power plants generate a major
fraction of electric power produced in the world.

12.1 MODELLING A STEAM divided into four major subsystems identified as is


POWER PLANT A, B, C and D on the diagram.

A steam power plant consists of the following main The subsystem A consists of a
components: furnace and chimney. Its function is to supply heat
energy to the boiler for steam generation. The heat
1. Boiler with its mountings and accessories energy may be obtained by burning of fossil fuel,
2. Turbine nuclear reaction, or by concentrating solar energy.
3. Condenser
Subsystem (B In the subsystem B, the working
4. Feed-water pump fluid passes through the series of four interconnected
5. Electric generator components and power is generated in this cycle.
6. Cooling tower This cycle is referred as steam cycle.
7. Circulating water pump The subsystem B is of prime focus in this
8. Chimney chapter, where the heat energy is converted into
9. Draught system work. It consists of a boiler (steam generator), a
10. Feed-water treatment plant turbine, a condenser, and a feed pump. The steam
generated in the boiler is passed through the turbine,
The basic components of a simplified steam
where it expands to a lower pressure, thus power is
power plant is shown in Fig. 12.1. To facilitate
generated. The steam leaving the turbine is passed
thermodynamic analysis, the whole plant can be
through the condenser, where it condenses to water
Vapour Power Cycles 381

and creates a low pressure at the turbine exit. The Back


cooling water is circulated in the condenser with It is another performance parameter for a power
the help of the subsystem C. The condensate is then plant and is defined as the ratio of pump work input
recirculated to the boiler with the help of the feed to the work developed by the turbine. It is denoted
pump. by rbw . The back work ratio is
Subsystem (C The subsystem C consists of the Pump work wp
rbw = = ...(12.2)
cooling tower and water-circulation arrangement. Turbine work wT
The circulated warm water from the condenser is
sent to the cooling tower, where its heat energy is
rejected to the atmosphere. The cooling water is
The work ratio for a power plant is defined as ratio
then recirculated through the condenser.
of the net work output of the cycle to the work
The subsystem D pertains to gen- developed by the turbine. It is denoted as rw; and
eration of electrical energy, and thus consists of a expressed as
generator. The generated electricity is supplied to a Net work output of the cycle
power grid through the substation. rw =
Turbine work
wnet
12.2 PERFORMANCE PARAMETERS = = 1- bwr ...(12.3)
wT
OF VAPOUR POWER CYCLE
Steam Rate
The thermal efficiency of any power cycle is ex- It is also called specific steam consumption. It
pressed as relates the power output to amount of steam
Net work done in the cycle necessary to produce it. It is the amount of steam
hth = required to produce 1 kWh (3600 kJ) of power. It is
Heat supplied in the cycle
wnet denoted by ssc and is expressed as
= ...(12.1)
qin
382 Thermal Engineering

Mass of steam in kg/h Reservoir at temperature TH


ssc =
Power output in kW
qin
ms ( kg/h )
=
ms (kg/s) ¥ wnet (kJ/kg) 4 1
3600 kJ/kWh Boiler
= ...(12.4)
wnet (kJ/kg)
Turbine WT
Heat Rate
Wp
It is the amount of heat required by a power plant to Compressor
produce 1 kWh of power. It is expressed in kJ/kWh.
(Heat input in kJ/s) ¥ (3600 s/h) 3 Condenser 2
Heat rate =
Net power output in kW
3600 qout
= (kJ/kWh) ...(12.5)
hth Reservoir at temperature TL

(a) Schematic
CARNOT VAPOUR POWER CYCLE p
TH
When we think of a power cycle of maximum ef- Saturation
curve
ficiency, the Carnot cycle immediately conjures up
in our mind. It is a cycle, which has maximum ef- 4 1
ficiency, operating between given temperature lim- TL

its and its efficiency is independent of properties of TH


working fluid.

3 2
TL

A Carnot vapour power cycle is executed within v


saturation dome of a pure substance. It uses a two- (b) p–v as diagram
phase fluid as the working medium as shown in
T
Fig. 12.2.
Figure 12.2(a) gives the arrangement of com- qin
ponents in the cycle, 12.2(b) shows Carnot vapour TH 4
1 Saturation
power cycle on p–v coordinates, 12.2(c) on T–s curve
coordinates, and 12.2(d) on h –s coordinates. The
boundary of the region in which liquid and vapour TL 2
3
are both present (the vapour dome) is also indicat- qout

ed.
The four processes in the cycle are as follows:
1. Reversible Adiabatic E Saturated S
S4 S1
steam expands in the turbine. The temperature
lowers from TH to TL. The state 2 is reached in the (c) T–s diagram
wet region.
Vapour Power Cycles 383

h
p1 Equation (12.7) is exactly same as equation
derived for an ideal gas. The efficiency of the Carnot
1
vapour power cycle depends on the operating
temperatures and is independent of properties of
p2
working substance.
4

3 However, in spite of the claim of theoretical higher


s
(d) h–s diagram thermal efficiency, the Carnot vapour power cycle
has certain practical difficulties as discussed below:
1. Isothermal heat transfer at constant tem-
During this pro- perature to or from the working substance is
cess, the condensation starts from state 2 and stops not difficult to obtain. It can be achieved by
at state 3 and the heat qL per unit mass is rejected in maintaining the working fluid in a two-phase
the condenser to the sink at TL. system in the boiler and condenser. But heat-
transfer processes in a two-phase system
Adiabatic The limit the maximum operating temperature
mixture of liquid and vapour is compressed to the in the cycle, thus limiting the thermal effi-
saturation liquid state 4 at boiler pressure. ciency.
I During 2. In the turbine, the dry saturated steam ex-
this process, a quantity of heat qH per unit mass is pands isentropically. The quality of steam
added in the boiler from the heat source at the tem- decreases during expansion. The presence of
perature TH. high moisture content in steam will lead to
erosion and wear of the turbine blades.
Vapour 3. Isentropic compression (process 3–4) in-
Power Cycle volves the compression of liquid–vapour
mixture to saturated liquid state. There are
Isothemal heat addition to a vapourising fluid in the three practical difficulties associated with
boiler; this process:
qin = TH (s1 – s4)
(a) Control of condensation at state 3, before
Isothermal heat rejected by the working sub-
reaching to saturated liquid state.
stance in the condeser;
(b) Compression of two-phase (water +
qout = TL (s1 – s4)
steam) system. Because of large spe-
The net work done of the cycle; cific volume of vapour than liquid, the
wnet = qin – qout compressor size and work input will
= TH (s1 – s4) – TL (s1 – s4) have to be large.
= (TH – TL )(s1 – s4) ...(12.6) (c) Higher compression work will reduce
The thermal efficiency of the cycle; the thermal efficiency of the plant.
wnet (T - TL ) ( s1 - s4 ) These practical difficulties limit the use of
hCarnot = = H
qin TH ( s1 - s4 ) Carnot cycle as a suitable model for design of
TH steam power plants.
= 1- ...(12.7)
TL
384 Thermal Engineering

A steam power plant operates on the (a) Quality of steam at the end of isentropic expansion
Carnot cycle using dry steam at 17.5 bar. The exhaust and at the end of isothermal heat rejection,
takes place at 0.075 bar into condenser. The steam con- (b) Heat added per cycle,
sumption is 20 kg/min. Calculate: (c) Net work developed in the cycle,
(d) The efficiency of the cycle,
(a) Power developed in the cycle,
(e) Work ratio.
(b) The efficiency of the cycle.
Solution
Solution
Given A Carnot engine operating on vapour power
Given A steam power plant operating on Carnot cycle
cycle
p1 = 17.5 bar p2 = 0.075 bar
p1 = 12 MPa p2 = 30 kPa
ms = 20 kg/min
ms = 0.1 kg
To Find
(i) Power developed, and To Find
(ii) Thermal efficiency of the cycle. (i) Quality of steam at the end of isentropic expansion
and at the end of isothermal heat rejection,
Analysis Properties of steam
(ii) Heat added per cycle,
At boiler pressure from Table A-12
(iii) Net work developed in the cycle,
p1 = 17.5 bar = 1750 kPa
(iv) The efficiency of the cycle, and
TH = 205.76°C = 478.77 K
(v) Work ratio.
h4 = hf = 878.48 kJ/kg
h1 = hg = 2796.43 kJ/kg
At condenser pressure from Table A-12,
p2 = 0.075 bar = 7.5 kPa
TL = 40.29°C = 3l3.3 K
The heat supplied in the cycle
qin = h1 – h4 = 2796.43 – 878.48
= 1917.95 kJ/kg
The Carnot efficiency is given as
T 313.3
hCarnot = 1 - L = 1 -
TH 478.77
= 0.3456 or 34.56%
The work done per kg in the cycle
wnet = hCarnot ¥ qin = 0.3456 ¥ 1917.95 Analysis Properties of steam
= 662.84 kJ/kg At boiler pressure, from Table A-12
The power developed in the cycle p1 = 12 MPa = 12000 kPa
Ê 20 ˆ TH = 324.75°C = 597.75 K
P = ms wnet = Á kg/s˜ ¥ (662.84 kJ/kg) h4 = hf = 1491.24 kJ/kg
Ë 60 ¯
h1 = hg = 2688.83 kJ/kg
= 220.95 kW
s4 = sf = 3.4972 kJ/kg ◊ K
A Carnot engine contains 0.1 kg of s1 = sg = 5.5 kJ/kg ◊ K
water. During the heat addition process, saturated liq- At condenser pressure, from Table A-12
uid is converted into saturated vapour. Heat addition p2 = 30 kPa
occurs at 12 MPa and heat rejection ocuurs at 30 kPa. TL = 69.13°C = 342.13 K
Determine: hf = 289.21 kJ/kg
Vapour Power Cycles 385

hfg = 2336.07 kJ/kg Note: Equation (12.7) will also give same
sf = 0.9439 kJ/kg ◊ K efficiency of Carnot cycle.
sfg = 6.8247 kJ/kg ◊ K (iv) Work ratio
(i) Quality of steam at the end of isentropic expansion Turbine work;
and at the end of isothermal heat rejection wT = h1 – h2 = 2688.83 – 1848.77
The state of steam after isentropic expansion in = 840.06 kJ/kg
the turbine; w 512
s1 = s2 = (sf + x2 sfg )@ 30 kPa rw = net = = 0.609
wT 840.06
5.5 = 0.9439 + x2 (6.8247)
It means 60.9% of turbine workdone will be
5.5 - 0.9439 available as net output of the cycle.
or x2 = = 0.6676
6.8247
The state of steam before isentropic compression
in the compressor; RANKINE CYCLE
s4 = s3 = (sf + x3 sfg )@ 30 kPa
3.4972 = 0.9439 + x3 (6.8247) Many of the practical difficulties associated with
the Carnot vapour power cycle are eliminated in
3.4972 - 0.9439
or x3 = = 0.374 Rankine cycle. The steam coming out of the boiler
6.8247
is usually in superheated state, and expands in the
Specific enthalpy at the state 2
turbine. After expanding in the turbine, the steam
h2 = (hf2 + x2 hfg2)@ 30 kPa
is condensed completely in the condenser. The
= 289.21 + 0.6676 ¥ 2336.07
Rankine Cycle shown in Fig. 12.3 is an ideal va-
= 1848.77 kJ/kg
pour power cycle and is used in steam power plants.
Specific enthalpy at the state 3
h3 = (hf3 + x3 hf g 3)@ 30 kPa
= 289.21 + 0.374 ¥ 2336.07
= 1163.2 kJ/kg
The heat supplied per kg of steam The four basic components of a vapour power plant
qin = h1 – h4 = 2688.83 – 1491.24 are shown in Fig. 12.4(a). Each component in the
= 1197.59 kJ/kg cycle is regarded as a control volume, operating at
(ii) Heat added per cycle Qin = ms qin steady state.
= (0.1 kg) ¥ (1197.59 kJ/kg) The vapour leaving the boiler enters the
= 119.75 kJ/cycle turbine, where it expands isentropically to the con-
The heat rejected per kg of steam denser pressure at the state 2. The work produced
qout = h2 – h3 = 1848.77 – 1163.2 by the turbine is rotary (shaft) work and is used to
= 685.57 kJ/kg drive an electric generator or machine.
Net work per kg of steam;
wnet = qin – qout The condenser is attached at the exit
= 1197.59 – 685.57 ª 512 kJ/kg of the turbine. The vapour leaving the turbine is
(iii) Net work done per cycle; wet vapour and it is condensed completely in the
Wnet = ms wnet = (0.1 kg) ¥ (512 kJ/kg) condenser to the state 3, by giving its latent heat to
= 51.2 kJ/cycle some other cooling fluid like water.
(iv) Thermal efficiency of the Carnot cycle The liquid condensate leaving the condens-
q 685.57 er at the state 3 is pumped to the operating pressure
hCarnot = 1 - out = 1 -
qin 1197.59 of the boiler. The pump operation is considered is-
= 0.4275 or 42.75% entropic.
386 Thermal Engineering
p1
The heat is supplied to the working fluid T
(feed water) in the boiler and thus vapour is gener- 1¢
ated. The vapour leaving the boiler is either satu-
qin
rated at the state 1 or superheated at the state 1¢, T1
1

depending upon the amount of heat supplied in the Saturation


curve
boiler. 4
p1
T2
3 2 2¢
p2
qout

Figure 12.4(b) shows Rankine cycle on p–v co- s


ordinates, Fig. 12.4(c) on T–s coordinates, and (c) T–s diagram for Rankine cycle
Fig. 12.4(d) on h–s coordinates. The liquid, vapour,
and wet vapour regions are also indicated with the h p1
help of saturation curve.

In all three diagrams above, the cycle 1–2–3–4–
1 Saturation
1 represents an ideal Rankine cycle using saturated curve
steam and the cycle 1¢–2¢–3–4 –1¢ represents an qin p2
ideal Rankine cycle with superheated steam at the
turbine entry. The Rankine cycle consists of the 4 2¢
2
following four internally reversible processes:
qout
3
qin
s
(d) h–s diagram for Rankine cycle
4 Boiler 1

Isentropic expansion of the working


Pump Turbine WT fluid in the turbine from boiler pressure to condenser
Wp pressure.
Heat rejection from the working fluid
3 Condenser 2 at constant pressure in the condenser till the fluid
reaches the saturated liquid state 3.
qout
Isentropic compression of the work-
(a) Basic components of vapour power cycle
ing fluid in the pump to the boiler pressure at the
p state 4 in the compressed liquid region.
Saturation Heat addition to working fluid at
curve
constant pressure in the boiler from state 4 to 1.
qin
4 1 1¢
Cycle
We assume 1 kg of working substance in the cycle
and applying steady flow energy equation to each
3
2 2¢
component in the cycle. If changes of kinetic and
qout potential energy are neglected then the steady-flow
v energy equation reduces to
(b) p–v diagram for Rankine cycle q – w = Dh ...(12.8)
Vapour Power Cycles 387

(i) For isentropic compression (q = 0) in the Thus the efficiency of Rankine cycle can be
pump (process 3–4); expressed as
– wp = h4 – h3 qin - qout q
Taking pump work negative; then hRankine = = 1 - out
qin qin
wp = h4 – h3 ...(12.9)
h2 - h3
where h3 is hf enthalpy of liquid at condenser = 1- ...(12.16)
h1 - h4
pressure p2.
h4 is the enthalpy of water at state 4, In order to avoid errosion and corrosion
calculated as on turbine blades, the wet steam is never
h4 = h3 + wp ...(12.10) allowed to enter the turbine. Figure 12.5
shows T–s and h–s (Mollier) diagrams for
Then the isentropic compression work wp is
an ideal Rankine cycle using wet steam at
obtained as
p1 turbine entry.
wp = Ú p2
v dp = v ( p1 - p2 ) ...(12.11)

where vf is the specific volume of liquid at


condenser pressure p2 . The negative sign
p1
from - Ú p2
vdp has been dropped to make
the pump work positive.
(ii) For constant-pressure heat addition
process in the boiler (w = 0):
q2–3 = qin = h1 – h4 ...(12.12)
(iii) For isentropic expansion process 1–2 in the
turbine (q = 0):
– wT = h2 – h1
or wT = h1 – h2 ...(12.13)
(iv) For constant-pressure heat removal process
2–3 in the condenser (w = 0):
q2–3 = qout = h3 – h2
Taking negative sign for heat rejection, then
qout = h2 – h3 ...(12.14)
The thermal efficiency of any power cycle is
expressed as
Net work done in the cycle w
h= = net
Heat supplied in the cycle qin R
For Rankine cycle, It is defined as the ratio of the actual thermal ef-
wnet = wT – wp ficiency of steam power plant to the corresponding
= (h1 – h2) – vf ( p1 – p2) Rankine efficiency. It is denoted by ‘hrelative’ and
For a thermodynamic cycle, the net work is expressed as
also equal to net heat transfer; Actual thermal efficiency
wnet = qin – qout hrelative = ...(12.17)
Rankine efficiency
= (h1– h4) – (h2 – h3) ...(12.15)
388 Thermal Engineering

A steam power plant has boiler and Specific enthalpy at the state 2
condenser pressures of 60 bar and 0.1 bar, respectively. h2 = (hf2 + x hfg2)@ 10 kPa
Steam coming out of the boiler is dry and saturated. The = 191.81 + 0.698 ¥ 2392.82
plant operates on the Rankine cycle. Calculate thermal = 1863.34 kJ/kg
efficiency. The pump work;
wp = vf (p1 – p2) = 0.001010 ¥ (6000 – 10)
= 6.05 kJ/kg
Enthalpy at the state 4;
h4 = h3 + wp = 191.81 + 6.05
= 197.86 kJ/kg.
Rankine cycle efficiency, Eq. (12.16)
h -h 1863.34 - 191.81
hRankine = 1 - 2 3 = 1 -
h1 - h4 2785.10 - 197.86
= 0.353 or 35.35%

A steam power plant works between


Solution
pressures of 40 bar and 0.05 bar. If the steam supplied
Given Rankine cycle with dry saturated steam is dry saturated and the cycle of operation is Rankine
p1 = 60 bar = 6000 kPa, cycle, find
p2 = 0.1 bar = 10 kPa (a) Cycle efficiency
To find Thermal efficiency of steam power plant. (b) Specific steam consumption

Analysis Properties of steam at principal states Solution


State 1: Dry saturated steam; from Table A-13
Given Rankine cycle with dry saturated steam
p1 = 6000 kPa,
p1 = 40 bar = 4000 kPa,
h1 = 2785.10 kJ/kg
p2 = 0.05 bar = 5 kPa
s1 = 5.8891 kJ/kg ◊ K
State 2 : Wet steam; To find
p2 = 10 kPa (i) Rankine cycle efficiency, and
hf2 = 191.81 kJ/kg (ii) Specific steam consumption.
hf g2 = 2392.82 kJ/kg Analysis Properties of steam at principal states (From
sf2 = 0.6492 kJ/kg ◊ K table A-13)
sf g2 = 7.5010 kJ/kg ◊ K State 1: Dry saturated steam;
State 3: Saturated liquid; p1 = 4000 kPa
p3 = 0.1 bar = 10 kPa h1 = 2800.36 kJ/kg
s1 = 6.0685 kJ/kg ◊ K
h3 = hf3 = 191.81 kJ/kg
State 2: Wet steam;
vf3 = 0.001010 m3/kg
p2 = 5 kPa
State 4: Compressed liquid;
hf2 = 137.79 kJ/kg
p4 = 6000 kPa,
hfg2 = 2423.66 kJ/kg
The state 2, after isentropic expansion can be defined
sf2 = 0.4763 kJ/kg ◊ K
by equating entropy at states 1 and 2; sfg2 = 7.9187 kJ/kg ◊ K
s1 = s2 = (sf + xsfg )@ 10 kPa State 3: Saturated liquid;
5.8891 = 0.6492 + x (7.5010)
p3 = 5 kPa;
5.8891 - 0.6492 h3 = hf3 = 137.79 kJ/kg
or x = = 0.698
7.5010 vf3 = 0.001005 m3/kg
Vapour Power Cycles 389

State 4: Compressed liquid; Solution


p4 = 4000 kPa
Given A Rankine cycle using dry and saturated steam;
h4 = h3 + wp
Mass flow rate of steam; ms = 9.5 kg/s
The state 2, after isentropic expansion can be defined
Turbine inlet pressure; p1 = 35 bar = 3500 kPa,
by equating entropy at states 1 and 2;
Condenser pressure, p2 = 0.2 bar
s1 = s2 = (sf + xsfg )@ 5 kPa
6.0685 = 0.4763 + x (7.9187) To find
6.0685 - 0.4763 (i) Pump work,
or x = = 0.7064
7.9187 (ii) Turbine work,
Specific enthalpy at the state 2 (iii) Rankine efficiency,
h2 = (hf2 + xhfg 2)@ 5 kPa (iv) Condenser heat flow.
= 137.79 + 0.7064 ¥ 2423.66 Assumptions
= 1849.84 kJ/kg (i) Each component in the cycle is analysed as
The pump work; control volume at steady state.
wp = vf ( p1 – p2) (ii) Turbine and pump operation are isentropic.
= 0.001005 ¥ (4000 – 5) (iii) No pressure drop in boiler and condenser.
= 4.015 kJ/kg (iv) All processes are internally reversible.
Enthalpy at the state 4; (v) Kinetic and potential energy effects in each
h4 = h1 + wp = 137.79 + 4.015 = 141.8 kJ/kg components are negligible.
Turbine work;
Analysis Properties of steam at principal states
wT = h1 – h2 = 2800.36 – 1849.84
State 1: Dry saturated steam;
= 952.52 kJ/kg
p1 = 3500 kPa
Net work of cycle;
h1 = 2802.25 kJ/kg
wnet = wT – wp = 952.52 – 4.015
s1 = 6.122 kJ/kg ◊ K
= 946.5 kJ/kg
Heat supplied;
qin = h1 – h4 = 2800.36 – 141.8
= 2658.56 kJ/kg
(i) Rankine cycle efficiency,
wnet 946.50
hRankine = = = 0.3562 or 35.62%
qin 2658.56
(ii) Specific steam consumption by Eq. (12.14);
3600 kJ/kWh 3600
ssc = = = 3.78 kg/kWh
wnet kJ/kg 946.50

In a Rankine cycle, the steam at inlet


to a turbine is dry satuarted at a pressure of 35 bar and
the exhaust pressure is 0.2 bar. Calculate
(a) Pump work,
(b) Turbine work,
(c) Rankine efficiency,
(d) Condenser heat flow.
Assume flow rate of 9.5 kg/s.
390 Thermal Engineering

State 2: Wet steam; (iv) Condenser heat flow;


p2 = 20 kPa Qin = ms (h2 – h3)
hf = 251.38 kJ/kg = 9.5 ¥ (2014.61 – 251.38)
hfg = 2358.33 kJ/kg = 16750.6 kW
sf = 0.8319 kJ/kg ◊ K
sf g = 7.0766 kJ/kg ◊ K A steam power plant operates on ideal
State 3: Saturated liquid; Rankine cycle. The steam enters the turbine at 3 MPa,
350°C and is condensed in the condenser at a pressure
p3 = 20 kPa
of 75 kPa. Determine thermal efficiency, back work ratio
h3 = hf3 = 251.38 kJ/kg
and work ratio of this cycle.
vf3 = 0.001017 m3/kg
State 4: Compressed liquid Solution
p4 = 3500 kPa
Given A steam power plant operating on ideal Rankine
h4 = h3 + wp
cycle.
The state 2, after isentropic expansion can be defined
Turbine inlet condition p1 = 3 MPa = 3000 kPa
by equating entropy at states 1 and 2;
T1 = 350°C
s1 = s2 = (sf + x sfg )@ 3.5 kPa
Condenser pressure, p2 = 75 kPa
6.122 = 0.8319 + x (7.0766)
6.122 - 0.8319 To find
or x = = 0.7476
7.0766 (i) Thermal efficiency of the Rankine cycle,
Specific enthalpy at the state 2 (ii) Back work ratio, and
h2 = (hf2 + xhfg 2)@20 kPa (iii) Work ratio.
= 251.38 + 0.7476 ¥ 2358.33
= 2014.61 kJ/kg
Schematic with given data
Pump work per kg of steam;
wp = vf (p1 – p2)
= 0.001017 ¥ (3500 – 20) = 3.54 kJ/kg
State 4; h4 = h3 + wp = 251.38 + 3.54
= 254.92 kJ/kg
(i) Pump power required;
Wp = ms wp = 9.5 ¥ 3.54 = 33.63 kW
(ii) Turbine Power output;
WT = ms (h1 – h2)
= 9.5 ¥ (2802.25 – 2014.61)
= 7482.58 kW
Net power of cycle;
Wnet = WT – Wp
= 7482.58 – 33.63 = 7448.95 kW
Total heat supplied to steam;
Qin = ms (h1 – h4)
= 9.5 ¥ (2802.25 – 254.92)
= 24199.63 kW
(iii) Rankine cycle efficiency,
W 7482.58
hRankine = net =
Qin 24199.63
= 0.3078 or 30.78%
Vapour Power Cycles 391

Analysis Properties of steam at principal states (ii) Back work ratio;


State 1: Superheated steam, Table A-14 Pump work wp
rbw = =
p1 = 3000 kPa Turbine work wT
T1 = 350°C, 3.03 kJ/kg
h1 = 3115.6 kJ/kg = = 0.0042
714 kJ/kg
s1 = 6.7428 kJ/kg ◊ K
wnet 711 kJ/kg
State 2: Wet steam, Table A-13 (iii) Work ratio; rw = = = 0.995
wT 714 kJ/kg
p2 = 75 kPa
hf2 = 384.36 kJ/kg
A steam turbine develops 5 kW, op-
hf g2 = 2278.6 kJ/kg
erating on an ideal Rankine cycle. It receives steam at
sf2 = 1.213 kJ/kg ◊ K
30 bar, and 300°C and exhausts it to a condenser at a
sf g2 = 6.2434 kJ/kg ◊ K
vacuum of 685 mm of Hg. The barometer reads 760 mm
State 3: Saturated liquid
of Hg. The condensate is then returned to the boiler by a
p1 = 75 kPa feed pump. Calculate
h3 = hf2 = 384.36 kJ/kg (a) Rankine cycle efficiency,
vf3 = 0.001037 m3/kg
(b) Dryness fraction of steam entering the condenser,
State 4: Compressed liquid
(c) back work ratio of this cycle,
p4 = 3000 kPa
(d) Mass flow rate of steam.
h4 = h3 + wp
The state 2, after isentropic expansion can be defined Solution
by equating entropy at states 1 and 2;
Given An ideal Rankine cycle
s1 = s2 = (sf + xsfg )@ 75 kPa
Turbine inlet condition p1 = 30 bar = 3000 kPa,
6.7428 = 1.213 + x (6.2434)
T1 = 300°C
6.7428 - 1.213
or x = = 0.885 Condenser vacuum, h2 = 685 mm of Hg
6.2434
Barometric pressure, hatm = 760 mm of Hg
Specific enthalpy at the state 2
h2 = (hf2 + x h fg2)@ 75 kPa To find
= 384.36 + 0.885 ¥ 2278.6 (i) Rankine cycle efficiency,
= 2401.6 kJ/kg (ii) Dryness fraction of steam entering the condenser,
The pump work; (iii) Back work ratio, and
wp = vf (p1 – p2) (iv) Mass flow rate of steam.
= 0.001037 ¥ (3000 – 75) = 3.03 kJ/kg
Analysis The absolute pressure in the condenser in mm
Enthalpy at the state 4;
of Hg
h4 = h3 + wp = 384.39 + 3.03
hcondenser = hatm – h2 = 760 – 685 = 75 mm of Hg
= 387.40 kJ/kg
The corresponding pressure in kPa
Turbine work;
wT = h1 – h2 = 3115.6 – 2401.6 101.325 kPa
p2 = (75 mm of Hg) ¥ = 10 kPa
= 714 kJ/kg 760 mm of Hg
Net work of cycle; Properties of steam at principal states
wnet = wT – wp State 1: Superheated steam; Table A-14,
= 714 – 3.03 ª 711 kJ/kg p1 = 3000 kPa
(i) Rankine cycle efficiency, Eq. (12.16) T1 = 300°C
h - h3 h1 = 2993.48 kJ/kg
hRankine = 1 - 2
h1 - h4 s1 = 6.5389 kJ/kg ◊ K
2401.6 - 383.36 State 2: Wet steam; Table A-13,
= 1- = 0.26 or 26% p2 = 10 kPa
3115.3 - 387.40
392 Thermal Engineering

Enthalpy at the state 4;


h4 = h3 + wp
= 191.81 + 3.0 = 194.81 kJ/kg
Turbine work;
wT = h1 – h2
= 2993.48 – 2070.62 = 922.85 kJ/kg
Net work of cycle;
wnet = wT – wp
= 922.85 – 3.0 ª 919.85 kJ/kg
Rankine cycle efficiency, Eq. (12.16)
h -h
hRankine = 1 - 2 3
h1 - h4
2070.62 - 191.81
= 1-
2993.48 - 194.81
= 0.3286 or 32.86%
Back work ratio;
Pump work
rbw =
Turbine work
wp 3.0 kJ/kg
= = = 0.0032
wT 922.85 kJ/kg
Mass flow rate of steam;
W 5 ¥ 103 kW
hf2 = 191.81 kJ/kg ms = =
hf g2 = 2392.82 kJ/kg wnet 919.85 kJ/kg
= 5.43 kg/s or 19568.4 g/h k
sf2 = 0.6492 kJ/kg ◊ K
sfg2 = 7.5010 kJ/kg ◊ K A Rankine cycle operates with steam
State 3: Saturated liquid; as working fluid. The saturated vapour enters the turbine
p3 = 10 kPa at 8.0 MPa and saturated liquid exists the condenser at a
h3 = hf3 = 191.81 kJ/kg pressure of 0.008 MPa. The net power output of the cycle
vf3 = 0.001001 m3/kg is 100 MW.
State 4: Compressed liquid; Determine for the cycle
p4 = 3000 kPa (a) thermal efficiency,
h4 = h3 + wp (b) back work ratio,
The state 2, after isentropic expansion can be defined (c) mass flow rate of steam in kg/h,
by equating entropy at states 1 and 2; (d) rate of heat transfer to the working fluid in the
s1 = s2 = (sf + x sfg )@ 10 kPa boiler is MW,
6.5389 = 0.6492 + x (7.5010) (e) rate of heat rejection in the condenser,
6.5389 - 0.6492 (f) mass flow rate of cooling water in kg/h, if cooling
or x = = 0.785 water enters the condenser at 15°C and leaves at
7.5010
Specific enthalpy at the state 2 35°C.
h2 = (hf2 + xhfg 2)@ 10 kPa
Solution
= 191.81 + 0.785 ¥ 2392.82
= 2070.62 kJ/kg Given A power plant working on Rankine cycle with
The pump work; saturated steam
wp = vf (p1 – p2) Turbine entry;
= 0.001001 ¥ (3000 – 10) ª 3 kJ/kg p1 = 8.00 MPa (8000 kPa) as a saturated steam
Vapour Power Cycles 393

State 2: Wet steam;


p2 = 8 kPa
hf2 = 173.37 kJ/kg
hfg2 = 2403.38 kJ/kg
sf2 = 0.59088 kJ/kg ◊ K
sfg2 = 7.64028 kJ/kg ◊ K
State 3: Saturated liquid;
p3 = 8 kPa
h3 = hf3 = 173.94 kJ/kg
vf3 = 0.001008 m3/kg
State 4: Compressed liquid;
p4 = 8000 kPa,
h4 = h3 + wp
The state of steam after isentropic expansion in the
turbine;
s1 = s2 = (sf + x sfg )@ 8 kPa
5.7431 = 0.59088 + x (7.64028)
5.7431 - 0.59088
or x = = 0.674
7.64028
Specific enthalpy at the state 2
h2 = (hf2 + x hfg 2)@ 8 kPa
= 173.37 + 0.674 ¥ 2403.38
= 1794.09 kJ/kg
The pump work;
wp = vf (p1 – p2)
Condenser exit; = 0.001008 ¥ (8000 – 8) = 8.055 kJ/kg
p3 = 0.008 MPa = 8 kPa as a saturated liquid Enthalpy at the state 4;
Net power output, h4 = h3 + wp
P = 100 MW = 173.37 + 8.055 = 181.43 kJ/kg
Turbine work;
To find
wT = h1 – h2 = 2757.94 – 1794.09
(i) Thermal efficiency of Rankine cycle,
= 963.85 kJ/kg
(ii) The back work ratio,
Net work of cycle;
(iii) The mass flow rate of steam, kg/h,
wnet = wT – wp
(iv) Heat transfer rate to the working fluid in the = 963.85 – 8.055 ª 955.79 kJ/kg
boiler,
The heat supplied per kg of steam
(v) Heat rejection rate in the condenser, and
qin = h1 – h4 = 2757.94 – 181.43
(vi) The mass flow rate of cooling water for its
= 2576.51 kJ/kg
temperature change from 15°C to 35°C.
(i) Thermal efficiency of Rankine cycle
Analysis The properties of steam at principal states w 955.79
(From Table A-13) hRankine = net = = 0.370 or 37%
qin 2576.51
State 1: Saturated steam; (ii) Back work ratio;
p1 = 8000 kPa wp 8.055 kJ/kg
h1 = 2757.94 kJ/kg rbw = =
wT 963.85 kJ/kg
s1 = 5.7431 kJ/kg ◊ K = 0.00835
394 Thermal Engineering

(iii) The mass flow rate of steam; (iii) Rate of work transfer from the cycle,
Power output (iv) Work ratio,
ms =
wnet (v) Specific steam consumption.
100 ¥ 103 kW Assumptions
=
955.79 kJ/kg (i) Each components of the cycle is in steady state.
= 104.625 kg/s or 376 ¥ 103 kg/h (ii) All processes of the working fluid are internally
(iv) Heat supply rate to working fluid in the boiler reversible.
Qin = ms qin (iii) The turbine and pump operate isentropically
= (104.625 kg/s) ¥ (2576.51 kJ/kg) (q = 0).
= 269.57 ¥ 103 kW (iv) The kinetic and potential energy effects are
(v) Rate of heat rejection in the condenser; negligible.
Qout = ms (h2 – h3) Schematic
= 104.625 ¥ (1794.09 – 173.378)
= 169.566 ¥ 103 kW
(vi) The mass flow rate of cooling water in the
condenser
Qout = mw Cpw (Tout – Tin )
169.566 ¥ 103 = mw ¥ 4.180 ¥ (35 – 15)
169.556 ¥ 103
mw =
4.180 ¥ (35 - 15)
= 2028.3 kg/s or 7.30 ¥ 106 kg/h

Steam enters the turbine as 100% sat-


urated vapour at 6 MPa and saturated liquid enters the
pump at a pressure of 0.01 MPa. If the heat rate to boiler
is 150 MW, determine
(a) the thermal efficiency,
(b) the mass flow rate of the steam,
(c) net rate of work transfer from the cycle,
(d) work ratio,
(e) specific steam consumption.

Solution
Given A power plant works on Rankine cycle with
saturated steam
Turbine entry:
p1 = 6.00 MPa (6000 kPa) as a saturated steam
Condenser exit;
p3 = 0.01MPa (10 kPa) as a saturated liquid Analysis The properties of steam at principal states
Heat rate to boiler, State 1: Saturated steam;
Qin = 150 MW p1 = 6000 kPa,
To find h1 = 2785.1 kJ/kg
(i) Thermal efficiency of Rankine cycle, s1 = 5.8891 kJ/kg ◊ K
(ii) The mass flow rate of steam, kg/h,
Vapour Power Cycles 395

State 2: Wet steam; (c) Net rate of work transfer from the cycle
p2 = 10 kPa p = ms wnet
hf2 = 191.81 kJ/kg = 57.97 ¥ 915.8
hfg2 = 2392.82 kJ/kg = 53094.16 kW or 53.09 MW
sf2 = 0.6492 kJ/kg ◊ K (d) Work ratio
sfg2 = 7.5010 kJ/kg ◊ K w
rw = net =
915.8
= 0.993
State 3: Saturated liquid; wT 921.8
p3 = 10 kPa (e) Specific steam consumption
h3 = hf3 = 191.81 kJ/kg 3600 kJ/kWh
ssc =
vf3 = 0.001001 m3/kg wnet
State 4: Compressed liquid; p4 = 6000 kPa, 3600 kJ/kWh
h4 = h3 + wp = = 3.93 kg/kWh
915.8 kJ/kg
The state of steam after isentropic expansion in the
turbine;
s1 = s2 = (sf + xsfg )@ 10 kPa 12.5 COMPARISON BETWEEN
5.8891 = 0.6492 + x (7. 5010) CARNOT AND RANKINE CYCLES
5.8891 - 0.6492
or x = = 0.698
7.5010 Figure 12.12 shows the graphical comparison be-
Specific enthalpy at the state 2 tween Rankine and Carnot cycles on p–v and T–s
h2 = (hf2 + x hfg 2)@ 10 kPa diagrams.
= 191.81 + 0.698 ¥ 2392.82
Cycle 1–2–3¢– 4¢–1 Carnot cycle with saturated
= 1863.3 kJ/kg
steam
The pump work;
Cycle 1≤–2¢–3¢– 4¢–1 Carnot cycle with super-
wp = vf ( p1 – p2)
heated steam
= 0.001008 ¥ (6000 – 10) = 6.0 kJ/kg
Enthalpy at the state 4; Cycle 1–2–3–4–1 Rankine cycle with saturated
h4 = h3 + wp = 191.81 + 6.0 = 197.81 kJ/kg steam
Turbine work; Cycle 1¢–2¢–3–4–1 Rankine cycle with super-
wT = h1 – h2 = 2785.1 – 1863.3 heated steam
= 921.8 kJ/kg The following facts can easily be acknowledged
Net work of cycle; from the above two diagrams:
wnet = wT – wp = 921.8 – 6.0 1. In the Rankine cycle, liquid water is pumped
= 915.8 kJ/kg during the process 3–4. Since the specific
The heat supplied per kg of steam volume of the working substance after com-
qin = h1 – h4 plete condensation at the state 3 becomes
= 2785.1 – 197.81 = 2587.29 kJ/kg very small, therefore, the back work ratio in
(a) Thermal efficiency of Rankine cycle; a Rankine cycle is almost negligible. While
w 915.8 the specific volume of liquid and vapour
hRankine = net = = 0.354 or 35.4%
qin 2587.29 mixture at the state 3¢ is large, thus a large
(b) The mass flow rate of steam; compression work is required in Carnot va-
Heat rate pour power cycle.
ms =
qin 2. There are higher rates of heat transfer in the
150 ¥ 103 kW boiler and condenser due to long processes.
= = 57.97 kg/s
2587.29 kJ/kg Consequently, the Rankine cycle requires
= 208.71 ¥ 103 kg/h
396 Thermal Engineering

T a lower steam flow rate than Carnot cycle.


1¢ Therefore, the plant size for Rankine cycle
is smaller.
1
4¢ 1≤ 3. The Rankine cycle uses a part of heat sup-
plied at constant temperature. Therefore, its
4
efficiency is lower than that of Carnot cycle.
3 3¢ 2 2¢ 4. In a Rankine cycle, the steam is superheated
at constant pressure 1–1¢, thus maintaining
s the pressure ratio in the cycle unaffected.
(a) T–s diagram
If Carnot vapour power cycle is to operate
p
Saturation with superheated steam then the pressure of
curve
steam must be reduced to keep the tempera-
ture constant. It means the heat is transferred
4 4¢ 1¢ 1
1≤ to steam as it proceeds for the expansion
during process 1–1≤–2¢. Such a type heat ad-
TH dition is almost impractical.

3 3¢ 2¢ 2 TL
AND RANKINE CYCLES
v
(b) p–v diagram Although the Carnot vapour power cycle has maxi-
mum efficiency, still the Rankine cycle is com-
monly used in steam power plants. The differences
between the two cycles are tabulated below:

Sr. No Aspect Carnot Vapour Power Cycle Rankine Cycle


1. Cycle It is a reversible cycle. It is an irreversible cycle.
2. Cyclic efficiency It has theoretically maximum efficiency. It has less thermal efficiency than that
of the Carnot vapour power cycle.
3. Heat addition Heat is added at constant temperature. Heat is added at constant pressure.
4. Superheated steam Use of superheated steam is practically It uses superheated steam and performs
difficult. better.
5. Condensation Condensation is terminated before Complete condensation of steam takes
being saturated liquid. place.
6. State after The mixture of water and steam exits at Only liquid water exists after conden-
Condensation the termination point. sation.
7. Pump work It requires a large pump work to handle It requires negligible pump work to
the two phase mixture. handle the liquid water only.
8. Steam quality Since it uses saturated steam, the It uses superheated steam in the cycle
after expansion moisture content at the end of expansion at the end of expansion, the quality of
is much higher which can lead to blade steam is not objectionable.
erosion.
9. Standard of cycle It is just a theoretical cycle and cannot Almost all thermal plants operate on
be used in practice. Rankine cycle.
Vapour Power Cycles 397

The boiler produces dry and satu- sg = 6.1869 kJ/kg


rated steam at 30 bar. The steam expands in the turbine At p2 = 20 kPa;
to a condenser pressure of 20 kPa. Compare the cyclic vf = 0.001017 m3/kg
work done and thermal efficiency of Carnot and Rankine TL = 60.06°C
cycles for these conditions. hf = 251.38 kJ/kg
hfg = 2358.33 kJ/kg
Solution
sf = 0.8319 kJ/kg ◊ K
Given Dry saturated steam pressure sfg = 7.0766 kJ/kg ◊ K
Boiler pressure p1 = 30 bar The specific enthalpies
Condenser pressure; p2 = 20 kPa h1 = 2804.2 kJ/kg
h4 = 1008.42 kJ/kg
To find
Carnot Cycle
(i) Work done by Carnot cycle,
(ii) Work done by Rankine cycle, Analysis
(iii) Thermal efficiency of Carnot cycle, The efficiency of Carnol cycle is given by
(iv) Thermal efficiency of Rankine cycle. TL (60.06 + 273)
hCarnot = 1 - =1-
Properties of steam TH ( 233.9 + 273)
At p1 = 30 bar; = 0.343 or 34.3%
hf = 1008.42 kJ/kg The heat supplied per kg of steam
hg = 2804.2 kJ/kg qin = h1 – h4 = 2804.2 – 1008.42
TH = 233.9°C = 1795.78 kJ/kg
T T

4 30 bar 1
TH Saturation p1 = 30 bar 1
curve

20 kPa
TL 2
3 4
p2 = 20 kPa
s 3 2

s
s3 s1
h
(kJ/kg)
h ar
(kJ/kg) ba
r 0b
0 =3
=3 p1
p1
1
1 2804.2
2804.2
Pa
20 k
k pa p2=
4 p2 = 20 4
1008.42
h2 2
2
251.38
3
3
s s
(a) T–s and h–s diagram for Carnot Cycle (b) T–s and h–s diagram for Rankine Cycle
398 Thermal Engineering

The work done per kg of steam


wnet = hCarnot qin = 0.343 ¥ 1795.78
= 615.86 kJ/kg
Rankine Cycle
Condition of steam at the state 2
s1 = s2 = (sf + x sfg )@20 kPa
6.1869 = 0.8319 + x ¥ (7.0766)
or x = 0.8319
Specific enthalpy at principal states;
State 1: h1 = 2804.2 kJ/kg
State 2: h2 = (hf + x hfg )@20 kPa
= 251.38 + 0.8319 ¥ 2358.33
= 2035 kJ/kg ◊ K
State 3: h3 = hf = 251.38 kJ/kg
The pump work;
wp = vf ( p1 – p2)
= 0.001017 ¥ (3000 – 20) = 3.03 kJ/kg
State 4: h4 = h3 + wp = 251.38 + 3.03 = 254.41 kJ/kg
Turbine work per kg of steam;
wT = h1 – h2 = 2804.2 – 2035 = 769.2 kJ/kg
Net work done by Rankine cycle
wnet = wT – wp = 769.2 – 3.03 = 766.17 kJ/kg
Heat supplied in Rankine cycle transfer decrease the availability of steam entering
qin = h1 – h4 = 2804.2 – 254.41 the turbine. However, these effects are negligible,
= 2549.79 kJ/kg and not shown in Fig. 12.14.
Thermal efficiency of Rankine cycle
Actual expansion process 1–2 in
w 766.17 the turbine is an irreversible process as shown in
hRankine = net = = 0.302 or 30.2%
qin 2549.79 Fig. 12.14. The entropy inceases during the actual
expansion process. Some heat may also transfer to
the surroundings. Thus the actual work developed
in the turbine is less than the work corresponding to
IN VAPOUR POWER CYCLE
isentropic process 1–2s. The isentropic efficiency
The Rankine cycle considered in Section 12.3 is an of the turbine is given as
ideal vapour cycle. In actual practice, all four pro- Actual work Actual enthalpy drop
hT = =
cesses of the cycle involve irreversibility and losses Isentropic work Isentropic enthalpy drop
and therefore, the efficiency of the actual vapour h1 - h2
= ...(12.18)
cycle is less than that of an ideal Rankine cycle. h1 - h2s
The losses associated with actual processes are
The pump losses are very simi-
shown in Fig. 12.14.
lar to turbine lossses. The actual compression
Piping Losses When the working fluid passes process 3–4 in the pump is an irreversible process.
through the tubes of the boiler and condenser, the The entropy increases during the actual compres-
pressure drops due to frictional effects and transfer sion process. Therefore, the actual work input in
of heat to surroundings. Both pressure drop and heat the pump is more than the work corresponding to
Vapour Power Cycles 399

the isentropic process 3–4s. The isentropic effi- h1 = 2960.68 kJ/kg,


ciency of the pump is given as s1 = 6.3614 kJ/kg ◊ K
Isentropic work input State 2: Wet steam; Table A-13
hp = p2 = 3.5 kPa
Actual work input
hf = 111.80 kJ/kg,
Isentropic enthalpy drop hfg = 2438.6 kJ/kg
=
Actual enthalpy drop sf = 0.3910 kJ/kg ◊ K
h4s - h3 sfg = 8.133 kJ/kg ◊ K
= ...(12.19)
h4 - h3 State 3: p2 = 3.5 kPa,
hf = 111.80 kJ/kg,
The condenser losses are rela- vf = 0.001003 m3/kg
tively small. The cooling of condensate below satu- State 4: Compressed liquid, p1 = 40 bar
ration temperature may be possible in the condens-
(a) For an ideal Rankine cycle 1–2s–3–4–1
er. Some additional heat is required to bring down
The state 2, after isentropic expansion can be
the saturation temperature of water. defined by equating entropy at states 1 and 2s;
Further, the air enters into the condenser with s1 = s2s = (sf + x sfg )@ 3.5 kPa
exhaust steam and from joints of the condenser. It 6.3614 = 0.3910 + x (8.133)
reduces the back pressure in the condenser. An air 6.3614 - 0.3910
pump is required to maintain the correct vacuum or x = = 0.734
8.133
in the condenser. Condenser losses are also small,
thus not incorporated in T–s and h–s diagrams of Schematic
qin
Fig. 12.14.
40 bar
4 Boiler 1
Example 12.11 A steam power plant operates on an 300°C
ideal Rankine cycle between a boiler pressure of 40 bar,
300°C and a condenser pressure of 0.035 bar. Calculate
cycle efficiency, work ratio, and specific steam consump- Pump Turbine Wnet

tion for Wp

(a) Ideal Rankine cycle


3.5 kPa
(b) For Rankine cycle, when expansion process has 3 Condenser 2

an isentropic efficiency of 80% qout


(a) Schematic diagram
Solution
(i) Cycle 1–2s–3–4–1 is an ideal Rankine cycle
Given A steam power plant operating on Rankine cycle
T
Boiler conditions; p1 = 40 bar, 300°C
Condenser pressure; p2 = 0.035 bar = 3.5 kPa 1
300°C
p1 = 40 MPa
To find
(i) Cycle efficiency,
4
(ii) Work ratio, and p2 = 3.5 kPa

(iii) Specific steam consumption. 3 2s

Analysis The properties of steam s


(b) T-s diagram
State 1: Superheated steam; Table A-14
p1 = 4000 kPa (ii) Cycle 1–2–3–4–1 is an irreversible Rankine cycle
T1 = 300°C Fig. 12.15
400 Thermal Engineering

Specific enthalpy after isentropic expansion at the Isentropic efficiency of the turbine is given as
state 2s; h -h wT
hT = 1 2 =
h2s = (hf + xhfg )@ 3.5 kPa h1 - h2s h1 - h2s
= 111.80 + 0.734 ¥ 2438.6 Actual turbine work;
= 1901.73 kJ/kg wT = hT (h1 – h2s )
The pump work; = 0.8 ¥ (2960.68 – 1901.73)
wp = vf (p1 – p2) = 847.48 kJ/kg
= 0.001003 ¥ (4000 – 3.5) Net work done per kg of steam;
= 4.0 kJ/kg wnet = wT – wp = 847.48 – 4.0
Enthalpy at the state 4; = 843.48 kJ/kg
h4 = h1 + wp (i) Cycle efficiency
= 111.80 + 4.0 = 115.80 kJ/kg wnet 843.48
Turbine work, hCycle = =
qin 2844.88
wT = h1 – h2s = 0.296 or 29.6%
= 2960.68 – 1901.73 w 843.48
= 1058.95 kJ/kg (ii) Work ratio; rw = net =
wT 847.48
Heat supplied in the boiler;
= 0.9952 or 99.52%
qin = h1 – h4 = 2960.68 – 115.80
(iii) Specific steam consumption rate
= 2844.88 kJ/kg
3600 kJ/kWh 3600 kJ/kWh
Net work done per kg of steam, ssc = =
wnet kJ/kg 843.48 kJ/kg
wnet = wT – wp
= 4.268 (kg/kWh)
= 1058.95 – 4.0 = 1054.95 kJ/kg
(i) The Rankine cycle efficiency
A steam power plant operates on a
w 1054.95 cycle. The pressure and temperature are designated in
hRankine = net =
qin 2844.88 Fig.12.16. The turbine efficiency is 90% and pump ef-
= 0.370 or 37% ficiency is 85%. Calcualte the thermal efficiency of the
cycle.
(ii) Work ratio;
wnet 1054.95 Solution
rw = =
wT 1058.95 Given A steam power plant with schematic as shown
= 0.996 or 99.6% in Fig.12.16;
(iii) Specific steam consumption rate
Properties of steam From Tables A-13, A-14
3600 kJ/kWh 3600 kJ/kWh
ssc = = State 1: At 3.8 MPa, 380°C; Superheated steam;
wnet kJ/kg 1054.95 kJ/kg
h1 = 3169.1 kJ/kg
= 3.41 (kg/kWh)
s1 = 6.7235 kJ/kg ◊ K
(b) For an irreversible Rankine cycle 1–2–3–4–1, we
have State 2: At 10 kPa, wet steam;
h1 = 2960.68 kJ/kg hf = 191.8 kJ/kg
h2s = 1901.73 kJ/kg hfg = 2392.8 kJ/kg
wp = 4◊0 kJ/kg sf = 0.6493 kJ/kg ◊ K
h4 = 115.80 kJ/kg sfg = 7.5009 kJ/kg
qin = 2844.88 kJ/kg State 3: At 10 kPa saturated liquid vf = 0.001009 m3/kg
hT = 0.8 State 4: At 5 MPa; compressed liquid
Vapour Power Cycles 401

6.7235 = 0.6492 + x (7.5010)


6.7235 - 0.6492
or x = = 0.8098
7.5010
Specific enthalpy after isentropic expansion at state 2s
h2s = (hf + x hfg )@ 10 kPa
= 191.8 + 0.8098 ¥ 2392.8
= 2129.5 kJ/kg
The isentropic efficiency of the turbine is given as
h -h wT
hT = 1 2 =
h1 - h2s h1 - h2 s
Actual turbine work per kg of steam;
wT = hT (h1 – h2s ) = 0.9 ¥ (3169.1 – 2129.5)
= 935.65 kJ/kg
Net work done in the cycle per kg of steam
wnet = wT – wp
= 935.65 – 5.92 = 929.73 kJ/kg
The heat supplied in the boiler per kg of steam
qin = h6 – h5
= 3213.6 – 171.8 = 3041.8 kJ/kg
The thermal efficiency of the cycle
w 929.73
State 5: At 4.8 MPa, 40°C Compressed liquid hcycle = net = = 0.3056 or 30.56%
qin 3041.8
h5 = hf @ 40°C = 191.8 kJ/kg
State 6: At 4 MPa, 400°C Superheated steam
h4 = 3213.6 kJ/kg
To find Thermal efficiency of the cycle.
VARIABLES ON RANKINE CYCLE

Assumptions
(i) Each component in the cycle is analysed as a The steam enters the condenser as a saturated mix-
control volume at steady state. ture of vapour and moisture at the turbine back
(ii) Compression and expansion are considered pressure p2. If this pressure is lowered, the satura-
adiabatic. tion temperature of exhausted steam decreases, and
(iii) Kinetic and potential energy effects are negligible. thus, the amount of heat rejection in the condens-
Analysis The isentropic enthalpy drop during er also decreases. The efficiency of the Rankine
process 3–4s; cycle increases by lowering the exhaust pressure.
h4s – h3 = vf ( p1 – p2) Figure 12.17 illustrates the effect of condenser
= 0.001009 ¥ (5000 – 10) = 5.035 kJ/kg pressure on the Rankine cycle.
The isentropic efficiency of the pump As turbine back pressure p2 decreases to p 2¢, the
h -h h -h heat rejection decreases by an area 2–3–3¢–2¢–2.
hp = 4s 3 = 4s 3
h4 - h3 wp The heat transfer to steam is also increased by an
Actual pump work per kg of steam; area a¢–4¢– 4–a–a¢. Thus, the net work done and
h -h 5.035 efficiency of the cycle increases.
wp = 4s 3 = = 5.92 kJ/kg
hp 0.85 However, there are limitations to this method.
The state 2s, after isentropic expansion can be defined These are
by equating entropy at states 1 and 2s; 1. Lowering the back pressure causes an
s1 = s2 s = (sf + xsfg )@ 10 kPa increase in moisture content of the steam
402 Thermal Engineering

leaving the turbine. It is an unfavourable


factor, because, if the moisture content of
steam in low-pressure stages of the turbine
exceeds 10%, there is a decrease in turbine
efficiency and erosion of turbine blade may
also be a very serious problem.
2. To maintain the high vacuum, the air ex-
traction pump will run continuously and its
work input will increase, thus reducing the
useful work.

Superheating of steam increases the mean tem- By increasing the boiler pressure, the mean tem-
perature of heat addition. The effect of superheated perature of heat addition increases, and thus raises
steam on the performance of the Rankine cycle is the thermal efficiency of the cycle. Figure. 12.19(a)
shown in Fig. 12.18(a). The increase in superheat is illustrates the effect of boiler pressure on Rankine
shown by the line 1–1¢. The hatched area 1–1¢–2¢– cycle efficiency.
2–1 represents an increase in net work done during By keeping the maximum temperature Tmax and
the cycle. The area under the curve 1–1¢ represents condenser pressure p2 constant if boiler pressure
increase in the heat input. Thus, both the net work increases, the heat rejection decreases by an area
done and heat transfer increase as a result of super- b¢–2¢–2–b–b¢. The net work done by the cycle
heating the steam to higher temperature. Therefore, remains almost same, thus, the Rankine cycle
the thermal efficiency of the Rankine cycle increas- efficiency increases, with an increase in maximum
pressure.
es. It is observed that the specific steam consump-
tion decreases as steam is superheated. The effect Figure 12.19(b) shows the variation of efficiency
of superheating is shown in Fig.12.18(b). and specific steam consumption with boiler pres-
sure. The graph shows that the thermal efficiency
Superheating of steam to higher temperature is
first increases, reaches to peak value and then de-
desirable, because the moisture content of steam
creases.
leaving the turbine decreases as indicated by the
state 2¢ in Fig. 12.18 (a). However, the metallurgical 1. As boiler pressure increases, the saturation
considerations, restrict the superheating of steam to temperature of feed water increases and the
a very high temperature. enthalpy of vapourisation reduces. Thus a
Vapour Power Cycles 403

Table 12.1

Sr No. Operating Work done Efficiency


variable
1. Decrease in Increases Increases
exhaust pressure
2. Increase in boiler No effect Increases
pressure
3. Superheating Increases Increases
steam

In a steam power plant operating


on an ideal Rankine cycle, the steam enters the turbine
at 3 MPa and 400°C and it is exhausted at 10 kPa.
Determine
(a) Thermal efficiency
(b) Thermal effeciency, if the steam is superheated to
500°C at 3 MPa, before it enters the turbine
(c) Thermal efficiency, if steam enters the turbine at
10 MPa and 400°C

Solution
Given An ideal Rankine cycle with different operating
variables
larger portion of heat is used to increase the To find Thermal efficiency of Rankine cycle in each
temperature of feed water to its saturation case
temperature. This non-isothermal process
Schematic
increases irreversibility and thus thermal ef-
Case (a): Turbine inlet: 3 MPa, 400°C: Cycle 1–2–3–4–1
ficiency decreases.
Case (b): Turbine inlet 3 MPa, 500°C: Cycle 1≤–2≤–3–
2. With increase in boiler pressure, the cycle 4–1≤
shifts toward left and the moisture contents Case (c): Turbine inlet 10 MPa, 400°C: Cycle 1¢–2¢–3–
of exhaust steam increases. It is an undesir- 4–1¢
able effect.
3. Specific steam consumption also decreases
first and then increases after reaching a mini-
mum level at 160 bar.
We can conclude that the efficiency of Rankine
cycle can be increased by lowering the condenser
pressure, by increasing the boiler pressure and by
superheating the steam. The quality of steam leav-
ing the turbine decreases by lowering condenser
pressure and by increasing boiler pressure, while it
improves by superheating. Table 12.1 presents the
summary of the above discussion.
404 Thermal Engineering

Analysis Case (b) The steam supplied to the turbine at 3 MPa and
Case (a): Steam enters the turbine at 3 MPa and 400°C. 500°C
Ideal Rankine cycle 1-2-3-4-1 Rankine cycle 1≤–2≤–3–4–1≤.
From steam tables, The properties of steam at states 3 and 4 will remain
State 1: p1 = 3 MPa, same but properties at states 1≤ and 2≤ will be different.
T3 = 400°C: Superheated steam State 1: Superheated steam at 3 MPa nd 500°C
h1 = 3230.9 kJ/kg h1≤ = 3456.5 kJ/kg
s1 = 6.9212 kJ/kg ◊ K s1≤ = 7.2338 kJ/kg ◊ K
State 2: Wet steam For the state 2≤
p2 = 10 kPa, s1≤ = s2≤ = (sf + x≤ sfg )@ 10 kPa
hf = 191.83 kJ/kg 7.2338 = 0.6492 + x (7. 5010)
hfg = 2392.87 kJ/kg 7.2338 - 0.6492
or x≤ = = 0.878
sf = 0.6492 kJ/kg 7.5010
sfg = 7.5010 kJ/kg ◊ K h2≤ = (hf + x≤ hfg )@ 10 kPa
State 3: Saturated liquid, = 191.81 + 0.878 ¥ 2392.82
p3 = 10 kPa = 2292.77 kJ/kg
h3 = hf = 191.83 kJ/kg Heat supplied per kg;
vf = 0.001010 m3/kg qin = h1≤ – h4 = 3456.50 – 194.85
State 4: Compressed liquid, = 3261.65 kJ/kg
p4 = 3 MPa Heat rejected per kg
The state 2 of steam after isentropic expansion in the qout = h2≤ – h3 = 2292.77 – 191.83 = 2100.94
turbine; q 2100.94
and hplant = 1 - out = 1 -
s1 = s2 = (sf + x sfg )@ 10 kPa qin 3261.65
6.9212 = 0.6492 + x (7. 5010) = 0.3559 or 35.59%
6.9212 - 0.6492 Comment: The increase in superheating temperature
or x = = 0.836
7.5010 results into increase in thermal efficiency of the plant as
Specific enthalpy at the state 2 well steam quality after expansion in the turbine.
h2 = (hf + x hfg )@ 10 kPa Case (c) With increase in boiler pressure to 10 MPa and
= 191.81 + 0.836 ¥ 2392.82 400°C
= 2192.27 kJ/kg Rankine cycle 1¢–2¢–3–4¢–1¢
The pump work; State 1: h1¢ = 3096.5 kJ/kg
wp = vf ( p1 – p2) s1¢ = 6.2120 kJ/kg ◊ K
= 0.001010 ¥ (3000 – 10) = 3.02 kJ/kg State 2: After isentropic expansion;
Enthalpy at the state 4; 6.2120 - 0.6492
h4 = h3 + wp = 191.83 + 3.02 = 194.85 kJ/kg x¢ = = 0.742
7.5010
Heat supplied in the boiler per kg of steam; h2¢ = (hf + x¢ hfg )@ 10 kPa
qin = h1 – h4 = 3230.90 – 194.85 = 191.83 + 0.742 ¥ 2392.87
= 3036.05 kJ/kg = 1967.34 kJ/kg
Heat rejected in the condenser per kg of steam; and h4¢ = h3 + vf (p1 – p2)
qout = h2 – h3 = 2192.27 – 191.83 = 191.83 + 0.001010 ¥ (10000 – 10)
= 2000.44 kJ/kg = 201.92 kJ/kg
The Rankine or plant efficiency qin = h1¢ – h4¢ = 3096.5 – 201.92
q 2000.44 = 2894.58 kJ/kg
hplant = 1 - out = 1 -
qin 3036.05 qout = h2¢ – h3 = 1967.34 – 191.83
= 0.3411 or 34.11% = 1775.51 kJ/kg
Vapour Power Cycles 405

qout 1775.51 In a reheat Rankine cycle, the steam is expanded


hplant = 1 - =1-
qin 2894.58 in a number of stages. After each stage of expan-
= 0.3866 or 38.66% sion, the steam is reheated in the boiler. Then, it
expands in the next stage of turbine and is finally
Comment: The increase in boiler pressure results in an
exhausted to the condenser.
increase in plant efficiency from 34.11% to 38.66%
Figure 12.21 shows the reheating process sche-
matically. Figures 12.22(a) and (b) show the reheat-
ing process 2–3 on T–s and h–s (Mollier) diagrams,
respectively. The steam at state 1 enters in the first
If the steam expands completely in a single stage stage of turbine and expands isentropically to the
then steam coming out the turbine is very wet. The state 2. The quality of steam at the state 2, is either
wet steam carries suspended moisture particles, just dry or slightly wet and thus it is taken back in
which are heavier than the vapour particles, thus the boiler and is reheated to original superheat tem-
deposited on the blades and causing its erosion. In perature T3 (= T1) at constant pressure p2. Then this
order to increase the life of the turbine blades, it is reheated steam at the state 3 enters the next stage
necessary to keep the steam dry during its expan- turbine and further expands to back pressure at the
sion. It is done by allowing the steam to expand to state 4.
an intermediate pressure in a high-pressure turbine,
and then taking it out and sending back to the boil- T Ist Stage expansion
er, where it is reheated at constant pressure, until 1 3
it reaches the inlet temperature of the first stage as

ting

IInd stage expansion


hea
shown in Fig. 12.21. This process is called reheat- Vaporisation

Re
ing during which heat is added to the steam. The re-
heated steam then further expands in the next stage
2
of the turbine. Due to reheating, the work output of 6
the turbine increases, thus improving the thermal Condensation
efficiency. 5 4

s
The reheat cycle is designed to take advantage of (a) T–s diagram
higher boiler pressure by eliminating the problem
h p1 p2
of excessive moisture content in the exhaust steam. Ist stage
expansion 3 T = Const
Superheated steam 1
g
tin

1
expansion
IInd stage
a

HP LP
he

Turbine Turbine
Re

WT
2 3 4
2
p3
Boiler Reheater
6 4

6
Feed Condenser 5
Pump
water
Wp qout s
(b) h–s diagram
406 Thermal Engineering

The amount of heat added during reheating its part is utilized to overcome the frictional resis-
qreheat = h3 – h2 tances. Thus, the kinetic energy produced is less
The total heat supplied per kg in two stages than that corresponding to theoretical isentropic
qin = (h1 – h6) + (h3 – h2) ...(12.20) enthalpy drop. Further, this friction is converted
For isentropic expansion in two stages, the total into heat, consequently, the steam becomes dry and
work done per kg of steam saturated, even superheated. This frictional heating
wT = (h1 – h2) + (h3 – h4) causes an increase in entropy and hence actual en-
thalpy drop is always less than the isentropic en-
The pump work per kg of steam;
p1
thalpy drop.
wp = h6 – h5 = - Ú p3
vdp Let us consider the expansion of steam in three-
stage turbine as shown in Fig. 12.23 on the h –s dia-
The net work done per kg of steam;
gram. The superheated steam initially at pressure
wnet = wT – wp p1 expands through three stages to exhaust pressure
= (h1 – h2) + (h3 – h4) – (h6 – h5) p4. The isentropic expansion from pressure p1 to p2
...(12.21) in the first stage of expansion is represented by a
p1
However, the pump work - Ú p3
vdp is very vertical line 1–2s. Due to friction and irreversibili-
ties, the actual state after expansion is 2 instead of
small in comparasion with turbine work, thus it is 2s. Therefore, the actual enthalpy drop (h1 – h2 ) is
neglected in most of the cases. less than the isentropic enthalpy drop (h1 – h2s ) and
The heat rejected in the condenser per kg of the difference between them (h2 – h2s ) is the loss
steam; due to irreversibilities, and thus the first stage ef-
qout = h4 – h5 ...(12.22) ficiency of the turbine can be expressed as
Then the efficiency of the turbine with reheating h - h2
is given by hstage I = 1
h1 - h2s
w q
hreheat = net = 1 - out Actual enthalpy drop
qin qin = ...(12.24)
Isentropic enthalpy drop
h4 - h5
= 1- ...(12.23)
( h1 - h6 ) + ( h3 - h2 ) The stage efficiency is identical to isentropic ef-
ficiency. If stage efficiency is known, then
It is evident from the T–s diagram Fig. 12.22(a)
h2 = h1 – hstage I (h1 – h2s ) ...(12.25)
that there is very less gain in thermal efficiency by
reheating the steam, only the quality of exhausted
steam is improved. However, the mean tempera-
ture of heat addition can be increased by including
the number of expansion and reheating processes.
Thus, the thermal efficiency of the cycle would fur-
ther increase.

In actual, when steam expands through the turbine,


a considerable friction is always involved when
steam glides over the blades. This friction resists
the flow of steam. The isentropic enthalpy drop is
not fully converted into kinetic energy but some of
Vapour Power Cycles 407

The locus of h2 at pressure line p2 can be marked imposed by the materials used to fabricate the su-
at state 2 on the h –s diagram as shown in Fig. 12.23. perheater, reheater and turbine. High pressure in the
The second stage isentropic expansion takes boiler also requires the piping that can withstand
place from the state 2 to 3s and actual enthalpy the stresses at elevated temperatures, Although
drop is these factors limit the gain that can be achieved by
h2 – h3 = hstage II (h2 – h3s ) superheating and reheating.
or h3 = h2 – hstage II (h2 – h3s ) ...(12.26) The operating pressures in the boilers have
The locus of h3 can also be marked in a similar gradually increased with improved materials and
way on the pressure line p3. Then steam expands methods of fabrication over the years. The higher
from state 3 to 4, and its actual enthalpy drop is operating pressures and maximum allowed cycle
h3 – h4 = hstage III (h3 – h4s ) temperatures have permitted a significant increase
in thermal efficiency of the cycle.
or h4 = h3 – hstage III (h3 – h4s ) ...(12.27)
If a vapour power plant is designed to operate
The sum of all isentropic enthalpy drops is re-
with boiler pressures exceeding the critical pressure
ferred as cumulative isentropic enthalpy drop and
of water (22.1 MPa) and turbine inlet temperature
is designated as hcum.
exceeding 600°C, such cycle is referred as the super
hcum = (h1 – h2s ) + (h2 – h3s ) + (h3 – h4s )
critical Rankine cycle as shown in Fig. 12.24. The
...(12.28)
Fig. 12.25 shows an ideal reheat cycle with super
If there was no irreversibility present during
critical steam pressure. At super critical pressure
expansion of steam, then steam would expand is-
entropically through all stages from pressure p1 to
pressure p4 as shown by the line 1–5s. The enthalpy
drop during this expansion is h1 – h5s referred as
isentropic enthalpy drop.
The ratio of cumulative isentropic enthalpy drop
to isentropic enthalpy drop from initial pressure to
final pressure is called the reheat factor and is given
by
hcum
RF =
h1 - h5s
h1 - h2 s + h2 - h3s + h3 - h4s
= ...(12.29)
h1 - h5s
The value of the reheat factor is always greater
than unity and its effect is to increase the final en-
thalpy drop through friction heating, so the turbine
efficiency improves in the same ratio. In actual
case, the efficiency gain due to friction is very less.
The turbine efficiency can be thus given by
hturbine = hstage ¥ RF ...(12.30)

The temperature of superheated steam entering the


turbine is restricted by metallurgical constraints
408 Thermal Engineering

(greater than 22.1 MPa), no phase change occurs (ii) Thermal efficiency of the cycle, and
during heat addition process 2–3. (iii) Steam rate in kg/kWh.
Properties of steam From Mollier chart
An steam power plant operates on a
theoretical reheat cycle. The steam from boiler at 150 bar At state 1: Superheated steam
and 550°C expands through the high-pressure turbine. It p1 = 150 bar
is reheated at constant pressure of 40 bar to 550°C and T1 = 550°C
expands through the low pressure turbine to a condenser h1 = 3450 kJ/kg
pressure of 0.1 bar. Draw T –s and h –s diagrams and find State 2: After isentropic expansion, the steam is also
(a) quality of steam at turbine exhaust, superheated
(b) Thermal efficiency of the cycle, p2 = 40 bar
(c) Steam rate in kg/kWh. T2 = 335°C
h2 = 3052 kJ/kg
Solution State 3: Superheated steam
Given A reheat Rankine cycle with p2 = 40 bar
p1 = 150 bar T1 = 550°C T3 = 550°C
p2 = 40 bar T3 = 550°C h3 = 3560 kJ/kg
p3 = 0.1 bar State 4: Wet steam
p3 = 10 kPa
To Find
x4 = 0.88
(i) Quality of steam at turbine exhaust, h4 = 2300 kJ/kg
State 5: Saturated liquid (from saturated steam table)
p3 = 10 kPa
vf = 0.001010 m3/kg
hf = 191.83 kJ/kg
State 6: Compressed liquid p1 = 150 bar
Analysis
(i) Quality of steam at turbine exhaust
Using Mollier (h –s) chart; draw a vertical straight
line 3–4 from point 3 (coordinates: 40 bar and
550°C) to a pressure line of 0.1 bar as shown in
h–s diagram of Fig. 12.26. The quality of steam at
intersection point 4 as
x = 0.88
(ii) Thermal efficiency of the cycle
The pump work;
wp = vf (p1 – p3)
= 0.001010 ¥ (150 ¥ 102 – 10)
= 15.14 kJ/kg
Enthalpy at the state 6;
h6 = h5 + wp
= 191.93 + 15.14 = 206.97 kJ/kg
The heat supplied per kg of steam
qin = h1 – h6 + h3 – h2
= 3450 – 206.97 + 3560 – 3052
= 3751.03 kJ/kg
Vapour Power Cycles 409

Turbine work per kg of steam;


wT = h1 – h2 + h3 – h4
= 3450 – 3052 + 3560 – 2300
= 1658 kJ/kg
wnet = wT – wp = 1658 – 15.14
= 1642.86 kJ/kg
Thermal efficiency;
wnet 1642.86
hcycle = = = 0.438 or 43.8%
qin 3751.03
(iii) Steam rate in kg/kWh
3600 3600
ssc = =
wnet 1642.86
= 2.19 kg/kWh

The steam is supplied to a turbine


at a pressure of 32 bar and a temperature of 410°C.
The steam then expands isentropically to a pressure of
0.08 bar. Find the dryness fraction of steam at the end of
expansion and thermal efficiency of the cycle.
If the steam is reheated at 5.5 bar to a temperaure of
395°C and then expands isentropically to 0.08 bar, what
will be the dryness fraction and thermal efficiency of the
cycle?

Solution
Given A steam power plant with superheated steam (iv) The kinetic and potential energy effects are
with and without reheating. negligible.
Turbine entry: p1 = 32 bar (3200 kPa) and 410°C as Properties of steam From Mollier chart
superheated steam At state 1: Superheated steam
Reheating; p2 = 5.5 bar (550 kPa) and 395°C p1 = 32 bar
Condenser exit; p3 = 0.08 bar (8 kPa) as a saturated T1 = 410°C
liquid h1 = 3250 kJ/kg
To find For simple Rankine cycle; State 3: Superheated steam after reheating
(i) Dryness fraction at the turbine exit, p2 = 5.5 bar
(ii) Thermal efficiency of cycle, T3 = 395°C
For reheat Rankine cycle, h3 = 3263 kJ/kg
(iii) Dryness fraction at the turbine exit, and State 5: Saturated liquid
(iv) Thermal efficiency of cycle. p3 = 8 kPa
vf3 = 0.001008 m3/kg
Assumptions
h5 = hf3 = 173.94 kJ/kg
(i) Each component of the cycle is in steady state. State 4: Compressed liquid p1 = 8000 kPa
(ii) All processes of working fluid are internally
reversible. Analysis
(iii) The turbine and pump operate isentropically (i) For simple Rankine cycle: Fig. 12.27(a) and
(q = 0). cycle 1 –4–5 –6;
410 Thermal Engineering

(a) Dryness fraction after isentropic expansion = 3250 – 177.16 + 3263 – 2426
in a single stage: = 3528.88 kJ/kg
Using Mollier (h–s) chart; draw a vertical The heat rejected per kg of steam
straight line 1–4 from the point 1 (coordi- qin = h4 – h5
nates: 32 bar and 410°C) to a pressure line = 2426 – 173.94 ª 2252 kJ/kg
of 0.08 bar as shown in h–s diagram of Thermal efficiency;
Fig. 12.27(a). The quality of steam at q 2252
intersection point 4 as hcycle = 1 - out = 1 -
qin 3528.88
x4 = 0.83 = 0.3618 or 36.18%
and enthalpy at the state 4
h4 = 2170 kJ/kg A steam power plant operates on
(b) Thermal efficiency of the cycle ideal reheat cycle. The steam enters the high pressure
The pump work; turbine at 150 bar and 600°C and after expansion in two
wp = vf (p1 – p3) stages, is exhausted at 10 kPa. If the moisture content
= 0.001010 ¥ (32 – 0.08) ¥ 102 of the exhausted steam should not exceed 10.4 percent,
= 3.22 kJ/kg determine
Enthalpy at the state 6; (a) the pressure at which the steam should be
h6 = h5 + wp = 173.94 + 3.22 reheated
= 177.16 kJ/kg (b) thermal efficiency of the cycle
The heat supplied per kg of steam Assume the steam is reheated to high pressure turbine
qin = h1 – h6 inlet temperature.
= 3250 – 177.16 = 3072.84 kJ/kg
Solution
The heat rejected per kg of steam
qin = h4 – h5 Given A reheat Rankine cycle with
= 2170 – 173.94 ª 1996 kJ/kg HP turbine inlet: p1 = 150 bar
Thermal efficiency; T1 = 600°C
q 1996 LP turbine inlet: T3 = 600°C
hcycle = 1 - out = 1 -
qin 3072.84 Condenser inlet: p3 = 10 kPa
x4 = 1 – 0.104 = 0.896
= 0.350 or 35.0%
(ii) For reheat Rankine cycle: To Find
Cycle 1–2 –3– 4 –5 – 6; Fig. 12.27(b) (i) Intermediate pressure of reheat.
(c) Dryness fraction after isentropic expansion (ii) Thermal efficiency of the reheat cycle.
in low pressure turbine Assumptions
Using Mollier (h –s) chart; draw a vertical (i) Each component of the cycle is in steady state.
straight line 3–4 from point 3 (coordinates:
(ii) All processes of working fluid are internally
5.5 bar and 395°C) to a pressure 0.08 bar as
reversible.
shown in h–s diagram of Fig. 12.27 (b). We
(iii) The turbine and pump operate isentropically (q =
get quality of steam at state 4 as
0).
x4 = 0.933
(iv) The kinetic and potential energy effects are
and enthalpy at state 4 negligible.
h4 = 2426 kJ/kg
(d) Thermal efficiency of the cycle Properties of steam
The heat supplied per kg of steam At state 1: Superheated steam: (from Mollier chart)
qin = h1 – h6 + h3 – h2 p1 = 150 bar
T1 = 600°C
Vapour Power Cycles 411

Analysis
(i) The intermediate pressure p2
Since the process 3– 4 is an isentropic process,
thus
s3 = s4
where s4 = (sf + x4 sfg )@10 kPa
= 0.6492 + 0.896 ¥ 7.5010
= 7.370 kJ/kg ◊ K
Hence s3 = 7.370 kJ/kg ◊ K
Using Mollier (h –s) chart; draw a vertical
straight line 3–4 from the point 4 (0.1 bar and
s3 = 7.370 kJ/kg ◊ K) to a temperature line of T3 =
600°C, as shown in h–s diagram of Fig. 12.28(b).
At intersection point, the pressure and enthalpy
found to be
p2 = p3 = 40 bar
h3 = 3675 kJ/kg
Therefore, the steam should be reheated at 40 bar.
(ii) Thermal efficiency of the cycle
The pump work;
wp = vf ( p1 – p3)
= 0.00101 ¥ (150 ¥ 102 – 10)
= 15.14 kJ/kg
Enthalpy at the state 6;
h6 = h5 + wp = 191.93 + 15.14
= 206.97 kJ/kg
Specific enthalpy at the state 4
h4 = (hf + x4 hfg )@ 10 kPa
= 191.83 + 0.896 ¥ 2392.82
h1 = 3580 kJ/kg
= 2335.8 kJ/kg
s1 = 6.6776 kJ/kg ◊ K
At the state 2, the pressure p2 = 40 bar
State 2: Quality of steam is unknown
sg2 = 6.070 kJ/kg ◊ K
s2 = s1 = 6.6776 kJ/kg ◊ K
and Tsat = 250.4°C
State 3: Superheated steam T3 = 600°C
At this state s2 = s1 = 6.6776 kJ/kg ◊ K > sg @ 40 bar
State 4: Wet steam; (from steam Table A-13)
Thus, steam is superheated at the state 2 and from
p3 = 10 kPa
Mollier diagram, we get
hf = 191.83 kJ/kg
h2 = 3150 kJ/kg, and T2 = 375°C
hfg = 2392.82 kJ/kg
The heat supplied per kg of steam
sf = 0.6492 kJ/kg ◊ K
qin = h1 – h6 + h3 – h2
sfg = 7.5010 kJ/kg ◊ K
= 3580 – 206.97 + 3675 – 3150
State 5: Saturated liquid
ª 3898 kJ/kg
p3 = 10 kPa
The heat rejected per kg of steam;
vf = 0.00101 m3/kg
qout = h4 – h5 = 2335.8 – 191.83
hf = 191.83 kJ/kg
= 2143.97 kJ/kg
State 6: Compressed liquid
p3 = 150 bar
412 Thermal Engineering

and thermal efficiency


q 2143.97
hcycle = 1 - out = 1 - = 0.4496
qin 3898
or = 44.96%

A steam power plant operates on an


ideal reheat Rankine cycle between the pressure limits of
9 MPa and 10 kPa. The mass flow rate of steam through
the cycle is 25 kg/s. Steam enters both stages of the tur-
bine at 500°C. If the moisture content of the steam exit-
ing the low-pressure turbine should not to exceed 10%;
determine
(a) the reheat pressure,
(b) total rate of heat input in the boiler,
(c) the thermal efficiency of the cycle.

Solution
Given A reheat Rankine cycle with
HP turbine inlet: p1 = 9 MPa,
T1 = 500°C
ms = 25 kg/s
LP turbine inlet: T3 = 500°C
Condenser inlet: p3 = 10 kPa,
x4 = 1 – 0.10 = 0.9
To Find
(i) The reheat pressure,
(ii) Total rate of heat input in the boiler, and
(iii) The thermal efficiency of the cycle.
vf = 0.00101 m3/kg
Properties of steam
hf = 191.83 kJ/kg
At state 1: Superheated steam (from Mollier Chart)
State 6: Compressed liquid
p1 = 9 MPa,
p1 = 9 MPa
T1 = 500°C
Analysis
h1 = 3386 kJ/kg,
(i) The intermediate pressure p2
s1 = 6.6575 kJ/kg ◊ K
Since the process 3– 4 is an isentropic process,
State 2: Quality of steam is unknown
thus
s2 = s1 = 6.6575 kJ/kg ◊ K
s3 = s4
State 3: Superheated steam; T3 = 500°C
Specific entropy at the state 4;
State 4: Wet steam; (from steam table A-13)
s4 = (sf + x4 sfg )@10 kPa
p3 = 10 kPa
= 0.6492 + 0.9 ¥ 7.5010
hf = 191.83 kJ/kg,
= 7.40 kJ/kg ◊ K
hfg = 2392.82 kJ/kg
Hence s3 = 7.40 kJ/kg ◊ K
sf = 0.6492 kJ/kg ◊ K
Using Mollier (h–s) chart; draw a vertical straight
sfg = 7.5010 kJ/kg ◊ K
line 3–4 from the point 4 (10 kPa and s3 =
State 5: Saturated liquid
7.40 kJ/kg ◊ K) to a temperature line of T3 = 500°C,
p3 = 10 kPa
Vapour Power Cycles 413

as shown in the h–s diagram of Fig. 12.29(b). entering the second stage turbine, where it expands to the
At intersection point, the pressure and enthalpy condenser pressure of 0.008 MPa. The net power output
found to be of the cycle is 100 MW. Determine
p2 = p3 = 20 bar (a) thermal efficiency of the cycle,
h3 = 3468 kJ/kg (b) mass flow rate of steam in kg/h,
Therefore, the steam should be reheated at 20 bar. (c) specific steam consumption in kg/kWh,
Similarly, the state 2 can be defined as (d) the rate of heat transfer from the condensing
s2 = s1 = 6.6575 kJ/kg ◊ K steam as it passes through the condenser in MW.
Using Mollier (h–s) chart; draw a vertical Discuss the effect of reheating on the vapour power
straight line 1–2 from the point 1 ( 90 bar and s1= cycle
6.6575 kJ/kg ◊ K) to a pressure line of 20 bar, we get
T2 = 300°C, Solution
h2 = 3015 kJ/kg
Given An ideal reheat Rankine cycle operates with
The steam is superheated at the state 2. steam as working fluid.
The pump work; HP turbine inlet; p1 = 8.0 MPa
wp = vf ( p1 – p3) T1 = 480°C
= 0.00101 ¥ (9 ¥ 103 – 10) LP turbine inlet; p3 = 0.7 MPa
= 9.08 kJ/kg T3 = 440°C
Enthalpy at the state 6; Condenser pressure; p4 = 0.008 MPa
h6 = h5 + wp = 191.93 + 9.08 Net Power output P = 100 MW
= 201 kJ/kg
Specific enthalpy at the state 4; To find
h4 = (hf + x4 hfg )@ 10 kPa (i) Thermal efficiency of the cycle.
= 191.83 + 0.9 ¥ 2392.82 (ii) Mass flow rate of steam in kg/h.
= 2345.35 kJ/kg (iii) Specific steam consumption in kg/kWh.
Total heat supplied per kg of steam (iv) Rate of heat transfer from the condensing steam
qin = h1 – h6 + h3 – h2 as it passes through the condenser, in MW.
= 3386 – 201 + 3468 – 3015 Assumptions
= 3638 kJ/kg (i) Each component in the cycle as control volume at
(ii) Rate of heat input in the boiler steady state.
Qin = ms qin = (25 kg/s) ¥ (3638 kJ/kg) (ii) All process are internally reversible.
= 90950 kW (iii) Turbine and pump operate isentropically.
The heat rejected per kg of steam; (iv) Kinetic and potential energy effects are negligible.
qout = h4 – h5 = 2345.35 – 191.83
Properties of steam at each principal states from Mollier
= 2153.52 kJ/kg
chart;
(iii) Thermal efficiency of the cycle
State 1: Superheated steam at
qout 2153.52
hcycle = 1 - = 1- p1 = 8.0 MPa
qin 3638.0 T1 = 480°C
= 0.408 or 40.8% h1 = 3380 kJ/kg
State 2: Wet steam at
The steam is the working fluid in p2 = 0.7 MPa
an ideal Rankine cycle with superheat and reheat. The s2 = s1
steam enters the first stage turbine at 8.0 MPa, 480°C and h2 = 2755 kJ/kg x2 = 0.98
expands to 0.7 MPa. It is then reheated to 440°C before
414 Thermal Engineering

8.0 MPa, 480°C State 5: Saturated liquid at


1
p3 = 0.008 MPa
HP LP
Turbine Turbine h5 = 173.93 kJ/kg ◊ K
wT = 100 MW
0.7 MPa
2 4 vf = 0.001008 m3/kg
qin
State 6: Compressed liquid at
Boiler Reheater
o
0.7 MPa, 440 C p6 = 8.0 MPa
3
8 MPa 0.008 MPa Analysis
6 Feed Pump Condenser
5 (i) Thermal efficiency of the cycle
water wp
qout
The pump work;
(a) Schematic
wp = vf (p1 – p3 )
T 1 = 0.001008 ¥ (8 – 0.008) ¥ 103
480oC
3 o
440 C = 8.06 kJ/kg
Enthalpy at the state 6;
8 MPa h6 = h5 + wp = 173.93 + 8.06
= 182 kJ/kg
0.7 MPa
6 2
The total turbine work per kg of steam;
wT = h1 – h2 + h3 – h4
0.008 MPa
5
= 3380 – 2755 + 3350 – 2410
4
= 1565 kJ/kg
Net work done per kg of steam
s
(b) T–s diagram wnet = wT – wp = 1565 – 8.06
h
ª 1557 kJ/kg
(kJ/kg) Total heat supplied per kg of steam
1 480°C °C qin = h1 – h6 + h3 – h2
3380 3 440
3350 kJ/kg = 3380 – 182 + 3350 – 2755
a
MP = 3793 kJ/kg
0.7
Pa Thermal efficiency;
8M 2
2755 kJ/kg
2430 wnet 1557
4 hcycle = = = 0.410 or 41.0%
6 8 MP
a qin 3793
0.00
(ii) Mass flow rate of steam
5
P 100 ¥ 103 kW
ms = =
s wnet 1557 kJ/kg
(c) h–s diagram
= 64.22 kg/s = 231.21 ¥ 103 kg/h
(iii) Specific steam consumption
State 3: Superheated steam at 3600 3600
ssc = = = 2.31 kg/kWh
p2 = 0.7 MPa wnet 1557
T3 = 440°C (iv) The rate of heat rejection in the condenser
h3 = 3350 kJ/kg Qin = ms (h4 – h5)
State 4: Wet steam at = 64.22 ¥ (2410 – 182)
p3 = 0.008 MPa = 143.4 ¥ 103 kW
s4 = s3 = 143.4 MW
h4 = 2410 kJ/kg x4 = 0.94
Vapour Power Cycles 415

Comment With superheating and reheating of steam in temperature and then the superheat at higher aver-
the Rankine cycle, the effects observed are (in compari- age temperature.
son with Example 12.8 given earlier) If the mean temperature Tm of heat addition
(i) The quality of steam leaving the turbine is as shown in Fig. 12.31(b) is assumed in such a
improved from x = 0.674 to 0.94. way that the area under curve 4–c–d–1 is equal
(ii) The efficiency of the cycle is increased by 41.0 – to the area under the curve a–b on T–s diagram
37.1 = 3.9%.
(Fig. 12.31), then
qin = h1 – h4 = Tm (s1 – s4 )
12.11 MEAN TEMPERATURE OF Mean temperature of heat addition;
HEAT ADDITION h1 - h4
Tm =
The heat-addition pattern of the simple Rankine s1 - s4
cycle is shown in Fig. 12.31(a). Initially, the heat Amount of heat supplied in the boiler
=
added as sensible heat to compressed liquid coming Change in entropy duriing heat addition
out the pump is at much lower average tempera- ...(12.31)
ture, the latent heat for vapourisation at constant Heat rejected per kg of steam;
qout = h2 – h3 = TL (s1 – s4 )
T 1 Then Rankine cycle efficiency;
g
tin

T (s - s )
ea

q
hRankine = 1 - out = 1 - L 1 4
rh
pe

qin Tm (s1 - s4 )
Su
d

c Water Steam
se

d
es

TL
\ hRankine = 1 -
te pr

...(12.32)
wa com

Tm
r
of
ing

where TL is temperature of heat rejection. The lower


at
He

value of temperature TL for given Tm will increase


Sensible Heat of Super
the thermal efficiency of the Rankine cycle. But the
heat vaporisation heat temperature of heat rejection cannot be lower than
q the temperature of surroundings. Therefore,
(a) Heat additional pattern to working fluid
hRankine = f (Tm) only ...(12.33)
p1
T Thus, the higher the mean temperature of heat
1 addition, the higher will be thermal efficiency of
qsup Rankine cycle.
qlatent
c 12.12 REGENERATIVE RANKINE CYCLE
TH qsensible
a d
Tm b
In a simple Rankine cycle, a significant amount of
4
heat is added for sensible heating of compressed
p1 liquid coming out the pump. The mean temperature
TL
3 2 at which sensible heat added is much lower than
p2 qout
the source temperature. Thus, the efficiency of the
s
Rankine cycle is much lower than that of Carnot
(b) Mean temperature of heat addition vapour power cycle. The efficiency of the Rankine
cycle can be improved by heating the feed water
regeneratively.
416 Thermal Engineering
qin

1 kg 1
Boiler
bi ne
The mean temperature of heat addition in the Tur
Rankine cycle can be improved by increasing the
heat supplied at high temperature such as increas- 4¢ 2
ing superheat, increasing boiler pressure and using 4
reheat. The mean temperature of heat addition can
Pump Condenser
also be increased by decreasing the amount of heat 3

supplied at lower temperatures. Wp qout


In saturated steam Rankine cycle shown in (a) Schematic of ideal regenerative cycle
Fig. 12.32, if the feed water enters the boiler at the
T
state 4¢ and all heat is supplied at constant temper-
(DS)water (DS)steam
atrure TH, then the cycle is called an ideal regen-
erative cycle, shown schematically in Fig. 12.33(a)
TH 4¢
and corresponding T–s diagram in Fig. 12.33(b). 1

DT
4 5
TL 3
2¢ 2

a b c d s
(b) Ideal regenerative cycle on T–s plot

h
(kJ/kg)
h1 1

In ideal regenerative cycle, the condensate leav-


ing the pump enters the turbine at the state 4 and 2

flows in counterflow direction to the steam flow. h3


Thus, it is possible to heat the feed water to steam 7 3
5 6
temperature at inlet to turbine. If at all points the
temperature difference between steam and feed wa- 4
s
ter is negligibly small then the heat transfer takes s7 s1 (kJ/kg.K)
place in reversible manner. For such a process; (c) h–s diagram for regenerative cycle
(DT ) Water = – (DT ) Steam
and (Ds)Water = – (Ds)Steam
During an ideal regeneration, the steam cools Then qin = h1 – h4¢ = TH (s1 – s4¢)
first to the state 5 and then expands to the state 2¢. and qout = h2¢ – h3 = TL (s2¢ – s3)
The slope of line 1–5 and 4¢–4 in Fig. 12.33(b) will Since s1 – s4¢ = s2¢ – s3
be identical at every point. The area a–4 – 4¢–b– a
qout T
and area c –5–1– d – c are equal and congruous. \ hReg = 1 -
=1- L ...(12.34)
Therefore, all the heat in the boiler is supplied at qin TH
constant temperature TH and all the heat is rejected The efficiency of an ideal regenerative cycle is
at constant temperature TL, both being reversible. thus equal to eficiency of the Carnot cycle. Since
Vapour Power Cycles 417

steam is first used to heat the feed water from 1–5


and then allows to expand from state 5 to state 2¢,
and therefore, the net work output of an ideal re-
generative cycle is less and steam rate will be more,
although it is more efficient as compared with sim-
ple Rankine cycle. However, such a proposition is
not practical for the following reasons:
1. The transfer of heat in reversible manner
takes place very slowly.
2. Heat exchange in the turbine is not mechani-
cally feasible.
3. The moisture content of steam in the turbine
will be very high.
In actual practice, advantage of regenerative
heating principle is used by extracting a part of
the steam from turbine at a certain stage of the ex-
pansion and it is used for heating of feed water in
separate feed-water heaters. This arrangement does
not reduce the dryness fraction of remaining steam
passing through the turbine.
If there were a large number of extraction stages
of steam for feed water heating, then the resulting
cycle would approach to be a Carnot cycle.

to be saturated at the state 6. Now this saturated


water is pumped by high pressure (HP) pump to the
The regenerative cycle with open feed-water heater boiler pressure state 7. With the regeneration, the
is shown in Fig. 12.34. A part of superheated steam average temperature at which heat is supplied has
which enters the turbine at the state 1, is extract- been increased, therefore, Rankine cycle efficiency
ed from the turbine at the intermediate state 2 of improves.
turbine- expansion process. The extracted steam Since the masses of steam flowing
is supplied to a heat exchanger known as feed wa- through the various components of the cycle vary,
ter heater. The remaining amount of steam in the thus the analysis of this cycle differs from previous
turbine expands completely to condenser pressure one.
(state 3). The condensate, a saturate liquid at state
Let 1 kg of steam be leaving the boiler and
4 is pumped isentropically by low pressure (LP)
entering the turbine.
pump to the pressure of extracted steam. The com-
pressed liquid at the state 5 enters the feed water m1 kg of steam per kg, is extracted at the state 2
heater and it mixes with steam extracted from the from the turbine at intermediate pressure p2.
turbine. Due to direct mixing process, the feed wa- (1 – m1) kg of steam per kg flow through the
ter heater is called open or direct-contact type feed- remaining part of the turbine during expansion
water heater. The portion of steam extracted is so from 2–3, condensation from 3–4 and pumping
adjusted to make the mixture leaving the feed water from 4–5.
418 Thermal Engineering

(1 – m1) kg of steam enters in open feed water Since Tm,Reg > Tm,Rankine
heater and mixed with m1 kg of steam blown from Thus the amount of heat supplied is decreased
the turbine at the state 2. After mixing, the mass of and efficiency of regenerative cycle will be higher
saturated liquid becomes 1 kg at the state 6 and it is than that of Rankine cycle. But at the same time,
pumped to boiler pressure at the state 7. the turbine work and heat rejection are also reduced
Applying steady flow energy equation to mixing due to extraction of steam at intermediate pressure.
process 2–6; Advantages of Regeneration
(1 – m1) h5 + m1 h2 = h6
1. It raises the temperature of feed water to
or h5 – m1 h5 + m1 h2 = h6
saturation temperature, and thus the amount
h6 - h5 of heat addition in the boiler reduces.
which gives, m1 = ...(12.35)
h2 - h5 2. The heat is added in the boiler at a higher
The heat supplied in the boiler average temperaure.
qin = h1 – h7 3. Open feed water heater serves as a deaerator
Heat rejected in the condenser to remove the air and other non-condensable
qout = (1 – m1) (h3 – h4) gases from the feed water, otherwise they
Turbine work, would cause corrosion.
wT = (h1 – h2) + (1 – m1) (h2 – h3) ...(12.36)
Pump work
wp = (h7 – h6) + (1 – m1) (h5 – h4) ...(12.37) Consider the Rankine cycle 1–2–3–4–5–6–7–8–
Net work done per kg of steam, 9 –10–1 with two stage feed-water heating oper-
ating between turbine inlet and condenser inlet as
wnet = wT – wp
shown on the T–s diagram in Fig. 12.35(a).
= (h1 – h2) + (1 – m1) (h2 – h3)
Applying steady-flow energy equation for the
– [(h7 – h6) + (1 – m1) (h5 – h4)]
mixing process 2–9;
...(12.38)
(1 – m1) h8 + m1 h2 = h9
Thermal efficiency of regenerative cycle;
h9 - h8
qout which gives, m1 =
hreg = 1 - h2 - h8
qin
Similarly, the steady-flow energy equation for
(1 - m1 ) ( h3 - h4 ) the mixing process 3–7;
= 1- ...(12.39)
h1 - h7 (1 – m1 – m2) h6 + m2 h3 = (1 – m1) h7
In the regenerative cycle, the feed water enters h7 - h6
the boiler at temperature T7 and its mean tempera- which gives, m2 = (1 - m1 )
h3 - h6
ture of heat addition is
The path 1–2–3–4 in Fig. 12.35(a) represents
qin h -h the states of decreasing mass of fluid during isen-
Tm, Reg = = 1 7 ...(12.40)
s1 - s7 s1 - s7 tropic expansion. If the same mass of steam (1 kg)
The mean temperature of heat addition without undergoes throughout regeneration and expansion,
heat generation for simple Rankine cycle operating the states would be represented by 1 –2¢–3≤–4¢ as
between same pressures p1 and p3 would be shown in Fig. 12.35(b). The turbine work with
decreasing mass and unit mass for above two types
qin h -h
Tm, Rankine = = 1 7 ...(12.41) of cycles would be;
s1 - s5 s1 - s5
Vapour Power Cycles 419

T wT = (h1 – h4¢) – (h9 – h8) – (h7 – h6 )


...(12.49)
1 kg 1 It is observed that the stepped cycle 1–2¢–3¢–
10 m1 kg 3≤– 4¢ –5–6 – 7–8 –9–10 –1 approximates the ideal
9 2
8 m2 kg regeneration cycle in Fig. 12.33. By addition of the
3
6 7
(1–m1–m2) kg
more number of feed water heaters, by extracting
5 4 the steam in number of stages from turbine, the
feed-water temperature could be raised, and it
s would give a closure approximation (Fig. 12.36) to
(a) Regenerative cycle with decreasing mass Carnot Cycle.
of steam during its expansion
T

1 kg 1
10
1 kg 2¢ Loss in
2
9
8 work output
1 kg 3¢¢
3
6 7 3¢
1 kg
5 4¢ 4

s
(b) Regenerative cycle for unit mass of fluid

By addition of feed-water heaters, the the heat


wT = (h1 – h2) + (1 – m1) (h2 – h3) rejection in the cycle decreases from (h4 – h5) to
(h4¢ – h5), the turbine work output decreases (area
+ (1 – m1 – m2) (h3 – h4) ...(12.42)
under 2– 2¢ + area under 3 –3¢) as shown by shaded
= (h1 – h2) + (h2¢ – h3¢) + (h3≤ – h4¢)
area in Fig. 12.35(b). The steam rate also increases
...(12.43)
with regeneration. But thermal efficiency of the
where (1 kg) ¥ (h2¢ – h3¢)
cycle increases.
= (1 – m1)(kg) ¥ (h2 – h3) ...(12.44)
and (1 kg) ¥ (h3≤ – h4¢) An ideal regenerative steam cycle
= (1 – m1 – m2) (kg) ¥ (h3 – h4) ...(12.45) operates with the steam entering the turbine at 30 bar
The cycle 1–2¢–3¢–3≤– 4¢–5–6 –7– 8–9–10–1 and 500°C and is exhausted at 0.1 bar. A feed water
heater is used, which operates at 5 bar. Calculate
using 1 kg working fluid throughout. The heat
(a) the thermal efficiency,
released by steam from 2 –2¢ is used to heat com-
(b) steam rate of the cycle, and
pressed water from the states 8 to 9.
(c) increase in average temperature of heat addition,
(1 kg) ¥ (h2 – h2¢) = (1 kg) (h9 – h8) ...(12.46)
efficiency, and steam as compare to an ideal
Similarly, Rankine cycle operates between same conditions.
(1 kg) ¥ (h3¢ – h3≤) = (1 kg) (h7 – h6) ...(12.47)
Rearranging Eq. (12.43) Solution
wT = (h1 – h4¢) – (h2 – h2¢) – (h3¢ – h3≤) Given An ideal regenerative steam cycle with
...(12.48) p1 = p7 = 30 bar T1 = 500°C
Using Eqs. (12.46) and (12.47) in Eq. (12.48), p3 = p4 = 0.1 bar p2 = p6 = 5 bar
the turbine work is
420 Thermal Engineering

h1 = 3455 kJ/kg,
s1 = 7.234 kJ/kg ◊ K
State 2: Superheated steam;
p2 = 5 bar,
T2 = 240°C
h2 = 2940 kJ/kg
s2 = 7.234 kJ/kg ◊ K
State 3: Wet steam;
p3 = 0.1 bar = 10 kPa,
x3 = 0.88
h3 = 2300 kJ/kg
State 4: Saturated liquid; (from steam tables);
p4 = 0.1 bar
vf 4 = 0.001010 m3/kg
h4 = hf 4 = 191.83 kJ/kg
State 5: Compressed liquid;
p5 = 5 bar = 500 kPa
State 6: Saturated liquid (from steam tables);
p6 = 5 bar;
h6 = hf6 = 641.21 kJ/kg
vf6 = 0.001094 m3/kg
s6 = 1.8606 kJ/kg ◊ K
State 7: Compressed liquid;
p7 = 30 bar = 3000 kPa
Analysis
(i) Regenerative cycle
The high-pressure pump work input per kg of
steam,
wp1 = vf @ 5 bar ¥ (p1 – p6)
= 0.001094 (30 – 5) ¥ 102
= 2.735 kJ/kg
Specific enthalpy at the state 7;
h7 = h6 + wp2 = 641.21 + 2.73
= 643.94 kJ/kg
To Find
The low pressure pump work input per kg of
(i) The thermal efficiency.
steam
(ii) Steam rate of the cycle.
wp2 = vf @ 0.1 bar ¥ (p5 – p4)
(iii) Increase in average temperature of heat addition,
= 0.001010 ¥ (5 – 0.1) ¥ 102
efficiency, and steam as compared to an ideal
= 0.5 kJ/kg
Rankine cycle operates between same conditions.
Specific enthalpy at the state 5;
Properties of steam From Mollier chart h5 = h4 + wp2
State 1: Superheated steam at turbine inlet = 191.83 + 0.5 = 192.32 kJ/kg
p1 = 30 bar, The mass of steam m1 extracted from turbine at
T1 = 500°C 5 bar can be obtained by using Eq. (12.35)
Vapour Power Cycles 421

h6 - h5 641.21 - 192.32 vf 4 = 0.001010 m3/kg


m1 = =
h2 - h5 2940 - 192.32 s4 = 0.6492 kJ/kg ◊ K
= 0.163 kg State 7: Compressed liquid;
Therefore, 0.163 kg of steam is extracted from the p7 = 30 bar
turbine for each kg of steam entering the turbine. The pump work input per kg of steam
Turbine work, wp = vf @ 0.1 bar ¥ (p7 – p4)
wT = h1 – h2 + (1 – m1) (h2 – h3) = 0.001010 ¥ (30 – 0.1) ¥ 102
= 3455 – 2940 + (1 – 0.163) (2940 – 2300) = 3.01 kJ/kg
= 1050.68 kJ/kg Specific enthalpy at the state 7;
The total pump work h7 = h4 + wp = 191.83 + 3.01
wp = wp1 + (1 – m1)wp2 = 194.85 kJ/kg
= 2.73 + (1 – 0.163) ¥ 0.5 Turbine work,wT = h1 – h3 = 3455 – 2300
= 3.1 kJ/kg = 1155 kJ/kg
Net work done per kg of steam Net work done per kg of steam
wnet = wT – wp = 1050.68 – 3.1 wnet = wT – wp = 1155 – 3.01
= 1047.58 kJ/kg = 1152 kJ/kg
The heat supplied in the boiler per kg of steam The heat supplied in the boiler per kg of steam
qin = h1 – h7 = 3455 – 643.94 qin = h1 – h7 = 3455 – 194.85
= 2811 kJ/kg = 3260.15 kJ/kg
(a) Thermal efficiency of the cycle (d) Thermal efficiency of the cycle;

wnet 1047.58 wnet 1152


hth, Reg = = hth, ideal = = = 0.3533
qin 2811 qin 3260.15
= 0.3725 or 37.26% or 35.33%
(b) Steam rate of the cycle; (e) Steam rate of the cycle;

3600 3600 3600 3600


ssc = = ssc = =
wnet 1047.58 wnet 1152

= 3.436 kg/kWh = 3.125 kg/kWh


(f) Average temperature of heat addition,
(c) Average temperature of heat addition,
Eq. (12.31);
Eq. (12.31);
qin 2811 qin 3260.15
Tm, Reg = = Tm, ideal = =
s1 - s6 7.234 - 1.8606 s1 - s4 7.234 - 0.6492
= 523.13 K = 494.1 K
(ii) Ideal Rankine cycle Changes compared to regenerative cycle
We have the properties of steam as Increase in efficiency with regeneration
State 1: Superheated steam at turbine inlet = hth, Reg – hth, ideal
h1 = 3455 kJ/kg, = 37.26 – 35.33 = 1.93%
s1 = 7.234 kJ/kg ◊ K Increase in average temperature of heat addition with
State 3: Wet steam; regeneration
h3 = 2300 kJ/kg = Tm, Reg – Tm, ideal
x3 = 0.88 = 523.13 K – 494.1 K
State 4: Saturated liquid; (from steam tables); = 29.0 K or 29°C
p4 = 0.1 bar Increase in steam rate with regeneration
h4 = hf 4 = 191.83 kJ/kg = 3.436 – 3.125 = 0.311 kg/kWh
422 Thermal Engineering

Consider a regenerative vapour State 2: Saturated steam;


power cycle with a feed-water heater. The steam enters p2 = 0.7 MPa
the first-stage turbine at 8.0 MPa, 500°C and expands h2s = 2750 kJ/kg
to 0.7 MPa, where some of the steam is extracted and State 3: Wet steam;
diverted to feedwater heater operating at 0.7 MPa. The p3 = 0.008 MPa = 8 kPa
remaining steam expands through the second stage tur- h3s = 2150 kJ/kg
bine to a condenser pressure of 0.008 MPa. The satu- State 4: Saturated liquid; (from steam tables);
rated liquid exits the feed-water heater at 0.7 MPa. The
p4 = 0.008 MPa = 8 kPa
isentropic efficiency of each turbine is 85%, while each
h4 = hf 4 = 173.88 kJ/kg
pump operates isentropically. If the net power output of
vf 4 = 0.001008 m3/kg
the cycle is 105 MW, determine
State 5: Compressed liquid;
(a) thermal efficiency of the cycle,
p5 = 0.7 MPa = 700 kPa
(b) the mass flow rate of steam entering the first
turbine stage. State 6: Saturated liquid (from steam tables);
p6 = 0.7 MPa;
Solution vf 6 = 0.001080 m3/kg
h6 = hf 6 = 697.23 kJ/kg
Given A regenerative vapour power cycle with feed-
water heater: State 7: Compressed liquid
First stage turbine inlet, p1 = 8.0 MPa, p7 = 8 MPa = 8000 kPa
T1 = 500°C Schematic with given data
Regeneration pressure, p2 = p5 = p6 = 0.7 MPa
Condenser pressure, p3 = p4 = 0.008 MPa
Isentropic efficiency, hT = 0.85
Power output plant, P = 105 MW
To find
(i) Thermal efficiency of the cycle.
(ii) Mass flow rate of steam entering the first turbine
stage.
Assumptions
(i) Each component in the cycle is analysed as a
control volume at steady state.
(ii) All processes of the working fluid are internally
reversible except expansion through the turbines.
(iii) The turbines, pumps and feed-water heater
operate adiabatically.
(iv) Kinetic and potential energy effects are negligible.
(v) Saturated liquid leave the feed water heater and
condenser at their pressures.
Properties of steam From Mollier chart
State 1: Superheated steam at turbine inlet
p1 = 8 MPa
T1 = 500°C
h1 = 3400 kJ/kg
Vapour Power Cycles 423

Analysis The high-pressure pump work input per kg of = 7.884 + (1 – 0.195) ¥ 0.697
steam = 8.4 kJ/kg
wp1 = vf @ 0.7 MPa ¥ (p1 – p6) Net work developed in the cycle
= 0.001080 (8 – 0.7) ¥ 103 wnet = wT – wp = 1029.77 – 8.4
= 7.884 kJ/kg = 1021.37 kJ/kg
Specific enthalpy at the state 7; Heat supplied to the steam in the boiler
h7 = h6 + wp2 = 697.22 + 7.884 qin = h1 – h7 = 3400 – 705.1
= 705.1 kJ/kg = 2694.9 kJ/kg
The low-pressure pump work input per kg of steam (i) Thermal efficiency of the regenerative cycle:
wp2 = vf @ 0.008 MPa ¥ (p5 – p4) w 1021.37
hth = net = = 0.378 or 37.8%
= 0.001008 ¥ (0.7 – 0.008) ¥ 103 qin 2694.9
= 0.697 kJ/kg (ii) The mass flow rate of steam entering the first
Specific enthalpy at the state 5; turbine:
h5 = h4 + wp2 = 173.88 + 0.697 Net power output P
ms = =
= 174.6 kJ/kg Net workdone per kg of steam wnet
The isentropic efficiency of first stage turbine 105 ¥ 103
= = 102.82 kg/s
h1 - h2 1021.37
h T1 = or 3.70 ¥ 105kg/h
h1 - h2 s
or h2 = h1 – hT1 (h1 – h2s) In a steam power plant, the condition
= 3400 – 0.85 ¥ (3400 – 2750) of steam at inlet to turbine is 20 bar and 300°C and the
= 2847.5 kJ/kg condenser pressure is 10 kPa. Two feed water heaters
The isentropic efficiency of second stage turbine operate at 5 bar and 1 bar. By neglecting the pump work,
h2 - h3 determine
hT2 =
h2 - h3s (a) The quality of steam at turbine exhaust,
or h3 = h2 – hT2 (h2 – h3s) (b) Masses of steam bled off at each pressure per kg
= 2847.5 – 0.85 ¥ (2847.5 – 2150) of steam entering the turbine,
= 2254.62 kJ/kg (c) Net work done per kg of steam flow,
The mass of steam m1 extracted from turbine at 5 bar (d) Thermal efficiency of the cycle, and
can be obtained by using Eq. (12.35) (e) Specific steam consumption.

h6 - h5 697.22 - 174.1
m1 = =
h2 - h5 2847.5 - 174.1
= 0.195 kg
Work done by first-stage turbine
wT1 = h1 – h2 = 3400 – 2847.5
= 552.5 kJ/kg
Work done by second-stage turbine
wT2 = (1 – m1) (h2 – h3)
= (1 – 0.195) ¥ (2845.5 – 2254.62)
= 479 kJ/kg
Total turbine work per kg of steam
wT = wT1 + wT2 = 552.5 + 479
= 1029.77 kJ/kg Solution
Total pump work input per kg of steam
Given A regenerative vapour power cycle with feed-
wp = wp1 + (1 – m1) wp2
water heater:
424 Thermal Engineering

First stage turbine inlet, p1 = 20 bar h7 = hf 7 = 417.5 kJ/kg


T1 = 300°C State 8: Compressed liquid;
No. of feed water heaters =2 p8 = 5 bar = 500 kPa,
Feed-water heater 1 pressure, p2 = p8 = p9 = 5 bar h8 = h7 = 417.5 kJ/kg
Feed-water heater 2 pressure, p3 = p6 = p7 = 1 bar (without pump work)
Condenser pressure, p4 = p5 = 10 kPa State 9: Saturated liquid (from steam tables);
p9 = 5 bar
To find
h9 = hf 9 = 641.21 kJ/kg
(i) The quality of steam at turbine exhaust.
State 10: Compressed liquid;
(ii) Masses of steam bled off at each pressure per kg
p10 = 20 bar = 2000 kPa
of steam entering the turbine.
h10 = h9 = 641.21 kJ/kg
(iii) Net work done per kg of steam flow.
(without pump work)
(iv) Thermal efficiency of the cycle.
(v) Specific steam consumption. Analysis
Properties at principal states From Mollier diagram: (i) The quality of steam at the state 4 (turbine
State 1: Superheated steam at turbine inlet exhaust) is
p1 = 20 bar x4 = 0.815
T1 = 300°C (ii) Masses of steam bled off at each pressure per kg
h1 = 3025 kJ/kg of steam entering the turbine
s1 = 6.7663 kJ/kg ◊ K The masses of steam m1 and m2 extracted from
turbine at pressures 5 bar and 1 bar, respectively;
State 2: Wet steam
h9 - h8 641.21 - 417.5
p2 = 5 bar m1 = =
h2 - h8 2725 - 417.5
x2 = 0.99
h2 = 2725 kJ/kg = 0.097 kg/kg of steam
s2 = s1 h7 - h6
and m2 = (1 - m1)
State 3: Wet steam; h3 - h6
p3 = 1 bar
Ê 417.5 - 191.83 ˆ
= (1 - 0.097) ¥ Á
Ë 2460 - 191.83 ˜¯
x3 = 0.92
h3 = 2460 kJ/kg
s3 = s1 = 0.09 kg/kg of steam
State 4: Wet steam; (iii) Net work done per kg of steam flow
p4 = 0.1 bar The work done by first-stage turbine
x4 = 0.815 wT1 = h1 – h2 = 3025 – 2725
h4 = 2140 kJ/kg = 300 kJ/kg
s4 = s1 The work done by second-stage turbine
State 5: Saturated liquid; (from steam tables); wT2 = (1 – m1) (h2 – h3)
p5 = 0.1 bar = (1 – 0.097) ¥ (2725 – 2460)
h5 = hf 5 = 191.83 kJ/kg = 239.19 kJ/kg
State 6: Compressed liquid; The work done by third-stage turbine
p6 = 1 bar = 100 kPa wT 3 = (1 – m1 – m2) (h3 – h4)
h6 = h5 = 191.83 kJ/kg = (1 – 0.097 – 0.09) ¥ (2460 – 2140)
(without pump work) = 260 kJ/kg
State 7: Saturated liquid (from steam tables); Total turbine work per kg of steam
p7 = 1 bar wT = wT1 + wT 2 + wT 2
Vapour Power Cycles 425

= 300 + 239.19 + 260 Solution


= 799.22 kJ/kg
Given A reheat–regenerative ideal vapour power cycle
In absence of pump work, the net work developed
with feed-water heater with
in the cycle
First-stage turbine inlet = 15 MPa, 600°C
wnet = wT = 799.22 kJ/kg
Intermediate pressures = 2 MPa and 0.5 MPa
The heat supplied to the steam in the boiler
Reheating = 2 MPa, 600°C
qin = h1 – h10
Condenser pressure = 10 kPa
= 3025 – 641.21 = 2381.75 kJ/kg
Power output, P = 120 MW
(iv) Thermal efficiency of the cycle,
wnet 799.22 To find
hth = = (i) Fraction of steam extracted from the turbine for
qin 2381.75
feed water heater,
= 0.3354 or 33.54%
(ii) Thermal efficiency of the cycle,
(v) Specific steam consumption;
(iii) Mass flow rate of steam in the cycle for network
3600 3600 output of 120 MW.
ssc = = = 4.5 kg/kWh
wnet 799.22
Assumptions
(i) Each component in the cycle is analysed as a
Consider a reheat regenerative va-
control volume at steady state.
pour power cycle with a feed-water heater. The steam
enter the first turbine at 15 MPa, 600°C and expands to (ii) All processes of the working fluid are reversible.
2 MPa. Then the steam is reheated to 600°C at the same (iii) Turbines, pumps and feedwater heaters operate
pressure. The steam for feed-water heater is extracted adiabatically.
from the low-pressure turbine at the pressure of 0.5 MPa (iv) No pressure drop in boiler, condenser and feed
and the remaining steam is further expanded to a con- water heaters.
denser pressure of 10 kPa. The working fluid experiences (v) Feed water heater is direct contact type.
no irreversibities, passes through turbines, pumps, boiler, (vi) The working fluid leaves the condenser and
reheater condenser. Determine feedwater heaters as saturate liquid.
(a) The fraction of steam extracted from the turbine (vii) Kinetic and potential energy effects are negligible.
for feed-water heater, (viii) 1 kg of steam flow through the boiler and first-
(b) Thermal efficiency of the cycle, and stage turbine.
(c) Mass flow rate of steam in kg/h, if cycle produces
Properties at principal states From Mollier diagram
120 MW.
State 1: Superheated steam at turbine inlet
p1 = 15 MPa = 150 bar
T1 = 600°C
h1 = 3580 kJ/kg
State 2: Superheated steam
p2 = 2 MPa = 20 bar
s2 = s1
h2 = 2985 kJ/kg
State 3: Superheated steam
p3 = 2 MPa = 20 bar
T3 = 600°C
h3 = 3695 kJ/kg
State 4: Superheated steam;
p4 = 0.5 MPa = 5 bar
426 Thermal Engineering

s4 = s3 The total pump work input


h4 = 3225 kJ/kg wp = wp1 + (1 − m1) wp2
State 5: Wet steam; = 15.86 + (1 − 0.148) × 0.5
p5 = 10 kPa = 0.1 bar = 16.28 kJ/kg
x5 = 0.94 Turbine work output,
h5 = 2440 kJ/kg wT = h1 − h2 + h3 − h4 + (1 − m1) (h4 − h5)
State 6: Saturated liquid; (from steam tables); = 3580 − 2985 + 3695 − 3225
p6 = 0.1 bar + (1 − 0.148) × (3225 − 2440)
vf6 = 0.001010 m3/kg = 1733.82 kJ/kg
h6 = hf6 = 191.83 kJ/kg Net work done per kg of steam
State 7: Compressed liquid; wnet = wT − wp = 1733.82 − 16.28
p7 = 5 bar = 500 kPa = 1717.54 kJ/kg
State 8: Saturated liquid (from steam tables); The heat supplied in the boiler per kg of steam
p8 = 5 bar qin = h1 − h9 + h3 − h2
vf8 = 0.001093 m3/kg, = 3580 − 647 + 3695 − 2985
h8 = hf8 = 641.21 kJ/kg = 3643 kJ/kg
State 9: Compressed liquid; (ii) Thermal efficiency of the cycle
p9 = 15 MPa wnet 1717.54
hth, reg = =
qin 3643
Analysis The high-pressure pump work input per kg of
steam = 0.471 or 47.1 %
wp1 = vf @ 5 bar ¥ ( p9 − p8) (iii) Mass flow rate of steam in kg/h
= 0.001094 (15 − 0.5) ¥ 103 P 120 ¥ 103
ms = =
= 15.86 kJ/kg wnet 1717.54
Specific enthalpy at the state 9; = 69.867 kg/s or 251.52 kg/h
h9 = h8 + wp2 = 641.21 + 15.86
= 657 kJ/kg
The low pressure pump work input per kg of steam
wp2 = vf @ 10 kPa ¥ ( p5 − p4)
= 0.001010 ¥ (5 − 0.1) ¥ 102 A closed feed-water heater is an indirect contact-
= 0.5 kJ/kg type feed water heater, usually a shell and tube type
Specific enthalpy at the state 7; heat exchanger, in which heat is transferred from
h7 = h6 + wp2 extracted steam to feed-water without mixing of
= 191.83 + 0.5 = 192.32 kJ/kg two fluid streams. Thus the two fluid streams can
(i) The fraction of steam extracted from the turbine be kept at different pressures. Figure 12.42 shows
for feed-water heater two arrangement for removing the condensate from
The mass of steam m1 extracted from the turbine the closed feedwater heater. In Fig. 12.42(a), the
at 5 bar for regeneration can be obtained by condensate is pumped forward to a feed-water line.
making mass balance on feed-water heater; In Fig. 12.42(b), the condensate is allowed to pass
(1 − m1) h7 + m1 h4 = h8 another feed-water heater or condenser through a
h -h 641.21 - 192.32 device called a steam trap. A trap is a type of valve
m1 = 8 7 =
h4 - h7 3225 - 192.32 that allows only liquid to be throttled to a lower
= 0.148 kg pressure, but it traps the vapour.
Therefore, 0.148 kg of steam is extracted from the A regenerative vapour power cycle with one
turbine for each kg of steam entering the turbine. closed feed-water heater with the condensate
Vapour Power Cycles 427

trapped into the condenser is shown is Fig. 12.43.


In this cycle, the working fluid passes isentropically
through the turbine stages and pumps, and without
any pressure drop in the components. As shown with turbine at intermediate pressure can be determined
the help of a T–s diagram, the total steam expands by applying mass and energy balance to the closed
through first stage turbine from state 1 to state 2. water heater.
At state 2, a part of steam (m1, kg) is extracted and
(m1 kg)h2 + (1 kg)h8 = (1 kg)h4 + (m1 kg)h5
is supplied to closed feed-water heater, where it
condenses, on outside of tubes, carrying the feed- h4 - h8
or m1 = kg/kg of steam
water. The saturated liquid at extraction pressure h2 - h5
exits the feed-water heater at state 5 and is routed ...(12.50)
to the condenser through a trap, where it is mixed Remark With the use of closed feed water heater,
with the condensate of steam passing the second- separate pumps to handle the feedwater for each
stage turbine. In the trap, the steam is throttled from heater are not required. But the main drawback of
state 5 to state 6. It is an irreversible process, and such heaters, that they do not heat the feedwater
thus shown by dotted line. Then total condensate very close to the saturation temperature as open
as saturate liquid at state 7 is pumped to boiler heaters do.
pressure and enters the feedwater at state 8. The
temperature of feedwater in the heater is raised to The steam at 30 bar, 400°C is
state 4. The cycle completes as working fluid enters supplied to a turbine and is expanded isentropically to
the boiler and heated at constant pressure from state a pressure of 3 bar, where some steam is extracted for
4 to state 1. a surface heater, in which the feed-water is heated to
130°C. The remaining steam from feed-water is cooled
Let 1 kg of steam leaving the in a drain cooler to 27°C. The feed-water before entering
boiler enters the first stage turbine. Figure 12.43. the feed heater is used as coolant in the drain cooler.
is labelled with the fraction of the steam at various The cooled drain water is then mixed with condensate
states. The mass fraction m1, extracted from the at 0.04 bar.
428 Thermal Engineering

Determine
(a) Mass of steam used for feedwater heating per kg
of steam entering the turbine.
(b) Thermal efficiency of the cycle.

Solution
Given A regenerative vapour power cycle with one
closed feed-water heater with operating conditions
First stage turbine inlet = 30 bar, 400°C
Extracted steam pressure = 3 bar
Feedwater temperature leaving the heater = 130°C
The temperature of feedwater in drain cooler = 27°C
Assumptions
To find
(i) Each component in the cycle is analysis as a
(i) Mass of steam extracted for feed-water heating control volume at steady state.
per kg of steam.
(ii) Turbines, pump and feed-water heaters operate
(ii) Thermal efficiency of the cycle. adiabatically.
Schematic with given data (iii) All processes of working fluid are reversible.
(iv) The extracted steam leaving the feed-water heater
30 bar as a saturate liquid.
1 kg 400°C
(v) In drain cooler, both fluid get equilibrium
1
Turbine Turbine WT temperature of 27°C.
I II
m1 kg (vi) 1 kg mass of steam flowing through the boiler and
qin first stage turbine.
3 bar 2 3 (1 – m1) kg
Boiler
3 bar (vii) Kinetic and potential energy effects are negligible.
0.04 bar (viii) The specific heat of water as 4.187 kJ/kg ◊ K.
Condenser
m1
7 Analysis Referring schematic and T–s diagram of
1 kg 130°C m1
4
Fig. 12.44.
8 Pump
Closed Drain State 7: Saturated liquid at
5
feedwater cooler Wp p7 = 0.04 bar (steam table)
heater m1
vf 7 = 0.001004 m3/kg
(a) Schematic
h7 = 121.44 kJ/kg
T State 8: Compressed liquid at
p8 = 30 bar, T8 = 27°C
1
Work input to pump
9 wp = vf 7 (p8 – p7)
30 bar = 0.001004 ¥ (30 ¥102 – 0.04 ¥ 102)
4 m1 2 = 3.0 kJ/kg
8 5 3 bar
The specific enthalpy of water at state 8
(1 – m1)
3 h8 = h7 + wp = 121.44 + 3.0
7 6 0.04 bar
= 124.44 kJ/kg
s State 1: Superheated steam at
(b) T–s diagram
p1 = 30 bar, T1 = 400°C
(From Mollier diagram)
h1 = 3230 kJ/kg, s1 = 6.868 kJ/kg ◊ K
Vapour Power Cycles 429

State 2: Wet steam at Solution


p2 = 3 bar
Given A regenerative vapour power cycle with two
h2 = 2700 kJ/kg, s2 = 6.868 kJ/kg ◊ K
closed feed-water heaters with operating parameters.
State 3: Wet steam at
First stage turbine inlet = 40 bar, 500°C
p3 = 0.04 bar
Intermediate pressures 10 bar and 1.1 bar
h3 = 2085 kJ/kg, s3 = 6.868 kJ/kg ◊ K
Condenser pressure 0.035 bar
State 4: Feed water at
p4 = 30 bar, T4 = 130°C To find
State 5: Saturated liquid, (i) The amount of steam extracted per kg of steam
p5 = p4 = 3 bar, for two closed water heaters.
h5 = hf 5 = 561.45 kJ/kg (ii) The work output of the plant per kg of steam.
(i) Mass of steam m1 extracted after first stage (iii) The thermal efficiency of the cycle.
turbine expansion Assumptions
Energy balance on closed feedwater heated and (i) Each component in the cycle is analysed as a
drain cooler combined; control volume at steady state.
Heat lost by steam = Heat gain by feedwater (ii) Turbines, pump and feedwater heaters operate
(m1 kg)(h2 – h5) = (1 kg) Cpw (T4 – T8) adiabatically.
1 ¥ 4.187 ¥ (130 - 27) (iii) All processes of working fluid are reversible.
or m1 =
2700 - 561.45 (iv) The extracted steam leaving the feedwater heater
= 0.20 kg/kg of steam as a saturate liquid.
(ii) Thermal efficiency of the cycle (v) 1 kg mass of steam flowing through the boiler and
Total work done per kg of steam turbines first stage turbine.
wT = (h1 – h2) + (1 – m1) (h2 – h3) (vi) Kinetic and potential energy effects are negligible.
= (3230 – 2700) + (1 – 0.02) ¥ (2700 – 2085)
Analysis Referring the Mollier diagram, we get spe-
= 530 + 492 = 1022 kJ/kg
cific enthalpies of steam:
Net work per kg of steam in the cycle
At State 1: Superheated steam
wnet = wT – wP = 1022 – 3.0 = 1019 kJ/kg
p1 = 40 bar, T1 = 500°C
Heat supplied per kg of steam in the cycle
h1 = 3450 kJ/kg, s1 = 7.08 kJ/kg ◊ K
qin = h1 – h4 = 3230 – 4.187 ¥ (130.0)
State 2: Superheated steam
= 2685.7 kJ/kg
p2 = 10 bar
w 1019
Thermal efficiency hth = net = h2 = 3034 kJ/kg, s2 = s1
qin 2685.7
State 3: Wet steam
= 0.379 = 37.9%
p3 = 1.1 bar
A regenerative vapour power cycle h3 = 2588 kJ/kg, s3 = s1
uses two closed feed-water heaters. The steam is supplied State 4: Wet steam
to the turbine at 40 bar, 500°C and is exhausted to p4 = 0.035 bar
condenser at 0.035 bar. The intermediate bleed pressures h4 = 2122 kJ/kg, s4 = s1
are obtained such that the saturation temperature State 5: Saturated liquid at p5 = 0.035 bar (From steam
intervals are approximately equal, giving pressures of 10 table)
and 1.1 bar.
hf 5 = 112 kJ/kg,
Determine the amount of steam extracted at each
vf 5 = 0.0010035 m3/kg
stage, the work output of the plant per kilogram of the
boiler stream and the cycle efficiency of the plant. Assume Pump work input
that the condensed steam from first feed-water heater is wp = vf 5 ( p6 – p5)
throttled to second water heater and from second water = 0.0010035 ¥ (40 – 0.035) ¥ 102
heater to condenser. = 4 kJ/kg
430 Thermal Engineering

40 bar, 500°C State 8: Liquid at p8 = 1.1 bar, throttled from state 7, thus,
Turbines
1 kg h8 = h7 = 762.79 kJ/kg
qin Boiler
State 9: Saturated liquid,
p9 = 1.1 bar
2 4
h9 = 428.2 kJ/kg
12 m1 kg 3
m2 (1 – m1 – m2) State 10: Liquid at p10 = 0.035 bar, throttled from state
10 bar
1.1 bar 9, thus,
1 kg Closed Closed
heater II heater I (1 – m1) Condenser h10 = h9 = 428.2 kJ/kg
11 qR
State 11: Heated liquid at p11 = 40 bar, with steam at
0.035 bar
6 5 1.1 bar, thus
Wp h11 = h9 = 428.2 kJ/kg
7 8 9
m1 feed State 12: Heated liquid at p12 = 40 bar with steam at
10 pump
m2 10 bar
T h12 = h7 = 762.79 kJ/kg
1 (i) Applying mass and energy balance to second
closed feed-water heater II,
40 bar
2
Smihi = Smehe
10 bar
12 h11 + m1h2 = h12 + m1h7
7 m1
m2
11
3 h12 - h11 762.79 - 428.2
6 9 8 1.1 bar m1 = =
5 h2 - h7 3034 - 762.79
10 0.035 bar 4
s
= 0.147 kg/kg of steam
(a) Example of a power plant layout. Applying mass and energy balance to, first closed
h
feed-water heated,
C

Smihi = Smehe

(kJ/kg)
500°C
40

3450 m2h3 + (1 – m1) h6 + m1h8 = h11 + m2h9


1
r
ba

2588 m2 + (1 – 0.147) ¥ 116 + 0147 ¥ 762.79


r
10

ba

= 428.2 + 428.2 m2
1
1.

3034 2
ba

It gives m2 = 0.10 kg/kg of steam


5
03
0.

2588 3 (ii) Heat supplied to 1 kg steam in cycle


2122
qin = h1 – h12 = 3450 – 762.79
4
= 2687.21 kJ/kg
Net work done per kg of steam in cycle
0 s
7.08 s (kJ/kg K) wnet = wT – wP
(b) h–s diagram for determination of specific = (h1 – h2) + (1 – m1) (h2 – h3)
entgalpies durning isentropic expansions. + (1 – m1 – m2) (h3 – h4) – wp
= (3450 – 3034) + (1 – 0.147)
(3034 – 2588) + (1 – 0.147 – 0.1)
State 6: Compressed liquid ¥ (2588 – 2122) – 4
p6 = 40 bar = 1143.32 kJ/kg
h6 = h5 + wp (iii) Thermal efficiency of the cycle
= 112 + 4 = 116 kJ/kg wnet 1143.32
hth = =
State 7: Saturated liquid, qin 2687.21
p7 = 10 bar = 0.4254 = 42.54%
h7 = 762.79 kJ/kg
Vapour Power Cycles 431

1 kg 15 MPa, 600°C
Consider a reheat-regenerative 1
vapour power cycle with two feed-water heaters. The
steam enters the first turbine at 15 MPa, 600°C and Boiler W1
expands to 4 MPa. Then some steam is extracted from the qin
4 MPa 1 Kg 2
turbine at this pressure for closed feed-water heater and 10 kPa

the remaining steam is reheated to 600°C at the same m1 3 4


5

pressure. The extracted steam is completely condensed 13 m2


Closed Open
in the heater and is then pumped to 15 MPa before it Mixing 260°C
feed- feed- qout
chamber 10 water water
mixes with the feed-water at 260°C at the same pressure. heater 9 heater 7 Condenser
The steam for open feed-water heater is extracted from 12 6
11 8
low pressure turbine at the pressure of 0.5 MPa and m1 (1 – m1) (1 – m1 – m2)
the remaining steam is further expanded to condenser Pump I Pump II Pump III
Wp1 Wp2 Wp3
pressure of 10 KPa. If the working fluid experiences no
(a) Schematic
irreversibities, as it passes thought turbines, pumps,
boiler, reheater condenser, determine T 1 3
600°C
(a) The fraction of steam extracted from the turbines 13 15 MPa
12
each time. 4 MPa
10 2
(b) Thermal efficiency of the cycle. 9 11 m1
8 0.5 Mpa
(c) Mass flow rate of steam in kg/h, if cycle produces m2 4
7
120 MW. 10 kpa
6
(1 – m1 – m2) 5
Solution
s
Given A reheat-regenerative ideal vapour power cycle (b) T–s diagram
with open and closed feed-water heater with T
1 3
First stage turbine inlet = 15 MPa, 600°C 600°C
Intermediate pressures = 4 MPa and 0.5 MPa
Reheating = 4 MPa, 600°C 4
2
Condenser Pressure = 10 kPa.
Pa

Pa
M
Pa
15

To find M
M

5 a 5
0. kP
4

(i) Fraction of steam extracted after first and second 10


stage expansion in the turbine. s
(ii) Thermal efficiency of the cycle. (c) h–s diagram for determination of specific enthalpies
(iii) Mass flow rate of steam in the cycle for network
output of 120 MW.
Assumptions (vii) 1 kg of steam flow through the boiler and first
(i) Each component in the cycle is analysed as a stage turbine.
control volume at steady state. Analysis
(ii) All process of the working fluid are reversible. The specific enthalpies at various states from steam
(iii) Turbines, pumps and feedwater heaters operate tables, and Mollier diagram.
adiabatically. h11 = 1087.31 kJ/kg vf11 = 0.001252 m3/kg
(iv) No pressure drop in boiler, condenser and h1 = 3582.3 kJ/kg h2 = 3154.3 kJ/kg
feedwater heaters.
h3 = 3674.4 kJ/kg h4 = 3014.3 kJ/kg
(v) The working fluid leaves the condenser and
h5 = 2335.8 kJ/kg h6 = 191.83 kJ/kg
feedwater heaters as saturated liquid.
h8 = 640.23kJ/kg vf8 = 0.001093 kJ/kg
(vi) Kinetic and potential energy effects are negligible.
432 Thermal Engineering

h10 = Cpw T10 = 1134.35 kJ/kg (at 260°C) m1h12 + (1 – m1 ) h10 = h13
vf6 = 0.001080.kJ/kg or h13 = 0.188 ¥ 1101.08
The work input per kg of steam in pump 1 + (1 – 0.188) ¥ 1134.35
wp1 = vf 12 (p13 – p12) = 1128.1 kJ/kg
= 1.001252 ¥ (15 ¥ 103 – 4 ¥ 103) (iii) Thermal Efficiency of the cycle:
= 13.772 kJ/kg The heat supplied in the boiler
The specific enthalpy of water at state 12 qin = h1 – h13 + (1 – m1) (h3 – h2)
h12 = h11 + wp1 = 1087.31 + 13.772 = 3582.3 – 1128.1 + (1 – 0.188)
= 1101.08 kJ/kg ¥ (3674.4 – 3154.3)
The pump 2, work input wp2 = vf 8 (p13 – p8) = 2876.5 kJ/kg
= 0.001093 ¥ (15 ¥103 – 0.5 ¥ 103) Heat rejected in the condenser
= 15.85 kJ/kg qout = (1 – m1 – m2) (h5 – h6)
The specific enthalpy of water at state 9 = (1 – 0.188 – 0.129) ¥ (2335.8 – 191.83)
h9 = h8 + wp2 = 640.23 +15.85 = 1464.33 kJ/kg
= 656.08 kJ/kg Thermal efficiency of the cycle
The pump III. work input qout 1464.33
hth = 1 – =1-
wp3 = vf 6 (p8 – p6) qin 2876.5
= 0.001080 ¥ (0.5 ¥ 103 – 10) = 0.491 or 49.1%
= 0.53 kJ/kg (iv) The mass flow rate in the cycle
The specific enthalpy of water at state 7, Net work done per kg of steam
h7 = h6 + wp3 wnet = hth qin = 0.491 ¥ 2876.5
= 191.83 + 0.53 = 192.36 kJ/kg = 1412.17 kJ/kg
(i) Let m1 fraction of steam extracted at 4 MPa and The mass flow rate of steam in the cycle
m2 fraction of steam extracted at 0.5 MPa. Power output 120 ¥ 103 kJ/s
m = =
The mass and energy balance on the closed Work done/kg 1412.17 kJ/kg
feedwater heater = 84.97 kg/s
Smihi = Smehe = 30.59 ¥ 104 kg/h
m1h2 + (1 – m1) h9 = m1h11 + (1 – m1) h10
or m1 (h2 – h9 – h11 + h10) = h10 – h9
With numerical values;
1134.35 - 656.08
m1 = The thermal efficiency of the regenerative vapour
3154.3 - 656.08 - 1087.31 + 1134.35
power cycle can be increased by incorporating
or m1 = 0.188 kg/kg of steam
several feedwater heaters at suitably intermediate
(ii) Energy and mass balance on open feedwater
pressures. However, the capital cost of plant also
heater
increases due to addition of heaters, piping, pumps
Smihi = Smehe
and valves. Therefore, the incremental increase in
m2h4 + (1 – m1 – m2) h7 = (1 – m1) h8
thermal efficiency achieved with each additional
3014.3 m2 + (1 – 0.188 – m2) ¥ 192.36
heater must justify its added cost.
= (1 – 0.188) ¥ 640.23
Figure 12.48 shows a schematic of vapour power
or 3014.3 m2 + 156.09 – 192.36 m2 = 519.47
cycle plant cycle with reheating and three closed
363.66
or m2 = = 0.129 kg/kg of steam feedwater hearers and one open feed-water heater.
2821.94
The power plants with multiple feed-water heaters
The specific enthalpy of water at state 13 is
must have at least one open feed-water heater,
determined by mass and energy balance in
operating above atmospheric pressure for proper
mixing chamber
Vapour Power Cycles 433

Qs
4
Boiler
WT

2 3
6 7 8
1
QR
Closed Closed Closed Condenser
heater De-aerating heater
14 heater
open
13 heater 10 9
12
Wp Wp
2 1
16 Main boiler Condensate
15 17 pump
feed pump
18

de-aeration of air and other gases from the feed- 6. The size of condenser is reduced.
water in order to minimise the corrosion. 7. In open feedwater heater, de-aeration takes
Analysis of regeneration cycle with multiple place to remove the air and non-condensable-
feed-water heater is similar to previous article. gases from the feedwater, otherwise they
Let 1 kg of steam enters the boiler and first stage would lead to corrosion in the system.
turbine and then quantify the fraction passing Disadvantages
through the various components. The fractions of
1. The vapour power plant becomes compli-
steam extracted are determined by mass and energy
cated.
balance principles.
2. Addition of heaters increase the cost of plant
and maintenance.
3. In feedwater heaters, the length of fluid flow
increases, which require more pump work.
Advantages 4. The specific steam consumption increases
with regeneration.
1. The heat is added in the boiler at a higher av-
erage temperature, thus, the heating process,
tends to be reversible. RANKINE CYCLE
2. The variation in temperature of working Figure 12.49 shows the indicator diagram for modi-
fluid in the boiler is minimised, thus thermal fied Rankine cycle for steam engine. High-pressure
stresses setup in the boiler are also reduced. steam enters the cylinder at constant pressure, it
3. Amount of heat addition is substantial re- expands isentropically in the cylinder up to a lower
duced, thus the efficiency of the cycle is im- pressure, which is called steam release point and
proved. then the steam is released at constant volume to a
4. The size of stage turbine is reduced, due to condenser or into atmosphere.
the reduced amount of steam passes through The expansion of steam is not taken to toe of the
these turbines. curve, i.e., the back (condenser) pressure or else the
5. Due to many extractions, the turbine drain- stroke of the engine will be very long , but the work
age is improved and erosion due to moisture obtained in this part is very small. Thus, the length
is reduced. of stroke reduces without appreciable reduction in
434 Thermal Engineering

Heat supplied to cycle


Q = h1 − hf 4

h1 - h2 + v2 ( p2 - p3)
h= …(12.50)
h1 - h f 4

12.14 CHARACTERISTICS OF THE


WORKING FLUID IN VAPOUR
POWER CYCLE

Water is the best suitable working fluid for majority


of vapour power plants. Because it has following
favourable characteristics:
1. It is easily available in large quantity at very
low cost.
2. It is non-toxic, chemically stable and rela-
tively poor corrosive.
3. It has a large change in specific enthalpy,
when it vapourises at ordinary pressures,
thus it limits the mass flow rate for a desired
power output.
4. Due to very low specific volume of liquid
water, the pumping work required is very
less, thus, the work ratio of the cycle is quite
power output of the engine. It is the modification in high.
Rankine cycle used for steam engines. 5. At low triple point temperature there is no
Cycle 5–1–2–3–4–5 is modified Rankine cycle. chance of solidification of water in the cycle.
The sequence of operations are shown in 6. Good heat transfer characteristics.
Fig. 12.49(a). Apart from certain desirable characteristics of
Work done by expansion of steam per water as a working fluid in vapour power cycle,
cycle in modified Rankine cycle. it does not fare well for high critical temperature,
controlled condenser pressure, and dry expansion,
w = Area of indicator diagram
because.
= Area 5 –1– c – 0 –5 + Area 1–2–
(i) Low critical temperature restricts the addi-
3 – b–c–1 – Area 3–4–0–b–3
tion of heat isothermally during vapourisa-

Ú
= p1v1 + pd v - p3 v2 tion at maximum temperature.
(ii) Very high pressure at high temperature,
or w = p1 1 + u1 − u2 − p3 2 which requires high strength of materials of
or w = (p1 1 + u1) − (u2 + p2 2) components used in the plant.
+ p2 2− p3 2
(iii) During condensation of steam a large change
= h1 − h2 + 2 (p2 − p3) in specific volume experiences a very low
pressure in the condenser. The air leakage
Vapour Power Cycles 435

problem at the joints in the condenser are


very commonly encountered.
(iv) The excessive formation of moisture dur-
ing expansion in the turbine is undesirable.
The moisture removal requires reheating of
wet steam at number of stages in the turbine.
Thus the cost of heat addition increases.

COGENERATION

In all the thermal cycles discussed so far, the ef-


fective conversion of heat energy into network is
considered valuable. Therefore, the various meth-
ods are used to improve the thermal efficiency of
the system. Typically, the thermal systems are used
by almost every industry, consist of a power gen-
erating unit and a heat recovery unit. The power
at state 3. The fraction m1 of steam is adjusted
generating unit develops electrical power, while the
such that working fluid leaves the process heating
heat recovery unit utilizes the remaining heat ef-
section as a saturated liquid. This fraction rejoins
fectively as a process heat. Some industries heavily
the condensed fraction (1 – m1) and the combined
rely on process heat are chemical, paper, pulp, oil
steam is returned to the boiler through pump for
production, refineries, food processing, textile and
next cycle. When no process steam is required, all
steel making, etc. The process heat in these indus-
of the steam generated by the boiler is allowed to
tries is usually supplied by steam at 5 to 7 bar and
expand through the turbine.
150 to 200°C.
With the cogeneration, a large fraction of the
Cogeneration is a method, which is used to
energy transferred to steam in the boiler is utilized
improve the energy resource utilization. The
as either process heat and or electric power. The
cogeneration method, produces the power as well
utilization fraction eu for a cogeneration plant is
as the required heat transfer (process heat) for some
devices. Network output + Process heat diverted
eu =
The main aim of the cogeneration is to produce Total heat input
power and process heat simultaneously, using an wnet + q process
=
integrated system at a lower cost than that would be qin
required to develop them individually. qout
Either a steam power cycle or gas power cycle or or eu = 1 - ...(12.51)
qin
even a combined cycle can be used as a power cycle
for cogeneration. where qout includes heat rejected in the condenser
and all undesirable heat losses during process heat.
A schematic of a cogeneration system using
The best designed cogeneration plant may have
vapour power cycle is shown in Fig. 12.50. The
utilization factor as high as 70%.
m1 fraction of the steam supplied to turbine is
extracted at an intermediate point 2 and diverted Consider the cogeneration plant,
to some process that requires the steam at this shown in Fig. Ex. 12.51. Steam enters the turbine at
condition. The remaining fraction (1 – m1) of the 7 MPa and 500°C. Some steam is extracted from the
steam in the turbine expands to condenser pressure turbine at 500 kPa for process heating. The remaining
436 Thermal Engineering

steam continues to expand to 5 kPa. Steam is then Solution


condensed at constant pressure and pumped to the boiler
pressure of 7 MPa. At times of high demand for process Given The schematic of a cogeneration plant with data.
heat, some steam leaving the boiler is throttled at 500 To find
kPa and is routed to the process heater. The extraction (i) Maximum rate of process heat.
fractions are adjusted so that steam leaves the process (ii) Power produced and utilization factor, when no
heater as a saturated liquid at 500 kPa. It is subsequently process heat is supplied.
pumped to 7 MPa. The mass flow rate of steam through (iii) The rate of process heat supply, when 10% of
the boiler is 15 kg/s. Disregarding any pressure drops steam is extracted, before it enters the turbine and
and heat losses in the piping and assuming the turbine 70% of steam is extracted from turbine at 500 kPa
and the pump to be isentropic, determine for process heat.
(a) the maximum rate at which process heat can be (iv) Utilization factor for case (iii).
supplied,
Assumptions
(b) the power produced and the utilization factor
(i) Each component in the cycle is analysis as a
when no process heat is supplied, and
control volume at steady state.
(c) the rate of process heat supply, when 10 percent (ii) The steam leaving the condenser and process heat
of the steam is extracted before it enters turbine as a saturated liquid.
and 70 percent of the steam is extracted from (iii) All process of working fluid are internally
turbine at 500 kPa for process heating, reversible, except throttling.
(d) utilization factor for case (iii). (iv) Kinetic and potential energy effects are negligible.
Analysis The properties of steam at various states are
taken from steam tables and Mollier diagram.
State 1, 2, 3: Superheated steam
p = 7 MPa, T = 500°C
h1 = h2 = h3 = 3410 kJ/kg
State 4: Superheated region after throttling
h4 = h3 = 3410 kJ/kg
State 5: Steam at p = 500 kPa, after first stage isentropic
expansion
s1 = s5
h5 = 2740 kJ/kg
x5 = 0.995
State 6: Steam after second stage expansion,
p = 5 kPa
s1 = s6
h6 = 2072 kJ/kg
x6 = 0.8
State 7: Saturated liquid at 500 kPa
h7 = hf 7 = 640.23 kJ/kg
v7 = vf 7 = 0.001095 m3/kg
State 8: Saturated liquid at p = 5 kPa
h8 = hf 8 = 137.82 kJ/kg
v8 = vf 8 = 0.001005 m3/kg
Work input to pump I,
wp1 = v8 (p9 – p8)
= 0.001005 ¥ (7 ¥ 103 – 5)
= 7.03 kJ/kg
Vapour Power Cycles 437

Work input to pump II, Mass of steam extracted from turbine,


wp2 = 0.001095 (7 ¥ 103 – 500) m5 = 0.7 ¥ 15 = 10.5 kg/s
= 7.11 kJ/kg Total mass of steam for precess heat
State 9: Compressed liquid at p = 7 Mpa m7 = 10.5 + 1.5 = 12 kg
h9 = h8 + wp1 = 144.85 kJ/kg
Applying mass and energy balance on the process
State 10: Compressed liquid at p = 7 Mpa heater
h10 = h7 + wp2 = 647.35 kJ/kg
Smihi = S me he + Q p
(i) Maximum rate of Process heat
Heat supply to turbine = 0 m4 h4 + m5 h5 = m7 h7 + Q p
Heat supply to process heat is 100% (via throttle 1.5 ¥ 3410 +10.5 ¥ 2740
value) = 12 ¥ 640.23 + Q p
Q Process,max = m (h4 – h7) or Q p = 26202.24 kW = 26.2 MW
= (15 kg/s) (3410 – 640.23) (kJ/kg)
(iv) The rate of work produced in the turbine
= 4.155 ¥ 104 kW
Utilization factor is 100% since no heat is rejected WT = m3 (h3 – h5) + ( m6 ) (h5 – h6)
of lost. where m3 = mass rate of steam entering the first
(ii) Power produced and utilization factor, when no stage turbine
process heat is supplied = m – m4 = 15 – 1.5 = 13.5 kg/s
Steam passes through turbine, m = 15 kg/s
m6 = mass rate of steam entering
Turbine work second stage turbine
WT = m (h1 – h6) = m3 - m5 = 13.5 – 10.5 = 3 kg/s
= (15 kg/s) (3410 – 2072) (kJ/kg) Thus WT = 13.5 ¥ (3410 – 2740) + 3
= 20070 kW ¥ (2740 – 2072) = 11049 kW
Work input to pump Rate of work input to pump
W p = m wp1 = (15 kg/s) ¥ (7.03 kJ/kg) W p = ( m6 ) wp1 + ( m7 ) wp2
= 105.45 kW
= 3 ¥ 7.03 + 12 ¥ 7.11 = 106.41 kW
Net work output of the plant
Net power output from the plant
Wnet = WT – W p Wnet = WT – W p
= 20070 – 105.45 = 19964.55 kW
= 11049 – 106.41 = 10942.6 kW
Heat supplied in the boiler,
The rate heat supplied to working fluid in the
Qin = m (h1 – h9) boiler
= (15 kg/s) ¥ (3410 – 144.35) (kJ/kg) Qs,3 = m7 (h3 – h10) + m6 (h3 – h9)
= 48977.25 kW = (12 kg/s)(3410 – 647.35) (kJ/kg)
Utilization factor, + (3 kg/s) ¥ (3410 – 144.85)(kJ/kg)
Wnet 19964.55 = 33151.8 + 9795.45
eu = =
Qin 48977.25 = 42947.25 kW
= 0.408 = 40.8% The utilization factor
Thus 40.8 percent of heat supplied is converted to Wnet + Q p
eu =
useful work. Qs,3
(iii) Rate of process heat supply 10942.6 + 26202.24
Mass of steam throttled, =
42947.25
m4 = 0.1 ¥ m = 0.1 ¥ 15 = 1.5 kg/s = 0.865 or 85%
438 Thermal Engineering

The binary vapour cycle is the combination of


two cycles. One operates in the high temperature
region, while other operates in low temperature
region. It uses two working fluids one with good
high temperature characteristics and another with
good operating characteristics at lower temperature.
In the binary vapour cycle, the two ideal Ranking
cycles are combined such that the condenser of
the high temperature cycle (also called topping
cycle) serves as the boiler for the lower temperature
cycle (bottoming cycle). Thus the heat rejection
of the topping cycle is used as a heat input to the
bottoming cycle.
Some common working fluids used in top-
ping cycle are mercury, sodium, potassium, and
sodium-potassium mixture. The mercury is a good
working fluid for binary vapour cycle. Its critical
temperature is 898°C, which is more than metal-
lurgical limit, thus heat can be supplied to mercury
isothermally at higher temperature, its critical pres-
sure is only about 18 MPa, thus safer in the cycle.
Although mercury cannot be used as sole working
fluid for entire cycle, since at atmospheric tem-
perature, 32°C, its saturation pressure is too low
0.07 Pa, which creates air leakage problems. At an
acceptable condenser pressure of 7 kPa, its satura-
tion temperature, 237°C, is high as minimum tem-
perature in the cycle. Further due to its low spe- POWER CYCLE
cific enthalpy, the mass flow rate of mercury in the
binary cycle is several times higher that of water. In order to improve the thermal efficiency of the
Therefore, the use of the mercury as a working fluid power plant, the certain innovative modification are
is limited to high temperature cycle only. made to the conventional power plants. The binary
vapour power cycle introduced above is an example
The schematic and T–s diagram of a binary
of such modification. A more popular modifica-
vapour cycle mercury and water are shown in
tions is a gas power cycle topping the vapour power
Fig. 12.52. The heat transfer between two cycles is
cycle, which is known as the combined gas-vapour
accomplished in an interconnecting heat exchanger,
cycle.
which serves as a condenser for the mercury cycle
and the boiler for steam cycle. It is evident from T–s The gas turbine cycles typically operate at
diagram that the binary vapour cycle approximate considerable higher temperature than vapour power
the Carnot cycle more closely than a cycle for the cycle. The gases leaving the gas turbine cycle are at
same temperature limits. Therefore, the efficiency very high temperature (usually above 500°C). One
of such cycle is more than steam cycle. way to utilise some of heat of high temperature
Vapour Power Cycles 439

gases for regeneration, thereby improving overall where Wgas = net power developed by the gas
fuel utilization. The combined cycle is shown in
turbine plant,
Fig. 12.53. The gases leaving the turbine of gas
power cycle transfer the heat to vapour power cycle Wvapour = net power developed by the steam
through a heat exchanger. cycle, and
The combined cycle has high average tempera- Qs = total rate of heat transfer to the
ture of heat addition for gas turbine cycle and low combined cycle.
average temperature of heat rejection from vapour These energy balance on the heat exchange
cycle, and thus an increased thermal efficiency. gives a relation.
The thermal efficiency of the combined cycle is ex- msteam (h3 – h2) = mgas (hd – he) ...(12.53)
pressed as For many applications, the combined cycle is
Wgas + Wvapour economical and they are increasingly being used
hcombined = ...(12.52)
Qs world wide for electric power generation.

Qs A combined gas turbine vapour


plant has a net power output of 45 MW. Air enters the
Combustor
compressor of the gas turbine at 100 kPa, 300 K, and
b c
is compressed to 1200 kPa. The isentropic efficiency of
Gas turbine the compressor is 84%. The condition at the inlet to the
Wgas
turbine is 1200 kPa, 1400 K. Air expands through the
Air inlet a turbine, which has an isentropic efficiency of 88%, to
d
Exhaust a pressure of 100 kPa. The air then passes through the
e interconnecting heat exchanger and is finally discharged
at 400 K. steam enters the turbine of the vapour power
3
Heat exchange cycle at 8 MPa, 400°C, and expands to the condenser
Wvapour pressure of 8 kPa. Water enters the pump as saturated
2 Vapour liquid at 8 kPa. The turbine and pump of the vapour cycle
cycle 4 have isentropic efficiencies of 90% and 80%, respectively.
Determine
Condenser
Pump Cooling (a) The mass flow rates of the air and the steam, each
1 water in kg/s, and the net power developed by the gas
Wp turbine and vapour power cycle, each in MW.
(a) Schematic (b) Thermal efficiency of the combined cycle.
c
T Solution
Given A combined gas-vapour power plant operates at
d steady state with
3
Output power, P = 45 MW
Gas compressor inlet = 100 kPa, 300 K
b e
Compressor exit = 1200 kPa
a2 hcomp = 0.84
1 4 Gas turbine inlet = 1200 kPa, 1400 K
s hT = 0.88
(b) T-s diagram Turbine exit = 100 kPa
Heat exchanger exit = 400 K gas
Inlet to steam turbine = 8 MPa, 400°C
440 Thermal Engineering

Condenser pressure = 8 kPa Assumptions


Pump inlet = Saturated water at 8 kPa (i) Each component is analyzed as a control volume
For steam cycle hT = 0.9 at steady state.
hP = 0.8 (ii) The turbines, compressor, pump, and intercon-
necting heat exchanger operate adiabatically.
To find
(iii) Kinetic and potential energy effects are negligible.
(i) Mass flow rate of air and vapour in kg/s.
(iv) There are no pressure drops for flow through the
(ii) Net power developed by gas turbine cycle and combustor, interconnecting heat exchanger, and
vapour power cycle, each in MW. condenser.
(iii) Thermal efficiency of the combined cycle. (v) An air-standard analysis is used for the turbine.
(iv) Full accounting of a availability increase of air The specific heat of air as Cp = 1.005 kJ/kg, g = 1.4
passing through the gas turbine combustor and
discuss the results. Analysis The properties of steam at various states
Schematic with given data State 1: Saturated liquid at
p1 = 8 kPa
Qin
h1 = hf 1 = 173.88 kJ/kg, vf = 0.0010084 m3/kg
Tc = 1400 K State 2: Compressed water at
Combustor
pc = pb = 1200 kPa p2 = 8 MPa
hcomp = 84% b c
hT = 88%
wps = vf 1 (p2 – p1)
Gas turbine Wgas =
Compressor Turbine = 0.0010084 ¥ (8 ¥ 103 – 8) = 8.06 kJ/kg
Ta = 300 K WT – Wc
pa = 100 kPa
h2s = h1 + wps = 173.88 + 8.06 = 181.94 kJ/kg
Air Inlet a
d w ps
Exhaust
Te = 400 K e
Further, hp =
T3 = 400°C wp
pe = p4 = 100 kPa
p3 = 8 MPa w ps 8.06
3 hT = 90% or wp = = = 10.075 kJ/kg
Heat exchanger hp 0.8
Wgas =
Turbine or h2 = 173.88 + 10.075 = 183.96 kJ/kg
WT – Wp
2 Vapour State 3: Superheated steam
cycle 4
p3 = 8 MPa, T3 = 400°C
Condenser h3 = 3138.30 kJ/kg, s3 = 6.3634 kJ/kg ◊ K
Pump Qout
1
State 4: Wet steam at
p1 = p4 = 8 kPa p4 = 8 kPa
hp = 80%
hf 4 = 173.88 kJ/kg, hfg4 = 2403.1 kJ/kg
c
sf 4 = 0.5926
T sfg4 = 8.2287 kJ/kg ◊ K, sg4 = 8.8213 kJ/kg ◊ K
sg4 > s3, the steam is wet at state 4
s e
Gabin e d
r l
tu cyc d Dryness fraction,
s
3
s3 - s f 4 6.364 - 0.5926
b x4s = =
bs e s f g4 8.2287
2 Vapour cycle = 0.701
a 2s
h4s = hf 4 + x4 hfg4
1 4s 4

s
= 173.88 + 0.701 ¥ 2403.1
= 1859.17 kJ/kg
(or from Mollier diagram h4s = 1860 kJ/kg)
Vapour Power Cycles 441

The isentropic efficiency of the vapour cycle turbine Similarly Td = Tc – hT (Tc – Tds)
h3 - h4 = 1400 – 0.88 ¥ (1400 – 688.32)
hT = = 773.72 K
h3 - h4s
Applying mass and energy balance to interconnecting
or h4 = h3 – hT (h3 – h4s)
heat exchanger
= 3138.30 – 0.9 ¥ (3138.30 – 1859.17)
S mi hi = S me he
= 1987 kJ/kg
mair (hd – he) = msteam (h3 – h2)
s4 = 6.7282 kJ/kg ◊ K (From Mollier diagram)
Work done by the steam turbine or mair Cp air (Td – Te) = msteam (h3 – h2)
wT, steam = h3 – h4 mair ¥ 1.005 (773.72 – 400)
= 3138.30 – 1987 = 1151.3 kJ/kg = msteam (3138.30 – 183.96)
Net work done per kg of steam in the vapour power
mair
cycle or = 7.865
msteam
wsteam = wT, steam – wp = 1151.3 – 10.075
= 1141.24 kJ/kg Net work done/kg of air in the gas power cycle
Analysis the gas power cycle. wgas = wT – wcomp
State a: Atmospheric air pa = 100 kPa, Ta = 300 K = (hc – hd) – (hb – ha)
State b: Compressed air, pb = 1200 kPa = Cp (Tc – Td) – Cp (Tb – Ta)
g -1 1.4 -1 = 1.005 ¥ [1400 – 773.72 – 699.26 + 300]
Êp ˆ g Ê 1200 ˆ 1.4 = 228.15 kJ/kg
Tbs = Ta Á b ˜ = 300 ¥ Á
Ë pa ¯ Ë 100 ˜¯ Net work output from combined cycle
= 610.18 K = Steam cycle work + Gas cycle work
State c: Heated compressed air 45 ¥ 103 kJ/s = msteam ¥ 1141.24 + mair ¥ 228.15
pc = 1200 kPa Using mair = 7.865 msteam , we get mass flow rates of
Tc = 1400 K steam and air
State d: Exhaust gas pd = 100 kPa msteam = 15.31 kg/s
Tc 1400 and mair = 15.31 ¥ 7.865 = 120.48 kg/s
Tds = g -1
= 1.4 -1 (ii) Net work developed in gas power cycle
Ê pc ˆ g Ê 1200 ˆ 1.4 mair ¥ wgas = 120.48 ¥ 228.15
ÁË p ˜¯ ÁË 100 ˜¯
d = 27.59 ¥ 103 kW
= 688.32 K = 27.50 MW
Using compressor and turbine efficiency Net work done in vapour power cycle
Tbs - Ta msteam ¥ wsteam = 15.31 ¥ 1141.24
hcomp =
Tb - Ta = 17.50 ¥ 103 kW
Tbs - Ta 610.18 - 300 = 17.5 MW
or Tb = Ta + = 300 +
hcomp 0.84
= 699.26 K

Summary

work as output is called power cycle. The vapour source temperature TH and sink temperature TL
power cycle is a cycle with a working substance is the Carnot vapour power cycle and its thermal
which alternatively vapourises and condenses. efficiency is given by
442 Thermal Engineering

TL produce 1 kWh (3600 kJ) of power. It is denoted


hCarnot = 1 - by ssc and is expressed as
TH
3600 kJ/kWh
Rankine cycle is an ideal cycle for vapour ssc =
power plants. Its thermal efficiency is function of wnet (kJ/kg)
heat supply and heat rejection and is given by Rankine cycle can be
improved by increasing the mean temperature of
h1 - h2 h - h3
h = = 1- 2 heat addition. The mean temperature of heat addi-
h1 - h4 h1 - h4 tion can be increasing by using superheat, reheat
Rankine cycle uses complete condensation and regeneration.
of steam and only liquid water is pumped back
to the boiler. The Rankine cycle requires very reducing moisture content of steam at the turbine
less pumping work (back work), therefore, it has exit.
practically higher efficiency than that of Carnot -
vapour power cycle. ing in the Rankine cycle reduce its thermal ef-
Back work ratio is defined as the ratio of pump ficiency. The isentropic efficiency of turbine is
work input to the work developed by the turbine. defined as
It is denoted by bwr, and given as Actual enthalpy drop
hT =
Pump work w p h4 - h3 Isentropic enthalpy drop
rbw = = =
Turbine work wT h1 - h2 The isentropic efficiency of the pump is given as
specific steam consumption and steam rate Isentropic enthalpy drop
hp =
is defined as the amount of steam required to Actual enthalpy drop

Glossary
Back work Work input to feed pump Heat Rate Amount of heat required by a power plant to
Back work ratio Ratio of pump work input to the work produce 1 kWh of power
developed by the turbine Reheating Heating of wet steam after its expansion in
Work ratio Ratio of the net work output to the work one stage for next stage
developed by the turbine Reheat factor Ratio of cummulative isentropic en-
Steam rate Amount of steam required to produce thalpy drop to isentropic enthalpy from initial pressure
1 kWh (3600 kJ) of power to final pressure

Review Questions
1. What are four basic components of a steam power 5. Draw the schematic for an ideal Rankine cycle.
plant? Write their function in brief. Draw p−v, T−s and h−s diagrams for this cycle.
2. What do you understand by steam rate and heat 6. What are methods which can lead to increase in
rate? What are their units? thermal efficiency of Rankine cycle?
3. Why Carnot cycle is not practical for a steam 7. What are the irreversibilities in a steam power
power plant? plant, which make its thermal efficiency less than
4. Explain how the quality of steam at turbine exit that of Rankine cycle?
gets restricted?
Vapour Power Cycles 443

8. What is reheating? What are the advantage of re- 12. What is the effect of regeneration on (a) specific
heat Rankine cycle? power output, (b) cycle efficiency, and (c) steam
9. What is the effect of reheating of steam on rate?
(a) specific power output, (b) cycle efficiency, and 13. How is the regeneration of steam done in
(c) steam rate? Carnotisation of Rankine cycle?
10. What is regeneration? Draw schematic and T−s 14. Explain the working and analysis of the regenera-
diagram for an ideal regenerative cycle. tive Rankine cycle with one feed-water heater.
11. Why ideal regeneration is not possible? Explain
in brief.

Problems
1. In a steam power cycle, the steam supply is at of 3 kg/s and it is exhausted at 0.1 bar. Calculate
15 bar, dry and saturated. The condenser pressure the followings
is 0.4 bar. Calculate the Carnot and Rankine ef- (a) Net power output
ficiencies of the cycle. Neglect pump work. (b) Steam rate
[25.9%, 23.54%] (c) Heat rejected in the condenser in kW
2. A steam power plant operates between a boiler (d) Rankine cycle efficiency
pressure of 42 bar and a condenser pressure of (e) Actual thermal efficiency of the plant, if the
0.035 bar. The steam enters the turbine just dry boiler efficiency is 90%
and saturated. Calculate for these limits the cycle [(a) 2636.4 kW, (b) 4.1 kg/kWh, (c) 5857.2 kW,
efficiency, work ratio, and specific steam con-
(d) 31.04%, (e) 27.94%]
sumption for
7. Steam at 20 bar, 360°C is expanded in a turbine
(a) Carnot cycle
to 0.08 bar. It then enters a condenser where it
(b) Ideal Rankine cycle is condensed to saturated liquid water. The feed
(c) Rankine cycle when expansion process has pump supplies saturated water back to the boiler.
an isentropic efficiency of 80% (a) Calculate the net work per kg of steam and
[(a) 42.2%,0.739, 4.91 kg/kWh, (b) 36.8%, 0.996, cycle efficiency for ideal Rankine cycle.
3.64 kg/kWh, (c) 29.4%, 0.995, 4.56 kg/kWh] (b) If the turbine and pump have each 80%
3. A simple Rankine cycle works between pressures isentropic efficiency, calculate percentage
of 28 bar and 0.06 bar, the initial condition of reduction in net work and cycle efficiency.
steam being dry saturated. Calculate the cyclic ef- [(a) 2983.41 kJ/kg, 32.55, (b) 20.1%, 20.1%]
ficiency, work ratio and specific steam consump- 8. A steam power plant operating on Rankine cy-
tion rate. [33.57%, 0.997, 4.049 kg/kWh] cle is supplied with steam at 10 bar and 200°C.
4. An ideal Rankine cycle uses superheated steam The condenser vacuum is 600 mm of mercury.
at 50 bar and 500°C. The condenser pressure is Barometer pressure is 750 mm of Hg. If the ef-
0.05 bar. Calculate cycle efficiency and specific ficiency of the plant relative to Rankine cycle is
steam consumption. [39.6%, 2.75 kg/kWh] 65%, calculate (a) specific steam consumption,
5. A steam turbine receives steam at 100 bar and and (b) work ratio.
600°C. It is exhausted steam at 2 bar. For the ideal [(a) 8.888 kg/kWh, (b) 0.998]
Rankine cycle, calculate net work, specific steam 9. A turbine used in a simple Rankine cycle receives
consumption, cycle efficiency and mean effective steam at 15 bar with 5% moisture. The steam
pressure. enters the condenser at a temperature of 29°C.
[995.5 kJ/kg, 3.62 kg/kWh, 32.03%, 11.71 bar] Calculate Rankine cycle efficiency.
6. A steam power plant operates on ideal Rankine [31.77%]
cycle, receives steam at 20 bar and 300°C at a rate
444 Thermal Engineering

10. A steam power plant operates on the Rankine efficiency, (c) relative efficiency, and (d) steam
cycle. The steam enters the turbine at a flow rate consumption for a power output of 100 kW.
of 500 kg/s, a pressure of 50 bar, a temperature of [(a) 400°C, (b) 29.28%, (c) 44.44%,
600°C and a velocity of 30 m/s. The steam leaves (d) 900 kg/h]
the turbine at a pressure of 7.5 bar and a velocity 15. The steam is supplied to a two-stage turbine at
of 100 m/s. Calculate the cycle efficiency and net 40 bar and 350°C. It expands in the first stage
power produced. [39.87%, 696.05 MW] turbine until it is just dry and saturated, then it
11. A steam turbine develops 5 MW, operating on is reheated to 350°C and expanded through the
Rankine cycle receives steam at 3.0 MPa and second-stage turbine. The condenser pressure is
300°C and exhausting it to condenser at a vacuum 0.035 bar. Calculate the work output and heat
of 685 mm of Hg. The barometer reads 760 mm supplied per kg of steam for plant. Assuming ide-
of Hg. Calculate al processes and neglecting feed-pump work, cal-
(a) Rankine cycle efficiency (b) Dryness fraction culate the specific steam consumption and cycle
of steam enetering the condenser (c) Mass flow efficiency.
rate of steam required per hour. [1290 kJ, 3362 kJ, 2.79 kJ/kWh, 38.4%]
[(a) 31.5%, (b) 0.785, (c) 20764 kg/h] 16. A reheat Rankine cycle operates between the
12. Two steam turbines A and B operate with same pressure limits of 26 bar and 0.04 bar. The steam
initial pressure of 15 bar and same degree of su- entering the HP turbine and LP turbine has a tem-
perheat 101.7°C. In turbine A, the exhaust pres- perature of 400°C. The steam leaves the HP tur-
sure is 0.1 bar, whereas in turbine B, the exhaust bine as dry saturated. Compare thermal efficiency
pressure is 0.03 bar, Calculate and steam rate of Rankine cycle without and with
(a) Steam rate of turbines A and B in kg/kWh reheating. Neglect the feed-pump work.
(b) Percentage reduction in steam consumption [(a) 36.02 %, and 36.94%, (b) 3.2
due to lower exhaust pressure in the and 2.66 kg/kWh]
turbine B 17. A reheat Rankine cycle receives steam at 3.5 MPa
(c) Percentage increase in thermal efficiency and 350°C to a HP turbine, where it expands is-
due to lower exhaust pressure in the entropically to 0.8 MPa. The steam is then passed
turbine B through the boiler and it is reheated to 350°C at
[(a) 4.33 kg/kWh and 3.712 kg/kWh, 0.8 MPa. The steam is then expanded isentropi-
(b) 14.22 %, (c) 12.94%] cally to 10 kPa in a LP turbine. Calculate (a) ther-
13. Steam expands in a turbine from 25 bar and 300°C mal efficiency, (b) quality of steam leaving the
to a condenser pressure of 20 kPa. Calculate LP turbine, (c) power developed for a steam flow
Rankine cycle efficiency. rate of 6 kg/min, and (d) thermal efficiency of the
(a) What would be the efficiency if the initial cycle without reheating, if it operates between
temperature of steam be 500°C instead of 3.5 MPa, 350°C and 10 kPa.
300°C? [(a) 34.7%, (b) 0.895 dry, (c) 114.71 kW,
(b) If the boiler pressure is increased to (d) 33.63%]
60 bar maintaining the steam temperature 18. In a single-heater regenerative cycle, the steam
at 500°C, calculate the cycle efficiency. enters the turbine at 30 bar, 400°C and the ex-
Assume condenser pressure remains con- haust pressure is 0.10 bar. The feed-water heater
stant in all cases. is a direct contact type which operates at 5 bar.
[27.64%, (a) 32.26%, (b) 36.63%] Find
14. A steam turbine has a specific steam consump- (a) the efficiency and the steam rate of the
tion of 9 kg/kWh. It is supplied with steam at cycle
10 bar. The steam is exhausted at 0.1 bar and (b) the increase in mean temperature of heat
0.902 dry to a condenser. Calculate (a) quality addition, efficiency and steam rate as com-
of steam supplied to turbine, (b) Rankine cycle pared to the Rankine cycle (without regen-
eration). Pump work may be neglected
Vapour Power Cycles 445

[(a) 36.08%, 43.85 kg/kWh, (b) 27.4°C, (b) efficiency of the cycle,
34.18%, 0.39 kg/kWh] (c) the specific steam consumption.
19. A steam turbine operates on simple regenerative [(a) 0.19 kg/kg, (b)37%, (c) 4.39 kg/kWh]
Rankine cycle. The steam is supplied dry and sat- 20. A regenerative Rankine cycle includes open
urated at 40 bar and is exhausted to condenser at feed-water heater. The steam enters the turbine
0.07 bar. The condensate is pumped at a pressure at 4 MPa and 400°C. The steam expands up to
of 3.5 bar at which it is mixed with bled steam 400 kPa and then some quantity of steam is ex-
from the turbine at 3.5 bar. The resulting mixture tracted from the turbine and is supplied to feed-
is saturated water at 3.5 bar is then pumped to the water heater. The water leaves the feed-water
boiler. Neglecting the pump work and for ideal heater as saturated liquid at 400 kPa. The remain-
cycle, calculate ing steam completes its expansion to 10 kPa.
(a) amount of bleed steam per kg of steam Calculate the cycle efficiency and steam rate.
supplied, [37.44%, 3.69 kg/kWh]

Objective Questions
1. Which one of the following is most popular va-
pour power cycle? Pump work
(a) Carnot cycle (b) Rankine cycle (b)
Turbine work
(c) Joule cycle (d) Binary cycle Net work output
2. Which one of the following has the maximum (c) (d) None of above.
Turbine work
thermal efficiency for a given range of tempera-
6. The back work ratio of Rankine cycle in compari-
tures?
son with Carnot vapour cycle is
(a) Carnot cycle (b) Rankine cycle
(a) less (b) more
(c) Joule cycle (d) Binary cycle
(c) same (d) none of the above
3. Why Carnot vapour power cycle is considered
7. In the Rankine cycle, the heat is added
non-practical cycle?
(a) isothermally
(a) Saturate steam enters the turbine
(b) at constant volume
(b) Pump handles water-vapour mixture
(c) at constant pressure
(c) Incompletely condensation of steam in the (d) adiabatically
condenser
8. Heat rate of a power plant is expressed as
(d) All of the above
Pump work 3600 kJ
4. Why is Rankine cycle considered as most practi- (a) (b)
cal cycle? Turbine work wnet (kJ/kg)
(a) Saturate steam enters the turbine 3600 (kJ)
(c) (d) none of the above
(b) Pump handles water-vapour mixture hth ( kWh )
(c) Complete condensation of steam in the con- 9. Specific steam consumption of a power plant is
denser expressed as
(d) None of the above Pump work 3600 kJ
(a) (b)
5. The back work ratio is defined as Turbine work wnet (kJ/kg)
Net work done in the cycle 3600 (kJ)
(a) (c) (d) none of the above
Heat supplied in the cycle hth ( kWh )
446 Thermal Engineering

10. Thermal efficiency of Rankine cycle can be im- 14. Rankine cycle with multiple regeneration can be
proved by steam approximated as
(a) superheating (b) reheating (a) ideal Brayton cycle
(c) regeneration (d) none of the above (b) Carnot vapour cycle
11. Specific steam consumption of Rankine cycle can (c) ideal Stirling cycle
be reduced by steam (d) none of the above
(a) superheating (b) reheating 15. In regenerative Rankine cycle, the feed water is
(c) regeneration (d) none of the above heated by
12. Rankine cycle efficiency of well maintained (a) exhaust steam (b) condensed steam
steam power plant is in the range of (c) bled-off steam (d) fresh steam
(a) 25 to 35% (b) 10 to 20% 16. The steam engines operate on
(c) 35 to 45% (d) 50 to 60% (a) simple Rankine cycle
13. With reheat Rankine cycle (b) ideal reheat Rankine cycle
(a) quality of exhausted steam is improved (c) Modified Rankine cycle
(b) net work out put of cycle increases (d) ideal regenerative cycle
(c) specific steam consumption decreases
(d) all of the above

16. (c) 15. (c) 14. (b) 13. (a) 12. (c) 11. (a) 10. (a) 9. (b)
8. (c) 7. (c) 6. (a) 5. (b) 4. (c) 3. (d) 2. (a) 1. (b)
Answers
Refrigeration 447

13
Refrigeration

Introduction
A refrigerator and a heat pump are both heat engines operating in reverse direction. In operation, the reversed
heat engine transfers heat energy from a low-temperature source to a high-temperature sink. The work input
is required to transfer the heat from cooler to hotter region. A refrigerator or air conditioner removes heat
energy from a low-temperature region while a heat pump delivers heat to a high-temperature region. The
gas refrigeration cycle, vapour compression refrigeration cycle, vapour absorption cycle, heat pump cycle
and refrigerant properties are explained in this chapter.

REFRIGERATION hours. Using specific enthalpy of fusion of ice as


333.43 kJ/kg, then
Refrigeration is a branch of science which deals (1000 kg) ¥ (333.43 kJ/kg)
with the transfer of heat from a low temperature 1 TR =
(24 h) ¥ (60 min/h)
region to a high-temperature region, in order to = 231.5 kJ/min
maintain a desired region at a temperature below
In practical calculations, 1 TR is taken as 210 kJ/min
than that of surroundings.
or 3.5 kW.
In the refrigeration process, heat is continuously
removed from a low temperature region to a high- Refrigeration
temperature medium by using a low boiling point
refrigerant. External power is required to carry out 1. Refrigeration is used for preservation of food
this process. Therefore, the refrigeration systems items, fruits, vegetables, dairy products, fish
are power-absorbing devices. and meats, etc.
2. Refrigeration is used for preservation of life-
Unit of Refrigeration saving drugs, vaccines, etc., in hospitals and
medical stores.
The capacity of refrigerating machines is measured
often in terms of tonnes of refrigeration. One tonne 3. It is used in operation theaters and intensive
of refrigeration is the amount of refrigerating effect care units (ICU) of hospitals.
(heat removed) produced by uniform melting of 4. It is used for making ice and preservation of
1 tonne (1000 kg) of ice from and at 0°C in 24 ice-creams.
448 Thermal Engineering

5. It is used for providing comfort air-condi-


Warm atmosphere at TH
tioning in offices, houses, restaurants, the- House
at TH
atres, hotels, etc.
6. It is used in industries for improving working QH QH
environment for their employees.
7. It is used for providing suitable working en- REF
Win HP
vironment for some precision machines and Win
precision measurement.
8. It is used in cold storages for preservation of QL
QL
seasonal vegetables, fruits, etc.
Cold refrigerated
9. It is used for preservation of photographic space at TL Cold atmosphere at TL
goods, archeological and important docu-
(a) Schematic of refrigerator (b) Schematic of heat pump
ments.
10. It is used for chilling beverages (soft drinks). Fig. 13.1
11. It is used for cooling of drinking water.
mance (COP), defined as
12. It is used for liquefaction of gases.
Desired output
13. It is used for processing of textiles, printing (COP) =
work, precision articles, photographic mate- Work input
rials, etc. For a refrigerator, it is the ratio of refrigerat-
14. It is used in control rooms and air-crafts. ing (cooling) effect to the work input required to
achieve that effect. Thus
13.2 REFRIGERATORS AND Refrigerating effect
(COP)R =
HEAT PUMPS Work input
Cooling effect
The devices that produces refrigeration effect are =
called refrigerators and the cycles on which they Work input
operate are called the refrigeration cycles. The Referring Fig. 13.1(a)
working fluids used in the refrigeration cycles are QL
called the refrigerants. (COP)R = ...(13.1)
Win
Another device that also transfers heat from a
Similarly, for a heat pump, supplying heat QH to
low-temperature region to a high temperature region
a high-temperature region at the cost of work input
is called heat pump. The refrigerators and heat
Win, Fig. 13.1(b), the coefficient of performance
pumps operate on the same cycle, but they differ in
their objectives. A heat pump maintains a region at Heating effect QH
(COP)HP = =
a higher temperature than that of its surroundings. Work input Win
It absorbs heat from a low-temperature region and Win + QL QL
supplies it to a warmer region. Schematics of a = =1+ ...(13.2)
Win Win
refrigerator and heat pump are shown in Fig. 13.1.
where quantities
The refrigerator removes heat QL from the low-
temperature region. While a heat pump supplies QL = quantity of heat removed from a
heat QH to a high-temperature region. low-temperature region,
The performance of a refrigerator and a heat QH = quantity of heat supplied to a
pump is expressed in terms of coefficient of perfor- high-temperature region,
Win = work input to cycle.
Refrigeration 449

From Eqs. (13.1) and (13.2), it is revealed that Ti = 23°C = 296 K


(COP)HP = (COP)R + 1 (COP)R = 3.5
htransmission = 0.85
Example 13.1 A household refrigerator with a COP Cpw = 4.1868 kJ/kg ◊ K
of 1.8 removes heat from a refrigerated space at a rate of hfg = 334.5 kJ/kg
90 kJ/min. Determine
(a) Electrical power consumed; and To find
(b) Heat rejected to surroundings (i) Power required by compressor motor,
(ii) Amount of heat transferred by the system per
Solution minute.

Given A household refrigerator Analysis The amount of heat removed by the system
(COP)R = 1.8 RE = 90 kJ/min per day
QL = Heat removed from water + Heat absorbed
To find
during phase change of water to ice
(i) Electrical power consumption, and = mCpw (Ti – 0°C) + mhfg
(ii) Heat rejected to surroundings. = 10 ¥ 103 ¥ 4.1868 ¥ (23 – 0) + 10 ¥ 103 ¥ 334.5
Analysis = 430.8 ¥ 104 kJ per day
(i) The coefficient of performance of a refrigerator is Rate of heat removal per minute
given as 430.8 ¥ 10 4 ( kJ/day )
RE = = 2991.67 kJ/min
(COP)R =
Refrigerating Effect RE
= ( 24 h ¥ 60 min/ h )
Work input Win The COP of a refrigerator is given as
Using the given values RE
(COP)R =
90 kJ/min Win
1.8 =
Win RE 2991.67
or Win = = = 854.76 kJ/min
or Win = 50 kJ/min = 0.833 kW (COP ) R 3.5
The electrical power consumption is 0.833 kW. = 14.25 kW
(ii) Heat rejected to surroundings Power required by compressor motor
QH = RE + Win
Win 14.25
= 90 kJ/min + 50 kJ/min = 140 kJ/min. P = = = 16.76 kW
htransmission 0.85
Example 13.2 An ice plant produces 10 ¥ 103 kg of
ice per day at 0°C using water at a temperature of 23°C.
Estimate the power required by the compressor motor, REFRIGERATION TERMINOLOGY
if the COP of the plants is 3.5 and the transmission
efficiency is 85%. Also find the amount of heat transferred 1. Refrigeration Load It is the amount of heat
from the system per minute. which must be removed per unit time from the
Take Cp (water) = 4.1868 kJ/kg ◊ K, and cold region. It is also known as the refrigeration
hfg(ice) = 334.5 kJ/kg capacity. It is measured in Tonnes of Refrigeration
(TR) and is designated as RE. Sometimes, it is also
Solution referred as refrigeraton effect.
Given Production of ice at 0°C
The refrigeration
mice = 10 ¥ 103 kg per day
capacity of a system decides the mass flow rate of
TL = 0°C = 273 K
a given refrigerant, when working under specified
conditions, i.e.,
450 Thermal Engineering

Mass flow rate of refrigerant The capillary tube is a long and


Refrigeration capacity narrow tube connecting the condenser directly to
m = the evaporator. The refrigerant passes through this
Refrigerating effect per kg of refriggerant
tube by capillary action (drop by drop), thus the
...(13.3)
fluid friction and flashing of liquid refrigerant into
3. It is the refrigeration vapour due to heat transfer, cause the pressure drop
effect produced at the evaporator of a refrigeration in the tube. For a given state of the refrigerant, the
system, thus equivalent to refrigeration capacity. pressure drop is directly proportional to length of
The evaporator is the device that consists of the the tube and inversely proportional to diameter of
long thin tubing where the refrigerant evaporates the tube.
by absorbing its heat of evaporation from the sur-
rounding medium. 13.4 TYPES OF REFRIGERATION
SYSTEMS
4. It is the term used in a household refrig-
erator. It is a place very near to evaporator coils, so The commonly used refrigeration systems are
the liquid can easily be freezed to solid state. It is listed below:
also called chiller.
(i) Gas refrigeration system
5. It is the deposition of ice on the evapo- (ii) Vapour compression refrigeration system
rating coil due to its very low temperature. When (iii) Vapour absorption refrigeration system
air passes over the extremely cold coils, the mois- (iv) Steam refrigeration system
ture in the air separates and deposits on coils and
gets solidified to form frosting. This frosting (ice) is GAS REFRIGERATION SYSTEMS
a very bad conductor of heat, and reduces the heat
transfer from cold region to refrigerant. Thus the In a gas refrigeration cycle, the gas such as air is
refrigeration load increases. Therefore, it is recom- used as a refrigerant. It transfer only its sensible
mended that this frost (ice) must be removed from heat and does not undergo a change of phase. The
the evaporator coils regularly. The removal of ice gas refrigeration systems have a number of appli-
from the evaporator coil is called the defrostation. cations. They are used to achieve very low tem-
perature for liquefaction of gases and for aircraft
cooling. The gas power cycles used earlier can be
used as gas refrigeration by simply reversing the
direction of process involved in these cycles. The
reversed Carnot cycle, reversed Brayton cycle and
reversed Stirling cycle are some gas refrigeration
cycles.
The main advantages of air refrigeration is its
free availability, light weight and eco-friendly.

As studied earlier in Section 6.22, if the direction of


Carnot engine is reversed, then the cycle works as a
refrigeration cycle. The reversed Carnot cycle will
theoretically have a maximum possible coefficient
Fig. 13.2
of performance, but it is not possible to construct
Refrigeration 451

a refrigerating machine which will work on the Net work input to cycle,
reversed Carnot cycle. Win = QH – QL
A reversed Carnot cycle using air as working = TH (S3 – S2) – TL(S3 – S2)
medium is shown on p–V and T–S diagrams in = (TH – TL ) (S3 – S2) ...(13.6)
Fig. 13.3(a) and (b), respectively. The coefficient of performance of the reversed
The cycle consists of four reversible processes Carnot cycle operation as refrigerator
in sequence. RE TL ( S3 - S2 )
Process 1–2 Isentropic expansion of air from (COP)R, rev = =
Win (TH - TL ) ( S3 - S2)
higher temperature TH to lower temperature TL
TL
Process 2–3 Heat removal from cold space in iso- = ...(13.7)
TH - TL
thermal manner at temperature TL
The coefficient performance of reversed Carnot
Process 3–4 Isentropic compression of air from cycle operating as heat pump
low temperature TL to high temperature TH
Heat rejected at TH QH
Process 4–1 Heat rejection isothermally to a (COP)HP, rev = =
Work input Win
medium at temperature TH
The refrigeration effect = Heat absorbed by air TH ( S3 - S2 ) TH
= =
during isothermal process 2–3 at temperature TL (TH - TL ) ( S3 - S2 ) TH - TL
RE = TL (S3 – S2) ...(13.4) ...(13.8)
Heat rejected at temperature TH
QH = TH (S3 – S2) ...(13.5)

p Although the reversed Carnot cycle gives maxi-


1
QH mum COP between two fixed temperatures and is
useful as a criterion of perfection, it has inherent
drawbacks, when a gas is used as a refrigerant.
4
1. It is almost impractical to transfer heat at
2 constant temperature during processes 2–3
QL and 4–1. Such heat transfer is only possible
3
with a very slow process.
0 V 2. Secondly, with the irreversibilities of the
(a) p–V diagram
compressor and expander, the isentro-
T pic compression and expansion cannot be
1 QH 4 achieved
TH
3. Further, isothermal processes are extremely
slow processes while isentropic processes
TL are extremely fast processes. Therefore,
2 QL 3
combination of these two processes within a
cycle is impossible.

Example 13.3 A refrigerator has a working tempera-


0 S
(b) T–S diagram ture in the evaporation and condenser coils of – 30°C
and 30°C, respectively. What is the maximum possible
Fig. 13.3 COP of the refrigerator?
452 Thermal Engineering

Solution Amount of heat removed from water–ice system per kg.


QL = Heat removed in cooling of water from 25°C to
Given A refrigerator operating between
0°C + Heat removed during formation of ice at
TL = – 30°C = 243 K
0°C + Heat removed from ice during its cooling
TH = 30°C = 303 K
from 0°C to –5°C.
To find Maximum possible COP of a refrigerator. = Cpw (TH – 0°C) + hfg + Cp,ice (0 – Tice)
Analysis The reversed Carnot cycle can give maximum = 4.18 ¥ (25 – 0) + 355 + 2.0 ¥ [0 – (–5)]
(COP)R, thus = 449.5 kJ/kg
TL 243 The COP of a Carnot refrigerator
(COP)R, rev = = = 4.05 TL
TH - TL 303 - 243 (COP)R, rev =
TH - TL
Example 13.4 A reversed Carnot cycle is used for 263
making ice at –5°C from water at 25°C. The tempera- = = 7.514
298 - 263
ture of the brine is –10°C. Calculate the quantity of ice Further, the COP can also be expressed as
formed per kWh of work input. Assume the specific heat
RE
of ice as 2 kJ/kg ◊ K, latent heat of ice as 335 kJ/kg and COP =
specific heat of water as 4.18 kJ/kg ◊ K. Win
RE
or 7.514 =
Solution 3600 kJ
Given Formation of ice at –5°C from water at 25°C or RE = 27051.43 kJ
TL = –10°C = 263 K Thus, the mass of ice formed with this refrigerating
TH = 25°C = 298 K effect
RE 27051.43
Tice = –5°C mice = = = 60.18 kg/kWh
QL 449.5
Win = 1 kWh = 3600 kJ
Cp, ice = 2 kJ/kg ◊ K
Cpw = 4.18 kJ/kg ◊ K 13.6 BRAYTON REFRIGERATION
hfg = 335 kJ/kg
To find The mass of ice formed.
The Bell Coleman refrigeration cycle is the reverse
Assumption The ambient temperature to be 25°C of the closed Brayton power cycle. The schematic
Analysis The schematic is shown in Fig. 13.4. and T–s diagrams of the reverse Brayton cycle are
shown in Fig. 13.5.
The refrigerant gas (may be air) enters the com-
pressor at the state 1 and is compressed to the state
2. The gas is then cooled at constant pressure in a
heat exchanger to the state 3. During cooling, the
gas rejects heat to the surroundings and approach-
es the temperature of the warm environment. The
gas is then expanded in an expander to the state 4,
where it attains a temperature, that is well below the
temperature of the cold region. The refrigeration ef-
fect is achieved through the heat transfer from the
cold region to gas as it passes from state 4 to 1 in a
heat exchanger and the cycle completes.
Fig. 13.4
Refrigeration 453

wc = h2 – h1 and wT = h3 – h4
The net work input to the cycle is
wnet = wc – wT
= (h2 – h1) – (h3 – h4) ...(13.9)
The refrigeration effect produced in the cycle
RE = Heat transfer from the cold region to
the refrigerant gas
= h 1 – h4 ...(13.10)
The coefficient of performance of the cycle is
the ratio of the refrigeration effect to the net work
input
RE h1 - h4
(COP)R = = ...(13.11)
wnet ( h2 - h1 ) - ( h3 - h4 )
The gas refrigeration cycle deviates from the re-
versed Carnot cycle because heat transfer processes
are not isothermal. In fact, the gas temperature var-
ies considerably during the heat-transfer process.
Figure 13.6 shows a T–s diagram which compares
the reversed Carnot cycle 1–2¢–3–4¢–1 and reversed
Brayton cycle 1–2–3–4–1. It reveals the following
facts:
1. The net work (area 1–2¢–3–4¢–1) required by
the reverse Carnot cycle is a fraction of that
required by the reverse Brayton cycle.
2. The reverse Carnot cycle produces greater
refrigeration effect (area under line 4¢–1) as
compare to reverse Brayton cycle (area un-
der curve 4 –1).
3. The mean temperature of heat rejection is
much greater and that of heat rejection is
Fig. 13.5 much lower in reverse Brayton.

The T–s diagram for an ideal Brayton cycle is


represented by the cycle 1–2s–3–4s–1, in which all
processes are internally reversible, and compression
and expansion are isentropic. The cycle 1–2–3–
4–1 includes the effect of irreversibilities during
adiabatic compression and expansion. The frictional
pressure drops have been ignored.
At steady state, the compression
and expansion work per kg of gas flow in the
Fig. 13.6
system are,
454 Thermal Engineering

Therefore, the COP of the reverse Brayton cycle Pressure ratio, p2/p1 : 3
is much lower than that of the vapour compression Turbine inlet, T3 : 300 K
cycle or the reversed Carnot cycle. To find
Advantages (i) Net power input in the cycle,
1. Air is a freely and easily available fluid. (ii) Refrigeration capacity, and
(iii) The COP of the cycle.
2. There is no danger of fire and toxic effects, if
it leaks. Schematic with given data
3. The weight of air refrigeration system per
tonne of refrigeration is less compared with
other refrigeration systems.
4. An air cycle can work as an open or closed
cycle system.
5. It is eco-friendly.
Disadvantages
1. The main drawback of an air refrigeration
system is its very low value of COP.
2. Air has only sensible heat and cannot trans-
fer heat at constant temperature across heat
exchangers. Thus, as the temperature differ-
ence decreases, its heat transfer capacity de-
creases.
3. The quantity of air required per tonne of re-
frigeration capacity is much larger than the
liquid refrigerants.
4. A turbine is required for the expansion of
air instead of a throtting device. Since air
contains some water vapour, thus danger of
frosting during expansion is more possible.
5. It has more running cost than other systems.

Example 13.5 Air enters the compressor of an ideal Fig. 13.7


Brayton refrigeration cycle at 1 atm and 270 K with a
volumetric flow rate of 1.5 m3/s. If the compressor pres- Assumptions
sure ratio is 3 and the turbine inlet temperature is 300 K, (i) Each component of the cycle is analyzed as a
determine control volume at steady state.
(a) the net power input, (ii) The turbine and compressor processes are
(b) the refrigeration capacity, and isentropic.
(c) coefficient of performance. (iii) There are no pressure drop through the heat
exchangers.
Solution (iv) Kinetic and potential energy effects are negligible.
Given An ideal Brayton refrigeration cycle operates (v) The working fluid is modelled as an ideal gas.
with air. (vi) Cold air standard assumptions with Cp = 1.005
Compressor inlet, T1 : 1 atm, 270 K, 1.5 m3/s kJ/kg ◊ K and g = 1.4.
Refrigeration 455

Analysis The specific enthalpy at each state of the absorbing heat at constant pressure, the air re-enters the
cycle. compressor, which is driven by the turbine. Find the COP
State 1: T1 = 270 K, p1 = 1 atm of the refrigerator, driving power required and air mass
h1 = Cp T1=1.005 ¥ 270 = 271.35 kJ/kg flow rate.
State 2: p2 = 3p1 = 3 atm
g -1 Solution
Ê 1.4 -1
p2 ˆ g
T2 = T1 Á ˜ = 270 ¥ (3) 1.4 = 370 K Given An aircraft cooling system
Ë p1 ¯ Compressor inlet, p1 = 100 kPa, T1 = 283 K
h2 = CpT2 = 1.005 ¥ 370 = 371.85 kJ/kg hC = 0.72
State 3: T3 = 300 K, p3 = 3 atm, Pressure ratio, rp : 2.5
h3 = Cp T3 = 1.005 ¥ 300 = 301.5 kJ/kg hT = 0.75
State 4: p4 = p1 = 1 atm Turbine inlet, T3 = 320 K
T3 300 RE = 3 TR = 10.53 kW
T4 = g -1
= 1.4 -1
= 219.35 K,
Ê p2 ˆ g Ê 3ˆ 1.4 To find
ÁË 1 ˜¯
ËÁ p1 ˜¯ (i) COP of refrigerator,
(ii) Mass flow rate of air, and
h4 = Cp T4 = 1.005 ¥ 219.35 = 220.44 kJ/kg
(iii) Power input.
The mass flow rate of air
V Schematic with given data
m =
v1
RT1 (0.287 kJ/kg ◊ K ) ¥ ( 270 K )
where v1 = =
p1 (101.325 kPa )
= 0.765 m3/kg
Given V = 1.5 m3/s
V 1.5
Then m = = = 1.96 kg/s
v 0.765
(i) Net power input in the cycle
Wnet , in = m [(h2 – h1) – (h3 – h4)]
= 1.96¥ [(371.85 – 271.35)
– (301.5 – 220.44)]
= 38.13 kW
(ii) The refrigeration capacity
RE = m(h1 – h4)
= 1.96 ¥ [271.35 – 220.44] = 99.78 kW
(iii) The coefficient of performance
RE 99.78
(COP)R, Brayton = = = 2.62
Wnet , in 38.13

Example 13.6 Air enters the compressor of an air-


craft cooling system at 100 kPa, and 283 K. Air is now
compressed to 2.5 bar with an isentropic efficiency of
72%. After being cooled to 320 K at constant pressure
in a heat exchanger, the air then expands in a turbine
to 1 bar with an isentropic efficiency of 75%. The cool- Fig. 13.8
ing load of the system is 3 tonnes of refrigeration. After
456 Thermal Engineering

Assumptions 1. COP of refrigerator


(i) Each component of the cycle is analyzed as a Heat absorbed per kg of air
(COP)R =
control volume at steady state. Net work input per kg of air
(ii) There are no pressure drops through the heat q 18.37
exchangers. = in = = 0.293
wnet 62.65
(iii) Kinetic and potential energy effects are negligible.
2. Mass flow rate of air in the system
(iv) The working fluid is modelled as an ideal gas.
Refrigerating effect
(v) Cold air standard assumptions with Cp = 1.005 ma =
Heat removed per kg of air
kJ/kg ◊ K and g = 1.4.
RE 10.53 kJ/s
Analysis Analysing each component separately; = = = 0.573 kg/s
qout 18.37 kJ/kg
Compressor The temperature after isentropic com- 3. Power input to refrigerator
pression Winput = ma wnet
g -1 1.4 -1
= (0.573 kg/s) ¥ (62.65 kJ/kg)
( )g
T2s = T1 rp = 283 ¥ ( 2.5) 1.4 = 367.7 K
= 35.91 kW
The isentropic efficiency of the compressor is given
by
Isentropic work T2 s - T1
hC = =
Actual work T2 - T1
367.7 - 283
or T2 = 283 + = 400.62 K
0.72 A schematic of an ideal vapour compression
refrigeration cycle and its T–s diagram are shown
The actual work input to compressor win
in Fig. 13.9.
win = h2 – h1 = Cp (T2 – T1)
= 1.005 ¥ (400.62 – 283) = 118.21 kJ/kg The vapour compression refrigeration cycle
consists of four processes discussed below:
Turbine The temperature T4s after isentropic expan-
Process 1–2 Isentropic compression of saturated
sion;
vapour in the compressor,
T3 320
T4s = g -1
= 1.4 -1
= 246.3 K Process 2–3 Constant pressure heat rejection in
( )
rp g (2.5) 1.4 the condenser,
The isentropic efficiency of the turbine is given as Process 3–4 Throttling of refrigerant in an expan-
sion device, and
Actual work output T -T
hT = = 3 4 Process 4–1 Constant pressure heat absorption in
Isentropic work output T3 - T4 s
evaporator.
Actual temperature after expansion in turbine,
T4 = 320 – 0.75 ¥ (320 – 246.3) = 264.47 K
In an ideal vapour compression cycle, the refrig-
Turbine work output per kg of air
erant enters the compressor at the state 1, as dry
wout = h3 – h4 = Cp (T3 – T4)
and saturated vapour, where it is compressed to a
= 1.005 ¥ (320 – 264.72) = 55.556 kJ/kg
relatively high pressure and temperature, the state
Net work input per kg to the plant
2. The refrigerant in the superheated state 2, enters
wnet = win – wout
the condenser and leaves as saturated liquid at the
= 118.21 – 55.556 = 62.65 kJ/kg of air
state 3, as a result of heat rejection to the surround-
Heat absorbed per kg of air
ings. The saturate liquid refrigerant at the state 3
qin = h1 – h4 = Cp (T1 – T4)
is throttled to the evaporator pressure by passing
= 1.005 ¥ (283 – 264.72) = 18.37 kJ/kg
through an expansion valve or a capillary tube. The
temperature of refrigerant at the state 4 drops below
the temperature of the refrigerated space.
Refrigeration 457

Warm
t
surroundings, nstan
v = co
TH T = constant

3 2 High
Condenser pressure

Pressure (bar)
High
pressure superheated

nt
vapours

ta
liquid

ns
co
Win
Compressor

=
Expansion

s
id
liqu

our
valve

vap
Low

ted
Low pres. T = constant
1 pressure

a
low temp. 4

ted
tur
refrigerant Evaporator vapours

Sa

tura
C
1 =

Sa
x

2
x

3
x
COLD refrigerator
space, TL Enthalpy (kJ/kg)

Fig. 13.10
(a) Basic components of a refrigerator

T
required for the calculation can directly be read off.
C
p=

The essential features of the diagram are given in


2 Fig. 13.10 and a typical refrigeration cycle is shown
qH in Fig. 13.11. The points 1, 2, 3 and 4 represent the
3 same state of refrigerant as on T–s diagram in Fig.
Win 13.9(b).
qL

4 1
Refrigeration
effect = h1 – h4
s
(b) T–s diagram for ideal vapour compression cycle.
Fig. 13.9

At the state 4, the refrigerant as wet mixture,


passes through an evaporator at constant pressure.
The refrigerant is now completely evaporated by
absorbing its latent heat from cold temperature (re-
frigerated) space. The heat absorbed during evapo-
ration of refrigerant is the latent heat and its magni-
tude is higher at low pressures.
The only throttling process 3– 4 is an irreversible Fig. 13.11
process and hence it is shown by dotted lines. The
remaining three processes are considered reversible. 13.7.2 Analysis of Vapour Compression
Therefore, the vapour compression cycle is not a Cycle
reversible cycle. All four components in a vapour compression cycle
are steady flow devices. Thus processes they per-
13.7.1 Vapour Compression Cycle on form are steady flow processes. In absence of any
Pressure–Enthalpy Diagram kinetic and potential energy changes, the steady
The pressure–enthalpy diagram is more convenient flow energy equation on unit mass basis reduces to
to represent refrigeration cycles, since the enthalpy q – w = Dh
458 Thermal Engineering

1. Evaporator (w = 0) mR = 5 kg/min.
qL = h1 – h4 ...(13.12) T1 = – 10°C = 263 K
2. Compressor (q = 0) T3 = 25°C
win = h2 – h1 ...(13.13) and tabulated properties of refrigerant.
3. Condenser (w = 0) To find
qH = h2 – h3 ...(13.14) (i) (COP)R ,
4. Expansion valve (q = 0, w = 0) (ii) Capacity of refrigerator.
h4 = h3 ...(13.15)
Analysis Properties of refrigerant at end states:
The coefficient of performance of refrigerator h2 = 1465.84 kJ/kg
and heat pump can be expressed as h3 = 298.9 kJ/kg
qL h -h h4 = 298.9 kJ/kg
(COP)R = = 1 4 ...(13.16) Using the Clapeyron equation for calculating entropy
win h2 - h1
change sfg2 during evaporation
q h -h
(COP)HP = H = 2 3 ...(13.17) h fg2 hg 2 - h f 2
win h2 - h1 sfg2 = =
Tsat2 Tsat2
where h1 = hg @ p1
h3 = hf @ p2 1465.84 - 298.9
= = 3.9159
298
Example 13.7 A vapour compression refrigerator Entropy at the state 2
cycle works between temperature limits of 25°C and – s2 = sf2 + sfg2 = 1.1242 + 3.9159
10°C. The vapour at the end of isentropic compression is = 5.0401 kJ/kg ◊ K
just dry. Assuming there is no subcooling, find the COP of h fg1 hg - h f 1
the system. Also find the capacity of refrigerator,if mass Similarly, sfg1 = = 1
Tsat1 Tsat1
flow rate of refrigerant is 5 kg/min.The properties of re-
frigerant are tabulated below. 1433.05 - 135.37
= = 4.9341
263
Temperature Enthalpy, kJ/kg Entropy of liquid, Entropy at the state 1
T, K hf hfg kJ/kg ◊ K s1 = sf1 + x1 sfg1 = 0.5443 + 4.9341x1
298 K 298.9 1465.84 1.1242 For isentropic compression,
263 135.37 1433.05 0.5443 s1 = s2 = 5.0401 kJ/kg ◊ K
or 5.0401 = 0.5443 + 4.9341x1
Solution or x1 = 0.911
Enthalpy at the state 1
Given A vapour compression cycle as shown on p–h
h1 = hf1 + x1 hfg1
diagram in Fig. 13.12.
= 135.37 + 0.911 ¥ (1433.05 – 135.37)
= 1317.55 kJ/kg
(i) COP of refrigerator
The compression work
win = h2 – h1 = 1465.84 – 1317.55
= 148.28 kJ/kg
Refrigeration effect
qin = h1 – h4
= 1317.55 – 298.9 = 1018.65 kJ/kg

Fig. 13.12
Refrigeration 459

qin 1018.65 For the process 3 – 4


Therefore, (COP)R = = = 6.87
win 148.28 h4 = h3 = 74.53 kJ/kg
Heat absorbed per kg of refrigerant
(ii) Capacity of refrigerator
qin = h1 – h4
RE = mR ¥ qin
= 178.61 – 74.53 = 104.08 kJ/kg
= 5 ¥ 1018.65
qin 104.08
= 5093.25 kJ/min or 84.88 kW Therefore, (COP)R = = = 3.176
win 32.77
Example 13.8 A refrigerator used R-12 as a work- Refrigerating Capacity,
ing fluid and it operates on an ideal vapour compression RE = mR ¥ qin = (0.03 kg/s) ¥ (104.08 kJ/kg)
cycle. The temperature of refrigerant in the evaporator = 3.12 kJ/s = 3.12 kW
is –20°C and in the condenser is 40°C. The refrigerant 3.12
= = 0.89 TR
is circulated at the rate of 0.03 kg/s. Determine the coef- 3.5
ficient of performance and capacity of refrigeration plant An ideal vapour compression system
in the TR. uses R-12 as the refrigerant. The system uses an evapo-
ration temperature of 0°C and a condenser temperature
Solution of 40°C. The capacity of the system is 7 TR. Determine
Given A vapour compression cycle as shown on T–s (a) The mass flow rate of refrigerant,
diagram (b) Power required to run the compressor,
T1 = – 20°C = 253 K (c) Heat rejected in the condenser,
T3 = 40°C (d) COP of the system.
mR = 0.03 kg/s Use the properties of R-12 from the table given below:
To find Temp. Pressure hf hg sf sg
(i) (COP)R, and °C bar kJ/kg kJ/kg kJ/kg kJ/kg◊K
(ii) Capacity in TR.
0 3.087 36.05 187.53 0.142 0.696
Analysis From R-12 table, we have saturation states 40 9.609 74.59 203.2 0.727 0.682
At –20°C h1 = 178.61 kJ/kg,
Take Cp for superheated vapour as 0.6 kJ/kg ◊ K.
s1 = 0.7082 kJ/kg
At 40°C h3 = hf 3 = 74.53 kJ/kg Solution
For process 1 – 2;
s1 = s2 = 0.7082 kJ/kg ◊ K Given An ideal vapour compression system as shown
in Fig. 13.14.
It gives h2 = 211.38 kJ/kg
h1 = 187.53 kJ/kg.
The compression work
s1 = 0.696 kJ/kg ◊ K
win = h2 – h1 = 211.38 – 178.61
h3 = h4 = 74.59 kJ/kg ◊ K
= 32.77 kJ/kg
sg2 = 0.682 kJ/kg ◊ K

Fig. 13.13 Fig. 13.14


460 Thermal Engineering

To find (iv) COP of the system


(i) Mass flow rate of refrigerant, Refrigerating effect h1 - h4
(COP)R = =
(ii) Power input to compressor, Work input h2 - h1
(iii) Heat rejected in the condenser, 187.53 - 74.59
= = 5.62
(iv) COP of the system. 207.63 - 187.53
Assumptions
Example 13.10 A refrigerator operates between tem-
(i) Refrigerant leaving the evaporator as saturated
perature limits of 30°C and –5°C. The refrigerant is 0.97
vapour.
dry before leaving the evaporator coil. Find the condition
(ii) Compression is isentropic. of refrigerant entering the evaporator and COP of sys-
(iii) No pressure drop in condenser and evaporator. tem. If the temperature rise of water circulating through
Analysis The state 2 can be obtained by equating the condenser is limited to 20°C, calculate mass flow rate
entropy at two states during compression. of the coolant.
Use the properties of refrigerant from table given
Ê Tsup ˆ below:
s1 = s2 = sg + C ps ln Á
Ë Tsat ˜¯
Temp. Enthalpy, Entropym Specific heat,
Ê Tsup ˆ kJ/kg kJ/kg ◊ K kJ/kg ◊ K
0.696 = 0.682 + 0.6 ¥ ln Á
Ë 40 + 273 ˜¯ °C hf hg sf sg Cp,L Cp,g

Ê Tsup ˆ 30 323.22 1465.38 1.2037 4.9839 5.024 3.35


or ln Á = 0.0233
Ë 313 ˜¯
–5 158.26 1431.89 0.63 5.4072 – –

or Tsup = 320.4 K Take Cp for superheated vapour as 3.35 kJ/kg ◊ K.


The enthalpy after isentropic compression
h2 = hg + Cps (Tsup – Tsat) Solution An ideal vapour compression system as
= 203.2 + 0.6 ¥ (320.4 – 313) shown in Fig. 13.15.
= 207.63 kJ/kg T1 = –5°C = 268 K
(i) Mass flow rate of refrigerant T2 = 30°C = 303 K
RE = 7 TR x1 = 0.97
= 7 ¥ 210 kJ/min = 1470 kJ/min (DT)w = 20°C,
The refrigerating effect per kg of refrigerant. Cps = 3.5 kJ/kg ◊ K
qin = h1 – h4 To find
= 187.53 – 74.59 = 112.94 kJ/kg (i) Dryness fraction of refrigerant, x4
The mass flow rate of refrigerant (ii) COP of the system,
RE 1470 (iii) Coolant rate in the condenser.
mR = = = 13.01 kg/min
qin 112.94
(ii) Power input to compressor
P = mR ( h2 - h1 )
Ê 13.01 ˆ
=Á kg/s˜ ¥ ( 207.63 - 187.53) ( kJ/kg)
Ë 60 ¯
= 4.35 kW
(iii) Heat rejected in the condenser
= mR ( h2 - h3 )
= 13.01 ¥ (207.63 – 74.59)
Fig. 13.15
= 1730.85 kJ/min
Refrigeration 461

Assumptions qin 1070.46


(COP)R = = = 6.685
(i) Refrigerant leaving the evaporator as saturated win 160.18
vapour.
(iii) Mass flow rate of water in condenser
(ii) Compression is isentropic.
Heat rejected per kg of refrigerant
(iii) No pressure drop in condenser and evaporator.
qout = h2 – h3 = 1553.86 – 323.22
(iv) Specific heat of water as Cpw = 4.187 kJ/kg ◊ K.
= 1230.64 kJ/kg
Analysis The enthalpy at the state 1 Water flow rate per kg of refrigerant
h1 = h f1 + x1( h fg1 - h f1 ) qout = mw Cpw (DT)w
= 158.26 + 0.97 ¥ (1431.89 – 158.26) 1230.64 = mw ¥ 4.187 ¥ (20°C)
= 1393.68 kJ/kg or mw = 14.7 kg/kg of refrigerant
The entropy at the state 1
s1 = s f1 + x1( s fg 1 - sf 1 ) Example 13.11 A simple R-12 plant is to develop
4 tonnes of refrigeration. The condenser and evapora-
= 0.63 + 0.97 ¥ (5.4072 – 0.63)
tor temperatures are 35°C and – 15°C, respectively.
= 5.2639 kJ/kg ◊ K
Determine
The state 2 can be obtained by equating entropy at the
(a) The mass flow rate of refrigerant in kg/s,
two states during compression.
(b) Volume flow rate handled by compressor in m3/s,
Ê Tsup ˆ
s1 = s2 = sg + C ps ln Á (c) The compressor discharge temperature,
Ë T ˜¯
sat (d) The pressure ratio,
Ê T2 ˆ (e) Heat rejected to condenser in kW,
5.2639 = 4.9839 + 3.35 ¥ ln Á
Ë 30 + 273 ˜¯ (f) Flash gas percentage after throttling,
Ê T ˆ (g) The COP, and
or ln Á 2 ˜ = 0.0836 (e) Power required to drive the compressor.
Ë 303 ¯
Compare this COP with COP of Carnot refrigerator
or T2 = 329.41 K
operating between temperatures of 35°C and –15°C.
The enthalpy after isentropic compression
h2 = hg + Cps (Tsup – Tsat) Solution
= 1465.38 + 3.35 ¥ (329.41 – 303)
Given A vapour compression cycle as shown on the
= 1553.86 kJ/kg
p–h diagram in Fig. 13.16.
Enthalpy at the end of condensation at the state 3;
T1 = –15°C = 258 K
h3 = hf@ 30°C = 323.22 kJ/kg ◊ K T3 = 35°C = 308 K
Enthalpy after throttling, at the state 4; RE = 4 TR = 4 ¥ 3.51 = 14.04 kW
h4 = h3 = 323.22 kJ/kg ◊ K
To find
(i) State of refrigerant after throttling
Enthalpy at state 4 can be expressed as; (i) The mass flow rate of refrigerant in kg/s,
h4 = h f 4 + x4 ( h fg4 - h f 4 ) (ii) Volume flow rate handled by compressor in m3/s,
323.22 = 158.26 + x4 ¥ (1431.89 – 158.26) (iii) The compressor discharge temperaturem T2,
or x4 = 0.13 (iv) The pressure ratio p2/p1,
(ii) COP of refrigerator (v) Heat rejected to condenser in kW,
The refrigerating effect per kg of refrigerant. (vi) Flash gas percentage after throttling,
qin = h1 – h4 (vii) The COP,
= 1393.68 – 323.22 = 1070.46 kJ/kg (viii) Power required to drive the compressor,
Work input per kg of refrigerant (ix) COP of Carnot refrigerator, and its comparison
with COP of VCC.
win = h2 – h1
= 1553.86 – 1393.68 = 160.18 kJ
462 Thermal Engineering

Heat removal rate


Qout = mR qout
= 0.126 ¥ 139.5
= 17.5 kW
(vi) Flash gas percentage after throttling
h3 = hf1 + x (hg1 – hf1)
233.5 = 186.28 + x ( 345 – 186.28)
233.5 - 186.28
or x = = 0.297
345 - 186.28
(vii) COP
Fig. 13.16 The compression work
win = h2 – h1 = 373 – 345 = 28 kJ/kg
Analysis From Table B-4, and p–h diagram, the prop-
q 111.5 kJ/kg
erties at various states. (COP)R = in = = 3.982
win 28 kJ/kg
At –15°C h1 = hg1 = 345 kJ/kg,
(viii) Power input to refrigerator
s1 = 1.563 kJ/kg
vg1 = 0.091 m3/kg Winput = mR win
p1 = 1.8 bar = (0.126 kg/s) ¥ (28 kJ/kg) = 3.528 kW
hf1 = 186.28 kJ/kg (ix) COP of Carnot refrigerator
T1 258 K
We have s1 = s2 = 1.563 kJ/kg, from p–h chart, we get (COP)R,Carnot = =
h2 = 373 kJ/kg T2 - T1 (308 - 258) ( K )
T2 = 46°C = 5.16
(COP ) R 3.982
p2 = 9 bar Relative COP = = = 0.771
(COP ) R, Carnot 5.16
At 35°C h3 = hf 3 = 233.5 kJ/kg
For process 3 – 4
h4 = h3 = 233.5 kJ/kg
(i) The mass flow rate of refrigerant in kg/s
Heat removed per kg of refrigerant The following analysis will show the effect of
qin = h1 – h4 = 345 – 233.5 = 111.5 kJ/kg change in operating conditions on the performance
The mass flow rate of refrigerant of the vapour compression cycle.
Refrigerating effect
mR = (a)
Heat removed per kg of air
The evaporator pressure is reduced to p1¢ from the
RE 14.04 kJ/s
= = = 0.126 kg/s existing pressure p1. The effect on the cycle is shown
qin 111.5 kJ/kg
on T–s and p–h diagrams of Fig. 13.17. By reducing
(ii) The volume flow rate of refrigerant the evaporator pressure, the evaporator temperature
VR = mR vg1 = 0.126 ¥ 0.091 decreases (favourable) but the refrigeration effect
= 0.01145 m3/s (h1¢ – h4¢) (area under the curve on T–s diagram)
(iii) The compressor discharge temperature T2 = 46°C decreases. The work input to the compressor (h2¢
(iv) The pressure ratio p2/p1 – h1¢) increases. The specific volume of vapour at
p2 9 bar low pressure is large, thus the volumetric efficiency
= =5
p1 a1.8 b r will also decrease. Therefore, it is not desirable to
(v) Heat rejected to condenser decrease the evaporator pressure.
Heat rejected per kg of refrigerant
(b)
qout = h2 – h3 = 373 – 233.5 = 139.5 kJ/kg
It is evident from T–s and p–h diagrams of
Refrigeration 463

(ii) Refrigerant effect decreases from 4–1 to 4¢–1.


(iii) The condenser temperature increases, there-
by increasing heat rejection.
(iv) The COP of the system decreases.
(c)
The effect of superheated vapour at the compression
suction is shown with the help of T–s and p–h
diagrams of Fig. 13.19.

Fig. 13.17

Fig. 13.18, that by increasing the condenser


pressure
(i) The work input to the compressor increases
from 1–2 to 1–2¢.

Fig. 13.19

refrigerant
vapour at suction from v1 to v1¢, thus the
compression work input increases as shown
by hatched area.
h1 –
h4) to (h1¢ – h4).
COP of the new system is given by
h¢ - h
COP¢ = 1 4
h2¢ - h1¢
( h1 - h4 ) + (h1¢ - h1 )
=
( h2 - h1) + [(h2¢ - h1¢ ) - ( h2 - h1)]
...(13.18)
Both the numerator and denominator in-
crease, thus the COP of the new system may
Fig. 13.18
increase, decrease or remain the same.
464 Thermal Engineering

It is the practical necessity to allow the refrig- dercooling of the condensed liquid and then
erant vapour to become slightly superheated at the throttling of refrigerant to evaporator pressure.
suction of the compressor in order to avoid any (b) The dry saturated vapour delivered to the com-
carry over of liquid refrigerant into the compressor. pressor, where it is compressed isentropically to
The amount of superheat should be kept minimum the condenser pressure.
in order to keep the compression work minimum. (c) The dry saturated vapour delivered to the com-
pressor and liquid after condensation is under-
(d) cooled by 10°C.
The condensed liquid can be cooled to a temperature
below the saturation temperature corresponding to Solution
the condenser pressure. This effect is shown with Given A refrigerator operating between a condenser
the help of T–s and p–h diagrams of Fig. 13.20 in pressure of 12.37 bar and evaporator pressure of 1.902
which the constant pressure line is shown left to bar.
the saturated liquid line. The effect of subcooling To find
(also called undercooling) is to move the line 3-4 (i) Refrigerating effect/ kg of refrigerant.
(throttling process) to the left of the diagrams. It (ii) COP of the system.
increases the refrigeration effect without increase
Assumptions
in compression work. Thus the COP of the system
(i) Each component in the all cycles is analysed as a
increases. Therefore, the subcooling of condensed
control volume at steady state.
liquid is desirable. (ii) Compression, condensation and evaporation
processes are internally reversible.
(iii) The kinetic and potential energy changes are
negligible.
Analysis
(i) The dry saturated vapour delivered to the
condenser and after condensation throttled
without sub-cooling
At 12.37 bar from properties of ammonia; Table
B-10,
h3 = hf = 332.8 kJ/kg ◊ K
h2 = hg = 1469.9 kJ/kg ◊ K
s2 = 4.962 kJ/kg ◊ K
At 1.902 bar
hf1 = 89.8 kJ/kg,
hg = 1420 kJ/kg
sf1 = 0.368 kJ/kg ◊ K,
sg = 5.623 kJ/kg ◊ K
Fig. 13.20 (a) The quality of refrigerant at the state 1;
s2 = s1 = sf1 + x1 sfg1 = sf1 + x1 (sg1 – sf 1)
Example 13.12 The pressure in the evaporator of an 4.962 = 0.368 + x1 (5.623 – 0.368)
ammonia refrigerator is 1.902 bar and the pressure in or x1 = 0.874
the condenser is 12.37 bar. Calculate the refrigeration The specific enthalpy of refrigerant at the
effect per unit mass of the refrigerant and (COP)R for the state 1;
following cycles:
h1 = hf1 + x1 (hg1 – hf1)
(a) The dry saturated vapour delivered to the con- = 89.8 + 0.874 ¥ (1420 – 89.8)
denser after isentropic compression and no un-
= 1251.8 kJ/kg
Refrigeration 465

The refrigerating effect


qin = h1 – h4 = h1 – h3 (∵ h4 = h3)
= 1251.8 – 332.8 = 919 kJ/kg
(b) The COP of the system;
Refrigeration effect qin
(COP)R = =
Work input h2 - h1
919
= = 4.21
1469.9 - 1251.8
(i)
(ii) The dry saturated liquid delivered to the
compressor
The properties of the refrigerant, Table B-10, 11
h1 = hf @ 1.902 bar = 1420 kJ/kg
h3 = hf @ 12.37 bar = 332.8 kJ/kg
h4 = h3 = 332.8 kJ/kg
s1 = sg = 5.623 kJ/kg ◊ K @ 1.902 bar
sg2 = 4.962 kJ/kg ◊ K
sg2 @ 12.37 < s2 = 5.623
Thus state 2 is superheated.
(ii)
Using p-h chart
h2 = 1698.5 kJ/kg
(a) The refrigeration effect;
qin = h1 – h4 = 1420 – 332.8
= 1087.2 kJ/kg
(b) The coefficient of performance;
Refrigeation effect qin
COP = =
Work input h2 - h1
1087.2
= = 3.9 (iii)
1698.5 - 1420
(iii) Dry saturated refrigerant vapour delivered to Fig. 13.21
compressor and liquid after condensation is
undercooled by 10°C Example 13.13 Consider a vapour compression re-
h1 = 1420 kJ/kg frigeration system using R-12 as a refrigerant. The maxi-
h2 = 1698.5 kJ/kg mum and minimum pressure of the cycle are 8 bar and
h3 = h4 1.2 bar respectively. At the compressor inlet , the vapour
temperature is –12°C and the temperature of liquid at
h3 = hf @ T3
the condenser outlet is 30°C. The required refrigerating
T3 = T3¢ – 10 = 32 – 10 = 22°C
load is 2.2 kW. The compressor runs to 600 rpm and has
h3 @ 22°C = 284.6 kJ/kg a volumetric efficiency of 75%. Find COP and swept vol-
(a) Refrigeration effect ume of compressor.
qin = h1 – h4
= 1420 – 284.6 = 1135.4 kJ/kg Solution
(b) COP of the system
Given A vapor compression refrigeration system:
qin 1135.4 p1 = 1.2 kPa, p2 = 8 bar
(COP)R = = = 4.08
h2 - h1 1698.5 - 1420 T1 = –12°C = 285 K, T3 = 30°C
RE = 2.2 kW, hvol = 0.75, N = 600 rpm
466 Thermal Engineering

To find VR 0.00232
(i) (COP)R, and or Vs = = = 0.00309 m3/s
hvol 0.75
(ii) Swept volume of compressor.
Swept volume rate can be expressed as
Assumptions N
Vs = Vs
(i) Each component in the all cycle is analysed as a 60
60
control volume at steady state. or Vs = 0.00309 ¥
600
(ii) Compression, condensation and evaporation
= 0.000309 m3 or 309 mm3
processes are internally reversible.
(iii) The kinetic and potential energy changes are
negligible.

The actual refrigeration cycle deviates from the


ideal vapour compression cycle, because of pressure
drop in tubes associated with fluid friction and heat
transfer to or from the surroundings. Further, the
compression will be polytropic involving friction
and heat transfer, thus migrating from an isentropic
one. The actual vapour compression cycle may
have some or all the processes deviate from an
ideal ones. The actual cycle might be approached
Fig. 13.22
one shown in Fig. 13.23 with the help of (b) T–s
Analysis Properties of refrigerant R-12 at end state:
diagram and (c) p–h diagram.
h1 = 354 kJ/kg, vg1 = 0.135 m3/kg The refrigerant leaving the evaporator at the
h2 = 388 kJ/kg, h3 = h4 = 226 kJ/kg, state 7 is slightly superheated in order to ensure
(i) COP of the system
completely dry refrigerant vapour entry to the com-
Heat removed per kg of refrigerant
pressor. In tube line connecting the evaporator with
qin = h1 – h4 = 354 – 226 = 128 kJ/kg
the compressor, the fluid friction causes a pressure
The compression work
drop of Dpev and temperature difference between
win = h2 – h1 = 388 – 354 = 34 kJ/kg
refrigerant and surroundings causes a heat transfer
to the refrigerant from the state 7 to 8. As a result,
q 128 kJ/kg
(COP)R = in = = 3.764 the refrigerant vapour at the state 8 gets superheated
win 28 kJ/kg
further, causing an increase in its specific volume,
(ii) Swept volume of compressor
and thus increase in compression work.
The mass flow rate of refrigerant
Due to suction effect of the compressor, the sud-
Refrigerating effect
mR = den pressure drop from the state 8 to the state 1
Heat removed per kg of air
takes place. The compression begins from the state
RE 2.2 kJ/s 1 and continues to the state 2. Actual compression
= = = 0.01718 kg/s
qin 128 kJ/kg is polytropic with friction and heat transfer, and
Volume flow rate through compressor thus departs from isentropic compression. At the
VR = mR vg1 = 0.01718 ¥ 0.135 compressor discharge, a sudden pressure drop is
= 0.00232 m3/s observed from the state 2 to the state 3.
The volumetric efficiency is given by During the process 3– 4, the refrigerant is first
Actual volume rate VR desuperheated to the state 3¢ and then condensed
hvol = =
Swept volume rate Vs during the process 3¢-4. A pressure drop further
Refrigeration 467

from the state 6 to 7. A slight pressure drop due to


WARM friction during evaporation makes the process 6 –7
environment at TH
irreversible.
Further, in order to have the effective heat trans-
QH
4 3 fer between the refrigerant and hot or cold region,
Condenser the refrigerant temperature in the evaporator is less
2
5 than the cold region and that in the condenser is
Compressor
Win higher than the warm atmospheric temperature. It
Expansion
valve makes the actual cycle further irreversible.
6 1
Evaporator 8 Example 13.14 Refrigerant 134a is the working fluid
7 in an ideal vapour compression refrigeration cycle, that
QL
operated between a cold region at 0°C and a warm region
COLD refrigerated
at 26°C. The saturate a vapour enters the compressor at
space at TL –10°C and the saturate liquid leaves the condenser at a
pressure of 9 bar. Determine for m = 0.08 kg/s (a) com-
pression power in kW, (b) refrigeration capacity in tonnes
(a) Schematic arrangement of actual vapour-compression
cycle and (c) coefficient of performance.

T 2 Solution
3
Given An ideal vapour compression refrigeration cycle
4 3¢ operates with refrigerant 134a as the working fluid
5 Mass flow rate, m = 0.08 kg/s
8 Cold region temperature = 0°C
7
6 1 Warm region temperature = 26°C
s Temperature of refrigerant vapour at compressor
(b) Actual VCC on T–s diagram inlet, T1 = –10°C
p Pressure of saturate liquid leaving the condenser =
9 bar
2
Dpc 4 To find
5 3
(i) Power input in compressor in kW,
Dpev 78
(ii) Refrigerating capacity in tonnes,
6
(iii) COP of the system.
1
h Schematic with given data
(c) Actual VCC on p-h diagram

Fig. 13.23

takes place in the tube line connecting the con-


denser and throttle valve. The liquid refrigerant is
slightly subcooled in the condenser from the state 4
to 5 in order to have at least saturated liquid before
throttling at the state 5.
The throttling process is represented by a
dotted line 5 – 6 and the refrigeration effect starts Fig. 13.24
468 Thermal Engineering

Assumptions Solution
(i) All the components of the cycle are analysed as a
Given A refrigerator operates with R-12 refrigerant
control volume at steady state.
with the operating conditions
(ii) All processes except throttling process are
mR = 0.05 kg/s
internally reversible.
(iii) Kinetic and potential energy effects are negligible. To find
(iv) Saturate vapour leaves the evaporator and saturate (i) Refrigeration effect,
liquid leaves the condenser. (ii) Work input to the compressor,
Analysis The properties of refrigerant 134a (iii) Isentropic efficiency of the compressor,
State 1: Saturate refrigerant vapour T1 = –10°C (iv) COP of the system.
h1 = 241.35 kJ/kg s1 = 0.9253 kJ/kg ◊ K Assumptions
State 2: Superheated refrigerant vapour p2 = 9 bar (i) All the components of the cycle are analysed as a
s2 = s1 = 0.9253 control volume in steady state.
h2 = 272.4 kJ/kg (by interpolation) (ii) Neglect kinetic and potential energy effects.
State 3: Saturate liquid at 9 bar
Schematic with given data
h3 = 99.56 kJ/kg
State 4: Liquid vapour mixture after throttling
h4 = h3 = 99.56 kJ/kg
(i) Compressor work input;
Wcomp = m (h2 – h1)
= 0.08 ¥ (272.4 – 241.35)
= 2.484 kW
(ii) Refrigerating capacity;
RE = m (h1 – h4)
= 0.08 (241.35 – 99.56)
= 11.343 kW
(11.34 kW )
= Fig. 13.25
(3.5 kW/ton)
= 3.24 ton of Refrigeration Analysis The properties of R-12 from Table B-5
(iii) Coefficient of performance; State 1: Superheated vapour,
Refrigerating effect p1 = 0.14 MPa,
(COP)R =
Work input T1 = –20°C
11.343 h1 = 179.01 kJ/kg, s1 = 0.7147 kJ/kg.
= = 4.567
2.484 State 2: Superheated compressed vapour at
p2 = 0.8 MPa, T2 = 50°C
Example 13.15 The refrigerant R-12 enters the com-
pressor of a refrigerator as a superheated vapour at 0.14 h2 = 213.45 kJ/kg
MPa and –20°C at a rate of 0.05 kg/s and leaves at 0.8 h2s = 210.08 kJ/kg s2 = s1 = 0.7147 kJ/kg ◊ K
MPa and 50°C. The refrigerant is cooled in the condens- State 3: Undercooled liquid refrigerant from Table B-4
er to 26°C and 0.72 MPa and is throttled to 0.15 MPa. p3 = 0.72 MPa, T3 = 26°C
Neglecting any heat transfer and pressure drop in the h3 = hf @ 26°C = 60.68 kJ/kg
connecting line between the components, determine State 4: Liquid-vapour mixture of refrigerant after
(a) rate of heat removal from the refrigerated space, throttling
(b) power input to the compressor, h4 = h3 = 60.68 kJ/kg
(c) the isentropic efficiency of the compressor, (i) Refrigerating effect
(d) coefficient of performance of the refrigerator. RE = Qin = mR (h1 – h4)
Refrigeration 469

= 0.05 ¥ (179.01 – 60.68) 3. The production of some refrigerant may be


= 5.92 kW or 1.69 TR hazardous to the environment.
(ii) Work input in compressor
Win = m r (h2 – h1) 13.8 VAPOUR ABSORPTION
= 0.05 ¥ (213.45 – 179.01)
= 1.722 kW
(iii) Isentropic efficiency hC of the compressor In a vapour compression refrigeration cycle, the
h2s - h1 210.08 - 179.01 temperature of saturated vapour leaving the evapo-
hC = = rator is increased by a compression process. Since
h2 - h1 213.45 - 179.01
= 0.902 = 90.2% the specific volume of vapour is relatively large,
(iv) Coefficient of performance therefore, the input work to the compressor is also
RE 5.92 kW large. The input work to the compressor can be re-
(COP)R = = = 3.43 duced significantly, if the refrigerant is compressed
Win 1.722 kW
in liquid state. The absorption refrigeration is based
on this approach.
Refrigeration A vapour absorption system operates with a
condenser, a throttle valve and an evaporator in the
same way as vapour compression system, but the
The followings are the advantages and disadvantages
compressor is replaced by an absorber, pump and
of vapour compression refrigeration system over
generator units as shown in Fig. 13.26.
air refrigeration system:
The low-pressure refrigerant vapour leaving the
Advantages
evaporator is absorbed by a secondary substance,
1. The vapour compression refrigeration sys- called an absorbent to form a strong liquid solution.
tem has a high value of coefficient of perfor- This liquid solution is then pumped at a higher
mance. pressure to the generator. The specific volume
2. The vapour compression refrigeration sys- of the liquid solution is much less than that of a
tem approaches reversed Carnot cycle ex- refrigerant vapour, and thus significant less work
cept for expansion of refrigerant in expan-
High temperature
sion device. source
3. The size of refrigeration system per tonne of QH
refrigeration is smaller. Generator
2
4. It has less running cost. Condenser
5. Since refrigerant has latent heat while air has
sensible heat only. 3
b
Strog solution

6. The refrigerant has large value of latent c


Weak solution

heat at lower pressure, and heat is absorbed Expansion Valve


by evaporation of low pressure liquid valve

refrigerant.
Absorber
7. The temperature of refrigerant in the evapo- 4 Wp
rator can easily be controlled by regulating Evaporator
Pump
1
the expansion valve.
QL a
Disadvantages
Refrigerated Cooling
1. It has high initial cost. region water

2. The leakage of refrigerant may cause harm- Fig. 13.26


ful effect.
470 Thermal Engineering

is required in the pump. The heat is supplied in the in the absorber to absorb the energy released due
generator, where the refrigerant vapourises from the to absorption of ammonia in the water. The strong
solution and leaves weak solution in the generator. ammonia–water solution is pumped to the generator
The refrigerant vapour enters the condenser and the through a heat exchanger, where it is preheated with
weak solution is again sent back to the absorber the help of hot weak solution returning to absorber,
through a pressure relief valve. thereby, reducing the heat supply in the generator.
The coefficient of vapour absorption refrigeration In the generator, the heat transfer from the source
system can be expressed as (solar energy) drives the ammonia vapour out of
Heat absorbed in evaporator the solution (endothermic process), leaving a weak
(COP)R = ammonia–water solution in the generator.
Heat input in generator + pump work
QL The ammonia vapour liberated passes to the
=
QH + W p condenser at the state 2 through a rectifier. The rec-
13.8.1 tifier removes the traces of water from the refriger-
ant if any, before it enters the condenser. It avoids
the formation of ice in the system. The remaining
Figure 13.27 shows a schematic arrangement weak solution returns back to the absorbent through
of ammonia–water vapour absorption system heat exchanger and valve. The condensed ammonia
using the solar energy for generator heating. The is expanded through an expansion valve and then
ammonia is used as refrigerant and water is used enters the evaporator, where it absorbs heat from
as absorbent. In the absorber, the ammonia vapour the low-temperature region.
coming out the evaporator at the state 1 is absorbed
by liquid water. The formation of this liquid
solution is exothermic, thus heat is released. The
solvency of ammonia in the water decreases as The electrolux absorption principle works on
temperature increases. Thus the cooling is required a three-fluid system. It uses natural circulation

Fig. 13.27
Refrigeration 471

in absence of any pump. Figure 13.28 shows a The ammonia liquid leaving the condenser en-
schematic of an electrolux refrigeration system. ters the evaporator, where it evaporates in the pres-
The refrigerant used in the system is ammonia, ence of H2 gas at low temperature corresponding
the absorbent is water and the third fluid hydrogen to its partial pressure. The ammonia and H2 gas
remains mainly in the evaporator for reducing the mixture passes to the absorber, where it mixes with
partial pressure of the refrigerant to enable it to weak ammonia–water solution coming from the
evaporate at low pressure and low temperature. The separator. The water absorbs the ammonia vapour
total pressure is constant throughout the system. and the solution becomes strong. The H2 gas sepa-
rates from the solution and returns to the evaporator.
The strong ammonia solution passes the generator,
where it is heated externally and ammonia vapour
drives out and rises to the separator. The moisture
from the vapour is separated out and a weak am-
monia solution returns back to the absorber. The
ammonia vapour from the separator enters the con-
denser, where it is condensed and then returns to
the evaporator.
With the certain modifications, the system can
be used in places, where electricity is not available.
It requires energy only in the form of heat (waste
Fig. 13.28 heat or solar energy) and no pump is necessary.

S.No. Aspect Vapour Absorption System Vapour Compression System


1. Energy input The vapour absorption system takes in low- Vapour compression system takes in
grade energy such as waste heat from furnace, high grade energy such as electrical
exhaust steam or solar heat for its operation. or mechanical energy for operation of
compressor used in the cycle.
2. Moving part It uses a small pump as moving part, which is It uses a compressor driven by an electric
run by a small motor. motor or engine.
3. Evaporator It can operate with reduced evaporator pressure, The refrigeration capacity decreases with
pressure with little decrease in refrigeration capacity. lowered evaporator pressure.
4. Load variation The performance of vapour absorption system The performance of vapour compression
does not change with load variation. system is very poor at partial load.
5. Evaporator xit
e In vapour absorption system, the liquid In a vapour compression system, it is
refrigerant leaving the evaporator does not put desirable to superheat vapour before
any bad effect on the system except to reduce leaving the evaporator, so no liquid can
the refrigeration effect. enter the compressor.
6. COP The COP of the system is poor. The COP of the system is excellent.
7. Capacity It can be built in capacities well above 1000 TR. For a single compression system, it is not
possible to have a system with more than
1000 TR capacity.
8. Refrigerant Water or ammonia is used as refrigerant. Chlorofluorocarbon, hydrochlofluoro
carbon and hydrofluorocarbon are used in
most of the system.
9. Lowest Since water is used as refrigerant, thus the With cascading, the temperature can
temperature lowest temperature attained is above 0°C. be lowered upto – 150°C or even less
temperature.
472 Thermal Engineering

STEAM JET REFRIGERATION er some of water is returned to flash chamber as


a make up water, and rest is used as feed water to
It uses water as refrigerant, which is quite safe like the boiler.
air. If the pressure exerted on the surface of water The steam jet refrigeration system uses waste
is reduced then saturation temperature also lowers steam returning to condenser. Thus it is quite
and water starts evaporating at lower temperature cheap, but the steam jet refrigeration systems are
due to reduced pressure. It is the basic principle for not used, when temperature below 5°C is required.
steam jet refrigeration system. It is widely used in food precessing plants for
The layout of steam jet refrigeration system is precooling of vegetables and concentration of fruit
shown in Fig. 13.29. It consists of an evaporator juices, gas plants, paper mills, breweries etc.
(flash chamber), a steam nozzle, an ejector and a
condenser. The steam expands through a nozzle
to form a high speed, jet, which draws (sucks)
the water from the flash chamber into the ejector. A refrigerator maintains a region at low temperature
Therefore, the pressure in the flash chamber gets by removing heat QL and it rejects heat QH to a high-
reduced, thereby results into further formation of temperature environment. However, the same basic
vapour (evaporation) in the (flash chamber). This cycle could maintain a region at higher temperature
evaporation extracts heat (latent heat which is high- by supplying heat QH, while absorbing heat QL
er at lower pressures) for phase change, thus reduc- from low temperature medium such as atmosphere,
ing the temperature of water in the flash chamber. lake, etc. Then the device, which maintains a region
The cold water is used for refrigeration. The mix- such as commercial building at higher temperature
ture of steam and water vapour is diffused in the than that of its surroundings is called a heat pump.
diverging part of the ventury tube to the exhaust Figure 13.30 shows a schematic of a heat pump.
pressure and fed to the condenser. After condens-

Building

QH
Condenser
2
3
Win
Expansion
Compressor
device
1
Evaporator
4

Fig. 13.30

The modern air-conditioning unit combines


both heating and cooling arrangement as shown
in Fig. 13.31. When cooling is required, it is op-
erated in the refrigeration mode and removes heat
QL from living space and rejects heat QH outside
the building environment. When heating of living
space is required in the winter, it absorbs heat QL
Fig. 13.29
from the environment and supplies heat QH to the
living space.
Refrigeration 473

REFRIGERANT
Expansion
valve
The refrigerant is a heat-carrying medium, which
Inside Outside undergoes the theromdynamics cycle of refrigera-
heat exchanger heat exchanger
tion (i.e., compression, condensation, expansion
Reversing and evaporation). In a refrigeration system, it ab-
valve
sorbs the heat from a low-temperature medium and
Heating mode discards the absorbed heat to a high-temperature
Cooling mode environment.
Wc

Compressor Refrigerant
Fig. 13.31 A good refrigerant should have the following prop-
erties:
In the most common type of vapour-compres- (i) The saturation pressure of the refrigerant at
sion heat pump for space heating, the evaporator a desired low temperature should be above
communicates thermally with the outside air. Such or equal to the atmospheric pressure in or-
air-source heat pumps can also be used to provide der to avoid leakage in the evaporator. The
cooling in the summer with the use of a reversing pressure at the condenser must not be exces-
valve, as illustrated in Fig. 13.31. The solid lines sively high for the same reasons.
show the flow path of the refrigerant in the heating (ii) The latent heat of evaporation at low tem-
mode as discussed above. To use the same compo- perature should be as high as possible to give
nents for cooling effect, the reverse valve is actu- a reasonably low mass flow rate of refriger-
ated and the refrigerant follows the path shown by ant for a given refrigeration capacity.
the dashed line. In the cooling mode, the outside (iii) The size of the compressor depends on the
heat exchanger becomes the condenser and the in- specific volume of the refrigerant at evapora-
side heat exchanger becomes the evaporator. tor pressure. Thus, the specific volume of the
refrigerant at the compressor suction should
The heat
not be high in order to avoid the large com-
pump is used for warming of homes and offices in
pressor for the required mass flow rate.
extreme cold climate. The heat pump also offers
distinct opportunity for industrial applications. (iv) The refrigerant should be chemically stable
A heat pump can provide heating and cooling of and should not react with lubricant used in
two dissimilar fluids simultaneously. The different reciprocating compressor and should be
industrial applications of heat pump are for miscible with oil.
(v) It should not be non-flammable, non-explo-
1. Purification of salty water. sive.
2. Concentration of juices, milk and syrups, (vi) It should be non-toxic. If toxic, then to a
dyes, chemicals, etc. limit, below the acceptable level.
3. Preparation of powder milk and table salt. (vii) It should have low specific heat of liquid for
4. For recovery of valuable solvents from dif- better heat transfer in condenser.
ferent manufacturing processes. (viii) The refrigerant should give high value of
5. For year round air-conditioning. COP with low power input per tonne of re-
frigeration.
474 Thermal Engineering

(ix) The refrigerant should have good thermal Azeotropes are mixtures that behave as a single
conductivity for better heat transfer in the substance. All components of the mixture evapo-
condenser and evaporator. rate and condense at the same conditions. The fre-
(x) The refrigerant must have freezing point quently used azeotropes are R-500, a CFC/HFC
temperature well below the lowest tempera- mixtures, and R-502—a HCFC/CFC mixture.
ture in the cycle. Zeotropes, or blends, are mixtures that do not
(xi) Other considerations are chemical stablility, always behave as a single substance. For instance,
non-corrosiveness, low cost and overall eco- they may not evaporate or condense at a constant
friendliness. temperature (called the temperature glide.)

Refrigerants may be broadly classified into follow- In early days, the commonly used refrigerants were
ing groups: ammonia, methyl chloride, sulphur dioxide, carbon
dioxide, ethylchloride, propane, and butane. Some
1. Primary refrigerants, and of them were toxic. Some had very high risk of
2. Secondary refrigerants. explosion and were irritants. Thus they became
The refrigerant which directly undergo the re- obsolete except ammonia and carbon dioxide.
frigeration cycle are called primary refrigerants, Apparently, the ideal refrigerants are in a chemi-
whereas the secondary refrigerants act only as heat cal group called either fluorinated hydrocarbons or
carrier. They are first cooled by primary refriger- halocarbons (now known as R-12). They are being
ants, and then are circulated through other media used since 1930, because of their excellent char-
to absorb heat. acteristics. They have good physical properties, are
The primary refrigerants are further classified non-toxic, stable and inexpensive.
into the following groups: (a) It is designated as R-729. Dry air is used as
All the halocarbon refrigerants a primary refrigerant in as air refrigeration system,
are divided into three subgroups according to their mainly used for aircraft refrigeration. It is also used
constituents. as a secondary refrigerant in domestic refrigerators.

(a) Chlorofluorocarbons (CFCs) composed of (b) It is designated as R-118. It is used in its


chlorine, fluorine and carbon atoms solid form as ice for chilling of beverages and other
Examples are R-11, R-12, and R-114. things. It is also used as refrigerant in vapour absorption
(b) Hydro chlorofluorocarbons (HCFCs) com- systems. Its high freezing-point temperature limits its
posed of hydrogen, chlorine, fluorine and use in a vapour compression system.
carbon atoms. Examples are R-22 and R-123. (c) It is designated as R-744. It
(c) Hydro fluorocarbons (HFCs) composed of is a non-toxic, non-irritating and non-flammable
hydrogen, fluorine and carbon. Examples are gas. It has extremely low freezing temperature,
R-134a (HFC-134a), and R-125 (HFC-125). but high operating pressure and temperatures in
refrigeration system. Its specific volume is also very
2. Inorganic such as air, water, ammo- low. Therefore, it requires a very small compressor.
nia and carbon dioxide. It has low COP, and thus is not preferred for
refrigerants, such as ethane, pro- commercial use. It is suitable for manufacturing of
pane, butane, etc. dry ice and marine purposes.
There are also mixtures of above substances that (d) It is designated as R-717. Its chemi-
are used as refrigerants. These are azeotropes and cal formula is NH3. It is a highly toxic gas, irritat-
zeotropes. ing to the eyes, nose and throat. It is non-corrosive
Refrigeration 475

to ferrous metals and attacks brass and bronze. Its


boiling point at atmospheric pressure is –33.3°C Ozone (O3) gas consists of three atoms of oxygen
and its freezing point is –78°C. Its low boiling-
per molecule. It is a vigorous oxidising agent
point temperature makes it favourable for use at
and it cleans the air in the atmosphere. Ozone is
well below 0°C without lowering its pressure be-
present in a layer in the earth’s atmosphere, known
low atmospheric. It is cheap and therefore, widely
as the stratosphere, about 11–50 km above the
used in large and commercial cold storages and ice
earth’s surface. The ozone layer blocks out most
plants. It is also used in vapour absorption systems.
of harmful ultraviolet (UV) radiation coming from
(e) It is a very popular refrigerant and the sun. The ozone layer has been progressively
designated as R-12. It is colourless, almost odour- depleting. One chlorine atom can destroy 100,000
less, non-toxic, non-flammable and non-irritating. ozone molecules by a continuous chain reaction.
It has a low boiling point temperature as –29°C at The effects of depletion of ozone layer over the
atmospheric pressure. It is non-reactive with lubri- earth include the following:
cating oils. It is relative costly and has low latent
heat. Thus, refrigeration system requires large mass 1. The ultraviolet rays in the solar radiation will
flow rate. It is used on small refrigerating machines. reach the earth surface and cause an increase
in skin cancer (most deadly form of cancer).
(f ) It is also a popular refrigerant and 2. An increase in eye diseases.
designated as R-22. It has a low boiling point 3. Reduction in immunity against disease.
temperature and is able to maintain a temperature
4. Harmful effects on crops, timber, wild life
up to – 40°C. It is widely used in air-conditioning
and marine life.
units and in household refrigerators. It can be used
with reciprocating and centrifugal compressors. The relative ability of a substance to deplete the
ozone layer is called the ozone depletion potential
(ODP). R-11 and R-12 have the highest value,
The important parameters to be considered in selec- ODP = 1.0. Table 13.1 lists some of the refrigerants
tion of a refrigerant are temperature of refrigerated and their ODP values. HCFCs have relatively low
space and type of equipment to be used. ODP, while HFCs do not cause any harm to the
The temperature of the refrigerant in the evapo- ozone layer (ODP = 0).
rator and the condenser are governed by the tem-
peratures of the cold and warm regions, respective-
ly. In order to have as effective heat transfer rate, a Refrigerants that are friendly to the ozone layer in
temperature difference of at least 10°C should be the atmosphere that protects the earth from harmful
maintained between the refrigerant and medium, ultraviolet rays are called eco-friendly refrigerants.
with which the system interacts thermally. These For example, R-134a is a chlorine-free refrigerant
temperatures also fix the operating pressures in the and, thus it is an eco-friendly refrigerant.
evaporator and condenser. Therefore, the selection
The chlorine refrigerants react with the ozone
of a refrigerant is based partly on the suitability of
layer and try to destroy the ozone layer in the up-
pressure–temperature relationship in the range of
per atmosphere by continuous chain reaction.
the particular application.
Hydrocarbons (HCs) and hydro fluorocarbon
It is generally avoided to have a very low pres-
groups provide an alternative to chlorinated refrig-
sure in the evaporator and an excessive high pres-
erant. They contain no chlorine atom and therefore,
sure in the condenser in order to minimize the leak-
have zero ozone depletion potential.
ages to or from the system.
476 Thermal Engineering

Sub Refrigerant Designation Chemical Formula Refrigerant Name ODP


Group
CFCs R-11 CCl3F Trichlorofluoro methane 1.0
R-12 CCl2F2 Dichlorodifluoro methane 1.0
R-113 CF2Cl–CFCl2 Trifluoro-tirchlore ethane 0.8
R-114 CF2Cl–CF2Cl Tetrafluoro-dichloro ethane 1.0
HCFCs R-22 CHClF2 Chlorodifluoro methane 0.05
R-123 CHCl2F3 Dichlorotrifluoro ethane 0.02
HFC R-134a CH2FCF3 Tetrafluoro ethane 0.0
Azeotrope R-500 R-12/R-152 HCFC and CFC mixture 0.74
R-717 NH3 Ammonia 0

Two most commom refrigerants, Freon-11 treaty outlines the phase reduction of compounds.
and Freon-12, are highly chlorinated and are very Production and use of some compounds have been
harmful for ozone depletion. Freon-11 is replaced banned and some will be banned in future.
by HCFC-123, which has 98% less ozone deletion The viable solutions include alternate halocarbon
potential and Freon-12 is replaced by R-134a refrigerants
(Tetrafluoroethane) with zero ODP.
1. For CFC-12 (R-12) used in automobile air
conditioning and household refrigerator;
HFC-134a (R-134a) is permanent substitute
The major sources of chlorine in the atmosphere (ODP = 0)
are the production of CFCs and HCFCs. The major 2. For CFC-11 (R-11) used in centrifugal com-
developed nations have agreed to control the use pressor, an interior substitute is HCFC-123
and manufacturing of CFCs and HCFCs and the until this group is phased out
United Nations through its environment programme
3. HCFC-22 (R-22) used in window and com-
has persuaded many nations to sign the Vienna mercial air-conditioning and refrigeration;
Convention 1987, a treaty specifically intended to
possible substitute is HFC mixture R-407c
control the production of substances which cause and R-410a.
ozone depletion. The Montreal Protocol of this

Summary
The removal of heat from a lower temperature The performance of refrigerators and heat pumps
region to a higher temperature region is called re- is measured in terms of the coefficient of perfor-
frigeration. Devices that produce refrigeration are mance (COP).
called refrigerators and cycles at which they oper- Desired cooling effect QL
ate are called the refrigeration cycle. The working (COP)R = =
Net work input Win
fluid used in the cycle is called the refrigerant.
The refrigerators used for heating purpose of a Desired heating effect QH
(COP)HP = =
space by transferring heat from a cold medium Net work input Win
are called the heat pumps.
Refrigeration 477

- compressor work to a pump work only. The used


pour compression cycle (VCC). In an ideal VCC, source energy is waste heat, solar energy, etc. The
the refrigerant enters the compressor as a satu- most commonly used vapour absorption refriger-
rated vapour where it is compressed to high pres- ation system is the ammonia–water system with
sure. The superheated vapour is then cooled to ammonia as refrigerant and water as absorbent.
saturate liquid state in the condenser before throt- The COP of such a system is
tling to evaporator pressure, where the refrigerant Refrigeration effect
evaporates, absorbs its enthalpy of evaporation (COP)R =
Heat input + Pump work
(latent heat) from the refrigerated space.
QL
refrigeration =
QH + Wp
cycles by reversing the directions of processes
involved. One of these is the reversed Brayton refrigerant for a refrig-
cycle, used for gas refrigeration systems. It is ex- eration cycle involves normal operating pressure
tremely used for aircraft cooling and liquefaction at very low and high temperatures, low specific
of gases. The COP of a gas refrigeration cycle is volume at evaporator pressure, a high latent heat
qin qin of evaporation at low temperature, chemical sta-
(COP)R = = bility, non-toxic, non-corrosive and overall eco-
wnet , in wcomp - wTurbine
friendly. CFCs and HCFCs contain large chlorine
lebels and thus cause depletion to the ozone layer
vapour absorption refrigeration system, where the in the stratosphere of the earth’s atmosphere.
refrigeration is absorbed by a transport medium Thus, their use and production are gradually
and is compressed in liquid form, the reduced phased out.

Glossary
Air refrigeration A refrigeration system in which air is Refrigerator A machine which produces refrigeration
the working medium. effect
Heat pump A machine which keeps a space at higher Secondary refrigerant A substance which acts as a heat
temperature than its surroundings carrier and exchanges heat with primary refrigerant only
Primary refrigerant A substance which undergoes Vapour absorption–Refrigeration A refrigeration sys-
refrigeration cycle tem in which refrigerant and its absorbent are used for
Refrigerant Working substance used in the refrigeration creating refrigeration effect without compressor
system Vapour compression–Refrigeration A refrigeration
Refrigeration effect Cooling effect system in which liquid and its vapour undergo refrigera-
Refrigeration Maintaining a space at low temperature tion cycle
than its surroundings

Review Questions
1. What are temperatures inside the fresh food and refrigeration? What is the basic formula for cal-
freezer compartments of a household refrigerator? culating the tonnage of refrigeration?
2. What is frosting? How can excessive frosting 4. What are the commonly used refrigerants for
harm the performance of a refrigerator? vapour compression refrigeration systems? Would
3. Define refrigerating effect. What is one tonne of water be a suitable refrigerant in a refrigerator?
478 Thermal Engineering

5. What do you understand by primary and second- 12. What are the causes of irreversibilities in an ac-
ary refrigerants? Explain in brief. tual refrigeration cycle ? Explain with the help of
6. How is the heat absorbed or removed from a low- a T–s diagram.
temperature source and transferred to high-tem- 13. What is the effect of lower evaporator pressure in
perature source in a vapour compression system? a VCC on its performance?
7. What is the difference between a refrigerator and 14. Consider two vapour compression refrigeration
a heat pump? cycles. The refrigerant enters the throttle valve
8. Why is the reversed Carnot cycle executed within as a saturated liquid at 30°C in one cycle and
a saturation curve and not a realistic model for as a subcooled liquid at 30°C in another cycle.
refrigeration cycles? The evaporator pressure for both cycles is same.
9. Prove that (COP)HP = (COP)R + 1 Which cycle do you think will have higher COP?
10. Sketch the vapour compression cycle on a T–s 15. Why are CFC phased out? Which are the alterna-
diagram and derive an expression for its COP. tives to CFCs?
11. What is the function of a condenser in a refrigera- 16. What is ozone depletion? What are the remedies
tion cycle? to save ozone in atmosphere?

1. If the power consumed by a reversible operates on Carnot refrigeration system, calculate


refrigeration compressor is 2 kW per tonne of the COP of the system. [5.7]
refrigeration, what is it COP? [1.75] 7. Refrigerant 134a enters the compressor of an
2. What is the refrigeration load in TR, when ideal vapour compression refrigeration system
20 m3/min of water is cooled from 13°C to 8°C? as saturated vapour at –16°C with a volumetric
[33.0 TR] flow rate of 1 m3/min. The refrigerant leaves the
3. Calculate the power required by a reversible condenser at 36°C, 10 bar. Determine
refrigerator to produce 400 kg of ice per hour at (a) the compressor power, in kW
–20°C from feed water at 25°C. Take latent heat (b) the refrigerating capacity, in tonnes
of ice formation as 335 kJ/kg ◊ K and specific heat (c) the coefficient of performance
of ice as 2.09 kJ/kg ◊ K. [9.92 kW] 8. The R-12 refrigerant is used in a refrigerator op-
4. A refrigerator working on Bell Coleman cycle erating between 36°C and –7°C. The refrigerant
operates between atmospheric pressure and 7 bar. is dry and saturated at the beginning of compres-
Air is drawn from a cold chamber at 13°C. Before sion. If the actual COP is 0.65 times the theoreti-
air expander, it is cooled to 28°C. Calculate COP, cal COP, calculate the net cooling produced per
if index of expansion and compression is 1.35 hour for a flow rate of 5 kg/min. The properties of
[1.53] R-12 are given:
5. The COP of a vapour compression refrigeration
Temp. °C hf hfg sf sfg
system is 3.0. If the compressor motor draws a
36 456.4 585.3 4.74 0.67
power of 10.5 kW at 91% motor efficiency, what
–7 412.4 570.3 4.76 —
is the refrigeration effect in TR of system?
[1.77 TR]
[9.84 TR]
9. The condenser and evaporator of a 15 TR ammo-
6. (a) Ice is formed at 0°C from water at 30°C. In the
nia refrigerating plant are maintained at 25°C and
refrigeration system the same water is used for
–10°C. The refrigerant is subcooled by 5°C in the
condenser cooling and the temperature of brine
condenser. The vapour leaving the evaporator is
is –15°C at evaporator. Considering the system
Refrigeration 479

0.97 dry. Calculate COP and power required. Use superheating. Calculate the mass flow rate, COP,
the following properties: degree of sub cooling and power input. The
refrigerant leaving the condenser is dry saturated
Temp. °C hf hfg sf sfg
liquid and leaving the evaporator is dry saturated
25 317.687 1483.18 1.4084 5.3175
vapour. The compression is isentropic. The
–10 154.056 1450.22 .8296 5.755 properties of Freon -12 are
[6.86, 7.65 kW] Pressure Temp. hf hfg sf sg Cp, f Cp,v
10. A refrigeration system of 15 tonnes capacity op- bar °C kJ/kg kJ/kg kJ/kg ◊ K kJ/kg ◊ K
erates on a vapour compression cycle using R-22 1.004 –30 8.854 174.076 0.371 0.7165 – 0.579
at an evaporator temperature of 5°C and condens- 13.663 55 90.201 207.766 0.3194 0.6777 1.074 –
ing temperature of 50°C. Calculate the refriger- [0.8346 kg/s, 1.8, 0, 38.9 kW]
ant mass flow rate and compressor intake volume 12. In an aircraft cooling system, air enters the com-
flow rate, if the volumetric efficiency is 72%. Use pressor at 0.1 MPa, 4°C and is compressed to 6
following properties: MPa with an isentropic efficiency of 72%. After
being cooled to 55°C at constant pressure in a
Pressure Temp. hf hfg vf vg
heat exchanger, the air then expands in a turbine
bar °C kJ/kg kJ/kg lit/kg lit/kg
to 0.1 MPa with an isentropic efficiency of 78%.
5.836 5 205.9 407.1 0.791 0.0404 The cooling load of the system is 3 tonnes of re-
19.423 50 263.3 417.7 .922 .0117 frigeration at constant pressure before re-entering
[0.365 kg/s, 0.01475 m3/kg] the compressor, which is driven by the turbine.
11. A 20 tonnes vapour compression refrigeration Assuming that air is an ideal gas, determine the
system using Freon-12 operates between an COP of refrigerator, driving power required and
evaporator pressure of 1.004 bar and a condenser the mass flow rate of air.
pressure of 13.663 bar. The system uses 10°C

Objective Questions
1. COP of a Carnot refrigeration cycle is greater than (a) decreases COP
(a) vapour compression cycle (b) increases COP
(b) reversed Brayton cycle (c) decrease refrigerating effect
(c) vapour absorption cycle (d) increases work input
(d) all the above 5. Heat is absorbed by a refrigerant during a refrig-
2. In an ideal vapour compression refrigeration cy- erant cycle
cle, which process is irreversible? (a) condenser (b) throttle valve
(a) Compression (b) Heat erjection (c) evaporator (d) compressor
(c) Throttling (d) Heat absorption 6. Heat is rejected by a refrigerant during a refriger-
3. Subcooling of refrigerant in vapour compression ant cycle
refrigeration cycle (a) condenser (b) throttle valve
(a) decreases COP (c) evaporator (d) compressor
(b) increases COP 7. In an ideal vapour compression refrigeration cy-
(c) decrease refrigerating effect cle, the refrigerant is in the form of superheated
(d) increases work input vapour before entering into
4. Superheating of vapour refrigerant at evaporator (a) condenser (b) throttle valve
in vapour compression cycle (c) evaporator (d) compressor
480 Thermal Engineering

8. In an ideal vapour compression refrigeration (c) carry the heat within refrigerated space
cycle, the refrigerant is in form of dry saturated (d) cool the water
vapour before entering into 12. A zeotropes are mixture of
(a) condenser (b) throttle valve (a) primary and secondary refrigerant
(c) evaporator (d) compressor (b) Ammonia and water
9. The refrigerant used in vapour absorption refrig- (c) CFCs and HFCs
eration system is (d) HCFCs and HFCs
(a) Freon-12 (b) CO2 13. Ozone depletion is caused due to
(c) ammonia (d) R-134a (a) use of HFCs
10. The COP of Carnot cycle when working as heat (b) use of HCs
pump is
(c) use of CFC
(a) less than when it works as refrigerator
(d) use ofAmmonia
(b) more than when it works as refrigerator
14. Refrigerant R-134a is
(c) less than when it works as heat engine
(a) an ecofriendly refrigerant
(d) equal to when it work as refrigerator
(b) mixture of hydrofluoro carbon
11. The secondary refrigerant acts to
(c) Halo carbone refrigerant
(a) undergo the refrigeration cycle
(d) all of above
(b) protect the ozone depletion

14. (d) 13. (c) 12. (c) 11. (c) 10. (b) 9. (c)
8. (d) 7. (a) 6. (a) 5. (c) 4. (d) 3. (b) 2. (c) 1. (d)
Answers
Ideal Gas Mixtures 481

14
Ideal Gas Mixtures

Introduction
A pure substance has been defined as a substance that is homogeneous and of constant chemical composition
throughout the mass. The homogeneous mixture of gases, if they do not react each other, are therefore, also
considered pure substances, for example, dry air, which is the mixture of oxygen, nitrogen, argon, and
traces of other gases. The properties of such mixtures can be obtained, correlated and tabulated or fitted by
equations like properties of any pure substance. It is therefore, important to calculate the properties of any
mixture from the properties of its constituents.
The mixtures considered in this chapter are composed of perfect gases and vapours. The properties of
such mixtures are important in combustion and air-conditioning calculations.

expressed as
mi mi
Consider a closed system of gas, which is composed xi = = i=k
…(14.3)
m
of k components, each a pure substance. The mass
of the mixture is equal to the sum of masses of its
Âm i
i =1
components. The ratio of mole number of a component to the
m = m1 + m2 + m3 + m4 + … + mk total mole numbers of the mixture is referred as
i=k mole fraction (y) and
= Âm i …(14.1)
ni ni
i =1 yi = = i=k
…(14.4)
where mi refers to mass of the ith component. n
The total number of moles of a mixture is the Ân i
i =1
sum of the number of moles of its components.
Dividing Eq. (14.1) by m (mixture mass) and
n = n1 + n2 + n3 + n4 + … + nk Eq. (14.2) by n (mixture moles), we can easily
i=k
obtain that the sum of mass fraction or mole
= Ân i …(14.2) fraction is equal to 1,
i =1
i=k i=k
The ratio of the mass of a component to the
mass of the mixture is referred as mass fraction (x),
or  xi = 1 and Ây i =1 ...(14.5)
i =1 i =1
482 Thermal Engineering

The mass of each component can be expressed n RuT


in terms of mole number n and molecular weight Using an ideal gas relation V = in Eq.
(14.9) p
M.
mi = ni Mi ni RuT n
pi = p = i p = yi p …(14.10)
i=k n RuT n
and total mass m= Â n Mi i …(14.6) The partial pressure of each constituent is that
i =1
pressure which the gas would exert, if it existed
The apparent (or average) molecular weight of alone at the temperature and volume of the mixture.
the mixture, M, is defined as the ratio of the to- The total pressure of the mixture is defined as
tal mass of the mixture, m, to the total number of
moles of the mixture, n ( n1 + n2 + n3 + º+ nk ) RuT
p =
V
m
M= ni RuT n2 RuT
n = + +…
V V
Inserting Eq. (14.6), the average molecular Thus p = p1 + p2 + p3 + … + pk
weight of a gas mixture = Spi …(14.11)
i=k i=k The sum of partial pressures of the constituents

m
 mi Ân M i i in a gas mixture is equal to the total pressure of the
i =1 i =1 mixture. It is known as Dalton’s law of partial pres-
M = = =
n n n sure or law of additive pressures.
i=k
= Ây M i i …(14.7)
i =1
The gas constant for a gas mixture can be evalu-
ated as
Ru
R= …(14.8)
M
Where Ru is universal gas constant (= 8314 J/kg
mol ◊ K)
It is based on the assumption that the each mixture
component behaves as an ideal gas, if it exists alone
at the pressure p and temperature T of the mixture.

In the absence of intermolecular forces, each com-


ponent in the gas mixture behaves as an ideal gas,
if it were alone at the temperature T and volume V
of the mixture. It follows that the individual com-
ponents would not exert the mixture pressure, but
rather a partial pressure.
The partial pressure pi of i th component in a gas The partial volume Vi of ith component in a gas
mixture is defined as mixture is defined as
ni RuT ni RuT
pi = …(14.9) Vi = = yiV …(14.12)
V p
Ideal Gas Mixtures 483

The partial volume of a gas component in a gas For each constituent in the gas mixture
mixture is the volume of that gas would occupy, if p1V = m1 R1T
it existed alone at the mixture pressure and tem- p2V = m2 R2T
perature. p3V = m3 R3T
The total volume of a gas mixture is expressed pkV = mk Rk T
as Adding and using Dalton’s low of partial pres-
RuT sures
V = (n1 + n2 + n3 + n4 + … + nk)
p pV = (m1 R1 + m2 R2 + m3 R3 + … + mk Rk)T
RuT RT RT RT comparing it with characteristic gas equation
= n1 + n2 u + n3 u + … + nk u pV = mRT
p p p p
we get apparent gas constant,
i=k
= V1 + V2 + V3 + … + Vk = ÂV i …(14.13) m R + m2 R2 + m3 R3 + º + mk Rk
R= 1 1
m
i =1
Thus, sum of partial volumes of the constituents = S xi Ri …(14.16)
in a gas mixture is equal to the total volume of the
mixture. It is known as Amagat–Leduce law of ad- Example 14.1 A vessel of volume 0.4 m3 contains
ditive volumes. 0.45 kg of carbon monoxide and 1 kg air, at 15°C.
Calculate the partial pressure of each constituents and
the total pressure in the vessel. The air contains 23.3%
oxygen and 76.7% nitrogen by mass. Take the molar
masses of carbon monoxide, oxygen and nitrogen as 28,
32 and 28 kg/k mol, respectively.
The specific volume of the gas mixture v is defined as
V V Solution
v= =
m m1 + m2 + m3 + º + mk Given A mixture of carbon monoxide and air in a
1 m1 m m m m vessel
or = + 2 + 3 + 4 +…+ k
v V V V V V V = 0.4 m3, mair = 1 kg,
1 1 1 1 Tair = 15°C = 288 K, mCO = 0.45 kg,
= + + +…+
v1 v2 v3 vk MCO = 28, MO2 = 32,
MN2 = 28
1
We have r = , therefore, density of a gas
v To find
mixture,
(i) Partial pressure of CO, O2 and N2 in the mixture,
r = r1 + r2 + r3 + r4 + … rk = Sri …(14.14) (ii) Total pressure of mixture.
The characteristic gas equation for ith component Assumptions
in a gas mixture can be expressed as (i) Each component in the mixture behaves like an
piV = ni RuT ideal gas.
m (ii) Mixture composition remains invariable.
where ni = i
Mi Analysis Calculating the mass of each component of
m
piV = i RuT air
Mi Mass of oxygen mO2 = 0.233 mair
R = 0.233 ¥ 1 = 0.233 kg
But Ri = u
Mi Mass of nitrogen, mN2 = 0.767mair = 0.767 kg
Therefore, piV = mi Ri T …(14.15)
484 Thermal Engineering

The specific gas constant of gas is given by = pCO + pO2 + pN2


Ru 8.314 kJ / kmol ◊ K = 0.9623 + 0.4345 + 1.64
R = = = 3.03 bar
M M ( kg / kmol)

8.314 Example 14.2 A volumetric analysis of a gaseous


Thus RCO = = 0.297 kJ/kg ◊ K
28 mixture gives the following results:
8.314 CO2 = 12.0% O2 = 4.0%
RO2 = = 0.259 kJ/kg ◊ K N2 = 82.0% CO = 2.0%
32
Determine the analysis of gas mixture on the mass
8.314
RN2 = = 0.297 kJ/kg ◊ K basis, the molecular weight and the gas constant on the
28 mass basis for the mixture. Assume ideal gas behaviour.
(i) The partial pressure of a gas is given by
Solution
m RT
pi = i i Given The components of gas mixture as
V
3
Since volume V = 0.4 m and T = 15 + 273 = 288 K, CO2 = 12%, O2 = 4%,
remain constant, the partial pressure of gas N2 = 82% CO = 2%
constituents are To find
m CO RCOT (i) Analysis of gas in mixture on mass basis,
pCO =
V (ii) Molecular weight of mixture, and
0.45 ¥ 0.297 ¥ 288 (iii) Gas constant of mixture.
=
0.4
= 96.23 kPa = 0.9623 bar Analysis
mO2 RO 2T (i) Analysis of component gas in mixture on mass
pO2 = basis is represented in Table 14.1.
V
0.233 ¥ 0.259 ¥ 288 (ii) Molecular weight of mixture M = 30.08 kg/kmol
= (iii) Gas constant for mixture
0.4
= 43.35 kPa = 0.4345 bar Ru 8.314
R = = = 0.276 kJ/kg ◊ K
mN 2 RN 2 T M 30.08
pN2 =
V
0.767 ¥ 0.297 ¥ 288 The gravitational analysis of air is
= 23.14% oxygen, 75.53% nitrogen, 1.28% argon and
0.4
0.05% carbon dioxide. Calculate the volumetric analysis
= 164.01 kPa ª 1.64 bar
and the partial pressure of each constituent in the
(ii) The total pressure of the gas mixture mixture, when the total pressure is 1 bar.
p = S pi
Table 14.1

Component % volume Mole fraction Molecular Mass kg/kmol of Mass basis mi


gas Weight Mi mixture, mi = yi Mi = ¥ 100
yi percentage m
CO2 12.0 0.12 44 5.28 17.55%
O2 4.0 0.04 32.0 1.28 4.26%
N2 82.0 0.82 28.0 22.96 76.33%
CO 2.0 0.02 28.0 0.56 1.86%
1.00 m = 30.08 100%
Ideal Gas Mixtures 485

Solution (ii) Using mole fractions to obtain partial pressures.


pi = yi p
Given Gravitational analysis of air as mixture of O2,
pO2 = 0.2093 ¥ 1 = 0.2093 bar
N2, argon and CO2.
pN2 = 0.781 ¥ 1 = 0.781 bar
To find pargon = 0.00092 ¥ 1 = 0.00092 bar
(i) Volumetric analysis, pCO2 = 0.00028 ¥ 1 = 0.00028 bar
(ii) Partial pressure of each constituent in air at p = 1
bar. Example 14.4 A vessel contains a gaseous mixture of
composition by volume 80% H2, and 20% CO. It is de-
Analysis For 1 kg of air as mass and molecular weight signed that the mixture should be made in the proportion
of constituents. 50% H2 and 50% CO by removing some of the mixture
mO2 = 0.2314 kg MO2 = 32 kg/kmol and adding some CO. Calculate per kmol of mixture the
mN2 = 0.7553 kg MN2 = 28 kg/kmol mass of mixture to be removed and the mass of CO to
margon = 0.0128 kg Margon = 40 kg/kmol be added. The pressure and temperature in the vessel re-
mCO2 = 0.0005 kg MCO2 = 44 kg/kmol main constant during the procedure.
Number moles of each gas constituents in the mixture Take the molar mass of hydrogen and CO as 2 kg/kmol
mi and 28 kg/kmol, respectively.
ni =
Mi
Solution
0.2314
nO2 = = 0.00723 Given A mixture of H2 and CO in a vessel
32
yH2 = 0.8, yCO = 0.2
0.7553 MH2 = 2, MCO = 28
nN2 = = 0.02697
28
To find
0.0128 (i) The mass of the mixture to be removed per kmol
nargon = = 0.00032
40 of mixture, and
0.0005 (ii) The mass of CO to be added per kmol of the
nCO2 = = 0.00001 mixture.
44
Total number of moles Analysis
n = S ni = 0.03453 (i) Since the pressure and temperature remain
The mole fraction is equal to volume fraction as well constant, thus the mass of gas in the vessel
as pressure fraction, thus remains constant. Therefore, the mass of mixture
removed = mass of CO added.
ni p V
yi = = i = i Let x kg of mixture to be removed and y kg CO
n p V
be added.
VO 2 0.00723 For a mixture the molecular weight
xO2 = = = 0.2093 or 20.93%
V 0.03453 M = Syi Mi
VN 2 0.02697 = 0.8 ¥ 2 + 0.2 ¥ 28 = 7.2 kg/kmol
xN2 = = = 0.781 or 78.1% The number of moles of mixture
V 0.03453
m
Vargon 0.00032 n =
xargon = = = 0.00092 or 0.92% M
V 0.03453
The moles of mixture to be removed
VCO2 0.00001
xCO2 = = = 0.00028 or 0.028% x kg x
V 0.03453 = = kmol
7.2 kg / kmol 7.2
486 Thermal Engineering

y (i) The mass fraction of oxygen


The moles of CO to be added = kmol and for
given conditions 28 mO2 1.8
xO2 = = = 0.4
x y m 3
=
7.2 28 The mass fraction of nitrogen
No. of moles of H2 in the mixture to be removed mN 2 1.2
x x x xN2 = = = 0.6
= yH = 0.8 ¥ = kmol m 3
2 7.2 7.2 9
The mass of the gas can also be expressed in
Ê xˆ
The moles of H2 in the vessel = Á 0.8 - ˜ kmol terms of number of moles and molecular weight;
Ë 9¯ mi = ni Mi
1 kg of new mixture contains 50% H2 and 50% mO2 1.2
CO, therefore \ nO2 = = = 0.0375
MO2 32
x
0.8 – = 0.5 mN 2 1.8
9 nN2 = = = 0.06428
or x = 9(0.8 – 0.5) = 2.7 kg/kmol MN 2 28
Thus the mass of mixture to be removed is 2.7 kg Hence, the total number of mole in the gas
per kmol. n = nO2 + nN2 = 0.0375 + 0.0642
(ii) The mass of CO to be added = 0.1017 moles
x y Thus, the mole fractions
We have =
7.2 28 0.0375
28 x 28 yO2 = = 0.368
or y = = ¥ 2.7 0.1017
7.2 7.2
0.0642
= 10.5 kg/kmol yN2 = = 0.633
0.1017
Thus the mass of CO to be added = 10.5 kg/kmol.
(ii) Apparent molecular weight
Example 14.5 A mixture of gases contains 1.2 kg of i=k
oxygen and 1.8 kg of nitrogen. The pressure and tempera- M = Ây M = x
i =1
i i O2 MO2 + xN2 MN2
ture of the mixture are 350 kPa and 300 K. Determine
for mixture. = 0.368 ¥ 32 + 0.633 ¥ 28
(a) mass and mole fraction of each constituent gas, = 29.5 kg/kmol
(b) average molecular weight, (c) the partial pres- (iii) The partial pressures
sures, The partial pressure of oxygen
(d) the specific gas constant, (e) the volume and (f) pO2 = yO2 p = 0.368 ¥ 350 = 128.9 kPa
the density. pN2 = yN2 p = 0.633 ¥ 350 = 221.55 kPa
(iv) The Apparent gas constant of mixture
Solution A number of gases with
Ru 8.314
mO2 = 1.2 kg mN2 = 1.8 kg R = = = 0.2812 kJ/kg ◊ K
M 29.5
p = 350 kPa T = 300 K
(v) The volume of gas mixture
To find
RuT
(i) Mass and mole fraction of each gas constituents, V = VO2 + VN2 = (nO2 + nN2)
(ii) Average molecular weight, p
(iii) The partial pressures, 8.314 ¥ 300
(iv) The specific gas constant, (0.0375 + 0.064) ¥ = 0.742 m3
350
(v) The volume, and
(vi) The density. (vi) The density of mixture
m 3 kg
Analysis The total mass of the mixture can be calcu- r = = = 4.13 kg/m3
lated as V 0.742 m3
m = mO2 + mN2 = 1.2 kg + 1.8 kg = 3 kg
Ideal Gas Mixtures 487

where hi is enthalpy per mole for ith component of


gas mixture.
The specific heats
The extensive properties of an deal gas mixture
can be obtained by an extension of Dalton’s law mCv = m1 Cv1 + m2 Cv2 + m3 Cv3 + … + mk Cvk
of partial pressure. Dalton’s law was reformulated = S mi Cvi
by Gibbs. The combined statement is known as m1 Cv1 + m2 Cv2 + m3 Cv3 + º + mk Cvk
or Cv =
the Gibbs–Dalton law. According to this theorem, m1 + m2 + m3 + º + mk
the internal energies, enthalpies, specific heats etc.
Smi Cv i
of a gas mixture are respectively equal to sum of = = S xi Cvi …(14.23)
internal energies, enthalpies, specific heats etc. of m
individual components, each at temperature and On the molal basis
volume of mixture. Thus
Cv = Syi Cvi …(14.24)
U = U1 + U2 + U3 + U4 + … + UK
= SUi …(14.17) m1 Cp1 + m2 C p2 + m3 C p3 + º mk C pk
and Cp =
or mu = m1 u1 + m2 u2 + m3 u3 + … + mk uk m1 + m2 + m3 + º + mk
= S mi ui Smi C pi
= = S xi Cpi …(14.25)
The specific internal energy of the mixture m
m1u1 + m2 u2 + m3u3 + º + mk uk On molal basis
u=
m1 + m2 + m3 + º + mk Cp = S yi C pi …(14.26)
Um
= = S xi ui …(14.18) The entropy of the gas mixture can be expressed
m as
On the molal basis S = S1 + S2 + S3 + … + Sk = SSi
U = n um = S ni ui = S mi si …(14.27)
1 The specific entropy of gas mixture
where u = S ni ui = Syi ui …(14.19)
n mi si + m2 s2 + m3 s3 + º mk sk
s=
where ui = internal energy per mole for ith m1 + m2 + m3 + º + mk
component of gas mixture. S mi si
= = S xi si …(14.28)
Similarly, the total enthalpy of a gas mixture m
H = H1 + H2 + H3 … + HK Further on molal basis S = Sni si
= S Hi …(14.20)
1
or mh = m1 h1 + m2 h2 + m3 h3 + … + mk hk and s = S ni si = S xi si …(14.29)
n
= S mi hi
Example 14.6 A mixture of ideal gases consists of
m h + m2 h2 + m3 h3 + º + mk hk 5 kg of nitrogen and 6 kg of carbon dioxide at a pressure
and h= 11
m1 + m2 + m3 + º + mk of 4 bar and a temperature of 27°C. Find
(a) The mole fraction of each constituent,
Smi hi
= = S xi hi …(14.21) (b) The equivalent molecular weight of the mixture,
m (c) The equivalent gas constant of the mixture,
On the molal basis (d) The partial pressures and partial volumes,
H = nhm = S ni hi (e) The volume and density of the mixture, and
(f) The Cp and Cv of the mixture.
1
and h = S ni hi = S yi hi …(14.22)
n
488 Thermal Engineering

If the mixture is heated at constant volume to 60°C, (iv) The partial pressures and partial volumes:
find the change in internal energy, enthalpy, and entropy pN2 = yN2 ¥ p = 0.570 ¥ 4 = 2.28 bar
of the mixture. pCO2 = yCO2 ¥ p = 0.435 ¥ 4 = 1.72 bar
Take g for CO2 = 1.286 and for N2 = 1.4 8.314
5¥ ¥ 300
mN 2 RN 2 T 28
VN2 = =
Solution p ( 4 ¥ 100 kPa )
Given A mixture of ideal gases = 1.113 m3
mN2 = 5 kg mCO2 = 6 kg mCO2 RCO2 T
VCO2 =
p = 4 bar T1 = 27°C + 273°C = 300 K p
V2 = V1 T2 = 60°C = 333 K 8.314
6¥ ¥ 300
g CO2 = 1.28 g N2 = 1.4 44
= = 0.85 m3
( 4 ¥ 100 kPa)
To find
(v) The volume and density of the gas mixture:
(i) Mole fraction of each constituent,
The total volume
(ii) The equivalent molecular weight of the mixture,
mRT1 (5 + 6) ¥ 0.237 ¥ 300
(iii) The equivalent gas constant of the mixture, V = =
p 400
(iv) The partial pressures and partial volumes,
(v) The volume and density of the mixture, = 1.954 m3
(vi) The Cp and Cv of the mixture, The density of the gas mixture:
(vii) Change in internal energy of mixture, m 11 kg
r = = = 5.63 kg/m3
(viii) Change of entropy of the mixture. V 1.954 m3
(vi) Cp and Cv of the mixture
Analysis Ru 8.314
(i) For calculation of number of moles, using relation CvN2 = =
MN 2 (g N 2 - 1) 28 ¥ (1.4 - 1)
mi = 0.742 kJ/kg ◊ K
ni =
Mi CpN2 = g N2 ¥ CvN2 = 1.4 ¥ 0.742
5 = 1.039 kJ/kg ◊ K
Thus n N2 = = 0.178
28 Ru
CvCO2 =
6 MCO2 (g CO2 - 1)
nCO2 = = 0.136
44 8.314
=
The total number of moles in the mixture: 44 ¥ (1.286 - 1)
n = nN2 + nCO2 = 0.178 + 0.136 = 0.6606 kJ/kg ◊ K
= 0.312 CpCO2 = gCO2 ¥ CvCO2 = 1.286 ¥ 0.6606
Therefore, mole fractions = 0.85 kJ/kg ◊ K
n N2 For mixture,
0.178
yN2 = = = 0.570 mN 2 Cp N 2 + mCO2 C p CO
n 0.312 Cp = 2

nCO 2 mN 2 + mCO2
0.136
yCO2 = = = 0.435
n 0.312 5 ¥ 1.039 + 6 ¥ 0.85
=
(ii) Apparent molecular weight of the mixture, M 5+6
M = yN2 MN2 + yCO2 MCO2 = 0.936 kJ/kg ◊ K
= 0.570 ¥ 28 + 0.435 ¥ 44 mN 2 Cv N 2 + mCO2 Cv CO2
Cv =
= 35. 1 kg/kg ◊ mole mN 2 + mCO2
(iii) Apparent gas constant of mixture, R: 5 ¥ 0.742 + 6 ¥ 0.6606
=
R 8.314 5+6
R = u = = 0.237 kJ/kg ◊ K = 0.697 kJ/kg ◊ K
M 35.1
Ideal Gas Mixtures 489

When mixture is heated at constant volume (ii) Each constituent of the gas mixture behaves as an
(vii) Change in internal energy of the mixture, ideal gas.
U2 – U1 = DU = mCv (DT) (iii) The kinetic and potential energy effects are
= 11 ¥ 0.697 ¥ (333 – 300) negligible.
= 253.22 kJ (iv) Polytropic expansion is internally reversible.
(viii) Change in enthalpy of the mixture, Analysis Using mi = ni Mi for conversion of volume
DH = H2 – H1 = mCp(DT) fraction to mass fraction for 1 mole of the mixture.
= 11 ¥ 0.936 ¥ (333 – 300)
= 339.76 kJ Constituents ni Mi mi = ni Mi xi = mi /m
(xi) Change in entropy of the mixture, = mass
fraction
ÊT ˆ Êv ˆ
DS = S2 – S1 = m Cv ln Á 2 ˜ + m R ln Á 2 ˜ CO2 0.13 44 5.72 0.187
Ë T1 ¯ Ë v1 ¯
O2 0.125 32 4.00 0.130
Ê 333 ˆ
= 11 ¥ 0.697 ln Á
Ë 300 ˜¯
+0 N2 0.745 28 20.86 0.682
= 0.80 kJ/K m = 30.58

The specific heat of the mixture


Example 14.7 In an engine cylinder, a gas has volu-
metric analysis of 13% CO2, 12.5% O2 and 74.5% N2. Smi C pi
Cp = = S xi Cpi
The temperature at the beginning of expansion is 1050°C m
and gas mixture expands reversibly through a volume ra- = 0.187 ¥ 1.235 + 0.131 ¥ 1.088 + 0.682 ¥ 1.172
tio 8: 1 according to the law pv1.2 = C. = 0.231 + 0.1425 + 0.799 = 1.1725 kJ/kg ◊ K
Calculate per kg of gas: The gas constant
(a) The work done Smi R1 R
(b) The heat flow R = = S xi Ri = Sxi u
m Mi
(c) The change of entropy per kg of mixture
8.314 8.314
The value of Cp for constituents CO2, O2 and N2 Thus R = 0.187 ¥ + 0.131 ¥
44 32
are 1.235 kJ/kg ◊ K, 1.088 kJ/kg ◊ K and 1.172 kJ/kg ◊ K
8.314
respectively. + 0.682 ¥
28
Solution = 0.0353 + 0.0340 + 0.2025
Given The volumetric analysis of a gas mixture: = 0.2718 kJ/kg ◊ K
CO2 = 13% O2 = 12.5% and Cv = Cp – R = 1.1725 – 0.2718
N2 = 74.5% = 0.90 kJ/kg ◊ K
T1 = 1050°C + 273°C = 1323 K The temperature after expansion
v2 1- n n -1
=8 pv1.2 = C T2 Ê v2 ˆ Ê v1 ˆ
v1 =Á ˜ =Á ˜
Cp,CO2 = 1.235 kJ/kg ◊ K Cp, O2 = 1.088 kJ/kg ◊ K, T1 Ë v1 ¯ Ë v2 ¯
Cp, N2 = 1.172 kJ/kg ◊ K 1.2 – 1
Ê 1ˆ
or T2 = ( 1323 K) ¥ Á ˜ = 872.85 K
To find Ë 8¯
(i) The work done,
(i) The work done by 1 kg of gas mixture,
(ii) The heat flow, and
R (T1 - T2 )
(iii) The change of entropy per kg of mixture. w =
n -1
Assumptions 0.2718 ¥ (1323 - 872.85)
=
(i) The gas mixture in the cylinder is analysed as 1.2 - 1
closed system at steady state. = 611.74 kJ/kg
490 Thermal Engineering

(ii) The heat flow per kg of gas mixture, mN 2 2.5


The change of internal energy nN2 = = = 0.0893
MN 2 28
Du = Cv (T2 – T1)
Total moles, n = nHe + nN2 = 0.25 + 0.0893 = 0.3393
= 0.90 ¥ (872.85 – 1323)
Mole fractions:
= – 405.13 kJ/kg.
The heat flow nN 2 0.0893
yN2 = = = 0.263
q = Du + w = – 405.13 + 611.74 n 0.3393
= 206.60 kJ/kg n 0.25
yHe = He = = 0.7368
(iii) The change of entropy per kg of mixture: n 0.3393
Ê T2 ˆ Ê v2 ˆ (i) Final partial pressures of constituents:
Ds = s2 – s1 = Cv ln Á ˜ + R ln Á ˜
Ë T1 ¯ Ë v1 ¯ (a) Final partial pressure of N2
p2,N2 = yN2 p2 = 0.263 ¥ (7 bar)
Ê 872.85 ˆ
= 0.90 ¥ ln Á + 0.2713 ¥ ln (8) = 1.84 bar
Ë 1325 ˜¯
(b) Final partial pressure of He
= – 0.3756 + 0.5652 = 0.1896 kJ/kg ◊ K p2,He = yHe p2 = 0.7368 ¥ (7 bar)
= 5.1576 bar
Example 14.8 A mixture consisting of 1 kg of He and
(ii) Final temperatures of mixture
2.5 kg of N2 at 25°C and 10 N/cm2, compressed in a re-
versible adiabatic process to 70 N/cm2. Calculate Total mass of mixture
m = mHe + mN2
(a) The final partial pressure of the constituents
= 1 + 2.5 = 3.5 kg
(b) The final temperature and the change in internal
energy of the mixture during the process. Specific heat of mixture
Molecular weight of He = 4 kg mN 2 Cp, N 2 + m He C p, He
Cp =
Cv for N2 = 0.743 kJ/kgK m
Cv for He = 3.14 kJ/kgK 2.5 ¥ 1.0 4+ 1 ¥ 5.2 3
=
Cp for N2 = 1.04 kJ/kgK 3.5
Cp for He = 5.23 kJ/kgK = 2.237 kJ/kg ◊ K
mN 2 Cv, N 2 + mHe Cv,He
Solution Cv =
m
Given A mixture of He and N2 2.5 ¥ 0.743 + 1 ¥ 3.14
mHe = 1 kg mN2 = 2.5 kg =
3.5
T1 = 25°C = 298 k, p1 = 10 N/cm2 = 1.0 bar = 1.4278 kJ/kg ◊ K
p2 = 70 N/cm2 = 7.0 bar, MHe =4 Ratio of two specific heats for mixture
Cv, N2 = 0.743 kJ/kg ◊ K, Cp, N2 = 1.04 kJ/kg ◊ K Cp 2.237
Cv, He = 3.14 kJ/kg ◊ K, Cp, He = 5.23 kJ/kg ◊ K g= = = 1.5667
Cv 1.4278
To find Temperature after isentropic compression process
(i) Final partial pressures of constituents, g –1
(ii) Final temperature of mixture, and Êp ˆ g
T2 = T1 Á 2 ˜
(iii) Change in internal energy of mixture during Ë p1 ¯
1.5667 - 9
process,
Ê 7ˆ 1.5667
= 298 ¥ Á ˜
Analysis The number of moles of constituents Ë 1¯
mHe 1 = 602.42 K or 329.42°C
nHe = = = 0.25
MHe 4
Ideal Gas Mixtures 491

(iii) Change in internal energy of mixture in internal energy of the system is zero.
DU = mCv (T2 – T1) DU = DUA + DUB = 0
= 3.5 ¥ 1.4278 ¥ (602.42 – 298) or DU = mA uA + mB uB = 0
= 1521.27 kJ For any ideal gas, the internal energy is the function
of absolute temperature and is expressed as
U = m Cv T
Thus, for a gas mixture before and after mixing
In the proceeding section, we have considered mCv T = mA CvA TA + mB CvB TB
the mixture of ideal gases that have already been m A Cv A TA + mB Cv B TB
or T =
formed, and we related the properties of mixture mCv
to the properties of the gas components. Now we Smi Cv i Tt
take up the process of mixing of ideal gases, that = …(14.30)
mCv
are initially separated. Such mixing of ideal gases
where Cv is the specific heat at constant volume of the
is an irreversible process since the gases can mix mixture. The change in internal energy DU can also be
spontaneously, but cannot separate to their initial calculated as
states, without external work input. In this section,
( nACv A + nBCvB) T = nA Cv ATA + nB Cv BTB
we relate the states of gas components before
mixing to the state of the mixture. nA Cv A TA + nB CvB TB
or T =
nA Cv A + nB CvB
S ni Cvi Ti
Consider the adiabatic mixing of two gases A and B = …(14.31)
S ni Cvi
at different pressures and temperatures in a closed
vessel by a thin diaphram as shown in Fig. 14.3(a). Applying the equation of state, the final pressure of
the mixture
The diaphram is ruptured, and the gases mix each
mRT
other and form a mixture as shown in Fig. 14.3(b). p = …(14.32)
The total volume and mass of the mixture are the V
sum of individual volumes and masses, receptively. The Where R is gas constant of mixture, m is mass of the
mixing process can be viewed as free expansion of each mixture. On molar basis, the pressure of mixture
gas and is thus irreversible. Since the mixing process is n RuT
p = …(14.33)
adiabatic and there is no work done, thus the internal V
energy of the system remains constant, and Since mixing is an irreversible process, thus the en-
U = UA + UB tropy must increase during the process. The increase in
But the internal energy of the each component is entropy of mixture equal to sum of increase in entropies
generally not same before and after mixing, but change
492 Thermal Engineering

of individual gas components, i.e., To find


DS = (S2 – S1)A + (S2 – S1)B > 0 …(14.34) (i) The pressure of oxygen after mixing,
where (S2 – S1)A (ii) Heat transferred to surroundings.
È ÊT ˆ Ê p A, 2 ˆ ˘ Analysis For the oxygen in vessel A
= mA ÍC pA ln Á ˜ - R ln Á ˜ ˙ …(14.35)
ÍÎ Ë TA ¯ Ë p A,1 ¯ ˙˚ pA VA = nA Ru TA
15 ¥ 102 ¥ VA = 0.5 ¥ 8.314 ¥ 323
where pA,1 is initial partial pressure of gas A, pA, 2 is the
final partial pressure of gas A in the mixture. VA = 0.895 m3
Similarly, The mass of oxygen in vessel A
(S2 – S1)B mA = nA MA
= 0.5 ¥ 32 = 16 kg
È Ê T ˆ Ê pB , 2 ˆ ˘
= mB ÍC pB ln Á ˜ - R ln Á ˜˙ …(14.36) The characteristic gas constant R for oxygen
ÍÎ Ë TB ¯ Ë pB ,1 ¯ ˙˚ Ru 8.314
R = = = 0.259 kJ/kg ◊ K
On molal basis, the change in entropy of ith M 32
component in the gas mixture; For vessel B.
DSi = (S2 – S1)i pB VB = mB R TB
È ÊTˆ Ê pi , 2 ˆ ˘ 6 ¥ 102 ¥ VB = 2.5 ¥ 0.259 ¥ 293
= ni ÍC pi ln Á ˜ - Ru ln Á ˜˙ …(14.37) VB = 0.316 m3
ÍÎ Ë Ti ¯ Ë pi ,1 ¯ ˙˚
Total volume of vessel A and B.
È ÊTˆ Ê V ˆ˘ V = VA + VB
or DSi = ni ÍCvi ln Á ˜ + Ru ln Á ˜ ˙ …(14.38)
ÍÎ Ë Ti ¯ Ë Vi ¯ ˙˚ = 0.895 + 0.316 = 1.211 m3
Total mass of oxygen after mixing
m = mA + mB =16 + 2.5 = 18.5 kg
Example 14.9 Two vessels A and B, both containing Final temperature after mixing = 27°C = 300 K
oxygen, are connected by a valve which open to allow
Final condition after mixing
the mixing of two streams. Before mixing the following
pV = m R T
properties are known
Using numerical values
Vessel A Vessel B p ¥ 1.211 = 18.5 ¥ 0.259 ¥ 300
pA = 15 bar pB = 6 bar or p = 1186.8 kPa ª 11.86 bar
TA = 50°C TB = 20°C Heat transferred to the surroundings
Contents = 0.5 kg mol Contents = 2.5 kg During mixing of two streams of oxygen
Temperature of oxygen after mixing reaches to 27°C W =0
Calculate final equilibrium pressure and amount of heat Q = U2 – U1
transferred to surroundings. Where
Take g = 1.4 U1 = Internal energy of two stream of O2
before mixing
Given Oxygen in vessels A and B
U2 = Internal energy of mixture
g = 1.4
we have U = m Cv T
R 0.259
Where Cv = = = 0.6475 kJ/kg ◊ K
g – 1 1.4 - 1
Then U1 = mA Cv TA + mB Cv TB
= 16 ¥ 0.6475 ¥ 323 + 2.5 ¥ 0.6475 ¥ 293
= 3820.57 kJ
and U2 = m Cv T = 18.5 ¥ 0.6475 ¥ 300 = 3593.62 kJ
Fig. 14.4
Ideal Gas Mixtures 493

Therefore, The internal energy after mixing


Q = 3593.62 – 3820.57 = – 226.94 kJ U2 = S (ni Cvi )T = T [(0.4035 ¥ 21.07)
+ (0.1253 ¥ 20.86)]
Example 14.10 A vessel of 1.5 m3 capacity contains = 11.118 T
oxygen at 7 bar and 40°C. The vessel is connected to an- For adiabatic mixing
other vessel of 3-m3 capacity containing carbon monox-
U2 = U1
ide at 1 bar and 15°C. A connecting valve is opened and
or 11.118T = 3413.8 kJ
the gases mix adiabatically. Calculate
or T = 307 K = 34°C
(a) Final temperature and pressure of the mixture
The pressure of the mixture is
(b) Change in entropy of the system
n RuT
For oxygen Cv = 21.07 kJ/kmol ◊ K, p =
V
for carbon monoxide Cv = 20.86 kJ/kmol ◊ K. (0.4035 + 0.1253) ¥ 8.314 ¥ 307
=
Solution (1.5 + 3)
= 300 kPa = 3 bar
Given Adiabatic mixing of two gases at constant total (ii) Change in entropy of the mixture:
volume. The initial state of two gases are shown in two DS = D SO2 + DSCO
separate boxes:
È Ê T ˆ Ê V ˆ˘
Oxygen Carbon monoxide DSO2 = nO2 ÍCvO ln Á ˜ + Ru ln Á ˜˙
ÍÎ 2 Ë TO2 ¯ Ë VO2 ¯ ˙˚
V = 1.5 m3 V = 3 m3
p = 7 bar p = 1 bar È Ê 307 ˆ Ê 4.5 ˆ ˘
= 0.4035 ¥ Í 21.07 ¥ ln Á + (8.314) ¥ ln Á
T = 40°C = 313 K T = 15°C = 288 K Î
˜
Ë 313 ¯ Ë 1.5 ˜¯ ˙˚
Cv = 21.07 kJ/kmol ◊ K Cv = 20.86 kJ/kmol ◊ K
= 3.52 kJ/K
To find È Ê T ˆ Ê V ˆ˘
DSCO = nCO ÍCvCO ln Á ˜ + Ru ln Á ˙
(i) The pressure and temperature of the mixture,
ÍÎ Ë TCO ¯ Ë VCO ˜¯ ˙˚
(ii) Change in entropy of the system.
È Ê 307 ˆ Ê 4.5 ˆ ˘
= 0.1253 ¥ Í 20.86 ¥ ln Á + 8.314 ¥ ln Á
Assumptions MO2 = 32, MCO = 28
Î
˜
Ë 288 ¯ Ë 3 ˜¯ ˙˚
Analysis = 0.589 kJ/K
(i) The number of moles of each gas constituent are and DS = 3.52 + 0.589 = 4.109 kJ/K
Ê pV ˆ
nO2 = Á
Ë RuT ˜¯ O
2

(7 ¥ 100 kPa ) ¥ (1.5 m3 )


= Another form of adiabatic mixing is the mixing of
(8.314 kJ / kmol ◊ K ) ¥ (313 K )
two streams of gases and to form a common gas
= 0.4035 kmol
(1 ¥ 100 kPa ) ¥ (3 m3 )
stream in steady state. It is shown schematically in
and nCO = Fig. 14.5. In absence of any changes in kinetic and
(8.314 kJ / kmol ◊ K ) ¥ ( 288 K )
potential energy the steady flow energy equation
= 0.1253 kmol
yields to
The total internal energy of two gases before
mixing m = m A + mB
U1 = Sni Cvi Ti and Q – W = m h – ( m A hA + mB hB)
= (0.4035 ¥ 21.07 ¥ 313) For adiabatic mixing, Q = 0
+ (0.1253 ¥ 20.86 ¥ 288) No work transfer, thus W = 0
= 3413.8 kJ Therefore, mh = m A hA + mB hB
494 Thermal Engineering

as
Using h = CpT, Cp,air = 1.02 kJ/kg ◊ K, Cp,CH4 = 2.37 kJ/kg ◊ K
hence mCpT = m A CpA TA + mB CpB TB
(iii) Dry air is treated as pure substance.
For more number of gas
mCpT = S mi Cpi Ti Analysis

S mi C pi Ti Mass flow rate of air mair = 17 mmethane


or T= …(14.39) = 17 ¥ 1.08 = 18.36 kg/s
mC p
Specific heat of mixture
On the molal basis
S mi C pi 1.08 ¥ 2.37 + 18.36 ¥ 1.02
S ni C pi Ti C = =
T= …(14.40) m 19.44
S ni C pi = 1.095 kJ/kg K
For change of entropy during mixing, the Eqs. (i) Applying the mass and energy balance on the
(14.35) and (14.37) may be used, by replacing pi, 2 control volume.
by mixture pressure p. m = mmethane + mair
= 1.08 + 18.36 = 19.44 kg/s
Example 14.11 Methane at 100 kPa, 15°C enters
an insulated mixing chamber, at a rate of 1.08 kg/s. It and mair hair + mCH 4 hCH4 = mh
is mixed adiabatically with air at 100 kPa, 160°C in an or 18.36 ¥ 1.02 ¥ 433 + 1.08 ¥ 2.37 ¥ 288
air/methane mass ratio of 17.0. The flow is steady and = 19.44 ¥ 1.095 T
kinetic and potential energy changes are negligible. The or T = 415.56 K = 142.56°C
ambient conditions are 100 kPa, 15°C. Determine (a) the (ii) Irreversibility during the mixing process is given
temperature of the mixture leaving the chamber, and (b) by
the irreversibility of the mixing per kg of methane. I = T0 DS
= T0 [ mCH 4 DsCH4 + mair Dsair]
Solution
For 1 kg of the methane gas
Given The adiabatic mixing of air and methane in
proportion of 17.0 È m ˘
i = T0 Í D sCH 4 + air D sair ˙
To find ÍÎ mCH 4 ˙˚
(i) Temperature of the mixture leaving the chamber. For each component gas, Ds can be obtained as
(ii) Irreversibility of mixing per kg of methane. ÊTˆ Ê pi,1 ˆ
Dsi = Cp ln Á ˜ – R ln Á
Assumptions Ë i¯
T Ë p ˜¯
(i) The mixing chamber as a control volume. Where p is the pressure of gas mixture, and pi, 1 is
(ii) The entering gases and exit mixture are regarded the partial pressure of gas in the mixture
as ideal gases with mean values of specific heats
Ideal Gas Mixtures 495

The number of moles for given mass can be


calculated as
m
ni = i
Mi
m CH 4 1.08 kg
nCH4 = = = 0.0675
M CH 4 16
mair 18.36
nair = = = 0.633
M air 28.97
Total moles, n = 0.0675 + 0.633 = 0.701 kmol
The mole fraction of methane
nCH4 0.0675
yCH4 = = = 0.0962
n 0.701
and yair = 1 – xCH4 = 0.9037
p i,1 Analysis
Then = yi
p The specific volume of air at state 1 is calculated as
Ê 415.56 ˆ Ê 8.134 ˆ RT1
DsCH4 = 2.37 ¥ ln Á – Á
Ë 288 ˜¯ Ë 16 ˜¯ v1 =
p1
¥ ln(0.0962) = 2.085 kJ/kg ◊ K
0.287 ¥ 305
Ê 415.56 ˆ Ê 8.134 ˆ = = 0.875 m3/kg
Dsair = 1.02 ¥ ln Á – Á 100
Ë 433 ˜¯ Ë 28.97 ˜¯
(i) The mass flow rate of dry air
¥ ln(0.9037) = – 0.012 kJ/kg
V1
mair =
and i = 288 ¥ ÈÎ 2.085 + 17 ¥ ( - 0.012)˘˚ v1
100 m3 /min
= 537.45 kJ/kg or mair = = 114.25 kg/min
0.875 m3 /min
Example 14.12 At steady state, 100 m3/min of dry air The mass flow rate of oxygen can be obtained by
at 32°C and 1 bar is mixed adiabatically with a stream of mass and energy balance at steady state
oxygen (O2)at 127°C and 1 bar to form a mixed stream at mair hair + mO2 hO2 = ( mair hair + mO 2 hO2)mixture
47°C and 1 bar. The kinetic and potential energy effects
or mair hair (T1) + mO 2 hO2 (T2)
are negligible. Determine (a) mass flow rates of dry air
and oxygen in kg/min, (b) the mole of fraction of dry air = mair hair (T3) + mO2 hO2 (T3)
and oxygen in the existing mixture, and (c) time rate of [hair (T3 ) - hair (T1 )]
Thus mO2 = mair
entropy production, in kJ/K.min. hO 2 (T2 ) - hO2 (T3 )
The specific enthalpies at given temperature can
Solution be obtained from properties of ideal gases; Tables
Given Adiabatic mixing of dry air and oxygen as A-8 and A-9, thus
mO 2 = (114.25 kg/min)
To find
(320.29 - 305.22) ( kJ / kg)
(i) Mass flow rates of dry air and oxygen in kg/min. ¥
Ê 1 ˆ
(ii) Mole fractions of dry air and oxygen in mixture. ÁË 32 kg/ kmol ˜¯ (11711 - 9325) ( kJ / kmol))
(iii) Time rate of entropy production in kJ/K ◊ min.
= 23.09 kg/min
Assumptions
The moles of gas components can be expressed as
Rair = 0.287 kJ/kg ◊ K, m
n =
RO2 = 0.259 kJ/kg ◊ K, Cp,air = 1.005 kJ/kg ◊ K, M
Cp,O2 = 0.928 kJ/kg ◊ K
496 Thermal Engineering

mair 114.25 (2) use of Amagat’s law of partial volumes


nair = =
Mair 28.97 in conjunction with equation of state of
= 3.94 kmol/min components gases
mO 2 23.09
(3) use of component compressibility factors.
nO2 = =
MO 32
2
= 0.721 kmol/min
Total number of moles. Dalton’s law of partial pressures can be used with
n = nair + nO2 = 3.94 + 0.721 reasonable accuracy in some range of pressure and
= 4.66 kmol/min temperature. However, the component pressures
(ii) The mole fraction should be evaluated from equation of state for
n 3.94 real gases such as van der Waals equation, Beattie
air yair = air = = 0.845
n 4.66 Bridgeman equation etc. we write
nO 0.721 i=k
oxygen yO2 = 2 =
n 4.66
= 0.155 p = p1 + p2 + p3 + … = Âp i
(iii) The entropy production during mixing is equal to i =1

rate of entropy increase during mixing where p1, p2 … denote the pressures that would
Sgen = DSair + DSO2 exert by individual component gas, if they would
be existed alone at temperature and volume of
È ÊTˆ Ê pi , 1 ˆ ˘
DSair = mair ÍC p ln Á ˜ - R ln Á ˙ mixture. By selecting suitable equation of state,
ÍÎ Ë T1 ¯ Ë p ˜¯ ˙˚ the pressure of each component gas in the mixture
= 114.25 can be calculated such as using Van der Waal’s
equation.
È Ê 320 ˆ ˘
¥ Í1.005 ¥ ln Á ˜¯ - 0.287 ¥ ln (0.845)˙ RiT a
Î Ë 305 ˚ pi = – 2i …(14.41)
vi – bi vi
= 11.034 kJ/k ◊ min where ai and bi are constants.
where,
pi, 1 = yair p = 0.845 bar
and pi, 2 = yO2 p = 0.115 bar
The Amagat’s law of additive volumes can also be
DSO2 = 23.09 used for gas mixture with considerable accuracy in
some ranges of pressure and temperature.
È Ê 320 ˆ ˘
¥ Í0.928 ¥ ln Á ˜ - 0.259 ¥ ln (0.115)˙ V = V1 + V2 + V3 + … = S Vi
Î Ë 400 ¯ ˚ Where V1, V2, V3 denote volumes of the individ-
= 8.153 kJ/min
ual components in the gas mixture, if they would be
Sgen = 11.034 + 8.153 = 19.19 kJ/K ◊ min
existed alone at pressure and temperature of mix-
ture.
In terms of molal volume.
V V V
When the components of a gas mixture deviate v = = 1 + 2 +…
n n1 n2
considerably from an ideal gas behaviour, then ni
the relations given above for ideal gas mixture Using yi = ,
n
cannot be used. However, the approximation can be yV y V
considered by we get v = 1 1 + 2 2 + …
n1 n2
(1) use of Dalton’s law of partial pressure
= y1 v1 + y2 v2 + … ...(14.42)
in conjunction with equation of state of
component gases
Ideal Gas Mixtures 497

By selecting a suitable equation of state, the the above equation reduces to law of additive vol-
molal specific volume of each component in gas umes.
mixture can be determined at pressure and tempera- The compressibility factor approach gives more
ture of mixture. accurate result when Zi’s are obtained by using
Amagat’s law of additive volumes. For a real gas
mixture
pV = Z n RuT …(14.44)
The compressibility factors for various components
The change in enthalpy and entropy of real gas
of gas mixture are available. Therefore, the value of
mixture can be calculated as
compressibility factor Z for real gas mixture can be
obtained as Dh = x1 Dh1 + x2 Dh2 + … = S xi Dhi
Z = y1 Z1 + y2 Z2 + y3 Z3 + … or Dh = yi D h1 + y2 D h2 + … = S yi Dhi
= S yi Zi …(14.43) and DS = x1 Ds1 + x2 Ds2 + … = S xi D si
If the compressibility factors Z1, Z2 etc. are eval- or D S = yi D s1 + y2 D s2 + … = S yi D si
uated at temperature and pressure of gas mixture,

Summary
the total pressure of mixture of ideal gases is
of its components: sum of pressures of its components gases, if each
i=k
existed alone at the temperature and volume of
m = m1 + m2 + m3 + … + mk = Âm
i =1
i the mixture.
Amagat–Leduce law states that the total volume
i=k of a mixture of ideal gases equals the sum of
n = n1 + n2 + n3 + n4 + … + nk = Ân
i =1
i
volumes of its components, if each existed alone
at the pressure and temperature of the mixture.

as from an ideal gas behaviour, then relation stated


m n above cannot be used. However, an approximation
xi = i and yi = i
m n is carried out by use of compressibility chart and
pi of ith competent in a gas viral expression, etc.
mixture is given as compressibility factor Z for a real gas mixture
ni RuT can be obtained as
pi = yi p =
V Z = y1 Z1 + y2 Z2 + y3 Z3 + … = S yi Zi
Vi of ith component in a gas where Z1, Z2, etc. are compressibility factor of
mixture real gas components evaluated at mixture pres-
ni RuT sure and temperature. The equation of state for
Vi = yiV =
p real gas mixture can be stated as
Dalton’s law of additive pressure states that pV = Z n RuT
498 Thermal Engineering

Glossary
Mass fraction Mass of a component divided by total Partial volume Volume of a component divided by
mass of mixture. total volume of mixture at same temperature and volume.
Mole fraction Mole of a component divided by total Apparent Molecular mass It is the ratio of mass and
moles of mixture. number of moles of a gas mixture.
Partial pressure Pressure of a component divided Apparent Gas Constant It is the ratio of the universal
by total pressure of mixture at same temperature and gas constant to apparent molecular mass of gas mixture
volume.

Review Questions
1. What are the mass fraction and mole faction? 4. What is an apparent gas constant for a gas mix-
2. Using definition of mass and mole fractions, ture? Can it be larger than the largest gas constant
derive a relation between them. in the mixture?
3. What is an apparent molecular weight for a gas 5. Express Dalton’s law of partial pressures. Does
mixture? Does the mass of every molecule in the this law hold exactly for ideal gas mixtures?
mixture equal to apparent molecular weight? 6. What is the difference between the total pressure
and the partial pressure?

Problems
1. A vessel of 0.35 m3 capacity contains 0.4 kg (b) 7.53%, (c) 31.872, 0.2608 kJ/kg ◊ K,
of carbon monoxide and 1 kg of air at 20°C. (d) 0.7641 m3]
Calculate: 3. A gaseous mixture, by its volumetric composi-
(a) Partial pressure of each constituents, tion, 78% H2, and 22% CO is contained in a ves-
(b) The total pressure in the vessel, and sel. It is desired that the mixture should be made
The gravitational analysis of air is to be taken as in proportion 52% H2 and 48% CO by remov-
23.3% oxygen and 76.7% nitrogen. ing some of the mixture and adding some CO.
[(a) 0.5068 bar, 1.9065 bar, 0.9943 bar, Calculate per mole of the mixture, the mass of
(b) 3.4076 bar] mixture to be removed, and mass of CO to be
added.
2. A vessel contains a mixture of 1 mole of CO2 and
4 moles of air at 1 bar and 20°C. Calculate for Assume that the pressure and temperature in the
mixture: vessel remains constant during the procedure.
(a) Mass of CO2, O2 and N2 and total mass, [2.57 kg to be removed,
(b) The percentage carbon content by mass, 9.32 kg CO to be added]
(c) The equivalent molecular weight and gas 4. A 0.825 m3 tank contains 0.95 kg oxygen, 0.82 kg
constant for the mixture, of nitrogen, 1.32 kg of carbon dioxide, and
0.091 kg of carbon monoxide at a temperature of
(d) The specific volume of mixture.
27°C. Calculate the total pressure of the mixture.
The volumetric analysis of air can be taken as
5. Carbon dioxide gas at 320 K is mixed with ni-
21% oxygen and 79% nitrogen.
trogen at 280 K in a steady flow apparatus, adia-
[(a) 44 kg, 26.88 kg, 88.48 kg, 159.36 kg,
Ideal Gas Mixtures 499

batically. Both gases flow at 100 kPa and have 11. 4 kg of carbon dioxide at 40°C and 1.4 bar is
the mole ratio of 2 : 1. Find the exit temperature mixed with 7 kg of nitrogen at 160°C and 1.0 bar
and total entropy generation per mole of the exit form a mixture at a final pressure 0.7 bar. The
mixture. process of mixing is adiabatic in a steady flow
Take Cp of CO2 = 0.85 and manner. Calculate
for N2; = 1.04 kJ/kg ◊ K. (a) final temperature of the mixture,
[308.7 K, 5.35 kJ/kmol ◊ K] (b) the change in entropy.
6. A gas mixture has volumetric analysis shows that Take Cp for CO2 = 0.85 kJ/kg ◊ K,
it consists of 20% carbon dioxide, 5% carbon and N2 = 1.04 kJ/kg ◊ K
monoxide and remaining is nitrogen. For mixture 12. A mixture consists of 60% nitrogen and 40% CO2
temperature and pressure of 45% C and 500 kPa. by mass. If the mixture is compressed from 2 bar,
Calculate (a) the partial pressures of the constitu- 5°C to 5 bar, 170°C, what is the entropy change
ents, (b) equivalent molecular weight, and (c) the of the mixture?
equivalent gas constant. 13. Two insulated tank A and B are connected by a
7. Flue gases consist of 81% nitrogen, 14% CO2, valve. Tank A has a volume of 1 m3 and initially
4.2% O2 and 0.8% CO on volumetric basis. contains Argon at 300 kPa, 10°C. Tank B has a
Calculate the density of flue gas mixture at 101.2 volume of 2 m3 and initially contains ethane at
kPa and 47°C. 200 kPa, 50°C. The valve is opened and remains
8. Air at 420 kPa, 80°C is introduced at the rate open, until the resulting gas mixture reaches to an
of 7.8 kg/min into a flowing air stream having a equilibrium state. Determine final pressure and
pressure of 102.2 kPa at 34°C. The pressure down temperature.
stream from the point of mixing is 101.8 kPa and Take Cp of argon = 0.520 kJ/kg ◊ K,
temperature is 42°C. Determine the mass flow ethane Cp = 1.766 kJ/kg ◊ K.
rate of air at 34°C. [242 kPa, 315.5 K]
9. A vessel of 1.8 m3 capacity contains oxygen at 14. A tank of capacity 0.45 m3 is insulated and di-
8 bar and 50°C. The vessel is connected with vided into two sections through a partition. One
another vessel of 3.6 m3 capacity containing car- section contains hydrogen at 3 bar and 130°C and
bon monoxide at 1 bar and 20°C. The connecting has volume of 0.3 m3 and other section contains
valve is opened and the gases mixed adiabatical- nitrogen at 6 bar and 30°C. The partition is then
ly. removed and gases are allowed to mix till they
Calculate reach an equilibrium state. Calculate:
(a) final temperature and pressure of the mix- (a) Temperature of mixture,
ture; (b) Pressure of mixture,
(b) the change of entropy of the system (c) The change in entropy for each component
Take CvO2 = 21.07 kJ/mol ◊ K and total values.
CvCO = 20.86 kJ/mol ◊ K. Assume
[(a) 437°C, 3.33 bar, (b) 5.411 kJ/K] Cp for N2 = 0.744 kJ/kg ◊ K,
10. A vessel of 6 m3 capacity contains two gases A Cv for H2 = 10.35 kJ/K
and B in proportion of 45% and 55%, respectively Cp (N2) = 1.041 kJ/kg ◊ K,
at 30°C. If the gas constant R for these gases is Cp (H2) = 14.47 kJ/kg
0.288 kJ/kg ◊ K and 0.295 kJ/kg ◊ K and total
[(a) 345.7 K, (b) 3.99 bar, (c) 0.00626 kJ/kg ◊ K,
weight of the mixture is 2 kg. Calculate
0.424 kJ/K, 0.43026 kJ/K]
(a) The partial pressure,
15. A mixture is made up of 25% N2, 35% O2, 20%
(b) The total pressure, and
CO2 and 20% CO by volume. Calculate:
(c) The mean value of R for the mixture.
(a) the molar mass of the mixture;
[(a) 0.13 bar, 0.164 bar, (b) 0.294 bar,
(b) C p and Cv for the mixture;
(c) 0.292 kJ/kg ◊ K]
500 Thermal Engineering

(c) g for the mixture; 17. A rigid insulated tank has two compartments:
(d) the partial pressure of each constituent Initially one contain 0.5 kmol of carbon diox-
when the total pressure is 1.5 bar; ide (CO2) at 27°C, 2 bar and the other contains
(e) the density of the mixture at 1.5 bar and 1 kmol of oxygen (O2) at 152°C, 5 bar. The gases
15°C. are allowed to mix while 500 kJ of energy are
[(a) 32.6 kg/kmol; (b) 30.84, 22.53 kJ/kmol ◊ K; added by electrical work. Determine
(c) 1.37; (d) 0.375, 0.525, 0.3, 0.3 bar; (a) the final temperature, in°C
(e) 2.04 kg/m3] (b) the final pressure, in bar
16. Two vessels are connected by a pipe in which (c) the change in energy, in kJ. for T0 = 20°C
there is a valve. One vessel of 0.3 m3 contains (d) the energy destruction, in kJ.
air at 7 bar and 32°C, and the other of 0.03 m3 18. Using the ideal gas model with constant specific
contains oxygen at 21 bar and 15°C. The valve heats, determine the mixture temperature, in K,
is opened and the two gases are allowed to mix. for each of two cases:
Assuming that the system is well insulated, (a) Initially, 0.6 kmol of O2 at 500 K is sepa-
calculate: rated by a partition from 0.4 kmol of H2 at
(a) the final temperature of the mixture; 300 K in a rigid insulated vessel. The parti-
(b) the final pressure of the mixture; tion is removed and the gases mix to obtain
(c) the partial pressure of each constituent; a final equilibrium state.
(d) the volumetric analysis of the mixture; (b) Oxygen (O2) at 500 K and a molar flow rate
(e) the values of Cp, Cv, R, M and g for the of 0.6 kmol/s enters an insulated control
mixture; volume operating at steady state and mixes
(f) the increase of entropy of the system per with H2 entering as a separate stream at
kilogram of mixture; 300 K and a molar flow rate of 0.4 kmol/s.
A single mixed stream exits. Kinetic and
(g) the change in internal energy and enthalpy
potential energy effects can be ignored.
of the mixture per kilogram if the vessel is
cooled to 10°C. 19. An insulated tank has two compartments con-
nected by a valve. Initially, one compartment
Assume that air consists only of oxygen and
contains 0.7 kg of CO2 at 500 K, 6.0 bar and the
nitrogen.
other contains 0.3 kg of N2 at 300 K, 6.0 bar. The
[(a) 27.9°C; (b) 8.27 bar; (c) 3.31, 4.96 bar;
valve is opened and the gases are allowed to mix
(d) 60% N2, 40% O2; (e) 0.987, 0.709 kJ/kg ◊ K;
until equilibrium is achieved. Determine
0.278 kJ/kg ◊ K; 29.91 kg/mol; 1.392;
(a) the final temperature, in K,
(f) 0.183 kJ/kg ◊ K; (g) 12.69, 17.67 kJ/kg]
(b) the final pressure, in bar,
(c) the amount of entropy produced, in kJ/K.

Objective Questions
1. In a mixture of gases, the mass fraction, xi is 2. In a mixture of gases, the mole fraction, yi in a
defined as mixture of
mi n mi n
(a) (b) i (a) (b) i
m n m n
mi ni mi n
(c) (d) (c) (d) i
n m n m
Ideal Gas Mixtures 501

3. The apparent molecular mass M of a gas mixture 7. Partial pressure of a gas in a gas mixture is defined
is given by as
m (a) pressure exerted by a gas at mixture volume
(a) (b) S xi Mi
n (b) pressure exerted by a gas at mixture
Ru temperature
(c) (d) all the above
R (c) pressure exerted by a gas at mixture volume
4. Total pressure of a gas mixture is given by and temperature
n RuT pi (d) none of the above
(a) (b)
V xi 8. Which of the following equations represents
(c) Spi (d) all the above specific internal energy of a gas mixture (xi =
5. Which one of the following is true statement mass fraction and yi = mole fraction)?
(a) m = nM (b) m = S xi ni (a) S xi ui (b) S yi ui
n (c) S xi Cvi (d) None of the above
(c) m = (d) none of the above
M 9. Which of the following equations represents spe-
6. According to Amagat–Leduce law, the partial cific enthalpy of a gas mixture (xi = mass fraction
volume of a gas in a gas mixture and yi = mole fraction)?
(a) volume occupied at partial pressure of gas (a) S xi hi (b) S yi hi
(b) volume occupied at total pressure of the gas (c) S xi C pi (d) none of the above
mixture 10. The specific heat of a gas mixture is given by
(c) volume occupied by a gas at mixture pres- S mi C pi
sure and temperature (a) S xi Cpi, (b)
m
(d) none of the above (c) Sni Cpi (d) all the above

10. (d) 9. (a)


8. (a) 7. (c) 6. (c) 5. (a) 4. (d) 3. (d) 2. (b) 1. (a)
Answers
502 Thermal Engineering

15
Psychrometry

Introduction
The Psychrometry is the study of the properties of moist air and is useful to engineers concerned with
heating, cooling, and ventilation of buildings, ships and aircrafts.
In most of air-conditioning applications, the atmospheric air is used at low temperatures (below 40°C).
The water vapour present in atmospheric air is at very low partial pressure and it behaves as a perfect gas.
Thus, the air-vapour mixture at low pressure and atmospheric temperature can be modeled as an ideal gas.
That is,
(i) the equation of state is pv = RT,
(ii) the enthalpy of the gas mixture is the function of temperature only, and
(iii) the vapour behaves in all respects as it existed alone at its partial pressure and temperature of the
mixture.

PSYCHROMETER wick. The temperature measured by a wick-covered


bulb is the temperature of the liquid water in the
The psychrometer is an equipment used to measure wick and thus is called the wet bulb temperature.
dry-bulb and wet-bulb temperatures in the labora-
tory. It is used in the sling method, and therefore,
the equipment is called a sling psychrometer as
shown in Fig. 15.1. It consists of a dry-bulb ther- 1. The dry air includes all components
mometer and a wet-bulb thermometer mounted of atmospheric air except water vapour (moisture).
side by side in a protective case that is attached to a Thus, the dry air is the mixture of oxygen, nitrogen,
handle by a swivel connection, so that the case can carbon dioxide, argon, etc. At low temperature
be rotated. The sling thermometer is rotated for one (cold conditions), its specific heat remains almost
minute in air and then readings are taken from the constant and is taken as 1.005 kJ/kg ◊ K.
thermometers. 2. The term moist air is the mixture of
The dry-bulb thermometer is directly exposed the dry air and water vapour, and each component
to the atmospheric air and measures the actual of the mixture behaves as an ideal gas at states
temperature of air. The bulb of a wet-bulb under considerations.
thermometer is covered with water soaked cotton
Psychrometry 503

the mixture. The corresponding partial pressure


of air becomes psat equal to saturation pressure at
temperature T and the partial pressure of air would
be pa = p – psat.

Some of the important psychrometric properties of


moist are defined below:

The dry-bulb temperature (DBT), designated


3. In atmospheric air, the mois- as T (or sometimes T db), of the mixture is the
ture appears in the form of superheated vapour as temperature measured by an ordinary thermometer
invisible gas at atmospheric pressure. The mixture placed in the air-vapour mixture.
of superheated vapour and dry air is called unsatu-
rated air.
The wet-bulb temperature (WBT), Twb, is the tem-
4. The saturated air is moist air perature measured by a thermometer whose bulb is
which contains maximum possible water vapour. covered by a thoroughly wetted cotton wick.
The partial pressure of water vapour in saturated
When air passes over the wetted cotton wick,
air is equal to the saturated pressure of steam cor-
some of the water evaporates, cooling effect is
responding to the temperature of moist air. Cooling
produced at the bulb and the temperature recorded
of saturated air causes the separation of moisture
is lower than that of the air stream.
and formation of fog.
Figure 15.3 shows the method of measurement
Figure 15.2 illustrates the states of saturated and of wet and dry bulb temperatures. The two ther-
unsaturated air. At the state 1, the partial pressure pv mometers are located in the stream of unsaturated
of water vapour is lower than the saturation pressure; air. As the air stream passes the wet cotton wick,
thus the water vapour is in superheated state, the some water evaporates and produces a cooling
air is unsaturated at the mixture temperature T and effect in the wick. The heat is transferred from air
total pressure p. At this state, partial pressure of air to wick with corresponding cooling. An equilibri-
is pa = p – pv. um condition is reached at which the wet bulb ther-
The state 2 is after isothermal cooling, when mometer indicates a temperature, that is lower than
saturated water vapour is at the temperature T of the dry bulb temperature.
504 Thermal Engineering

T
Moist air

pv

Tatm
1
Tdp
2
s

Fig. 15.4 Cooling of vapour at constant pressure


brings dew point temperature
Fig. 15.3
and dry-bulb temperature Dew-point Depression

Wet-bulb Depression The difference between the dry-bulb temperature


and dew-point temperature is called as dew-point
The difference between the dry-bulb temperature depression.
and wet-bulb temperature is called as wet-bulb Dew-point depression = T – Tdp ...(15.2)
depression.
Wet-bulb depression = T – Twb ...(15.1) Relative Humidity
The amount of wet-bulb depression depends on
It is defined as the ratio of actual mass of water
moisture content of air. For completely dry air, the
vapour to the mass of saturated vapour produced
wet-bulb depression is maximum, because dry air
in a mixture of air and water vapour at the same
is capable of absorbing maximum amount of mois-
temperature and pressure. It is denoted by f (phi).
ture. On the other hand, when air is saturated, it
Since water vapour behaves like an ideal gas,
cannot absorb moisture any more and the wet bulb
m p V /Rv T p
temperature reading equals the dry bulb tempera- thus f = v = v = v ...(15.3)
ture readings. It means, wet bulb depression ap- msat psat V /Rv T psat
proaches zero for saturated air. where pv =partial pressure of vapour in the
mixture
Dew-point Temperature psat =saturation pressure of vapour at
temperature of mixture
The dew-point temperature, Tdp, of an air–vapour
T = mixture temperature
mixture is a temperature at which the vapour starts
It can also be expressed as
to condense, when it is cooled at constant pres- rfg vg
p
sure. The partial condensation of water vapour on f = v = = ...(15.4)
psat rg vf g
window panes in winter, or on pipes carrying cold
water encounters dew most often. The formation of
dew on the grass is the best example. It occurs due
to cooling of water vapour at constant pressure to The specific humidity is also called humidity ratio
its saturation temperature. or absolute humidity. It is defined as ratio of mass
Figure 15.4 illustrates the dew point on a T–s of water vapour to the mass of dry air in a given
diagram. The air–vapour mixture at the state 1 is volume of the mixture. It is denoted by w (omega)
superheated. It is cooled at constant pressure pv to mv
w = ...(15.5)
the state 2, where the saturation temperature reach- ma
es and condensation of vapour begins. The temper- where the subscript a refers to dry air and v refers
ature at state 2 is dew point temperature. to vapour. Since both the vapour and mixture are
Psychrometry 505

considered as ideal gases, thus h of moist air per kg of dry air is expressed as
pv V p V Mv or ma h = ma ha + mv hg
mv = = v m
Rv T Ru T or h = ha + v hg
ma
pa V p V Ma
and ma = = a = (ha + w hg) kJ/kg of dry air ...(15.11)
Ra T RuT
where ha = enthalpy of dry air
pv Mv = Cp Tdb = 1.005 T
Therefore, w = ...(15.6)
pa Ma hg = enthalpy of water vapour calculated as
For an air–water vapour mixture(Mv = 18 and = 2500 + 1.88 T kJ/kg of water vapour
Ma = 28.97), it reduces to T = dry-bulb temperature in °C
pv pv
w = 0.622 = 0.622 ...(15.7)
pa p - pv
where p is the total pressure of the mixture ( pa +
pv), thus pa = p – pv. Figure 15.5 shows a closed system consisting of
atmospheric air, a mixture of water vapour and dry
air occupying volume V, at mixture pressure p and
It is defined as the ratio of actual humidity ratio mixture temperature T. The mixture is assumed
to the humidity ratio of saturated air at the same to follow ideal gas equation. Thus, the mixture
temperature and total pressure. It is designated as m pressure
and is expressed as n Ru T
p =
V
w
m = ...(15.8) m
w sat Introducing n = ;
M
pv m Ru T
Using w = 0.622 from Eq. (15.7), and at p = ...(15.12)
p - pv MV
saturation state where n, m, M and Ru denote the number of moles,
psat mass, molecular weight and universal gas constant,
wsat = 0.622
p - psat respectively.
pv ( p - psat ) p - psat Temperaure, T Pressure, P
then m= =f ...(15.9)
psat ( p - pv ) p - pv
mp – m pv = f ( p – psat)
Using pv = f psat from Eq. (15.3), then
or mp = f ( p – psat + m psat ) Moist Air
Mixture m, n
= f [ p – (1 – m) psat ] ª fp Dry air ma, na
Since (1 – m)psat ª 0 Vapour mv, nv

\ m =f ...(15.10)

Volume = V
The enthalpy of moist air is the sum of enthalpy of Boundary

dry air and that of water vapour. Thus, the enthalpy


506 Thermal Engineering

The partial pressure pa of dry air in the mixture; To find Humidity ratio
ma Ru T Analysis From steam tables, the saturation pressure of
pa = ...(15.13)
MaV water at 30°C
psat = 4.246 kPa
The partial pressure pv of water vapour in the
The partial pressure of water vapour in mixture
mixture
pv = fpsat
mv Ru T = 0.7 ¥ 4.246 = 2.97 kPa
pv = ...(15.14)
Mv V The partial pressure of air,
where ma, mv denote the mass of dry air and water pa = p – pv = 95 – 2.97 = 92.03 kPa
vapour, respectively and Ma and Mv are respective The specific humidity by using Eq. (15.7)
molecular weights in the mixture. pv
w = 0.622
The total pressure of the mixture is the sum of pa
partial pressure of water vapour and dry air present 2.97
in the mixture. = 0.622 ¥
92.03
p = p a + pv ...(15.15) = 0.02007 kg/kg/of dry air
The saturation pressure of water corresponding
to dew point can be obtained from steam tables, Example 15.2 A sample of 450 gram of moist air at
which is equal to the partial pressure of water 22°C, 101 kPa and 70% relative humidity is cooled to
vapour in air. But it is difficult to measure the dew- 5°C, while keeping the pressure constant. Determine (a)
point temperature accurately. the initial humidty ratio (b) dew point temperature, and
(c) amount of water vapour that condenses.
The wet-bulb temperature of the mixture is
easily measured with a wetted wick thermometer. Solution
The partial pressure pv of vapour in the mixture
can be calculated by using Dr Willis H Carrier’s Given A sample of moist air is cooled at constant
pressure
equation
m = 0.45 kg T1 = 22°C = 295 K
( p - pv¢ ) (T - Twb ) ¥ 1.8
pv = pv¢ - ...(15.16) p = 101 kPa p = constant
2800 - 1.3 ¥ (1.8 T + 32) f = 70% T2 = 5°C
where pv = partial pressure of water vapour at dry To find
bulb temperatutre in mixture, (i) Initial humidity ratio,
pv¢ = saturation water pressure at wet bulb (ii) Dew point temperature, and
temperature, (iii) Amount of water vapour that condenses.
p = total pressure of moist air,
Schematic with given data
T = dry-bulb temperature,
Twb = wet-bulb temperature.

Atmospheric air at 95 kPa, 30°C has


a relative humidity of 70 per cent. Determine humidity
ratio.

Solution
Given Atmospheric air
p = patm = 95 kPa T = 30°C = 303 K
f = 0.7
Psychrometry 507

Assumptions The amount of water vapour condensed


(i) 1 kg of moist air sample as a closed system. mw = mv1 – mv2 = 0.0109 – 0.00535
(ii) The gas phase of air and vapour can be treated as = 0.0055 kg/kg of dry air
an ideal gas mixture. In the sample of 0.450 kg
mw = 0.45 ¥ 0.0055 = 0.0025 kg
Analysis From steam tables at 22°C;
psat1 = psat @ 22°C = 0.02645 bar Example 15.3 An air–water vapour mixture is con-
The partial pressure of water vapour pv1 tained in a rigid closed vessel with a volume of 35 m3 at
pv1 = f psat1 1.5 bar, 120°C, and f = 10%. The mixture is cooled at
= 0.7 ¥ 0.02645 = 0.0185 bar = 1.85 kPa constant volume with its temperature decrease to 22°C.
(i) The humidity ratio at initial state Determine (a) the dew point temperature corresponding
pv1 to initial state, (b) the temperature at which the condensa-
w1 = 0.622 tion actually begins, and (c) amount of water condensed.
p - pv1
1.85 Solution
= 0.622 ¥
101 - 1.85
Given An air–vapour mixture in a closed vessel,
= 0.011 kg/kg of dry air
V = 35 m3 p1 = 1.5 bar
(ii) Dew-point temperature is the saturation tem-
T1 = 120°C = 393 K f1 = 0.1
perature corresponding to partial pressure pv1.
Interpolating between 16°C and 17°C, we get V1 = V2 T2 = 22°C
Tdp = 16.267°C
(iii) The amount of condensate,
mw = Initial amount of water vapour
– Final amount of water vapour
= mv1 – mv2
For 1 kg of dry air and moisture,
1 kg = ma + mv1
mv1
Using w1 = ;
ma
m Ê1 ˆ
1 kg = v1 + mv1 = mv1 Á + 1˜
w 1 Ëw ¯ To find
w1 (i) Dew point temperature,
or mv1 =
w1 + 1 (ii) The temperature, at which condensation starts,
0.011 (iii) Amount of water condensed.
=
0.011 + 1 Assumptions
= 0.0109 kg/kg of dry air
(i) The contents of vessel are taken as closed system.
The mass of dry air/kg,
(ii) The gas phase of air and vapour can be treated as
ma = 1 – 0.0109 = 0.9891 kg
ideal gases.
Further, humidity ratio w 2 at state 3, i.e., at 5°C
(iii) When liquid water is present, the vapour exists at
psat = 0.00872 bar = 0.872 kPa
saturation temperature.
0.622 ¥ 0.872
w2 = Analysis
101 - 0.872
(i) The partial pressure of vapour at the state 1;
= 0.0054 kg/kg of dry air
pv1 = f1 psat1
The mass of water vapour
psat1 = psat @ 120°C = 1.985 bar
mv2 = w 2 ma = 0.0054 ¥ 0.9891
Thus pv1 = 0.1 ¥ 1.985 = 0.1985 bar
= 0.00535 kg/kg of dry air
508 Thermal Engineering

The saturation temperature corresponds to results from adding water vapour adiabatically to
0.1985 bar the atmospheric air in a steady flow manner until it
Tsat = 60°C becomes completely saturated.
It is the dew point temperature A steady stream of unsaturated air with specific
(ii) The specific volume of vapour in the mixture humidity w1 and temperature T1 is passed through
Ru T an insulated duct containing a pool of water as
vv1 =
Mv1 pv1 show in Fig. 15.8(a). As air flows over the water,
Ê 8.314 ˆ Ê 393 K ˆ some water evaporates and its vapour mixes with
= Á ˜ ¥Á
Ë 18 ¯ Ë 0.1985 ¥ 100 kPa ˜¯ the air stream. The moisture content of air increases
and its temperature decreases. If the duct is long
= 9.145 m3/kg
enough, the air stream will come out completely
Interpolating, vv1 = vg, gives T = 56°C
saturated (f = 100%) at the temperature T2 which is
Thus the condensation will begin at 56°C.
called adiabatic saturation temperature.
(iii) The mass of water initially present in moist air
If make up water is supplied to the duct at
V 35 the rate at which it evaporates then the above
mv1 = = = 3.827 kg
vv1 9.145 process can be treated as a steady flow process.
The propetrties of steam at T = 56°C at the state 2, Figure 15.8(b) illustrates the adiabatic saturation
vf 2 = 1.0022 ¥ 10–3 m3/kg, process on the T–s diagram.
vg 2 = 15.445 m3/kg
The vapour has a two-phase mixture having Moist air Saturated air
w1, f1,T1 w2, f2 = 100%, T2
specific volume of 9.145 m3/kg
vv1 = vf 2 + x2 (vg2 – vf 2) . .
ma ma
9.145 = 1.0022 ¥ 10–3 + x2 (15.445 .
mv
.
mv
1 1 Liquid water 2
– 1.0022 ¥ 10–3)
or x2 = 0.178
insulation
The mass of water vapour at state 2
mv2 = x2 mv1 = 0.178 ¥ 3.827 = 0.681 kg
Make up water
The mass of water vapour condensed saturated liquid at T2
. .
m w = mv1 – mv2 = 3.827 – 0.681 mass flow rate = mv – mv
1
= 3.146 kg (a) Adiabatic saturator
T

Adiabatic
saturation
temperature
In atmospheric air, the relative humidity is always 1
less than 100%. The water-vapour pressure is lower
T2 2
than the saturation pressure. If this air is exposed
Tdp Dew-point
to liquid water, some liquid evaporates and mixes temperature
with the air. The specific humidity of air increases.
If such a process is carried in an insulated duct then
the air temperature will also decrease due to latent
s
heat of absorption during evaporation of water. (b) Process representation of T–s diagram
The adiabatic saturation temperature of atmo-
spheric air is defined as the temperature which
Psychrometry 509

At steady state, the mass-flow rate of dry Analysis


air entering the saturator ma must be equal to the (i) Partial pressure of water vapour
mass-flow rate of dry air leaving the saturator. The The specific humidity is expressed as
mass flow rate of make up water is the difference pv
between the exiting and entering vapour flow rates w = 0.622
p - pv
denoted by mv2 - mv1, respectively. These flow
using numerical values
rates are labelled on Fig. 15.8(a). In absence of
kinetic and potential energy changes, the steady- pv
0.016 = 0.622 ¥
flow energy equation reduces to 100 - pv
Enthalpy of mixture in + enthalpy of liquid or 100 – pv = 38.875 pv
water added = Enthalpy of mixture out. or pv = 2.5 kPa
ma ha1 + mv1 hg1 + ( mv2 - mv1 ) hf 2 (ii) Relative humidity f;
= ma ha2 + mv2 hg2 …(15.17) From steam table, the saturation pressure psat cor-
responding to dry-bulb temperature of 28°C;
Dividing both sides by ma ,
psat = 0.03778 bar = 3.778 kPa
ha1 + w 1 hg1 + (w 2 – w1) hf 2 = ha2 + w 2 hg2
pv 2.5
or w 1 (hg1 – hf 2) = ha2 – ha1 f = = = 0.661
psat 3.778
+ w 2 (hg2 – hf 2)
For ideal gas behaviour = 66.1%
(iii) Dew-point temperature
w 1 (hg1 – hf 2) = Cpa (T2 – T1) + w 2 hfg2
The dew-point temperature is the saturation tem-
C pa (T2 - T1 ) + w 2 hf g2 perature corresponding to partial pressure of va-
or w1 = ...(15.18)
hg1 - h f 2 pour. Thus, saturation temperature corresponds to
2.5 kPa = 21.1°C
psat2
where w 2 = 0.622 (iv) Specific enthalpy
p2 - psat2 h = ha + whg
and hfg2 = hg2 – hf 2 where ha = Cp Tdb = 1.005 ¥ (28°C)
The temperature of completely saturated air is = 28.14 kJ/kg.
called wet bulb temperature (wbt). hg = 2500 + 1.88 Tdb = 2500 + 1.88 ¥ 28
= 2552.64 kJ/kg
Example 15.4 The air at 28°C and 1 bar has a spe-
h = 28.14 + 0.016 ¥ 2552.64
cific humidity of 0.016 kg per kg of dry air. Determine (a)
= 6898 kJ/kg
partial pressure of water vapour, (b) relative humidity,
(c) dew point temperature, and (d) specific enthalpy. Example 15.5 The pressure of the air entering and
leaving the adiabaltic saturator is 1 bar. The air enters
Solution The moist air with
at 30°C and leaves as saturated air at 20°C. Calculate
p = 1 bar = 100 kPa
the specific humidity, relative humidity of the air–vapour
T = 28°C = 301 K mixture entering.
w = 0.016 kg/kg of dry air
Solution
To find
(i) Partial pressure of water vapour, pv1, Given Adiabatic saturator with
(ii) Relative humidity f, p1 = p2 = 1 bar = 100 kPa
(iii) Dew point temperature, Tdp, T1 = 30°C
(iv) Specific enthalpy. T2 = 20°C
f2 = 100%
510 Thermal Engineering

To find At entrance of adiabatic saturator


(i) Specific humidity, and
(ii) Relative humidity. The psychrometric chart is a graphical presentation
of properties of air–water vapour mixture. These
Assumptions
charts are available in a number of different forms
(i) Ideal gas mixture;
but the most commonly used chart is the w–DBT
(ii) Constant specific heat of air; as Cpa = 1.0035
chart. The typical layout of the chart is shown in
kJ/kg ◊ K.
Fig. 15.9. It has the following features:
Analysis Properties of steam at 20°C (Table A-12)
1. The chart is based on the standard atmo-
At 30°C hg1 = 2556.3 kJ/kg (From Table A-12)
spheric pressure, i.e., 760 mm of Hg or
psat1 = 4.246 kPa
101.325 kPa.
psat2 = 2.339 kPa
2. The chart is a plot of humidity ratio and va-
hfg2 = 2454.1 kJ/kg
pour pressure, on the vertical axis and dry-
hf2 = 83.96 kJ/kg
bulb temperature on the horizontal axis.
(i) Since the water vapour leaving the saturator is
3. On the left end of the chart, the saturation
completely saturated (f2 = 100%), thus
curve represents the state of saturated air at
pv2 = psat2
different temperatures.
The specific humidity of air leaving the saturator
4. Saturation curve is a curve of 100% relative
pv2
w 2 = 0.622 humidity. Other constant relative humidity
p - pv2 curves also have the same shape.
2.339 5. The constant wet-bulb temperature lines
= 0.622 ¥
100 - 2.339 have a downhill appearance to the right.
= 0.0149 kg/kg of dry air 6. The constant specific-volume lines are
The specific humidity of air entering is calculated steeper than constant wet-bulb temperature
as lines.
C pa (T2 - T1 ) + w 2 h fg2 7. The constant-enthalpy lines are parallel to
w1 =
( hg1 - h f 2 ) the constant wet-bulb temperature lines.
Using these values, Therefore, constant enthalpy and constant
1.0035 ¥ ( 20 - 30) + 0.0149 ¥ 2454.1
w1 =
2556.3 - 83.96
= 0.0107 kg/kg of dry air
(ii) The specific humidity of air entering can also be
expressed as
pv1
w1 = 0.622
p - pv1
pv1
fi 0.0107 = 0.622 ¥
100 - pv1
or 100 – pv1 = 58.13 pv1
or pv1 = 1.69 kPa
pv1 1.69
Thus f1 = = = 0.398 = 39.8%
psat1 4.246
Psychrometry 511

wet bulb temperature are represented on the Example 15.7 For the moist air at 30°C DBT and
same line. 15°C WBT, calculate
8. As shown in Fig. 15.10, on the saturation (a) Specific humidity,
line, the dry-bulb temperature T, wet-bulb (b) Enthalpy,
temperature Twb and dew-point temperature (c) Relative humidity,
Tdp are identical. (d) Dew-point temperature.

Solution
Given The moist air
T = 30°C Twb = 15°C
15°C Tdp = 15°C
To find
e
urv w (i) Specific humidity (w),
nc
Tw

io
rat
T = 15°C

tu (ii) Enthalpy (h),


b
=

Sa
15

(iii) Relative humidity (f),


°C

(iv) DPT (Tdp).


T
Analysis From psychrometric chart at intersection
Fig. 15.10 point of T = 30°C and Twb = 15°C
(i) Specific humidity w = 0.00422 kg of dry air
(ii) Enthalpy h = 42.2 kJ/kg of dry air
The psychrometric chart is used extensively in
(iii) Relative humidity f = 17.5%
air-conditioning applications. An ordinary cool-
(iv) Dew point temperature Tdp = 2.3°C
ing or heating process of air without change in its
moisture content (w = constant) is represented by Example 15.8 Air at 40°C DBT and 25°C WBT is
the horizontal line. Any deviation from the horizon- cooled down in an air-conditioning plant to 25°C DBT
tal line indicates that the moisture content of air is and 60% RH.
changed during the process. Calculate the heat to be removed per kg of air if the
COP of the unit is 3.5. Also, find the work required to
Example 15.6 The ambient conditions of air are cool 3 kg of air.
40°C DBT and 20°C WBT. Find the
(a) Relative humidity, Solution
(b) Specific humidity. Given Moist air
State 1: 40°C DBT and 25°C WBT
Solution
State 2: 25°C DBT and f = 0.6
Given Ambient air with System COP = 3.5 and m = 3 kg
T = 40°C and Twb = 20°C
To find
To find (i) Heat removed per kg of air, and
(i) Relative humidity, and (ii) Work input to system to cool 3 kg of air.
(ii) Specific humidity.
Analysis
Analysis From the psychrometric chart at intersection (i) From the psychrometric chart,
point of 40°C DBT and 20°C WBT,
At 40°C DBT and 25°C WBT, enthalpy of air;
Relative humidity = 14.5%
h1 = 76.5 kJ/kg of dry air
Specific humidity = 0.0066 kg/kg of dry air
512 Thermal Engineering

At 25°C DBT and 60% RH, enthalpy of air; keeping specific humidity w constant. Therefore,
h2 = 55.2 kJ/kg of dry air simple heating and cooling processes appear as a
Heat removed per kg of dry air horizontal line on the psychrometric chart as shown
qL = h1 – h2 in Fig. 15.12. The process 1–2 represents a sensible
= 76.5 – 55.2 = 21.3 kJ/kg heating process while the process 2–1 represents a
(ii) The coefficient of performance of a refrigerator is sensible cooling process.
given by
RE q By passed Conditioned
Tc
(COP)R = = L Moist air
T2 air
Win win w2 = w1
w1, T1, f1
f 2 < f1
qL 21.3
or win = = = 6.085 kJ/kg Cooling
(COP ) R 3.5 or
heating
Work input for 3 kg of air coil
Win = m win = 3 ¥ 6.085 (a) Schematic
= 18.26 kJ
h2

AIR-CONDITIONING PROCESS h1

The basic thermodynamic processes of moist 1 c 2 w


air are illustrated on the psychrometric chart in
Fig. 15.11. These processes are
1. Sensible heating and cooling T1 T2
2. Humidification and dehumidification DBT
3. Humidification with heating/cooling (b) Representation on psychrometric chart
4. Dehumidification with heating/cooling Fig. 15.12
5. Mixing of two air streams.
The sensible heat-transfer rate is given by
Q = ma (h2 – h1) = ma Cp (T2 – T1)
Humidification

= ma C pa (T2 - T1) + ma w C pv (T2 - T1)


ca nd
n
ifi a
tio
id ing

or Q = ma (1.005 + 1.88 w) (T2 – T1) ...(15.19)


um at
Dehumidification H He

Cooling ma = mass flow rate of air, kg/s


Heating w
Where Cpa = specific heat of air, 1.005 kJ/kg ◊ K
ca d
n

Cpv = specific heat of vapour, 1.88 kJ/kg ◊ K


id an
tio
m g
hu olin
ifi
de Co

By-Pass Factor of Heating and Cooling Coils


As shown in Fig. 15.12(a), the air flows over the
DBT
coil, and some part of the air directly passes the duct
Fig. 15.11 Basic air-conditioning processes
without coming in contact of cooling or heating
coil. Such a stream is referred as the by-passed
stream. Therefore, the temperature of air coming
The sensible heating or sensible cooling process in- out of the duct is not equal to the coil temperature.
volves only change in dry bulb temperature, while Thus, the by-pass factor (BPF) is defined as
Psychrometry 513

w1 = w2
Temperature difference between
coil and exit air p1 = p2 = 1 bar
BPF =
Temperature differrence between To find
coil and entering air (i) The rate of heat transfer, and
(ii) Relative humidity at the exit.
Tc - Texit T - T2
= = c ...(15.20) Schematic
Tc - Tinlet Tc - T1
where Tc is the temperature of the coil, and T1 and
T2 are inlet and exit temperatures of air.

The efficiency of heating and cooling coils is the ra-


tio between actual temperature change to the maxi-
mum possible temperature change.
For heating or cooling coil
Actual temperature change
hh =
Maximum possible temperature change
T2 - T1
= ...(15.21)
Tc - T1
T2 - T1 - Tc + Tc
=
Tc - T1
(Tc - T1 ) - (Tc - T2 ) Assumption Specific gas constant for dry air as
=
Tc - T1 R = 0.287 kJ/kg ◊ K

(Tc - T2 ) Analysis
or hh = 1 - = 1 – BPF ...(15.22)
Tc - T1 (i) Heat transfer rate
From psychrometric chart
Example 15.9 Moist air enters a duct at 10 °C,
State 1: 10°C DBT and f = 80%,
80% relative humidty, and a volumetric flow rate of 150
m3/min. The mixture is heated as it flows through the duct h1 = 25.7 kJ/kg
and leaves at 30°C. No moisture is added or removed and State 2: w1= w2 and 30°C DBT,
the mixture pressure remains approximately constant at h2 = 45.9 kJ/kg
1 bar. For steady-state operation, determine (a) the rate The heat transfer per kg of dry air
of heat transfer, and (b) relative humidity at exit. q = h2 – h1
Changes in kinetic and potential energy can be = 45.9 – 25.7 = 20.2 kJ/kg
ignored and use a psychrometric chart for the solution. The mass-flow rate of moist air can be obtained
by using perfect gas relation
Solution pV
m = 1 1
Given Moist air enters a duct and is heated RT1
At inlet: T1 = Tdb1 = 10°C = 283 K (100 kPa ) ¥ (150 m3/min)
=
f1 = 80% = 0.8 (0.287 kJ/kg ◊ K) ( 283 K )
3
V = 150 m /min = 185 kg/min
At exit: T2 = Tdb2 = 30°C = 303 K
514 Thermal Engineering

Then the total heat transfer rate to air in the duct Mass of moist air
Q = mq V 300 m3
m = = = 339 kg
= (185 kg/min) ¥ (20.2 kJ/kg) v g 0.885 m3/kg
= 3737 kJ/min
The heat transfer to air
(ii) Relative humidity at exit
Q = m (h2 – h1)
From psychrometric chart, at the state 2, 30° = 339 ¥ (86.4 – 76.5) = 3356 kJ
DBT, w 2 = w 1
f 2 = 23.1% Example 15.11 Air is cooled from 39°C DBT and
29% RH to 24°C at the rate of 5 m3/s. Calculate the
Example 15.10 A quantity of air having a volume of capacity of the cooling coil if the surface of the cooling
300 m3 at 30 °C DBT and 25°C WBT is heated to 40 °C coil is 20 °C. Also, calculate the by pass factor.
DBT. Calculate the amount of heat added, and relative
humidity at both the states. Solution

Solution Given A quantity of air is cooled sensibly;


At inlet: T1 = 39°C
Given A quantity of air is heated sensibly;
f1 = 29%
At inlet: T1 = 30°C 3
V = 5 m /s
Twb1 = 25°C
At exit: T2 = 24°C
V = 300 m3
w1 = w2
At exit: T2 = 40°C = 313 K Tcoil = 20°C
w1 = w2
To find
To find (i) Cooling capacity, and
(i) The rate of heat transfer, and (ii) By-pass factor.
(ii) Relative humidity at inlet and exit.

Analysis From the psychrometric chart Analysis From the psychrometric chart
State 1: 30°C DBT and 25°C WBT State 1: 39°C DBT and f1 = 29%
h1 = 76.5 kJ/kg h1 = 75 kJ/kg
vg = 0.885 m3/kg vg = 0.9 m3/kg
f1 = 65% State2: w 1 = w 2 and 24°C DBT
State 2: w1 = w2 and 40°C DBT, h2 = 60 kJ/kg
h2 = 86.4 kJ/kg
f1 = 39%
Psychrometry 515

Mass flow rate of air,


V 5 m3/ kg
ma = = = 5.56 kg/s
v g 0.9 m3/ kg
(i) Capacity of cooling coil
Q = ma (h1 – h2)
= 5.56 ¥ (75 – 60)
= 83.4 kW or 23.83 TR
(ii) By-pass factor
Tc - Texit Tc - T2
BPF = =
Tc - Tinlet Tc - T1
20 - 24
= = 0.21
20 - 39
Example 15.12 Atmospheric air at 760 mm of Hg,
15 °C DBT and 11°C WBT enters a heating coil, whose
temperature is 41°C. The by-pass factor of the coil is 0.5.
Calculate DBT, WBT and the relative humidity of air
leaving the coil. Also, calculate the sensible heat added
to air per kg of dry air. fi T2 = 28°C
State 2: Using psychrometric chart at coordinates of w 1 =
Solution w 2 and 28°C DBT, and wet bulb temperature and relative
Given A quantity of air is heated sensibly; humidity are
At inlet: T1 = 15°C Twb2 = 16°C f2 = 29%
Twb1 = 11°C and h2 = 46 kJ/kg
p1 = 760 mm of Hg = 101.3 kPa Sensible heat transfer to air
Heating coil Tc = 41°C q = h2 – h1
BPF = 0.5 = 46 – 31.8 = 14.2 kJ/kg
and w1 = w2
To find
(i) T2, Twb2, f2 of air leaving the coil,
(ii) The rate of heat transfer.
When the moisture content of air is increased at
Analysis The sensible heating process is shown constant temperature, the process is referred as
schematically and on a psychrometric chart in Fig. 15.16. humidification [process (1 – 2)], and when moisture
(i) T2, Twb2, f2 of air leaving the coil; is reduced from the moist air at constant tempera-
From psychrometric chart ture the process is referred as dehumidification
[process (2 – 1)].
State 1: 15°C DBT and 11°C WBT
During humidification and dehumidification,
T1 = 15°C Twb1 = 11°C
the specific humidity w and humidity ratio f, both
h1 = 31.8 kJ/kg and f1 = 62%
change. The processes are shown in Fig. 15.17.
The by-pass factor is given by Eq. (15.20)
Actually, there is no method by which one can
Tc - Texit Tc - T2 obtain simply humidification or dehumidification
BPF = =
Tc - Tinlet Tc - T1 of air. These processes are accomplished by either
41 - T2 simultaneous heating or cooling.
0.5 =
41 - 15
516 Thermal Engineering

Example 15.13 Moist air at 22°C and a wet-bulb


temperature of 9°C enters a steam spray humidifier. The
mass flow rate of the dry air is 90 kg/min. Saturated water
vapour at 110°C is injected into the mixture at a rate of
52 kg/h. There is no heat transfer with the surroundings
and pressure is constant throughout at 1 bar. Determine
(a) the humidity ratio, and (b) temperature of air leaving
the humidifier.

Solution
Given Humidification of moist air with the help of
steam injection
T1 = 22°C Twb1 = 9°C
ma = 90 kg/min Tsat = 110°C
ms = 52 kg/h = 0.867 kg/min
p = 1 bar = 100 kPa
This process is generally used in winter air-condi-
tioning. It involves warming and humidifying of
air simultaneously. In this process, the air passes
through a humidifier where the hot water (or steam)
is injected, both the humidity ratio and dry-bulb
temperature increase. The process of heating and
humidification is shown by the line 1–2 on the psy-
chometric chart in Fig. 15.18.

To find
(i) The specific humidity of air leaving the humidifier,
(ii) Exit temperature of moist air.
Analysis
(i) The humidity ratio at the exit
The mass flow rate balance during the flow
through control volume between states 1–2
ma1 = ma2 (dry air)
mv1 + ms = mv2 (water vapour)
Dividing both sides by ma
mv1 ms mv2
+ =
ma ma ma
1
ms
or w2 = w1 +
ma
From pychrometric chart against coordinates of
22°C DBT and 9°C WBT
w1 = 0.002 kg/kg of dry air
h1 = 27.2 kJ/kg of dry air
Psychrometry 517

Thus, the specific humidity at the exit


(0.867 kg/min)
w 2 = 0.002 +
(90 kg/min )
= 0.0116 kg/kg of dry air
(ii) The exit temperature of moist air
From steam table, A-12
hs = hg @ 110°C = 2691.47 kJ/kg
The enthalpy of leaving air
m
h2 = h1 + s hs
ma
= (27.2 kJ/kg)
0.867 kg/min
+ ¥ ( 2691.47 kJ/kg)
90 kg/min
or h2 = 53.11 kJ/kg of dry air
Using psychrometric chart at coordinates of h2 =
53.11 kJ/kg and w 2 = 0.0116 kg/kg, the DBT and
relative humidity are
T2 = 24°C
f2 = 61%
P
The atmospheric air at 25°C DBT
and 12°C WBT is flowing at a rate of 100 m3/min through The mass flow rate of atmospheric air
a duct. The dry saturated steam at 100°C is injected into Va 100 m3/min
ma = = = 118.48 kg/min
the air stream at a rate of 72 kg/h. Calculate the specific v1 0.844 m3/kg
humidity, DBT, WBT, relative humidity and enthalpy of (i) The specific humidity at the exit
air leaving the duct.
m 1.2
w 2 = w1 + s = 0.0034 +
Solution ma 118.48
= 0.0135 kg/kg of dry air
Given Humidification of atmospheric air with the help
(ii) The exit temperature of moist air
of steam injection
From steam tables
T1 = 25°C Twb1 = 12°C
hs = hg @ 100°C = 2676 kJ/kg
Va = 100 m3/min Tsat = 100°C
The enthalpy of leaving air
ms = 72 kg/h = 1.2 kg/min
m
h2 = h1 + s hs
To find ma
(i) The specific humidity, DBT, WBT, f2 of air 1.2
= 33.2 + ¥ 2676
leaving the humidifier, and 118.48
(ii) Specific enthalpy of air leaving. = 60.3 kJ/kg of dry air
Analysis From pychrometric chart against coordinate Using psychrometric chart at coordinates of h2
of 25°C DBT and 12°C WBT = 60.3 kJ/kg and w 2 = 0.0135 kg/kg, and locate
point 2. The properties of moist air at state 2 are
w1 = 0.0034 kg/kg of dry air
DBT T2 = 26°C
h1 = 33.2 kJ/kg of dry air
WBT Twb2 = 21°C
v1 = 0.844 m3/kg
RH f2 = 65%
518 Thermal Engineering

For negligible heat transfer to surroundings and


in absence of work transfer, kinetic and potential
Cooling and humidification, simultaneously can be energy losses, the steady flow energy equation
accomplished by evaporative cooling. It involves yields to
either spraying of liquid water into air or forcing (ha2 + w 2 hg2) = (w2 – w1)hf1 + (ha1 + w 1 hg1)
the air through a soaked pad that is kept saturated ...(15.23)
with water as in desert coolers, also known as where hf1 is the specific enthalpy of the water
evaporative coolers. A schematic of an evaporative entering the control volume.
cooler is shown in Fig. 15.21(a). The evaporative cooling follows a line of constant
wet-bulb temperature on the psychrometric chart as
Water at Tw
1 2 shown in Fig. 15.21(b). Thus
Twb = constant ...(15.24)
h ª constant ...(15.25)
T1 T2
f1 f2 The evaporative cooling (humidification and
ha1 ha2 cooling) takes place in cooling towers, evaporative
hg1 hg2 condensers and desert coolers.

Example 15.15 Air at 37°C and 10% relative humid-


Soaked pad
ity enters an evaporative cooler with a volumetric flow
(a) Evaporative cooler rate of 140 m3/min. Moist air leaves the cooler at 20°C.
The water is added to the soaked pad of the cooler at
20°C and evaporates fully into the moist air. There is
no heat transfer with the surroundings and the pressure
is constant throughout the process at 1 atm. Determine
Tw1

(a) the mass flow rate of water to the soaked pad, and

w
Tw2

(b) relative humidity of the moist air at the exit of the


w2 evaporative cooler.
2
w2 – w1

Solution
w1
1
Given Humidification of moist air with the help of
T1 – T2
evaporative cooling.
T2 T1 T1 = 37°C f1 = 10% = 0.1
Dry-bulb temperature
V = 140 m3/min T2 = 20°C
(b) Representation of procession psychrometric
T3 = 20°C p = 1 atm = 101.3 kPa
chart

The low humidity air enters the humidifier at the


state 1, and when it passes through a water soaked
pad a part of water evaporates and gets mixed with
air. The energy for evaporation is extracted from
the water body and air stream, thus both water and
air get cooled. Therefore, the stream of air leaving
the humidifier at the state 2 is rich in moisture and
low in temperature.
Psychrometry 519

To find w2 p 0.0109 ¥ 101.3


(i) The mass flow rate of water to the soaked pad. or pv2 = =
w 2 + 0.622 0.0109 + 0.622
(ii) The relative humidity of moist air leaving the
= 1.744 kPa
evaporative cooler.
At 20°C, the saturation pressure of vapour,
Assumptions psat2 = 0.02339 bar = 2.339 kPa
(i) Specific gas constant of air, R = 0.287 kJ/kg ◊ K. pv2 1.744
Thus f2 = = = 0.745 = 74.5%
(ii) Specific heat at constant pressure of air, Cp = psat2 2.339
1.005 kJ/kg ◊ K.
Alternatively, the relative humidity f2 can be
Analysis obtained from psychrometric chart with w2 =
(i) The mass flow rate of water to the soaked pad 0.0109 and T2 = 20°C, the intersecting point gives
The mass conservation gives f2 = 75%.
mw = mv2 – mv1
= ma (w2 – w1) ...(i)
The saturation pressure psat1 at 37°C is 6.289 kPa It is based on the principle of adsorption, i.e., capil-
pv1 = f1 psat1 = 0.1 ¥ 6.289 kPa
lary action. Thermodynamically, an adsorption pro-
= 0.6289 kPa
cess is the nerve of the adiabatic saturation process.
The specific humidity at the state 1
pv1 In this process, the air is passed over the adsorbing
w1 = 0.622 surface (chemicals are silica gel, activited alumina,
p - pv1
etc. which have affinity for moisture). As the water
0.6289 kPa
= 0.622 ¥ vapour comes in contact of these substances, the
101.3 - 0.6289 kPa
= 0.00388 kg/kg of dry air moisture gets condensed out of air and gives up its
The mass flow rate of dry air at the state 1 latent heat, resulting into rise in temperature of air.
pV 101.3 ¥ 140 The dehumidification and heating process is shown
ma = = = 159.4 kg in Fig. 15.23.
RT 0.287 ¥ 310
The specific humidity w2 can be obtained by
using steady-flow energy Eq. (15.23)
ha2 + w 2 hg2 = (w 2 – w1) hf 1 + ha1 + w1hg1
or w 2 (hg2 – hf 1) = ha1 – ha2 + w1(hg1 – hf1)
C p (T1 - T2 ) + w1 ( hg1 - h f1 )
or w2 = ...(ii)
hg2 - h f1
From steam table A-12
hg1 = hg @ 37°C = 2568.9 kJ/kg
hg2 = hg @ 20°C = 2538.1 kJ/kg
hf1 = hf @ 20°C = 83.96 kJ/kg
Thus from Eq. (ii)
1.005 ¥ (37 - 20) + 0.00388 ¥ ( 2568.9 - 83.96)
w2 =
2538.1 - 83.96
= 0.0109 kg/kg of dry air When a moist air stream is cooled at constant
Substituting in Eq. (i) pressure to a temperature below its dew point
mw = 159.4 ¥ (0.0109 – 0.00388) temperature, some of the water vapour initially
= 1.12 kg/min present would condense. Fig. 15.24 shows the
(ii) The relative humidity of the moist air schematic of a dehumidifier and cooling process on
The partial pressure of water vapour in air psychrometric chart.
pv2
w 2 = 0.622
p - pv2
520 Thermal Engineering

Cooling 2 Heating Solution


1 coil coil 3
Given Dehumidification and cooling of moist air:
f2 = 100% T1 = 40°C f1 = 60%
3
T2 < T 1 T2 = 25°C Va = 40 m /min
Moist w2 = w1
. air T3 > T2 TADP = TO = 20°C
ma w3 = w2
T1, w1 To find
(i) Dew-point temperature,
(ii) Mass of water drained out per hour,
Condensed (iii) Capacity of cooling coil, and
. water
mw (iv) By pass factor of coil.
(a) schematic of dehumidifier
Analysis The process of cooling and dehumidification
is shown on psychrometric chart in Fig. 15.25.
1 w1 h
0 % 1 =
10 11
5
f= 3
kJ
/kg
w3
2 w h
o
31 C 1 0.029
2 =
76
kJ/
kg
0.02
c 2

DBT T2 T3 T1
w
(b) Representation of dehumidification and
cooling on psychrometic chart o
DBT 20 C 25oC 40 C
o

The moist air enters the state 1, flows across


Inlet conditions: f1 = 60%, T1 = 40°C
a cooling coil, some of the water vapour initially
From psychrometric chart;
present in the moist air condenses and a saturated
v1 = 0.925 m3/kg
moist air mixture leaves the dehumidifier section
w1 = 0.029 kg/kg of air
at the state 2. Since the moist air leaving the
h1 = 115 kJ/kg
cooling coil is saturated at temperature below the
Exit conditions: T2 = 25°C
temperature of the moist air entering, the moist
TC = 20°C
air stream might be unsuitable for direct use in
w2 = 0.02 kg/kg of air
occupied space. Thus this stream passes the heating
h2 = 76 kJ/kg
section and is brough to the comfort condition at
the state 3. (i) Dew-point temperature corresponding to the
state 1 is 31°C.
Example 15.16 Air at 40°C DBT and 60% RH The mass flow rate of dry air,
is cooled to 25°C DBT. It is achieved by cooling and V 40
ma = =
dehumidification. Air flow rate is 40 m3/min. Using va1 0.925
psychrometric chart, calculate: = 43.24 kg/min or 2595.6 kg/h
(a) the dew point temperature, (ii) Mass of moisture removed
(b) mass of water drained out per hour, mw = ma (w1 – w2)
(c) Capacity of cooling coil. = 2595.6 ¥ (0.029 – 0.02)
If the apparatus dew-point temperature (ADPT) is = 23.36 kg/h
20°C, find the by pass factor of coil.
Psychrometry 521

(iii) Capacity of cooling coil


The heat transfer;
Q = ma (h1 – h2) = 43.24 ¥ (115 – 76)
= 1686.36 kJ/min
Q (kJ/min) 1686.36
Coilc apacity = =
( 210 kJ/min/Ton) 210
= 8.03 Ton
(iv) By pass factor
The by-pass factor is given by Eq. (15.33)
T - Texit Tc - T2
BPF = c =
Tc - Tinlet Tc - T1
20 - 25 (ii) Heat removal rate
= = 0.25
20 - 40 Q = ma (h1 – h2)
= 11.63 ¥ (85.5 – 39.5)
Example 15.17 Air enters a window air conditioner = 534.98 kJ/min or 8.916 kW
at 1 atm, 30°C and 80% RH at a rate of 10 m3/min and
it leaves at saturated at 14°C. A part of moisture, which Example 15.18 Moist air at 30°C and 50% relative
condenses during the process is also removed at 14°C. humidity enters a dehumidifier operating at steady state
Determine the rate of heat and moisture removal from with a volumetric flow rate of 280 m3/min. The moist air
air. passes over a cooling coil and water vapour condens-
es. The condensate leaves the dehumidifier saturated at
Solution 10°C. Saturated moist air leaves in a separate stream at
Given Dehumidification and cooling of moist air: the same temperature. During the process, the pressure
remains constant at 1.013 bar and heat transfer to the
p1 = 1 atm T1 = 30°C
3 surroundings is negligble. Determine (a) the mass flow
f1 = 80% Va = 10 m /min rate of dry air, (b) rate of water condensation in the de-
T2 = 14°C f2 = 100% humidifier in kg/kg of dry air flowing through the control
To find volume, and (c) the required refrigerating capacity in
(i) Rate of moisture removal, tonnes.
(ii) Rate of heat transfer.
Solution
Analysis The process of cooling and dehumidification
is shown on the psychrometric chart in Fig. 15.26. Given Dehumidification of moist air over a cooling
Inlet conditions: f1 = 80% T1 = 30°C coil.
V = 280 m3/min T1 = 30°C = 303 K
From psychrometric chart;
f1 = 50% = 0.5 T2 = 10°C
v1 = 0.86 m3/kg
p = 1.013 bar = 101.3 kPa
w1 = 0.022 kg/kg of air
h1 = 85.5 kJ/kg To find
Exit conditions: T2 = 24°C f2 = 100% (i) The mass flow rate of dry air,
w2 = 0.0102 kg/kg of air (ii) The rate of condensation of water vapour in
h2 = 39.5 kJ/kg dehumidifier in kg/kg of dry air,
The mass flow rate of dry air (iii) Required refrigerating capacity in tonnes.
V 10
ma = a = = 11.63 kg/min Assumptions
va1 0.86
(i) The control volume is shown in Fig. 15.27.
(i) Removal rate of moisture from air
(ii) Moist air leaves the cooling coil as saturated at
mw = ma (w1 – w2) 10°C.
= 11.63 ¥ (0.022 – 0.0102) (iii) Specific gas constant for dry air as R = 0.287
= 0.1372 kg/min or 8.23 kg/h kJ/kg ◊ K.
522 Thermal Engineering

Cooling (iii) Refrigeration effect


coil
Heat removal rate
Q = ma (h1 – h2)
= 320 ¥ (64 – 27.2) = 11776 kJ/min
Moist air Saturated mixture
The refrigeration effect in Tons of refrigeration
f1 = 50% at 10°C (1 TR = 210 kJ/min)
T1 = 303 K
.. 11776
V = 280 m3/min TR = = 56.07 Tons
Control valve 210

(a) Condensate water at 10°C

It is a common practice in air conditioning that


/kg
64 kJ w the fresh and recirculated moist air streams are
50%
/kg mixed as shown in Fig. 15.28. During such mixing,
27 .2 kJ 1
0.0136
normally no heat is transferred to the surroundings,
o
10 C
0.0076
and thus mixing is almost adiabatic.
2

o o
10 C 30 C
DBT
(b) Process on psychrometric chart

Analysis
(i) The mass-flow rate of dry air
From psychrometric chart
The inlet conditions: p1 = 101.3 kPa
f1 = 50% T1 = 303 K
w1 = 0.0136 kg/kg of air
Applying mass and energy balance on the
va = 0.875 m3/kg
1 control volume
h1 = 64 kJ/kg
Exit conditions
T2 = 10°C, f2 = 100% ma1 + ma2 = ma3 (dry air) ...(15.26)
h2 = 27.2 kJ/kg mv1 + mv2 = mv3 (water vapour)
The mass flow rate of dry air Using mv = w ma , we get
V 280 w 1 ma1 + w 2 ma2 = w3 ma3 (water vapour)
ma = = = 320 kg/min
va1 0.875 w1 ma1 + w 2 ma2 w1ma1 + w 2 ma2
or w3 = =
(ii) Rate of moisture removal from air ma3 ma1 + ma2
mw ...(15.27)
= (w1 – w2) = 0.0136 – 0.0076
ma
= 0.006 kg/kg of dry air
ma1 h1 + ma2 h2 = ma3 h3
Psychrometry 523

ma1 h1 + ma 2 h2 Humidity of mixed stream


or h3 = ...(15.28) w1 ma1 + w 2 ma2
ma3 w3 =
ma3
where h = ha + w hg; the specific enthalpy of 0.0122 ¥ 25 + 0.0024 ¥ 5
entering and exiting water vapour are evaluated at =
30
saturated vapour values at the respective dry-bulb = 0.01056 kg/kg of air
temperatures. Specific enthalpy of mixed stream
ma1 h1 + ma 2 h2
Example 15.19 25 kg of air at 25°C DBT and 61% h3 =
RH is mixed with 5 kg of air at 5°C DBT and 30% RH. ma3
Calculate the condition of mixed air. 25 ¥ 56 + 5 ¥ 11
= = 48.5 kJ/kg
30
Solution From psychrometric chart; using coordinates w3 and
Given Mixing of two streams of air h3, temperature and humidity of mixed stream
m1 = 25 kg, T1 = 25°C f1 = 61% T3 = 22.5°C, f3 = 60%
m2 = 5 kg T2 = 5°C f2 = 30%
Example 15.20 142 m3/min moist air at 5°C with
To find Condition of mixed stream of air. specific humidity of 0.002 kg/kg of dry air is mixed adia-
Assumptions batically with 425 m3/min of moist air stream at 24°C
and 50% relative humidity. If the pressure is constant
(i) There is no heat transfer with the surroundings.
throughout at 1 bar. Determine (a) the humidity ratio,
(ii) The pressure remains constant throughout at and (b) the temperature of the mixed stream.
1 atm.
Schematic Mixing of two streams is shown on psy- Solution
chrometric chart in Fig. 15.29. Given Adiabatic mixing of two moist air streams as
shown on Fig. 15.30.
To find
(i) Humidity ratio w3 of mixed stream, and
(ii) Temperature of the mixed stream.
Analysis From psychrometic chart
Coordinates: w1 = 0.002 and T1 = 5°C
v1 = 0.79 m3/kg
h1 = 10 kJ/kg
Coordinates: f2 = 50% and 24°C
v2 = 0.855 m3/kg w2 = 0.0094 kg/kg
h2 = 47.8 kJ/kg
Analysis From psychometric chart;
Mass of inlet stream 1;
Coordinates: 25°C DBT and 61% RH
V1 142
w1 = 0.0122 kg/kg of dry air ma1 = = = 179.746 kg/min
h1 = 56 kJ/kg v1 0.79
Coordinates: 5°C DBT and 30% RH Mass of inlet stream 2;
w2 = 0.0024 kg/kg of dry air V2 425
ma2 = = = 497.07 kg/min
h2 = 11 kJ/kg v2 0.855
The mass balance (i) Specific humidity of mixture can be obtained by
ma1 + ma2 = ma3 Eq. (15.27)
or 25 kg + 5 kg = 30 kg
524 Thermal Engineering
.
V1 = 142 m3/min Example 15.21 1 kg air at 40°C dry-bulb tempera-
T1 = 5°C ture and 50% relative humidity is mixed with 2 kg of air
w1 = 0.002 1
at 20°C dry-bulb temperature and 20°C dew-point tem-
3 perature. Determine for the mixed stream (b) tempera-
ture, and (c) specific humidity.
w3 = ?
T3 = ?
2 Solution
Control boundary
.
V2 = 425 m3/min Given Mixing of two streams of air
T2 = 24°C m1 = 1 kg T1 = 40°C f1 = 50%
f2 = 50%
(a) Mixing of two stream of air m2 = 2 kg T2 = 20°C Tdp = 20°C
To find
47
.8

(i) Specific humidity, and


kJ
/k
g

(ii) Temperature of the mixture.


w
2
0.0094
10

Assumptions
kJ
/
kg

w3 (i) There is no heat transfer with the surroundings.


50

3
(ii) The pressure remains constant throughout at
f=

1 0.002
1 atm.
Schematic Mixing of two stream is shown on psychro-
metric chart in Fig. 15.31.
T 5¢C T3 24¢C
DBT

(b) Mixing procession psychrometric chart

w1 ma1 + w 2 ma2
w3 =
ma3
Using numerical values
0.002 ¥ 179.746 + 0.0094 ¥ 497.07
w3 =
179.746 + 497.07
= 0.0077 kg/kg of dry air
(ii) Temperature of the mixed stream Analysis From psychometric chart;
Specific enthalpy of mixture, Eq. (15.28); Coordinates: 40°C DBT and 50% RH,
ma1 h1 + ma2 h2 w1 = 0.0236 kg/kg of dry air
h3 =
ma3 Coordinates: 20°C DBT and 20°C DPT,
Using numerical values f2 = 100%
179.746 ¥ 10 + 497.07 ¥ 47.8 and w2 = 0.0148 kg/kg of dry air
h3 =
179.746 + 497.07
(i) Specific humidity of mixture can be obtained by
= 37.7 kJ/kg
Eq. (15.38)
From psychometric chart at coordinates
w3 = 0.0077 and h3 = 37.7 kJ/kg w1 ma1 + w 2 ma2 w1ma1 + w 2 ma2
w3 = =
T3 = 19°C ma3 ma1 + ma2
Psychrometry 525

Using numerical values


0.0236 ¥ 1 + 0.0148 ¥ 2
w3 =
1+ 2
= 0.0177 kg/kg of dry air
(ii) Temperature of mixed stream
Draw a straight line between points 1 and 2 on
the psychrometric chart and locate the point 3
corresponding to w3 = 0.0177, we get the dry-
bulb temperature.
T3 = 26.8°C

processes that can take place between air entry at


An air washer is a system in which the air is passed the state 1 and air exit at the state 2. These are the
through a spray of water. The condition of air following:
leaving the washer depends on the mean surface
1. When the mean temperature of water drop-
temperature of water droplets.
lets is greater than the dry-bulb temperature
According to the requirement of conditioned of air, the air is heated and humidified, pro-
air, the water used for spray is externally heated cess 1–2A.
or cooled or simply re-circulated by a pump as
2. When the mean temperature of water drop-
shown in Fig. 15.32. The make-up water is added to
lets is equal to the dry-bulb temperature of
maintain the water level in the washer constant. The
air, the air is humidified, with an increase in
louvers are provided to the loss of water droplets.
its enthalpy, process 1–2B.
3. When the mean temperature of water drop-
lets is less than the dry-bulb temperature of
air but greater than the wet-bulb temperature
of air, the air is cooled and humidified with
increase in its enthalpy, process 1–2C.
4. When the mean temperature of water drop-
lets is equal to the wet-bulb temperature
of air, the air is saturated adiabatically,
process 1–2D.
5. When the mean temperature of water drop-
lets is equal to the dew-point temperature
of air, the air is cooled with decrease in its
enthalpy, process 1–2E.
6. When the mean temperature of water drop-
The droplets of water in the washer act as wetted lets is less than the dew-point temperature of
surface. When air passes over it, its treatment air, the air is cooled and dehumidified, pro-
depends on vapour pressure and temperature of cess 1–2F.
water. Figure 15.33 shows a range of psychrometric
526 Thermal Engineering

Summary
air The humidity ratio w is defined as the ratio of the
is psychrometry. mass of vapour in the atmospheric air to the mass
The temperature measured by an ordinary ther- of dry air:
mometer is termed as dry-bulb temperature. m pv
w = v = 0.622
The dew-point temperature of dry air and water ma p - pv
vapour mixture is defined as the saturation tem- where p = pa + pv = total pressure of mixture.
perature corresponding to the partial pressure of The degree of saturation m is defined as the ratio
the vapour in the mixture. of actual humidity ratio to the humidity ratio of
The adiabatic saturation temperature of atmo- saturated air at the same temperature and pres-
spheric air is defined as the temperature which sure:
results from adding water vapour adiabatically w
m =
to the atmospheric air in steady flow, until it be- w sat
comes saturated.
The enthalpy of moist air is the sum of enthalpy
The wet-bulb temperature is the temperature of dry air and water vapour:
measured by a thermometer whose bulb is cov-
h = ha + whg
ered with water soaked cotton wick and placed in
= 2500 + 1.88 T kJ/kg of water vapour.
a stream of air.
The psychrometric chart is a graphical represen-
The relative humidity f is defined as the ratio of
tation of properties of atmospheric air.
actual mass of vapour to mass of vapour required
Air washer is a system in which the air is passed
to produce a saturated mixture of air at the same
through a spray of water.
pressure and temperature:
mv p
f = = v
msat psat

Glossary
DBT Temperature indicated by an ordinary thermom- Relative humidity Ratio of partial pressure of vapour to
eter total pressure of mixture
Dehumidification Reduction of water vapour from Sensible heating/cooling Heat transfer with only
moist air change in DBT
DPT Temperature at which vapour starts to condense Specific humidity Mass of water vapour per kg of dry
Humidification Addition of water vapour in air air
Psychrometry Study of moist air WBT Temperature indicated by thermometer, when its
bulb is covered by a wick thoroughly wetted.
Psychrometry 527

Review Questions
1. What is the dew-point temperature? 8. When are the dry-bulb and dew-point tempera-
2. List the properties of moist air that a psychromet- tures identical?
ric chart provides for an air-conditioning engi- 9. What is an adiabatic saturation? When does the wet-
neer. bulb temperature equal the saturation temperature?
3. Sketch a psychrometric chart and indicate the 10. How is the enthalpy of air–water vapour mixture
lines of constant wet-bulb temperature and con- defined?
stant relative humidity. 11. What do you understand by evaporative cooling?
4. What is the difference between dry air and atmo- List three important characteristics of evaporative
spheric air? cooling systems.
5. Define (a) specific humidity, and (b) relative 12. What is sensible heating or cooling?
humidity. 13. Explain the process of cooling and dehumidifica-
6. The moist air is passed through a cooling section tion.
where it is cooled and dehumidified. How do the 14. Explain the process of heating and humidification.
specific humidity and relative humidity of air 15. Prove that the specific humidity is given by
change during this process? pv
w = 0.622
7. In the summer, the outer surface of a glass filled p - pv
with iced water frequently ‘sweats’. How can you where pv = partial pressure of water vapour
explain this sweating? p = total pressure of air

Problems
1. The moist air at 1 atm has 32°C DBT and 26°C perature, mass of water vapour and mass of dry
WBT. Calculate air for 100 m3 of moist air and partial pressure of
(a) Partial pressure of water vapour, water vapour and air.
(b) Specific humidity, [0.016 kg/kg of dry air, 21.25°C, 1.817 kg,
(c) Dew point temperature, 113.54 kg, 2.544 kPa, 98.78 kPa]
(d) Relative humidity, 5. On a hot summer day, the ambient conditions are
(e) Degree of saturation, 40°C DBT, 20% RH. A desert cooler is used to
(f ) Enthalpy of mixture. increase the RH to 80%. Show the process on a
[(a) 0.03 bar (b) 0.0186 kg/kg of dry air, psychrometric chart and by using it, calculate the
temperature of exit air and minimum temperature
(c) 24.1°C, (d) 62.5% (e) 0.614, ( f ) 80.55 kJ/kg]
to which the air can be cooled by a well-designed
2. A sling psychrometer reads 40°C DBT and 36°C
desert cooler. [24.8°C, 22°C]
WBT. Calculate (a) specific humidity, (b) relative
6. DBT and WBT of atmospheric air at 1 atm are
humidity, and (c) dew point.
25 and 15°C, respectively. Determine (a) specific
[0.0238, 50%, 28.2°C]
humidity, (b) relative humidity, and (c) enthalpy
3. Calculate the amount of heat removed per kg of
of air.
dry air, if the initial condition of air is 35°C DBT,
[(a) 0.0065 kg/kg of dry air,
70% RH and final condition is 25°C DBT and
(b) 33.2%, (c) 41.8 kJ/kg]
60% RH. [46 kJ/kg]
7. The air at 50°C has a relative humidity of 100%.
4. The moist air has 60% RH at 1 atm and 30°C.
What is its dew-point temperature? What mass
Calculate the specific humidity, dew point tem-
528 Thermal Engineering

of liquid water per kg of dry air will result if the steadily in an insulated mixing chamber so that
mixture is cooled to 10°C at constant pressure of the resulting temperature is 25°C. What is the rate
90 kPa? [50°C] of mass flow of the cooler air required and what
8. Consider 100 m3 of atmospheric air at 100 kPa, would be the relative humidity of air leaving?
15°C and 40% relative humidity. Find mass of 13. Air enters a dehumidifier at 34°C, 101 kPa and
water and humidity ratio. What is the dew point 60% RH. After being dehumidified, the air is
of the mixture? [0.51 kg, 0.0043 kg, 1.4°C] heated to 20°C, with RH being 48%. Determine
9. Air at 10°C, 90% relative humidity and air at the temperature of air leaving the demudifier, if
30°C, 90% relative humidity are mixed steadily the air is saturated.
and adiabatically. The mass-flow rate of the 14. Saturated air at 20°C at a rate of 70 m3/min is
colder stream is twice that of the other stream. mixed adiabatically with the outside air at 35°C
What would be the state after mixing? and 50% RH at a rate of 30 m3/min. Assuming
10. 10 m3/min of air at 1 atm and 20°C with 90% RH that the mixing process occurs at a pressure of
is mixed with 20 m3/min of air at 1 atm and 40°C 1 atm, determine the specific humidity, the
with 20% RH. Calculate the resulting state of the relative humidity, and dry bulb temperature and
mixture. [h3 = 61 kJ/kg, T3 = 35.5°C] volume flow rate of the mixture.
11. Consider a 500-litre rigid tank containing air– [0.0157 kg/kg of dry air, 80%,
water vapour mixture at 100 kPa, 35°C with a 26° C, 100.13 m3/min]
70%, RH. The system is cooled until the water 15. Air at 1°C DBT and 80% relative humidity mixes
just begins to condense. Determine the final adiabatically with an air stream at 18°C DBT,
temperature in the tank and the heat transfer for 40% relative humidity in the ratio of 1 to 3 by
the process. [28.2°C, –2.77 kJ] volume. Calculate the temperature and per cent
12. A stream of air at 1 atm, 30°C, 80% relative saturation of the mixture. Take the barometric
humidity flows at a rate of 28.0 kg/min. It is pressure 1.013 bar throughout constant.
proposed to mix air at 1 atm, 15°C with a stream, [13.6°C and 48%]

Objective Questions
1. The psychrometry is (b) mixture of superheated vapour and air
(a) study of cooled air (b) study of hot air (c) moist air containing maximum possible
(c) study of moist air (d) none of the above moisture
2. A psychrometer is an instrument, which measures (d) none of the above
(a) wet-bulb emperature
t 5. Temperature at which condensation of vapour
(b) dry-bulb temperature begin is called
(c) dew-point emperature
t (a) boiling temperature
(d) dry-bulb and wet-bulb temperatures (b) saturation temperature
3. Temperature recorded by an ordinary thermocou- (c) dew-point temperature
ple is known as (d) all of the above
(a) wet-bulb emperature
t 6. Humidity ratio is defined as ratio of
(b) dry-bulb temperature (a) mass of water vapour to mass of dry air in
(c) dew-point emperature
t the mixture
(d) saturation temperature (b) mass of water vapour to mass of total mixture
4. Saturated air is (c) mass of dry air to mass of water vapour
(a) mixture of moisture and vapour (d) none of the above
Psychrometry 529

7. The degree of saturation is defined as (b) horizontal and uniform


(a) mass of water vapour to mass of dry air in (c) vertical
the mixture (d) straight and inclined
(b) ratio of actual humidity ratio to humidity 14. On a psychrometric chart, the dew point tempera-
ratio of saturated air ture lines are
(c) ratio of actual mass of water vapour to mass (a) curved
of saturated vapour (b) horizontal and uniform
(d) none of the above (c) vertical
8. Humidity ratio is called (d) straight and inclined
(a) relative humidity (b) specific humidity 15. On a psychrometric chart, the dry-bulb tempera-
(c) degree of humidity (d) none of the above ture lines are
9. Wet-bulb depression is equal to (a) curved
(a) sum of dry-bulb temperature and wet bulb (b) horizontal and uniform
temperature (c) vertical
(b) average of dry-bulb temperature and wet- (d) straight and inclined
bulb temperature 16. For a moist air, if DBT is 15°C, WBT is 15°C and
(c) difference of dry-bulb temperature and wet- DPT is also 15°C, the saturation temperature will
bulb temperature be
(d) product of dry-bulb temperature and wet- (a) 15°C (b) 25°C
bulb temperature, one of the above (c) 10°C (d) 35°C
10. On a psychrometric chart, the horizontal scale 17. The process of cooling of air at same humidity
shows ratio is known as
(a) wet-bulb emperature
t (a) sensible heating (b) sensible cooling
(b) dry-bulb temperature (c) humidification (d) dehumidification
(c) dew-point temperature 18. The process of adding moisture to air at constant
(d) absolute humidity dry-bulb temperature is known as
11. On a psychrometric chart, the vertical scale shows (a) sensible heating (b) sensible cooling
(a) wet-bulb emperature
t (c) humidification (d) dehumidification
(b) dry-bulb temperature 19. The process of removing moisture from air at
(c) dew-point temperature constant dry-bulb temperature is known as
(d) absolute humidity (a) sensible heating (b) sensible cooling
12. On a psychrometric chart, the wet-bulb tempera- (c) humidification (d) dehumidification
ture lines are 20. During the process of adiabatic saturation,
(a) curved (a) DBT decreases
(b) horizontal but not uniform (b) specific humidity increase
(c) vertical (c) RH increases
(d) straight and inclined (d) all of the above
13. On a psychrometric chart, the relative humidity
lines are
(a) curved
20. (d) 19. (d) 18. (c) 17. (b)
16. (a) 15. (c) 14. (b) 13. (a) 12. (d) 11. (d) 10. (b) 9. (c)
8. (b) 7. (b) 6. (a) 5. (d) 4. (c) 3. (b) 2. (d) 1. (c)
Answers
530 Thermal Engineering

16
Fuels and Combustion

Introduction
In the preceding chapters, our discussion was limited to non-reactive systems, in which chemical composition
remains unchanged during a process with transfer of sensible internal energy and latent internal energy. In
reactive systems (fuels), the chemical internal energy associated with destruction and formation of chemical
bonds between atoms is transferred during a reaction. Such a chemical reaction is called combustion.
In this chapter, focus is put on types of fuels and combustion reactions, amount of air required for
complete combustion of fuel, air–fuel ratio, enthalpy of reaction and calorific values of fuels.

FUELS interactive bonds of the molecules of the fuel


and oxygen are broken and rearrangement of
A fuel is simply a combustible substance. It burns atoms in new molecular combination takes place.
in the presence of oxygen and releases heat energy. During this process, the energy is released in the
The various types of fuels, like liquid, solid and form of heat. Therefore, the combustion is an
gaseous fuels, are available for burning in boilers, exothermic chemical reaction. Though, during
furnaces and other combustion equipment. a combustion process, the reacting molecules
Most fuels contain carbon and hydrogen as main rearrange themselves after the combustion but the
constituent; thus they are called hydrocarbon fuels, total number of atoms of each element remain
and designated with the chemical formula Cn Hm. unchanged following the law of conservation of
The hydrocarbon fuels consist of all phases such mass.
as coal (a solid fuel), petroleum (a liquid fuel),
and natural gas (a gaseous fuel). These fuels are 16.2 CHARACTERISTIC OF AN
available for burning in boilers, furnaces and other IDEAL FUEL
combustion equipments.
Each fuel consists of a certain amount of bonded The selection of the right type of fuel depends
energy in between its constituent elements. This on various factors such as availability, storage,
energy is called the chemical energy and it is in handling, pollution and landed cost. The knowledge
the form of internal energy. In combustion, the of fuel properties helps in selecting the right fuel
for the right purpose and efficient use of fuel.
Fuels and Combustion 531

A good fuel should have the following qualities: moisture content with low fixed carbon of about
60%. Its appearance is brown and it contains high
(i) It should have a high heating value.
ash and low heating value. It is a low-grade fuel
(ii) It should be free from moisture and non-
having a calorific value about 21, 000 kJ/kg. It is
combustible matter, i.e., ash, etc.
too brittle, and hence it is converted into briquetts,
(iii) Its products of combustion should not be
which can be easily handled. It burns with heavy
harmful and pollutant.
smoke. There are comparatively large deposits
(iv) It should have moderate ignition tempera- of lignite in Kashmir, Neyvelli and Rajasthan.
ture. High ignition temperature may cause Neyvelli Power Plant uses this type of fuel.
difficulty in combustion and low ignition
temperature may cause fire hazards. Bituminous C Its formation is next to lignite.
(v) It should be easy to transport and store in It is shining black in appearance. It contains about
minimum space. 70% carbon and the remaining as a large percentage
(vi) It should have moderate rate of combustion of volatile matter. Thus, it is easy to ignite and it
and controlled combustion. burns with yellow smoky flames. It has an average
(vii) It should have high combustion efficiency. calorific value of 31, 500 kJ/kg. In India, its reserves
(viii) It should be readily available at low cost. are available in Bihar, MP, West Bengal and Orissa.
It is used for producing producer gas.
COAL It is the next generation of bituminous
coal, and has the highest rank. It is a hard coal
Coal is the most important solid fuel and its main
composed of about 90% carbon with little volatile
constituent is carbon. It also contains some amount
content and practically no moisture. It is brittle
of oxygen, hydrogen, nitrogen, sulphur, moisture
and has a shining black lusture. It burns without
and ash. Coal is a mineral substance of vegetable
flame and smoke but is difficult to ignite. It is non-
origin. It is the result of decay of vegetable matter
cracking and it has minimum ash, sulphur, volatile
which accumulated under earth millions of years
matter and moisture content. It has highest heating
ago and got transformed by the action of heat and
value, approximately 36,000 kJ/kg. In India, its
pressure. Its transformation occurs progressively.
reserves are in Kashmir and the eastern Himalayas.
It is used as fuel in steam power plants.
Coal
The coal passes through different stages during its It is obtained by heating of wood
formation from vegetation. These stages are listed up to a temperature of 280°C in the absence of
and discussed below: atmospheric air. During this process, the volatile
matter and water are expelled and the residue
It is a spongy humidified substance and is charcoal. It is an excellent fuel but costly. Its
the first stage at which the fuel is derived from calorific value is about 28,000 kJ/kg.
the wood and vegetable matter. Thus, it possesses
constituents very nearest to those of wood with It is obtained from bituminous coal by the
30% moisture. It has lowest calorific value about carbonization process, i.e., the bituminous coal is
14,500 kJ/kg. It is mainly used as a fuel in gas- heated strongly in the absence of air for 48 hours.
producer plants. Other varieties of coal are derived Thus, the volatile matter present in the coal is driv-
from peat. en off. Coke is dull black in colour, hard, brittle,
and porous. It consists of 90 to 95% carbon, small
Lignite is found next to peat. It is a soft quantities of sulphur, hydrogen, nitrogen and phos-
coal, composed mainly of volatile matter and phorus. It is a smokeless and clear fuel. Its average
532 Thermal Engineering

calorific value is 32,500 kJ/kg. It is used in steam (a) Proximate Analysis


power plants, blast furnaces and gas producers. Proximate analysis indicates the percentage
by mass of the fixed carbon, volatiles, ash, and
It is produced from finely grounded
moisture content in coal. The amount of fixed
low-grade coal by the moulding operation. The coal
carbon and volatile matter directly contribute to the
powder is mixed with a suitable binder and pressed
heating value of coal.
together to form blocks or briquettes. The binding
material may be pitch, coal tar, crude oil or clay. By Fixed carbon acts as a main heat generator during
this method, the heating value of low-quality fuel burning. It consists mostly of carbon but also con-
is improved. tains some hydrogen, oxygen, sulphur and nitrogen.
Fixed carbon gives a rough estimate of the heating
Pulverised Crushed coal in fine powder value of coal.
form is called pulverized coal. During combustion,
it floats and thus comes in better contact of air, are methane, hydrocarbons, hydro-
improving the combustion efficiency. Pulverised gen and carbon monoxide, and incombustible gases
coal has flexibility of control, complete combustion like carbon dioxide and nitrogen found in coal.
with less excess air and high flame temperature. High volatile matter content indicates easy ignition
Pulverised coal is used in cement industries and in of fuel.
furnaces. is an impurity that will not burn. The ash con-
tent is important in the design of the furnace grate,
Coal combustion volume, pollution control equipment
It is difficult to give an exact mass analysis of coal, and ash-handling systems of a furnace.
since its composition varies considerably from one in coal decreases the heat content per kg
geographical location to another. The following of coal. The moisture content in coal ranges from
two type of analysis of coal are most popular: 0.5 to 10%. A typical proximate analysis of various
types of coal is given in the Table 16.1.

Table 16.1

Percentage by mass of fuel


Fuel Inherent Volatile Fixed Ash %
Moisture % matter % Carbon %
Anthracite 2 6 75 17
Bituminous coal 6 21 35 38
Indonesian coal 9 30 47 14

Table 16.2

Percentage by mass of dry fuel


Fuel Rank C H2 O2 N2 S Mineral matter
Anthracite 101 88.2 2.7 1.7 1 1.2 5.2
Medium-rank coal 401 81.8 4.9 4.4 1.8 1.9 5.2
Low-rank coal 902 75.0 4.6 10.7 1.6 2.1 6.0
Coke – 90.0 0.4 1.9 – – 7.7
Fuels and Combustion 533

(b) Ultimate Analysis Advantages of Liquids Fuel over Solid Fuels


The ultimate analysis gives an accurate chemical 1. The calorific value of liquid fuels is higher
analysis by mass of important elements of fuel than solid fuels.
such as carbon, hydrogen, oxygen, sulphur, etc. It 2. They can be stored in comparatively less
is useful in determining the quantity of air required space.
for combustion and the volume and composition of 3. Their combustion rate can be easily con-
the combustion gases. This information is required trolled.
for the calculation of flame temperature and the
4. During its burning, ash formation is almost
flue duct design, etc. Such analysis may vary from
absent and its burning is clean.
one sample to another within the group also and
5. They have higher thermal efficiency.
hence cannot be taken as a reference guide. Typical
6. They are non-corrosive and light in weight.
ultimate analyses of various coals are given in
Table 16.2. 7. They are easy to ignite and easy to shutting
of operation.
LIQUID FUELS 8. Semi-skilled labour can be employed.
Disadvatages of Liquid Fuels
Most liquid hydrocarbon fuels are a mixture of
hydrocarbons that are derived from natural pe- 1. Liquid fuels are costly.
troleum, called the crude oils. Some of the these 2. Their reserves are limited and will be
are gasolene, diesel, methanol, kerosene, and fuel depleted soon.
oil. Within each classification, there is a variety of 3. There is greater risk of fire hazard.
grades and each is the mixture of a large number of 4. Costly containers are required for their
different hydrocarbons. For convenience, the com- transportation and storage.
position of certain hydrocarbons in a fuel is repre- 5. They cannot be burn in the presence of
sented by a single hydrocarbon. For an example, insufficient air.
gasolene—a mixture of more than 40 hydrocar-
bons—is treated as octane C8H18, diesel as dodec- GASEOUS FUEL
ane C12H26, methanol as CH3OH.
The light diesel oil, furnace oil and LSHS (low Gaseous fuels are the chemically simplest among
sulphur heavy stock) are predominently used in in- the three groups. The two primary sources of
dustrial applications. The ultimate analysis of cer- gaseous hydrocarbon fuels are natural gas wells
tain liquid fuels is given in Table 1.3. and certain chemical manufacturing process.

Table 16.3

Fuel Carbon Hydrogen Sulphur Ash, etc.


Petrol (Iso-octane) 85.1 14.9 0.01 –
Commercial Petrol 85.5 14.4 0.1 –
Benzole 91.7 8.0 0.3 –
Kerosene 86.3 13.6 0.1 –
Diesel oil 86.3 12.8 0.15 –
Light diesel oil 86.2 12.4 1.5 –
Furnace oil 86.1 11.8 3 –
Residual fuel oil 88.3 9.5 1.2 1.0
534 Thermal Engineering

LPG (liquified petroleum gas) is mainly a industrial heating and power generation application.
mixture of propane and butane. LPG is a mixture It has a calorific value of about 17,000 kJ/m3.
of those hydrocarbons which are in gaseous Carbon monoxide, coal gas, biogas, methane,
phase at normal atmospheric pressure, but may be ethane, are some gaseous fuels. Table 16.4 shows
condensed to the liquid state at normal temperature, a typical analysis of natural gas by volume at
by the application of moderate pressures. atmospheric pressure and temperature.
LPG vapour is denser than air, butane is about Advantages of Gaseous Fuel
twice as heavy as air and propane about 1.5 times
as heavy as air. Consequently, the vapour may flow 1. The supply of fuel can be easily and accu-
along the ground and into drains sinking to the rately controlled.
lowest level of the surroundings and be ignited at 2. The gaseous fuels are free from impurities.
a considerable distance from the source of leakage. 3. The combustion of gaseous fuel is possible
Natural gas contains methane as the main con- with very less excess air.
stituent and other gases in small amounts. Natural 4. They produce relatively less smoke and
gas has high calorific value and it burns without pollution.
smoke. It is lighter than air and disperses into air 5. They are capable of attaining very high
easily in case of leak. temperatures.
Coal gas is obtained by the carbonization 6. The gaseous fuels are light in weight.
process of coal. The quality of coal gas depends 7. Transportation of gaseous fuels is easy.
on the quality of coal used and the temperature
Disadvantages
of carbonization. It contains mainly H2, CO and
various hydrocarbons. Its calorific value is about 1. They require larger storage capacity.
22, 500 kJ/m3. It is used in towns for street lighting 2. They are highly flammable and thus can
and domestic lighting purposes. cause fire hazards.
Producer gas is obtained by partial combustion
of coal, coke and charcoal in a mixed air stream. 16.6 CONVERSION OF VOLUMETRIC
Its manufacturing cost is low and it has a calorific ANALYSIS TO GRAVIMETRIC
value of about 6500 kJ/m3. It is used in furnaces. ANALYSIS
Water gas is a mixture of H2 and CO and is
If the volumetric analysis of a mixture is known,
produced by passing steam over incandescent coke.
it can easily be converted into gravimetric (mass)
Blast furnace gas is a by-product in the produc- analysis by using the relation
tion of pig iron in the blast furnace. It is used to
mi = ni Mi ...(16.1)
preheat the blast in the furnace. It is used as fuel in th
where mi = mass of the i constituent
metallurgical furnaces. It has calorific value of the
order of 3700 kJ/m3. ni = number of moles of the ith constituent
Coke oven gas is obtained as a by-product during Mi = molar mass (molecular weight) of the
the formation of bituminous coal. It is used for ith constituent

Table 16.4

Methane Ethane Propane Butane Nitrogen Carbon dioxide


CH4 C2 H6 C3H8 C4H10 N2 CO2
92.6% 3.6% 0.8% 0.3% 2.6% 0.1
Fuels and Combustion 535

The volume percentage of a mixture constituent 16.7 CONVERSION OF GRAVIMETRIC


is considered as mole fraction and when the molar ANALYSIS TO VOLUMETRIC
mass is multiplied to mole fraction, it results into ANALYSIS
mass of the constituent. The conversion of vol-
ume percentage to mass percentage is referred as The reverse of the above procedure, which converts
conversion of volumetric analysis to gravimetric the mass fraction to mole fraction by using the
analysis. The following steps should be taken into m
same relation as ni = i is referred as conversion
consideration for such conversion. Mi
of gravimetric analysis to volumetric analysis. It is
1. Start with volume percentage of each con-
generally applied to gaseous fuels. The following
stituent.
steps should be followed for such a conversion.
2. Multiply the volume percentage of each con-
stituent by its molecular weight. 1. Start with the percentage of mass (mass
3. Take the sum of products of the step 2 above. fraction) of each constituent.
4. Find the percentage analysis by mass as 2. Divide mass fraction of each constituent by
mi its molecular mass.
¥ 100 . 3. Take the sum of quotients of the step 2 above.
S mi
n
The following example illustrates the conversion 4. Obtain the percentage volume as i ¥ 100 .
S ni
of volume percentage to mass percentage of
The following example illustrates the conversion
constituents in the gas mixture.
of gravimetric analysis to volumetric analysis.
Example 16.1 The percentage analysis of a gas by
Example 16.2 A gaseous mixture contains CO2 =
volume is given as
16%, CO= 1%, O2 = 8% and N2 = 75% by mass. Convert
CO2 = 5.5% CO = 38.3%
this gravimetric analysis to volumetric analysis.
CH4 = 0.4% O2 = 0.1%
H2 = 52.8% N2 = 2.9% Solution According to the steps given above, the
Obtain the percentage analysis by mass. conversion is taken into tabular form, as

Solution The solution is presented in the following Consti- Mass Mole- Number of Volume
tabular form. tuent fraction cular moles, percentage
mi mass Mi mi ni
ni =
Consti- % Mole- Mass % Mass in
Mi S ni
tuent Volume cular m i = ni Mi mixture = CO2 16 44 0.364 10.93%
ni weight mi CO 1 28 0.035 1.040%
¥ 100
Mi S mi O2 8 32 0.25 7.50%
CO2 5.5 44 242.0 16.02% N2 75 28 2.68 80.53%
CO 38.3 28 1072.4 70.98% S ni = 3.327 100%
CH4 0.4 16 6.4 0.423%
O2 0.1 32 3.2 0.211%
H2 52.8 2 105.6 6.99 COMBUSTION
N2 2.9 28 81.2 5.37
S mi = 1510.8 100% Combustion Terminology
(i) The components of fuel that exist
before the combustion reaction are called the
reactants.
536 Thermal Engineering

(ii) The components that exist after 1


combustion reaction are called the products. Both 1H2 + O2 Æ H2O ...(16.2)
2
the reactants and products are pure substances Introducing the molar masses,
before and after the chemical reaction and we can
2 kg H2 + 16 kg O2 Æ 18 kg H2O
calculate the properties change.
Hence, the mass of the reactants is equal to the
Chemical reaction
Reactants æææææææææ Æ Products mass of products. When using the number of moles,
Heat Release
1
(iii) The combustion is an exo- 1 mole H2 + mole O2 Æ 1 mole of H2O
2
thermic process in which the rapid oxidation of
Thus the number of moles of the product differs
combustible elements of a fuel takes place with
from the number of moles of reactants.
release of heat energy. The combustion process is
a fuel-burning process in the presence of oxygen. (ii) All the combustion equations
The combustion is called the complete combustion follow the ideal gas equation and general gas
when all the carbon present in the fuel is burned to equations:
carbon dioxide, all the hydrogen is burned to water pV = n Ru T
vapour, all the sulphur is burned to sulphur dioxide where n = number of moles of a gas
and all other combustible elements are fully oxi-
V = molar volume, m3/kmol
dised. When these conditions are not fulfilled or the
Ru = Universal gas constant
constituents of fuel are not fully oxidised then com-
= 8314.3 J/kmol ◊ K
bustion is called incomplete combustion.
(iii) It states that molecular mass
(iv) The intimate contact of
of all perfect gases occupy the same volume under
fuel and oxygen cannot initiate the combustion pro-
similar conditions of pressure and temperature. At
cess. The fuel must be brought to its ignition tem-
NTP, the molar volume
perature to start the combustion. The ignition tem-
perature of a fuel element is the lowest temperature nRuT
V0 =
at which it starts burning. The minimum ignition p
temperature for gasolene is 260°C, for carbon it is (1 kmol) ¥ (8.314 kJ/kmol ◊ K) ¥ (273 K)
=
400°C, for hydrogen it is 580°C, for carbon monox- (101.325 kPa)
ide it is 610°C and for methane it is 630°C. 3
= 22.41 m /kmol

Combustion
There are certain physical laws on which the (a) Combustion of Solid and Liquid Fuels
analysis of combustion process is based. The soilid and liquid fuels usually consist of carbon,
(i) During a chemi- hydrogen and sulphur. These constituents burn with
cal reaction, the mass is conserved, that is, the mass rapid oxidation to produce heat. The products of
of the reactants equals the mass of the products. combustion may be CO, CO2, H2O and SO2. The
The total mass of each chemical element must be basic combustion reactions for these three elements
same on both sides of the equation even though the are the following:
elements exist in different chemical compositions (i)
in reactants and products. However, the number of
moles of products may differ from the number of 2C + O2 Æ 2CO
moles of reactants. For example,
Fuels and Combustion 537

Introducting the molar masses; 1


1 volume CO + volume O2 Æ 1 volume of CO2
(2 ¥ 12) kg C + 32 kg O2 Æ 56 kg CO 2 ...(16.7)
4 7
or 1 kg C + kg O2 Æ kg CO ...(16.3) It can be interpreted as one volume of carbon
3 3 1
It indicates that 1 kg of carbon requires 4/3 kg monoxide requires volume of oxygen to produce
2
of oxygen to produce 7/3 kg of carbon monoxide. 1 volume of carbon dioxide.
(ii) (ii) 2

C + O2 Æ CO2 1
H2 + O2 Æ H2O ...(16.8)
Using molar masses; 2
12 kg C + 32 kg O2 Æ 44 kg CO2 ...(16.4) On mole basis
8 11 1
or 1 kg C + kg O2 Æ kg CO2 1 mole H2 + mole O2 Æ 1 mole H2O
3 3 2
8 1
It means 1 kg of carbon requires kg of oxygen or 1 volume H2 + volume O2 Æ 1 volume H2O
11 3 2
to produce kg of carbon dioxide.
3 1
Thus, 1 volume of H2 combines with volume
2
(iii)
of O2 to form 1 volume of H2O.
2CO + O2 Æ 2CO2 On mass basis,
Using molar masses; 2 kg H2 + 16 kg O2 Æ 18 kg H2O
56 kg CO + 32 kg O2 Æ 88 kg CO 1 kg H2 + 8 kg O2 Æ 9 kg H2O ...(16.9)
4 11 Thus, 1 kg H2 requires 8 kg of O2 to form 9 kg
or 1 kg CO + kg O2 Æ kg CO2 ...(16.5)
7 7 of water vapour.
4
That is, 1 kg of carbon monoxide requires kg (iii)
7
of O2 to produce
11
kg of carbon dioxide. CH4 + 2O2 Æ CO2 + 2H2O
7 Mole basis:
( ) 1 mole CH4 + 2 mole O2 Æ 1 mole CO2
S + O2 Æ SO2 + 2 mole H2O ...(16.10)
Molar mass basis It means 1 mole of methane gas combines with 2
32 kg S + 32 kg O2 Æ 64 kg SO2 moles of oxygen gas to produce 1 mole of CO2 and
2 moles of water vapour.
or 1 kg S + 1 kg O2 Æ 2 kg SO2 ...(16.6)
Mass basis:
Thus 1 kg sulphur requires 1 kg O2 to produce
2 kg SO2. 16 kg CH4 + 64 kg O2 Æ 44 kg CO2 + 36 kg H2O
11 9
(b) Combustion of Gaseous Fuel 1 kg CH4 + 4 kg O2 Æ kg CO2 + kg H2O
4 4
When dealing with gases, the volumes or number
...(16.11)
of moles may be substituted for gas elements.
That is, 1 kg of CH4 gas reacts with 4 kg of O2 to
(i)
11 9
produce kg of CO2 and kg of H2O.
2 mole of CO + 1 mole of O2 Æ 2 mole of CO2 4 4
538 Thermal Engineering

( )

C2H4 + 3O2 Æ 2CO2 + 2H2O


1 mole C2H4 + 3 mole O2 Æ 2 mole
CO2 + 2 mole H2O ...(16.12)
1 m of ethylene completely burns with 3 m3
3

oxygen to produce 2 m3 of CO2 and 2 m3 of H2O.


( )
C8H18 + 12.5O2 Æ 8CO2 + 9H2O
1 mole C8H18 + 12.5 mole O2 Æ 8 mole
CO2 + 9 mole H2O ...(16.13)
The above equations are helpful in calculating
the amount of oxygen required to burn the fuel
completely.
The combustion equation for any hydrocarbon
(say, C10H22) can be written as
C10H22 + a O2 Æ b CO2 + d H2O
Equating C, H, O on both sides, we get
b = 10
2d = 22 or d = 11
2a = 2b + d or a = 15.5
Then the equation becomes
C10H22 + 15.5 O2 Æ 10 CO2 + 11 H2O Fig. 16.1

Combustion
The objective of good combustion is to release Instead of supplying only oxygen for the chemical
all of the heat in the fuel. This is accomplished by reactions, atmospheric air is supplied. Atmospheric
controlling the three T’s of combustion which are air is the mixture of mainly O2 and N2 with small
(1) temperature high enough to ignite and maintain traces of argon, carbon dioxide, water vapour, etc.
ignition of the fuel, (2) turbulence for intimate mix- The composition of dry air on molar basis is given
ing of the fuel and oxygen, and (3) time sufficient by 78.09% of N2, 20.95% of O2, 0.93% of argon
for complete combustion. and 0.03% of CO2. The molecular weight of this
Too much or too little fuel with the available mixture is 28.97 kg/kmol. For all calculations, argon
combustion air may potentially result in unburned and carbon dioxide are considered as an additional
fuel and carbon monoxide generation. A very spe- nitrogen, because they are also inert gases like N2
cific amount of O2 is needed for perfect combus- and appear in very small quantities. Accordingly,
tion and some additional (excess) air is required for air is considered to consist of 79% of nitrogen and
ensuring complete combustion. However, too much 21% of oxygen by volume or on a molar basis.
excess air will result in heat and efficiency losses.
With this assumption, the molar (volume) ratio of
Figure 16.1 shows complete, good and incomplete
combustion, respectively. 0.79
nitrogen to oxygen is = 3.76, i.e., air supplied
0.21
Fuels and Combustion 539

for combustion contains 3.76 mole of nitrogen with Ê8 ˆ


each mole of oxygen. Total oxygen required = Á C + 8H + S˜ kg
Ë3 ¯
The mass composition can be derived from this
molar composition by using relation m (mass) = All the oxygen supplied with air must be con-
n (moles) ¥ M (mol. weight). Thus, it is found that sumed by the fuel elements to form the completely
air consists of 23% O2 and 77% N2 on mass basis. oxidised products. No free oxygen should appear in
The composition of air taken for calculations is the products. If (O) kg of oxygen is already present
given in Table 16.5. in the fuel then minimum mass of oxygen required
for complete combustion
8
Table 16.5 = C + 8H + S - O
3
Molar basis Mass basis 8 Ê Oˆ
= C + 8Á H - ˜ + S ...(16.14)
0.21 mole O2 + 0.79 mole 0.23 kg O2 + 0.77 kg N2 3 Ë 8¯
N2 = 1 mole of air = 1 kg of air \ minimum mass of air required becomes,
1 mole O2 + 3.76 mole N2 1 kg O2 + 3.347 kg N2 =
100 È 8 Ê Oˆ ˘
= 4.76 mole of air 4.347 kg of air mth = Í C + 8 ÁË H - ˜¯ + S˙ ...(16.15)
Molecular weights 23 Î 3 8 ˚
MO2 = 32, MN2 = 28, Mair = 28.97 (kg/kmol) since air contains 23% of oxygen by mass.

16.10 AMOUNT OF AIR REQUIRED Complete


FOR COMBUSTION Gaseous Fuel
Consider 1 m3 of gaseous fuel consisting of CO, H2,
Complete CH4, C2H4, etc. Let the volume of these gases in the
mixture be CO m3, H2 m3, CH4 m3 and C2H4 m3,
The minimum amount of air that supplies just
respectively. From Eqs. (16.7), (16.8), (16.10)
sufficient oxygen for the complete combustion of
and (16.12), respectively, the minimum volume of
all the carbon, hydrogen and sulphur present in
oxygen required for complete combustion
the fuel is called the theoretical amount of air or
= 0.5 CO + 0.5 H2 + 2 CH4 + 3 C2H4 (m3)
stoichiometric air. It is also called the chemically
...(16.16)
correct amount of air required for complete 3
combustion. If fuel itself contains O2 m of oxygen then vol-
ume of oxygen required for complete combustion
For 1 kg of fuel consisting of carbon, hydrogen
and sulphur, the minimum amount of O2 required = 0.5 CO + 0.5 H2 + 2 CH4 + 3 C2H4 O2 (m3)
can be obtained with the help of basic combustion. ...(16.17)
Equations (16.4), (16.6), (16.9) respectively as The atmospheric air contains 21% of oxygen
by volume. Hence, the minimum volume of air
8
1 kg of carbon requires kg of O2 required for complete combustion is
3
100
Ê8 ˆ Vth = ¥ [0.5 CO + 0.5 H2 + 2 CH4
\ C kg of carbon requires Á C˜ kg of O2 21
Ë3 ¯ + 3 C2H4 O2] (m3) ...(16.18)
Similarly,
S kg of sulphur requires (S) kg of O2.
H kg of hydrogen requires (8 H) kg of O2. In actual combustion process, the amount of air
supplied is either greater or less than the theoretical
540 Thermal Engineering

amount. The amount of air actually supplied is usu- Air–fuel ratio can also be interpreted on mole
ally expressed in terms of per cent of stoichiometric basis as the ratio of number of moles of air to the
air. number of moles of fuel. But the former definition
The amount of air in excess, which is necessary is normally used.
to burn fuel completely is called the excess air. This Calculation for the air–fuel ratio for CH4 on
can be expressed as a per cent excess air in terms of mass basis by Eq. (16.21);
stoichiometric or theoretical amount of air as Mass of air
Percent excess air = number of moles of air ¥ Mair
Actual air used - Stoichiometric air required = 2 ¥ (1 mole of O2 + 3.76 mole N2) ¥ 28.97
=
Stoichiometric air required = 9.52 mole air ¥ 28.97 = 275.8 kg
...(16.19) The mass of fuel, CH4 = 1 mole ¥ MCH4 = 16 kg
Similarly, if the actual amount of air supplied 278.8 kg
is less than the theoretical amount then it can be Thus A/F = = 17.237
16 kg
expressed as per cent deficiency of air. For an ex-
ample, consider the complete combustion of meth-
ane with 150% of theoretical air (50% excess air). ANALYSIS OF FLUE GASES
The balanced chemical equation is
CH4 + (1.5) ¥ 2 (O2 + 3.76 N2) Æ CO2 When the volumetric analysis of products of
combustion is known. then the actual air–fuel ratio
+ 2H2O + O2 + 11.28 N2 ...(16.20)
can be obtained by using the following methods:
Here the amount of air per mole of fuel is
1.5 times the theoretical amount determined by 1. Carbon balance method
Eq. (16.13). 2. Hydrogen balance method
3. Carbon- Hydrogen balance method

It is the ratio of actual air–fuel ratio to the theo-


retical air–fuel ratio for complete combustion. The If all carbon present in the fuel burns completely
reactants form a rich mixture when the equivalence without leaving traces in flue gases then carbon
ratio is less than unity. Accordingly, for excess air balance method is used to obtain the actual amount
supplied with fuel, the equivalence ratio is always of air used to form the dry products of combustion.
greater than unity. In this method, the mass of carbon present in the
fuel is balanced with mass of carbon present in the
dry flue gases. It is a quite accurate method.
Let C be the mass of carbon in 1 kg of fuel; and
A frequently used quantity in the analysis of the
CO2 and CO are the masses of carbon dioxide and
combustion processes is the air–fuel ratio. It is
carbon monooxide, respectively, formed per kg of
usually expressed on mass basis and is defined as
flue gases. Then
the ratio of mass of the air to the mass of the fuel 3
during combustion process. It is designated as A/F. 1 kg of CO2 contains kg of carbon (∵ 1 kg
11
Mass of air 11
A/F = ...(16.21) of carbon produces kg of CO2), and
Mass of fuel 3
3
Simply A/F ratio is the equivalent amount of air 1 kg of CO contains kg of carbon (∵ 1 kg of
7
required for burning of 1 kg of fuel completely. 7
carbon produces kg of CO);
3
Fuels and Combustion 541

Then the mass of Carbon per kg of dry flue gases To find


Ê3 3 ˆ (i) Theoretical air–fuel ratio on molar basis,
= Á CO 2 + CO˜ kg/kg of flue gases
Ë 11 7 ¯ (ii) Theoretical air–fuel ratio on mass basis.

The mass of dry flue gases per kg of fuel burnt Assumptions


mdry flue gases = (i) The air contains 23% O2 and 77% N2 by mass.
(ii) Complete combustion.
Mass of carbon present in 1 kg of fuel
Analysis
Mass of carbon presen nt in 1 kg of flue gases
(i) Pure carbon
...(16.21) Combustion reaction of carbon
The mass of vapour formed per kg of fuel during C + O2 Æ CO2
combustion 1 mole of C combines with 1 mole of O2 to form
mH2O = 9 ¥ H2 present per kg of fuel 1 mole of CO2. But one mole of O2 associates
Then the total mass of dry flue gases per kg of with 3.76 moles of nitrogen and 4.76 moles of air.
fuel burnt Thus
mflue gases = mdry flue gases + mH2O ...(16.22) moles of air with 1 mole O2
= 4.76 moles of air per mole of O2
A/F ratio on molar basis
Number of moles of air
When some solid carbon is suspected in combustion =
Number of moles of carbon
products, then the carbon-balance method cannot 4.76
give an accurate mass of air used in combustion. In = = 4.76
1
such a situation, the hydrogen method is used. Minimum mass of air required for complete com-
In the hydrogen-balance method, the mass of bustion of 1 kg of carbon
hydrogen in the fuel composition is balanced with 100 Ê 8 ˆ
= ¥ kg per kg of carbon˜
the mass of hydrogen per kg of flue gases. 23 ÁË 3 ¯
100 8
B M = ¥ ¥ 1 = 11.59 kg/kg of C
23 3
If the Orasat apparatus cannot measure the percent- Theoretical air–fuel ratio
age of nitrogen accurately, the masses of carbon Mass of air 11.59 kg
= = = 11.59
and hydrogen are balanced in fuel composition Mass of carbon 1 kg
and products of combustion. It is a very accurate (ii) Pure hydrogen
method. Combustion reaction of hydrogen
Calculate the theoretical air to fuel H2 + 1 O2 Æ H2O
2
ratio on molar and mass basis for the following fuels:
1 mole of H2 combines with 1 mole of O2 to
(a) Pure carbon, (b) pure hydrogen (c) petrol 2
(C8.5H18.4 ) (d) Heptane (C7H16 ) (e) Methanol (CH3OH) form 1 mole of H2O. Thus
moles of air = 1 mole of O2 ¥ 4.76 moles of air
Solution 2
= 2.38 moles of air/mole of H2
Given The fuels: A/F ratio on molar basis
(i) Pure carbon (ii) Pure hydrogen Number of moles of air
(iii) Petrol (C8.5 H18.4 ) (iv) Heptane (C7 H16 ) =
Number of moles of hydrogen
(v) Methanol (CH3OH) 2.38
= = 2.38
1
542 Thermal Engineering

Minimum mass of air required for complete com- Mass of 1 mole of heptane (C7 H16 )
bustion of 1 kg of hydrogen = 7 ¥ 12 + 16 ¥ 1 = 100 kg
100 Mass of 52.36 moles of air
= ¥ (8 kg per kg of H 2 )
23 = 52.36 ¥ 28.97 =1516.87 kg
100 Theoretical air–fuel ratio
= ¥ 8 ¥ 1 = 34.78 kg/kg of H2
23 Mass of air 1516.87 kg
= =
Theoretical air–fuel ratio Mass of Heptane 100 kg
Mass of air 34.78 kg = 15.168
= =
Mass of hydrogen 1 kg (v) Methanol (CH3 OH)
= 34.78 Combustion reaction for methanol
(iii) Petrol (C8.5 H18.4 ) CH3OH + 1.5O2 Æ CO2 + 2H2O
Combustion reaction for petrol 1 mole of methanol combines with 1.5 moles of
C8.5H18.4 + 13.1 O2 Æ 8.5 CO2 + 9.2 H2O O2 to form 1 moles of CO2 and 2 mole of H2O.
Thus,
1 mole of petrol combines with 13.1 moles of O2
to form 8.5 mole of CO2 and 9.2 mole of H2O. moles of air = 1.5 moles of O2 ¥ 4.76 moles
Thus = 7.14 moles of air/mole of methnol
moles of air = 13.1 moles of O2 ¥ 4.76 moles A/F ratio on molar basis
= 62.356 moles of air/mole of petrol Number of moles of air
=
A/F ratio on molar basis Number of moles of methnol
Number of moles of air 7.14
= = = 7.14
Number of moles of petrol 1
Mass of 1 mole of methanol (CH3OH)
62.356
= = 62.356 = 1 ¥ 12 + 1 ¥ 16 + 4 ¥ 1 = 32 kg
1
Mass of 7.14 moles of air
Mass of 1 mole of petrol (C8.5H18.4 )
= 7.14 ¥ 28.97 = 206.845 kg
= 8.5 ¥ 12 + 18.4 ¥ 1 = 120.4 kg
Theoretical air–fuel ratio
Mass of 62.356 moles of air
Mass of air 206.845 kg
= 62.356 ¥ 28.97 =1806.45 kg = =
Mass of methanol 32 kg
Theoretical air–fuel ratio
= 6.464
Mass of air 1806.45 kg
= =
Mass of petrol 120.4 kg Example 16.4 Determine the air–fuel ratio and the
= 15.00 theoretical amount of air required by mass for complete
(iv) Heptane (C7H16) combustion of a fuel containing 85% of carbon, 8%
Combustion reaction for heptane of hydrogen, 3% of oxygen, 1% of sulphur and the
C7H16 + 11 O2 Æ 7 CO2 + 8 H2O remaining as ash. If 40% of excess air is used, what is the
1 mole of heptane combines with 11 moles of volume of air at 27°C and 1.05 bar pressure? Does this
O2 to form 7 moles of CO2 and 8 moles of H2O. represent per kg of fuel?
Thus,
Solution
moles of air = 11 moles of O2 ¥ 4.76 moles
= 52.36 moles of air/mole of heptane Given The composition of fuel by mass. For 1 kg of
A/F ratio on molar basis fuel
Number of moles of air C = 0.85 kg H2 = 0.8 kg
=
Number of moles of heptane O2 = 0.03 kg S = 0.01 kg
52.36 Ash = 0.03 kg
= = 52.36
1 Excess air = 40% T1 = 27°C = 300 K
p1 = 1.05 bar = 105 kPa
Fuels and Combustion 543

To find (i) Theoretical air required for complete combustion


(i) Minimum amount of air required, 100 È 8 Ê Oˆ ˘
(ii) Air–fuel ratio by mass, and = Í C + 8 Á H - ˜ + S˙
23 Î 3 Ë 8 ¯ ˚
(iii) Volume of air required with 40% excess air.
100 È 8 ˘
= ¥ ¥ 0.8 + 8 ¥ 0.12 + 0.0 ˙
Assumptions 23 ÍÎ 3 ˚
(i) The air contains 23% O2 and 77% N2 by mass.
= 13.45 kg/kg of fuel
(ii) Complete combustion.
(ii) Number of moles of C;
(iii) Specific gas constant for air as 0.287 kJ/kg ◊ K.
m 0.8
nC = = = 0.067 mole
Analysis M 12
(i) Minimum mass of air required for complete Number of moles of hydrogen;
combustion, 0.12
nH2 = = 0.12 mole
1
100 È 8 Ê Oˆ ˘
mth = Í C + 8 Á H - ˜ + S˙ Combustion reaction with theoretical air
23 Î 3 Ë 8¯ ˚ 0.067 C + 0.12 H + x(O2 + 3.76 N2) Æ a CO2
100 È 8 Ê 0.03 ˆ ˘ + bH2O + 3.76N2
= ¥ Í ¥ 0.85 + 8 ¥ Á 0.08 - + 0.01˙
23 Î 3 Ë 8 ˜¯ ˚ The element balance yields to
= 12.55 kg/kg of fuel a = 0.067 mole,
(ii) Air–fuel ratio by mass = 12.55 : 1 and b = 0.06 mole
(iii) Since air supplied is 40% in excess, the actual and 2x = 2a + b or x = 0.097 mole
amount of air supplied is equal to Total moles of products
ma = 1.4 ¥ 12.55 = 17.57 kg. = 0.067 + 0.06 + 3.76 ¥ 0.097
The volume occupied by air can be calculated as = 0.492 mole
follows: Hence volumetric analysis
pV = ma RT 0.067 ¥ 100
CO2 = = 13.62%
105V = 17.57 ¥ 0.287 ¥ 300 K 0.492
or, V = 14.407 m3 0.06 ¥ 100
H2O = = 12.2%
0.492
Example 16.5 The gravimetric analysis of a sample 3.76 ¥ 0.097 ¥ 100
of coal gives 80% of carbon, 12% of H2 and 8% of ash. N2 = = 74.13%
0.492
Calculate the theoretical air required and analysis of
products by volume. Example 16.6 A producer gas has the following
percentage composition by volume, H2 =15%, CH4 =
Solution 2%, CO = 20%, CO2 = 6%, O2 = 3% and N2 = 54%.
Given Gravimetric analysis of a sample of coal If 50% excess air is supplied for combustion, determine
C = 0.8, H2 = 0.12 the volume of air supplied per m3 of gas and volumetric
analysis of the combustion products.
ash = 0.08
To find Solution
(i) Theoretical air required, and Given A producer gas with following percentage
(ii) Analysis of products by volume. composition
Analysis Considering 1 kg coal, H2 = 15% CH4 = 2%
mass of carbon = 0.8 kg, and CO = 20% CO2 = 6%
mass of hydrogen = 0.12 N2 = 54% O2 = 3%
Excess air = 50%
544 Thermal Engineering

To find 0.28
CO2 = ¥ 100 = 14.31%
(i) Volume of air supplied per m3 of producer gas. 1.9565
(ii) Analysis of dry products of combustion by
0.0925
volume. O2 = ¥ 100 = 4.73%
1.9565
Analysis Minimum volume of air required, Eq. (16.18) 1.584
N2 = ¥ 100 = 80.96%
100 1.9565
Vth = ¥ [0.5 CO + 0.5 H2 + 2 CH4
21
+ 3 C2H4 O2] (m3) Example 16.7 A gas engine is supplied with natural
100 gas of the following composition. CH4 = 93%, C2 H6 =
= ¥ [(0.5 ¥ 0.2 + 0.5 ¥ 0.15 + 2
21 3%, N2 = 3%, CO = 1%. If the A/F ratio is 30 by volume,
¥ 0.02) – 0.03] (m3) calculate the analysis of the dry products of combustion.
3
= 0.88095 m It can be assumed that the stoichiometric A/F ratio is less
Actual volume of air supplied than 30.
Vact = 1.5 ¥ 0.88095 = 1.3214 m3
Excess air supplied Solution
= Vact Vth = 1.3214 0.88095 Given A natural gas with composition
= 0.44045 m3 CH4 = 93% C2H6 = 3% N2 = 3%
The constituents of flue gases are CO2, O2, and N2 CO = 1% A/F = 30 by volume
Excess oxygen supplied
To find Analysis of dry products of combustion by
21 volume.
= ¥ 0.44045 = 0.0925 m3
100
Volume of O2 in flue gases = 0.0925 m3 Analysis It is mentioned that the stoichiometric air–
Volume of N2 in fuel = 0.54 m3 fuel ratio is less than 30. Thus the excess air is supplied
with fuel. The combustion equation
Volume of N2 in air supplied
79 0.93 CH4 + 0.03 C2H6 + 0.01 CO + 0.03 N2 + 30 (0.21
= ¥ Volume of actual air supplied O2 + 0.79 N2) Æ aCO2 + bH2O + c O2 + d N2
100
79 The element balance yields
= ¥ 1.3214 = 1.044 m3
100 C: a = 0.93 + 0.03 ¥ 2 + 0.01 or a = 1
Total volume of N2 in flue gases H2: 2b = 0.93 ¥ 4 + 0.03 ¥ 6,
= 0.54 + 1.044 = 1.584 m3 or b = 1.95
Volume of CO2 in the fuel = 0.06 m3 O2: 2a + b + 2c = 0.01 + 30 ¥ 0.21
Volume of CO2 produced by combustion of 0.2 m3 of or c = 1.18
CO = 0.2 m3 N2: 2d = 0.03 ¥ 2 + 30 ¥ 0.79
Volume of CO2 produced by combustion of 0.02 m3 or d = 23.73
of CH4 = 0.02 m3 Total moles of dry products are 1 + 1.18 + 23.73
Total volume of CO2 in flue gases = 25.91 kmol
= 0.06 + 0.2 + 0.02 = 0.28 m3 The analysis of dry products by volume
Total volume of flue gases 1
= Volume of O2 + volume of N2 + volume of CO2 CO2 = ¥ 100 = 3.86%
25.91
= 0.0925 + 1.584 + 0.28 = 1.9565 m3 1.18
The analysis of dry products by volume O2 = ¥ 100 = 4.55%
25.91
% volume of element A 23.73
Volume of element A N2 = ¥ 100 = 91.56%
= ¥ 100 25.91
Total volume of flue gases
Fuels and Combustion 545

Example 16.8 A sample of dry anthracite has the O2 required by 0.005 kg of S


following composition by mass: = 0.005 ¥ 1 = 0.005 kg
C = 90% H = 3% O = 2.5% SO2 produced = 0.005 ¥ 2 = 0.01 kg
N = 1% S = 0.5% ash = 3% The total O2 required
Calculate = 2.4 + 0.24 + 0.005 = 2.645 kg
(a) Stoichiometric air–fuel ratio, O2 already present with fuel = 0.025 kg
(b) the actual air–fuel ratio and dry and wet analysis Thus, net O2 required for combustion
of products of combustion by mass and by volume = 2.645 – 0.025 = 2.62 kg
when 20% excess air is supplied. N2 required = 3.347 ¥ 2.62 kg O2 = 8.77 kg
Theoretical mass of air required
Solution = 2.62 + 8.77 = 11.39 kg
Given A sample of dry anthracite with mass composi- Stoichiometric A/F ratio
tion
Mass of air 11.39 kg
C = 90% H = 3% O = 2.5% = =
Mass of fuel 1 kg
N = 1% S = 0.5% ash = 3% = 11.39
To find (ii) Actual air–fuel ratio with 20% excess air
(i) Stoichiometric air–fuel ratio, and = 1.2 ¥ stoichiometric A/F
(ii) Actual air–fuel ratio, dry and wet analysis of = 1.2 ¥ 11.39 = 13.67
products of combustion by mass and volume O2 supplied = 1.2 ¥ 2.62 kg = 3.144 kg
when 20% excess air is supplied. Excess O2 supplied
= 3.144 – 2.62 = 0.524 kg
Assumptions
N2 supplied = 1.2 ¥ 8.77 = 10.524 kg
(i) Each mole of O2 is accompanied by 3.347 kg N2.
Table 16.8.1 on next page shows dry and wet analysis
(ii) Nitrogen is an inert gas.
of product of combustion.
(iii) Combustion is complete.
Analysis Example 16.9 Determine the air–fuel ratio on both
(i) For complete combustion of 1 kg of anthracite, mass and molar basis for the complete combustion of
the chemical equations are the following: octane (C8 H18), with (a) theoretical amount of air, (b)
150% theoretical air.
Carbon C + O2 Æ CO2
8 11 Solution
1 kg C + kg O2 Æ kg CO2
3 3
Given Octane C8H18 burns completely with
8
0.9 kg C require 0.9 ¥ O2 = 2.4 kg O2 (i) theoretical
3
(ii) excess air.
11
Carbon dioxide produced = 0.9 ¥ = 3.3 kg
3 To find Air–fuel ratio.
1
Hydrogen H2 + O2 Æ H2O Assumptions
2
(i) Each mole of oxygen is accompanied with 3.76
1 kg H2 + 8 kg O2 Æ 9 kg H2O moles of nitrogen.
O2 required for 0.03 kg of H2 (ii) Nitrogen is an inert gas.
= 0.03 ¥ 8 = 0.24 kg (iii) Complete combustion of octane.
H2O produced = 0.03 ¥ 9 = 0.27 kg
Sulphur S + O2 Æ SO2 Analysis
1 kg S + 1 kg O2 Æ 2 kg SO2 (i) The chemical reaction for complete combustion
C8H18 + a O2 Æ b CO2 + c H2O
546 Thermal Engineering

Table 16.8.1

Products Mass mi kg/kg mi mi


Wet analysis Dry analysis
m1 m2
CO2 3.30 22.559% 22.98%
H2O 0.27 1.846% ––
SO2 0.01 0.068% 0.07%
O2 0.524 3.582% 3.65%
N2 10.524 71.944% 73.30%

m1 = Âm
wet
i = 14.628 kg 100.00% 100.00%

m2 = Âm
Dry
i = 14.358 kg

Table 16.8.2

Products Mass mi Mol. weight mi Wet Analysis Dry Analysis


Mole ni = ni ni
kg/kg Mi kg/kmol Mi
n2 n2

CO2 3.3 44 0.075 15.55% 16.05%


H2O 0.27 18 0.015 3.11% –
SO2 0.01 64 0.00015 0.032% 0.033%
O2 0.524 32 0.0163 3.38% 3.50%
N2 10.524 28 0.376 77.96% 80.43%

n1 = Ân
Wet
i = 0.4823 100.00% 100.00%

n2 = Ân
Dry
i = 0.4673

Using mass conservation, we get The mass of fuel,


a = 12.5, b = 8, and c = 9, and mf = nf Mf = 1 ¥ 114
Thus, the chemical equation is = 114 kg
C8H18 + 12.5 O2 Æ 8 CO2 + 9 H2O The mass of air,
ma = na Ma
On the molar basis
= 59.5 kmol ¥ 28.97 kg/kmol
1 kmol C8H18 + (12.5 ¥ 4.76) kmol of air Æ
= 1723.715 kg
8 kmol CO2 + 9 kmol H2O
The air–fuel ratio on the mass basis
The moles of air required, Mass of air 1723.715
na = 12.5 ¥ 4.76 = 59.5 kmol/ kmol of fuel A/F = = = 15.12
Mass of fuel 1 14
Number of kmol of air 59.5 (ii) 150% theoretical air
A/F = =
Number of kmol of fuel 1 Actual moles of air required
= 59.5 = 1.5 ¥ theoretical moles
On the mass basis, using, mi = ni Mi = 1.5 ¥ 59.5 = 89.5 kmol/ kmol of fuel
Fuels and Combustion 547

(ii) The mass of products of combustion


Thus, A/F ratio on molar basis is 89.25.
Similarly, on mass basis A/F Mass of CO2 formed
= 1.5 ¥ 15.12 = 22.68 =
11
kg per kg of C ¥ 0.78 kg
3
Example 16.10 A steam boiler uses pulverised coal = 2.86 kg/kg of fuel
in the furnace. The ultimate analysis of coal by mass is Mass of H2O formed
given as = 9 kg per kg of H2 = 9 ¥ 0.03
C = 78% H2 = 3% = 0.27 kg/kg of fuel
O2 = 3%, S = 1% Mass of SO2 formed
Ash = 10% moisture = 5% = 2 kg per kg of S = 2 ¥ 0.01
Excess air supplied is 30%. Calculate the mass of air = 0.02 kg/kg of fuel
to be supplied and mass of products of combustion per kg Mass of N2 associated with air
of coal burnt.
= 0.77 ¥ mtotal = 0.77 ¥ 17.465
Solution = 13.448 kg/kg of fuel
The mass of oxygen with combustion products
Given The ultimate analysis of coal
= Mass of oxygen with excess air
C = 78% H2 = 3% mass of air in fuel
O2 = 3% S = 1% = 0.23 ¥ mex 3% in fuel on mass basis
Ash = 10% moisture = 5% = 0.23 ¥ 4.03 0.03
Excess air supplied = 30% of theoretical air = 0.897 kg/kg of fuel
To find
Example 16.11 The gravimetric analysis of coal
(i) Mass of air supplied for combustion gives 80% of carbon, 8% of hydrogen, 4% of moisture
(ii) Mass of products of combustion per kg of fuel and 8% of ash. Actual air supplied is 18 kg per kg of
burnt coal. Calculate the theoretical amount of air required,
Analysis if 80% of carbon is burned to CO2 and the remaining
to CO. Also determine the volumetric composition of dry
(i) Mass of air supplied for combustion products of combustion.
The theoretical amount of air required for
complete combustion of 1 kg of fuel Solution

100 È 8 Ê Oˆ ˘ Given The gravimetric analysis of coal as


mth = Í C + 8 Á H - ˜ + S˙ kg/kg of fuel
23 Î 3 Ë 8 ¯ ˚ C = 80% H2 = 8% moisture = 4%
Ash = 8% ma = 18 kg/kg of coal
100 È 8 Ê 0.03 ˆ ˘
= Í ¥ 0.78 + 8 ¥ Á 0.03 - ˜ + 0.01˙
23 Î 3 Ë 8 ¯ ˚
To find

= 13.435 kg/kg of fuel (i) Theoretical air required


Excess amount of air supplied, (ii) Volumetric analysis of dry combustion products
mex = 0.3 ¥ mth = 0.3 ¥ 13.435 Analysis Since 80% of carbon burns to CO2, the mass
= 4.03 kg/kg of fuel of carbon burns to
Total amount of air supplied for combustion of CO2 = 0.8 ¥ 0.8 = 0.64 kg
fuel; Remainder carbon burns to
mtotal = mth + mex = 13.435 + 4.03 CO = 0.8 – 0.64 = 0.16 kg
= 17.465 kg/kg of fuel
548 Thermal Engineering

The minimum oxygen required for formation of CO2 (c) the A/F ratio, wet and dry analysis by volume of
and H2O the exhaust gas for a mixture strength of 120%.
8
= C + 8H Solution
3
8
= ¥ 0.64 + 8 ¥ 0.08 = 2.346 kg Given The burning of ethyl alcohol (C2 H5 OH)
3
The oxygen required for formation of CO To find
4 (i) Stoichiometric air–fuel ratio,
= ¥ 0.16 = 0.213 kg
3 (ii) A/F ratio, the dry and wet analysis for products of
Total oxygen required combustion for mixture strength of 90%,
= 2.346 + 0.213 = 2.56 kg (iii) A/F, ratio, the dry and wet analysis by volume of
products of combustion for mixture strength of
The minimum mass of air required
120%.
100
= ¥ 2.56 = 11.13 kg/kg of coal Analysis
23
A/F ratio = 11.13:1 (i) The combustion reaction with minimum air
The dry products of combustion per kg of coal burnt Combustion reaction for methanol;
11 C2H5OH + 3 O2 Æ 2CO2 + 3 H2O
Mass of CO2 formed = kg per kg of C ¥ 0.64
3
= 2.3467 kg 1 mole of ethyl alcohol combines with 3 moles
7 O2 to form 2 moles of CO2 and 3 moles of H2O.
Mass of CO formed = kg per kg of C ¥ 0.16 Thus,
3
= 0.3733 kg moles of air = 3 moles of O2 ¥ 4.76 moles
Mass of N2 with air = 0.77 ¥ mtotal = 14.28 mole/mole of ethanol
= 0.77 ¥ (18 kg air) Mass of 1 mole of ethnol (C2H5OH)
= 13.86 kg = 2 ¥ 12 + 1 ¥ 16 + 6 ¥ 1 = 46 kg
Mass of O2 supplied with excess air Mass of 14.28 moles of air
= O2 in actual air – O2 used in combustion = 14.28 ¥ 28.97 = 413.69 kg
= 0.23 ¥ 18 – 2.56 = 1.58 kg Stoichiometric A/F ratio
The volumetric analysis of dry products of combustion Mass of air 413.69 kg
= =
is given in the table below. Mass of ethanol 46 kg
= 8.99
Products Mass Molecular No. of % volume (ii) When mixture strength becomes 90%, the actual
mass mole ni
mi kg m ¥ 100 8.99
ni = i S ni air fuel ratio = = 9.99
Mi Mi 0.9
1
It means the amount of air supplied is times
CO2 2.3467 44 0.0533 8.728% 0.9
CO 0.3733 28 0.0133 2.181% that of stoichiometric air. Then the combustion
O2 1.5800 32 0.0493 8.08% reaction,
N2 13.8600 28 0.495 81.0% 1
C2 H5 OH + [3O2 + 3 ¥ 3.76 N2] Æ 2CO2
Total Sni = 0.611 100% 0.9
+ 3H2O + 0.33O2 + 12.53 N2
Example 16.12 If ethyl alcohol (C2 H5 OH) is burned The products of combustion are
in a petrol engine, calculate 2 mole CO2 + 3 mole H2O + 0.33 mole O2
(a) the stoichiometric air fuel ratio. + 12.53 mole N2
(b) the A/F ratio, the wet and dry analysis by volume Total moles of products = 17.86 mole
of exhaust gas for a mixture strength of 90%.
Fuels and Combustion 549

Wet analysis by volume Total dry moles of products = 1 + 1 + 9.4


2 = 11.4 kmol
CO2 = ¥ 100 = 11.19%
17.86 Dry products analysis by volume
3 1
H2O = ¥ 100 = 16.79% CO2 = ¥ 100 = 8.77%
17.86 11.4
0.33 1
O2 = ¥ 100 = 1.85% CO = ¥ 100 = 8.77%
17.86 11.4
12.53 9.4
N2 = ¥ 100 = 70.15% N2 = ¥ 100 = 82.46%
17.86 11.4
For dry products analysis by volume
Total moles of dry products = 2 + 0.33 + 12.53 Example 16.13 A fuel having a chemical formula
= 14.86 moles C12 H26 is burnt with 50% excess air. Calculate the
2 stoichiometric air required and percentage analysis of
CO2 = ¥ 100 = 13.46% products of combustion including water vapour.
14.86
0.33
O2 = ¥ 100 = 2 Solution
14.86
Given A fuel with chemical formula C12 H26
12.53
N2 = ¥ 100 = 84.32% Excess air = 50%
14.86
(iii) For mixture strength of 120%. To find
8.99 (i) Mass of stoichiometric air required,
actual air–fuel ratio = = 7.49
1.2 (ii) Analysis of products of combustion.
Then combustion reaction is
Analysis
1
C2H5OH + [3O2 + 3 ¥ 3.76 N2] Æ a CO2 (i) For complete combustion of fuel with chemical
1.2
+ b CO + 3H2O + 9.4 N2 formula C12H26, the chemical reaction is
The element balance on two sides of reaction C12H26 + 18.5 (O2 + 3. 76 N2) Æ 12 CO2
C: a+b =2 + 13 H2O + 69.56 N2
3
O2: 2a + b + 3 = 1 + ¥2 A minimum of 18.5 moles of O2 is required for
1.2
or 2a + b = 3 complete combustion
Solution yields to The mass of O2 required
a = 1 and b = 1 = 18.5 ¥ 32 = 592 kg/kmol of fuel
Therefore, the products of combustion are Mass of minimum air required
1 kmol CO2 + 1 kmol CO + 3 kmol H2O 100
+ 9.4 kmol N2 = ¥ 592 = 2574 kg/kmol
23
Total moles = 14.4 kmol Mass of one mole of fuel
Wet analysis by volume = 12 ¥ 12 + 26 ¥ 1 = 170 kg/kmol
1 Mass of air per mole of fuel
CO2 = ¥ 100 = 6.94% A/F ratio =
14.4 Mass of 1 mole of fuel
1 2574
CO = ¥ 100 = 6.94% = = 15.14
14.4 170
3 (ii) When 50% excess air is supplied;
H2O = ¥ 100 = 20.83%
14.4 Mass of actual air supplied
9.4 = 15.14 ¥ 1.5 = 22.71 kg/kmol
N2 = ¥ 100 = 65.29%
14.4
550 Thermal Engineering

Excess O2 supplied Example 16.14 The dry exhaust gas from an oil
= 18.5 ¥ 0.5 = 9.25 kg/kmol of fuel engine has the following composition by volume:
Actual O2 supplied CO2 = 8.85%,
= 18.5 ¥ 1.5 = 27.75 kg/kmol of fuel CO = 1.2%,
Actual N2 supplied O2 = 6.8% and N2 = 83.15%
= 3.76 ¥ 18.5 ¥ 1.5 The fuel oil has a percentage composition by mass as
= 104.34 kg/kmol of fuel C = 84% H2 = 14% and O2 = 2%
Therefore, actual chemical reaction Determine
C12H26 + 27.75 (O2 + 3.76 N2) Æ 12CO2 (a) Mass of carbon per kg of dry flue gases
+ 13H2O + 9.25 O2 + 104.34 N2 (b) The air–fuel ratio by mass
Products of combustion
Mass of CO2 produced per kg of fuel Solution
Mass of 12 moles of CO2 Given The composition of fuel oil by mass. For 1 kg
=
Mass of 1 mole of fuel of fuel
12 ¥ 44 C = 0.84 kg H2 = 0.14 kg O2 = 0.02 kg
= = 3.1 kg/ kg of fuel
170 Composition of flue gases by volume:
CO2 = 8.85% CO = 1.2% O2 = 6.8%
Mass of H2O produced
N2 = 83.15%
13 ¥ 18
= = 1.376 kg/kg of fuel
170 To find
Mass of excess O2 (i) Mass of carbon per kg of dry flue gases,
9.25 ¥ 32 (ii) The air–fuel ratio, and
= = 1.741 kg/kg of fuel
170 (iii) Mass of excess air supplied.
Mass of N2 accompanied with air Analysis
104.34 ¥ 28 (i) Mass of carbon supllied per kg of dry flue gases:
=
170 The conversion of volumetric analysis of exhaust
= 17.185 kg/kg of fuel gases in gravimetric analysis is tabulated below:
Percentage analysis of these products
Consti- % Molecular Mass Mass
Constituent Products mi Mass fraction tuent Volume weight mi = fraction in
mass ni Mi ni Mi flue gases
mi mi
= ¥ 100 =
S mi S mi
CO2 3.1 13.25% CO2 0.0885 44 3.894 0.1311
H2O 1.376 5.88% CO 0.0120 28 0.336 0.0113
O2 1.741 7.44% O2 0.0680 32 2.176 0.0733
N2 17.185 73.43% N2 0.8315 28 23.282 0.7842
Total Smi = 23.04 100% Smi = 1 kg
29.688
Percentage excess air supplied
Excess mass of air 5.94 The mass of carbon in exhaust gases per kg of
= ¥ 100 = ¥ 100 flue gases
Theoretical mass of air 14.29
3 3
=
5.94
¥ 100 = 41.17% = kg CO 2 + kg CO
14.29 11 7
Fuels and Combustion 551

3 3 The conversion of volumetric analysis of exhaust


= ¥ 0.1311 + ¥ 0.0113 gases in gravimetric analysis is tabualted below:
11 7
= 0.0406 kg/kg of dry flue gases Consti- % Molecular Mass Mass
(ii) Air–fuel ratio by mass tuent ni Volume Mi mi = niMi fraction in
The mass of dry flue gases formed per kg of fuel weight flue gases
Mass of carbon in 1 kg of fuel mi
=
= S mi
Mass of Carbon in 1 kg of flue gases
0.84 kg CO2 0.090 44 3.960 0.1332
= = 20.69 kg/kg of flue gases CO 0.015 28 0.42 0.0141
0.0406 kg
The mass of vapour (H2O) formed from H2 O2 0.070 32 2.24 0.0753
present in fuel N2 0.825 28 23.1 0.7772
= 9 H = 9 ¥ 0.14 = 1.26 kg S mi = 29.72 1 kg
Total mass of flue gases
The mass of carbon in exhaust gases per kg of
= mass of dry flue gases + mass of vapour
flue gases
= 20.69 + 1.26 = 21.95 kg/kg of flue gases 3 3
Actual air supplied per kg of fuel for its = kg CO 2 + kg CO
11 7
combustion 3 3
= ¥ 0.1332 + ¥ 0.0141
= Total mass of flue gases 1 11 7
= 21.95 1 = 20.95 kg = 0.04237 kg/kg of dry flue gases
Mass of air uesd 20.95 kg The mass of dry flue gases formed per kg of fuel
Air–fuel ratio = =
Mass of fuel 1 kg Mass of carbon in 1 kg of fuel
= 20.95 =
ue gases
Mass of carbon in 1 kg of flu
0.85 kg
Example 16.15 A fuel oil has mass composition as = = 20.06 kg/kg of flue gases
0.04237 kg
C = 85% H2 = 13% O2 = 2%
The dry exhaust gases have the following volumetric The mass of vapour (H2O) formed from H2
composition: present in fuel
CO2 = 9%, CO = 1.5%, = 9 H = 9 ¥ 0.13 = 1.17 kg
O2 = 7%, N2 = 82.5% Total mass of flue gases
Determine the mass of air supplied per kg of fuel and = mass of dry flue gases + mass of vapour
percentage of excess air supplied. = 20.06 + 1.17 = 21.23 kg/kg of flue gases
Actual air supplied per kg of fuel for its
Solution combustion
Given The composition of fuel oil by mass. For 1 kg = Total mass of flue gases 1
of fuel = 21.23 1 = 20.23 kg
C = 0.85 kg H2 = 0.13 kg O2 = 0.02 kg (ii) Percentage of excess air supplied
Composition of flue gases by volume: Theoretical mass of air required for complete
CO2 = 9% CO = 1.5% O2 = 7% combustion
N2 = 82.5%
100 È 8 Ê Oˆ ˘
ma = Í C + 8 Á H - ˜ + S˙
To find 23 Î 3 Ë 8¯ ˚
(i) Mass of air supplied per kg of fuel, and
100 È 8 Ê 0.02 ˆ ˘
= ¥ Í ¥ 0.85 + 8 ¥ Á 0.13 - + 0.0 ˙
8 ˜¯
(ii) The percentage of excess air supplied.
23 Î 3 Ë ˚
Analysis = 14.29 kg/kg of fuel
(i) Mass of air supllied per kg of fuel
552 Thermal Engineering

Excess air supplied per kg of fuel, The complete volumetric analysis of dry products
= Actual air Theoretical air
= 20.23 14.29 = 5.94 kg/kg of fuel Constituent (a) Number of % volume of
moles (ni) constituent
Percentage excess air supplied n
Mass of excess air = i
= S ni
Theoretical mass of air
CO2 0.55 8.39%
5.94 kg
¥ 100 = ¥ 100 O2 0.411 6.27%
14.29 kg
N2 5.59 85.34%
= 41.56%
Total S ni = 29.688 100%
Example 16.16 The coal gas supplied to a gas engine
has the following composition: Example 16.17 A fuel has ultimate analysis as 88%
H2 = 50.5% CO = 10% CH4 = 26% of carbon, 4.4% of hydrogen, and the remaining being
C4H8 = 4% O2 = 0.4% CO2 = 3% incombustible.
N2 = 6%
A partial volumetric analysis of dry products of
The fuel burns with some excess air and its air–fuel combustion shows 13.2% of CO2, 3.2% of O2 and it is
ratio by volume is worked out to be 7. Calculate the suspected that some CO is also present.
analaysis of dry products of combustion by volume.
Calculate
Solution (a) Complete volumetric analysis of dry exhaust
gases
Given The volumetric analysis of coal gas: (b) Air to fuel ratio by mass
H2 = 50.5% CO = 10% CH4 = 26% Assume all carbon and hydrogen is burned.
C4H8 = 4% O2 = 0.4% CO2 = 3%
N2 = 6% Solution
Air–fuel ratio with excess air = 7 Given The ultimate analysis for 100 kg of fuel
To find The volumetric analysis of dry exhaust gases C = 88 kg H2 = 4.4 kg and remaining is
incombustible
Assumption 1 mole of air consists of 0.21 mole of O2
Composition of flue gases by volume:
and 0.79 mole of N2 by volume.
CO2 = 13.2% O2 = 3.2%
Analysis The chemical reaction for coal gas
To find
0.506 H2 + 0.1 CO + 0.26 CH4 + 0.04 C4H8
+ 0.004 O2 + 0.03 CO2 + 0.06 N2 + 7 ¥ [0.21O2 + 0.79 N2] (i) Complete volumetric analysis of dry exhaust
gases
Æ a CO2 + b H2O + c O2 + d N2
(ii) Air–fuel ratio by mass
The values of a, b c, and d are to be worked out by
balancing the elements of fuel. The element balance Analysis
yields (i) Complete volumetric analysis of dry exhaust
C 0.1 + 0.26 + 0.04 ¥ 4 + 0.03 = a or a = 0.55 gases
H2 0.506 + 0.26 ¥ 4 + 0.04 ¥ 8 = 2b or b = 1.186 88
For 100 kg of fuel, there are moles of carbon
O2 0.1 + 0.004 ¥ 2 + 0.03 ¥ 2 + 7 ¥ 0.21 ¥ 2 = 2 c 12
or c = 0.411 4.4
and moles of H2. The chemical reaction for
N2 0.06 ¥ 2 + 7 ¥ 0.79 ¥ 2 = 2d or d = 5.59 2
Summation of moles of dry products ( excluding moles the fuel is
of H2O)
88 4.4
S DP = a + c + d C+ H2 + x (O2 + 3.76 N2) Æ a CO2
12 2
= 0.55 + 0.411 + 5.59 = 6.55 + b CO + 2.2 H2O + c O2 + 3.76x N2
Fuels and Combustion 553

The values of a, b c, and x are to be worked out by 100


balancing the elements of fuel. = ¥ (8.82 ¥ 32) = 1227 kg
23
The element balance yields
The air–fuel ratio by mass
88
C = a + b or b = 7.33 a Mass of air uesd 1227 kg
12 = =
O2 2x = 2a + b + 2.2 + 2c Mass of fuel 100 kg
or c = x 0.5a 4.765 = 12.27
Summation of moles of dry products
SDP = a + b + c + 3.76 x
The volume fraction of CO2 is 0.132 and O2 is
16.13 FLUE GAS ANALYSIS—ORSAT
0.032. Thus,
a
0.132 =
S DP
c The Orsat analysis technique is a quick, simple and
and 0.032 =
S DP inexpensive method for volumetric analysis of the
a combustion products. An Orsat gas analyzer uses a
or a + b + c + 3.76 x =
0.132 chemical absorption technique to determine mole
c fractions of CO2, CO and O2 in an exhaust gas.
and a + b + c + 3.76 x =
0.032 Since it cannot measure H2O content, the exhaust
The solution of these two algebraic equations gas sample is cooled to atmospheric temperature,
leads to so most of the water vapour in the combustion
a = 5.52 products gets condensed out. Therefore, the Orsat
b = 1.81 analysis is treated as an analysis of the dry com-
c = 1.3 bustion products. The Orsat apparatus also does not
x = 8.82 detect unburnt hydrocarbons and free hydrogen in
The moles of N2, nN2 = 3.76; x = 33.16 the products.
It results into the following chemical reaction:
7.33 C + 2.2 H2 + 8.82 (O2 + 3.76 N2) Æ
5.52 CO2 + 1.81CO + 2.2 H2O + 1.3 O2
The Orsat apparatus is shown in Fig. 16.2. It con-
+ 33.16 N2
sists of a burette, levelling bottle and three pi-
The complete volumetric analysis
pettes to absorb CO2, O2 and CO. These pipettes
Constituent Number of % Volume of are interconnected by means of a manifold fitted
(a) moles (ni) constituent with cocks. Each pipette is also fitted with a num-
n ber of small glass tubes (not shown in Fig. 16.2)
= i
S ni to increase the surface area within the pipette. The
CO2 5.52 13.21 measuring burette is surrounded by a water jacket
CO 1.81 4.33 to keep the temperature and density of the gas con-
stant. The first pipette contains caustic soda (KOH),
O2 1.30 3.11
which absorbs carbon dioxide, the second pipette
N2 33.16 79.35
contains pyrogallic acid to absorb oxygen content
Total Sni = 29.688 100
of the gas and third one contains cuprous chloride
to absorb carbon monoxide. Further, the apparatus
(ii) Air–fuel ratio
is also equipped with a three-way cock to connect
For 100 kg of fuel, mass of air supplied
100
the apparatus either to gases or to the atmosphere.
= ¥ (Moles of O2 ¥ Mol. mass of O2)
23
554 Thermal Engineering

Fig. 16.2

approximation, the flue gases must be passed


A sample of gas is drawn in the measuring burette through absorption pipettes in the same sequence
by lowering the levelling bottle. The sample of flue as discussed above.
gas is taken as 100 cc, and any excess gas is ex-
Example 16.18 The volumetric analysis of a gas is
pelled to the atmosphere via three-way cock. The as follows:
cock A is opened and the flue gas is first passed H2 = 49.4% CO = 18% CH4 = 20%
through the first pipette containing KOH, where C4H8 = 2% O2 = 0.4% N2 = 6.2% CO2 = 4%
CO2 component of the gas is absorbed. The pro- Calculate
cess is repeated several times to ensure complete
(a) Stoichiometric air fuel ratio,
absorption of CO2 in the KOH solution. The level-
(b) Wet and dry analysis of products of combustion, if
ling bottom is then lowered to suck the gas back in the actual mixture is 20% weak.
the measuring burette. The difference in level of the
gas gives the percentage of CO2 by volume in the Solution
flue gases.
Given The volumetric analysis of a gas for 1 m3 as
Now the cock A is closed, the procedure is re-
H2 = 0.494 CO = 0.18 CH4 = 0.2
peated with the second and third pipettes by open-
C4H8 = 0.02
ing cocks B and C, respectively and the volumetric
O2 = 0.004 N2 = 0.062 CO2 = 0.04
percentage analysis of O2 and CO in the flue gases
is measured. To find
When the percentage of CO2, O2 and CO have (i) Stoichiometric air–fuel ratio, and
been determined then the remaining gas in the sam- (ii) The wet and dry analysis of products of combus-
ple is assumed to be nitrogen, N2. tion if the actual mixture is 20% weak.
The Orsat apparatus gives the volumetric analy- Assumptions
sis of dry flue gases, since steam is condensed at (i) Each molecule of oxygen is accompanied by
atmospheric temperature. In order to get good 3.76 moles of nitrogen.
Fuels and Combustion 555

(ii) The nitrogen remains as inert gas. The moles of air associated with 0.853 mole of
(iii) Since it is a weak mixture, thus excess air is O2;
needed for complete combustion. = 0.853 ¥ 4.76 mole = 4.060 mole
Stoichiometric fuel ratio
Analysis mole of air 4.060
(i) For complete combustion of each element, the O2 = =
mole of fuel gas 1
required and products produced: = 4.060
Hydrogen (ii) When mixture is 20% weak, excess air by 20% is
1 needed.
H2 + O2 Æ H2O
2 Actual air–fuel ratio = 1.2 ¥ 4.060 = 4.872
1 Oxygen associated with air supplied
1 mole of H2 requires mole of O2 to produce
2 4.872
= = 1.023 moles
1 mole of H2O, 1.88 mole of N2. Thus, 0.494 4.76
Excess O2 supplied
mole of H2
= 1.023 – 0.853 = 0.170 mole
1 Nitrogen associated with O2 supplied
requires = 0.494 ¥ = 0.247 mole of O2
2 = 3.76 ¥ 1.023 = 3.846 moles
produces = 0.494 of H2O, Total nitrogen in products
= 3.846 + 0.062 = 3.908 moles
1
CO: CO + O2 Æ CO2 Analysis by volume of wet and dry products of
2
combustion
0.18 mole of CO requires
Product Mole/mole of fuel % by % by
1
= 0.18 ¥ = 0.09 mole of O2 volume volume
2 (dry wet
produces = 0.18 mole CO2 products
products)
CH4 CH4 + 2O2 Æ CO2 + 2H2O
CO2 0.18 + 0.2 10.92 9.01%
0.2 mole CH4 + (0.2 ¥ 2 mole) (O2) Æ
+ 0.08 + 0.04 = 0.5 ––
0.2 mole CO2 + (0.2 ¥ 2 mole) H2O
H2O 0.494 + 0.4 + 0.08 — 17.54%
Thus, 0.2 mole of CH4 requires
= 0.974
= 0.4 mole of O2
O2 0.170 3.71 3.07%
produces = 0.2 mole CO2 + 0.4 mole H2O
N2 3.908 85.36 70.39%
C4H8: C4H8 + 6O2 Æ 4CO2 + 4H2O
Total dry = 4.578 100.00 100.00
For 0.02 mole; 0.02 C4H8 + (0.02 ¥ 6) O2 Æ (0.02
Total wet = 5.552
¥ 4) CO2 + (0.02 ¥ 4) H2O
0.02 mole of C4H8 Example 16.19 A fuel having the formula C7 H16
requires = 0.12 mole O2 is burnt with 10% excess air. Assuming 90% carbon is
produces = 0.08 mole CO2 + 0.08 mole H2O burnt to CO2 and the remaining to CO, determine the
Thus, the O2 mole requires for complete volumetric analysis of dry flue gases and air to fuel ratio
combustion by mass.
= 0.247 + 0.09 + 0.4 + 0.12
= 0.857 mole Solution
The fuel gas contains 0.004 mole of O2; thus Given A fuel with chemical formula C7H16
Net O2 mole requires = 0.857 – 0.004 Excess air supplied = 10% and 90% carbon to CO2
= 0.853 mole
To find
(i) Air fuel ratio by mass, and
(ii) Volumetric analysis of dry flue gases.
556 Thermal Engineering

Analysis Total mole of carbon in the fuel


(i) Air–fuel ratio = 0.063 + 0.007 = 0.07
The mass of 1 mole of fuel C7H16 The chemical reaction on the molar basis.
= 7 ¥ 12 + 16 ¥ 1 0.07C + 0.08 H + 0.121O2 + 0.463N2 Æ
= 100 kg/mole of fuel 0.08
0.063CO2 + 0.007CO + H2O + a O2
The mass fraction of carbon in fuel 2
7 ¥ 12 + 0.463N2
= = 0.84 Equating the constants for O2
100
0.007
Mass fraction of hydrogen in the fuel 0.121 = 0.063 + + 0.04 + a
2
16
= = 0.16 It gives a = 0.0145
100
Total moles of dry flue gases
The minimum mass of air required to burn 1 kg of = moles of [CO2 + CO + O2 + N2]
fuel Volumetric composition
100 È8 ˘ = 0.063 + 0.007 + 0.0145 + 0.463
= Í 3 C + 8H ˙
23 Î ˚ = 0.5475
100 È 8 ˘ 0.063
= ¥ 0.84 + 8 ¥ 0.16 ˙ %C O2 by volume = ¥ 100 = 11.5%
Í
23 Î 3 0.5475
˚
0.007
= 15.3 kg/kg of fuel % CO by volume = ¥ 100 = 1.278%
Thus, the stoichiometric air fuel ratio 0.5475
15.3kg air 0.0145
= = 15.3 % O2 by volume = ¥ 100 = 2.648%
1kg fuel 0.5475
But 10% excess air is supplied, thus mass of 0.463
% N2 by volume = ¥ 100 = 84.56%
actual air supplied per kg of fuel 0.5475
= 15.3 ¥ 1.10 = 16.834 kg/kg of fuel Total = 100%
Actual A/F ratio
Example 16.20 The exhaust gas of a gasoline-fueled
= 16.834
automobile engine was cooled to 20°C and then subject-
(ii) Volumetric analysis of combustion products
ed to Orsat gas analysis. The results were
The mass of carbon burnt to CO2
CO2 = 7.2% CO = 0.8% O2 = 9.9% N2 = 82.1%
mCO2 = 0.84 ¥ 0.9 = 0.756 kg/kg of fuel
Determine
The mass of carbon burnt to CO
(a) the hydrocarbon model of the fuel,
mCO = 0.84 ¥ 0.1 = 0.084 kg/kg of fuel
(b) the composition of fuel on mass basis,
The moles of elements in products coming from
(c) the air–fuel ratio on mass basis,
fuel;
(d) the theoretical air used in the combustion process.
0.756
nCO2 = = 0.063
12 Solution
0.084
nCO = = 0.007 Given The exhaust gas analysis of a gasoline engine
12
0.16 To find
nH2 = = 0.08
2 (i) The hydrocarbon model of the fuel.
Ê 23 ˆ 16.834
n O2 = Á ¥ = 0.121 (ii) The composition of fuel on the mass basis.
Ë 100 ˜¯ 32
(iii) The air fuel ratio on mass basis.
Ê 77 ˆ 16.834
nN2 = Á ¥ = 0.463 (iv) The percentage theoretical air used in the
Ë 100 ˜¯ 28 combustion process.
Fuels and Combustion 557

Assumptions 100 È 8 ˘
= ¥ ¥ 0.845 + 8 ¥ 0.155˙
(i) Since the analysis of Orsat apparatus is carried 23 ÍÎ 3 ˚
out at 20°C, we consider that all the water in the = 15.18 kg/kg of fuel
combustion products has condensed out.
(ii) Steady flow chemical reaction.
(iii) The combustion reaction of 100 moles. 16.14 HEAT GENERATED BY
(iv) The each oxygen mole in the reaction is COMBUSTION
accompanied with 3.76 moles of nitrogen.
During a chemical reaction, some chemical bonds
Analysis in the fuel are broken and other new ones are
(i) The chemical reaction for a hydrocarbon is formed. The chemical energy associated with these
Cn Hm + a (O2 + 3.76 N2) Æ 7.2 CO2 + 0.8 CO bonds is released or absorbed in the form of heat.
+ 9.9 O2 + b (H2O) + 82.1 N2 That is,
The balancing the elements of reaction Energy of reactants = Energy of products
Carbon, C n = 7.2 + 0.8 = 8.0 + Chemical energy (as heat)
Hydrogen, H2 m = 2b In absence of any change in kinetic and potential
Nitrogen, N2 3.76 a = 82.1 or a = 21.9 energies, the energy change of a system during a
Oxygen, O2 a = 21.9 chemical reaction will be due to change in state and
0.8 b a change in chemical composition. That is,
= 7.2 + + 9.9 +
2 2 Esys = Estate + Echemical ...(16.23)
or b = 8.8 When products formed during a chemical reac-
Thus, m = 2b = 17.6 tion are at the inlet state of reactants then Estate
And the model of hydrocarbon becomes = 0 and the energy change of the system is due to
Cn Hm = C8H17.6 change in the chemical compostion only.
(ii) Mass of 1 mole of fuel = 12 ¥ 8 + 17.6 ¥ 1
= 113.6 kg/mole of fuel
Mass of O2 moles in the fuel
The heat generated during a reaction does not
= 21.9 ¥ 32 = 700.8 kg
depend only on the chemical energy of reactants
Mass of air associated with this O2
but also on pressure and temperature at which the
100 reaction takes place. Further, after the chemical
= ¥ 700.8
23 reaction, the composition of the system is also
= 3047 kg/mole of fuel
different from that at the beginning of the process.
Mass of air 3047 In such cases pref = 1 atm and Tref = 25°C (= 298 K) is
A/F ratio = =
Mass of fuel 113.3 known as the standard reference state. The property
= 26.82 values at the standard reference state are designated
(iii) Mass percentage of carbon in the fuel by the superscript ‘°’, such as h°, u°, etc.
8 ¥ 12 At standard reference state, the enthalpy values
= ¥ 100 = 84.51%
113.6 can be assigned to elements for use in analysis
Mass percentage of H2 in the fuel of a reacting system. For some chemically stable
17.6 ¥ 1 elements, the property value (like enthalpy) at the
= ¥ 100 = 15.5%
113.6 standard reference state is assigned a zero value.
(iv) Theoretical air–fuel ratio
The mass of air required for complete combustion
100 È 8 ˘ The enthalpy of an element at the standard
= C + 8H ˙
23 ÍÎ 3 ˚
reference state is called the enthalpy of formation.
558 Thermal Engineering

The enthalpy of formation is the energy released rejected from the reaction chamber to surroundings
or absorbed when the products are formed from its during the process to keep the product CO2 at 25°C
element, the products and elements both being Tref and 1 atm.
and pref. It is denoted as h f∞, i , where the subscript f
indicates formation, the subscript i means element
(species) and superscript ‘°’ means the standard
reference state of 1 atm and 25°C. It is measured in
kJ/kmol. Table 16.6 presents the values of enthalpy
of formation for several elements.
Consider the formation of CO2 from its elements, Fig. 16.3 2
carbon (graphite) and oxygen. Carbon and oxygen
both enter the reaction chamber at 25°C and 1 atm Since the process does not involve any work
and react completely at steady state to form CO2 at interaction, therefore, the steady-flow energy equa-
25°C and 1 atm. tion in absence of change in potential and kinetic
C + O2 Æ CO2 + Q (Heat) energies yields to
The formation of CO2 from carbon is an exo- Q = H = HP – HR ...(16.24)
thermic reaction and therefore, chemical energy is where HP = enthalpy of product
relased in the form of heat and this heat must be HR = enthalpy of reactants, both are at the
same state
Table 16.6 Dh °f

Substance Formula State Molar mass (kg/ Enthalpy of formation


kmol) Dh f (kJ/kmol)
Carbon C Graphite 12.01 0
Carbon C Diamond 12.01 1900
Hydrogen H2 Gas 2.016 0
Oxygen O2 Gas 32.00 0
Nitrogen N2 Gas 28.01 0
Carbon monoxide CO Gas 28.01 110 530
Carbon dioxide CO2 Gas 44.01 393 520
Water H2O Liquid 18.02 285 830
Water H2O Gas 18.02 241 820
Methane CH4 Gas 16.04 74 850
Ethylene C2H2 Gas 28.05 52 280
Ethane C2H6 Gas 30.07 84 680
Propylene C3H6 Gas 42.08 20 410
Propane C3H8 Gas 44.09 103 850
Butane C4H10 Gas 58.12 126 150
Pentane C5H12 Gas 72.15 146 440
Octane C8H18 Gas 114.22 208 450
Benzene C6H6 Gas 78.11 82 930
Methyle alcohol CH3OH Gas 32.08 200 890
Methyle alcohol CH3OH Liquid 32.08 238 810
Ethyle alcohol C2H5OH Gas 46.07 235 310
Ethyle alcohol C2H5OH Liquid 46.07 277 690
Fuels and Combustion 559

The products and reactants both are at same state,


and thus the enthalpy change during the process
is only due to change in chemical composition of
the system. This energy is referred as enthalpy of
formation.
The enthalpy of formation is defined as the
enthalpy of reaction at a specified state for the
formation of a substance from its elements in their
most stable form. The stable forms of elements
means forms such as H2 and O2 for hydrogen and
oxgyen instead of H and O, the monoatomic forms. Fig. 16.4 2 2
The stable form of carbon is graphite instead of
diamond, etc.
enthapy is called the enthalpy of reaction, hR . In
The enthalpy of formation for all stable elements
this case, the reaction is written as
such as O2, N2 and C is assigned as value zero at
the standard reference state. That is, h f∞ = 0, for all CO + (1/2)O2 Æ CO2 + hR
stable elements Thus, the enthalpy of reaction can be defined as
Now reconsider the formation of CO2 (product) the difference between the enthalpy of products and
from its elements carbon and oxygen at 25°C and 1 enthalpy of reactants at same specificed state for a
atm during a steady flow process. The enthalpy of chemical reaction.
formation of reactants C and O2, HR = 0 and The formation of CO2 from reactants C and O2
Q = HP – HR = – 393, 520 kJ/kmol leads to
C + O2 Æ CO2 + h f∞, co2 ...(16.25)
or h f∞ ,co2 = – 393, 520 kJ/kmol
And formation of CO from reactants leads to
The negative sign is used to account for the fact C + (1/2)O2 Æ CO + h f∞, co ...(16.26)
that the enthalpy of 1 kmol of CO2 at 25°C and
1 atm is 393,520 kJ less than that of 1 kmol of C and Subtracting Eq. (16.26) from (16.25) and trans-
1 kmol of O2 at the same state. In other words, the posing CO to the left, leads to

reaction is leaving the system. According to sign CO + (1/2)O2 Æ CO2 + hR,co ...(16.27)

convention, this heat is negative. Thus the enthalpy where hR,co = h f∞, co2 - h f∞, co
of formation of substances produced by exothermic
reactions are considered negative. = 393,520 – (– 110,530)
= – 28,990 kJ/kmol
For a combustion process, the enthalpy of
reaction is referred as enthalpy of combustion
The enthalpy of formation concept can be used
hc , which can be interpreted as the amount of heat
to obtain the change in chemical energy during a
released during a steady-flow combustion process,
reaction.
when 1 kmol of (1 kg) of fuel burns completely at a
Consider the formation of CO2 from its elements,
specificed temperature and pressure. It is expressed
CO and oxygen, at standard reference state.
as
CO + (1/2)O2 Æ CO2 + Q (Heat)
In this case, the term ‘enthalpy of formation’ hc = Ân h -Ân h
e e i i (kJ/kmol) ...(16.28)
cannot be used since one of the reactants, CO, is P R

not a stable element. In such a case, the change in where ni hi = h ∞f , i + D hi (for i th species)
560 Thermal Engineering

where, n’s represent the number of moles of To find Enthalpy of combustion.


reactants and products per mole of fuel and h ’s Analysis The chemical reaction is
represent the enthalpy per mole.
CH4 + 2O2 Æ CO2 + 2H2O
When the enthalpy of combustion is expressed
The enthalpy of reaction for 1 kmol of CH4 (fuel).
on a unit mass basis, it is designated as hc and
measured in kJ/kg. If the enthalpy of formation
data are available for all reactants and products, the
hc = Ân h - Ân h
P
e e
R
i i

enthalpy of combustion can be calculated directly (i) The liquid water in the products
from Eq. (16.28) as illustrated in Examples 16.21 Using the values from Table 16.6;
and 16.22.

Example 16.21 Calculate the enthalpy of combustion


Ân h
R
i i = 1h f∞, CH ( g )
4

at 25°C of ethyl alcohol, C2H5OH. = ( 1 kmol) ¥ (– 74 850 kJ/kmol)


= – 74 850 kJ/kmol
Solution
Given Ethyl alcohol at 25°C
and Ân h
P
e e = 1 h f∞, CO2 + 2 h ∞f , H 2 O ( l )

To find Enthalpy of combustion = ( 1 kmol) ¥ (– 393,520 kJ/kmol)


+ (2 kmol) ¥ (– 285 830 kJ/kmol)
Analysis The chemical reaction is
= – 965 180 kJ/kmol
C2H5OH + 3O2 Æ 2CO2 + 3H2O
The enthalpy of reaction for 1 kmol of C2H5OH (fuel). Then hc = – 965 180 – (– 74 850)
= – 890 330 kJ/kmol
hc = Ân h - Ân h
P
e e
R
i i Dividing it by the molar mass of methane
890330 kJ/kmol
Referring to Table 16.6, hc = -
16.04 kg/kmol
Ân h
R
i i = 3h ∞f , O2 + 1h ∞f, C2 H5 OH = 55 506.85 kJ/kg
(ii) The water vapour in the products
= ( 3 kmol) ¥ 0 + (1 kmol)
Using the values from Table 16.6;
(– 235,310 kJ /kmol)
= – 235,310 kJ/kmol
Ân h i i = 1h f∞, CH 4 ( g )
and Ân h
P
e e = 2 h f∞, CO2 + 3h ∞f , H 2 O R
= – 74 850 kJ/kmol
= 2 kmol ¥ (– 393,520 kJ/kmol) +
(3 kmol) ¥ (– 241,820 kJ/kmol) and Ân h
P
e e = 1h f∞, CO2 + 2 h f∞, H 2 O ( g )
= – 1512500 kJ/kmol
hc = – 1512500 – (– 235,310) = ( 1 kmol) ¥ (– 393,520 kJ/kmol)
= – 12,77,190 kJ/kmol + (2 kmol) ¥ (– 241 820 kJ/kmol)
= – 877 160 kJ/kmol
Example 16.22 Calculate the enthalpy of combustion Then hc = – 877 160 – (– 74 850)
of gaseous methane in kJ/kg of fuel
= – 802 310 kJ/kmol
(a) at 25°C, 1 atm with liquid water in the products
Dividing it by the molar mass of methane
(b) at 25°C , 1 atm with water vapour in products
802310 kJ/kmol
hc = -
Solution 16.04 kg/kmol
= 50 019.32 kJ/kg
Given Fuel as gaeous methane at 25°C and 1 atm
Fuels and Combustion 561

16.15 CALORIFIC VALUE, OR HEATING by deducting the heat necessary to form the vapour
VALUE OF FUEL from hydrogen. That is,
lower calorific value (LCV)
The term ‘calorific value’ is most commonly used = HCV – mv hfg ...(16.29)
in conjunction with the combustion of fuel. The where mv = mass of water vapour formed per kg of
calorific value of a fuel is defined as the amount fuel,
of heat energy liberated by complete combustion hfg = the heat of vaporisation of water at
of unit quantity of a fuel. It is also called heating 25°C and 1 atm,
value of the fuel and it can also be considered as an
= 2441.5 kJ/kg ◊ K
absolute value of enthalpy of combustion. That is,
The two calorific values are also related as
calorific (heating) value = | hc |
The calorific value is measured in kJ/kg or HCV = LCV + ( n h f∞ ) H 2O kJ/kg of fuel ...(16.30)
kJ/kmol for solid and liquid fuels and kJ/m3 for
where n is number of kmol/kg of H2O and hf g is
gaseous fuels.
the enthalpy of vaporisation of water at 25°C.
There are two aspects of relation for heat of
The two units of calorific value can be correlated
formation (or reaction) and calorific value, which
as
should be kept in mind.
Calorific value of fuel in kJ/kg
1. The calorific value of the fuel is the absolute Calorific value of fuel in kJ/kmol
value of enthalpy of formation (or reaction), =
Molecular weight of fuel (kg/kmol))
but expressed per unit quantity of fuel (i.e.,
reactants) rather than the products. ...(16.31)
2. By convention, the calorific value is positive, Thus, the calorific value in kJ/kg is obtaind by
it has opposite sign convention to that for dividing the calorific value in kJ/kmol by the re-
enthalpy of formation (or reaction). spective molecular weight.
The calorific values of some fuels in kJ/kmol are
presented in Table 16.7.

Basically, all fuels are hydrocarbons, thus their


main constituents are carbon and hydrogen. During
the combustion process, carbon burns to carbon di- Normally, fuels contain carbon, hydrogen, oxygen,
oxide and hydrogen reacts with oxygen and forms nitrogen, ash and sulphur. The nitrogen and ash are
water vapour. The magnitude of the calorific value inert to the combustion. The carbon, hydrogen, sul-
depends on the phase of water vapour in the com- phur and oxygen are participating in the combus-
bustion products. tion. Let C, H, O and S be the percentage of these
When combustion products are cooled to the constituents, respectively in the fuel.
reactant’s temperature, the water vapour gets con- The calorific value of these constituents can be
densed and the heat of its vaporisation is recovered. obtained from Table 16.7 as
The calorific value thus obtained is called the high- 393520 kJ/kmol
For carbon (C) =
er calorific value (HCV) or gross calorific value. 12 kg/kmol
The lower calorific value (LCV) or net calorific ª 33,000 kJ/kg
value is the amount of heat released by complete
286043
combustion of unit quantity of fuel, when the va- For hydrogen (H) =
pour carries its heat of vaporisation. It is obtained 2
= 1,43,000 kJ/kg (HCV)
562 Thermal Engineering

Table 16.7

Substance Symbol Approximate HCV LCV


Molecular weight H2O(l) H2O(g)
Acetylene C2H2 26 1300566 1256520
Benzene C6H6 78 3303878 3171736
n-Butane C4H10 58 2879190 2658955
Isobutane C4H10 58 2870817 2650580
Carbon (graphite) C 12 393520 ––
Carbon monoxide CO 28 283192 ––
Ethane C2H6 30 1560997 1428855
Ethelene C2H4 28 1411982 1323930
n-heptane C7H18 100 4856962 4504584
n-hexane C6H14 86 4197760 3889430
Methane CH4 16 890994 802900
n-octane C8H18 114 5516163 5119738
n-pentane C5H12 72 3538685 3274400
Propane C3H8 44 2221622 2045475
Hydrogen H2 2 286043 242000
Sulphur S 32 293120 ––

242000 value is the lower calorific value of the fuel and can
= similarly be obtained as
2
= 1,21,000 kJ/kg (LCV) Ê Oˆ
LCV = 330 C + 91.6 S + 1210 Á H - ˜
293120 Ë 8¯
For sulphur (S) = = 9,160 kJ/kg
32 ...(16.33)
For 100 kg of fuel, the total heat produced after
Example 16.23 Calculate the calorific value of eth-
complete combustion ane.
Ê Oˆ
= 33000 C + 9160 S + 143000 Á H - ˜ kJ/kg Solution
Ë 8¯
The higher calorific value (HCV) of fuel Given Ethane (C2H6) as fuel

Ê Oˆ To find Calorific value (absolute value of enthalpy of


33000 C + 9160 S + 143000 Á H - ˜ kJ combustion) of ethane
Ë 8¯
=
100 kg Analysis The molecular weight of ethane is 30.
Fraction of carbon content per kg of fuel
Ê Oˆ
= 330 C + 91.6 S + 1430 Á H - ˜ kJ/kg 2 ¥ 12
Ë 8¯ mc = = 0.8 kg/kg of fuel
30
...(16.32)
It is also known as Dulong’s formula, where C, S, Fraction of hydrogen content per kg of fuel
H and O are percentage constituents, respectively in 6 ¥1
mH2 = = 0.2 kg/kg of fuel
1 kg of fuel. If the lower calorific value of hydrogen 30
as 121000 kJ/kg is used then the obtained heating
Fuels and Combustion 563

The heat released by complete combustion Ê 5.6 ˆ


= 330 ¥ 84.4 + 1430 ¥ Á 4 -
Q = mC (HCV)c + mH2(HCV)H2 Ë 8 ˜¯
Ê 393, 520 ˆ Ê 286 043 ˆ = 32,571 kJ/kg
= 0.8 ¥ Á kJ/kg˜ + 0.2 ¥ Á kJ/kg˜
Ë 12 ¯ Ë 2 ¯ (ii) Lower calorific value Using Eq. (16.33) in
= 54,839 kJ/kg of fuel absence of sulphur percentage,
Ê Oˆ
Example 16.24 A sample of gobar gas contains 55% LCV = 330 C + 1210 Á H - ˜
Ë 8¯
methane and the rest of CO2. What is the calorific value ?
Ê 5.6 ˆ
= 330 ¥ 84.4 + 1210 Á 4 -
Solution Ë 8 ˜¯

Given A sample of gobar gas with 55% CH4 and 45% = 31,845 kJ/kg
CO2
To find Calorific value of gobar gas.
Analysis Since CO2 does not participale in combustion,
it remains inert to combustion. And the mass fraction of The apparatus used for determining the calorific
CH4 in gobar gas is 0.55 kmol/kmol of fuel. value of fuels is known as a calorimeter. The basic
For 1 kmol of CH4 is principle used in determining the calorific value
HCV = 890,994 kJ/kmol of fuels is that a known quantity of fuel is burned
Thus, for 0.55 kmol/kmol of CH4 in controlled environment and the heat energy
= 0.55 ¥ 890 994 = 490,047 kJ/kmol liberated is transferred to a medium of known
mass and specific heat and the rise in temperature
Example 16.25 The ultimate analysis of a sample of the medium is measured. Though there are
of coal has the following analysis: carbon-84.4%,
various types of calorimeters available, we shall
hydrogen-4% , oxygen-5.6% and the remainder is ash.
only discuss Bomb calorimeter and Junker’s gas
Calculate the higher and lower calorific values of the
fuel. calorimeters used for determining the calorific
value of fuels.
Solution
Given Ultimate analysis of a sample of coal
BOMB CALORIMETER
C = 84.4% H = 4% The calorific value of powdered and liquid fuels is
O = 5.6% Ash = 6% (by calculation) determined at constant volume in the bomb calo-
To find rimeter. It resembles the shape of a bomb, and thus
(i) Higher calorific value of fuel, it is known as the bomb calorimeter. It is shown in
(ii) Lower calorific value of fuel. Fig. 16.5.
Assumptions
(i) Complete combustion.
(ii) Ash as inert to combustion. The fuel is burnt in a strong steel chamber, known as
bomb, which is immersed in a known mass of water.
Analysis The fuel sample is placed in a crucible inside the
(i) Higher calorific value of a fuel Using bomb, which is filled with oxygen under a pressure
Eq. (16.32) in absence of percentage of sulphur above 25 atm. It is then ignited by an electrically
(S = 0).
heated platinum wire. The combustion thus takes
Ê Oˆ place at constant volume, the fuel burns almost in
HCV = 330 C + 1430 Á H - ˜ kJ/kg
Ë 8¯ a constant-pressure environment due to the high
564 Thermal Engineering

Fig. 16.5 bomb calorimeter

pressure of oxygen. To reduce any losses of heat, After experimentation, the bomb is taken out
the calorimeter is also provided with additional from its housing. The products of combustion
water jacket and air. A motor-driven stirrer is used are released through the release valve. Then it is
to keep the water temperature uniform around the opened, and the unburnt fuse wire, if any, is col-
bomb and an accurate thermometer (Beckman type) lected and weighed. A temperature–time curve is
is immersed in water to measure the temperature plotted.
accurately. The measured temperature rise is corrected for
various losses. The allowance for combustion of
fuse wire is determined from the weight of the fuse
A known quantity of fuel sample as a briquette and its known calorific value. The water equiva-
is placed into the crucible and a fuse wire is con- lence of a calorimeter must be used in calculation
nected with the electrodes as shown in Fig. 16.6. to accommodate its allowance.
The bomb is then placed in a calorimeter with a The heat released by combustion of fuel is ab-
weighed quantity of water. After making necessary sorbed by water surrounding the bomb and calo-
connections, the stirrer is started and temperature rimeter. Thus an energy balance yields to
measurements are taken every minute. At the end Mass of fuel ¥ calorific ⎫ (Mass of water equivaent
of the fifth minute, a charge is fired and tempera- value + mass of fuse ⎪ = of calorimeeter) × specific
wire burn ¥ calorific ⎬ heat of water × corrected
ture readings are taken carefully every 10 seconds ⎪
value of fuse wire ⎭ temperature rise
during this period. When the temperature readings
begin to fall, the frequency of readings may be re- mf CV + mfuse CV1
duced to one every minute. = (mw + me) Cpw [(T2 – T1) + Tc]
Fuels and Combustion 565

Connections to external
firing circuit Electrical
Drive connections

Cover

Release
valve
Calorimeter Oxygen
inlet valve
Terminal
External
insulated
One- housing
from bomb
Wire way
clamps valve Calorimeter

Stirrer Housing of
calorimeter

Crucible Platinum
Fuse wire wire Fixed range
thermometer
(or Beckman
thermometer)

Water
Nylon
feet

Fig. 16.6 bomb calorimeter

( mw + me ) C pw [(T2 - T1 ) + Tc ] - mfuse CV1 equivalent of the apparatus was determined as 2500 g.


CV = Calculate the calorific value of the coal in kJ/kg.
mf
...(16.34)
Pre Temp. Heating Temp. Cooling Temp.
where firing °C time °C period °C
Tc = radiation correction to temperature, it is Period (min) (min) time
obtained from graphical presentation of time (min)
observation before and after firing 0 25.730 6 27.840 10 27.880
mf = mass of fuel 1 25.732 7 27.880 11 27.878
mfuse = mass of fuse wire 2 25.734 8 27.883 12 27.876
mw = mass of water filled in calorimeter 3 25.736 9 27.885 13 27.874
me = water equivalent of the calorimeter 4 25.738 14 27.872
CV1 = Calorific value of fuse wire 5 25.740 15 27.870
T2 – T1 = Observed temperature difference
Solution
The bomb calorimeter measures a higher calo-
rific value of fuel. If a liquid fuel is being tested, it Given
is contained in a gelatin capsule and the firing may Mass of the coal, mf = 0.825 g
be assisted by paraffin of known calorific value in Total water equivalent of apparatus = 2500 g
the crucible.
To find Calorific value of the fuel in kJ/kg.
Example 16.26 The table given below gives the Assumptions
results of a calorimeter test of a sample of coal. The mass
of the coal burned was 0.825 gram and the total water (i) Negligible mass of fuse wire.
566 Thermal Engineering

(ii) Total water equivalent consists of mass of water Energy balance on the calorimeter yields to
and water equivalent of calorimeter. Heat liberated by fuel and fuse wire
(iii) Complete combustion of fuel. = Heat absorbed by water and calorimeter
Analysis The rate of cooling, mf CV + mfuse CV1
= (mw + me) Cpw [(T2 – T1) + Tc]
27.880 - 27.870
r = = 0.002°C/min Inserting numerical values
15 - 10
(2.25 ¥ 10–3 ¥ 26,480) + (0.025 ¥ 10–3 ¥ 6700)
Correction in the temperature,
= (2000 cc ¥ 10–3 kg/cc + me) ¥ 4.187
r 0.002
Tc = ◊ t firing = ¥ ( 4) = 0.004°C ¥ (5.2 + 0.1)
2 2 or 22.19 me + 44.3822 = 59.7475
Calorific value of the fuel, me = 0.6924 kg
mw Cpw [(T2 - T1 ) + Tc ] = 692.4 gram
CV =
mf
( 2500 ¥ 10 - 3 kg) ¥ ( 4.187 kJ/kg.K ) JUNKER’S GAS CALORIMETER
¥ [( 27.885 - 25.740) + 0.004]
= The calorific value of a gaseous fuel is determined
(0.825 ¥ 10 - 3 kg) by Junker’s or Boy’s gas calorimeter. The metered
= 27,202.8 kJ/kg gaseous fuel is continuously supplied to the calo-
rimeter at constant pressure, where it burns in the
Example 16.27 The following observations were tak-
presence of air as shown in Fig. 16.7. The products
en during a bomb calorimeter test to determine the water
of combustion are cooled to the initial temperature
equivalent of the calorimeter:
of the reactants by continuous circulating water.
Mass of the fuel burnt = 2.25 g
The gas temperature and pressure are measured
Calorific value of the fuel = 26,480 kJ/kg
and the amount of the gas burned is referred to
Mass of the fuse wire burnt = 0.025 g
25°C and 1.013 bar. The temperature rise of circu-
Calorific value of the fuse wire = 6700 kJ/kg
lating water is measured and the condensate from
Quantity of water in the calorimeter
the products of combustion is collected. This test is
= 2000 cc
carried out for a fixed time period. The water flow
Temperature rise during burning of fuel
rate and weight of condensate are measured. Thus,
= 5.2°C in 5 minutes
we have
Drop in temperature after burning of fuel
= 0.2°C in 5 minutes (Volume of fuel at 1.013 bar, 25°C)
Specific heat of water = 4.187 kJ/kg ◊ K ¥ Calorific value = Mass of water circulated
¥ specific heat ¥ temperature rise of water
Solution
Given Data as above V ¥ CV = mw Cpw ¥ ( T )w ...(16.35)

To find Water equivalent of the calorimeter. pgas Vgas


where V = ¥T
pTgas
Analysis The rate of cooling, 298 pgas Vgas
0.2
= ¥
r = = 0.04°C/min 1.013 ¥ 100 Tgas
5
Correction in temperature where p = 1.013 bar, atmospheric pressure
r 0.04 T = 25°C, standard reference temperature
= ◊ t firing = ¥ 5 = 0.1°C
2 2 In both the bomb calorimeter test and Junker’s
calorimeter test, the steam formed on the combus-
Fuels and Combustion 567

Constant
level tank

Flow restrictor
Overflow

Water
Water change- outlet Water from
over from waste temp. main supply
to collecting cylinder
Water
inlet
Effluent
temp.
gas temp.

Gas
temp.
Gas
Cooling meter Gas from the
B-Burners
coils main after a
quadrant valve
Water
level Gas pressure
To
waste
Condensate

Gas pressure regulator

Fig. 16.7

tion is condensed and the heat of steam formation Cpw = 4.2 kJ/kg ◊ K
is recovered. Thus, they measure a higher calorific T = 25°C
value of fuel. p = 760 mm of Hg
ms = 0.055 kg
Example 16.28 The following results were obtained hfg = 2442.5 kJ/kg ◊ K
during the trail on the gas calorimeter: Gas supplied
= 0.08 m3 at 32°C; Pressure of gas = 40 mm of water, To find
Barometric reading = 750 mm; Temperature of water at (i) Higher calorific value of the gas.
inlet and outlet 30°C and 38°C, respectively; Mass of (ii) Lower calorific value of the gas.
cooling water circulated = 24 kg; Specific heat of water
Assumptions
= 4.2 kJ/kgK; Steam condensed = 0.055 kg. Determine
(i) Density of water as 1000 kg/m3.
the higher and lower calorific value of the fuel at 25°C.
Assume standard barometric pressure = 760 mm of Hg, (ii) Density of mercury as 13.6 ¥ 103 kg/m3.
Take hfg = 2442.5 kJ/kg at 25°C. (iii) Acceleration due to gravity g = 9.81 m/s2.
Analysis
Solution
(i) The absolute pressure of the gas supplied
Given A test for determination of calorific value of a pgas = pgage + patm
gas = 40 mm of H2O + 750 mm of Hg
Vgas = 0.08 m3 T1 = 30°C
Ê 40 ˆ
= ¥ 9.81 ¥ Á
1000
Ë 1000 ˜¯
Tgas = 32°C = 305 K T2 = 38°C
pgas = 40 mm of water above 750 mm of Hg
Ê 750 ˆ
mw = 24 kg + 13.6 ¥ 103 ¥ 9.81 ¥ Á
Ë 1000 ˜¯
568 Thermal Engineering

= 392.4 + 100062 = 1,00,454.4 Pa ( T )water = 38 – 30 = 8°C


= 100.45 kPa Then the higher calorific value of the gas
Standard barometric pressure
mw Cpw ( DT ) w 24 ¥ 4.2 ¥ 8
p = 760 mm of Hg CV = =
V 0.0775
Ê 760 ˆ
= 13.6 ¥ 103 ¥ 9.81 ¥ Á ¥ 10–3 = 10,403.5 kJ/m3
Ë 1000 ˜¯
(ii) The lower calorific value of the gas
= 101.3 kPa
LCV = HCV – Mass of steam/m3 ¥ hfg
The volume of gas at standard reference state
0.055
pV pgasVgas = 10403.5 - ¥ 2442.5
= 0.0775
T Tgas
298 100.45 ¥ 0.08 = 8670 kJ/m3
or V = ¥ = 0.0775 m3
101.3 305

Summary
enthalpy of combustion.
chemical energy between its constituent elements. calorific value of the fuel is defined as the
The combustion is an exothermic process, in amount of heat energy liberated by complete
which the rapid oxidation of fuel takes place with combustion of a unit quantity of a fuel. It is also
a release of heat energy. called the heating value. It is measured in kJ/kg
or kJ/m3.
by mass and 21% of oxygen and 79% of nitrogen
by volume. The theoretical amount of air that the reactant’s temperature then the water vapour
supplies just sufficient oxygen for complete gets condensed and the heat of its vaporisation is
combustion of all elements of the fuel is termed recovered. Thus, the calorific value is called the
as stoichiometric air required. The air–fuel ratio higher calorific value or gross calorific value.
is defined as the ratio of mass of air to the mass net calorific value is the amount
fuel during the combustion process. of heat released by complete combustion of
unit quantity of fuel, when water vapour in the
on the volume basis. It uses the chemical- products is in gaseous form.
absorption technique. higher calorific value of powdered
enthalpy of formation is defined as the and liquid fuels is determined at constant volume
enthalpy of a substance at a specified state due to in the bomb calorimeter, while calorific value of
its chemical composition. During a combustion a gaseous fuel is determined by Junker’s or Boy’s
process, the amount of heat released is called the calorimeter at constant pressure.

Glossary
Fuel A combustible substance which burns in the Pulverised coal Crushed coal in fine power form
presence of oxygen and releases heat energy Reactants The components of fuel that exist before the
Briquettes Blocks, produced from finely grounded combustion reaction
low-grade coal by moulding operation Products The components that exist after combustion
reaction
Fuels and Combustion 569

Combustion An exothermic reaction with oxygen, in Equivalence Ratio Ratio of actual air–fuel ratio to the
which heat energy is released theoretical air–fuel ratio for complete combustion
Ignition temperature Lowest temperature at which a Standard reference state pref = 1 atm and Tref = 25°C
fuel starts burning (= 298 K).
3 T’s of Combustion Temperature, turbulence and time Enthalpy of Formation Enthalpy of an element at
for complete combustion standard reference state
Stoichiometric air Minimum amount for the complete Enthalpy of reaction Difference between the enthalpy
combustion of fuel elements of products and enthalpy of reactants at the same
Air fuel ratio Ratio of mass of the air to the mass of specificed state
the fuel Calorific value Amount of heat energy liberated by
complete combustion of unit quantity of a fuel

Review Questions
1. Define fuels and classify their various types. 11. Define (a) reactants, (b) products, (c) combustion,
2. Enumerate the advantages of liquid and gaseous and (d) ignition temperature.
fuels over solid fuels. 12. Write the physical laws of combustion.
3. What do you understand by higher and lower 13. Write the basic combustion equations.
calorific values of fuels? 14. Write the composition of air on mass and molar
4. Explain the procedure to determine the high- basis.
er calorific value of a solid fuel. 15. Define (a) stoichiometric air–fuel ratio (b) Excess
5. Explain the working of Boy’s gas calorimeter air (c) and equivalence ratio.
with the help of a neat sketch. 16. Define enthalpy of formation and enthalpy of
6. What is the use of Orsat apparatus ? Discuss its combustion.
working with the help of a neat sketch. 17. Explain the procedure to determine the calorific
7. Write the characteristic of an ideal fuel. value of gaseous fuel.
8. Explain the various stages of coal. 18. Define calorific value, and differentiate between
9. Write the steps involved in conversion of volu- net and gross calorific values.
metric analysis to gravimetric analysis. 19. Explain the construction and working of bomb
10. Write the steps involved in conversion of gravi- calorimeter.
metric analysis to volumetric analysis. 20. Define minimum and excess air for combustion
and air fuel ratio.

Problems
1. The volumetric analysis of a fuel is given as 2. The ultimate analysis of a solid fuel by mass
follows: gives
CO2 = 14%, CO = 1%, O2 = 5% and N2 = 80%. C = 86%, H2 = 12%, O2 = 1%, and S = 1%.
Obtain the flue gas composition by mass. Calculate the theoretical amount of air required
[CO2 = 20.23%, CO = 0.92%, for complete combustion at NTP and its volume
O2 = 5.26% and N2 = 73.59%] occupied. Assume specific volume of air at NTP
is 0.773 m3/kg. [14.1 kg, 10.9 m3]
570 Thermal Engineering

3. The mass analysis of coal is as follows: 9. In the actual combustion of benzene in an engine,
C = 77.2%, H2 = 5.2%, S = 1.25%, N2 = 1.5%, the A/F ratio was 12/1. Calculate the analysis of
O2 = 8.65 and the remainder is ash. Determine the wet products of combustion.
the theoretical amount air required for complete [CO2 13.38%; CO 3.94%, H2O 8.66%;
combustion and the volumetric analysis of prod- N2 74.03%]
ucts of combustion with 25% excess air. 10. The ultimate analysis of a sample of petrol was
[10.33 kg/kg of fuel; CO2 = 13.8%, O2 = 4.1%, C-85.5% and H-14.5%. Calculate the
H2O = 5.6%, SO2 = 0.1% and N2 = 76.4%] (a) stoichiometric A/F ratio
4. The volumetric analysis of flue gases obtained (b) A/F ratio when the mixture strength is 90%
from the combustion of an unknown hydrocarbon (c) A/F ratio when the mixture strength is
fuel is CO2 = 12.1%, CO = 0.5%, O2 = 3.2% and 120%
N2 = 84.2%.
(d) analysis of the dry products for (ii) and (iii)
Determine the excess air supplied in percentage
(e) the volume flow rate of the products
of theoretical air. [15%]
through the engine exhaust per unit rate of
5. The analysis of coal used in a boiler is car- fuel consumption for (iii) when the pressure
bon-82%, hydrogen-10%, oxygen-4% and ash- is 1.013 bar and the temperature is 110°C
4%.
[14.76/1; 16.4/1; 12.3/1; CO2 13.38%;
The boiler consumes coal at the rate of 0.2 kg/s
O2 2.24%; N2 84.38%; CO2 8.67%;
with 30% excess air. Calculate the
CO 8.79%; N2 82.54%;
(a) Volume flow rate of air at intake—the
15.11 m3/s per kg/s]
intake conditions are 1 bar and 20°C. Take
11. The ultimate analysis of a sample of petrol was
R = 0.287 kJ/kg ◊ K.
C-85.5% and H-14.5%. The analysis of the
(b) Percentage composition of dry flue gases
dry products gave 14% of CO2 and some O2.
by mass
Calculate the A/F ratio supplied to the engine and
[(a) 2.8 m3/s, (b) CO2 18%, O2 = 5.3%,
the mixture strength. [15.72/1; 94%]
N2 = 76.7%] 12. In an engine test, the dry product analysis was
6. During a trial on a boiler, the coal sample has CO2-15.5%; O2-2.3% and the remainder was N2.
been tested and analysed as carbon-89%, hydro- Assuming that the fuel burned was a pure hydro-
gen-4%, oxygen-3%, Sulphur-1% and the re- carbon, calculate the ratio of carbon to hydrogen
mainder being ash. Determine the minimum mass in the fuel, the A/F ratio used, and the mixture
of air required for complete combustion of 1 kg strength. [11.5; 14.84/1; 89.5%]
of coal. If 60% excess air is supplied, estimate the 13. A quantity of coal used in a boiler had the follow-
percentage of dry flue gases by mass. ing analysis : 82% of C; 5% of H; 6% of O; 2%
[11.53 kg/kg of coal, CO2 = 17.14%, of N, 5% of ash. The dry flue gas analysis showed
O2 = 8.34%, N2 = 74.52%] 14% of CO2 and some oxygen. Calculate the
7. A petrol sample contains 15% of hydrogen, 85% (a) oxygen content of the dry flue gas,
of carbon, and 50% excess air is supplied to ensure (b) A/F ratio and the excess air supplied.
complete combustion. Determine the percentage
[5.52%; 14.29/1; 31.8%]
analysis of dry products of combustion by mass.
14. The products of combustion of a hydrocarbon
[CO2 = 14.1%, O2 = 7.9%, N2 = 78%]
fuel, carbon to hydrogen ratio 0.85:0.15, are
8. A composition of petrol on mass basis was C =
found to be CO2-8%, CO-1%, O2-8.5%. Calculate
85.5%, and H2 = 14.5%. It burns with 20% excess
the A/F ratio for the process by two methods and
air. Calculate the precentage analysis of dry prod-
hence check the consistency of the data.
ucts of combustion by mass and convert them to
[23.70, 23.53]
volume basis.
Fuels and Combustion 571

15. A bomb calorimeter is used to determine the (b) If 50% excess air is supplied, determine the
calorific value of coal. Results obtained are as volumetric analysis of dry products of com-
follows: bustion.
Mass of coal = 1.5 g [(a) 11.583 kg/kg of fuel; CO2 = 27.00%;
Mass of water = 3.75 kg N2 = 73%, (b) CO2 = 18.31%,
Water equivalent of calorimeter = 1.0 kg O2 = 7.4% and N2 = 74.29%]
Temp. rise of cooling water = 2.62°C 19. The ultimate analysis of coal is as follows:
Cooling correction factor = 0.018°C C = 82%, H2 = 6%, O2 = 4% and remainder is ash.
Specific heat of water = 4.2 kJ/kg ◊ K Determine the amount of theoretical air required
Determine the calorific value of coal. for complete combustion. If the actual air sup-
[29458 kJ/kg] plied is 40% in excess and 80% of the given
16. The following data was obtained during an carbon is burnt to CO2 and the remainder to CO,
experimental determination of the calorific value determine the volumetric analysis of dry products
of a sample of coal by a bomb calorimeter. of combustion.
Carbon = 90% [11.41 kg/kg of fuel; O2 = 7.22%, N2 = 80.31%,
Hydrogen = 5% CO2 = 9.98% and CO = 2.49%]
Mass of coal = 0.8 g 20. A gaseous fuel has the following volumetric com-
Mass of fuse wire = 0.03 g position:
Calorific value of fuse wire = 7000 J/g CO2 = 5.5%, CO = 38.3%, CH4 = 0.4%,
O2 = 0.1%, H2 = 52.8% and N2 = 2.9%.
Mass of water in calorimeter = 2000 g
Determine the air–fuel ratio by volume and the
Water equivalent of colorimeter = 400 g
analysis of dry products of combustion, if 10%
Temp. rise of cooling water = 3.2°C
excess air is supplied.
Cooling correction = 0.02°C
[2.42:1; CO2 = 18.2%, O2 = 1.9%,
Calculate higher and lower calorific values of the
N2 = 79.9%]
fuel. Take hfg = 2465 kJ/kg.
21. A fuel mixture with volumetric analysis of 94.4%
[45815.5 kJ/kg, 44708 kJ/kg]
of CH4, 3.4% of C2H6, 0.6% of C3H8, 0.5% of
17. The following results were obtained during ex- C4H10 and 1.1% of N2, burns completely with
perimentation on a Boy’s gas calorimeter: 20% excess air in a reactor operating steadily.
Gas used = 2 ¥ 10–3 m3 at 30°C The fuel flow rate is 0.1 kmol/h. Determine the
Water circulated = 0.5 kg volume flow rate of air in kmol/h.
Temperature rise of water = 10.1°C 22. A coal sample has gravimetric analysis of 77.39%
Manometer reading = 60 mm of Hg of carbon, 4.1% of hydrogen, 5.31% of oxygen,
Barometer pressure = 750 mm of Hg 1.62% of nitrogen, 1.1% of sulphur and the rest
Specific heat of water = 4.2 kJ/kg ◊ K is ash. For complete combustion, 110% of the
Steam condensed = 0.0015 kg theoretical amount of air is supplied. Calculate
Calculate higher and lower calorific values of the the air–fuel ratio on the mass basis.
gas at 1 atmosphere and 25°C. 23. Decane (C10H22) burns with 95% of theoretical
[10117.3 kJ/m3, 8368 kJ/m3] air, producing a gaseous mixture of CO2, CO,
18. The percentage analysis of the fuel used in a boil- H2O and N2. Determine the
er by mass is as follows: (a) air–fuel ratio on a molar basis,
C = 90%, H2 = 3.5%, O2 = 3%, N2 = 1%, S = 1% (b) analysis of products on a dry molar basis.
and the remainder is ash. 24. A boiler uses coal which has the mass composition
(a) Find theoretical amount of air required and as: C = 84%, H2 = 3% and the remainder as ash.
the mass analysis of dry products of com- The Orsat apparatus gave the volumetric analysis
bustion. as follows: CO2 = 11.5%, O2 = 8.4% and N2 =
572 Thermal Engineering

80.1%. Calculate the air–fuel ratio and the mass excess or per cent deficiency of air, whichever is
of excess. appropriate.
[17.8:1; 7.0 kg] 27. A certain kind of petrol consists of 86% of car-
25. The volumetric analysis of dry exhaust gases as bon and 14% of hydrogen by mass. If the fuel is
determined by an Orsat apparatus is CO2 = 10%, burnt with 20% excess air and the combustion
CO = 11.5% and O2 = 8%. is complete, calculate volumetic composition of
The fuel used has the mass composition as C = products of combustion.
80%, H2 = 6%, O2 = 7% and the remainder being [CO2 = 10.97%, H2O = 10.73%,
ash. Determine by weight the air supplied per kg O2 = 3.27%, N2 = 75%]
of fuel and the excess air supplied. 28. The enthalpy of combustion of propane gas,
[17 kg/kg of fuel; 5.95 kg/kg of fuel] C3H8, at 25°C with H2O in the products in the
26. 100 kmol of propane (C3H8) together with liquid phase is – 50360 kJ/kg. Calculate the
3572 kmol of air enters a furnace per unit time. enthalpy of combustion with H2O in the vapour
The products are carbon dioxide, carbon monox- phase per unit mass of fuel and per unit amount
ide and unburnt fuel. Determine the percentage of substance of fuel.
[ – 46364 kJ/kg; – 2040030 kJ/kmol]

Objective Questions
1. The main constituents of a fuel are (c) anthracite (d) pulverised coal
(a) hydrogen and oxygen 7. Coking is
(b) carbon and hydrogen (a) burning of coal in a furnace
(c) sulphur and hydrogen (b) formation of coke in a boiler furnace
(d) sulphur and oxygen (c) heating of coal in absence of air
2. Which of the following elements is not a constitu- (d) none of the above
ent of coal? 8. The fuel produced when wood is heated with a
(a) Hydrogen (b) Manganese limited supply of air at 280°C is
(c) Nitrogen (d) Carbon (a) coke (b) wood charcaol
3. The amount of heat generated per kg fuel is (c) bituminous coal (d) briquetted coal
known as
9. The ultimate analysis of coal is done to determine
(a) higher calorific value the percentage of
(b) lower calorific value
(a) carbon (b) ash
(c) calorific value
(c) sulphur (d) moisture
(d) none of the above
10. The symptom showing incomplete combustion of
4. The variety of coal having lowest calorific value
coal is
is
(a) presence of free carbon in exhaust
(a) steam coal (b) lignite
(b) presence of oxygen in exhaust
(c) anthracite, (d) bituminous coal
(c) presence of free nitrogen in exhaust
5. The variety of coal having highest calorific value
(d) presence of free CO in exhaust
(a) steam coal (b) lignite
11. Calorific value of a liquid or solid fuel is the
(c) anthracite, (d) bituminous coal
amount of heat liberated in kJ by
6. The fuel generally used in cement industries and
(a) complete combustion of 1 kg of fuel
metallurgical furnaces is
(b) complete combustion of 1 m3 of fuel
(a) hard coke (b) lignite
Fuels and Combustion 573

(c) temperature rise of fuel by 1°C 17. One kg of CO requires 4/7 kg of oxygen and the
(d) none of the above mass of carbon dioxide produced is
12. The calorific value of a gaseous fuel is the amount 7 8
(a) kg (b) kg
of heat liberated in kJ by 3 3
(a) complete combustion of 1 kg of fuel 11 11
(c) kg (d)
(b) complete combustion of 1 m3 of fuel 3 7
(c) temperature rise of fuel by 1°C 18. The mass of carbon per kg of flue gas is given by
(d) none of the above 11 3 3 3
(a) CO 2 + CO (b) CO 2 + CO
13. A good fuel has 3 7 11 7
(a) low ignition point and high calorific value 3 11 7 11
(b) low ignition point and low calorific value (c) CO 2 + CO (d) CO 2 + CO
7 3 3 3
(c) high ignition point and high calorific value
19. Which of the follwing is not a petroleum product?
(d) high ignition point and low calorific value
(a) Petrol (b) Kerosene
14. The amount of heat obtained by complete com-
(c) Methylated sprit (d) Lubricating oil
bustion of 1 kg of fuel when the products of com-
20. Liquid fuels as compared to solid fuels have the
bustion are cooled to the temperature of supplied
calorific value.
air is known as
(a) higher (b) lower
(a) calorific value
(c) same (d) none of the above
(b) higher calorific value
21. A bomb calorimeter is used to determine
(c) lower calorific value
(a) higher calorific value of solid or liquid fuel
(d) none of the above
(b) lower calorific value of solid or liquid fuel
15. One kg of carbon requires 4/3 kg of oxygen and
the mass of carbon monoxide produced is (c) higher calorific value of gaseous fuel
(d) lower calorific value of gaseous fuel
7 8
(a) kg (b) kg 22. In a bomb calorimeter, the fuel burns at constant
3 3
(a) volume (b) pressure
11 11
(c) kg (d) (c) temperature (d) entropy
3 7
23. The gas having higher calorific value is
16. One kg of carbon requires 8/3 kg of oxygen and
the mass of carbon dioxide produced is (a) water gas (b) coke-oven gas
(c) blast-furnace gas (d) producer gas
7 8 24. The gas having lower calorific value is
(a) kg (b) kg
3 3 (a) coal gas (b) coke-oven gas
11 11 (c) blast-furnace gas (d) producer gas
(c) kg (d)
3 7

24. (c) 23. (b) 22. (a) 21. (a) 20. (a) 19. (c) 18. (b) 17. (d)
16. (c) 15. (a) 14. (c) 13. (a) 12. (b) 11. (a) 10. (d) 9. (a)
8. (b) 7. (c) 6. (d) 5. (c) 4. (b) 3. (c) 2. (b) 1. (b)
Answers
574 Thermal Engineering

17
Steam Generators

Introduction
A steam boiler or steam generator is a closed vessel in which water is heated, vaporised and converted into
steam at a pressure higher than the atmospheric pressure. The heat energy required for steam generation is
produced by burning fuel in the furnace. The steam produced in the boiler may be used for producing power,
for industrial processes or for heating purposes. All power plants as well as industries are using steam.
The advantages of the use of steam are the following:
1. It is capable of supplying process heat at constant temperature while condensing.
2. It is cheap, and can be produced everywhere.
3. It is a clean, odourless and tasteless source of heat energy.
4. It can be used repeately again and again as well as first used for power generation and then for process
heating.
5. Its flow rate can easily be controlled and readily distributed.
The main applications of steam are the following:
1. The heat content of steam is large and thus it is suitable for process heating (for curing, drying, etc.) in
many indusrtries like sugar mills, textile mills, and chemical industries.
2. It is also used for power generation in thermal power plants.
3. Due to its large heat content, steam is used for cooking items like steamed rice, idlies, etc.
4. Steam can also be used for heating buildings and producing hot water in winter.
5. Steam is also used for creation of vacuum, ejection of gases and sterlisation.

to such a vessel, which is wholly or partly under


pressure when the steam is shut off.
The Indian Boiler Regulations (IBR) Act 1923
An IBR Steam Boiler means any closed vessel by the Government of India, was enacted to
exceeding 22.75 litres in capacity and which is used consolidate and amend the law relating to steam
expressively for generating steam under pressure. boilers. The main features of Indian Boiler
It includes any mounting or other fitting attached Regulations are as follows:
Steam Generators 575

1. The boiler should be operated after it has 1. According to Relative Passage of Water and Hot
been registered with the chief inspector of Gases
boilers. (a) Water Tube Boiler A boiler in which the water
2. The maximum working pressure of the flows through a number of small tubes which are
boiler has to be determined by the Boiler surrounded by hot combustion gases, e.g., Babcock
Inspector. In any circumstances, the operator and Wilcox, Stirling, Benson boilers, etc.
should not run the boiler above this pressure.
(b) Fire Tube Boiler The hot combustion gases pass
3. In case of any accident, the boiler owner
through the boiler tubes, which are surrounded
should submit a report containing full details
by water, e.g., Lancashire, Cochran, locomotive
of nature and cause of accident within
boilers, etc.
24 hours of occurrence of the accident.
4. The rules and regulation, procedure for 2. According to Water Circulation Arrangement
registration, inspection, decision upon
(a) Natural Circulation Water circulates in the
maximum working pressure and other
boiler due to density difference of hot and cold
conditions, etc., are subjected to revision
water, e.g., Babcock and Wilcox boilers, Lancashire
by a Central Board under the control of the
boilers, locomotive boilers etc.
Government of India.
5. Violation of any law in the Boiler Act is (b) Forced Circulation A water pump forces the
liable to prosecution and punishment with water along its path, therefore, the steam generation
fine.

The boiler system comprises of a feed-water system,


steam system and fuel system. The feed-water
system supplies treated water to the boiler and
Water tube B Flue gases
regulates it automatically to meet the steam to chimney
demand. Various valves and controls are provided
to access for maintenance and monitoring.
The steam system heats and vaporises the feed
water and controls the steam produced in the boiler. Grate
Furnace
Steam is directed through a piping system to the
(a)
application. Throughout the system, steam pressure
is regulated using valves and monitored with steam
pressure gauges. Chimney
The fuel system consists of all equipment used to
supply of fuel to generate the necessary heat. The
equipment required in the fuel system depends on
the type of fuel used in the system.
Fire tube
Fire Bridge
Water

There are a large number of boiler designs, but (b)


they may be classified according to the following
criteria:
576 Thermal Engineering

rate increases, e.g., Benson, La Mount, Velox 7. According to Charge in the Furnace
boilers, etc. (a) Pulverised fuel,
3. According to the Use (b) Supercharged fuel, and
(c) Fluidised bed combustion boilers
(a) Stationary Boiler These boilers are used for
power generation in thermal power plants or
process steam in plants.
Generally, a boilers consists of the following parts:
(b) Portable BoilerThese are small units of mobile
The boiler drum consists of a shell and
boilers and are used for temporary uses at the sites.
end heads. The shell of the boiler consists of one or
(c) LocomotiveThese are specially designed boilers. more steel plates bent into the cylindrical form and
They produce steam to drive railway engines. riveted or welded together. The ends of the shell are
closed by means of the flat or curved plates called
(d) Marine Boiler These are used on ships.
boiler head and a boiler drum is formed.
4. According to Position of the B Horizontal,
inclined or vertical boilers It is also called foundation and is
constructed of bricks. It supports the boiler drum
5. According to Position of Furnace and other components. It forms the wall of the
(a) Internally Fired The furnace is located inside furnace, combustion chamber and passage to flue
the shell, e.g., Cochran, Lancashire boilers, etc. gases.

(b) Externally Fired The furnace is located outside It is the space located below the furnace
the boiler shell, e.g., Babcock and Wilcox, Stirling and consists of cast-iron bars upon which the fuel
boilers, etc. is burned. The air can pass through the spaces
between the bars and can support the combustion
6. According to Pressure of Steam Generated
process, the ash can fall down through these spaces.
Low-Pressure Boiler A boiler which produces steam
It is the space above the grate and
at pressures of 15–20 bar is called a low-pressure
below the boiler shell. It is the space where the
boiler. This steam is used for process heating.
volatile matter and combustible gases are burnt and
Medium-Pressure Boiler It has a working pressure flue gases are generated.
of steam from 20 bar to 80 bar and is used for power
It is the hot mixture of products of
generation or combined use of power generation
combustion, generated in the furnace.
and process heating.
High-Pressure Boiler It produces steam at a The hot gas passage in the boiler is known
pressure of more than 80 bar. as flue. It provides the direction to the hot gases to
pass around the boiler.
(b) Sub-critical Boiler If a boiler produces steam at
a pressure which is less than the critical pressure It is the surface of the boiler
(221.1 bar), it is called sub-critical boiler. which is exposed to hot flue gases on one side,
water on other side.
(c) Supercritical BoilerThese boilers produce steam
at a pressure greater than critical pressure. These It is a mechanical system for charging
boilers do not have an evaporator and the water of coal to the furnace and keep the firing continued.
directly flashes into steam and thus they are called The closely spaced water tubes
once through boilers. arranged near the furnace wall form a layer like
Steam Generators 577

a wall and hence are called water wall. The tube 4. Its erection time should be less and its parts
surface in the water wall receives the heat by should be easily dismantable.
radiation. 5. The boiler should have reliable controls and
safety apparatus.
The space of the boiler shell
6. It should have high rate of heat transfer and
occupied by water is called the water space. The
better combustion efficiency.
level of water in the boiler can be seen through the
7. It should be able to accomodate the load
water level indicator.
variation.
The entire space of boiler shell 8. It should occupy less floor space.
which is not occupied by the water is called steam 9. It should be trouble free, and require less
space. attention and less maintenance.
10. It should be free from manufacturing defects.
The water supplied to the boiler
is called feed water. The pump which supplies the 11. Mud should not get deposited on the heating
water is called feed pump. surface. Soot or scale should not be deposited
on the tubes.
It is the pressure of steam 12. All parts of the boiler should be accessible
generated in the boiler and superheater. for cleaning and inspection.
13. It should conform to IBR acts.
The feed water supplied by the
feed pump is heated by the waste hot gases before
they escape to the chimney. Therefore, some of the
waste heat is recovered by feed water and plant
efficiency improves. The following factors should be considered, while
selecting a boiler:
Similar to feed water heating,
1. The steam-generation rate of the boiler at
the fresh air going to the furnace is also preheated
working pressure,
to improve the combustion process. The air-heating
2. The type of fuel used and its rate of burning,
system device is known as the air preheater.
3. Availability of fuel and water,
This is the device which heats 4. Floor space occupied by the boiler,
the saturated steam generated in the boiler. The 5. The type of load—steady, fluctuating, etc.
superheaters are located above the furnace and 6. Initial cost, running cost and maintenance
they increase the heat content of steam without cost of the boiler.
increasing its pressure.

A good boiler should have the following


See Table 17.1
characteristics:
1. The boiler should have maximum steam-
generation rate with minimum fuel con-
sumption.
2. It can be started or stopped quickly. It is the simplest form of an internal furnace,
3. Its initial cost, running and maintenance cost vertical fire-tube boiler as shown in Fig. 17.2. It is
should not be high. a portable boiler and it requires a small floor space.
578 Thermal Engineering

Sr. Fire-tube boiler Water-tube boiler


1. The hot flue gases pass through tubes and water Water passes through tubes and hot flue gases
surrounds them. surround them.
2. These are operated at low pressures up to 20 bar. The working pressure is high enough, up to 250 bar
in supercritical boilers.
3. The rate of steam generation and quality of steam The rate of steam generation and quality of steam
are low, therefore, not suitable for power generation. are better and suitable for power generation.
4. Load fluctuations cannot be handled. Load fluctuations are easily handled.
5. It requires more floor area for a given output. It requires less floor area for a given output.
6. They are bulky and difficult to transport. These are light in weight, hence transporation is not
a problem.
7. Overall efficiency is up to 75%. Overall efficiency with an economiser is up to 90%.
8. Water does not circulate in a definite direction. Direction of water circulation is well defined.
9. The drum size is large and damage caused by If any water tube is damaged, it can be easily
bursting is large. replaced or repaired.
10. Less initial cost, but cost per unit is more. Initial cost is very high, but cost per unit is low.
11. Simple in design, easy to erect and low maintenance Complex design, difficult to erect and high
cost. maintenance cost.
12. Less skill is required for efficient operation. Skilled operators are required.
13. The treatment of feed water is not very essential, as Treatment of feed water is very essential as small
overheating due to scale formation cannot burst the scale deposits inside the tubes can cause overheating
thick shell. and bursting.
14. Used in process industry. Use in large power plants.

The steam rating normally does not exceed 2500 kg


per hour and pressure is limited to 10 bar.
It consists of a vertical, cylindrical
shell, equipped with a fire box in the bottom, water
space in the middle and steam space in the upper
portion. The grate is placed at the bottom of the fire
box and coal is fired in the fire box. An ash pit is
located at the bottom of the grate to collect the ash
of burnt coal, which is periodically removed.
One or more cross tubes are either flanged or
riveted to the water space and are located in the
fire box to increase the heating surface area and
to improve the water circulation. A short chimney
is connected at the top of the fire box to discharge
the waste flue gases at some greater height. Man
hole and hand holes are provided for cleaning the
interior of the boiler shell and cross tubes.
Steam Generators 579

The boiler consists of a pressure gauge, water-


level indicator, safety valve, steam stop valve and a
man hole as mountings to provide safety and ease
of working.
Fuel burns on the grate in the fire box.
The resulting hot flue gases are allowed to pass
around the cross tubes. The water surrounding the
cylindrical fire box also receives heat by convection
and radiation. Thus steam is produced. The water
circulation in the boiler depends on the density
difference in the water, created by temperature
difference in the water.

It is a vertical, coal-
or oil-fired, fire-tube boiler. It is the modification of
a simple vertical boiler with increase in the heating
surface area. The flue gases from the furnace are
passed through a number of small tubes surrounded
by water.
The man hole is provided in the the crown of
As shown in Fig. 17.3, a Cochran boiler consists boiler for periodic cleaning and maintenance. A
of a cylindrical shell with a hemispherical crown, mud hole is provided at the bottom for draining
grating, fire box, combustion chamber, number out the muddy water from the boiler. The pressure
of smoke tubes, smoke box, chimney and various gauge, water gauge, blow off cock, feed check
mountings. The grate is placed at the bottom of the valve, feed pump, fusible plug and chimney are
hemispherical furnace. The coal is fed into the grate provided for proper functioning of the boiler.
through the fire door and ash formed is collected in The Cochran boilers are made in sizes from
the ash pit located just below the grate, and then it 1 m to 3 m in diameter, 2 m to 6 m heigh. Its steam
is removed manually. generation rate is approximately 3600 kg/h with
The fuel is burnt on grating. The hot- working pressure limited to 11 bar.
flue gases pass through a short flue to a combustion Salient Features
chamber, small horizontal smoke tubes and are then
1. The spherical crown and spherical shape of a
collected in the smoke box, from where they are
fire box are the special features of this boiler.
discharged to the atmosphere through the chimney.
These shapes require least material for a
The heat is transferred to water by radiation given volume.
through the dome of the fire place and by convection 2. It is very compact and requires minimum
from the walls of the smoke tubes. On heating, the floor area.
water is vaporised and converted into steam. The
3. Any type of fuel can burn in the boiler.
generated steam is collected in the steam space
4. It is well suited for small industries.
above the water. This steam is then taken for use
5. It gives about 70% thermal efficiency with
through the main steam stop valve.
coal firing.
580 Thermal Engineering

low water and high steam alarm gives an audio


It is a horizontal, internally fired, fire tube, natural signal for low water level and high steam pressure.
circulation, stationary boiler. It is a widely used The blow off cock, and feed check valve are also
boiler due to its good steam-generation capacity. provided in front of the boiler and the fusible plug
This boiler can also be used for power generation at is provided in the main flues just over the grates
a moderate steam pressure of 15 bar. to prevent the overheating of boiler tubes by
extinguishing the fire, when water level falls below
As shown in Fig. 17.4, the boiler a particular level.
consists of a large shell supported by refractory
brick masonry. The cylindrical shell is usually 2 The fuel is burnt at the grating and the
to 3 m in diameter and 7 to 9 m long. Two large, hot gases travel along internal flue tubes followed
horizontal and parallel flue gas tubes pass through by flue passage A and then in side passages B and
shell. The fire place is located in front of the flue C. The flue gases are then collected in the chamber
tubes. In brick work, a flue passage A below the before they lead to the atmosphere through a
boiler shell, two flue passages B and C at the sides chimney. The hot flue gases transfer its maximum
of boiler are formed. The flue passages B and C are heat contents to water during its long passages. The
connected to a chamber and then to the chimney. water is converted into steam and collected in the
steam space in the shell and it is then taken out
The dampers in the form of sliding doors are
through the steam stop valve for use.
located at the end of side flues to control the flow of
gases. They regulate the combustion rate as well as Special Features
steam-generation rate. These dampers are operated 1. Its heating surface area per unit volume is
by a chain passing over a pulley at the front of the considerably large.
boiler. 2. Its maintenance is easy.
The boiler is also provided with usual mountings 3. This boiler can easily handle the load
like pressure gauge, water level indicator, steam fluctuation to large steam capacity.
stop valve, safety valve, low water and high steam
4. It is highly suitable for process industries.
safety valve, man hole on the top of the shell. The
Steam Generators 581

Cornish Boiler Thus, heating rate remains unaffected. The steam-


The Cornish boiler is very similar to a Lancashire generation capacity is 6500 kg/h at a pressure of
boiler. It is also a horizontal, fire tube, internally 10.5 bar.
fired, natural circulation, stationary boiler. However,
Locomotive Boiler
it differs from a Lancashire boiler in two respects.
It is also an internally fired, horizontal, multi-tube,
1. It is small in size.
mobile, fire-tube boiler. The Locomotive boiler
2. It has only one flue tube.
generates steam at a pressure of about 25 bar with
The Cornish boiler as shown in Fig. 17.5 consists a steam rate of 60–70 kg/h per square metre of the
of a horizontal cylindrical shell of 1.25 m to 1.75 m heating surface. This steam rate is quite high and
diameter and 4 m to 7 m long. It has flat ends. thus it is mostly used on locomotives for generating
The flue tubes containing the furnace are located steam to drive a steam engine train.
in the centre of the boiler shell. The products of
A view of the locomotive boiler is
combustion from the fire grate first pass forward
shown in Fig 17.6. It consists of the following main
through the central flue tube and then return by the
parts:
two side flues to the front of the boiler and again
pass to the back end of the boiler along the bottom 1. Boiler shell (horizontal)
flue and finally get discharged through chimney. 2. Fire box surrounded by water
The various mountings are provided on the boiler 3. Smoke tubes
to provide safety. These are pressure gauge, water- 4. Super-heater tubes
level indicator, steam stop valve, fusible plug, blow 5. Smoke box
off cock, high steam and low water safety valve, 6. Chimney
feed check valve and man hole. 7. Blast pipe
It has a unique feature that any sediments in 8. Damper
the feed water are deposited at the bottom of the 9. Steam Dome with regulator
shell, which is not in the zone of hottest flues gases.
10. Safety valve and other mountings

Fig. 17.5 Cornish boiler


582 Thermal Engineering

It has horizontal shell, 1.5 m is diameter and through a nozzle at the top of the blast pipe (exhaust
4 m long. The coal is fed into the fire box through pipe). The jet of steam draws the flue gases to the
the fire door and burnt on the grate. The entire fire atmosphere through the short chimney, and thus
box is properly heated by deflecting the flue gases creates sufficient suction in the fire box to suck the
from the grate by a bridge. The heat is transferred to fresh air.
water through the walls of the fire box, smoke tubes
and superheater tubes.
The flue gases are formed due to The Scotch Marine boiler is the most commonly
combustion of coal in presence of air on the grate. used fire-tube marine boiler. It has a large heating
These gases rise up and are deflected by a brick surface area for the space occupied. It has excellent
arch for their proper distribution to pass through the steaming capacity of about 1000 kg/h at a pressure
smoke tubes and over superheater tubes and then of 17 bar. It is compact in size and occupies a
finally get discharged into the atmosphere through small floor space. It is a self-contained boiler and
a short chimney. supported by a cradle, securely fastened to the
The steam generated is collected in the steam frame of the ship. Adjustable stays hold the boiler
space above the water in the boiler drum. A steam in place in the cradle.
regulator is located in the steam dome and is Figure. 17.7 shows the schematic
operated by a long regulator rod from the engine arrangement of a Scotch Marine boiler. The boiler
cabin by the driver. When this valve is opened, the consists of a cylindrical shell that houses one to
wet steam passes through the superheater header four cylindrical, corrugated steel furnaces. These
and to the superheated tubes located in a smoke furnaces are internally fired and surrounded by
tube. As steam passes through superheated tubes, it water. A combustion chamber is located above the
picks up additional heat and becomes superheated. furnace and it is also surrounded by water. The
The superheated steam is then supplied to the steam boiler has a number of tubes passing from the front
engine. plate of combustion chamber to the front plate of
Since the long chimney cannot be installed on the shell. These tubes are surrounded by water.
a locomotive engine, thus sufficient suction effect The fuel burns in the furnace on the
(natural draught) cannot be created. Therefore, grate. The hot flue gases resulting due to burning
the exhausted steam from the engine is discharged of fuel move to the combustion chamber. Then they
Steam Generators 583

is shown in Fig. 17.8 and its construction and


operation are stated below.
Construction
1. A horizontal Steam and Water Drum This is the
main part of the boiler. It is supported by a steel
structure at a certain height and is independent of
brick works. The size of the boiler drum is small as
compared with the boiler drum of a fire-tube boiler
of same capacity. It contains water and steam. All
safety and control devices are mounted over the
boiler drum.
2. A Bundle of Steel Tubes The front end of the boiler
drum is connected to the uptake header (water box)
travel to the smoke box through the fire tubes and by a short tube and the rear end is connected to the
finally get discharged to the atmosphere through downtake header (water box) by a long tube. In
uptake and chimney. The heat is transferred to between the headers, a number of small-diameter
water around the furnace, combustion chamber, steel tubes are fitted at an angle of 5° to 15° with the
and fire tubes and steam is generated. horizontal to promote the water circulation. These
steel tubes are arranged in the combustion chamber
in a zigzag manner so that more surface area of the
tube is exposed to hot gases.
3. Combustion Chamber It is the space above
The babcock and Wilcox boiler is probably the the grate, below the front end of the drum where
first water tube boiler designed and widely used. combustion of fuel takes place. This chamber
A schematic of the Babcock and Wilcox boiler is enclosed by brickwork and it is lined from
584 Thermal Engineering

inside by fire bricks. Doors are provided to give it is finally passed through the superheater tubes
access for cleaning, inspection and repairing. for its superheating. The superheated steam is then
The Combustion chamber is divided into three available for use.
separate compartments by baffles. Thus, the first
Special Features
compartment above the furnace is the hottest and
the last chamber is of lowest temperature. This 1. Its evaporating capacity is quite high
makes the path of hot gases longer before leaving compared with other boilers (20,000 to
the boiler through the chimney. The superheater is 40,000 kg/h). The operating pressure lies
placed between the drum and water tubes. During between 11.5 to 17.5 bar.
the first turn of the hot gases, the gases are passed 2. The draught losses are minimum.
over superheater tubes. Dampers are provided at 3. The defective tubes can be replaced easily.
the rear end of the chamber to regulate the fresh air 4. The entire boiler rests on an iron structure,
supply for maintaining proper combustion of fuel. independent of brick structure.
4. Safety and Control Devices Safety and control
devices are called mountings, as basically these
devices are mount over a boiler drum. These are the It is a water-tube boiler and has two water steam
safety valve, pressure gauge, water-level indicator, drums at the top and a mud drum at the bottom.
feed check valve, steam-stop valve, blow of cock, These drums are connected by a number of small
fusible plug and man hole. bent tubes through which water flows. The use of
bent tubes avoids the thermal stresses at the joints
The water is pumped by a feed pump near the drum due to expansion and contraction.
and it enters the drum through the feed check valve Figure 17.9 shows the principle working of a
up to the prespecified level so that the headers and Stirling boiler. The steam drums containing water
tubes are flooded always. When the combustion and steam are connected in series by the tubes
takes place above the grate, the products of hot above and below the water level. The upper tubes
gases come out and rush through each compartment are steam-circulating tubes and are used to equalise
of the combustion chamber. Hence, the front the pressure and the lower water circulating tubes
portion of the tubes has highest temperature and the maintain same level of water in all drums.
rear portion has the lowest. When water is heated
Water is supplied to the first drum, which then
inside the tubes, it becomes lighter and rises up
passes to the mud drum through the rear bank of
in the tube. Due to continuous heat supply, some
the tubes. The suspended impurities are collected
of the water gets vaporised into steam inside the
in the mud drum. The water from the mud drum is
tubes and the mixture of water and steam enters the
circulated to other upper drums.
boiler drum through the uptake header. The cold
The flue gases from the grate rise above and
water from the boiler drum comes down through
pass over the bent tubes and then are deflected by
the downtake header and enters the lower end of baffles for their proper distribution. Finally they are
the water tubes for getting heated further. This discharged to the atmosphere through the chimney.
natural circulation of water remains continuous due
to difference in temperature. Such a circulation is
called thermosiphon system.
The steam generated gets collected in the steam
space above water space in the boiler drum. In The packaged boiler is so called because it comes
order to remove all water particles from the steam, as a complete package. Once delivered to site, it
Steam Generators 585

Fig. 17.9

requires only steam, water pipe work, fuel supply The salient features of package boilers are
and electrical connections to be made for it to (i) Small combustion space and high heat re-
become operational. Package boilers are generally lease rate resulting in faster evaporation,
of a shell type with fire-tube design so as to achieve
(ii) Large number of small-diameter tubes lead-
high heat transfer rates by both radiation and ing to good convective heat transfer,
convection (refer Fig. 17.10).
(iii) Forced or induced draft systems resulting in
good combustion efficiency,
(iv) Number of passes resulting in better overall
heat transfer,
(v) Higher thermal efficiency levels compared
with other boilers.
These boilers are classified based on the number
of passes—the number of the hot combustion gases
passes through the boiler. The combustion chamber
is taken as the first pass after which there may
be one, two or three sets of fire tubes. The most
Fig. 17.10 common boiler of this class is a three-pass unit with
586 Thermal Engineering

two sets of fire-tubes and with the exhaust gases Steam

exiting through the rear of the boiler. Combustion


gases
Coal
Pulverized Fuel Boiler
Most coal-fired power station boilers use pulverized
coal, and many of the larger industrial water-tube
boilers also use this pulverized fuel. This technology
Water
is well developed, and there are thousands of units
around the world, accounting for well over 90% of
Ash
coal-fired capacity. The coal is ground (pulverised) Compressed air
to a fine powder, so that less than 2% is 300 micro
Fig. 17.12
metre (mm) and 70–75% is below 75 microns, for
bituminous coal. supported on a fine mesh. The fuel in powder form
The pulverised coal is blown with a part of is fed downward. The air velocity supports the fuel
the combustion air into the boiler plant through a in the air and creates bubble formation, vigorous
series of burner nozzles. Secondary and tertiary turbulence and rapid mixing and combustion of
air may also be added. Combustion takes place fuel, and the bed is said to be fluidized.
at temperatures from 1300 –1700°C, depending If the sand in a fluidized state is heated to the
largely on coal grade. Particle residence time in the ignition temperature of the coal and the coal is
boiler is typically 2 to 5 seconds, and the particles injected continuously in to the bed, the coal will burn
must be small enough for complete combustion to rapidly, and the bed attains a uniform temperature
have taken place during this time. due to effective mixing. Proper air distribution is
This system has many advantages such as ability very important for maintaining uniform fluidisation
to fire varying quality of coal, quick responses to across the bed.
changes in load, use of high pre-heat air temperatures, Fluidised bed combustion has significant
etc. One of the most popular systems for firing advantages over conventional firing systems and
pulverized coal is the tangential firing using four offers multiple benefits, namely, fuel flexibility,
burners corner to corner to create a fireball at the reduced emission of noxious pollutants such as
centre of the furnace as shown in Fig. 17.11. SOx and NOx, compact boiler design and higher
combustion efficiency.

Supercharged Boiler
In a supercharged boiler, the combustion is carried
under the pressure in the combustion chamber.
More mass of air is supplied by increasing the
density of air in an air compressor. Thus more
fuel can burn in the presence of more oxygen. The
exhaust gases from the combustion chamber are
Fig. 17.11 used to spin a gas turbine. The gas turbine drives a
rotary compressor to supply compressed air to the
Fluidised Bed Combustion Boiler combustion chamber. The supercharging boiler has
certain advantages:
In the fluidised bed combustion boiler, Fig. 17.12,
the slightly compressed air is passed upward through 1. With the compressed air, the convection
a finely divided bed of solid particles such as sand heat-transfer coefficient increases, which
Steam Generators 587

increases the heat-transfer rate. Thus, less with the use of small-diameter tubes. It helps in
combustion space is required in comparing high rate of heat transfer to water flowing inside
to a conventional boiler. the tubes. Therefore, a large number of small
2. A part of the gas turbine can be used to drive diameter tubes in a zigzag manner are used for
other auxiliaries. water circulation in forced circulation. Further, use
3. Rapid start and efficient combustion. of a short tube reduces the pressure loss and gives
better control over the quality of steam.

The
A boiler is called a high-pressure boiler when it steam is generated at a pressure between 80 bar to
operates with a steam pressure above 80 bar. The 300 bar and temperature of 450°C to 585°C with
high-pressure boilers are widely used for power two superheaters in series. The use of such steam
generation in thermal power plants. is very suitable for power generation. It increases
If the feed-water pressure increases, the satura- thermal efficiency of the plant and reduces the
moisture contents in low pressure stages of
tion temperature of water rises and the latent heat
expansion in the turbine.
of vaporisation decreases. The feed water can be
heated to saturation temperature in the economizer Modren high-
with the help of waste heat recovery from the pressure boilers use the heat transfer by radiation
exhaust gases escaping to the chimney. Then the along conduction and convection. The total heat-
boiler supplies only latent heat of vaporisation and receiving equipment is divided into several parts,
superheat. Thus, a boiler operating at high pressure so they can easily be located in various zones of
will require less heat addition for steam generation. the furnace for most efficient heat transfer to the
water circuit.

5. Improved Method of heating The high pressure


High-pressure boilers have the following unique
boilers use the following methods of heating for
features:
improved heat transfer rate:
l. Forced circulation of water
2. Large number of small-diameter tubes (i) Evaporation of water above critical pressure
3. Higher steam pressure and temperature of steam
4. Improved mode of heat transfer (ii) Heating of water by mixing superheated
steam for high heat transfer rate
5. Improved method of heating
(iii) Increasing the combustion air velocity over
6. Pressurised combustion
the tube
7. Compactness
8. High efficiency For increasing the
9. Intensive heating combustion rate and thus heat-release rate,
10. Once through construction pressurised air is used in the furnace. It gives large
amount of heat in a small space.
In all modren high
pressure boilers, the forced circulation of water is The high rate of heat transfer
maintained with the help of pumps. It increases the inside the boiler reduces the overall size of the
mean temperature of heat addition and evaporation boiler and the boiler becomes compact.
capacity of the boiler.
High-pressure boilers have
The better firing methods, monitoring, furnace condi-
surface-area-to-volume-ratio (area density) increases tions, control of flue gases and water velocity.
588 Thermal Engineering

Thus, the efficiency of such boilers ranges from 85


to 90%.

The furnace temperature in


high-pressure boilers is high enough, and therefore,
70% of heat is transferred to water by radiation,
which is a faster way of heat transfer.

10. Once Through Construction In high-pressure


boiler operating at and above the critical pressure,
the water directly flashes into steam in the tube
itself. It eliminates the need of a boiler drum.

1. High-pressure boilers use the forced circula-


tion of water which ensures the positive cir-
culation of water and increased evaporative
capacity.
2. They require less heat for vaporisation.
3. They are compact, and thus require less floor
water from the drum and delivers to the tubes of
space.
the evaporating section, where water is heated in a
4. Due to high velocity of water, the tendency
large number of small-diameter tubes and a mixture
of scale formation is minimised.
of steam and water is formed. This mixture is stored
5. All the parts are uniformly heated, and the in the drum. The convective superheater draws the
danger of overheating is minimised. wet steam from the drum and heats the steam for
6. The steam can be raised quickly to meet the its superheating. The superheated steam is supplied
variable load requirements without use of to a prime mover. The La Mont boiler generates
complicated control devices. approximately 50 tonnes of steam per hour at a
7. The plant efficiency is increased. pressure of 130 bar and a temperature of 500°C.
8. With the use of high pressure, the steam
Advantages of a La Mont Boiler
generation is economical.
1. With the use of small diameter tubes, the
high heat-transfer rate is maintained.
The La Mont boiler is a high-pressure, water- 2. The multiple-tube circuit gives flexibility for
tube type boiler. It works on a forced-circulation suitable location of heat transfer equipments.
principle. The water circulation is maintained by a 3. With forced circulation of water through the
centrifugal pump. Figure 17.13 shows the schematic tubes, a high evaporation rate is achieved.
of a La Mont boiler. The feed water is circulated
through the water walls and drums continuously
and prevents the tubes from being overheated. This boiler is a water-tube boiler and also uses forced
The feed water first passes through the circulation of water. It uses superheated steam for
economiser. Most of the sensible heat is supplied to vaporisation of feed water in the evaporator. The
the feed water in the economiser. Then water enters hot flue gases from the furnace are mainly used for
the boiler drum. A water circulation pump draws superheating of steam.
Steam Generators 589

Figure. 17.14 shows the schematic of a Loeffler Thus the operation is salient and clean.
boiler. The high-pressure feed pump supplies the 3. It is able to respond rapidly to sudden varia-
water to the economiser, where water is heated and tions of load.
then delivered to the evaporator. In the evaporator, 4. With the use of a radiant superheater, it
the feed water is further heated with the help of makes better heat recovery from the boiler
superheated steam. The steam produced is then furnace.
drawn from the evaporator drum by a steam- 5. The high circulation rate of steam over the
circulating pump and is then forced through tubes tubes causes low temperature difference
in the combustion chamber, where the radiant between the tube and steam.
superheater is located before entering into the
convective superheater. Thus, the steam becomes
superheated. Approximately, two-thirds of the
It is a subcritical boiler using supercharged furnace
superheated steam is sent to the evaporator and the
at 2–3 atmospheric pressure. The flue gas from the
remaining is available for use. The flue gases from
boiler drives the gas turbine, which in turn drives
the combustion chamber move to the convection
a rotary compressor to supply high-pressure air to
superheater and to the economiser before
the furnace.
proceeding to the atmosphere through the chimney.
The Loeffler boiler generates steam approximately The Velox boiler is operated on the principle that
at 100 tonne per hour at 140 bar. when the gas velocity exceeds the sonic velocity
(velocity of sound), the heat transfer rate from the
Advantages of a gas becomes much higher than that achieved with
1. This boiler uses forced circulation of water, subsonic flow.
and is thus capable to carry high salt concen- The schematic arrangement of Velox boiler is
trations than any other type. shown in Fig 17.15. The air is compressed in an
2. It uses superheated steam in the evaporator axial flow compressor to a pressure of 2.5 bar. In
for generation of steam from heated water. presence of this pressurised air in the furnace, there
is a high combustion rate, and thus a high rate of
heat release. The generated flue gases pass through
a nozzle section, where the velocity of flue gases is
increased to sonic velocity.
The flue gases enter the evaporator section with
sonic velocity and heat the water and steam. The
flue gases coming out of the evaporator are further
passed over a convective superheater, where a
portion of their heat content is used to superheat the
steam. The gases coming out of the superheater are
used to drive a gas turbine. The gas turbine drives
the air compressor. The flue gases coming out of
the gas turbine pass through the economiser for
heating the feed water.
The feed water after receiving heat in the
economiser is pumped into the evaporator section
tubes. The mixture of steam and water thus formed
enters the separator with a spiral flow. The circular
590 Thermal Engineering

motion thus separates the heavier water particles The feed pump increases the water pressure to
by throwing them outward on the walls. This effect supercritical pressure and forces the water through
separates the steam from water. The separated tubes. It first passes through the economiser, where
steam then enters the convection superheater before it is heated. Then it passes through the radiant
going to use. The removed water from steam in the water heater, where the water is further heated
separator is again passed to the evaporator section and its temperature increases to almost critical
tubes with the help of a pump. temperature. It then enetrs the transit heater, gets
converted into steam and then passes through
the convective superheater and finally becomes
The Benson boiler is a high-pressure (supercritical), available for applications.
drumless, once-through, water-tube boiler. The The thermal efficiency of a Benson boiler
boiler uses forced-circulation heat-transfer mechanism reaches up to 90% and it generates approximately
and uses oil as fuel. It operates at a pressure of 135 tonnes of steam per hour. It can be started
250 bar, which is more than the critical pressure within 15 minutes to produce the required flow rate
of water, and thus the latent heat of vaporisation of steam.
becomes zero. This boiler as shown in Fig. 17.16 Advantages
does not have any drum. The feed water enters one
1. It requires less floor space.
end of the tube and comes out as superheated steam
2. Its weight is 20% less than the other boilers.
from the other end. Thus, it is also called a once-
through boiler. 3. It can be started or stopped very quickly.
Steam Generators 591

A large number of industrial boilers are designed


to operate between working pressure ranges from
125 bar to 300 bar. When a boiler operates at a
pressure below the critical pressure of water, i.e.,
221 bar, then it is called a sub-critical boiler and it
4. Parts can be transported very easily and can consists of an economiser, evaporator and superheater.
be assembled at the site because there is no When a boiler operates at a pressure greater than
drum. 221 bar, it is termed as a supercritical boiler and
5. It can be operated economically. it consists of an economiser and superheater only.
6. Explosion hazards are less severe because It does not have an evaporator, because at critical
it consists of only small-diameter tubes and pressure or above it, the enthalpy of evaporation
has very less storage capacity. becomes zero. Its special features are the following:
1. It has rapid heat-transfer rate.
2. The temperature of water can be raised to
It is a supercritical boiler. It is very similar to the the critical temperature in the economiser,
Benson boiler, except that its evaporator section and thus the boiler has a very high thermal
consists of spiral tubes instead of straight tubes. Its efficiency.
water circuit is shown in Fig. 17.17. 3. The pressure level is more stable and
Water is forced into the economiser by a feed therefore, the boiler gives better response.
pump, where it is heated and then it enters spiral 4. The boiler is more suitable with load
evaporating tubes, where water flashes into steam. fluctuations.
This steam then passes the superheater before going 5. Due to absence of a two-phase mixture,
to some application. the problem of corrosion and erosion is
minimised.
592 Thermal Engineering

steam boiler or steam generator is a closed high-pressure boilers are widely used for power
vessel in which water is heated, vaporised and generation in thermal power plants.
converted into steam at a pressure higher than the The packaged boiler comes as a complete
atmospheric pressure. package. It requires only the steam, water-pipe
water-tube boiler, the water flows through a work, fuel supply and electrical connections to be
number of small tubes, which are surrounded by made for it to become operational.
hot combustion gases, while in a fire-tube boiler, fluidised-bed combustion boiler, the
the hot combustion gases pass through the boiler slightly compressed air is passed upward through
tubes, and water surrounds them. a finely divided bed of solid particles, while the
A boiler is called a high-pressure boiler, when it fuel in powder form is fed downward.
operates with a steam pressure above 80 bar. The

Boiler The steam boiler is a closed vessel in which Packaged boiler A fire-tube boiler, which comes as a
water is heated, vaporised and converted into steam complete package. It requires only electricity, fuel, water
IBR Indian Boiler Regulation Act and steam connection at sight.
Steam system It collects, controls and distributes the Pulverised-fuel boiler A boiler which uses pulverised
steam produced in the boiler coal as fuel in the furnace
Fuel system It includes all equipment used to provide Fluidised-bed combustion boiler In this boiler, fuel in
fuel to generate the necessary heat powder form is fed in the presence of slightly compressed
Fire-tube boiler A boiler in which flue gases pass air. The air velocity supports fuel in air and creates
through large-diameter tubes and water surrounds them bubble formation, vigorous turbulence, rapid mixing and
combustion of fuel.
Water-tube boiler A boiler in which water flows
through a number of small-diameter tubes, which are Supercharged boiler In this boiler, the combustion is
surrounded by hot flue gases. carried under pressure on the combustion chamber. More
mass of air is supplied by increasing the density of air in
High-pressure boiler A boiler which produces steam
an air compressor.
at more than 80 bar pressure.

Review Questions
1. What is a boiler? 7. What are the factors affecting the selection of a
2. What is the definition of ‘boiler’ according to boiler?
IBR? 8. What are the characteristics of a good boiler?
3. What is the difference between a steam boiler and 9. Explain the construction and working of a simple
a steam generator? vertical boiler with the help of a neat sketch.
4. What are the requirements of boilers? 10. Explain the working of a Cochran boiler with the
5. Classify the different types of boilers. help of a neat sketch.
6. Differentiate a water-tube boiler from a fire-tube 11. Explain the working of a Lancashire boiler with
boiler. the help of a neat sketch.
Steam Generators 593

12. Explain the working of a locomotive boiler with 15. Explain the working principle of a Velox boiler
the help of a neat sketch. with the help of a neat sketch.
13. Explain the working of Babcock and Wilcox 16. Explain the construction and working of a Loffler
boiler with the help of a neat sketch. boiler with the help of a neat sketch.
14. Explain the construction and working of a La 17. What are the advantages of operating a boiler at
Mont boiler with the help of a neat sketch. or above the critical pressure of water?
18. Explain the working of a supercharged boiler.

Objective Questions
1. A closed vessel is termed as a boiler if (c) an internally fired boiler
(a) its pressure exceeds 10 bar (d) a water-tube boiler
(b) its volume exceeds 22.75 litres 7. Which one of the following is a water-tube boiler?
(b) it consists of mountings (a) Babcock and Wilcox boiler
(d) it consists of accessories (b) Stirling boiler
2. Feed-water system of a boiler consists of (c) La Mont boiler
(a) supply of fuel (d) All of the above
(b) supply of water and steam generation 8. Which one of the following is a high-pressure
(b) supply of water to condenser boiler?
(d) supply of steam to utilities (a) Babcock and Wilcox boiler
3. In a fire-tube boiler (b) Stirling boiler
(a) water flows through the tubes (c) La Mont boiler
(b) flue gas flows through the tubes (d) All of the above
(b) fire is produced in the tubes 9. Which one of the following is a once-through
(d) flue gas surrounds the tube boiler?
4. In a water-tube boiler (a) Loeffler boiler
(a) water flows through the tubes (b) Benson
(b) flue gas flows through the tubes (c) La Mont boiler
(b) fire is produced in the tubes (d) All of the above
(d) flue gas surrounds the tube 10. Which one of the following is a bent-tube boiler?
5. A Cornish boiler is (a) Babcock and Wilcox boiler
(a) multi-tubular boiler (b) Stirling boiler
(b) a water-tube boiler (c) La Mont boiler
(c) a fire tube boiler (d) All of the above
(d) flue gas surrounds the tube 11. The rate of steam generation in water-tube boilers
as compared to fire-tube boilers is
6. Locomotive boiler is
(a) less (b) same
(a) non-portable boiler
(c) more (d) none of the above
(b) an externally fired boiler
11. (a) 10. (b) 9. (b)
8. (c) 7. (d) 6. (c) 5. (c) 4. (a) 3. (b) 2. (b) 1. (b)
Answers
594 Thermal Engineering

18
Boiler Mountings and
Accessories

Introduction
In accordance with the Indian Boiler Regulations (IBR), mountings are essential fittings for safe working
and control of a boiler.
The boiler accessories are those devices which are fitted either inside or outside the boiler to improve the
performance of a boiler or help in proper working of a boiler plant.

BOILER MOUNTINGS 9. Man hole 1 number


10. Mud box 1 number
The boiler mountings are the different fittings
and devices which are mounted on a boiler shell Safety Valve
for proper functioning and safety. These form an
Safety valves are located on the top of the boiler.
integral part of the boiler. These are in two groups:
They guard the boiler against the excessive high
(a) Mountings for Safety pressure of steam inside the drum. If the pressure
1. Safety valve 2 numbers of steam in the boiler drum exceeds the working
2. High pressure and low water safety valve pressure then the safety valve allows to blow-off
on Lancashire and Cornish boiler 1 number a certain quantity of steam to the atmosphere, and
each thus the pressure of steam falls in the drum. The
3. Water-level indicator 2 numbers escape of steam makes an audible noise as alarm to
warn the boiler attendant.
4. Fusible plug 1 number
There are four types of safety valves.
(b) Mountings for Control
1. Dead-weight safety valve
5. Pressure gauge 1 number 2. Spring-loaded safety valve
6. Steam stop valve 1 number 3. Level-loaded safety valve
7. Feed check valve 1 number 4. High steam and low water safety valve
8. Blow off cock 1 number
Boiler Mountings and Accessories 595

(i) Dead-Weight Figure 18.1 shows Figure 18.2 shows the schematic of a spring-
the schematic of a dead-weight safety valve. It is loaded safety valve. The device has two steam
very similar to the dead weight (whistle) loaded on passages in the form of ∪ and two valves. These
a pressure cooker and functions in a similar way. valves close the steam passages under the action of
A gunmetal valve rests on a gunmetal seat. The a central helical spring. The operating pressure of
gunmetal seat is mounted on a steel steam pipe. the valves is adjusted by varying the tension in the
The valve is fastened to a weight carrier. The dead spring. The extended lever is provided to check the
weights in the form of cylindrical discs are placed function of the valve from time to time.
on the carrier. Therefore, the total weight placed on
Lever
the carrier acts downward. This is the weight of the
cast-iron carrier and the valve itself. This weight W
Valve
is calculated on the basis of the working pressure p
and cross-sectional area a of the valve.
Steam Spring
Passage

p (Steam pressure)

Fig. 18.2 Spring-loaded safety valve

When the upward force of steam exceeds the


downward spring tension, the valves open and
some steam escapes to the atmosphere. Thus lowers
Fig. 18.1 Dead-weight safety valve
the steam pressure in the boiler and the valves are
When the force due to steam pressure (p ¥ a) again closed under the spring force.
is less than the total dead weight on the valve, (iii) Lever-Loaded The schematic of
the valve remains closed. When the force due to a lever-loaded safety valve is shown in Fig. 18.3.
steam pressure exceeds the total dead weight acting The body of the valve is fastened on the top of
downward, the valve lifts up from the seat and some the boiler shell. A gunmetal valve is placed on the
quantity of steam escapes to the atmosphere, thus steam passage formed in the casing. A cast-iron
reducing the steam pressure in the boiler shell, and lever attached to a fulcrum on one end and loaded
the valve is again closed. The dead-weight safety
valves are only used on stationary boilers.
(ii) Spring-Loaded The dead-weight
safety valve cannot be used on locomotive and
marine boilers. The jerks, pitching and rolling may
change the load on the valve and it can open the
valve frequently under working pressure.
The spring-loaded safety valve is used on
locomotive marines and on high-pressure boilers. Fig. 18.3 Lever-loaded safety valve
596 Thermal Engineering

by weight on the other end keeps the valve on the It consists of a lever A hung inside the boiler
seat in a closed position. shell and hinged at the fulcrum C. One side of the
When the upward force due to steam pressure lever A is loaded by a balance weight and the other
exceeds the load on the valve, the valve opens, side carries a float immersed in water. The high-
and allows some quantity of steam to escape. The steam valve V1 is loaded by a lever safety valve. In
pressure of steam in the boiler falls and the valve the centre of the valve V1, a hemispherical valve V2,
again rests on the seat. is located. This hemispherical valve is connected
As the weight is acting through a long lever, a with a valve rod and projects with a knife edge
small weight can give a large thrust on the valve on the lever A. The hemispherical valve is loaded
according to the equation directly by central dead weights.
When the water level falls below the normal
W (L1 + L2) = p a L1.
level in the boiler shell, the float end of the lever A
where a is the cross-sectional area of steam passage. lowers down and the valve rod is pushed up to open
the hemispherical valve V2. The steam leaks with a
(iv) High-Steam and Low Water This
louder noise and sounds an alarm to the attendant.
valve is a combination of two valves as shown
in Fig. 18.4. It is used in Cornish and Lancashire When steam pressure exceeds the maximum
boilers. One of the valves is lever loaded and is working pressure, the steam valve V1 lifts up with
operated when steam pressure in the boiler exceeds the valve V2 and steam leaks out, thereby decreasing
the working pressure. The second valve operates the steam pressure.
and blows off steam with a louder noise, when
Water-Level Indicator
water level in the boiler falls below the normal
level. The water level indicator is located in front of the
boiler in such a position that the level of water can
Fulcrum
Strut easily be seen by the attendant. Two water-level
indicators are used on all boilers.
A water-level indicator consists of a metal tube
Adjustable and a strong glass tube with markings. The upper
weight Lever B and lower ends of these tubes are connected to two
Weight
Steam gunmetal hollow pipes. The upper pipe has a steam
cock and the lower pipe has a water cock which are
High steam bolted to the boiler plate by two flanges. The upper
Hemispherical valve, V1 pipe is opened to steam and the lower pipe is opened
valve, V2
(Low water valve) to water with the help of steam and water cocks,
Rod
respectively. The drain cock is used frequently to
ensure that the water and steam cocks are clear.
Knife edge
During the boiler operation, the steam cock and
Fulcrum C
water cock remain opened while the drain cock is
Dead Balance kept closed. During the normal operation, the two
Lever A weight weight balls provided inside the gunmetal pipe remain in
position as shown in Fig. 18.5. Hence, the water can
Float
reach the glass gauge and its level can be seen.
In case the glass gauge breaks accidently, the
Fig. 18.4 High-steam and low water safety valve water and steam simultaneously rush out through
Boiler Mountings and Accessories 597

Boiler front
plate
Cast iron Normal position
pipe of Ball
Bourdon tube
Pointer
Plug
+
Gear sector

Steam cock Glass-tube gauge

Metal Water level in


tube the boiler

Plug Fluid pressure


(a) Bourdon pressure gauge
Ball
Cast iron pipe
Pressure gauge
Water cock
Drain cock Boiler cover

Fig. 18.5 Water-level indicator


3 way Steam space
the gunmetal pipes. The force is exerted on two balls valve
connects to
and they are carried away by water and steam and standard
guage
the passages are closed. The water and steam cocks
Syphon tube
are then closed and the glass gauge is replaced.

Pressure Gauge (b) Pressure gauge with syphon tube


A pressure gauge is fitted in front of the boiler in Fig. 18.6
such a position that the operator can conveniently the effect of high temperature steam on the gauge
read it. It reads the pressure of steam in the boiler components. The steam pressure is transferred by
and is connected to the steam space by a siphon water to the Bourdon pressure gauge.
tube.
The most commonly used gauge is the Bourdon Fusible Plug
pressure gauge. Figure 18.6(a) illustrates the
Bourdon pressure gauge. It consists of an elliptical It is a very important safety device which protects
spring Bourdon tube. One end of this tube is the fire-tube boiler shell against overheating. It
connected to the siphon tube and other end is is located just above the furnace in the boiler. It
connected by levers and gears to pointer. consists of a gunmetal plug fixed in a gunmetal
body with a fusible molten metal as shown in
When fluid pressure acts on the Bourdon tube,
Fig. 18.7.
it tries to make its cross-section change from
elliptical to circular. In this process, the lever end Hollow plug
of the tube moves out as indicated by an arrow. The
tube movement is magnified by the mechanism
and given to pointer to move over a circular scale
indicating the pressure.
The siphon tube, is shown in Fig. 18.6(b),
it connects the steam space of the boiler to the Crown plate Fusible metal
Bourdon gauge is filled with water in order to avoid Fig. 18.7 Fusible plug
598 Thermal Engineering

During the normal boiler operation, the fusible over the spindle prevents the handle being removed
plug is covered by water and its temperature does unless the cock is closed.
not rise to its melting state. But when the water
level falls too low in the boiler, it uncovers the
fusible plug. The furnace gases heat up the plug, the The feed check valve is fitted to the boiler, slightly
fusible metal of the plug melts, and the inner plug below the working level in the boiler. It is used to
falls down. The water and steam then rush through supply high-pressure feed water to the boiler. It also
the hole and extinguish the fire before any major prevents the returning of feed water from the boiler
damage occurs to the boiler due to overheating. if the feed pump fails to work.
A feed check valve consists of two valves:
feed valve and check valve as shown in Fig. 18.9.
The function of the blow-off cock is to discharge The feed valve is operated by a hand wheel for
mud and other sediments deposited in the bottom- its opening or closing. The check valve operates
most part of the water space in the boiler, while the automatically on its seat, up and down under the
boiler is in operation. It can also be used to drain off pressure difference of water.
the boiler water. Hence it is mounted at the lowest
Hand wheel
part of the boiler. When it is open, water under the
pressure rushes out, thus carrying sediments and
mud. To Boiler

The blow-off cock as shown in Fig. 18.8 consists


of a conical hollow gunmetal plug type of valve, Check valve
which fits accurately into the corresponding hole
in the casing. The plug valve has a hole, which
Water in
when brought in line with the hole in the casing,
by rotating the plug, causes the water to flow out Fig. 18.9 Feed check valve
of the boiler.
The flow of water through the cock can be In normal working, the feed-water pressure is
stopped by rotating the plug valve in such a way more than the boiler pressure, thus the check valve
that its solid side comes in the line of the hole in remains open. But in case of its failure, the boiler
the casing. The stuffing box prevents the leakage pressure becomes more than the feed water, the
of water at the valve shank. The guard provided valve rests on its seat and closes the water passage
Guard and prevents its reverse flow.

Gland
Steam Stop Valve
Valve
shank Valve The steam stop valve is located on the highest part
Casing
of the steam space. It regulates the steam supply for
use. The steam stop valve can be operated manually
Inlet Out let or automatically.
A hand-operated steam stop valve is shown in
Fig. 18.10. It consists of a cast-iron body and two
flanges at right angles. One flange is fastened to the
Set screw
boiler shell and the other is fastened to the steam
Fig. 18.8 pipe. A steel valve connects the hand wheel through
Boiler Mountings and Accessories 599

Hand wheel
(ii) Economiser
(iii) Air preheater
(iv) Feed-water pump
Valve body
(v) Steam injector
Spindle (vi) Steam separator
Valve (vii) Steam trap
Steam out (viii) Boiler draught equipments
Valve
seat
Flanges
It is a heat exchanger in which products of heat of
Steam in combustion are utilized to dry the wet steam and to
Fig. 18.10 Steam stop valve
make it superheated by increasing its temperature.
During superheating of the steam, pressure remains
the spindle. When the hand wheel is rotated, constant, and its volume and temperature increase.
the spindle also rotates and carries the valve up A super-heater consists of a set of small-diameter
or down to open or close the valve. The spindle U tubes in which steam flows and takes up the heat
passes through a stuffing box and glands in order to from hot flue gases.
prevent the leakage. The smaller diameter tubes have lower pressure
When the spindle is rotated anticlockwise, the stresses and withstand better. The tube material
valve lifts up and steam is allowed to pass through should be carefully selected, because the tubes
the clearance between the valve and its seat. The are subjected to high temperature, pressure and
amount of steam passing the valve is controlled thermal stresses. The maximum steam temperature
by the valve lift. When the hand wheel is rotated at the superheater exit is about 540°C. The
clockwise, the valve rests on its seat and closes the superheaters and re-heaters, which are operating at
steam passage. this temperature, are made of special high-strength
alloy steels, which have high strength and corrosion
18.1.8 Man Hole and Mud Box resistance.
The man hole is provided on the boiler shell at Superheaters are classified as convective, radiant
a convenient position so that a person can enter and of combination types.
through it, inside the boiler for cleaning and In the convective superheater, the heat of the
inspection purposes. hot flue gases is transferred to the surface of the
The mud box is placed at the bottom of the boiler superheater by convection. These are located in the
to collect mud discharged through the blow-off path of hot flue gases.
cock. Therefore, it is connected with the blow-off In a radiant superheater, the heat of the
cock. combustion is transferred to the surface of the
superheater by thermal radiation. These are located
in one or more walls of the furnace. These are used
in high-pressure boilers.
The accessories are mounted on the boiler to In a combination type of superheater, the heat
increase its efficiency. These units are optional on is transferred to the surface of the tubes by both
an efficient boiler. The following accessories are modes of heat transfer. The radiant superheaters are
normally used on a modern boiler: occasionally used in combination with convective
(i) Superheater superheaters and are arranged in series.
600 Thermal Engineering

When the boiler load increases, the rate of energy and then it is taken for use through the valve B via
absorption by the furnace water walls (radiant the uptake header. The superheating of steam is
superheater) does not increase as rapidly as the controlled by controlling the quantity of flue gases
steam flow rate. Thus, a radiant superheater shows by operating the dampers manually.
a decrease in steam temperature with an increase If superheated steam is not needed or the
in boiler load and steam flow as shown in Fig. superheater is under maintenance, the valves B and
18.11(a) and (b). A convective superheater shows C are closed and steam is then taken out through
an opposite variation in the steam temperature the valve A.
with change in the boiler load. These opposite
characteristics can be utilised to control the final Economiser
temperature of steam from the combination type of
An economiser is a heat exchanger used for
superheaters.
heating the feed water before it enters the boiler.
The economiser recovers some of waste heat of
Combined
Combined hot flue gases going to the chimney thus it helps in
Steam temperature

Steam temperature

e
c tiv improving the boiler efficiency. It is placed in the
ve
Ra on
C path of flue gases at the rear end of the boiler just
e
tiv

di
at
ec

ive before the air preheater.


nv

Rad
Co

iativ
e The most popular economiser is Green’s
economiser and it is shown in Fig. 18.13.
Per cent load Per cent steam flow Green’s economiser consists of a set of vertical
(a) (b)
cast-iron pipes joined with horizontal lower and
Fig. 18.11 Superheater characteristics upper headers. The cold feed water flows through
the vertical pipes via the lower header. The hot
Figure 18.12 shows a schematic of Surgden’s flue gases pass over them transferring heat to the
superheater. It consists of two mild steel headers. water. The heated water is supplied to the boiler via
The ∪-shaped steel tubes are connected to these the upper header. The scrappers are provided on
headers. pipes, which move up and down slowly by means
The steam generated into the boiler passes of chains and sprockets to avoid the soot depositon
the valve C and enters the superheater (∪) tubes on the pipe surface. The soot collected in the soot
through the intake header. The steam is made dry chamber can be removed from the door.
and superheated in these tubes by supplying heat Each economiser is equipped with a safety
valve, a drain valve, a release valve, pressure gauge
A Main steam B
pipe and thermometers.
Stop valve

Intake
Boiler C
Uptake header The function of an air preheater is similar to that
header
Water level of an economiser. It recovers some portion of the
Steel tubes waste heat of hot flue gases going to the chimney,
(U-shaped) and transfers the same to the fresh air before it enters
the combustion chamber. A tubular air preheater is
shown in Fig. 18.14.
Dampers
Due to preheating of air, the furnace temperature
increases. It results in rapid combustion of fuel
Fig. 18.12 Superheater
with less soot, smoke and ash. The high furnace
Boiler Mountings and Accessories 601

Sprocket

Chain
Stop valve
Safety
valve
Feed water Upper header
outlet

Flue gases Vertical water


Scraper

Stop valve
Lower Header Feed water inlet

Soot chamber Door

Fig. 18.13 Green’s economiser


Hot gases out expanding steam is given to feed water. Thus the
pressure and velocity of water increase.
Cold air in
Figure 18.15 shows an injector. It consists of a
set of nozzles—steam nozzle, suction jet, delivery
nozzle, steam and water inlet passages, delivery
passage, spindle handle and a overflow valve.
Baffles

Hot air out

Hot
gases in
Hopper

Soot gate

Fig. 18.14 Tubular air preheater

temperature can permit a low-grade fuel with less


atmospheric pollution. The air preheater is placed
between the economiser and chimney.

Steam Injector
A steam injector lifts and forces the feed water
into the boiler. It is usually used for vertical and
locomotive boilers and can be accomodated in a
small space. It is less costly and does not have any
moving parts. Thus, operation is silent.
In a steam injector, the water is delivered to the
boiler by steam pressure. The kinetic energy of the Fig. 18.15 Steam injector
602 Thermal Engineering

When the steam valve is opened with the help motor or by a prime mover. It utilizes a centifugal
of a spindle and handle, the steam from the boiler force of the impeller for pumping water.
enters the steam nozzles. The steam pressure drops
in the nozzles and steam velocity increases. The (b) These are positive-
high-velocity steam creates a partial vacuum in displacement type pumps. The most popular type
the steam jet. Thus, the water from the feed tank of reciprocating feed pump used in a boiler is a
is sucked and mixed with steam and the steam duplex feed pump. It consists of a steam engine and
condenses. This mixture then enters the delivery a water pump side by side.
tube, where the kinetic energy of the mixture is The reciprocasting water pump is driven by the
converted to pressure energy. reciprocating steam engine as shown in Fig. 18.16.
When the pressure of water delivered by the The piston rods of the water pump and steam engine
nozzle is less than the boiler pressure, the water are coupled together. So the steam engine and water
leaving the injector is passed through the overflow pump operate simultaneously. The steam generated
valve to the overflow pipe. But when sufficient in the boiler is used to run the steam engine. The
pressure in the delivery tube is raised, the water is steam engine drives the water pump, Double acting
supplied to the boiler. pumps are commonly used for medium-size boilers.

Feed Pump Steam Traps

The feed pump delivers feed water at a pressure A steam trap is a valve device that drains away
higher than that in the boiler. The commonly used the condensed steam and air automatically from
feed pumps are the following: the steam pipe, steam jackets or steam separator
without discharging the steam. The purpose of
(a) It is a high-speed centrifugal installing the steam traps in the process equipment is
pump and is used to deliver a large quantity of to obtain fast heating of the product and equipment
water into the boiler. It consists of an impeller and by keeping the steam lines and equipment free of
a casing. The impeller shaft is driven by an electric condensate, air and non-condensable gases.

Fig. 18.16 Feed pump


Boiler Mountings and Accessories 603

Functions of The three important Steam Separator


functions of steam traps are A steam separator is installed on the steam main
as well as on the branch lines to separate any water
is formed particles present from the steam and to improve the
quality of the steam going to the units. It is installed
very close to units on main steam pipes. By change
incondensible gases of direction of steam, steam seperators cause the
condensed water particles to be separated out and
Types of Steam Traps
delivered to a point where they can be drained away
There are three basic types of steam traps discussed as condensate through a conventional steam trap. A
below: few types of separators are illustrated in Fig. 18.18.
1. Thermostatic Steam Traps operated by changes in
fluid temperature.
In the steam space, condensed steam and dry
steam are present. But as a result of any heat
loss, the temperature of condensed steam falls. A
thermostatic trap will pass the condensate when this
lower temperature is sensed. As steam reaches the
trap, the temperature increases and the trap closes.
2. Mechanical Steam Traps operated by changes in
fluid density.
This range of steam traps operates by sensing
the difference in density between the steam and
condensate. These steam traps include ‘ball float
traps’ and ‘inverted bucket traps’. In the ‘ball
float trap’, the float ball rises in the presence of
condensed steam, opens a valve which passes the
denser condensate out. A ball float steam trap is
shown in Fig. 18.17.
3. Thermodynamic Steam Traps operated by changes
in fluid dynamics.
Thermodynamic steam traps rely partly on the
formation of flash steam from the condensate.
Air cock

Fig. 18.17 steam trap Fig. 18.18 Steam separators


604 Thermal Engineering

Summary
(a) Boiler Mountings (b) Boiler Accessories

Name Function Name of the accessory Function

1. Safety valves Do not allow boiler pressure 1. Economiser Preheating the feed water by
to rise beyond its safe pressure utilising the heat of waste flue
gases
2. Water-level Shows working level of water
indicator in the boiler 2. Superheater Increase the temperature of
steam at constant pressure
3. Pressure gauge Indicates working pressure of
beyond saturation
boiler
3. Anti-priming Filter out moisture from
4. Steam stop valve Regulates the amount of
pipe (anti- outgoing steam
outgoing steam
priming devices)
5. Feed check valve Checks the amount of feed
water going to the boiler and
does not allow its return 4. Air preheater Preheat the fresh air by using
Allows to drain water from the heat of waste flue gases
6. Blow-off cock
boiler 5. Steam injector Lift and force the feed water
Allows a person to go inside into the boiler
7. Man hole
the boiler drum for repairs,
etc.
8. Mud hole (with Facilates removal of heavy
door) impurities settled in the boiler
drum
9. Fusible plug Stops the boiler if its heating
surface is overheated due to
low water level
10. Dual function
safety valves
(a) High pressure, Allows escape of steam in
low water safety case of (i) unsafe higher
valve pressure, or (ii) unsafe low
water level
(b) Low water, high Whistles by blowing
water safety steam in case of (i) unsafe low
valve or (ii) high water level in the
drum
11. Feed pipe Lead the feed water to the
inside of the boiler
Boiler Mountings and Accessories 605

Review Questions
1. List the boiler mountings and acccessories. (c) Blow-off cock
2. Discuss various types of safety valves. (d) Feed check value
3. Explain the high steam and low water safety
7. Explain the working of a steam stop valve.
valve.
8. Why are superheaters used on boilers? Explain
4. Explain the working of a water-level indicator
their working.
with a neat diagram.
9. Write the advantages of using feed water heaters
5. Explain the working of Bourdon’s pressure
on boilers.
gauge.
10. Explain the function of an air preheater in a
6. Discuss in brief with their function
boiler.
(a) Fusible plug 11. Write the function of a steam separator and a
(b) Man hole steam trap.

Objective Questions
1. Which one of the following is essential for 6. Which one of the following equipment prevents
operation of a boiler? the boiler against excessive pressure?
(a) Safety valve (b) Economiser (a) Pressure gauge
(c) Superheater (d) Injector (b) Steam stop valve
2. Which one of the following is not boiler (c) Fusible plug
mounting? (d) Safety valve
(a) Superheater (b) Feed check valve 7. Which one of the following safety valves is used
(c) Man hole (d) Fusible plug on stationary boilers?
3. Which one of the following is an accessory on a (a) Dead-weight safety valve
boiler? (b) Spring-loaded safety valve
(a) Pressure gauge (c) High-steam and low water valve
(b) Blow-off cock (d) Lever-loaded safety valve
(c) Economiser 8. Which one of the following safety valves is used
(d) Feed check valve on locomotive boilers?
4. Which one of the following must be used in pair? (a) Dead-weight safety valve
(a) Pressure gauge (b) Spring-loaded safety valve
(b) Water level indicator (c) High-steam and low water valve
(c) Fusible plug (d) Lever-loaded safety valve
(d) All of the above 9. The function of a fusible plug is to
5. Which boiler generally uses the high steam and (a) Superheat the steam
low water safety valve? (b) Extinguish the fire
(a) Cochran boiler (c) Maintain constant temperature
(b) Cornish boiler (d) Increase flue gas temperature
(c) Lancashire boiler 10. The function of superheater is to
(d) Stirling boiler (a) Superheat the steam
606 Thermal Engineering

(b) Extinguish the fire 13. Which one of the following is the correct
(c) Maintain constant temperature sequence of accessories in a boiler plant?
(d) Preheat the feed water (a) Boiler economiser superheater chimney
11. The function of an air preheater is to (b) Economiser boiler superheater chimney
(a) Increase the fuel consumption (c) Economiser air preheater superheater
(b) Heat the fresh air chimney
(c) Burn fuel efficiently (d) Economiser boiler preheater chimney
14. The function of a steam trap is to
12. The function of an ijector in a boiler plant is to (a) Lift the steam
(a) Lift the feed water (b) Separate the steam
(b) Lift and force the feed water (c) Discharge the condesate
(c) Inject the fuel (d) Heat the steam
(d) Heat the feed water

14. (c) 13. (b) 12. (b) 11. (b) 10. (a) 9. (b)
8. (b) 7. (a) 6. (d) 5. (c) 4. (b) 3. (c) 2. (a) 1. (a)
Answers
Boiler Draught and Performance 607

19
Boiler Draught and
Performance

Introduction
In a steam-generating unit, the hot flue gases are to be moved from the furnace to the places where the
heat can be transferred and these gases are discharged in the atmosphere at a certain height through the
chimney. For continuous flow of gases, a certain pressure difference is to be maintained inside the boiler.

Boiler Draught
BOILER DRAUGHT (DRAFT)
The detailed classification of boiler draught is
The boiler draught may be defined as the small given below:
pressure difference which causes the continuous
Draught
flow of gases inside the boiler. In other words, the
draught is a small pressure difference between the
air outside the boiler and gases within the furnace Natural or Artificial
chimney draught
or chimney. draught

Steam jet Mechanical


draught or fan
The boiler draught performs the following func- draught
tions:
1. It forces a sufficient quantity of air into the Induced Forced Induced Forced Balanced
furnace for proper combustion of fuel. draught draught draught draught draught

2. It circulates the hot flue gases through the


flue tubes, superheater, economiser, air
preheater etc. DRAUGHT
3. It discharges the hot flue gases to the
atmosphere through the chimney. The draught obtained by use of a chimney is called
natural or chimney draught. A chimney is a veritcal
608 Thermal Engineering

long cylindrical structure made of either brick Advantages


masonary, reinforced concrete or steel. A chimney 1. Easy to construct.
carries the products of combustion to such a height 2. No power is required for producing the
before discharging so that they will not be harmful draught.
to the surroundings. The draught produced by the 3. Long life of chimney.
chimney is due to the density difference between
4. No maintenance is required.
the column of hot flue gases inside the chimney
and the equivalent column of cold air outside the Disadvantages
chimney. 1. Tall chimney is required.
Figure 19.1 shows a schematic arrangement of a 2. Poor efficiency.
chimney of height H metres above the grate level. 3. Efficiency decreases with increase in outside
patm
temperature.
4. No flexibility to create more draughts to take
Equivalent column of

peak loads.
hot flue gases
Column of
cold air

H
Chimney
The amount of natural draught produced by a
Grate Boiler
level chimney mainly depends on the height of the
chimney, the temperature of hot flue gases and
atmospheric air. Refer to Fig. 19.2.
Fig. 19.1
Let ma = Mass of air supplied in kg/kg of fuel
The pressure at grate level in the combustion ma + 1 = Mass of flue gases, kg/kg of fuel
chamber pa = atmospheric pressue, N/m2
p1 = atmospheric pressure + pressure due to To = Absolute temperature at N.T.P = 273 K
column of hot flue gases in the chimney. Ta = Temperature of atmospheric air, K
= pa + rg g H Tg = Average temperature of flue gases, K
Similarily, pressure at grate level outside the H = Height of chimney, m
chimney h = Draught required in mm of water
p2 = atmospheric pressure + pressure due to
cold air column of height H The specific volume of air at NTP
= pa + ra gH RT0 (0.287 kJ/kg ◊ K) ¥ (273 K)
v0 = =
p0 (101.325 kPa)
where ra and rg represent the densities of cold air
and hot flue gases, respectively. = 0.7732 m3/kg
Since the density of hot flue gases is less than pa
H¢ of hot flue gases
Equivalent column

that of cold air, thus pressure difference will act


on the grate level, which will cause flow of fresh
Column H of
cold air

Main
air in the combustion chamber. The net pressure flue
Chimney
difference Economiser

Dp = p2 – p1 = ( ra – rg ) gH ...(19.1) Grate Boiler


level
The pressure difference causing the flow of gases
is known as static draught and is generally very Air pre-heater
small. Thus, it is measured by water manometers. Fig. 19.2
Boiler Draught and Performance 609

Since the volume of fuel is negligible as Equating (19.5) and (19.6), we get
compared to the volume of air supplied per kg of
È 1 Ê m + 1ˆ 1 ˘
fuel, therefore, the volume of flue gases can be h = 353H Í - Á a ˜ ¥ ˙ (mm of water)
taken equal to the volume of air. ÍÎ Ta Ë ma ¯ Tg ˙˚ ...(19.7)
The volume of atmospheric air outside the Equation (19.7) represents the theoretical value
chimney of natural draught which depends on chimney height
Va V m v and weather conditions. The draught is directly
= 0 = a 0
Ta T0 T0 proportional to chimney height and it decreases
ma ¥ 0.7732 with increase in temperature of atmospheric air or
or Va = ¥ Ta ...(19.2) decrease in temperature of flue gases. The actual
273
m draught available would be less than the theoretical
and density of air ra = a draught because of friction losses offered by the
Va
273 1 353 passages.
= ¥ =
0.7732 Ta Ta
...(19.3)
Similarly, the volume of hot flue gases inside the Let H ¢ be the height of hot gases, equivalent to the
chimney column of cold outside air H and produce the same
V0 0.7732 pressure difference, i.e.,
Vg = Tg = maTg
T0 273 Dp = rg gH ¢
and density of flue gas Ê m + 1ˆ 353
Using rg = Á ¥
m +1 Ë m ˜¯ Tg
rg = a
Vg Equating with Eq. (19.5),
( ma + 1) 273 ( ma + 1) 353 Ê ma + 1ˆ 353
= ¥ = ¥ ...(19.4)
0.7732 ma Tg ma Tg ÁË m ˜¯ ¥ T ¥ gH ¢ =
a g
The total static pressure difference Dp in N/m2 È 1 Ê m + 1ˆ 1 ˘
353 gH Í - Á a ˜ ¥ ˙
can be obtained by using ra and rg in Eq. (19.1); ÍÎ Ta Ë ma ¯ Tg ˙˚
È 1 Ê m + 1ˆ 1 ˘ ÈÊ ma ˆ Tg ˘
Dp = 353 gH Í - Á a ˜ ¥ ˙ (N/m2) or H¢ = H ÍÁ ¥ - 1˙
ÍÎ Ta Ë ma ¯ Tg ˙˚ ˜
ÍÎË ma + 1¯ Ta ˙˚
...(19.5)
(metres of hot gases) ...(19.8)
This pressure difference can be expressed in
terms of the water column (mm) as
Dp = (r gh)w
The theoretical velocity of hot flue gas flowing
Where rw = 1000 kg/m3 through chimney
h Vg = 2gH ¢ ...(19.9)
hw = h (mm of water) = ( metre)
1000 However, the actual velocity of the flue gases
Ê h ˆ will be less than the theoretical velocity given by
\ Dp = (1000 kg/m3 ) ¥ g Á m
Ë 1000 ˜¯ Eq. (19.9), due to frictional losses. If hf be the
= gh (N/m2) ...(19.6) frictional losses of height of column of flue gases
610 Thermal Engineering

then the velocity of flue gases in the chimney,


Ê ma ˆ 1 1
Vg = 2 g ( H ¢ - h f ) = 4.43 (H ¢ - h f ) ...(19.10) or mg = C Á ˜ ¥ - 2 ...(19.15)
Ë ma + 1¯ TaTg Tg
It can be arranged as Ap
where constant C = 2 gH
hf R
Vg = 4.43 H ¢ ¥ 1- = k H¢ ...(19.11)
H¢ For maximum discharge rate, differentiating
Eq. (19.15) with respect to Tg and equating it to
hf zero,
where k = 4.43 1 - ...(19.12)

dmg d ÏÔ Ê ma ˆ 1 1 ¸Ô
=C Ì Á ¥ - ˝ =0
dTg Ô Ë ma + 1˜¯ TaTg Tg 2 Ô
The value of k has been found experimentally
dTg
k = 0.825 for brick chimney, and Ó ˛
= 1.1 for steel chimney. 1

The mass of flue gases flowing through a d ÏÔÊ ma ˆ 1 1 ¸Ô 2


ÌÁ ˜ ¥ - 2˝ =0
chimney of cross-sectional area A dTg ÔË ma + 1¯ TaTg Tg Ô
Ó ˛
mg = rg AVg (kg/s)
- 12
Êp ˆ 1 ÏÔÊ m ˆ 1 1 ¸Ô
= r g Á D 2 ˜ Vg 0 = ÌÁ
a
˜ ¥ - 2˝
¥
Ë4 ¯ 2 ÔÓË ma + 1¯ TaTg Tg Ô˛
4 mg
or D2 = ÏÔÊ m ˆ 1 Ê 1 ˆ 2 ¸Ô
pr g Vg ÌÁ
a
¥ Á - ˜ + ˝
Ë m + 1˜¯ Ta Ë Tg2 ¯ Tg3 Ô
mg ÓÔ a ˛
or D = 1.128 ...(19.13)
r g Vg Ê ma ˆ 1 2
fi -Á ¥ + =0
Ë ma + 1˜¯ TaTg 2 Tg 3

Ê ma ˆ 1 2
or ÁË m + 1˜¯ ¥ T = T
a a g
The velocity of flue gases through the chimney
without any losses Tg Ê m + 1ˆ
or = 2Á a
Vg = 2 gH ¢ when hf = 0 Ta Ë ma ˜¯
...(19.16)
Using H¢ from Eq. (19.8), we get
ÏÔÊ ma ˆ Tg ¸Ô Thus, for maximum discharge through the chim-
Vg = 2 gH ÌÁ ˜ ¥ - 1˝ ...(19.14) ney, the absolute temperature of flue gases should
ÓÔË ma + 1¯ Ta Ô˛
be greater than twice the absolute atmospheric tem-
The density of flue gases is given by perature.
p T
rg = Using the value of g in Eq. (19.8), we get
RTg Ta
Therefore, the rate of mass of flue gases dis- ÈÊ ma ˆ Ê m + 1ˆ ˘
charged H ¢max = H ÍÁ ˜ ¥ 2 Á a ˜ - 1˙
ÍÎË ma + 1¯ Ë ma ¯ ˙˚
mg = AVg rg
=H ... (19.17)
È ˘
ÔÏÊ ma ˆ Tg Ô¸ ˙ p For maximum discharge, the height of the hot-
= A Í 2 gH ÌÁ ˜ ¥ - 1 ˝ ¥
Í
Î ÔÓË ma + 1¯ Ta Ô˛ ˙˚ RTg gas column should be equal to the height of the
Boiler Draught and Performance 611

chimney. Similarly, draught in mm of water to Ta = Atmospheric temperature in K


height of chimney, g = Acceleration due to gravity as 9.81 m/s2
È1 1 ˘ 176.5 H The efficiency of the chimney is directly propor-
h¢max = 353H Í - ˙ = ...(19.18) tional to the height of the chimney. The efficiency
Î Ta 2Ta ˚ Ta
of a chimney is less than 20%. It is obvious from
the above statement that the chimney is very insuf-
ficient means of producing the draught.
A certain minimum flue-gas temperature is required
to produce a given draught with a given height of
the chimney. The temperature of the flue gases
leaving the chimney in case of natural draught has The actual natural draught is always less than the
to be higher than that in an artificial draught. It leads calculated value due to the following reasons:
to higher thermal losses through the flue gases and 1. The frictional resistance between chimney
therefore, poor boiler efficiency in case of natural walls and flue gases (approx. 1mm of water).
draught. In artificial fan draught, the temperature of 2. Frictional losses during the flow through the
the flue gases discharging through chimney can be bends in gas flow passages (approx. 1.25 mm
lowered to any desired value and the recovered heat of water).
can be used in economiser and air preheater, etc., 3. Frictional resistance offered due to path of
without change in draught developed. flue gases in furnace with baffles, grates,
The efficiency of the chimney is defined as superheaters, economisers, and air preheater.
the ratio of the energy equivalent of draught in
metre head or in N-m per kg of gases, produced The total natural draught losses are nearly 20%
by artificial draught fan to the energy equivalent in of the total static draught produced by the chimney.
N-m per kg of gases of the additional heat carried Therefore, the available draught is approximately
away by the flue gases to create natural draught. 80% of the total static draught calculated theoreti-
cally.
Energy equivalent of artificial draught
hchimney =
Energy equivalent of hoot flue gases Example 19.1 A thermal power plant has a chimney
required to create natural draught draught of 3.5 cm of H2O column. The flue-gas tempera-
ture flowing through the chimney is 280°C and the ambi-
H ¢g ent temperature is 15°C. The amount of air supplied/kg
= ...(19.19)
C p (Tg - Tg¢ ) of the fuel is 20 kg. Calculate the height of the chimney.
where
Solution
H ¢ = Column of hot flue gases, equivalent to
artificial draught in metre head Given The chimney of a thermal power plant
h = 3.5 cm = 35 mm of water,
Ê ma Tg ˆ
= HÁ ¥ - 1˜ (metres) Tg = 280°C = 553 K
Ë m2 + 1 Ta ¯ Ta = 15°C = 288 K,
Cp = Specific heat of hot flue gases in J/kg ◊ K ma = 20 kg/kg of fuel
Tg = Temperature of hot flue gases discharged
To find The height of the chimney.
through chimney in case of natural
draught in K Analysis The height of the chimney in terms of the
¢
Tg = Temperature of hot flue gases discharged water column
through chimney in case of artificial È 1 Ê m + 1ˆ 1 ˘
h = 353H Í - Á a ˜ ¥ ˙ ( mm of water)
draught in K ÍÎ Ta Ë ma ¯ Tg ˚˙
612 Thermal Engineering

Example 19.3 A thermal power station works on


natural draught. The height of the chimney is restricted to
40 m. The ambient temperature of the air is 20°C and the
temperature of the flue-gas passing through the chimney
at its base is 300°C. The air–fuel ratio is 17:1. Calculate
the diameter of the chimney at the base, if head lost due
Fig. 19.3 to friction is 25% of the ideal draught.
È 1 Ê 20 + 1ˆ 1 ˘ Solution
35 = 353H Í -Á ˜ ¥ ˙
Î 288 Ë 20 ¯ 553 ˚
Given The chimney of a thermal power plant
35
H = = 63.0 m H = 40 m Ta = 20°C = 293 K
353 ¥ 1.573 ¥ 10 -3 Tg = 300°C = 573 K ma = 17 kg/kg of fuel
hf = 0.25 H¢ mg = 17 + 1
Example 19.2 Find the minimum height of the = 18 kg/kg of fuel
chimney required to produce a draught of 16 mm of H2O,
if 19 kg of air is required/kg of fuel burnt on the grate. To find Diameter of the chimney.
The mean temperature of flue gases inside the chimney is Analysis The natural draught in terms of hot-gas
330°C and the atmpospheric temperature is 30°C. column H ¢ is given by
ÈÊ ma ˆ Tg ˘
Solution H ¢ = H ÍÁ ˜¯ ¥ T - 1˙ (metres of hot gases)
Ë
ÍÎ am + 1 a ˙˚
Given H =? h = 16 mm of H2O
ÈÊ 17 ˆ 573 ˘
ma = 19 kg/kg of fuel = 40 ¥ ÍÁ ˜¥ - 1˙ = 33.879 m
Tg = 330°C + 273 = 603 K ÍÎË 17 + 1¯ 293 ˙˚
Ta = 30°C + 273 = 303 K Actual draught produced by chimney
Hact = H ¢ – hf
To find Height of the chimney
= H ¢ – 0.25 H ¢ = 0.75 ¥ 33.879 = 25.4 m
Actual velocity of flue gases in the chimney
Vg = 2gH act = 2 ¥ 9.81 ¥ 25.4 = 22.32 m
Density of flue gases
Ê m + 1ˆ 353
rg = Á a ˜ ¥
Ë ma ¯ Tg
Fig. 19.4 Ê 17 + 1ˆ 353
= Á ¥ = 0.65 kg/m3
Ë 17 ˜¯ 573
Analysis The draught produced in mm of H2O column
is given by, The diameter of the chimney
È 1 Ê m + 1ˆ 1 ˘ mg
h = 353H Í - Á a ˜ ¥ ˙ (mm of water) D = 1.128 ¥
r g Vg
ÍÎ Ta Ë ma ¯ Tg ˙˚
Now substituting the values, 18
= 1.128 ¥ = 1.255 m
0.65 ¥ 22.32
È 1 Ê 19 + 1ˆ 1 ˘
16 = 353H Í -Á ˜¯ ¥ 603 ˙ mm of H2O
Î 303 Ë 19 ˚
Example 19.4 A boiler uses 16 kg of air per kg of fuel,
= 353 ¥ H ¥ 1.55 ¥ 10 –3 = 0.5487H when the fuel consumption is at the rate of 1800 kg/h.
16 Actual draught required is 20 mm of water when all
\ H = = 29.15 m
0.5487 losses are considered. The surrounding air temperature
is 27°C and the flue-gas temperature is 277°C.
Boiler Draught and Performance 613

Determine the chimney height and its diameter Ê p 2ˆ


if actual velocity of the flue gases is 0.35 times the ÁË 4 D ˜¯ = 1.4846
theoretical velocity due to roughness of interior surfaces
of the chimney. Diameter of chimney D = 1.375 m

Solution Example 19.5 A boiler is equipped with a chimney


of 30 m height. The ambient temperature is 25°C. The
Given A boiler with a chimney temperature of flue gases passing through the chimney is
ma = 16 kg/kg of fuel m f = 1800 kg/h 300°C. If the air flow is 20 kg/kg of fuel burnt, find
h = 20 mm of water Ta = 27°C = 300 K (a) draught produced
Tg = 277°C = 550 K Vact = 0.35 Vg (b) the velocity of flue gases passing through chimney
if 50% of the theoretical draught is lost in
To find
friction
(i) Chimney height, and
(ii) Diameter of chimney. Solution
Analysis The draught in mm of water is expressed Given A boiler with a chimney
È 1 Ê m + 1ˆ 1 ˘ H = 30 m
h = 353H Í - Á a ˜ ¥ ˙ (mm of water) Ta = 25°C + 273 = 298 K
ÍÎ Ta Ë ma ¯ Tg ˚˙
Tg = 300°C + 273 = 573 K
È 1 Ê 16 + 1ˆ 1 ˘ ma = 20 kg/kg of fuel
20 = 353H Í -Á ˜ ¥ ˙
Î 300 Ë 16 ¯ 550 ˚ hf = 0.5 ¥ theoretical draught = 0.5H¢
H = 40.43 m To find
Density of flue gases (i) Draught produced, and
353 Ê ma + 1ˆ (ii) Velocity of flue gas.
rg = ¥
Tg ËÁ ma ¯˜
353 Ê 16 + 1ˆ
= ¥ = 0.683 kg/m3
550 ÁË 16 ˜¯
Mass-flow rate of flue gases,
mf 1800
mg = ( ma + 1) ¥ = (16 + 1) ¥ = 8.5 kg/s
3600 3600
Draught equivalent to hot-gas column
Fig. 19.5
ÈÊ ma ˆ Tg ˘
H¢ = H ÍÁ ˜ ¥ - 1˙ (metres of hot gases)
ÍÎË ma + 1¯ Ta ˙˚ Analysis

ÈÊ 16 ˆ 550 ˘ (i) The draught produced is given by Eq. (19.5)


= 40.43 ¥ ÍÁ ˜¥ - 1˙ = 29.33 m
ÍÎË 16 + 1¯ 300 ˙˚ È 1 Ê m + 1ˆ 1 ˘
Dp = 353 gH Í - Á a ˜ ¥ 2
˙ (N/m )
Actual velocity of flue gases ÍÎ Ta Ë ma ¯ Tg ˙˚
Vact = k 2 g H ¢ = 0.35 ¥ 2 ¥ 9.81 ¥ 29.33 = 8.396 m/s È 1 Ê 20 + 1ˆ 1 ˘
= 353 ¥ 9.81 ¥ 30 ¥ Í -Á ˜¯ ¥ 573 ˙
Further, mass-flow rate of flue gases
Î 298 Ë 20 ˚
mg = rg AVact
= 158.25 N/m2
Êp ˆ (ii) The velocity of flue gases,
8.5 = 0.682 ¥ Á D 2 ˜ ¥ 8.396
Ë4 ¯
Vg = 4.43 H ¢ - h f
614 Thermal Engineering

Height of hot-gas column Mass-flow rate of flue gases,


ÈÊ ma ˆ Tg ˘ mg = rg AVg
H¢ = H ÍÁ ˜ ¥ - 1˙
ÍÎË ma + 1¯ Ta p
˚˙ = 0.648 ¥ ¥ ( 2) 2 ¥ 24.6 = 50.11 kg/s
4
ÈÊ 20 ˆ 573 ˘
= 30 ¥ ÍÁ ˜¥ - 1˙ = 24.93 m
ÍÎË 20 + 1¯ 298 ˙˚ Example 19.7 A boiler plant uses 1500 kg of coal per
H¢ – hf = (1 – 0.5) H¢ = 0.5 ¥ 24.93 = 12.4688 m hour with the formation of 16 kg of dry flue gases per kg
of coal. The probable draught losses are
\ Vg = 4.43 ¥ 12.4688 = 15.64 m/s
Through fuel bed = 4.5 mm of water column
Example 19.6 Find the mass-flow rate of flue gases Boiler passages = 7.8 mm of water column
through the chimney when the draught produced is equal Straight breeching = 3 mm of water column
to 20 mm of water column. The temperature of gases is Velocity of gas flow = 0.5 mm of water column
300 °C and the ambient temperature is 30 °C. The mass Bends in breeeching = 0.4 mm of water column
of air used is 19 kg per kg of fuel burnt. Diameter of the
The expected outside temperature is 27 °C and mean
chimney is 2 m. Neglect the losses.
temperature of flue gases in the chimney is 250 °C. The
Solution actual draught may be taken as 0.8 times the theoretical
draught and the velocity of flow coefficient as 0.65. Find
Given A boiler with a chimney the height of the chimney.
ma = 19 kg/kg of fuel D =2m
h = 20 mm of water Ta = 30°C = 303 K Solution
Tg = 300°C = 573 K Given A boiler plant
To find Mass-flow rate of flue gases m f = 1500 kg/h
Tg = 250°C + 273 = 523 K
Analysis The draught in mm of water is expressed
Ta = 27°C + 273 = 300 K
È 1 Ê m + 1ˆ 1 ˘ ma + 1 = 16 kg/kg of fuel
h = 353H Í - Á a ˜ ¥ ˙ (mm of water)
ÍÎ Ta Ë ma ¯ Tg ˙˚ hact = 0.8 ¥ theoretical draught
È 1 Ê 19 + 1ˆ 1 ˘ k = 0.65
20 = 353 H Í -Á ˜¥ ˙ Total draught losses = 4.5 + 7.8 + 3 + 0.5 + 0.4
Î 303 Ë 19 ¯ 573 ˚
= 16.2 mm
H = 38.72 m
Height of hot-gas column To find
ÈÊ ma ˆ Tg ˘ (i) Height of chimney, and
H ¢ = H ÍÁ ˜ ¥ - 1˙ (ii) Diameter of chimney.
ÍÎË ma + 1¯ Ta ˙˚
ÈÊ 19 ˆ 573 ˘ Analysis Minimum theoretical draught required
= 38.72 ¥ ÍÁ ˜¥ – 1˙ = 30.84 m
ÍÎË 19 + 1 ¯ 303 ˙˚ =
Draught losses 16.2
= = 20.25 mm of water
Velocity of flue gases; 0.8 0.8
The draught in mm of water is expressed as
Vg = 2 g H ¢ = 2 ¥ 9.81 ¥ 30.84
È 1 Ê m + 1ˆ 1 ˘
= 24.6 m/s h = 353H Í - Á a ˜ ¥ ˙ (mm of water)
Density of flue gases ÍÎ Ta Ë ma ¯ Tg ˚˙
353 Ê ma + 1ˆ È 1 Ê 16 ˆ 1 ˘
rg = ¥ -Á ˜ ¥
Tg ÁË ma ˜¯
20.25 = 353 H Í ˙
Î 300 Ë 15 ¯ 523 ˚
353 Ê 19 + 1ˆ Height of chimney; H = 44.338 m
= ¥ = 0.648 kg/m3
573 ÁË 19 ˜¯
Boiler Draught and Performance 615

Example 19.8 How much air is used per kg of coal Ê 21.17 + 1ˆ 353
= Á ¥
burnt in a boiler having a chimney of 35-m height to Ë 21.17 ˜¯ 643
create a draught of 20 mm of water? The temperature
= 0.575 kg/m3
of gases in the chimney is 370°C and the boiler-house
temperature is 34°C. Does this chimney satisfy the Under the maximum discharge condition, the
condition for maximum discharge? Also, find the height of draught produced in mm of water, Eq. (19.18);
hot-gas column under maximum condition of discharge. 176.5 H 176.5 ¥ 35
h¢max = =
Ta 307
Solution = 20.122 mm of water
Given The chimney of a boiler; The equivalent column of hot gases
h = 20 mm of water Tg = 370°C = 643 K rw ¥ hmax
¢
H ¢max =
Ta = 34°C = 307 K H = 35 m rg

To find (1000 kg/m3 ) ¥ ( 21.12 ¥ 10 -3 m)


=
(i) The mass of air used per kg of coal, (0.575 kg/m3 )
(ii) Whether the chimney satisfy the condition for = 36.73 m
maximum discharge, and
Example 19.9 A 40-m high chimney is discharging
(iii) Height of hot gas column for maximum discharge.
flue gases at 350°C, when the ambient temperature is
Analysis 30°C. The quantity of air supplied is 18 kg per kg of fuel
(i) The mass of air used per kg of coal burnt. Determine
The height of chimney in terms of water column (a) draught produced in mm of water,
(b) equivalent draught in metres of hot-gas column,
È 1 Ê m + 1ˆ 1 ˘
h = 353H Í - Á a ˜ ¥ ˙ (c) efficiency of the chimney, if minimum temperature
ÍÎ Ta Ë ma ¯ Tg ˙˚ of artificial draught is 150°C; the mean specific
(mm of water) heat of flue gases is 1.005 kJ/kg ◊ K,
È 1 Ê m + 1ˆ 1 ˘ (d) the percentage of the heat spent in natural
20 = 353 ¥ 35 ¥ Í -Á a ˜ ¥ ˙ draught system, if the net calorific value of the
ÍÎ 307 Ë m a ¯ 643 ˚˙ fuel supplied be 30600 kJ/kg, and
ma + 1
= 1.0536 (e) the temperature of chimney gases for maximum
ma discharge in a given time and what would be the
or ma = 18.66 kg of air/kg of coal corresponding draught in mm of water produced.
(ii) For maximum discharge;
Tg Ê m¢ + 1ˆ Solution
= 2Á a ˜
Ta Ë ma¢ ¯ Given The chimney;
Ê m¢ + 1ˆ H = 40 m Tg = 350°C or 623 K
643
or = 2Á a ˜ Ta = 30°C = 303 K ma = 18 kg/kg of fuel
307 Ë ma¢ ¯
Tg¢ = 150°C = 423 K Cp = 1.005 kJ/kg ◊ K
or m¢a = 21.17 kg of air per kg of coal CV = 30600 kJ/kg
Since m¢a > ma, thus the chimney does not satisfy
the condition of maximum discharge. To find
(iii) Height of hot-gas column for maximum discharge (i) Draught produced in mm of water,
For maximum discharge condition, the density of (ii) Equivalent draught in metres of hot-gas column,
flue gases using Eq. (19.4); (iii) Efficiency of the chimney,
( ma¢ + 1) 353 (iv) The percentage of the heat spent in natural
rg = ¥ draught system, and
ma¢ Tg
616 Thermal Engineering

(v) The temperature of chimney gases for maximum Corresponding draught,


discharge and corresponding draught in mm of È 1 Ê 18 + 1ˆ 1 ˘
water. h = 353 ¥ 40 ¥ Í -Á ˜¯ ¥ 639.66 ˙
Î 303 Ë 18 ˚
Analysis = 23.3 mm of water
(i) Draught in mm of water
È 1 Ê m + 1ˆ 1 ˘
h = 353H Í - Á a ˜ ¥ ˙ (mm of water)
ÍÎ Ta Ë ma ¯ Tg ˙˚
È 1
Usually, the static draught produced by the
Ê 18 + 1ˆ 1 ˘
h = 353 ¥ 40 ¥ Í -Á ˜¯ ¥ 623 ˙ chimney is not sufficient to meet the requirement
Î 303 Ë 18 ˚ of draught of a boiler that varies from 30 to 350 mm
h = 22.676 mm of water column. When artificial draught is used on
(ii) Draught equivalent to hot-gas column the boiler then the waste heat carried by the flue
ÈÊ ma ˆ Tg ˘ gases can be better utilized in an economiser, air
H ¢ = H ÍÁ ˜¯ ¥ T - 1˙ (metres of hot gases)
Ë
ÍÎ am + 1 a ˙˚ preheater, etc.
An artificial draught produced by a fan or
ÈÊ 18 ˆ 623 ˘
= 40 ¥ ÍÁ ˜¥ - 1˙ = 37.915 m a blower is known as mechanical draught and
ÍÎË 18 + 1¯ 303 ˙˚ that produced by steam jet is known as steam jet
(iii) Efficiency of the chimney draught.
H ¢g
hchimney =
C p (Tg - Tg¢ )
37.915 ¥ 9.81 Draught produced by a fan or blower may be of
=
1005 ¥ (623 - 423) three types: (a) induced, (b) forced, and (c) balanced
= 0.00185 or 0.185% draught.
(iv) The percentage of the heat spent in natural
(a) The fan is placed near the
draught system
base of the chimney as shown in Fig. 19.6. The
Extra heat carried away by flue gases in the
natural draught system per kg of fuel burnt
fan draws the flue gases from the furnace. So the
QEx = mg Cp (Tg – Tg¢)
pressure above the fuel bed is reduced below the
= (18 + 1) ¥ 1.005 ¥ (623 – 423)
atmospheric pressure. The fresh air rushes to the
= 3819 kJ/kg of fuel
furnace and after combustion, the flue gases get
Percentage heat spent discharged through the chimney in the atmosphere.
Q 3819
= Ex ¥ 100 = ¥ 100
CV 30600
= 12.48%
(v) Temperature of chimney gases for maximum
discharge and corresponding draught in mm of
water
Using Eq. (19.16),
Ê m + 1ˆ
Tg = 2 Ta Á a ˜ Fig. 19.6
Ë ma ¯
(b) The fan or blower is located
Ê 18 + 1ˆ
= 2 ¥ 303 ¥ Á = 639.66 K near or at the base of the boiler grate to force
Ë 18 ˜¯
atmospheric air on to the furnace under pressure.
This pressure helps in circulation of flue gases
Boiler Draught and Performance 617

through components of the boiler and then through (vi) Smoke formation is less.
chimney to atmosphere. It is shown in Fig. 19.7. (vii) Tendancy for air leakage in the furnace is
less.

Fans are used in forced and induced draught


arrangements. The fan is driven by an electric
motor. The power required to drive the fan can be
calculated as under.
Let Dp =pressure difference caused by fan
Fig. 19.7 (draught) in kPa,
(c) A combination of induced V = volume flow rate of combustion air/
and forced draught in a boiler is known as a gases at fan conditions, m3/h
balanced draught. A forced draught fan located near hf = fan efficiency
the grate supplies air under the pressure through the The power required to drive the fan
furnace and an induced draught fan located near D pV
the chimney base, draws in flue gases through the P= ...(19.20)
hf
economiser, air preheater, etc., and discharges them
into the atmosphere through a chimney. Figure 19.8 Expressing draught in terms of water column
illustrates the balanced draught system. D p = rw g h
where rw = 1000 kg/m3,
g = 9.81 m/s2 and h in mm, therefore,
3 2 Ê h ˆ
D p = (1000 kg/m ) ¥ (9.81 m/s ) ¥ Á m
Ë 1000 ˜¯
= 9.81h (N/m2)
9.81hV
Then P = (watts)
hf
Fig. 19.8
9.81 hV
or P = (kW) ...(19.21)
1000 ¥ h f
Over Natural Draught
Forced Draught Power, PFD In a forced draught
(i) Forced draught puts better control on com- system, the fan is located before the combustion
bustion. chamber and it handles only atmospheric air.
(ii) Low-grade fuel can be efficiently burnt.
Let v0 = Specific volume of air at NTP
(iii) Overall efficiency of a thermal power plant
is higher due to better heat recovery from RT0 (0.287 kJ/kg.K) ¥ (273 K)
= =
the exhaust flue gases in economisers and p0 (101.325 kPa)
preheaters. = 0.7732 m3/kg
(iv) The height of the chimney can be reduced ma = Mass of air supplied, kg/kg of fuel
than that required in natural draught.
m f = Rate of fuel firing, kg/s
(v) Higher the rate of fuel-burning capacity
The volume of air supplied to furnace at NTP;
on the grate, higher the heat-supply rate at 3
higher effective temperature. V0 = v0 ma m f = 0.7732 ma m f (m /s)
618 Thermal Engineering

The volume of air supplied to the furnace at fan The temperature of flue gases leaving the boiler in
conditions; each case is 200°C and of air in the boiler house is at
V0 0.7732ma m f 33°C. The air supplied is 18.5 kg per kg of fuel and mass
Va = Ta = Ta of coal fired per hour is 1820 kg. Assume a fan efficiency
T0 273
of 80% in both cases.
ma m f
or Va = Ta ...(19.22)
373 Solution
Using in Eq. (19.21), the power required to drive Given
the forced-draught fan h = 45 mm of water Tg = 200°C or 473 K
9.81h ma m f Ta = 33°C = 306 K ma = 18.5 kg/kg of fuel
P = Ta (kW) ...(19.23)
1000 ¥ 353h f m f = 1820 kg/h h f = 0.8

Induced Draught Power, PFD In an induced To find


draught system, the fan is located near the base of (i) Power required to drive induced-draught fan.
the chimney and it handles the products of combus- (ii) Power required to drive forced-draught fan.
tion, i.e., volume of flue gases. Analysis
Let ma =Mass of air supplied, kg/kg of fuel (i) Power for Induced draught fan, Eq. (19.25)
ma + 1 =Mass of flue gases, kg/kg of fuel 9.81h ma m f
mf =Rate of fuel firing, kg/s P = Tg
1000 ¥ 353h f
m f (ma + 1) =
Mass flow rate of flue gases across 9.81 ¥ 45 ¥ 18.5 ¥ (1820 / 3600)
the induced draught fan, kg/s = ¥ 473
1000 ¥ 353 ¥ 0.8
The volume of air supplied to furnace at NTP; = 6.91 kW
Mass flow rate of flue gases (ii) Power for forced draught fan, Eq. (19.23)
Vg =
Density of flue gases 9.81h ma m f
P = Ta (kW)
m f ( ma + 1) 1000 ¥ 353h f
= (m3/s)
rg 9.81¥ 45 ¥ 18.5 ¥ (1820 / 3600)
= ¥ 306
( ma + 1) 353 1000 ¥ 353 ¥ 0.8
Using rg = ¥ from Eq. (19.4)
ma Tg = 4.47 kW
m f ma
then Vg = Tg (m3/s) ...(19.24) Example 19.11 Compare the fan power input for
353 forced and induced draught fans and also compare the
Using in Eq. (19.21), the power required to drive quantity of heat carried away by the flue gases per kg of
the forced-draught fan fuel fired in each case. Assume specific heat of flue gases
9.81h ma m f as 1.005 kJ/kg ◊ K.
P= Tg (kW) ...(19.25)
1000 ¥ 353h f
Types of Outside air Flue gas Mass of
If induced-draught and forced-draught fans are draught temperature leaving boiler air in
equally efficient and produce same draught then °C Temperature, kg/kg of
°C fuel
Power of ID fan Tg
= ...(19.26) Forced 27 150 15
Power of FD fan Ta draught
Induced 27 150 15
Example 19.10 Calculate the motor power required
draught
to drive a fan which maintains a draught of 45 mm of Chimney 27 370 20
water under the following conditions for (a) induced- draught
draught fan, (b) forced-draught fan.
Boiler Draught and Performance 619

Solution In a forced steam-jet draught, the jet of live


steam from the boiler is introduced into the ash pit.
Given
The high velocity jet of steam draws in atmospheric
Tg¢ = 150°C or 423 K Tg = 370°C or 643 K air and forces it on to the furnace, fire tubes and fi-
Ta = 27°C = 300 K ma1 = 15 kg/kg of fuel nally discharges to the atmosphere through chimney.
ma2 = 20 kg/kg of fuel Cpg = 1.005 kJ/kg ◊ K The steam-jet draught system is economical,
To find simple in operation and maintenance free and re-
(i) Comparision of power input to induced and quires very little space for installation. But draught
forced draught fans, and produced by a steam jet is very low and can be in-
(ii) Comparision of heat carried away by flue gases. stalled only on small boilers. It has also great dis-
advantage that it cannot be operated until proper
Analysis
steam pressure is available. The passing on to the
(i) Comparision of power input to induced and furnace also carries away some quantity of heat
forced draught fan, Eq. (19.26)
along with flue gases.
Power of ID fan Tg 423
= = = 1.41
Power of FD fan Ta 300
The heat carried away by flue gases in artificial
(i) It is a very simple and economical artificial
draught
draught.
Q1 = (ma1 + 1) Cpg (Tg¢ – Ta)
= (15 + 1) ¥ 1.005 ¥ (423 – 300) (ii) Low-grade fuel can be burnt efficiently.
= 1977.84 kJ/kg of fuel (iii) It is automatically adjusted to suit the boiler
The heat carried away by flue gases in natural requirement, because the exhaust steam from
draught the steam engine is used to produce draught.
Q2 = (ma2 + 1)Cpg (Tg¢ – Ta) (iv) It does not require any maintenance cost.
= (20 + 1) ¥ 1.005 ¥ (643 – 300) (v) In a forced steam-jet draught, the fire bars
= 7239 kJ/kg of fuel are kept cool by the steam, thus adhering of
Q2 7239 clinker to the fire bars are prevented.
Further, = = 3.66
Q1 1977.84
In case of natural draught, the flue gases carry
3.66 times more heat than that in artificial draught.
OF BOILERS

Steam Jet Draught The performance of boiler is calculated for


comparing the boilers for its evaporating capacity,
Artificial draught can easily be produced by a steam energy consumption and heat balance sheet. The
jet of either induced type or forced type. quantity of heat supplied to vaporise water from
In an induced steam-jet draught, the exhausted and at 100°C and 1 atm is referred as evaporation
steam from an steam engine or live steam from the unit.
boiler is introduced into the smoke box or in the
chimney in the form of a jet. A partial vacuum is
created, which draws in air through the ash pit,
fuel bed and boiler flue tubes into the smoke box, The quantity of steam generated by a boiler at full
from where they are discharged to the atmosphere load is called the evaporative capacity of the boiler.
through chimney. Such an arrangement is used on a The performance of boilers is often measured in
locomotive boiler. terms of evaporative capacity. The evaporative
620 Thermal Engineering

capacity of a boiler can be expressed on the basis of = hg + Cps (Tsup – Tsat) for superheated
time, heating area, or fuel burned. These are steam
Evaporation rate hf1 = Enthalpy of water at feed temperature
Mass of steam generated m f = Mass of fuel burned per hour
= (kg/h) me = Equivalent mass of dry and saturated
Time period in hours
...(19.27) steam generated from and at 100°C
The evaporation per kg of fuel
or Evaporation rate in kg of steam/h/m2
Mass of steam generated per hour
Mass of steam generated per hour ma =
= Mass of fuel burned per hour
Heating area of grate in m 2 ms
= ...(19.30)
...(19.28) mf
or Evaporation rate in kg of steam/kg of fuel The total heat supplied to water in the boiler
Mass of steam generated per hour = ma (h – hf1)
= and the equivalent evaporation
Mass of fuel burned per houur
...(19.20) ma ( h - h f1) ma ( h - h f1 )
me = =
h fg @ 100∞C 2257
(kg of steam/kg of fuel) ...(19.31)
The quantity of steam generated by a boiler depends
on the following four important parameters:
(i) feed-water temperature, (ii) working pressure, It is defined as the ratio of heat received by 1 kg
(iii) fuel used, and (iv) quality of steam generated. of feed water for evaporation under actual working
In practical situations, the different boilers use feed conditions to that received by 1 kg of water
water at different temperatures, operate at different evaporated from and at 100°C. It is denoted by Fe
pressures and generate different qualities of steam. and expressed as
Therefore, such boilers cannot be compared unless ( h - h f1) h - h f1
some standard parameters are adopted. Hence feed Fe = = ...(19.32)
h fg @ 100∞C 2257
water is taken at 100 °C and working pressure is
at 1.01325 bar. At this state, water evaporates at It can also be expressed as
100°C and requires 2257 kJ/kg of latent heat to m
Fe = e
convert it into steam at 100°C. ma
Equivalent evaporation is defined as the amount Equivalent evaporation from and at 100∞C
of dry and saturated steam generated from feed =
Evaporation per kg of fuel
water at 100°C at normal atmospheric pressure. In ...(19.33)
short, it is defined as the amount of steam generated
from and at 100°C.
Let ms = Mass of steam generated at pressure
p and temperature T per hour in the Although the equivalent evaporation of steam in the
boiler boiler is a fair indication of the performance of the
boiler, it does not take into account the quantity of
h = Enthalpy of steam
fuel burned in the furnace of the boiler. Hence, the
= hf + x hfg for wet steam
term boiler efficiency is used for comparison of the
= hg for dry and saturated steam performance of boilers.
Boiler Draught and Performance 621

The boiler efficiency is defined as the ratio


of actual heat utilized in producing steam to the
amount of heat librated by burning of fuel.
The percentage heat utilised in the superheater is
Heat required to produce steam
defined as the ratio of heat absorbed by the steam
from feed water
hboiler = in the superheater for its superheating to the heat
Heat librated by burning of fuel liberated by combustion of fuel. Mathematically;
ms ( h - h f1 ) % heat utilised in superheater
= ...(19.34)
m f ¥ CV Heat utilised in superheater
=
where ms = Rate of mass of steam generation, Heat liberated by combustion off fuel
kg/h ms (hsup - h) (hsup - h)
m f = Rate of mass of fuel firing, kg/h = = ...(19.38)
m f CV CV
h = Total enthalpy of steam, kJ/kg
hf1 = Enthalpy of water at feed Where
temperature ms = Rate of mass of steam generation, kg/h
CV = Calorific value of fuel in kJ/kg m f = Rate of mass of fuel firing, kg/h
m h =Total enthalpy of steam entering the
In Eq. (19.34), the quantity s = ma (evapora-
mf superheater, kJ/kg
tion per kg of fuel). Hence, the boiler efficiency can = hf + x hfg
also be expressed as hsup = Enthalpy of superheated steam leaving
m ( h - h f1 ) the super-heater, kJ/kg;
hboiler = a ...(19.35)
CV = hg + Cps (Tsup – Tsat)
CV = calorific value of fuel in kJ/kg.

It is defined as the ratio of heat absorbed by the feed


water in the economiser to the heat available to flue
It is expressed as
gases in the economiser.
Heat utilised in economiser Heat utilised in boiler plant
heconomiser = hoverall =
Heat available of flue gases Heat produced by burning of fuel
...(19.36) ...(19.39)
More often, it is necessary to obtain the per-
centage heat utilised in an economiser rather than
the economiser efficiency. The percentage heat
utilised in the economiser is defined as the ratio According to the American Society of Mechanical
of heat absorbed by the feed water in the econo- Engineers (ASME), the boiler power is expressed
miser to the heat liberated by combustion of fuel. as
Mathematically; Equivalent evaporation from and at
% heat utilised in economiser 100∞C per hour
=
Heat utilised in economiser 21.296
= ...(19.40)
Heat liberated by combustion off fuel
ms (Dh) feed water
= Example 19.12 Calculate the equivalent evaporation
m f CV of a boiler per kg of coal fired, if the boiler produces
m (Dh) feed water 50,000 kg of wet steam per hour with a dryness fraction
= a ...(19.37)
CV
622 Thermal Engineering

of 0.95 and operating at 10 bar. The coal burnt per hour x = 9.6
in the furnace is 5500 kg and feed-water temperature is mf = 10 lit = 10 ¥ 10–3 m3
40°C. SG = 0.85
Dt2 = 11 min., 25 s
Solution
Tf1 = 35°C CV = 43125 kJ/kg
Given Steam generation in a boiler
To find Thermal effciency of boiler, hboiler .
p = 10 bar x = 0.95
ms = 50,000 kg/h m f = 5500 kg/h Analysis The absolute pressure of generated steam
Tf1 = 40°C p = patm + pg = 6 + 1 = 7 bar
Density of fuel
To find Equivalent evaporation of the boiler per kg of
rf = SG ¥ rwater = 0.85 ¥ 1000
coal fired
= 850 kg/m3
Analysis Properties of steam at 10 bar (1000 kPa) Properties of steam at 7 bar
Tsat = 179.91°C hf = 697.20 kJ/kg, hfg = 2064.85 kJ/kg
hf = 762.79 kJ/kg Enthalpy of water at 35°C
hfg = 2015.29 kJ/kg hf1 = 146.66 kJ/kg
Enthalpy of feed water at 40°C; The specific enthalpy of steam at 7 bar
hf1 = 167.54 kJ/kg h = hf + x hfg
The mass of steam generated per kg of coal fired = 697.20 + 0.96 ¥ 2064.85
ms 50000 = 2763.01 kJ/kg
ma = = = 9.09 kg/kg of fuel
mf 5500 Rate of steam generation
The specific enthalpy of steam at 10 bar ms 75
ms = = = 0.1315 kg/s
h = hf + x hfg Dt1 9.5 ¥ 60
= 762.79 + 0.95 ¥ 2015.29 = 2677.31 kJ/kg Rate of fuel consumption
The equivalent evaporation mf (10 ¥ 10 -3 m3 ) ¥ (850 kg/m3 )
ma ( h - h f1) mf = =
me = Dt 2 (11 min. ¥ 60 s/min + 25 s)
2257 = 0.0124 kg/s
9.09 ¥ (2677.31 - 167.54)
= Thermal efficiency of boiler
2257
ms ( h - h f 1 )
= 10.1 kg/kg of fuel h =
m f CV
Example 19.13 In an experiment on a small oil-fired 0.1315 ¥ (2763.01 - 146.66)
boiler, the steam produced is at 6 bar gauge. The quality =
0.0124 ¥ 43125
of steam produced is found out to be 0.96 dry. 75 litres
= 0.6429 or 64.29%
of water is converted into steam in 9.5 minutes. The fuel
is a light diesel oil with specific gravity of 0.85 and calo- Example 19.14 Calculate the equivalent evaporation
rific value of 43125 kJ/kg. 10 litres of oil is consumed in from and at 100°C for a boiler, which receives water at
11 minutes and 25 seconds. The feed-water temperature 60°C and produces steam at 1.5 MPa and 300°C. The
is 35°C. Determine the boiler efficiency. Take atmospher- steam-generation rate is 16000 kg/h. Coal is burnt
ic pressure as 1 bar. at the rate of 1800 kg/h. The calorific value of coal is
34750 kJ/ kg. Also calculate the thermal efficiency of the
Solution
boiler.
Given Steam generation in a small oil-fired boiler If the thermal efficiency of the boiler increases by
pg = 6 bar patm = 1 bar 5% due to use of an economiser, find the saving in coal
ms = 75 litres = 75 kg Dt1 = 9.5 min consumption per hour.
Boiler Draught and Performance 623

Solution The saving in fuel consumption


= m f – m f1 = 1800 – 1682 = 118 kg/h
Given Steam generation in a boiler
p = 1.5 MPa Tsup = 300°C Example 19.15 A boiler generates 8 kg of steam per
ms = 16000 kg/h m f = 1800 kg/h kg of fuel burnt at a pressure of 12 bar from feed water
Tf1 = 60°C CV = 34750 kJ/kg entering at 80°C. The boiler is 75% efficient and its
h2 = h1 + 0.05 factor of evaporation is 1.15. Calculate
(a) Degree of superheat and temperature of steam
To find generated,
(i) Equivalent evaporation from and at 100°C, (b) Calorific value of fuel in kJ/kg,
(ii) Thermal effciency of boiler, h1, (c) Equivalent evaporation in kg of steam per kg of
(iii) Saving in fuel consumption, when boiler effciency fuel.
increases by 5%. Take specific heat of superheated steam as 2.3 kJ/kg ◊ K.
Analysis Properties of superheated steam at 1.5 MPa
Solution
(1500 kPa) and 300°C
h = 3038.9 kJ/kg Given A boiler with a superheater
Enthalpy of water at 60°C ma = 8 kg/kg of fuel Cps = 2.3 kJ/kg ◊ K
hf1 = 251.1 kJ/kg hboiler = 0.75
(i) The mass of steam generated per kg of coal fired p = 12 bar Fe = 1.15
ms 16000 Tf1 = 80°C
ma = = = 8.89 kg/kg of fuel
mf 1800 To find
The equivalent evaporation from and at 100°C (i) Degree of superheat and temperature of steam
ma ( h - h f1 ) generated,
me = (ii) Calorific value of fuel in kJ/kg, and
2257
8.89 ¥ (3038.9 - 251.1) (iii) Equivalent evaporation in kg of steam per kg of
= fuel.
2257
= 10.1 kg/kg of fuel Analysis The properties of steam
(ii) Thermal efficiency of boiler At 12 bar: Tsat = 188°C
ms ( h - h f 1 ) hg = 2784.83 kJ/kg
h1 = At 80°C: hf1 = 334.88 kJ/kg
m f CV
(i) Degree of superheat and temperature of steam
16000 ¥ (3038.9 - 251.1)
= generated
1800 ¥ 34750
Factor of evaporation
= 0.7131 or 71.31%
h - hf 1 Èhg + C ps (Tsup - Tsat ) - h f ˘
(iii) Saving in fuel consumption, when boiler effciency Fe= =Î 1˚

increases by 5% 2257 2257


With use of economiser, thermal effciency of 2784.83 + 2.3 ¥ (Tsup - Tsat ) - 334.88
1.15 =
boiler; 2257
h2 = h1 + 0.05 = 0.7131 + 0.05 = 0.7631 2595.55 = 2452.25 + 2.3 ¥ (Tsup – Tsat)
ms ( h - h f 1 ) (Tsup – Tsat) = 62.3°C
Then h2 =
m f 2 CV The temperature of superheated steam is
16000 ¥ (3038.9 - 251.1) Tsup = 188 + 62.3 = 250.3°C
or m f2 = Specific enthalpy of superheated steam;
0.7631 ¥ 34750
= 1682 kg/h h = hsup = hg + Cps (Tsup – Tsat)
624 Thermal Engineering

= 2784.83 + 2.3 ¥ (250.3 – 188) Analysis


= 2928.12 kJ/kg ◊ K (i) Heat produced by complete burning of fuel
(ii) Calorific value of fuel Q1 = m f ¥ CV
The boiler efficiency is given as = 227 ¥ 30,000 = 681 ¥ 104 kJ/h
ms ( h - h f ) ma ( h - h f 1) Heat utilized in generation of steam
hboiler = =
m f ¥ CV CV Q2 = ms (h – hf1)
= 2060 ¥ (2750 – 4.187 ¥ 105)
8 ¥ ( 2928.12 - 334.88)
0.75 = = 475.93 ¥ 104 kJ/h
CV
The efficiency of boiler
Calorific value of fuel;
Heat utilised in generation of steam
20745.92 hboiler =
CV = = 27661.22 kJ/kg Heat produced by burn ning of fuel
0.75
475.93 ¥ 10 4 kJ / h
(iii) Equivalent evaporation hboiler =
ma ( h - h f 1) 681 ¥ 10 4 kJ / h
me = = 0.6988 or 69.88%
2257
(ii) Heat utilized by economiser
8 ¥ ( 2928.12 - 334.88)
= = Heat used to raise the temp. of water from 15°C
2257 to 105°C.
= 9.19 kg/kg of fuel Q3 = ms Cpw (Tf1 – T1)
= 2060 ¥ 4.187 ¥ (105 – 15)
Example 19.16 Calculate the efficiency of (a) boiler,
(b) economiser, and (c) whole plant having the following = 77.62 ¥ 104 kJ/h
data: Heat available in flue gases passing to economiser
(a) Boiler Mass of the feed water = 2060 kg/h Q4 = Heat of flue gases above atmospheric
Mass of the coal burnt = 227 kg/h temperature
Calorific value of coal = 30,000 kJ/kg = mg ¥ Cpg ¥ (Tg – Ta )
Enthalpy of steam produced = 2750 kJ/kg = 4075 ¥ 1.30 ¥ (370 – 18)
(b) Economiser = 147.74 ¥ 104 kJ/h
Inlet temperature of feed water = 15°C The efficiency of economiser
Exit temperature of feed water = 105°C Heat utilised in economiser
heconomiser =
Atmospheric air temperature = 18°C Heat available of flue gases
Temperature of flue gases entering = 370°C 77.62 ¥ 10 4 kJ
= = 52.53%
Mass of flue gases = 4075 kg/h 147.74 ¥ 10 4 kJ
Specific heat of flue gases = 1.3 kJ/kg ◊ °C (iii) Overall efficiency of plant
Solution Heat utilised in boiler plant
hoverall =
Heat produced by burning of fuel
Given A boiler plant with an economiser
m f = 227 kg/h ms = 2060 kg/h 475.93 ¥ 10 4 + 77.62 ¥ 10 4
=
CV = 30,000 kJ/kg C h = 2750 kJ/kg 681 ¥ 10 4
T1 = 15°C Tf1 = 105°C = 0.8128 or 81.28%
Tg = 370°C mg = 4075 kg/h
Cpg = 1.3 kJ/kg,°C Example 19.17 A boiler generates steam at the rate
of 6000 kg/h at a pressure of 800 kPa with a dryness
To find fraction of 0.98. The feed water is supplied at 40°C. If the
(i) Efficiency of boiler, efficiency of the boiler is 75%, calculate the rate of coal
(ii) Efficiency of economiser, and consumption, which has a calorific value of 31000 kJ/kg.
(iii) Efficiency of boiler plant. What is equivalent evaporation from this boiler?
Boiler Draught and Performance 625

If the superheater is used with the boiler and The equivalent evaporation
temperature of superheated steam reaches 250°C, then
ma ( h - h f1)
(a) what is the equivalent evaporation from the boiler, me =
and (b) what is the thermal efficiency of boiler? 2257
Take specific heat of superheated steam as Cps = 9.08 ¥ (2727.45 - 167.54)
=
2.27 kJ/kg ◊ °C 2257
= 10.30 kg/kg of fuel
Solution (c) Boiler with superheater
Given A boiler plant with economiser The specific enthalpy of water
ms = 6000 kg/h p = 800 kPa hsup = hg + Cps (Tsup – Tsat)
x = 0.98 Tf1 = 40°C = hf + hfg + Cps(Tsup – Tsat)
h1 = 0.75 CV = 31000 kJ/kg = 720.9 + 2047.5 + 2.27
Tsup = 250°C Cps = 2.27 kJ/kg◊ K ¥ (250 – 170.40)
= 2949.09 kJ/kg
To find
Heat utilised by steam,
(i) Rate of coal consumption without superheater in
qu = hsup – hf1 = 2949.09 – 167.54
the boiler,
= 2781.55 kJ/kg
(ii) Equivalent evaporation from boiler without su-
Then thermal efficiency of boiler
perheater, and
(iii) Equivalent evaporation from boiler with super- Heat utilsed in generation of steam
hboiler =
heater. Heat produced by burnning of fuel
(iv) Efficiency of boiler plant. ms qu
=
Analysis The properties of steam: m f ¥ CV
At 800 kPa Tsat = 170.40°C (6000 kg/h) ¥ (2781.55 kJ/kg)
=
hf = 720.9 kJ/kg (660.62 kg/h) ¥ (31000 kJ/kg)
hfg = 2047.5 kJ/kg
= 0.8149 or 81.49%
At 40°C hf1 = 167.54 kJ/kg
(i) Boiler without superheater Example 19.18 In a boiler trial of one-hour duration,
The specific enthalpy of water the following observations were made:
h = hf + x hfg Steam generated = 5250 kg
= 720.9 + 0.98 ¥ 2047.5 Fuel burnt = 695 kg
= 2727.45 kJ/kg Calorific value of fuel = 30200 kJ/kg
(a) Mass of coal fired per hour Steam condition = 0.94
Thermal efficiency of boiler Boiler (steam) pressure = 11 bar (gauge)
Heat used in generation of steam Temperature of hot well = 47°C,
hboiler =
Heat produced by burning of fuel Temp. of steam leaving superheater = 240°C
ms ( h - h f 1 ) Calculate:
= (a) Equivalent evaporation per kg of coal without
m f ¥ CV
and with superheater
6000 ¥ (2727.45 - 167.54) (b) Boiler efficiency with and without superheater
mf =
0.75 ¥ 31000 Assume atmospheric pressure as 100 kPa.
= 660.62 kg/h
(b) The evaporation rate per kg of coal fired Solution
m 6000
ma = s = Given A trial on boiler with superheater
mf 660.62
ms = 5250 kg/h x = 0.94
= 9.08 kg/kg of fuel
626 Thermal Engineering

pg = 11 bar patm = 100 kPa = 1 bar ms ( h1 - h f 1)


m f = 695 kg/h CV = 30200 kJ/kg =
m f ¥ CV
Tf1 = 47°C Tsup = 240°C
5250 ¥ (2665.65 - 196.80)
=
To find 695 ¥ 30200
(i) Equivalent evaporation without and with super- = 0.6175 or 61.75%
heater, With superheater;
(ii) Efficiency of boiler with and without superheater.
ms ( h2 - h f 1)
hboiler =
Assumption The specific heat for superheated steam m f ¥ CV
as 2.1 kJ/kg ◊ K.
5250 ¥ (2894.03 - 196.80)
=
Analysis The absolute pressure of steam in the boiler 695 ¥ 30200
p = pg + patm = 11 + 1 = 12 bar = 0.6746 or 67.46%
The properties of steam:
At 12 bar (1200 kPa): Tsat = 188°C, Example 19.19 The following reading was recorded
during a boiler trial:
hf = 798.64 kJ/kg,
Feed water = 2400 kg/h
hfg = 1986.19 kJ/kg,
Ambient temperature = 33°C
At 47°C: hf1 = 196.8 kJ/kg.
Feed water temperature = 42°C,
(i) Equivalent evaporation:
Fuel used = 205 kg/h
Evaporation rate per kg of fuel;
Composition of fuel by mass:
m 5250
ma = s = = 7.554 kg/kg of fuel Carbon = 84%, Hydrogen = 9.27%, Oxygen = 6.73%
mf 695
The specific enthalpy of steam without superheat- Calorific value of fuel = 39500 kJ/kg
ing Average chimney temp. = 307°C
h1 = hf + x hfg = 798.64 + 0.94 ¥ 1986.19 Height of chimney = 32 m
= 2665.65 kJ/kg Steam pressure = 11.054 bar (gauge)
The equivalent evaporation without superheater Barometric pressure = 710 mm of Hg
Air supplied in excess = 50%
ma ( h1 - h f 1)
me = Steam condition = 0.96
2257
Calculate
7.554 ¥ (2665.65 - 196.80)
= (a) Boiler efficiency,
2257
(b) Equivalent evaporation from and at 100°C,
= 8.26 kg/kg of fuel
(c) Draught produced in mm of water.
The specific enthalpy of water with superheating
h2 = hf + hfg + Cps (Tsup – Tsat) Solution
= 798.64 + 1986.19 + 2.1 ¥ (240 – 188)
Given A boiler plant with an economiser
= 2894.03 kJ/kg
ms = 2400 kg/h x = 0.96
The equivalent evaporation with superheater
ma ( h2 - h f1) Ta = 33°C
me = pg = 11.054 bar
2257
7.554 ¥ (2894.03 - 196.80) patm = 710 mm of Hg
= H = 32 m Tf1 = 42°C
2257
= 9.02 kg/kg of fuel m f = 205 kg/h CV = 39500 kJ/kg
(ii) Thermal efficiency of boiler Tg = 307°C ma = 1.5 mth
Without superheater; Composition of fuel by mass:
Heat used in generation of steam Carbon = 84%
hboiler =
Heat produced by burningg of fuel
Boiler Draught and Performance 627

Hydrogen = 9.27% 100 È 8 Ê 0.0673 ˆ ˘


= ¥ Í ¥ 0.84 + 8 Á 0.0927 - + 0˙
Oxygen = 6.73% 23 Î 3 Ë 8 ˜¯ ˚
To find = 12.67 kg/kg of fuel
(i) Efficiency of boiler, The mass of actual air supplied;
(ii) Equivalent evaporation from and at 100°C, ma = 1.5 mth = 1.5 ¥ 12.67
(iii) Draught produced in mm of water. = 19 kg/kg of fuel
The draught produced in water column
Analysis The pressure in bar corresponding to 710 mm
of Hg È 1 Ê ma + 1ˆ 1 ˘
h = 353H Í -Á ˜ ¥ T ˙ (mm of water)
710 mm of Hg ÍÎ a
T Ë m ¯ g ˙˚
patm = (1.0325 bar) ¥ a
760 mm of Hg È ˘
1 Ê 19 + 1ˆ 1
= 0.9466 bar = 353 ¥ 32 ¥ Í -Á ˜¯ ¥ (307 + 273) ˙
Î ( 33 + 273) Ë 19 ˚
The absolute pressure of steam in the boiler
p = pg + patm = 11.054 + 0.9466 = 12 bar = 16.415 mm of water
The properties of steam
Example 19.20 The following readings were record-
At 12 bar (1200 kPa) ed during two hour trial on a boiler:
hf = 798.64 kJ/kg Feed water supplied = 14000 kg
hfg = 1986.19 kJ/kg Boiler working pressure = 10 bar
At 42°C: hf1 = 175.92 kJ/kg Dryness fraction of steam = 0.96
The specific enthalpy of steam; Temp. of feed water entering economiser = 35°C
h = hf + x hfg = 798.64 + 0.96 ¥ 1986.19 Temp. of feed water leaving economiser = 90°C
= 2705.38 kJ/kg Temperature of steam leaving superheater = 250°C
(i) Thermal efficiency of boiler Coal burnt = 1500 kg
Heat used in generation of steam Calorific value of coal = 33500 kJ/kg
hboiler =
Heat produced by burningo f fuel Calculate:
ms ( h - h f 1)
= (a) Enthalpy received by feed water in economiser,
m f ¥ CV boiler and superheater,
2400 ¥ (2705.38 - 175.92) (b) Percentage of heat utilized in economiser, boiler
=
205 ¥ 39500 and superheater,
= 0.7497 or 74.97% (c) Overall thermal efficiency of plant,
(ii) Equivalent evaporation from and at 100°C (d) Overall equivalent evaporation from and at
Evaporation rate per kg of fuel; 100°C.
2400
ma = = 11.7 kg/kg of fuel
205 Solution
The equivalent evaporation
Given A trial on boiler equipped with economiser and
ma ( h - h f 1 )
me = superheater:
2257 t = 2 hours ms = 14000 kg
11.7 ¥ (2705.38 - 175.92)
= mf = 1500 kg p = 10 bar
2257
x = 0.96 CV = 33500 kJ/kg
= 13.12 kg/kg of fuel
T1 = 35°C Tf 1 = 90°C
(iii) Draught produced in mm of water
Tsup = 250°C
Theoretical mass of air required for complete
combustion of fuel To find
100 È8 Ê Oˆ ˘ (i) Enathalpy received by feed water in economiser,
mth = Í C + 8 Á H - ˜ + S˙ boiler and superheater,
23 Î 3 Ë 8 ¯ ˚
628 Thermal Engineering

(ii) Percentage of heat absorbed in economiser, boiler Percentage of heat absorbed in the boiler
and superheater, q 2315.48
= 2 ¥ 100 = ¥ 100
(iii) Overall thermal efficiency of plant, qu 2795.91
(iv) Overall equivalent evaporation from and at = 82.82%
100°C. Percentage of heat absorbed in superheater
q 246.05
Analysis The properties of steam: = 3 ¥ 100 = ¥ 100
qu 2795.91
At 10 bar: Tsat = 179.91°C
hf = 762.8 kJ/kg = 8.80%
hfg = 2014.3 kJ/kg (iii) Overall thermal efficiency of the plant
At 35°C: h1 = 146.66 kJ/kg Total heat utilised in boiler plant
hoverall = ¥ 100
At 90°C: hf1 = 381.04 kJ/kg Heat supplied by burniing of fuel
ms qu
At 10 bar and 250°C: = ¥ 100
m f ¥ CV
hsup = 2942.58 kJ/kg
(7000 kg/h) ¥ 2795.91 kJ/kg)
The steam generation per hour; = ¥ 100
(750 kg/h) ¥ (33500 kJ/kg)
ms = ms = 14000 kg = 7000 kg/h = 77.9%
Dt 2 hour
(iv) Overall equivalent evaporation from and at
The coal supplied per hour, 100°C
mf 1500 kg Evaporation rate per kg of fuel,
mf = = = 750 kg/h
Dt 2 hour ms 7000 kg/h
ma = =
(i) Enthalpy received by feed water in economiser, mf 750 kg/h
boiler and superheater: = 9.33 kg/kg of fuel
Heat supplied per kg of water in economiser; ma (hsup - h f 1)
\ me =
q1 = hf 1 − h1 2257
= 381.04 − 146.66 = 234.38 kJ/kg 9.33 ¥ (2942.58 – 381.04)
=
Heat supplied per kg of water in boiler; 2257
q2 = hf + x hfg − hf1 = 10.58 kg/kg of fuel
= 762.8 + 0.96 ¥ 2014.3 − 381.04
= 2315.48 kJ/kg Example 19.21 A trial on a water-tube boiler gave
the following data:
Heat supplied per kg of steam in superheater;
Boiler working pressure = 12 bar
q3 = (1 − x) hfg + Cps (Tsup − Tsat )
Degree of superheat = 77°C
= hsup − (hf + x hfg )
Temperature of feed water = 80°C
= 2942.58 − (762.8 + 0.96 ¥ 2014.3)
Feed water supplied per hour = 4000 kg
= 246.05 kJ/kg
Coal fired per hour = 450 kg
(ii) Percentage of heat absorbed in economiser,
boiler and superheater Calorific value of dry coal = 31000 kJ/kg
Total heat utilised per kg of formation of Ash = 45 kg
superheated steam from water at 35°C Percentage of combustible in ash = 10%
qu = q1 + q2 + q3 Calorific value of combustible in ash = 30000 kJ/kg
= 234.38 + 2315.48 + 246.05 Moisture in coal = 4.5%
= 2795.91 kJ/kg Calculate
Percentage of heat absorbed in the economiser (a) Efficiency of the boiler plant including the
q 234.38 superheater,
= 1 ¥ 100 = ¥ 100
qu 2795.91 (b) Efficiency of boiler and furnace combined.
= 8.38% Take specific heat of superheated steam as 2.1 kJ/kg ◊ K.
Boiler Draught and Performance 629

Solution Actual heat produced per hour,


Qact = 429.75 ¥ 31000 − 4.5 ¥ 30000
Given A trial on a boiler equipped with a superheater:
= 13187250 kJ/kg
ms = 4000 kg/h mf = 450 kg/h Combined efficiency of boiler and furnace
CV1 = 31000 kJ/kg CV2 = 30000 kJ/kg Qu 10446600
hcombined = ¥ 100 = ¥ 100
p = 12 bar Tsup − Tsat = 77°C Qact 13187250
Ash = 45 kg Tf 1 = 80°C = 79.21%
moisture in coal = 4.5%
Combustible in ash = 10% Example 19.22 A boiler is supplied with 15,900 kg of
Ash = 45 kg Cps = 2.1 kJ/kg ◊ K feed water at 15°C, during a trial period of 8 hours and
20 minutes. At the end of the trial, the weight of water
To find was 680 kg less than that of commencement of trial. The
(i) Efficiency of the boiler plant including the super- steam produced at the pressure of 12.5 bar is 95% dry.
heater, and The coal having the calorific value of 30,000 kJ/kg
(ii) Efficiency of boiler and furnace combined. is fired at the rate of 275.8 kg/h. Calculate (a) actual
Analysis The properties of steam: evaporation rate, (b) equivalent evaporation, and (c)
thermal efficiency of the boiler.
At 12 bar Tsat = 188°C
hg = 2784.83 kJ/kg Solution
At 80°C hf 1 = 334.88 kJ/kg
Given A trial on a boiler
(i) Efficiency of the boiler plant including superheat-
er mwater = 15,900 kg Tf 1 = 15°C
Specific enthalpy of superheated steam; t = 8 h, 20 min = 500 min
h = hsup = hg + Cps (Tsup − Tsat) mmake up = 680 kg m f = 275.8 kg/h
= 2784.83 + 2.1 ¥ 77 CV = 30,000 kJ/kg
x = 0.95 p = 12.5 bar
= 2946.53 kJ/kg ◊ K
Heat utilised to produce steam; To find
Qu = ms (hsup − hf1) (i) Actual evaproation,
= 4000 ¥ (2946.53 − 334.88) (ii) Equivalent evaporation, and
= 10446600 kJ/h (iii) Boiler efficiency.
Mass of dry coal;
4.5 Analysis The total water evaporated during the trial
mdf = m f - ¥ mf ms = 680 kg + 15,900 kg = 16,580 kg
100
4.5 (i) Actual evaporation rate
= 450 - ¥ 450 = 429.75 kg/h
100 16580 kg
ms =
Heat produced by burning of coal; 500 min
Qs = mdf CV1 = 429.75 ¥ 31000 = 33.16 kg/min or 1989.6 kg/h
= 13322250 kJ/h Actual evaporation in terms of evaporation per kg
Boiler efficiency; coal burnt
Q 10446600 1989.6 kg/h
hboiler = u ¥ 100 = ¥ 100 ma = = 7.214 kg/kg of coal
Qs 13322250 275.8 kg/h
From steam table
= 78.41% At 12.5 bar
(ii) Efficiency of boiler and furnace combined hf = 806.9 kJ/kg,
Combustible matter in ash per hour hfg = 1977.5 kJ/kg
10 At 15°C hf 1 = 62.98 kJ/kg
= 45 ¥ = 4.5 kg
100
630 Thermal Engineering

Enthalpy of steam produced; Analysis The properties of steam:


h = hf + x hfg From steam table;
= 806.9 + 0.95 ¥ 1977.5 At 12.5 bar hf = 806.9 kJ/kg
= 2685.52 kJ/kg hfg = 1977.5 kJ/kg
The amount of heat supplied to steam per kg At 105°C hf1 = 440.13 kJ/kg
h − hf 1 = 2685.52 − 62.98 (i) The heat rate of the boiler
= 2622.54 kJ/kg Qu = Heat utilised by water during
(ii) Equivalent evaporation formation of steam
ma ( h - h f 1) Specific enthalpy of steam;
me =
2257 h = hf + x hfg
7.214 ¥ 2622.54 kJ/kg = 806.9 + 0.97 ¥ 1977.5
=
2257 = 2725.07 kJ/kg
= 8.38 kg/kg of fuel Enthalpy increase of steam in the boiler;
(iii) Boiler efficiency = h − hf 1 = 2725.07 − 440.13
Heat utilised by water during formation of steam = 2284.94 kJ/kg
Qu = ms (h – hf 1) Heat rate of the boiler;
= (1989.6 kg/h) ¥ (2622.54 kJ/kg) Qu = ms (h − hf 1)
= 52.178 ¥ 105 kJ/h = 18000 ¥ 2284.94
Heat supplied by fuel; = 411.289 ¥ 105 kJ/h
Qs = m f ¥ CV = 275.8 ¥ 30,000 (ii) Equivalent evapoartion
= 82.5 ¥ 105 kJ/h Evapoartion rate per kg of fuel;
Boiler efficiency m 18000
ma = s = = 8.823 kg/kg of fuel
Heat utilised mf 2040
hboiler = ¥ 100
Heat supplied ma ( h - h f 1)
5
me =
52.178 ¥ 10 kg/h 2257
= ¥ 100 = 63.24%
5
82.5 ¥ 10 kg/h 8.823 ¥ 2284.94
=
2257
Example 19.23 A steam generator evaporates 18000 = 8.932 kg/kg of fuel
kg/h of steam at 12.5 bar and a quality of 0.97 dry from
(iii) Thermal efficiency
feed water at 105°C, when coal is fired at 2040 kg/h. If
Heat supplied by fuel;
the higher calorific value of coal is 27400 kJ/kg, find the
Qs = m f ¥ CV = 2040 ¥ 27400
(a) heat rate of the boiler in kJ/h,
= 558.96 ¥ 105 kJ/h
(b) equivalent evaporation, and
Boiler efficiency
(c) thermal effciency.
Heat utilised
hboiler = ¥ 100
Solution Heat supplied
411.289 ¥ 105 kg/h
Given A steam generator; = ¥ 100 = 73.61%
558.96 ¥ 105 kg/h
ms = 18000 kg/h p = 12.5 bar
x = 0.97 Tf1 = 105°C
m f = 2040 kg/h CV = 27400 kJ/kg
To find
(i) The heat rate of the boiler, In a boiler, the heat is produced by burning of fuel
(ii) Equivalent evaporation, and in the presence of atmospheric air. A part of this
(iii) Thermal efficiency of boiler. heat is used to generate the steam and the remaining
Boiler Draught and Performance 631

portion is lost. The energy diagram for a boiler may CO = Volumetric percent of CO in flue gases
be given as follows: The mass of dry gases formed per kg of fuel,
mg = Mass of air supplied + Mass of
Carbon present in the fuel
= ma1 + mC (kg/kg of fuel)

q2 = mg Cpg (Tg − Ta ) (kJ/kg of fuel) ...(19.43)


where, Cpg = Specific heat of combustion gases
Tg = Temperature of hot flue gases
We express each quantity per kg of fuel burnt in Ta = Atmospheric temperature
the following manner. (c) Heat Carried by Moisture per kg of Fuel
Heat generated by combustion of 1 kg (or 1 m3
The moisture formed per kg of fuel
for gaseous) fuel in the furnace:
= 9 H (kg/kg of fuel)
qin = CV
If some moisture is present in the fuel, total
where CV = Lower calorific value of fuel, kJ/kg.
mass of moisture;
(a) Heat Absorbed by Water During its Heating and mv = fraction mass of moisture in fuel + mass of
Evaporation vapour formed
The mass of steam generation per kg of fuel Heat carried by moisture
Mass of steam generated per hour q3 = mv [hg + Cps (Tg − Tsat) − Cpw Ta]
ma =
Mass of fuel fired per houur (kJ/kg of fuel) ...(19.44)
(kg of steam per kg of fuel) where,
q1 = ma (h − hf 1) (kJ/kg of fuel) ...(19.41) hg = Total specific enthalpy of steam at partial
where ma = Mass of steam generated per kg of pressure of steam in flue gases
fuel Cps = Specific heat of superheated steam ª 2.1
h = Specific enthalpy of steam, kJ/kg kJ/kg ◊ K
hf1 = Initial specific enthalpy of feed Cpw = Specific heat of water ª 4.2 kJ/kg ◊ K
water, kJ/kg Ta = Temperature of boiler house
(b) Heat Carried by Dry Flue Gases Tg = Temperature of hot flue gases
Tsat = Saturation temperature at partial pressure
The flue gases carry the dry products of combus-
of steam in flue gases
tion as well as vapour generated due to combustion
of hydrogen in fuel. (d) Heat Lost Due to Incomplete Combustion
Actual air supplied per kg of fuel burnt, If carbon present in fuel is burnt to CO instead of
CO2, the combustion is said to be incomplete.
N2 ¥ C
ma1 = (kg/kg of fuel) 1 kg of carbon releases 33800 kJ of heat, if it
33 ¥ (CO + CO 2 )
burns to CO2, and
...(19.42)
1 kg of carbon releases 10120 kJ of heat, if it
where, burns to CO.
C = Mass fraction of carbon in fuel heat lost due to incomplete combustion
N2 = Volumetric percent of N2 in flue gases = 33800 − 10120 = 23680 kJ/kg of coal.
CO2 = Volumetric percent of CO2 in flue gases
632 Thermal Engineering

If the percentages of CO and CO2 in the gases by Example 19.24 The following data were obtained
volume are known, the mass of carbon burnt to CO during a boiler trial:
CO ¥ C Mass of steam = 700 kg/h
=
CO 2 + CO Temperature of feed water = 60°C
and heat lost due to incomplete combustion Steam pressure = 10 bar
CO ¥ C Oil consumption = 55 kg/h
q4 = ¥ 23680 (kJ/kg of fuel) CV of oil = 44000 kJ/kg
CO 2 + CO
...(19.45) Dryness fraction of steam = 0.98
The heat lost due to incomplete combustion can Percentage composition of oil by mass:
be reduced by supplying excess quantity of air in C = 85%, H2 = 14%, Ash = 1%
turbulent motion. Analysis of dry flue gases by volume:
CO2 = 12.5%, O2 = 4.5%, N2 = 83%
(e) Heat Lost Due to Unburnt Fuel
Temp. of flue gases leaving the boiler = 350°C
If muf is the mass of unburnt fuel per kg of fuel Boiler room temperature = 25°C
then Specific heat of flue gases = 1.02 kJ/kg ◊ K
q5 = muf ¥ CV ...(19.46) Partial pressure of steam = 0.08 bar
where Heating surface area = 21.4 m2
muf = Mass of unburnt fuel per kg of fuel, Find
CV = Calorific value. (a) equivalent evaporation per kg of fuel from and at
100°C,
(f ) Unaccounted Heat Losses
(b) equivalent evaporation per sq. m of heating area,
The boiler furnace and hot surfaces are exposed (c) thermal efficiency of the boiler,
to atmosphere, therefore, some heat is radiated and (d) heat balance sheet on the basis of 1 kg of fuel and
convected to the surroundings. on the percentage basis.
Unaccounted heat losses
Solution
= qin − (q1 + q2 + q3 + q4 + q5) kJ/kg of fuel
...(19.47) Given A trial on a boiler
Mass of steam, ms = 700 kg/h
Heat Balance Sheet
Temp. of feed water, Tf 1 = 60°C
Heat % Heat utilisation kJ/kg of fuel % Steam pressure = 10 bar
supplied/
kg of fuel
Oil consumption, m f = 55 kg/h
CV of oil = 44000 kJ/kg
qin = CV 100 (a) Heat used to q1 = ma ¥
generate steam (h − hf 1) x = 0.98
(b) Heat carried q2 = mg Cpg ¥ Percentage composition of oil by mass:
away by dry (Tg − Ta) C = 85% H2 = 14% Ash = 1%
flue gases
(c) Heat carried Analysis of dry flue gases by volume:
q3 = mv ¥
away by steam CO2 = 12.5% O2 = 4.5% N2 = 83%
in dry flue gases (hsup − CpwTa)
Tg = 350°C
(d) Heat CO ¥ C
lost due to q4 = Boiler room temperature,
CO 2 + CO
incomplete ¥ 23680 Ta = 25°C
combustion
Cpg = 1.02 kJ/kg ◊ K
(d) Heat
q5 = muf ¥ CV Partial pressure of steam = 0.08 bar
lost due to
incomplete Heating surface area = 21.4 m2
unburnt fuel
(e) Unaccounted q6 = qin − q1 − To find
heat losses q2 − …
(i) Equivalent evaporation per kg of fuel from and at
100%
100°C,
Boiler Draught and Performance 633

(ii) Equivalent evaporation per sq. m of heating area, (iii) Thermal efficiency
(iii) Thermal efficiency of the boiler, and Heat utilised in boiler,
(iv) Heat balance sheet on the basis of 1 kg of fuel and Qu = ms (h − hf 1)
on percentage basis. = (700 kg/h) ¥ (2486.66 kJ/kg)
Assumption The specific heat of superheated steam, = 1740662 kJ/h
Cps = 2.1 kJ/kg ◊ K Heat supplied by fuel,
Analysis The properties of steam: Qs = m f CV
At 10 bar (1000 kPa) pressure = (55 kg/h) ¥ (44000 kJ/kg)
hf = 762.79 kJ/kg = 2420000 kJ/h
hfg = 2015.29 kJ/kg Efficiency,
At 60°C Qu
hboiler = ¥ 100
hf 1 = 251.11 kJ/kg Qs
The specific enthalpy of steam,
(1740662 kJ/h)
h = hf + x hfg = ¥ 100 = 71.93%
(2420000 kJ/h)
= 762.79 + 0.98 ¥ 2015.29
= 2737.77 kJ/kg (iv) Heat balance sheet
Heat supplied to steam in the boiler (a) Heat supplied by 1 kg of fuel,
= h − hf1 = 2737.77 − 251.11 qin = CV = 44000 kJ/kg
(b) Heat absorbed by water per kg of fuel during
= 2486.66 kJ/kg
its heating and evaporation,
(i) Equivalent evaporation per kg of fuel from and at
q1 = ma (h − hf 1)
100°C
= (12.72 kg/kg of fuel)
Mass of steam generated per kg of fuel
¥ (2486.66 kJ/kg)
700 kg/h = 31630.31 kJ/kg of fuel
ma = = 12.72 kg/kg of fuel
55 kg/h (c) Actual air required per kg of fuel burnt,
The equivalent evaporation, N¥C 83 ¥ 85
ma ( h - h f 1) ma = =
33 ¥ (CO + CO 2 ) 33 ¥ (0 + 12.5)
me =
2257 = 17.1 kg/kg of fuel
(12.72 kg/kg of fuel) ¥ (2486.66 kJ/kg) Since 0.85 kg carbon is also present per kg of
=
(2257 kJ/kg) fuel, thus the mass of dry gases formed per kg
= 14 kg of steam/per kg of fuel of fuel,
(ii) Equivalent evaporation per sq.m of heating area mg = ma + mC
Mass of steam generated per sq.m of heating = 17.1 + 0.85 = 17.95 kg/kg of fuel
surface (d) Heat carried by dry flue gases,
700 kg/h q2 = mg Cpg (Tg − Ta)
ma =
21.4 m 2 = 17.95 ¥ 1.02 ¥ (350 − 25)
= 32.71 kg/m2 of heaing area/h = 5944.8 kJ/kg of fuel
Equivalente vaporation, (e) Heat carried by moisture per kg of fuel
ma ( h - h f 1) The moisture formed per kg of fuel,
me =
2257 mv = 9 H = 9 ¥ 0.14 = 1.26 kg/kg of fuel
(32.71 kg/m 2 grate area) ¥ (2486.66 kJ/kg) Properties of steam at 0.08 bar (8 kPa)
= hg = 2577 kJ/kg
2257 kJ/kg
Tsat = 41.54°C
= 36 kg of steam/m2 of heating area/h
634 Thermal Engineering

Heat carried by moisture (b) Make complete heat balance sheet on the basis of
q3 = mv [hg + Cps (Tg − Tsat) − Cpw Ta] 1 kg of coal and on percentage basis.
= 1.26 ¥ [2577 + 2.1 ¥ (350
− 41.54) − 4.2 ¥ 25] Solution
= 3931 kJ/kg of fuel Given A trial on a boiler
Heat lost by radiation and convection, etc. Mass of steam, ms = 14000 kg/h
q4 = qin − q1 − q2 − q3 Temp. of feed water, Tf 1 = 55°C
= 44000 − 31630.31 − 5944.8 − 3931 Steam pressure = 128 N/cm2
= 2493.9 kJ/kg of fuel Tsup = 250°C
Heat Balance Sheet Coal consumption, m f = 1600 kg/h
CV of coal = 35000 kJ/kg
Heat % Heat utilisation kJ/kg of fuel % Analysis of dry flue gases by volume
supplied/
CO2 = 9.4% O2 = 1.1%
kg of fuel
N2 = 79.5% Tg = 310°C
qin = CV 100 (a) Heat used to q1 = 31648.4 71.92%
Boiler room temperature, Ta = 25°C
= 44000 generate steam
Cpg = 1.02 kJ/kg ◊ K
kJ/kg (b) Heat carried q2 = 5944.8 13.51% Partial pressure of steam = 0.08 bar
away by dry flue
Percentage composition of coal by mass
gases
C = 85.2% H2 = 4.8%
(c) Heat carried q3 = 3931 8.93%
away by steam
Ash = 10% Moisture = 1.8%
in dry flue gases q6 = 2493.9 To find
5.66%
(d) Unaccounted (i) Heat carried away by excess air in kJ/kg of fuel,
heat losses (ii) Heat balance sheet on the basis of 1 kg of coal
100% and on percentage basis.
Assumption The specific heat of superheated steam,
Example 19.25 The following data were recorded Cps = 2.1 kJ/kg ◊ K.
during a boiler trial:
Boiler room temperature = 25°C Analysis
Temperature of feed water = 55°C (i) Heat carried away by excess air
Mass of steam = 14000 kg/h Flue gases produced per kg of fuel burnt,
Steam pressure = 128 N/cm2 N¥C 79.5 ¥ 85.2
ma1 = =
Steam temperature = 250°C 33 ¥ (CO + CO 2 ) 33 ¥ (0 + 9.4)
Coal consumption = 1600 kg/h
= 21.83 kg/kg of fuel
CV of coal = 35000 kJ/kg
The theoretical amount of air required for
Analysis of flue gases by volume: complete combustion of 1 kg of fuel
CO2 = 9.4% O2 = 1.1% N2 = 79.5%
100 È8 Ê Oˆ ˘
Percentage composition of coal by mass: mth = Í C + 8 Á H - ˜ + S˙ kg/kg of fuel
23 Î3 Ë 8¯ ˚
C = 85.2% H2 = 4.8% Ash = 10%
Moisture = 1.8% 100 È 8 ˘
= ¥ Í ¥ 0.852 + 8 ¥ (0.048) + 0 ˙
Temp. of flue gases leaving the boiler = 310°C 23 Î 3 ˚
Partial pressure of steam = 0.08 bar = 11.55 kg/kg of fuel
Specific heat of air and dry flue gases = 1.02 kJ/kg ◊ K The mass of excess air supplied per kg of fuel
Calculate mex = 21.83 − 11.55
(a) Heat carried by excess air, = 10.28 kg/kg of fuel
Boiler Draught and Performance 635

Heat carried by excess air (d) Heat carried by moisture per kg of fuel,
= mex Cpg ¥ (Tg – Ta) Properties of steam at 0.08 bar (8 kPa)
= 10.28 ¥ 1.02 ¥ (310 – 25) hg = 2577 kJ/kg
= 2988.4 kJ/kg of fuel Tsat = 41.54°C
The properties of steam Heat carried by moisture
At 128 N/cm2 (1280 kPa) hfg = 2785 kJ/kg q3 = mv [hg + Cps (Tg − Tsat) − Cpw Ta]
Tsat = 190.5°C = 0.45 ¥ [2577 + 2.1 ¥
At 55°C hf 1 = 230.3 kJ/kg (310 − 41.54) − 4.2 ¥ 25]
The specific enthalpy of steam, = 1366.1 kJ/kg
h = hg + Cps (Tsup − Tsat) Heat lost by radiation and convection, etc.
= 2785 + 2.1 ¥ (250 − 190.5) q4 = qin − q1 − q2 − q3
= 35000 − 23446.9 − 6476.8 − 1366.1
= 2909.95 kJ/kg
= 3710.2 kJ/kg of fuel
Heat supplied to steam per kg
= h − hf 1 = 2905.95 − 230.3 Heat Balance Sheet
= 2679.65 kJ/kg Heat % Heat utilisation kJ/kg of fuel %
Mass of steam generated per kg of fuel supplied/
14000 kg/h kg of fuel
ma = = 8.75 kg/kg of fuel
1600 kg/h qin = CV 100 (a) Heat used to q1 = 23446.9 67.00%
= 35000 generate steam
(ii) Heat balance sheet (b) Heat carried
kJ/kg q2 = 6476.8 18.50%
(a) Heat supplied by 1 kg of fuel, away by dry flue
gases
qin = CV = 35000 kJ/kg
(c) Heat carried q3 = 1366.1 3.90%
(b) Heat absorbed by water per kg of fuel during away by steam
its heating and evaporation, in dry flue gases
(d) Unaccounted q6 = 3710.2 10.60%
q1 = ma (h − hf 1)
heat losses
= 8.75 ¥ 2679.65 kJ/kg
100%
= 23446.9 kg/kg of fuel
(c) The mass of products formed per kg of fuel,
Example 19.26 A boiler is fired with an oil having
mp = Mass of air + Mass of combustible
percentage analysis by mass: C = 84% and H2 = 16%.
matter in the fuel
The air initially at 15°C is first passed through the flue
= ma1 + mC = 21.83 + 0.9 gas air preheater and then supplied to the boiler furnace
= 22.73 kg/kg of fuel at 260°C. The air supply is 25% in excess of theoretical
(10% ash in fuel) air. Calculate the temperature of flue gas as it enters the
The moisture formed per kg of fuel, heater, if the flue gas is leaving the heater at 137°C.
= 9 H = 9 ¥ 0.048 Assume Cp for air and flue gas as 1.003 kJ/kg ◊ K and
= 0.432 kg/kg of fuel for superheated steam as 2.1 kJ/kg ◊ K
Fuel contains 0.018 kg moisture per kg of
fuel, total mass of moisture Solution
mv = 0.432 + 0.018 = 0.45 kg/kg of fuel Given Oil composition by mass
Mass of dry flue gases C = 84% H2 = 16%
mg = mp − mv = 20.73 − 0.45 Ta1 = 15°C Ta2 = 260°C
= 22.28 kg/kg of fuel Tg2 = 137°C mact = 1.25 mth
Heat carried by dry flue gases, Cpg = 1.003 kJ/kg ◊ K Cps = 2.1 kJ/kg ◊ K
q2 = mg Cpg (Tg − Ta) To find Temperature of flue gas inlet to air preheater.
= 22.28 ¥ 1.02 ¥ (310 − 25) Analysis Theoretical air required for complete
= 6476.8 kJ/kg of fuel combustion
636 Thermal Engineering

100È8 ˘ (a) Calculate the air leakage into the economiser


mth = Í 3 C + 8H ˙ per minute, if the fuel is used in the boiler is at
23Î ˚
50 kg/min.
100 È 8 ˘
= ¥ 0.84 + 8 ¥ 0.16 ˙ (b) Also, find the reduction in temperature of flue
23 ÍÎ 3 ˚ gas due to air leakage if the atmospheric air
= 15.3 kg/kg of oil temperature is 15°C and flue-gas temperature
Actual air supplied is 350°C.
ma = mth + 0.25 mth = 1.25 ¥ 15.3 Assume Cp for air as 1 kJ/kg ◊ K and flue gas as 1.1
= 19.13 kg/kg of oil kJ/kg ◊ K.
Mass of products formed
Solution
mp = ma + 1 = 19.13 + 1 = 20.13 kg/kg of oil
Mass of vapour formed Given
mv = 9H = 9 ¥ 0.16 = 1.4 kg/kg of oil Fuel composition by mass: C = 80%
Mass of dry flue gases Incombustibles = 15%
mg = mp − mv = 20.13 − 1.44 Ta1 = 15°C Tg1 = 350°C
= 18.69 kg/kg of oil m f = 50 kg/min
Cpa = 1.0 kJ/kg ◊ K
Cpg = 1.1 kJ/kg ◊ K
Volumetric composition of flue gas before and after
economiser.
Fig. 19.9
To find
Energy balance on air preheater (i) Mass of air leakage into the economiser per
Heat gain by air minute, and
= Heat given by dry flue gases (ii) Reduction in temperature of flue gas due to air
+ heat given by water vapour leakage.
maCpa (Ta2 − Ta1)
= mg Cpg (Tg1 − Tg2) + mv Cps (Tg1 − Tg2)
19.13 ¥ 1.003 ¥ (260 − 15)
= 18.69 ¥ 1.003 ¥ (Tg1 − 137) + 1.44 ¥ 2.1
¥ (Tg1 − 137)
or 21.77 ¥ (Tg1 − 137) = 4700.91
Temperature of flue gas;
Tg1 = 352.93°C Fig. 19.10
Example 19.27 During a trial on a boiler with Analysis
economiser, the following results were obtained:
(i) Mass of air per kg of fuel entering economiser
CO2 O2 N2 N¥C 80.3 ¥ 80
ma1 = =
% volumetric analysis of flue 8.3 11.4 80.3 33 ¥ (CO + CO 2 ) 33 ¥ (0 + 8.3)
gas entering economiser = 23.45 kg/kg of fuel
% volumetric analysis of flue Mass of air per kg of fuel leaving economiser;
gas leaving economiser 7.9 11.5 80.6 N¥C 80.6 ¥ 80
ma2 = =
The fuel contains 80% carbon and it does not contain 33 ¥ (CO + CO 2 ) 33 ¥ (0 + 7.93)
N2. But the fuel contains 15% incombustible matter. = 24.73 kg/kg of fuel
Boiler Draught and Performance 637

Mass of air leakage into economiser; 2. Feed-Water Preheating using an Economiser


= ma2 − ma1 = 24.73 − 23.45 The flue gases leaving a boiler are at temperatures
= 1.28 kg/kg of fuel of 200 to 300°C. Thus, there is a potential to
The air leaks into economiser per minute recover some of heat from these gases. The flue-
= (50 kg fuel/min) ¥ (1.28 kg of air/min) gas exit temperature from a boiler is usually kept
= 64 kg/min at 200°C, so that the sulphur oxides in the flue
(ii) Reduction in temperature gas do not condense and cause corrosion in heat
At enterance to the economiser, the mass of flue transfer surfaces. When a clean fuel such as natural
gas products formed per kg of fuel; gas, LPG or gas oil is used, the economy of heat
mg = Mass of combustible + Mass of air recovery must be worked out, as the flue gas
= (1 – 0.15) + 23.45 temperature may be well below 200°C. Thermal
= 24.3 kg/kg of fuel efficiency can be increased up to 5%.
Energy balance on economiser;
mg Cpg Tg + ma Cpa Ta = (ma + ma) Cpg Tm 3. Combustion Air Preheat
where Tm is the temperature of mixture of exhaust The preheating of combustion air also saves the
gases leaving the economiser. fuel consumption. It improves thermal efficiency
With numerical values; by 1%, if the combustion air temperature is raised
24.3 ¥ 1.1 ¥ 350 + 1.28 ¥ 1 ¥ 15 by 20°C.
= (24.3 + 1.28) ¥ 1.1 ¥ Tm
or 28.138 Tm = 9355.5 + 19.2 = 9374.7 4. Incomplete Combustion
or Tm = 333.17°C Incomplete combustion is result of a shortage of air
Reduction in temperature of flue gas due to air or a surplus of fuel or poor distribution of fuel. It
leakage;
is from the colour or smoke, and must be corrected
= Tg − Tm = 350 − 333.17 = 16.83°C immediately.
In case of oil- and gas-fired systems, formation
CO or smoke with normal or high excess air
19.6
indicates burner system problems. A frequent cause
OPPORTUNITIES
of incomplete combustion is the poor mixing of
The various energy-saving opportunities in a fuel and air at the burner. With coal firing, unburned
boiler system can be related to combustion, heat carbon can result into a big loss. Non-uniform fuel
transfer, avoidable losses, high auxiliary power size could be one of the reasons for incomplete
consumption, water quality and blowdown. These combustion.
opportunities are discussed below.
5. Excess Air Control
1. Stack (Chimney) Temperature
Excess air is required to ensure complete combus-
The flue gases temperature should be as low as tion. The optimum excess air level for maximum
possible. However, it should not be reduced to boiler efficiency occurs when the sum of the losses
such value at which water vapour in the exhaust due to incomplete combustion and loss due to heat
condenses on the chimney walls. The fuels in flue gases is minimum. This level varies with fur-
containing significant sulphur at low temperature nace design, type of burner, fuel and process vari-
can lead to sulphur dew-point corrosion. Flue-gas ables. It can be determined by conducting tests with
temperatures above 200°C indicate potential for different air–fuel ratios.
waste heat recovery.
638 Thermal Engineering

6. Radiation and Convection Heat Loss achieving energy savings. Generally, combustion
The external surfaces of a boiler shell are always air control is effected by throttling dampers fitted
hotter than the surroundings. The surfaces thus lose at forced and induced draft fans. Though dampers
heat to the surroundings depending on the surface are simple means of control, they lack accuracy,
area and the difference in temperature between the giving poor control characteristics at the top and
surface and the surroundings. bottom of the operating range. In general, if the
The heat loss from the boiler shell is normally a load characteristic of the boiler is variable, there is
fixed energy loss, irrespective of the boiler output. a possibility of replacing the dampers by a variable
With modern boiler designs, this may represent speed drive fan and blowers.
only 1.5% on the gross calorific value at full rating,
but will increase to around 6%, if the boiler operates
at only 25 per cent of capacity. The maximum efficiency of the boiler does not
occur at full load, but at about two-thirds of the
7. Automatic Blowdown Control full load. If the load on the boiler decreases further,
Uncontrolled continuous blowdown results into efficiency also tends to decrease. At zero output,
wastage of useful heat energy. Automatic blow- the efficiency of the boiler is zero, and any fuel
down controls can be installed that sense and re- fired is used only to supply the losses.
spond to boiler water conductivity and pH value. A
11. Proper Boiler Scheduling
10% blowdown in a 15 kg/cm2 boiler results in 3%
efficiency loss. Since the optimum efficiency of boilers occurs at
65–85% of full load. Therefore, a fewer number of
8. Reduction of Scaling and Soot Losses boilers at higher loads, should be operated than to
In oil and coal-fired boilers, soot buildup on tubes operate a large numbers at low loads.
acts as resistance to heat transfer. Any such deposits
12. Boiler Replacement
should be removed on a regular basis. Rise in stack
temperatures indicates excessive soot build-up. The potential savings from replacing a boiler depend
Also, same result will occur due to scaling on the on the expected increase in overall efficiency. A
water side. High exit gas temperatures at normal change in a boiler can be financially attractive, if
excess air indicate poor heat transfer performance. the existing boiler is
This condition can result from a gradual build-up of (i) old and inefficient,
gas-side or waterside deposits. Waterside deposits
(ii) not capable of firing cheaper substitution
require a review of water-treatment procedures and
fuel,
tube cleaning to remove deposits. An estimated 1%
(iii) over or under-sized for present requirements,
efficiency loss occurs with every 22°C increase in
(iv) not designed for ideal loading conditions.
stack temperature.
9. Variable Speed Control for Fans, Blowers and
Pumps
Variable speed control is an important means of
Boiler Draught and Performance 639

Summary
near or at the base of the boiler grate to force
pressure difference which causes the flow of atmospheric air on to the furnace under pressure.
gases inside the boiler.
The draught obtained by use of a chimney is boiler is known as balanced draught.
called natural or chimney draught. But the static is defined as the amount
draught produced by the chimney is not sufficient of dry and saturated steam generated from feed
to meet the requirement of the draught of a boiler. water at 100°C at a normal atmospheric pressure.
Most commonly, the artificial draught is used on It is expressed as
the boilers. The waste heat carried by the flue ma ( h - h f1 ) ma ( h - h f1 )
gases can be better utilized in an economiser, air me = = (kg/kg of fuel)
h fg @100∞C 2257
precheater, etc, with the help of artificial draught.
artificial draught produced by a fan or a boiler efficiency is defined as the ratio of
blower is known as mechanical draught and that actual heat utilized in producing steam to amount
produced by a steam jet is known as a steam jet of heat liberated by burning of fuel.
draught. In the induced draught system, the fan Enthalpy rise of water in the boiler
is placed near the base of the chimney. While in hboiler =
Heat librated by burninng of fuel
the forced draught, the fan or blower is located

Glossary
Boiler Draught A small pressure difference between Factor of evaporation Ratio of heat received by 1 kg
air outside the boiler and gases within the furnace or of feed water for evaporation under actual working
chimney condition to that received by 1 kg of water evaporated
Natural draught The draught obtained by use of a from and at 100°C
chimney Boiler Efficiency Fraction of heat utilized in producing
Artificial Draught The draught produced artificially by steam of heat librated by burning of fuel
a fan or a blower Economiser efficiency Fraction of heat absorbed by
Steam jet draught Draught produced by steam jet the feed water in the economiser of heat available of flue
Evaporation rate Mass of steam generated per hour gases
Equivalent evaporation from
Equivalent evaporation Amount of steam generated and at 100∞C per hour
from and at 100°C. Boiler Power: =
21.296

Review Questions
1. What is boiler draught and how it is produced by 5. Derive an expression for maximum discharge
a chimney? through a chimney.
2. Compare natural and artificial draughts. 6. State briefly the difference between systems of
3. Explain a steam jet draught with its features. producing draught in a boiler.
4. Compare force and induced draught. 7. Derive an expression for draught produced in
640 Thermal Engineering

terms of height of chimney, ambient and flue 10. State the advantages of artifical draught over nat-
gas temperatures. State clearly the assumptions ural draught.
made. 11. Define evaporative capacity and equivalent evap-
8. Establish a condition for maximum discharge of oration of the boiler.
flue gases through a chimney of given height. 12. What is the factor of evaporation?
9. Define equivalent evaporation, factor of evapora- 13. Define boiler efficiency.
tion, boiler efficiency. 14. Establish the energy balance in a boiler. How can
its performance be improved?

Problems
1. A chimney of 30-m height is discharging hot gas- by a chimney of 30-m height, when air–fuel ratio
es at 320°C, when outside temperature is 30°C. used in the combustion is 20. The atmospheric air
The air–fuel ratio is 20. Determine the temperature is 27°C. [274.75°C]
(a) draught produced in mm of water column, 5. Find the draught produced in mm of water by a
(b) temperature of gases for maximum dis- chimney of 40-m height. The mass of the flue
charge in a given time and what would be gases is 20 kg/kg of fuel burnt in the combustion
the draught produced corresponding? chamber. The temperature of flue gases and ambi-
[(a) 16.2 mm (b) 362.3°C] ent are 270°C and 23°C, respectively.
2. How much air is used per kg of coal in a boil- Assuming diameter of the chimney is 150 cm
er having a chimney of 30-m height to create a and 30% of the theoretical draught is lost due to
draught of 20 mm of water, when temperature friction, find the mass of flue gases discharged
of flue gases in the chimney is 350°C and tem- through the chimney per minute.
perature of the boiler house is 35°C? Does the [20.33 mm, and 1464.65 kg/min]
chimney satisfy the condition of maximum dis- 6. A boiler plant generates one tonne of steam per
charge? [18.3 kg, No] hour at a pressure of 12 bar and 50°C of super-
3. A 30-m high chimney is discharging flue gases at heat, the boiler details are:
561 K, when the ambient temperature is 294 K. Feed water temperature 50°C,
The quantity of air supplied is 18 kg of air per kg Grate area 0.6 m2,
of fuel burnt. Determine Heating surface = 12 m2.
(a) draught in mm of water column produced Determine actual and equivalent evaporation
by chimney. (a) per hour, (b) per m2 of grate area per hour, and
(b) equivalent draught in metre of hot gas (c) per m2 of heating surfcace area per hour.
column. 7. A boiler generates steam at a rate of 40 tonnes/h
(c) velocity of flue gases in chimney, if 50% consuming coal at 4000 kg/h of calorific value
of draught is lost in friction at the grate and 35,000 kJ/kg. The steam pressure is 15 bar and
passages. dry saturated. Calculate (a) factor of evaporation,
(d) draught produced in mm of water and tem- (b) equivalent evaporation from and at 100°C,
perature of flue gases under the condition of and (c) efficiency of the boiler.
maximum discharge. [(a) 1.237, (b) 12.37 kg/kg of fuel,
[(a) 16.1 mm, (b) 24.3 m, (c) 15.45 m/s, (c) 79.77%]
(d) 18 mm and 620K] 8. A boiler generates steam at 18 bar and 325°C
4. Find the minimum temperature of flue gases re- when feed water is supplied at 41.45°C. The
quired to produce a draught of 15 mm of water thermal efficiency of the boiler is 80%. It uses
Boiler Draught and Performance 641

furnace oil of CV 45500 kJ/kg. The steam is sup- Quantity of steam generated per hour = 7000 kg
plied to a turbine producing 500 kW power, hav- Determine thermal efficiency of the plant and
ing a specific steam consumption of 10 kg/kWh. percentage heat utilised in the economiser, boiler
Calculate the furnace-oil consumption in kg/h and superheater.
and equivalent evaporation. [92.84%, 7.16%, 72.85%, 12.84%]
[395.71 kg/h; 6382.55 kg/h] 13. The data were collected during a boiler trial for a
9. A boiler generates 500 kg/h of steam at 16 bar duration of one hour:
and 300°C from feed water at 30°C. Coal used is Pressure of steam = 9.8 bar
60 kg/h of CV 30,000 k/kg. Find Temp. of steam leaving the superheater = 250°C
(a) equivalent of evaporation Feed-water temperature = 25°C
(b) boiler efficiency Calorific value of coal = 30,000 kJ/kg
[(a) 10.75 kg/kg of coal, (b) 80.9%] Quantity of coal burned per hour = 95 kg
10. The equivalent of evaporation of a boiler from Quantity of steam generated per hour = 650 kg
and at 100°C is 10.4 kg/kg of fuel. The CV of Ash collected = 9 kg
fuel is 29800 kJ/kg. CV of ash = 2500 kJ/kg
Determine efficiency of boiler. If the boiler pro- Quantity of air used per kg of fuel = 19 kg
duces 15000 kg of steam per hour at 20 bar from a Flue-gas temperature = 350°C
feed water at 40°C and the fuel used is 1650 kg/h,
Boiler-room temperature = 30°C
determine the condition of steam produced.
Mean sp. heat of flue gases = 1.005 kJ/kg ◊ K
[(a) 78.76% (b) x = 0.9734]
Determine equivalent evaporation, boiler effi-
11. Calculate the thermal efficiency and equivalent
ciency, % heat losses in flue gases, ash and radia-
evaporation from and at 100°C of a boiler from
tion.
which the following data were obtained during a
[8.55 kg/kg of coal, 64.35%, 21.44%,
trial: steam pressure = 10.6 bar, steam tempera-
ture = 260°C, feed-water temperature = 38°C, 0.79%, 13.42%]
water evaporated 10 kg per kg of coal of calorific 14. The observation recorded during a boiler trial are
value = 33450 kJ/kg. as given below:
[62.48%, 9.26 kg/kg of coal] Duration of trial = 60 min
12. The following observations were taken during a CV of coal burnt = 30, 200 kJ/kg
test on a steam boiler: Steam generated = 5250 kg
Pressure of steam = 9.8 bar Coal burnt = 695 kg
Temp. of steam leaving the superheater = 250°C Boiler pressure = 12 bar
Feed-water temp. entering the economiser = Dryness fraction of steam = 0.94
25°C Temp of steam leaving superheater = 250°C
Feed-water temp. leaving the economiser = 80°C Temperature of hot well = 45°C
Dryness fraction of steam entering the superheat- Calculate with and without superheater:
er = 0.95 (i) Equivalent evaporation from and at 100°C
Calorific value of coal = 30,000 kJ/kg (ii) Boiler efficiency
Quantity of coal burned per hour = 750 kg (iii) Heat supplied by superheater
642 Thermal Engineering

Objective Questions
1. The draught in a boiler is provided to Tg Ê H ˆ
(a) = 2Á
(a) force the air on the furnace Ta Ë H + 1˜¯
(b) force the hot gases on superheater
(c) discharge the flue gases through chimney Tg Ê m + 1ˆ
(b) = 2Á a ˜
(d) all of the above Ta Ë ma ¯
2. The boiler draught is defined as Tg Ê ma ˆ
(c) = 2Á
(a) pressure difference between furnace bed Ta Ë ma + 1˜¯
and chimney base
(d) none of the above
(b) pressure difference between feed pump and
7. The chimney efficiency is given by
injector
(c) temperature difference between economiser È m Tg ˘
(a) HÍ a ¥ - 1˙
and air preheater Î ma + 1 Ta ˚
(d) all of the above È1 m +1 1 ˘
3. The natural draught is produced by (b) g H Í - a ¥ ˙
ÍÎ Ta ma Tg ˙˚
(a) fan before the furnace
(b) fan after the fan È m Tg ˘
(c) gH Í a ¥ - 1˙
(c) chimney height m
Î a + 1 Ta ˚
(d) none of the above (d) none of the above
4. The actual velocity of flue gases through the 8. The balanced draught is produced by
chimney is given by
(a) chimney draught + induced fan draught
(a) rgh (b) forced fan draught and induced fan draught
(b) rgh (c) forced fan draught + chimney draught
(d) none of the above
(c) 2gH ¢ 9. The steam jet draught is used in
(a) Cochran boiler
(d) 2g ( H ¢ - h f )
(b) Lancashire boiler
5. The draught produced in terms of hot flue-gas (c) Locomotive boiler
column is given by (d) none of the above
È m Tg ˘ 10. The evaporation rate of a boiler is defined as
(a) HÍ a ¥ - 1˙
m
Î a + 1 Ta ˚ Mass of steam generated
(a)
È ˘ Time period in hours
(b) 353 H Í 1 - ma + 1 ¥ 1 ˙
ÍÎ Ta ma Tg ˙˚ Mass of steam generated per hour
(b)
(c) 2gH ¢ Heating area of grate in m 2
Mass of steam generated per hour
(d) 2g ( H ¢ - h f ) (c)
Mass of fuel burned per houur
6. The condition of maximum discharge through a (d) all of the above
chimney is given by
Boiler Draught and Performance 643

11. The equivalent evaporation is defined as (b) 2257 kJ/kg


(a) steam generated at 100°C (c) 2276 kJ/kg
(b) dry and saturated steam generated at 100°C (d) 2557 kJ/kg
from feed water at 100°C 14. The boiler efficiency is defined as
(c) steam generated at 1 bar and at 100°C Heat utilised in steam generated/h
(d) none of the above (a)
Heat liberated by fuel buurning
12. Which one of following is standard condition for
equivalent evaporation? Heat utilised in steam generated
(b)
(a) Pressure 1 bar and generation at 100°C Heat liberated by fuel burnning/h
(b) From feed water at 100°C and 1 bar Heat utilised in steam generated/h
(c) From and at 100°C and 1 bar (c)
Heat liberated by fuel buurning/h
(d) None of the above
(d) none of above.
13. The enthalpy of evaporation at 100°C
(a) 2527 kJ/kg

14. (c) 13. (b) 12. (c) 11. (b) 10. (d) 9. (c)
8. (b) 7. (d) 6. (b) 5. (a) 4. (d) 3. (c) 2. (a) 1. (d)
Answers
644 Thermal Engineering

20
Steam Engines

Introduction
The steam engine is a slow speed reciprocating
machine, in which the energy of steam is converted
into mechanical work. The steam is used as a
working fluid. Steam is prepared outside the engine
in the boiler, and thus steam engine is also called an
external combustion engine.
Steam engines have played a very important
role in India, being a major factor in the Industrial
revolution and Indian Railways. Nowadays, steam
engines are replaced by internal combustion engines,
steam and gas turbines. In some plants, the steam
engines are used to operate air compressors, pumps,
and other auxiliaries. The exhaust steam from these
engines is used for space heating and industrial
process work.
Figure 20.1 shows the sectional view of steam-
Fig. 20.1
engine.

2. According to action of operation


20.1 CLASSIFICATION OF STEAM
ENGINES (a) Single acting engine
(b) Double acting engine
Steam engines can be classified in the following 3. According to expansion stages
manner: (a) Single cylinder engine
1. According to position of cylinder axis (b) Compound steam engine
(a) Vertical engine 4. According to speed of operation
(b) Horizontal engine, and (a) Low speed engine (less than 100 rpm)
(c) Inclined steam engine (b) Medium speed engine (100 to 250 rpm)
(c) High speed engine (more than 250 rpm)
Steam Engines 645

3. It is a cylindrical part and is made of


(a) Condensing engine Steam after expansion cast iron. It is fitted with piston rings to provide
in the cylinder exhausted to condenser. sealing between the cylinder and piston body. The
(b) Non-condensing engine Steam after ex- piston reciprocates in the cylinder.
pansion in the cylinder is exhausted into at-
4. It is the link which connects the
mosphere.
piston and cross head. The piston rod reciprocates
inside the cylinder through the stuffing box and
20.2 CONSTRUCTION OF STEAM gland.
ENGINE
5. The cross-head provides
The constructional features of a simple steam bearings to a pin, which is joined to a connecting
engine as shown in Fig. 20.2 are given below. rod while it connects the piston rod rigidly. It
reciprocates on parallel guides and provides linear
1. It is made of cast iron, and it supports
motion to the piston rod.
the moving parts of the engine. It rests on the
foundation. 6. The connecting rod is made
of carbon or alloy steel. It is a component which
2. It is the heart of the engine. It is takes up the force of the piston through the piston
attached with a steam chest. It is made of cast rod and cross-head and transmits it to the crank. It
iron and it forms the chamber in which the piston converts reciprocating motion of the piston rod into
reciprocates. One end of the cylinder is closed by a rotary motion. The small end of the connecting rod
cylinder cover, known as the cover, end, while the is joined to the cross-head by pin (gudgeon pin) and
other end is known as the crank end. the larger end is connected to crank by crank pin.

1. Cylinder 2. Cylinder cover 3. Piston rod 4. Piston 5. Cross-head 6.


Fly wheel 14. Pulley 15. Steam chest 16. D-slide valve 17. D-slide valve rod

Fig. 20.2
646 Thermal Engineering

7. It is made of forged steel, it is into the cylinder on the cover end, and at the same
supported on the main bearing and is provided with time it also opens the steam port B for escape of
a crank pin supported with webs. It supports the used steam from the cylinder to the exhaust pipe E.
fly wheel at one end and the output pulley on the Figure 20.4 shows another position of the
other end. The rotary motion of the crank shaft is D-slide valve which covers both the steam ports for
mechanical energy. steam entry. It is the steam cut-off. The steam in the
cylinder can expand and do work until the piston
8. It is mounted on the crankshaft and
reaches the end of the stroke.
it connects the D-slide valve through the eccentric
rod. The eccentric converts the rotary motion of
the crank into reciprocating motion of the D-slide
valve.

9. It is located in the steam chest.


It controls the steam entry into the cylinder and its
exhaust. The cross-section of the D-slide valve is
like the letter D, and hence it is called D-slide valve. Fig. 20.4

10. It is mounted on the end of the


crankshaft. It is made of cast iron. It stores energy Figure 20.5 shows the position of the piston
in the form of inertia, when energy is in excess and and D-slide valve at the completion of the stroke.
it gives energy back, when it is in deficit. In other The steam enters the port B while the used steam
words, it minimises the speed fluctuations on the escapes from the port A.
engine.

20.3 WORKING OF A DOUBLE-


ACTING STEAM ENGINE

Steam engines are generally of double-acting type.


Double acting means that both back and front faces
of the piston are arranged as working faces. Fig. 20.5
The D-slide valve controls the steam entry into
the cylinder on one end and its exhaust from the
other end simultaneously. 20.4 HYPOTHETICAL INDICATOR
Figure 20.3 illustrates one position of the D-slide DIAGRAM OF STEAM ENGINE
valve which opens the steam port A for steam entry
The steam engine works on the modified Rankine
cycle discussed in Chapter 12; Section 12.12. A
hypothetical indicated diagram is drawn with the
following assumptions:
1. Instantaneous opening and closing of ports,
2. Steam enters the engine at boiler pressure,
3. Steam leaves the engine at condenser pressure,
Fig. 20.3 4. No throttling effect due to any restriction in
the port,
Steam Engines 647

5. No compression of steam, p
p1 5 1
6. No heat transfer losses, and
7. Piston moves in the cylinder frictionlessly.
Area 5-1-b-0-1
20.4.1 Hypothetical Indicator Diagram
without Clearance
The hypothetical indicator diagram for a steam
engine without clearance volume is shown in
b
Fig. 20.6. The sequence of operations are as follows: 0 V1
(a) Steam admission

p1 1

Area 1-2--3 -a -b-1

2
pb 3
b a
0 V1 V2
(b) Expansion 1-2 and steam release 2-3
Fig. 20.6
p

Steam Admission The steam at boiler pressure p1


enters the cylinder at the state 5 and steam supply
is cut off at the state 1 and it is known as the cut-
off point. The area 5–1–b –0 –5 represents the work
done during steam entry into the cylinder as shown
in Fig. 20.7(a). Area 3-4 - 0 -a-3
pb 4 3

The steam expands in the cylinder a


0 V2
from the state 1 to the end of stroke of the piston at
(c) Exhaust 3-4
the state 2. The expansion is hyperbolic. Area under
Fig. 20.7
the curve 1–2 represents the work done during
hyperbolic expansion as shown in Fig. 20.7(b).

The exhaust port opens at the The line 3–4 represents the exhaust of
point 2, and steam is released and pressure drops steam into the condenser. The state 4 represents
suddenly from p2 to the condenser pressure pb the piston position at the top dead centre (without
(called back pressure). The point 2 is known as the clearance). The fresh steam enters the cylinder at
point of steam release. The line 2–3 represents the the state 5 for the next cycle. The area 3– 4– 0 –a–3
steam release process. of Fig. 20.7(c) represents the work required for
steam exhaust from the cylinder.
648 Thermal Engineering

Work done by expansion of steam per operation of all the processes are exactly same as
cycle in the steam engine; that of the indicator diagram shown in Fig. 20.6,
W = Area of indicator diagram except the effect of the clearance volume, Vc.
= Area 5–1–b –0 –5 + Area 1–2 –3–a –b –1 The steam enters the cylinder at the point 4, the
– Area 3– 4–0–a –3 steam pressure in the cylinder suddenly rises to the
boiler pressure p1. This process is represented by
= p1V1 +
Ú pdV - p Vb 2 the line 4–5.
ÊV ˆ Work done During the Cycle
or W = p1V1 + p1V1 ln Á 2 ˜ – pbV2
Ë V1 ¯ W = Area 5–1–2–3–4–5 of indicator diagram
or W = p1V1 (1 + ln re ) – pb V2 ...(20.1) = Area a–5–1–b + Area b –1–2 – 3 –d
V2 – Area 3– 4–a–d

Ú pdV – pb (V2 – Vc )
where re = known as expansion ratio.
V1 = p1 (V1 – Vc ) +
Mean effective pressure of the cycle can be ÊV ˆ
obtained as = p1V1 – p1Vc + p1Vl 1n Á 2 ˜ – pb Vs
Ë V1 ¯
Work done
pm = = p1V1 [1 + ln (re )] – p1Vc – pb Vs ...(20.3)
Swept volume V
where re = 2 , expansion ratio.
p1V1 ( 1 + ln re ) - pb V2 V1
=
V2 Mean effective pressure, pm
V1
= p1
V2
[1 + ln re ] - pb pm =
Work done
=
W
Stroke volume Vs
or pm = p1 r [1 + ln (re)] – pb ...(20.2)
pV pV V
1 V1 pm = 1 1 [1 + ln ( re ) ] - 1 c - pb s ...(20.4)
where r= = , cut-off ratio. Vs Vs Vs
re V2 V
Let c = c , clearance ratio.
Vs
20.4.2 Hypothetical Indicator Diagram with
Clearance Volume V - Vc
r = 1 , cut-off ratio.
Vs
Figure 20.8. shows a hypothetical indicator diagram \ V1 = rVs + Vc
for a steam engine cylinder with clearance. The
V1
or =r+c
Vs
Using, we get
pm = p1 ( r + c) [1 + ln (re)] – p1 c – pb ...(20.5)
If neglecting the effect of clearance then
Vc = 0 and c = 0,
Then W = p1V1 [1 + ln (re )] – pb V2
(∵ V2 = Vs)
and pm = p1 r [1 + ln (re )] – pb
Thus we obtain the same relations given by
Eqs. (20.2) and (20.3) of work done and mean ef-
fective pressure, respectively for indicator diagram
Fig. 20.8 -
without clearance.
Steam Engines 649

20.4.3 Hypothetical Indicator Diagram ÊV ˆ


with Clearance and Compression - pb (V2 - V4 ) - pb V4 ln Á 4 ˜
Ë Vc ¯
Figure 20.9 shows a hypothetical indicator diagram È Ê V2 ˆ ˘
= p1V1 Í1 + ln Á ˜ ˙ - p1Vc
for a steam engine with compression of steam in ÍÎ Ë V1 ¯ ˙˚
clearance volume. ÊV ˆ
- pb (Vs + Vc - V4 ) - pb V4 ln Á 4 ˜ ...(20.7)
Ë Vc ¯
V2
Let re = , expansion ratio
V1
V1 - Vc
r = , cut-off ratio
Vs
V
c = c , clearance volume
Vs
V - Vc
a= 4 , fraction of stroke volume
Vs
completed at the start of compression,
Using, we get
V1 = rVs + Vc and V4 = aVs + Vc
Fig. 20.9
Therefore, work done,
W = p1V1 [1 + ln (re )] – p1Vc – pb [Vs + Vc
Process 6 –1 is steam admission, Process 1–2 È a Vs + Vc ˘
hyperbolic expansion, Process 2–3 is steam release – a Vs – Vc ] – pb (aVs + Vc ) ln Í ˙
as considered in previous indicator diagrams. Î Vc ˚
Process 3–4 Exhaust of steam into condenser, the = p1V1 [1 + ln (re )] – p1Vc – pb (1 – a) Vs
exhaust terminates at the state 4. Ê a + cˆ
– pb (a Vs + Vc ) ln Á ...(20.8)
Process 4–5 Compression of remaining steam in Ë c ˜¯
the cylinder. The mean effective pressure
Process 5–6 Fresh steam enters from boiler at the Work done W
pm = =
state 5 for the next cycle. Its pressure rise suddenly Swept volume Vs
to boiler pressure p1. p1V1 pV V
= [1 + ln ( re )] - 1 c - pb s (1 - a )
Work Done Vs Vs Vs
W = Area of indicator diagram 6–1–2–3–4–5–6. Ê aV V ˆ Èa + c ˘
- pb Á s + c ˜ ln Í
= Area a–6–1– d + Area d–1–2–3–e Ë Vs Vs ¯ Î c ˙˚
– Area b–4–3–e– Area a–f–5–4–b = p1(r + c) [1 + ln (re )] – p1c – pb (1 – a)
ÊV ˆ
= p1 (V1 - Vc ) + p1 V1 ln Á 2 ˜ Ê a + cˆ
Ë V1 ¯ – pb (a + c) ln Á ...(20.9)
Ë c ˜¯
ÊV ˆ
- p b (V2 - V4 ) - pb V4 ln Á 4 ˜ ...(20.6) Case (a) If neglecting the effect of compression,
Ë Vc ¯ i.e., a = 0, we get
ÊV ˆ W = p1V1 [1 + ln (re )] – p1Vc – pb Vs
W = p1V1 + p1 V1 ln Á 2 ˜ - p1Vc
Ë V1 ¯ and pm = p1 (r + c) [1 + ln (re )] – p1c – pb
650 Thermal Engineering

These are the relations of work done and mean 3. The inlet port does not close instantaneously,
effective pressure for the hypothetical indicator it closes gradually. Thus, the indicator
diagram Fig. 20.8, without compression. diagram is rounded off at the cut-off point.
Case (b) If neglecting the effect of compression 4. The expansion of steam in the cylinder is
and clearance, i.e., a = 0 and c = 0, we get not true hyperbolic. It is due to continuous
heat exchange between the steam and
W = p1V1 [1 + ln (re)] – pb V2 [∵ Vs = V2]
cylinder. During steam entry and early part
and pm = p1 r [1 + ln (re)] – pb
of expansion, the steam is hotter and heat is
These are relations for hypothetical diagram, transferred to cylinder walls, causing steam
Fig. 20.6 without clearance and compression. to condense. In the later part of the stroke,
the steam is colder than cylinder walls, and
ACTUAL INDICATOR DIAGRAM thus heat is transferred to steam, causing
Figures 20.6, 20.8 and 20.9 are hypothetical indi- reheating of steam.
cator diagrams for a steam engine. They are theo- 5. The steam release occurs before the end of
retical representations of the steam engine cycle expansion stroke, and the exhaust port opens
where all practical losses are assumed negligible. gradually, and thus the curve is rounded off
However, an indicator diagram taken with the help at the toe of the diagram.
of an instrument indicator as shown in Fig. 20.10, 6. The exhaust pressure is slightly higher than
is called actual indicator diagram. the back pressure since the steam is being
forced out of the cylinder.
7. Some steam remains in the cylinder, and is
compressed before the fresh steam enters the
cylinder.

The work lost per cycle can be reduced by


1. Rapid opening and closing of valves by
using Corliss valves or drop valves
Fig. 20.10
2. Steam jacketing, which makes the process
20.5.1 Actual Indicator Diagram vs nearly hyperbolic and minimises condensa-
Hypothetical Indicator Diagram tion
3. Reducing back pressure by liberal exhaust
The close examination of the two types of diagrams port opening.
reveals the following facts:
1. The steam pressure drops considerably be-
tween the boiler and cylinder. This pressure
The diagram factor is defined as the ratio of the
drop is due to condensation of some quantity
area of actual indicator diagram to the area of the
of steam into the pipeline and due to wire
hypothetical indicator diagram. It is denoted by K.
drawing (throttling) effect at the inlet port.
2. There is a gradual pressure drop during the Area of the actual indicator diagram
K=
steam supply into the cylinder up to the cut- Area of hypothetical indicator diagram
off point. It is due to condensation of some ...(20.10)
steam in the cylinder due to heat transfer.
Steam Engines 651

Mean height of the actual ms = Mass of cushion steam


indicator diagram + Mass of steam admitted per stroke
or K= ...(20.11) = Mass of cushion steam
Mean height of hypotheetical
indicator diagram Mass of steam used per hour
+ ...(20.13)
actual mean effective pressure No. of cycles per hour
or K= The mass of steam supplied per stroke is called
Hypothetical mean effective prressure
...(20.12) cylinder feed and the steam left behind in the
clearance space is known as cushion steam and it
Therefore,
acts as a buffer for slowing down the piston at the
Actual work done/cycle
end of the stroke. The mass of cushion steam can
= K ¥ hypothetical work done be obtained with the help of calibrated indicator
and Actual MEP( pm,act) diagram as shown in Fig. 20.11.
= K ¥ hypothetical mean effective pressure

CYLINDER CONDENSATION

Steam engines are generally double acting.


Steam from the boiler enters on both sides of the
cylinder. Further, at the end of expansion, the steam
temperature reduces due to decrease in steam
pressure and the cylinder becomes cool. When
the fresh steam enters the cylinder and it comes
in contact of the colder surface of the cylinder,
Fig. 20.11
some steam condenses into the cylinder without
doing any work. Some steam also condenses during On the compression curve, the steam may
expansion. This is called cylinder condensation. be assumed just dry and saturated, because, the
Due to condensation of steam in the early part compression tends to dry the steam. By choosing
of the working stroke, there is increase in steam a point b on the compression curve, the volume
consumption rate as high as 20%. The cylinder and pressure are noted. From the steam table, the
condensation may be reduced by specific volume of dry steam at this pressure is used
(i) Using superheated steam, for calculating the mass of cushion steam.
(ii) Steam jacketing of the cylinder, which keeps Mass of cushion steam
the cylinder surface at high temperature, Volume on the compression curve
(iii) By compressing some portion of the ex- =
Sp. volume corresponding topre ssure
hausted steam,
...(20.14)
(iv) By compounding of the steam engine, and
(v) By operating the engine at higher speed.
MISSING QUANTITY

As discussed in the previous section, some quantity


of steam condenses inside the cylinder during the
The total mass of steam in the cylinder is the sum of early part of the admission without doing any work.
the mass of fresh steam admitted per stroke and the This quantity of condensed steam is considered as
mass of steam left behind in the clearance volume missing quantity.
from the previous cycle.
652 Thermal Engineering

The magnitude of missing quantity can be


obtained with the help of the indicator diagram of
the steam engine shown in Fig. 20.12. An expansion The steam consumption of a steam engine may be
curve is drawn assuming the steam to be dry and defined as the amount of steam in kg consumed
saturated at every point throughout the expansion by the engine per hour. The steam consumption
stroke. It gives the theoretical volume occupied by of an engine can be obtained from the theoretical
steam at any point. It follows the saturation curve. indicator diagram as
Mass of steam admitted per cycle
ms = Volume of steam swept up to cut-off ¥
density of steam
p 2 1 1
= d L ¥ r ¥ = ( rVs) (kg) ...(20.17)
4 v v
where
d = bore of cylinder, m
L = stroke length of piston, m
r = cut-off ratio,
Fig. 20.12 v = specific volume of steam at admission
pressure, m3/kg
The horizontal difference between the actual = x vg for wet steam
expansion curve and saturation curve is the loss of = vg for dry and saturated steam
volume due to wetness of steam and is considered Tsup
as a missing quantity. = vg for superheated steam
Tsat
The line AC represents the volume of steam
For a single-acting steam engine which makes N
in the cylinder, if steam was dry and saturated at
rotations per minute (rpm),
pressure p1 and the line AB represents the actual
p 1
volume of expanding steam in the cylinder at the ms = d 2 L ¥ ¥ 60 N (kg/h) ...(20.18)
same pressure p1. Therefore, the missing quantity 4 v
(volume of steam) at the point B is Equation (20.18) represents the steam consump-
tion on one side of the piston. For a single-acting
BC = AC – AB
steam engine, it is the total steam consumption
= vg (1 – x) ...(20.15)
per hour. Generally, the steam engines are double
Sometimes, the missing quantity is also ex- acting. For double-acting steam engines, the total
pressed as the difference between the actual mass steam consumption is twice the above amount, pro-
of steam in the cylinder and indicated mass of dry vided
and saturated steam.
1. the cut-off ratio on both sides of the piston is
Missing mass of steam = ms (1 – x) ...(20.16) same
The missing quantity is mainly due to condensa- 2. the volume occupied by the piston rod is
tion of steam but a small amount is due to leakage negligible
past the piston. Towards the end of the expansion
Thus, for a double-acting steam engine
stroke, the reheating of steam takes place, thus
some missing quantity is recovered. p 1
ms = d 2 L ¥ ¥ (2 ¥ 60) N (kg/h)
4 v
...(20.19)
Steam Engines 653

More often, the steam consumption of a steam load applied on the engine corresponding to rated
engine is given in specific steam consumption. The speed is fixed and is called full load.
specific steam consumption (ssc) or steam rate is During the operation of the engine, as load
defined as the amount of steam required in kg to varies, the speed of the engine will also vary. If
develop 1 kWh (3600 kJ) of work. It can be obtained the mass flow rate of steam is kept constant, the
by dividing the amount of steam consumed per efficiency of the engine decreases at loads other
hour by the engine power (kW) units developed by than the full load. Therefore, it is imperative to keep
the engine. It is measured in kg/kWh and may be the engine speed constant by controlling the mass
expressed on the basis of either indicated power or flow rate and other properties of steam by a device.
brake power. Thus a device which controls the mass flow rate of
Mass of steam consumption, kg/h steam in the engine according to applied load on
ssc = the engine is called governor, and the process of
Power produced by engine, kW W
controlling speed is called governing.
(kg/kWh) ...(20.20)
The function of a governor on a steam engine is
Figure 20.13 shows the specific steam consump-
to maintain the constant speed of the engine with
tion for throttle and cut-off governings.
minimum fluctuations, irrespective of the load on
the engine. In other words, the engine should be
able to adjust its power output according to the load
with minimum fluctuations in speed.
There are two types of governing techniques
used on steam engines:
(i) Throttle governing
(ii) Cut-off governing

Fig. 20.13 In this method of governing, the cut-off point is


kept constant, but steam pressure is regulated by
a throttle valve. The steam from the boiler at a
20.10 STEAM COMPRESSION IN
THE CYLINDER pressure p1 is throttled to some lower pressure
according to reduction in load on the engine as
In order to minimise condensation of live steam shown in Fig. 20.14. The lower pressure of steam
during its admission into the cylinder, the exhaust during the admission process reduces the work
port is closed earlier and some quantity of steam re- developed by the engine.
maining in the cylinder is compressed. Steam pres-
sure and temperature increase, causing the cylinder
surface temperature to rise. Then fresh steam from
the boiler enters the cylinder, thus reducing steam
condensation. The work of compression is compen-
sated by reduction in cylinder condensation.

Most steam engines are designed to work on rated Fig. 20.14


speed at which their efficiency is maximum. The
654 Thermal Engineering

During a throttle control governing, the indi- regulates the mass of steam entering the cylinder.
cated power developed by an engine is varied, The cut-off governing is made with the help of
and the steam consumption rate is also varied. a slide valve operating under the control of a
Measurements reveal that the steam consumption centrifugal governor.
rate is linearly proportional to the indicated power The variation of steam consumption rate against
and thus is related as the indicated power developed in cut-off governing
ms = a IP + C ...(20.21) is shown in Fig. 20.17.
where a is the slope of the straight line, C is the
intersection point at no load.
Figure 20.15 shows a graph of the steam
consumption and indicated power for a throttle
governing. The linear line on the graph is called
Willan’s line.

Fig. 20.17

No. Throttle control Cut-off governing


Fig. 20.15 governing
1. The throttling is an During cut-off
irreversible process, governing the
thus the available pressure remains
As indicated by its name, in this method of energy is lost during constant, thus
governing, the inlet steam pressure is kept constant, throttling. the enthalpy of
but the steam cut-off point is controlled according steam also remains
constant.
to load on the engine. As load falls on the engine,
the steam cut-off point is shifted towards the left 2. At part load, the steam At part load, the mass
enters cylinder at lower of steam supplied is
as shown in Fig. 20.16. The cut-off governing
pressure, thus the reduced by shifting
steam consumption rate the steam cut-off
increases. towards left, thus
steam consumption
rate is reduced.
3. Due to throttle Thermal efficiency of
governing, the thermal the engine does not
efficiency of the engine change with cut-off
is lowered. governing.
4. It is simple and easy to It is complicated and
operate. costly.
Fig. 20.16
Steam Engines 655

The brake power (BP) is measured by dynamom-


20.12 POWER OUTPUT OF
STEAM ENGINE eters like rope brake, prony brake or hydraulic
dynamometers. The rope brake dynamometers are
Piston Speed extensively used in laboratories. A schematic of a
rope brake dynamometer is shown in Fig. 20.18.
It is the linear distance travelled by a piston per A rope wrapped over the fly wheel (brake drum)
second and is expressed as applies frictional resistance to the rotation of the
2LN brake drum, mounted on the engine shaft.
V= (m/s) ...(20.22)
60
Spring balance, S

The theoretical power developed by the engine


inside the cylinder as shown by the area of an
indicated diagram is known as indicated power. It
is designated as IPth and expressed as the product of Pulley
force and velocity of the piston.
Rope
IPth = M.E.P ¥ Cross sectional area of piston
¥ Piston velocity
2LN n D
= pm A = pm L A
60 60
n
= pm Vs ...(20.23)
60
where n = N, for single-acting engine
= 2 N, for double-acting engine
N = rotation per minute (rpm) of engine.
Dead weights, W

Actual Indicated Power


Fig. 20.18
The actual indicated power developed by an engine,
n Let W = Weight applied on rope, N (= mg)
IP = K pm L A ( kW ) ...(20.24)
60 S = Spring balance reading, N
n N = Speed of crank shaft, rpm
= pm, act LA
60 R = Effective radius of brake drum, m
If mean effective pressure is used in kN/m2, A in displacement
Then BP = Force ¥
m2, L in m, then the indicated power is expressed time
in kW. (W - S ) N
= ¥ 2p R ( kW) ...(20.25)
1000 60
Frictional Power
It is the power available (output) at the crank shaft
of the engine. However, it is less than the power The frictional power is the difference between
developed in the cylinder due to various frictional indicated power and brake power, i.e.,
losses. FP = IP – BP ...(20.26)
656 Thermal Engineering

20.13 EFFICIENCIES OF A
STEAM ENGINE

The efficiencies of a steam engine relate power


developed and heat supplied by steam into the
engine.
Heat supplied by steam to engine
Qs = ms ¥ (h1 – h2) ...(20.27) Fig. 20.19
where ms = Mass flow rate of steam (kg/s)
h1 = Specific enthalpy of steam entering
the engine (kJ/kg) BP
hoverall = ...(20.31)
h2 = Specific enthalpy of exhaust steam m f ¥ CV
(kJ/kg) m f = Rate of fuel supply, kg/s
CV = Calorific value of fuel, kJ/kg

A double-acting steam engine has a


It is defined as the ratio of actual indicated power cylinder bore of 250 mm and a stroke of 300 mm. The dry
developed in the engine cylinder to heat supplied saturated steam is supplied from the boiler at a pressure
by steam. of 10 bar. The condenser pressure is 0.3 bar. The engine
IPact has a speed of 160 rpm and a diagram factor of 0.8. If cut-
hith = ...(20.28) off takes place at 0.35 of the stroke, neglect the clearance
ms ( h1 - h2 )
and calculate the indicated power of the engine.

Solution
It is defined as the ratio of brake power to heat Given A double-acting steam engine without clearance
supplied by steam. d = 250 mm L = 300 mm
BP p1 = 10 bar = 1000 kN/m2
hbth = ...(20.29) pb = 0.3 bar = 30 kN/m2
ms ( h1 - h2 )
K = 0.8 r = 0.35
N = 160 rpm n = 320 working strokes/min

It is defined as the ratio of brake power to indicated To find Indicated power of the engine.
power developed by the steam engine.
BP hbth
hmech = = ...(20.30)
IP hith
Figure 20.19 shows the variation of mechanical
and brake thermal efficiencies with increase of
load.

It is defined as the ratio of brake power to energy


supplied by the fuel for steam generation in the
Fig. 20.20
boiler.
Steam Engines 657

Analysis The stroke volume of the cylinder


p p
Vs = V2 = d 2 ¥ L = ¥ (0.2 5) 2 ¥ (0.3)
4 4
= 0.0147 m3
The volume of cylinder at cut-off
V1 = rVs = 0.35 ¥ 0.0147 = 0.0051 m 3
V 0.0147
Expansion ratio, re = 2 = = 2.857
V1 0.0051
Theoretical mean effective pressure as
pm = p1 r [1 + ln (re)] – pb
= 10 ¥ 0.35 [1 + ln (2.857)] – 0.3
= 6.874 bar Fig. 20.21
Actual mean effective pressure
pm, act = K ¥ pm = 0.8 ¥ 6.874 = 5.499 bar The mean effective pressure is given by
Indicated power Work done W
pm = =
pm, act LA n Swept volume Vs
IP = ¥
60 60 15 ¥ 0.45 Vs ÈÎ1 + ln ( 2.33)˘˚ - 15 ¥ 0.05 Vs - 1.5 Vs
or pm =
Ê 320 ˆ Vs
= (5.499 ¥ 100) ¥ (0.0147) ¥ Á
Ë 60 ˜¯ = 6.75 (1.8473) – 0.75 – 1.5 = 10.22 bar
= 43.19 kW Alternatively, the mean effective pressure for a steam
engine cylinder with clearance can also be obtained by
Example 20.2 The steam enters a steam engine at
using Eq. (20.5),
15 bar and exhausts at 1.5 bar. The steam supply is cut
pm = p1 (r + c) [1 + ln (re)] – p1c – pb
off at 40% of the stroke. The clearance volume is 5% of
= 10.22 bar
the swept volume. Calculate the mean effective pressure.
Example 20.3 The area of an indicator diagram
Solution
is 25 cm2. The swept volume is 0.15 m3. Calculate the
Given A steam engine theoretical mean effective pressure. The indicator
p1 = 15 bar pb = 1.5 bar diagram is drawn to the following scales:
r = 0.4 Vc = 0.05 Vs 1 cm = 1 bar along the pressure axis
1 cm = 0.02 m3 along the volume axis
To find The mean effective pressure.
Assumption Diagram factor, K = 1 Solution

Analysis For indicator diagram shown in Fig. 20.21; Given Indicator diagram parameters,
V1 = Vc + rVs = 0.05 Vs + 0.4 Vs = 0.45 Vs A = 25 cm2 Vs = 0.15 m3
V2 = Vc + Vs = 0.05 Vs + Vs = 1.05 Vs Spring constant = 1 bar/cm
Expansion ratio; To find The mean effective pressure.
V 1.05Vs
re = 2 = = 2.333 Analysis The length of the indicator diagram
V1 0.45Vs
Swept volume
Hypothetical work transfer =
Scale on volume axis
W = p1V1 ÈÎ1 + ln ( re )˘˚ - p1Vc - pb Vs
0.15 m3
= = 7.5 cm
0.02 m3/cm
658 Thermal Engineering

The mean effective pressure is Analysis


Area of indicator diagram (i) Diagram factor
pm = ¥ Spring constant
Length of indicator diagram Expansion ratio in the cylinder
25 1 1
= ¥ 1 = 3.33 bar re = = = 5.0
7.5 r 0.2
The mean effective pressure in the cylinder
Example 20.4 A single-cylinder, double-acting pm = p1 r [1 + ln (re)] – pb
steam engine gives an indicated power of 50 kW, when = 1000 ¥ 0.2 ¥ [1 + ln (5.0)] – 60
running at 210 rpm. The engine bore is 0.25 m and the = 462 kPa
stroke is 0.38 m. The steam is supplied at 10 bar, dry and Area of cylinder
saturated and the back pressure is 0.6 bar, while cut-off p 2 p
takes place at 20% of the stroke. Calculate the diagram A= d = ¥ (0.25)2 = 0.049 m2
4 4
factor. The actual indicated power developed is given by
If steam consumption is 600 kg/h, calculate the n
indicated thermal efficiency. IP = pm, act LA
60
420
Solution \ 50 = pm, act ¥ 0.38 ¥ 0.049 ¥
60
Given The double-acting steam engines or pm,act = 383 kPa
d = 0.25 m L = 0.38 m The diagram factor,
N = 210 rpm n = 2 N = 420 cycles/min pm, act 383
p1 = 10 bar = 1000 kPa K = = = 0.828
pm 462
pb = 0.6 bar = 60 kPa (ii) Indicated thermal efficiency
IP = 50 kW From steam table
r = 0.2 At 10 bar, dry and saturated steam
ms = 600 kg/h h1 = hg @ 10 bar = 2777.08 kJ/kg
To find At 0.6 bar h2 = hf @ 0.6 bar = 359.9 kJ/kg
(i) Diagram factor, and Heat supplied to steam engine,
(ii) Indicated thermal efficiency. Qs = ms ¥ (h1 – h2)
= (600 kg/h) ¥ (2777.08 – 359.9)
Assumptions
= 1450308 kJ/h = 402.86 kW
(i) Engine cylinder without clearance.
Indicated thermal efficiency
(ii) Neglecting effect of piston rod on underside of
piston. IP 50
hith = =
(iii) The steam condenses completely after expansion. Qs 402.86
= 0.1241 or 12.41%
p
5
10 bar 1
Example 20.5 A rope-brake dynamometer is used to
measure the brake power of a steam engine. The net weight
applied at the rim of the brake drum of 1 m diameter is
130 kg. The engine runs at 300 rpm. Determine the brake
2 power.

0.6 bar 3
4 Solution

VC
V Given A rope-brake dynamometer
V1 m = 130 kg D =1m
V2
N = 300 rpm
Fig. 20.22
Steam Engines 659

To find Brake power 250


= (8.194 ¥ 100) ¥ 0.4 ¥ 70.68 ¥ 10 –3 ¥
Analysis The brake power is calculated as 60
= 96. 54 kW
2p N T Effective area of cylinder on crank side
BP =
60 p 2 2 p
where torque, A2 = ( d - dcr ) = ¥ (0.32 - 0.0 62 )
4 4
T = Force ¥ Drum radius = 0.0678 m2
D 1 Power developed by engine from crank side
= mg ¥ = 130 ¥ 9.81 ¥ = 637.65 Nm
2 2 N
300 IP2 = pm L A2
Then BP = 2p ¥ ¥ 637.65 = 20032 W 60
60 250
(8.194 ¥ 100) ¥ 0.4 ¥ 0.0678 ¥
= 20.03 kW 60
= 92.67 kW
Example 20.6 A double-acting steam engine has Total power developed by the engine
steam admission at 12 bar with cut-off at 1/3 of stroke. IP = IP1 + IP2 = 96.54 + 92.67 = 189.21 kW
The exhaust is at 0.2 bar. The cylinder bore is 0.3 m
and the stroke is 0.4 m. The engine runs at 250 rpm. Example 20.7 A double-acting steam engine with a
Determine the power developed considering the piston bore of 30 cm and a stroke of 40 cm has steam admission
rod diameter is 6 cm on the crank side. at 12 bar and the exhaust is at 1 bar. Cut-off occurs when
the total volume is equal to 0.25 of stroke volume. The
Solution engine runs at 250 rpm.
Given A double-acting steam engine If the clearance is 8% of strokes, calculate the power
p1 = 12 bar p2 = 0.2 bar developed, neglecting the effect of piston rod.
1 1
V1 = V2 or r = Solution
3 3
\ re = 3 Given A double-acting steam engine
d = 0.3 m L = 0.4 m d = 30 cm = 0.3 m L = 40 cm = 0.4 m
N = 250 rpm p1 = 12 bar = 1200 kPa pb = 1 bar = 100 kPa
n = N = 250 cycles for each side V1 = 0.25Vs N = 250 rpm
dcr = 6 cm n = 2 N = 500 rpm
To find Power developed by the engine. Vc = 0.08Vs or c = 0.08
Assumptions To find Power developed by the engine.
(i) Cylinder without clearance. Assumptions
(ii) Diagram factor K = 1. (i) Neglecting the compression of steam in the
(iii) Same mean effective pressure on both sides of cylinder.
piston. (ii) Diagram factor K = 1.
Analysis The mean effective pressure,
pm = pr [1 + ln (re)] – pb
1
= 12 ¥ [1 + ln (3)] - 0.2 = 8.194 bar
3
Area of cylinder on cover end,
p 2 p
A1 = d = ¥ (0.3)2 = 70.68 ¥ 10–3 m2
4 4
Power from cover-end side
N
IP1 = pm LA
60
Fig. 20.23
660 Thermal Engineering

Analysis The volume of cylinder at the point of cut-off To find


V1 = 0.25Vs (i) Diameter of the cylinder.
and V2 = Vc + Vs = (0.08 + 1) Vs = 1.08Vs (ii) Stroke length.
V1 - Vc (0.25 - 0.08) Vs
Cut-off ratio r = = = 0.17 Analysis The volume after expansion
Vs Vs V2 = Vc + Vs = cVs + Vs
V 1.08
Expansion ratio re = 2 = = 4.32 = 0.05Vs + Vs = 1.05Vs
V1 0.25 and volume at cut-off
The mean effective pressure in the cylinder V1 = rVs + Vc = 0.4Vs + 0.05Vs = 0.45Vs
pm = p1(r + c) [1 + ln (re )] – p1c – pb
V2 1.05 Vs
= 12 ¥ (0.17 + 0.08) [1 + ln (4.32)] – 12 ¥ 0.08 – 1 The expansion ratio, re = = = 2.333
V1 0.45 Vs
= 5.43 bar = 543 kPa
Power developed IP The theoretical mean effective pressure
n Êp ˆ 500 pm = p1 (r + c) [1 + ln (re )] – p1 c – pb
= pm LA = 543 ¥ 0.4 ¥ Á ¥ (0.3) 2 ˜ ¥
60 Ë 4 ¯ 60 = 8 ¥ (0.4 + 0.05) ÈÎ1 + ln ( 2.333)˘˚ - 8 ¥ 0.05 - 1.1
= 127.935 kW = 5.15 bar
Actual mean effective pressure
Example 20.8 Find the cylinder diameter and length pm, act = pm ¥ K = 5.15 ¥ 0.8 = 4.12 bar
of stroke for a double-acting steam engine developing
60 kW, when the cut-off takes place at 0.4 of stroke. The Indicated power
steam is supplied at 8 bar and it is exhausted at 1.1 bar. Ê nˆ
The clearance volume is 5% of the stroke volume. The IP = pm, act ¥ LA Á ˜
Ë 60 ¯
engine runs at 120 rpm. Neglect the effect of compression.
Êp ˆ Ê 240 ˆ
Take the diagram factor as 0.8 and stroke is 1.5 times of 60 = (4.12 ¥ 100) ¥ (1.5 d ) ¥ Á d 2 ˜ ¥ Á
diameter. Ë 4 ¯ Ë 60 ¯˜
60 ¥ 4 ¥ 60
d3 = = 0.03090
Solution 4.12 ¥ 100 ¥ 1.5 ¥ p ¥ 240
Given A double-acting steam engine with clearance. d = 0.3138 m = 31.38 cm
V - Vc Stroke L = 1.5d = 47.07 cm
IP = 60 kW r = 1 = 0.4
Vs
c = 0.05 p1 = 8 bar Example 20.9 In a steam engine the clearance
pb = 1.1 bar K = 0.8 volume is 5% of the swept volume and the back pressure
is 1.15 bar. If the compression is at 0.3 of the stroke, find
L = 1.5d N = 120 rpm
the pressure at the end of compression stroke.
n = 2N = 240 cycle/min
Find also the mean effective pressure, if the steam
supply pressure is 13.7 bar, and cut-off occurs at 4.0%
of the stroke.

Solution
Given A steam engine with clearance and compression.
Vc = 0.05 Vs c = 0.05
p1 = 13.7 bar pb = 1.15 bar
V4 – Vc = 0.3Vs a = 0.3 and r = 0.4
To find
(i) p5, the pressure after compression.
Fig. 20.24 (ii) pm, the mean effective pressure of the cycle.
Steam Engines 661

Assumptions Stroke = 1.5 ¥ Bore = 30 cm


(i) The compression and expansion are hyperbolic. Net brake load = 1.5 kN
(ii) All processes are reversible. Effective brake diameter = 160 cm
Supply steam condition = Dry and Saturated at 9 bar
Condensate collected = 800 kg/h
Condenser pressure = 0.07 bar
Exhaust steam quality = 0.7 dry
Determine
(a) Indicated power,
(b) Brake power,
(c) Mechanical efficiency,
(d) Indicated thermal efficiency,
(e) Brake thermal efficiency.

Fig. 20.25 Solution

Analysis Given A trial on a double-acting steam engine


(i) The pressure at end of compression p5 pm = 105 N/cm2 = 1050 kN/m2
For the process 4–5, the pressure and volume can N = 240 rpm
be related as n = 2 N (double acting engine) = 480 rpm
p4 V4 = p5 V5 d = 20 cm L = 1.5 d = 30 cm
where V4 = Vc + 0.3Vs = 0.05Vs + 0.3Vs (W – s) = 1.5 kN
= 0.35Vs 160 cm
R = = 80 cm = 0.8 m
V5 = Vc = 0.05Vs 2
p1 = 9 bar p2 = 0.07 bar
V4 0.35
Thus p5 = p4 = 1.15 ¥ = 8.05 bar x1 = 1.0 x2 = 0.7
V5 0.05 ms = 800 kg/h
(ii) The mean effective pressure
pm = p1(r + c) [1 + ln (re)] – p1c To find
Ê a + cˆ (i) Indicated power, IP
– pb (1 – a) – pb (a + c) ln Á
Ë c ˜¯ (ii) Brake power, BP
(iii) Mechanical efficiency, hmech
V2 V + Vs (0.05 + 1)Vs
where re = = c = (iv) Indicated thermal efficiency, hith
V1 Vc + r Vs (0.0 5+ 0.4)Vs
(v) Brake thermal efficiency, hbth .
= 2.333
Analysis
Thus pm = 13.7 ¥ (0.4 + 0.05) [1 + ln (2.333)] (i) Indicated power
n
– 13.7 ¥ 0.05 – 1.15 ¥ (1 – 0.3) – 1.15 ¥ (0.3 IP = pm, act ¥ LA ¥
60
Ê 0.3 + 0.0 5ˆ p 480
+ 0.05) ¥ ln Á = 1050 ¥ 0.3 ¥ ¥ (0.2)2 ¥
Ë 0.05 ˜¯ 4 60
= 11.39 – 0.685 – 0.805 – 0.7832 = 79.168 kW
= 9.115 bar 2 p RN
(ii) Brake power BP = (W – s)
60
Example 20.10 The following data was obtained 240
during a trial on a double-acting steam engine. = (1.5 kN ) ¥ 2 p ¥ (0.8 m) ¥
60
Average mean effective pressure = 105 N/cm2 = 30.16 kW
Speed = 240 rpm
662 Thermal Engineering

(iii) Mechanical efficiency


BP 30.16
hmech = = = 0.3809 or 38.09%
IP 79.168
(iv) Indicated thermal efficiency
From steam table
At 9 bar, h1 = hg = 2773.94 kJ/kg
At 0.07 bar, hf 2 = 163.4 kJ/kg, and
hfg = 2409.2 kJ/kg
Enthalpy of exiting steam
h2 = hf 2 + x2 hf g2
= 163.4 + 0.7 ¥ 2409.2
= 1849.84 kJ/kg
Heat supplied to steam engine,
Qs = ms ¥ (h1 – h2)
= (800 kg/h) ¥ (2773.94 – 1849.84)
= 739280 kJ/h = 205.35 kW
IP 79.168
hith = = = 0.3855 or 38.55%
Qs 205.35
(v) Brake thermal efficiency Fig. 20.26
BP 30.16
hbth = = = 0.1468 or 14.68% k = 0.8 bar/cm
Qs 205.38
N = 120 rpm hmech = 0.8
Example 20.11 During a trial on a vertical double- To find
acting steam engine, the following observations were (i) Indicator power, and
taken:
(ii) Brake torque.
Indicator card area (cover-end side) = 10.5 cm2
Indicator card area (crank-end side) = 10.2 cm2 Analysis
Length of indicator card = 7.6 cm (i) The mean effective pressure from an indicator
Spring constant = 0.8 bar/m diagram is obtained as
Speed = 120 r.p.m Area of indicator diagram
pm =
Diameter of piston = 21.5 cm Length of indicator card
Diameter of piston rod = 3.7 cm ¥ spring constant
Stroke = 30.5 cm On the cover side
Calculate the indicator power. If the mechanical A
efficiency of the engine is 80% at the given load, find the pm1 = 1 ¥ spring constant (k)
l
brake torque required. 10.5 cm 2
= ¥ (0.8 bar/cm)
7.6 cm
Solution
= 1.1052 bar = 110.52 kPa
Given A double-acting vertical steam engine. p
Indicator A1 = 10.5 cm2 Acover = d 2 = ¥ (0.215) 2 = 0.0363 m 2
4 4
d = 21.5 cm N
and IP1 = pm1 L Acover
A2 = 10.2 cm2 60
dcr = 3.7 cm 120
= 110.52 ¥ 0.305 ¥ 0.0363 ¥
l = 7.6 cm 60
L = 30.5 cm = 2.447 kW
Steam Engines 663

On the crank side To find Thermal efficiency of the engine at given


A 10.2 cm 2 conditions.
pm2 = 2 ¥ k = ¥ (0.8 bar/cm )
l 7.6 cm Assumption Steam condenses completely, after expan-
= 1.0737 bar = 107.37 kPa sion.
p
Acrank = ( d 2 - dcr2
) = ¥ (0.2152 - 0.0372 ) Analysis The graphical representation of mass
4 4
consumption rate v/s power output of the engine is shown
= 0.0352 m2
in Fig. 20.27.
N
and IP2 = pm2 L Acrank ¥ Using the relation ms = a IP + C
60
2820 = a ¥ 150 + C
120
= 107.37 ¥ 0.305 ¥ 0.0352 ¥ and 5000 = a ¥ 375 + C
60
= 2.305 kW
Total indicated power of engine
IP = IP1 + IP2 = 2.447 + 2.305
= 4.752 kW
(ii) Brake power,
BP
hmech =
IP
or BP = hmech ¥ IP = 0.8 ¥ 4.752
= 3.80 kW
The brake power can also be expressed as
2p N T
BP = Fig. 20.27
60
120
3.80 = 2p ¥ ¥T Solving these equations, we get
60 a = 9.6889 kg/kWh
Brake torque, C = 1366.67 kg/h
T = 0.3024 kN-m = 302.4 Nm The linear law
ms = 9.6889 IP + 1366.67
Example 20.12 A throttle-governed steam engine
uses 2820 kg of steam per hour, when developing an At IP3 = 225 kW
output power of 150 kW and 5000 kg/h of steam when ms = 9.6889 ¥ 225 + 1366.67
developing 375 kW. Estimate the thermal efficiency of the = 3546.67 kg/h = 0.985 kg/s
engine when developing an output of 225 kW, assuming Power output
Thermal efficiency hth =
the steam supply is dry and saturated at 10 bar and the ms ¥ ( h1 - h2 )
exhaust is at 0.25 bar. From steam table at 10 bar, dry and saturated steam
h1 = hg @ 10 bar = 2776.2 kJ/kg
Solution
At 0.25 bar h2 = hf @ 0.25 bar = 272.0 kJ/kg
Given A throttle-governed steam engine 225
ms1 = 2820 kg/h Then hth = = 0.0912 = 9.12%.
0.985 ¥ ( 2776.2 - 272)
IP1 = 150 kW
ms2 = 5000 kg/h
Example 20.13 The cylinder of a double-acting
IP2 = 375 kW
steam engine is 30 cm in diameter and has 44-cm stroke.
IP3 = 225 kW The steam enters the cylinder at 6 bar and leaves at
p1 = 10 bar, dry and saturated 1.2 bar. The steam is cut-off at 0.4 of stroke and com-
pb = 0.25 bar pression starts from 0.8 of return stroke. The clearance
664 Thermal Engineering

is 10% of the swept volume. The engine runs at 150 rpm. Volume after expansion
Assuming hyperbolic expansion and compression, cal- V2 = Vs + Vc = 1.1 Vs
culate (a) Indicated power produced, and (b) the steam Expansion ratio
consumption per kWh. Assume diagram factor of 0.9. V 1.1
re = 2 = = 2.2
V1 0.5
Solution
Volume at the start of compression
Given A double-acting engine with clearance and V4 - Vc
a = = (1 - 0.8) = 0.2
compression. Vs
d = 30 cm L = 44 cm p1 = 6 bar \ V4 = 0.2 Vs + Vc = 0.3Vs
r = 0.4 V4 – Vc = (1 – 0.8) Vs
Mean effective pressure,
c = 0.10
pm = p1(r + c) [1 + ln(re)] – p1c
pV = C pb = 1.2 bar
K = 0.9 È Ê a + cˆ˘
– pb Í(1 - a ) + (a + c) ln Á ˙
N = 150 rpm n = 2N = 300 rpm Î Ë c ˜¯ ˚
To find = 6 ¥ (0.4 + 0.1) ÈÎ1 + ln ( 2.2)˘˚ - 6 ¥ 0.1
(i) Indicated power produced, and È Ê 0.2 + 0.1ˆ ˘
- 1.2 Í(1 - 0.2) + (0.2 + 0.1) ln Á ˙
(ii) Steam consumption kg/kWh.
Î Ë 0.1 ˜¯ ˚
Assumptions = 6 ¥ 0.5 ¥ (1 + 0.788) – 0.6
(i) Dry and saturated steam eners the cylinder. – 1.2 ¥ [0.8 + 0.3 ¥ 1.0986]
(ii) Negligible area of piston rod. = 5.36653 – 0.6 – 1.355 = 3.40 bar
Actual mean effective pressure,
pm,act = K pm = 0.9 ¥ 3.40 = 3.06 bar.
Indicated power developed,
n n
IP = pm, act LA = pm, act Vs ¥
60 60
300
= (3.06 ¥ 100) ¥ 0.0311 ¥
60
= 47.6 kW
From steam tables at 6 bar, vg = 0.316 m3/kg
Theoretical mass per cycle,
r1 Vs
Fig. 20.28 mth =
g
Analysis The stroke volume 0.4 ¥ 0.0311
= = 0.0393 kg/stroke
p 2 p 0.316
Vs = d ¥L= ¥ (0.3)2 ¥ 0.44
4 4 Actual mass consumtion per cycle,
= 0.0311 m3 m
m1 = th =
0.0393
= 0.0437 kg/stroke
Clearance volume, K 0.9
Vc = 0.1Vs Total steam consumption,
Cut-off volume ms = m1 ¥ No of effective strokes/hour
V1 – Vc = 0.4 Vs = 0.0437 ¥ 300 ¥ 60 = 787.34 kg/h
V1 = 0.4 Vs + Vc Specific steam consumption,
= (0.4 + 0.1) Vs = 0.5 Vs m 787.37
ssc = s = = 16.54 kg/kWh
IP 47.6
Steam Engines 665

Example 20.14 The following readings were taken Swept volume on the crank side
during a trial of a single-cylinder, double-acting, non- p 2
Vs2 = (d – d 2cr) ¥ L
condensing steam engine running at 240 rpm. 4
p
Cylinder diameter, d = 300 mm = (0.32 – 0.052) ¥ 0.45
Srtoke length, L = 450 mm 4
= 0.0309 m3
Piston rod diameter, dcr = 50 mm
The indicated power developed,
Cut-off = 30% of stroke
N N
Length of indicator diagram l = 55 mm IP = pm1Vs1 + pm2Vs2 ¥
60 60
Area of indicator diagram 240
for cover end A1 = 1620 mm2 = (3.534 ¥ 100) ¥ 0.0318 ¥
60
Area of indicator diagram 240
+ (3.185 ¥ 100) ¥ 0.0309 ¥
for crank end A2 = 1460 mm2 60
Spring constant k = 0.12 bar/mm = 44.96 + 39.40 = 84.36 kW
Effective circumference of (ii) Brake power developed by engine
the brake wheel, pD = 8.344 m pDN
BP = (W - S )
Dead load on the brake W = 1900 N 60
240
Reading of the spring balance S = 190 N = (1900 - 190) ¥10 - 3 ¥ (8.344 m) ¥
60
Pressure of steam supplied p1 = 10.5 bar = 57.0 kW
Dryness fraction of steam (iii) Mechanical efficiency
supplied x = 0.92 BP 57.0
Find (a) indicated power, (b) brake power, (c) hmech = = = 0.6766 or 67.66%
IP 84.36
mechanical efficiency, (d) specific steam, consumption, (iv) Specific steam consumption
and (e) brake thermal efficiecy.
Mass flow rate of steam
Cut-off volume
Solution ms = ¥ No of effective stroke/h
Specific volume
Given Data as above Total swept volume;
To find (i) IP (ii) BP (iii) hmech (iv) specific steam Vs = Vs1 + Vs2
consumption on IP and BP basis, and (v) brake thermal = 0.0318 + 0.0309 = 0.0627 m3
efficiency Cut-offv olume
V1 = 0.3 ¥ Vs = 0.3 ¥ 0.0627
Assumption Diagram factor, K = 1.
= 0.0188 m3
Analysis No. of effective stroke/cycle = 1 (on each side)
(i) Indicated power No. of cycle per hour = 240 ¥ 60 = 14400
Area of indicator diagram ¥ Spring constant Specific volume of dry saturated steam at
pm =
Length of indicator diagram 10.5 bar;
vg = 0.1854 m3/kg
On cover end;
Specific volume of supplied steam with x = 0.92
A ¥ k 1620 ¥ 0.12
Pm1 = 1 = = 3.534 bar dry.
l 55
v = x vg @ 10.5 bar
On crank end;
= 0.92 ¥ 0.1854 = 0.1705 m3/kg
A ¥ k 1460 ¥ 0.12
pm2 = 2 = = 3.815 bar Steam consumption rate,
l 55
V 0.0188
Swept volume on the cover side ms = 1 = ¥ 1 ¥ 14400 = 1587.17 kg/h
p 2 0.1705
V s1 = d ¥L Specific steam consumption on IP basis
4
p m 1587.17
= ¥ (0.3)2 ¥ 0.45 = 0.0318 m3 = s = = 18.81 kg/kWh
4 IP 84.36
666 Thermal Engineering

Specific steam consumption on BP basis Assumptions


m 1587.17 (i) Negligible clearance in the cylinder.
= s =
BP 57.0 (ii) Steam condenses completely at the end of expan-
= 27.84 kg/kWh sion.
(v) Brake thermal efficiency
Analysis The expansion ratio,
Brake power
hbth = 1 1
Heat supply rate re = = = 2.5
r 0.4
Heat supplied/kg = h1 – h2
Theoretical mean effective pressure as
At 10.5 bar, hf = 772 kJ/kg,
pm = p1 r [1 + ln (re)] – pb
and hfg = 2005.9 kJ/kg
= 9 ¥ 0.4 ¥ [1 + ln (2.5)] – 1.4 = 5.5 bar
h1 = 772 + 0.92 ¥ 2005.9
Actual mean effective pressure,
= 2617.42 kJ/kg
pm, act = K ¥ pm = 0.8 ¥ 5.5 = 4.4 bar or 440 kPa
h2 = 0 (since engine is non condensing)
qin = 2617.42 – 0 = 2617.42 kJ/kg Indicated power,
n
Total heat supplied IP = pm, act LA ¥
60
1587.17
Qs = ms qin ¥ 2671.42 = 1177.77 kW Ê p ˆ Ê 480 ˆ
3600 22.5 = 440 ¥ (1.25 d ) ¥ Á d 2 ˜ ¥ Á
BP 57 Ë 4 ¯ Ë 60 ˜¯
hbth = = = 0.0483 or 4.83%
Qs 1177.77 or d3 = 6.51 ¥ 10–3 m3

Example 20.15 A single-cylinder, double-acting (i) Bore = 0.1867 m or 187 mm


steam engine is supplied with steam at 9 bar, dry and (ii) Stroke L = 1.25 d = 234 mm
saturated and exhausted at 140 kPa. Cut-off takes place Swept volume,
at 0.4 of stroke. Determine the necessary cylinder bore p
and piston stroke. The engine develops 22.5 kW. Assume Vs = d 2 L
4
a diagram factor of 0.8, stroke of 1.25 times the bore, p
= ¥ (0.187) 2 ¥ (0.234) = 0.00642 m3
speed of 240 rpm and hyperbolic expansion. 4
If actual steam consumption is 1.5 times the indicated The steam consumption per cycle
steam consumption, calculate the probable steam con- From steam table, at 9 bar
sumption and indicated thermal efficiency. vg = 0.2148 m3/kg
Indicated mass per cycle,
Solution
r1 Vs 0.4 ¥ 0.00642
m1 = =
Given A double-acting steam engine K g 0.8 ¥ 0.2148
p1 = 9 bar p2 = 140 kPa = 1.4 bar = 0.0149 kg/cycle
x1 = 1.0 r = 0.4 Steam consumption per hour,
IP = 22.5 kW K = 0.8 m = m1 ¥ No of effective strokes/hour
N = 240 rpm = m1 ¥ (n ¥ 60)
n = 2 N (double acting engine) = 480 rpm = ¥ 480 ¥ 60 = 430.4 kg/h
0.0149
L = 1.25d mact = 1.5mth (iii) Actual steam consumption
To find ms = 1.5 ¥ m = 1.5 ¥ 430.4 = 645.58 kg/h
(i) Bore of cylinder, d, (iv) Indicated thermal efficiency
(ii) Piston stroke, L, From steam tables,
(iii) Actual steam consumtion, mact . At 9 bar h1 = hg = 2773.94 kJ/kg
(iv) Indicated thermal efficiency, hith . At 1.4 bar h2 = hf2 = 458.4 kJ/g
Steam Engines 667

Heat supplied to steam engine, be varied by varying cylinder diameters. Therefore,


Qs = ms ¥ (h1 – h2) the diameter of the intermediate cylinder will be
= (645.58 kg/h) ¥ (2773.94 – 458.4) larger than the high-pressure cylinder, as steam
= 1494881.57 kJ/h = 415.24 kW pressure has been dropped and its volume has cor-
IP 22.5 respondingly increased. By the same reasoning,
Then hith = = = 0.0541 or 5.41% the low-pressure cylinder will have a still larger
Qs 415.24
diameter.

20.14.1 Limitations of Simple


Steam Engine
If steam expands completely in single expansion
within the cylinder, the steam engine is called a The compounding of steam engines is necessary
simple steam engine. because of the following limitations of a simple
Nowadays the modern boilers generate steam steam engine.
at very high pressure and the expansion of steam 1. If high-pressure steam expands in a single
is carried out in more than one cylinder. Thus stage of expansion to condenser pressure, it
an engine is called a compound steam engine. A requires a larger expansion ratio and thereby
hypothetical p–V diagram for a triple expansion bigger stroke length.
compound steam engine is shown in Fig. 20.29. 2. A steam engine with bigger stroke length
The whole hyperbolic expansion is divided into will have larger engine size and more
three parts by horizontal lines. The high-pressure imbalance forces.
cylinder receives the steam from the boiler and it 3. With use of high pressure steam, the pressure
rejects exhaust steam directly to the intermediate difference on two sides of the piston will
increase, which will cause leakage past the
pressure cylinder and the intermediate cylinder
piston.
rejects the exhaust steam directly to the low-
4. To handle the high-pressure steam, the
pressure cylinder. The three cylinders are designed
cylinder and other parts should have to be
in such a manner that the same amount of work is
made stronger. It will further increase the
done in each cylinder. weight of the engine and large variation in
As steam expands, its volume increases and turning moment. Thus vibrations will occur
hence a large space is required in low- pressure cyl- on the engine.
inders. The length of all cylinders is same, they all 5. With use of larger expansion ratio, the
have the same length of stroke, but the volume must temperature variation in the cylinder would
be large, which will result into larger steam
condensation in the cylinder and thereby
increase in steam consumption rate.

20.14.2 Advantages of Compound


Steam Engine
Limitations of simple steam engines can be over-
come by compounding a steam engine. The com-
pound steam engines have following advantages.
(i) The pressure difference per cylinder is
reduced, which reduces steam leakage and
Fig. 20.29 stroke length.
668 Thermal Engineering

(ii) With reduced pressure difference in a (ix) Due to lighter reciprocating parts of an
cylinder, the temperature variation is also engine, the engine vibrations are reduced.
reduced, which will result into less cylinder (x) In case of any breakdown, the engine can run
condensation. at reduced load.
(iii) Low pressure range in a cylinder also results
in less variation in the turning moment. 20.14.3 Disadvantages of Compound
(iv) Since high-pressure steam enters the HP Steam Engine
cylinder, it should only be made stronger to Apart from several attractive advantages, a com-
withstand high pressures and conusequently, pound steam engine has the following disadvan-
the successive cylinders will be lighter in tages:
weight. It reduces the weight-to-power ratio
(i) It has high initial cost.
of the engine.
(ii) Its size is large and it requires large floor
(v) The engine can be started at any position.
space.
(vi) The thermal efficiency of the engine improves.
(iii) It has complex construction, thus mainte-
(vii) The cylinder condensation can also be
nance is difficult.
reduced by reheating steam after expansion
(iv) Radiation losses are large from a high-
in each cylinder.
pressure cylinder.
(viii) High speed of engine is possible due to
perfect mechanical balancing. (v) Its wear and tear are large and it has
lubrication trouble.

Summary
external governing, the mass of
combustion engine that converts heat energy of steam consumed by an engine is linearly propor-
steam into mechanical work. tional to indicated power and is thus related as
ms = a IP + C
type, both back and front faces of the piston are governing, the inlet steam pressure
arranged as working faces. is kept constant, but the steam cut-off point is
diagram factor K is defined as the ratio of controlled according to load on the engine.
area of the actual indicator diagram to area of the is called
hypothetical indicator diagram. cylinder feed, and the steam left behind in the
clearance space is known as cushion steam.
in contact of the surface of the cylinder, which
is colder than the steam, some steam condenses the basis of either indicated power or brake power
into the cylinder without doing any work. This as
quantity of steam is considered as a missing Mass of steam consumption, kg/h
ssc =
quantity. Power produced by engine, kW W
governor on a steam engine is (kg/kWh)
to maintain the constant speed of the engine with n
minimum fluctuations, irrespective of the load IP = K pm L A (kW)
60
on the engine. There are two types of governing
techniques used on the steam engines: crank shaft of the engine,
(i) Throttle governing (W - S ) N
(ii) Cut-off governing BP = ¥ 2p R ( kW)
1000 60
Steam Engines 669

Glossary
Cross-head Link between piston rod and connecting Cylinder feed Mass of steam supplied per stroke
rod Cushion steam Steam left behind in the clearance
D-slide valve It controls the steam entry into the space
cylinder and its exhaust Missing quantity Quantity of steam condenses inside
Eccentric It converts the rotary motion of the crank the cylinder during admission without doing any work
into reciprocating motion of the D-slide valve Piston Speed Linear distance travelled by the piston
Diagram factor Ratio of area of actual indicator per second
diagram to area of hypothetical indicator diagram

Review Questions
1. Describe the hypothetical and actual indicator 6. Draw an expression for indicated work of a
diagram for a steam engine and explain what you double-acting steam engine without clearance
understand by diagram factor. volume.
2. Explain with neat diagrams the governing meth- 7. What is cylinder condensation. How is it reduced?
ods of a simple steam engine. 8. Compare throttle control governing and cut-off
3. Explain ‘Willan’s line’ for steam engine. governing.
4. Draw the actual and hypothetical indicator dia- 9. What are limitations of a simple steam engine?
gram for a steam engine. Establish an expression State pointwise.
for theoretical work done per stroke and the mean 10. Why are steam engines compounded? State the
effective pressure. State any assumptions made. advantages of compounding of steam engines.
5. Write the name of parts of a simple steam engine.

Problems
1. The following observations were made during 2. Find the mean effective pressure and work done
a trial on a single-cylinder, double-acting steam per stroke of an engine working at an admission
engine: pressure of 6 bar and a back pressure of 1.0 bar if
Bore = 250 mm; Stroke = 200 mm; Effective the cut-off ratio is 0.5 of the stroke. The cylinder
brake load = 800 N; Brake radius = 50 cm; diameter is 20 cm and its stroke is 30 cm. Neglect
Steam admission = 7 bar, 200°C; Mean effective clearance and compression.
pressure = 250 kPa; Speed = 200 rpm; Condensate [4.08 bar; 384.3 Nm]
collected = 200 kg/h; Temperature of condensate 3. Determine the actual mean effective pressure
= 50°C. for a reciprocating steam engine which receives
Determine (i) brake power, (ii) mechanical ef- steam at 10 bar and exhausts at 0.4 bar. The cut-
ficiency, (iii) indicated specific steam consump- off is at 0.4 of the stroke with clearance volume as
tion, and (iv) brake thermal efficiency. 10% of the stroke. Assume diagram factor = 0.8.
[(i) 8.377 kW, (ii) 51.2%, (iii) 12.22 kg/kWh, [6.033 bar]
(iv) 5.72%] 4. If in Problem (3), diameter of the cylinder is
21 cm and stroke length is 30 cm, determine the
670 Thermal Engineering

IP. of the engine is running at 150 rpm. Assume 1.05 bar. The expansion ratio for the cylinder is
the engine to be double acting. [31.34 kW] 2.5 and it develops 190 kW at a speed of 250 rpm.
5. Calculate IP, BP and mechanical efficiency of Determine the diameter and stroke length of the
a double-acting steam engine whose data as cylinder, if the stroke is 1.3 times its diameter.
follows: [30.97 cm; 40.27 cm]
imep = 2.5 bar; diameter of the cylinder = 24 cm; 8. The following are the readings taken during a
stroke = 40 cm; speed = 120 rpm; net brake load trial on a double-acting steam engine.
= 1060 N; brake drum radius = 1 m. Stroke = 30 cm; diameter of cylinder = 20 cm;
[18.095 kW; 13.32 kW; 73.6%] speed = 119 rpm; imep = 8 N/cm2; mass of fuel
6. Find the required cylinder diameter and the supplied = 0.08 kg/min; calorific value of fuel
length of the stroke for a steam engine to develop used = 30000 kJ/kg; net brake load = 300 N;
60 kW of indicated power at 150 rpm. The boiler steam used = 1.2 kg/min; back pressure =
pressure is 10 bar, cut-off at 0.4 of the stroke, back 10.33 N/cm2; steam admission pressure = 40
pressure is 1 bar and the diagram factor is 0.7. N/cm2, dry-saturated.
Neglect clearance and compression and assume Calculate IP, BP, mechanical efficiency, indicated
the stroke length to be 1.5 times the diameter of thermal efficiency and the overall efficiency of
the cylinder. [72.94 cm; 41.91 cm] the plant. [2.99 kW; 2.243 kW; 75.2%;
7. A double-acting steam engine is supplied with 7.475%; 5.608%]
steam at 9.8 bar and exhausts at a pressure of

Objective Questions
A. Choose the correct answer.
1. The ratio of clearance volume to the swept diagram and the theoretical indicator dia-
volume is known as gram
(a) expansion ratio (b) cut-off ratio (d) none of the above
(c) compression artio (d) clearance artio 5. The diagram factor is
2. The ratio of the volume of cut-off to the swept (a) less than unity (b) equal to unity
volume is known as (c) greater han
t unity (d) zero
(a) expansion ratio (b) cut-off ratio 6. The theoretical mean effective pressure is given
(c) compression artio (d) clearance artio by the expression
3. Mean effective pressure is obtained if the work p1
(a) [1 – ln (r)] – pb
done is divided by r
(a) total volume of the cylinder p1
(b) [1+ ln (r)] – pb
(b) swept volume of the cylinder r
(c) clearance volume of the cylinder p1
(c) [1+ ln (r)] + pb
(d) none of the above r
p1
4. Diagram factor is defined as (d) [1– ln – (r)] + pb
r
Area of the theroetical indicator diagram where p1 = initial pressure of steam entering the
(a)
Area of the actual indicatoor diagram cylinder
area of the actual indicator diagram Pb = back pressure of steam in the
(b)
area of the theoretical indicatoor diagram cylinder
(c) product of the area of the actual indicator r = ratio of expansion of steam
Steam Engines 671

7. The actual mean effective pressure is equal to (c) steam after expanding from one cylinder is
(a) the theoretical mean effective pressure plus admitted into another cylinder to expand
diagram factor further
(b) the theroretical mean effective pressure (d) none of the above
minus diagram factor 14. The large expansion ratio in a single cylinder of a
(c) the theroretical mean effective pressure steam engine results in
multiplied by diagram factors (a) much condensation of steam
(d) the theoretical mean effective pressure (b) large stroke of piston
divided by diagram factor (c) large temperature range
8. For the same cylinder specifications, speed and (d) all of the above
initial pressure, and cut-off, the condensing 15. By compounding a steam engine, the temperature
engine develops-in comparison with a non- range is
condensing engine. (a) reduced (b) constant
(a) less power (b) more power (c) increased (d) none of above
(c) the same power (d) none of the above (e) receiver ype
t
9. For the same cylinder specifications, speed, 16. In throttle governing, intake steam
initial pressure, cut-off and back pressure, in (a) pressure is varied
comparison with single-acting engine, a double-
(b) temperature is varied
acting engine develops
(c) volume is varied
(a) the same power
(d) none of the above
(b) 50% more power
17. In cut-off governing, intake steam
(c) approximately 100% more power
(a) pressure is varied
(d) none of the above
(b) temperature is varied
10. The indicator diagram is taken with the help of a/
an (c) mass is varied
(a) crankshaft (b) connecting or d (d) none of the above
(c) eccentric (d) indicator
B. Fill in the blanks.
11. The steam in a steam engine entrapped in the
1. A steam engine works on modified ______ cycle.
cylinder after the exhaust valve closes is known
2. When steam is admitted from both sides of a
as
steam engine, it is called ________.
(a) cushion steam
3. A double-acting engine of the same size and
(b) missing quantity
speed will produce _______ the power as a
(c) clearance volume steam single-acting engine.
(d) none of the above 4. A double-acting engine requires _________ to
12. The saturation curve is the curve showing the connect the piston rod to the connecting rod.
volume of the steam in the cylinder during 5. A stuffing box and gland are used to reduce
expansion in that steam engine if the dryness _________ of steam.
fraction of the steam is
6. A fictitious constant pressure acting on the piston
(a) 0.85 (b) 0.90 to produce the same amount of work as in the
(c) 0.95 (d) unity actual engine is called __________.
13. A steam engine is said to be a compound engine if 7. The expansion in the steam engine follows
(a) steam enters at the same initial pressure in _________ law.
more than one cylinders placed side by side 8. The popularly used steam valve is called _______.
(b) steam exhausts at the same exhaust pressure 9. The pressure at which the cylinder is connected to
in more than one cylinder placed side by the exhaust is called _________.
side
672 Thermal Engineering

10. If the exhaust port is closed before the dead centre 6. The reduction in exhaust pressure will ______ the
then the steam will be _________. engine efficiency.
11. In order to have exhaust pressure below 7. Throttle governing will cause an _________ in
atmospheric pressure, a _________ is used. steam consumption.
12. Steam jacketing is used to reduce cylinder 8. If a well-designed governor is used, the engine
_________. speed will _________.
13. The ratio of actual mep to the theoretical MEP is 9. The cylinder temperature will _________ due to
called _________. steam jacketing.
10. Cylinder condensation _________ due to steam
C. Answer using ‘increases’, ‘decreases’ or ‘remains jacketing.
constant’. 11. A fly wheel _________ the cyclic torque
1. If cut-off is decreased, the IP will ________. variation.
2. If cut-off is decreased, the steam consumption 12. The power developed _________ as speed is
will _________. decreased, other factors remaining unchanged.
3. If the power developed is larger than the power 13. Steam consumption _________ due to clearance.
required for the driven machine, the engine speed 14. The MEP _________ if cylinder condensation
will _________. increases.
4. If cut-off is increased, the efficiency will 15. The diagram factor _________ when cylinder
_________. condensation is reduced.
5. The reduction in exhaust pressure will ______ the
power developed.

8 Remain constant:
1, 2, 4, 10, 11, 12, 14 Decreases:
Increases: 3, 5, 6, 7, 9, 13, 15
Answers C

13. diagram factor


condensation 12. condenser 11. 10. compressed
release pressure 9. D-slide valve 8. 7. hyperbolic
mean effective pressure 6. leakage 5. 4. cross head and guide
twice 3. double acting 2. 1. Rankine
Answers B

17. (c)
16. (a) 15. (a) 14. (d) 13. (c) 12. (d) 11. (c) 10. (d) 9. (c)
8. (b) 7. (c) 6. (b) 5. (a) 4. (b) 3. (b) 2. (b) 1. (d)
Answers A
Steam Nozzles 673

21
Steam Nozzles

Introduction
A nozzle is a device of varying cross-sectional area, in which the pressure energy of fluid is converted into
kinetic energy. The mass of steam passing through any section of the nozzle remains constant. When fluid
passes through the nozzle, the pressure of fluid decreases with increase in velocity. Nozzles are used in
stream turbines, gas turbines, Jet engines, flow measuring devices, fuel injection and carburetion systems of
IC engines, spray paintings, etc.

The steam nozzles are classified as (i) convergent


nozzles, (ii) divergent nozzles, and (iii) convergent–
divergent nozzles. Divergent nozzle

Convergent Nozzle
A convergent nozzle is shown in Fig. 21.1 Its cross- A convergent–divergent nozzle is shown in
sectional area decreases continuously from its Fig. 21.3. Its cross sectional area first decreases
entrance to exit. It is used, when back pressure is from its entrance to throat and then increases from
equal to or greater then critical pressure. throat to exit. The throat is a portion of the nozzle

Convergent nozzle

A divergent nozzle is shown in Fig. 21.2. Its cross-


sectional area increases continuously from its
entrance to exit. It is used when the back pressure
is less than critical pressure. Converging–diverging nozzle
674 Thermal Engineering

which has smallest cross-section. The convergent– nozzle, superheated steam at high pressure enters
divergent nozzle is used when back pressure is less the nozzle with negligible velocity. As steam
than the critical pressure. It is widely used in steam flows in the nozzle, the steam pressure decreases
and gas turbines. with increase in its velocity. At the same time, the
enthalpy of steam also decreases. This enthalpy
drop is utilised for converting potential energy into
kinetic energy.
The following point should be noted for flow
Steam flow through a nozzle is considered adiabat- through the nozzle.
ic, since during the expansion of steam in a nozzle,
neither heat is supplied to the nozzle nor heat is re- Q = 0, there is no heat transfer.
jected from the nozzle. However, the work is per- W = 0, no work interaction is in-
formed by increasing kinetic energy of steam. volved during flow through noz-
zle.
Dpe = 0, fluid experiences usually very
The mass-flow rate of steam passing through a little or no change in its eleva-
nozzle is considered constant with respect to time. tion between inlet and outlet.
The mass-flow rate ( ms ) of any section of nozzle Using steady-flow energy equation, we get,
can be expressed as V22 – V12
AV h 2 – h1 + =0 …(21.4)
ms = c …(21.1) 2
v
where V1 = Inlet velocity of steam, m/s
where Ac = Cross-sectional area of nozzle, m2
V2 = Exit velocity of steam, m/s
V = Velocity of steam, m/s
h1 = Enthalpy of steam at nozzle entrance,
v = Specific volume of steam, m3/kg
J/kg
Accordingly, the velocity of steam at the entrance
h2 = Enthalpy of steam at nozzle exit, J/kg.
m v 4 ms v1
V1 = s 1 = …(21.2) Using Eq. (21.4), the velocity of steam at the
A1 p d12 nozzle exit can be expressed as
and velocity of steam at the exit
m v 4 ms v2 V2 = V12 + 2 ( h1 - h2) …(21.5)
V2 = s 2 = …(21.3)
A2 p d22
Normally, the velocity of steam at nozzle
Consider isentropic flow of steam through a
entrance is very small as compared to velocity of
nozzle as shown in Fig. 21.4. At the inlet of the
steam at nozzle exit, hence entrance velocity V1
is neglected. Thus, Eq. (21.5) is modified as

V2 = 2( h1 - h2) …(21.6)

When the magnitude of enthalpies of steam


is used in kJ/kg (as directly obtained from steam
tables) then

V2 = 2 ¥ 1000 ¥ ( h1 - h2)

= 44.72 ¥ ( h1 - h2) (m/s) …(21.7)


Steam Nozzles 675

where A2 = cross-sectional area, m2, and v2 =


specific volume of steam, m3/kg. Using v2 from
Since the fluid flow through the nozzle is isentropic, Eq. (21.8), we get
therefore, the pressure and specific volume of steam È n –1˘
are related as A2 2n Í Ê p2 ˆ n ˙
ms = p1v1 Í1 – Á ˜ ˙
pvn = C (constant) Êp ˆ -
1 n –1 Ë p1 ¯
v1 Á 2 ˜ Í ˙
where n is the index of expansion.
n
Î ˚
Ë p1 ¯
For isentropic flow through the nozzle, the en-
thalpy drop (h1 – h2) is equal to – vdp. Ú A Ê p ˆn
1
2n
È
Í Ê p2 ˆ n ˙
n –1˘

Therefore, the kinetic energy gain when velocity = 2 Á 2˜ p1v1 Í1 – Á ˜ ˙


v1 Ë p1 ¯ n –1 Ë p1 ¯
of approach is negligible; Í ˙
Î ˚
V22 n
2
= – vdp =Ún –1
(p1 v1 – p2 v2)
2n Ê p2 ˆ n p1v1
2 È
Í Ê p2 ˆ n ˙
n– 1 ˘

= A2 ¥ 2 Í1 – ÁË p ˜¯ ˙
n Ê pv ˆ n – 1 ÁË p1 ˜¯ v1
= p1 v1 Á1 – 2 2 ˜ Í 1 ˙
n –1 Ë p1v1 ¯ Î ˚
For isentropic flow
È 2 n +1 ˘
1
-
1 2n p1 ÍÊ p2 ˆ n Ê p2 ˆ n ˙
or ms = A2 –Á ˜
v2 Ê
= Á ˜
p1 ˆ n Ê p2 ˆ n
…(21.8) n – 1 v1 ÍÁË p1 ˜¯ Ë p1 ¯ ˙
=Á ˜ Í ˙
v1 Ë p2 ¯ Ë p1 ¯ Î ˚
È – ˘
1 …(21.10)
V22 n Í p2 Ê p2 ˆ n ˙ It is an expression for mass-flow rate of steam at
then = p1 v1 Í1 –
2 n –1 p1 ÁË p1 ˜¯ ˙ the nozzle exit.
Í ˙
Î ˚
È 1– ˘
1
n Í Ê p2 ˆ n ˙
= p1 v1 Í1 – Á ˜ ˙ Equation (21.10) gives the mass-flow rate of steam
n –1 Ë p1 ¯
Í ˙ through a nozzle. It is evident that the mass-flow
Î ˚
rate remains same throughout its cross-section.
È n –1˘ The mass-flow rate of steam will be maximum at
2n Í Ê p2 ˆ n ˙ the throat, where the cross section A2 is minimum.
or V2 = p1v1 Í1 – Á ˜ ˙ … (21.9) p
n –1 Ë p1 ¯ The value of pressure ratio 2 at the throat is
Í ˙
Î ˚ p1
Using in continuity equation, the mass flow rate called the critical pressure ratio. For maximum
of steam through the nozzle
discharge rate, differentiating Eq. (21.10) with
AV p
ms = 2 2 respect to pressure ratio 2 at throat and equating
v2 p1
it to zero;
È n –1˘
A 2n Í Ê p2 ˆ n ˙ dm
= 2 p1v1 Í1 – Á ˜ ˙ =0
v2 n –1 Ë p1 ¯
Í
Î
˙
˚ d Ê p2 ˆ
Ë p1 ¯
676 Thermal Engineering

d 2n p1
0= ms = A2
d Ê p2 ˆ n – 1 v1
Ë p1 ¯
È n
¥
2 n
¥
n +1 ˘
Ï È 2 n +1 ˘¸ ÍÊ 2 ˆ n –1 n Ê 2 ˆ n –1 n ˙
ÍÁË n + 1˜¯ –Á
ÔÔ 2n p1 ÍÊ Ê p2 ˆ n p2 ˆ n ˙ ÔÔ Î Ë n + 1˜¯ ˙
˚
Ì A2 ÍÁ ˜ –Á ˜ ˙˝
Ô n – 1 v1 Ë p1 ¯ Ë p1 ¯
Í ˙Ô È 2 n +1 ˘
ÔÓ Î ˚ Ô˛
2 n p1 ÍÊ 2 ˆ n –1 Ê 2 ˆ n –1 ˙
= A2 –Á
The quantities A2, p1, v1, n are constant with n – 1 v1 ÍÁË n + 1˜¯ Ë n + 1˜¯ ˙
Í ˙
p Î ˚
respect to 2 . Thus, we can write above equation
as p1 2
2n p Ê 2 ˆ n –1
È 2 n +1 ˘ = A2 ¥ 1 ¥Á
d ÍÊ p2 ˆ n Ê p2 ˆ n ˙ n – 1 v1 Ë n + 1˜¯
0= ÍÁË p ˜¯ – ÁË p ˜¯ ˙
d Ê p2 ˆ Í 1 1 ˙ È n +1 2 ˘
Ë p1 ¯ Î ˚ –
Í Ê 2 ˆ n –1 n –1 ˙
¥ 1 –
Í ÁË n + 1˜¯ ˙
n +1
2
–1 –1 Î ˚
2Ê p2 ˆ n n + 1Ê p2 ˆ n
0 = –
n ÁË p1 ˜¯ n ÁË p1 ˜¯
2 È n –1 ˘

= A2 2n p1 Ê 2 ˆ n –1 Í Ê 2 ˆ n –1 ˙
¥ ¥ Í1 – ÁË n + 1˜¯ ˙
n – 1 v1 ÁË n + 1˜¯
2–n 1 Í ˙
Î ˚
Ê p2 ˆ n Ê p2 ˆ n
or 2Á ˜ = (n + 1) Á ˜ 2
Ë p1 ¯ Ë p1 ¯
2n p Ê 2 ˆ n –1 È Ê 2 ˆ ˘
= A2 ¥ 1 ¥Á Í1 – Á ˜˙
2–n

1
n – 1 v1 Ë n + 1˜¯ ÍÎ Ë n + 1¯ ˙˚
Êp ˆ n Ê p2 ˆ n n +1
or Á 2 ˜ ÁË p ˜¯ =
Ë p1 ¯ 1 2 2
2n p Ê 2 ˆ n –1 È n + 1 – 2 ˘
= A2 ¥ 1 ¥Á Í ˙
n – 1 v1 Ë n + 1˜¯
1– n
Ê p2 ˆ n n +1 Î n +1 ˚
or ÁË p ˜¯ =
2 2
1
2n p Ê 2 ˆ n –1
n mmax = A2 ¥ 1 ¥Á …(21.12)
p2 Ê n + 1ˆ 1 – n n + 1 v1 Ë n + 1˜¯
= Á
Ë 2 ˜¯
or
p1 It is evident from Eq. (21.12), that the maximum
n
Ê 2 ˆ n –1 mass flow rate of steam depends on initial state (p1,
= Á …(21.11) v1) of steam and throat area, A2 and is independent
Ë n + 1˜¯
of the final state of steam. Therefore, the diverging
This pressure ratio is called critical pressure portion of the nozzle does not affect the mass-flow
ratio, and the pressure p2 at the throat is called rate, but it accelerates the steam leaving the nozzle.
critical pressure.
Substituting critical pressure in the expression
of mass flow, Eq. (21.10), we get the maximum The velocity of steam at the throat corresponding to
mass discharged through the nozzle maximum mass flow rate can be obtained by using
critical pressure ratio in Eq. (21.9).
Steam Nozzles 677

È n
¥
n –1 ˘
2n Í Ê 2 ˆ n –1 n ˙ Consider a nozzle attached to a large reservoir as
Vmax = p1 v1 Í1 – Á ˙
n –1 Ë n + 1˜¯ shown in Fig. 21.5. The reservoir contains steam
Í ˙
Î ˚ at high and steady pressure p1. Since the steam
2n Èn +1 – 2˘ velocity at the inlet to nozzle is relatively small,
= p1v1 Í ˙ thus it is considered negligible. The pressure at the
n –1 Î n +1 ˚
exit plane of the nozzle is the back pressure pb.
2n
or Vmax = p1v1 …(21.13)
n +1

Generally, the pressure at the throat is designated


as p*. The critical pressure ratio is defined as the
pressure ratio which maximizes the mass flow rate
of steam to be discharged by the nozzle.
n
p* Throat pressure Ê 2 ˆ n – 1
= =Á
p1 Inlet pressure Ë n + 1˜¯
The values of critical pressures for different
values of expansion index n are calculated below:

1. Saturated Steam When the steam is dry


saturated at the nozzle inlet, the index of expansion
is taken as 1.135;
1.135
p* Ê 2 ˆ 1.135 –1
\ = Á = 0.577 …(21.14)
p1 Ë 1.135 + 1˜¯

2. Superheated Steam When the steam is


superheated at the nozzle inlet, n = 1.30. Let the pressure at the exit plane vary gradually.
1.30 Its effects are the following:
p* Ê 2 ˆ 1.30 –1
\ = Á = 0.545 …(21.15) 1. When back pressure pb = p1, there is no
p1 Ë 1.30 + 1˜¯
mass flow ms = 0, through the nozzle. This
3. Wet Steam For wet stream at the nozzle inlet, corresponds to Case 1 of Fig. 21.5(b).
the index of expansion is considered as n = 1.113. 2. If the back pressure pb is reduced to the
1.113 state 2, it causes the pressure to decrease
p* Ê 2 ˆ 1.113 –1 in flow direction which results in certain
\ = Á = 0.582 …(21.16)
p1 Ë 1.113 + 1˜¯ mass-flow rate of steam through the nozzle.
Further, decrease in back pressure results in
4. Air When air enters a nozzle inlet, the index of greater mass-flow rate.
expansion is considered as g = 1.4 and 3. If the back pressure pb is further reduced, the
1.4 mass-flow rate reaches its minimum possible
p* Ê 2 ˆ 1.4 –1 value. This case is represented by state 3 on
= Á = 0.528
p1 Ë 1.4 + 1˜¯
678 Thermal Engineering

Fig. 21.5(b). At this state, the pressure is portion is small. Thus, most of the friction occurs in
called critical pressure and the velocity of the divergent portion of the nozzle and h–s diagram
flow is called sonic velocity. takes the shape as shown in Fig. 21.6. The nozzle
4. If the back pressure is further reduced to efficiency for such a nozzle is defined as the ratio of
states 4 and 5, less than p2, neither the mass actual enthalpy drop to the isentropic enthalpy drop
flow rate nor flow velocity in the nozzle is h1 – h3
affected and the nozzle is said to be choked. hN = …(21.17)
h1 – h3s
For a choked nozzle, the mass-flow rate
where h1 is the enthalpy of steam at the nozzle inlet
reaches the maximum possible value for
and h3 is the enthalpy of fluid at the exit for the actual
given conditions.
nozzle, while h3s is the exit enthalpy for a nozzle
For a convergent–divergent nozzle, the sonic under isentropic conditions. The nozzle efficiency
velocity reaches at the throat, where cross-section varies from 90% to 99%. The larger nozzles have
is minimum. The flow is subsonic in convergent more efficiency than the smaller nozzles.
portion, and supersonic in the divergent portion of
the nozzle.

21.3 FLOW THROUGH ACTUAL


NOZZLES

In the preceeding section of this chapter, the flow


through the nozzle is approximated to be isentropic.
Actual expansion through nozzles is non-isentropic
flow. It is due to presence of irreversibility at the
surface of flow and within the fluid itself. The
The nozzle efficiency can also be defined in
primary cause of irreversibility in nozzles is the
terms of actual kinetic energy and kinetic energy
presence of frictional effects, which are due to
corresponds to isentropic flow at nozzle exit as
(i) friction between fluid and wall surface of Actual kinetic energy at nozzle exit
nozzle, and hN =
Kinetic energy at nozzlle exit for
(ii) friction within the fluid itself. isentropic flow at same exit pressure
The friction between the wall surface and fluid
V32
molecules makes the expansion adiabatic but not =
isentropic. The energy lost in overcoming the fric- V32s
tion is used to reheat the steam and the enthalpy and Another cause of irreversibility in the nozzles is
entropy of steam increases during the process. flow separation, which induces strong turbulence
The convergent portion of the nozzle is smaller near the nozzle wall. Flow separation occurs when
than the divergent portion. Thus, the wall friction the angle of divergence in a convergent–divergent
is small in the convergent portion as compared nozzle is made too large. Consequently, the flow
to the divergent portion. The fluid friction is also area increases faster than the fluid expands. Thus,
small in convergent portion than in the divergent the included (cone) angle of the divergent duct is
portion, since the fluid velocity in the convergent usually kept less than 20°C.
Steam Nozzles 679

The frictional losses in the nozzle depends upon Example 21.1 A nozzle is to be designed to expand
material of construction, size, shape and surface steam at the rate of 0.10 kg/s from 500 kPa, 210°C to
because the wall surface in the large nozzles 100 kPa. Neglect inlet velocity of steam. For a nozzle
occupies a smaller portion of total flow volume. efficiency of 0.9, determine the exit area of the nozzle.
The effect of friction in a nozzle can be summa-
Solution
rized as
Given Expansion of steam through a nozzle
(i) Reduction in enthalpy drop,
m = 0.1 kg/s
(ii) Reheating of fluid,
p1 = 500 kPa
(iii) Reduction in exit velocity, T1 = 210°C = 483 K
(iv) Increase in sepecific volume, p2 = 100 kPa
(v) Decrease in mass flow rate. V1 = 0
hN = 0.9
To find The exit area of the nozzle.
Actual flow velocity of steam through a nozzle is

V 2 = 44.72 ¥ k ( h1 – h2 ) (m/s) …(21.18)

where k is called friction coefficient of the nozzle.

The nozzle velocity coefficient (CV) is also an


important parameter and can be expressed as
Actual velocity at the nozzle exit
CV = Velocity at the nozzle ex
xit with isentropic
flow and same exit pressure Assumption Dpe = 0
V
= 3 = hN …(21.19) Analysis From Mollier diagram, the properties of
V3s steam
It follows that the velocity coefficient is equal to State 1: Superheated steam at 500 kPa and 210°C,
the square root of the nozzle efficiency; h1 = 2877 kJ/kg s1 = 7.1039 kJ/kg ◊ K
The mass-flow rate through the nozzle is the After isentropic expansion, s1 = s2
design consideration, because it is affected by State 2: Wet steam; p2 = 100 kPa
irreversibilities. An important parameter, coefficient h2s = 2580 kJ/kg s2s = 7.1039 kJ/kg ◊ K
of discharge (CD), relates the actual mass flow with Isentropic enthalpy drop in nozzle
mass flow under the isentropic conditions for the Dhisentropic = h1 – h2s = 2877 – 2580 = 297 kJ/kg
same nozzle. Actual enthalpy drop;
Dhact = hN ¥ Dhisentropic
Actual mass-flow rate
CD = = 0.9 ¥ 297 = 267.3 kJ/kg
mass-flow rate with esentropic flow
= 267.3 ¥ 103 J/kg
m Actual enthalpy at the state 2;
= …(21.20)
ms h2 = h1 – Dhact
= 2877 – 267.3 = 2609.7 kJ/kg
680 Thermal Engineering

On the Mollier chart, locate the intersection point 1.35


of the pressure of 100 kPa and specific enthalpy of Ê 2 ˆ 1.35 –1
or p2 = p* = 10 ¥ Á
2609.7 kJ/kg. At this point, we get Ë 1.135 + 1˜¯
x2 = 0.971 and v2= 1.645 m3/kg = 5.77 bar

Steam velocity V2 = 44.72 ¥ Dh act (ii) Throat area


From Mollier chart, at 5.77 bar after isentropic
= 44.72 ¥ 267.3 = 731.14 m/s expansion
mv2 0.10 ¥ 1.645 h2 = 2675 kJ/kg x2 = 0.962
A2 = =
V2 731.14 vg@ 5.77 bar = 0.328 m3/kg (from steam table)
= 0.225 ¥ 10–3 m2 Specific volume at throat;
v2 = x2 vg = 0.962 ¥ 0.328
Example 21.2 Calculate the critical pressure and
= 0.316 m3/kg
throat area per unit mass-flow rate of steam, expanding
The exit velocity of steam
through a convergent–divergent nozzle from 10 bar, dry
saturated, down to atmospheric pressure of 1 bar. Assume V2 = 44.72 ¥ ( h1 h2 )
that the inlet velcocity is negligible and that the expasion = 44.72 ¥ 2778 – 2675
is isentropic.
= 453.87 m/s
Solution Using continuity equation
mv 1 ¥ 0.316
Given Isentropic expansion of dry saturated steam A2 = s 2 =
V2 453.87
though convergent–divergent nozzle = 6.96 ¥ 10–4 m2 = 696 mm2
V1 = 0 p1 = 10 bar
ms = 1 kg/s p3 = 1 bar, s = C Example 21.3 Steam enters a convergent–divergent
nozzle at 2 MPa and 400°C with a negligible velocity and
mass-flow rate of 2.5 kg/s and it exits at a pressure of
300 kPa. The flow is isentropic between the nozzle
entrance and throat and overall nozzle efficiency is
93 per cent. Determine (a) throat, and (b) exit areas.

Solution
Given Expansion of superheated steam through a
convergent–divergent nozzle.
ms = 2.5 kg/s hN = 0.93
Inlet conditions Superheated steam;
To find p1 = 2 MPa = 2000 kPa
(i) Critical pressure, and T1 = 400°C, V1 = 0
(ii) Throat area. Exit conditions
p2 = 300 kPa
Assumptions
(i) The expansion is in thermal equilibrium. To find
(ii) Frictionless flow of steam through nozzle. (i) Throat area of nozzle, and
(iii) Isentropic index for expansion of dry saturated (ii) Exit area of nozzle.
steam is 1.135.
Analysis
Analysis (i) Throat area of nozzle
(i) Critical pressure The critical pressure ratio for superheated steam;
n
p* Ê 2 ˆ n -1 p2
= Á = 0.546
p1 Ë n + 1˜¯ p1
Steam Nozzles 681

MP
a Therefore, at actual exit state, the properties are
1 2
3247.6 p3 = 300 kPa h3 = 2814.5 kJ/kg
400°C
v3 = 0.6764 m3/kg
p*
2 The velocity at nozzle exit
3076.1
Pa V3 = 44.72 ¥ h1 – h3
0k
h(kJ/kg) 30
= 44.72 ¥ 3247.6 - 2814.5
2781.9 3
3s = 930.7 m/s
And the exit area of nozzle
mv 2.5 ¥ 0.6764
A3 = s 3 =
V3 930.7
0 s = 1.817 ¥ 10–3 m2 = 18.17 cm2
7.127

7.2 kJ/kg.k Example 21.4 In a convergent–divergent nozzle,


the steam enters at 15 bar and 300°C and leaves at a
pressure of 2 bar. The inlet velocity to the nozzle is
or p2 = 0.546 ¥ 2000 = 1092 kPa 150 m/s. Find the required throat and exit areas for mass-
The steam properties; flow rate of 1 kg/s. Assume nozzle efficiency to be 90 per
Inlet p1 = 2 MPa cent and Cps = 2.4 kJ/kg ◊ K
T1 = 400°C
Solution
h1 = 3247.6 kJ/kg
s1 = 7.127 kJ/kg ◊ K Given Expansion of superheated steam through a
Throat p2 = 1092 kPa convergent–divergent nozzle.
s1 = s2 = 7.127 kJ/kg ◊ K ms = 1 kg/s, hN = 0.90, Cps = 2.4 kJ/kg ◊ K
h2 = 3076.1 kJ/kg Inlet conditions Superheated steam;
v2 = 0.242 m3/kg p1 = 15 bar = 1500 kPa, T1 = 300°C
The velocity of steam at throat V1 = 150 m/s
Exit conditions
V2 = 44.72 ¥ h 1– h2
p3 = 2 bar = 200 kPa
= 44.72 ¥ 3247.6 - 3076.1
To find
= 585.66 m/s
(i) Throat area of nozzle,
Throat area
ms v2 2.5 ¥ 0.242 (ii) Exit area of nozzle.
A2 = =
V2 585.66 Schematic with given data
= 1.033 ¥ 10–3 m2 = 10.33 cm2
(ii) Exit Isentropic state 3s
p3 = 300 kPa
h3s = 2781.9 kJ/kg s2 = s3s
The enthalpy of steam at the actual exit state 3 is
determined by using nozzle efficiency
h1 – h3
hN =
h1 – h3s
or h3 = h1 – hN (h1 – h3s)
= 3247.6 – 0.93 ¥ (3247.6 – 2781.9)
= 2814.5 kJ/kg
682 Thermal Engineering

Analysis The velocity at nozzle exit


(i) Throat area of nozzle V3 = 44.72 ¥ h01 – h3
The critical pressure for superheated steam
= 44.72 ¥ 3031.25 - 2667.38
p2
= 0.546 = 963.15 m/s
p1 And the exit area of nozzle
or p2 = 0. 546 ¥ 15 bar = 8.19 bar mv 1 ¥ 0.85
A3 = s 3 =
The steam properties V3 963.15
Inlet p1 = 15 bar, T1 = 300°C, = 8.82 ¥ 10–4 m2 = 882 mm2
h1 = 3020 kJ/kg
Throat p2 = 8.19 bar, s1 = s2 = 7.127 kJ/kg ◊ K Example 21.5 Calculate the throat and exit diameters
h2 = 2900 kJ/kg of a convergent–divergent nozzle, which will discharge
v2 = 0.3 m3/kg 820 kg of steam per hour at a pressure of 8 bar
Exit state Isentropic state 3s, p3 = 2 bar superheated to 220°C into a chamber having a pressure
of 1.5 bar. The friction loss in the divergent portion of the
h3s = 2640 kJ/kg s1 = s3s
nozzle may be taken as 0.15 of the isentropic enthalpy
Since steam enters the nozzle with a velocoty,
drop.
therefore, the stagnation enthalpy at inlet
V12 Solution
h01 = h1 +
2 Given Expansion of superheated steam through a
(150) 2 convergent–divergent nozzle.
= (3020 kJ/kg) + (kJ/kg)
2000 ms = 820 kg/h = 0.2278 kg/s
= 3031.25 kJ/kg Inlet conditions: Superheated steam;
The velocity of steam at throat p1 = 8 bar T1 = 220°C
V2 = 44.72 ¥ h01 – h2 Exit conditions: p3 = 1.5 bar Dhf = 0.15 Dh2–3
= 44.72 ¥ 3031.25 2900 To find
= 512.34 m/s (i) Throat area of nozzle, and
Throat area; (ii) Exit area of nozzle.
ms v2 1 ¥ 0.3
A2 = =
V2 512.34
= 5.855 ¥ 10–4 m2 = 585.5 mm2
The enthalpy of steam at the actual exit state 3 is
determined by using nozzle efficiency
h –h
hN = 01 3
h01 – h3s
or h01 – h3 = hN (h01 – h3s)
= 0.93 ¥ (3031.25 – 2640)
= 363.86 kJ/kg
and h3 = 3031.25 – 363.86
= 2667.38 kJ/kg
The specific volume at the intersection point
of pressure of 2 bar and specific enthalpy of Assumptions
2667.38 kJ/kg is
(i) Inlet velocity of fluid is negligible.
v3 = 0.85 m3/kg (From Mollier chart)
(ii) No change in potential energy.
(iii) Flow of fluid in convergent portion is isentropic.
Steam Nozzles 683

Analysis The critical pressure ratio for expansion of Example 21.6 A convergent–divergent nozzle is
superheated steam is required to discharge 2 kg of steam per second. The
p* nozzle is supplied with the steam at 7 bar and 180°C and
= 0.546 when k = 1.3 discharge takes place against the back pressure of 1 bar.
p1
The expansion up to throat is isentropic and the frictional
The critical pressure p* = 0.546 ¥ 8 bar = 4.368 bar resistance between throat and exit is equivalent to 63
The critical pressure is greater than the exit pressure kJ/kg of steam. The approach velocity to the nozzle is
of steam, thus expansion will continue in divergent 75 m/s and throat pressure is 4 bar. Estimate
portion also.
(a) Suitable areas of throat and exit,
The steam properties;
(b) Overall efficiency of the nozzle based on enthalpy
Inlet: p1 = 8 bar T1 = 220°C; drop between inlet pressure, temperature and exit
h1 = 2880 kJ/kg pressure.
Throat: p2 = 4.36 bar, s1 = s2 = 7.127 kJ/kg ◊ K
h2 = 2740 kJ/kg, v2 = 0.435 m3/kg Solution
Exit state: Isentropic state 3s Given Expansion of superheated steam through a
p3 = 1.5 bar h3s = 2465 kJ/kg, x = 0.94 convergent–divergent nozzle.
(i) Throat area of nozzle ms = 2 kg/s
The velocity of steam at throat Inlet conditions: Superheated steam;
V2 = 44.72 ¥ h1 – h2 p1 = 7 bar, T1 = 180°C, V1 = 75 m/s
Throat condition: p2 = 4 bar, s1 = s2
= 44.72 ¥ 2880 – 2740 = 529.13 m/s Exit conditions: p3 = 1 bar, Dhf = 63 kJ/kg

Throat area; To find


mv 0.2278 ¥ 0.435 (i) Throat area of nozzle,
A2 = s 2 =
V2 529.13 (ii) Exit area of nozzle, and
(iii) Overall efficiency of the nozzle.
= 1.87 ¥ 10–4 m2 = 187.2 mm2
The enthalpy of steam at actual exit state 3 (p3 =
1.5 bar)
h3 = h3s + Dhf
= 2465 + 0.15 ¥ (2740 – 2465)
= 2506.25 kJ/kg
The specific volume at intersection point of
pressure of 1.5 bar and specific enthalpy of
2506.25 kJ/kg is
v3 = 1.16 m3/kg (From Mollier chart)
The velocity at nozzle exit
V3 = 44.72 ¥ h1 – h3
= 44.72 ¥ 2880 – 2506.25
= 864.55 m/s Analysis
(ii) Exit area of nozzle
(i) Throat area of nozzle
mv3 0.2278 ¥ 1.16 The steam properties
A3 = =
V3 864.55 Inlet: p1 = 7 bar T1 = 180°C
= 3.0564 ¥ 10–4 m2 = 305.64 mm2 h1 = 2830 kJ/kg
684 Thermal Engineering

Throat: p2 = 4 bar s1 = s2 = 7.127 kJ/kg ◊ K Example 21.7 A turbine having a set of 16 nozzles
h2 = 2720 kJ/kg v2 = 0.48 m3/kg receives steam at 20 bar and 400°C. The pressure of
Exit state: Isentropic state 3s steam at the nozzle exit is 12.0 bar. If the discharge rate
p3 = 1 bar h3s = 2480 kJ/kg is 260 kg/min and the nozzle efficiency is 90%, calculate
s1 = s3s the cross-sectional area at the nozzle exit.
The stagnation enthalpy at inlet If the steam has a velocity of 80 m/s at entry to the
nozzle, find the percentage increase in discharge.
V12
h01 = h1 +
2 Solution
(75) 2
= (2830 kJ/kg) + (kJ/kg)
2000 Given Expansion of steam through set of nozzles
= 2832.8 kJ/kg
Case 1 N = 16, p1 = 20 bar, T1 = 400°C
The velocity of steam at throat
hN = 0.9, p2 = 12 bar, ms = 260 kg/min
V2 = 44.72 ¥ h01 – h2
Case 2 V1 = 80 m/s
= 44.72 ¥ 2832.8 – 2720 To find
= 475 m/s (i) Cross-sectional area at nozzle exit,
(ii) Percentage increase in discharge, if inlet velocity
(i) Throat area
approaches 80 m/s.
ms v2 2 ¥ 0.48
A2 = =
V2 475 Assumptions
–3 2
= 2.021 ¥ 10 m (i) For the first case, the inlet velocity to the nozzle is
= 2021 mm2 negligible.
The enthalpy of steam at the actual exit (ii) For superheated steam, the isentropic index is
state 3 ( p3 = 1 bar) k = 1.3.
h3 = h3s + Dhf = 2480 + 63
= 2543 kJ/kg
The specific volume at the intersection point
of pressure of 1 bar and specific enthalpy of
2543 kJ/kg is
v3 = 1.6 m3/kg (From Mollier chart)
The velocity at nozzle exit
V3 = 44.72 ¥ h01 – h3

= 44.72 ¥ 2832.8 – 2543


= 761.31 m/s
(ii) Exit area of nozzle
Analysis The critical pressure ratio for expansion of
ms v3 2 ¥ 1.6
A3 = = superheated steam is
V3 761.31
p*
= 4.203 ¥ 10–3 m2 = 4203 mm2 = 0.546 when k = 1.3
p1
(iii) Overall efficiency of the nozzle
The critical pressure p* = 0.546 ¥ 20 bar = 10.92 bar
h1 – h3
hoverall = The critical pressure is less than the exit pressure of
h1 – h3s
steam from nozzle. Thus the nozzle is convergent type
2830 – 2543 only.
= = 0.82 or 82%
2830 – 2480
Steam Nozzles 685

From Mollier chart or steam tables Example 21.8 The steam is supplied to a nozzle at
State 1: Nozzle entry p1 = 20 T1 = 400°C a rate of 1 kg/s from an inlet condition of 10 bar, dry
h1 = 3247.6 kJ/kg saturated and exit at 1 bar pressure. The efficiency of the
State 2: Nozzle exit, s1 = s2s p2 = 12 bar nozzle for the convergent portion is 95 per cent and that
h2s = 3100 kJ/kg of the divergent portion is 90 per cent. Determine

Case 1 Using nozzle efficiency (a) throat and exit diameters of nozzle,
h – h2 (b) length of nozzle, if divergent cone angle of the
hN = 1 nozzle is 14°,
h1 – h2 s
or h1 – h2 = hN (h1 – h2s) (c) The power in kW corresponds to exit velocity of
the steam.
= 0.9 ¥ (3247.6 – 3100) = 132.84 kJ/kg
and h2 = 3247.6 – 132.84 = 3114.76 kJ/kg Solution
From Mollier chart at p2 = 12 bar, and h2 = Given Expansion of dry saturated steam through a
3114.76 kJ/kg convergent–divergent nozzle.
Specific volume v2 = 0.275 m3/kg ms = 1 kg/s p1 =10 bar x1 = 1.0
The exit velocity of steam p2 = 1 bar hN, conv = 0.95 hN,div = 0.9
V2 = 44.72 ¥ h1 – h2 = 44.72 ¥ 132.84 To find
= 515.42 m/s (i) Throat diameter of nozzle,
Mass-flow rate of steam per nozzle (ii) Exit diameter of nozzle,
ms Ê 260 ˆ 1 (iii) Length of nozzle for cone angle a = 14°,
m1 = =Á kg/s˜ ¥ = 0.271 kg/s
N Ë 60 ¯ 16 (iv) Power at nozzle exit.

A2 V2 Assumptions
Further, m1 =
v2 (i) Inlet velocity to nozzle is negligible.
(ii) Isentropic index of expansion of dry saturated
m1v2 0.271 ¥ 0.275
or A2 = = steam is 1.135.
V2 515.42
(iii) Steam expansion is in thermal equilibrium.
= 1.44 ¥ 10–4 m2
Case 2 If initial velocity of steam is taken as 80 m/s
then initial stagnation enthalpy
V12 (80) 2
h01 = h1 + = 3247.6 +
2000 2000
= 3250.8 kJ/kg
h01 – h2¢ = hN (h01 – h2s) = 0.9 ¥ (3250.8 – 3100)
= 135.72 kJ/kg
The velocity of steam at nozzle exit
V 2¢ = 44.72 ¥ h01 – h¢2 = 44.72 ¥ 135.72
= 521 m/s
Percentage increase in velocity Analysis
(i) The critical pressure ratio at throat
V2 - V2¢ 521 – 515.42
= ¥ 100 = ¥ 100 p*
V2 515.42 = 0.577
p1
= 1.08%
or p2 = p* = 0.577 ¥ 10 bar = 5.77 bar
686 Thermal Engineering

From Mollier chart


State 1: Nozzle entry r3
14° r2
p1 = 10 bar x = 1.0
h1 = 2778 kJ/kg
State 2: After isentropic expansion at throat
and p2 = 5.77 bar s1 = s2s L
h2s = 2676 kJ/kg
Using nozzle efficiency in convergent portion
h1 – h2
hN, conv = Exit velocity of steam
h1 – h2 s
V3 = 44.72 ¥ h1 – h3
or h2 = h1 – hN, conv (h1 – h2s)
= 2778 – 0.95 ¥ (2778 – 2676) = 44.72 ¥ 2778 – 2418.2
= 2681.1 kJ/kg = 848.28 m/s
From Mollier chart at p2 = 5.77 bar and h2 = A3V3
2681.1 kJ/kg, and ms =
v3
x2 = 0.965 v2 = 0.327 m3/kg
The velocity of steam at throat 1 ¥ 1.694
or A3 =
848.28
V2 = 44.72 ¥ h1 – h2
= 1.977 ¥ 10–3 m2
= 44.72 ¥ 2778 – 2681.1 p 2
A3 = d3
= 440.2 m/s 4
The mass-flow rate of steam is given by It gives d3 = 0.0504 m = 5.04 cm
ms = A2 V2 r3 – r2 d –d
(iii) tan (a/2) = = 3 2
v2 L 2L
1 ¥ 0.327 5.04 – 3.75
or A2 = or L = = 8.0 cm
440.2 2 tan 7∞
= 7.428 ¥ 10–4 m2 = 7.428 cm2 (iv) Power at nozzle exit
p 2 = kinetic energy of steam
Further, A2 = d2
4 V32 1 ¥ (848.28) 2
= ms =
or d2 = 3.75 cm 2 2
(ii) State 3s: After isentropic expansion in divergent = 359.79 ¥ 103J/s
section of the nozzle, = 359.79 kW
h3s = 2389.0 kJ/kg at s2 = s3s
Using nozzle efficiency of divergent section Example 21.9 The steam is supplied to a nozzle at
15 bar, 350°C and exits at 1 bar. If the divergent portion
h2 – h3
hN, div = of the nozzle is 80 mm long and the throat diameter is
h2 – h3s 6 mm, determine the cone angle of the divergent portion.
or h3 = h2 – hN, div (h2 – h3s) Assume 12% of total enthalpy drop is lost in the friction
= 2681.1 – 0.9 ¥ (2681.1 – 2389) in the divergent portion. Also calculate the velocity and
= 2418.2 kJ/kg temperature of steam at the throat.
From Mollier chart at
Solution
p3 = 1 bar h3 = 2418.2 kJ/kg
x3 = 0.886 Given Expansion of steam through convergent–diver-
and v3 = 1.694 m3/kg gent nozzle.
Steam Nozzles 687

p1 = 15 bar T1 = 350°C p2 = 1 bar (i) Temperature of steam at throat


hN = 1 – 0.12 = 0.88 L = 80 mm d2 = 6 mm = 270°C
(ii) Velocity of steam at throat
To find
(i) Temperature of steam at throat, V2 = 44.72 ¥ h1 – h2
(ii) Velocity of steam at throat, and = 44.72 ¥ 3150 – 2992
(iii) Cone angle of divergent portion of nozzle.
= 562.14 m/s
Assumptions The mass-flow rate of steam through nozzle
(i) Isentropic expansion in convergent portion of Ê pˆ 2
nozzle. A2 V2 ÁË 4 ˜¯ d2 V2
(ii) Inlet velocity of steam to nozzle is negligible. ms = =
v2 v2
(iii) Isentropic index for steam expansion as 1.3.
Ê pˆ
¥ (6 ¥ 10 –3 ) 2 ¥ 562.14
ËÁ 4 ¯˜
=
0.24
= 0.0662 kg/s
Using nozzle efficiency for divergent portion
h –h
hN, div = 2 3
h2 – h3s
or h3 = h2 – hN (h2 – h3s)
= 2992 – 0.88 ¥ (2992 – 2580)
= 2629.44 kJ/kg
From Mollier chart at
p3 = 1 bar h3 = 2629.44 kJ/kg
v3 = 1.75 m3/kg
The velocity at nozzle exit
Analysis The critical pressure ratio for nozzle V3 = 44.72 ¥ 3150 – 2629.44
p* = 1020.32 m/s
= 0.546
p1 The exit area of the nozzle
Critical pressure of steam mv 0.0662 ¥ 1.75
A3 = s 3 =
p2 = p* = 0.546 ¥ 15 bar = 8.19 bar V3 1020.32
which is greater than exit pressure of steam. Thus the = 1.1354 ¥ 10–4 m2
steam expands in divergent portion of nozzle also. = 113.54 mm2
From Mollier chart; p
Further, A3 = d32 fi d3 = 12.02 mm
Nozzle entry: p1 = 15 bar, T1 = 350°C 4
h1 = h01 = 3150 kJ/kg (iii) Cone angle of divergent portion Refer Fig.
Throat: (critical) condition: p2 = 8.19 bar 21.17.
s1 = s2
h2 = 2992 kJ/kg L = 80 mm
v2 = 0.24 m3/kg,
T2 = 270°C a/2
a d2 d* = 6 mm d3 = 12.02 mm
Nozzle exit: State 3s
p3 = 1 bar, s1 = s3s
h3s = 2580 kJ/kg
688 Thermal Engineering

Ê aˆ r –r d – d2 Since steam is superheated at the entrance of the


tan Á ˜ = 3 2 = 3 nozzles, thus isentropic index
Ë 2¯ L 2L
k = 1.3
12.06 – 6
= = 0.0383 p2
2 ¥ 80 and critical pressure ratio, = 0.546
p1
a
or = 2.156° Thus, throat pressure p2 = 0.546 p1 = 5.46 bar
2
which is greater than exit pressure, thus the expansion
Cone angle, a = 4.31° remains continue in divergent protion.
Entry of steam: p1 = 10 bar, T1 = 250°C, h1 = 2943 kJ/kg
Example 21.10 The nozzles of a turbine are
supplied with superheated steam at 10 bar, 250°C. Throat condition:
The steam leaves the nozzles at a pressure of 1.0 bar. p2 = 5.46 bar s1 = s 2
The steam consumption for the turbine is 16 kg/kWh, h2 = 2805 kJ/kg v2 = 0.366 m3/kg
when it develops 225 kW. If the throat diameter is Nozzle exit: h3s = 2515 kJ/kg ◊ K s1 = s 3
0.8 cm, determine the number of nozzles required and x = 0.947 vg3 = 1.694 m3/kg
exit diameter of the nozzles, assuming that 10% of the
Velocity at throat; V2 = 44.72 ¥ h1 – h2
total heat drop is lost in overcoming the friction in the
divergent portion only. Neglect the velocity of approach. = 44.72 ¥ 2943 – 2805
= 525.35 m/s
Solution
The mass flow rate per nozzle at throat,
Given Expansion of steam through nozzles of a turbine AV
m1 = 2 2
p1 = 10 bar T1 = 250°C V1 = 0 v2
p3 = 1.0 bar P = 225 kW ssc = 16 kg/kWh p
(0.8 ¥ 10 - 2 ) 2 ¥ 525.35
d2 = 0.8 cm hN = 1 – 0.1 = 0.9 4
=
0.366
To find
= 0.0721 kg/s
(i) Number of nozzles used in turbine,
(ii) Exit diameter of the nozzle. (i) Number of nozzles required
ms 1 kg/s
= = = 13.86 = 14 nozzles
m1 g 0.0721 k /s
The enthalpy at the nozzle exit is determined by
using nozzle efficiency
h2 – h3
hN =
h2 – h3s
or h3 = 2805 – 0.9 ¥ (2805 – 2515)
= 2544 kJ/kg
The velocity at nozzle exit
V3 = 44.72 ¥ 2943 – 2544
= 893.28 m/s
The exit area of the nozzle
m1 ( xv g3 ) 0.0721 ¥ 0.947 ¥ 1.694
A3 = =
Analysis The mass-flow rate through the nozzles is V3 893.28
determined as = ¥ 10−4 m2
1.295
sss ¥ P 16 ¥ 225 (ii) Exit diameter of the nozzle
ms = = = 1 kg/s p
3600 3600 A3 = d32
4
Steam Nozzles 689

and critical pressure ratio,


4 A3 4 ¥ 1.295 ¥ 10 - 4 p2
or d3 = =
p p = 0.546
p1
= 0.01284 m = 12.84 mm
Throat pressure p2 = 0.546 p1
Example 21.11 A steam turbine develops 190 kW = 0.546 ¥ 11.8 bar
with a consumption of 18 kg/kWh. The pressure and = 6.44 bar
temperature of steam enering the nozzle are 11.8 bar which is greater than exit pressure, hence the
and 220°C, respectively. The steam leaves the nozzles at expansion continues in divergent portion.
1.18 bar. The diameter of nozzle at the throat is 8 mm.
Entry of steam: p1 = 11.8 bar, T01 = 220°C
find the number of nozzles.
h1 = 2850 kJ/kg
If 8 per cent enthalpy drop is lost due to friction in
Throat condition: p2 = 6.44 bar
diverging part of the nozzle, determine the diameter at
the exit of the nozzle and exit velocity of leaving steam. s1 = s2
h2 = 2740 kJ/kg
Solution v2 = 0.368 m3/kg
Nozzle exit: h3s = 2525 kJ/kg ◊ K
Given Expansion of steam through nozzles of a turbine
s1 = s3, x3 = 0.93
p1 = 11.8 bar, T1 = 220°C ssc = 18 kg/kWh
vg3 = 1.44 m3/kg
p3 = 1.18 bar P = 190 kW d2 = 8 mm
hN = 1 – 0.08 = 0.92 Velocity at throat;
To find V 2 = 44.72 ¥ 2850 – 2740
(i) Number of nozzles used in turbine, = 469.04 m/s
(ii) Exit diameter of the nozzle, and The mass flow rate per nozzle at throat
(iii) Exit velocity of leaving steam. p
(8 ¥ 10 – 3 ) 2 ¥ 469.04
A2 V2 4
m1 = =
v2 0.368
= 0.064 kg/s
(i) Number of nozzles required
ms 0.95 kg/s
= =
m1 0.064 kg/s
= 14.83 ª 15 nozzles
The enthalpy at the nozzle exit is determined by
using nozzle efficiency
h2 – h3
hN, div =
h2 – h3s
or h3 = 2740 – 0.92 ¥ (2740 – 2525)
= 2542.2 kJ/kg
(ii) The velocity at nozzle exit
Analysis The mass flow rate through the nozzles is
determined as V3 = 44.72 ¥ 2850 – 2542.2
sss ¥ p 18 ¥ 190 = 784.58 m/s
ms = = = 0.95 kg/s
3600 3600 The exit area of the nozzle
Since steam is superheated at the entrance of the m1 ( xv g3 )
nozzles, thus isentropic index A3 =
V3
k = 1.3
690 Thermal Engineering

Analysis The mass flow rate through the nozzles is


0.064 ¥ (0.93 ¥ 1.44)
= determined as
784.58
sss ¥ P
= 1.0924 ¥ 10– 4 m2 ms =
3600
(iii) Exit diameter of the nozzle
9 ¥ 220
p 2 = = 0.55 kg/s
A3 = d3 3600
4
(i) Throat pressure
4 A3 4 ¥ 1.0924 ¥ 10 – 4 Since steam is wet at the entrance of the nozzles,
or d3 = =
p p thus isentropic index
= 0.01179 m = 11.79 mm k = 1.113
and critical pressure ratio,
Example 21.12 Steam at a pressure of 15 bar 1.113
and a dryness fraction of 0.97 is discharged through p2 Ê 2 ˆ 1.113 – 1
= Á = 0.582
a convergent–divergent nozzle to a back pressure of p1 Ë 1.113 + 1˜¯
0.2 bar. The steam consumption is 9 kg/kWh. If the power
developed is 220 kW, determine Throat pressure
p2 = 0.582 p1
(a) Throat pressure,
= 0.582 ¥ 15 bar = 8.73 bar
(b) Number of nozzles required, if each nozzle has
throat of rectangular cross-section of 4 mm ¥ 8 mm. which is greater than exit pressure, hence the
expansion continues in divergent portion.
(c) The cross-sectional area of the exit rectangle
if 12% of the overall isentropic enthalpy drop (ii) Number of nozzles required
reheats steam by friction in divergent portion Entry of steam: p1 = 15 bar x1 = 0.97
h1 = 2735 kJ/kg
Solution Throat condition:
p2 = 8.73 bar s1 = s2
Given Expansion of steam through a convergent
divergent nozzles h2 = 2660 kJ/kg
p1 = 15 bar x1 = 0.97 ssc = 9 kg/kWh v2 = 0.24 m3/kg
p3 = 0.2 bar P = 220 kW A2 = 4 mm ¥ 8 mm Nozzle exit: h3s = 2105 kJ/kg ◊ K s1 = s3,
hN = 1 – 0.12 = 0.88 x3 = 0.785, vg3 = 6.1 m3/kg.
Velocity at throat;
To find
V2 = 44.72 ¥ 2735 – 2660
(i) Throat pressure,
(ii) Number of nozzles used in turbine, and = 387.28 m/s
(iii) Exit cross section of the nozzle. The mass flow rate per nozzle at throat
A2 V2 (8 ¥ 4) ¥ 10 - 6 ¥ 387.28
m1 = =
v2 0.24
= 0.0516 kg/s
Number of nozzle
ms 0.55 kg/s
= = 10.65 ª 11 nozzles
m1 0.0516 kg/s
The enthalpy at the nozzle exit is determined by
using nozzle efficiency in divergent portion
h1 – h3
hN, div =
h1 – h3s
Steam Nozzles 691

or h3 = 2735 – 0.88 ¥ (2735 – 2110) Fig. 21.22(a) and (b), respectively. The point a is
= 2185 kJ/kg on saturation line, where in normal condition, the
(iii) Exit cross-sectional area of the nozzle condensation of steam begins. However, if the point
The velocity at nozzle exit a is reached in the divergent section of the nozzle,
V3 = 44.72 ¥ 2735 – 2185 the condensation does not occur at this point. The
steam exists as a vapour between points a and b,
= 1048.77 m/s
but the temperature is lower than the saturation
The exit area of the nozzle
temperature for a given pressure. The point b is
m1 ( xv g3 ) 0.0516 ¥ (0.785 ¥ 6.1)
A3 = = known as metastable state. The point b lies on the
V3 1048.77 pressure line p2 produced from the superheated
= 2.356 ¥ 10– 4 m2 = 235.6 mm2 region.

Point on
saturation line

a b

When steam expands in the nozzle, its pressure and Point where
temperature drop simultaneously, but steam does condensation occurs
1 very abruptly
not start condensing, and expands as superheated
vapour even as it reaches the saturation line. This
process is very quick, the residence time of steam
velocity is very high, and there may not be sufficient The temperature of supersaturated vapour at p2
time for necessary heat transfer and formation of is Tb, which is less than the saturation temperature
liquid droplets. Consequently, the condensation of T2 corresponding to p2. This vapour is said to be
vapour is delayed for a little while. This phenomenon supercooled, and the degree of supercooling (or
is known as supersaturation and steam, which exits undercooling) is given by (T2 – Tb).
in the wet region without containing any mositure The degree of supersaturation is defined as the
is called supersaturated steam. Such expansion of ratio of actual pressure at the point b to the saturation
steam is called a supersaturated expansion. The pressure corresponding to the temperature Tb of
point at which condensation occurs may be within steam at the point b.
the nozzle or after the vapour leaves the nozzles. pb
Degree of supersaturation = ...(21.21)
pb, sat
The locus of points where the condensation will
The steam below the saturation curve up to the take place regardless of initial temperature and
point where condensation begins neither in stable pressure of steam at the nozzle entrance is called
equilibrium nor in unstable equilibrium. Since the Wilson line. The Wilson line lies between 95
fluid is a homogenous vapour below saturation and 96 per-cent dryness fraction curves in the
temperature, the steam in this condition is said to saturation region as shown on the h–s diagram
be in a metastable state. Fig. 21.22(b) and it is approximated by the 96%
Consider superheated steam is expanding dryness fraction line. Therefore, the high-velocity
isentropically in a convergent–divergent nozzle steam flowing through the nozzle is assumed to
as shown in Fig. 21.21. The expansion process begin to condense when the 96% dryness fraction
is represented on T–s and h–s diagrams in line in crossed.
692 Thermal Engineering

T p1 2. Exit velocity is slightly reduced.


1
3. The specific volume is reduced, thus the
mass flow rate is increased by 2 to 5%.
p2 4. Entropy is slightly increased, because abrupt
p2 2
a condensation takes place at constant enthal-
py line.
b
Since the steam during supersaturted expansion
s
(a)
remains in superheated state, it can be treated as a
gas with an isentropic index of 1.3 and problems of
h p1 supersaturated flow can be solved without use of
1
Mollier chart.

a p2 Example 21.13 Determine the throat area, exit area


Saturation
and exit velocity for a steam nozzle to pass a mass flow of
b
2 line 0.2 kg/s, when inlet conditions are 10 bar and 250°C and
pb
p2 Wilson line
the final pressure is 2 bar. Assume expansion is isentropic
(x = 0.96) and inlet velocity is negligible. Use pv1.3 = constant. Do
s not use h–s chart.
(b)
Solution
Given Steam expansion through a nozzle
m = 0.2 kg/s p1 = 10 bar
It is observed that the enthalpy drop in supersaturated T1 = 250°C p3 = 2 bar
flow (hl – hb) is less than the enthalpy drop (h1 – h2) 1.3
s1 = s2 pv = C, V1 = 0
for expansion in equilibrium conditions. Thus, the Specific volume of steam at 10 bar, 250°C,
exit velocity for supersaturated flow is less than that v1 = 0.233 m3/kg (from steam tables)
for equilibrium flow. However, the difference in
enthalpy drop is small and square root of enthalpy To find
drop is used for evaluating exit velocity. Thus, the (i) Throat area of nozzle,
effect on the exit velocity is also small. (ii) Exit velocity of steam from nozzle, and
Further, the specific volume at exit with (iii) Exit area of nozzle.
supersaturated flow vb is considerably less than the Analysis
specific volume at exit with equilibrium flow, v2. (i) For given, n = 1.3, the critical pressure ratio
Now the mass-flow rate through an area A2 as given p2
earlier for equilibrium flow, = 0.546
p1
A2 Vb The pressure of steam at throat
ms = …(21.22)
vb p2 = 0.546 ¥ 10 bar = 5.46 bar
Since vb < v2, thus the mass flow rate with The velocity of steam at the throat, Eq. (21.9);
supersaturated flow is greater than the mass flow
rate with equilibrium flow. È n –1˘
Ê 2n ˆ Í Ê p2 ˆ n ˙
The effect of supersaturation flow can be V2 = ÁË n – 1˜¯ p1v1 Í1 – ÁË p ˜¯ ˙
summarised below: Í 1 ˙
Î ˚
1. Actual heat drop is slightly reduced.
Steam Nozzles 693

Ê 2 ¥ 1.3 ˆ Solution
5 2
= ÁË 0.3 ˜¯ ¥ (10 ¥ 10 N / m )
Given Metastable expansion of steam through nozzles
È 0.3 ˘ without friction
Í Ê 5.46 ˆ 1.3 ˙ ms = 5.2 kg/s N = 6 nozzles
¥ (0.233 m3/ kg) ¥ Í1 – Á
Ë 10 ˜¯ ˙ p1 = 30 bar T1 = 350°C = 623 K
Í ˙
Î ˚ p2 = pb = 4 bar
= 513 m/s
The specific volume at the throat To find
1 1 (i) Diameter of nozzles at exit,
Ê p1 ˆ n Ê 10 ˆ 1.3
v2 = v1 Á ˜ = 0.233 ¥ Á (ii) Degree of undercooling,
Ë p2 ¯ Ë 5.46 ˜¯ (iii) Degree of supersaturation,
3
= 0.371 m /kg (iv) Loss in available enthalpy drop,
Area of throat, using continuity equation (v) Entopy increase,
m v2 0.2 ¥ 0.371 (vi) Ratio of mass-flow rate with metastable expansion
A2 = =
V2 513 to that of expanssion in thermal equilibrium.
= 1.446 ¥ 10–4 m2
= 1.446 cm2
(ii) Exit velocity of steam,
È n –1˘

V3 = Ê 2n ˆ Í Ê p3 ˆ n ˙
ÁË n – 1˜¯ p1v1 Í1 – ÁË p ˜¯ ˙
Í 1 ˙
Î ˚
È 0.3 ˘
= Ê 2 ¥ 1.3 ˆ 5 Í Ê 2 ˆ 1.3 ˙
(10 ¥ 10 ¥ 0.233) ¥ Í1 - Á ˜ ˙
ËÁ 0.3 ¯˜ Ë 10 ¯
Í ˙
Î ˚
= 791.5 m/s
(iii) The specific volume of steam at exit,
1 1
Ê p ˆn Ê 10 ˆ 1.3
v3 = v1 Á 1 ˜ = 0.233 ¥ Á ˜
Ë p3 ¯ Ë 2¯ Assumptions
= 0.803 m /kg 3 (i) Negligible inlet velocity of steam
(ii) Index of steam expansion, n = 1.3.
The exit area of nozzle,
m v3 0.2 ¥ 0.803 Properties of steam From Mollier chart,
A3 = =
V3 791.5 Steam inlet: 30 bar, 350°C
= 2.030 ¥ 10– 4 m2 = 2.030 cm2 h1 = 3115 kJ/kg v1 = 0.09 m3/kg
State 2: After expansion in thermal equilibrium,
Example 21.14 5.2 kg/s of steam at 30 bar and p = 4 bar
350°C is supplied to a group of six nozzles on a wheel
h2 = 2675 kJ/kg v2 = 0.46 m2/kg
diameter maintained at 4-bar pressure. Determine for
metastable expansion, Analysis
(a) Diameter of nozzles at exit without friction, (i) Diameter of nozzles
(b) Degree of under–cooling and supersaturation, In metastable state, the steam behaves as gas,
(c) Loss in available heat drop due to ireversibiltiy,
(d) Increase in entropy, È n –1˘
Ê n ˆ Í Ê pb ˆ n ˙
(e) Ratio of mass-flow rate with metastable expansion h1 – hb = Á p1v1 Í1 – Á ˜ ˙
Ë n – 1˜¯ Í Ë p1 ¯ ˙
to that of expansion in thermal equilibrium.
Î ˚
694 Thermal Engineering

1.3 From steam tables at 4 bar, Tsat = 143.63°C


= ¥ 30 ¥ 102 ¥ 0.09 Degree of subcooling (undercooling)
1.3 – 1
= Tsat – Tb = 143.63 – 118.33 = 25.3°C
È 1.3 – 1 ˘
Í Ê 4 ˆ 1.3 ˙ (iii) Degree of supersaturation
¥ Í1 – Á ˜ ˙ Saturation pressure corresponding to 118.33°C,
Ë 30 ¯
Í ˙ from steam tables; pb, sat ª 1.88 bar
Î ˚
= 435 kJ/kg Degree of supersaturation
The velocity at the metastable state b, pb 4 bar
= = 2.12
È n –1˘ pb, sat a1.88 b r
Ê 2n ˆ Í Ê pb ˆ n ˙
Vb = ÁË n – 1˜¯ p1v1 Í1 – ÁË p ˜¯ ˙ (iv) Loss in available enthalpy drop
Í 1 ˙ Isentropic enthalpy drop,
Î ˚
Dhisentropic = h1 – h2 = 3115 – 2675
Ê 2 ¥ 1.3 ˆ 5 = 440 kJ/kg
= ÁË 0.3 ˜¯ ¥ (30 ¥ 10 ¥ 0.09) Loss in available enthalpy drop
= Dhisentrop – (h1 – hb) = 440 – 435
È 0.3 ˘ = 5 kJ/kg
Í Ê 4 ˆ 1.3 ˙
=¥ Í1 - ÁË 30 ˜¯ ˙
(v) Increase in entropy
Í ˙ Loss in enthalpy drop
Î ˚ Ds =
= 932.8 m/s (Tsat + 273)

The specific volume of steam at the state b, 5 kJ / kg


= = 0.012 kJ/kg ◊ K
1 1 (143.63 + 273)
Ê p ˆn Ê 30 ˆ 1.3
vb = v1 Á 1 ˜ = 0.09 Á ˜ (vi) Ratio of mass-flow rate of two types of expansions
Ë pb ¯ Ë 4¯ Velocity at nozzle exit corresponds to isentropic
enthalpy drop (expansion in thermal equilibrium)
= 0.424 m3/kg
Mass flow rate per nozzle V2 = 2Dhisentropic = 2 ¥ 440 ¥ 103
ms 5.2
m = = = 0.867 kg/s = 938 m/s
N 6
Then the mass-flow rate for equilibrium
The area at nozzle exit, (continuity equation)
conditions
m v2 0.867 ¥ 0.424
Ab =
Vb
=
932.8 A2 V2 Ab Vb 3.939 ¥ 10 - 4 ¥ 938
m= = =
v2 v2 0.46
= 3.939 ¥ 10–4 m2
= 0.803 kg/s
p 2 4 Ab
and Ab = d fid=
4 p ms 0.867
Ratio of mass flow = = = 1.078
= 0.0224 m = 22.4 mm m 0.803
(ii) Degree of undercooling
Example 21.15 A convergent–divergent nozzle re-
Temperature Tb after expansion is determined as ceives steam at 7 bar and 200°C and expands it isentopi-
n –1 0.3 cally to 3 bar. Neglect the inlet velocity, calculate the exit
Tb Ê pb ˆ n Ê 4 ˆ 1.3 area required for a mass flow rate of 0.1 kg/s
= = Á ˜ = 0.628
T1 ÁË p1 ˜¯ Ë 30 ¯
(a) when flow is in equilibrium throughout,
or Tb = 0.628 ¥ (623 K) = 391.33 K (b) when the flow is supersaturated with pv1.3 =
= 118.33°C constant.
Steam Nozzles 695

Soltuion (ii) When steam expansion is supersaturated


Here, h1 = 2846 kJ/kg
Given Expansion of steam though a convergent–
v1 = 0.3001 m3/kg (from steam tables)
divergent nozzle.
After expansion of steam as a gas
p1 = 7 bar T1 = 200°C V1 = 0 1 1
p2 = 3 bar ms = 0.1 kg/s n = 1.3 Ê p ˆn Ê 7 ˆ 1.3
vb = v1 Á 1 ˜ = 0.3001 ¥ Á ˜
To find Exit area of nozzle, when Ë pb ¯ Ë 3¯
(i) Expansion of steam is in thermal equilibrium, = 0.576 m3/kg
(ii) Expansion of steam is supersaturated. The velocity of steam after supersaturation
Analysis expansion

(i) When expansion is in thermal equilibrium È n –1˘


Ê 2n ˆ Í Ê pb ˆ n ˙
throughout Vb = ÁË n – 1˜¯ p1v1 Í1 – ÁË p ˜¯ ˙
From Mollier chart and steam table Í 1 ˙
Î ˚
h1 = 2846 kJ/kg h2 = 2682 kJ/kg
vg2 = 0.6057 m3/kg Ê 2 ¥ 1.3 ˆ 5
x2 = 0.98 = ÁË 0.3 ˜¯ ¥ 7 ¥ 10 ¥ 0.3001
The velocity of steam at nozzle exit
V2 = 44.72 ¥ 2846 – 2682 È 0.3 ˘
Í Ê 3 ˆ 1.3 ˙
= 572.7 m/s =¥ Í1 - ÁË 7 ˜¯ ˙ = 568.63 m/s
Using continuity equation to obtain exit area of Í ˙
Î ˚
nozzle,
Using continuity equation, for exit area
ms ( xv g2 ) m vb
A2 = Ab =
V2 Vb
0.1 ¥ (0.98 ¥ 0.6057) m vb 0.1 ¥ 0.576
= = =
572.7 Vb 568.63
= 1.036 ¥ 10–4 m2 = 1.0127 ¥ 10– 4 m2 = 101.27 mm2
= 103.6 mm2

Summary

area, in which the pressure energy of a fluid is small as compared to exit velocity of fluid at
converted into kinetic energy. the nozzle exit. Therefore, the inlet velocity is
considered negligible. The exit velocity of fluid
and temperature. Thus ideal gas relations are from a nozzle can be expressed as
not applicable to steam. An isentropic index g
V 2 = 44.72 ¥ ( h1 - h2)
is replaced by n, the ratio of specific heats, in
certain relations for steam.
n = 1.135 for dry saturated steam and within the fluid makes the fluid flow non-
n = 1.3 for superheated steam isentropic but adiabatic. The nozzle efficiency is
critical pressure ratio for dry saturated steam defined as
is 0.577 and that for superheated steam is 0.546.
696 Thermal Engineering

h1 – h2 Acutal enthalpy drop degree of supersaturation is defined as the


hN = = ratio of actual pessure to the saturation pressure
h1 – h2 s Isentropic enthalpy drop
corresponding to temperature of steam at the
location condensation.
when it reaches the saturation line and exits as a
supersaturation,
supersaturated substance in the wet region. Such
1. Actual heat drop is slightly reduced
expansion is said to be supersaturated expansion.
Supersaturation state is in non-equilibrium, and 2. Exit velocity is slightly reduced
thus is called metastable state. 3. The specific volume is reduced, thus the
mass flow rate is increased by 2 to 5%

Glossary
Critical pressure Pressure of steam at the throat Metastable State Supersaturation state of a fluid
Critical velocity Velocity of steam at throat corresponds Degree of supercooling Difference between saturation
to maximum discharge condition temperature and temperature of supersaturated vapour
Supersaturation Existance of fluid in superheated state Degree of supersaturation Ratio of actual pressure to
in wet region the saturation pressure corresponding to temperature of
steam condensate.

Review Questions
1. What is the function of a steam nozzle? 6. What is the significance of critical pressure ratio
2. What are the types of nozzles? Explain with neat on discharge through the nozzle?
diagrams. 7. Discuss the effect of irreversibilities on nozzle ef-
3. Derive an equation for discharge through the noz- ficiency with the help of a T–s diagram.
zle. 8. Discuss the supersaturated flow of steam.
4. Derive an expression for condition of maximum 9. Define metastable state and degree of supersatu-
discharge. ration.
5. Define critical velocity, critical pressure ratio. 10. What are the effects of supersaturation flow?

Problems
1. Steam enters a convergent–divergent nozzle at The back pressure of the nozzle is maintaind at
11 bar, dry and saturated at a rate of 0.75 kg/s, 0.85 bar. The throat area is 2.4 cm2. Find the
and expands isentropically to 2.7 bar. Neglect the maximum mass-flow rate and the required exit
inlet velocity and assuming the expansion follows area. Assume the index of expansion to be 1.35
a law pv1.135 = constant, determine throughout. [0.3546 kg/s, 6.575 cm2]
(a) the area of the nozzle throat, 3. Steam enters a convergent nozzle at a pressure
(b) the area of the nozzle exit. of 5 bar and 180°C and a velocity of 200 m/s.
[(a) 474 mm2, (b) 646 mm2] The discharge pressure is maintained at 3 bar.
2. Steam at a pressure of 10 bar and 0.95 dry Determine the required throat area for a mass-
expands in a convergent–divergent nozzle. flow rate of 0.5 kg/s and a nozzle efficiency of
96%. [6.284 cm2]
Steam Nozzles 697

4. Dry, saturated steam at a pressure of 10 bar is Also, determine the degree of undercooling at the
discharged through a nozzle at a back pressure of throat. [486.8 m/s; 4.41 cm2; 3°C]
1 bar. Determine the throat and exit diameters for 11. Steam is expanded in a nozzle from 8 bar, 200°C
a mass-flow rate of 1 kg/s and nozzle efficiency of to a pressure of 1.4 bar. Determine the isentropic
90%. [6.99 cm2; 18.17 cm2] heat drop, actual heat drop and the degree of
5. Steam at 12 bar and 220°C is supplied to a steam subcooling.
turbine at the rate of 6.5 kg per kW per hour when [316 kJ/kg; 299.6 kJ/kg; 66°C]
the turbine develops 220 kW. The back pressure is 12. Steam at a pressure of 20 bar, 250°C expands
maintained at 1.2 bar. The nozzle throat diameter in a convergent–divergent nozzle up to the exit
is 0.8 cm. The fricition losses in the convergent pressure of 2 bar. Assuming a nozzle coefficient
portion are negligible. Find the mass flow rate and of 0.94 for supersaturated flow up to the throat
number of the nozzle required and the diameter at and nozzle efficiency as 90%, find
the exit of the nozzle. (i) Velocity at throat,
[0.07234 kg/s; 6 nozzles; 1.183 cm] (ii) Mass-flow rate, if the throat diameter is
6. A convergent–divergent nozzle is supplied steam 1 cm,
(iii) Velocity and diameter of the nozzle at the
at a pressure of 10 bar, dry, saturated. The exit
pressure is 1 bar. The friction losses are equivalent exit.
to 5% of heat drop in the convergent portion and [(i) 469.2 m/s, (ii) 0.205 kg/s, (iii) 1.55 cm]
10% of heat drop in the divergent portion. For a 13. Steam at a pressure of 15 bar and 200°C is
throat diameter of 1.47 cm, determine the mass supplied to a nozzle and it leaves at a pressure of
flow rate and the exit diameter of the nozzle. 1 bar. Expansion is supersaturated up to the throat
Also, find the semi cone angle of the divergent and in thermal equilibrium afterwards. For the
portion if its length is 2.85 cm. mass-flow rate of 0.1 kg/s, find the exit velocity,
exit area, degree of undercooling at the throat and
[0.236 kg/s; 2.37 cm; 8.65°]
its degree of supersaturation.
7. A nozzle is supplied with steam at 15 bar, 250°C
[956.6 m/s; 1.5 cm2; 33°C; 2.375]
with negligible velcosity and it leaves the nozzle
14. Steam at 15 bar and 200°C is supplied to a
at 1.2 bar pressure. There are 10% friction losses
convergent–divergent nozzle against a back
throughout the passage of the nozzle. Determine pressure of 4 bar. Expansion is supersatuated up
the maximum mass-flow rate and the exit diam- to the throat and the nozzles are rectangular in
eter, if the throat diameter is 0.8 cm. shape, its width being 2.5 times the breadth. For a
[0.1011 kg/s; 1.364 cm] mass-flow rate of 0.3 kg/s, find
8. Eight nozzles are required to discharge steam (i) Dimensions of the nozzle at the exit,
to an impulse turbine from a pressure of 10 bar (ii) Degree of undercooling and supersaturation,
and 350°C. The back pressure of the turbine is (iii) Increase in entropy.
maintained at 0.14 bar. The nozzle efficiency is [(i) 0.8415 cm, 2.1 c, (ii) 23°C, 2.375,
90%. If the throat diameter for each nozzle is (iii) 0.005 kJ/kg ◊ K]
0.8 cm, find the total mass-flow rate and exit 15. Steam is expanded in a set of nozzles from
diameter of each nozzle. [0.516 kg/s; 2.283 cm] 10 bar and 200°C to 5 bar. What type of nozzle it
9. Steam at 10 bar and 0.95 dry expands in a nozzle is? Neglect the initial velocity, find the minimum
up to 0.12 bar pressure. The mass-flow rate is area of nozzle required to allow a mass flow rate
0.5 kg/s. The index of expansion may be of 3 kg/s under the given conditions. Assume is-
calculated using Zeuner’s equation given by n = entropic expansion of steam. [21 cm2]
1.035 + 0.1x where, ‘n’ is the index and ‘x’ is the 16. Dry, saturated steam at 9 bar expands through
dryness fraction. [2069 cm; 7.763 cm] two nozzles to 1 bar. The throat diameter of the
10. 40.86 kg/min. of steam is supplied to a conver- nozzle is 2.5 mm. Assuming nozzle efficiency of
gent–divergent nozzle at 11 bar and 220°C. If 90%, find the quantity of steam used per hour and
the expansion is under metastable conditions, power produced by steam at the nozzle exit.
determine the velocity and the area at the throat. [43.2 kg/h and 4 kW]
698 Thermal Engineering

Objective Questions
1. Steam nozzle converts 7. Nozzle efficiency is defined as
(a) heat energy of steam into pressure energy (a) ratio of isentropic heat drop to useful heat
(b) heat energy of steam into kinetic energy drop
(c) pressure energy of steam into heat energy (b) ratio of useful heat drop to isentropic heat
(d) pressure energy of steam into potential drop
energy (c) product of useful heat drop and isentropic
2. A nozzle is designed for heat drop
(a) maximum pressure at the outlet (d) none of the above
(b) maximum discharge 8. When a nozzle operates with maximum mass
flow, it is said to be
(c) maximum pressure and maximum discharge
(a) under expanding flow
(d) maximum kinetic energy at the outlet
(b) over expanding flow
3. Ideal expansion of steam through a nozzle is con-
sidered (c) choked flow
(d) none of the above
(a) isothermal
9. For supersaturated flow in the nozzle, the dis-
(b) adiabatic
charge
(c) polytropic
(a) increases
(d) none of the above
(b) decreases
4. Presence of frictional effect during flow through
(c) remains constant
the nozzle
(d) none of the above
(a) reduces the exit velocity
10. For the flow through the convergent and divergent
(b) increases the exit velocity
nozzle, whole of friction loss is assumed
(c) has no effect on exit velocity
(a) in the converging portion
(d) none of the above
(b) in the divergent portion
5. Frictional losses in the nozzle
(c) between inlet and outlet
(a) reduces the heat drop
(d) none of the above
(b) increases the heat drop
11. The presence of friction in the nozzle
(c) has no effect on heat drop
(a) increases the final dryness fraction of steam
(d) none of the above
(b) decreases the final dryness fraction of steam
6. Actual exit velocity of steam from a nozzle con-
(c) it does not affect the dryness fraction of
sidering friction coefficient k and negligible inlet
steam
velocity, can be expressed as
(d) none of the above
(a) V 2 = 91.53 ¥ k ( h1 – h2 ) 12. When the back pressure of a nozzle is below the
(b) V 2 = 100 ¥ k ( h1 – h2 ) critical pressure, the nozzle is said to be
(a) under expanding flow
(c) V 2 = 44.72 ¥ k ( h1 – h2 ) (b) over expanding flow
(c) choked flow
(d) V 2 = 9.81 ¥ k ( h1 – h2 )
(d) none of the above

12. (c) 11. (a) 10. (b) 9. (a)


8. (c) 7. (b) 6. (c) 5. (a) 4. (a) 3. (b) 2. (d) 1. (b)
Answers
Steam Turbines 699

22
Steam Turbines

Introduction
A steam turbine is a prime mover in which rotating shaft work is developed by a steadily flowing fluid.
The work is produced by changing the momentum of steam as it passes through a rotor of the turbine. The
change in angular momentum of steam causes the torque on the rotor, thus the rotor spins. Therefore, a
steam turbine is a rotodynamic machine.
The steam turbine essentially contains two parts: (i) nozzles, and (ii) moving curved blades. The high-
velocity steam jet coming out of the nozzle impringes on the curved blades, and the steam glides tangentially
on the curved blades fixed on the periphery of a moving wheel. The motive force is exterted on the blades
due to change in direction of flowing steam which causes the blades to rotate.
Since the steam undergoes a continuous steady flow process and the speed of fluid is very high, thus a
turbine handles a large mass of fluid and produces a large power. Steam turbines are used to drive electric
generators in power plants to produce electricity. They are also used to propel large ships, ocean liners,
submarines, and so on. The small steam turbines are used to drive the fans, compressors and pumps.

The first steam turbine was made by Hero of


Alexandria, 2000 years ago, i.e., in the first century
AD. This turbine worked on the pure reaction
principle and no power was produced by it. In fact,
it was considered as an amazing toy as shown in
Fig. 22.1. It consisted of a hollow ball mounted in
between two pivots. The ball was equipped with
two converging tubes (nozzles). The generated
steam was supplied to the ball through one pivot.
The steam was expanded through the converging
tubes to atmospheric pressure, thus causing a
reactive force on the ball and making it to rotate In the year 1888, Gustaf de Laval of Stockholm,
between the pivots. one of the prominent pioneers of the steam turbine,
700 Thermal Engineering

built a reaction turbine on the principle of Hero’s The torque acting on the rotating wheel of radius
turbine, which attained a speed of 42000 rpm with R is given by
a tip speed of 180 m/s. However, the efficiency was T = FR = ms R [V1 cos a + V2 cos b]
very low and hence commercial turbines could not ...(22.2)
be built on the pure reaction principle. In the year The linear velocity of blades
1884, about four years prior to the development of 2p NR p DN
the de Laval reaction turbine, Sir Charles Parson u = =
60 60
expressed that moderate surface velocities and The rate of work done by the blades
rotor speeds were essential to turn it into a prime Power, P = Force ¥ linear blade velocity
mover. This gave the principle of compounding of
= ms [V1 cos a + V2 cos b ] ¥ u
the turbine in series.
2p NR
= ms [V1 cos a + V2 cos b ]
60
2p N T
or P= (W ) ...(22.3)
60
Figure 22.2 shows a steam jet gliding on a vane where N = rotation per minute of rotating wheel.
tangentially at an angle a and leaving the vane at T = ms R(V1 cos a + V2 cos b )
an angle b after gliding over the curved surface of
the vane. Due to change in angular momentum of
the steam jet, the blade moves with a linear velocity
u. According to Newton’s second law, the change
in momentum is directly proportional to the force The steam turbines can be classified according to
acting on the blades.
A
(i) Impulse turbines,
(ii) Reaction turbines, and
(iii) Impulse–reaction turbines.
S R R
(i) Axial flow turbine,
(ii) Radial flow turbine, and
(iii) Mixed flow turbine.
S
(i) Single stage turbine, and
(ii) Multi-stage turbine.

Force applied on the vane


The impulse turbines work on the principle of
= Change in momentum of jet
impulse. The steam coming out the boiler expands
= mass flow rate ¥ change in velocity
in the stationary nozzles, where the kinetic energy
F = ms [V1 cos a – (–V2 cos b)] of steam increases at the cost of enthalpy drop. This
= ms [V1 cos a + V2 cos b)] ...(22.1) high-velocity steam jet is directed to glide over the
where ms = mass-flow rate of steam, kg/s curved blades mounted on the periphery of the
V1 = Velocity of steam jet entering, m/s rotor (rotating wheel). When the steam jet glides
V2 = Velocity of steam leaving, m/s tangentially over the curved surface of blades, the
Steam Turbines 701

direction of steam flow is changed. It changes the


momentum of the high-velocity steam jet and thus
impulsive force is exerted in normal direction of
steam jet, which ultimately provides motive force
at the turbine shaft and it spins rapidly. Figure 22.3
shows a schematic of a simple impulse wheel. It
consists of a rotor having a number of curved
blades fixed round the circumference.

The reaction turbines work on the principle


of Newton’s third law of motion; the reaction
(backward) force is always developed opposite
to a certain action. In these turbines the rotation
is caused by the reaction force generated by the
momentum change of fluid accelerating through passing through the moving blades causes
the nozzles that are attached to the rotor (moving a further increase in kinetic energy within
wheel), like a nozzle on rotating shaft as shown in these blades, giving rise to a reaction and
Fig. 22.4 (a). adds to the propelling force, which is applied
through the rotor to the turbine shaft.

Turbine varied (converging type).


The impulse–Reaction turbine works on the
The shape of nozzles in the reaction turbine is
principle of impulse as well as the principle of
not the same as is that of a simple impulse turbine.
reaction.
Instead, they are two-dimensional nozzles, like
channels formed in the passage between the blades of
in fixed nozzles as well as in moving blades. each row. In the reaction turbine, the entire pressure
702 Thermal Engineering

Blade Clearance
Nozzle Casing

Fresh Exhaust
Steam Steam

Labyrinth
packing Roter
drop is obtained gradually and continuously over
a series of fixed and moving blades in succession.
The fixed blades act as guide vanes to regulate the Shaft Shaft
direction of steam flow as well as to expand the Bearing

Blade Motion
steam to a higher velocity. Parson’s turbine is an Nozzle
example of an impulse–reaction turbine.

A simple impulse turbine consists of a set of p1 Velocity


nozzles, a rotor mounted on a shaft, one set of
Pressure of steam

Exit velocity
moving blades attached to the rotor and the casing.
V2
A simple impulse turbine is schematically shown Pressure
Condenser
in Fig. 22.6. The uppermost portion of the diagram pressure
shows a longitudinal section of the upper half of the
V1

p2
turbine, the middle portion of the diagram shows
Entering velocity of steam
the shape of the nozzle and blading and the lower
portion shows the variation of the absolute velocity
and absolute pressure during the flow of steam
through the nozzles and blades. jet cannot be effectively utilized in a single row
In the impulse turbine, the complete expansion of blades. It is therefore, necessary to incorporate
steam (from boiler pressure to condenser pressure) some improvement. It is achieved by using more
takes place in stationary nozzles, the enthalpy than one set of nozzles, blades and rotors in series,
drops and kinetic energy of steam increases, which so that either the steam jet pressure or the steam jet
is converted in to shaft work. The steam passes velocity is regulated by the turbine in stages.
on moving blades at constant pressure, but with
gradual reduction in its velocity.
p1 = p2 The steam supplied to a single-wheel impulse
In an impulse turbine, the absolute velocity of turbine expands completely in the nozzle and leaves
steam coming out the nozzles is about 1050 m/s. If with a high absolute velocity. The steam is directed
this high-velocity steam jet is used on a single row on the rotor at an angle a. Therefore, the absolute
of blades, the rotor speed may reach as 30,000 rpm, inlet velocity has two components tangential
which is too high for practical considerations. and axial as shown in Fig. 22.7. The tangential
Further, the steam leaving the moving blade is component (with direction of rotation) is called the
also at high velocity, thus involves a loss of energy, whirl velocity and is mainly responsible for useful
which is commonly known as carry-over loss or work, while axial (vertical) component is called
leaving loss. Accordingly, the velocity of steam axial or flow velocity, allows the flow of steam
Steam Turbines 703

Vr2 = Relative velocity of steam at outlet, m/s


Vf2 = Axial velocity at discharge, m/s
Vw2 = Whirl velocity (component of V2 in the
direction of rotation)
b = Steam jet outlet angle, degree
f = Blade outlet angle, degree
The relative velocity at the inlet Vr1 is the vector
difference of the absolute velocity V1 and the blade
velocity u. The relative velocity Vr1 makes an angle
q with the axis of rotation. Since the steam does
not expand in the moving blades, thus the relative
velocity Vr2 at the exit in absence of any frictional
effects, will be equal to relative velocity at the inlet.
However, if there is friction then the two relative
velocities are related as
Vr2 = k Vr1 ...(22.4)
where k is called the blade velocity constant and
its value varies from 0.7 to 0.9, which takes into
account the loss of velocity due to friction.

across the wheel. Since the blades are moving with


a velocity u, then the velocity of steam relative to We have already drawn the velocity diagrams for
the blade at the inlet is called the relative velocity, velocities for moving blades at its inlet and exit on
which makes an angle q with the blade. two different planes. Since the blade velocity u is
To summarise, the various terms used in the common in both the vector diagrams of Fig. 22.7,
vector diagram of velocity used in Fig. 22.7 and thus both diagrams can be combined together on
Fig. 22.8 are as given below: using common blade velocity u, as shown in Fig.
( ) Inlet Vector Diagram 22.8 by superimposing one on the other.
V1 = Absolute velocity of steam jet coming out
the nozzle, m/s
a = Angle of steam jet in respect to plane of
blade rotation, degree
u = Linear velocity of blade, m/s
Vr1 = Relative velocity of steam at inlet, m/s
q = Blade inlet angle, degree
Vw1 =Whirl velocity, (component V1 in the
direction of rotation), m/s
Vf1 = axial or flow velocity of steam, m/s
(ii) Exit Vector Diagram
V2 = Absolute velocity of steam at blade outlet,
m/s
1. Draw a horizontal line. Mark a segment AB
equal to blade velocity u to the scale.
704 Thermal Engineering

2. From the point A draw a line AC inclined at 2p NR 2p NT


nozzle angle a and length equal to absolute or W = ms ( Vw) = (W ) ...(22.7)
60 60
velocity of steam V1 (to the scale). where T = ms R (Vw) is turning moment that acts on
3. Join points B and C. The line BC represents the rotor.
relative velocity Vr1 at the inlet. Vw = Vw1 + Vw2
4. From the point C, draw a vertical line CF. Horse power developed,
It represents axial ( flow) velocity Vf1 at the
W
inlet. hp =
5. Measure the angle CBF as q as blade inlet 746
angle.
6. Calculate the relative velocity at the outlet
The vertical component of the absolute velocity is
Vr2 = kVr1.
responsible to provide axial thrust on the bearings
7. From the point B, draw a line BD equal to and other components and therefore, is undesirable.
BC inlined f (blade ouitlet angle). The line Axial thrust on the wheel is obtained due to
BD represents relative velocity at the outlet. difference between the velocities of flow (axial
8. Join points A and D. The line AD represents velocity) at entrance and outlet of blade.
absolute velocity V2 of steam at the outlet. Axial thrust
9. Draw a vertical line DE from the point D. = Mass flow rate ¥ change of axial velocity
The line DE represents axial (flow) velocity
= ms ¥ (Vf 1 – Vf2) ...(22.8)
Vf2 of steam at outlet.
The axial thrust on the wheel must be balanced
10. Measure EA as whirl velocity Vw2 and AF as
or thrust bearing may be used to take up the load.
whirl velocity Vw1.

In a single-stage impulse turbine, the amount of


energy supplied to the blades is equal to kinetic
(i) By Newton’s second law of
energy of entering steam, i.e., V12/2. Blade
motion, the tangential force is proportional to the
efficiency, also called diagram efficiency, can be
rate of change of momentum, i.e.,
calculated as
Tangential force
Blade efficiency,
= Rate of mass flow ¥ change in
velocity in tangential direction Work done on the blade
hb =
F = ms [Vw1 – (–Vw2)] Energy supplied to the blade
or F = ms [Vw1 + Vw2] ...(22.5) u Vw
hb =
where Vw2 is opposite in direction of Vw1. V12/ 2
ms = Rate of mass flow of steam.
2u Vw
or hb = ...(22.9)
(ii) Work done on the Blade V12
Work = Force ¥ displacement The blade efficiency may also be evaluated as
Rate of work done,
Change in kinetic energy per kg
W = Force ¥ blade velocity hb =
Energy supplied per kg
= ms (Vw1 + Vw2) u = ms u Vw ...(22.6)
Steam Turbines 705

V12 - V22
= ...(22.10)
V12 Due to presence of friction, when steam passes over
the blades, the existing relative velocity is always
It will be maximum, when V22 is minimum, i.e.,
less than inlet relative velocity.
when b = 90° or discharge is axial.
Thus Vr2 = k Vr1
where k is called the blade velocity coefficient.
A stage of an impulse turbine consists of a nozzle
set and a moving wheel. The stage efficiency
correlates the enthalpy drop in the nozzle and work
done in the stage. The stage efficiency is sometimes
referred as gross efficiency. It is expressed as The work done per kg of steam flow in a single-
Work done on blade stage impulse turbine is given by
hstage =
Energy supplied per stage w = u Vw = u (Vw1 + Vw2)
From combined velocity triangle, Fig. 22.8;
u Vw
= ...(22.11) Vw1 = V1 cos a = Vr1 cos q + u
Dh
where Dh = h1 – h2 specific enthalpy drop of and Vw2 = V2 cos b = Vr2 cos f – u
steam in the nozzle. Using above, we get work done per kg of steam
Further, the stage efficiency in a single stage
= Blade efficiency ¥ Nozzle efficiency w = u (Vr1 cos q + Vr2 cos f)
hstage = hb ¥ hN ...(22.12) If there is no friction and for symmetrical blades,
Nozzle efficiency is given by Vr1 = Vr2 and q = f
Then the work done per kg of steam is
V12
hN = ...(22.13) w = 2uVr1 cos q = 2u(V1 cos a – u)
2(h0 - h1) ...(22.16)
with h0 = Enthalpy of steam at the entrance of Now blade or diagram efficiency is given by
nozzle, J/kg, and
2u ( V1 cos a - u )
h1 = Enthalpy of steam at nozzle exit, J/kg hb =
V12
2
4u Ê uˆ
ÁË cosa - V ˜¯
The work delivered as a shaft output is always less = ...(22.17)
than the work calculated from the velocity diagram V1 1
due to mechanical friction. Thus, the net efficiency Introducing the blade-speed ratio;
is defined as the ratio of work delivered at the shaft u
s =
to total energy supplied in the stage. V1
Net work output at shaft hb = 4s (cos a – s)2
Net efficiency =
total energy supplied to the stage = 4s cos a – 4s2 ...(22.18)
or hoverall = hN ¥ hStage ¥ hmech. ...(22.14) For a given steam velocity V1 and blade velocity
Energy converted to heat by blade friction u, it is clear from Eq. (22.16) that the work done
= Loss of kinetic energy during flow over blades per kg of steam would maximum, when cos a
= ms ¥ (Vr21 – Vr22) ...(22.15) = 1 or a = 0. For zero value of a, the axial flow
component would be zero. But it is essential to have
706 Thermal Engineering

an axial flow component to allow the steam to reach Ê 2u ˆ


the blades and to clear the blades on leaving. As wmax = 2u Á ¥ cos a - u˜
Ë cos a ¯
a increases, the work done on the blades reduces,
but at the same time surface area of blades reduces, = 2u2 ...(22.21)
thus there are less frictional losses. With these
Example 22.1 The steam expands isentropically in
conflicting requirements, the blade angle a is
a simple impulse turbine from 12 bar, 250°C with an
generally kept between 15° and 30°. For a given
enthalpy of 2935 kJ/kg to an enthalpy of 2584 kJ/kg at
value of a, the optimum blade-speed ratio, s for 0.1 bar. The nozzle makes 20° with blade motion and
maximum diagram efficiency can be obtained by the blades are symmetrical. Calculate the blade velocity
differentiating Eq. (22.18) with respect to s and that produces maximum efficiency for a turbine speed of
equating it to zero i.e., 3600 rpm. Assume that the steam enters the nozzle with
d hb d negligible velocity.
= [4s cos a – 4s2] = 0
ds ds
Solution
or 4 cos a – 8s = 0
u cosa Given A single-stage impulse turbine
or s= = ...(22.19)
V1 2 p0 = 12 bar T0 = 250°C pb = 0.1 bar
It is the condition for maximum efficiency h0 = 2935 kJ/kg h1 = 2584 kJ/kg a = 20°
for impulse turbine. The maximum blade or dia- N = 3600 rpm f =q V0 = 0
gram efficiency can be obtained by substituting To find Blade velocity.
cosa
s= in Eq. (22.18). Analysis The change in specific enthalpy of steam
2
Dh = h0 – h1 = 2935 – 2584 = 351 kJ/kg
4 cos 2 a 4 cos 2 a
hb, max = - The steam jet velocity
2 4 V1 = 44.72 ¥ Dh (∵ V0 = 0)
= cos2 a ...(22.20)
or V1 = 44.72 ¥ (351 kJ/kg) = 837.83 m/s
The variation of blade efficiency with blade- For maximum velocity, Eq. (22.19)
Ê uˆ u
=
cos a
speed ratio, s Á = ˜ is plotted in Fig. 22.9. V1 2
Ë V1 ¯
Further, the blade-speed ratio yields V1 cos a 837.83 ¥ cos 20∞
or u = = = 393.65 m/s
2 2
2u
V1 =
cosa Example 22.2 The steam leaves the nozzles of a
Substituting in Eq. (22.16), we get maximum single-row impulse turbine at 900 m/s. The nozzle angle
work done on the blade is 20° and blade angles are 30° at inlet and outlet.
Calculate the blade velocity and work done per kg of
steam. Assume the flow over the blade is frictionless.

Solution
Given A single-row impulse turbine
V1 = 900 m/s, a = 20°
q = f = 30°
To find
(i) Blade velocity, and
(ii) Work done per kg of steam.
Steam Turbines 707

Analysis The velocity diagram is constructed as with a nozzle angle of 20° and leaving the blade in
explained below: the axial direction. The ratio of blade velocity to whirl
(i) Draw a horizontal line and choose a point A and velocity of steam is 0.6. Sketch the velocity diagram and
draw a line AC inlined at 20° to the horizontal calculate.
line. Its length represents absolute steam velocity (a) Blade velocity,
of 900 m/s. (b) Work done per kg of steam.
(ii) Draw a vertical line CD through the point C. It
represents inlet flow velocity, Vf1. Solution
(iii) Draw a line BC through the point C making Given A single-stage, single-row impulse turbine
an angle of (90° – 30° =) 60° with vertical line V1 = 1200 m/s a = 20°
CD. Line BC will also make blade inlet angle of u
b = 90° (Axial discharge) = 0.6
q = 30° with horizontal and will represents the Vw
relative velocity Vr1 at inlet. To find
(i) Blade velocity, and
(ii) Work done per kg of steam.
Assumption Adiabatic turbine and expansion of steam
without friction.
Analysis The velocity diagram is constructed as
explained below:
(i) Draw a horizontal line and choose a point A and
draw a line AC inlined at 20° to the horizontal
line. Its length represents absolute steam velocity
of 1200 m/s.
(ii) Draw a vertical line CD through the point C. It
represents inlet flow velocity, Vf1.

(iv) Measure length of segment AB along horizontal


line. Its magnitude represents blade velocity u =
315 m/s.
(v) Draw the line BE through the point B at angle of
f = 30° and length is equal to Vr 2 = Vr1.
(vi) Join A and E. Line AE represents absolute
velocity of steam at exit.
(vii) Draw a vertical line EF through point E to
represent flow velocity Vf 2 at outlet.
(viii) Measure the length of line FD = Vw2 + Vw2 =
1062 m/s.
(iii) For axial flow turbine Vw2 = 0; hence Vw = Vw1 =
Work done per kg of steam
length of line AD = 1128 m/s.
w = uVw = 315 ¥ 1062
Calculate u = 0.6Vw = 676.8 m/s.
= 334530 J/kg = 334.53 kJ/kg
(iv) Mark a point B along the horizontal line
In a single-stage, single-row impulse AD. Length AB represents blade velocity u =
turbine, the steam is entering at a velocity of 1200 m/s 676.8 m/s.
708 Thermal Engineering

Work done per kg of steam To find


w = u Vw = 676.8 ¥ 1128 (i) Power developed by turbine,
= 763430.4 J/kg = 763.43 kJ/kg (ii) diagram efficiency, and
(iii) Stage efficiency.
Example 22.4 The steam at 4.9 bar and 160°C is
supplied to a single-stage impulse turbine at a mass Assumptions
flow rate of 30 kg/min, from where it is exhausted to a (i) Total pressure drop in nozzle only,
condenser at a pressure of 19.6 kPa. The blade speed is
(ii) Adiabatic turbine and isentropic expansion of
300 m/s. The nozzles are inclined as 25° to the plane of
steam.
wheel and the outlet blade angle is 35°.
Properties of steam from Mollier diagram
Neglecting friction losses, determine (a) theoretical
power developed by the turbine, (b) diagram efficiency, At 4.9 bar and 160°C
and (c) stage efficiency. h0 = 2777 kJ/kg s0 = 6.82 kJ/kg ◊ K
At 19.6 kPa and s1 = s0 = 6.82
Solution
h1 = 2275 kJ/kg
Given A single-stage impulse turbine
Analysis The change in specific enthalpy of steam
p0 = 4.9 bar pb = 19.6 kPa
Dh = h0 – h1 2777 – 2275 = 502 kJ/kg
T0 = 160°C a = 25°
The steam jet velocity (V0 = 0)
u = 300 m/s f = 35°
ms = 30 kg/min V1 = 44.72 ¥ Dh
or V1 = 44.72 ¥ (502 kJ/kg) = 1002 m/s
T
r Velocity diagram
ba
4.9 For vector diagram, using
160°C 0
i a V1 = 1002 m/s u = 300 m/s
kP
.6
19 a = 25° b = 35°
1 (i) By measuring EF,
Vw = 1200 m/s
s
Power developed in the stage
0 (a)
P = ms uVw
Vw = 1200 m/s
Ê 30 ˆ
= Á kg/s˜ ¥ (300 m/s) ¥ (1200 m/s)
Vw Vw Ë 60 ¯
2 u = 300 m/s 1 D
F A B
a f q = 180 ¥ 103 W = 180 kW
b
(ii) Diagram (blade) efficiency
/s
0m

Vr 2u Vw
52

1 hb =
V12
2 =

Vr V
2 1 =
10
V

02
m/
s 2 ¥ (300 m/s) ¥ (1200 m/s)
=
E C (1002 m/s) 2
= 0.717 or 71.7%
Vr = Vr (iii) Stage efficiency
1 2
u Vw
(b) hstage =
Dh
Fig. 22.12
Steam Turbines 709

(300 m/s) ¥ (1200 m/s) Analysis


= 3
(502 ¥ 10 J/kg) (i) From the Mollier diagram
= 0.717 or 71.7% h0 @ 4 bar, 160°C = 2780 kJ/kg
h1 @ 0.15 bar = 2265 kJ/kg,
Example 22.5 A single-stage impulse turbine is sup- when s0 = s 1
plied with steam at 4 bar and 160°C and it is exhausted at Dh = h0 – h1 = 2780 – 2265 = 515 kJ/kg
a condenser pressure of 0.15 bar at the rate of 60 kg/ min. Actual heat drop,
The steam expands in a nozzle with an efficiency of 90%. Dhact = hN ¥ Dh
The blade speed is 250 m/s and the nozzles are inclined = 0.9 ¥ 515 = 463.5 kJ/kg
at 20° to the plane of the wheel. The blade angle at the
Steam jet velocity,
exit of the moving blade is 30°. Neglecting friction losses
in the moving blades, determine, V1 = 44.72 ¥ Dhact
(a) Steam jet velocity, = 44.72 ¥ ( 463.5 kJ/kg)
(b) Power developed,
= 962.8 m/s
(c) Blade efficiency,
The velocity diagram can be drawn as in
(d) Stage efficiency.
Fig. 22.13.
Solution By measurement we get,
Vw1 + Vw2 = FD = 1280 m/s
Given The single-stage impulse steam turbine with
(ii) Power developed,
p0 = 4 bar, 160°C pb = 0.15 bar
P = ms V w u
ms = 60 kg/min hN = 0.9
u = 250 m/s a = 20° Ê 60 ˆ
=Á kg/s˜ ¥ (1280 m/s) ¥ (250 m/s)
f = 30° Ë 60 ¯

To find = 320 ¥ 103 W = 320 kW


(i) Steam jet velocity, (iii) Blade efficiency,
(ii) Power developed, 2Vw u
(iii) Blade efficiency, and hb =
V12
(iv) Stage efficiency.
2 ¥ (1280 m/s) ¥ ( 250 m/s)
Assumptions =
(962.8 m) 2
(i) Isentropic expansion in the turbine.
= 0.6904 = 69.04%
(ii) No frictional losses, thus Vr1 = Vr2.
(iv) Stage efficiency,
Vw u
hs =
Dh
(1280 m/s) ¥ ( 250 m/s)
=
(515 ¥ 103 J/kg)
= 0.621 = 62.1%

Example 22.6 The rotor of an impulse turbine is of


260 mm diameter and runs at 20,500 rpm. The nozzle
angle is 20°, and issues a steam jet with a velocity of
910 m/s. The mass flow rate through the turbine nozzle’s
blading is 2.0 kg of steam per second. Draw the velocity
diagram and calculate the (a) tangential force on blades,
710 Thermal Engineering

(b) axial force on blades, (c) power developed by the No, friction loss, Vr2 = Vr1 = 654.5 m/s
turbine wheel, (d) efficiency of the blading, (e) inlet angle For shockless flow, q = f = 29°
of blades for shockless inflow of steam. (i) Tangential force on the blade

Solution F = ms (Vw1 + Vw2) = ms Vw


= (2 kg/s) ¥ (1114 m/s)
Given A single-stage impulse turbine
= 2228 N = 2.228 kN
D = 260 mm N = 20,500 rpm a = 20°
(ii) Axial force on the blade
V1 = 910 m/s ms = 2.0 kg/s q=f
= ms (Vf1 – Vf2)
To find
= (2 kg/s) ¥ (325.5 – 325.5) = 0
(i) Tangential force on the blades,
(iii) Power developed by the turbine
(ii) Axial force on the blades,
P = Fu
(iii) Power developed by the turbine wheel,
= (2.228 N) ¥ (279 m/s)
(iv) Efficiency of the blading, and
= 621.6 kN m/s = 621.6 kW
(v) Inlet angle of the blades for shockless inflow of
(iv) Blade efficiency
steam.
2u Vw
Assumptions hb =
V12
(i) Adiabatic turbine and isentropic flow of steam in
2 ¥ ( 279 m/s) ¥ (1114 m/s)
the turbine, and =
(910 m/s) 2
(ii) No friction on the turbine blade, thus Vr1 = Vr2.
= 0.75 = 75%
Analysis The linear blade velocity
(v) Inlet angle of blade for shockless inflow, q = 90°
p DN Ê 20500 ˆ
u= = p ¥ (0.26 m) ¥ Á rps = 279 m/s
60 Ë 60 ˜¯
The measurements on velocity diagram is shown in
Fig. 22.14, reveal that
Relative velocity at inlet, Vr1 = 654.5 m/s
Axial velocity at inlet, Vf1 = 325.5 m/s
In the earlier section, the effect of friction is
Axial velocity at outlet, Vf2 = 325.5 m/s
neglected and relative velocity at outlet, Vr2 is
Whirl velocity at inlet, Vw1 = 854 m/s
assumed equal to the inlet relative velocity, Vr1. In
The total whirl velocity, Vw = 1114 m/s
actual practice, the blades offer frictional flow of
Inlet blade angle, q = 29°
steam over them, and thus the exit relative velocity
Steam Turbines 711

is always less than the inlet relative velocity. This


frictional effect reduces the relative velocity in the
order of 10 to 15% as steam passes over the blades.
The two relative velocities are related as
Vr2 = k Vr1
where k is called the blade-velocity coefficient or
friction coefficient. The modified velocity diagram
is shown in Fig. 22.15.
Mark the point G on BC such that
BG = k BC = kVr1
Now with B as centre and BG as radius, draw
an arc along BD to locate the position D. Then
complete the outlet velocity triangle in the usual
manner.
Example 22.7 Steam leaves the nozzle of a single-
stage impulse turbine at 840 m/s. The nozzle angle is 18°
and the blade angles are 29° at the inlet and outlet. The
friction coefficent is 0.9.
Calculate
(a) Blade velocity,
(b) Steam mass flow rate in kg/h to develop 300 kW
power.

Solution
Given The single-stage impulse steam turbine
V1 = 840 m/s a = 18°
Vr
f = q = 29° k = 1 = 0.9
Vr2
P = 300 kW
To find
(i) Blade velocity, u (m/s)
When the blades of an impulse turbine are designed (ii) The mass flow rate in (kg/h).
in such a way that the steam leaving the blades at
its tip at 90° to the direction of blade peripheral Analysis The velocity diagram can be drawn as shown
velocity u then the turbine is called axial flow in Fig. 22.17.
turbine. For an axial flow turbine, b = 0; V2 = V f2 (i) Draw a horizontal line. At the point A, with a =
and V w2 = 0. 18° and V1 = 840 m/s (to the scale) draw a line
A typical velocity diagram is shown in Fig. 22.16. AC.
(ii) At the point C, draw a vertical line CD. Through
The power developed by axial flow turbine
the point C with an angle 90° – q = 61°, with
(Vw2 = 0)
vertical line; draw a line BC, whose measurement
P = ms uV w = ms uV w1 ...(22.23) gives us Vr1 = 535 m/s.
712 Thermal Engineering

(iii) With Vr2 = 0.9 Vr1 = 481.5 and q = 29°, draw a Example 22.8 The steam is supplied to a de-Laval
line BE. turbine at a velocity of 1000 m/s at an angle of 20°. The
(iv) Meet BE = V2. blade velocity is 300 m/s and the blades are symmetrical.
(v) Meet EF, its measurement gives Vf2. The mass-flow rate of the steam is 0.5 kg/s. Allowing a
friction factor of 0.8, determine.
(a) Blade efficiency
(b) Power developed
(c) Stage efficiency, if the nozzle efficiency is 95%

Solution
Given A de-Laval steam turbine
V1 = 1000 m/s a = 20°
u = 300 m/s q =f
Vr
ms = 0.5 kg/s k = 2 = 0.8
Vr1
hN = 0.95
To find
The measurement shows that (i) Blade efficiency,
u = 330 m/s (ii) Power developed, and
Vw = 890 m/s (iii) Stage efficiency.
(b) Power developed
Assumptions
m uV
P = s w kW (i) Adiabatic expansion in the turbine.
1000
(ii) Change in potential energy is negligible.
ms ¥ 330 ¥ 890
300 =
1000 Analysis The velocity diagram for given data is drawn
or ms = 1.02 kg/s as shown below. The measurements reveal that
Vw = 1162 m/s
Alternatively, the velocities can be worked out
trignometrically as follows: (i) Blade efficiency,
From D ACF, 2u Vw
hb =
AD = V1 cos a V12
or Vw1 = 840 cos 18° = 798.9 m/s
From Ds ACD and BCD
CD = AC sin a = BC sin q
or V1 sin a = Vr1 sin q
840 sin 18∞
\ Vr1 = = 535.4 m/s
sin 29∞
Vr2 = k Vr1 = 0.9 ¥ 535.4 = 481.9 m/s
u = Vw1 – Vr1 cos 29°
= 798.9 – 535.4 cos 29° = 330.6 m/s
Vw2 = Vr2 cos f – u = 481.9 ¥ cos 29° – 330.6
= 421.5 – 330.6 = 90.9 m/s
Vw = Vw1 + Vw2 = 798.9 + 90.9 = 889.89 m/s
Steam Turbines 713

2 ¥ 300 ¥ 1162
= = 0.697 = 69.7%
(1000) 2

(ii) Power developed,


u Vw
p = ms
1000
300 ¥ 1162
= 0.5 ¥ = 174.3 kW
1000
(iii) Stage efficiency,
hstage = Nozzle efficiency
¥ Blade efficiency
= 0.95 ¥ 0.697 = 0.662 = 66.2%

Example 22.9 In a de-Laval turbine, steam issues from


(i) Draw a line AB equal to the blade velocity
the nozzles with a velocity of 850 m/s. The nozzle angle
350 m/s.
is 20°. Mean blade velocity is 350 m/s and the blades
(ii) From the point A, draw a line AC equal to V1 =
are equiangular. The mass-flow rate is 1000 kg/min. The
850 m/s at an angle 20°.
friction factor is 0.8. Determine:
(iii) Meet the points B and C, and measure angle CBF,
(a) Blade angles,
q = 34° and length BC (Vr1) = 535 m/s.
(b) Axial thrust on the bearings,
(iv) From the point B, draw a line BD equal to 0.8 Vr1
(c) Power developed in kW,
at an angle equal to f = 34°.
(d) Blade efficiency,
The measurements show that
(e) Stage efficiency, if nozzle efficiency is 93%.
inlet relative velocity, Vr1 = 535 m/s
Solution exit relative velocity, Vr2 = 0.8 Vr1
= 428 m/s.
Given The de-Laval turbine
(i) Nozzle angle b = f = 34°
V1 = 850 m/s a = 20°
Axial velocities,Vf1 = 300 m/s, and Vf2 = 240 m/s
u = 350 m/s
The total whirl velocity, Vw = Vw1 + Vw2
ms = 1000 kg/min = 16.667 kg/s
= 790 + 20 = 810 m/s
f =q hN = 0.93
Vr1 (ii) Axial thrust on bearings
k = = 0.8 Axial force = ms (Vf1 – Vf2)
Vr2
= (16.667 kg/s) ¥ (300 – 240)
To find = 1000 N
(i) Blade angles, (iii) Power developed,
(ii) Axial thrust on the bearings, m uV 16.667 ¥ 350 ¥ 810
(iii) Power developed in kW, P= s w =
1000 1000
(iv) Blade efficiency, and = 4725 kW
(v) Stage efficiency. (iv) Diagram or blade efficiency,
Assumptions 2u Vw
hb =
(i) Adiabatic turbine. V12
(ii) Change in potential energy of the steam during 2 ¥ 350 ¥ 810
=
expansion is negligible. (850) 2
Analysis The velocity diagram for given data is drawn = 0.7847 or 78.47%
as shown in Fig. 22.19, with some convenient scale.
714 Thermal Engineering

(v) Stage efficiency (i) Inlet angle of moving blade, q = 24°.


hstage = hb ¥ hN Inlet angle of fixed blades, b = 42.5°
= 0.7847 ¥ 0.93 = 0.7298 or 72.98% By measurement,
Vw = EF = 977 m/s
Example 22.10 The nozzles of a de-Laval turbine Vr1 = AC = 612.5 m/s
deliver 1.5 kg/s of steam at a speed of 800 m/s to a ring Vr2 = 0.75 Vr1 = 459.4 m/s
of moving blades having a speed of 200 m/s. The exit
Vf1 = FC = 247 m/s
angle of the nozzle is 18°. If the blade velocity coefficient
Vf2 = 194 m/s
is 0.75 and the exit angle of the moving blades is 25°,
calculate. 2u Vw
(ii) Diagram efficiency, hb =
(a) Inlet angle of moving and fixed blades, V12
(b) Diagram efficiency, 2 ¥ 200 ¥ 977
or hb =
(c) Energy lost in blades per second, (800) 2
(d) Power developed, and = 0.6106 or 61.06%
(e) Axial thrust on the turbine rotor. (iii) Energy lost in blades
Vr2 - Vr2
Solution De = 1 2

2
Given A de-Laval turbine (612.5) 2 - ( 459.4) 2
=
ms = 1.5 kg/s 2
a = 18° f = 25° = 82065 Nm/kg
u = 200 m/s V1 = 800 m/s = 82.065 kJ/kg
Vr (iv) Power developed
k = 2 = 0.75 V u
Vr1 P = ms w
To find 1000
977 ¥ 200
(i) Inlet angle of moving and fixed blades, = 1.5 ¥ = 293.1 kW
1000
(ii) Diagram efficiency, (v) Axial thrust on rotor
(iii) Energy lost in blades per second, Fa = ms ( Vf 1 - Vf 2)
(iv) Power developed, and = 1.5 ¥ (247 – 194)
(v) Axial thrust on the turbine rotor. = 79.5 N
Analysis The velocity diagram can be drawn as shown
Example 22.11 A simple de Laval steam turbine,
in Fig. 22.20. By measurement, we get,
nozzle ring is suplied with 200 kg/h of dry steam at 10 bar.
The exhaust pressure is 0.6 bar. The nozzle angle is 20°
and nozzle efficiency is 85%. The blade outlet angle is 28°
and blade speed is 200 m/s. The blade velocity coefficient
is 0.78. The power loss due to fan action and disc friction
is 1.5 kW. The losses reappear as heat. Calculate
(a) Net output at the shaft
(b) Dryness fraction of steam in exhaust passage,
when speed is small
(c) The dryness fraction of steam as it leaves the
turbine rotor bladding

Solution
Given A simple steam turbine
ms = 200 kg/h = 0.055 kg/s
Steam Turbines 715

a = 20° f = 28° Vf1 = FC = 320 m/s


u = 200 m/s p0 = 10 bar (dry steam) Vf2 = 272 m/s
Vr Theoretical power developed,
p1 = 0.6 bar k = 2 = 0.78
Vr1 Pth = ms u Vw
hN = 0.85 = ¥ 200 ¥ 1140 = 12540 Nm/s
0.055
Power loss in fan action and friction = 1.5 kW. = 12.54 kW
Net power output,
To find
P = 12.54 – 1.5 = 11.04 kW
(i) Net output at shaft,
(ii) Dryness fraction of exhaust steam, after heating
(ii) Dryness fraction of exhaust steam, after heating by frictional heat:
by frictional heat, and
When the speed of the turbine is small, then all
(iii) Dryness fraction of steam as leaves the turbine the frictional heat will be utilized to heat the
rotor. steam. Therefore, the change in enthalpy will be
Analysis The properties of steam at nozzle inlet and 1.5 kW
Dh1 = 480 + = 507.27 kJ/kg
exit 0.55 kg/s
At 10 bar (dry and saturated) The enthalpy at the turbine exhaust
h1 = 2780 kJ/kg (from Mollier diagram) h2¢ = h1 – Dh1
At 0.6 bar (s1 = s2) = 2780 – 507.27 = 2272.73 kJ/kg
h2 = 2300 kJ/kg From Mollier diagram at 0.6 bar and 2272.73
x2 = 0.845 kJ/kg
Change in enthalpy, x2¢ = 0.88
Dh = 2780 – 2300 = 480 kJ/kg (iii) Dryness fraction of steam as it leaves the turbine
(i) The absolute velocity of steam at nozzle exit rotor
V1 = 44.72 hN ( D h) Dryness fraction of steam as it leaves the turbine
rotor at h = 2300 kJ/kg and s1 = s2.
= 44.72 ¥ 0.85 ¥ 480 x2 = 0.845
= 903.3 m/s
The velocity diagram can be drawn as shown Example 22.12 A simple impulse turbine has a mean
in Fig. 22.21. By measurement, inlet angle of blade-ring diameter of 70 cm and runs at 3000 rpm. The
moving blade, q = 26°. speed ratio is 0.46 and the discharge is axial. The nozzle
Vw = EF = 1140 m/s angle is 21° and blade friction factor is 0.95. Determine
Vr1 = BC = 731 m/s (a) Blade angles,
Vr2 = 0.78 Vr1 = 570 m/s (b) Theoretical specific power output.

Solution
Given A simple impulse turbine
D = 70 cm N = 3000 rpm
b = 90°
u
speed ratio, s = = 0.46
V1
a = 21° k = 0.95
To find
(i) Blade angles, and
(ii) Theoretical specific power output.
716 Thermal Engineering

Analysis Alternatively
(i) For axial discharge, b = 90° Vw1 = V1 cos a = 239 ¥ cos 21
= 223.12 m/s
The blade velocity,
\ BF = Vw1 – u = 223.12 – 110
p DN p ¥ 70 ¥ 10 -2 ¥ 3000 = 113.12 m/s
u = =
60 60 CF = Vf 1 = V1 sin a = 239 sin 21
= 110 m/s
= 85.65 m/s
The nozzle exit velocity,
u 110 \ V r1 = ( BF ) 2 + (CF ) 2
V1 = = = 239 m/s
s 0.46
= (113.12) 2 + (85.65) 2
The relation between two relative velocities.
Vr2 = 141.93 m/s
= 0.95 CF
Vr1 tan q =
BF
The velocity diagram can be constructed as
It gives the inlet angle of the moving blades
follows:
Ê 85.65 ˆ
(a) Draw a horizontal line EF. Mark AB = u = q = tan -1 Á = 37.12°
110 m/s. Ë 113.12 ˜¯
(b) At point A, a = 21° and V1 = 239 m/s, draw a Vr2 = kVr1 = 0.95 ¥ 141.93 = 133.95 m/s
line AC. u 110
(c) Meet BC, it gives q = 37.12°, inlet blade cos f = =
Vr2 133.95
angle.
or, exit angle of the moving blades,
(d) Using centre B and radius BG = 0.95 BC,
f = 34.8°
draw a chord to cut vertical line AD at point
D. Vw2 = 0
(e) Meet BD, DA and CF.
Example 22.13 In an impulse turbine (with a single
The velocity diagram for given data can be drawn row wheel), the mean diameter of the blade is 1.05 m and
as shown in Fig. 22.22. speed is 3000 rpm. The nozzle angle is 18°. The ratio of
blade speed to steam speed is 0.42 and ratio of relative
velocity at outlet from the blades to that at inlet is 0.84.
The outlet angle of the blade is to be made 3° less than
the inlet angle. The steam-flow rate is 10 kg/s. Draw the
velocity diagram for blades and derive the following:
(a) Tangential thrust on the blades,
(b) Axial thrus on the blades,
(c) Power developed in the blades,
(d) Blading efficiency.

Solution
Given A simple impulse turbine
D = 1.05 m N = 3000 rpm
The measurements on the velocity diagram give, f = q – 3°
whirl velocity Vw = 224 m/s u
speed ratio, s = = 0.42
The specific power output, V1
u Vw 110 ¥ 224 a = 18° k = 0.84
P = =
1000 1000 ms = 10 kg/s
= 24.64 kW/kg of steam
Steam Turbines 717

To find Vf1 = FC = 122.5 m/s


(i) Tangential thrust on the blades, Vf2 = 98.5 m/s.
(ii) Axial thrus on the blades, (i) Tangential thrust on the blades,
(iii) Power developed in the blades, and F = ms (Vw1 + Vw2) = 10 ¥ 395 = 3950 N
(iv) Blading efficiency. (ii) Axial thrust on the blades,
Fa = ms (Vf1 – Vf2)
Analysis The blade velocity,
= ¥ (122.5
10 – 98.5) = 240 N
p DN p ¥ 1.05 ¥ 3000
u = = = 164.93 m/s (iii) Power developed in the blades,
60 60 m uV 10 ¥ 164.93 ¥ 395
The nozzle exit velocity, P = s w =
1000 1000
u 164.93
V1 = = = 392.7 m/s = 651.47 kW
s 0.42
(iv) Blading efficiency,
The relation between two relative velocities.
Vr2 2u Vw 2 ¥ 164.93 ¥ 395
hb = =
= 0.84 V12 (392.7) 2
Vr1
= 0.691 or 69.1%
The velocity diagram can be constructed as shown in
Fig. 22.23.
Example 22.14 The steam leaves the nozzle of a
(i) Draw a horizontal line EF. Mark AB = u = simple impulse turbine of outlet area 15.5 cm2 at 920 m/s.
164.93 m/s. The steam coming out of the nozzle is 0.91 dry at 150 kPa
(ii) At the point A, a = 18° and V1 = 392.7 m/s, draw absolute. The blade inlet and outlet angles are 30°, and
a line AC. the blade velocity is 230 m/s. If the friction factor is
(iii) Meet BC, it gives q = 30°, inlet blade angle. 0.8, find (a) the nozzle angle, (b) the power developed,
Length of the line BC represents Vr1. (c) the diagram efficiency, and (d) the axial thrust on the
(iv) Draw the line BD to represent Vr2 = 0.84 Vr1 at an blading.
angle f = q – 3° = 27°.
(v) Meet AD to represent V2. Solution
(vi) Meet ED to represent Vf2. Given A simple impulse turbine with
(vii) Meet CF to represent Vf1. Nozzle outlet area Ac = 15.5 cm2
The measurements on the velocity diagram give Inlet velocity of steam, V1 = 920 m/s
Vw = EF = 395 m/s Quality of steam, x = 0.91
Vr1 = BC = 242.5 m/s Steam pressure, p1 = 150 kPa
Vr2 = 0.84 Vr1 = 203.7 m/s Blade angles, f = q = 30°
Blade velocity, u = 230 m/s
Friction factor, k = 0.8
To find
(i) Nozzle angle a,
(ii) Power developed in the turbine,
(iii) Diagram efficiency,
(iv) Axial thrust on the blading.
Assumptions
(i) Adiabatic turbine.
(ii) Change in potential energy of the steam during
expansion in negligible.
718 Thermal Engineering

Analysis The velocity diagram for the given data is (iii) Diagram or blade efficiency
drawn as shown in Fig. 22.24 with some convenient scale. 2u Vw 2 ¥ 230 ¥ 1099
hb = 2
=
(i) Draw a horizontal line EF. Mark segment AB = u V1 (920) 2
= 230 m/s. = 0.597 or 59.7%
(ii) From point B draw a line BC at angle 30°. (iv) Axial thrust on the blading
(iii) From point A with radius AC = V1 = 920 m/s, F = ms (Vf1 – Vf2)
draw a chord to cut the line BC at point C. = 1.353 ¥ (378 – 294)
(iv) With angle f = 30° and Vr2 = 0.8Vr2, draw a line = 113.65 N
BD.
(v) Join AD, ED and CF to complete the velocity Example 22.15 Steam issues from the nozzle of a
triangle. simple impulse turbine with a velocity of 610 m/s. The
nozzle angle is 20° and the diameter of the rotor is 62 cm
and runs at 9500 rpm. The blade outlet angle is 30° and
the friction factor is 0.8. Calculate the power developed
per kg. of steam and the diagram efficiency.

Solution
Given A single impulse steam turbine
V1 = 610 m/s a = 20°
D = 62 cm N = 9500 rpm
f = 30° k = 0.8
To find
(i) The power developed per kg of steam, and
The measurements show that inlet relative velocity, (ii) Diagram efficiency.
Vr1 = 715 m/s Assumptions
Exit relative velocity,
(i) Adiabatic turbine.
Vr2 = 0.8, Vr1 = 572 m/s
(ii) No change in potential energy of steam.
(i) Nozzle angle a = 23° (iii) Negligible steam velocity at the nozzle inlet.
Axial velocities,
Vf1 = 378 m/s, Vf2 = 294 m/s Analysis The linear blade velocity
The total whirl velocity, p DN
u =
Vw = Vw1 + Vw2 = 840 + 259 60
= 1099 m/s p ¥ (0.62 m) ¥ (9500 rpm )
= = 308.4 m/s
(ii) The specific volume of steam at 150 kPa 60
vg = 1.15828 m3/kg The velocity diagram is shown in Fig. 22.25. The
Mass-flow rate of steam measurements reveal that
-4 The inlet relative velocity,
A V 15.5 ¥ 10 ¥ 920
ms = c 1 = Vr1 = length of line BC = 339 m/s
x vg 0.9 ¥ 1.15828
Inlet blade angle, q = 37°
= 1.353 kg/s
Power developed, The relative velocity of steam at outlet,
m uV Vr2 = k Vr1
P = s w
1000 = 0.8 ¥ 339 = 271.2 m/s
1.353 ¥ 230 ¥ 1099 The net whirl velocity,
=
1000 Vw = Vw1 – Vw2
= length of line EF = 510 m/s
= 341.97 kW
Steam Turbines 719

(i) Power developed/kg of steam,


u Vw 308.4 ¥ 510
P = =
1000 1000 This design was developed by an American engineer
= 157.28 kJ/kg CG Curtis in year 1985. In velocity compounding,
(ii) Diagram or blade efficiency the steam is expanded to an exhaust pressure in
2u Vw 2 ¥ 308.4 ¥ 510 a single set of nozzles. The kinetic energy of the
hb = =
V12 (610) 2 steam is absorbed in two or three sets of moving
= 0.845 = 84.5% blades. The pressure and velocity variation for a
three sets of moving blades are shown in Fig. 22.26
It consists of a set of nozzles, three rows of
moving blades and two rows of guide blades. The
guide blades are arranged between moving blades
The compounding of an impulse turbine is done in the reverse manner.
to reduce rotational speed of the turbine to the On passing through moving blades, the steam
practical limits. In a simple impulse turbine, a rotor gives up its partial kinetic energy. During the
speed of 30,000 rpm is almost impractical to use in passage through the moving blades, the steam
practice. The compounding of an impulse turbine pressure remains constant. Then the steam enters
is achieved by using more than one set of nozzles, the stationary guide blades. The guide blades
blades, rotors in a series, keyed to a common collect the steam coming out of the moving blades,
shaft; so that either the steam pressure or steam jet adjust its direction of motion and direct it on the
velocity can be regulated by the turbine in stages. next row moving blades. Then steam enters the
Thus, compounding of steam turbine improves the next row of guide blades and then moving blades
performance of the turbine. in similar manner. During the passage through the
There are three main types of compounding in guide blades, there is no change in pressure and
impulse turbines. velocity of steam.
The velocity of the steam jet is increased in the
(i) Velocity compounded steam turbine
nozzles only and it drops gradually, after being
(ii) Pressure compounded steam turbine
utilized successively by all rows of moving blades
(iii) Pressure velocity compounded turbine till it is finally discharged to the condenser. The
720 Thermal Engineering

Terminology for Vector Diagram of Velocity


Compounded Steam Turbine The suffix f indicates
quantity for first row and s indicates for second row.
u = Linear velocity of blade, m/s
V1f = Absolute velocity of steam jet at inlet to
blade, m/s
af = Inlet angle of steam jet in respect to plane
of blade rotation, degree
Vr1f = Relative velocity of steam at inlet, m/s
qf = Blade inlet angle, degree
Vw1f = Whirl velocity at inlet, m/s
Vf 1f = Flow velocity of steam air inlet; m/s
V2f , bf, Vr2 f , ff , Vw2 f , Vf2f are corresponding
values at the outlet of the first row of moving blades
and the inlet to the first row stationary guide blades.
V1s, as, Vr1s, qs, Vw1s, Vf1s are corresponding
values at the inlet to the second row of moving
blades.
V2s, bs, Vr2 f , ff , Vw2f , Vf2f are corresponding
values at the outlet of the second row of moving
blades and inlet to the second row stationary guide
blades, and so on.
The work done per kg of steam on the first row
of moving blades
wf = u (Vw1f + Vw2 f )
The work done per kg of steam on the second
velocity, diagram for a two rows of moving blades row of moving blades
is shown in Fig. 22.27.
ws = u (Vw1s + Vw2s)
Steam Turbines 721

Total work done per kg of steam on the two-row


wheel
wtotal = wf + ws
= u(Vw1f + Vw2 f) + u (Vw1s + Vw2s)
= u(Vwf + Vws) ...(22.22)
Power output
P = ms u (Vwf + Vws) ...(22.23)
where Vwf = Vw1f + Vw2f , total whirl velocity of first
row of moving blades, and
Vws = Vw1s + Vw2s, total whirl velocity of
second row of moving blades.
Blade of diagram efficiency
Work done on the blade
hb =
Energy supplied to the blade
u ( Vw f + Vws )
hb = ...(22.24)
1 2
V1
2
Stage efficiency,
Work done on blade
hstage =
Energy supplied per stage
u ( Vwf + Vws )
= ...(22.25)
Dh

Instead of expanding the steam completely in a


single set of nozzles, the expansion of steam is
splitted into a number of phases by arranging a
This arrangement is equivalent to splitting up the
number of moving and fixed blades in series, thus
whole pressure drop into a series of small pressure
the rotor speed is obtained in practical range. The
drops, hence arrangement is known as pressure
fixed blades act as nozzles. The steam expands
compounding.
equally in all rows of fixed blades. Thus, this
arrangement can be viewed as a number of simple The lower part of Fig. 22.28 shows the velocity
impulse machines in series on the same shaft, and pressure distribution. Steam enters the first row
allowing the exhaust steam from one turbine to of the nozzle, a small pressure drop takes place with
enter the nozzle of the succeeding turbine. Each increase in steam velocity. The steam passes over
of the simple impulse machines would be termed the first row of moving blades, The steam pressure
as a “stage” of the turbines, since each stage remains constant, but steam velocity decreases.
comprises its set of nozzles and moving blades. It constitutes one stage. The variation of pressure
Fig. 22.28 shows schematic arrangement of three velocity gets repeated for a number of stages until
stage impulse turbine. condenser pressure is reached.
722 Thermal Engineering

In a pressure-compounding steam turbine, a


partial enthalpy of steam is transformed into kinetic
energy in each stage. Hence, steam velocity is
much lower than the simple impulse and velocity-
compounded steam turbines, and thus the operation
is salient and more efficient. The example of
pressure-compounded steam turbine is Rateau and
Zeolly turbine.

This is a combination of pressure and velocity


compoundings. The total pressure drop of steam is
divided into a number of stages as done in pressure
compounding. Each stage has a number of rows of
fixed and moving blades working as an independent
velocity compounded stage.
Each stage is separated from the adjacent stage
by a row of stationary ring of nozzles for expansion
of steam for the next stage. The set of moving and
fixed blades is used for velocity compounding and
a set of nozzle rings in between stages is utilized for
pressure compounding. Such type of compounding
offers a larger pressure drop in each stage with less
number of stages. Therefore, the turbine is simple
in construction and compact in size.
The diagrammatic arrangement shown in
Fig. 22.29, explains the principle of working,
construction and pressure-velocity diagrams.

Example 22.16 In an impulse turbine, the steam


issues from the nozzle with a speed of 600 m/s and blade Find the diagram efficiency under these conditions and
speed is 120 m/s. The velocity is compounded by passing power output for steam flow rate of 5 kg/s.
the steam through a ring of moving blades, through a
ring of fixed blades and finally through a ring of moving What would be maximum possible diagram efficiency
blades. for given steam inlet velocity and nozzle angle?
The nozzle angle is 18° and the blade exit angles and
relative velocity coefficients are the following: Solution
Given A velocity compounded impulse turbine with
Blades exit Velocity
Nozzle angle af = 18°
angle coefficient
Inlet velocity of steam,
First row moving blades 20 ° 0.8 V1f = 600 m/s
Fixed-row blades 25° 0.85 Blade velocity, u = 120 m/s
Second-row moving blades 30° 0.9 Mass flow rate, ms = 5 kg/s
Steam Turbines 723

Moving Blade exit angles,


ff = 20°, and fs = 30°
Fixed blade exit angle, as = 25°
Friction factors, k1 = 0.8,
k2 = 0.85, and k3 = 0.9.
To find
(i) Diagram efficiency,
(ii) Power developed by the turbine,
(iii) Maximum possible diagram efficiency.
Assumptions
(i) Adiabatic turbine.
(ii) Change in potential energy of the steam during
expansion in negligible.
Analysis The velocity diagram for the given data is
drawn as shown in Fig. 22.30 with some convenient scale.
(i) Draw a horizontal line AB equal to blade velocity
u = 120 m/s to a convenient scale.
(ii) From the point A draw a line AC to represent V1f
= 600 m/s at angle a = 18° to horizontal line.
(iii) Join the point B and C by a line BC to represent
Vr1f relative velocity at inlet. Second-row moving blade angle
(iv) From the point B, draw line BD equal to Vr2f = fs = 30°,
0.8Vr1f at an angle f1f = 20°.
The total whirl velocity of first stage; on measurement
(v) Join points A and D to represent absolute velocity
Vws = Vw1s + Vw2s = 200 m/s
of steam at exit of first stage.
(i) Blade or diagram efficiency,
The measurements show the following
Inlet relative velocity, 2u ( Vw f + Vws )
hb =
Vr1f = 480 m/s V12f
Exit relative velocity, 2 ¥ 120 ¥ (790 + 200)
Vr2f = 0.8, Vr1f = 368 m/s =
(600) 2
First row moving blade inlet angle
= 0.66 or 66%
ff = 20°,
(ii) Power developed by the turbine,
Absolute velocity of steam at exit of first stage
V2f = 280 m/s ms u (Vwf + Vws )
P =
The total whirl velocity of first stage 1000
Vwf = Vw1f + Vw2f = 790 m/s 5 ¥ 120 ¥ (790 + 200)
= = 594 kW
The second-row velocity diagram is also constructed 1000
in a similar manner. (iii) Maximum possible diagram efficiency:
Blade velocity, u = 120 m/s hb,max = cos2af = cos2 (18°)
Steam velocity, V1s = 0.85 V2f = 238 m/s at an angle = 0.9045 or 90.45%
as = 25°
Exit relative velocity, Example 22.17 The following particulars relate to a
Vr2s = 0.9 Vr1s two-row velocity compounded impulse wheel:
724 Thermal Engineering

Steam velocity at nozzle outlet = 650 m/s (v) Join points A and D to represent absolute velocity
Mean blade velocity = 125 m/s of steam at exit of first stage.
The nozzle outlet angle = 16°
Outlet angle of first row of moving blades = 18°
Outlet angle of fixed guide blades = 22°
Outlet angle of second moving blades = 36°
Steam flow rate = 2.5 kg/s
The ratio of relative velocity at the outlet to that at the
inlet is 0.84 for all blades.
Determine the following:
(a) Axial thrust on blades,
(b) The power developed, and
(c) The efficiency of the wheel.

Solution
Given A two-row velocity compounded impulse
turbine with
Nozzle angle af = 16°
Inlet velocity of steam, V1f = 650 m/s
Blade velocity, u = 125 m/s
Mass flow rate, ms = 2.5 kg/s
Moving-blade exit angles,
ff = 18°, and fs = 36°
Fixed blade exit angle, as = 22°, The measurements show the following:
Friction factors, k1 = k2 = k3 = 0.84. Given that
To find Vr2f = 0.84 Vr1f and qf = 20°
(i) Axial thrust on the blades, Flow velocities of steam for first row wheel
(ii) Power developed, and Vf1f = 180 m/s, and Vf2 f = 138 m/s.
(iii) Efficiency of wheel. The total whirl velocity of first stage
Vwf = Vw1f + Vw2f = 924 m/s
Assumptions The second-row velocity diagram is also constructed
(i) Adiabatic turbine. in a similar manner.
(ii) Change in potential energy of the steam during Blade velocity, u = 125 m/s
expansion is negligible. Steam velocity, V1s = 0.84 V2f an angle
Analysis The velocity diagram for the given data is as = 22°
drawn as shown in Fig. 22.31 with some convenient scale. Exit relative velocity,
(i) Draw a horizontal line AB equal to the blade Vr2s = 0.84 Vr1s and fs = 30°,
velocity u = 125 m/s to a convenient scale. Flow velocities of steam for second-row wheel
(ii) From the point A draw a line AC to represent V1f Vf1s = 122 m/s, and Vf2s = 107 m/s.
= 650 m/s at angle a = 16° to horizontal line. The total whirl velocity of first stage
(iii) Join points B and C by a line BC to represent Vr1f Vws = Vw1s + Vw2s = 324 m/s
relative velocity at inlet. (i) Axial thrust on the blades,
(iv) From the point B, draw line BD equal to Vr2f = Fa = ms [(Vf1f – Vf2f) + (Vf1s – Vf2s)]
0.8Vr1f at an angle ff = 18°. = 2.5 ¥ [(180 – 138) + (122 – 107)]
= 142.5 N
Steam Turbines 725

(ii) Power developed by the wheel,


ms u (Vwf + Vws )
P =
1000
2.5 ¥ 125 ¥ (924 + 324)
=
1000
= 390 kW
(iii) Blade or diagram efficiency,
2u ( Vw f + Vws )
hb =
V12f
2 ¥ 125 ¥ (924 + 324)
=
(650) 2
= 0.7384 or 73.84%

Example 22.18 In a two stage velocity compounded


impulse turbine, the steam leaves the nozzle at a velocity
of 675 m/s, when blade speed is 150 m/s. The nozzle
angle is 20°, while, the discharge angles are 25° for first-
row blades, 25° for fixed blades and 30° for the second-
row blades. There is 10% loss in velocity during passage
of steam through each row of blades. For the steam flow
rate of 5 kg/s, calculate the power output of the turbine
and diagram efficiency.
Exit velocities
Solution
Vr2f = k Vr1f = 0.9 ¥ 536.5 m/s = 482.85 m/s
Given A two-stage, velocity-compounded steam = Line BD through point B at angle, qf = 25°
turbine V2f = 352.8 m/s Line AD through the point A to meet D.
V1f = 675 m/s u = 150 m/s af = 20° Vw2 f = 287.5 m/s Length of the line AE.
as = 25° qf = 25° qs = 30°
For the second stage;
ms = 5 kg/s k = 0.9
u = 150 m/s Line GH,
To find V1s = k V2f = 0.9 ¥ 352.8 m/s = 317.52 m/s
(i) Power developed, and = Line GI at angle a = 25°
(ii) Blade efficiency. By measurement;
Vr1s = 192 m/s Line HI through H to meet I.
Analysis The velocity diagram for first stage of turbine
is drawn as shown in Fig. 22.32(a). Vw1s = 287.5 m/s Line GL through G to meet L.
The measurements reveal that Exit velocities
Vr2s = kVr1s = 0.9 ¥ 192 m/s = 172.8 m/s,
Inlet velocities = Line HJ through point H at angle, qs = 30°
u = 150 m/s Line segment AB Vw2 s = 0,
V1f = 675 m/s Line AC at angle a = 20° Total whirl velocities;
Vr1f = 536.5 m/s Line BC through B to meet C Vwf = Vw1f + Vw2f
Vf 1f = 231 m/s Length of line CF = 634.5 + 287.5 = 922 m/s
Vw1f = 634.5 m/s Line AF through A to meet F Vws = Vw1s + Vw2s
= 287.5 + 0 = 287.5 m/s
726 Thermal Engineering

Power output
P = ms u (Vwf + Vws)
= 5 ¥ 150 ¥ (922 + 287.5)
= 907125 W or 907.125 kW
Blade or diagram efficiency
u (Vwf + Vws )
hb =
1 2
V1
2
(150 m/s) ¥ (922 m/s + 287.5 m/s)
=
1
(675 m/s) 2
2
= 0.796 or 79.6%

Example 22.19 The nozzles of a two-row velocity


compounded impulse turbine, have outlet angle of 22°
and utilise an isentropic enthalpy drop of 220 kJ/kg of
steam. All moving and guide blades are symmetrical and
mean blade velocity is 150 m/s. Assume an isentropic
efficiency of for the nozzle as 90%. Calculate the specific
power output produced by each kg of steam. The velocity
at the inlet to nozzle can be neglected.

Solution Vr1f = 493.5 m/s Line BC through B to meet C,


Given A two-stage velocity compounded steam turbine angle ff = 28.5°
u = 150 m/s Vw 1f = 583.5 m/s Line AF through A to meet F
af = 22° Exit velocities
Dh = 220 kJ/kg Vr2f = kVr1f = 1 ¥ 493.5 m/s = 493.5 m/s
For symmetrical blades = Line BD through point B at angle, qf = 28.5°
qf = ff bf = bs Vw2f = 283.5 m/s Line EA through A to meet E
qs = fs hN = 0.9 V0 = 0 V2f = 368.7 m/s Line AD at angle bf = 40°
From velocity diagram of second stage of turbine
To find Specific power output. Inlet velocities
Assumption No friction between blades and steam, u = 150 m/s Line GH
k = 0. V1s = V2f = 368.7 m/s Line GI at angle as = 40°
Vw1s = 282.5 m/s Line GL through G to meet L
Analysis The exit velocity from nozzles
Vr1s = 224.7 m/s Line HI through H to meet I,
V1 = 44.72 ¥ hN Dh angle fs = 54°
= 44.72 ¥ 0.9 ¥ 220 = 629.26 m/s Exit velocities
The velocity diagram for first stage of turbine is Vr2s = Vr1s = 224.7 m/s, Line HJ at an angle
drawn as shown in Fig. 22.33(a). qs = 54°
The measurements reveal that Vw2s = –18 m/s, Length of line GK.
Inlet velocities Specific power output
u = 150 m/s Line AB = u (Vw1f + Vw2f + Vw1s + Vw2s)
V1f = 629.26 m/s Line AC at angle a = 22° = 150 ¥ (583.5 + 283.5 + 282.5 – 18)
bf = ff = 28.5° (For symmetrical blades) = 169725 W or 169.725 kW
Steam Turbines 727

Vws = Vw 1s + Vw 2s
Example 22.20 A compounded impulse turbine has
two rows of moving blades separated by a fixed row of H K
G 22°
blades. The steam leaves the nozzle at an angle of 20° 30° qs
with the direction of motion of blades. The blades exit

V r1
s
angles are I st moving 30°, fixed 22° and 2 nd moving 30°.

r2

s
V
J V
1s =
If the adiabatic heat drop of nozzle is 186.2 kJ/kg with V
2f
nozzle efficiency of 90%, calculate the blade velocity I
u
necessary, if the final velocity of steam is to be axial.
Vwf = 790 m/s
Assume a loss of 15% in the relative velocity of all blade
Vw 2f Vw1f
passages. Also obtain the blade efficiency and stage
efficiency. A B
E b1 20° 30° q1 F
V2f
Solution V
f 2f
V r 1f V
.8 Vr
Given A velocity compounded impulse turbine with =0 V
1f = 1f f 1f
V r 2f 57 8.9
Nozzle angle a f = 20° D 3m G
/s
Moving blade exit angles,
C
ff = 30°, and fs = 30°
Fixed blade exit angle, as = 22°, and bs = 90° Fig. 22.34
Friction factors, k1 = k2 = k3 = 0.85.
6. From point H with radius equal to Vr1s draw an
Dh = 186.2 kJ/kg, hN = 0.9
arc to cut the line GI at point I. The length GI
To find represent absolute velocity of steam at inlet to
(i) Blade velocity, second row.
(ii) Diagram efficiency, and 7. Calculate the absolute velocity of steam at exit to
(iii) Stage efficiency. first stage
V1s Length GI
Analysis The velocity of steam coming out of the V2f = =
nozzle k2 0.85
V1f = 44.72 Dh hN = 186.2 ¥ 0.9 First-row velocity triangle
= 578.93 m/s 1. Draw a horizontal line AB equal to the blade
The velocity diagram for axial discharge turbine is velocity u.
drawn in Fig. 22.34. 2. From the point A draw an arc with the radius
equal to V2f .
Second-row velocity triangle
3. From the point B draw a line BD at angle a = 30°.
1. Draw a horizontal line GK. Mark the segment GH
4. Join points A and D to represent V2f .
as unknown blade velocity u.
5. Join points B and D by a line BD to represent Vr2f
2. From the point G draw a vertical line GJ to
relative velocity at outlet.
represent V2s for axial flow turbine.
6. Calculate the relative velocity of steam at inlet to
3. Draw a line HJ at an angle of 30° as second-row
first stage
moving blade angle. The length HJ represents
Vr2 f Length BD
outlet relative velocity Vr2s of second row blaes. Vr1f = =
4. The relative velocity at inlet to second-row k1 0.85
Vr Length HJ 7. From the point B, draw an arc radius equal to Vr1f .
moving blades Vr1s = 2s = .
k3 0.85 8. From the point A, draw line AC at an angle a1f =
5. Now draw a line GI at an angle of 22° fixed row 20° to cut the arc at point C.
blade angle. 9. Joint points B and C to complete the velocity
triangle of first stage.
728 Thermal Engineering

10. Measure the length of line AC and calculate the


scale of triangle as
Length AC
Scale =
578.93
11. Measure the length AB and calculate the blade ve-
locity u by using scale of triangle. The measure-
ments show that
(i) Blade velocity u = 117 m/s (Length AB)
The total whirl velocity of first stage
Vwf = Vw1f + Vw2f = 762 m/s
The total whirl velocity of first stage
Vws = Vw1s + Vw2s = 234 m/s
(ii) Blade or diagram efficiency
2u ( Vw f + Vws)
hb =
V12f
2 ¥ 117 ¥ (762 + 234)
=
(578.93) 2
= 0.6954 or 69.54%
(iii) Stage efficiency:
u ( Vwf + Vws)
hstage =
Dh
117 ¥ (762 + 234)
=
186.2 ¥ 103
= 0.6258 or 62.58%

In a reaction turbine, the moving blades have


converging steam passage. Therefore, when steam
passes over the moving blades, it expands with
a drop in steam pressure and increase in kinetic
energy. Thus in a reaction turbine, the steam jet
leaves the moving blades with higher velocity
than it enters the blades. The higher velocity steam
jet coming out of the moving blades, reacts on
the blades and causes them to rotate in opposite
direction. The Parson turbine which is named after
its inventor, Sir Charles A Parson, in 1884 is a good turbine, the steam does not expand completely in
example of a reaction turbine. It developed 7.5 kW the stationary nozzle, but it expands in the fixed
running at 17000 rpm. as well as moving blades, both acting as nozzles.
In modern steam turbines, both impulse and The motive force is partly impulsive and partly
reaction principles are used simultaneously. In this reaction force due to continuous expansion of
Steam Turbines 729

steam in fixed and moving blades. The work output there is a full admission. Fig. 22.35(b) shows the
from the turbine is due to both impulsive and effect of friction, when steam glides over the fixed
reactive forces. Therefore, these turbines are also and moving blades. The actual enthalpy drop is less
called the impulse-reaction turbines. However, the than the isentropic enthalpy drop.
work produced by reactive force is higher than the The reaction turbines are also compounded to
impulsive force, and hence these turbines are also reduce the rotor speed. The variation of pressure
simply referred as reaction turbines. and velocity through a three-stage reaction turbine
The pressure falls continuously as the steam is shown in Fig. 22.36.
flows over the fixed and moving blades of each In an impulse turbine, the steam pressure
stage. The steam velocity increases in each set of remains constant while steam flows through the
the fixed blades while it decreases in the moving moving blades and no thrust is exerted by the
blades. steam in the direction of the rotor axis, while in
There are a number of rows of moving blades the reaction turbine, the axial thrust is considerable
attached to the rotor and an equal number of fixed due to pressure difference of either sides of moving
blades attached to the casing. Thus these are referred blades. Dummy pistons and thrust bearings are
as rotor blades and stator blades, respectively. used to balance this axial thrust.
The fixed blades are set in the reversed direction
of moving blades as shown in Fig. 22.35(a). Due
to the fixed blades at the entrance, the steam is
admitted for the whole circumference and hence As discussed above, in reaction turbines, the steam
expands continuously while passing over the rings
of fixed and moving blades. It is accomplished by
using a tapered rotor with progressively increasing
blade height. The effect of expansion of steam on
the moving blade is to increase the relative velocity
at the exit. Therefore, the relative velocity Vr 2 is
always greater than the relative velocity at the inlet
Vr1. Thus the inlet and outlet velocity diagrams are
shown with common blade velocity u in Fig. 22.37.

The degree of reaction of a reaction turbine is


defined as the ratio of the enthalpy drop in moving
730 Thermal Engineering

blades to the total enthalpy drop in the stage. It is Vf


designated as L and may be defined as It gives L = [cot f – cot q] …(22.29)
2u
Enthalpy drop in the movig blades In a reaction turbine, the terms ‘degree of
L= …(22.26)
Total ehthalpy drop in thee stage reaction’ is a measure of proportion of work done
Figure 22.35(b) shows the actual enthalpy drop by the reaction effect. For the simple impulse
when steam glides over the fixed and moving turbine, L = 0, no heat drop in moving blades; for
blades. the Parson’s type of blading, which has the same
h0 = Enthalpy of the steam at inlet of fixed section for both the fixed and moving blades,
blades, L = 0.5 (Dhf = Dhm), then
h1 = Enthalpy of the steam at entry of 1 Vf
moving blades, and = [cot f – cot q]
2 2u
h2 = Enthalpy of the steam at exit from the It gives u = Vf (cot f – cot q) …(22.30)
moving blades. The blade velocity u can also be written as
The total enthalpy drop in a stage u = Vf (c ot a – cot b) …(22.31)
= Enthalpy drop in fixed blades + Enthalpy Comparing Eqs. (22.30) and (22.31), we get
drop in moving blades q = b and f = a
= h1 – h0 + h2 – h1 It indicates that a reaction turbine with 50%
= Dhf + Dhm degree of reaction will have moving and fixed
The total enthalpy drop for a stage (Dhm + Dhf) blades of same shape.
is equal to work done by the steam in the stage and For Parson’s reaction turbine, the inlet and outlet
it equals to, velocity diagram are symmetrical with common
Dhm + Dhf = u (Vw1 + Vw2) …(22.27) blade velocity u as shown in Fig. 22.38 and
Hence, V1 = Vr 2 and V2 = Vr1
Degree of reaction,
Vr22 – Vr21
L = …(22.28)
2u (Vw1 + Vw2 )
Vr2 = Vf 2 cosec f,
and Vr1 = Vf 1 cosec q
(Vw1 + Vw2) = Vf1 cot q + Vf 2 cot f
The velocity of flow is generally constant while
the steam passes over the blade ring,
i.e., Vf 1 = Vf 2 = Vf (say)
\ degree of reaction,
( Vf cosec f ) 2 - ( Vf cosec q ) 2
L =
2u Vf (cot q + cot f )
Vf (cosec 2 f - cosec 2 q )
or L =
2u (cot f + cot q ) The blade or diagram efficiency of a turbine is
Vf È (cot f + 1) - (cot q + 1) ˘
2 2 given as
or L = Í ˙ Work done in stage
2u ÍÎ cot f + cot q ˙˚ hb =
Energy supplied in the stage
Vf È cot f - cot q ˘
2 2
= Í ˙ u ( Vw1 + Vw 2 ) w
2u ÍÎ cot f + cot q ˙˚ = = …(22.32)
D hf + D hm Dh
Steam Turbines 731

Refer velocity triangle, Fig. 22.38; u


Using s = , we get
The work done per kg of steam V1
w = u (Vw1 + Vw2) V12
= u (V1 cos a + Vr2 cos f – u) …(22.33) Dh = [1 – s2 + 2s cos a] …(22.37)
2
We have a = f, Vr2 = V1, then Substituting Eqs (22.34) and (22.37) in
w = u (V1 cos a + V1 cos a – u) Eq. (22.32), then the blade or diagram efficiency is
= u (2V1 cos a – u)
V12 ( 2 s cos a - s 2 ) 2 ( 2 s cos a - s 2)
È 2u V u2 ˘ hb = =
= V 12 Í 2 1 cosa - 2 ˙ V12 1 - s 2 + 2 s cos a
ÍÎ V1 V1 ˙˚ [1 - s 2 + 2 s cos a ]
2
u 2 (1 + 2 s cos a - s 2) - 2
Using s = , blade-speed ratio, we get =
V1
1 + 2 s cos a - s 2
w = V 12 (2s cos a – s2) …(22.34)
2
Energy input to the blades in a stage hb = 2 – …(22.38)
1 + 2 s cosa - s 2
Dh = Kinetic energy supplied to the fixed
blades + Kinetic energy supplied to
the moving blades
From Eq. (22.38), it is evident that the blade
Kinetic energy supplied to fixed blades
efficiency would be maximum, when the term (1 +
V12 2s cos a – s2) becomes maximum.
=
2 Using the condition of maxima, differentiating
Kinetic energy supplied to the moving blades the above term with respect to blade speed ratio, s
Vr2 - Vr21 and equating to zero.
= 2
2 d
(1 + 2s cos a – s2) = 0
Vr2 - Vr21 ds
V12
\ Dh = + 2 …(22.35) 2 cos a – 2s = 0
2 2
or s = cos a …(22.39)
For the symmetrical triangles,
Substituting in Eq. (22.38), we get
Vr2 = V1
2
V12 V12 - Vr21 hb, max = 2 –
\ Dh = + 1 + 2 cos a - cos 2 a
2
2 2
2
Vr21 =2–
= V 12 – …(22.36) 1 + cos 2 a
2
Refer Fig. 22.38 for DABC 2 + 2 cos 2 a - 2
=
V r21 = u2 + V12 – 2u V1 cos a 1 + cos 2 a

u 2 + V12 - 2u V1 cosa 2 cos 2 a


Then Dh = V 12
– = …(22.40)
2 1 + cos 2 a
V12 - u 2 + 2u V1 cosa The variation of diagram efficiency with blade
= speed ratio, s for the simple impulse turbine and a
2
2 È 2 ˘ reaction stage shown in Fig. 22.39. The efficiency
V Ê uˆ u
= 1 Í1 - Á ˜ + 2 cosa ˙ curve for reaction turbine is flat for maximum value
2 Í Ë V1 ¯ V1 ˙
Î ˚ of blade speed ratio.
732 Thermal Engineering

hb 2
2cos a Sp. Volume of steam = vg @ steam pressure
2
1 + cos a
2p rh Vf 1
2
cos a Then ms = …(22.42)
vg
Reaction
turbine
Example 22.21 Show that for a Parson’s reaction
Simple
Impulse turbine, the degree of reaction is 50%.
turbine
Solution The degree of reaction of a reaction turbine
is defined as
Enthalpy drop in the moving blades
u
s= V
L =
cos a Total enthalpy drop in thhe stage
cos a 1
2 D hm
=
D hm + D hf
In terms of velocities, the enthalpy drop in moving
blades
Vr2 - Vr2
Dhm =
2 1
…(i)
Figure 22.40 shows the blade arrangement in a 2
reaction turbine. The blade height of a particular Enthalpy drop in fixed blades, with assumption that
stage can be determined by continuity equation, the velocity of steam entering the fixed blades is equal
i.e., to the absolute velocity of steam leaving the previous
Mass-flow rate of steam, moving blades.

Area flow ¥ flow velocity V12 - V22


ms = Dhf = …(ii)
Specific volume of steam 2
The velocity diagram for Parson’s reaction turbine is
Area of flow,
shown in Fig. 22.38.
Ac = Mean circumference ¥ Blade height
q = b and f = a
or Ac = 2p r h or [p (D + h)] h (m2) …(22.41) Thus, the inlet and outlet velocity diagram are
Velocity of flow = Vf 1 (m/s) symmetrical with common blade velocity u, thus using
in Eq. (ii)
V1 = Vr2 and V2 = Vr1
Vr2 - Vr2
Dhf =
2 1

2
Total enthalpy drop
Vr2 - Vr2 Vr2 - Vr2
Dhm + Dhf =
2 1 2 1
+
2 2
= 2 Dhm …(iii)
Degree of reaction of Parson reaction turbine
D hm
L = = 0.5 or 50% Proved
2 D hm
Steam Turbines 733

Example 22.22 A Parson reaction turbine running at Vf1 V1 sin a


400 rpm with 50% reaction develops 75 kW per kg of the tan q = =
Vw1 - u V1 cos a - u
steam. The exit angle of the blade is 20° and the steam
velocity is 1.4 times the blade velocity. Determine 300.2 sin 20∞
= = 1.51715
(a) Blade velocity, 300.2 cos 20∞ - 214.42
(b) Blade inlet angle. or q = 56.61°

Solution Example 22.23 In a Parson reaction turbine, the


angles of receiving tips are 35° and of discharging tips,
Given A Parson reaction turbine
20°. The blade speed is 100 m/s. Calculate the tangential
a = f = 20° ms = 1 kg/s V1 = 1.4 u
force, power developed, diagram efficiency, and axial
N = 400 rpm P = 75 kW L = 0.5 thrust of the turbine, if its steam consumption is 1 kg/min.
To find
Solution
(i) Blade velocity, and
(ii) Blade inlet angle. Given A Parson reaction turbine
a = f = 20° ms = 1 kg/min u = 100 m/s
Analysis
q = f = 35° L = 0.5
(i) Blade velocity, u as shown in Fig. 22.41
To find
(i) Tangential force,
(ii) Power developed,
(iii) Diagram efficiency, and
(iv) Axial thrust.
Analysis Velocity diagram is constructed as shown in
Fig. 22.42.
(i) Draw a horizontal line AB to some convenient
scale to represent to blade velocity u = 100 m/s.
(ii) Through the point A, draw a line AC at an angle of
20°.
Inlet whirl velocity, (iii) Through the point B, draw a line BC at an angle of
Vw1 = V1 cos a = V1 cos 20° = 0.9397 V1 35° which intersects at the point C.
= 0.9397 ¥ 1.4 u = 1.31558 u (iv) Measure length of AC = V1 = 220 m/s and BC =
Exit whirl velocity, Vr1 = 130 m/s.
Vw2 = Vw1 – u (since a = f, Vr2 = V1) (v) Through the point C, draw a vertical line CF to
= 1.31558 u – u = 0.31558 u represent the flow velocity at inlet.
Power developed
ms u ( Vw 1 + Vw2 )
P = kW
1000
1 ¥ u [1.31558 u + 0.31558 u ]
or 75 =
1000
or u2 = 45.979 ¥ 103
It gives u = 214.42 m/s
(ii) Inlet blade angle,
V1 = 1.4 u = 1.4 ¥ 214.42 = 300.2 m/s
Vf1 = Vf2 = V1 sin a, Vw1 = V1 cos a
734 Thermal Engineering

(vi) Measure length of AF = Vw1 = 207 m/s. the annulus area can be neglected. Calculate
(vii) Since Vr2 = V1, and a = q ; draw a line BD at angle (a) the rotational speed of the wheel,
of 20°. (b) the diagram power,
(viii) Join point A with a line AD, which will represent (c) the diagram efficiency,
absolute velocity of steam at outlet i.e., V2 = (d) the enthalpy drop of the steam in this stage.
130 m/s.
(ix) Draw a vertical line DE through point D to Solution
represent flow velocity Vf2 at outlet.
Given A reaction turbine.
(x) Measure length of segment EA = Vw2 = 107 m/s.
p1 = 2.7 bar, (dry and saturated steam)
(a) Tangential force
h = 40 mm 0.04 m,
= ms (Vw1 + Vw2)
3
= (1/60) ¥ (207 + 107) V 1 = 90 m/s a = 20° Vf = u
4
= 5.233 N ms = 9000 kg/h = 2.5 kg/s
(b) Power developed;
To find
ms u ( Vw1 + Vw 2 )
P = kW (i) Rotational speed of the wheel,
1000
(ii) Diagram power,
(1/60) ¥ 100 ¥ [207 + 107]
= (iii) Diagram efficiency, and
1000
(iv) Enthalpy drop of the steam in this stage.
= 0.5244 kW
(c) Diagram efficiency, Assumptions
Work done in a stage (i) Degree of reaction in the turbine is 50%.
hb = (ii) Flow of steam in the turbine is frictionless and
Energy supplied in the stage
adiabatic.
The work done per kg of steam in a stage
w = u (Vw1 + Vw2) Analysis The velocity diagram is shown in Fig. 22.43
= 100 ¥ (207 + 107) = 31,400 J/kg (i) The flow velocity is given as
Energy supplied per kg of steam in a stage
3
Eq. (22.36) Vf = u = V1 sin a = 90 sin 20°
4
Vr2 = 30.78 m/s
Dh = V12
1
– 4
2 Then u = Vf = 41.04 m/s
3
(130 m/s) 2
= (220 m/s)2 – The mass-flow rate of the steam is given by
2 V A
= 39950 J/kg ms = f
v
31400 J/kg
Thus hb = = 0.786 or 78.6% where A = annular area,
39950 J/kg
Vf = flow velocity, and
(d) Axial thrust; Fa = ms (Vf 2 – Vf1) = 0 (∵ Vf 2 v = vg @ 2.7 bar = 0.6686 m3/kg
= V f 1)
30.78 A
Therefore, 2.5 kg/s =
Example 22.24 A stage of steam turbine with Parson 0.6686
blading delivers dry, saturated steam at 2.7 bar from the It gives, A = 0.054 m3
fixed blades at 90 m/s. The mean blade height is 40 mm, Further, A = 2p r h = 2p ¥ r ¥ 0.04
and the moving blade’s exit angle is 20°. The axial
It gives r = 0.215 m
velocity of steam is three quarter of the blade velocity at
Then the wheel rotational speed
the mean radius. The steam is supplied to the stage at the
rate of 9000 kg/h. The effect of the blade tip thickness on Blade speed u
N = =
2p r 2p r
Steam Turbines 735

The energy input to moving blade per stage per


kg of steam
Vr2
V 12
1
= –
2
53.312
= 902 – = 6679.02 J/kg
2
Total energy supplied to moving blade
= 2.5 ¥ 6679.02 = 16697.55 J/s
Work done
Then hstage =
Energy supplied
13.14 ¥ 103 J/s
= = 0.787 = 78.7%
16697.55 J/s
(iv) Enthalpy drop in the moving blades can be
obtained from stage efficiency as
Workdone per kg of steam
hstage =
Enthalpy dorp per stage
13.14/2.5
or Enthalpy drop = = 8.348 kJ/kg
0.787
41.04 Example 22.25 At a stage in a reaction turbine,
= = 30.24 rev/s the mean blade ring diameter is 1 m. The turbine runs
2p ¥ 0.216
= 1814 rpm at 3000 rpm. The blades are designed for a degree of
(ii) The diagram power reaction of 50% with exit angles of 30° and inlet angles
of 50°. The turbine is supplied with a steam at 10000 kg/h
m uV
P = s w and the stage efficiency is 85%. Determine
1000
We have (a) power output of the stage,
Vw1 = V1 cos a = 90 cos 20° = 84.57 m/s (b) specific enthalpy drop in kJ/kg,
Vw2 = Vr2 cos f – u = V1 cos f – u (c) percentage increase in relative velocity of steam
(∵ a = f and Vr1 = V1) over moving blades,
= 90 cos 20° – 41.04 = 43.53 m/s (d) the specific steam consumption.
\ Vw = Vw1 + Vw2 = 84.57 + 43.53
Solution
= 128.10 m/s
2.5 ¥ 41.04 ¥ 128.1 Given A 50% reaction turbine with
and P = = 13.14 kW
1000 L = 0.5 D =1m N = 3000 rpm
(iii) Diagram efficiency or stage efficiency a = f = 30° q = b = 50°
The relative efficiency at the inlet ms = 10000 kg/h = 2.778 kg/s hstage = 0.85
V r21 = V 12 + u2 – 2V1 u cos a To find
= 902 + 41.042 – 2 ¥ 90 ¥ 41.04 cos 20° (i) The power output of the stage,
= 2842.58 (ii) Specific enthalpy drop (Dh) in kJ/kg,
or Vr1 = 53.31 m/s (iii) Percentage increase in relative velocity of steam
Note: The magnitude of Vf1, Vw1, Vw2, Vr1, can over moving blades, and
also be obtained by measurements on vector (iv) Specific steam consumption.
diagram.
736 Thermal Engineering

Analysis The blade velocity is given by The percentage increase in relative velocity of
p DN p ¥ (1 m) ¥ (3000 rpm) steam over moving blades
u = = Vr2 - Vr1
60 60 = ¥ 100
= 157.08 m/s Vr1
The velocity diagram for given turbine is constructed 363.38 - 229.6
and shown in Fig. 22.44. = ¥ 100 = 58.27%
229.6∞
(iv) The specific steam consumption
m
ssc = s ¥ 3600
P
ssc u Vw ssc ¥ P
Since 1 kWh = 3600 kJ = =
1000 ms
2.778 ¥ 3600
Thus ssc = = 49.58 kg/kWh
201.72

Example 22.26 A reaction turbine runs at 3000 rpm


and the steam consumption is 20000 kg/h. The pressure
of steam at a certain pair is 2 bar, its dryness fraction
is 0.93 and the power developed by the pair is 50 kW.
From the measurements
The discharge blade angle is 20° for both the fixed and
V1 = 351.8 m/s, Vw1 = 304.7 m/s moving blades and the axial velocity of flow is 0.72 times
Vf1 = 229.6 m/s Vw2 = 157.62 m/s the blade velocity. Find the drum diameter and the blade
The total whirl velocity height. Take the tip leakage steam as 8%. Neglect the
Vw = Vw1 + Vw2 = 304.7 + 157.62 blade thickness.
= 462.32 m/s
(i) The power developed by the turbine Solution
ms u Vw Given A reaction turbine with
P =
1000 N = 3000 rpm ms = 20000 kg/h
157.08 ¥ 462.32 p = 2 bar x = 0.93
= 2.778 ¥
1000 P = 50 kW a = f = 20°
= 201.72 kW Vf 1 = Vf 2 = 0.72 u
(ii) The stage efficiency is given by Leakage of steam = 8% = 0.08
Power developed
hstage =
Total enthalpy drop per stage
P
=
ms Dh
Thus the specific enthalpy drop can be obtained
as
P 201.72
Dh = =
ms hstage 2.778 ¥ 0.85
= 85.43 kJ/kg
(iii) The exit relative velocity can be expressed as
Vr2 cos f = Vw2 + u
To find
157.62 + 157.08 (i) Drum diameter, and
or Vr2 = = 363.38 m/s
cos 30 (ii) Blade height.
Steam Turbines 737

Assumptions Using the value of (d + h)2 from Eq. (iv)


(i) 50% Parson reaction turbine, and 20000 p ¥ 0.1341 h ¥ 113.09
(ii) No heat loss form the casing of the turbine. =
3600 0.8237
Analysis h = 0.096 m
The actual mass flow rate of steam, Thus, D = 0.3662 – 0.096 = 0.270 m
m act = theoretical mass flow ¥ (1 – tip leakage)
20000 22.10 COMPARISON BETWEEN
= ¥ (1 – 0.08) = 5.11 kg/s
3600 IMPULSE AND REACTION
The specific volume of steam TURBINES
vs = x vg @ 2 bar = 0.93 ¥ 0.8857
The differences between the impulse and reaction
= 0.8237 m3/kg
turbines are summarized in Table 22.1 given below.
p ( D + h) N
The blade speed, u =
60 Table 22.1 Comparison between impulse and reaction
p ( D + h) turbines
u = ¥ 3000 = 157.08 (D + h) …(i)
60
S.No. Impulse Turbines Reaction Turbines
The axial flow velocity
1. Pressure drop takes Pressure drop takes
Vf1 = 0.72 u = 113.09 (D + h) …(ii)
place in nozzles only place in both fixed
From velocity triangle, DACF and pressure remains blades (nozzles) and
Vf1 constant in moving moving blades. Thus,
= tan 20°
Vw 1 blades. This means pressures value is
that the relative different on the two
113.09 ( D + h)
or Vw1 = = 310.73 (d + h) …(iii) velocity remains sides of the moving
tan 20∞
same in moving blades. Continuous
and Vw2 = Vw1 – u = 310.73 (D + h) – 157.08 (d + h) blades or reduces expansion of steam
= 153.65 (d + h) slightly on account of means relative
The total whirl velocity friction. velocity in moving
Vw = Vw1 + Vw2 blades increases
= 370.73 (D + h) + 153.65 (D + h) and volume of flow
= 464.381 (D + h) changes.
Power developed by the turbine 2. Blade shape is Blade shape is
m uV profile type and its aerofoil type and
P = act w
1000 manufacturing is its manufacturing is
5.11 ¥ 157.08 ( D + h) ¥ 464.38 ( D + h) simple. difficult.
or 50 =
1000 3. Blade passage is Blade passage is
of constant cross- of variable cross-
or (D + h)2 = 0.1341 …(iv)
sectional area since sectional area
or D + h = 0.3662 …(v)
there is no expansion. (converging type) due
Further, the theoretical mass-flow rate to expansion.
p ( D + h) h Vf1
ms = 4. Diaphragm contains Fixed blades attached
vs the nozzles and the to casing serve as
20000 p ¥ ( D + h) h ¥ 113.09 ( D + h) rotor construction is nozzles and the rotor
= disc or wheel type. construction is drum
3600 0.8237
type.
Contd.
738 Thermal Engineering

5. Because of large Because of small decrease in kinetic energy of steam coming out of
pressure drop in pressure drop in each the nozzle. The decrease in kinetic energy is caused
nozzles, the number stage, the number of due to these causes.
of stages are less. stages are larger for
1. Heat loss from steam before entering the
the same pressure
drop. Reaction
nozzle.
turbines are multi- 2. The friction in the nozzle reduces available
stage turbines only. enthalpy drop, which is less than isentropic
6. Because of large Because of small
enthalpy drop. Thus, the actual velocity
pressure drop, the pressure drop, the leaving the nozzle is less than that obtained
blade speed and blade speed and with isentropic expansion.
steam speed are steam speed are 3. Viscous friction between steam particles.
larger. Also, the small. The diagram 4. Flow deflection in the nozzle.
diagram efficiency efficiency vs blade 5. Development of boundary layer in nozzle.
decreases rapidly speed/steam speed
6. Turbulence in the nozzle.
with the change in ratio curve is flat
designed blade speed/ for large values, so
steam speed ratio. greater working range
is available. Steam leaves the boiler and reaches the condenser
7. Overall friction losses Leakage losses are after passing through the main valve, regulating
are more compared to more compared to valves, nozzles, clearance spaces between nozzles
leakage losses. friction losses. and moving blades, diaphragm and rotating shaft,
8. Occupies less space Occupies more space etc. Further, there is large pressure difference inside
per unit power. per unit power. and outside, from one location to another location
9. Suitable for small Suitable for medium across these devices. Therefore, the steam leakages
power. and large power. take place through (i) main valve and regulating
valves, (ii) seals and glands, (iii) spaces between
nozzles and moving blades, (iv) space between
diaphragm and shafts of turbine, and (v) space
between moving blade rings and turbine casing.
The total energy contents of steam supplied to a
The steam leakages through these devices carry
turbine is not completely converted into mechanical
energy, which is a wastage of steam.
energy. There are certain energy losses, which
occur inside a turbine. These are
1. Admission losses Frictional resistance is offered during flow of steam
2. Leakage losses through nozzles, on moving and stationary blades.
3. Friction losses In most of the turbines, the wheel is rotating in
4. Exhaust losses a space full of steam. The viscous friction at the
5. Radiation and convection losses wheel surface causes admission loss as steam
6. Losses due to moisture passes from nozzle to wheel. The effect of partial
admission creates eddies in the blade channels and
7. Carry over losses
blade windage losses.
The surface of the curved moving blades and
stationary blades offers resistance, which increases
In actual practice, the flow through the nozzle is not with increase in relative velocity between them and
isentropic, but accompanied with losses, causing a roughness of blade surface.
Steam Turbines 739

The energy loss also takes place when the steam the steam flow rate by means of valves interposed
jet turns along the curvature of the blade surface. between the boiler and turbine. The steam turbine
The turning losses depend on the angle of turning. may be governed by the following possible
methods:
1. Throttle governing
The energy content of steam is not fully utilized in
2. Nozzle control governing
the turbine. The exhaust steam coming out of the
turbine and entering the condenser carries some of 3. By-pass governing
kinetic energy and useful enthalpy, which is direct 4. Combination of any of the above two meth-
energy loss. ods
5. Emergency governing

The steam turbine is operated at a relatively high


temperature. Therefore, some of the heat energy of In throttle governing, the pressure of steam is
steam is radiated and convected from the body of reduced by passing the steam through a restricted
the turbine to its surroundings. These are the direct passage like partially opened steam stop valve in
losses and are minimized by proper insulation. order to maintain the speed of the turbine constant
at part load. The throttle governing is most widely
used on small turbines, because of its low initial
The steam passing through the last stage of the cost and simple mechanism.
turbine has very high velocity and it contains large For the turbines of small power plants, the
moisture. The liquid particles have lesser velocity valves are light and a centrifugal governor shown
than that of vapour particles. Hence the liquid in Fig. 22.46 may be used to actuate the valve
particles obstruct the flow of vapour particles in the directly. The centrifugal governor consists of flying
last stage of turbine and therefore, a part of kinetic balls attached on the arm connected to the sleeve.
energy of steam is lost. If the dryness fraction of The sleeve moves up and down axially on a rotating
steam falls below 0.88, the erosion and corrosion of shaft geared to the turbine shaft.
blades can also take place.
As the load on the turbine decreases, the turbine
shaft speed increases. With increase in speed, the
flying ball of the governor flies apart and raises
When steam passes from one stage to the next stage the sleeve which operates a lever through fulcrum
through the diaphram, some energy loss takes place, and actuates the main valve to close partially and
which is referred as carry-over loss. Therefore, the reduce the mass flow rate of steam.
kinetic energy of steam available at succeeding row For large steam turbines, there is considerable
of moving blades for utilization is less than that at friction at the valves and an oil-operated servo
the exit of preceding row. It is due to formation of system can be used to boost the sensitivity of the
eddies in the annular space between the nozzle (Or lever connected to the governor sleeve. The small
fixed blades) and moving blades. deflection of the lever of the governor is amplified
with the help of the relay system shown in Fig. 22.47.
It consists of a throttle valve (A), relay piston (B),
differential lever (C), frictionless pilot valves (D),
The purpose of governing of steam turbine is to servomotor and piston spindle (E) connected to
maintain its speed as constant, irrespective of its the throttle valve. A differential lever is attached at
load. The turbine speed is controlled by varying one end of the governor sleeve and the other end to
740 Thermal Engineering

the throttle valve spindle and at some intermediate throttle valve partially. The downward movement of
point, to the pilot valve spindle. The pilot valves are piston spindle (E) also lowers the pilot valves (D),
two small piston valves, which cover two ports in both ports are covered and the relay piston position
an oil chamber without any lap. Pipes G are open to is locked. The steam pressure and its flow rate
oil drain tank. Any deflection of the sleeve causes reduce, and thus the turbine power output reduces
the displacement of pilot valves in the oil chamber till its speed falls fairly near to normal value.
through a differential lever. Accordingly, the oil If the load on the turbine increases, the lever
under the pressure enters either the upper half or deflection opens the lower half passage for oil entry
lower half of the chamber. and an opposite operation of the piston opens the
If the load on the turbine decreases, the excessive throttle valve for more steam flow and thus more
power of the turbine will accelerate the rotor and power output of turbine till the speed again matches
causes the governor sleeve to lift. Thus, the lever with normal value.
deflection will raise the pilot valves spindle (D1) and The measurement shows that the steam con-
the upper port is opened to oil entry and the lower sumption rate, ms (kg/h) is linearly proportional to
port for oil exit. The high-pressure oil enters the the turbine load during throttle-control governing.
cylinder and pushes the relay piston (B) downward. Figure 22.48(a) shows a graph of the steam con-
Thus, the piston spindle (E) descends and closes the sumption and load on turbine. The linear line on the
Steam Turbines 741

graph is called Willan’s line given by of nozzles is controlled by a separate valve. When
ms = a L + C ...(22.43) load on the turbine decreases, the required number
where a is the steam rate, kg/kWh of nozzles can be shut off. The nozzle control
L is load on turbine, kW usually affects the first stage of the turbine only.
C is no load steam consumption Other stages remain unaffected. In nozzle control
governing, the mass flow rate of steam is regulated
In throttle governing, the pressure of steam
rather than regulating steam pressure. It has the
is reduced at turbine entry. Thus, the energy
advantage of using steam at full boiler pressure and
availability is reduced. Further, if throttle governing
reduction in steam consumption rate at part loads.
is used at low loads, the turbine efficiency is
drastically reduced. Figure 22.49 shows a schematic arrangement of
nozzle control governing. It consists of three sets of
nozzle as N1, N2 and N3 having 5, 4 and 12 nozzles,
respectively. The opening and closing of these sets
In nozzle control governing, the nozzles are grouped of nozzles is controlled by valves V1, V2 and V3.
in sets of two, three and more groups and each set
742 Thermal Engineering

When a steam turbine is overloaded, additional Every steam turbine is also provided with an
fresh steam is admitted through a by-pass valve to emergency governor, which comes in operation,
later stages of the turbine. Within the economical when
loads, the turbine is governed by a speed governor 1. speed of shaft exceeds beyond 110%,
through nozzle control or throttle governing. But 2. balancing of turbine is disturbed,
for all loads greater than economical load, a by- 3. lubrication system fails, and
pass valve is opened, allowing some fresh steam
4. the vacuum in the condenser is too less or
from the steam chest to enter the later stage of the
coolant supply to the condenser is inad-
turbine.
equate.
Steam Turbines 743

In a back pressure turbine, steam enters at boiler


There are dual demand of power and process
pressure and after expansion to some intermediate
heating simultaneously in several industries, such
pressure, it is exhausted into a pipe, which leads
as papermaking, textile, sugar, chemical dying,
to a process plant. This system may be used when
refining, pulp, brewing, etc. For power generation,
power generated by expanding steam from boiler
high pressure, superheated steam is required in a
pressure to heating pressure is equal to or greater
turbine, while the saturated steam at moderate
than the power requirement of the plant.
pressure fulfils the requirement of process heating.
The steam power plants convert approximately
40% of heat supplied into useful work and reject
remaining 60% of heat as waste heat to cooling In a pass-out or extraction turbine, the steam enters
water in the condenser. For process heating, the at boiler pressure, and after its expansion to some
low-pressure saturated steam gives away its latent intermediate pressure between inlet and exhaust,
heat at constant temperature. In this situation, it is some steam is extracted for process heating and the
not economical to install two boilers to generate remaining steam is allowed to expand in the turbine
steam at different pressures for power generation to condenser pressure.
and process heating. It is better to design one plant The choice between back-pressure turbine and
to offer both services by suitable modification. Such extraction-condensing turbine depends mainly
a plant is known as a combined heat and power or on the quantities of power and heat, quality of
co-generation plant. heat, and economic factors. The extraction points
There are two types of steam turbines most of steam from the turbine could be more than
widely used in combined power and process plants: one, depending on the temperature levels of heat
back-pressure and the extraction-condensing types, required by the processes.
shown in Fig. 22.50.
744 Thermal Engineering

Summary
rotodynamic machine,
in which a mechanical rotating shaft work is Work done on the blade
developed by steady flow of steam through it. hb =
Energy supplied to the blade
The motive force in a steam turbine is obtained by
changing the momentum of a high-velocity jet of 2u Vw V12 - V22
= =
steam, impinged on the curved blades fixed on the V12 V12
rotor. s=
impulse turbine works on the principle u cosa
becomes equal to .
of impulse, Newton’s second law of motion. V1 2
The impulse force is generated by the change Work done on the blade
in momentum of steam gliding on the blades, hstage =
Energy supplied per stage
causing the rotor to spin.
u Vw
A reaction turbine works on the principle of =
Dh
Newton’s third law of motion, in which the
rotation is caused by the reaction force generated
by the momentum change of the flowing steam. Fa = ms (Vf 1 – Vf 2)
reaction turbines do not operate on
the rotational speed of the turbine to the practical pure reaction, but they utilize the principle of
limits. There are three types of compounded impulse and reaction simultaneously. Therefore,
impulse turbines. these are also called impulse-reaction turbines.
The Parson’s reaction turbine operates on 50% of
In the velocity-compounded impulse turbine,
degree of reaction.
the steam expands to exhaust pressure in a single
set of nozzles and the kinetic energy of the steam
is absorbed by two or three sets of moving blades. Enthalpy drop in the moving blades
L =
In the pressure-compounded impulse turbine, Total enthalpy drop in thhe stage
the steam expands gradually into a number of
D hm
stages. Each stage comprises of its own set of =
nozzles and moving blades. Dh
The pressure-velocity compounded impulse
turbine, is a combination of pressure and velocity as
compounding. u ( Vw1 + Vw2 ) 2
hb = = 2-
Dh 1 + 2 s cosa - s 2
impulse turbine is given by
s = cos a
W = ms u (Vw1 + Vw2) = ms u Vw (W )

Glossary
Turbine A rotadynamic machine Whirl velocity Tangential component of velocity,
Impulse turbine Steam expands in nozzles only and responsible for useful work
blades spin due to change in momentum of steam Flow velocity Axial component of velocity, allows the
Reaction turbine Steam expands in fixed and moving steam flow across the wheel
blades, thus reactive force acts on blades to cause them Relative velocity Velocity of steam relative to blade
to rotate speed
Steam Turbines 745

Stage A set of fixed blades (nozzles) and moving blades Blade-velocity coefficient Ratio of outlet relative
Stage efficiency Ratio of work done on blade to energy velocity to inlet relative velocity
supplied per stage Axial flow turbine Turbine, in which steam leaves
Blade-efficiency Ratio of work done on blade to energy blades at 90° to blade velocity u
supplied to blade Compounding Using more than one set of nozzles and
Blade-speed ratio Ratio of blade velocity to absolute moving blades in series keyed on same shaft
steam velocity Degree of reaction Ratio of enthalpy drop in moving
blades to total enthalpy drop in a stage

Review Questions
1. Distinguish between impulse and reaction tur- 11. Explain the working of an impulse reaction tur-
bines. bine.
2. Explain the principle of working of an impulse 12. Draw the velocity triangle for a 50% reaction
turbine. turbine and derive the expressions for work done
3. Write the details of a simple impulse turbine. and blade efficiency.
4. Draw the velocity-triangle diagram for an im- 13. Define degree of reaction.
pulse turbine blades and derive the expressions 14. Prove that for a 50% reaction turbines, a = f and
for work done and axial thrust. q = b.
5. Define blade efficiency and stage efficiency. 15. Prove that the shape of fixed blades and moving
6. Derive an expression for maximum blade effi- blades is identical in a 50% reaction turbine.
ciency for an impulse turbine in terms of blade 16. Prove that V1 = Vr2 and V2 = Vr1 in a 50% reaction
speed ratio. turbine.
7. Why are steam turbines compounded? Explain. 17. Derive an expression for optimum stage efficien-
8. Describe the various methods of compounding in cy of a reaction turbine.
an impulse turbine. 18. Explain how the flow rate can be obtained from
9. What is the pressure–velocity compounding? given blade height, mean diameter and steam
Write its advantages. condition at that stage.
10. Prove that the net efficiency of a simple turbine is
given by
hnet = hstage ¥ hN ¥ hmech.

Problems
1. Steam issues from the nozzles of a single impulse 2. Steam leaves the nozzle of a single impulse wheel
turbine at 850 m/s. The blades are moving at turbine at 900s. The nozzle angle is 20° and the
350 m/s. The blade tip angles at inlet and exit are blade angles are 30° at inlet and outlet. What
each 36°. The steam enters the blades without is the blade velocity and the work done per kg
shock and the flow over the blades is frictionless. of steam? Assume the flow over the blades as
Determine (a) the angle at which the nozzles are frictionless. [270 m/s, 307.8 kJ/kg]
inclined to the direction of motion of the blades, 3. The mean diameter of the blades of an impulse
and (b) the diagram efficiency. turbine with a single-row wheel is 1 m and the
[(a) 22°, (b) 86%] speed of rotation is 3,000 rpm. The nozzle angle
746 Thermal Engineering

is 18°, the ratio of blade speed to steam speed 8. The blade angles for a de-Laval turbine are
is 0.42, the ratio of the relative velocity at the designed for the maximum efficiency for which
outlet from the blades to that at inlet is 0.84. The the blade velocity is fixed at 200 m/s and the blade
outlet angle of the blade is to be 3° less than the inlet and discharge tip angles are 30°. Determine
inlet angle. The steam flow is 7 kg/s. Determine the required steam jet velocity and the nozzle
(a) tangential force on the blades, (b) power angle, neglecting friction over the ring of moving
developed on the blades, (c) blading efficiency, blades. Hence, determine the diagram efficiency
and (d) axial thrust on the blades. of the turbine.
[(a) 2205 N (b) 346.18 kW [V1 = 407.5 m/s; a = 16°; h = 92.4%]
(c) 84.8% (d) 8.5 N] 9. Steam at a velocity of 400 m/s relative to the
4. In a single-stage impulse turbine, the steam moving blades enters an impulse turbine at an
jet leaves the nozzles at 20° to the plane of the angle of 30°. The blade velocity is 250 m/s.
wheel at a speed of 670 m/s and it enters the The work developed in the blades is estimated
moving blades at an angle of 35° to the drum to be 165.54 kW/kg. Assuming the blades to
axis. The moving blades are symmetrical in be symmetrical in shape, determine the blade
shape. Determine the blade velocity and diagram efficiency and blade-velocity coefficient.
efficiency. [83.5%; 0.9]
[300 m/s; 89.4%] 10. From the pair of blades of an impulse turbine, the
5. Steam with a velocity of 800 m/s enters an steam leaves the moving blades at an absolute
impulse turbine ring and drives the rotor at velocity of 130 m/s. The nozzles have a discharge
3000 rpm. The jet angle is 20° and the mean drum angle of 20° and the steam enters the blades
diameter is 1.4 m. Assuming that inlet and exit without shock. Determine the following for the
angles of the moving blades are equal and a blade maximum efficiency conditions.
velocity coefficient of 0.85, find
(a) The blade angles
(a) the blade angles (b) The steam jet and blade velocity
(b) diagram efficiency (c) The stage efficiency, if the nozzle efficiency
(c) power developed per kg per second of is 90%.
steam flow Assume equiangular moving blades.
(d) stage efficiency, if the nozzle efficiency is [(a) 36°, (b) V1 = 435 m/s; u = 204 m/s;
95% (c) 79.2%]
[(a) 28° (b) 56% (c) 179.37 kW (d) 53.2%] 11. A single-stage impulse turbine nozzle issues a
6. The velocity of jet of steam entering a de-Laval steam jet at a velocity of 450 m/s at an angle of
turbine is 500 m/s. The nozzles are inclined at 18° to the plane of wheels. The blade-speed ratio
20° to the direction of blades. The blade speed is is 0.42. The blade velocity coefficient is 0.9 and
200 m/s and the exit angle of the moving blades is the exit angle of the moving blades is 27°. Draw a
25°. For mass flow rate of 5 kg/s, determine velocity diagram for the stage and determine
(a) exit velocity of steam (a) axial thrust on the bearings,
(b) diagram efficiency (b) power developed.
(c) power developed. Assume the mass-flow rate of steam to be
[(a) 162 m/s (b) 90.4% (c) 225.95 kW] 10 kg/s. [(a) 221.7 N, (b) 872.6 kW]
7. Show that in an impulse turbine having a fixed 12. In an impulse turbine, the steam enters a ring of
nozzle angle a, equiangular blades with blade moving blades with a relative velocity of 209 m/s.
velocity coefficient ‘k’, the maximum blade The nozzles are inclined at 20° to the plane of
efficiency is given by, the wheel. The moving blades are symmetrical
in shape and the blade velocity is 157 m/s. The
Ê1+ k ˆ
hmax = Á cos 2 a friction losses in the blades are equivalent to
Ë 2 ˜¯
Steam Turbines 747

14%. For an axial thrust of 171.6 N, determine to the motion of the blades. Calculate the power
the following: developed if the mass-flow rate is 5 kg/s.
(a) Mass flow rate of steam, [89.2 kW]
(b) Steam jet velocity, 17. One expansion in a reaction turbine has 8 pairs
(c) Power developed. of blades of an outlet angle of 20°. The mean
[(a) 98.1 kg/s, (b) 350 m/s, (c) 489.3 kW] diameter of the expansion is 50 cm and the rotor
13. In a reaction turbine, the diameter of the rotor speed is 3000 rpm. The ratio of blade speed to
is 2 m and its speed is 840 rpm. The steam con- steam speed is 0.8. The efficiency for the stage
sumption amounts to 870 kg/min. The height of is 80%. Determine the power developed and the
the blade at a particular stage is 15 cm. The exit adiabatic heat drop during the expansion for a
angle of the nozzle and the moving blades is 25°. steam-flow rate of 5 kg/s.
The pressure at this stage is 0.3 bar and steam is [320.17 kW; 76.8 kJ/kg]
0.98 dry. Estimate the power developed and the 18. A reaction turbine runs at 600 rpm and consumes
heat drop in kJ/s. [199.4 kW; 17.63 kJ/s] 18000 kg/h of steam. The exit angle of the fixed
14. In a stage of an impulse turbine provided with and moving blades are 20°. The axial velocity of
a single-row wheel, the mean diameter of the flow is 0.75 times the blade velocity. Determine
blade ring is 80 cm and the speed of rotation is the drum diameter and blade height of a particu-
3,000 rpm. The steam issues from the nozzle with lar stage where steam pressure is 2 bar and it is
a velocity of 275 m/s and the nozzle angle is 20°. 0.95 dry. Assume the power developed to be 15
The inlet and outlet angles of the blades are equal, kW and the tip leakage of steam as 7%.
and due to friction in the blade channels the rela- [0.9728 m; h = 5.05 cm]
tive velocity of the steam at outlet from the blade 19. Deduce an expression for work done per stage of
is 0.86 times the relative velocity of steam enter- a reaction turbine and determine the condition for
ing the blades. What is the power developed in maximum efficiency.
the blading when the axial thrust on the blades is If the blade speed of a Parson’s reaction turbine
12.2 kg? is 288 m/s and exit angles are 20°, determine the
15. Steam issues from the nozzles of a de-Laval tur- work done per kg and the maximum diagram
bine with a velocity of 920 m/s. The nozzle angle efficiency of the turbine.
is 20°, the mean diameter of the blades is 25 cm [u = V1 cos a; 82678 Nm]
and the speed of rotation is 20,000 rpm. The 20. In a stage of 50% Parson’s reaction turbine, the
steam flow through the turbine is 0.18 kg/s. If the steam consumption is 1800 kg/h and it runs at
ratio of relative velocity at outlet from the blades 300 rpm. The discharge blade tip angles are
to that at inlet is 0.82, calculate 20° for both fixed and moving blades. The axial
(a) Tangential force on blades velocity of flow is 0.7 times the blade velocity.
(b) Work done on blades per second Determine the drum diameter and blade height
(c) Power of the wheel of a particular turbine pair where the pressure
(d) Efficiency of blading is 2.0 bar of steam, 0.95 dry and the power
developed amounts to 3.75 kW.
(e) Axial force on blades,
[91.89 cm, 11.41 cm]
(f ) Inlet angle of blades for shock-less in flow
21. At a particular stage of 50% reaction turbine, the
of steam.
pressure is 1.4 bar and steam is 0.9 dry. The inlet
Assume that the outlet angle of blades is equal to
and outlet angles are 35° and 20°, respectively.
the inlet angle.
The blade velocity is 67 m/s. Determine the blade
16. In Problem 15, in a Parson’s reaction turbine the
height, if the ratio of drum diameter to blade
exit angles of the moving blades is 20° and the
height is 8.0 for a mass-flow rate of 4.5 kg/s.
absolute discharge velocity of steam from the
Also, find the power developed.
moving blades is 50 m/s in the direction at 115°
[5.9 cm; 63.93 kW]
748 Thermal Engineering

22. In a stage of impulse reaction turbine, operat- equiangular. The blade friction factor is 0.86.
ing with 50% degree of reaction, the blades are Determine the power developed, if axial thrust on
identical in shape. The outlet angle of moving blade-end bearing of the rotor is 118 N.
blades is 19° and the absolute discharge velocity 24. In a two-stage, velocity-compounded steam tur-
of steam is 100 m/s in the direction at 100° to bine, the mean blade speed is 150 m/s, while
the motion of blades. If the rate of flow of steam steam velocity as it is issued from the nozzle is
through the turbine is 15000 kg/h, calculate the 675 m/s. The nozzle angle is 20°, the exit angle
power developed by the turbine in kW. of first-row moving blades, fixed blades and
23. In a stage of implulse reaction turbine provided second-row moving blades are 25°, 25°, and 30°,
with a single-row wheel, the mean diameter of respectively. The blade friction coefficient is 0.9.
blades is 1 m, it runs at 3000 rpm. The steam If the steam-flow rate is 4.5 kg/s, determine
issues from the nozzle at a velocity of 350 m/s (i) power output, and (ii) diagram efficiency
and the nozzle angle is 20°. The rotor blades are

Objective Questions
1. The steam turbines is a 7. In a reaction turbine, steam expands
(a) rotary machine (a) in the nozzle only
(b) reciprocating machine (b) in the moving blades only
(c) rotodynamic machine (c) in the fixed and moving blades
(d) none of the above (d) none of the above
2. Thermodynamic efficiency of a steam turbine is 8. A simple impulse turbine consists of
(a) less than a steam engine (a) one set of nozzles and one set of moving
(b) less than a Diesel engine blades
(c) less than a petrol engine (b) two sets of nozzle and one set of moving
blades
(d) none of the above
(c) one set each of fixed and moving blades
3. From inlet to exit of steam nozzle, the pressure
(d) none of the above
(a) increases (b) decreases
9. Tangential components of velocity is called
(c) remains constant (d) none of the above
(a) relative velocity (b) flow velocity
4. From inlet to exit of moving blades in case of im-
pulse turbine, the pressure (c) whirl velocity (d) absolute velocity
(a) increases (b) decreases 10. Axial components of velocity is called
(c) remains constant (d) none of the above (a) relative velocity (b) flow velocity
5. From inlet to exit of moving blades in case of a (c) whirl velocity (d) absolute velocity
reaction turbine, the pressure 11. The stage efficiency is also called
(a) increases (b) decreases (a) blade efficiency
(c) remains constant (d) none of the above (b) diagram efficiency
6. In an impulse turbine, steam expands (c) gross efficiency
(a) in the nozzle only (d) none of the above
(b) in the moving blades only 12. The blade efficiency is also called
(c) in the fixed and moving blades (a) stage efficiency
(b) diagram efficiency
(d) none of the above
Steam Turbines 749

(c) gross efficiency 19. In a pressure-compounded impulse steam tur-


(d) none of the above bine, as compared to velocity compounding, the
13. The stage efficiency of a steam turbine is also number of stages is
given by (a) less (b) more
Nozzle efficiency (c) same (d) none of the above
(a)
Blade efficiency 20. The degree of reaction is defined as
Blade efficiency Enthalpy drop in moving blades
(b) (a)
Nozzle efficiency Enthalpy drop in fixed bladess
(c) nozzle efficiency ¥ blade efficiency Enthalpy drop in moving blades
(d) none of the above (b)
Total enthalpy drop
14. The overall efficiency of a steam turbine is given
Enthalpy drop in fixed blades
by (c)
hNozzle Total enthalpy drop
(a) ¥ hstage
hBlade (d) none of the above
hBlade 21. For a Parson reaction turbine, the degree of reac-
(b) ¥ hstage tion is
hNozzle
(c) hNozzle ¥ hStage ¥ hMech (a) 80% (b) 50%
(d) none of the above (c) 75% (d) none of the above
15. The blade-speed ratio is given by 22. For a Parson reaction turbine, the condition for
Absolute velocity maximum efficiency is given by
(a) u
Blade velocity (a) = 2 cosa
Blade velocity V1
(b) u
Absolute velocity (b) = cosa
Blade velocity V1
(c) u
Absolute velocity (c) = cos 2 a
V1
(d) none of the above
(d) none of the above
16. In a velocity-compounded impulse steam turbine,
23. In throttle governing of steam turbines
steam expands in
(a) mass-flow rate of steam is regulated
(a) one set of nozzles only
(b) pressure of steam is regulated
(b) more than one set of nozzles
(c) additional steam is supplied
(c) fixed and moving blades
(d) none of the above
(d) none of the above
24. In nozzle control governing of steam turbines
17. In a pressure-compounded impulse steam tur-
(a) mass-flow rate of steam is regulated
bine, steam expands in
(b) pressure of steam is regulated
(a) nozzles and fixed blades only (c) additional steam is supplied
(b) moving blades only (d) none of the above
(c) fixed and moving blades both
25. In by-pass governing of steam turbines
(d) none of the above
(a) mass-flow rate of steam is regulated
18. In a pressure-compounded impulse steam turbine,
(b) pressure of steam is regulated
pressure drop over each ring of moving blades
(c) additional steam is supplied
(a) remains constant (b) is increasing (d) none of the above
(c) is decreasing (d) none of the above 25. (c)
24. (a) 23. (b) 22. (b) 21. (b) 20. (b) 19. (b) 18. (a) 17. (a)
16. (a) 15. (b) 14. (c) 13. (c) 12. (b) 11. (c) 10. (b) 9. (c)
8. (a) 7. (c) 6. (a) 5. (b) 4. (c) 3. (b) 2. (d) 1. (c)
Answers
750 Thermal Engineering

23
Steam Condensers

Introduction
In thermal power plants, condense are used to condense the exhaust steam from a steam turbine to obtain
maximum efficiency and also to convert the turbine exhaust steam into pure water so that it may be reused
in the steam generator as feed water.
In this chapter, the classification, construction and working of jet and surface condensers are explained
in the first part. Then the condenser and vacuum efficiency with the methods of their augmentation are
discussed. The cooling tower and cooling pond are discussed at the end of the chapter.

CONDENSER 1. The condenser lowers the back pressure at


the turbine exhaust. Thus, steam expands
A condenser is a device in which vapour condenses through a higher pressure ratio across the
to liquid phase at saturation temperature and con- turbine. It results into
stant pressure. During condensation, the working (i) increased work done per cycle,
substance changes its phase from vapour to liquid (ii) improved thermal efficiency of the
and rejects latent heat. A condenser maintains a cycle, and
very low pressure, due to sudden decrease in spe- (iii) reduced steam consumption.
cific volume of the working substance. In a steam
2. The condenser enables the recovery and
condenser, the cooling of steam is accomplished
recirculation of pure feed water into the
by circulating water as a cooling agent in the con-
plant. Thus,
denser. The exhaust pressure in the condenser is
(i) the cost of water softening plant is
maintained nearly 7 to 8 kPa which corresponds to
reduced, and
condensate temperature of nearly 40°C.
(ii) it also saves the cost of fresh water to
be supplied to the boiler.
3. The condenser enables the removal of air
In a steam power plant, the condenser plays a very and non-condensable gases from steam.
important role. The main functions of a steam Thus the heat-transfer rate is improved and
condenser are listed below: tube corrosion is reduced.
Steam Condensers 751

There are mainly two types of condensers:


A condenser unit consists of a condenser, air- (i) Jet condenser
extraction pump, cooling-water circulating pump,
(ii) Surface condenser
feed-water extraction pump and a cooling tower
as shown in Fig. 23.1. The brief function of each
element is given below:
(i) It condenses the exhaust steam In jet condensers, the exhaust steam and cooling
water come in direct contact and mix up together.
from the turbine with the help of circulating water.
Thus, the final temperature of condensate and
(ii) Condensation Extraction Pump It extracts the cooling water leaving the condenser is same. The
cooling water is sprayed on the exhaust steam to
condensate from the bottom of the condenser and
cause rapid condensation.
supplies it to the hot well. The feed water from the A jet condenser is very simple in design and
hot well is further pumped to the boiler. cheaper than a surface condenser. It can be used
when cooling water is cheaply and easily available.
(iii) It removes the air and However, the condensate cannot be reused in the
non-condensable gases from the condenser. boiler, because it contains impurities like dust, oil,
metal particles, etc.
(iv) Circulating Pump It takes in the water from
The jet condensers are also classified as
the sump of cooling tower and circulates it through
the condenser tubes. (a) Low-level jet condenser
(v) The hot water coming out of (i) Counter-flow type
the condenser is sprayed from a certain height. A (ii) Parallel-flow type
small quantity of water evaporates in air and the (b) High-level jet condenser
remaining water is cooled in the cooling tower. (c) Ejector jet condenser
This cooled water is again recirculated into the
condenser.

Elements of a condensing plant


752 Thermal Engineering

23.5.1
Jet Condenser Jet Condenser
The schematic diagram of a low-level counter-flow In this type of condenser, the exhaust steam and
jet condenser is shown in Fig. 23.2. Exhaust steam cooling water both flow in the same direction. The
is supplied from the bottom side of the condenser steam usually enters at the top of the condenser
and it flows upwards while the cooling water is and the cooling water just below it from the side as
supplied from the top of the condenser. shown in Fig. 23.3. Other arrangements are similar
to a counter-flow jet condenser. The mixture of
condensate, coolant and air is extracted with the
help of a wet air pump. This limits the vacuum
created in the condenser up to 600 mm of Hg
(approx 6 kPa).
Exhaust
steam

Cooling Condenser
water shell

Wet air pump


Condensate, air
and coolant

The water flows downward through a series of


baffles or trays. As steam comes in contact with Condenser
falling water, it gets condensed. The air-extraction The schematic of a high-level jet condenser is
pump, located at the top of the condenser sucks shown in Fig. 23.4. This condenser is also called
the air and any uncondensed vapour. The air pump a barometric condenser. The condenser shell is
maintains enough vacuum in the condenser shell, installed at a height greater than that of atmospheric
causing the cooling water to be lifted up to a height pressure in water column, i.e., 10.33 m. A long
of approximately 5.5 m. A pump for water supply tail pipe, more than 10.33 m in height, is attached
is only needed if it is to be lifted more than 5.5 m between the bottom of the condenser and the hot
in height. The condensate extraction pump at the well. The pressure at the bottom of the pipe is equal
bottom of the shell extracts the liquid condensate to the atmospheric pressure, while at its top in the
and cooling water and discharges it to a hot well, condenser shell, the vacuum is maintained. This
from where it may be fed to the boiler, if cooling allows the condensate and coolant to fall from the
water suits it. The excess amount of condensate condenser under gravity without any extraction
from the hot well flows into the cooling pond by an pump.
overflow pipe.
Steam Condensers 753

Air

Air-extraction
pump

Cooling
Baffles
Water

Exhaust steam

Condensate and coolant

More than 10.33 m of


barometric head
Tail pipe
Circulating pump

Over flow

Hot well

Cooling pond

The cooling water is supplied by a water- Disadvantages


circulation pump. The water enters from the top of (i) Due to the high level of the condenser, long
the condenser and the exhaust steam enters at the pipelines are installed. Hence, initial cost is
bottom of the condenser shell. The water stream is high.
broken into fine spray of droplets by baffles. The (ii) It requires more head room with costly
air released from steam and water flows towards the support structure.
top, where it is extracted by an air-extraction pump. (iii) It is not readily accessible for maintenance.

Ejector Condenser
Condenser
In this condenser, the momentum of flowing water
Advantages
is used to remove the mixture of condensate and
(i) It requires less floor space. coolant from the condenser without the use of
(ii) It requires only a circulating pump of low any extraction pump. The schematic of an ejector
capacity. The condensate falls due to the condenser is shown in Fig. 23.5. The exhaust
action of gravity. steam enters the condenser shell at a side through
754 Thermal Engineering

and it can be used as a feed water in the boiler.The


surface condensers are classified in two groups:
(i) Shell-and-tube type, and
(ii) Evaporative type.

Condenser
In this condenser, a large number of tubes are
packed in a shell, with their axes parallel to the
shell. The cooling water flows inside the tubes,
while steam enters the shell side and condenses
over the tubes. The baffles are commonly placed
in the shell to force the steam to flow along path
across the shell and to increase the heat-transfer
rate and to maintain the uniform spacing between
the tubes.
Shell-and-tube type surface condenser can be
further classified according to the number of shell-
and-tube passes involved. A surface condenser in
which water makes one U turn in the water box, is a
a non-return valve, the cooling water enters from one-shell pass and two-tube pass surface condenser
the top of the condenser under a water head of 5 as shown in Fig. 23.6. Similarly, the surface
to 6 m (approx. 1.5 bar) and passes over a series of condenser that involves two passes in the shell and
converging nozzles and attains a high velocity. At four passes of the tube is called a two-shell pass and
the same time, vacuum is created in the side gap of four-tube pass condenser. The surface condensers
the nozzles, drawing in the exhaust steam through require two pumps, i.e., wet air pump to remove air
the truncated divergent cones (nozzles). and condensate and a water pump to circulate the
The mixing of steam and water causes the cooling water under the pressure through the tubes.
condensation and hence the vacuum further These are mainly used in large power plants and
increases in the side of the nozzles. In the central chemical industries.
passage, the water and condensate get momentum, The surface condensers are classified according
which forces the mixture of condensate and water to direction of steam flow:
and air out of the shell. (i) down flow, (ii) central flow, and (iii) inverted
flow.
Exhaust Cooling water
steam Baffles out
In the surface condenser, the exhaust steam and
cooling water do not come in physical contact,
rather they are separated by a heat-transfer wall. Water
box
The exhaust steam passes over the outer surface of
the tubes and the cooling water flows through the Tubes
Shell
tubes. Since the cooling water does not mix with Condensate
Cooling
water in
the condensing steam, the condensate remains pure
Steam Condensers 755

(i) Condenser The sectional


view of a down-flow surface condenser is shown
in Fig. 23.7. The exhaust steam enters the top of
the condenser shell and flows downward over the
water tube. The water tubes are double passed. The
cold water flows in the lower side first and then
in the upper side in reverse direction. It enables
the maximum heat transfer rate for a condenser.
The extraction pump connected at the bottom of
the condenser draws the condensate out of the
condenser.

(iii) The steam


enters the bottom of the shell and air extraction
pump connected at the top. Steam flows upward
first and subsequently, returns to the bottom of
the condenser. The condensate extraction pump is
connected at the bottom of the shell to extract the
condensate.

Condenser
In this type of condenser, the evaporation of some
cooling water provides cooling effect, thereby
steam condenses. An evaporative condenser is
The air pump extracts the air and non-condens- shown in Fig. 23.9.
able gases from the condenser. The separated air The steam to be condensed is passed through
under the baffle is cooled to maximum extent. Due the finned (grilled) tubes. The cooling water is
to cooling of air, its specific volume is reduced.
Therefore, the power consumption for operation of
pump is reduced.
(ii) The sectional
view of a central-flow surface condenser is shown
in Fig. 23.8. The suction pipe of an air extraction
pump is located at the centre of the condenser tubes.
The steam flows radially inward. The condensate is
collected at the bottom of the shell from where it is
taken out by the condensate extraction pump. The
steam gets access to the entire periphery of tubes,
and thus a large surface area for heat transfer is
available as compared to a down flow condenser. Evaporative steam condenser
756 Thermal Engineering

sprayed over the outer surface of the tubes. Cool 3. It requires costly maintenance and skilled
dry air is also blown over the tube surface, which workers.
causes the evaporation of a thin film of water over 4. It requires large floor area.
the tubes. This evaporation cools the water and
extracts latent heat from the steam in the pipe, thus
steam condenses. The water collected in the sump
is mixed with make-up water and again pumped for
Jet condenser Surface condenser
spray. The condensate is extracted with the help of
(mixing type) (non-mixing type)
a wet pump. The air passing over the tubes carries
1. Exhaust steam and Exhaust steam
moisture and it is drawn by an induced draft fan
cooling water mix condenses on outer
located at the top.
together, thus the surface of tubes
The evaporative condensers are most suitable for steam condenses due through which water
small plants, where supply of cold water is limited. to physical contact. flows. Hence, the two
These are most popular with refrigerant condensing fluids do not mix.
units and for chemical equipments.
2. Condensate contains Condensate is pure
impurities, and and can be used
cannot be reused. repeatedly as feed
water to the boiler.
Advantages 3. Heat is exchanged Indirect heat
1. The surface condenser lowers the back by direct contact of exchange, thus a
pressure (7 to 8 kPa) of steam at the turbine cooling water, and large quantity of
exit, and thus allows the expansion of steam thus for condensation cooling water is
through a higher pressure ratio. of steam, less required.
quantity of water is
2. It has high vacuum efficiency, and is thus
required.
suitable for large power plants.
4. Construction of a jet Construction is
3. It gives a pure condensate which can be
condenser is simple, slightly complicated,
recirculated as feed water to the boiler. thus the condenser is thus initial higher
4. Since the condensate is reused, it saves the less costly. installation cost.
cost of fresh water to be circulated and the
5. Maintenance is Costly maintenance,
cost of its chemical treatment. simple and cheap. requires skilled
5. It requires low power input for air-extraction worker.
pump. 6. It requires small It requires large floor
6. Since the cooling water is in indirect contact floor space. space.
of steam, low quality cooling water can be
7. More power is Less power is needed
used in the condenser. required for air for air pump.
7. The cooling of condensate can be controlled pump.
by regulating the flow of cooling water.
8. Less power is needed More power is
Disadvantages for water pumping. needed for water
1. Indirect cooling takes place in the condenser, pumping
and thus large cooling water is required. 9. It has low vacuum It has high vacuum
2. Construction is complicated, requires higher efficiency, thus is efficiency, thus is
less suitable for large more suitable for
installation cost.
plants. large plants.
Steam Condensers 757

The rate of heat gain by cooling water


Qw = mass flow rate ¥ specific heat ¥
temp. rise of water
The steam condenser is used to condense several = mw ¥ Cpw (Tc,o – Tc,i ) ...(23.2)
thousand kilograms of steam per hour in a The energy balance on a condenser leads to
power plant. The steam releases its latent heat of
Qw = Qsteam
condensation and sometimes a portion of sensible
heat also. Therefore, a huge amount of cooling water or mw ¥ Cpw (Tc,o – Tc,i )
is required for steam condensation. Approximately, = msteam [hf + x hfg – hfc ]
30 to 50 kg of cooling water is required per kg of msteam ( h f + x h fg – h fc )
steam to be condensed. or mw = ...(23.3)
C pw (Tsat – Tc )
Figure 23.10 shows a schematic of a steam
condenser with steam and cooling water flowing where Cpw is the specific heat of water =
through it. 4.187 kJ/ kg ◊ K and mw and msteam are the mass-
flow rate of cooling water and exhaust steam
respectively into the condenser.

The condenser is a heat exchanger and its efficiency


can be defined as the ratio of actual temperature
rise to the maximum possible temperature rise of
cooling water.
Actual temperature rise of
cooling water
hcondenser =
Maximum possible tem mperature
Usually, the condensate loses some of its sensible rise of cooling water
heat, and thus the temperature Tc of the condensate
( DT )c
leaving the condenser is less than the saturation or hcondenser = ...(23.4)
temperature of the condensate. The temperature ( DT ) max
difference (Tsat – Tc ) is known as undercooling or For a condenser, the actual temperature rise of
subcooling of the condensate. water is
The rate of heat loss by exhaust steam (DT )c =Exit temperature of water – Inlet
Qsteam = sum of latent heat and subcooling heat temperature of water
of the condensate = Tc, o – Tc,i ...(23.5)
= msteam [xhfg + Cpw (Tsat – Tc )] and the maximum possible temperature rise of
= msteam [Cpw Tsat + xhfg – Cpw Tc ] cooling water is
Using Cpw Tsat = hf (sensible heat of exhaust DTmax = Sat. temp. corresponding to absolute
steam), and pressure in condenser – Water inlet
Cpw Tc = hfc temperature
= sensible heat of condensate leaving = Tsat – Tc,i
the condenser in a hot well Tc ,o - Tc ,i
Then Qsteam = msteam [hf + x hfg – hfc ] ...(23.1) Then hcondenser = ...(23.6)
Tsat - Tc ,i
758 Thermal Engineering

Example 23.1 A surface condenser is designed to Inlet temperature of cooling water = 15°C
handle 12000 kg of steam per hour. The steam enters at Outlet temperature of cooling water = 30°C
8 kPa, 0.9 dry. The condensate leaves the condenser at Mass of cooling water per kg of steam = 32 kg
the corresponding saturation temperature. Calculate the Assuming that all the heat lost by the exhaust steam is
rate of cooling water, if cooling water temperature rise is taken up by the circulating water, determine the dryness
limited to 12°C. fraction of the steam as it enters the condenser.

Given Data of a surface condenser Given Data from a trial on a surface condenser
msteam = 12000 kg/h psat = 8 kPa psat = 5.628 kPa Tc = 32°C
x = 0.9 DTc = 12°C Tc, i = 15°C Tc, o = 30°C
To find The mass-flow rate of cooling water in the mw = 32 kg/kg of steam
condenser. msteam = 1 kg

Assumptions To find Dryness fraction of the steam entering the


condenser.
(i) No air present in the condenser.
(ii) No heat loss from the condenser body. Assumptions
(iii) The specific heat of cooling water as 4.187 (i) No air present in the condenser.
kJ/kg ◊ K. (ii) No heat loss from the condenser body.
Analysis The properties of steam at 8 kPa (from steam (iii) The specific heat of cooling water as 4.187
tables A-13): kJ/kg ◊ K.
Tsat = 41.5°C, Analysis The properties of steam from steam tables:
hfg = 2403.1 kJ/kg At 5.628 kPa hf = 146.66 kJ/kg,
The energy balance for the condenser hfg = 2418.62 kJ/kg
Heat given by exhaust steam At 32°C hfc = 134.2 kJ/kg
= Heat gain by cooling water ...(i) The energy balance on the condenser
Heat given by exhaust steam Heat given by exhaust steam
= msteam ¥ Latent heat of condensation = Heat gain by cooling water ...(i)
= msteam [x hfg ] ...(ii) Heat given by exhaust steam
Heat gain by cooling water = msteam [hf + xhfg – hfc] ...(ii)
= mw Cpw DTc ...(iii) Heat gain by cooling water = mw Cpw (Tc, o – Tc, i )
Using Eqs. (ii) and (iii) in Eq. (i), with numerical Using Eq. (ii) and (iii) in Eq. (i), with numerical
values, values,
12000 ¥ 0.9 ¥ 2403.1 = mw ¥ 4.187 ¥ 12 1 ¥ [146.66 + 2418.62 x – 134.2]
= 32 ¥ 4.187 ¥ (30 – 15)
Mass of cooling water;
or 2418.62 x = 2009.76 – 12.46 = 1997.3
12000 ¥ 0.9 ¥ 2403.1
mw = Dryness fraction of steam; x = 0.826
4.187 ¥ 12
= 5,16,548.84 kg/h Example 23.3 The steam is supplied to a steam
turbine at 3.0 MPa and 300°C. The expansion of steam
Example 23.2 The following data refers to a test of
is carried out isentropically to a condenser vacuum of
the surface condenser of a steam turbine.
713 mm of Hg. The barometer reads 758 mm of Hg. The
Absolute pressure of the steam entering condenser temperature is 20°C and rise in temperature
the condenser = 5.628 kPa of cooling water is 12°C. Determine
Temperature of condensate leaving the (a) quality of steam entering the condenser, and
condenser = 32°C
Steam Condensers 759

(b) quantity of cooling water circulated per kg of


steam.
Assume no air is present in the condenser.

Given Data from steam turbine and condenser


p1 = 30.0 MPa T1 = 300°C
pg = 713 mm of Hg patm = 758 mm of Hg
Tc = 20°C DTw = 12°C
msteam = 1 kg No air in condenser
To find
(i) Dryness fraction of the steam entering the
condenser. Example 23.4 The following data were recorded
(ii) Quantity of cooling water circulated per kg of from a test of a surface condenser:
steam. Inlet temperature of circulating water = 21°C
Exit temperature of circulating water = 35°C
Assumptions
Vacuum i n t he c ondenser = 704.7 mm of Hg
(i) The specific heat of cooling water as 4.187
Barometer reading = 760 mm of Hg
kJ/kg ◊ K.
Calculate the efficiency of the condenser.
(ii) No heat loss from the condenser body.
Analysis The absolute pressure in the condenser,
pabs = patm – pg = 758 – 713 = 45 mm of Hg Given Data from a test of a surface condenser
101.325 Tc, i = 21°C Tc, o = 35°C
= 45 mm of Hg ¥ ª 6.0 kPa
760 mm of Hg pg = 704.7 mm of Hg patm = 760 mm of Hg
The quality of steam entering the condenser can be
To find Condenser efficiency.
found by using Mollier diagram for isentropic expansion.
Choose coordinates 30.0 MPa and 300°C on the Assumptions
Mollier diagram and draw a vertical line till it intersects (i) Density of mercury, r = 13600 kg/m3.
the 6 kPa line. Read the dryness fraction at the intersection (ii) Acceleration due to gravity, g = 9.81 m/s2.
point. It is found to be
Analysis The absolute pressure in the condenser
x = 0.77
pabs = patm – pg = 760 – 704.7 = 55.3 mm of Hg
The properties of steam from steam tables:
This absolute pressure can be expressed in kPa as
At 6.0 kPa hf = 151.5 kJ/kg,
hfg = 2416.0 kJ/kg 101.325 kPa
pabs = 55.3 mm of Hg ¥
At 20°C hfc = 83.9 kJ/kg 760 mm of Hg
Heat given by exhaust steam = 7.38 kPa
= msteam [hf + x hfg – hfc] ...(i) From steam tables; the saturation temperature at
Heat gain by cooling water = mw Cpw DTw ...(ii) 7.38 kPa
Equating Eq. (i) and (ii) and using numerical values Tsat = 40°C
1 ¥ [151.5 + 0.77 ¥ 2416 – 83.9] Temperature rise of cooling water,
= mw ¥ 4.187 ¥ 12 (DT )c = Tc, o – Tc, i = 35 – 21 = 14°C
or 1927.92 = 50.244 mw Maximum possible temperature rise of water,
1927.92 (DT )max = Tsat – Tc, i = 40 – 21 = 19°C
Mass of cooling water; mw = ( DT )c 14
50.244 Thus hcondenser = = = 0.736 or 73.6%
= 38.37 kg/kg of steam ( DT ) max 19
760 Thermal Engineering

At any barometric reading other than standard


reading of 760 mm of Hg, the vacuum gauge
reading is corrected to standard barometric reading
as
When the steam condenses in a closed vessel, the Corrected vacuum in mm of Hg
vapour phase of working substance (water) changes = Standard barometer reading
to liquid phase, and thus its specific volume reduces – (Actual barometer reading
to more than one thousand times. Due to change – Vacuum gauge reading) ...(23.8)
in specific volume, the absolute pressure in the
condenser falls below the atmospheric pressure and The abosolute pressure in the condenser can be
a high vacuum is created. determined as
This lower pressure in the condenser permits pabs = 760 mm – corrected vacuum in mm
maximum expansion of steam in the turbine, and of Hg ...(23.9)
more work is developed. The minimum pressure
that can be attained depends on the temperature of
condensate and air present in the condenser. For a mixture of non-reactive gases, the total
The term vacuum means the pressure below the (mixture) is given by
atmospheric pressure. The vacuum in the condenser
m A RAT mB RBT
is generally measured in mm of mercury (Hg). p = +
The vacuum is the negative gauge pressure and V V
is equal to the differences of barometric pressure = pA + pB + … + S pi ...(23.10)
and absolute pressure in the condenser as shown in where pi is the pressure of ith gas component at the
Fig. 23.12. mixture temperature T and volume V.
Vacuum gauge According to Dalton’s law, the total pressure of
a non-reactive mixture is equal to the sum of partial
pressure of its components, if each component
Condenser
would occupy the same volume alone at the mixture
Barometric

temperature.
pressure

Vacuum (gauge)
The condenser has a mixture of wet steam and
Absolute pressure
air. The total pressure p in the condenser is the sum
of partial pressure of steam, psat and air, pa. The
saturation pressure of steam psat can be obtained
from the steam table at condensate (saturation)
temperature. According to Dalton’s law,
p = psat + pa
The absolute pressure in the condenser or pa = p – psat ...(23.11)
= Atmospheric pressure – Vacuum With the help of partial pressure of air, the mass
gauge pressure ...(23.7) of air can be determined as
= Barometer reading – Vacuum gauge
paV
reading ma = ...(23.12)
The vacuum is usually referred to a standard RaTsat
atmospheric pressure as 101.325 kPa or 760 mm where V is the volume of the condenser, Tsat is the
of Hg, i.e., condensate temperature and Ra is the characteristic
760 gas constant of air, which is equal to 0.287 kJ/kg ◊ K.
1 kPa = mm of Hg
101.325
Steam Condensers 761

(iii) In case of a jet condenser, some air comes


In the steam condenser, a mixture of steam and in with the injected water in which it is
air is present. Therefore, the absolute pressure in dissolved.
the condenser is the sum of partial pressures of
steam and air. The presence of air in the condenser
Condenser
disturbs the vacuum, and thus the actual vacuum is
less than that could be attained in the condenser, The presence of air in the condenser is a far serious
if only steam is present. If no air is present in the concern. It affects the performance of the condenser
condenser then absolute pressure in the condenser to a great extent, thereby, the performance of steam
is the saturation pressure of steam and thus the power plant. The presence of air into the condenser
maximum vacuum would be attained. puts the following effects:
The vacuum efficiency is defined as ratio of the (i) The presence of air lowers the vacuum in
actual vacuum to the maximum possible vacuum. the condenser. Thus the back pressure of the
Actual vacuum in the condenser plant increases, and consequently, the work
hvacuum = output of the turbine reduces.
Maximum possible vacuum (gaugge )
(ii) The presence of air also lowers the partial
pg
= ...(23.13) pressure of steam and hence lower saturation
pg,max temperature. The steam with lower saturation
where pg is the actual vacuum temperature has higher latent heat. A large
= Barometric pressure quantity of cooling water is required to get
– Absolute pressure in condenser the desired result in the condenser.
= patm – p (iii) Air forms the film adjacent to the tube
Maximum possible vacuum surface in the condenser. Air has very poor
pg, max = Barometric pressure – Sat. pressure thermal conductivity. Hence, the rate of heat
of steam corresponding to conden- transfer from vapour to cooling medium is
reduced.
sate temperature
= patm – psat (iv) The presence of air in the condenser corrodes
to the metal surfaces. Therefore, the life of
Condenser condenser is reduced.

All types of condensers require air extraction pump A vacuum of 712 mm was obtained
to remove the air present in the condenser. The with the barometer reading of 753 mm of Hg. Correct
main sources of air present in the condenser are the the vacuum to a standard barometer reading of
following: 760 mm of Hg.

(i) The ambient air leaks to the condenser


chamber at the joints and glands which are
Given Vacuum reading = 712 mm of Hg
internally under pressure lower than that of
Actual barometer reading = 753 mm of Hg
ambient. It can be reduced by taking utmost
Standard barometer reading = 760 mm of Hg
care while designing and making vacuum
joints. To find Corrected vacuum in the condenser.
(ii) Another source of air is the dissolved air Analysis The corrected vacuum in the condenser is
with feed water. The dissolved air in feed given by
water enters into the boiler and it travels = Standard barometer reading
with steam into the condenser. Its quantity – (Actual barometer reading – Actual vacuum reading)
depends upon the quality of feed water. = 760 mm – (753 mm – 712 mm) = 719 mm of Hg
762 Thermal Engineering

Example 23.6 Steam enters a condenser at 35°C. The the quantity of water required per minute and the
barometer reading is 760 mm of mercury. If the vacuum minimum height of tail pipe required above the level of
of 690 mm is recorded, calculate the vacuum efficiency. condensate in the hot well.

Given Vacuum in a steam condenser Given Data from a barometric jet condenser
Tsat = 35°C msteam = 3000 kg/h
Barometer reading = 760 mm of Hg p = 100 mm of Hg
Vacuum reading, pg = 690 mm of Hg x = 0.92
Tc, i = 15°C Tc, o = 40°C
To find Vacuum efficiency
To find
Analysis The saturation pressure of steam at 35°C;
(i) Quantity of cooling water circulated per minute.
(from steam table A)
(ii) Minimum height of tail pipe above condensate
psat = 5.628 kPa
760 mm of Hg level in hot well.
= (5.628 kPa) ¥
101.325 kPa Assumptions
= 42.21 mm of Hg
(i) Standard atmospheric pressure of 760 mm of Hg.
Max. possible vacuum
(ii) No heat loss from the condenser body.
= Barometer reading – Sat. pressure
(iii) The specific heat of cooling water as 4.187
of condensing steam
kJ/kg ◊ K.
pg, max = 760 – 42.21 = 717.79 mm of Hg
(iv) The density of water as 1000 kg/m3.
The vacuum efficiency
Actual vacuum in the condenser Analysis
hvacuum =
Maximum possible vacuum (gaugge ) (i) Quantity of cooling water circulated per min
690 mm of Hg The absolute pressure in the condenser in kPa;
= = 0.961 or 96.1%
717.79 mm of Hg 101.325
p1 = 100 mm of Hg ¥
760 mm of Hg
Example 23.7 A barometric jet condenser deals with = 13.33 kPa
3000 kg of steam per hour. The steam is 0.92 dry. The The properties of steam from steam tables:
absolute pressure in the condenser is 100 cm of mercury. At 13.33 kPa hf = 215.5 kJ/kg,
Cooling water enters at 15°C and mixed condensate and hfg = 2373.65 kJ/kg
cooling water leaves the condenser at 40°C. Calculate At 40°C hfc = 165.54 kJ/kg
Heat given by exhaust steam,
Qw = msteam [hf + xhfg – hfc] ...(i)
Heat gain by cooling water;
Qw = mw Cpw (Tc, o – Tc, i ) ...(ii)
Equating Eq. (i) and (ii) and using numerical
values
Ê 3000 ˆ
ÁË 60 ˜¯ ¥ [215.5 + 0.92 ¥ 2373.65 – 167.54]

= mw ¥ 4.187 ¥ (40 – 15)


or 111585.9 = 104.675 mw
Mass of cooling water
111585.9
mw = = 1066 kg/min
104.675
Steam Condensers 763

(ii) Height of tail pipe above condensate level in hot Analysis


well (i) The vacuum in the condenser
The atmospheric pressure acts at the condensate The mass of water in the tail pipe acts downward,
level in the hot well. It can be expressed as thus creating vacuum in the condenser. The gauge
patm = abs. pressure in condenser + gauge pressure (vacuum) pressure in fluid column is expressed as
offered by water column in tail pipe pg = r gh
or patm = p1 + pg = p1 + rwater ghwater Therefore, equating the gauge (vacuum) pressure
or 101.325 ¥ 103 in water and mercury columns;
= 13.33 ¥ 103 + 1000 ¥ 9.81 ¥ hwater rwater ghwater = rHg g hHg
Height of tail pipe; Using the numerical values;
101.325 ¥ 103 - 13.33 ¥ 103 1000 ¥ 9.81 ¥ (8.5 m) = 13590 ¥ 9.81hHg
hwater = 1000 ¥ 9.81 ¥ 8.5
1000 ¥ 9.81 or hHg =
13590 ¥ 9.81
= 8.97 m
= 0.625 m or 625 mm of Hg
Example 23.8 3000 kg of wet steam with a dryness (ii) Absolute pressure in the condenser
fraction of 0.95 is condensed per hour in a barometric The absolute pressure in the condenser
condenser. The minimum height of the tail race above the pabs = patm – pvacuum
hot well is 8.5 m. The barometric pressure is 760 mm = 760 mm – 625 mm
of Hg. The cooling water enters the condenser at 25°C = 135 mm of Hg
and the mixture of condensate and cooling water exit This pressure can be obtained in kPa as
temperature is 50°C. Calculate 101.325 kPa
pabs = 135 mm of Hg ¥
(a) Vacuum in the condenser in mm of Hg, 760 mm of Hg
(b) Absolute pressure in the condenser in kPa, ª 18 kPa
(c) Mass of cooling water required without under- (iii) Mass of cooling water required without under-
cooling. cooling
The properties of steam from steam tables:
At 50°C hf = 209.31 kJ/kg,
Given A barometric jet condenser hfg = 2382.75 kJ/kg
msteam = 3000 kg/h The energy balance on the condenser
x = 0.95 Heat given by exhaust steam without undercool-
Hwater = 8.5 m ing,
Barometer reading = 760 mm of Hg Qsteam = msteam [hf + x hfg ]
Tc, i = 25°C Heat gain by cooling water,
Tc, o = 50°C Qw = mw Cpw (Tc, o – Tc, i )
Making energy balance using numerical values
To find
3000 ¥ [209.31 + 0.95 ¥ 2382.75]
(i) Vacuum in the condenser in mm of Hg.
= mw ¥ 4.187 ¥ (50 – 25)
(ii) Absolute pressure in the condenser in kPa.
or 7418767.5 = 104.67 mw
(iii) Mass of cooling water required without
Mass of water needed;
undercooling.
7418767.5
mw = = 70877.68 kg/h
Assumptions 104.67
(i) No heat loss from the condenser body.
Example 23.9 The temperature in a surface condens-
(ii) The specific heat of cooling water as 4.187
er is 37.31°C and the vacuum is 698 mm of Hg and the
kJ/kg ◊ K.
barometer reads 755.2 mm of Hg. Correct the vacuum
(iii) The density of water as 1000 kg/m3.
reading to a standard barometer of 760 mm and hence,
(iv) The density of mercury as 13590 kg/m3. determine (i) the partial pressure of steam and air, and
764 Thermal Engineering

(ii) the mass of air associated with one kg of steam. Take Example 23.10 A condenser of 0.75 m3 capacity
R = 0.287 kJ/kg ◊ K for air. contains saturated steam and air at a temperature of
45°C and a pressure of 0.13 bar abs. Air also leaks
further into the condenser, increases the pressure to
Given A surface condenser 0.28 bar abs. and temperature falls to 38°C. Calculate
Tsat = 37.31°C the mass of air which has leaked in. Take R for air as
Vacuum, pg = 698 mm of Hg 0.287 kJ/kg ◊ K.
Ra = 0.287 kJ/kg ◊ K
Barometer pressure, patm = 755.2 mm of Hg
Standard barometer reading = 760 mm of Hg Given Steam and air mixture in a condenser
V = 0.75 m3 Tsat1 = 45°C
To find
p1 = 0.13 bar = 13 kPa
(i) Correct vacuum reading to standard atmospheric
After air leaks into the condenser
pressure,
p2 = 0.28 bar Tsat2 = 38°C
(ii) Partial pressure of steam and air, and
R = 0.287 kJ/kg ◊ K
(iii) Mass of air associated with 1 kg of steam.
To find Mass of air leaks the into the condenser.
Analysis
(i) Corrected vacuum reading Analysis The saturation pressure of steam at 45°C
= Standard barometer reading – (from steam tables A-12)
(Actual barometer reading – Vacuum reading) psat1 = 9.59 kPa
= 760 – (755.2 – 698) The initial partial pressure of air in the condenser
= 702.8 mm of Hg pa1 = p1 – psat1 = 13 kPa – 9.59 kPa
(ii) Partial pressure of steam and air = 3.41 kPa
The saturation pressure of steam at 37.31°C Mass of air present intially in the condenser
(from steam tables) Volume of air = Volume of steam
psat = 6.439 kPa Va = 7.5 m3
vg = 22.585 m3/kg of steam pa1Va
The partial pressure of steam is psat = 6.439 kPa Mass of air; ma1 =
RaTsat1
The absolute pressure of steam in the condenser
p = patm – pg = 755.2 – 698 (3.41 kPa) ¥ (7.5 m3 )
=
= 57.2 mm of Hg (0.287 kJ/kg ◊ K) ¥ (45 + 273)(K)
101.325 kPa = 0.28 kg
= 57.2 mm of Hg ¥
717.79 mm of Hg After air leaks into the condenser, the saturation
= 7.63 kPa pressure of steam at 38°C,
Using Dalton’s law of partial pressure, the psat2 = 6.62 kPa
pressure of air in the condenser The final partial pressure of air in the condenser,
pa = p – psat = 7.63 kPa – 6.439 kPa pa2 = p2 – psat2 = 28 kPa – 6.62 kPa
= 1.187 kPa = 21.38 kPa
(iii) Mass of air associated with 1 kg of steam Mass of air present finally in the condenser,
Specific volume of air = Specific volume of steam pa2Va (21.38 kPa) ¥ (7.5 m3 )
va = 22.585 m3/kg of steam ma2 = =
RaTsat2 (0.287 kJ/kg ◊ K) ¥ (38 + 273)(K)
p v
ma = a a = 1.796 kg
RaTsat
The air leaking into the condenser,
(1.187 kPa) ¥ (22.585 m3/kg) = ma2 – ma1 = 1.796 kg – 0.28 kg = 1.516 kg
=
(0.287 kJ/kg ◊ K) ¥ (37.31 + 273)(K)
= 0.301 kg/kg of steam
Steam Condensers 765

Example 23.11 A vacuum of 710 mm of Hg was Ideal vacuum


recorded in a condenser when the barometer reads pg, max = Standard barometer reading
755 mm of Hg. The temperature of the condensate – Steam pressure
was 25°C. Calculate the pressure of steam and air in 760 mm of Hg
= 760 – 3.17 kPa ¥
the condenser and mass of air per kg of steam. Also, 101.325 kPa
calculate the vacuum efficiency. = 736.24 mm of Hg
pg 710 mm of Hg
\ hvacuum = =
pg,max 736.24 mm of Hg
Given A steam condenser = 0.964 or 96.4%
Vacuum, pg = 710 mm of Hg
patm = 755 mm of Hg Example 23.12 The absolute pressure in the con-
Tsat = 25°C denser is 11.56 kPa when the barometer reads 1 bar. The
condenser temperature is 40°C. Calculate the partial
To find
pressure of air, vacuum efficiency and mass of air present
(i) Partial pressure of steam and air, in the condenser per kg of steam.
(ii) Mass of air associated with 1 kg of steam,
(iii) Vacuum efficiency.
Assumption R for air as 0.287 kJ/kg ◊ K. Given Absolute pressure in the condenser,
p = 11.56 kPa
Analysis
Barometric pressure, patm = 1 bar = 100 kPa
(i) The absolute pressure in the condenser
The condenser temperature, Tsat = 40°C
p = patm – pg = 755 – 710 = 45 mm of Hg
101.325 kPa To find
= 45 mm of Hg ¥ (i) Partial pressure of air,
760 mm of Hg
ª 6 kPa (ii) Vacuum efficiency, and
(iii) Mass of air present in the condenser per kg of
The saturation pressure of steam at 25°C (from steam.
steam tables)
Assumption The gas constant for air as 0.287 kg/kg ◊ K.
psat = 3.17 kPa
vg = 43.36 m3/kg Analysis
The partial pressure of steam in the condenser: (i) Partial pressure of air in the condenser
psat = 3.17 kPa The saturation pressure of steam at condensate
The partial pressure of air in the condenser temperature of 40°C
pa = p – psat = 6 kPa – 3.17 kPa psat = 7.384 kPa
= 2.83 kPa The partial pressure of air,
(ii) Mass of air present in the condenser pa = p – psat = 11.56 kPa – 7.384 kPa
Sp. Volume of air = 4.176 kPa
= Specific volume of steam (ii) Vacuum efficiency
= 43.36 m3/kg The vacuum pressure in the condenser,
pa va (2.83 kPa) ¥ (43.36 m3/kg) pg = patm – p = 100 kPa – 11.56 bar = 88.44 kPa
\ ma = =
RaTsat (0.287 kJ/kg ◊ K) ¥ (25 + 273)(K) Maximum possible vacuum in the condenser
= 0.0143 kg/kg of steam pg, max = p – psat = 100 – 7.384
(iii) Vacuum efficiency = 92.616 kPa
Actual vacuum, pg 88.44 kPa
hvacuum = =
pg = 710 mm of Hg pg,max 92.616 kPa
= 0.9549 or 95.49%
766 Thermal Engineering

(iii) Mass of air present in the condenser per kg of – (Actual barometer reading
steam – Actual vacuum reading)
Volume of air/kg of steam = 760 mm – (766 mm – 716 mm)
= Specific volume of steam at 40°C = 710 mm of Hg
or va = 19.5219 m3/kg of steam (from (ii) The saturation pressure of steam corresponds to
steam tables) temperature, Tsat = 35°C
The mass of air psat = 5.628 kPa
p v (4.176 kPa) ¥ (19.5219 m3/kg) Ê 760 mm of Hg ˆ
ma = a a = = 5.628 ¥ Á
RaTsat (0.287 kJ/kg ◊ K) ¥ (40 + 273)(K) Ë 101.325 kPa ˜¯
= 0.9705 kg/ kg of steam = 42.21 mm of Hg
Maximum possible vacuum;
Example 23.13 During a trial on a condenser, the
pg,max = Actual barometer reading
following readings were recorded:
– saturation pressure
Barometer reading = 766 mm of Hg
= 766 mm – 42.21 mm
Actual vacuum recorded by gauge = 716 mm of Hg
= 723.79 mm of Hg
Temperature of exhaust steam = 35°C
The vacuum efficiency;
Temperature of hot well = 29°C pg
Actual vacuum
Inlet temperature of cooling water = 15°C hvacuum = =
Max. possible vacuum pg,max
Outlet temperature of cooling water = 24°C
716 mm
Calculate = = 0.9892 = 98.92%
723.79 mm
(a) Corrected vacuum to standard barometer reading
of 760 mm of mercury, (iii) Undercooling of condensate
(b) Vacuum efficiency, = Condensate temperature
(c) Undercooling of condensate, and – hot well temperature
(d) Condenser efficiency. = Exhaust steam temp. – temperature
of condensate coming out the condenser
= 35°C – 29°C = 6°C
(iv) Condenser efficiency;
Given Data from the trial on a steam condenser:
Actual temperature rise of
Actual barometer reading = 766 mm of Hg
cooling water
Standard barometer reading = 760 mm of Hg hcondenser =
Maximum possible tem mperature
Actual vacuum in the condenser, pg = 716 mm of Hg
rise of cooling water
Exhaust steam temperature, Tsat = 35°C
Tc,o - Tc,i 24 - 15
Cooling water Tc, i = 15°C = =
Tc, o = 24°C Tsat - Tc,i 35 - 15
Hot well temperature, Tc = 29°C = 0.45 or 45%

To find Example 23.14 During trial on a steam condenser,


(i) Corrected vacuum, the following observations were recorded.
(ii) Vacuum efficiency, Condenser vacuum = 680 mm of Hg
(iii) Undercooling of condensate, and Barometer reading = 764 mm of Hg
(iv) Condenser efficiency. Mean condenser temperature = 36.2°C
Hot well temperature = 30°C
Assumption No air present in the condenser.
Condensate formed per hour = 1780 kg
Analysis Inlet temperature of cooling water = 20°C
(i) The corrected vacuum Outlet temperature of cooling water = 32°C
= Standard barometer reading Quantity of cooling water circulated = 1250 kg/min
Steam Condensers 767

Determine The maximum possible vacuum in the condenser


(a) Condenser vacuum corrected to standard barom- pg, max = Actual barometer reading
eter, – sat. pressure of condensate
(b) Vacuum efficiency, = 764 – 45 = 719 mm of Hg
(c) Condenser efficiency, and Then
(d) Condition of steam entering the condenser. Actual vacuum in the condenser
hvacuum =
Assume R for air = 0.287 kJ/kg ◊ K, Maximum possible vacuum
Specific heat of water = 4.186 kJ/kg ◊ K 676 mm
= = 0.9402 or 94.02%
719 mm
(iii) Condenser efficiency
Given Data from a trial on a surface condenser: Actual temperature rise of water,
Barometer reading, patm = 764 mm of Hg (DT )c = Tc, o – Tc, i = 32 – 20 = 12°C
The absolute pressure in the condenser
Condenser vacuum, pg = 680 mm of Hg
Condensate mean temp. Tsat = 36.2°C p = barometer reading – Vacuum gauge reading
Hot well temperature, Tc = 30°C = 764 – 680 = 84 mm of Hg
Cooling water Tc, i = 20°C 101.325 kPa
= 84 ¥ = 11.12 kPa
Tc, o = 32°C 760 mm
Mass of steam condensed, msteam = 1780 kg/h Saturation temperature of steam corresponding to
psat = 11.12 kPa (from steam tables)
Mass of water circulated, mw = 1250 kg/min
Tsat = 48°C
R = 0.287 kJ/kg ◊ K Maximum possible temperature rise of cooling
Cpw = 4.186 kJ/kg ◊ K water
To find (DT )max = Tsat – Tc, i = 48 – 20 = 28°C
(i) Condenser vacuum corrected to standard barom- ( DT )c 12
Then hcondenser = =
eter reading, ( DT ) max 28
(ii) Vacuum efficiency, hvacuum = 0.4286 or 42.86%
(iii) Condenser efficiency, hcondenser (iv) The quality of exhaust steam, x
(iv) Quality of exhaust steam, x. From steam tables,
At 11.12 kPa, hf = 200.1 kJ/kg and
Analysis
hfg = 2387.2 kJ/kg
(i) Condenser vacuum corrected to standard barom- At 30°C hfc = 125.75 kJ/kg
eter reading Heat energy given by steam
= Standard barometer reading = Heat gain by circulating water
– (Actual barometer reading Heat energy given by steam,
– Vacuum reading) Qsteam = msteam [hf + x hfg – hfc )]
= 760 – (764 – 680) Heat gain by cooling water,
= 676 mm of Hg Qw = mw Cpw (DT )c
(ii) The vacuum efficiency Using numerical values and equating two
The saturation pressure of steam corresponding quantities of heat energy;
to condensate temperature of 36.2°C 1760
¥ [200.1 + x ¥ 2387.2 – 125.75]
760 mm 60
psat = 6 kPa = 6 ¥
101
P .325 k a = 1250 ¥ 4.186 ¥ (32 – 20)
= 45 mm of Hg or 2387.2 x = 2140.57 – 74.35 = 2066.22
2066.22
or x = = 0.865
2387.2
768 Thermal Engineering

Example 23.15 The following observation were psat = 5.628 kPa


made during a test on a surface condenser: 760 mm
= 5.628 kPa ¥ = 42.21 mm of Hg
Barometer reading = 760 mm of Hg 101
P .325 k a
Condenser vacuum = 705 mm of Hg The maximum possible vacuum in the condenser,
Mean temperature of the condensate = 35°C pg, max = barometer reading – saturation pressure
Condensate collected = 2000 kg/h of condensate
Quantity of cooling water circulated = 60,000 kg/h = 760 – 42.21 = 717.79 mm of Hg
Rise in temperature of cooling water = 16°C (i) The vacuum efficiency
Hot well temperature = 28°C Actual vacuum in the condenser
hvacuum =
Inlet temperature of cooling water = 20°C Maximum possible vacuum
Calculate 705 mm
= = 0.982 or 98.2%
(a) Vacuum efficiency, 717.79 mm
(b) Condenser efficiency, (ii) Condenser efficiency
(c) Quality of steam entering the condenser, Actual temperature rise of water, (DT )c = 16°C
(d) Mass of air present per m3 of the condenser Saturation temperature of steam corresponds to
volume. psat = 7.332 bar (from steam tables)
Tsat = 40°C
Maximum possible temperature rise of cooling
Given Data from the test on a surface condenser: water
Barometer reading patm = 760 mm of Hg (DT )max = Tsat – Tc, i = 40 – 20 = 20°C
Condenser pressure, pg = 705 mm of Hg ( DT )c 16
Then hcondenser = = = 0.8
Condensate mean temperature Tsat = 35°C ( DT ) max 20
Mass of steam condensed, msteam = 2000 kg/h = 80%
Mass of water circulated, mw = 60,000 kg/h (iii) The quality of exhaust steam, x
(DT )c = 16° Heat given by steam
Hot well temperature, Tc = 28°C = Heat gain by circulating water
Tc, i = 20°C Heat given by steam
To find Qsteam = msteam [ hf + x hfg – hfc ]
Heat gain by cooling water,
(i) Vacuum efficiency, hvacuum
Qw = mw Cpw (DT )c
(ii) Condenser efficiency, hcondenser
From steam tables:
(iii) Quality of exhaust steam, x
At 35°C, hf = 146.66 kJ/kg and
(iv) Mass of air present in condenser, ma per m3
condenser volume. hfg = 2418.62 kJ/kg
At 28°C hfc = 117.4 kJ/kg
Assumptions Using the numerical values
(i) The specific heat of water as 4.187 kJ/kg ◊ K. 2000 ¥ [146.66 + 2418.62 x – 117.4]
(ii) The specific gas constant for air as 0.287 kJ/kg ◊ K. = 60,000 ¥ 4.187 ¥ 16
Analysis The absolute pressure in the condenser or 2418.62 x = 2009.76 – 29.26 = 1980.5
p = barometer reading – Vacuum gauge reading 1980.5
or x = = 0.8188
= 760 – 705 = 55 mm of Hg 2418.62
101.325 kPa (iv) Mass of air present in the condenser per m3 of
= 55 ¥ = 7.332 kPa condenser volume
760 mm
Saturation pressure of steam, corresponds to The partial pressure of air
condensate temperature of 35°C pa = p – psat
Steam Condensers 769

= 7.332 kPa – 5.628 kPa


= 1.705 kPa
p V
ma = a a
RTsat
(1.705 kPa) ¥ (1 m3 )
=
(0.287 kJ/kg ◊ K) ¥ (35 + 273)(K)
= 0.0193kg

The presence of air increases the back pressure in


the condenser. Therefore, the air should be removed
continuously out of the condenser. For removal
of air from the condenser, there are two types of
systems used: capable to maintain a relatively low vacuum, thus it
(i) Dry-air extraction system, and is only suitable for small power plants.
(ii) Wet-air extraction system. It consists of solid bucket-type plunger, which
is flat at the top and conical at the bottom surface.
(i) In this system, The plunger reciprocates in the pump barrel. The
the dry pump removes only moist air from the delivery valves are mounted in the cover of the pump
condenser. A separate pump is used for extraction barrel. These valves are also called head valves.
of condensate from the bottom of the condenser. The plunger has several ports along its periphery.
The air pump is usually connected at the top of These ports communicate with the condenser.
the condenser. The opening of air extraction pump When the plunger is at the top position, the
is separated from opening for incoming steam by a condensate and air from the condenser is collected
screen or baffle. It avoids the steam being directly in the conical portion of the barrel. When the piston
removed by the air extraction pump and permits moves down, the partial vacuum is created above
the cooling of air prior to entry in the air-extraction the plunger. The air and condensate rush into the
pump. barrel above the plunger through ports in it. At this
position the head valves are in a closed position
(ii) In this system, a
and sealed by water. When the piston moves up, the
wet air extraction pump extracts -air with conden-
condensate with air is compressed slightly above
sate from the condenser. The air is extracted at the
atmospheric pressure and delivery valves open.
temperature of condensate without cooling. The
The condensate-air mixture is discharged above the
pump is connected at the bottom of condenser.
barrel cover. The condensate flows over the weirs
The pump may be reciprocating or rotary type. to the hot well.
The following two pumps are commonly used in a A relief valve is provided at the base of the
steam power plant. cylinder. This valve opens to the atmosphere to
1. Edward’s air pump 2. Steam jet air ejector release the pressure in case the pressure inside
increases above atmospheric pressure.

The most commonly used reciprocating type of air


extraction pump is Edward’s air pump as shown The working principle of a steam-jet air ejector is
in Fig. 23.14. It is a wet-air extraction pump. It is shown in Fig. 23.15. It consists of a convergent–
770 Thermal Engineering

To find Capacity of air pump in m3/min.


Assumption The specific gas constant for air as
0.287 kJ/kg ◊ K.
Analysis The mass-flow rate of air in the condenser
Ê Steam condensed in kg/h ˆ
ma = Á + 2˜ kg/h
Ë 2000 ¯
(22000 kg/h)
divergent nozzle and a diffuser. The high-pressure = + 2 = 13 kg/h
2000
steam from the boiler enters the nozzle A, where its From steam tables:
kinetic energy increases and pressure reduces. The At condensate temperature of 30°C
pipe C is connected to the condenser from where psat = 4.25 kPa and vf = 0.001004 m3/kg
moist air is sucked by low-pressure steam at the The absolute pressure in the condenser
location B. The momentum of steam jet carries the pabs = p = patm – pg = 760 – 700 = 60 mm of Hg
mixture of steam and moist air in to the diffuser D,
101.325 kPa
where its velocity decreases and pressure increases = 60 mm of Hg ¥ = 8 kPa
717.79 mm of Hg
before discharge.
Using Dalton’s law of partial pressure of air in the
If sufficient number of ejectors are installed, condenser
a very low pressure can be maintained in the pa = p – psat = 8 kPa – 4.25 kPa = 3.75 kPa
condenser. The steam ejectors are simple in con- From ideal-gas relation, the volume flow rate of air
struction, cheap, highly efficient and without any
ma Ra Ta
moving parts. These are suitable for large power Va =
pa
plants, where a high condenser vacuum is required.
(13 kg/h) ¥ (0.287 kJ/kg ◊ K) ¥ (30 + 273)(K)
=
Example 23.16 During a trial on a condenser, the (3.75 kPa)
following readings were recorded: = 301.46 m3/h
Condenser vacuum = 716 mm of Hg The volume rate of steam condensed
Barometer reading = 760 mm of Hg Vsteam = msteam ¥ v f
Mean condenser Temperature = 30°C = 22000 ¥ 0.001004 = 22.088 m3/h
Condensate collected = 22000 kg/h Total volume to be handled by wet pump per hour
The air entering the condenser is given by the V = Va + Vsteam = 301.46 + 22.088
equation = 323.55 m3/h = 5.393 m3/min.
Ê Steam condensed in kg/h ˆ The discharge capacity of wet pump
ma = Á + 2˜ kg/h
Ë 2000 ¯ Volume flow rate
=
Calculate the discharge capacity of wet air pump, Volumetric efficiency
which removes air and condensate in m3/min. Take 5.392 m3 / min
volumetric efficiency of the pump as 80%. = = 6.74 m3/min
0.8

Example 23.17 The air leakage into a surface


Given Data from the test on a surface condenser: condenser operating with a steam turbine is estimated
Barometer reading, patm = 760 mm of Hg as 84 kg/h. The vacuum near the inlet of the air pump is
Condenser vacuum pressure, pg = 700 mm of Hg 700 mm of Hg, when the barometer reads 760 mm of Hg.
Hot well temperature, Tc = 30°C The temperature at the inlet of vacuum pump is 20°C.
Mass of steam condensed, msteam = 22000 kg/h Calculate
Volumetric efficiency, hvol = 0.8 (a) minimum capacity of air pump in m3/h,
Steam Condensers 771

(b) The diameter of reciprocating air pump to remove The discharge capacity of wet pump
the air, if it runs at 200 rpm with L/D ratio of 1.5 Volume flow rate
and volumetric efficiency of 100%, =
Volumetric efficiency
(c) Mass of vapour extracted per minute.
20.8 m3/min
=
1.0
= 20.8 m3/min
Given A surface condenser:
Tsat = 20°C ma = 84 kg/h (ii) Diameter of pump
hvol = 1.0 L/D = 1.5 The pump extraction rate can be expressed as
N = 200 rpm patm = 760 mm of Hg Êpˆ 2
pg = 700 mm of Hg Va = ÁË ˜¯ D L N hvol
4
To find Êpˆ 2
or 20.8 = Á ˜ D ¥ (1.5 D ) ¥ 200 ¥ 1.0
(i) Capacity of air pump, Ë 4¯
(ii) Cylinder diameter, and
(iii) Mass of vapour extracted. or D3 = 0.088278 m3 = 88278 cm3
Diameter D = 44.52 cm
Assumptions (iii) Mass of vapour extracted
(i) For air R = 0.287 kJ/kg ◊ K. Assuming some steam also flows with air through
(ii) Air pump handles dry air only. the air-extraction pump, the mass flow rate
Analysis The mass-flow rate of air into the condenser Va 20.8
per minute msteam = =
vf 0.001002
84
ma = = 1.4 kg/min = 20778.2 kg/min
60
At condensate temperature of 20°C, Example 23.18 A surface condenser deals with
psat = 2.34 kPa and vf = 0.001002 m3/kg 13625 kg of steam per hour at a pressure of 0.09 bar.
The absolute pressure in the condenser, The steam enters 0.85 dry and the temperature of the
pabs = p = patm – pg = 760 – 700 condensate and air extraction pipes is 36°C. The air
= 60 mm of Hg leakage amount to be 7.26 kg/h.
101.325 kPa Determine
= 60 mm of Hg ¥
717.79 mm of Hg (a) the surface area required, if the average heat
= 8 kPa transmission rate is 3.97 kJ/cm2
Using Dalton’s law, the partial pressure of air in the (b) the cylinder diameter of dry air pump, if it is to
condenser be a single-acting reciprocating type, runs at
pa = p – psat = 8 kPa – 2.34 kPa 60 rpm with a stroke-to-bore ratio of 1.25 and a
= 5.66 kPa volumetric ratio of 0.85
(i) Capcaity of air pump
From ideal gas relation, the volume flow rate of
air Given A surface condenser:
ma Ra Ta msteam = 13625 kg/h p = 0.09 bar = 9 kPa
Va =
pa x = 0.85 Tsat = 36°C
(1.4 kg/min) ¥ (0.287 kJ/kg ◊ K) ¥ (20 + 273)(K) ma = 7.26 kg/h hvol = 0.85
=
(5.66 kPa) L/D = 1.25 q = 3.97 kJ/cm2
= 20.8 m3/h N = 60 rpm
772 Thermal Engineering

To find
(i) Surface area of condenser, and
(ii) Cylinder diameter of dry air pump. Power plants, air-conditioning systems and some
industries produce a large quantity of waste heat
Assumptions
in the form of hot water. In the present scenario,
(i) For air R = 0.287 kJ/kg ◊ K.
in most of the places, the water supply is limited
(ii) Air pump handles air only.
and thermal pollution is also a serious concern.
Analysis From steam tables, Therefore, the waste heat from water must be
At 0.09 bar = 9 kPa rejected for its re-use.
hf = 183.3 kJ/kg and The cooling tower is a direct-contact type of
hfg = 2397.7 kJ/kg heat exchanger. It is a semi-enclosed, evaporative
At 36°C, psat = 5.94 kPa cooler. The water is sprayed through a certain
hfc = 150.86 kJ/kg height and an air current passes over it. Some water
The heat transfer rate from steam per second evaporates and the heat of evaporation is extracted
Q = msteam [hf + x hfg – hc ] from falling water and surrounding air, thus both
water and air cool.
13625
= ¥ È183.3 + 0.85 ¥ 2397.7 - 150.86 ˘˚
3600 Î The cooling towers are classified as shown:
= 7836.2 kJ/s Cooling towers

The surface area required,


Q 7836.2 kJ/s Natural draft Mechanical
A = = = 1973.85 cm2 (hyperbolic) draft
q 3.97 kJ/cm 2 .s
The mass-flow rate of air into the condenser per
Cross Counter Cross Counter
minute flow flow flow flow
7.26
ma = = 0.121 kg/min
60
The partial pressure of air into the condenser, A natural-draft cooling tower is tall and hyperbolic
pa = p – psat = 9 – 5.94 = 3.06 kPa in construction. It looks like a large chimney and
The volume-flow rate into the condenser, works as an ordinary chimney. The chimney effect
ma Ra Tsat is created by the hyperbolic profile of the tower. As
Va =
pa air comes in contact of heated falling water, air is
(0.121 kg/s) ¥ (0.287 kJ/kg ◊ K) ¥ (36 + 273)(K) heated, becomes lighter and rises up in the tower.
=
(3.06 kPa) The cold and heavier outside air fills the vacant
= 3.506 m3/min space creating an air flow. These towers do not
The pump extracts the air with this discharge rate, i.e., require any external power to induce the air, but their
construction cost is much more than mechanical-
Êpˆ 2
Va = Á ˜ D L N hvol draft cooling towers. These are constructed from
Ë 4¯
RCC and have base diameters ranging 75 to 100 m
Êpˆ and heights between 100 m to 150 m.
or 3.506 = Á ˜ D 2 ¥ (1.25 D ) ¥ 60 ¥ 0.85
Ë 4¯ In the cross-flow arrangement, the water is
or D3 = 0.0700 m3 = 0.07 ¥ 106 cm3
sprayed downward near the base of the tower on
a fill material. Air is induced radially inward and
or D = 41.2 cm
flows in across the water.
Steam Condensers 773

In the counter-flow arrangement as shown in


Fig. 23.16, air enters through a peripheral section at
the bottom of the tower and flows upward through
a descending water spray made from a suitable
height.
The heat exchanger section in cross-and counter-
flow natural-draft cooling towers occupies a small
portion of the tower, while the remaining structure
is used to promote the chimney effect. These towers
are only suitable in desert regions, where moisture
content in air is very low.
is discharged from the top of the tower. Since some
of the warm water is evaporated into the air stream,
an equivalent amount of make-up water is added to
the cycle.

1. The natural-draft cooling towers have a


higher construction cost than mechanical
draft towers.
2. Natural-draft cooling towers require unrea-
sonable high height. They use only a small
portion for heat exchange between hot
water and air, while mechanical draft cool-
ing towers utilize the whole height and space
These are induced-draft cooling towers. These are for heat exchange.
compact in size with low height. The air movement
3. Natural-draft cooling towers cannot control
through the tower is created by induced fans,
the temperature of outlet water precisely.
located at the top of the tower.
However, the mechanical cooling towers
An induced draft counter flow cooling tower is enable better control on heat transfer
shown in Fig. 23.17. The warm water coming from process, which is favourable for varying
the condenser is pumped to the top of the tower load on power plant and changing ambient
and is sprayed into air stream. The falling water conditions.
passes through a series of baffles, thus the water
breaks into fine droplets to promote evaporation.
The atmospheric air is drawn in by an induced fan
flowing upward, counter to the direction of the The cooling ponds are large open tanks. The hot
falling water droplets. As the two streams interact, water is sprayed into air and is cooled by air as it
a small fraction of water stream evaporates into falls into the pond as shown in Fig. 23.18. The cold
moist air and cools the remaining falling water. The water from the pond is recirculated to the condenser
cooled water is collected at the bottom of the tower with the help of a pump. In the cooling pond, the
and is pumped back to the condenser. The moist air cooling effect is mainly due to evaporation of
774 Thermal Engineering

sprayed water and water from the upper surface of


the pond.
The cooling ponds are suitable for small capacity
steam plants or where land is cheaply available.
The water loss due to air drift (windage) is high and
it is unprotected against dust and dirt.

is a device, used for vapour conden- in the condenser in order to provide large
sation to liquid state at saturation temperature and expansion of steam through the turbine. The
constant pressure. The condensers are of basically vacuum efficiency is defined as
two types: jet and surface condensers. Actual vacuum in the condenser
hvacuum =
Maximum obtained vacuum
water mix together. The cooling water is sprayed pg
=
on the steam, which causes rapid condensation. pg, max
These are further classified as
(i) low evel,
l vacuum is disturbed and actual vacuum is less
(ii) high level, and than the vacuum that could be obtained in absence
(iii) ejector-type jet condensers. of air in the condenser.
surface condensers, the steam and cooling Actual vacuum = Barometric pressure –
water do not physically interact. The exhaust Absolute measured pressure
steam condenses over the outer surface of the in the condenser
tubes, through which cooling water flows. The
Maximum = Barometric pressure –
condensate coming out of the condenser is pure,
vacuum Saturation pressure of steam
thus it is used as feed water in the boiler.
at condenser temperature.
power plants, refrigerating plant and for chemical
air from the condenser. The removal air helps to
equipments.
maintain the desired vacuum in the condenser.
water in the large surface condenser. It uses the
the condenser and the condensate is extracted by
principle of water evaporation.
the condensate extraction pump, while the wet
Actual temperature rise of
air pump is used to remove both condensate and
cooling water
hcondenser = air from the condenser. Edward’s wet air pump
Maximum possible tem mperature is suitable for small power plants, while steam-
rise of water jet air ejectors help to maintain very low back
vacuum is the pressure measured below pressure in the condenser, and are thus useful for
atmospheric pressure. The vacuum is maintained large power plants.
Steam Condensers 775

Glossary
Condenser A device in which vapour condenses to Surface condenser Exhaust steam and cooling water
liquid at saturation temperature and constant pressure interact indirectly for heat transfer
Jet condensers A device in which the exhaust steam Condenser efficiency Ratio of actual temperature rise
and cooling water come in direct contact and mix up to the maximum possible temperature rise of cooling
together water
Barometric Condenser A high-level jet condenser Vacuum The pressure below the atmospheric pressure
having a tail pipe longer than 10.33 m. Vacuum efficiency Ratio of the actual vacuum to the
maximum possible vacuum

Review Questions
1. Write the function and applications of the 12. Explain the working of an evaporative condenser.
condenser. 13. Explain the working of a cooling pond with a
2. Why is the steam condenser used in each power diagram.
plant unit? 14. Explain the working of a natural-draft cross-flow
3. Classify the steam condenser. cooling tower with a diagram.
4. Describe the details and working of a low-level 15. Define condenser efficiency.
jet condenser. 16. Define vacuum and how it is corrected to standard
5. Explain the working principle of a jet condenser. barometric reading?
6. Explain the working of a barometric jet condenser. 17. Define vacuum efficiency?
7. Explain the working of an ejector-type condenser. 18. Write the source of air into a condenser.
8. Why does a barometric jet condenser not require 19. Differentiate between jet and surface condensers.
a water-extraction pump? Explain. 20. State Dalton’s law of partial pressures.
9. Why does an ejector-type jet condenser not 21. Explain the determination of quantity of circulat-
require a water-extraction pump? Explain. ing water in a condenser.
10. Explain the principle of working of a surface 22. What do you mean by the terms ‘undercooling of
condenser. condensate’ and ‘hot well’?
11. Explain the working of a shell-and-tube type
surface condenser.

Problems
1. Exhaust steam having a dryness fraction of temperature is 37.31°C. The air leakage into
0.85 enters a surface condenser at a pressure the condenser is 5 kg per 10,000 kg of steam.
of 9.81 kPa and is condensed to water at 38°C. Determine (a) the volume of air to be dealt with
The circulating water enters at 15°C and leaves the dry air pump per kg of steam entering the
at 30°C. Calculate the mass of cooling water condenser, and (b) the mass of water vapour
required per kg of exhaust steam. [32.88 kg] associated with this air. The barometric reading is
2. The vacuum at the air-extraction pump in 760 mm of Hg. [(a) 0.0628 m3, (b) 0.00267 kg]
a condenser is 706 mm of mercury and the
776 Thermal Engineering

3. 80% dry steam enters a surface condenser Barometer reading = 760 mm of Hg; Recorded
where the vacuum is 92.85 kPa, the barometer vacuum = 700 mm of Hg; Mean temperature of
reads 101.18 kPa and is condensed to 37.31°C. condensation = 34°C; Hot-well temperature =
The temperature of the hot well is 32.2°C. The 27°C. Mass of condensate = 2120 kg/h; Mass of
circulating water enters at 15.5°C and leaves at cooling water = 66000 kg/h; Rise in temperature
30°C. Determine of cooling water = 16°C.
(a) the mass of air extracted per kg of steam, Find
(b) the mass of circulating water required per (a) the state of steam entering the condenser,
kg of steam, and (b) mass of air present m3 of condenser, and
(c) the vacuum efficiency. (c) vacuum efficiency.
[(a) 0.4 kg, (b) 32.14 kg/kg of steam, [(a) x = 0.8492 (b) ma = 0.0304 kg/m3
(c) 98%] (c) hvaccum = 97.2%]
4. The following observations were made on a 9. The following observations were recorded during
condensing plant in which the temperature a test on a steam condenser:
of condensation was measured directly by Recorded condenser vacuum = 71 cm of Hg;
thermometers; the recorded vacuum was 710 mm Barometer reading = 76.5 cm of Hg, Mean
of Hg and the barometer reads 765 mm of Hg. condenser temperature = 34°C, Condensate
The mean temperature of condensation and hot collected = 1800 kg/h; Mass of cooling water
well was 34.25°C and 28°C, respectively. The = 5700 kg/h; Cooling water temperature rise =
condensate leaves the condenser at the rate of 17.5°C.
2000 kg/h. The cooling water circulates in the Calculate
condenser at the rate of 64,000 kg/h and its inlet (a) Corrected vacuum to standard barometer of
and outlet temperature was recorded 14.5°C and 76 cm of Hg,
30°C, respectively. Calculate (a) the state of
(b) vacuum efficiency,
steam entering the condenser, and (b) the mass of
(c) condenser efficiency, and
air present per m3 of condenser volume.
(d) state of steam entering the condenser.
[(a) 0.85, (b) 0.497 kg]
Assume inlet temperature of cooling water =
5. A vacuum of 712 mm was obtained with a
8.5°C. [(a) 70.5 cm of Hg (b) 97.91%
barometer reading of 753 mm of Hg. Correct
the vacuum to a standard barometric reading of (c) 55.76% (d) 0.9667]
76 cm of Hg. [71.9 cm of Hg] 10. The vacuum gauge reading of a condenser is 71 cm
6. Following data were recorded during testing of a of Hg and the mean temperature of the condenser
condenser: is 35°C. The air leakage into the condenser is
1 kg per 2000 kg of steam. Determine the volume
Vacuum = 700 mm;
of air to be handled by the dry air pump per kg of
Barometer reading = 754 mm of Hg;
steam and the mass of vapour associated with air.
Condensate temperature = 18°C. Barometer reading = 76 cm.
Find the partial pressure of air and steam in the [0.0424 m3/kg of steam, 0.00168 kg]
condenser and the mass of air/kg of steam.
11. The following observations were made on a
[pa = 0.05135 bar; ps = 0.071976 bar; steam condensing plant:
3.775 kg/kg of steam] Barometer reading = 76 cm of Hg; Recorded
7. The vacuum reading of a condenser is 70.5 cm condenser vacuum = 70 cm of Hg; Mean
of Hg when the barometer shows 76 cm of Hg temperature of condensate = 35°C; Condensate
and the condensate temperature is 31°C. Find the collected 16.75 kg/min; Cooling water =
vacuum efficiency. [97.06%] 39600 kg/h; Rise in temperature of cooling water
8. The following observations were made on a = 14°C; Inlet temperature of cooling water =
steam condensing plant: 25°C.
Steam Condensers 777

Calculate 37.31°C. The temperature of the hot well is


(a) Mass of air present per m3 of condenser 32.2°C. The circulating water enters at 15.5°C
volume, and leaves at 30°C. Determine (a) the mass of
(b) Dryness fraction of steam at entry to air extracted per kg of steam, (b) the mass of
condenser, circulating water required per kg of steam, and
(c) The vacuum efficiency, and (c) the vacuum efficiency.
(d) The effectiveness (efficiency) of the 13. A surface condenser is designed to handle
condenser. 10,000 kg of steam per hour. The steam enters
[(a) 0.02692 kg/m3 (b) 0.87 at 0.08 bar and 0.9 dry and the condensate leaves
at corresponding saturation temperature. the
(c) 97.51% (d) 84.64%]
pressure is constant throughout the condenser.
12. Exhaust steam having a dryness fraction of 0.8
Calculate the cooling water flow rate per hour, if
enters a surface steam condenser where the
the cooling water temperature rise is 10°C.
vacuum is 696.5 mm of mercury (barometer
reading = 759 mm of Hg) and is condensed to

Objective Questions
1. The condensation of steam in a condenser takes
place at (a) they require less coolant
(a) constant pressure (b) condensate can be reused
(b) constant temperature (c) they are more efficient
(c) constant pressure and constant temperature (d) none of the above
(d) none of the above 6. In evaporative condensers, the condensing of
2. During condensation process, the temperature of steam is achieved
the condensing fluid (a) by rejecting heat to surrounding air
(a) remains constant (b) by rejecting heat to coolant
(b) decreases (c) by evaporation of some coolant
(c) increases (d) none of the above
(d) none of the above 7. Dalton’s law of partial pressure, applicable to
3. The function of a condenser in a steam power condensers, states that
plant is (a) pa = pabs + psat
(a) to reduce back pressure (b) psat = pabs + pa
(b) to condense the exhaust steam (c) pabs = pa+ psat
(c) to reduce specific volume of fluid (d) none of the above
(d) all of the above 8. The absolute pressure in a condenser is given by
4. In a high-level jet condenser, the condenser shell (a) pabs = patm + pvacuum
is installed at a height of (b) pabs = patm – pvacuum
(a) more than 5.5 m (c) pabs = pvacuum
(b) more than 10.33 m (d) none of the above
(c) less than 10.33 m 9. The vacuum efficiency of a condenser is defined
(d) none of the above as
5. The surface condensers are preferred in steam Actual absolute pressure in condeser
(a)
power plant, because Atmospheric pressure
778 Thermal Engineering

Atmospheric pressure 11. Air leakage into the condenser reduces


(b) (a) turbine output
Absolute pressure in condenser
(b) cooling capacity
Actual vacuum in the condeser
(c) (c) life of condenser
Maximum possible vacuum
(d) all of the above
(d) none of the above
12. The vacuum maintained in a condenser depends
on
10. The condenser efficiency is defined as
(a) pressure of cooling water
Temp. rise of coolant
(a) (b) temperature of cooling water
Temp. drop of condensate
(c) back pressure maintained in the condenser
Temp. drop of condensate (d) all of the above
(b)
Temp. rise of coolant
Actual temp. rise of coolant
(c)
Maximum possible temp. rise off coolant
(d) none of the above

12. (b) 11. (d) 10. (c) 9. (c)


8. (b) 7. (c) 6. (c) 5. (b) 4. (b) 3. (d) 2. (a) 1. (c)
Answers
Internal Combustion Engines 779

24
Internal Combustion Engines

Introduction
An internal combustion engine is a machine that converts chemical energy in a fuel into mechanical energy.
Fuel is burnt in a combustion chamber, releases its chemical energy in the form of heat, which is converted
into mechanical energy with the help of a reciprocating piston and crank mechanism.
Two principal types of reciprocating internal combustion engines are in general use: the Otto-cycle engine
and the Diesel engine. The Otto-cycle engine, named after its inventor, the German technician Nikolaus
August Otto, is the familiar gasoline engine used in automobiles and airplanes. The Diesel engine, named
after the French-born German engineer Rudolf Christian Karl Diesel, operates on a different principle and
usually uses oil as a fuel. It is employed in electric-generation and marine-power plants, in trucks and buses,
and in other automobiles. Both Otto-cycle and Diesel-cycle engines are manufactured in two-stroke and
four-stroke cycle models.

24.1 CLASSIFICATION OF IC 3. According to method of ignition


ENGINES (a) Spark ignition
The internal combustion engines are usually of (b) Compression ignition
reciprocating type. The reciprocating internal 4. According to fuel-feeding system
combustion engines are classified on the basis of (a) Carburetted engine
the thermodynamic cycle and mechanical method (b) Engine with fuel injection
of operation, type of fuel used, type of ignition, 5. According to charge feeding system
type of cooling system and cylinder arrangement, (a) Naturally aspirated engine
etc. The detailed classification is given below:
(b) Supercharged engine
1. According to piston strokes in the working 6. According to cooling system
cycle
(a) Air-cooled engine
(a) Four-stroke engine
(b) Water-cooled engine
(b) Two-stroke engine
7. According to number of cylinders
2. According to fuel used in the cycle
(a) Single cylinder engine
(a) Petrol engine (b) Diesel engine
(c) Gas engine (d) Multi–fuel engine (b) Multi-cylinder engine
780 Thermal Engineering

8. According to speed of the engine 1. It is the heart of the engine. The piston
(a) Low-speed engine reciprocates in the cylinder. It has to withstand
(b) Medium-speed engine high pressure and temperature, and thus it is made
strong. Generally, it is made from cast iron. It is
(c) High-speed engine
provided with a cylinder liner on the inner side and
9. According to position of engine a cooling arrangement on its outer side. For two-
(a) Horizontal engine stroke engines, it houses exhaust and transfer port.
(b) Vertical engine
2. The top cover of the cylinder,
(c) V-engine towards TDC, is called cylinder head. It houses the
The petrol engines use low compression ratio. spark plug in petrol engines and fuel injector in
The fuel and air mixture as a charge is ignited by a Diesel engines. For four stroke cycle engines, the
high intensity spark. Therefore, they are also called cylinder head has the housing of inlet and exhaust
spark ignition (SI) engines. The Diesel engines use valves.
high compression ratio, and the compressed charge
3. It is the reciprocating member of the
is autoignited. Therefore, they are also called
engine. It reciprocates in the cylinder. It is usually
compression ignition (CI) engines.
made of cast iron or aluminium alloys. Its top
surface is called piston crown and bottom surface
is called piston skirt. Its top surface is made flat for
The essential parts of Otto-cycle and Diesel- four-stroke engines and deflected for two-stroke
cycle engines are the same. Actually an internal engines.
combustion engine consists of a large number of 4. Piston Rings Two or three piston rings are
parts and each part has its own function. A few of provided on the piston. The piston rings seal the
them are shown in Fig. 24.1 and listed below: space between the cylinder liner and piston in order
Exhaust valve
to prevent leakage (blow by losses) of high-pressure
Inlet valve Valve spring gases, from cylinder to crank case.
Exhaust port
5. It is a rotating member. It makes circular
Inlet port Cylinder head motion in the crank case (its housing). Its one end
is connected with a shaft called crank-shaft and the
Cooling fins other end is connected with a connecting rod.
Piston rings
Piston
Cylinder
Cylinder liner Piston pin 6. It is the housing of the crank and
body of the engine to which cylinder and other
Connecting rod
engine parts are fastened. It also acts as a ground
for lubricating oil.
Crank case
7. It is a link between the piston
Crank pin
and crank. It is connected its one end with a crank
Crank and on the other end with a piston. It transmits
shaft Crank case
power developed on the piston to a crank shaft
Crank through crank. It is usually made of medium carbon
steel.
8. It is the shaft, a rotating member,
Fig. 24.1
which connects the crank. The power developed by
Internal Combustion Engines 781

the engine is transmitted outside through this shaft. mixture, as a charge, is supplied to engine
It is made of medium carbon or alloy steels. cylinder through suction valve or port.
9. Cooling Fins or During (b) It is provided with a Diesel
combustion, the engine releases a large amount of engine. The diesel is taken from the fuel
heat. Thus the engine parts may be subjected to a tank, its pressure is raised in the fuel pump
temperature at which engine parts may not sustain and then it is delivered to fuel injector.
their properties such as hardness, etc. In order to
keep the engine parts within safe temperature 17. It is mounted on the crank shaft and
limits, the cylinder and the cylinder head are is made of cast iron. It stores energy in the form
provided with a cooling arrangement. The cooling of inertia, when energy is in excess and it gives
fins are provided on light duty engines, while a back energy when it is in deficit. In other words,
cooling water jacket is provided on medium and it minimizes the speed fluctuations on the engine.
heavy duty engines.
24.3 OTTO CYCLE ENGINES:
10. It is provided on four-stroke
engines. It carries two cams, for controlling the
opening and closing of inlet and exhaust valves. The ordinary Otto-cycle engine is a four-stroke
engine; that is, its piston makes four strokes, two
11. This valve controls the admission toward the cylinder head (TDC) and two away from
of charge into the engine during a suction stroke.
the head (BDC). By suitable design, it is possible to
12. The removal of exhausted operate an Otto-cycle as a two-stroke cycle engine
gases after doing work on the piston is controlled with one power stroke in every revolution of the
by the valve. engine. Thus, the power of a two-stroke cycle
engine is theoretically double that of a four-stroke
13. It is the passage which carries cycle engine of comparable size. These engines are
the charge from carburettor to the engine. also called spark ignition engines.
14. It is the passage which
carries the exhaust gases from the exhaust valve to
the atmosphere. All essential operations are carried out in one
revolution of the crank shaft or two strokes of the
15. (a) It is provided on petrol
piston. Therefore, the engine is called a two-stroke
engines. It produces a high-intensity spark
or two-stroke cycle engine.
which initiates the combustion process of
the charge. (a)
A two-stroke petrol engine is shown in Fig. 24.2.
(b) It is provided on Diesel
It consists of a cylinder, cylinder head, piston,
engines. The Diesel fuel is injected in the
cylinder at the end of the compression piston rings, connecting rod, crank, crank case,
through a fuel injector under very high crank shaft, etc. The charge (air–fuel mixture) is
pressure. prepared outside the cylinder in the carburettor. In
the simplest type of two-stroke engine, the ports are
16. (a) It is provided with a petrol provided for charge inlet and exhaust outlet, which
engine for preparation of a homogeneous are uncovered and closed by the moving piston.
mixture of air and fuel (petrol). This The suction port S with a reed-type valve is used for
782 Thermal Engineering

the purpose of discharging the burnt gases from the


cylinder. The spark plug is located in the cylinder
head.
(b) Operation
In a two-stroke engine, the inlet, transfer and exhaust
ports are covered and uncovered by a moving piston.
The following operations take place in a two-stroke
or in one cycle of the engine.
(i) Charge Transfer and Scavenging When the piston
is nearer to the crank case (bottom dead centre),
the transfer port and exhaust port are uncovered
by the piston as shown in Fig. 24.3(a). A mixture
of air and fuel as a charge, slightly compressed in
Fig. 24.2
the crank case, enters through the transfer port T
and drives out the burnt gases of the previous cycle
induction of charge into the crank case, the transfer through the exhaust port E.
port T is used for transfer of charge from the crank In a two-stroke engine, the piston top is made
case to the cylinder and the exhaust port E serves deflected. Therefore, the incoming charge is

Spark plug

Cylinder
Exhaust port (E )
E

T T
Piston

S
Connecting rod Suction port (S)
Crank case
Crank

(a) Charge transfer and scavenging (b) Start of compression

Cylinder Deflector
Exhaust port (E )
E

Transfer port (T ) T

S Inlet port
S

(c) Compression and Suction (d) Power and exhaust


Fig. 24.3
Internal Combustion Engines 783

directed upward, and aids in sweeping of the burnt in the cylinder. The burning of the charge generates
gases out of the cylinder. This operation is known the pressure in the cylinder.
as scavenging (a gas-exchange process).
( ) The burning gases apply
As the piston moves upward, the fresh charge pressure on the top of the piston, and the piston is
passes into the cylinder for 1/6th of the revolution forced downward as a result of pressure generated.
and the exhaust port remains open a little longer
As the piston descends through about 80%
than the transfer port.
of the expansion stroke, the exhaust port E is
( ) As the piston moves uncovered by the piston, and the combustion gases
upward, both the transfer port and exhaust port are leave the cylinder by pressure difference and at
covered by the piston and the charge trapped in the same time, the underside of the piston causes
the cylinder is compressed by the piston’s upward compression of charge taken into crank case as
movement as shown in Fig. 24.3(b). At the same shown in Fig. 24.3(d).
time, a partial vacuum is created into the crank case,
( ) The slightly compressed charge in
the suction port S opens by moving the crank and
the crank case passes through the transfer port and
the fresh charge enters the crank case [Fig. 24.3 (c)].
enters the cylinder as soon as it is uncovered by
( ) When the piston reaches at its end the descending piston and when it approaches the
of stroke nearer to the cylinder head or at the top bottom dead centre, the cycle is completed.
dead centre, a high-intensity spark from the spark The p-V diagram and port-timing diagram for
plug ignites the charge and initiates the combustion a two-stroke petrol engine are shown in Figs. 24.4
and 24.5.
784 Thermal Engineering

(c) Port-Timing Diagram 1. 50 cc–70 cc engines are used in mopeds,


lawn moovers and non-gear vehicles.
TDC
2. 100–150 cc engines are commonly used in
Suction in crank case and scooters and motor cycles.
combustion in cylinder head
IGN 3. 250 cc two-stroke engines are used in high-
IPU IPC powered (racing) motor cycles.
80°
4. These engines can also be used in small

Exp
Compression

electric generator sets, pumping sets and


ansion
motor boats.

100°
EPC 120° EPU
TPC
Charge
TPU All operations are carried out in four strokes of
Transfer
the piston, i.e., two revolutions of the crank shaft.
Exhaust Therefore, the engine is called a four stroke engine.
BDC (a) Constructional Details
IPU = Inlet port uncovered 40° before TDC Similar to a two-stroke engine, it also consists of
IPC = Inlet port closed 40° after TDC
a cylinder, cylinder head attached with spark plug,
EPU = Exhaust port uncovered 60° before BDC
EPC = Exhaust port closed 60° after BDC piston attached with piston ring, connecting rod,
TPU = Transfer port uncovered 50° before BDC crank, crank shaft, etc., as shown in Fig. 24.6. In a
TPC = Transfer port closed 50° after BDC
IGN = Spark ignition 15-20º before TDC four-stroke engine, valves are used instead of ports.
There are suction and exhaust valves. These valves
Fig. 24.5
are operated by cams attached on a separate shaft,
called a cam shaft. It is rotated at half the speed of
(d) Applications
a crank shaft.
Two-stroke gasoline engines are used where sim-
plicity and low cost are main considerations. These (b) Operation
engines have a little higher specific fuel consump- The travel of the piston from one dead centre
tion. to another is called piston stroke and a four-

Intake valve
Spark plug
Combustion
chamber

Piston
Carburetor
Cylinder

Crank shaft Air filter

Fuel filter
Fuel supply

Fig. 24.6
Internal Combustion Engines 785

stroke cycle consists of four strokes as suction, exhaust valve opens and burnt gases are expelled
compression, expansion and exhaust strokes. to surroundings by upward movement of the piston
as shown in Fig. 24.7(d). This stroke is completed
The suction valve opens, exhaust
when the piston approaches the top dead centre.
valve remains closed as shown in Fig. 24.7(a).
Thus, one cycle of a four stroke petrol engine is
The piston moves from the top dead centre to the
completed. The next cycle begins with piston
bottom dead centre, the charge (mixture of fuel and
movement from the top dead centre to the bottom
air prepared in the carburettor) is drawn into the
dead centre.
cylinder.
Figure 24.8 shows the p–V diagram with a
When the piston moves schematic of a four-stroke petrol engine.
from the bottom dead centre to top dead centre, and (c) Valve Timing
the suction valve is closed, exhaust valve remains
Theoretically, in a four-stroke cycle engine, the
closed as shown in Fig. 24.7(b). The trapped
inlet and exhaust valves open and close at dead
charge in the cylinder is compressed by the upward
centres as shown in Fig. 24.9(a).
moving piston. As the piston approaches the top
A typical valve-timing diagram for a four-stroke
dead centre, the compression stroke completes.
petrol engine is shown in Fig. 24.9(b). The angular
At the end of the compression
positions in terms of crank angle with respect to
stroke, the compressed charge is ignited by a high- TDC and BDC position of piston are quoted on the
intensity spark created by a spark plug, combustion diagram.
starts and the high-pressure burning gases force the When the inlet valve and exhaust valve remain
piston downward as shown in Fig. 24.7(c). The gas open simultaneously, it is called a valve operlap.
pressure performs work, therefore, it is also called
(d) Applications
working stroke or power stroke. When the piston
approaches the bottom dead centre in its downward These engines are mostly used on automobiles,
stroke then this stroke is completed. In this stroke, motor cycles, cars, buses, trucks, aeroplanes, small
both valves remain closed. pumping sets, mobile electric generators, etc.
Nowadays, the four-stroke petrol engines have
When the piston moves from the been replaced by four-stroke Diesel engines for
bottom dead centre to the top dead centre, only the most applications.

Fuel-air
mixture in Spark plug Gases out

(a) Suction (b) Compression (c) Power (d) Exhaust


Fig. 24.7
786 Thermal Engineering

P
3
Heat
supply

Ex
pa
nsi
on
2

Com
pre
s sion 4
Heat release p
Suction
1 3
Exhaust
V
TDC
Vc Vs BDC

Cylinder 2
TDC Piston BDC IGN
Exhaust valve 4
EVO
Spark
Gudgen pin EVC Exhaust
5 1
plug patm IVC
Crank
Cylinder head IVO Suction
Suction valve
V
Vc V1
Vs
Connecting rod Crank pin TDC BDC

(a) Theoretical p–V diagram for a four-stroke petrol engine (b) Actual p–V diagram for four stroke petrol engine

Fig. 24.8

TDC TDC

EVC EVC
S
ark
Sp IVO 10°
IVO
Ex
n
mpression

10°
ressio

Su
Ex

pansion
30°
Exhaust

Suc

15°
aust
pansion

ction
tion

p
E xh
m
Co
o

35° 50°
C

IVC IVC
EVO

EVO
BDC BDC
(a) Theoretical valve timing diagram of a (b) The typical valve timing diagram for a
four stroke cycle four stroke petrol engine

IVO = Inlet valve opens when piston at TDC IVO : Inlet valve opens about 15° before TDC
IVC = Inlet valve closes, when piston reaches BDC IVC : Inlet valve closes 20° – 40° after BDC to take advantage
S = Spark produces, when piston reaches TDC of rapidly moving gas
EVO = Exhaust valve opens when piston at BDC S : Spark occurs 20°–40° before TDC
EVC = Exhaust valve closes, when piston at TDC EVO : Exhaust valve opens about 50° before BDC
EVC : Exhaust valve close about 0° to 10° after TDC
Fig. 24.9

DIESEL ENGINES except they take in only air as charge during


suction, and fuel is injected at the end of the
All engines using diesel as a fuel operate on the compression stroke. The Diesel engines have a fuel
Diesel cycle. They work similar to a petrol engine injector instead of a spark plug in the cylinder head
Internal Combustion Engines 787

as shown in Fig. 24.10. The diesel engines use a the transfer port and exhaust port are uncovered
high compression ratio in the range of 14 to 21. by the piston and the slightly compressed air
The temperature of intake air reaches quite a high enters into the cylinder through the transfer port
value at the end of compression. Therefore, the and helps to scavenge the remaining burnt gases
injected fuel is self ignited. The Diesel engines use from the cylinder as shown in Fig. 24.11(a). The
a hetrogeneous air–fuel mixture, ratio ranging from charge transfer and scavenging continue till the
20 to 60. piston completes its downward stroke and further,
it moves upward and covers the transfer port.
( ) After covering the
transfer port, the exhaust port is also covered by the
upward moving piston. As both ports are covered
by the piston in Fig. 24.11(b), the air trapped in the
cylinder is compressed during the forward stroke of
the piston. As the piston moves towards the cylinder
head, a partial vacuum is created in the crank case,
the inlet port opens and fresh air enters the crank
case, Fig. 24.11(c).
( ) Near the end of the
compression stroke, the fuel is injected at a very
high pressure with the help of the fuel pump and
injector. The injected fuel is self ignited in the
presence of hot air and combustion starts. The
piston is forced downward by very high pressure
of burnt gases and power is transmitted to the crank
shaft.
( ) Near the end of the power stroke,
the exhaust port is uncovered first by the piston
Fig. 24.10
and the products of combustion start leaving the
cylinder as a result of pressure difference as shown
Two-stroke Diesel Engine in Fig. 24.11(d).

The operation of a two-stroke Diesel engine ( ) The slightly compressed air in the
is similar to a petrol engine, except it takes air crank case passes through the transfer port and
as charge and fuel is injected at the end of the enters the cylinder as soon as it is uncovered by
compression stroke. It uses a high compression the descending piston and when it approaches the
ratio. Therefore, the injected fuel is self-ignited. bottom dead centres, the cycle is completed.
Operation Theoretical p–V Diagram

Both inlet and exhaust take place through the The theoretical p–V digaram for a two-stroke
cylinder ports which are covered and uncovered by Diesel engine is shown in Fig. 24.12. The valve
the piston. timing diagram for a two-stroke Diesel engine is
very similar to that of a two-stroke petrol engine
() When the piston as shown in Fig. 24.9 except, only air is inducted
is nearer to the crank case (bottom dead centre), in the crank case and fuel is injected at the end of
788 Thermal Engineering

Fuel injector

Cylinder
Exhaust port (E)

Piston

Connecting rod
Crank case
Crank

(a) Scavenging (b) Start of compression

Fuel injector

Cylinder Deflector
E

Transfer port

S Inlet port

(c) Compression and suction (d) Exhaust


Operations of a two-stroke Diesel engine

shown in Fig. 24.13. One cycle of a four-stroke


Diesel engine is completed in four strokes of the
piston or two revolutions of the crank shaft.
(a) Working of Engine
The four-stroke Diesel engine operates in a similar
manner as a four-stroke petrol engine. A schematic
of a four-stroke Diesel engine is shown in Fig.
24.13. The details of operations are discussed
below.

Fig. 24.12 1. Suction Stroke The inlet (suction) valve opens,


the exhaust valve remains closed, only air is drawn
the compression stroke, instead of spark from the into the cylinder as the piston moves from the top
spark plug. dead centre to the bottom dead centre. This stroke
ends as the piston approaches the bottom dead
Four-stroke Diesel Engine centre. Fig. 24.14(a)
A four-stroke Diesel engine contains a fuel injector, 2. Compression Stroke As the piston moves from
fuel pump, cylinder, cylinder head, inlet and the bottom dead centre to the top dead centre, the
exhaust valves, piston attached with piston rings, inlet valve closes, exhaust valve remains closed as
connecting rod, crank shaft, cams, camshaft, etc. shown in Fig. 24.14(b). The air trapped into the
Internal Combustion Engines 789

Only air
inlet Fuel injector

(a) Suction (b) Compression


Gases out

Fig. 24.13
(c) Power (d) Exhaust
cylinder is compressed in the cylinder till the piston
Fig. 24.14
approaches the top dead centre. The air temperature
reaches about 800°C by compression. At the end
of the compression stroke, the fuel is injected at stroke are expelled out from the cylinder by upward
very high pressure into the compressed hot air. The movement of the piston.
temperature of hot compressed air is sufficient to The theoretical p–V diagram is shown in
ignite the injected fuel. Thus, ignition takes place Fig. 24.15 for a four-stroke Diesel engine operation.
inside the cylinder.
(b) Valve-Timing Diagram
During this stroke, both valves
Theoretically, the inlet and exhaust valves open at
remain closed as shown in Fig. 24.13(c). The piston
dead centres as shown in Fig. 24.9(a). A typical
at the top dead centre is pushed by expansion of
valve-timing diagram for a four-stroke Diesel
burning gases. Actual work is obtained during this
engine is shown in Fig. 24.16.
stroke due to the force obtained by high pressure
burning gases. Therefore, this stroke is called (c) Applications
power stroke or working stroke.
The four-stroke Diesel engine is one of the most
During this stroke, the piston popular prime movers. It is manufactured from
moves from the bottom dead centre to the top 50 mm to 1000 mm cylinder bore with speeds
dead centre, exhaust valve opens and the inlet ranging from 100 rpm to 4500 rpm. It has wide
valve remains closed. Burnt gases of the previous applications. Some of these are
790 Thermal Engineering

Fig. 24.15

TDC

EVC
Combust ion
FVO FVC

IVO 15°15°
IVO : Inlet valve opens 10° to 30° before TDC
25°
E xp

25° IVC : Inlet valve closes 20°–50° after BDC


pression

Suctio

EVO : Exhaust valve opens approximately 40° before BDC


ansion
Exhaust

EVC : Exhaust valve closes aproximately 15° after TDC


n
Com

FVO : Fuel injection starts 5° to 15° before TDC


25° 40° FVC : Fuel injection stops 15° to 25° after TDC

IVC EVO

BDC

Fig. 24.16

1. Small pumping sets for agriculture, 6. Small power plants, mobile electric generat-
2. Construction machinery, ing plants,
3. Air compressor and drilling jigs, 7. Boats and ships,
4. Tractors, jeeps, cars, taxies, buses, trucks, 8. Power saws, Bulldozers, tanks, etc.
5. Diesel-electric locomotives,
Internal Combustion Engines 791

According to Petrol Engine Diesel Engine


1. Basic cycle It operates on constant-volume cycle. It operates on constant-pressure cycle.
2. Fuel used It uses gasolene or petrol as fuel. It uses diesel and oils as a fuel.
3. Fuel induction The air–fuel mixture is prepared in the The Diesel engine takes in only air during
carburettor and inducted into the engine the suction stroke, and it is compressed. At
cylinder during the suction stroke. the end of the compression stroke the fuel
is injected under the high pressure by a fuel
injector.
4. Ignition of charge The charge (air–fuel mixture) is ignited Fuel is injected in very hot air, therefore, it
by a high-intensity spark produced at the is self-ignited.
spark plug.
5. Compression ar tio It uses less compression ratio, usually It uses high compression ratio, range of
range of 4 to 10. 14 to 21.
6. Pressure rise Lower and controlled rate of pressure rise; High rate of pressure variation, so engine
therefore, operation is salient and smooth. operation is rough, and noisier.
7. Efficiency For the same compression ratio, the It has lower efficiency for same
efficiency of petrol engine is better. compression ratio.
8. Pollution Comparatively lower pollution for same Higher pollution for same power output.
power output.
9. Weight It has comparatively less number of parts, It uses large number of sturdier parts, thus
thus is less in weight. engine is heavy.
10. Cost Engines are cheaper. Costlier engine due to complicated parts.
11. Maintenance It requires less and cheaper maintenance. It requires costlier and large maintainance.
12. Starting Very easy to start due to lower compression Very difficult to start due to higher com-
ratio. pression ratio.

According to Two-Stroke Engine Four-Stroke Engine


1. Working stroke There is one working stroke in each There is one working stroke in two
revolution. Hence engine has more even revolutions. Hence engine has uneven torque
torque and reduced vibration. and large vibration.
2. Engine design It uses ports and hence engine design is It uses valves, therefore, mechanism
simple. involved is complex.
3. Mechanical The working cycle completes in one Working cycle completes in two revolution,
efficiency revolution and hence it has high mechanical hence, it has more friction, thus less
efficiency. mechanical efficiency.
4. Scavenging The burnt gases are not completely driven It has separate stroke for explusion of burnt
out. It results in dilution of fresh charge. gases, thus ideally no dilution of fresh
charge.
Contd.
792 Thermal Engineering

Contd.

5. Thermal fficiency
e Poor thermal efficiency due to poor Very good thermal efficiency.
scavenging and escaping of charge with
exhaust gases.
6. Cost Less cost due to less parts in engine. More cost due to large number of parts.
7. Maintenance Cheaper and simple. Costlier and slightly complex.
8. Weight Lighter engine body. Heavier engine body.

24.7 ADVANTAGES AND (viii) Actually, the gas dilution in a two-stroke


cycle engine is comparatively less than the
four-stroke cycle engine.
(ix) Engine cost is also less than the four-stroke
The two-stroke cycle engines have cycle engine.
the following advantages over four-stroke cycle
engine: The two-stroke cycle engines
have the following disadvantages over four-stroke
(i) The two-stroke engines are simple in design, cycle engines:
manufacturing and operation. Hence they are
used in small power engines such as scooters, (i) The effective compression in a two-stroke
mopeds, motor cycles and autorickshas, etc. cycle engine is lower for the same stroke.
(ii) The two-stroke cycle engine develops net (ii) Due to less effective cooling arrangement in
work as positive because it works above two-stroke cycle engines, some amount of
atmospheric pressure. lubrication oil burns during the combustion
(iii) The turning moment of the two-stroke cycle of charge, thus more lubrication oil is
engine is more uniform, thus a smaller consumed.
flywheel is required. (iii) A two-stroke cycle engine is more noisy
(iv) There are less frictional losses due to absence because of sudden release of exhaust gases
of valves, thus higher mechanical efficiency in the port.
is obtained in two-stroke cycle engines. (iv) In a two-stroke cycle engine, the scavenging
(v) All the operations in two-stroke cycle engines (gas exhange process) is poor. The fresh
complete in two strokes, i.e., one revolution charge pushes the burnt gases out. In this
of the crank shaft, thus theoretically, a two- process, some fresh charge always flows
stroke cycle engine develops twice the power with the exhaust gases.
of the four-stroke cycle engine for the same (v) More wear and tear take place in a two-
size and same speed of the engine. stroke cycle engine.
(vi) The two-stroke cycle engine is much lighter (vi) Thermal efficiency of a two-stroke cycle en-
and more compact for same power output gine is less due charge loss during scaveng-
than a four-stroke cycle engine. ing.
(vii) Due to less parts in a two-stroke cycle
engine, the maintenance required is very
less. Further, the maintenance cost is very The air–fuel ratio is defined as the ratio of mass of
low. air to the mass of fuel. The engines operating on
Internal Combustion Engines 793

different loads and speeds require different air–fuel requirement by a carburetted engine at constant
ratios. The fuel and air are mixed to form the three speed and full throttle opening.
types of mixtures. A petrol engine requires a rich mixture for
(i) Chemically correct mixture (stoichiometric starting and idling (no load condition). When an
mixture) engine operates at part loads of up to 75% of the
designed load, the mixture required is slightly
(ii) Rich mixture
weak. But on acceleration and maximum load
(iii) Lean mixture
conditions, the engines requires a rich mixture
Chemically correct or stoichiometric air– (such as A/F ratio of 13).
fuel mixture is one, which has just sufficient air In a CI engine, irrespective of load condition at
for complete combustion of fuel. The mass of a given speed, the engine sucks almost a constant
air required for complete combustion of 1 kg quantity of air. According to load condition, the
of particular fuel (A/F ratio) is computed from a quantity of fuel is injected into the cylinder, thus
chemical equation. This computed value for most A/F ratio varies from cycle to cycle. The overall A/F
of the hydrocarbons is usually approximated to be ratio that is used in a CI engine ranges from 80 : 1
15. at no load and 20 : 1 at full load. Actually, in a CI
A mixture which contains more air than the engine, the mixture is heterogeneous with different
stoichiometric requirement is called a lean mixture A/F ratios in different areas of the combustion
chamber. There may be some pockets where there
(example A/F ratio of 17 : 1, 19 : 1, etc.)
is only air or fuel, having rich or lean mixture. But
A mixture which contains less air than the there are always certain areas where the A/F ratio is
stoichiometric required air is called a rich mixture, within the combustible limit and combustion starts
example A/F ratio of 12 : 1, 10 : 1, etc. from such areas and spreads over the combustion
A petrol engine can operate on A/F ratios in chamber.
the range 10 : 1 to 20 : 1, but it does not perform For a CI engine, the amount of fuel required
satisfactorily at extreme ratios. The air–fuel ratio varies directly with load on the engine, thus the
used in the engine has a considerable influence on curve of bsfc remains almost horizontal with brake
the engine performance. The mixture correspond- power as shown in Fig. 24.18. Figure 24.19 shows
ing to maximum power is called the best power the gross fuel consumption against brake power.
mixture and its A/F ratio is approximately 13 : This line in the graph is called Willan’s line and it
1. The mixture corresponding to minimum fuel indicates that the fuel consumption of a CI engine
consumption per kWh of power is called the best increases as load increases.
economy mixture and its A/F ratio is approximately
The fuel system provides the correct
16 : 1. Figure 24.17 shows a typical A/F mixture
air–fuel mixture in the cylinder for efficient

Fig. 24.17
Fig. 24.18
794 Thermal Engineering

(ii) the vapourisation characteristics of fuel


(iii) the temperature of the incoming air, and
(iv) the carburetter design.
Fuel consumption

The carburetters are used on most SI engines for


preparation of combustible air–fuel mixture as a
charge. Figure 24.21 shows a simple carburetter,
which provides an air–fuel mixture for normal range
at a single speed. It consists of a float chamber, fuel
fp bp
discharge nozzle and metering orifice, a venturi, a
Fig. 24.19 throttle valve and a chock valve.
The basic carburetter as shown in
burning. There are basically two methods available;
Fig. 24.21 is built around a hollow tube called a
one is called carburation and is particularly used
throat. The downward motion of the piston creates
for petrol engines, and the other is fuel injection
a partial vacuum inside the cylinder that draws air
used for Diesel engines. There are various designs
into the carburetter’s throat and past a nozzle that
of each system, but only the basic principle will be
sprays fuel. The mixture of air and fuel produced
discussed in the chapter.
inside the carburetter is delivered to the cylinder for
combustion.

A throttle valve at the base of the


The process of preparation of combustible mixture carburetter controls the amount of charge sucked
by mixing the proper amount of fuel with air through the engine by the partial vacuum in the
before admission to the engine cylinder is called cylinder. The driver opens the throttle valve by
carburation. A device which atomises the fuel and pressing down the accelerator. When throttle valve
mixes it with air and prepares the charge for Otto opens wider, more air flows through the carburetter,
cycle engine is called carburetter. delivering larger amounts of fuel to the engine.
When the piston descends in the cylinder, it The driver can regulate or close the opening of
creates vacuum, the charge is drawn through the the throttle valve by decreasing pressure on the
carburetter into the cylinder through intake valve accelerator.
and induction manifold.
The venturi is a gradually decreasing
cross-sectional hollow tube. The venturi is the
narrowing of the carburettor’s throat. Air rushing
The factors affecting the carburation process are through the narrow part speeds up. At the same
(i) the engine speed time, air pressure against the sides of the passage-
Carburetter way decreases, creating a partial vacuum inside the
Inlet valve Exhaust
Fuel Intake manifold
valve throat. This partial vacuum draws fuel through the
0.5 bar nozzle and into the air.
Air
1 atm
0.3 bar Cylinder
The fuel that enters the carburetter
is stored in a reservoir called a float chamber or float
bowl. A float and needle valve system maintains
Fig. 24.20 a constant level of gasolene in the float chamber.
Internal Combustion Engines 795

Fig. 24.21

The float floats on the gasolene surface and closes The choke is a device
the needle valve. If the level of fuel in the float that can partially block air from entering into the
chamber falls below the designed level, the float carburetter. If the throttle valve is open and the
goes down, thereby opening the fuel supply valve, choke valve is closed, the vacuum from the engine
and fuel enters the float chamber. As fuel reaches is strong enough inside the carburetter to draw more
the designed level, the needle closes the fuel fuel from all the nozzles. This added fuel produces
supply valve. The fuel level in the float chamber is a rich air–fuel mixture to help a cold engine get
maintained slightly below the tip of the discharge started. Once the engine warms–up, the choke is
jet of the nozzle in order to avoid overflow through shut off.
the jet, when the carburetter is not in operation.
Air–fuel Ratio A carburetter can be adjusted to
Idle and Transfer Ports In addition to the main mix larger or smaller amounts of air with the fuel.
nozzle in the venturi portion of the carburetter, two An idling engine at normal operating temperature
other nozzles, or ports, deliver fuel to the engine. requires an air-to-fuel ratio of about 15 : 1 (by
The idle port is located below the venturi and allows weight) to completely burn the fuel. Raising or
the engine to get fuel when air flow through the lowering the air flow makes the mixture either
carburetter is minimum, such as when the engine lean (containing less fuel) or rich (containing more
is idling at a low speed. Fuel from the idle port is fuel). A lean mixture produces a cleaner, hotter
drawn into the cylinder by the engine vacuum. The combustion for normal speeds, but not enough fuel
idle port supplies enough fuel to keep the engine for starting the engine efficiently or allowing it to
running at slow speeds. Fuel from the main nozzle produce more power. A rich mixture burns easily
is necessary to run the engine at normal operating in the engine but produces more pollutants as by-
speeds. products.
796 Thermal Engineering

The carburetter is adjusted to provide a rich 5. It gives a proper spray pattern to the fuel
mixture for cold engine starts because the rich droplets in order to mix it into air in a short
mixture burns easier and longer. As the engine period.
warms up, the carburetter alters the air–fuel ratio 6. It supplies equal quantity of fuel to all
for a leaner mixture. cylinders in case of a multicylinder engine.

A simple carburetter discussed above suffers from Systems


the following drawbacks. In CI engines, two methods of fuel injection are
1. It can only provide required A/F ratio at one used:
throttle position at constant speed. As speed (a) Air injection system, and
or throttle position changes, the mixture is (b) Solid injection system.
either richer or leaner.
(a) This system is also
2. At a wider open throttle, the air flow rate
called indirect injection (IDI) system. In this
increases at the venturi throat, the air density
method, the fuel is injected into the cylinder by
and the pressure of air decrease, while the
means of compressed air. The air is compressed in
fuel density remains constant. Thus, a
a compressor to a pressure higher than that at the
simple carburetter provides a progressively
end of the compression stroke. Then air and fuel
rich mixture with increasing throttle opening
are injected through the fuel nozzle into the engine
or speed of engine.
cylinder. The advantages and disadvantages of air
3. It fails to supply a richer mixture at the time
injection systems are given below.
of engine start.
4. It is not able to supply a rich mixture during
idling. 1. It provides better atomisation and distribu-
5. It fails to supply a rich mixture during tion of fuel.
acceleration and overload. 2. Inferior-quality fuel can also burn efficiently.
3. The charge burns completely and the engine
gives better thermal efficiency.
The fuel-injection system is the most important 4. The chance of choking of the fuel valve is
component in the working of a CI engines. The negligible due to compressed air supply.
engine performance, i.e., power output, fuel
economy, etc., depend on the effectiveness of a fuel The air-injection system is little used nowadays due
injection system. A typical fuel injection system to the following reasons:
functions in the following ways.
1. It requires multistage compression for high
1. It measures the correct quantity of fuel to be pressure of air. The large number of parts,
injected per cycle. the intercoolers, etc., make the system heavy,
2. It injects fuel in the cylinder at the correct complicated and expensive.
time. 2. Separate mechanical linkage is required for
3. It controls the rate of fuel injection. operation of fuel valve and compressor.
4. It atomizes the fuel into very fine droplet- 3. Due to large number of moving parts, the
like spray. mechanical efficiency of the engine reduces.
Internal Combustion Engines 797

4. More space is required for the engine.


5. Charge burns in the combustion chamber
very near to the injection nozzle, which may
lead to overheating and burning of the valve
and its seat.
(b) It is also called airless
injection system or direct injection (DI) system. In
this system, the liquid fuel is directly injected into
the combustion chamber without use of compressed
air. It is also called mechanical injection.
The fuel-injection systems for Diesel engines
employ a high-pressure fuel pump, which increases
1. It is compact and simple in construction. the pressure of the fuel to about 120 to 200 bar and
2. It does not require compressed air. this fuel is injected through the nozzle(s) into the
3. It has better control on the quantity of fuel to hot air present into the combustion chamber at the
be injected. end of the compression stroke.

1. It requires very high accuracy in the fuel The basic principle of a fuel-injection system can be
injector and fuel pump. understood with the help of Fig. 24.23. It consists
2. With this system, inferior quality of fuel of a spring loaded, plunger-type pump. The plunger
cannot be injected. is activated through a push rod from the cam shaft.
3. The prepared charge is more heterogenous. When the follower on the push rod is at the
minimum lift position of the cam, the spring forces
the plunger for its lowest position. Thus a suction is
created in the barrel and the fuel from the main tank
The fuel-injection system consists of mainly the flows into the barrel through the fuel filter. When
following components. the cam rotates and reaches its maximum lift, the

(i) Fuel tank


(ii) Fuel feed pump to supply fuel from the fuel
tank to the injector
(iii) Fuel filter to separate dust and dirt in the fuel
(iv) Fuel-injection pump to meter and pressurise
the fuel for injection
(v) Governor to ensure the correct quantity of
fuel according to load and speed
(vi) Fuel piping and injectors to take the fuel from
the pump and distribute in the combustion
chamber by atomizing it into fine droplets
A typical arrangement of various components
for a solid fuel-injection system used in Diesel
engines is shown in Fig. 24.22. Fig. 24.23
798 Thermal Engineering

plunger is lifted upwards, the inlet valve closes and 2. It measures the correct quantity of fuel at the
the fuel is forced through the delivery valve. beginning of the plunger stroke and spilling
When the fuel-operating pressure is reached, the back the excess fuel.
fuel from the injector is injected into the cylinder. 3. The axial distance travelled by the plunger in
The spring pressure above the valve rod in the each stroke is the same. The angular rotation
injector is used to set the fuel injection pressure. of the plunger by rack decides the length of
effective stroke for fuel injection.
4. Using the plunger stroke, the rack brings the
Nowadays, the ‘jerk pump’ fuel-injection system end of the fuel delivery by suddenly spilling
is universally used over the whole range of CI off the fuel from the cylinder.
engines. The jerk pump as shown in Fig. 24.24, The upper part of the helical groove in the
is a precision equipment and consists of plunger plunger controls the uncovering of the spill port.
in a barrel, a very close fit. A cam gives vertical The timing of the opening of spill port by the helix
movement to the plunger, while a rack controls is thus decided by the angular movement of the
its angular movement in the barrel. The plunger is plunger. The plunger is rotated by the rack, which
provided with a helical groove and the barrel has a is moved in or out by the governor. By changing
supply port, spill port and a spring-loaded delivery the angular position of the helical groove in the
valve. plunger, the length of the stroke, during which fuel
This system functions in the following way: is delivered, can be varied and thereby the quantity
1. It uses a variable stroke of the plunger. of fuel to be delivered to the cylinder is also varied
accordingly.
Internal Combustion Engines 799

The fuel at high pressure passes to the spring Effective stroke O Effective stroke
loaded injector, where the needle is set to lift at a commences ends
Quantity of
predetermined pressure in the delivery line. flue injected
Effective
stroke
Operation of The operation of a fuel A
S
F S
pump as plunger undergoes a stroke is illustrated
with the help of Fig. 24.25.

( ) ( ) Plunger Rack D

1. The plunger is at its down stroke. Plunger

2. The fuel inlet port F is uncovered and the (c) (d)


fuel enters the barrel above the plunger
and into the recesses in the plunger,
Fig. 24.25(a). 2. The trapped fuel in the barrel is now forced
3. As the plunger rises up, it closes the port F to pass the check valve through the orifice O
4. Fuel flows out through the spill port S, to the injector as shown in Fig. 24.25(c).
because recesses in the plunger align with 3. The injection of fuel starts.
the port S as shown in Fig. 24.25(b). 4. The injection continues during the upward
5. A spring force acts on the check valve and it stroke of the plunger till the helical recess on
does not open. the plunger uncovers the spill port S.
6. The effective stroke ends and no fuel is 5. The check valve closes the orifice, the fuel
delivered to the injector. injection stops. The effective stroke of the
plunger ends as shown in Fig. 24.25(d).
( ) ( ) 6. The remaining fuel escapes through the spill
1. During the upward stroke of the plunger, port.
the rack rotates the plunger in such a way Fuel Pump Operation for Partial Load
that the fuel inlet port and spill port, both are
closed. 1. During upward stroke, the plunger is
rotated by the rack to the position shown in
Fig. 24.25(e).
O No fuel injected 2. The trapped fuel is forced to pass the check
valve through the orifice O.

F S
F
S

S
F

Helical
groove Rack D Plunger

(a) (b)
Fig.24.25
800 Thermal Engineering

3. The fuel injection starts.


4. But for partial load on the engine, the spill
port S is uncovered sooner as shown in Fig.
24.25(f).
5. The effective stroke is shortened.
6. The quantity of fuel injected is small.
It is to remember that the axial distance travelled
by the plunger in each stroke is the same. The
angular rotation of the plunger by the rack decides
the length of the effective stoke.

The fuel pump in its effective stroke supplies the


fuel to the fuel injector. The fuel injector delivers the
fuel under pressure into the combustion chamber.
The fuel injector serves to fulfil the following tasks:
1. It atomizes the fuel into fine droplets.
2. It distributes the fuel uniformly by proper
spray pattern. Fig. 24.26
3. It prevents the injection of fuel on the
cylinder walls and piston top.
4. It controls the start and stop of fuel
instantaneorusly. Combustion in SI Engines
A cross-sectional view of a typical fuel injector The homogeneous mixture of fuel and air prepared
is shown in Fig. 24.26. The injector assembly in carburetter as charge is inducted in the spark
consists of ignition engine. This charge is compressed in the
(i) a needle valve cylinder during compression stroke. At the end of
(ii) compression spring the compression stroke, the combustion is initiated
by a high intensity spark produced by an electric
(iii) a nozzle
discharge. The typical stages of combustion are
(iv) adjusting screw with lock nut
shown in Fig. 24.27.
(v) an injector body
The compression spring exerts the force on 30
I II III
I Ignition lag C
the nozzle valve through the spindle to close it. II Propagation of flame
When the fuel is supplied under high pressure by III Afterburning
Pressure (bar)

20
a fuel pump, as it overcomes the spring force, the
Spark

nozzle valve lifts and the fuel is sprayed into the D


combustion chamber in finely atomized particles. 10
B
As the fuel pressure falls, the nozzle valve is pushed
A
on its seat by a spring force. The amount of fuel TDC
Moto
ring
injected is regulated by the duration of open period 0
100 80 60 40 20 8 0 20 40 60 80
of the nozzle valve. The pressure of fuel injection Crank angle (deg)
can be adjusted by spring tension by means of an
Fig. 24.27
adjusting screw.
Internal Combustion Engines 801

Pressure
First S It is referred as the preparation B
phase (ignition lag) in which fuel elements become
ready to react chemically with the oxygen present Ignition
Po
A D we
ion
in compressed air. The chemical process depends press r
Com
on temperature and pressure and nature of charge. BDC TDC BDC
Time
The growth and development of the propagating B¢

nucleus of flame takes place in this phase. (a) Normal combustion

Pressure
Second S The second stage occurs B
C
when the piston is at the top dead centre and the Ignition
Po
flame is propagated at a faster rate throughout the A D we
ion r
combustion chamber and reaches to the farthest end press
Com
BDC TDC BDC
of the cylinder. The rate of heat release in this stage B¢ C¢ Time
depends on turbulence intensity and reaction rate. (b) Combustion with detonation
The rate of pressure rise is proportional to the rate
Fig. 24.28
of heat release. The initial gas force is exerted on
the piston for its power stroke.
shows normal combustion when the flame travels
Third S This stage starts from the instant from the spark plug to the farthest end of the com-
at which maximum pressure is reached in the bustion chamber and pressure wave travels in one
cylinder. This stage occurs during earlier part of the direction only.
expansion stroke and the flame velocity decreases,
Figure 24.28(b) shows the combustion with
and the rate of combustion becomes low. There is
detonation. The advancing flame front compresses
no pressure rise during this stage.
the unburned charge BB’D in the end zone of the
Detonation in SI Engine combustion chamber, thus raising its pressure and
temperature. The temperature of this charge is also
The engine knock occurs when some of the unburnt
increased due to heat transfer from the burning
gases ahead of the flame in SI engine are ignited
charge. If the temperature of the end charge reaches
spontaneously. The unburnt gas ahead of the
flame is compressed as flame propagates through the self-ignition temperature of the fuel, and remains
the mixture, and the temperature and pressure of for some duration, the auto-ignition of charge takes
the unburnt mixture rise. If the temperature of place leading to knocking combustion.
the unburnt mixture at some instant exceeds the During auto-ignition, another pressure wave
self-ignition temperature of fuel remaining in starts traveling in the opposite direction to the
the mixture, the auto-ignition of unburnt mixture main pressure wave. When the two pressure waves
occurs. This phenomenon is called detonation, or collide, a severe pressure pulse is generated. The
knocking. gas in the combustion chamber is subjected to
Detonation in the combustion chamber generates vibration along the pressure pulse until the pressure
a shock wave which traverses from the end gas pulse is subsidized to an equilibrium state. The
region and an expansion wave which traverses into gas vibration can force the combustion chamber
the end gas region. The two waves collide at the to vibrate. An objectionable audible sound can be
boundary of the combustion chamber and interact heard from the engine.
to produce high-amplitude severe pressure pulses. Detonation during combustion can cause total
The phenomenon of detonation may be illus- engine failure. The gas vibration can scrub the
trated with the help of Fig. 24.28. Figure 24.28(a) chamber walls causing increased heat loss.
802 Thermal Engineering

Combustion in CI Engines
80
In compression ignition engines, only air is sucked
and compressed to a high pressure and high 1 23 4
temperature. The fuel is injected into this highly
60 Compression
compressed air in the combustion chamber. The

Pressure (bar)
pressure
Start of
jet of fuel disintegrates into tiny droplets. These combustion
droplets evaporate in the presence of compresed Motoring
40 (non-firing)
air and a mixture of air and fuel vapour formed at Start of
injection
some locations. As soon as the temperature of this
0.001 s
mixture attains self-ignition temperature, the auto- 20 Injection
ignition takes place. Atmospheric
The combustion in CI engines can be considered
0
to take place in four stages as shown in Fig. 24.29: 100 80 60 40 20 TDC 20 40 60 80 100
1. Ignition delay period, Time (degree of crankshaft rotation)

2. Period of uncontrolled combustion, Fig. 24.29


3. Period of controlled combustion, and
4. Period of after burning. 2. Period of Uncontrolled During
the ignition delay period, most of the fuel
1. This phase is also called admitted would have been evaporated and formed
preparatory phase. In this phase, the injected a combustible mixture with compressed air. By
fuel droplets air first heated and evaporated by that time the pre-flame reactions would have
absorbing heat from the surrounding compressed been also completed. The auto-ignition of the
air. It reduces the temperature of the thin layer of combustible mixture starts from different places of
surrounding air and some time elapses before the the combustion chamber simultaneously. It leads to
temperature of the mixture again reaches the self uncontrolled combustion and rapid pressure rise.
ignition temperature.
The pressure rise during uncontrolled combus-
The time period required to start actual burning
tion depends on the length of the delay period.
of mixture after injection of the first fuel droplets
Longer the delay period, rapid the pressure rise,
into the combustion chamber is called ignition delay
since more fuel would have been accumulated in
period. The ignition delay period is the combination
the combustion chamber.
of physical and chemical delays.
The physical delay is the time lapse between the 3. Period of Controlled The uncon-
beginning of fuel injection and start of chemical trolled combustion is followed by controlled com-
reaction. During this period, fuel jet is atomized, bustion. The temperature and pressure in the second
evaporated, mixed with compressed air and raises stage are already quite high. Hence, the ignition
its self-ignition temperature. delay reduces and fuel injected in the combustion
The chemical delay is the time lapse for chamber burns faster as it gets oxygen. During this
development of inflammation after the beginning of stage, the pressure rise can be controlled by con-
reaction. The chemical delay depends on temperature trolling the fuel-injection rate.
of compressed air. It is analogous to ignition lag in
an SI engine. Combustion does not
The ignition delay period influences the engine come to an end with the end of the fuel-injection
performance, combustion rate, exhaust quality and process. The burning of unburnt and partially
engine knock. burnt fuel particles left in the combustion chamber
Internal Combustion Engines 803

continue as they come in contact of oxygen. The valve is partially closed. It minimizes the charge
duration of this process is called the period of after supply to the engine, and thus speed reduces. So
burning. the engine speed can be adjusted to normal speed.

Quality Governing
The CI engines are governed by controlling the
The internal combustion engines are governed to quantity of fuel to be injected per cycle. Since the
operate them at constant speed at all load conditions. amount of air inducted per cycle into the cylinder
remains almost constant. By varying the quantity
Quantity Governing of fuel, the quality of charge is varied according to
The SI engines are quantity governed by the opening load condition. Therefore, the CI engines are called
and closing of the throttle valve, which regulates quality governed engines.
the mass-flow rate of charge into the cylinder. The governor in CI engines controls the move-
The position of the throttle valve is regulated by a ment of the rack which rotates the plunger in the
centrifugal-type speed governor. barrel. The rotation of the plunger governs the
helix opening with a spill port. As illustrated in
The working principle of a centrifugal governor
Fig. 24.30, the balls of the governor rotate at the
is illustrated in Fig. 24.30. Two arms are hinged at
speed of the engine shaft. When the load on the
the top of the spindle and two revolving balls are
engine decreases, the speed of the shaft suddenly
fitted on the other ends of these arms. The arms are
increases. Therefore, the rotating balls fly out,
connected to sleeve through two links. The speed causing the sleeve to raise through connected le-
of the crank shaft is transmitted to the spindle vers. The upward movement of the sleeve will draw
through a pair of bevel gears. The rotation of the the rack of fuel injection pump. The rack drives the
spindle of the governor causes the weights to move pinion and the plunger rotates axially, bringing the
centrally outward due to a centrifugal force. It helix in communication of spill port. Thus, the ef-
makes the sleeve move in an upward direction. This fective stroke of the plunger ends and reduces the
movement of the sleeve is transmitted to the throttle amount of fuel delivered to the engine.
valve through a connected lever. Thus the throttle When the load on the engine increases, the speed
of the governor decreases, and the sleeve position
comes down on the spindle. The lever controls the
rack for longer effective stroke of the plunger. Thus
more fuel is injected into the cylinder to meet the
load condition.

It is used for governing the speed of gas engines.


When the load on the gas engine suddenly
decreases, the speed of the engine increases to
a very high value. The governor sleeve rises, the
lever attached to the sleeve lifts the pecker block so
that the gas valve does not open. Hence only air is
supplied to the engine and no power is developed
inside cylinder. The engine speed comes back to
Fig. 24.30 normal and vice-versa.
804 Thermal Engineering

supply the voltage to the primary circuit of an


ignition system. It is charged by an engine-driven
The SI engines use lower compression ratio and dynamo.
self-ignition temperature of gasoline is higher.
For initiation of combustion of charge, an ignition It is an electrical switch which
system is required. The ignition system provides is used to allow and stop the flow of electrical
the high intensity spark needed to ignite the fuel. energy from source to primary circuit by turning
The ignition system produces, distributes, and it on or off.
regulates electric sparking that ignites fuel vapour The induction coil is an iron core,
in the combustion chambers. wrapped with primary and secondary windings of
Two broad categories differently sized wires. The primary winding has
of ignition systems are defined as 100 to 200 turns of relatively thick wire, whereas
the secondary winding has approximately 20,000
1. Battery ignition system turns of fine wire.
2. Magneto ignition system
The ballast resistance is an
The whole ignition system can be divided into
electrical resistance which is provided in series of
primary and secondary circuits. The primary circuit
primary winding to regulate the current in a primary
consists of a battery or magneto as a source of
circuit.
primary current, ignition switch, ballast resistance,
primary windings, contact breaker points and A capacitor is a device that
capacitor. One end of the capacitor is connected to temporarily stores electric charge. In the ignition
the contact breaker and the other is grounded (to the system, a capacitor helps to produce a sharply
engine its elf ). defined cut-off current when the breaker points
The secondary circuit consists of a secondary open. The capacitor also absorbs the surge of high-
winding (of large number turns of fine wires), voltage electricity as it moves from the coil to the
distributor and spark plugs. points. In doing so, the capacitor minimizes arcing
across the breaker points when they open, thus
increasing their service life.
An ignition system consists of the following The distributor serves two primary
components: functions. It routes high-voltage pulses to indi-
1. Source of electric current—battery or vidual cylinders in the correct sequence and with
magneto precise timing. It also houses a mechanical switch-
2. Ignition switch ing system involving breaker points—two contact
3. Ballast resistance points that open to interrupt the flow of electric cur-
rent.
4. Ignition coil
5. Contact breaker A wire conductor carries the pulses of current
6. Capacitor from the coil to the distributor, which routes them
through other wires to individual spark plugs. The
7. distributor
spark plugs deliver sparks that ignite the fuel.
8. Spark plug
The battery is the source of electrical
energy. It produces electrical energy from chemical The battery ignition system is shown in Fig. 24.31.
energy stored in it. A 16–12 V battery is used to The battery is used as a source of electrical energy.
Internal Combustion Engines 805

Fig. 24.31

When the ignition switch is turned on and the When the rotating cam opens the breaker points,
rotating cam makes the contact on breaker points, the flow of low-voltage current stops and the
a 12-V current flows from the battery in primary magnetic field collapses, inducing a high-voltage
winding through the ignition switch, and the surge of about 20,000 volts in the secondary
primary circuit is completed through the ground. winding. This high voltage current passes to the
The magnetic field is set up around the secondary distributor which connects the spark plug in correct
winding. sequence, depending upon the firing order of the
engine.
806 Thermal Engineering

Advantages of B Ignitions S system is popular on motorcycles, racing cars, and


1. It gives constant voltage irrespective of a variety of small engines.
speed of the engine. The advantages and disadvantages of magneto
2. It gives better spark at low speed and starting ignition systems are listed below.
of engine. Advantages
3. It is reliable and requires very less mainte- 1. The system is more reliable due to absence
nance, except battery and contact points. of battery and connecting cables.
4. The battery is charged by dynamo run by 2. This system generates secondary voltage
engine. according to engine speed, thus it is more
Disadvantages of B Ignition S suitable for medium and high speed engines.
1. The system is bulky and occupies more 3. The system is very light and compact, and
space. requires very less space.
2. At higher speed, the sparking voltage 4. The spark intensity improves as engine
decreases. speed increases.
3. If battery discharges, the engine cannot be 5. The system is much cheaper.
started as induction coil fails to operate. 6. It require very less and cheaper maintenance.
Disadvantages
Magneto Ignition System
1. The system is not useful at low speed and
A magneto is an electric generator that provides starting of the engine, because at low speed,
the current for ignition in systems that do not have it produces poor spark.
batteries. It is mounted to the engine. When engine 2. The powerful spark at high speed may cause
is started, it supplies kinetic energy to the magneto. burning of electrodes of the spark plug.
The magneto then converts this energy to electrical
energy. The schematic of magneto ignition system is
shown in Fig. 24.32. Ignition System and Magneto
The difference between battery and magneto Ignition System
ignition systems is in the source of electrical energy,
The battery ignition system uses a battery which
all other components being the same. A magneto
converts the stored chemical energy into electrical
Spark plug energy. A magneto is an electrical generator which
Distributor generates low voltage. The graphical comparison of
current produced in two systems with engine speed
Coil
is shown in Fig. 24.33.
It is evident from Fig. 24.33 that for a magneto
Primary winding ignition system, the starting current is poor. But
Secondary winding as speed increases, the current also increases. For
Rotating magnet a battery ignition system, the starting current is
Cam (Two poles shown)
excellent and it gives good spark at low speeds.
Contact Therefore, for low speed engines, the battery
breaker
ignition system is more suitable.
Ignition The diffrences between battery ignition systems
Condenser switch
and magneto ignition systems are given in the table
Fig. 24.32 as follows.
Internal Combustion Engines 807

hth, Diesel

The spark plug is a device which conducts a high-


6
Current at breaker (Amp5)

voltage current from ignition system and produces


5 spark to ignite the compressed charge into the
Magneto current
4 Battery current
combustion chamber. The spark plug as shown in
Fig. 24.34, mainly consists of
3

0 The central electrode is surrounded by a ceramic


2000 4000 6000
Spark per minute body. It has a threaded base, which is screwed at the
Fig. 24.33 top of the engine cylinder. Two electrodes on the
base of the spark plug project into the combustion

Sl. No. Aspect Battery Ignition system Magneto Ignition system


1. Source of nergy
e Conversion of chemical energy into Conversion of kinetic energy into electrical
electrical energy. energy.
2. Primary current Obtained from battery Obtained from magneto.
3. Starting Easy start, because battery gives good Poor spark at start and low speed of engine.
spark.
4. Low speed Good spark, thus no problem. Poor spark at low speed, thus engine runs
eratically.
5. High Speed Current for spark decreases as engine Current for spark increases with speed of
speed increases. engine, thus excellent spark at high speed.
6. Space required System is bulky and requires more space. System is light weight and compact, thus
less space is required.
7. Maintenance System requires more maintenance and it Very less and cheap maintenance, due to
is difficult to start engine when battery is absence of battery.
in discharge condition.
8. Applications In cars, light commercial vehicles. On two wheelers and racing cars, aircrafts,
etc.

Electronic Ignition chamber. The upper end of the central electrode


Electronic ignition systems use semiconductors and is connected to a high tension cable from the
other solid-state electronic components to switch distributor. The other electrode is located at a small
current flow on and off in the coil, eliminating the distance from the central electrode and is welded
need for breaker points. Automobile manufacturers to a steel shell of the plug and is grounded with the
began installing electronic ignition systems in the engine body. The gap between the two electrodes
1970s and 1980s in an effort to produce cleaner, is known as spark gap. The high-tension current
more efficient engines. passes through the central electrode and jumps over
the spark gap to grounded electrode and produces
808 Thermal Engineering

Terminal 6. It must offer maximum resistance to erosion,


burning away the spark points.
7. It must be heat resistant not to cause pre-
ignition of charge within cylinder, when it is
hot.
Insulator

The firing order is a sequence in which the firing


takes place in various cylinders of a multicylinder
engine. The firing order of a multicylinder engine is
arranged to obtain an uniform torque and power by
burning of charge, reduced vibrations and balancing
of the complete engine. The firing order depends on
Steel shell the number of cylinders in the engine and it may
Gasket differ from engine to engine. The firing order for
Insulator
some mullicylinder engines are given below:
Centre electrode Number of cylinders Firing order
Spark gap 2-cylinder engine 1, 2
Side electrode
(grounded) 3-cylinder engine 1, 3, 2
4-cylinder engine 1, 3, 4, 2 or 1, 2, 4, 3
Fig. 24.34
6-cylinder engine 1, 5, 3, 6, 2, 4 or 1, 4, 2,
the high intensity spark to ignite the fuel vapour in 6, 3, 5
the combustion chamber. The intensity of spark is 7-cylinder engine 1, 6, 2, 5, 8, 3, 7, 4 or 1,
greatly affected by the spark gap. 8, 7, 3, 6, 5, 4, 2
1. If the gap is too large, there may be possibility
of no spark due to insufficient high voltage.
2. If the gap is too small, there may be
possibility of very less spark, which is
insufficient to ignite the charge. The temperature of gases inside the engine cylinder,
due to combustion, reaches a quite high value (more
Requirements of a Good Spark Plug than 2500°C) during a cycle. If an engine is allowed
1. It must withstand high temperature attained to run without external cooling, the cylinder walls,
in the combustion chamber. cylinder liner, and piston will tend to attain the
2. It must produce a good spark under all average temperature of the hot gases to which they
working conditions. are exposed, which may be of the order of 1000
3. It must provide suitable insulation between to 1500°C. At such high temperatures, obviously,
two electrodes to prevent short circuiting. the metal will lose its strength and piston will
4. It must maintain proper gap between the two expand considerably and sieze the liner. Of course,
electrodes. theoretically, the thermal the efficiency of the engine
5. It must offer very high resistance to current will be better without cooling, but in actual practice,
leakage. this engine will not run longer. If the cylinder wall
temperatrue exceeds 265°C, the lubricating oil
Internal Combustion Engines 809

starts losing its viscosity, starts evaporating, thus excessive cooling. However, the excessive cooling
tending towards lubrication failure. Also, the high is not as harmful as overheating. But undercooling
temperature may cause excessive stresses in some is undesirable due to the following reasons.
parts, making them useless for further operations.
1. At low temperature, starting of the engine
Therefore, the internal combustion engines are
becomes difficult.
provided with cooling arrangement, which keeps
2. At low temperature, there is poor vaporisation
the engine temperature well within the safe working
of fuel, the combustion is not proper and the
temperature limits. A typical heat balance for a
engine runs eratically.
reciprocating internal combustion engine is shown
in Fig. 24.35. 3. At low temperature, the viscosity of lubri-
cating oil increases, it offers more frictional
resistance, and thus the output of the engine
Coolling = 30% decreases.
Radiation 4. Undercooling of the engine may change the
Heat supplied
Exhaust
unaccounted
}
= 32%
valve clearance and settings.
to Engine
5. Overcooling may reduce engine life due to
100% Useful
work } = 28%
corrosion and carbon deposits.
Friction = 10
In general, undercooling affects the economy
and life of the engine.
Fig. 24.35
There are two basic types of cooling systems
used in reciprocating engines to absorb and
dissipate the heat from hot cylinders.
The following are the harmful effects of engine 1. Air-cooling system, and
overheating: 2. Liquid-cooling system
1. High temperature reduces the strength of the
Air-Cooling System
piston and cylinder liner.
2. Overheating may lead to burning of lubri- In an air-cooling system, the outer surface of the
cants, thus there may be possibility of lu- cylinder and cylinder head is cooled by air flowing
brication failure and metal to metal contact, over them. To increase the heat transfer rate from
thus more heat generation due to friction. the surface, the metallic fins are cast on the cylinder
3. The overheating may cause uneven expan- and cylinder head. These fins increase the heat
sion of the piston and cylinder that may lead transfer area, and thereby heat transfer rate.
to piston seizure. Air cooling system is a very simple, reliable
4. The overheated cylinder or piston may lead and maintenance-free cooling system, with no
to pre-ignition of charge in SI engines. operating cost. It is very suitable for small engines
5. As temperature of the cylinder increases, the of automobiles.
volumetric efficiency decreases, and hence Applications
the power output of the engine is reduced. 1. It is used in small engines, i.e., motor cycles,
scooters, mopeds, aeroplanes, and combat
tanks, where speed of the vehicle gives a
The cooling system in an internal combustion good velocity to the air to cool the engine.
engine should provide adequate cooling but not 2. It is also used in small stationary engines
used for agriculture and industries.
810 Thermal Engineering

Advantages Radiator Thermostat

1. The design of the engine becomes simpler Air


with an air-cooling system. Fan

2. There is no cooling pipe radiator, fan pump Pump


Engine
and liquid cooling jacket, and hence the
engine has less weight. Fig. 24.36
3. In an air-cooled engine, the cylinder wall
temperature is relatively higher. Thus there suction side of the pump. The pump is mounted on
is more output from the engine. the engine and driven by a crank shaft with a fan
4. No danger of coolant leakage, coolant belt.
freezing, etc.
5. Installation, assembly, dismantling of the The fan is mounted in front of the radiator
engine are quick and simple. and is driven by a belt-pulley arrangement. The fan
6. The weight per kW of an air-cooled engine draws air through the spaces between the radiator
is less than that of a water-cooled engine. tube and fins, thus bringing down the temperature
7. The engine is almost maintenance free. of the water flowing in the tubes.

Disadvantages The water passages between the


1. Non-uniform cooling of engine. double walls of the cylinder and cylinder head are
called the water jacket. The water jacket is usually
2. The compression ratio of the engine is
cast as an integral part of the cylinder block and
limited due to high wall temperature.
head. The jacket covers the entire length of the
3. The volumetric efficiency of the engine is
stroke in order to avoid unequal thermal expansion
less than a water-cooled engine.
of cylinder and to prevent the breakdown of
4. It produces aerodynamic noise.
lubricating oil film by excessive temperature.
5. It can be used for only small-sized engines
due to its small capacity of heat dissipation. The radiator is basically a compact
heat exchanger. It is provided with a large surface
area for effective heat transfer. It consists of an
In a liquid-cooling system, water or other solutions upper header and lower header. Between these
flow through the water jacket around the cylinder headers, there is a core of the radiator, which
and cylinder head to absorb the heat. The hot consists of a large number of elliptical or circular
liquid coming out of the water jacket is cooled in brass tubes, pressed into a large number of brass
the radiator, where circulated air absorbs the heat fins. As hot water flows from top to down in the
from the radiator. The cold liquid coming out of radiator core, it transfers its heat to the radiator fins
the radiator is again pumped to the water jacket for from where the heat is picked up by circulating air.
absorbing heat. The lower cylinder temperature
The water-cooling system is explained with the results into poor performance and rough operation
help of a block diagram shown in Fig. 24.36. of the engine. In cold starting, if water is circulated
The water-cooling system mainly consists of a through the radiator, as the engine starts, the
radiator, fan water pump and thermostat. circulation of cold water in the water jacket brings
The pump maintains the water circulation down the cylinder temperature continuously and
through the water jacket around the engine. The the engine will take a long time to reach the safe
bottom side of the radiator is connected to the operating temperature.
Internal Combustion Engines 811

The thermostat is an instrument which auto- water cooled engine, while in a high-output
matically maintains the preset minimum tempera- engine, it is difficult to circulate the correct
ture and permits a quick warm up of the engine quantity of air in an air-cooled engine.
after starting. The thermostat is located in the up- Limitations
per hose connection and its opening and closing is
1. It requires pure water supply for proper
controlled by the water temperature in the cooling
functioning.
system. During the warm-up period, the thermo-
stat is closed and the water pump circulates the 2. The pump absorbs considerable power and it
water through the water jacket only as shown in reduces the output of the engine.
Fig. 24.36. When the preset operating temperature 3. In case of failure of cooling system, the
(70°C) is reached, the thermostatic valve opens and engine may get a serious demage.
allows the water to circulate through the radiator. 4. Cost of the system is considerably high.
The preset operating temperature range varies from 5. The system requires continuous maintenance
60°C to 76°C. of its parts.
In order to avail the advantages of
higher boiling temperature of water, the pressurised
water is used in the radiator. The pressure is built The lubrication is the supply of oil between two
up within the system due to continuous pumping. A surfaces having relative movement. The objects of
pressure cap is fitted with two valves, a safety valve lubrication are
loaded by compression spring, and a vacuum valve.
1. To minimize the friction between the parts
When the coolant is cold, both valves are closed.
having relative motion.
But as the engine warms up, the coolant temperature
rises and reaches the desired preset pressure. If 2. To reduce the wear and tear of moving parts.
pressure in the system exceeds the preset value, the 3. To cool the surfaces by carrying away the
safety valve opens and releases some of the gases heat generated due to friction.
and liquid to maintain the desired pressure. 4. To seal the space between piston rings and
cylinder liner.
5. To absorb the shocks between bearings and
other parts and consequently, reduce noise.
Advantages 6. To act as cleaning agent and remove dirt,
grit and any deposits that might be present
1. Efficient cooling as compared to air-cooling between the moving parts.
system.
2. Fuel consumption of liquid-cooled engines
is less than that of air-cooled engines.
A good lubricating oil should have the following
3. Liquid-cooled engines require less frontal
characteristics:
area.
4. For water-cooled engines, the cooling system The viscosity of the oil
can be located conveniently anywhere on the should not be changed with temperature rise.
automobile. Some vehicles have it at the
rear, while in air cooled engines, it is not It ensures the adherence to the bearings
possible. and spread over the surface. This property makes
the oil smooth and very important in boundary
5. Size does not pose a serious concern in the
lubrication.
812 Thermal Engineering

The oil must have high strength to (i) Mist lubrication system,
avoid metal to metal contact and seizure under (ii) Wet sump lubrication system, and
heavy loads. (iii) Dry sump lubrication system
The oil should not react
with surfaces and any deposit in the cylinder.
It is a very simple system of lubrication. In this
It should be low to allow the flow of system, the small quantity of lubricating oil
lubricant at low temperature to the oil pump. (usually 2 to 3%) is mixed with the fuel (preferably
The lubricating oil gasoline). The oil and fuel mixture is introduced
should not burn inside the cylinder, otherwise it through the carburetter. The gasoline is vaporised
will leave heavy deposit and poisonous exhaust. and oil in the form of mist enters the cylinder via the
Therefore, the flash point and fire point of the crank case. The droplets of oil strike the crank case,
lubricating oil must be high. lubricate the main and connecting rod bearings and
the rest of the oil lubricates the piston, piston rings
The oil should not have a and cylinder.
tendency to form deposits by reacting with air,
The system is preferred in two-stroke engines
water, fuel or the products of combustion.
where crank case lubrication is not required.
The oil should act as cleaning agent In a two-stroke engine, the charge is partially
inside the engine and should carry any deposits with compressed in a crank case, so it is not possible to
it. It should also have non-foaming characteristics, have the oil in crank case.
low cost, and be non-toxic. This system is simple, low cost and maintenance-
free because it does not require any oil pump, filter,
etc. However, it has certain serious disadvantages.
Lubrication Therefore, it is not popular among the lubrication
The main parts of the engine which need lubrication system. Its disadvantages are the following:
are 1. During combustion in the engine, some
(i) Main crank shaft bearings lubricating oil also burnt and it causes heavy
(ii) Big end or crank pin bearings exhaust and forms deposits on the piston
(iii) Gudgen pin bearings crown, exhaust port and exhaust system.
(iv) Piston rings and cylinder walls 2. Since the lubricating oil comes in contact of
acidic vapours produced during the combus-
(v) Timing gears
tion, it gets contaminated and may result in
(vi) Cam shaft and cam shaft bearings
the corrosion of the bearings surface.
(vii) Valve mechanism
3. When the vehicle is moving downhill, the
(viii) Valve guide, valve tappets, rocker arms throttle is almost closed, and the engine
(ix) Governor suffers lack of lubrication as supply of fuel
(x) Water pump bearing is less. It is a very serious drawback of this
system.
4. There is no control over the supply of
lubricating oil to the engine. In normal
The various lubrication systems used for lubricating
operating conditions, the two-stroke engines
the above parts of an internal combustion engine
are classified as
Internal Combustion Engines 813

are always over-oiled. Thus consumption of


oil is also more.
5. This system requires thorough mixing of oil
and fuel prior to admission into the engine.
It requires either separate mixing or use of
some additives.

In the wet-sump lubrication system, the bottom


of the crank case contains an oil pan or sump that
serves as oil supply, oil storage tank and oil cooler.
The oil dripping from the cylinders, bearings and
other parts, falls under gravity back into the sump,
from where it is picked up by pump and recirculated
Fig. 24.37
through the engine lubrication system. There are
three types of wet-sump lubrication system. big end of the connecting rod, crank pin, gudgen
(i) Splash system, pin, piston rings and cylinder.
(ii) splash and pressure system, and
(iii) full-pressure system.
(i) It is used on small,
stationary four-stroke engines. In this system, the
cap of the big end bearing on the connecting rod is
provided with a scoop which strikes and dips into
the oil-filled troughs at every revolution of the crank
shaft and oil is splashed all over the interior of crank
case into the piston and over the exposed portion of
the cylinder as shown in Fig. 24.37. A hole is drilled
through the connecting rod cap through which the
oil passes to the bearing surface. Oil pockets are
provided to catch the splashed oil over all the main
bearings and also the cam shaft bearing. From these
pockets, oil passes to the bearings through drilled Fig. 24.38
hole. The surplus oil dripping from the cylinder
flows back to the oil sump in the crank case. (iii) In this
system, the lubricating oil is supplied by a pump
(ii) This system is under pressure to all parts requiring lubrication as
a combination of splash and pressure system as shown in Fig. 24.39. The oil under the pressure is
shown in Fig. 24.38. In this system, the lubricating supplied to main bearings of the crank shaft and
oil is supplied under the pressure to main and cam camshaft. Holes drilled through the main crank
shaft bearings. The oil is also directed in the form shaft bearing journals, communicate oil to big
of spray from nozzle or splashed by a scoop or end bearing and small end bearings through a hole
dipper on the big end to lubricate bearings at the drilled in the connecting rod. A pressure gauge is
814 Thermal Engineering

2. Brake power
3. Frictional power
4. Fuel consumption
5. Air consumption
6. Brake thermal efficiency
7. Indicated thermal efficiency
8. Mecahnical Efficiency
9. Volumetric efficiency
10. Air–fuel ratio
The distribustion of energy produced by the
combustion of fuel is shown in Fig. 24.40.

Fig. 24.39

provided to confirm the circulation of oil to various


parts.
This system provides sufficient lubrication to
all parts and is favoured by most of the engine
manufacturers. Thus, it is used in most heavy-duty
and high-speed engines.

In this system, the oil supply is carried from an


external tank. The oil from the sump is pumped by
Fig. 24.40
means of a scavenging pump through filters to the
external storage tank. The oil from the storage tank
IP)
is pumped to the engine cylinder through an oil
cooler. The oil pressure may vary from 3 to 8 bar. It is defined as the rate of work done on the piston
The dry-sump lubrication system is generally by burning of charge inside the cylinder. It is
used for heavy-duty engines. evaluated from an indicated diagram obtained from
the engine. It is the gross power produced by the
engine. It is calculated as
IP = Indicated mean effective pressure ¥
Swept volume rate
An engine is selected for a particular application on p L An k
the basis of its power output and rated speed. Other = mi ( kW ) ...(24.1)
60
factors include capital cost and operational cost.
where k = Number of cylinders
Therefore, certain measurements and calculations
pmi = Indicated mean effective pressure, (kPa
are required to judge the performance of an engine.
or kN/m2)
These are
L = Stroke length, (m)
1. Indicated power
Internal Combustion Engines 815

A = (p /4) d 2, cross-sectional area of cylin- = N for two-stroke engine


der of bore, d, (m) N
= for a four-stroke engine
n = Number of working strokes per minute, 2
when engine has a speed of N rotations
per minute FP)
= N for two-stroke engines It is the part of the indicated power which is used
N to overcome the frictional effects within the engine.
= for four-stroke engines
2 The friction power also includes power required to
operate the fuel pump, lubrication pump, valves,
BP) etc. Therefore, it is given as the difference between
It is the net power available at the engine shaft the indicated power and brake power.
for external use. It is measured by the brake or FP = IP – BP ...(24.4)
dynamometer (refer rope brake dynamometer,
Fig. 20.18), which can be loaded to measure the
brake power of the engine. It is calculated as A calibrated burette can be used for fuel-
Brake power = brake load (F ) ¥ velocity of consumption measurement as shown in Fig. 24.41.
brake drum (2p R N/60) A quantity of fuel is taken from the fuel tank
2p N ( FR) 2p NT through a valve and then the valve is closed. The
or BP = = ( kW ) ...(24.2) fuel flows into the running engine only from
60, 000 60, 000
graduated burette. The time, Dt is recorded for
where, F = Braking force, (N)
a known volume of fuel-consumption. The fuel
R = Effective radius of the brake drum (m)
consumption rate is then given as
= 1/2 (Dia. of brake drum + Dia. of the
rope)
T = F R, the torque is the product of force
F and effective radius R of the brake
drum
N = Speed of the engine in rpm
Brake power can also be obtained in terms of
brake mean effective pressure ( pmb) as
BP =Brake mean effective pressure ¥
Fig. 24.41
Swept volume rate
p L An k Vfuel ¥ r fuel ¥ 3600
= mb (kW) ...(24.3) mf = (kg/h) ...(24.5)
60 Dt
where, k = Number of cylinders where, Vfuel = Volume of fuel in m3 used in time
pmb = Brake mean effective pressure, (kPa Dt,
or kN/m2) rfuel = Density of fuel, and
L = Stroke length, (m) Dt = Time in seconds.
A = (p/4) d 2, cross-sectional area of cylin-
der of bore, d, (m) sfc)
n =Number of working strokes per It is defined as the ratio of the mass of fuel
minute, when engine has a speed of N consumed per hour per unit power output (BP).
rotation per minute
816 Thermal Engineering

It is also designated as Bsfc (brake specific fuel where, Va = Volume flow rate of air, m3/s
consumption). It is a parameter which decides the do = Diameter of orifice, m
economical power production from an engine.
Cd = Coefficient of discharge of orifice
m f ( kg/h )
Bsfc or sfc = ( kg/kWh ) ...(24.6) g= 9.81 m/s2, acceleration due to gravity
BP ( kW ) r h
The specific fuel consumption in kg/kWh ha = Head of air = w w (m),
rair
based on the indicated power (IP) is called the hw = Water column in m and
Isfc (indicated specific fuel consumption) and is rw = density of water,
expressed as
m f ( kg/h ) rair = density of air.
Isfc = ( kg/kWh ) ...(24.7) The mass-flow rate of air through the orifice can
IP ( kW )
be calculated as
ma = rair ¥ Va ...(24.9)
Air-consumption rate of an engine can be effectively
calculated by means of an orifice meter installed in
an air box. It is the ratio between the mass of the air and mass
of the fuel supplied to the engine. It is expressed as
ma ( mass flow rate of air )
A/F = ...(24.10)
m f ( mass flow rate of fuel )
Theoretically, the correct (stoichiometric) air–
fuel ratio is 15. But the combustion of air–fuel
mixture can take place in A/F ratio ranges from
12 to 19 for petrol engines and 20 to 60 in Diesel
engines.

EFFICIENCIES OF IC ENGINES

hbth)
Fig. 24.42 The power output of an engine is obtained from the
combustion of charge. Thus the overall efficiency
An orifice is fitted to an air-tight air box. Inlet
of an engine is given by brake thermal efficiency,
manifold is connected to the air box through a
i.e.,
flexible pipe. A U-tube manometer is installed on
Brake power BP
the air box to measure the pressure depression in the hbth = = ...(24.11)
Energy supply rate m f ¥ CV
water column (hw ), when the engine sucks the air. A
rubber diaphragm is also installed to minimize the where, m f = mass flow rate of the fuel (kg/s)
pressure pulsation. CV = Calorific value of fuel, (kJ/kg)
The volume flow rate of air passing through
the orifice, in m3/s, can be calculated by using the hith)
relation The indicated thermal efficiency is defined as the
Va = Aorifice ¥ velocity of air ratio of the indicated power to the heat supply rate,
which can be expressed as i.e.,
IP
p hith = ...(24.12)
Va = do2 Cd 2 g ha ...(24.8) m f ¥ CV
4
Internal Combustion Engines 817

hmech) The volumetric efficiency can also be defined as


It is the ratio of the brake power and indicated the ratio of the volume of the charge inducted in the
power. cylinder, measured at NTP to the swept volume of
BP the cylinder.
hmech = ...(24.13) V
IP hvol = act ...(24.18)
It can also be expressed as Vs
Brake thermal efficiency Example 24.1 A rope-brake dynamometer was used
hmech = to measure the brake power of a single cylinder, four-
Indicated thermal efficiency
stroke cycle petrol engine. It was found that the torque
h
= bth ...(24.14) due to brake load was 175 Nm and the engine makes
hith 500 rpm. Determine the brake power developed by the
Brake mean effective pressure engine.
and hmech =
Indicated mean effective pressure
p Solution
= mb ...(24.15)
pmi Given A single-cylinder, four-stroke petrol engine with
a rope-brake dynamometer
k =1
T = 175 Nm N = 500
It is the ratio of actual thermal efficiency to air
standard efficiency of the engine. It is sometimes To find Brake power
referred as efficiency ratio. It is expressed as Analysis The brake power is given by
Brake thermal efficiency 2p N T 2p ¥ 500 ¥ 175
hRelative = ...(24.16) BP = = = 9.16 kW
Air standard efficiency 60, 000 60, 000
Relative efficiency for most of the engines varies A four-cylinder, four-stroke petrol
from 75 to 95% with air standard efficiency. engine develops indicated power of 14.7 kW at 1000 rpm.
The mean effective pressure is 5.5 bar. Calculate the bore
hvol) and stroke of the engine, if the stroke is 1.5 times the bore.
It is defined as the ratio of the mass of the actual
Solution
charge inducted into the cylinder to the mass of the
charge corresponding to the swept volume, or Given A four-cylinder, four-stroke cycle petrol engine

Actual mass flow rate of the charge N


hvol = k =4 n =
2
Density ¥ Swept volume per second
IP = 14.7 kW N = 1000 rpm
ma (kg/s) pm = 5.5 bar = 550 kPa
= ...(24.17)
Êp 2 ˆ n L = 1.5 d
ra Á d L˜
Ë4 ¯ 60 To find (i) Bore (ii) Stroke
where, ra = Density of inlet charge,
Analysis The power of an engine is given by
d = bore, L = stroke
L Ank
n = Number of effective suction strokes IP = pm
60
per cycle per minute
(1.5 d ) Ê p 2 ˆ 1000
n = N for a two-stroke engine 14.7 = 550 ¥ ¥Á d ˜ ¥ ¥4
60 Ë4 ¯ 2
N
= for a four-stroke engine d3 = 6.806 ¥ 10– 4 m 3
2
818 Thermal Engineering

Bore, d = 0.08796 m or 87.96 mm The brake specific fuel consumption


Stroke L = 1.5 d = 131.94 mm. m f (kg/h) 8.78
Bsfc = =
Example 24.3 A four-cylinder, two-stroke cycle BP (kW) 30
petrol engine develops 30 kW at 2500 rpm. The mean = 0.293 kg/kWh
effective pressure on each piston is 8 bar and mechanical
efficiency is 80%. Calculate the diameter and stroke Example 24.4 The following results were obtained
of each cylinder, if the stroke to bore ratio is 1.5. Also from a test on a single-cylinder, four-stroke Diesel engine.
calculate the fuel consumption of the engine, if the brake Diameter of the cylinder is 30 cm, stroke of the piston is
thermal efficiency is 28%. The calorific value of the fuel 45 cm, indicated mean effective pressure is 540 kPa and
is 43900 kJ/kg. engine speed is 240 rpm. Calculate the indicated power
of the engine.
Solution
Given A four-cylinder, two-stroke cycle petrol engine Solution
k =4 BP = 30 kW Given A single-cylinder, four-stroke Diesel engine
N = 2500 rpm n =N d = 30 cm = 0.3 m L = 45 cm = 0.45 m
hmech = 0.8 L = 1.5 d pmi = 540 kPa N = 240 rpm
hbth = 0.28 CV = 43900 kJ/kg k =1
pm = 8 bar = 800 kPa N
n = = 120 working stroke per minute
To find 2
(i) Bore of cylinder,
To find Indicated power
(ii) Stroke of pistion, and
(iii) Fuel consumption rate (Bsfc). Analysis The cross-sectional area of the cylinder

Analysis The mechanical efficiency is given as p 2 p


A = d = ¥ (0.3) 2 = 0.07068 m2
4 4
BP
hmech =
IP Indicated power (IP)
30 kW
\ IP = = 37.5 kW pmi L A n k
0.8 IP =
60
(i) The indicated power is expressed as (540 kPa ) ¥ (0.45 m) ¥ (0.07068 m 2 ) ¥ 120 ¥ 1
p L A nk =
IP = m 60
60 = 34.353 kW
Êp ˆ
800 ¥ (1.5d ) ¥ Á d 2 ˜ ¥ 2500 ¥ 4
Ë4 ¯ Example 24.5 In a test of a single-cylinder, four-
or 37.5 =
60 stroke Diesel engine, the following data were recorded.
or d3 = 0.000238 m3 Indicated mean effective pressure = 755 kPa
Bore d = 0.062 m or 62 mm cylinder diameter = 10 cm
(ii) Stroke L = 1.5d = 93 mm piston stroke = 15 cm
(iii) The brake thermal efficiency is given by engine speed = 480 rpm
BP brake wheel diameter = 62.5 cm
hbth =
m f CV net load on the brake wheel = 170 N
30 kW Calculate
or mf =
0.28 ¥ ( 43900 kJ/kg) (a) indicated power,
= 0.00244 kg/s or 8.78 kg/h (b) brake power, and
(c) the mechanical efficiency of the engine.
Internal Combustion Engines 819

Solution Solution
Given A single-cylinder, four-stroke Diesel engine Given A petrol engine
pmi = 755 kPa N = 480 rpm IP = 30 kW BP = 26 kW
N N = 1000 rpm Bsfc = 0.35 kg/kWh
n = = 240 working stroke/min.
2 CV = 43900 kJ/kg
k =1 d = 10 cm = 0.1 m
To find
L = 15 cm = 0.15 m Dbrake = 62.5 cm
(i) Indicated thermal efficiency, hith
or Rbrake = 31.25 cm = 0.3125 m
(ii) Brake thermal efficiency, hbth
F = 170 N
(iii) Mechanical efficiency, hmech
To find
Analysis Fuel consumption rate,
(i) Indicated power,
m f = Bsfc ¥ BP
(ii) brake power, and
= 0.35 ¥ 26 = 9.1 kg/h = 2.53 ¥ 10–3 kg/s
(iii) mechanical efficiency
(i) Indicated thermal efficiency (hith),
Analysis The cross-sectional area of the cylinder
IP
p hith =
A = ¥ (0.1 m) = 7.854 ¥ 10 -3 m 2 m f ¥ CV
4
30
(i) Indicated power (IP) =
2.53 ¥ 10 -3 ¥ 43900
p L Ank
IP = mi = 0.27 = 27%.
60
755 ¥ 0.15 ¥ 7.854 ¥ 10 -3 ¥ 240 ¥ 1 (ii) Brake thermal efficiency (hbth),
=
60 BP
= 3.557 kW hbth =
m f CV
(ii) Brake power, 26
2p N T 2p N ( F Rbrake ) =
BP = = 2.53 ¥ 10 -3 ¥ 43900
60, 000 60, 000 = 0.234 = 23.4%.
2p ¥ ( 480 rpm) ¥ (170 N ¥ 0.3125 m) (iii) Mechanical efficiency (hmech),
=
60, 000 BP 26
= 2.67 kW hmech = = = 0.867 = 86.7%.
IP 30
(iii) Mechanical efficiency (hmech)
BP 2.67 Example 24.7 The mechanical efficiency of a single-
hmech = = = 0.750 = 75%. cylinder, four-stroke engine is 80%. The friction power is
IP 3.557
estimated to be 26 kW. Calculate the indicated power and
Example 24.6 The following results refer to a test on brake power developed by the engine.
a petrol engine:
Indicated power = 30 kW Solution
brake power = 26 kW Given A single-cylinder, four-stroke engine
Engine speed = 1000 rpm hmech = 0.80 FP = 26 kW
Bsfc = 0.35 kg/kWh
To find
CV of fuel used = 43900 kJ/kg
(i) Indicated power, and
Calculate
(ii) Brake power.
(a) Indicated thermal efficiency,
(b) Brake thermal efficiency, and Analysis The mechanical efficiency of an engine is
(c) Mechanical efficiency. given by
820 Thermal Engineering

BP To find
hmech = or BP = 0.8 IP
IP (i) Mechanical efficiency, hmech
The friction power of the engine is given as (ii) Air consumption, and
FP = IP – BP (iii) Brake thermal efficiency, hbth
26 kW = IP – 0.8 IP = 0.2 IP Analysis
26
or IP = = 130 kW (i) Mechanical efficiency (hmech)
0.2
Indicated power,
Then BP = IP – FP = 130 – 26 = 104 kW.
IP = BP + FP = 420 + 76 = 496 kW
Example 24.8 A Diesel engine has a brake thermal BP 420
hmech = = = 0.8467
efficiency of 30%, if the calorific value of the fuel is IP 496
42000 kJ/kg. Calculate the brake specific fuel consump- = 84.67%.
tion. (ii) Fuel consumption rate,
A 22
Solution ma = m f ¥ = (195 kg/h) ¥
F 1
Given A Diesel engine with = 4290 kg/h or 71.5 kg/min
hbth = 0.3 (iii) The brake thermal efficiency of an engine is
CV = 42000 kJ/kg expressed as
BP
To find Brake specific fuel consumption. hbth =
m f ¥ CV
Analysis The brake thermal efficiency of an engine is 420 kW
=
expressed as Ê 195 ˆ
BP ÁË 3600 kg/s˜¯ ¥ ( 42000 kJ/kg)
hbth =
m f ¥ CV = 0.1846 or 18.46%
BP
or = hbth CV Example 24.10 An engine is used in a process which
mf
requires 100 kW of brake power with a mechanical
= 0.3 ¥ 42000 = 12600 kJ/kg
mf efficiency of 78%. The engine uses 1 kg of fuel per
or = 7.936 ¥ 10–5 kg/kJ minute. If a simple modification in design reduces the
BP
engine friction by 8 kW then what will be the percentage
mf
and Bsfc = ¥ 3600 = 0.287 kg/kWh saving in fuel consumption? Assume indicated thermal
BP efficiency remains same.
Example 24.9 A two-stroke, Diesel engine develops Solution
a brake power of 420 kW. The engine consumes
195 kg/h of fuel and air–fuel ratio is 22:1. Calorific value Given An engine with
of the fuel is 42000 kJ/kg. If 76 kW of power is required to BP = 100 kW hmech = 0.78
overcome the frictional losses, calculate m f1 = 1 kg/min. hith = constant
(a) Mechanical efficiency, Change in FP = 8 kW
(b) Air consumption,
To find Percentage saving in fuel consumtion.
(c) Brake thermal efficiency.
Analysis
Solution (i) Mechanical efficiency (hmech),
Given A two-stroke Diesel engine with BP
hmech =
BP = 420 kW FP = 76 kW IP
A/F = 22 : 1 CV = 42000 kJ/kg BP 100
or IP1 = = = 128.21 kW
m f = 195 kg/h hmech 0.78
Internal Combustion Engines 821

Friction power develops a brake power of 6 kW, while the air–fuel ratio
FP1 = IP – BP = 128.21 – 100 is 20 : 1. CV of fuel is 42550 kJ/kg and the ambient air
= 28.21 kW density is 1.18 kg/m3. Calculate
After modified design of engine, (a) The volumetric efficiency,
FP2 = 28.21 – 8 = 20.21 kW (b) Brake specific fuel consumption.
IP2 = BP + FP2 = 100 + 20.21 = 120.21
For same indicated thermal efficiency Solution
hith1 = hith2 Given A single-cylinder, four-stroke Diesel engine
IP1 IP2 with
or =
m f1 ¥ CV m f 2 ¥ CV k =1 d = 85 mm
N 1800
128.21 120.21 L = 110 mm n = = = 900
or = 2 2
1 ¥ CV m f 2 ¥ CV
ma = 0.56 kg/min A/F = 20 : 1 P = 6 kW
or m f 2 = 0.9376 kg/min CV = 42550 kJ/kg ra = 1.18 kg/m3
Percentage saving in fuel consumption
To find
m f1 - m f 2 1 - 0.9376
= ¥ 100 = ¥ 100 (i) The volumetric efficiency, and
m f1 1 (ii) Brake specific fuel consumption.
= 6.24%
Analysis
Calculate the brake mean effective (i) Volumetric efficiency
pressure of a four-cylinder, four -stroke Diesel engine Swept volume rate,
having a 100-mm bore and 120-mm stroke which p 2 p
develops a power of 42 kW at 1200 rpm. Vs = d L n k = ¥ (0.085) 2 ¥ 0.11 ¥ 900 ¥ 1
4 4
= 0.5617 m3/min
Solution
Mass of air,
Given A four-cylinder, four-stroke Diesel engine with ma = Vs ra = 0.5617 ¥ 1.18
k =4 d = 100 mm = 0.10 m
= 0.663 kg/min
N = 1200 rpm L = 120 mm = 0.12 m
Volumetric efficiency,
N
BP = 42 kW n = Actual mass of air
2 hvol =
To find The brake mean effective pressure. Mass correponds to swept volume
0.56
Analysis The brake mean effective pressure relates = ¥ 100 = 84.5%
brake power as 0.663
p L Ank (ii) Brake specific fuel consumption:
BP = mb
60 Actual mass of air 0.56
mf = =
where, pmb is brake mean effective pressure A/F ratio 20
Êpˆ = 0.028 kg/min or 1.68 kg/h
pmb ¥ 0.12 ¥ Á ˜ ¥ (0.1) 2 ¥ 1200 ¥ 4
Ë 4¯ m f ( kg/h ) 0.028
\ 42 kW = Bsfc = = = 0.28 kg/kWh
60 ¥ 2 BP ( kW) 6
or pmb = 1114.08 kPa = 11.14 bar
Example 24.13 Calculate the brake mean effective
Example 24.12 A single cylinder, 4-stroke Diesel pressure of a four-cylinder, two-stroke engine of 100 mm
engine running at 1800 rpm has an 85 mm bore and a bore, 125 mm stroke, when it develops a torque of
110-mm stroke. It takes 0.56 kg of air per minute and 490 Nm.
822 Thermal Engineering

Solution (iii) The mechanical efficiency is expressed as


BP hbth
Given A four-cylinder, two-stroke engine with hmech = =
IP hith
k =4 d = 100 mm
hbth 0.3
n =N L = 125 mm or hith = = = 0.375 = 37.5%
hmech 0.8
T = 490 Nm
Example 24.15 A four-cylinder, four-stroke petrol
To find The brake mean effective pressure of the engine
engine has a 10-cm bore, 15-cm stroke and uses a
Analysis The brake power is given by compression ratio of 6. The engine develops 25 kW
2p N T p L Ank indicated power at 2000 rpm. Find the mean indicated
BP = = mb pressure and air standard efficiency.
60, 000 60
Here n = N for two-stroke engine Also calculate the fuel consumption per hour, if the
2p T indicated thermal efficiency is 30%. Take the calorific
pmb = value of fuel as 42 MJ/kg.
1000 L A k
2p ¥ 490 Solution
=
Êpˆ
1000 ¥ 0.125 ¥ Á ˜ ¥ (0.1) 2 ¥ 4 Given A four-cylinder, four-stroke petrol engine
Ë 4¯
k =4 d = 10 cm = 0.1 m
= 784 kPa
L = 15 cm = 0.15 m r = 6
Thus, the break mean effective pressure is 7.84 bar.
IP = 25 kW N = 2000 rpm
Example 24.14 A single-cylinder, C1 engine with a N
n= hith = 0.3
brake thermal efficiency of 30% uses diesel oil having a 2
CV = 42 MJ/kg
calorific value of 42000 kJ/kg. If its mechanical efficiency
is 80%, calculate (a) Bsfc, (b) Isfc, and (c) hith. To find
(i) Air standard efficiency,
Solution
(ii) Indicated thermal efficiency, and
Given A single-cylinder CI engine with (iii) Fuel consumption per hour.
hbth = 0.3 CV = 42000 kJ/kg
Assumptions
hmech = 0.8
(i) The petrol engine works on air standard Otto
To find cycle.
(i) Bsfc (ii) Isfc (iii) hith (ii) The ratio of specific heats g = 1.4 for air.
Analysis Analysis
(i) The brake thermal efficiency relates brake power (i) Air standard efficiency of Otto cycle:
as 1
BP hOtto = 1 - g -1
hbth = r
m f CV
1
mf = 1 - 1.4 - 4 = 0.511 or 51.1%
1 1 1 (6 )
or = = = kg/kJ
BP hbth CV 0.3 ¥ 42000 12600 (ii) Indicated mean effective pressure
mf 1 pmi L An k
and Bsfc = ¥ 3600 = ¥ 3600 IP =
BP 12600 60
= 0.286 kg/kWh
pmi ¥ (0.15m) ¥ (p /4) ¥ (0.1m) 2 ¥ 2000 ¥ 4
Isfc 25 kW =
(ii) hmech = 60 ¥ 2
Bsfc or pmi = 318.3 kPa = 3.183 bar
Thus Isfc = 0.8 ¥ 0.286 = 0.229 kg/kWh
Internal Combustion Engines 823

(iii) Fuel consumption per hour = (735 kg/m3) ¥ (6.74 ¥ 10–3 m3/h)
IP = 4.954 kg/h = 1.376 ¥ 10–3 kg/s
We have hith =
m f ¥ CV The brake thermal efficiency
IP 25 kW BP
or mf = = hbth =
hith CV 0.3 ¥ 42 ¥ 103 kJ/kg m f ¥ CV
16.18
= 1.984 ¥ 10–3 kg/s = 7.14 kg/h = = 0.266
1.376 ¥ 10 -3 ¥ 44200
Example 24.16 A four-cylinder, four-stroke petrol = 26.6%
engine has a bore of 57 mm and a stroke of 90 mm. Its (iv) The brake-specific fuel consumption
rated speed is 2800 rpm, torque is 55.2 Nm. The fuel m f ( kg/h ) 4.954
consumption is 6.74 lit/h. The density of the petrol is Bsfc = =
BP ( kW ) 16.18
735 kg/m3 and petrol has a calorific value of 44200 kJ/kg.
Calculate BP, bmep, brake thermal efficiency and brake- = 0.306 kg/kWh
specific fuel consumption.
Example 24.17 The following data and results refer
Solution to a test on a four-cylinder, four-stroke car engine:
cylinder bore = 7.5 cm
Given A four-cylinder, four-stroke petrol engine with
piston stroke = 9 cm
d = 57 mm = 0.057 m L = 90 mm = 0.09 m
engine-to-rear-axle ratio = 39 : 8
T = 55.2 Nm N = 2800 rpm
wheel diameter with tyre fully inflated = 65 cm
N
k =4 n = The petrol consumption is 0.250 kg for a distance of
2
4 km, when the car was travelling at a speed of 60 kmph.
V f = 6.74 lit/h rf = 735 kg/m3 CV of fuel is 44200 kJ/kg.
CV = 44200 kJ/kg If the mean effective pressure is 5.625 bar, calculate the
To find indicated power and indicated thermal efficiency.
(i) BP,
Solution
(ii) Brake mep,
(iii) hbth , and Given A four-cylinder, four-stroke car engine with
(iv) Brake-specific fuel consumption. d = 7.5 cm = 0.075 m
L = 9 cm = 0.09 m
Analysis
T = 55.2 Nm
(i) The brake power of the engine can be obtained as
k =4
2p N T 2p ¥ 2800 ¥ 55.2 Dwheel = 65 cm = 0.65 m
BP = =
60, 000 60, 000 Distance = 4 km
= 16.18 kW Engine rear axle ratio = 39 : 8
(ii) The Bmep (brake mean effective pressure) is speed = 60 km/h = 16.667 m/s
given by mf = 0.250 kg
L Ank CV = 44200 kJ/kg
BP = pmb
60
Bmep = 5.625 bar = 562.5 kPa
p (0.057) 2 ¥ 2800 N
16.18 = pmb ¥ 0.09 ¥ ¥ ¥4 n =
4 60 ¥ 2 2
or pmb = 755.1 kPa = 7.55 bar To find
(iii) The mass flow rate of the petrol (i) Indicated power, and
m f = rf V f (ii) Indicated thermal efficiency.
824 Thermal Engineering

Analysis (c) the brake thermal efficiency, and


60 ¥ 1000 (d) brake-specific fuel consumption in kg/kWh.
Velocity of car = = 1000 m/min
60
= p Dwheel Ntyre Solution
Revolution of tyre, Given A single-cylinder, two-stroke cycle engine
1000
Ntyre = pmi = 550 kPa
p ¥ 0.65
T = 628 Nm
= 490 rotation per min.
N = 360 rpm
Using rear-to-axle ratio, the speed of car engine
n = N = 360 working stroke/min
490 ¥ 39
N = = 2388 rpm d = 21 cm = 0.21 m
8
Cross-sectional area of the cylinder L = 28 cm = 0.28 m
p 2 p m f = 8.16 kg/h
A = d = ¥ (0.075) 2
4 4 CV = 42700 kJ/kg
= 0.00441 m2 k =1
(i) Indicated power To find
L Ank
IP = pmi (i) Mechanical efficiency,
60
1 (ii) Indicated thermal efficiency,
0.09 ¥ 0.00441 ¥ 2388 ¥ ¥ 4
2 (iii) Brake thermal efficiency, and
= 562.5 ¥
60 (iv) Brake-specific fuel consumption.
= 17.776 kW
Analysis Cross-sectional area of the cylinder
(ii) Indicated thermal efficiency (hith),
p 2 p
Time for 4-km distance A = d = ¥ (0.21) 2 = 0.0346 m2
4 4
Distance 4 km
= = (i) Indicated power
Car speed 60
km/s L Ank
3600 IP = pmi
= 240 seconds 60
Fuel mass 0.250 kg 0.28 ¥ 0.0346 ¥ 360 ¥ 1
Fuel rate m f = = = 550 ¥
time 240 s 60
= 32.0 kW
= 0.00104 kg/s
IP 17.776 Brake power
hith = =
m f CV 0.00104 ¥ 44200 2p N T 2p ¥ 360 ¥ 628
BP = =
= 0.386 or 38.6% 60, 000 60, 000
= 23.675 kW
Example 24.18 The following data and results refer Mechanical efficiency, hmech:
to a test on a single-cylinder, two-stroke cycle engine: BP ( 23.675 kW )
indicated mean effective pressure = 550 kPa hmech = =
IP (32.0 kW )
cylinder diameter = 21 cm
= 0.7397 ª 74%
piston stroke = 28 cm
(ii) Indicated thermal efficiency (hith);
engine speed = 360 rpm
brake torque = 628 Nm IP
hith =
fuel consumption = 8.16 kg/h m f CV
calorific value of fuel = 42700 kJ/kg (32.0 kW )
=
Calculate Ê 8 . 16 ˆ
kg/s˜ ¥ ( 42700 kJ/kg)
(a) mechanical efficiency, ËÁ 3600 ¯
(b) the indicated thermal efficiency,
= 0.330 = 33%
Internal Combustion Engines 825

(iii) Brake thermal efficiency (hbth); L Ank


IP = pmi
BP 60
hbth =
m f CV p (0.3) 2 300
= 600 ¥ 0.45 ¥ ¥ ¥ ¥1
( 23.675 kW ) 4 60 2
=
Ê 8.16 ˆ
ÁË 3600 kg/s˜¯ ¥ ( 42700 kJ/kg) = 47.71 kW

= 0.244 = 24.4% (ii) The brake power,


(iv) Brake-specific fuel consumption (Bsfc); 2p N T
BP =
60, 000
m f ( kg/h ) (8.16 kg/h )
Bsfc = = where, T = brake load ¥ effective brake radius
BP ( kW ) ( 23.675 kW )
Dbrake + drope 1.8 + 0.0 2
= 0.3446 kg/kWh Rbrake = =
2 2
Example 24.19 The following observations were = 0.91 m
recorded during a test on a single cylinder, four-stroke T = 1.5 ¥ 0.91 = 1.365 kNm
oil engine: = 1365 Nm
Bore 300 mm 2p ¥ 300 ¥ 1365
Stroke 450 mm Thus BP = = 42.88 kW
60, 000
Speed 300 rpm (iii) Mechanical efficiency,
Imep 6 bar
BP 42.88
brake load 1.5 kN hmech = = = 0.8988
IP 47.71
Brake drum diameter 1.8 m
Brake rope diameter 2 cm = 89.88%
Calculate
Example 24.20 A single-cylinder, four-stroke Diesel
(a) Indicated power, engine works on the following data:
(b) brake power, and
Cylinder bore = 15 cm
(c) mechanical efficiency.
Stroke = 25 cm
Solution Speed = 250 rpm
Area of indicator diagram = 6 cm2
Given A single-cylinder oil engine with following data:
Length of the indicator diagram = 9 cm
d = 300 mm = 0.3 m
Spring constant = 7.5 bar/cm
L = 450 mm = 0.45 m
Brake specific fuel consumption = 0.24 kg/kWh
N = 300 rpm
CV of fuel = 42000 kJ/kg
pmi = 6 bar = 600 kPa
Diameter of brake wheel = 70 cm
Wbrake = 1.5 kN
Rope diameter = 3.5 cm
Dbrake = 1.8 m, drope = 2 cm = 0.02 m
Brake load = 40 kg (392.4 N)
k =1
Calculate
N
n = (a) brake power,
2 (b) indicated mean effective pressure,
To find (c) indicated power,
(i) IP (ii) BP (iii) hmech (d) mechanical efficiency, and
(e) indicated thermal efficiency.
Analysis
(i) The indicated power,
826 Thermal Engineering

Solution (v) Indicated thermal efficiency (hith);


mf
Given A single-cylinder, four-stroke Diesel engine: Given Bsfc =
N BP
n = k =1 mf
2 or 0.24 =
d = 15 cm = 0.15 m L = 25 cm = 0.25 m 3.775
N = 250 rpm Ai = 6 cm2 or m f = 0.906 kg/h = 2.516 ¥ 10– 4 kg/s
l = 9 cm Spring constant = 7.5 bar/cm IP
Bsfc = 0.24 kg/kWh CV = 42000 kJ/kg Thus hith =
m f ¥ CV
Dbrake = 70 cm drope = 3.5 cm
4.6
Wbrake = 392.4 N =
2.516 ¥ 10 -4 ¥ 42000
To find
= 0.435 or 43.5%
(i) BP (ii) Imep (iii) IP
(iv) hmech (v) hith Example 24.21 A full-load test was conducted on
Analysis a two-stroke engine and the following results were
obtained:
Effective brake radius
Speed = 500 rpm
Dbrake + drope
70 + 3.5
Rbrake = = Brake load = 500 N
2 2
= 36.75 cm = 0.3675 m imep = 3 bar
T = Load ¥ Effective brake radius Oil consumtion = 5 kg/h
= Wbrake Rbrake Jacket water temperature rise = 35°C
Jacket water flow rate = 7 kg/min.
T = 392.4 ¥ 0.3675 = 144.2 Nm
A/F ratio by mass = 30
(i) The brake power
Exhaust gas temperature = 350°C
2p N T Room temperature = 25°C
BP =
60, 000 Atmospheric pressure = 1 bar
2p ¥ 250 ¥ 144.2 Cylinder diameter = 22 cm
\ BP = = 3.775 kW
60000 Stroke = 28 cm
(ii) Indicated mean effective pressure (Imep) Brake diameter = 1.6 m
Area of indicator diagram CV of fuel = 42000 kJ/kg
pmi =
Length of the indicator diagram Proportion of H2 by mass in fuel = 15%
¥ Spring constant Specific heat of exhaust gas = 1.0 kJ/kg ◊ K
6 cm 2 Specific heat of dry steam = 2.0 kJ/kg ◊ K
= ¥ 7.5 bar/cm Calculate
9 cm
= 5.0 bar or 500kPa (a) indicated thermal efficiency
(iii) Indicated power (IP) (b) Specific fuel consumtion
(c) Volumetric efficiency based on atmospheric
L Ank conditions
IP = pmi
60 Draw up a heat balance sheet for test.
p (0.15) 2 250
= 500 ¥ 0.25 ¥ ¥ ¥ ¥1
4 60 2 Solution
= 4.6 kW
Given A single-cylinder, two-stroke Diesel engine:
(iv) Mechanical efficiency (hmech)
n = N = 500 rpm A/F = 30
BP 3.775
hmech = = = 0.820 = 82.0% d = 22 cm = 0.22 m L = 28 cm = 0.28 m
IP 4.6
Internal Combustion Engines 827

pmi = 3 bar = 300 kPa m f = 5 kg/h (iii) Volumetric efficiency


CV = 42000 kJ/kg Wbrake = 500 N Actual mass flow rate into engine
Dbrake = 1.6 m mw = 7 kg/min ma = (A/F) m f = 30 ¥ 5 = 150 kg/h
Tg = 350°C (DT )w = 35°C = 0.4167 kg/s
Ta = 25°C = 298 K pa = 1 bar = 100 kPa Actual volume flow rate of air;
Cpg = 1.0 kJ/kg ◊ K Cps = 2.0 kJ/kg ◊ K ma RTa 0.4167 ¥ 0.287 ¥ 298
Va = =
pa 100
To find = 0.03563 m3/s
(i) Indicated thermal efficiency, The volumetric efficiency
(ii) Specific fuel consumption, V 0.03563
(iii) Volumetric efficiency based on atmospheric con- hvol = a =
Vs 0.0887
ditions, and
= 0.4017 or 40.17%
(iv) Heat balance sheet.
Heat Balnace sheet: On minute basis;
Assumptions Heat supplied per minute by fuel
(i) Single cyclinder engine, i.e. k = 1 Ê 5 ˆ
(ii) Specific gas constant for air, R = 0.287 kJ/kg ◊ K Q in = m f CV = Á kg/min˜ ¥ 42000
Ë 60 ¯
(iii) Specific heat of water Cpw = 4.187 kJ/kg ◊ K = 3500 kJ/min
Analysis The swept volume rate Heat equivalent to BP
p 2 p 500 Q1 = BP ¥ 60 = 20.96 ¥ 60
Vs = d L n k = ¥ (0.22) 2 ¥ 0.28 ¥ ¥1 = 1257.6 kJ/min
4 4 60
= 0.0887 m3/min Heat lost to cooling water
(i) Indicated power Q 2 = mw Cpw(DT ) = 7 ¥ 4.187 ¥ 35
= 1025.81 kJ/min
IP = pmiVs = 300 ¥ 0.0887 = 26.6 kW
Heat lost to exhaust gases
Heat supplied by fuel
Mass of exhaust gases formed/kg of fuel
Qin = m f CV = 5 ¥ 42000 A Ê 5 ˆ
= 210000 kJ/h = 58.33 kW mex = m f ¥ = Á kg / min˜ ¥ 30
F Ë 60 ¯
Indicated thermal efficiency = 2.5 kg/min
IP 26.6 kW Mass of H2O formed during combustion
hith = =
Qin (58.33 kJ/s) me x = 9H2 ¥ Mass of fuel used per minute
= 0.456 or 45.6% 5
= 9 ¥ 0.15 ¥ = 0.1125 kg/min
(ii) Specific fuel consumption 60
The brake drum radius Total mass of exhaust gases (dry) per minute
D 1.6 mg = Mass of wet exhaust – Mass of H2O
Rbrake = brake = = 0.8 m
2 2 formed
Brake torque T = Load ¥ Effective brake radius = 2.5 – 0.1125 = 2.3875 kg/min
= Wbrake Rbrake (a) Heat lost to dry exhaust gases
= 500 ¥ 0.8 = 400 N-m Q 3 = mg Cpg (DT )g
Brake power = 2.3875 ¥ 1.0 ¥ (350 – 25)
2p N T 2p ¥ 500 ¥ 400
BP = = = 775.93 kJ/min
60, 000 60000
(b) H2O formed exit in superheated state at
= 20.94 kW
350°C, thus heat carried away by steam
The brake-specific fuel consumtion
Q4 = mH2O (Cpw (Tsat – Ta) + hfg + Cps (Tg – Ta))
mf (5 kg/h)
Bsfc = = = 0.1125 [(4.187 ¥ (100 – 25) + 2257 + 2.0
BP ( 20.94 kW ) ¥ (350 – 25)]
= 0.238 kg/kWh = 362.36 kJ/min
828 Thermal Engineering

Heat Balance Sheet N


n = , R = 0.287 kJ/kg ◊ K
2
Particulars Quntity Percentage k =6
Credit (input) d = 9.5 cm = 0.095 m
Heat supplied by fuel 3500 kJ/min 100% L = 12 cm = 0.12 m
Debit (output) N = 2400 rpm
Heat equivalent to BP 1256.4 kJ/min 35.9% W = 550 N
Heat carried by coolant 1025.81 kJ/min 29.3% pa = 75 cm of Hg
Heat carried away by rf = 0.83 kg/lit.
dry flue gases 773.93 kJ/min 22.17% t = 19.3 s/100 cc consumtion
Heat carried away by Ta = 25°C = 298 K
steam 362.3 kJ/min 10.35% do = 3 cm = 0.03 m
Unaccounted heat lost 79.8 kJ/min 2.28% Cd = 0.6 cm
ho = 14.5 cm of Hg
C : H ratio = 83 : 17 by mass in fuel
Example 24.22 The brake power of a 6-cylinder,
4-stroke Diesel engine, absorbed by a hydraulic To find
dynamometer is given by (i) Brake mean effective pressure,
WN (ii) Brake-specific fuel consumption, (Bsfc),
BP = (kW )
20,000 (iii) Percentage excess air, and
where W is the brake load in newtons and N is the (iv) Volumetric efficiency, hvol .
rotational speed in rpm. The air consumption is measured
Analysis
by an air box with a sharp edge orifice. The following
readings were recorded. (i) The brake mean effective pressure
Bore of cylinder = 9.5 cm The brake power is given by
Stroke of piston = 12 cm WN
BP = (kW)
Speed of engine = 2400 rpm 20, 000
Brake load = 550 N Using numerical values,
Ambient pressure = 75 cm of Hg 550 ¥ 2400
BP = = 66 kW
Fuel density = 0.83 kg/litre 20, 000
Time for 100 cc fuel consumtion = 19.3 seconds The brake power is also given by
Room temperature = 25°C pmb L An k
C : H ratio in fuel mass = 83 : 17 BP =
60
Orific diameter = 3 cm 66 ¥ 60
Cd (orific) = 0.6 or pmb =
p 2400
Manometer head across the orifice = 14.5 cm of Hg 0.12 ¥ ¥ (0.095) 2 ¥ ¥6
4 2
Calculate = 646.61 kPa or 6.47 bar
(a) Brake mean effective pressure (ii) Brake specific fuel consumtion
(b) Brake specific fuel consumtion The mass-flow rate of fuel
(c) Percentage excess air 100 cc Ê 3600 s/h ˆ
mf = ¥ (0.83 kg/lit ) ¥ Á
(d) Volumetric efficiency (1000 cc/lit ) Ë 19.3 s ˜¯
Assume R = 0.287 kJ/kg ◊ K for air.
= 15.48 kg/h
Solution m f ( kg/h ) 15.48
Bsfc = = = 0.2345 kg/kWh
BP ( kW ) 66
Given A six-cylinder, four-stroke Diesel engine:
Internal Combustion Engines 829

(iii) Prercentage excess air Actual volume suction rate


3 3
Pressure of ambient air; V f = 0.077 m /s ¥ (60 s/min) = 4.62 m /min
750 mm of Hg
pa = ¥ 101.325 kPa Va 4.62
760 mm of Hg hvol = = = 0.754 or 75.4%
= 99.9917 @ 100 kPa Vs 6.124
Density of ambient air,
p 100 Example 24.23 A 6-cylinder, four-stroke Diesel
ra = a = = 1.17 kg/m3 engine has a bore of 33.75 cm and a stroke of 37.5 cm. It
RTa 0.287 ¥ 298
is tested at half-load conditions and gave the following
Since fuel contains 0.83 kg of carbon and 0.17 kg observations:
of hydrogen in 1 kg of fuel, thus the theoretical
Brake power = 142 kW
mass of air required per kg of fuel for complete
Engine speed = 350 rpm
combustion
Indicated mean effective pressure = 3.72 bar
100 È 8 ˘
mth = C + 8H ˙
23 ÍÎ 3
Fuel consumption rate = 44 kg/h
˚
Calorific value of fuel = 44800 kJ/kg
100 È 8 ˘
= ¥ ¥ 0.83 + 8 ¥ 0.17˙
23 ÍÎ 3
Air consumption rate = 38.6 kg/min
˚
Water flow rate in the jacket = 60.2 kg/min
= 15.53 kg/kg of fuel
Rise in temperature of water = 32°C
Theoretical air supply rate to burn the fuel
completely Piston cooling oil flow rate = 34.96 kg/min
mth = mth ¥ m f Cp of piston oil = 2.1 kJ/kg ◊ K
Rise in cooling oil temperature = 20 °C
= 15.53 ¥ 15.48 = 240.4 kg/h
Exhaust gas temperature = 210 °C
The head of air in orifice meter
Ambient air temperature = 25°C
hHg r Hg (0.145 m) ¥ (13600 kg/m3 )
ha = = Cp of exhaust gases = 1.05 kJ/kg ◊ K
ra 1.17 kg/m3
Cp of cooling water = 4.187 kJ/kg ◊ K
= 1685.4 m
Fuel contains 14% H2 by mass.
Actual volume of air supplied through orificre
meter Prepare the heat balance sheet on minute and
p 2 percentage basis . Calculate the specific fuel consumption
Va = do Cd 2 g ha at full load, assuming frictional power and indicated
4
p thermal efficiency do not change with load variation.
= ¥ (0.03) 2 ¥ 0.6 ¥ 2 ¥ 9.81 ¥ 1685.4 Assume partial pressure of water vapour in exhaust
4
gases to be 0.06 bar.
= 0.077 m3/s
The actual mass-flow rate of air Solution
ma = ra Va = 1.17 ¥ 0.077
= 0.0902 kg/s or 324.85 kg/h Given Six-cylinder, four-stroke Diesel engine:
Percentage excess air supplied N
k =6 N = 350 rpm, n =
324.85 - 240.4 2
= ¥ 100 = 35.12% d = 33.75 cm L = 37.5 cm
240.4
= 0.3375 m = 0.375 m
(iv) Volumetric efficiency (hmech)
pmi = 3.72 bar = 372 kPa BP = 142 kW
Swept volume per minute
Loading =H alf m f = 44 kg/h
p
Vs = ¥ d2 Lnk CV = 44800 kJ/kg ma = 38.6 kg/min
4
p 2400 mw = 60.2 kg/min moil = 34.96 kg/min
= ¥ (0.095) 2 ¥ 0.12 ¥ ¥6
4 2 (DT )w = 32°C (DT )oil = 20°C
= 6.124 m3/min Tg = 210°C Ta = 25°C = 298 K
830 Thermal Engineering

pv = 0.06 bar = 6 kPa Cpw = 4.187 kJ/kg ◊ K Heat carried by superheated water vapour at
Cpg = 1.05 kJ/kg ◊ K Cp,oil = 2.1 kJ/kg ◊ K 0.06 bar in exhaust gases
= mass of H2 = 0.14 mf Q5 = mH 2 O hsup
To find where hsup = hg @0.06 bar + Cps (Tsup – Ta)
(i) Heat balance sheet, and = 2567.5 + 2.1 ¥ (210 – 25)
(ii) Specific fuel consumtion at full load. = 2985.5 kJ/min
Then Q5 = 0.924 ¥ 2985.5 = 2758.6 kJ/min
Analysis
(i) Indicated power Heat Balance Sheet
n
IP = pmi L A k Particulars Quantity Percentage
60
p 350 Credit (input)
= 372 ¥ 0.375 ¥ ¥ (0.3375) 2 ¥ ¥6 Heat supplied by fuel, 32853.3 kJ/min 100%
4 2 ¥ 60
= 218.4 kW Qin
Heat supplied by fuel Debit (output)
44 Heat equivalent to BP,
Qin = m f CV = ¥ 44800
60 Q1 8520 kJ/min 25.93%
= 32853.3 kJ/min Heat carried by
The heat equivalent to BP cooling water, Q2 8065.83 kJ/min 24.55%
Q1 = 142 ¥ 60 = 8520 kJ/min Heat carried by
Heat carried by cooling water cooling oil, Q 3 1468.32 kJ/min 4.47%
Q 2 = mw Cpw (DT )w Heat carried away by
dry flue gases, Q4 7460.95 kJ/min 22.70%
= 60.2 ¥ 4.187 ¥ 32
Heat carried away by
= 8065.83 kJ/min
steam formed, Q5 2758.6 kJ/min 8.40%
Heat carried by cooling oil
Unaccounted heat lost
Q 3 = moil Cp, oil (DT )oil = Qin − Q1 − Q2
= 34.96 ¥ 2.1 ¥ 20 − Q3 − Q4 − Q5 4579.6 kJ/min 13.94%
= 1468.32 kJ/min
Mass of exhaust gases formed per minute
(ii) Specific fuel consumtion at full load:
mex = ma + m f
Brake power at full load conditions
44 BPfull = 2 BPhalf = 2 ¥ 142 = 284 kW
= 38.6 + = 39.33 kg/min
60
Frictional power at half-load condition
The amount of water vapour formed;
FP = IPhalf – BPhalf = 218.4 – 142 = 76.4 kW
44
mH 2 O = 9 H2 = 9 ¥ 0.14 ¥
60 Since the FP remains constant, thus IP at full-
= 0.924 kg/min load condtions
Mass of dry exhaust gases; IPfull = BPfull + FP = 284 + 76.4 = 360.4 kW
mg = mex – mH2 O = 39.33 – 0.924 Indicated thermal efficiency at half-load
= 38.409 kg/min conditions
Heat carried by dry exhaust gases IPhalf ( 218.4 ¥ 60 kJ/min)
hith = =
Q4 = mg Cpg (Tg – Ta) Qin (32853.6 kJ/min)
= 38.409 ¥ 1.05 ¥ (210 – 25) = 0.3988
= 7460.95 kJ/min
Internal Combustion Engines 831

The heat input rate at full load 762


pa = ¥ 101.325 kPa = 101.59 kPa
IPfull 360.4 kW 760
Qin, full = = = 903.71 kW
hith 0.3988 Swept volume of four cylinders of engine
Fuel consumption rate at full load p p
Vs = 4 ¥ d 2 L = 4 ¥ ¥ (0.085) 2 ¥ 0.13
Qin, full 4 4
903.71 kW
m f , full = = = 2.95 ¥ 10–3 m3
CV 44800 kJ/kg
Actual volume of air drawn
= 0.02017 kg/s or 72.62 kg/h
Va = 0.7 Vs = 0.7 ¥ 2.95 ¥ 10–3
The brake specific fuel consumtion
= 2.065 ¥ 10–3 m3
m f , full (72.62 kg/h )
bsfc = = The mass of air inducted in cylinders
BPfull ( 284 kW )
paVa 101.59 ¥ 2.065 ¥ 10 -3
= 0.255 kg/kWh ma = =
RTa 0.287 ¥ 290
Example 24.24 A 4-cylinder, four-stroke engine has = 2.52 ¥ 10–3 kg
an 85-mm bore and 130-mm stroke. It develops 30 kW Theoretical mass of air inducted per minute
of power at 1500 rpm and it runs at 20% rich mixture.
N 1500
If the volume of air in the cylinder measured at 17°C ma = ma = 2.52 ¥ 10 -3 ¥
2 2
and 762 mm of mercury is 70% of the swept volume,
theoretical air–fuel ratio is 14.8, calorific value of fuel = 1.89 kg/min
is 45000 kJ/kg and mechanical efficiency of the engine Theoretical fuel used per minute
is 90%, calculate (a) indicated thermal efficiency, and ma 1.89
m f , th = = = 0.1277 kg/min
(b) brake mean effective pressure. A/F 14.8
Assume R = 0.287 kJ/kg ◊ K for air. When using 20% rich mixture, then

Solution m f , act = 1.2 m f , th


= 1.2 ¥ 0.1277 = 0.1532 kg/min
Given Four-cylinder, four-stroke engine
or = 2.554 ¥ 10–3 kg/min
k =4
N Indicated power
N = 1500 rpm, n =
2 BP 30
d = 85 mm = 0.085 m IP = = = 33.33 kW
hmech 0.9
L = 130 mm = 0.13 m
(i) Indicated thermal efficiency
m f , act = 1.2 m f , th
Heat supplied by fuel
BP = 30 kW
CV = 45000 kJ/kg Q in = m f , act CV = 2.554 ¥ 10–3 ¥ 45000
Ta = 17°C = 290 K = 114.93 kW
pa = 762 mm of Hg Indicated thermal efficiency of the engine
Vact = 0.7Vs IP 33.33 kW
hith = = = 0.29 or 29%
A/F = 14.8 Qin 114.93 kW
hmech = 0.90 (ii) Brake mean effective pressure
R = 0.287 kJ/kg ◊ K Brake power is given by
To find n n
BP = pmb L A k = pmb Vs
(i) Indicated thermal efficiency, 60 60
BP 30
(ii) Brake mean effective pressure. or pmb = =
n 1500
Vs 2.95 ¥ 10 -3 ¥
Analysis The atmospheric pressure corresponds to 60 2 ¥ 60
762 mm of Hg = 813.56 kPa
832 Thermal Engineering

3. It is used to reduce the size of engine to fit in


a limited space such as in marine engines.
The charging of an engine refers to supply of charge 4. It is used to increase the power output from
into the engine cylinder during suction stroke. In a an existing engine.
naturally aspirated engine, the charge is inducted The supercharged Otto and Diesel cycles are
into the cylinder below atmospheric pressure. Thus shown in Fig. 24.43.
the mass of charge is less than that corresponding
to atmospheric pressure. Therefore, the volumetric
efficiency of the engine is less than 100%. Supercharged Engine
Supercharging is a process which supplies the Sr. Aspect Naturally Supercharged
charge above atmospheric pressure. Super means No. aspirated or engine
better. So supercharge means increased mass of conventional
engine
charge that is inducted into the cylinder during the
suction stroke. The mass of charge can be increased 1. Volumetric It is always In most of
by increasing the density of charge. The charge is efficiency less than the cases, it
100% is more than
compressed prior to admission into the cylinder, 100%
thereby enabling the greater mass of charge with 2. Mechanical Less Sightly better
same total piston displacement. The device used for efficiency than naturally
compressing the charge is called supercharger. aspirated
The supercharging is used in racing cars and in engines
3. Combustion Depends on Always good
heavy-duty diesel engines. due to close
turbulence of
Objectives of Supercharging charge spacing of
fuel molecules
1. It is used to overcome the effects of high 4. Output Less More power
altitude, such as high aircrafts and engines output
run on mountains. 5. Peak pressure Less Higher
2. It reduces the weight of engine per kW 6. Peak
power developed. temperature Less Much higher

p p
3 2 3

Ex
Ex pa
pa ns
nsi +ve Loop io
o n n
2 Co
+ve Loop mp
re
ss
ion
Com 4 4
pre
ssio
n
Suction Suction
7 1 7 1
+ve Loop +ve Loop
6 5 6 5
patm patm
Exhaust Exhaust

Vc Vs Vc Vs

V V
(a) Supercharge otto cycle (b) Supercharged Diesel cycle
Fig. 24.43
Internal Combustion Engines 833

Summary
system is used on two wheelers, racing cars and
that converts chemical energy of fuel into aircrafts.
mechanical energy. internal combustion engines are subjected
to very high temperature during combustion of
cycle in two strokes of the piston. The three charge. Due to overheating of engine, there may
ports; inlet, transfer, and exhaust ports are used be uneven expansion in some parts, burning of
for suction, transfer and discharge of charge, lubricant, valve seats, etc. Therefore, the engine
respectively. A deflector-shapped piston is used should be provided with adequate cooling
to direct the charge inside the cylinder. arrangement.

cycle in four strokes of the piston as suction, maintenance-free and is widely used on two
compression, expansion and exhaust stroke. They wheelers and light-duty engines.
are widely used on motor cycles, cars, buses,
trucks and aeroplanes. Due to good thermal of heat during combustion, and therefore, they
efficiency of four-stroke engines, the specific fuel are water cooled. The water-cooling arrangement
consumption is less. consists of a pump, a fan, a water jacket around
Petrol engines use low compression ratio in the engine and a radiator.
the range of 4 to 10, while Diesel engines use
high compression ratio usually in the range of moving parts. The mist lubrication system is
14 to 21. The petrol engines induct carburetted used in two-stroke engines, while all four-stroke
homogeneous air–fuel mixture as charge into engines use wet or dry sump lubrication system.
the cylinder while Diesel engines induct only air
during suction, and diesel is injected at the end supplies the charge to the cylinder above
of the compression stroke. The fuel burns in the atmospheric pressure. Thus, the volumetric
presence of hot air. Therefore, the Diesel engines efficiency of the engine improves and the engine
are also called compression ignition (CI) engines. produces more power.
-
gines, while Diesel engines are quality-governing cylinder of the engine where the admitted charge
engines. The hit-and-miss governing is used for pushes the combustion products out of the
high speed gas engines. cylinder. Scavenging takes place in two-stroke
ignition system is used to produce the high- engines at the end of the expansion stroke. Poor
intensity spark for initiation of combustion in the scavenging may lead to dilution of charge, thus
petrol engine. The battery ignition system is used less power is obtained from the engine.
on heavy-duty engines, while magneto ignition

Glossary
A/F Mass ratio of air to fuel CI engine Compression ignition (Diesel) engine
BDC Bottom dead centre Carburation Preparation of combustible mixture for
Bore Internal diameter of cylinder petrol engine
Brake power Power available from the engine for Clearance volume Volume left in the centre, when
external use piston is at TDC
834 Thermal Engineering

Compression ratio Ratio of maximum volume to SI engine Spark ignition (petrol) engine
minimum volume in the cylinder Scavenging Gas exchange process in two-stroke
Friction power Difference between indicated power engines
and brake power Specific fuel consumption Mass of fuel (kg/h) con-
Fuel injector Device to inject the pressurised fuel into sumed per unit power output (BP)
cylinder Stoichiometric air Amount of air just sufficient for
Fuel pump Device to pressurise the fuel for injection complete combustion
IC engines Internal combustion reciprocating engines Stroke Linear distance between TDC and BDC
Indicated power Power developed on the piston by Supercharging Charging of engine cylinder above
combustion gases inside the cylinder atmospheric pressure
Magneto An electric generator, which converts kinetic Swept volume Piston displacement volume in cylinder
energy into electrical energy TDC Top dead centre
Mean effective pressure Ratio of net work done to Thermal efficiency Ratio of brake power to the heat-
swept volume in the cycle supply rate
Mechanical efficiency Ratio of brake power to the Thermostat An instrument which controls the
indicated power temperature

Review Questions
1. What is an internal combustion engine? 14. Define brake-specific fuel consumption.
2. Write the classification of internal combustion 15. What is carburation? Draw a schematic for fuel-
engines. injection system in a Diesel engine.
3. List the parts of an IC engine. 16. Draw a simple carburetter and write the function
4. Explain the construction, working and applica- of its parts.
tions of a two-stroke petrol engine. 17. Draw a neat diagram of a jerk-type fuel-injection
5. Explain construction, and working of a two- pump and explain its working.
stroke Diesel engine. 18. How are the following engines governed?
6. Explain the working of a four-stroke petrol (i) SI engines (ii) CI engines
engine. 19. Explain the abnormal combustion in SI engines.
7. Discuss the construction of a four-stroke petrol 20. What are the harmful effects of overheating of an
engine. engine? Explain.
8. Explain the working of a four-stroke Diesel 21. Why is the cooling arrangement provided with
engine. each internal combustion engine?
9. Why are four-stroke engines preferred over two- 22. What are the undesirable effects if an engine is
stroke engines? undercooled?
10. Compare petrol and Diesel engines. 23. Discuss the air-cooling system with its merits and
11. Compare two-stroke and four-stroke engines. demerits.
12. Define charge. What do you mean by 24. How is the liquid-cooling arrangement better
(a) Stoichiometric mixture than the air-cooling arrangement on an internal
(b) Rich mixture combustion engine?
(c) Lean mixture 25. Discuss water-cooling system for an internal
13. Draw the curves for best power and best economy combustion engine.
on A/F ratio vs Power output plot. 26. What are the advantages of increasing pressure of
the coolant in a liquid-cooling system?
Internal Combustion Engines 835

27. What are the functions of lubrication? 34. Explain the battery ignition system with a neat
28. What are the desirable properties of lubricating diagram.
oil? 35. Explain magneto ignition system.
29. Explain the mist lubrication system. 36. What is supercharging? What are the effects of
30. Explain splash lubrication system. supercharging on engine performance?
31. Discuss the pressure-feed lubrication system. 37. Explain the working of a fuel pump.
32. What is an ignition system? What are the 38. Explain the working of a fuel injector.
requirements of an ignition system for the SI 39. What is scavenging? How does scavenging take
engines? place in a two-stroke petrol engine? Explain.
33. Why is the battery ignition system preferred on 40. Explain (a) quality governing, and (b) quantity
most of the automobiles? governing of IC engine.

Problems
1. The engine of a car has four cylinders of 70 mm indicated thermal efficiency is 38% and the
bore, and 75 mm stroke. The compression ratio is mechanical efficiency is 80%. Calculate (a) brake
8. Determine the cubic capacity of the engine and power, and (b) heating value of the fuel.
the clearance volume of each cylinder. [(a) 73.7 kW, (b) 41992.9 kJ/kg]
[1154 cc, 41.21 cc] 7. An engine develops an indicated power of 125
2. A four-cylinder, four-stroke petrol engine has kW and delivers a brake power of 100 kW.
a bore of 80 mm and a stroke of 80 mm. The Calculate (a) frictional power, and (b) mechanical
compression ratio is 8. Calculate the cubic efficiency of the engine. [(a) 25 kW, (b) 80%]
capacity of the engine and clearance volume of 8. A single-cylinder, four-stroke cycle oil engine
each cylinder. What type of engine is this? is fitted with a rope brake dynamometer. The
[(a) 1608.4 cc, (b) 57.4 cc, diameter of the brake wheel is 600 mm and the
(c) Square engine] rope diameter is 26 mm. The brake load is 170 N.
3. A four-cylinder, four-stroke Diesel engine is If the engine runs at 450 rpm, calculate the brake
to develop 30 kW at 1000 rpm. The stroke is power of the engine. [2.5 kW]
1.4 times the bore and the indicated mean 9. A single-cylinder, four-stroke Diesel engine
effective pressure is 6.0 bar. Determine the bore having a displacement volume of 790 cc is tested
and stroke of the engine. [176 mm, 246 mm] at 300 rpm. When a braking torque of 49 Nm is
4. A 42.5-kW engine has a mechanical efficiency applied, analysis of the indicator diagram gives
of 85%. Find the indicated power and friction a mean effective pressure of 980 kPa. Calculate
power. If the friction power is constant with the the brake power and mechanical efficiency of the
load, what will be the mechanical efficiency at engine. [(a) 1.54 kW, (b) 79.4%]
60% of the load? [50 kW, 7.5 kW, 77.3%] 10. A four-stroke petrol engine delivers a brake
5. Calculate the brake mean effective pressure of a power of 36.8 kW with a mechanical efficiency
four-cylinder, four-stroke Diesel engine having a of 80%. The air–fuel ratio is 15 : 1 and the fuel
150-mm bore and 200-mm stroke which develops consumption is 0.4068 kg/kWh. The calorific
a brake power of 73.6 kW at 1200 rpm. value of the fuel is 42000 kJ/kg. Calculate
[5.206 bar] (a) indicated power, (b) friction power, (c) brake
6. An engine is using 5.2 kg of air per minute, thermal efficiency, (d) indicated thermal
while operating at 1200 rpm. The engine requires efficiency, and (e) total fuel consumption.
0.2256 kg of fuel per hour to produce an indicated [(a) 46 kW, (b) 9.2 kW, (c) 21%,
power of 1 kW. The air–fuel ratio 15 : 1. The (d) 26.25%, (e) 15.12 kg/h]
836 Thermal Engineering

11. During a trial of four-stroke Diesel engine, the (a) Air fuel ratio
following observations were recorded: (b) Indicated and brake thermal efficiencies
Area of indicator diagram = 475 mm2 [(a) 20, (b) 32.875, 28.6%]
Length of the indicator diagram = 62 mm 13. A twin cylinder, two-stroke internal combustion
Spring constant = 1.1 bar/mm engine is operating with a speed of 4000 rpm.
Bore = 100 mm The fuel consumption is 10 litres per hour. The
Stroke = 150 mm indicated mean effective pressure is 7.5 bar.
Speed = 375 rpm Specific gravity of the fuel is 0.78. CV of fuel =
Determine (a) indicated mean effective pressure, 42 MJ/kg, A/F = 16, hvol = 75%, hmech = 80%,
and (b) indicated power. average piston speed = 600 m/min. Determine
[(a) 8.43 bar, (b) 3.1 kW] the dimensions of the cylinder. Also, calculate the
brake thermal efficiency.
12. A four-stroke, gasolene engine develops a brake
power of 410 kW. The engine consumes 120 kg [d = 69.1 mm, L = 75 mm, hith = 30.91%]
of fuel in one hour and air consumption is 14. A single-cylinder, four-stroke CI engine has a bore
40 kg/min. The mechanical efficiency is 87% and and stroke of 75 mm and 100 mm respectively.
the fuel heating value is 43000 kJ/kg. Determine Find the bmep, if the torque is 25 N-m.
the following: [35.55 bar]

Objective Questions
1. Which one of the following parts does not exist in (a) charging (b) scavenging
an IC engine? (c) combustion (d) none of the above
(a) Crank shaft (b) Cam shaft 7. In a four–stroke Diesel engine, during suction
(c) Piston rod (d) Connecting rod stroke
2. Stoichiometric air–fuel ratio of petrol is roughly (a) fuel–air mixture is inducted
(a) 50 : 1 (b) 25 : 1 (b) only fuel is inducted
(c) 15 : 1 (d) 1 : 1 (c) only air is inducted
3. A two-stroke engine has (d) none of the above
(a) two ports (b) three ports 8. The function of venturi in the carburetter is
(c) two valves (d) three valves (a) to decrease the air velocity
4. In a two-stroke engine, one power stroke is (b) to increase the velocity
obtained in (c) to decrease the fuel flow
(a) one revolution of the crank shaft (d) to increase the manifold vacuum
(b) two revolutions of the crank shaft 9. Which one of the following is true for a Diesel
(c) four revolutions of the crank shaft engine?
(d) none of the above (a) It has high compression ratio.
5. In a four-stroke engine, one power stroke is (b) It does not have a spark plug.
obtained in (c) It has large noise and vibrations.
(a) one revolution of the crank shaft (d) All of the above
(b) two revolutions of the crank shaft 10. The air–fuel ratio of a petrol engine is controlled
(c) four revolutions of the crank shaft by
(d) none of the above (a) fuel njector
i (b) fuel pump
6. In a two-stroke engine, the gas exchange process (c) carburetter (d) none of the above
is called
Internal Combustion Engines 837

11. In an IC engine, the exhaust pressure in the (a) misfiring


cylinder is (b) detonation
(a) equal to the atmospheric pressure (c) knocking
(b) below the atmospheric pressure (d) longer ignition delay
(c) above the atmospheric pressure 19. In a supercharged engine, induction air
(d) none of the above (a) is supplied at higher density
12. In a two-stroke IC engine, the piston top has a (b) mixed with fuel
deflector for (c) performes better scavenging
(a) better combustion of fuel (d) none of the above
(b) better scavenging of exhaust gases 20. In a Diesel engine, ignition occurs due to high
(c) better mixing of air and fuel (a) density ofcharge
(d) better charging of the cylinder (b) temperature of air–fuel mixture
13. The pumping power is required in the following (c) temperature of compressed air
stroke(s): (d) intensity spark on spark plug
(a) Suction tsroke 21. Thermal efficiency of an IC engine indicates
(b) Exhaust tsroke percentage of
(c) Compression tsroke (a) BP converted into IP
(d) Suction and exhaust stroke (b) heat converted into work
14. In a Diesel engine, fuel consumption against (c) IP converted into BP
brake power is (d) heat lost into exhaust
(a) parabolic (b) linear 22. Mechanical efficiency of an IC engine indicates
(c) hyperbolic (d) non-predictable percentage of
15. Which one of following is not a governing (a) BP converted into IP
method used in IC engines? (b) heat converted into work
(a) Quality governing (c) IP converted into BP
(b) Quantity governing (d) heat lost into exhaust
(c) Injection governing 23. By decreasing clearance volume, the volumetric
(d) Hit-and-miss governing efficiency of an IC engine
16. Which one of following is a governing method (a) remains constant (b) increases
used on Diesel engines? (c) decreases (d) cannot asy
(a) Quality governing 24. By use of lubrication, which efficiency of an IC
(b) Quantity governing engine improves
(c) Injection governing (a) volumetric efficiency
(d) Hit-and-miss governing (b) mechanical efficiency
17. Which one of following is a governing method (c) charging efficiency
used on petrol engines? (d) indicated thermal efficiency
(a) Quality governing 25. By use of cooling, which efficiency of an IC
(b) Quantity governing engine decreases
(c) Injection governing (a) volumetric efficiency
(d) Hit and miss governing (b) mechanical efficiency
18. Increase in compression ratio in an Otto cycle (c) charging efficiency
engine may cause (d) thermal efficiency
25. (d)
24. (b) 23. (b) 22. (c) 21 (b) 20. (c) 19. (a) 18. (b) 17. (b)
16. (a) 15. (c) 14. (b) 13. (d) 12. (b) 11. (c) 10. (c) 9. (d)
8. (b) 7. (c) 6. (b) 5. (b) 4. (a) 3. (b) 2. (c) 1. (c)
Answers
838 Thermal Engineering

25
Reciprocating Air Compressor

Introduction
An air compressor is a machine which takes
in atmospheric air, compresses it with the help
of some mechanical energy and delivers it at
higher pressure. It is also called air pump. An
air compressor increases the pressure of air by
decreasing its specific volume using mechanical
means. Thus compressed air carries an immense
potential of energy. The controlled expansion
of compressed air provides motive force in air
motors, pneumatic hammers, air drills, sand-blasting machines and paint sprayers, etc.
The schematic of an air compressor is shown in Fig. 25.1. The compressor recieves energy input from a
prime mover (an engine or electric motor). Some part of this energy input is used to overcome the frictional
effects, some part is lost in the form of heat and the remaining part is used to compress air to a high pressure.

4. Cleaning purposes,
5. Blast furnaces,
Compressed air has wide applications in industries 6. Spray painting and spraying fuel in Diesel
as well as in commercial equipment. It is used in engines,
1. Air refrigeration and cooling of large build- 7. Hard excavation work, tunneling, boring,
ings, mining, etc.
2. Driving pneumatic tools in shops like drills, 8. Starting of heavy-duty diesel engines,
rivetters, screw drivers, etc. 9. Operating air brakes in buses, trucks and
3. Driving air motors in mines, where electric trains etc.
motors and IC engines cannot be used 10. Inflating automobile and aircraft tyres,
because of fire risks due to the presence of 11. Supercharging internal combustion engines,
inflammable gases, etc. 12. Conveying solid and powder materials in
pipelines,
Reciprocating Air Compressor 839

13. Process industries, 2. Double-acting compressor is a compressor in


14. Operating lifts, hoists, crains and to operate which suction, compression and delivery of gas
pumps etc. take place on both sides of the piston and two cycles
15. Pump sets for oil and gas transmission line, take place during one revolution of the crank shaft.
16. Automobile suspension system. 3. Single-stage compressor is a compressor in which
the compression of gas to final delivery pressure is
carried out in one cylinder only.
The compressors are mainly classified as 4. Multistage compressor is a compressor in which
(i) Reciprocating compressors, and the compression of gas to the final pressure is
carried out in more than one cylinder in series.
(ii) Rotary compressors.
5. Pressure ratio is defined as the ratio of absolute
The air compressors can broadly be classified as
discharge pressure to absolute suction pressure.
6. Free air is the air that exists under atmospheric
condition.
7. Compressor displacement volume is the volume
created when the piston travels a stroke. It is given
as
p
V = d2 L ...(25.1)
4
where d is the bore of the cylinder and L the is
A reciprocating compressor is used to produce stroke of the piston.
high-pressure gas. It uses the displacement of into
piston in the cylinder for compression. It handles a the compressor is expressed in m3/s and is given as
low mass of gas and a high pressure ratio.
V = Volume inducted per cycle ¥ No. of
The rotary compressors are used for low and
inductions per revolution ¥ Number of revolutions
medium pressures. They usually consist of a bladed
per second
wheel or impeller that spins inside a circular
For the single-acting reciprocating compressor,
housing. They handle a large mass of gas.
only one cycle (thus, one induction) takes place for
These compressors may be single stage or
each revolution of the crank. Thus, for a compressor
multistage to increase the pressure ratio.
without clearance
p 2 N
25.3 V = d L ...(25.2)
4 60
For the double-acting reciprocating compressor,
In connection to reciprocating compressors, the the induction takes place on both sides of the piston
following terms are defined: for each revolution. Thus,
1. Single-acting compressor is a compressor in p 2 Ê 2N ˆ
V = d LÁ ...(25.3)
which suction, compression and delivery of a gas 4 Ë 60 ˜¯
take place only on one side of the piston during a
9. Capacity of a compressor is the actual quantity
cycle of one revolution of the crank shaft.
of air delivered per unit time at atmospheric
conditions.
840 Thermal Engineering

It is the discharge
10. Free Air delivery (FAD)
volume of the compressor corresponding to
ambient conditions.
A machine which takes in air or gas during suction
11. Piston speed is the linear speed of the piston stroke at low pressure and then compresses it to
measured in m/min. It is expressed as high pressure in a piston–cylinder arrangement
Vpiston = 2 L N ...(25.4) is known as a reciprocating compressor. External
work must be supplied to the compressor to achieve
required compression. This work is used to run
the compressor. A part of the work supplied to
Compressed air systems consist: intake air filters, the compressor is lost to overcome the frictional
inter-stage coolers, after coolers, air dryers, resistance between rubbing surfaces of the piston
moisture drain traps, receivers, piping network, and cylinder. The cylinder of air compressor is
control valves and lubricators. cooled to minimise the work input.
1. They prevent dust from The air compressed by a reciprocating compres-
entering the compressor. Dust causes sticking of sor cannot directly be used for an application. The
valves, scoured cylinders, excessive wear, etc. reciprocating motion of the piston gives rise to
pulsating flow through the discharge valve of the
2. These are placed between compressor. Thus, the compressed air is discharged
consecutive stages of multistage compressor. They from the air compressor to an air receiver.
reduce the temperature of compressed air, before it
enters the next stage of compression.

3. They remove heat of compression Figure 25.2 shows the sectional view of a single-
and moisture in the air by reducing the temperature stage air compressor. It consists of a piston,
in a water-cooled heat exchanger, after compression cylinder with cooling arrangement, connecting rod,
is completed. crank, inlet and delivery valves. The piston fitted
with piston rings, reciprocates in the cylinder. The
4. The remaining traces of moisture, prime mover (an engine or electric motor) drives
after an after-cooler are removed by using air the crank shaft, the crank rotates and converts
dryers, for using compressed air in instruments and rotary motion into reciprocating motion of piston
pneumatic equipment. The moisture is removed by
using adsorbents like silica gel or activated carbon,
Water jacket
or refrigerant dryers, or heat of compression dryers.
Receiver Atmospheric
Delivery value Inlet value
Pressure pressure
5. Moisture drain traps are
TDC
used for removal of moisture in the compressed air.
Stroke volume
Clearance

These traps are manual drain cocks, timer based/


volume
Water jacket

Water jacket

automatic drain valves, etc. Piston

6. Air Receivers are cylindrical tanks into which


the compressed air is discharged after final stage BDC
of compression from the air compressor. Receiver
acts as storage tank and it helps to reduce pulsa-
tions and pressure variations from the compressed
in the discharge line.
Reciprocating Air Compressor 841

with the help of a connecting rod. The cylinder The double-acting air compressor is shown in
head consists of spring-loaded inlet and delivery Fig. 25.3. Its construction is very similar to that of
valves, which are operated by a small pressure a single-acting air compressor, except for two inlet
difference across them. The light spring pressure and two delivery valves on two ends of the cylinder
gives a rapid closing action. The piston rings seal in order to allow air entry and delivery on two sides
the gap between the piston and cylinder wall. The of the piston. When the piston compresses the air
cylinder is surrounded by a water jacket or metallic on its one side, it creates suction on the other side.
fins for proper cooling of air during compression. Thus, the suction and compression of air take place
on two sides of the piston simultaneously.

Water jacket
Induction
To receiver

Delivery or next stage


to receiver

Water
Water

air in the cylinder is compressed by piston as shown


Air Compressor by the curve 1-2.
During the compression stroke, as air pressure
As the piston moves in a downward stroke (from
reaches a value, which is slightly more than the
TDC to BDC), any residual compressed air left in
pressure of compressed air acting outside the
the cylinder from the previous cycle expands first.
delivery valve, the delivery valve opens and the
On further movement of the piston, the pressure in
compressed air is discharged from the cylinder
the cylinder falls below the atmospheric pressure.
to storage tank. At the end of the compression
The atmospheric air pushes the inlet valve to
stroke, the piston once again moves downward, the
open and fresh air enters the cylinder as shown
pressure in the cylinder falls below the atmospheric
in Fig. 25.4. The line c-1 represents the induction
pressure, the delivery valve closes and inlet valve
stroke. During this stroke, the compressed air in
opens for next cycle. The suction, compression and
the storage tank acts on the delivery valve, thus
delivery of air take place with two strokes of the
it remains closed. As the piston begins its return
piston which is one revolution of the crank.
stroke from BDC to TDC, the pressure in the
Figure 25.4 shows the p –V diagram for a
cylinder increases, and closes the inlet valve. The
reciprocating compressor without clearance. The
processes are summarized below:
842 Thermal Engineering

Process c–1 Suction stroke—inlet valve opens


and air enters the compressor at constant pressure
p1
Process 1–2 Polytropic compression of air from
pressure p1 to pressure p2
Process 2–d Discharge of compressed air through
delivery valve at const. pressure p2
Process d–c No air in the cylinder and return of
piston for suction stroke
= Area 2–d –0 –b–2 + Area 1–2 –b –a –1
– Area 1– c – 0 – a –1
These three areas are shown in Fig. 25.5 as (a),
The theoretical p –V diagram for single-stage,
(b) and (c), respectively.
single-acting reciprocating air compressor without
clearance is shown in Fig. 25.4. The net work done Area 2–d – 0 – b –2 = p2V2 (Flow work during
in the cycle is equal to the area behind the curve on discharge at constant
p –V diagram and it is the work done on air. pressure p2)
Indicated work done on the air per cycle
= Area behind the curve, i.e., area c –1–2–d–c
Ú
Area 1–2 –b –a –1 = – pdV (Piston dispalce-
ment work from p1 to p2, –
Reciprocating Air Compressor 843

sign is taken for compres- pm)


sion) It is a hypothetical average pressure, which if acted
Area 1–c – 0 – a –1 = p1V1 (Flow work during on the piston during the entire compression stroke
suction at constant pressure will require the same power input as required
p1) during the actual cycle.
During compression process 1–2; the pressure Net work input in a cycle,
and volume are related as Win = pm ¥ (Swept volume)
pV n = C (constant) = pm ¥ Vs
p V - p1V1
Ú
Thus we get – pdV = 2 2 W Work output
Thus pm = net = ...(25.10)
n -1 Vs Swept volume
Therefore, the total indicated work input to From a given indicator diagram the indicated
compressor is mean effective pressure can be obtained as
p V - p1V1 Area of indicator diagram (mm 2 )
Win = p2V2 + 2 2 – p1V1 ...(25.5) pm =
n -1 Length of the indicator diag ram (mm)
È 1 ˘ ¥ Spring constant (kPa/mm) ...(25.11)
= ( p2V2 – p1V1) Í - 1˙
Î n -1 ˚
n
Win = ( p2V2 - p1V1 ) (kJ/cycle) ...(25.6)
n -1
1. Indicated (IP) The work done on air
Using charecteristic gas equation as per unit time is called indicated power input to
pV = ma R T the compressor. The power required by an air
Equation (25.6) can be modified as compressor, running at N rpm is given as
n Indicated power IP
Win = ma R (T2 – T1) (kJ/cycle) ...(25.7)
n -1 = Work input per cycle ¥ No. of
Other expression for indicated work can be cycles per unit time
derived by arranging Eq. (25.7) as W Nk
or IP = in (kW) ...(25.12)
n ÈT ˘ 60
Win = ma R T1 Í 2 - 1˙ From an indicated diagram, It is calculated as
n -1 T
Î 1 ˚
IP = Indicated mean effective pressure
It is convenient to express the temperature T2 in ¥ Swept volume rate
terms of delivery and intake pressure ratio. p L AN k
n -1 = mi (kW) ...(25.13)
Êp ˆ n
60
T2 = T1 Á 2 ˜ where for Eq. (25.12) and Eq. (25.13);
Ë p1 ¯ Win = Indicated work input per cycle
È n -1 ˘ pmi = Indicated mean effective pressure, (kPa or
n ÍÊ p2 ˆ n ˙ kN/m2)
Then Win = ma R T1 ÍÁ ˜ - 1˙ (kJ/cycle)
n -1 Ë p1 ¯ L = Stroke length, (m)
Í ˙
Î ˚ ...(25.8) A = (p/4)d 2, cross-sectional area of cylinder of
È n -1 ˘ bore, d, (m)
n ÍÊ p2 ˆ n ˙ N = number of rotation per minute
or Win = p 1V1 ÍÁ ˜ - 1˙ (kJ/cycle)
n -1 Ë p ¯ k = number of suction per revolution of crank
Í 1 ˙
Î ˚ ...(25.9) shaft
where V1 is the volume inducted per cycle. = 1 for single-acting reciprocating compressor
= 2 for double-acting reciprocating compres-
sor
844 Thermal Engineering

2. Brake (BP) The actual power (brake The rate of work input to compressor, Eq. (25.7)
power or shaft power) input to the compressor is n
Win = ma R(T2 – T1)
more than the indicated power because some work n -1
is required to overcome the irreversibilities and 1.35
= ¥ (1.4 kg/min) ¥ (0.287 kJ/kg ◊ K)
mechanical frictional effects. 1.35 - 1
¥ (461.46 – 290) (K)
Brake power; = 265.72 kJ/min
BP = Indicated power + Frictional Indicated power input;
power ...(25.14) W ( 265.72 kJ/min)
IP = in = = 4.43 kW
hmech The mechanical 60 (60 s/min)
efficiency of the compressor is given by Note: The indicated work input to compressor can also
Indicated power be calculated by using Eq. (25.8).
hmech = ...(25.15)
Brake power
Example 25.2 A single-acting, single-cylinder re-
The brake power is derived from a driving motor ciprocating air compressor has a cylinder diameter of
or engine. The input of a driving motor can be 200 mm and a stroke of 300 mm. Air enters the cylinder
expressed as at 1 bar; 27°C. It is then compressed polytropically to
Shaft power (or brake power) 8 bar according to the law pV 1.3= constant. If the speed
Motor power = of the compressor is 250 rpm, calculate the mass of air
Mechanical efficiency of
compressed per minute, and the power required in kW for
motor annd drive
...(25.16) driving the compressor.

Example 25.1 A single-stage reciprocating air Solution


compressor takes in 1.4 kg of air per minute at 1 bar Given A single-acting, single-cylinder reciprocating
and 17°C and delivers it at 6 bar. Assuming compression air compressor
process follows the law pV1.35 = constant, calculate d = 200 mm = 0.2 m L = 300 mm = 0.3 m
indicated power input to compressor. p1 = 1 bar = 100 kPa p2 = 8 bar
N = 250 rpm T1 = 27°C = 300 K
Solution
n = 1.3
Given A single-stage reciprocating air compressor
To find
ma = 1.4 kg/min p1 = 1 bar
(i) The mass of air compressed in kg/min,
T1 = 17°C = 290 K p2 = 6 bar
(ii) Power input to compressor in kW.
n = 1.35 bar
Law pV1.35 = C Assumptions
(i) Negligible clearance volume in the cylinder.
To find Indicator power input to compressor.
(ii) Air as an ideal gas with R = 0.287 kJ/kg ◊ K.
Assumptions
Analysis The swept volume of the cylinder per cycle
(i) Negligible clearance volume in the compressor. Êpˆ
(ii) No throttling effects on valve opening and clos- Vs = V 1 = Á ˜ d 2 L
Ë 4¯
ing.
(iii) Air as an ideal gas with R = 0.287 kJ/kg ◊ K. Êpˆ
= Á ˜ ¥ (0.2 m) 2 ¥ (0.3 m)
Ë 4¯
Analysis The delivery temperature of air
= 9.424 ¥ 10–3 m3
n -1 1.35 -1 The mass of air, using perfect gas equation
Ê p2 ˆ n Ê 6 ˆ 1.35
T2 = T1 Á ˜ = ( 290 K ) ¥ Á ˜ p1V1 (100 kPa ) ¥ (9.424 ¥ 10 -3 m3 )
Ë p1 ¯ Ë 1¯ ma = =
RT1 (0.287 kJ/kg ◊ K ) ¥ (300 K )
= 461.46 K
= 0.0109 kg/cycle
Reciprocating Air Compressor 845

The mass flow rate of air;


1.25 Ê 20 ˆ
ma = mass of air ¥ number of suction/min = ma N = ¥ kg/s˜ ¥ (0.287 kJ/kg ◊ K)
1.25 - 1 ÁË 60 ¯
= 0.0109 ¥ 250 = 2.74 kg/min ¥ (425.4 – 303) (K)
Temperature of air after compression = 58.55 kW
n -1 The motor (brake) power
Êp ˆ n
T2 = T1 Á 2 ˜ BP =
IP
=
58.55 kW
= 73.18 kW
Ë p1 ¯ hmech 0.8
1.3-1
Ê 8ˆ 1.3
= (300 K ) ¥ Á ˜ = 484.75 K Example 25.4 A single-cylinder, double-acting, re-
Ë 1¯
ciprocating air compressor receives air at 1 bar; 17°C,
The work input to compressor, Eq. (25.7) compresses it to 6 bar according to the law pV1.25= con-
n stant. The cylinder diameter is 300 mm. The average pis-
Win = ma R (T2 – T1)
n -1 ton speed is 150 m/min at 100 rpm. Calculate the power
1.3 required in kW for driving the compressor. Neglect clear-
= ¥ (2.74 kg/min) ¥ (0.287 kJ/kg ◊ K) ance.
1.3 - 1
¥ (484.75 – 300) (K)
Solution
= 629.56 kJ/min or 10.49 kW
Given A double-acting, single-cylinder reciprocating
Example 25.3 A single-acting, single-cylinder recip- air compressor
rocating air compressor is compressing 20 kg/min. of air d = 300 mm = 0.3 m p1 = 1 bar = 100 kPa
from 110 kPa, 30°C to 600 kPa and delivers it to a receiv- p2 = 6 bar N = 100 rpm
er. Law of compression is pV1.25 = constant. Mechanical T1 = 17°C = 290 K n = 1.25
efficiency is 80%. Find the power input to compressor, k =2 Vpiston = 150 m/min
neglecting losses due to clearance, leakages and cooling.
To find Power input to compressor in kW.
Solution
Analysis The piston speed is given as
Given A single-stage reciprocating air compressor Vpiston = 2 LN
ma = 20 kg/min p1 = 110 kPa Stroke of piston;
T1 = 30°C = 303 K p2 = 600 kPa Vpiston 150 m/ min
Law pV 1.25 = C L = =
2N 2 ¥ (100 rotation/ min)
hmech = 0.8 = 0.75 m
To find Power input to compressor. The swept volume of the cylinder per cycle
Êpˆ p
Assumptions Vs = V1 = Á ˜ d 2 L = ¥ (0.3 m) 2 ¥ (0.7 5m)
Ë 4¯ 4
(i) Negligible clearance volume in the compressor.
(ii) No throttling effects on valve opening and closing. = 0.053 m3
(iii) Air as an ideal gas with R = 0.287 kJ/kg ◊ K. The indicated work input to compressor by Eq. (25.9)

Analysis The delivery temperature of air È n -1 ˘


p 1V1 ÍÍÊ p2 ˆ ˙
n n
n -1
Win =
ÁË p ˜¯ - 1˙
1.25 -1 n -1
Êp ˆ n Ê 600 ˆ 1.25 Í 1 ˙
T2 = T1 Á 2 ˜ = (303 K ) ¥ Á Î ˚
Ë p1 ¯ Ë 110 ˜¯
È 1.25 -1 ˘
Ê 6 ˆ 1.25
¥ 100 ¥ 0.053 ¥ ÍÍÁ ˜ ˙
= 425.4 K 1.25
= - 1˙
The indicated power input to compressor, 1.25 - 1 Ë 1¯
Í ˙
n Î ˚
IP = ma R (T2 – T1) = 11.42 kJ/cycle
n -1
846 Thermal Engineering

For a double-acting reciprocating compressor, the The mass flow rate of air per minute,
indicated power pV (100 kPa) ¥ (1 m3 )
ma = 1 =
Win N k RT1 (0.287 kJ/kg ◊ K) ¥ (290 K)
IP = (kW)
60 = 1.2 kg/min
11.42 ¥ 100 ¥ ( 2 for double acting) The temperature of air after compression
= n -1
60 1.35 -1
Êp ˆ n Ê 7 ˆ 1.35
= 38.1 kW T2 = T1 Á 2 ˜ = ( 290 K ) ¥ Á ˜
Ë p1 ¯ Ë 1¯
Example 25.5 A single-stage, single-acting, recip- = 480.28 K
rocating air compressor takes in 1 m3 air per minute at The rate of work input to compressor, Eq. (25.7)
1 bar and 17°C and delivers it at 7 bar. The compressor n
W = ma R (T2 – T1)
runs at 300 rpm and follows the law pV1.35= constant. n -1
Calculate the cylinder bore and stroke required, assum- 1.35
= ¥ 1.2 ¥ 0.287 ¥ 480.28 – 290
ing stroke-to-bore ratio of 1.5. Calculate the power of the 1.35 - 1
motor required to drive the compressor, if the mechanical = 252.77 kJ/min
efficiency of the compressor is 85% and that of motor Indicated power required;
transmissions is 90 %. Neglect clearance volume and ( 252.77 kJ/min)
take R = 0.287 kJ/kg ◊ K for air. IP = = 4.21 kW
(60 s/min)
Solution The brake power input to compressor;
IP ( 4.21 kW)
Given A single-stage, single-acting, reciprocating air Brake power = = = 4.956 kW
compressor hmech 0.85
3 The motor power required;
V = 1 m /min p1 = 1 bar = 100 kPa
T1 = 17°C = 290 K p2 = 7 bar Brake power ( 4.956 kW)
Motor power = =
N = 300 rpm n = 1.35 htransmission 0.9
htransmission = 0.9 hmech = 0.85 = 5.5 kW
L/d = 1.5 R = 0.287 kJ/kg ◊ K
To find
(i) Cylinder bore, and strokes,
(ii) Motor power.
The work done on the gas for compression can
Analysis Volume sucked in per cycle
be minimized when the compression process is
V 1 m3/min 1 executed in an internally reversible manner, i.e.,
Vs = = = m3
N k (300 rpm) ¥ 1 300 by minimizing the irreversibilities. The other way
The cylinder (swept) volume also given as, of reducing the compression work is to keep the
p 2 p Êpˆ specific volume of gas as small as possible during
Vs = d L = d 2 (1.5d ) = 1.5 ¥ Á ˜ d 3 compression process. It is achieved by keeping
4 4 Ë 4¯
Equating two equations the gas temperature as low as possible during
Êpˆ 1 the compression. Since specific volume of gas is
1.5 ¥ Á ˜ d 3 = proportional to temperature, therefore, the cooling
Ë 4¯ 300
arrangement is provided on the compressor to cool
We get
the gas during the compression.
cylinder bore,
d = 0.1414 m = 141.4 mm
For better understanding of the effect of
and stroke;
cooling during compression process, we consider
L = 1.5 d = 212.10 mm
three types of compression processes executed
Reciprocating Air Compressor 847

between same pressure levels ( p1 and p2); an For an isothermal process; using
isentropic compression 1–2≤ (involves no cooling), C
a polytropic compression 1–2 (involves partial p = (Since pV= C )
V
cooling) and an isothermal compression 1–2¢ V2 Ê V1 ˆ
(involves perfect cooling) as shown in Fig. 25.6. We get –
Ú
V1
pdV = p1V11n Á ˜
Ë V2 ¯
(a) The indicated compression work per cycle For isothermal process;
for a polytropic compression process 1–2 is
V1 p
given by Eq. (25.9) p2V2 = p1V1 and = 2
V2 p1
È n -1 ˘
n ÍÊ p2 ˆ n ˙ Êp ˆ
WPoly = p 1V1 ÍÁ ˜ - 1˙ \ Wiso = p1V11n Á 2 ˜ ...(25.18)
n -1 Ë p1 ¯ Ë p1 ¯
Í ˙
Î ˚
where V1 is the volume of the air inducted per cycle.
(b) Isentropic compression process 1–2≤ The three processes are plotted on a p–V diagram
An equation for indicated work input can be in Fig. 25.6 for same inlet state and exit pressure.
obtained as Eq. (25.9) by replacing n by g. The area of the indicator diagram is the measure of
That is, compression work. The only type of compression
g È g -1 ˘ can influence the magnitude of area of indicator
Wisentropic = p 1V1 ÍÊ p2 ˆ g - 1˙ diagram and length of the line 2–d.
g -1 ÍÁË p ˜¯ ˙
Í 1 ˙ It is interesting to observe from this diagram
Î ˚
that among the three processes considered, the area
...(25.17) with isentropic compression is maximum. Thus it
(c) Isothermal compression process 1–2¢: With requires maximum work input and with isothermal
perfect cooling (T2 = T1); compression, the area of indicator diagram is
Indicated work input for isothermal com- minimum. Thus, the compressor with isothermal
pression is given by area c–1–2¢– d – c. compression will require minimum work input.
Area c –1–2¢–d –c = Area a –1 –2¢– b + Area
b –2¢– d – 0 – Area a –1 – c– 0
V2
Wiso = – Ú
V1
pdV + p2V2 – p1V1 This term is seldom used in practice for reciprocating
compressors. The adiabatic efficiency of an air
p compressor is defined as the ratio of isentropic
Delivery 2¢ 2 2¢¢ work input to actual work input.
p2 pV n = C
d Isentropic work input
pV = C hadiabatic = ...(25.19)
Actual work input
Co
m
pr
es
sio pV g = C Compressor
n
It compares the indicated work input to isothermal
1 work input to the compressor and it is defined as
p1
c Suction ratio of isothermal work input to indicated work
b a
V1
V input
0 V2 Isothermal work input
hcomp = ...(25.20)
Indicated work input
848 Thermal Engineering

If the very high pressure ratio


It compares the actual work done on the gas with is required then air is compressed in stages.
isothermal compression work, and is defined as the The intercooler is used between two stages of
ratio of isothermal work input to actual work input compression for cooling of compressed air after
during compression, i.e., one stage, before entering the next stage. The
water jackets are also used around the cylinder of
Isothermal work input compressor of each stage.
hiso = ...(25.21)
Actual work input
If the com-
pressor has large surface to volume ratio, the great-
er surface area will be available for heat transfer
and cooling will be more effective.
As illustrated with the help of Fig. 25.6, the
compression of gas in isothermal manner requires It is possible by choosing a cylinder with large
minimum work input. With isothermal compression, bore and short piston stroke. The large cylinder
the temperature remains constant throughout the head dissipates heat in a much more effective way,
compression process. which contains hottest compressed air all the time.
T2 = T1 Example 25.6 A single-acting, single-stage recip-
Isothermal compression is only possible when all rocating air compressor of 250–mm bore and 350–mm
the heat generated during compression is dissipated stroke runs at 200 rpm The suction and delivery pres-
to cooling medium around the cylinder wall. It is sures are 1 bar and 6 bar, respectively. Calculate the
possible, when the compressor runs very slowly. theoretical power required to run the compressor under
In actual practice, the compression process each of the following conditions of compression:
should approach isothermal compression even with (a) isothermal,
high-speed compressors. The various methods are (b) polytropic n = 1.3, and
adopted to reduce the temperature of gas during the (c) isentropic, g = 1.4.
compression and keep it more closely to isothermal Neglect the effect of clearance and also calculate
compression. isothermal efficiency in each of the above cases.
It was an old method in which Solution
water was injected into the cylinder during
compression of air to keep the temperature of air Given A single-acting, single-stage reciprocating air
constant. But this method has certain disvantages, compressor:
and thus became obsolete. p1 = 1 bar = 100 kPa p2 = 6 bar
d = 250 mm = 0.25 m L = 350 mm
It is commonly and success- N = 200 rpm
fully used practice for all types of reciprocating and three types of compression with n = 1, 1.3 and 1.4
compressors. The water is circulated around the
To find
cylinder through the water jacket which helps to
(i) Theoretical power required to run the compressor
cool the air during compression.
for
For a small-capacity compressor, (a) Isothermal compression,
the effective cooling can be achieved by attaching (b) Polytropic compression,
fins of conducting material around the cylinder. The (c) Isentropic compression.
fins increase surface area of the cylinder for heat (ii) Isothermal efficiency in each case.
transfer.
Reciprocating Air Compressor 849

(ii) Isothermal efficiency


Isothermal work
hiso = ¥ 100
Actual work
(a) For isothermal compression,
hiso = 100%
(b) For polytropic compression,
10.261
hiso = ¥ 100 = 80.8%
12.7
(c) For isentropic compression,
10.261
hiso = ¥ 100 = 76.67%
13.38

Analysis The cylinder volume (without clearance)


Êpˆ Êpˆ
V1 = Vs = Á ˜ d 2 L = Á ˜ ¥ (0.25 m)2 ¥ (0.35 m)
Ë 4¯ Ë 4¯
The clearance volume is the space left in the
= 0.0171 m3 cylinder when the piston reaches its topmost
The volume flow rate of air position, i.e., TDC. It is provided
V1 = Cylinder volume ¥ No of suctions per second
200
(i) to avoid the piston striking the cylinder head,
= 0.0171 ¥ = 0.0572 m3/s and
60
(i) Theoretical power required (ii) to accommodate the valve’s actuation inside
(a) For isothermal compression, the cylinder, because suction and delivery
valves are located in the clearance volume.
Êp ˆ
Power Wiso = p1V1 ln Á 2 ˜
Ë p1 ¯ A compressor should have the smallest possible
Ê 6ˆ clearance volume, because the compressed air
= 100 ¥ 0.0572 ¥ ln Á ˜ left in the clearance volume, first re-expands in
Ë 1¯
= 10.261 kJ/s or 10. 261 kW
the cylinder during suction, thus reducing suction
(b) Polytropic compression,
capacity.
The ratio of clearance volume to swept volume
È n -1 ˘
n ÍÊ p2 ˆ n ˙ is defined as the clearance ratio or percentage
WPoly = p1V1 ÍÁ ˜ - 1˙
n -1 Ë p ¯ clearance. The value of clearance ratio may vary
Í 1 ˙
Î ˚ from 2 to 10%.
È 1.3-1 ˘
Ê 1.3 ˆ ÍÊ 6 ˆ 1.3 ˙
= Á ¥ 100 ¥ 0. 0572 ¥ - 1
Ë 1.3 - 1˜¯ ÍÁË 1 ˜¯ ˙
Í ˙ 1. The volume of air taken in per stroke is less
Î ˚
= 12.7 kW than the swept volume, thus the volumetric
(c) Isentropic compression, efficiency decreases.
È g -1 ˘
g ÍÊ p2 ˆ g ˙ 2. More power input is required to drive the
Wisentropic = p1V1 ÍÁ ˜ - 1˙
g -1 Ë p ¯ compressor for same pressure ratio, due to
Í 1 ˙
Î ˚ increase in volume to be handled.
È 1.4 -1 ˘ 3. The maximum compression pressure is
= Ê 1.4 ˆ ¥ 100 ¥ 0.0572 ¥ ÍÊ 6 ˆ 1.4 ˙
- 1˙
Á ˜
Ë 1.4 - 1¯ ÍÁ ˜ Ë 1¯ controlled by the clearance volume.
Í ˙
Î ˚
= 13.38 kW
850 Thermal Engineering

Similar to derivation of Eq. (25.9), the compres-


sion work equivalent to area 1–2–e–d, can be ob-
The clearance volume is generally kept very small. tained with index of compression nc.
The work done on the air in the clearance space Area 1–2–e–d;
during compression stroke is approximately equal È nc -1 ˘
to the work done by the air when it re-expands nc ÍÊ p2 ˆ nc ˙
WComp = p1V1 ÍÁ ˜ - 1˙ ...(25.22a)
during suction stroke. Therefore, the work of nc - 1 Ë p1 ¯
Í ˙
compression is not affected by clearance space in Î ˚
the compressor. But the mass of air inducted is The work done by gas during expansion (index
reduced, and thus the volumetric efficiency of the ne ) can be also obtained as above
compressor will be less. Area 3–e–d– 4;
Figure 25 . 8 shows an indicator diagram for a È ne -1 ˘
reciprocating air compressor with clearance. After ne ÍÊ p3 ˆ ne ˙
WExpan = p 4V4 ÍÁ ˜ - 1˙
delivery of compressed air, the air remaining in ne - 1 Ë p4 ¯
Í ˙
the clearance volume at pressure p2 expands, when Î ˚
the piston proceeds for the next suction stroke. As È ne -1 ˘
soon as the pressure p1 reaches at the state 4, the ne ÍÊ p2 ˆ ne ˙
induction of fresh charge starts and continues to the = p1V4 ÍÁ ˜ - 1˙ ...(25.22b)
ne - 1 Ë p ¯
Í 1 ˙
end of the stroke at state 1. Î ˚
Since p4 = p1 and p3 = p2
Net work of compression;
È nc -1 ˘
nc ÍÊ p2 ˆ nc ˙
Win = p1V1 ÍÁ ˜ - 1˙
nc - 1 Ë p1 ¯
Í ˙
Î ˚
È ne -1 ˘
ne ÍÊ p2 ˆ ne ˙
– p1V4 ÍÁ ˜ - 1˙ ...(25.23)
ne - 1 Ë p ¯
Í 1 ˙
Î ˚
If indices of compression and expansion are
same, i.e., nc = ne = n, then
È n -1 ˘
n Ê
Í 2 n
p ˆ ˙
Win = p1(V1 – V4) ÍÁ ˜ - 1˙ ...(25.24)
n -1 Ë p1 ¯
Í ˙
Î ˚

Example 25.7 An ideal single-stage, single-acting


The indicated work done is given by the area reciprocating air compressor has a displacement volume
1–2–3–4 –1 on a p–V diagram. of 14 litre and a clearance volume of 0.7 litre. It receives
the air at a pressure of 1 bar and delivers it at a pressure
Indicated work
of 7 bar. The compression is polytropic with an index of
= Area 1 –2–3–4–1 1.3 and re-expansion is isentropic with an index of 1.4.
= Area 1–2–e–d – Area 3– e–d–4 Calculate the net indicated work of a cycle.
Reciprocating Air Compressor 851

Solution È 1.4 -1 ˘
1.4 ÍÊ 7 ˆ 1.4 ˙
Given A single-stage, single-acting reciprocating air – ¥ 100 ¥ 0.00281 ¥ ÍÁË 1 ˜¯ - 1˙
1.4 - 1 Í ˙
compressor. Î ˚
p1 = 1 bar = 100 kPa p2 = 7 bar = 3.61 – 0.731 = 2.88 kJ/cycle
Vs = 14 litre Vc = 0.7 litre
Example 25.8 A single-stage, double-acting recipro-
nc = 1.3 ne = 1.4
cating air compressor takes in 14 m3 of air per minute
To find Indicated work input per cycle. measured at 1.013 bar and 15°C. The delivery pressure
is 7 bar and the compressor speed is 300 rpm. The com-
pressor has a clearance volume of 5% of swept volume
with a compression and re-expansion index of n = 1.3.
Calculate the swept volume of the cylinder, the delivery
temperature and the indicated power.

Solution
Given A single-stage, double-acting reciprocating air
compressor, with
3
V1 = 14 m /min T1 = 15°C = 288 K
N = 300 rpm n = 1.3
p1 = 1.03 bar = 101.3 kPa p2 = 7 bar
Vc = 0.05Vs
To find
(i) The swept volume of cylinder,
(ii) The delivery temperature, and
Analysis The total volume of cylinder; (iii) Indicated power.
V1 = Vs + Vc
Assumptions
= 14 + 0.7 = 14.7 litre or 0.0147 m3
(i) No throttling effect on valve opening and closing.
The volume V4 after re-expansion of compressed air
(ii) Effect of piston rod on underside of cylinder is
in clearance space
1 1
negligible.
Ê p ˆ ne Ê 7 ˆ 1.4 (iii) Air as an ideal gas with specific gas constant R =
V 4 = V3 Á 2 ˜ = 0.7 ¥ Á ˜ 0.287 kJ/kg ◊ K.
Ë p1 ¯ Ë 1¯
= 2.81 litre or = 0.00281 m3 Analysis The p –V diagram for given data of compressor
is shown in Fig. 25.10.
Indicated work input per cycle, Eq. (25.23)
È nc -1 ˘
nc ÍÊ p2 ˆ nc ˙
Win = p 1V1 ÍÁ ˜ - 1˙
nc - 1 ÍË p1 ¯ ˙
Î ˚
È ne -1 ˘
ne ÍÊ p2 ˆ ne ˙
– p1V4 ÍÁ ˜ - 1˙
1 - ne ÍË p1 ¯ ˙
Î ˚
È 1.3 -1 ˘
1.3 ÍÊ 7 ˆ 1.3 ˙
= ¥ 100 ¥ 0.0147 ¥ ÍÁ ˜ - 1˙
1.3 - 1 Ë 1¯
Í ˙
Î ˚
852 Thermal Engineering

(i) Swept volume of cylinder


The total volume of cylinder,
V1 = Vs + Vc = Vs + 0.05 Vs = 1.05 Vs The actual indicator diagram on a p–V plane for
The volume inducted per cycle a single-stage reciprocating air compressor is
Volume induction per minute shown in Fig. 25.11. It is similar to theoretical
V1 – V4 =
No. of suction per revolution ¥ one (Fig. 25.8) except for induction and delivery
No. of revolution per minute processes. The variation during these processes is
14 m3/min due to valve action effects.
=
2 suction per rev. ¥ 300 rev./min
= 0.0233 m3/cycle
The volume V4 after re-expansion of compressed
air.
1 1
Ê p ˆn Ê p ˆn
V 4 = V 3 Á 3 ˜ = Vc Á 2 ˜
Ë p4 ¯ Ë p1 ¯
1
Ê 7 ˆ 1.3
= 0.05 Vs ¥ Á
Ë 1.013 ˜¯
= 0.221 Vs
Then V1 – V4
= 1.05 Vs – 0.221 Vs = 0.829 Vs
or 0.829 Vs = 0.0233 m3/cycle
\ swept volume,
0.0233
Vs = = 0.0281 m3/cycle
0.829 There must be some pressure difference across
The swept volume of the cylinder is 0.0281 m3. the valves to operate them. During the suction
(ii) The delivery temperature of air process 4–1, the pressure drops in the cylinder
n -1 until the inlet valve is forced by atmospheric air to
Êp ˆ n
open. During the suction stroke, the piston creates
T2 = T 1 Á 2 ˜
Ë p1 ¯ vacuum in the cylinder. Thus pressure reduces and
1.3-1
Ê 7 ˆ 1.3
atmospheric air enters the cylinder.
= 288 ¥ Á
Ë 1.013 ˜¯ Similarly, during delivery process 2–3, some
= 450 K = 177°C more pressure is required to open the delivery valve
(iii) Indicated power and to displace the compressed air through narrow
valve passage. Thus, gas throttling takes place
È n -1 ˘
n ÍÊ p2 ˆ n ˙ during delivery, reducing the pressure gradually to
IP = p1 V ÍÁ ˜ - 1˙
n -1 Ë p ¯ the state 3.
Í 1 ˙
Î ˚ The waviness of lines during these processes
1.3 is due to valve bounce and wire drawing effect
= ¥ (101.3 kPa)
1.3 - 1 through the valves.
È 1.3 -1 ˘
Ê 14 ˆ ÍÊ 7 ˆ 1.3 ˙
¥ Á kg/s˜ ÍÁ - 1
Ë 60 ¯ Ë 1.013 ˜¯ ˙
Í ˙
Î ˚ Actual volume sucked into the cylinder during the
= 57.58 kW
suction stroke is always less than the swept volume.
Reciprocating Air Compressor 853

It is due to swept volume, and piston displacement volume.


(i) resistance offered by inlet valve to incoming The volumetric efficiency can be expressed in
air, terms of effective volume and piston displacement
volume as;
(ii) temperature of incoming air, and
V -V V + Vc - V4 V + Vc - V4
(iii) back pressure of residual gas left in the hvol = 1 4 = s = s
clearance volume. V1 - Vc Vs + Vc - Vc Vs
Vc V4 Vc
The volumetric efficiency, hvol of the air com- = 1+ - ¥ ...(25.27)
pressor is defined as the ratio of actual volume of Vs Vs Vc
air sucked into the compressor, measured at at- V
Introducing c = c as clearance ratio, and using
mospheric pressure and temperature to the piston Vc = V3, then Vs
displacement volume.
ÊV ˆ
In terms of mass ratio, the volumetric efficiency hvol = 1 + c – c Á 4 ˜ ...(25.28)
is defined as the ratio of actual mass of air sucked Ë V3 ¯
per stroke to the mass of air corresponding to piston For expansion of gas in clearance volume
displacement volume at atmospheric conditions. 1 1
V4 Ê p ˆ ne Ê p ˆ ne
hvol =
Actual mass sucked = Á 3˜ = Á 2˜
Mass corresponding to swept volume at V3 Ë p4 ¯ Ë p1 ¯
1
Atmmospheric pressure and temperature
Ê p ˆ ne
...(25.25) Then hvol =1+c–c Á 2˜ ...(25.29)
Ë p1 ¯
Effective swept volume
= ...(25.26) If index of expansion and index of compression
Piston displacement volume
are same, then
Figure 25.12 shows an indicator diagram for 1 1
a reciprocating air compressor showing effective Ê p2 ˆ ne Ê p ˆ nc V
ÁË p ˜¯ = Á 2˜ = 1
1 Ë p1 ¯ V2
ÊV ˆ
and hvol = 1 + c – c Á 1 ˜ ...(25.30)
Ë V2 ¯
The volumetric efficiency decreases with
Êp ˆ
pressure ratio Á 2 ˜ in the compressor, its
Ë p1 ¯
variation is shown in Fig. 25.13. The factors which
lower volumetric efficiency are the following:
Re-expansion
of residual compressed air in the clearance space
will reduce effective suction stroke (V1 – V4)
and therefore, the mass of fresh air entering into
the cylinder reduces and volumetric efficiency
decreases.

Obstruction due to
narrow valve passage causes throttling of air in
the cylinder. Throttling reduces the pressure in the
854 Thermal Engineering

following reasons:
1.0
Volumetric efficiency, hvol

1. Obstruction at inlet valve It offers the


0.8 resistance to air flow through the narrow
passage of valve.
0.6
2. Re-expansion of high pressure air in
0.4 clearance volume It reduces effective suction
stroke.
0.2
3. Presence of hot cylinder walls of compressor
Pressure ratio (p2/p1)
Air gets heated as it enters the cylinder.
Thus, it expands and reducing the mass of
air sucked into the cylinder.
In the actual indicator diagram as shown in
cylinder during intake stroke and discharge pressure
Fig. 25.11, the air is sucked at a pressure and
during delivery stroke, and thus the pressure ratio
temperature which are lower than that of free
in the cylinder decreases. It leads to reduced FAD
(atmospheric) air. Using the property relation for
and volumetric efficiency.
an ideal gas as
With high speed of pf Vf p (V - V )
the compressor, the pressure drop across the inlet = 1 1 4
Tf T1
valve and delivery valve increases. Further, the Then free air delivery (FAD)
temperature of compressed air increases due to p1 T f
less available time for cooling. Both these factors Vf = (V1 - V4 ) ...(25.31)
p f T1
reduce volumetric efficiency of the compressor.
where the suffix f denotes free (ambient) conditions
The heated cylinder while the suffix 1 indicates actual suction condi-
walls increase the temperature of the intake air. tions. Then volumetric efficiency with respect to
Thus, the specific volume of air increases which free air delivery;
will reduce FAD and volumetric efficiency of the Vf p1 T f Ê V1 - V4 ) ˆ
compressor. hvol, overall = =
V -V
1 c p T ÁË V - V ) ˜¯
f 1 1 c
Leakages across the
piston will reduce the vacuum during suction and È 1 ˘
p1 T f Í Ê p2 ˆ ne ˙
mass of compressed air above the piston. Both = 1+ c - c Á ˜ ...(25.32)
p f T1 Í Ë p1 ¯ ˙
these effects will increase compression work input Í ˙
Î ˚
and decrease in volumetric efficiency.
Example 25.9 A single-stage, single-acting recip-
rocating air compressor receives air at 1.013 bar, 27°C
and delivers it at 9.5 bar. The compressor has a bore =
The volume of compressed air corresponding 250 mm, and stroke = 300 mm and it runs at 200 rpm.
to atmospheric conditions is known as free air The mass-flow rate of air is 200 kg/h. Calculate the volu-
delivery (FAD). FAD is the volume of compressed metric efficiency of the compressor.
air measured in m3/min, reduced to atmospheric
pressure and temperature. Solution
The free air delivered volume is less than Given A single-stage, single-acting reciprocating air
the compressor displacement volume due to the compressor
Reciprocating Air Compressor 855

p1 = 1.03 bar = 101.3 kPa T1 = 27°C = 300 K pf = 1.03 bar = 101.3 kPa
N = 200 rpm p2 = 9.5 bar Tf = 27°C = 300 K
d = 250 mm = 0.25 m L = 300 mm = 0.3 m p2 = 7 bar
mact = 200 kg/h FAD, Vf = 14 m3/min
p1 = 0.95 bar = 95 kPa
To find The volumetric efficiency of the compressor.
T1 = 45°C = 318 K
Assumptions nc = ne= n = 1.3
(i) Neglecting clearance volume. Vc = 0.05 Vs
(ii) Specific gas constant of air, R = 0.287 kJ/kg ◊ K.
To find
Analysis The volume swept per cycle (i) Indicated power, and
Êpˆ (ii) Volumetric efficiency.
V1 = Á ˜ d 2 L
Ë 4¯ Assumptions
Êpˆ (i) The compression and expansion are reversible.
= Á ˜ ¥ (0.25 m) 2 ¥ (0.3 m)
Ë 4¯ (ii) Air as an ideal gas with R = 0.287 kJ/kg ◊ K.
= 0.0147 m3 Analysis The p–V diagram for given data is shown in
The mass of air inducted per cycle Fig. 25.14.
p1V1 (101.3 kPa) ¥ (0.0147 m3 )
ma = =
RT1 (0.287 kJ/kg ◊ K) ¥ (300 K)
= 0.0173 kg/cycle
The mass-flow rate per hour
ma = mass per cycle ¥ No. of suctions/
revolution ¥ No. of revolutions/h
= (0.0173 kg/cycle) ¥ (1 suction/rev)
¥ (200 ¥ 60 rev/h)
= 207.6 kg/h
The mass of air actually sucked, mact = 200 kg/h
Actual mass sucked
Thus hvol =
Mass corresponds to swept volume at
atmosspheric pressure and temperature
200 kg
= = 0.963 or 96.3%
207g.6 k
The mass-flow rate corresponding to FAD at
Example 25.10 A single-stage, double-acting recip- atmospheric conditions, pf and Tf ;
rocating air compressor has a FAD of 14 m3/min mea- pf Vf
sured at 1.013 bar and 27°C. The pressure and tempera- ma =
RT f
ture of the cylinder during induction are 0.95 bar and
101.3 ¥ 14
45°C. The delivery pressure is 7 bar and the index of = = 16.47 kg/min
0.287 ¥ 300
compression and expansion is 1.3. Calculate the indicat-
The temperature T2 after compression,
ed power required and volumetric efficiency. The clear-
n -1
ance volume is 5% of the swept volume.
Êp ˆ n
T2 = T 1 Á 2 ˜
Solution Ë p1 ¯
1.3 -1
Ê 7 ˆ 1.3
Given A single-stage, double-acting reciprocating air = (318 K) ¥ Á = 504.18 K
compressor Ë 0.95 ˜¯
856 Thermal Engineering

(i) The indicated power,


n
IP = ma R (T2 – T1)
n -1
1.3 Ê 16.47 ˆ
= ¥ kg/s˜ ¥ (0.287 kJ/kg ◊ K)
1.3 - 1 ÁË 60 ¯
¥ (504.18 K – 318 K)
= 63.56 kW
(ii) The volumetric efficiency can be obtained by
using Eq. (25.32)
hvol, overall
È 1˘
p1 T f Í Ê p2 ˆ n ˙
= Í1 + c - c Á ˜ ˙
p f T1 Í Ë p1 ¯ ˙
Î ˚
È 1 ˘ Analysis
0.95 ¥ 300 Í Ê 7 ˆ 1.3 ˙
= ¥ 1 + 0.05 - 0.05 ¥ Á
1.013 ¥ 318 Í Ë 0.95 ˜¯ ˙
(i) Free air delivery (FAD)
Í ˙ The swept volume;
Î ˚
= 0.723 or 72.3% Êpˆ
Vs = V1 – V3 = Á ˜ d 2 L
Ë 4¯
Example 25.11 A single-stage, single-acting recipro-
Êpˆ
cating air compressor has a bore of 20 cm and a stroke = Á ˜ ¥ (0.2 m) 2 ¥ (0.3 m)
Ë 4¯
of 30 cm. The compressor runs at 600 rpm. The clearance
volume is 4% of the swept volume and index of expan- = 0.009425 m3
sion and compression is 1.3. The suction conditions are Clearance volume
at 0.97 bar and 27°C and delivery pressure is 5.6 bar. Vc = 0.04 Vs = 0.04 ¥ 0.009425
The atmospheric conditions are at 1.01 bar and 17°C. = 3.77 ¥ 10–4 m3
Determine, Total volume
(a) The free air delivered in m3/min V1 = Vs + Vc
(b) The volumetric efficiency referred to the free air = 0.009425 + 3.77 ¥ 10–4
conditions
= 0.0098 m3
(c) The indicated power
The volume V4, after re-expansion of compressed
air in clearance space,
Solution
1 1
Given Single-stage, single-acting reciprocating air V4 Ê p ˆn Ê p ˆn
= Á 3˜ = Á 2˜
compressor V3 Ë 4¯
p Ë p1 ¯
d = 20 cm = 0.2 m L = 30 cm = 0.3 m 1
N = 600 rpm n = 1.3 –4 Ê 5.6 ˆ 1.3
V4 = 3.77 ¥ 10 ¥ Á
p1 = 0.97 bar = 97 kPa T1 = 27°C = 300 K Ë 0.97 ˜¯
pf = 1.01 bar = 101 kPa Tf = 17°C = 290 K = 0.00145 m3
Vc = 0.04 Vs p2 = 5.6 bar Effective swept volume,
V1 – V4 = 0.0098 – 0.00145 = 0.00835 m3
To find
The free air delivery per cycle,
(i) Free air delivery, m3/min,
p1 T f
(ii) Volumetric efficiency at ambient conditions; Vf = (V1 – V4)
hvol, overall, and p f T1
(iii) Indicated power.
Reciprocating Air Compressor 857

0.97 ¥ 290 p1 = 1 bar = 100 kPa p2 = 7 bar


= ¥ 0.00835
1.01 ¥ 300 N = 300 rpm T1 = 27°C = 300 K
= 0.00775 m3/cycle Vf = 2 m3/min pf = 1.03 bar = 103 kPa
Free air delivered per minute Vc = 0.05 Vs Tf = 20°C = 293 K
= Vf ¥ No. of cycle per minute hmech = 0.8 nc = 1.3, and
= 0.00775 ¥ 600 = 4.65 m3/min ne = 1.35
(ii) The volumetric efficiency referred to free air To find
conditions
(i) Volumetric efficiency,
Vf 0.00775
hvol, overall = = (ii) Indicated power, and
Vs 0.009425 (iii) Brake Power.
= 0.822 or 82.2%
(iii) Indicated power
Indicated work input using Eq. (25.24)

È n -1 ˘
n ÍÊ p ˆ n ˙
Win = p1 (V1 – V4) ÍÁ 2 ˜ - 1˙
n -1 ÍË p1 ¯ ˙
Î ˚
È 1.3 -1 ˘
1.3 ÍÊ 5.6 ˆ 1.3 ˙
= ¥ 97 ¥ 0.00835 ¥ ÍÁ -1˙
1.3 - 1 Ë 0.97 ¯˜
Í ˙
Î ˚
= 1.75 kJ/cycle
Since N cycles take place within a minute, the
indicated power
ÊNˆ
IP = Win Á ˜
Ë 60 ¯ Analysis
Ê 600 ˆ
= (1.75 kJ/cycle) ¥ Á cycle/s˜ (i) Volumetric efficiency
Ë 60 ¯
Clearance volume; Vc = V3 = 0.05 Vs
= 17.5 kW The volume V4 after re-expansion of compressed
air in clearance space
Example 25.12 A single-stage, double-acting air
1
compressor delivers air at 7 bar. The pressure and
Ê p ˆ ne
temperature at the end of the suction stroke are 1 bar V4 = V 3 Á 2 ˜
and 27°C. It delivers 2 m3 of free air per minute when the Ë p1 ¯
compressor is running at 300 rpm. The clearance volume 1
is 5% of the stroke volume. The pressure and temperature Ê 7 ˆ 1.35
= 0.05 Vs ¥ Á ˜ = 0.2113Vs
of the ambient air are 1.03 bar and 20°C. The index Ë 1¯
of compression is 1.3, and index of expansion is 1.35. The total volume of cylinder;
Calculate V1 = Vs + Vc = 1.05 Vs
(a) Volumetric efficiency of the compressor, Effective swept volume
(b) Indicated power of the compressor, V1 – V4 = 1.05 Vs – 0.2113 Vs = 0.8386 Vs
(c) Brake Power, if mechanical efficiency is 80%. The volumetric efficiency
V - V4 0.8386Vs
Solution hvol = 1 =
Vs Vs
Given A single-stage, double-acting reciprocating air = 0.8386 or 83.86%
compressor
858 Thermal Engineering

Free air delivered per cycle is given as


Ê 300 ˆ
p1 T f = (0.856 kJ/cycle) ¥ Á cycle/s˜ ¥ 2
Vf = (V1 – V4) Ë 60 ¯
p f T1
= 8.558 kW
Thus volume of air inducted per min at suction
(iii) The brake (shaft) power
condition
p f T1 Indicated power 8.558
V1 - V4 = V f Brake power = =
p1 T f hmech 0.8
(1.03 bar) ¥ (300 K) = 10.7 kW
= ( 2 m3/min) ¥
(1 bar) ¥ (293 K)
= 2.109 m3/min Example 25.13 A single-stage, double-acting recip-
It is the volume sucked per minute, which can rocating air compressor works between 1 bar and 10 bar.
also be expressed as The compression follows the law pV1.35 = constant. The
piston speed is 200 m/min and the compressor speed is
V1 - V4 = hvol Vs ¥ N ¥ No. of suction per
revolution 120 rpm. The compressor consumes an indicated power
of 62.5 kW with volumetric efficiency of 90%. Calculate
2.109 m3/min = 0.8386 Vs ¥ (300 rpm) ¥ 2
or Vs = 0.00419 m3 (a) diameter and stroke of the cylinder
Now V1 = 1.05 ¥ 0.00419 = 0.0044 m3 (b) Clearance volume as percentage of stroke volume
and V4 = 0.2113 ¥ 0.00419
Solution
= 8.856 ¥ 10–4 m3
(ii) Indicated power Given A single-stage, single-acting reciprocating air
Indicated work input per cycle, Eq. (25.23) compressor
p1 = 1 bar = 100 kPa p2 = 10 bar
È nc -1 ˘
nc ÍÊ p2 ˆ nc ˙ N = 120 rpm Vpiston = 200 m/min
Win = p 1V1 ÍÁ ˜ - 1˙
nc - 1 hvol = 0.9 IP = 62.5 kW
ÍË p1 ¯ ˙
Î ˚ and pV1.35 = C
È ne -1 ˘
ne ÍÊ p2 ˆ ne ˙ To find
– p1V4 ÍÁ ˜ - 1˙
1 - ne Ë p ¯ (i) Cylinder bore and piston stroke,
Í 1 ˙
Î ˚ (ii) Clearance ratio.
1.3
= ¥ 100 ¥ 0.0044 Analysis
1.3 - 1
(i) Cylinder bore and piston stroke
È 1.3 -1 ˘
ÍÊ 7 ˆ 1.3 ˙ The indicated work input per cycle for double
¥ ÍÁ ˜ - 1˙ acting compressor can be obtained as
Ë 1¯
Í ˙
Î ˚
Ê 60 ˆ
1.35 Win = IP s/cycle˜
– ¥ 100 ¥ 8.856 ¥ 10–4 ËÁ 2 N ¯
1.35 - 1
È 1.35 -1 ˘ Ê 60 ˆ
= (62.5 kJ/s) ¥ Á s/cycle˜
ÍÊ 7 ˆ 1.35 ˙ Ë 2 ¥ 120 ¯
¥ ÍÁ ˜ - 1˙
Ë 1¯
Í ˙ = 15.625 kJ/cycle
Î ˚
= 1.080 – 0.224 = 0.856 kJ/cycle Further,
È n -1 ˘
The indicated power input to compressor n ÍÊ p2 ˆ n ˙
Win = p 1(V1 – V4) ÍÁ ˜ - 1˙
ÊNˆ n -1 Ë
Í 1p ¯ ˙
IP = Win Á ˜ ¥ 2 (for double acting) Î ˚
Ë 60 ¯
Reciprocating Air Compressor 859

1.35 (b) the power required, if mechanical efficiency is


15.625 = ¥ (100 kPa) 85%,
1.35 - 1
(c) speed of the compressor.
È 1.35 -1 ˘
ÍÊ 10 ˆ 1.35 ˙
¥ (V1 – V4) ¥ ÍÁ ˜ - 1˙ Solution
Ë 1¯
Í ˙
Î ˚ Given A single-stage, single-acting reciprocating air
or (V1 – V4) = 0.0496 m3 compressor
The volumetric efficiency is given as p1 = 1 bar p2 = 6 bar
V1 - V4 ma = 0.6 kg/min T1 = 30°C = 303 K
hvol =
Vs L = 150 cm d = 100 mm
Stroke volume c = 0.03 n = 1.3
0.0496 m3 hmech = 0.85
or Vs = = 0.0551 m3
0.9 To find
The piston speed is given by
(i) Volumetric efficiency,
Vpiston = 2 LN
(ii) Power required by compressor, and
Stroke length; (iii) Speed of the compressor.
200 Assumptions
L = = 0.833 m or 833 mm
2 ¥ 120
(i) Suction takes place at free air conditions.
Further, the stroke volume can be expressed as
(ii) The specific gas constant for air R = 0.287
Êpˆ kJ/kg ◊ K.
Vs = Á ˜ d 2 L
Ë 4¯
Analysis
0.0551
Bore d = (i) The volumetric efficiency is given by
(p /4) ¥ 0.833
1
= 0.290 m or 290 mm Ê p ˆn
hvol = 1 + c – c Á 2˜
(ii) Clearance ratio Ë p1 ¯
The volumetric efficiency is given by 1
1 Ê 6 ˆ 1.3
Ê or = 1 + 0.03 – 0.03 ¥ Á ˜
p2 ˆ n Ë 1¯
hvol = 1 + c – c Á ˜
Ë p1 ¯ = 0.910 or 91.0%
1
(ii) Power required by compressor
Ê 10 ˆ 1.35
or 0.9 = 1 + c – c ¥ Á ˜ Indicated power input
Ë 1¯
È n -1 ˘
Clearance ratio; n ÍÊ p2 ˆ n ˙
c = 0.222 or 2.22% IP = ma R T1 ÍÁ ˜ - 1˙
n -1 Ë
Í 1p ¯ ˙
Î ˚
Example 25.14 A single-stage, single-acting recipro-
cating air compressor delivers 0.6 kg/min of air at 6 bar. 1.3 Ê 0.6 kg ˆ
= ¥ ¥ 0.287
The temperature and pressure at the suction stroke are 1.3 - 1 ÁË 60 s ˜¯
30°C and 1 bar, respectively. The bore and stroke are
È 1.3 -1 ˘
100 mm and 150 mm respectively. The clearance volume ÍÊ 6 ˆ 1.3 ˙
is 3% of the swept volume and index of expansion and ¥ 303 ¥ ÍÁ ˜ - 1˙
Ë 1¯
compression is 1.3. Determine, Í ˙
Î ˚
(a) the volumetric efficiency of compressor, = 1.929 kW
860 Thermal Engineering

Brake power required; d = 25 cm drod = 2.5 cm, n = 1.3


IP 1.929 T1 = Tf = 20°C = 293 K n = 1.3 cm
BP = = = 2.27 kW
hmech 0.85 For indicator diagram,
(iii) Speed of the compressor a1 = 7.6 cm2 a2 = 7.8 cm2
The volume-flow rate of air at suction conditions l = 6.75 cm k = 200 kPa/cm
ma RT1
V1 - V4 = To find
p1
(i) Overall efficiency,
0.6 ¥ 0.287 ¥ 303
= (ii) Isothermal efficiency,
100
(iii) Mechanical efficiency, and
= 0.5217 m3/min (iv) Volumetric efficiency.
Piston-displacement volume rate
Assumptions
V1 - V4 0.5217 m3/min
Vs = = (i) Suction takes place at free air conditions.
hvol 0.91
(ii) Negligible clearance volume on head and tail side
= 0.5733 m3/min of the cylinder.
(iii) Specific gas constant for air R = 0.287 kJ/kg ◊ K.
Êpˆ
Further, Vs = Á ˜ d 2 L N
Ë 4¯ Analysis The suction pressure corresponds to 745 mm
of Hg;
\ 0.5733
745
Êpˆ p1 = ¥ (101.3 kPa) = 99.3 kPa
= Á ˜ ¥ (0.1 m)2 ¥ (0.1 5m) ¥ N 760
Ë 4¯
Absolute delivery pressure,
Speed of compressor; p2 = p1 + pg2 = 99.3 + 700 = 799.3 kPa
N = 487 rpm Mass of air inducted per second

Example 25.15 A single-stage, double-acting recip- p1V f . kPa) ¥ (5.5 m3 /min)


(9 93
ma = =
rocating air compressor is driven by an electric motor RT f (0.287 kJ/kg ◊ K) ¥ ( 293 K)
consuming 40 kW, compresses 5.5 m3/min air according = 6.49 kg/min or 0.1082 kg/s
to the law pV1.3 = constant. It receives atmospheric air Temperature after polytropic compression
at 20°C and 745 mm of Hg barometer and delivers at a n -1
gauge pressure of 700 kPa. Calculate isothermal, volu- Êp ˆ n
T2 = T 1 Á 2 ˜
metric, mechanical and overall efficiencies for the follow- Ë p1 ¯
ing data from an indiactor diagram for head and tail end. 1.3 -1
Length of the indicator diagram = 6.75 cm Ê 793.3 ˆ 1.3
= (293 K) ¥ Á
Area of the head end = 7.6 cm2 Ë 93.3 ˜¯
Area of the tail end = 7.8 cm2 = 474.12 K
Spring scale = 200 kPa/cm Actual power input to air in the compressor;
The diameter of the piston and piston rod are 25 and n
Win = ma R(T2 – T1)
2.5 cm, respectively. The stroke length is 30 cm. The n -1
compress or runs at 300 rpm. 1.3
= ¥ 0.1082 ¥ 0.287 ¥ ( 474.12 - 293)
1.3 - 1
Solution
= 24.37 kW
Given A single-stage, double-acting reciprocating air
(i) Overall efficiency
compressor.
p1 = 745 mm of Hg pg2 = 700 kPa Win 24.37 kW
hOverall = =
N = 300 rpm V f = 5.5 m3/min BP 40 kW
= 0.6093 or 60.93%
BP = 40 kW L = 30 cm
Reciprocating Air Compressor 861

Isothermal power input to the compressor; Total displacement volume per minute from head
Êp ˆ and tail ends of the cylinder;
Wiso = p1V f ln Á 2 ˜ Vtotal = (A1 + A2) L N
Ë p1 ¯
= (0.049 + 0.0486) ¥ 0.3 ¥ 300
Ê 5.5 m3 ˆ Ê 793.3 ˆ = 8.78 m3/min
= (99.3 kPa) ¥ Á ˜ ¥ ln Á
Ë 60 s ¯ Ë 93.3 ˜¯ (iv) Volumetric efficiency
= 19.48 kW Vf 5.5 m3/min
hvol = =
(ii) Isothermal efficiency Vtotal 8.78 m3/min
Wiso 19.48 = 0.626 or 62.6%
hiso = = = 0.487 or 48.7%
BP 40
Example 25.16 During the overhauling of an old
For head end of the cylinder compressor, a distance piece of 9 mm thickness is
Cross-sectional area, inserted accidentally between the cylinder head and
Êpˆ Êpˆ cylinder. Before overhaul, the clearance volume was
A1 = Á ˜ d 2 = Á ˜ ¥ (0.25 m) 2 3 per cent of the swept volume. The compressor receives
Ë 4¯ Ë 4¯
2 atmospheric air at 1 bar and it is designed to deliver air
= 0.049 m
at a gauge pressure of 7 bar with a stroke of 75 cm. If
Indicated mean effective pressure;
the compression and re-expansion follow the law pV1.3 =
a1 7.6 cm 2 constant, determine the percentage change in
pm1 = k= ¥ ( 200 kPa/cm)
l 6.75 cm (a) volume of free air delivered,
= 225.18 kPa (b) power necessary to drive the compressor.
For tail end of the cylinder
Cross-sectional area, Solution

Êpˆ Given A reciprocating air compressor before and after


A2 = Á ˜ ÈÎd 2 - d 2p ˘˚
Ë 4¯ overhaul
p1 = 1 bar pg2 = 7 bar
Êpˆ
= Á ˜ ¥ ÈÎ(0.25 m) 2 - (0.025 m) 2 ˘˚ t = 9 mm Vc = 0.03Vs
Ë 4¯
L = 75 cm n = 1.3
= 0.0486 m2
Indicated mean effective pressure; To find
2 (i) Percentage change in volume of free air delivered,
a2 7.8 cm
pm2 = k= ¥ ( 200 kPa/cm) (ii) Percentage change in power necessary to drive
l 6.75 cm
the compressor.
= 231.11 kPa
Total indicated power input to head and tail sides Analysis The absolute pressure of delivered air
of the compressor p2 = p1 + pg2 = 1 bar + 7 bar = 8 bar
pm1 L A1 N pm 2 L A2 N The clearance space
IP = +
60 60 Before overhaul,
225.18 ¥ 0.3 ¥ 0.049 ¥ 300 Lc1 = 0.03L = 0.03 ¥ 75 = 2.25 cm
= After overhaul,
60
231.11 ¥ 0.3 ¥ 0.0486 ¥ 300 Lc2 = Lc1 + t = 2.25 cm + 0.9 cm
+
60 = 3.15 cm
or IP = 4.965 + 5.054 = 33.4 kW The clearance volume V4 after re-expansion of
(iii) Mechanical efficiency compressed air in clearance space
1 1
IP 33 .4 kW Ê p ˆn Ê 8 ˆ 1.3
hmech = = V4 = V 3 Á 2 ˜ = V 3 ¥ Á ˜ = 4.95Vc
BP 40 kW Ë p1 ¯ Ë 1¯
= 0.835 or 83.5%
862 Thermal Engineering

Using cylinder cross-section area A, before overhaul; Solution


V4 = 4.95 ¥ 2.25 A = 11.14 A cm3
Given A 4-cylinder, single-stage, double-acting recip-
After overhaul,
rocating air compressor
V4a = 4.95 ¥ 3.15 A = 15.59 A cm3
No. of cylinders = 4 k =2
Total volume of compressor
p1 = 1 bar = 100 kPa T1 = 27°C = 300 K
Before overhaul; 3
p2 = 7 bar V f = 30 m /min
V1 = Vs + A Lc1 = 75 A + 2.25A N = 300 rpm pf = 1 bar
= 77.25 A cm3 c = 0.04 Tf = 17°C = 290 K
After overhaul; hmech = 0.85 n = 1.32
V1a = Vs + A Lc2 = 75 A + 3.15A = 78.15 A cm3 L/d = 1.2
The effective suction volume
To find
Before overhaul;
(i) Volumetric efficiency,
V1 – V4 = 77.25 A cm3 – 11.14 A cm3
(ii) Indicated power,
= 66.11 A cm3
(iii) Size of motor (brake power), and
After overhaul; (iv) Bore of cylinder and piston stroke.
V1a – V4a = 78.15 A cm3 – 15.59 A cm3
= 62.56 A cm3 Analysis
(i) Percentage change in FAD (i) Volumetric efficiency
Clearance ratio;
66.11 A - 62.56 A
= ¥ 100 = 5.37% c = 0.04
66.11 A
The volumetric efficiency;
(ii) Indicator work input 1
È n -1 ˘ Ê p ˆn
ÍÊ p2 ˆ n ˙ hvol = 1+ c - c Á 2˜
n Ë p1 ¯
W1 = p1 (V1 – V4) ÍÁ ˜ - 1˙
n -1 ÍË p1 ¯ ˙ 1
Î ˚ Ê 7 ˆ 1.32
= 1 + 0.04 - 0.04 ¥ Á ˜
In calculation of indicated work input, all terms except Ë 1¯
(V1 – V4) will remain constant before and after overhaul. = 0.8653 or 86.53%
Therefore, percentage change in indicated work input (ii) Indicated power
will be equal to percentage change in FAD, i.e. % change The effective swept volume;
in work input = 5.37% pf Vf T
V1 - V4 = ¥ 1
Tf p1
Example 25.17 A 4-cylinder, single-stage, double
(1 bar) ¥ (30 m3/min) Ê 300 K ˆ
acting air compressor delivers air at 7 bar. The pressure = ¥Á
( 290 K) Ë 1 bar ˜¯
and temperature at the end of the suction stroke are 1 bar
and 27°C. It delivers 30 m3 of free air per minute when = 31.03 m3/min or 3
0.517 m /s
the compressor is running at 300 rpm. The pressure Indicated power;
and temperature of the ambient air are 1 bar and 17°C. È n -1 ˘
n ÍÊ p2 ˆ n ˙
The clearance volume is 4% of the stroke volume. The IP = p 1( V1 -V4 ) ÍÁ ˜ - 1˙
stroke-to-bore ratio is 1.2. The index of compression and n -1 Ë
Í 1 p ¯ ˙
expansion is 1.32. Calculate Î ˚
1.32
(a) Volumetric efficiency of the compressor, = ¥ (100 kPa)
1.32 - 1
(b) Indicated power of the compressor,
È 1.32 - 1 ˘
(c) Size of motor, if mechanical efficiency is 85%, 3 Í Ê 7 ˆ 1.32 ˙
¥ (0.517 m /s) ¥ ÍÁ ˜ - 1˙
(d) Cylinder dimensions. Ë 1¯
Í ˙
Î ˚
= 128.60 kW
Reciprocating Air Compressor 863

(iii) The motor (brake) power T1 = 20°C + 25°C = 45°C or 318 K


IP 128.6 p2 – pd = 150 kPa
BP = = = 151.3 kW hcomp = 0.85
hmech 0.85
hmech = 0.95
(iv) Cylinder dimensions
The piston displacement volume of one cylinder To find
can be obtained by using volumetric efficiency as
(i) Indicated power necessary to drive the
V1 - V4 compressor,
Vs =
No. of cylinders ¥ hvol (ii) Brake power, and
31.03 (iii) Cylinder dimesions for same bore and stroke size.
= = 8.965 m3/min
4 ¥ 0.8653
The piston displacement volume rate with L =
1.2 d; for double-acting cylinder can be expressed
as
Êpˆ
Vs = Á ˜ d 2 L N k
Ë 4¯
Êpˆ
= Á ˜ d 2 ¥ (1.2 d ) ¥ 300 ¥ 2
Ë 4¯
8.965
Bore d = 3
(p /4) ¥ 1.2 ¥ 300 ¥ 2
= 0.251 m or 251 mm
Stroke; L = 1.2 d = 301.4 mm

Example 25.18 A twin-cylinder single-stage, single-


acting reciprocating air compressor running at 300 rpm
has pressure ratio of 8. The clearance is 3 per cent of Analysis The suction pressure
the swept volume. It compresses 30 m3/min free air at
p1 = pf – 8 kPa = 101.3 – 8 = 93.3 kPa
101.3 kPa and 20°C according to pV1.3 = constant. The
temperature rise during suction stroke is 25°C . The loss Using the property relation for an ideal gas as
of pressure through intake and discharge valve is 8 and pf Vf p1 (V1 -V4 )
150 kPa, respectively. Determine =
Tf T1
(a) Indicated power required by the compressor,
(b) Brake power, assuming compression efficiency of It gives
85% and mechanical efficiency of 95% , pf Vf T1 101.3 ¥ 30 318
V1 -V4 = ¥ = ¥
(c) Cylinder dimesions for same bore and stroke size. Tf p1 293 93.3

Solution = 35.35 m3/min


(i) The indicated power input to the compressor
Given A single-stage, single-acting reciprocating air
compressor È n -1 ˘
n ÍÊ p2 ˆ n ˙
No. of cylinders = 2 c = 0.03 IP = p1(V1 - V4 ) ÍÁ ˜ - 1˙
n -1 Ë
Í 1p ¯ ˙
N = 300 rpm n = 1.3
3
Î ˚
V f = 30 m /min pf = 101.3 kPa
1.3 Ê 35.5 ˆ È 1.3 -1 ˘
Tf = 20°C = 293 K pf – p1 = 8 kPa = ¥ 93.3 ¥ Á ¥ Í(8) 1.3 - 1˙
p2 1.3 - 1 Ë 60 ˜¯ ÍÎ ˙˚
=8 L =d
p1 = 146.7 kW
864 Thermal Engineering

(ii) The brake power input to the compressor therefore, there is a decrease in fresh air
Brake power; induction.
Indicated power 146.7 2. With high delivery pressure, the delivery
BP = =
hcomp ¥ hmech 0.85 ¥ 0.95 temperature increases. It increases specific
= 181.67 kW volume of air in the cylinder, thus more
(iii) Cylinder dimesions for same bore and stroke size: compression work is required.
The volumetric efficiency is given as 3. Further, for high pressure ratio, the cylinder
1
size would have to be large, strong and heavy
Ê p ˆn
hvol = 1 + c – c Á 2˜ working parts of the compressor will be
Ë p1 ¯
needed. It will increase balancing problem
= 1 + 0.03 – 0.03 ¥ (8)1/1.3 and high torque fluctuation will require a
= 0.8814 heavier flywheel installation.
Further, the volumetric efficiency is also given as
(V1 - V4 ) All the above problems can be reduced to
hvol = minimum level with multistage compression.
Vs
\ total displacement volume for two cylinders;
(V1 - V4 ) 35.35
Vs = =
hvol 0.8814 As discussed in preceding sections, the compressor
3
= 40.1 m /min requires minimum work input with isothermal
For a two-cylinder, single-acting reciprocating air compression. But the delivery temperature T2
compressor, the displacement volume per minute increases with pressure ratio and the volumetric
is also expressed as efficiency decreases as pressure ratio increases.
Êpˆ All the above problems can be reduced to
Vs = 2 ¥ Á ˜ d 2 L N
Ë 4¯ minimum level by compressing the air in more than
For d = L; one cylinders with intercooling between stages, for
Êpˆ the same pressure ratio. The compression of air in
40.1 = 2 ¥ Á ˜ d 3 ¥ 300 two or more cylinders in series is called multistage
Ë 4¯
compression. Air cooling between stages provides
It gives bore and stroke sizes as
the means to achieving an appreciable reduction
d = L = 0.44 m or 440 mm
in the compression work and maintaining the air
temperature within safe operating limits.

Compression
Usually, the pressure ratio for a single-stage
1. The gas can be compressed to a sufficiently
reciprocating air compressor is limited to 7. Increase
high pressure.
in pressure ratio in a single-stage reciprocating
air compressor causes the following undesirable 2. Cooling of air is more efficient with
effects: intercoolers and cylinder wall surface.
3. By cooling the air between the stages of
1. Greater expansion of clearance air in the compression, the compression can be
cylinder and as a consequence, it decreases brought to isothermal and power input to the
effective suction volume (V1 – V4) and compressor can be reduced considerably.
Reciprocating Air Compressor 865

4. By multistaging, the pressure ratio of each


stage is lowered. Thus, the air leakage past
the piston in the cylinder is also reduced.
5. The low pressure ratio in a cylinder improves
volumetric efficiency.
6. Due to phasing of operation in stages,
in a multistage compressor, the negative
and positive forces are balanced to a large
extent. Thus, more uniform torque and better
mechanical balance can be achieved.
7. Due to low pressure ratio in stages, the
compressor speed could be higher for same
isothermal efficiency.
8. Low working temperature in each stage
helps to sustain better lubrication.
9. The low-pressure cylinder is made lighter
and high-pressure cylinders are made cylinder at intermediate pressure p2 and temperature
smaller for reduced pressure ratio in each T2, related with inlet pressure p1 and temperature T1
stage. as
n -1
T2 Êp ˆ n
= Á 2˜
T1 Ë p1 ¯
Figure 25.18 shows a schematic for two-stage
The air is then cooled in an intercooler, if
compression. The air at p1 and T1 is first drawn
intercooling is complete (perfect), the air will
into the first stage or low pressure (LP) cylinder.
enter the HP cylinder at the same temperature at
It is partially compressed to some intermediate
which it enters the LP cylinder. The second-stage
pressure, p2 and temperature T2 and is then
compression cycle in an HP cylinder is shown by
discharged to an intercooler which ideally cools
cycle 5–6 –7– 8 – 5. The line 1–2–9 represents the
the air to its initial temperature T1. The cooled air
single-stage compression from initial pressure p1 to
then enters the second stage or high pressure (HP)
delivery pressure p3. The shadded area 2–9–6–5–2
cylinder and is compressed to a delivery pressure
represents the saving in compression work obtained
p3 and temperature T3. The corresponding indicator
by intercooling.
diagram is shown on a p–V plane in Fig. 25.19.
The total indicator work
The cycle 1 –2 –3 – 4 –1 represents first-stage
compression cycle. The air is discharged from LP Win = WLP + WHP
È n -1 ˘
n ÍÊ p2 ˆ n ˙
= p1 (V1 – V4) ÍÁ ˜ - 1˙
n -1 Ë p1 ¯
Í ˙
Î ˚
È n -1 ˘
n ÍÊ p3 ˆ n ˙
+ p1 (V5 – V8) ÍÁ ˜ - 1˙
n -1 Ë p2 ¯
Í ˙
Î ˚
866 Thermal Engineering

In terms of mass of air inducted per cycle; temperature T1. Therefore, the heat rejected in
intercooler
È n -1 ˘
n ÍÊ p2 ˆ n ˙ Qcooling = ma Cp (T2 – T1) (kJ) ...(25.35)
Win = ma R T1 ÍÁ ˜ - 1˙
n -1 Ë p ¯ Using Cp = g Cv , we get
Í 1 ˙
Î ˚
g Cv - nCv (g - n)
È n -1 ˘ Cn = = Cv
n -1 n -1
n ÍÊ p3 ˆ n ˙
+ ma R T1 ÍÁ ˜ - 1˙ Then total heat rejected;
n -1 Ë p2 ¯
Í ˙
Î ˚ (γ − n )
n Qstage = ma C v (T2 – T1) + ma Cp (T2 – T1)
Win = ¥ n −1
n -1 ⎡ (γ − n ) ⎤
n -1 n -1
= ma ⎢ C v + C p ⎥ (T2 – T1) (kJ)
È ˘ ⎣ n − 1 ⎦
Ê
Í 2p ˆ n Ê p ˆ n ˙
ma R T1 ÍÁ ˜ +Á ˜3
- 2˙ (kJ/cycle) ...(25.36)
Ë p1 ¯ Ë p2 ¯
Í ˙ For heat rejected per kg of air
Î ˚
...(25.33) ⎡ (γ − n ) ⎤
qstage = ⎢ C v + C p ⎥ (T2 – T1) (kJ/kg)
pf Vf p1 (V1 - V4 ) p2 (V5 - V8 ) ⎣ n −1 ⎦
where, ma = = = ...(25.37)
RT f RT1 RT1
Since same mass of air is handled by both iagram for a
cylinders, the suffix f represents free air conditions.
For given mass-flow rate ma (kg/s), the The actual indicator diagram on a p–V plane for
indicated power a two-stage reciprocating air compressor is shown
È n -1 n -1 ˘ in Fig. 25.20. The variation during suction and
n ÍÊ p2 ˆ n Ê p3 ˆ n ˙ delivery processes is due to valve action effects.
IP = ma R T1 ÍÁ ˜ +Á ˜ - 2˙ (kW)
n -1 Ë p ¯ Ë p2 ¯ The indicator diagrams for low-pressure and
Í 1 ˙
Î ˚ high-pressure cylinders overlap due to pressure
...(25.34) drop taking place in intercooler and clearance
effect.
During the suction process, pressure drops in
Compression
the cylinder until the inlet valve is forced to open
In a two-stage air compressor, air rejects heat by air. Similarly, during delivery process , some
(i) during compression process, and (ii) after more pressure is required to open the delivery
compression in intercooler. valve and to displace the compressed air through
Qstage = Qcomp + Qcooling a narrow valve passage. Thus, gas throttling takes
Heat rejected during polytropic compression place during delivery, which reduces the pressure
process gradually.
Qcomp = ma Cn (T2 – T1) (kJ)
C - nCv
where Cn = p , the polytropic specific
n -1
heat, measured in kJ/kg ◊ K.
The intermediate pressure p2 influences the work
For perfect intercooling, the temperature of
to be done on the gas and its distribution between
air after cooling should be reduced to initial
Reciprocating Air Compressor 867

n -1
Ê 1ˆ n 1 n -1 1- 2 n
or ÁË p ˜¯ ( p2 )- n - ( p3 ) n ( p2 ) n =0
1
1- n 1 n -1 1- 2 n
or ( p1 )n ( p2 )- n = ( p3 ) n ( p2 ) n

1 2 n -1 n -1 n -1
- +
or ( p2 ) n n = ( p1 ) n ( p3 ) n
2( n -1) n -1
or ( p2 ) n = ( p1 p3 ) n
Therefore,
p22 = p1 p3 ...(25.38)
p2 p
or = 3 ...(25.39)
p1 p2
It is proved that for minimum compression
work, the conditions required are the following
stages. The intermediate pressure, which makes 1. The pressure ratio of each stage should be
work input minimum, is always important. the same.
The total power input to a two-stage reciprocating 2. The pressure ratio of any stage is the square
air compressor with complete intercooling is given root of overall pressure ratio, for a two stage
by Eq. (25.34); compressor.
È n -1 n -1 ˘ 3. Air after compression in each stage should
n ÍÊ p2 ˆ n Ê p3 ˆ n ˙
IP = ma R T1 ÍÁ ˜ +Á ˜ - 2˙ be cooled to intial temperature of air intake.
n -1 Ë p1 ¯ Ë p2 ¯
Í ˙ 4. The work input to each stage is same.
Î ˚
If p1, T1 and p3 are fixed then the optimum value Consider multistage compression with z stages.
of intermediate pressure p2 for minimum work Then
input can be obtained by applying condition of p2 p p
minima, i.e., = 3 = 4 =…
p1 p2 p3
d ( IP ) p ( z + 1)
=0 = = X (say) ...(25.40)
dp2
pz
È n -1 n -1 ˘ Then p2 = X p1; p3 = Xp2 = X 2p1;
n d ÍÊ p2 ˆ n Ê p3 ˆ n ˙
ma RT1 ÍÁ ˜ +Á ˜ - 2˙ = 0 p4 = Xp3 = X 2p2 = X 3p1
n -1 dp2 ÍË p ¯ Ë p ¯
1 2 ˙ and p(z + 1) = Xpz = … = X z p1
Î ˚
p
n -1
n -1 n -1 1- n
or X z = ( z + 1)
Ê 1ˆ n d d p1
ÁË p ˜¯ dp2
( p2 ) n + ( p3 ) n
dp2
( p2 ) n = 0
1 p ( z + 1)
n -1 or X= z
n -1 p1
Ê 1 ˆ n Ê n - 1ˆ
ÁË p ˜¯ (
ÁË n ˜¯ 2 np ) -1
= z (Pressure ratio through compressor)
1
n -1 Ê 1- n
1- n ˆ ...(25.41)
+ ( p3 ) n Á ( ) -1
˜ p2 n =0
Ë n ¯
868 Thermal Engineering

T1 = 20°C = 293 K T3 = 25°C = 298 K


p2 p3
= 7.0 = 6.0
p1 p2
Inserting Eq. (25.39) in Eq. (25.34), we get total n = 1.3 R = 0.287 kJ/kg ◊ K
minimum power,
To find Power input to compressor.
È n -1 ˘
Ê n ˆ ÍÊ p2 ˆ n ˙ Analyis The mass of air compressed per minute, using
IPmin = 2 ¥ Á m RT1 ÍÁ - 1˙
Ë n - 1˜¯ Ë p1 ˜¯
a perfect gas equation
Í ˙
Î ˚ p1V1
ma =
...(25.42) RT1
= 2 ¥ power required for one stage (101.3 kPa) ¥ ( 25 m3/min)
In terms of overall pressure ratio =
(0.287 kJ/kg ◊ K ) ¥ ( 293 K )
1
= 30.11 kg/min
p2 p1 p3 p3 Ê p3 ˆ 2
= = = Temperature of air after first-stage compression;
p1 p1 p1 ÁË p1 ˜¯ n -1 1.3 -1
Total minimum power; Êp ˆ n Ê 7ˆ 1.3
T2 = T1 Á 2 ˜ = ( 293 K ) ¥ Á ˜
n -1 Ë p1 ¯ Ë 1¯
È ˘
Ê n ˆ ÍÊ p3 ˆ 2 n ˙ = 459.08 K
IPmin = 2 ¥ Á ˜ ma RT1 ÍÁ ˜ - 1˙
Ë n - 1¯ Ë p ¯ The work input in LP cylinder compressor,
Í 1 ˙
Î ˚ IPLP =
n
ma R(T2 – T1)
...(25.43) n -1
1.3
This expression can be extended to z stages of = ¥ 30.11 ¥ 0.287 ¥ (459.08 – 300)
1.3 - 1
compression.Total minimum power;
= 6219.24 kJ/min or 103.65 kW
È n -1 ˘ Temperature of air after second-stage compression;
Ê n ˆ ÍÊ p( z +1) ˆ z n ˙
IPmin = z ¥Á m RT1 ÍÁ ˜ - 1˙ n -1 1.3-1
Ë n - 1˜¯
a
ÍË 1 ¯
p ˙ Êp ˆ
T4 = T3 Á 4 ˜
n Ê 6ˆ
= ( 298 K ) ¥ Á ˜
1.3
Î ˚ Ë p3 ¯ Ë 1¯
...(25.44) = 450.59 K
1 The work input in HP cylinder of compressor,
Ê pz +1 ˆ z n
where the pressure ratio in each stage = Á
Ë p1 ˜¯
IPHP = ma R(T4 – T3)
n -1

Example 25.19 Calculate the power required to


compress 25 m3/min atmospheric air at 101.3 kPa, 20°C
to a pressure ratio of 7 in an LP cylinder. Air is then cooled
at constant pressure to 25°C in an intercooler, before
entering HP cylinder, where air is again compressed to a
pressure ratio of 6. Assume polytropic compression with
n = 1.3 and R = 0.287 kJ/kg ◊ K.

Solution
Given A two-stage reciprocating air compressor with
imperfect inercooler;
V1 = 25 m3/min p1 = 101.3 kPa
Reciprocating Air Compressor 869

1.3 The density of incoming air


= ¥ 30.11 ¥ 0.287 ¥ (450.59 – 298)
1.3 - 1 p1 (100 kPa)
r1 = =
= 5714 kJ/min or 95.23 kW RT1 (0.287 kJ/kg ◊ K) ¥ (293 K)
Total power input to the compressor = 1.189 kg/m3
IP = IPLP + IPHP The mass-flow rate of air into LP cylinder
= 103.65 kW + 95.23 kW = 198.88 kW ma = r1 V1 = 1.189 ¥ 0.589 = 0.7 kg/s
Temperature of air after first-stage compression;
Example 25.20 The LP cylinder of a two-stage g -1 1.4 -1
double-acting reciprocating air compressor running Êp ˆ g Ê 3ˆ 1.4
at 120 rpm has a 50-cm diameter and 75-cm stroke. T2 = T1 Á 2 ˜ = ( 293 K ) ¥ Á ˜
Ë p1 ¯ Ë 1¯
It draws air at a pressure of 1 bar and 20°C and
= 401.04 K
compresses it adiabatically to a pressure of 3 bar. The air
is then delivered to the intercooler, where it is cooled at
constant pressure to 35°C and is then further compressed
polytropically (index n = 1.3) to 10 bar in HP cylinder.
Determine the power required to drive the compressor.
The mechanical efficiency of the compressor is 90% and
motor effieincy is 86%.

Solution
Given A two-stage, double-acting reciprocating air
compressor with imperfect intercooling;
N = 120 rpm d1 = 50 cm = 0.5 m
L = 75 cm = 0.75 m p1 = 1 bar
T1 = 20°C = 293 K p2 = 3 bar
p1V 1g = p2V 2g p2 = p3 = 3 bar
T3 = 35°C = 308 K p4 = 10 bar The IP input to LP cylinder,
p3V3n = p4 V4n with n = 1.3 k =2 g
IPLP = ma R (T2 – T1)
hmech = 0.9 hMotor = 0.86 g -1
To find Motor power input to drive the compressor. 1.4
= ¥ 0.7 ¥ 0.287 ¥ (401.04 – 293)
1.4 - 1
Assumptions
= 75.97 kW
(i) The effect of the piston rod is negligible on the Temperature of air after second-stage compression;
cylinder volume. n -1 1.3-1
(ii) For air: R = 0.287 kJ/kg ◊ K, Cp = 1.005 kJ/kg ◊ K Êp ˆ n Ê 10 ˆ 1.3
and g = 1.4. T4 = T3 Á 4 ˜ = (308 K ) ¥ Á ˜
Ë p3 ¯ Ë 3¯
Analysis The volume-flow rate of air to LP cylinder = 406.64 K
p 2 N The work input in HP cylinder of compressor,
V1 = d LP L k
n
4 60 IPHP = ma R (T4 – T3)
p Ê 120 ˆ n -1
= ¥ (0.5 m) 2 ¥ (0.7 5m) ¥ Á rps˜ ¥ 2 1.3
4 Ë 60 ¯ = ¥ 0.7 ¥ 0.287 ¥ (406.64 – 308)
1.3 - 1
= 0.589 m3/s
= 85.87 kW
870 Thermal Engineering

Total power input to the compressor È 1.35 -1 ˘


IP = IPLP + IPHP 1.35 ÍÊ 7 ˆ 1.35 ˙
= ¥ ma RT ¥ ÍÁ ˜ - 1˙
= 75.97 kW + 85.87 kW = 161.85 kW 1.35 -1 Ë 1 ¯
Í ˙
Motor power input Î ˚
= 2.53 m RT
IP 161.85
= = Percentage saving in power due to multistage com-
hmech hMotor 0.9 ¥ 0.86
pression
= 209.1 kW IPsingle - IPmulti 2.53 - 2.213
= ¥ 100 = ¥ 100
IPsingle 2.53
Example 25.21 Find the percentage saving in work
input by compressing air in two stages from 1 bar to = 12.56%
7 bar instead of one stage. Assume a compression index
Example 25.22 2 kg/s of air enters the LP cylinder
of 1.35 in both the cases and optimum pressure and
of a two-stage, reciprocating air compressor. The overall
complete intercooling in a two-stage compressor.
pressure ratio is 9. The air at inlet to compressor is at
Solution 100 kPa and 35°C. The index of compression in each
cylinder is 1.3. Find the intercooler pressure for perfect
Given A two-stage air compressor with perfect intercooling. Also, find the minimum power required for
intercooling compression, and percentage saving over single-stage
p1 = 1 r ba p3 = 7 bar n = 1.35 compression.
To find Saving in work in comparison with single-
Solution
stage compression.
Given A two-stage, single-acting, reciprocating air
Assumptions
compressor with perfect intercooling
(i) Single-acting reciprocating air compressor. ma = 2 kg/s p1 = 100 kPa
(ii) Compressions and expansions are reversible T1 = 35°C = 308 K p3 = 9 bar
processes. n = 1.3
(iii) No effect of valve opening and closing on
induction and delivery processes. To find
(i) Intermediate pressure,
Analysis Power required to drive compressor
(ii) Power required to drive the compressor, and
The minimum power input for two-stage compression
(iii) Percentage saving in work in comparison with
with perfect intercooling;
single-stage compression.
È n -1 ˘
Ê n ˆ ÍÊ p3 ˆ 2 n ˙ Assumptions
IPmulti = 2 ¥ Á ma RT1 ÍÁ ˜ - 1˙
Ë n -1˜¯ Ë
Í 1p ¯ ˙ (i) For air; R = 0.287 kJ/kg ◊ K and Cp = 1 kJ/kg ◊ K.
Î ˚
(ii) No effect of valve opening and closing on
È 1.35 -1 ˘ induction and delivery processes.
Ê 1.35 ˆ ÍÊ 7 ˆ 2 ¥1.35 ˙
= 2¥Á ¥ m RT ¥ ÍÁË 1 ˜¯ - 1˙
Ë 1.35 -1˜¯
a
Analysis
Í ˙
Î ˚
(i) For perfect intercooling
= 2.213 m RT
Power input with single-stage compression from 1 bar or p2 = p1 ¥ p3 = 1 ¥ 9 = 3 bar
to 7 bar; p2
or =3
È n -1 ˘ p1
n ÍÊ p3 ˆ n ˙
IPsingle = ma RT1 ÍÁ ˜ - 1˙ (ii) Power required to drive the compressor
n -1 Ë
Í 1p ¯ ˙
Î ˚ The minimum power input for two-stage
compression with perfect intercooling;
Reciprocating Air Compressor 871

È n -1 ˘ T1 = 300 K p3 = 10 bar
Ê n ˆ ÍÊ p3 ˆ 2 n ˙ n = 1.3 R = 0.287 kJ/kg ◊ K
IPmulti = 2¥Á ˜ ma RT1 ÍÁ ˜ - 1˙
Ë n -1¯ ÍË p1 ¯ ˙ Cp = 1 kJ/kg ◊ K k =1
Î ˚
To find
È 1.3 -1 ˘
Ê 1.3 ˆ ÍÊ 9 ˆ 2 ¥1.3 ˙ (i) Power required to drive the compressor,
= 2¥Á ¥ 2 ¥ 0. 287 ¥ 308 ¥ ÍÁË 1 ˜¯ -1˙
Ë 1.3 -1˜¯ Í ˙ (ii) Saving in work in comparison with single-stage
Î ˚ compression,
= 442.1 kW
(iii) Isothermal efficiency, and
(iii) Percentage saving in work of comparison with
(iv) Heat transferred in intercooler.
single-stage compression
Power input with single stage compression from Assumptions
1 bar to 9 bar; (i) Given conditions leads to perfect intercooling.
È n -1 ˘ (ii) Compressions and expansions are reversible
n ÍÊ p3 ˆ n ˙ processes.
IPsingle = n -1 ma RT1 ÍÁ p ˜ - 1˙
Ë
Í 1 ¯ ˙ (iii) No effect of valve opening and closing on
Î ˚ induction and delivery processes.
È 1.3 -1 ˘
1.3 ÍÊ 9 ˆ 1.3 ˙ Analysis For perfect intercooling
= ¥ 2 ¥ 0.287 ¥ 308 ¥ ÍÁ ˜ - 1˙
1.3 -1 Ë 1¯ or p2 = p1 ¥ p3 = 1 ¥ 10 = 3.162 bar
Í ˙
Î ˚ p2
or = 3.162
= 505.92
kW p1
Saving in power due to multistage compression (i) Power required to drive compressor
= IPsingle – IPmulti = 505.92 – 442.1 The minimum power input for two-stage
= 63.82 kW compression with perfect intercooling;
Per cent saving È n -1 ˘
IPmulti - IPsingle Ê n ˆ ÍÊ p3 ˆ 2 n ˙
63.82 IPmulti = 2¥Á ma RT1 ÍÁ ˜ - 1˙
=
IPsingle
¥ 100 =
505.92
¥ 100 Ë n -1˜¯ ÍË p1 ¯ ˙
Î ˚
= 12.61% Ê 1.3 ˆ
= 2¥Á ¥ 0.1 ¥ 0.287 ¥ 300
Ë 1.3 -1˜¯
Example 25.23 A two-stage, single-acting, recipro-
cating air compressor takes in air at 1 bar and 300 K. È 1.3 -1 ˘
ÍÊ 10 ˆ 2 ¥1.3 ˙
Air is discharged at 10 bar. The intermediate pressure ¥ ÍÁ ˜ - 1˙
Ë 1¯
is ideal for minimum work and perfect intercooling. The Í ˙
Î ˚
law of compression is pV1.3 = constant. The rate of dis- = 22.7 kW
charge is 0.1 kg/s. Calculate Power input with single-stage compression from
(a) power required to drive the compressor, 1 bar to 10 bar;
(b) saving in work in comparison with single stage È n -1 ˘
compression, n ÍÊ p3 ˆ n ˙
IPsingle = ma RT1 ÍÁ ˜ - 1˙
(c) isothermal efficiency, n -1 ÍË p1 ¯ ˙
(d) heat transferred in intercooler. Î ˚
1.3
Take R = 0.287 kJ/kg ◊ K and Cp = 1 kJ/kg ◊ K. = ¥ 0.1 ¥ 0.287 ¥ 300
1.3 -1
Solution È 1.3 -1 ˘
ÍÊ 10 ˆ 1.3 ˙
¥ ÍÁ ˜ - 1˙
Given A two-stage, single-acting, reciprocating air Ë 1¯
Í ˙
compressor with perfect intercooling Î ˚
ma = 0.1 kg/s p1 = 1 bar = 100 kPa = 26.16 kW
872 Thermal Engineering

(ii) Saving in power due to multistage compression


= IPsingle – IPmulti = 26.16 – 22.7
= 3.46 kW
Isothermal power input,
Êp ˆ
IPiso = ma RT1 ln Á 3 ˜
Ë p1 ¯
Ê 10 ˆ
= 0.1 ¥ 0.287 ¥ 300 ¥ ln Á ˜
Ë 1¯
= 19.825 kW
(iii) Isothermal efficiency;
IPiso 19.825
hiso = =
IPact 22.7
= 0.8733 or 87.33%
Temperature after first-stage compression
n -1 1.3 -1
Êp ˆ n Ê 3.162 ˆ 1.3 Analysis For perfect intercooling
T2 = T1 Á 2 ˜ = (300 K ) ¥ Á
Ë p1 ¯ Ë 1 ˜¯ 1 1
p2 p p Ê p ˆ 3 Ê 2000 ˆ 3
= 391.3
K = 3 = 4 = Á 4˜ = Á = 2.732
p1 p2 p3 Ë p1 ¯ Ë 98 ˜¯
(iv) Heat rejection rate in the intercooler
Q = ma Cp (T2 – T1) (i) Power input to compressor
= ¥ 0.1
1.0 ¥ (391.3 – 300)
È n -1 ˘
= 9.13 kW Ê n ˆ ÍÊ p3 ˆ 3 n ˙
IP3, stage = 3¥Á p V
1 1 ÍÁ ˜¯ - 1˙
Ë n -1˜¯ Ë
Í 1p ˙
Example 25.24 In a three-stage compressor, air is
Î ˚
compressed from 98 kPa to 20 bar. Calculate for 1 m3 of
air per second, Ê 1.3 ˆ
= 3¥Á ¥ (98 kPa )
(a) Work under ideal condition for n = 1.3, Ë 1.3 -1˜¯
(b) Isothermal work,
(c) Saving in work due to multi staging, È 1.3 -1 ˘
3 ÍÊ 2000 ˆ 3 ¥1.3 ˙
¥ (1 m /s) ¥ ÍÁ - 1
Ë 98 ˜¯
(d) Isothermal efficiency. ˙
Í ˙
Î ˚
Solution
= 332.62 kW
Given An ideal three-stage reciprocating air compres-
sor (ii) Isothermal work
p1 = 98 kPa p2 = 20 bar = 2000 kPa Êp ˆ
n = 1.3 3 IPiso = p1V1 ln Á 4 ˜
V1 = 1 m /s Ë p1 ¯
To find
Ê 2000 ˆ
= (98 kPa) ¥ (1m3/s) ¥ ln Á
(i) Power input to compressor, Ë 98 ˜¯
(ii) Isothermal work,
(iii) Saving in work due to multi-staging, and = 295.56 kW
(iv) Isothermal efficiency. Power required in single-stage compression,

Assumptions È n -1 ˘
Ê n ˆ ÍÊ p4 ˆ n ˙
IP1, stage = Á p V
1 1 ÍÁ - 1˙
(i) Neglecting clearance volume. Ë n -1˜¯ ÎË p1
˜¯
˚
(ii) Perfect inetercooling.
Reciprocating Air Compressor 873

È 1.3 -1 ˘ Assumptions
Ê 1.3 ˆ ÍÊ 2000 ˆ 1.3 ˙ (i) Air as an ideal gas, with R = 0.287 kJ/kg ◊ K.
= Á ¥ 98 ¥ 1 ¥ ÍÁË 98 ˜¯ - 1˙
Ë 1.3 -1˜¯ Î ˚ (ii) Compression and expansion are reversible
polytropic.
= 427.08 kW
(iii) Saving in work due to multistaging Analysis The stroke (swept) volume of LP cylinder
= IP1,stage – IP3,stage Êpˆ Êpˆ
Vs = Á ˜ d12 L1 = Á ˜ ¥ (0.25 m) 2 ¥ (0.25 m)
= 427.08 kW – 332.62 kW = 94.42 kW Ë 4¯ Ë 4¯
Percentage saving; = 0.01227 m3
Saving 94.42
= ¥ 100 = ¥ 100
IP1,stage 427.08
= 22.1%
(iv) Isothermal efficiency
IPiso 295.56
hiso = ¥ 100 = ¥ 100
IPact 332.62
= 88.85%

Example 25.25 A two-stage, single-acting recipro-


cating air compressor has an LP cylinder bore and stroke
of 250 mm each. The clearance volume of a low-pressure
cylinder is 5% of the stroke volume of the cylinder. The
intake pressure and temperature are 1 bar and 17°C, re-
spectively. Delivery pressure is 9 bar and the compressor
runs at 300 rpm. The polytropic index is 1.3 throughout. The volumetric efficiency of LP cylinder is given by
1
The intercooling is complete and intermediate pressure is
Ê p ˆn
3 bar. The overall efficiency of the plant including electric hvol, LP = 1 + c 1 – c1 2
ÁË p ˜¯
driving motor is 70%. Calculate 1 1
(a) the mass-flow rate through the compressor, and Ê 3 ˆ 1.3
= 1 + 0.05 – 0.05 ¥ Á ˜ = 0.933
Ë 1¯
(b) energy input to electric motor.
It is also expressed as
Solution V1 -V4
hvol, LP =
Given Two-stage, single-acting reciprocating air com- Vs
pressor or V1 – V4 = hvol, LP Vs = 0.933 ¥ 0.01227
LP cylinder: d1 = 250 mm = 0.25 m = 0.01145 m3
L1 = 250 mm = 0.25 m The mass of air inducted per cycle into LP cylinder
c1 = 0.05 Vs or c = 0.05 p (V - V )
ma = 1 1 4
p1 = 1 bar = 100 kPa RT1
T1 = 17°C = 290 K (100 kPa) ¥ (0.01145 m3 )
p2 = 3 bar p3 = 9 bar or ma =
(0.287 kJ/kg ◊ K) ¥ (290 K)
n = 1.3 k =1
= 0.0137 kg/cycle
N = 300 rpm hOverall = 0.7
(i) Mass-flow rate of air
To find The mass flow rate of air per minute
(i) The mass-flow rate of air through the compressor, ma = ma N k
(ii) Power input to electric motor.
874 Thermal Engineering

= 0.0137 ¥ 300 ¥ 1 = 4.13 kg/min Introducing volumetric efficiency of respective


= 0.069 kg/s cylinders;
(ii) Motor power input p1hvol 1Vs1 = p2 hvol 2 Vs2
The indicated power input to the compressor with = p3hvol3 Vs3 = … ...(25.47)
1/ 2 Piston displacement volume as
p2 Ê p3 ˆ
=
p1 ÁË p1 ˜¯ Êpˆ
Vs = Á ˜ d 2 L , then
È n -1 ˘ Ë 4¯
2n ÍÊ p2 ˆ n ˙ Êpˆ Êpˆ
IP = ma RT ÍÁ ˜ - 1˙ p1hvol 1 Á ˜ d12 L1 = p2 hvol 2 Á ˜ d22 L2
n -1 Ë
Í 2p ¯ ˙ Ë 4¯ Ë 4¯
Î ˚
2 ¥ 1.3 Êpˆ
= ¥ 0.069 ¥ 0.287 ¥ 290 = p3 hvol 3 Á ˜ d32 L3 = …
1.3 -1 Ë 4¯
È 1.3 -1 ˘ ...(25.48)
¥ ÍÊ 3 ˆ 1.3 - 1˙ Usually, the stroke length for all cylinders is
ÍÁË 1 ˜¯ ˙
Î ˚ same, i.e.,
= 14.32 kW
L1 = L2 = L3 = …
Motor power
\ p1hvol1 d12 = p2 hvol2 d22 = p3h vol3 d 32
IP 14.32
= = = 20.46 kW =… ...(25.49)
hOverall 0.7
If all cylinders have same clearance ratio, then
hvol 1 = hvol2 = hvol3 = …
and p1d12 = p2 d22 = p3 d32 = … ...(25.50)
Equations (25.49) and (25.50) are used to
In an air compressor, the mass of air inducted calculate the cylinder dimension for multi-stage
through each cylinder is same. Therefore, compressors.
m1 = m2 = m3 = … Example 25.26 A two-stage, single-acting, recip-
or r1 (V1 – V4) = r2 (V5 – V8) = r3 (V9 – V12) rocating air compressor with complete intercooling re-
=… ...(25.45) ceives atmospheric air at 1 bar and 15°C, compresses it
where (V1 – V4), (V5 – V8) and (V9 – V12) represent polytropically (n = 1.3) to 30 bar. If both cylinders have
effective suction volume per cycle taken in LP, the same stroke, calculate the diameter of the HP cylin-
der, if the diameter of the LP cylinder is 300 mm.
intermediate, and HP cylinders, respectively and
r1, r2, r3 are corresponding densities of air. The Solution
density of air can be expressed as
Given Two-stage, single-acting reciprocating air com-
p
r= pressor
RT
LP cylinder: d1 = 300 mm p1 = 1 bar
p1 p
\ (V1 - V4 ) = 2 (V5 - V8 ) T1 = 15°C = 288 K T3 = 15°C = 288 K
RT1 RT3 p3 = 30 bar n = 1.3 k = 1
p
= 3 (V9 - V12 ) = … To find Diameter of HP cylinder.
RT5
For perfect intercooling, isothermal conditions Assumptions
prevail, i.e., (i) Compression in both cylinders is reversible.
T1 = T3 = T5 = … (ii) Negligible clearance in both cylinders.
\ p1 (V1 – V4) = p2 (V5 – V8) = p3 (V9 – V12) (iii) No effect of valve opening and closing on
=… ...(25.46) induction and delivery processes.
Reciprocating Air Compressor 875

compresses it isentropically in the LP cylinder to inter-


mediate pressure, where air cools to its initial tempera-
ture and then again compresses polytropically (n = 1.3)
in the HP cylinder. The clearance volume and pressure
ratio in both cylinders are 5% of the swept volume and 2,
respectively. Determine the stroke volume of HP cylinder
for 60 litre swept volume of LP cylinder.

Solution
Given Two-stage, single-acting reciprocating air com-
pressor
Vs,LP = 60 litres V3 = 0.05 Vs,LP = 3 litres
T1 = 15°C = 288 K
T3 = 15°C = 288 K
p2 p3
Analysis For perfect intercooling, the pressure ratio = 2.0 = 2.0
p1 p2
per stage
V7 = 0.05 Vs, HP n = 1.3 k=1
p2 p
= 3 = p1 ¥ p3 = 1¥ 30 = 5.477
p1 p2 To find Stroke volume (V5 – V7) of HP cylinder.
p2 = 5.477 p1 = 5.477 ¥ 1 = 5.477 bar
The total volume of LP cylinder
Êpˆ
V1 = Vs, LP = Á ˜ d12 L
Ë 4¯
The total volume of HP cylinder
Êpˆ
V3 = Vs, HP = Á ˜ d22 L
Ë 4¯
Equation (25.46), for perfect intercooling, without
clearance leads to
p1V1 = p2 V3
Ê p1 ˆ 1
or Vs,HP = Vs,LP Á ˜ = Vs,LP ¥
Ë p2 ¯ 5.477
= 0.1825 Vs,LP
Without clearance, with same pressure ratio and with Assumptions
perfect intercooling, the volumetric efficiency will be
(i) For air g = 1.4.
same. Expressing the above equation in terms of cylinder
(ii) For LP cylinder, the re-expansion of air is
diameters
isentropic.
Êpˆ 2 Êpˆ 2 (iii) For HP cylinder, the re-expansion of air is
ÁË 4 ˜¯ d HP L = 0.1825 ¥ ÁË 4 ˜¯ ¥ (300 mm) ¥ L polytropic.

or dHP = 0.1825 ¥ (300 mm 2 ) Analysis The total volume of LP cylinder


V1 = Vs, LP + V3 = 60 lit + 3 lit = 63 litres
= 128.18 mm
The volume of air after first-stage compression
Example 25.27 A two-stage, single-acting, recipro- 1
Ê p ˆg
cating air compressor receives atmospheric air at 15°C, V2 = V1 Á 1 ˜
Ë p2 ¯
876 Thermal Engineering

1 Stroke to bore ratio for LP cylinder


Ê 1 ˆ 1.4 = 1:2
= (63 lit ) ¥ Á ˜ = 38.4 litres
Ë 2¯ Stroke of HP cylinder = Stroke of LP cylinder
The temperature of air after first-stage compression Calculate
g -1 1.4 -1 (a) Cylinder diameters,
Ê p2 ˆ g Ê 2 ˆ 1.4
T2 = T1 Á ˜ = ( 288 K ) ¥ Á ˜ (b) Power input, neglecting clearance volume.
Ë p1 ¯ Ë 1¯
= 351 K Solution
The total volume of HP cylinder
Given Two-stage, single-acting reciprocating air com-
V5 = Vc,HP + Vs,HP = Vc,HP + 20 Vc,HP pressor k = 1
= V7 + 20 V7 = 21V7 FAD = V1 = 6 m3/min p1 = 1 bar = 100 kPa
The volume of air after re-expansion in HP cylinder T1 = 27°C = 300 K p2 = 6 bar
1 1 p3 = 30 bar T3 = 27°C = 300 K
Ê p ˆn Ê 2 ˆ 1.3
V8 = V7 Á 7 ˜ = V7 ¥ Á ˜ = 1.704 V7 n = 1.3 N = 300 rpm
Ë p8 ¯ Ë 1¯
hmech = 0.85
In intercooling, air is cooled to initial temperature, LP cylinder: L1/d1 = 1.2
and volume of air reduces from (V2 – V3) to (V5 – V8) at and L2 = L1
constant pressure. Therefore, and Negligible clearance
V2 - V3 V -V
= 5 8 To find
T2 T1
(i) Cylinder dimensions,
T1 288
or V5 – V8 = (V2 - V3 ) = (38.4 - 3) ¥ (ii) Power input to compressor.
T2 351
Assumptions
= 29.046 litres
(i) Air as an ideal gas, with R = 0.287 kJ/kg ◊ K.
or 21 V7 – 1.704 V7 = 29.046 litres
(ii) Compression and expansion are reversible
29.046 polytropic.
or V7 = = 1.505
21 - 1.704 (iii) Volumetric efficiency of both cylinders as 100%.
or V5 = 21 ¥ 1.505 = 31.61 litres
Analysis
Stroke volume of HP cylinder
(i) Cylinder diameters
V5 – V7 = 31.61 – 1.505 = 30.10 litres
The stroke (swept) volume rate of LP cylinder
Example 25.28 In a trial on a two-stage, single-act- with hvol = 1.0
ing, reciprocating air compressor, following data were
recorded:
Free air delivery per min = 6 m3
Free air conditions = 1 bar, 27°C
Delivery pressure = 30 bar
Compressor speed = 300 rpm
Intermediate pressure = 6 bar
Temperature at the inlet of HP cylinder
= 27°C
Law of compression = pV1.3
Mechanical efficiency = 85%
Reciprocating Air Compressor 877

Êpˆ ma = 4.5 kg/min = 0.075 kg/s


V1 = FAD = Á ˜ d12 L1 N k
Ë 4¯ k =1
p1 = 1.013 bar = 101.3 kPa
Êpˆ
or 6 m3/min = Á ˜ ¥ d12 ¥ (1.2 d1 ) ¥ 300 ¥ 1 T1 = 17°C = 290 K
Ë 4¯ p3
=9 n = 1.3
or d1 = 0.276 m or 276 mm p1
For same stroke length and with hvol = 1.0, the c1 = c2 = 0.05 Vs
diameters of LP and HP cylinders are related as N = 300 rpm R = 0.287 kJ/kg ◊ K
p1 d12 = p2 d22
To find
1 ¥ (276)2 = 6 ¥ d22
(i) Minimum indicated power, and
or d2 = 112.67 mm
(ii) Swept volumes of LP and HP cylinders.
(ii) Power input to compressor
The mass-flow rate of air into compressor
p1V1 (100 kPa) ¥ (6 m3/min)
ma = =
RT1 (0.287 kJ/kg ◊ K) ¥ (300 K)
= 6.9686 kg/min or 0.116 kg/s
The indicated power input to two-stage, single-
acting air compressor
È n -1 n -1 ˘
n ÍÊ p2 ˆ n Ê p3 ˆ n ˙
IP = ma RT1 ÍÁ ˜ +Á ˜ - 2˙
n -1 Ë
Î 1p ¯ Ë p2 ¯ ˚
1.3
= ¥ 0.116 ¥ 0.287 ¥ 300
1.3 -1
È 1.3 -1 1.3 -1 ˘
ÍÊ 6 ˆ 1.3 Ê 30 ˆ 1.3 ˙
¥ ÍÁ ˜ +Á ˜ - 2˙
ÎË 1 ¯ Ë 6 ¯ ˚
Analysis
= 84.9 kW
(i) Minimum indicated power
Brake power input
For perfect intercooling
IP 48 .9 kW p2 = p1 ¥ p3 = p1 ¥ 9 p1 = 3 p1
= = = 99.88 kW
hmech 0.85
p2
or =3
p1
Example 25.29 In a single-acting, two-stage recipro-
cating air compressor handles 4.5 kg of air per minute, The minimum power input, Eq. (25.42)
and compresses it from 1.013 bar 17°C through a pres- È n -1 ˘
sure ratio of 9. The index of compression and expansion Ê n ˆ ÍÊ p2 ˆ n ˙
IPmin = 2¥Á ma RT1 ÍÁ ˜ - 1˙
in both stages is 1.3. If the intercooling is complete,find Ë n -1˜¯ Ë
Î 1p ¯ ˚
the minimum indicated power and cylinder swept vol-
Ê 1.3 ˆ
umes required. Assume the clearance volume of both the = 2¥Á ¥ 0.075 ¥ 0.287
stages are 5% of their respective stroke volumes and the Ë 1.3 - 1˜¯
compressor runs at 300 rpm. Take R = 0.287 kJ/kg ◊ K. È 1.3 -1 ˘
ÍÊ 3 ˆ 1.3 ˙
¥ 290 ¥ ÍÁ ˜ - 1˙ = 15.61 kW
Solution Ë
Î 1 ¯ ˚
Given A single-acting, two-stage reciprocating air (ii) The stroke (swept) volume of LP cylinder
compressor with perfect intercooling The mass of air inducted per cycle into LP
878 Thermal Engineering

cylinder If the compressor runs at 440 rpm, the clearance


m 4.5 kg/min ratios for LP and HP cylinders are 0.04 and 0.06,
ma = a =
N k (300 rotation/min) ¥ 1 respectively, calculate the swept and clearance volumes
for each cylinder.
= 0.015 kg/cycle
Efffective swept volume of LP cylinder; Solution
m RT 0.015 ¥ 0.287 ¥ 290
(V1 – V4) = a 1 = Given A single-acting, two-stage reciprocating air
p1 101.3 compressor with perfect intercooling
= 0.0123 m3/cycle ma = 10.5 kg/min = 0.175 kg/s
The volumetric efficiency of LP and HP cylinders p1 = 1 bar = 100 kPa T1 = 27°C = 300 K
1
p3 = 16 bar n = 1.3
Ê p2 ˆ n
hvol = 1 + c1 – c1 Á ˜ c1 = 0.04 c2 = 0.06
Ë p1 ¯
N = 440 rpm k =1
1
Ê 3 ˆ 1.3 To find
= 1 + 0.05 – 0.05 ¥ Á ˜ = 0.933
Ë 1¯ (i) Indicated power,
It is also expressed as (ii) Isothermal efficiency,
V1 - V4 (iii) FAD,
hvol =
Vs, LP (iv) Heat transfer in intercooler, and
(v) Swept volumes of LP and HP cylinders.
V1 - V4 0.0123
or Vs, LP = = Assumptions
hvol 0.933
3 (i) Air an ideal gas, R = 0.287 kJ/kg ◊ K,
= 0.0131 m /cycle
(ii) Compressions and expansions are reversible
The swept volume of LP cylinder is 0.0131 m3/
processes, and
cycle.
(iii) Suction takes place at free air conditions.
Using Eq. (25.46) for ratio of LP and HP
cylinders, Analysis For perfect intercooling, the pressure ratio
p1 (V1 – V4) = p2 (V5 – V8) per stage
p1 1 p2 p
V5 – V8 = (V1 - V4 ) = ¥ (0.0123) = 3 = p1 ¥ p3 = 1 ¥ 16 = 4
p2 3 p1 p2
= 0.0041 m3/cycle
Then swept volume of HP cylinder
V5 - V8 0.0041
Vs, HP = = = 0.00440 m3
hvol 0.933

Example 25.30 A single-acting, two-stage recipro-


cating air compressor with complete intercooling deliv-
ers 10.5 kg/min of air at 16 bar. The compressor takes in
air at 1 bar and 27°C. The compression and expansion
follow the law pV1.3 = Const. Calculate
(a) Power required to drive the compressor
(b) Isothermal efficiency
(c) Free air delivery
(d) Heat transferred in intercooler
Reciprocating Air Compressor 879

(i) Indicated power input for two-stage compressor 1


Ê 4 ˆ 1.3
È n -1 ˘ = 1 + 0.04 – 0.04 ¥ Á ˜
Ê n ˆ ÍÊ p2 ˆ n ˙ Ë 1¯
IPmin =2¥Á ˜ ma RT1 ÍÁ ˜ - 1˙
= 0.923 or 92.3%
Ë n - 1¯ ÎË p1 ¯ ˚
The volume handled per cycle by LP cylinder
Ê 1.3 ˆ
= 2¥Á ¥ 0.175 ¥ 0.287
Ë 1.3 -1˜¯ (V1 – V4)
FAD Vf 9.04
È 1.3 -1 ˘ = = =
ÍÊ 4 ˆ 1.3 ˙ No. of cycle per minute N k 440 ¥ 1
¥ 300 ¥ ÍÁ ˜ - 1˙
Ë 1¯ = m3/cycle
0.0205
Í ˙
Î ˚ Further, it can also be expressed as
= 49.23 kW V1 - V4
(ii) Isothermal efficiency hvol, LP =
Vs, LP
Power input for isothermal compression from
V1 - V4 0.0205
1 bar to 16 bar pressure or Vs, LP = =
hvol , LP 0.923
Êp ˆ
IPiso = ma RT1 ln Á 3 ˜ = 0.0222 m3/cycle
Ë p1 ¯
For HP cylinder
Ê 16 ˆ
= 0.175 ¥ 0.287 ¥ 300 ¥ ln Á ˜ 1
Ë 1¯ Ê p ˆn
= 41.77
kW hvol, HP = 1 + c 2 – c2 Á 3 ˜
Ë p2 ¯
Isothermal power 41.77 kW
hiso = = 1
Actual power 49.23 kW
Ê 4 ˆ 1.3
= 0.848
or 84.8% = 1 + 0.06 – 0.06 ¥ Á ˜
Ë 1¯
(iii) Free air delivery (FAD )
= 0.885 or 85.5%
ma RT1
V f = V1 -V4 = The effective swept volume handled by HP
p1
cylinder with perfect intercooling
(10.5 kg/min) ¥ (0.287 kJ/kg ◊ K) ¥ (300 K) p2 (V5 – V8) = p1(V1 – V4)
=
(100 kPa) p
or (V5 – V8) = 1 ¥ (V1 - V4 )
= 9.04 m3/min p2
(iv) Heat transferred in intercooler
1
Temperature after compression in each stage =¥ 0.0205 = 0.005125 m3/cycle
4
n -1 1.3 -1
The piston displacement volume of HP cylinder;
Êp ˆ n Ê 4ˆ 1.3
T2 = T 1 Á 2 ˜ = 300 ¥ Á ˜ V5 - V8 0.005125
Ë p1 ¯ Ë 1¯ Vs, HP = =
hvol , HP 0.885
= 413.1 K
Heat transfer rate in intercooler = 0.0058 m3/cycle

Q = ma Cp (T2 – T1) Example 25.31 A two-stage, double-acting, re-


= 10.5 ¥ 1.005 ¥ (413.1 – 300) ciprocating air compressor operating at 300 rpm, re-
= 1193.5 kJ/min or = 19.89 kW ceives air at 1 bar and 27°C. The bore of LP cylinder is
(v) Swept volumes 360 mm and its stroke is 400 mm. Both cylinders have
equal stroke and equal clearance of 4% . The LP cylinder
The volumetric efficiency of LP cylinder
1
discharges air at a pressure of 5 bar. The air then passes
Ê p ˆn through an intercooler to cool air to its initial tempera-
hvol, LP = 1 + c1 – c1 Á 2 ˜ ture. Pressure drops in intercooler to 4.75 kPa. Finally
Ë p1 ¯
air is discharged from HP cylinder at 15 bar. The index
880 Thermal Engineering

of compression and expansion in both cylinders is 1.3. Cp The pressure ratio for LP cylinder;
= 1.005 kJ/kg ◊ K, and R = 0.287 kJ/kg ◊ K. p2
=5
Calculate p1
(a) Heat rejected in the intercooler, The volumetric efficiency of LP cylinder;
1
(b) Diameter of HP cylinder,
Ê p ˆn
(c) Power required to drive HP cylinder. hvol, LP = 1 + c1 – c1 Á 2 ˜
Ë p1 ¯
1
Solution Ê 5 ˆ 1.3
= 1 + 0.04 – 0.04 ¥ Á ˜ = 0.902
Given Two-stage, double-acting reciprocating air com- Ë 1¯
pressor It is also expressed using effective stroke volume rate
k =2 N = 300 rpm ( V1 - V4 ) of LP cylinder as
p1 = 1 bar = 100 kPa T1 = 27°C = 300 K
V1 - V4
p2 = 5 bar p5 = 4.75 bar hvol, LP =
T3 = 27°C = 300 K p6 = 15 bar Vs, LP
n = 1.3 d1 = 360 mm or V1 - V4 = Vs, LP ¥ hvol, LP
L1 = 400 mm c1 = c2 = 0.04 = ( 24.42 kg/min) ¥ 0.902 = 22.02 m3/min
Cp = 1.005 kJ/kg ◊ K R = 0.287 kJ/kg ◊ K
The mass flow rate of air into the LP cylinder;
L2 = L1
p1 (V1 - V4 ) (100 kPa) ¥ (22.02 m3/min)
To find ma = =
RT1 (0.287 kJ/kg ◊ K) ¥ (300 K)
(i) Heat rejected in the intercooler,
(ii) Diameter of HP cylinder, and = 25.584 kg/min or 0.426 kg/s
(iii) Power required to drive HP cylinder. Temperature after compression in each stage
n –1 1.3 -1
Êp ˆ n Ê 5ˆ 1.3
T2 = T1 Á 2 ˜ = 300 ¥ Á ˜
Ë p1 ¯ Ë 1¯
= 434.93 K
(i) Heat transfer rate in the intercooler
Q = ma Cp (T2 – T1)
= 25.584 ¥ 1.005 ¥ (434.93 – 300)
= 3469.33 kJ/min or 57.82 kW
(ii) Diameter of HP cylinder
The effective swept volume rate of HP cylinder
ma RT1
V5 - V8 =
p5
25.584 ¥ 0.287 ¥ 300
=
475
3
Analysis The stroke (swept) volume per minute of LP = 4.637 m /min
cylinder The volumetric efficiency of HP cylinder;
1
Êpˆ 2 Ê p ˆn
Vs, LP = V1 - V3 = Á ˜ d1 L1 N k hvol, HP = 1 + c2 – c2 Á 3 ˜
Ë 4¯ Ë p5 ¯
Êpˆ 1
= Á ˜ ¥ (0.3 6 m) 2 ¥ (0.4 m) ¥ 300 ¥ 2 Ê 15 ˆ 1.3
Ë 4¯ = 1 + 0.04 – 0.04 ¥ Á
Ë 4.75 ˜¯
= 0.943
= 24.42 m3/min
Reciprocating Air Compressor 881

Further, the volumetric efficiency can be ex- (b) Heat rejected in after-cooler, if delivered air is
pressed in terms of piston displacement volume cooled to initial temperature,
rate of HP cylinder as (c) Diameter of intermediate and HP cylinders,
V5 - V8 (d) Power required to drive compressor, if its
hvol, HP =
Vs, HP mechanical efficiency is 85%.

V5 - V8 4.637 Solution
or Vs, HP = = = 4.917 m3/min
hvol , HP 0.943
Given Three-stage, double-acting reciprocating air
which can be further, expressed as compressor
Êpˆ k =2 N = 300 rpm
Vs, HP = Á ˜ d2 2 L1 N k p1 = 1 bar = 100 kPa T1 = 27°C = 300 K
Ë 4¯
p2 = 5 bar p5 = 4.8 bar
Êpˆ
or 4.917 = Á ˜ ¥ d22 ¥ 0.4 ¥ 300 ¥ 2 T3 = 27°C = 300 K p6 = 20 bar
Ë 4¯
p9 = 19.8 bar p10 = 75 bar
or d2 = 0.1615 m or 161.5 mm n1 = 1.3 n2 = 1.32
(iii) Power required to drive HP cylinder n3 = 1.35 d1 = 360 mm
È n -1 ˘ L1 = 400 mm c1 = c2 = c3 = 0.04
n ÍÊ p3 ˆ n ˙ Cp = 1.005 kJ/kg ◊ K R = 0.287 kJ/kg ◊ K
IP = p5 ( V5 - V8 ) ÍÁ ˜ - 1˙
n -1 ÎË 5 ¯
p ˚ L3 = L2 = L1 hmech = 0.85
1.3 Ê 4.637 ˆ To find
= ¥ 4.75 ¥ 102 ¥ Á
1.3 -1 Ë 60 ˜¯ (i) Heat rejected in each stage in intercooler and
È 1.3 -1 ˘ during compression,
ÍÊ 15 ˆ 1.3 ˙ (ii) Heat rejected in after-cooler,
¥ ÍÁ ˜¯ - 1˙
ÎË 4. 75 ˚ (iii) Diameter of intermediate and HP cylinders,
= 48.34 kW (iv) Power required to drive compressor.
Analysis The stroke (swept) volume per minute of LP
Example 25.32 A three-stage, double-acting, recip-
cylinder
rocating air compressor operating at 300 rpm, receives
Êpˆ 2
air at 1 bar and 27°C. The bore of LP cylinder is 360 mm Vs, LP = V1 - V3 = Á ˜ d1 L1 N k
Ë 4¯
and its stroke is 400 mm. Intermediate cylinder and HP
cylinder have same stroke as LP cylinder. The clearance
volume in each cylinder is 4 % of the stroke volume . The
LP cylinder dischrges air at a pressure of 5 bar, the inter-
mediate cylinder discharges at 20 bar and air is finally
discharged by the HP cylinder at 75 bar. The air is cooled
in intercoolers to initial temperature after each stage of
compression. A Pressure drop of 0.2 bar takes place in
intercooler after each stage. The index of compression
and expansion for an LP cylinder is 1.3, for intermediate
cylinder is 1.32 and for HP cylinder is 1.35. Neglect the
effect of piston rod and assume Cp = 1.005 kJ/kg ◊ K, and
R = 0.287 kJ/kg ◊ K.
Calculate
(a) Heat rejected in each stages in intercooler and
during compression,
882 Thermal Engineering

Êpˆ After second stage,


= Á ˜ ¥ (0.3 6 m) 2 ¥ (0.4 m) ¥ 300 ¥ 2
Ë 4¯ n2 -1
Êp ˆ n2
= 24.42 m /min 3
T6 = T5 Á 6 ˜
Ë p5 ¯
The volumetric efficiencies of LP, intermediate and
1.32 -1
HP cylinders;
Ê 20 ˆ 1.32
= 300 ¥ Á
Ë 4.8 ˜¯
1 = 424.00 K
Ê p ˆ n1
hvol, LP = 1 + c1 – c1 Á 2 ˜ After third stage
Ë p1 ¯
1 n3 -1
Ê 5 ˆ 1.3 Êp ˆ n3
= 1 + 0.04 – 0.04 ¥ Á ˜ = 0.902 T10 = T9 Á 10 ˜
Ë 1¯ Ë p9 ¯
1.35 -1
For IP cylinder
Ê 75 ˆ 1.35
= 300 ¥ Á
Ë 19.8 ¯˜
1 = 423.71 K
Ê p ˆ n2
hvol, IP = 1 + c2 – c2 Á 6 ˜
Ë p5 ¯ Specific heat at constant volume;
1 R 0.287
Ê 20 ˆ 1.32 Cv = = = 0.717 kJ/kg ◊ K
= 1 + 0.04 – 0.04 ¥ Á g -1 1.4 -1
Ë 4.8 ˜¯
= 0.922
(i) Heat rejection rate in each stage
For HP cylinder
(g - n)
1 Qcomp = ma Cv (T2 – T1)
Ê n -1
p10 ˆ n3
hvol, IP = 1 + c3 – c3 Á
Ë p9 ˜¯ Qint ercooler= ma Cp (T2 – T1)
1
Ê 75 ˆ 1.35 Heat rejcetion rate in first stage (during compres-
= 1 + 0.04 – 0.04 ¥ Á
Ë 19.8 ˜¯
= 0.9327 sion and in intercooler);
È (g - n1 ) ˘
The effective stroke volume ( V1 - V4 ) of air per Q1 = ma Í Cv + C p ˙ (T2 – T1)
Î n1 - 1 ˚
minute in LP cylinder;
V1 - V4 = Vs, LP ¥ hvol, LP È (1.4 - 1.3) ˘
= 25.584 ¥ Í ¥ 0.717 + 1.005˙
= (24.42 m3/min) ¥ 0.902 = 22.02 m3/min Î 1.3 - 1 ˚
The mass flow rate of air in the compressor; ¥ (434.93 – 300)
3 = 4295 kJ/min
p1 (V1 - V4 ) (100 kPa) ¥ (22.02 m /min)
ma = = Heat rejcetion rate in second stage (during com-
RT1 (0.287 kJ/kg ◊ K) ¥ (300 K)
pression and in intercooler);
= 25.584 kg/min or 0.426 kg/s
È (g - n2 ) ˘
Temperature after compression in each stage Q2 = ma Í Cv + C p ˙ (T6 – T5)
Î n2 - 1 ˚
After first stage,
È (1.4 - 1.32) ˘
n1 -1 = 25.584 ¥ Í ¥ 0.717 + 1.005˙
Ê Î 1 . 32 - 1 ˚
p2 ˆ n1
T2 = T1 Á ˜ ¥ (424 – 300)
Ë p1 ¯
1.3 -1
= 3757 kJ/min
Ê 5ˆ 1.3 Heat rejction rate in third stage compression;
= 300 ¥ Á ˜ = 434.93 K
Ë 1¯ Ê g - n3 ˆ
Q3 = ma Á Cv (T10 – T9)
Ë n3 - 1 ˜¯
Reciprocating Air Compressor 883

Ê 1.4 - 1.3 5ˆ The effective swept volume rate of HP cylinder


= 25.584 ¥ Á ¥ 0.717
Ë 1.35 - 1 ˜¯ V9 - V12 =
ma RT9
p9
¥ (423.71 – 300)
= 324.18 kJ/min 25.584 ¥ 0.287 ¥ 300 K
=
Heat rejection rate in after-cooler 1980
Q3 = ma Cp (T10 – T1) = 1.1125 m3/min
= 25.584 ¥ 1.005 ¥ (423.71 – 300) V9 - V12 1.1125
Vs, HP = = = 1.192 m3/min
= 3180.82 kJ/min hvol , HP 0.9327
(ii) Diameter of IP and HP cylinders which can be further, expressed as
The effective swept volume rate of IP cylinder Êpˆ
1.192 = Á ˜ ¥ d32 ¥ 0.4 ¥ 300 ¥ 2
ma RT1 Ë 4¯
V5 - V8 =
p5 or d3 = 0.07954 m or 79.54 mm
25.584 ¥ 0.287 ¥ 300 K The power input to compressor
=
480 Since T1 = T5 = T9,
= 4.588 m3/min È Ï n1 -1 ¸
Further, the volumetric efficiency can be ex- Í n1 ÔÊ p2 ˆ n1 Ô n
IP = ma RT1 Í Ì - 1˝ + 2
Ô Á ˜ Ô
pressed using piston displacement volume rate of n - 1
ÍÎ 1 Ó 1 Ë p ¯ ˛ n2 -1
IP cylinder as
Ï n2 -1 ¸ Ï n3 -1 ¸˘
V5 - V8 ÔÊ p ˆ n2 Ô n3 ÔÌÊ p10 ˆ n3 Ô˙
hvol, IP = Ì 6
Á ˜
˝
-1 + Á ˜ - 1˝ ˙
ÓÔË p5 ¯ ˛Ô n3 -1 ÓÔË p9 ¯ ˛Ô ˙˚
Vs, IP

V5 - V8 4.588 = 25.584 ¥ 0.287 ¥ 300


\ Vs, IP = = = 4.976 m3/min
hvol , IP 0.922 È Ï 1.3 -1 ¸ Ï 1.32 -1 ¸
Í 1.3 ÔÊ 5 ˆ 1.3 Ô 1.32 ÔÊ 20 ˆ 1.32 Ô
which can be further, expressed as ¥ Í ÌÁ ˜ - 1˝ + ÌÁ ˜¯ - 1˝
Í 1 . 3 - 1 Ô Ë 1 ¯ Ô 1 .32 - 1 Ô Ë 4 . 8 Ô
Êpˆ Î Ó ˛ Ó ˛
Vs, IP = Á ˜ d2 2 L1 N k
Ë 4¯ Ï 1.35 -1 ¸˘
1.35 ÔÊ 75 ˆ 1.35 Ô˙
Êpˆ + ÌÁ ˜ - 1˝˙
or 4.976 = Á ˜ ¥ d22 ¥ 0.4 ¥ 300 ¥ 2 1.35 - 1 ÔË 19.8 ¯ Ô˙
Ë 4¯ Ó ˛˚
or d2 = 0.1624 m or 162.4 mm = 2202.78 ¥ (1.949 + 1.705 + 1.59)
Similarly, for HP cylinder; = 11,551.4 kJ/min or 192.34 kW

Summary

volume and increases the pressure of a quantity cylinder reciprocating compressor is


of air by mechanical means. The reciprocating p N
V = d 2L
compressor handles a small quantity of gas 4 60
and produces very high pressure, while rotary double-acting reciprocating compressor,
compressors are used to handle a large volume of the induction takes place on both sides of the
gas and to produce low and medium pressures. piston for each revolution. Thus
884 Thermal Engineering

p 2 Ê 2N ˆ pm L A N k
V = d LÁ IP = (kW)
4 Ë 60 ˜¯ 60
where N is the speed of compressor in rotations where k = 1 for single acting and k = 2 for double
per minute. acting reciprocating compressor.
The capacity of a compressor is the actual
quantity of air delivered per unit time at given by
atmospheric conditions. Free Air delivery (FAD ) Indicated power
hmech =
is the discharge volume of the compressor Brake power
corresponding to ambient conditions.
Piston speed is the linear speed of the piston or engine. The input of the driving motor can be
measured in m/min. It is expressed as expressed as
Vpiston = 2 L N Motor power
dicated work input to a single-stage, Shaft power (or Brake power)
single-acting reciprocating compressor without =
Mechanical efficiency of motorr and drive
clearance is
È n-1 ˘
n ÍÊ p2 ˆ n ˙ compressor is
Win = p1V1 ÍÁ ˜ - 1˙ (kJ/cycle) Actual Mass sucked
n -1 ÎË p1 ¯ ˚ hvol =
lume is provided in the cylinder Mass corresponds to swept volume at
atmosspheric pressure and temperature
to accomodate valves. The clearance ratio c is the 1
ratio of clearance volume to the swept volume. Ê p ˆ ne
The clearance ratio for a reciprocating air = 1 + c – c Á 2˜
Ë p1 ¯
compressor is usually 2 to 10%. The work input
-
to the compressor with clearance ratio c is
es the compression work.
È nc -1 ˘
nc ÍÊ p2 ˆ nc ˙
Win = p 1V1 ÍÁ ˜ - 1˙ intercooler, the work input per cycle is
nc - 1 ÎË p1 ¯ ˚
È ne -1 ˘ È n -1 n -1 ˘
ne ÍÊ p2 ˆ ne ˙ n ÍÊ p2 ˆ n Ê p3 ˆ n ˙
– p1V4 ÍÁ ˜ - 1˙ Win = ma RT1 ÍÁ ˜ +Á ˜ - 2˙
ne - 1 Ë p ¯ n -1 Ë
Î 1p ¯ Ë p2 ¯ ˚
Î 1 ˚
where nc = index of compression and ne = index where p2 and T2 are intermediate pressure and
of expansion. temperature, respectively.
If both indices are same, i.e., nc = ne then
È n -1 ˘ Qcooling = ma Cp (T2 – T1) (kJ)
n ÍÊ p2 ˆ n ˙ and heat rejected during polytropic compression
Win = p 1(V1 – V4) ÍÁ ˜ - 1˙
n -1 ÎË p1 ¯ ˚ is
ciency of a compressor is defined (g - n)
Qcomp = ma Cv (T2 – T1)
as the ratio of isothermal work input to actual n- 1
work input.
compressor would be minimum when a stage
is given as pressure ratio is
p2 p
IP = Win per cycle = 3
¥ No of compression per unit time p1 p2
Ê Nk ˆ and the minimum compression power for a two-
= Win Á
Ë 60 ˜¯ stage compressor
ndicated diagram, the indicated power È n -1 ˘
Ê n ˆ ÍÊ p2 ˆ n ˙
is obtained in terms of indicated mean effective IPmin = 2 ¥ Á ma RT1 ÍÁ ˜ - 1˙
pressure, pm as Ë n - 1˜¯ ÎË p1 ¯ ˚
Reciprocating Air Compressor 885

Glossary
Reciprocating compressor A reciprocating machine, Pressure ratio The ratio of absolute discharge pressure
used to compress the air during each stroke of piston to absolute suction pressure
Rotary compressor A machine which compresses the Free air Air that exists under atmospheric condition
air by dynamic action Free Air delivery (FAD) Discharge volume of com-
Single-acting compressor A compressor in which all pressor corresponding to ambient conditions
actions take place only one side of the piston during a Compressor Capacity Quantity of air delivered per unit
cycle time at atmospheric conditions
Double-acting compressor A compressor in which Inter-stage coolers Used to cool the air in between
suction, compression and delivery of gas take place on stages of compression
both sides of the piston After coolers Used to remove the moisture in the air
Single-stage compressor A compressor in which the by cooling it
compression of gas to final delivery pressure is carried Air-dryers Removes traces of moisture after after-cool-
out in one cylinder only er is used
Multistage compressor A compressor which com- Moisture drain traps used for removal of moisture in
presses the gas to the final pressure in more than one cyl- the compressed air
inder in series
Air receiver Storage tank used to store the compressed
air

1. What is an air compressor? Why is it an important 11. Explain the working of double-acting reciprocat-
machine? ing air compressor.
2. Write the uses of compressed air. 12. Derive an expression for indicated work of a re-
3. Classify the air compressors. ciprocating air compressor by neglecting clear-
4. How do the suction and delivery valve activate in ance.
reciprocating air compressor? 13. Why is the clearance volume provided in each re-
5. State the main parts of reciprocating air compres- ciprocating compressor? Is it desirable to have a
sor. high clearance volume in a compressor?
6. Differentiate between 14. What is clearance ratio? Write the effect of clear-
(i) Single-acting and double-acting compres- ance volume on performance of a reciprocating
sors compressor.
(ii) Single-stage and multistage compressors. 15. Derive an expression for indicated work of a
7. Why is a cooling arrangement provided with all reciprocating air compressor by considering its
compressors? clearance.
8. Define swept volume, and deduce it for single- 16. Define volumetric efficiency and prove that
1
cylinder, single-acting and double-acting com-
Ê p ˆ ne
pressor having bore d, stroke L, and speed N rpm. hvol = 1 + c – c Á 2˜
Ë p1 ¯
9. Write the construction of a single-acting, single-
stage reciprocating air compressor. where each term has its usual meaning.
10. Explain the working of a single-acting reciprocat- 17. Define volumetric efficiency. How is it affected
ing air compressor. by (i) pressure ratio, (ii) speed of compressor, and
886 Thermal Engineering

(iii) throttling across the valves? Explain in brief. 23. Define overall volumetric efficiency. Discuss the
18. What are the advantages of multistage compres- parameters in brief, which affect it.
sion over single-stage compression? 24. Show the effect of increase in compression ratio
19. Why is the intercooler provided between stages? in single-stage reciprocating compressor on a
20. Prove that in a reciprocating air compressor, with p–V diagram and give its physical explanation.
perfect intercooling, the work done for compress- 25. Draw the indicator diagram for single-stage, dou-
ing the air is equal to heat rejected by the air. ble-acting reciprocating air compressor on a p–V
21. What is an after-cooler? Why is it provided with diagram.
an air compressor. 26. What are the advantages of using an after-cooler
22. Prove that for complete intercooling between two with an air compressor, when air under pressure
stages, the compression work would be minimum has to be stored over long periods?
when intermediate pressure 27. What is the effect of intake temperature and pres-
sure on output of an air compressor?
p2 = p1 ¥ p3

Problems
1. Calculate the bore of the cylinder for a double- and discharges air at 6.5 bar. It has a mechanical
acting, single-stage reciprocating air compressor efficiency of 80% and a clearance ratio of 8.9%.
runs at 100 rpm with average piston speed of Calculate
150 m/min. The indicated power input is 50 kW. (a) the volumetric efficiency,
It receives air at 1 bar and 15°C and compresses it (b) mean effcetive pressure,
according to pV1.2 = constant to 6 bar. (c) brake power.
[349 mm] [(a) 75% (b) 1.06 bar (c) 1.48 kW]
2. A single-acting, single-cylinder reciprocating air 5. Calculate the power required to drive a single-
compressor has a cylinder diameter of 300 mm stage, single-acting reciprocating air compressor
and a stroke of 400 mm. It runs at 100 rpm. to compress 8 m3/min of air, receiving at 1 bar,
Air enters the cylinder at 1 bar; 20°C. It is then 20°C to 7 bar. The index of compression is 1.3.
compressed to 5 bar. Calculate the mean effective Also, calculate the percentage saving in indicated
pressure and indicated power input to compressor, power by compressing the same mass of air
when compression is
(a) in two stages with optimum intercooler
(a) isothermal,
pressure and perfect intercooling,
(b) according to the law pV1.2 = constant,
(b) in two stages with imperfect intercooling
(c) adiabatic.
to 27°C, intercooler pressure remaining the
Calculate isothermal efficiency for each case.
same as in case in (a),
Neglect clearance.
(c) in three stages with optimum intercooler
[(a) 1.61 bar, 7.58 kW, 100% (b) 1.85 bar,
and perfect cooling.
8.7 kW, 87.2% (c) 2.043 bar, 9.63 kW, 78.8%]
[32.8 kW (a) 11.3% (b) 10.2% (c) 14.63%]
3. An air compressor takes in air at 100 kPa, 300 K.
6. A power cylinder of 0.5 m3 capacity is charged
The air delivers at 400 kPa, 200°C at the rate of
with compressed air without after-cooling it
2 kg/s. Determine minimum compressor work
at 170 bar from a four-stage compressor with
input. [312.7 kW]
perfect intercooling between stages and working
4. A single-acting, single-cylinder reciprocating
in best conditions. What are the most economical
air compressor receives 30 m3 of atmopheric air
intermediate pressure?
per hour at 1 bar and 15°C . It runs at 450 rpm
[3.611bar 13.04 bar and 47.08 bar]
Reciprocating Air Compressor 887

7. The free air delivered by a single-stage, double- Data: Air inlet conditions = 1.013 bar and
acting reciprocating air compressor, measured at 15°C; delivery pressure = 8 bar; clearance volume
1 bar and 15°C is 16 m3/min. The suction takes = 7% of swept volume; index of compression and
place at 96 kPa and 30°C and delivery pressure re-expansion = 1.3; mechanical efficiency of the
is 6 bar. The clearance volume is 4% of swept drive between motor and compressor = 87%;
volume and mean piston speed is limited to motor power output = 23 kW
300 m/min. D etermine (4.47 m3/min;72.7% 297 mm)
(a) power input to compressor, if mechanical 11. The LP cylinder of a two-stage, double-acting
efficiency is 90% and compression efficien- reciprocating air compressor running at 120 rpm
cy is 85% has a 50-cm diameter and 75-mm stroke. It
(b) Bore and stroke if compressor runs at receives air at 1 bar and 20°C and compresses it
500 rpm adiabatically to 3 bar. Air is then delivered to an
Assume index of compression and expansion intercooler, where it is cooled at constant pressure
= 1.3. to 35°C and then furter compressed to 10 bar in
[(a) 83.6 kW (b) 290 mm and 300 mm] HP cylinder. Determine the power required of an
8. A double-acting, single-stage reciprocating air electric motor to drive a compressor. Assume the
compressor has a bore of 330 mm, stroke of mechanical efficiency of the compressor as 90%
350 mm, clearance of 5%, and runs at 300 rpm. and of the motor as 86%. [212.9 kW]
It receives air at 95 kPa and 25°C . The delivery 12. A reciprocating air compressor takes in air at
pressure is 4.5 bar and the index of compression 40°C and 1.013 bar in the daytime.
is 1.25. The free air conditions are 1.013 bar and (a) Calculate the percentage increase of mass
20°C. Determine output in the night, if the night temperature
(a) FAD, is 10°C.
(b) heat rejected during the compression, and (b) If the compressor is shifted to a hill station,
(c) power input to compressor, if its mechanical where the barometric pressure is 0.92 bar,
efficiency is 80%. calculate percentage decrease in output,
[(a) 14.51 m3/min (b) 817.4 kJ/min. assuming suction temperature to be same at
(c) 56.82 kW] two places.
9. A Four cylinder, double acting reciprocating (c) Calculate the pressure ratio of the
air compressor is used to compress 30 m3/min compressor at two places, if the law of
of air at 1 bar and 27°C to a pressure of 16 bar. compression is pV1.25 = constant, if delivery
Calculate the size of motor required and cylinder gauge pressure is 7 bar at both places.
dimensions for the following data: [(a) 10.61% (b) 9.18% (c) at first place
speed of compressor = 320 rpm, clearance ratio 4.81% and second place 5.24]
4%, stroke to bore ratio 1.2, hmech = 82%, index 13. A three-stage single-acting reciprocating air
of compression and expansion, n = 1.3. compressor has perfect intercooling. The
Assume air gets healed by 12°C during suction. pressure and temperature at the end of the suction
[241 kW, 263 mm, 315.6 mm] stroke in an LP cylinder are 1.013 bar and 15°C,
10. A single-acting, single-cylinder air compressor respetively. If 8.4 m3 of free air is delivered by the
running at 300 rev/min is driven by an electric compressor at 70 bar per minute and work done is
motor. Using the data given below, and assuming minimum, calculate
that the bore is equal to the stroke, calculate (a) LP and IP cylinder delivery pressures,
(a) free air delivery, (b) ratio of cylinder volumes,
(b) volumetric efficiency, (c) total indicated power.
(c) bore and stroke. Neglect clearance and assume n = 1.2.
[(a)4.16 bar, 17.05 bar (b) 2.02 (c) 676.8 kW]
888 Thermal Engineering

Objective uestions
1. For isothermal compression in a compressor, the 9. In a reciprocating air compressor, inlet and
compressor should run at delivery valves actuate
(a) very high speed (b) very slow speed (a) by separate cam mechanism
(c) constant speed (d) none of above (b) by pressure difference
2. A reciprocating compressor handles (c) by use of compressed air
(a) large volume for high pressure ratio (d) none of the above
(b) large volume for low pressure ratio 10. In a reciprocating air compressor, the work input
(c) small volume for high pressure ratio is minimum when compression is
(d) small volume for low pressure ratio (a) isentropic (b) polytropic
3. Usually, the index of actual compression is (c) isothermal (d) isobaric
(a) near to 1 (b) 1.3 to 1.4 11. What is the sequence of processes in a reciprocat-
(c) 1.1 to 1.3 (d) 1.4 to 1.6 ing air compressor?
4. Which of the following process takes place in an (a) Compression, expansion, and constant vol-
air compressor? ume discharge
(a) Specific volume of air decreases (b) Induction, compression and constant pres-
(b) Pressure of air increases sure discharge
(c) Mechanical energy is supplied (c) Induction, expansion and constant pressure
(d) All of above discharge
5. For which one of the following applications, the (d) Induction, compression and constant vol-
compressed air is not used? ume discharge
(a) Driving air motors 12. Work input in a reciprocating air compressor is
(b) Oil and gas transmission given by
(c) Starting of I.C. engines È n ˘
(d) Transmission of electrical energy n -1 Í Ê p2 ˆ n-1 ˙
(a) p1v1 Í1 + Á ˜ ˙
6. Reciprocating compressor is n Î Ë p1 ¯ ˚
(a) a positive displacement machine
(b) a negative displacement machine È n ˘
n -1 ÍÊ p2 ˆ n-1 ˙
(c) a dynamic action machine (b) p1v1 ÍÁ ˜ - 1˙
(d) none of above
n ÍË p1 ¯ ˙
Î ˚
7. Air dryers are used in an air compressor
È n-1 ˘
(a) before air entry into cylinder n -1 ÍÊ p2 ˆ n ˙
(c) p1v1 ÍÁ ˜ - 1˙
(b) before entering air receiver n Ë p ¯
Í 1 ˙
(c) between two stages Î ˚
(d) after leaving air receiver È n -1 ˘
8. Air receiver used in an air compressor is used to n -1 ÍÊ p2 ˆ n ˙
(d) p1v1 ÍÁ ˜ + 1˙
(a) cool the air after compression n ÍË p1 ¯ ˙
(b) eliminate the pulsation Î ˚
(c) supply the air to utility 13. The isothermal efficiency of a reciprocating air
(d) to separate the moisture compressor is given by
Reciprocating Air Compressor 889

Indicated power n -1
(a) Êp ˆ n
Isothermal power (b) 1 - c + c Á 2 ˜
Ë p1 ¯
Isothermal power 1
(b)
Indicated power Ê p ˆn
(c) 1 + c - c Á 2 ˜
Ë p1 ¯
Isothermal power 1
(c)
Brake power Ê p ˆn
(d) 1 - c + c Á 2 ˜
Ë p1 ¯
Brake power
(d) 18. The maximum pressure ratio in a single-stage
Isothermal power
reciprocating air compressor is limited to
14. The compressor efficiency of a reciprocating air
(a) 2 (b) 4
compressor is given by
(c) 7 (d) 10
Indicated power 19. Multistage compression in a reciprocating air
(a)
Isothermal power compressor improves
Isothermal power (a) isothermal efficiency
(b) (b) volumetric efficiency
Indicated power
(c) mechanical balance
Isothermal power (d) all of above
(c)
Brake power 20. Ideal intermediate pressure p2 for two-stage
reciprocating air compressor is given by
Brake power
(d)
Isothermal power (a) p1 ¥ p3 (b) p1 ¥ p3

15. The isothermal efficiency of a reciprocating air p3 p3


compressor can be improved by use of (c) (d)
p1 p1
(a) water jacketing (b) external fins
21. Heat rejection rate per stage of air with perfect
(c) intercooler (d) all of the above
intercooler is given by
16. The clearance volume in a reciprocating air
compressor È (g - n) ˘
(a) ma Í Cv + C p ˙ (T2 – T1)
(a) reduces work input Î n -1 ˚
(b) reduces suction capacity È (g - n) ˘
(c) reduces discharge pressure (b) ma ÍC p - Cv ˙ (T2 – T1)
Î n -1 ˚
(d) all of the above
17. The volumetric efficiency of a reciprocating air È ( n - 1) ˘
(c) ma ÍC p - Cv ˙ (T2 – T1)
compressor is defined as Î g -1 ˚
n -1
Êp ˆ n È ( n - 1) ˘
(a) 1 + c - c Á 2 ˜ (d) ma ÍC p + Cv ˙ (T2 – T1)
Ë p1 ¯ Î g -1 ˚

21. (a) 20. (b) 19. (d) 18. (c) 17. (c)
16. (b) 15. (d) 14. (b) 13. (c) 12. (c) 11. (b) 10. (c) 9. (b)
8. (b) 7. (b) 6. (a) 5. (d) 4. (d) 3. (c) 2. (c) 1. (b)
Answers
890 Thermal Engineering

26
Rotary Compressor

Introduction
Rotary compressors are used to supply continuous pulsation free compressed air. They have rotor(s)
and casing in place of piston cylinder arrangement. They are compact, well balanced, and high speed
compressors. They have low starting torque thus they are directly coupled with prime-mover. They handle
large mass of gas and are suitable for low and medium pressure ratios.
The special features of rotary compressors are:
÷ Designed to provide pulsation-free air,
÷ 100% continuous duty,
÷ Quiet operation,
÷ Energy efficient at full load,
÷ Extended service intervals,
÷ Reliable long life,
÷ Improved air quality,
÷ Low starting torque.
Fans and blowers are also rotary machines, which are used for supplying air or gas. These machines are
differentiated by the method used to move the air, and by the system pressure, at which they are operating.
The ratio of the discharge pressure to the suction pressure is used to define the fans, blowers and compressors
as shown in Table 26.1.

Table 26.1

Equipment Pressure ratio Pressure rise in mm of water


Fan up to 1.11 1136
Blower 1.11 to 1.20 1136 - 2066
Compressor more than 1.20 more than 2066

Some compressors are suitable only for low pressure ratio work such as for scavenging and supercharging
of engines and for various applications of exhausting and vacuum pumping. For a pressure ratio above 9 bar,
vane-type rotary machines can be used to boost the pressure.
Rotary Compressor 891

26.1 CLASSIFICATION OF ROTARY In a rotary positive displacement type of air


COMPRESSORS compressor, the air is compressed by being trapped
in the reduced space formed by two sets of engaging
The air compressors are either reciprocating types surfaces. In a non-positive displacement or steady
or rotary types. The rotary air compressors can flow type of compressor, the air flows continuously
broadly be classified as through them and pressure is increased due to
Rotary dynamic action.
compressors The rotary compressors have adiabatic compres-
sion. They have high speed and no cooling arrange-
Positive Dynamic action or ment is provided during compression.
displacement type steady flow type

ROOTS BLOWER COMPRESSOR


Roots Lysholm Screw Vane Centrifugal Axial
blower type type type flow type
Roots blower is a positive dispacement compressor.
It is also called lobe compressor. The roots blower

Comparison between Reciprocating and Rotary Compressors

S. No. Aspect Reciprocating compressor Rotary compressor


1. Pressure ratio Discharge pressure of air is high. Discharge pressure of air is low. The
The pressure ratio per stage may be pressure ratio per stage may be in
in order of 4 to 7. order of 3 to 5.
2. Handled volume Quantity of air handled is low and is Larger quantity of air can be handled
limited to 50 m3/s. and it is about 500 m3/s.
3. Speed of compressor Low speed of compressor. High speed of compressor.
4. Vibrational Problem Due to reciprocating action, greater Rotary parts of machine, thus it
vibrational problem, the parts of has less vibrational problems. The
machine are poorly balanced. machine parts are fairly balanced.
5. Size Size of compressor is bulky for Compressor size is small for given
given discharge volume. discharge volume.
6. Air supply Air supply is intermittent. Air supply is steady and continuous.
7. Purity of compressed air Air delivered from the compressor is Air delivered from the compressor
dirty, since it comes in contact with is clean and free from dirt.
lubricating oil and cylinder surfaces.
8. Compression efficiency Higher with pressure ratio more Higher with compression ratio less
than 2. than 2.
9. Maintanance Higher due to reciprocating parts. Lower due to balanced rotary parts.
10. Mechanical efficiency Lower due to several sliding parts. Higher due to less sliding parts.
11. Lubrication Complicated lubrication system. Simple lubrication system.
12. Initial cost Higher. Lower.
13. Flexibility Greater flexibility in capacity and No flexibility in capacity and
pressure range. pressure range.
14. Suitability For medium and high pressure ratio For low and medium pressures and
and low and medium gas volume. large volumes.
892 Thermal Engineering

To Receiver
Lobed Lobed
Discharge rotor
rotor

Casing

Air
Air

Air Inlet Air Inlet Air Inlet


(a) Two-lobed roots blower (b) Initial part of suction process (c) Continuation of suction process

Discharge port Discharge

Air
Air
Air

(d) Compression process (e) Start of discharge process (f) Completion of discharge process
Fig. 26.1

is essentially a low-pressure blower and is limited 26.1(b). The air is drawn into the casing to fill the
to a discharge pressure of 1 bar in single-stage space. The flow of gas in the casing space continues
design and up to 2.2 bar in a two-stage design. Its till both rotors change their position as shown in
discharge capacity is limited to 1500 m3/min and it Fig. 26(c).
can run up to 7000 rpm. With further movement of the lobed rotor, the
This type of rotary compressor air is trapped between one rotor, when its tip tuches
consists of two or more lobed rotors and casing the casing as shown in Fig. 26(d). This part of the
with inlet and outlet passage of air. The lobed rotors blower is not open to suction port. But the air flows
rotate in an air tight casing with the help of gears in into the space created by rotation of other rotor.
external housing. The compressor inlet is open to This rotor is also carrying out the same cycle as
atmospheric air at one side and it is open to delivery first rotor after 90°.
side at the other side. The two lobes of the roots The trapped volume of air is not internally
blower is shown in Fig. 26.1(a). compressed, it is only displaced at high speed from
One of the rotors is connected to drive. The suction side to delivery side. Continued rotation
second rotor is gear driven from the first. Thus, of lobes opens the discharge port as shown in
Fig. 26 (e).
both rotors rotate with the same speed. The profile
of the lobes is made cycloidal or involute in order to Since the compressed air at higher pressure is
seal the inlet side from the delivery side. present at the delivery side, when the rotor lobe
uncovers the exit port, some pressurised air enters
The rotation of rotors creates space into the space between the rotor and casing of the
in the casing at the entry port as shown in Fig. compressor. This flow of air is called back flow of
Rotary Compressor 893

air. This back flow of air continues until the pressure p2


Let = rp, pressure ratio. Dividing numerator
in the blower gets equalised. After back flow, the p1
air is compressed irreversibly at constant volume. and denominator by p1V1, we get
Finally, at higher pressure, the air is delivered from g -1
g Ï ¸
the blower to receiver as shown in Fig. 26.1(f ). ( )
Ì r g - 1˝
g -1 Ó p ˛
The process of compression can be represented hroots =
( rp - 1)
by constant volume line on p–V plane as shown in g -1
Ï ¸
Fig. 26.2.
g Ó
( )
Ì rp g - 1˝
˛
= ¥ ...(26.3)
g -1 ( rp - 1)
p2 The efficiency of roots blower decreases with
Irreversible increase in pressure ratio. However, the compressor
pressure is suitable to give a pressure ratio between range of
rise due to
p1
back flow 1 to 3.6.
Applications
V1 1. The root blowers are used for scavenging
Fig. 26.2 and supercharging of two-stroke internal
combustion engines.
Consider a volume of V1 is trapped between the 2. It is also used for low-pressure supply of air
lobe and casing at atmospheric pressure p1. The air in steel furnaces, sewage disposal plants,
is compressed to delivery pressure p2. The actual low-pressure gas boosters, and for blower
work done on air; service in general.

Ú
Wact = - Vdp = V1 ( p2 - p1 ) ...(26.1) A root blower compresses 1 m3 of air
per second from a pressure of 1.01325 bar to 1.8 bar.
The ideal work input for compression is Find the power required to run the compressor and its
isentropic work input. The theoretical work input efficiency.
to compress air from atmospheric pressure p1 to
delivery pressure p2. Solution
Given A root blower with
Ï g -1 ¸
ÔÊ p2 ˆ g
3
g Ô V = 1 m /s
Wisen = p1V1 ÌÁ ˜ - 1˝ ...(26.2) p1 = 1.01325 bar = 101.325 kPa
g -1 ÔË p1 ¯ Ô
Ó ˛ p2 = 1.8 bar = 180 kPa

The efficiency of the roots blower can be defined To find


as the ratio between adiabatic work to the actual (i) Power required to run the compressor, and
work input. Mathematically; (ii) Compressor efficiency.
Adiabatic work input Assumptions
hroots =
Actual work input (i) Compression of air at constant volume.
(ii) Compression is reversible.
Ï g -1 ¸
g ÔÊ p2 ˆ g Ô (iii) Index of isentropic compression be 1.4.
p1V1 ÌÁ ˜ - 1˝
g -1 ÔË p1 ¯ Ô Analysis
= Ó ˛ (i) The actual power required to run the compressor
V1 ( p2 - p1 ) P = V ( p2 – p1)
894 Thermal Engineering

= (1 m3/s) ¥ (180 – 101.325) (kPa) The mechanical efficiency is given by


= 78.675 kW IP
hmech =
Ideal power required to compress the air BP
or Brake power,
Ï g -1 ¸
g ÔÊ p2 ˆ g Ô IP 43.05
Pise = p1V1 ÌÁ ˜ - 1˝ BP = = = 53.81 kW
g -1 ÔË p1 ¯ Ô hmech 0.8
Ó ˛
Ï 1.4 -1 ¸
1.4 ÔÊ 1.8 ˆ 1.4 Ô VANE-TYPE COMPRESSOR
= ¥ 101.325 ¥ 1ÌÁ ˜¯ - 1˝
1.4 - 1 ÔË 1 . 01325 Ô
Ó ˛ An arrangement of a typical vane-
= 63.28 kW
(ii) Compressor efficiency;
type compressor is shown in Fig. 26.3. It consists
of an air-tight circular casing, in which a drum
Adiabatic work input
hroots = rotates about an ecentric centre of casing. The drum
Actual work input
consists of a set of spring-loaded vanes. The slots
63.28 kW
= = 0.8043 or 80.43% are cut in the drum to accomodate the vanes. The
78.675 kW
drum rotates in anticlockwise direction. During the
1 kg of air per second is taken into a rotation of the drum, the vanes remain in contact
root blower compressor at 1 bar and 27°C. The delivery with the casing. Size of inlet passage is larger than
pressure of air is 1.5 bar. Calculate the motor power the size of outlet in the compressor.
required to run the compressor, if mechanical efficiency
is 80%. Springs
Co
m
pr
n

Solution es
io
ct

sio
Su

n Rotor
Given A root blower with
m = 1 kg/s p1 = 1 bar = 100 kPa
Inlet Discharge
T1 = 27°C = 300 K p2 = 1.5 bar = 150 kPa
hmech = 0.8
To find Power required to run the compressor.
Casing
Assumptions Vanes
(i) Compression of air at constant volume.
(ii) Compression is reversible. Fig. 26.3
(iii) Specific gas constnat for air as 0.287 kJ/kg ◊ K.
As the drum rotates, the volume of air
Analysis The volume flow rate of air taken into the V1 at atmospheric pressure p1 is trapped between
compressor the vanes, drum and casing. Air gets compressed
m RT1
V = due to two operations performed on air. First the
p1
compression begins due to decreasing volume
(1 kg/s) ¥ (0.287 kJ/kg ◊ K ) ¥ (300 K ) between the drum and casing. The volume is reduced
=
(100 kPa ) to V2 and pressure increases to p2. Secondly, the
= 0.861 m3/s air is compressed due to back flow of compressed
The power required to run the compressor air in the receiver. Then the air is compressed at
IP = V (p2 – p1) constant volume to a pressure p3. The first part
= (0.861 m3/s) ¥ (150 – 100) (kPa) of compression follows adiabatic compression
= 43.05 kW
Rotary Compressor 895

process and the second part follows constant- V2 ( p3 - p2 )


volume process. The process of compression is =
È g -1 ˘
shown on the p–V diagram in Fig. 26.4 g ÍÊ p2 ˆ g ˙
p1V1 ÍÁ ˜ - 1˙ + V2 ( p3 - p2 )
g -1 Ë ¯
p
ÍÎ 1 ˙˚
...(26.5)
1. The vane-type compressor requires less
work input compared to roots blower for
same capacity and pressure ratio.
2. Vane-type compressors are commonly used
to deliver air up to 150 m3/min at a pressure
ratio up to 8.5.
3. Vane-type compressors can run up to
3000 rpm.
Fig. 26.4
4. Vane-type compressors are used for
supercharging of IC engines and supply of
Work input for adiabatic compression;
air to cupola.
Ï g -1 ¸ 5. These are portable compressors used for
g ÔÊ p2 ˆ g Ô
W12 = p1V1 ÌÁ ˜ - 1˝ construction purpose.
g -1 Ë
Ô 1p ¯ Ô
Ó ˛ Example 26.3 Calculate the power required to run
Work input for constant volume compression; the vane compressor and its efficiency, when it handles
W23 = V2 ( p3 – p2) 6 m3 of air per minute from 1 bar to 2.2. bar. The pressure
Total work input for compression within a vane; rise due to compression in the compressor is limited to
1.6 bar. Take the mechanical efficiency of compressor as
Ï g -1 ¸ 80%.
g ÔÊ p2 ˆ g Ô
Wvane = p1V1 ÌÁ ˜ - 1˝
g -1 Ë
Ô 1p ¯ Ô Solution
Ó ˛
+ V2 ( p3 – p2) Given A vane compressor with
V = 6 m3/min = 0.1 m3/s p1 = 1 bar = 100 kPa
If there are N vanes within the drum, then total
p2 = 1.6 bar = 160 kPa p3 = 2.2 bar = 220 kPa
work input;
hmech = 0.8
È Ï g -1 ¸
Í g ÔÊ p2 ˆ g Ô To find
WN vane = N Í p1V1 ÌÁ ˜ - 1˝
Íg - 1 ÔË p1 ¯
(i) Brake power required to run the compressor,
Ô
Î Ó ˛ (ii) Efficiency of the compressor.
˘ Assumptions
˙
+ V2 ( p3 - p2 ) ˙ ...(26.4) (i) Compression is reversible.
˙ (ii) Specific heat ratio for air as 1.4.
˚
The efficiency of a vane compressor can be Analysis The isentropic compression power from
state1 to state 2;
expressed as
Work input for constant- Ï g -1 ¸
g ÔÊ p2 ˆ g Ô
volume compression IP1 = p1V1 ÌÁ ˜ - 1˝
hvane, comp = g -1 Ë
Ô 1p ¯ Ô
Total work inputf or compression Ó ˛
896 Thermal Engineering

Ï 1.4 -1 ¸ To find
1.4 ÔÊ 1.6 ˆ 1.4 Ô (i) Work input to root blower and vane-type com-
= ¥ (100 kPa ) ¥ (0.1 m3/s) ¥ ÌÁ ˜¯ - 1˝
1.4 - 1 ÔË 1 Ô pressor.
Ó ˛
= 5.03 kW Assumptions
The volume of air after compression to 1.6 bar; (i) Compression is reversible.
1 1 (ii) Specific heat ratio for air as 1.4.
Ê p ˆg Ê 1 ˆ 1.4
= V1 Á 1 ˜ = (0.1 m3/s) ¥ Á
Ë 1.6 ¯˜
V2 Analysis
Ë p2 ¯
3 (i) For a root blower compressor,
= 0.07148 m /s p
Work done due to back flow from the state 2 to state 3 Pressure ratio, 2 = 1.5
p1
IP2 = V2 ( p3 – p3) or p2 = 1.5 p1 = 1.5 ¥ 1.013
= (0.07148 m3/s) ¥ (220 – 160) (kPa) = 1.5195 bar or 151.95 kPa
= 4.29 kW work input per revolution
Total power input for compression of air Wroot = V1 ( p2 – p1) = 0.05 ¥ (151.95 – 101.3)
IP = IP1 + IP2 = 5.03 + 4.29 = 9.32 kW = 2.53 kJ
The mechanical efficiency is given by (ii) For vane-type compressor:
IP p
hmech = Pressure ratio, 3 = 1.5
BP p1
or p3 = 1.5 p1 = 1.5 ¥ 1.013
(i) The Brake power required to run the motor or
= 1.5195 bar or 151.95 kPa
shaft power
Intermediate pressure;
IP 9.32 p2 = p1 + 0.5 (p3 – p1)
BP = = = 11.65 kW
hmech 0.8 = 101.3 + 0.5 ¥ (151.95 – 101.3)
(ii) Efficiency of the vane compressor, Eq. (26.5); = 126.625 kPa
Work input for constant The volume of air after intermediate compression
1 1
volume compression Ê p ˆg Ê 101.3 ˆ 1.4
hvane comp = V2 = V1 Á 1 ˜ = (0.05 m3 ) ¥ Á
Total work inputf or compression Ë p2 ¯ Ë 126.625 ¯˜
4.29 kW = 0.0426 m3/rev
= = 0.46 or 46%
9.32 kW Work input per revolution for vane-type
compressor
Example 26.4 Compare the work required for com-
Ï g -1 ¸
pression in a root blower and vane blower compressors g ÔÊ p2 ˆ g Ô
for the following particulars: WVane = p1V1 ÌÁ ˜ - 1˝
g -1 Ë
Ô 1p ¯ Ô
Intake volume = 0.05 m3 per revolution Ó ˛
Inlet pressure = 1.013 bar + V2 ( p2 – p1) = 0
Pressure ratio = 1.5 Ï 1.4 -1 ¸
For the vane-type compressor, internal compression 1.4 ÔÊ 126.625 ˆ 1.4 Ô
= ¥ 101.3 ¥ 0.05 ¥ ÌÁ ˜¯ - 1˝
takes place through half the pressure range. 1.4 - 1 ÔË 101 . 3 Ô
Ó ˛
Solution + 0.0426 ¥ (151.95 – 126.625)
= 1.167 + 1.079 = 2.246 kJ/rev
Given Compression with
V1 = 0.05 m3/rev The vane-type compressor requires less
p1 = 1.013 bar = 101.3 kPa work input for compression of given volume of air for
Pressure ratio = 1.5 same pressure ratio.
Rotary Compressor 897

26.4 LYSHOLM COMPRESSOR—A the male rotor is made with four lobes along the
SCREW COMPRESSOR length of the rotor that meshes with similarly
formed correspondingly six helical flute on the
The screw compressors are most commonly used auxiliary (female) rotor.
rotary air compressor. They are single stage helical The auxiliary rotors seal the working space
or spiral lobe, oil flooded screw air compressor. between the suction and pressure side. In the course
They are simple in design and have few wearing of rotation, main and auxiliary rotors generate a
parts, and thus easy to install, maintain and operate. V-shaped space for the air drawn in, which becomes
There are available in wide range of pressure smaller and smaller right up to the end, between the
ratio and capacity. Lubricated types are available in rotor lobes and the cylinder walls.
sizes ranging from 200 to 2000 m3 per hour with Because of the number of male lobes, there
discharge pressure up to 10 bar. are four compression cycles per revolution which
The oil free rotary screw air compressor uses means that the resulting compressed air has small
specially designed machine. It compresses air pulsations compared to a reciprocating compressor.
without oil in the compression chamber. These
compressors are air or water cooled machines.
They deliver oil-free air and are available in sizes As rotors rotate, the air is drawn through the inlet
up to 30,000 m3 per hour and pressure upto 15 bar. port to fill the space between the male lobe and
Alf Lysholm had developed the modern screw female flute. As rotors continue to rotate, the air
compressor, which is known as Lysholm compressor. is moved past the suction port and sealed in the
Its construction and working are discussed below. interlobe space. The trapped air is moved axially
and radially and is compressed by direct volume
reduction as enmeshing of lobes of compressor
Screw compressors are equipped with two mating progressively reduces the space occupied by the
helical grooved rotors housed within a cylindrical gas with increase in pressure. Simultaneously, with
casing equipped with inlet and discharge ports as this process, the oil is injected into the system. The
shown in Fig. 26.5. oil seals the internal clearances and it absorbs the
heat energy generated during compression. The
compression of air continues until the interlobe
space communicates with discharge port in the
casing. The compressed air leaves the casing
through the discharge port.
The working parts of compressor never get
severe operating temperatures, since the cooling
Fig. 26.5 takes place right inside the compressor. The oil is
separated out in the oil separator and cooled down
The main (male) rotor is normally driven by in an oil cooler and is returned back to compressor
either an electric motor or an engine and transforms through an oil filter.
about 85–90% of the energy received at the The internal volume ratio of a screw compressor
coupling into pressure and heat energy. The number is defined as the ratio of the volume of flute at the
of lobes (or valleys) on the rotors will vary from start of compression process to volume of the same
one compressor manufacturer to another. Usually, flute as it begins to open to discharge port.
898 Thermal Engineering

CENTRIFUGAL COMPRESSOR The impeller rotates inside the casing. The impeller
is usually forged or die casting of aluminium alloy.
The centrifugal compressors are dynamic action The centre of the impeller is called the eye. The
compressors. These compressors have appreciably eye of the impeller is connected with the drive shaft.
different characteristics as compared to reciprocat- The casing of the compressor has a volute shape. A
ing machines. A small change in compression ratio diffuser ring is housed in the radial portion of the
produces a marked change in compressor output casing.
and efficiency. Centrifugal machines are better
suited for applications requiring very high capaci-
ties, typically above 3000 m3/min and a moderate
As the impeller rotates, the air enters radially
pressure ratio of 4 to 6. They are preferred due to
into the impeller eye with low velocity V1 at
their simplicity, light weight and ruggedness.
atmospheric pressure p1. Due to centrifugal action
The centrifugal air compressor is an oil-free of the impeller, the air comes radially out and
compressor by design. The oil-lubricated running during its movement, it is guided by the blades
gear is separated from the air by shaft seals and at- within the impeller.
mospheric vents. It is a continuous duty compres-
The high velocity of the impeller increases the
sor, with few moving parts, and is particularly suit-
momentum of air, causing rise in static pressure,
ed to high volume applications, especially where
temperature and kinetic energy of air. The pressure,
oil-free air is required.
temperature and velocity of air leaving the impeller
are p2, T2 and V2 , respectively.
The air leaving the outside edge of the impeller
The basic components of a typical centrifugal enters into the diffuser ring where its kinetic energy
compressor includes an impeller, diffuser and is converted into pressure energy. Thus, the static
casing as shown in Fig. 26.6. The impeller is a pressure of air is further increased. The air is then
radial disc with a series of radial blades (vanes). collected in the casing and discharged from the
Tip
Impeller
compressor. The change in pressure and velocity
eye of air passing the impeller and diffuser passage are
Air in take Shaft
shown in Fig. 26.7.

In a centrifugal air compressor, the air enters the


(a) Impeller eye impeller radially and leaves axially. Hence the
Discharge
scroll
Air
flow

Impeller
Diffuser passages Blades

(b) Sectional view of centrifugal compressor


Fig. 26.7
Fig. 26.6
Rotary Compressor 899

The work transfer per kg of air to centrifugal


compressor can also be obtained by using steady-
flow energy equation
Ê V2 V2 ˆ
q – w = ( h2 - h1 ) + Á 2 - 1 ˜ + Dpe
Ë 2 2 ¯
For centrifugal air compressor, q = 0,
Change in potential energy, D pe = 0;
Omitting the negative sign for work input, the
compresson work per kg of air
Ê V2 ˆ Ê V2 ˆ
w = Á h2 + 2 ˜ - Á h1 + 1 ˜
Ë 2 ¯ Ë 2 ¯
or w = (h02 – h01) = Cp (T02 – T01)
ÊT ˆ
= Cp T01 Á 02 - 1˜ ...(26.8)
Ë T01 ¯
The static state and stagnation state are shown in
Fig. 26.9. The stagnation temperatures T01 and T02
can be expressed as
g -1
Fig. 26.8 Êp ˆ g
T01 = T1 Á 01 ˜
Ë p1 ¯
g -1
blades are designed in such a way that the air enters Êp ˆ g
and leaves the blades without shock. T02 = T2 Á 02 ˜
Ë p2 ¯
Using usual notation, let
g -1
u1 = Blade velocity at inlet T02
and = Ê p02 ˆ g
Vr1 = Relative velocity of blades at inlet T01 ÁË p ˜¯
01
V1 = Absolute velocity of inlet air
Hence the work input to compressor per kg of
Vf1 = Inlet flow velocity air can be expressed as
a = Air inlet angle g -1
Ê ˆ
b = Blade angle at inlet Á Ê p02 ˆ g
w = Cp T01 Á - 1˜ ...(26.9)
u2, Vr2, V2, Vf2, q, and f are the corresponding Á Ë p01 ˜¯ ˜
ÁË ˜¯
values at outlet.
Work input per kg of air
w = u1Vw1 + u2 Vw2
Since the working fluid enters radially, i.e., a =
90°, thus Vw1 = 0
Hence work input by blade per kg of air
w = u2 Vw2 (J/kg) ...(26.6)
The above equation is known as Euler’s equation
or Euler’s work.
If m is the mass flow rate of air in kg/s, then
power input to compressor
Fig. 26.9
P = m u2 Vw 2 (Joules) ...(26.7)
900 Thermal Engineering

If V1 = V2, then stagnation pressure, p01 = air in the impeller, i.e.,


p1(static pressure) and p02 = p2, T01 = T1;
Vr12 - Vr22 u22 - u12
Ê g -1 ˆ D ke = + ...(26.16)
Ê 2 2
p2 ˆ g
w = Cp T1 Á Á ˜ - 1˜ ...(26.10)
Á Ë p1 ¯ ˜ The first term above Eq. (26.16) indicates the
ÁË ˜¯ pressure rise in the impeller due to diffusion action,
and the second term represents the pressure rise
in the compressor due to centrifugal action of the
impeller.
For constant mass flow rate of air, the width of the Total pressure rise in the compressor is equal to
blades of impeller can be calculated as follows: work input to the compressor.
AV1 \ degree of reaction
m = (Continuity equation) Pressure rise in the impeller
v1 Rd =
where v1 = specific volume air at inlet Pressure rise in the compressoor
A = p D1B1, with D1, diameter of impeller at Vr12 – Vr22u22 – u12
+ ( Vr2 – Vr22 ) + (u22 – u12 )
inlet and B1 as width of impeller blades at inlet. = 2 2 = 1
As air trapped radially, V1 = Vf1 u2 Vw2 2u2 Vw2

p D1 B1 (u22 - Vr22 ) + ( Vr12 - u12 )


\ m = Vf1 ...(26.11) or Rd = ...(26.17)
v1 2u2 Vw2
m v1 From the inlet velocity triangle of Fig. 26.8;
or B1 = ...(26.12)
p D1 Vf1 Vr12 – u12 = Vf21 = Vf22 (since Vf 1 = Vf 2 )
Similarly, the width of impeller blade at the ...(26.18)
outlet can be obtained by using the suffix 2 in From outlet velocity trinagle of Fig. 26.8;
Eq. (26.12) Vf22 + (u2 - Vw2 ) 2 = Vr22
m v2
B2 = ...(26.13) or Vr22 = Vf22 + u22 + Vw22 – 2u2 Vw2
p D2 Vf 2
or u22 – Vr22 = 2u2 Vw2 – Vf22 – Vw22 ...(26.19)
If the number and thickness of blades are
considered then the effective area of the blade will Using Eqs. (26.18) and (26.19) in Eq. (26.17),
be we get
A = p D – nt 2u2 Vw2 - Vf22 - Vw22 + Vf22
Rd =
where n = number of blades and t = thickness 2u2 Vw 2
of blade. Vw2
m v1 or Rd = 1 - ...(26.20)
Then B1 = ...(26.14) 2u2
(p D1 - nt ) Vf1
m v2
and B2 = ...(26.15)
(p D2 - nt ) Vf 2 Since the flow velocity remains constant at inlet and
outlet, therefore, the inlet and exit velocity triangles
can be drawn on the common base. Since air enters
The degree of reaction is defined as the ratio of at right angles to the impeller blade, thus the air
static pressure rise in the impeller to the total static inlet angle at the impeller is equal to 90°, i.e., a
pressure rise in the compressor. The pressure rise in = 90°. Combined velocity triangle for centrifugal
the impeller is equal to change in kinetic energy of compressor is shown in Fig. 26.10.
Rotary Compressor 901

Fig. 26.11
Fig. 26.10

Procedure
Compressor
1. Draw a vertical line AB to represent the flow
velocity and it remains constant at inlet and The following losses occur in a centrifugal
exit. compressor, when air flows through the impeller:
2. The horizontal line CA represents the blade 1. Friction between moving air layers and
velocity u1 at inlet. impeller blades and friction between air
3. The line CB inclined at the blade angle b layers moving with relative velocities,
represents relative velocity Vr1 of blade at 2. Shock at entry, and
the inlet. 3. Turbulence caused in air.
4. The line DB inclined at blade angle f
represents the relative velocity Vr2 of the These losses cause an increase in enthalpy of air
blade at the outlet. without increasing the pressure of air. Therefore,
the actual temperature of air coming out of the
5. The line DE represents the blade velocity u2
compressor is more than the temperature of air
of impeller at outlet.
at the inlet. The actual work input for the same
6. Join the line EB. It represents absolute
pressure ratio is more due to irreversibilities. The
velocity V2 of air at the outlet inclined at
actual and isentropic compression for pressure ratio
angle q with respect to the horizontal.
is shown in Fig. 26.12.
From the combined velocity triangle, the work Since the cooling arrangement is not provided
input to the compressor per kg of air in dynamic compressors, the ideal compression
w = u2 Vw2 (J/kg)
Power input; P = m w = m u2 Vw2 (W)
If the air flow through the impeller blade is radial
(ideal case), the velocity diagram at the outlet takes
the shape as shown in Fig. 26.11.
In this case, the blade velocity at the outlet
becomes equal to whirl velocity at the outlet, i.e.,
u2 = Vw2. The work input per kg of air is
w = u 22 (J/kg) ...(26.21)
The exit whirl velocity Vw2 of air cannot be
greater than the blade tip velocity. Thus, it is the
limiting case and it is the maximum work supplied Fig. 26.12
to air per kg.
902 Thermal Engineering

process in dynamic compressors is isentropic Actual work input


compression. But the actual work input for fw =
Euler work input
compression is always more than isentropic work
input for compression through same pressure ratio. C p (T02 - T01 )
= ...(26.26)
The isentropic efficiency (hisen ) of a dynamic u2 Vw2
compressor is given by
Pressure coefficient is defined as the ratio of
Isentropic work input h02 s - h01 isentropic work to Euler work. It is designated as
hisen = =
Actual work input h02 - h01 f p.
...(26.22)
Isentropic work input
If specific heat Cp of air remains constant then fp =
Euler work input
T02 s - T01
hisen = ...(26.23) C p (T02 s - T01 )
T02 - T01 = ...(26.27)
u2 Vw2
If inlet velocity of air is equal to air exit velocity,
i.e., V1 = V2 then Using Eq. (26.23), and assuming radial vanes of
T -T impeller (u2 = Vw2), then
hisen = 2 s 1
T2 - T1
hisenC p (T02 - T01 )
Isentropic temperature rise fp = ...(26.28)
= ...(26.24) u22
Actual temperature rise
where suffix 1 indicates the inlet state, 2s indicates From Eq. (26.26), we get
the state after isentropic expansion, 2; the state after
Cp (T02 – T01) = fw u2 Vw2
actual expansion, and the properties with suffix 0
indicates corresponding stagnation properties. Substituting the Vw2 as fs u2 ; from Eq. (26.25),
we get
Cp (T02 – T01) = fw fs u22
A centrifugal compressor has maximum work input
Using in Eq. (26.28), we get;
when u2 = Vw2. In actual practice, the whirl velocity
Vw2 is always less than the blade tip velocity u2. The hisenfw f s u22
difference between blade tip velocity u2 and whirl fp = = fw fs hisen ...(26.29)
u22
velocity Vw2 is known as slip, i.e., slip = u2 – Vw2.
The slip factor is defined as the ratio of whirl
velocity to blade tip velocity. It is designated as fs
and expressed as There are usually three types of impeller blade
V shapes used in a centrifugal compressor. These are
Slip factor, fs = w2 ...(26.25)
u2
1. Backward curved blades (q < 90°)
2. Radial blades (q = 90°)
3. Forward curved blades (q > 90°)
The actual work done per kg of air by compressor
is always greater than the impeller work u2 Vw2 due Figure 26.13 shows the geometry of backward,
to fluid friction and windage losses. Work factor or radial and forward curved vanes and performance
power input factor is defined as ratio of actual work of these vanes. The centrifugal action on the curved
input to Euler work input. It is designated as fw and vanes creates bending moment and induces bending
is given as stresses.
Rotary Compressor 903

radial vane impeller, the diffuser contributes about


one half of the overall static pressure rise.
A diffuser consists of curved vanes, which are
used to minimize the whirl of high speed air with
smallest possible flow path and diameter. The flow of
air through the diffuser vanes may be approximated
as logarithmic spiral path. For fast diffusion, the
axis of vanes is straight and tangential to spiral.
Usually, the number of diffuser passages are less
than the impeller passages for more uniform and
smooth flow.
Figure 26.14 shows a typical curved vanes
diffuser along with an impeller. The clearance
provided between the impeller and diffuser rings
acts as a vaneless diffuser and it functions to
Fig. 26.13
1. smooth out velocity variation betweem the
impeller tip and diffuser vanes
The pressure head of delivered air decreases 2. reduce circumferential pressure gradient at
with increase in mass-flow rate for backward the impeller tip
curved vanes, while the pressure head increases for 3. reduce the velocity at the entry of vanes
forward curved vanes. But for radial curved vanes,
Vaned
the pressure head of delivered air remains constant diffuser
with mass-flow rate.
The backward vanes are normally used with
q = 20° to 25°, except for delivery of air at high
head. The radial vane is the compromise between
Impeller
backward and forward curved vanes. Therefore,
the radial vane impeller is most commonly used
in a centrifugal compressor due to the following Diffuser vanes Diffuser passage
reasons: Fig. 26.14
1. Radial-vane geometry is simple, thus vanes
can be manufactures easily.
2. Radial vanes have lowest bending stress for At constant impeller speed, the decrease
given diameter and speed as compared to in pressure ratio leads to an increase in mass-flow
forward and backward curved vanes. rate and hence the density of compressed air is
3. Radial vanes have a constant pressure head decreased. Consequently, the radial velocity of air
in the impeller as well as in the diffuser. increases, which increases the absolute velocity of
4. A radial-vane impeller has good efficiency air at impeller exit and incidence angle at diffuser
and high pressure head. vane tip. The slope of the characteristic curve
decreases and finally the point A is reached as
shown in Fig. 26.15. The mass-flow rate of fluid
cannot be increased beyond the point A. This point
In a centrifugal compressor, the diffuser converts
is called choking state.
kinetic energy of air into static pressure head. For a
904 Thermal Engineering

Surging in the line causes unstable compressor


operation. Surging occurs when the operating points
of the compressor get into the unstable area of the
operating curve. During surging, the compressor
shows cyclic flow and back-flow of the compressed
air resulting into high vibrations, pressure shocks
and over heating. The breakdown of flow due to
persistent surging may lead to heavy damages.
Consequences of surging can
Fig. 26.15 include

1. Rapid flow and pressure oscillations causing


Consider a compressor is running at process instabilities
constant speed and the full valve is open on 2. Rising temperatures inside the compressor
delivery side at a pressure ratio located by the point 3. Tripping of the compressor
B on the characteristic curve shown in Fig. 26.15. 4. Mechanical damage
If any resistance is placed in the delivery path of a Mechanical damage can include
compressor, or by partial closing of the valve, the
mass-flow rate decreases with increase in pressure ÷ Radial bearing load during the initial
ratio, and the operating point will shift towards left phase of surging
on characteristic curve, say at the point C from the ÷ Thrust bearing load due to loading and
point B. unloading
If resistance is further increased in the delivery ÷ Seal rubbing
path, the mass-flow rate of air through the ÷ Stationary and rotating part contact, if
compressor decreases and the compressor operating thrust bearing is overloaded
point will shift toward left till it reaches the point
D, the operating point for maximum pressure ratio.
If resistance is further increased, the mass-flow
rate will decrease and reach the zone D–E on the
curve, with decrease in pressure ratio also. In this Sr. Reciprocating Centrifugal
situation, the pressure in downstream line will be No. Compressor Compressor
more than the pressure of air at the actual delivery 1. Geater noise and Compratively salient
of the compressor. This situation leads to a stop of vibrations operation
fluid flow and sometimes, flow of fluid in reverse 2. Poor mechanical Better mechanical
direction. efficiency due to large efficiency due to
Within a short interval of time, the pressure is sliding parts absence of sliding
built up within the compressor due to accumulation parts
of mass; and the compressor may again start to 3. Installation cost is Installation cost is
deliver the fluid. If downstream conditions remain higher lower
unchanged then fluid flow will again break down, as 4. Pressure ratio up to Pressure ratio up to 4
pressure of delivered air is subsidized and the cycle 5 to 8
will be repeated with high frequency. This situation
of instability is known as surging or pumping. Contd.
Rotary Compressor 905

5. Higher pressure It is not suitable for Analysis The ratio of two specific heats
ratio up to 500 atm. multistaging Cp 1.00
g = = = 1.4
is possible with Cv 0.716
multistaging of The temperature of air after isentropic compression
compressor g -1 1.4 -1
6. It runs intermittantly It runs continuously Êp ˆ g Ê 2ˆ 1.4
T2s = T1 Á 2 ˜ = (300 K ) ¥ Á ˜
and delivers pulsating and delivers steady Ë p1 ¯ Ë 1¯
air and pulsating free air = 365.7 K
7. Less volume is Large volume is (i) Isentropic efficiency is given by;
handled handled
T2 s - T1 Isentropic temperature rise
8. More maintenance is Less maintenance is hisen = =
T2 - T1 Actual temperature rise
required required
365.7 - 300
= = 0.8423 or 84.23%
9. Weight of compressor Comparatively less 378 - 300
is more weight (ii) The power input for compression
P = mC p (T2 – T1) = 0.5 ¥ 1.0 ¥ ( 378 – 300)
10. It operates at low It operates at high
speed speed = 39 kW

11. Isothermal efficiency Isentropic efficiency Example 26.6 A centrifugal compressor compresses
should be better should be better air from 1 bar at 15°C to 2.15 bar, 95°C. The mass of air
12. Higher compression Higher compression delivered is 2.2 kg/s and no heat is added to the air from
efficiency at pressure efficiency, if pressure external sources during compression. Find the efficiency
ratio more than 2 ratio less than 2 of the compressor relative to ideal adiabatic compression
13. Suitable for low Suitable for high and estimate the power absorbed. Also, find the change in
discharge and high discharge and low entropy of air during compression.
pressure ratio pressure ratio
Solution
Example 26.5 In a centrifugal compressor, the air Given A centrifugal compressor with
enters at 27°C and leaves at 105°C. The air is compressed m = 2.2 kg/s p1 = 1 bar
through a pressure ratio of 2. Calculate the isentropic T1 = 15°C = 288 K p2 = 2.15 bar
efficiency and power required by the compressor, if 30 kg T2 = 95°C = 368 K Q =0
of air is compressed per minute. Take Cp = 1.00 kJ/kg ◊ K
and Cv = 0.716 kJ/kg ◊ K. To find
(i) Isentropic efficiency of the compressor,
Solution (ii) Power required to run the compressor,
Given A centrifugal compressor with (iii) Entropy change during compression process.
m = 30 kg/min = 0.5 kg/s Assumptions
p2
= 2.0
p1 For air Cp = 1.00 kJ/kg ◊ K
T1 = 27°C = 300 K g = 1.4 and R = 0.287 kJ/kg ◊ K
T2 = 105°C = 378 K
Analysis The temperature of air after isentropic
Cp = 1.00 kJ/kg ◊ K compression
Cv = 0.716 kJ/kg ◊ K g -1 1.4 -1
Êp ˆ g Ê 2.15 ˆ 1.4
T2s = T1 Á 2 ˜ = ( 288 K ) ¥ Á
Ë 1 ˜¯
To find
Ë p1 ¯
(i) Isentropic efficiency of the compressor, and
= 358.4 K
(ii) Power required to run the compressor.
906 Thermal Engineering

(i) Isentropic efficiency is given by


T2 s - T1 358.4 - 288
hisen = =
T2 - T1 368 - 280
= 0.880 or 80%
(ii) The power input for compression
In absence of change in kinetic and potential
energies and heat transfer, the steady-flow energy
equation for compressor leads to
win = h2 – h1 = Cp (T2 – T1) Fig. 26.16
= 1.005 ¥ ( 358.4 – 288)
= 70.75 kJ/kg The linear velocity of impeller at the outlet
The power input p D2 N p ¥ 0.5 ¥ 2000
P = mwin u2 = = = 52.36 m/s
60 60
= 2.2 ¥ 70.75 = 155.65 kW With the use of blade velocities and blade angles at the
(iii) Entropy change during compression process: inlet and outlet, the velocity triangle can be constructed
The entropy change during a process can be as follows:
obtained as (i) The horizontal line CA represents the blade
È ÊT ˆ Ê p ˆ˘ velocity u1 = 31.416 m/s at inlet.
D S = m ÍC p ln Á 2 ˜ - R ln Á 2 ˜ ˙
ÍÎ Ë T1 ¯ Ë p1 ¯ ˚˙ (ii) Draw line CB inclined at 22°.
È
(iii) From the point D, draw line DB inclined at angle
Ê 368 ˆ Ê 2.15 ˆ ˘
= 2.2 ¥ Í1.005 ¥ ln Á - 0.287 ¥ ln Á ˙ f = 40°; which cuts line CB at point B.
Î Ë 288 ˜¯ Ë 1 ˜¯ ˚
(iv) Draw a vertical line AB to represent the flow
= 0.586 kJ/K velocity.
(v) Draw the line DE to represent blade velocity u2 =
Example 26.7 A centrifugal compressor running
52.36 m/s.
at 2000 rpm has internal and external diameters of the
(vi) Join the line EB to represent exit velocity of air
impeller as 300 mm and 500 mm, respectively. The blade
V2.
angles at inlet and outlet are 22° and 40°, respectively.
The air enters the impeller radially. Determine the work From the velocity triangles, the measurements give
done by the compressor per kg of air and degree of Vw2 = Length of AE = 37.3 m/s,
reaction. Vr1 = Length of CB = 34.1 m/s,
Vr2 = Length of DB = 19.9 m/s.
Solution (i) Work input per kg of air
Given A centrifugal compressor with w = Vw2 u2 = 37.3 ¥ 52.36 = 1953.02 J/kg
N = 2000 rpm D1 = 300 mm = 0.3 m = 1.953 kJ/kg
b = 22° D2 = 500 mm = 0.5 m (ii) Degree of reaction;
f = 40° Pressure rise in the impeller
Rd =
To find Pressure rise in the compressoor
(i) Work input to the compressor, (u22 - Vr2 ) + ( Vr2 - u12 )
2 1
=
(ii) Degree of reaction. 2u2 Vw 2
Analysis The linear velocity of impeller at inlet (52.36 2 - 19.92 ) + (34.12 - 31.416 2 )
=
p D1N p ¥ 0.3 ¥ 2000 2 ¥ 52.36 ¥ 37.3
u1 = =
60 60 = 0.6455 or 64.55%
= 31.416 m/s
Rotary Compressor 907

Example 26.8 A centrifugal compressor running Vr1 = Length of CB = 11 m/s


at 1440 rpm, handles air at 101 kPa and 20°C and Vr2 = Length of DB = 4.7 m/s
compresses it to a pressure of 6 bar isentropically. The Vf1 = 3 m/s
inner and outer diameters of the impeller are 14 cm and Work input per kg of air
28 cm, respectively. The width of the blade at the inlet is w = Vw2 u2 = 17.5 ¥ 21.12 = 369.6 J/kg
2.5 cm. The blade angles are 16° and 40° at entry and (i) Mass-flow rate of air
exit. Calculate mass-flow rate of air, degree of reaction,
m = r1 A1 Vf1 = r1 p D1 B1 Vf1
power input and width of blades at outlet.
Density of air
Solution p 101
r = 1 =
RT1 0.287 ¥ 293
Given A centrifugal compressor with
= 1.201 kg/m3
N = 1440 rpm D1 = 14 cm = 0.14 m
\ m = 1.201 ¥ p ¥ 0.14 ¥ 0.025 ¥ 3
b = 16° D2 = 28 cm = 0.28 m
= 0.0396 kg/s
f = 40° p1 = 101 kPa
(ii) Power input to run the compressor
p2 = 6 bar = 600 kPa T1 = 20°C = 293 K
P = m w = 0.0396 ¥ 369.6 = 14.61 W
B1 = 2.5 cm = 0.025 m
(iii) Degree of reaction
To find
(u22 - Vr2 ) + ( Vr2 - u12 )
(i) Mass flow rate of air, Rd = 2 1

(ii) Power input to the compressor, 2u2 Vw 2


(iii) Degree of reaction, and ( 21.122 - 4.72 ) + (112 - 10.56 2 )
=
(iv) Width of blade at outlet. 2 ¥ 17.5 ¥ 21.12
= 0.5864 or 58.64%
Assumption The specific gas constant as 0.287 kJ/kg ◊ K.
(iv) Width of blade at outlet
Analysis The linear velocity of impeller at the inlet Flow velocity at outlet
p D1N p ¥ 0.14 ¥ 1440 Vf2 = Vf1 = 3.0 m/s
u1 = = = 10.56 m/s
60 60 Temperature of air at the outlet
The linear velocity of impeller at the outlet g -1 1.4 -1
p D2 N p ¥ 0.28 ¥ 1440 Êp ˆ g Ê 600 ˆ 1.4
T2 = T1 Á 2 ˜ = 293 ¥ Á
u2 =
60
=
60
= 21.12 m/s
Ë p1 ¯ Ë 101 ˜¯
With the use of blade velocities and blade angles at = 487.48 K
inlet and outlet, the velocity triangle can be constructed Density of air at the outlet
as follows: p 600
r2 = 2 =
From the velocity triangles, the measurements give RT2 0.287 ¥ 487.48
Vw2 = Length of AE = 17.5 m/s = 4.288 kg/m3
From continuity equation;
m = r2 pD2 B2 Vf2
m 0.0396
or B2 = =
r2p D2 Vf 2 4.288 ¥ p ¥ 0.28 ¥ 3
= 0.0035 m = 3.5 mm

Example 26.9 A centrifugal compressor runing at


12000 rpm delivers 600 m3/min of free air. The air is
compressed from 1 bar and 27°C to a pressure ratio of
Fig. 26.17 4 with an isentropic efficiency of 85%. The blades are
radial at the impeller outlet and flow velocity of 60 m/s
908 Thermal Engineering

may be assumed throughout constant. The outer radius (ii) Power input to compressor
of the impeller is twice the inner one and slip factor is Work input per kg of air
0.9. Calculate w = Cp (T2 – T1) = 1.005 ¥ (471.52 – 300)
(a) Final temperature of air = 172.37 kJ/kg
(b) Power input to compressor Power input
(c) Impeller diameter at inlet and outlet P = mw = 11.61 ¥ 172.37 = 2001.3 kW
(d) Width of impeller at inlet (iii) Impeller diameters at inlet and outlet
For radial blades, the work input to compressor
Solution
with slip is given by
Given A centrifugal compressor with w = fs u 22 (J/kg)
N = 12000 rpm r2 = 2 r1 Using numerical values and equating with work
p1 = 1 bar = 100 kPa T1 = 27°C = 300 K obtained above
p2
=4 Vf1 = 60 m/s 0.9 ¥ u22 = 172.37 ¥ 103 (J/kg)
p1 It gives u2 = 437.63 m/s
hisen = 0.85 The linear blade velocity at impeller tip is given
V = 600 m3/min = 10 m3/s by
fs = 0.9 p D2 N
u2 =
60
To find
It gives impeller diameter at outlet
(i) Final temperature of air, 437.63 ¥ 60
(ii) Power input to the compressor, D2 =
p ¥ 12000
(iii) Impeller diameter at inlet and outlet, and = 0.6965 m or 69.65 cm
(iv) Width of impeller at inlet. Impeller diameter at inlet
The specific heat of air as 1005 J/kg ◊ K D 69.65
Assumption D1 = 2 =
and g = 1.4 2 2
= 34.825 cm
Analysis
(iv) Width of impeller at inlet, Eq. (26.12)
(i) Final temperature of air m v1 V
g -1 B1 = =
Êp ˆ g p D1 Vf1 p D1 Vf1
T2s = T1 Á 2 ˜
Ë p1 ¯ 10 m3/s
=
1.4 -1 p ¥ 0.34825 ¥ 60
= 300 ¥ ( 4) 1.4 = 445.79 K = 0.1523 m or 15.23 cm
The isentropic efficiency is given by
Example 26.10 A centrifugal compressor handles
T2 s - T1
hisen = 600 kg/min of air.The ambient air conditions are 1 bar
T2 - T1
and 27°C. The compressor runs at 18000 rpm with an
Final temperature of air is given by isentropic efficiency of 80%. The air is compressed in
T2 s - T1 the compressor from 1 bar static pressure to 4 bar total
T2 = + T1
hisen pressure. The air enters the impeller eye with a velocity
of 150 m/s with no prewhirl. Take the ratio of whirl speed
445.79 - 300
= + 300 to tip speed as 0.9. Calculate
0.85
(a) rise in total temperature during compression if
= 471.52 K or 198.52°C change in kinetic energy is negligible
The mass flow rate of air (b) tip diameter of impeller
pV 100 ¥ 10 (c) power required
m = = = 11.61 kg/s
RT 0.287 ¥ 300 (d) Eye diameter, if hub diameter is 10 cm
Rotary Compressor 909

p02
Solution T 02 p2
Given A centrifugal compressor with 02s 2
T02
N = 18000 rpm Dh = 10 cm = 0.1 m T2 2s
p1 = 1 bar = 100 kPa T1 = 27°C = 300 K p01
p02 = 4 bar = 400 kPa V1 = 150 m/s

10 cm
Dh
p1

D2
Vw 2 01
= 0.9 hisen = 0.8 T01
u2
T1 1
m = 600 kg/min = 10 kg/s Vw1 = 0
s
To find
(i) Rise in total temperature of air, Fig. 26.18
(ii) Tip diameter of the impeller,
The isentropic efficiency in terms of stagnation
(iii) Power input, and
temperatures may be given as
(iv) Eye diameter.
T -T
hisen = 02 s 01
Assumption The specific heat of air as 1005 J/kg ◊ K T02 - T01
and g = 1.4 Actual rise in stagnation temperature is given by
Analysis T02 s - T01 134.6
T02 – T01 = =
(i) Rise in total temperature of air hisen 0.8
Stagnation temperature at the inlet of compressor = 168.25°C
V12 150 2
T01 = T1 + = 300 + (ii) Tip diameter of impeller, D2
2C p 2 ¥ 1005 Work input to compressor per kg of air
= 311.19 K w = Cp (T02 – T01) = 1.005 ¥ 168.25
The stagnation pressure at compressor inlet = 169.09 kJ/kg
g
Work input per kg of air to compressor is also
Ê T ˆ g -1
p01 = p1 Á 01 ˜ given by
Ë T ¯
1 w = u2 Vw2 (J/kg)
Ê Vw2 ˆ
1.4 or 169.09 ¥ 103 = 0.9 u 22 ÁË∵ u = 0.9˜¯
Ê 311.19 ˆ 1.4 -1 2
= (1 bar ) ¥ Á or u2 = 433.45 m/s
Ë 300 ˜¯
The blade velocity is given by
= 1.137 bar p D2 N
Stagnation temperature after isentropic compres- u2 =
60
sion u2 ¥ 60 433.45 ¥ 60
g -1
or D2 = =
pN p ¥ 18000
Êp ˆ g
T02s = T01 Á 02 ˜ = 0.4599 m ª 46 cm
Ë p01 ¯
(iii) Power input to compressor
1.4 -1
Ê 4 ˆ 1.4
P = mw = 10 ¥ 169.09 = 1690.9 kW
= 311.19 ¥ Á
Ë 1.137 ˜¯ (iv) Eye diameter, D1
The density of air at compressor inlet is given by
= 445.79 K
p 100
Isentropic rise in stagnation temperature r1 = 1 =
RT1 0.287 ¥ 300
= T02 s – T01
= 445.79 – 311.19 = 134.6°C = 1.161 kg/m3
910 Thermal Engineering

The mass-flow rate through impeller eye can be The stagnation pressure at compressor inlet
given by g

m =
p 2
4
(
D1 - Dh2 ¥ V1 ¥ r1 ) p01
Ê T ˆ g -1
= p1 Á 01 ˜
Ë T ¯
1
or 10 =
p 2
4
(
D1 - 0.12 ¥ 150 ¥ 1.161 ) 1.4
Ê 248.1ˆ 1.4 -1
= (0.23 bar ) ¥ Á
10 ¥ 4 Ë 217 ˜¯
or D1 = + 0.12 = 0.3675 bar
p ¥ 150 ¥ 1.161
The blade velocity at exit is given by
= 0.288 m = 28.8 cm
p D2 N
u2 =
Example 26.11 An aircraft engine is fitted with a 60
p ¥ 0.54 ¥ 15000
single-sided centrifugal compressor. The aircraft flies or u2 = = 424.11 m/s
with a speed of 900 km/h at an altitude, where the 60
The whirl velocity at exit
pressure is 0.23 bar and temperature is 217 K. The inlet
duct of the impeller eye contains fixed vanes, which gives Vw2 = fs u2 = 0.9 ¥ 424.11 = 381.7 m/s
the air prewhirl of 25° at all radii. The inner and outer The power input factor is given as
diameter of the eye are 180 and 330 mm, respectively. C p (T02 - T01 )
fw =
The diameter of the impeller tip is 540 mm and u2 Vw2
rotational speed is 15000 rpm. Estimate the stagnation The stagnation temperature at exit
pressure at compressor outlet when the mass flow rate is fw u2 Vw2
210 kg/ minute. T02 = + T01
Cp
Neglect losses in inlet duct and fixed vanes, and
assume that the isentropic efficiency of the compressor 1.04 ¥ 424.11 ¥ 381.7
= + 248.1
is 80%. Take slip factor as 0.9 and power input factor 1005
as 1.04. = 415.62 K
The isentropic efficiency in terms of stagnation
Solution temperatures may be given as
T -T
Given A single-sided centrifugal compressor of an hisen = 02 s 01
airfraft with T02 - T01
N = 15000 rpm Dh = 180 mm = 0.15 m Isentropic stagnation temperature at exit is given by
D1 = 330 mm = 0.33 m D2 = 540 mm = 0.54 m T02s = hisen(T02 – T01) + T01
p1 = 0.23 bar = 23 kPa T1 = 217 K = 0.8 ¥ (415.62 – 248.1) + 248.1
V1 = 900 km/h hisen = 0.8 = 382.11 K
m = 210 kg/min = 3.5 kg/s Stagnation pressure after compression
g
Vw2 Ê T ˆ g -1
fs = = 0.9 fw = 1.04 p02 = p01 Á 02 s ˜
u2 ËT ¯ 01
To find Stagnation pressure at compressor outlet. 1.4
Ê 382.11ˆ 1.4 -1
= 0.3675 ¥ Á
Assumption The specific heat of air as 1005 J/kg ◊ K Ë 248.1 ˜¯
and g = 1.4
= 1.667 bar
Analysis The velocity of air with reference to aircraft
900 ¥ 1000
V1 = = 250 m/s
3600
Stagnation temperature at the inlet of compressor Axial compressors are aerofoil (blade) based
V12 2
250 rotary compressors. The gas flows parallel to the
T01 = T1 + = 217 + = 248.1 K
2C p 2 ¥ 1005 axis of rotation in axial flow compressors and gas
Rotary Compressor 911

is continuously compressed. The several rows of process and then guide and redirect the fluid onto
aerofoil blades are used to achieve large pressure the next stage of moving blades without shock.
rise in the compressor. The blades are made in aerofoil section to
The axial compressors are generally multi-stage reduce the losses caused by shocks, turbulence and
machines; each stage can give a pressure ratio of boundary separation. The annular area for air flow
1.2 to 1.3. The axial flow compressors are suitable is gradually reduced from the inlet to the outlet of
for higher pressure ratios and are generally more the compressor. The rotor of an axial compressor is
efficient than radial compressors. made aerodynamic.
Axial compressors are widely used in gas turbine
plants and small power stations. They are also used
in industrial applications such as blast-furnace air, The work input to a rotor shaft is transferred by
large-volume air-separation plants, and propane moving blades to air, thus accelerating the air flow.
dehydrogenation. Axial compressors are also used The spaces between moving blades and casing
for supercharging. They are also used to boost form the diffuser passages, and thus the velocity
the power of automotive reciprocating engines by of air decreases as air passes through them and
compressing the intake air. results into increase in pressure and enthalpy. The
air is then further diffused in stator blades which
are also arranged to form diffuser passages. The
An axial air compressor consists of a large number fixed blades also guide the air to flow at an angle
of rotating blade rows, fixed on a rotating drum, for smooth entry of next row of moving blades.
and stator (fixed) blade rows fixed on the casing of The temperature rise of air is almost same in
the compressor as shown in Fig. 26.19. A pair of moving as well as in fixed blades and axial velocity
rotating and stationary blades is called a stage. of air remains constant throughout the compressor.
The moving blades act as a series of fans and
the fixed blades act as guide vanes and diffuser. The
moving blades are imparting energy into the fluid,
and the fixed blades convert a part of kinetic energy The velocity triangles at inlet and outlet of
of the fluid into pressure energy through diffusion- moving blades of an axial-flow air compressor is

Inlet guide vanes Stator (Casing) Delivery vanes

S R Air Delivered
S R
R
S
R
S

Rotating drum
Air inlet
Drive shaft

Air Delivered

Moving Fixed
blades (R) blades (S)

Fig. 26.19
912 Thermal Engineering

shown in Fig. 26.20. The following points should


be considered for the construction of a velocity
triangle for an axial-flow compressor. Since the blade speed u and flow velocity Vf remain
1. The blade velocity remains same at inlet and constant at the inlet and outlet, thus the combined
outlet, i.e., u1 = u2 = u. velocity triangle can be constructed as below.
2. Flow velocity also remains constant, i.e., Vf1 1. Draw the line AB to represent the blade
= Vf2. velocity u.
3. Relative velocity at outlet is less than that at 2. Through the point A, draw a line at an
inlet, i.e., Vr2 < Vr1. inclination of 90° – a.
4. Both the whirl velocity components lie in 3. Through the point B, draw a line at an
the same plane. inclination of 90° – b.
The work input per stage per kg of air 4. The above two lines intersect at the point C,
w = u(Vw2 – Vw1) line AC represents the air inlet velocity V1
= Cp (T02 – T01) ...(26.30) and line BC represents relative velocity of
Power required to drive the compressor air Vr1 at inlet.
P = m w = m u (Vw2 – Vw1) ...(26.31) 5. The vertical line CD through the point C
From inlet and outlet velocity triangles, we get represents the velocity Vf1 at inlet.
Vw1 = Vf1 tan a 6. Through the point B draw a line at an
inclination of 90° – f.
and Vw2 = Vf2 tan q ...(26.32)
7. Along the inclined line, locate the point E
given that Vf1 = Vf2 = Vf (say)
with line EF, representing velocity Vf2 (= Vf1)
Therefore, work input to axial flow compressor
at the outlet.
can also be given by
8. Join the line AE to represent absolute
w = u Vf (tan q – tan a) ...(26.33)
velocity V2 of air at the outlet.
From Eqs. (26.30) and (26.33), we get
u(Vw2 – Vw1) = u Vf (tan q – tan a) ...(26.34)

Fig. 26.21

As discussed earlier, the degree of reaction is


defined as the ratio of static pressure rise in rotor
to the total static pressure rise in the compressor.
Mathematically,
Degree of reaction
Fig. 26.20
Pressure rise in rotor blade
Rd =
Pressure rise in compressor
Rotary Compressor 913

Vr12 - Vr22 Ê g -1 ˆ
Á Ê p02 ˆ g
2 = C p hisen ¥ T01 Á - 1˜ ...(26.39)
or Rd = Á Ë p01 ˜¯ ˜
u( Vw 2 - Vw1 ) ÁË ˜¯
( Vr12 - Vr22 ) Equation (26.39) demonstrate that for a given
= ...(26.35)
2u( Vw 2 - Vw1 ) pressure ratio and isentropic efficiency, the work
input to the compressor is directly proportional
From the inlet velocity triangle CDB;
to the initial temperature of air in the stage. Thus,
V r21 = V f21 + (Vf1 tan b)2 a compressor consisting of more than one stage
= Vf21 + Vf21 tan2 b ...(i) of equal isentropic efficiency, will require more
Similarly, from the outlet velocity trinagle BEF; work input, because it receives fluid at increased
temperature from preceeding stage. The two axial
V r22 = V f22 + V f22 tan2 f ...(ii)
compressors of different pressure ratios will have
Since V f1 = Vf2 = Vf (say) different work input and overall efficiency.
Then V r21 – V r22 = V f2 (tan2 b – tan2 f) ...(26.36)
Substituting Eqs. (26.34) and (26.36) in Eq. (26.35),
we get The polytropic efficeincy is the isentropic efficiency
Vf2 (tan 2 b - tan 2 f ) of one stage of a multistage, axial flow air
Rd = ...(26.37)
2u Vf (tan q - tan a ) compressor. The stage efficiency remains constant
for all stages of the compressor.
From symmetry of velocity triangles, we get
Figure 26.22 shows a stage of multistage axial
b = q and a = f
flow compressor. DT0s is the isentropic stagnation
Usually, the degree of reaction in axial flow temperture drop and DT0 is the actual temperature
1 drop during the stage. Thus, the polytropic effciency
compressor is taken 50%, i.e., Rd = ;
2 is defined as
1 Vf (tan b + tan f )
\ =
2 2u
u
or = tan b + tan f ...(26.38)
Vf
With 50% reaction blading, the axial compressor
has symmetrical blades and losses in the compressor
are drastically reduced.

The actual work input per kg of air is given by


w = Cp (T02 – T01)
which can be written as Fig. 26.22
Ê T - T01 ˆ Isentropic tempature drop dT0 s
w = Cp Á 02 ¥ (T02s – T01) hpoly = =
Ë T02 s - T01 ˜¯ Actual temperature drop dT0
= Cphisen ¥ (T02s − T01) ...(26.40)
ÊT ˆ For an irreversible polytropic process, the
= Cph isen ¥ T01 Á 02 s - 1˜ stagnation pressure and specific volume are related
Ë T01 ¯
as
914 Thermal Engineering

p0 vn0 = Z (compressibility factor; a constant) Ê n ˆ T dp0


or p0 = Z r 0n Using Á ˜ = 0 from Eq. (26.43), and
Ë n - 1¯ p0 dT0
Differentiating both sides, we get
subsituting in Eq. (26.45),
p
dp0 = n Z r n0 –1dr0 = n 0 dr0 ...(26.41) Ê g - 1ˆ dp0 T
r0 hpoly = Á ¥ 0
From characteristic gas equation, r0 can be ˜
Ë g ¯ p0 dT0
expressed as Ê dT ˆ Ê g - 1ˆ dp0
p or hpoly Á 0 ˜ = Á
r0 = 0 Ë T0 ¯ Ë g ˜¯ p0
RT0
On differentiation; we get Integrating between two states, we get
1 È T dp - p dT ˘ ÊT ˆ Ê g - 1ˆ Ê p02 ˆ
hpoly ln Á 02 ˜ = Á ln
dr0 = Í 0 0 2 0 0 ˙
R ÍÎ
...(26.42) Ë T01 ¯ Ë g ˜¯ ÁË p01 ˜¯
T0 ˙˚
Using dr0 in Eq. (26.41); Êp ˆ
ln Á 02 ˜
Ê g - 1ˆ Ë p01 ¯
p0 1 È T dp - p0 dT0 ˘ or hpoly = Á ...(26.46)
dp0 = n ¥ Í 0 0 ˙ Ë g ˜¯ Ê T02 ˆ
r0 R ÍÎ T02 ˙˚ ln Á
Ë T01 ˜¯
1 È T dp - p dT ˘
= nRT0 ¥ Í 0 0 2 0 0 ˙ Equation (26.46) expresses the polytropic
R ÍÎ T0 ˙˚ efficiency in terms of stagnaton pressure ratio and
È T dp - p0 dT0 ˘ stagnation temperature ratio.
= nÍ 0 0 ˙
Î T0 ˚ 26.6.8
dT0
or dp0 = n dp0 – np0
T0
dT0 1. It is defined as the ratio of axial
np0 = ndp0 – dp0 = dp0(n – 1) velocity (Vf) to blade velocity (u) . It is designated
T0
as ff.
Actual stage temperature; Vf
Ê n - 1ˆ T0 ff =
dT0 = dp0 Á ...(26.43) u
Ë n ˜¯ p0 From Eq. (26.38), u = Vf (tan a + tan b )
Similarly, for isentropic compression path, Vf
\ ff =
stagnation pressure and temperature are related as Vf ( tan a + tan b )
Ê g - 1ˆ T0
dT0s = dp0 Á ...(26.44) 1
Ë g ˜¯ p0 = ...(26.47)
tan a + tan b
Subsituting Eqs. (26.43) and (26.44) in From symmetry of inlet and outlet velocity
Eq. (26.40) to express the polytropic efficiency; triangles for 50% reaction,
Ê g - 1ˆ T0 b = q and a = f
dp0 Á
Ë g ˜¯ p0 The flow coefficient can also be expressed as
hpoly =
Ê n - 1ˆ T0 ff =
1
...(26.48)
dp0 Á
Ë n ˜¯ p0 tan f + tan q
Ê g - 1ˆ Ê n ˆ 2. It is defined as the ratio of
= Á ...(26.45)
Ë g ˜¯ ÁË n - 1˜¯ actual work input to kinetic energy corresponding
Equation (26.45) expresses the polytropic to mean peripheral velocity. It is also called head
efficiency in terms of polytropic and isentropic coefficient and designated as fh.
exponents.
Rotary Compressor 915

C p DTact u ( Vw2 - Vw1 ) 2 ( Vw2 - Vw1 ) which is less than the predetermined value. Thus,
fh = = = the flow becomes unsteady, periodic reversal.
u2 / 2 u2 / 2 u
The surging state (from M to U on characteristic
Ê tan b - tan f ˆ curve of Fig. 26.23) is dangerous for operation of a
= 2Á ˜¯ ...(26.49)
Ë tan q + tan f compressor of axial flow compressor also.
3. It is defined as ratio When the velocity of fluid in the
of isentropic work input to kinetic energy compressor reaches sonic velocity, the mass-flow
corresponding to mean peripheral velocity. It is rate through the compressor reaches a maximum
denoted by fp. value. This situation is called choking. At choking
C p DTisen condition, the pressure ratio in the compressor
fp = = hisen fh ...(26.50)
u2 / 2 becomes unity, i.e., there is no compression.
Choking means constant mass flow irrespective of
pressure ratio.
The losses in an axial-flow compressor can be Figure 26.23 shows the characteristic curve for
divided in three groups: an axial flow compressor. The choking occurs at the
The blade geometry of an axial point C. In region M to C on the curve, the flow is
compressor is two dimensional. The air flow along stable. Decrease in mass flow rate will result into
the profile of blade experiences skin friction. increased pressure rise.
Further, the different streams of air are mixed after
passing on blades. These losses lead to pressure
loss of compressed air.
When air flows through the
annulus passage of the compressor, it experiences
growth of boundary layer and skin friction.
Therefore, there is loss of pressure of compressed
air.
In an axial-flow air compres-
sor, the certain secondary flows are generated by Fig. 26.23
combined effect of curvature of blade and growth
of boundary layer in the annulus. The air is deflect- A compressor stalling is a situation of
ed by curvature of blades and bends in pipe, etc. It abnormal air flow resulting from a reduction in
causes loss in pressure of compressed air. the lift coefficient generated by an airfoil within
the compressor. The stalling is the separation of
26.6.10 flow from blade surface at low flow rates. At the
large value of incidence, the flow separation occurs
Surging is defined as a self-oscillation of at suction side of the blade, which is referred
the discharge pressure and flow rate, including a as positive stalling. Negative stalling is due to
flow reversal. Every centrifugal or axial compressor separation of flow occuring on the delivery side of
has a characteristic combination of maximum head the blade due to large value of negative incidence.
and minimum flow. Beyond this point, surging Compressor stalls hamper the compressor
will occur. During surging, a flow reversal is often performance, which can differ in severity from a
accompanied by a pressure drop. Surging is caused momentary compression loss to a complete loss of
when mass-flow rate of fluid is reduced to a value compression.
916 Thermal Engineering

Assumption The specific heat of air as 1005 J/kg ◊ K


and g = 1.4
FLOW COMPRESSORS

Sr. Centrifugal Axial-flow


No. Compressor Compressor
1. Air flows radially in Air flows parallel to
the compressor. axis of shaft.
2. Low maintenance and High maintenance and
running cost. running cost. Fig. 26.24
3. Low starting torque Requires high starting
Analysis The temperature after isentropic compression
required. torque.
g -1
4. Not suitable for Suitable for only 1.4 -1
Êp ˆ g
multistaging. multistaging. T2s = T1 Á 2 ˜ = 300 ¥ (5) 1.4
Ë p1 ¯
5. Suitable for low Suitable up to a
pressure ratios up to 4. pressure ratio of 10. = 475.14 K
6. For given mass-flow It requires less frontal The isentropic efficiency is given by
rate, it requires, larger area. T -T
hisen = 2 s 1
frontal area. T2 - T1
7. Isentropic efficiency Isentropic efficiency Final temperature of air is given by
80 to 82%. 86 to 88%. T2 s - T1 475.14 - 300
T2 = + T1 = + 300
8. Better performance at Poor performance at hisen 0.85
part load. part load. = 506 K
(i) Work input to compressor per kg of air
Example 26.12 An axial-flow compressor having w = Cp (T2 – T1) = 1.005 ¥ (506 – 300)
10 stages works with 50% degree of reaction. It com- = 207 kJ/kg
presses air with a pressure ratio of 5. The inlet condi-
Work input per kg is also given by Eq. (26.33),
tions of air are 27°C and 100 kPa. The air enters the
w = uVf (tan q – tan a ) ¥ No. of stages
compressor with a velocity of 110 m/s. The mean speed
of the rotor blade is 220 m/s. The isentropic efficiency of Using symmetry of velocity triangles, q = b and
the compressor is 85%. Calculate the work input per kg numerical values;
of air and blade angles. 207 ¥ 103 = 220 ¥ 110 ¥ (tan b – tan a) ¥ 10
tan b – tan a = 0.855 (i)
Solution
Further form Eq. (26.38) with q = b;
Given An axial-flow air compressor with u
= tan b + tan a
Rd = 0.5 Nstages = 10 Vf
p1 = 100 kPa T1 = 27°C = 300 K 220
p2 = 500 kPa V1 = 110 m/s or = tan b + tan a
110
hisen = 0.85
or tan b + tan a = 2 ...(ii)
u = 220 m/s
Solving Eqs. (i) and (ii), we get
To find
2 tan b = 2.855
(i) Power input, and
or b = 55°
(ii) Blade angles.
and a = 29.79°
Rotary Compressor 917

Example 26.13 An axial-flow compressor draws air pressure and temperature are 1 bar and 300 K, respec-
at 20°C and delivers it at 50°C. Assuming 50% reaction, tively. The mean blade velocity is 180 m/s. The degree of
calculate the velocity of flow, if blade velocity is 100 m/s, reaction is 50% at mean radius with relative air angles
work factor is 0.85.Take Cp = 1 kJ/kg ◊ K, Assume a = of 12° and 32° at rotor inlet and outlet, respectively. The
10°, and b = 40°. Find the number of stages. work done factor is 0.9. Calculate
(a) Stagnation polytropic efficiency,
Solution (b) Inlet temperature and pressure,
Given An axial-flow air compressor with (c) Number of stages,
Rd = 0.5 T1 = 20°C = 293 K (d) Blade height in first stage, if ratio of hub-to-tip
T2 = 50°C = 323 K a = 10° diameter is 0.42, mass-flow rate is 19.5 kg/s.
b = 40° u = 100 m/s
Solution
fh = 0.85 Cp = 1 kJ/kg ◊ K
Given An axial-flow air compressor with
To find
Rd = 0.5 p01 = 100 kPa
(i) Flow velocity, and T01 = 300 K m = 19.5 kg/s
(ii) Number of stages. p02
=4 u = 180 m/s
p01
Analysis The blade velocity and flow velocity are
related as hisen = 0.85 fh = 0.9
u a = f = 12° b = q = 32°
= tan b + tan a
Vf rh = 0.42r1
100 To find
or = tan 40° + tan 10°
Vf (i) Stagnation Polytropic efficiency,
or 100 = 1.015Vf (ii) Inlet temperature and pressure,
or Vf = 98.48 m/s (iii) Number of stages, and
Further, (iv) Blade height in first stage.
Vw1 = Vf tan a = 98.48 ¥ tan 10°
Assumption The specific heat of air as 1005 J/kg ◊ K
= 17.36 m/s
and g = 1.4.
and Vw2 = Vf tan q = Vf tan b = 98.48 ¥ tan 40°
= 82.63 m/s Analysis
Work done per stage (i) Stagnation polytropic efficiency:
w1 = u (Vw2 – Vw1)fh Stagnnation temperature after isentropic com-
= 100 ¥ (82.63 – 17.36) ¥ 0.85 pression
= 5548.33 J/kg
Theoretical work required for a compressor
w = Cp (T2 – T1) = 1005 ¥ (323 – 293)
= 30150 J/kg

Number of stages
Theoretical work 30150
= =
Work input per stage 5548.33
= 5.43 ª 6 stages

In an axial flow compressor, overall


stagnation pressure ratio achieved is 4 and overall stag-
nation isentropic efficiency is 85%. The inlet stagnation
Fig. 26.25
918 Thermal Engineering

g -1 = 180 ¥ (134.31 – 45.69) ¥ 0.9


1.4 -1
Êp ˆ g = 14357.46 J/kg
T02 = T01 Á 02 ˜ = 300 ¥ ( 4) 1.4
Ë p01 ¯ The stganation temperature is given by
= 445.8 K V12
T01 = T1 +
The isentropic efficiency is given by 2C p
T -T The static temperature at inlet
hisen = 02 s 01
T02 - T01
V12 219.752
Final stagnation temperature of air T1 = T01 – = 300 –
2C p 2 ¥ 1005
T -T
T02 = 02 s 01 + T01 = 276 K
hisen
Inlet pressure can be expressed as
445.8 - 300
= + 300 g 1.4
0.85 Ê T ˆ g -1 Ê 276 ˆ 1.4 -1
p1 = p01 Á 1 ˜ = 1¥ Á
= 471.52 K Ë T01 ¯ Ë 300 ˜¯
The stagnation polytropic efficiency
= 0.746 bar
Êp ˆ (iii) Number of stages
ln 02
Ê g - 1ˆ ÁË p01 ˜¯ Total work input to compressor per kg of air in all
hpoly =
ÁË g ˜¯ ÊT ˆ stages;
ln Á 02 ˜
Ë T01 ¯ wT = Cp (T02 – T01)
= 1005 ¥ (471.52 – 300)
Ê 4ˆ
ln Á ˜ = 172385 J/kg
Ê 1.4 - 1ˆ Ë 1¯
= Á ˜ No. of stages
Ë 1.4 ¯ Ê 471.52 ˆ
ln Á
Ë 300 ˜¯ wT 172385
= = ª 12
= 0.8759 or 87.59% wstage 14356.44
(ii) Inlet temperature and pressure: (iv) Blade height in first stage
The blade velocity-to-flow velocity ratio is given Density of air entering the first stage
by Eq. (26.38); p 0.746 ¥ 100
u r = 1 = = 0.9425 kg/m3
= tan a + tan b = tan 12° + tan 32° RT1 0.287 ¥ 276
Vf
The mass-flow rate is given by
= 0.8374 m = r Ac Vf = r p (r 12 – r 2h)Vf
180 or 19.5 = 0.9425 ¥ p ¥ r 12 (1 – 0.422) ¥ 214.95
or Vf = = 214.95 m/s
0.8374 (∵ rh = 0.42r1)
Inlet air velocity; or r12 = 0.03720
Vf 214.95 Itgi ves r1 = 0.1928 m
V1 = = = 219.75 m/s
cos a cos12∞ and rh = 0.42 r1 = 0.081 m
Whirl velocity at inlet; Height of blade in first stage
Vw1 = Vf tan a = 214.95 ¥ tan 12° = r1 – rh = 0.1928 – 0.081
= 45.69 m/s = 0.1118 m or 11.18 cm
Whirl velocity at outlet;
Vw2 = Vf tan q = 214.95 ¥ tan 32° Example 26.15 An axial-flow compressor has a
constant axial velocity of 150 m/s and 50% reaction . The
= 134.31 m/s
mean daimeter of the blade ring is 35 cm and speed is
Work input per kg of air per stage
15,000 rpm.. The exit angle of the blade is 27°. Calculate
wstage = u (Vw2 – Vw1) fh blade angle at inlet and work done per kg of air.
Rotary Compressor 919

Solution Work input per kg of air


w = u (Vw2 – Vw1)
Given An axial flow air compressor with
= 274.89 ¥ ( 198.5 – 76.5)
Rd = 0.5 Vf1 = 150 m/s
= 33536.5 J/kg or 33.53 kJ/kg
D = 35 cm = 0.35 m a = f = 27°
N = 15,000 rpm An axial-flow compressor of 50%
To find reaction has a blade outlet angle of 30°. The flow velocity
is 0.5 times the mean blade velocity. The speed of the
(i) Blade angle b at inlet, and
rotor is 7500 rpm. The stagnation condition of air at the
(ii) Work input per kg of air.
entry is 1.013 bar and 5°C and the static pressure at this
Analysis The mean velocity of the blade ring section is 0.91 bar. Draw the velocity triangle and find the
power required to run the compressor, mass-flow rate and
p DN p ¥ 0.35 ¥ 15000 mean diameter of rotor. The mean flow area is 0.35 m2.
u = =
60 60
= 274.89 m/s Solution
Given An axial-flow air compressor with
Rd = 0.5 Vf1 = 0.5 u
p01 = 1.013 bar T01 = 5°C = 278 K
p1 = 0.91 bar a = f = 30°
N = 7500 rpm A = 0.35 m2
To find
(i) Mean diameter of rotor,
(ii) Power input per kg of air, and
(iii) Mass flow rate of air.
Fig. 26.26
Assumption The specific heat of air as 1005 J/kg ◊ K
Construction of velocity triangles and g = 1.4
(i) Draw a horizontal line AB to represent the blade
velocity u = 274.89 m/s.
(ii) Through the point A, draw an inclined line at
angle 90° – a = 63°.
(iii) Mark the intersection point C and draw a vertical
line CD representing flow velocity of 150 m/s at
inlet.
(iv) Through the point B, draw an inclined line BE at
angle 90° – 27° = 63°.
(v) Mark the intersection point E and draw a vertical Fig. 26.27
line EF representing flow velocity of 150 m/s at
outlet. Analysis The static temperature
(vi) Join the point E with A and the point C with B. g -1 1.4 -1
(vii) Line CB represents relative velocity Vr1 at the Ê p ˆ g Ê 0.91 ˆ 1.4
T1 = T01 Á 1 ˜ = 278 ¥ Á
inlet and the line EA represents relative velocity Ë p01 ¯ Ë 1.013 ˜¯
Vr2 at the outlet.
= 269.6 K
(viii) Measure the –AEF or –DCB = 53°.
The stagnation temperature is given by
(ix) Measure AD = Vw1 = 76.5 m/s, and AF = Vw2=
198.5 m/s. V12
T01 = T1 +
2C p
920 Thermal Engineering

It gives (vii) Length of line BE represents relative velocity at


outlet Vr2 = V1.
V1 = 2C p (T01 - T1 ) = 2 ¥ 1005 ¥ ( 278 - 269.6)
(viii) From the point B, meet the point C by a line BC
= 130 m/s to represent Vr1 = V2 = 195.5 m/s.
Flow velocity (ix) Length of line AF represents Vw2 = 160 m/s.
Vf1 = Vf2 = V1 cos a
Mean diameter of rotor:
= 130 ¥ cos 30° = 112.5 m/s
Vf The mean velocity of blade ring,
112.5
Blade velocity, u = = = 225 m/s p DN
0.5 0.5 u =
For 50% reaction in axial flow compressor 60
60 u 60 ¥ 225
a = f, and b = q or D = = = 0.573 m
Construct the velocity triangle as follows: p N p ¥ 7500
(i) Draw an inclined line AC to represent air velocity, Power input per kg of air
V1 = 130 m/s at an angle (90°–30°) = 60°. w = u(Vw2 – Vw1)
(ii) Draw a vertical line CD, which represents axial = 225 ¥ (160 – 65)
velocity of air. Its length is equivalent to Vf1 = 21375 J/kg or 21.37 kJ/kg
112.5 m/s.
Mass-flow rate of air
(iii) The line segment AD, represents whirl velocity at
p1 0.91
the inlet. Its length is equivalent to Vw1 = 65 m/s. Density of air r = =
(iv) Extend the line AD to the point B to represent u = RT1 0.287 ¥ 278
225 m/s. = 1.176 kg/m3
(v) From the point B, draw a line BE inlined at angle Mass-flow rate, m = rAVf1 = 1.176 ¥ 0.35 ¥ 112.5
(90° – 30°) = 60°. = 46.3 kg/s
(vi) Locate the point E on the line BE by drawing a
vertical line EF = Vf2 = Vf1 = 112.5 m/s.

Summary
- Lysholm compressor is a patented screw
tons and give a continuous, pulsation-free com- compressor, a single-stage helical lobe, oil
pressed air. The rotary compressors are mainly flooded air compressor.
classified as rotary positive-displacement type air centrifugal compressors are dynamic action
compressor and steady-flow type compressor. compressors. The centrifugal air compressor is
an oil free compressor by design. Centrifugal
air is compressed by being trapped in the reduced machines are better suited due to their simplicity,
space formed by two sets of engaging surfaces. light weight and ruggedness.
In a non-positive dispacement or steady-flow type Axial compressors are dynamic action, rotating,
compressor, the air flows continuously through aerofoil blade compressors. Axial flow compres-
them and pressure is increased due to dynamic sors produce a continuous flow of compressed
action. gas, and have the benefits of high efficiencies and
roots blower is essentially a low-pressure large mass flow capacity, particularly in relation
blower and is limited to a discharge pressure of to their cross-section.
1 bar in single-stage design and up to 2.2 bar in a
two-stage design.
Rotary Compressor 921

Glossary
Degree of Reaction Ratio of static pressure rise in Pressure coefficient Ratio of isentropic work to Euler work
impeller to the total static pressure rise in the compressor Choking State of maximum mass-flow rate
Slip factor Ratio of whirl velocity to blade tip velocity Surging State of alternatively forward and backward
Euler’s work Product of blade velocity and whirl flow of fluid
velocity (w = u2 Vw2) Stalling Separation of flow from blade surface
Work factor Ratio of actual work input to Euler work Polytropic Efficiency The isentropic efficiency of one
input stage of a multistage axial flow air compressor

Review Questions
1. Define rotary compressor. Classify them. 8. Discuss the effect of impeller blade shape on
2. Differentiate between positive displacement and performance of centrifugal compressor.
negative displacement compressors. 9. Define slip, slip factor and pressure coefficient.
3. Compare reciprocating compressor with a rotary 10. Explain the construction and working of a diffuser
compressor. in a centrifugal compressor.
4. Explain construction and working of a roots 11. Explain the phenomenon of surging and its effects
blower. in the compressor.
5. Explain construction and working of a vane-type 12. Prove that the work input per kg of air in an axial
compressor. flow compressor is w = u Vw (tan b – tan a)
6. Explain working and construction of a screw 13. Compare axial flow compressor with centrifugal
compressor. one.
7. Describe the principle of operation, construction 14. What is stalling in an axial flow compressor?
and working of centrifugal compressor.

Problems
1. Compare the work inputs required for a roots The air enters the impeller radially. Determine the
blower and a vane-type compressor having work done by a compressor per kg of air. Also
same volume inducted of 0.3 m3/rev. The inlet calculate the degree of reaction.
pressure is 1.013 bar and pressure ratio is 1.5 in [1.95 kJ/kg, 64.5%]
a compressor. For vane-type compressor, assume 4. A rotary compressor handles 3 kg of air per
half the compression takes place through half the second, runing at 2400 rpm. The internal and
pressure range. [1.52 kJ, 1.352 kJ] external diameters of the impeller are 120 mm
2. A roots blower compresses 0.08 m3 of air from and 240 mm, respectively. The impeller angle at
100 kPa to 150 kPa per revolution. Calculate the exit is 35°. The air enters the impeller radially
compressor efficiency. [85.95%] with 7 m/s. Calculate the vane angle at inlet.
3. A centrifugal compressor running at 2000 rpm Also calculate the power required to drive the
has internal and external diameters of impeller as compressor. [25°, 1.83 kW]
300 mm and 500 mm, respectively. The vane angle 5. A centrifugal compressor runs at 8000 rpm,
at inlet and outlet are 22° and 40°, respectively. handles 4.8 m3/s from 1 bar and 20°C to
922 Thermal Engineering

The index of compression is 1.5. The flow angle is 12°. The blade angle at the inlet and exit
velocity is 65 m/s, same at the inlet and outlet are 35° and 27°, respectively. [2318 kW, 66.55]
of compressor. The inlet and outlet impeller 9. An axial flow compressor, with a compression
diameters are 320 mm and 620 mm, respectively. ratio of 4, draws air at 20°C and delivers it at
Calculate (a) blade angle at inlet and outlet, 97°C. The blade velocity and flow velocity are
(b) absolute angle at tip of impeller, and (c) width constant throughout the compressor. The blade
of blade at inlet and outlet. velocity is 300 m/s. Air enters the blade at an
[(a) b = 25.88°, f = 34.2°, (b) q = 21.6°, angle of 12°. Calculate the flow velocity, work
(c) 7.34 cb and 2.89 cm] done per kg of air and degree of reaction. Take
6. A centrifugal compressor runs at 1440 rpm, the inlet stagnation temperature of 305 K.
compresses air from 101 kPa , 20° to a pressure [152 m/s, 77.38 kg/s, 46.2%]
of 6 bar isentropically. The inner and outer 10. An axial flow compressor with 50% degree of
diameters of the impeller are 140 mm and reaction has blades with inlet and outlet angles
280 mm, respectively. The width of blades at of 45° and 10°, respectively. The pressure ratio is
inlet is 25 mm. The blade angles are 16° and 40° 6 and isentropic efficiency is 85%, when the air
at entry and exit. Determine the mass-flow rate inlet temperature is 40°C. The blade velocity is
of air, degree of reaction, power developed and 200 m/s. The blade velocity and axial velocity are
width of blade at outlet. constant throughout the compressor. Calculate
[0.4 kg/s, 14.78 W, 58.6%, 3.5 mm] the number of stages required, when work factor
7. A centrifugal compressor handles 16.5 kg/s of is (a) unity, and (b) 0.89 for all stages.
air with total pressure ratio of 4. The speed of [(a) 9 (b)10]
the compressor is 15000 rpm. Inlet stagnation 11. Air from a quiescent atmosphere, at pressure of
temperature is 20°, slip factor is 0.9, power input 1 bar and 300 K enters a centrifugal compressor,
factor is 1.04 and isentropic efficiency as 80%. fitted with radial vanes and the air leaves the
Calculate diameter of impeller and power input diffuser with negligible velocity. The tip diameter
to the compressor. [55.67 cm, 2951.7 kW] of the impeller is 450 mm and the compressor
8. An axial-flow compressor stage has a mean rotates at 18000 rpm. Neglect all losses and
diameter of 600 mm and runs at 15000 rpm. calculate the temperature and pressure of air
The mass-flow rate through the compressor is as it leaves the compressor. Take g = 1.4, Cp =
50 kg/s. Calculate the power required to drive the 1.005 kJ/kg ◊ K, and fs = 0.9. [188°C, 4.5 bar]
compressor and degree of reaction, if the inlet

Objective Questions
1. Which one of the following is a non-positive type 3. In a roots blower, the compression process can be
rotary compressor? represented by
(a) Vane blower (a) isothermal ine
l
(b) roots blower (b) isentropic line
(c) Centrifugal compressor (c) constant-volume inel
(d) Lysholm compressor (d) constant-pressure ine l
2. A machine is called a compressor when it has a 4. In a roots blower, the pressure is increased due to
pressure ratio (a) rotation of lobes
(a) up to 1.11 (b) up to 1.2 (b) increase in mass
(c) more than 1.2 (d) none of the above
Rotary Compressor 923

(c) back flow of air 9. The degree of reaction in a centrifugal compressor


(d) reduction in volume of air is defined as
5. In a centrifugal compressor, the increase in Pressure rise in diffuser
(a)
pressure is due to Pressure rise in impeller
(a) back flow of air Pressure rise in impeller
(b) dynamic action (b)
Pressure rise in compressor
(c) intermittent flow
Pressure rise in diffuser
(d) reduction in volume (c)
Pressure rise in compressor
6. The efficiency of a roots blower is given as
(d) none of the above
Isentropic work input
(a) 10. Pressure coefficient is defined as
Actual work input
Actual work input
Actual work input (a)
(b) Euler work input
Isentropic work input
Blade velocity
Actual work input (b)
(c) Whirl velocity
Isothermal work input
Isentropic work input
Isothermal work input (c)
(d) Euler work input
Actual work input
(d) none of the above
7. The stages are arranged in a multisatge
11. Forward curved impeller vane has a blade exit
compressor in
angle of
(a) parallel (b) series
(a) < 90° (b) > 90°
(c) cross connected (d) none of the above
(c) = 90° (d) none of the above
8. In a centrifugal compressor, the pressure rise
12. At the state of choking in compressor,
takes place in
(a) mass-flow rate reaches a minimum value
(a) impeller only
(b) mass-flow rate reaches a maximum value
(b) diffuser only
(c) pressure ratio reaches a maximum value
(c) casing only
(d) pressure ratio reaches a minimum value
(d) impeller and diffuser both

12. (b) 11. (b) 10. (c) 9. (b)


8. (d) 7. (b) 6. (a) 5. (b) 4. (c) 3. (c) 2. (c) 1. (c)
Answers
924 Thermal Engineering

27
Gas Turbine Plant

Introduction
Gas turbine is a rotary internal combustion engine. It consists of a compressor, a combustion chamber
and a turbine. The gas turbine uses continuous gas flow as the working medium by which heat energy is
partially converted into mechanical energy. Gas is produced in the engine by the combustion of certain
fuels. Stationary nozzles discharge jets of this gas against the blades of a turbine wheel. The impulse force
of the jets causes the shaft to spin.
A simple-cycle gas turbine is shown in Fig. 27.1. Atmospheric air is drawn into an axial compressor,
where it is compressed to a higher pressure and temperature. This hot compressed air is passed through a
combustion chamber, where the fuel in gaseous or liquid-spray form is injected and combustion takes place
with the help of burner. The resulting combustion products expand through a turbine to produce power.
The exhaust gases leaving the turbine are finally discharged into the atmosphere. Most of the turbine power
is used to drive the compressor and auxilliary devices and the remainder is the net plant output, which is
available to generate electricity, to propel an aircraft or for other purpose.
Air

Safety valve

Starting Generator
motor

Turbine Axial flow


compressor

Exhaust

Combustion
chamber

Burner Fuel

Fig. 27.1 A simple gas turbine plant


Gas Turbine Plant 925

5. The speed of the gas turbines is higher than


the IC engines.
The gas turbine plants have their applications in the 6. The gas turbines use cheaper fuel such as
following fields: paraffin type, oil residues, or powdered
coal, while special-grade oils are used in IC
1. They are widely used in the aircraft field, engines to avoid knocking in the engine.
i.e., for propulsion of turbo jet and turbo 7. The combustion is complete in a gas turbine,
propeller engines. thus exhaust is less polluting as compared to
2. With availability of natural gas fuel, the IC engines.
gas turbines are increasingly being used for 8. In gas turbines, the ignition and lubrication
electric power generation. systems are much simpler.
3. These are also used in marine propulsion. 9. Since the gas turbine parts are not subjected
4. The gas turbines are used in conjunction to large pressure, so they can be made lighter.
with thermal power plants as cogeneration 10. Due to low specific weight, the gas turbines
for power production. are particularly suitable for use in aircrafts.
5. The gas turbines are used in high-speed
racing cars. Demerits
6. The gas turbines are also used to run the 1. Thermal efficiency of the gas turbine power
railway locomotives. plant is lower (15 to 20%) as compared with
7. These are used to operate hovercrafts. IC engines (25 to 30%).
8. These are used to run turbo (high speed) 2. It is difficult to start a gas turbine as
pumps. compared to an IC engine.
9. These are used to run the rotary compressors. 3. The gas turbine blades require a special
10. The gas turbines are also used in cross cooling system.
country pipe and charging stations. 4. The manufacturing of gas turbine blades is
difficult and costly.
5. For the same power output, the gas turbine
produces exhaust gas approximately five
times more than that of an IC engine.
The gas turbines have the following merits and
demerits over reciprocating IC engines.
Merits
1. Since the gas turbine is a rotary engine, its The brief comparison between a gas turbine plant
mechanical efficiency approaches 95%, and a steam turbine power plant is given below:
quite high compared with IC engines.
2. The torque on the shaft of a gas turbine is 1. Steam turbines use steam as working fluid.
continuous and uniform, thus a gas turbine Steam is the cheapest medium and can be
does not require a flywheel, while without generated with the help of easily available
flywheel, IC engine cannot run smoothly. fuels. The gas turbines are using costly fuel
which can burn efficiently in its combustion
3. The weight of the gas turbine per kW power
chamber.
is less than the IC engine.
2. Steam is prepared outside the turbine, thus it
4. Power developed by a gas turbine per kg of
is an external combustion engine, while the
air is more than of IC engine.
combustion chamber is an integral part of
926 Thermal Engineering

gas turbine, thus it is an internal combustion The compressed air is mixed with fuel and
engine. combustion occurs in the the combustion chamber.
3. The steam turbine is in highly developed stage The combustion gases leaving the turbine after
after long experience and modifications, expansion are expelled to the atmosphere. Thus, the
thus it has higher thermal efficiency than gas cycle becomes an open cycle as shown in Fig. 27.2.
turbines.
Fuel Combustion
4. A steam power plant includes boiler plant,
chamber
turbine, condensing plant including cooling
tower and continuous supply of water.
Compressor Turbine Wnet
All the equipments of steam, power plant
require large space and are bulky. A gas
turbine plant includes less number of parts,
Air Combustion
i.e., compressor, combustion chamber and products
turbine. The combustion chamber is also Fig. 27.2 Open-cycle hgas turbine
compact. Thus, gas turbine is lighter engine
and frequently used on aircrafts. (b) Closed-cycle Gas Turbine In a closed-cycle gas
5. A gas turbine uses lower pressure ratio of the turbine, the working substance air undergoes the
working fluid as compared to steam pressure cycle, repeatedly. The combustion process is re-
in a steam turbine. placed by heat addition from external source and
6. Except blade cooling, no cooling is required exhaust process is replaced by heat rejection pro-
in gas turbine plants, where a large amount cess to the surroundings.
of cooling water is required in the condenser
of steam power plants. Heat
Heat exchanger, 1
7. Gas turbines run at higher speed than steam Combustion
turbines. chamber
3
2
8. A gas turbine plant can easily be started or
stopped as compared to a steam power plant. Compressor Turbine Wnet

9. Due to absence of components, such as


4
condenser, feed-water treatment plant, 1 Heat
exchanger, 2
cooling turbine, etc., the auxiliary equipment
in a gas turbine are small and lighter as Heat
compared to steam turbine. Fig. 27.3 Closed-cycle gas turbine
10. The number of skilled man power required to
operate the gas turbine plant is comparatively
(ii) According to Combustion Process
much less than for steam power plant.
(a) Constant-pressure Combustion Gas Turbine
Theoretically, it operates on air standard Brayton
cycle (Fig. 27.5) in which the combustion takes
place at constant pressure by slow burning fuel.
The gas turbines are rotary engines and can be (b) Constant-volume (Explosive) Gas Turbine It is
classified as follows: also called explosive gas turbine and theoretically,
(i) According to Cycle of Operation it operates on the Atkinson power cycle.
(a) Open-cycle Gas TurbinesThe fresh charge The atmospheric air is compressed in a
is supplied to the compressor in each cycle. compressor from the pressure p1 to the pressure
Gas Turbine Plant 927

Fuel p2. The compressed air is supplied to a combustion


Fuel pump chamber when the inlet valve opens. Fuel is
Inlet valve
p3
injected through a fuel pump and burnt with the
Combustion
p2 chamber help of a spark plug at constant volume, thus
Nozzle pressure and temperature further increase to p3 and
T3, respectively.
Compressor Turbine Wnet The hot combustion products expand in a nozzle
partially to increase the velocity of fluid and then
p1, T1
expand in a turbine to develop mechanical power.
Air Combustion
products

Fig. 27.4

Differences between closed-cycle and open-cycle gas turbines are tabulated below.
Sr. No. Criterion Closed-cycle gas turbine Open-cycle gas turbine
1. Cycle of operation It works on closed cycle. The working It works on open cycle. The fresh charge is
fluid is recirculated again and again. It supplied to each cycle and after combustion
is a clean cycle. and expansion, it is discharged to the
atmosphere.
2. Working fluid The gases other than air like helium Air–fuel mixture is used which leads to
or helium–CO2 mixture can be used, lower thermal efficiency.
which has more favourable properties.
3. Manner of he
at nput
i The heat is transferred indirectly Direct heat supply. It is generated in the
through a heat exchanger. combustion chamber itself.
4. Quality of heat ni put The heat can be supplied from any It requires high-grade heat energy for
source like waste heat from some generation of power in a gas turbine.
process, nuclear heat and solar heat
using a concentrator.
5. Type of fuel used Since heat is transferred externally, Since combustion is an integral part of the
so any type of fuel; solid, liquid or system, thus it requires high-quality liquid
gaseous or a combination of these can or gaseous fuel for burning in a combustion
be used for generation of heat. chamber.
6. Efficiency High thermal efficiency for given lower Lower thermal efficiency for same
and upper temperature limits. temperature limits.
7. Part load efficiency Better. Comparatively less.
8. Size of he
t plant Reduced size per MWh of power Comparative large size for same power
output. output.
9. Blade life Since combustion products do not Direct contact with combustion products,
come in direct contact of turbine blade, the blades are subjected to higher thermal
thus there is no blade fouling and stresses and fouling and hence shorter
longer blade life. blade life.
10. Control Better control on power production. Poor control on power production.
11. Cost Plant is complex and costly. Comparatively simple plant and less costly.
928 Thermal Engineering

addition and heat rejection processes taking


place in a heat exchanger. However, this
pressure drop is less significant, and is thus
The gas turbines usually operate on open cycle neglected for simplicity of analysis as shown
as shown Fig. 27.2. The open-cycle gas turbine in Fig. 27.6(b).
cycle can be modelled as a closed-cycle gas 2. Because of the frictional effects within the
turbine, as shown in Fig. 27.3 by using air-standard compressor and turbine,
assumptions. (a) the actual work input into the
Air is compressed in a compressor, heated compressor is more than the isentropic
in a heat exchanger, expanded in a turbine and work, and
cooled again to initial state before re-entering (b) the actual work output of the gas tur-
the compressor. This modified cycle operates on bine is less than the isentropic work. It
air standard Brayton Cycle, shown in Fig. 27.5, results in a marked decrease in the net
explained in Article 11.14. work output of the gas turbine plant.
p
qin
3. The specific heat of the working substance
increases as temperature increases.
2 3

T
pv g = Const. 3

4
2 4s
4 2s
1
qout
v
0 1
s
T 0
(a) Actual gas turbine cycle
3
qin all processes are irreversible
T 3
st.
on qin
p =C 4
2 nst.
Co 4
p= 4s
1 qout 2
2s
s
0 qout
1
Fig. 27.5 Air standard Brayton cycle 0 s
(b) Gas turbine cycle with irreversible
compression and expansion
Fig. 27.6
turbine cycle

The actual gas turbine cycle in Fig. 27.6(a) differs In absence of any changes in kinetic and
from an ideal Brayton cycle in the following potential energies; the steady-flow energy equation
manner. on a unit mass basis, as
q – w = Dh = hexit – hinlet
1. Due to frictional effects, the working fluid
Assuming constant specific heat of air, the heat
experiences a pressure drop during heat
supplied to and rejected from air are
Gas Turbine Plant 929

qin = h3 – h2 = Cp (T3 – T2) T


3
qout = h4 – h1 = Cp (T4 – T1)
Work developed per kg by turbine a
kP
wT = h3 – h4 = Cp (T3 – T4) ...(27.1) 00
10
=
Work input to compressor p2 4
2
4s
wC = h2 – h1 = Cp (T2 – T1) ...(27.2) 2s
kPa
Net work of the gas turbine plant 100
300 K p1 =
wnet = Sq or Sw 0
1 3
s
= qin – qout or wT – wC
Fig. 27.7 An actual gas turbine cycle
Thermal efficiency of gas turbine cycle
C p (T4 - T1 ) Solution
w q
hThermal = net = 1 - out = 1 -
qin qin C p (T3 - T2 ) Given An air-standard gas turbine cycle with
...(27.3) p1 = 100 kPa T1 = 300 K
rp = 10 T3 = 1400
In order to obtain an appreciable net work from
hT = 0.8 hC = 0.8
the plant, the compressor and turbine must be 3
V = 5 m /s
highly efficient. The isentropic efficiencies of the
compressor and gas turbine are defined as To find
(i) Thermal efficiency of the cycle,
Actual work output
hT = (ii) The back work ratio, and
isentropic work output
(iii) The net power developed in kW.
h3 - h4 C p (T3 - T4 ) T3 - T4
= = = ...(27.4) Assumptions
h3 - h4 s C p (T3 - T4 s ) T3 - T4 s
(i) Each device operates in a steady flow manner.
Isentropic work input (ii) Compression and expansion are adiabatic.
hC = (iii) No pressure drop during flow of gas through heat
Actual work input
exchangers.
h2 s - h1 C p (T2 s - T1 ) T2 s - T1 (iv) Kinetic and potential energy effects are negligible.
= = = ...(27.5)
h2 - h1 C p (T2 - T1 ) T2 - T1 (v) The working fluid is air as an ideal gas with
where the states designated without suffix represent Cp = 1.005 kJ /kg ◊ K, g = 1.4.
actual states of working fluid and states designated Analysis The T–s diagram for given data of a gas
with suffix s denote the state after isentropic turbine plant is shown in Fig. 27.7. Analysing each
process. device in a steady flow manner.

Example 27.1 Air enters the compressor of a Compressor The temperature after isentropic compres-
gas turbine plant operating on air-standard cycle at sion
g 1 1.4 -1
100 kPa and 300 K with a volumetric flow rate of 5 m3/s. g
The compressor pressure ratio is 10. The turbine inlet T2s = T1( rp ) = 300 ¥ (10) 1.4 = 579.2 K
temperature is 1400 K. The turbine and compressor each The isentropic compressor work per kg of air,
has an isentropic efficiency of 80%. Calculate wC = h2s – h1 = Cp (T2s – T1)
(a) the thermal efficiency of the cycle, = 1.005 ¥ (579.2 – 300) = 280.6 kJ/kg
(b) the back work ratio, The compressor efficiency is given by
(c) the net power developed in kW. Insentropic work input wC
hC = =
Actual work input win
930 Thermal Engineering

Actual work input; The power output


wC 280.6 P = ma wnet = 5.80 ¥ 191.85
win = = = 350.75 kJ/kg = 1114.1 kW
hC 0.8
Actual work input per kg to the compressor can also Example 27.2 A gas turbine unit has a pressure
be expressed as ratio of 6 and maximum cycle temperature of 610°C. The
win = h2 – h1 = Cp (T2 – T1) isentropic efficiency of the turbine and compressor are
win 350.75 0.82 and 0.8, respectively. Calculate the power output in
or T2 = T1 + = 300 + = 649 K kW of an electric generator, geared to the turbine, when
Cp 1.005
air enters the compressors at 15°C at a rate of 16 kg/s.
Heat supplied The heat supplied to air in heat exchanger Take Cp = 1.005 kJ/kg ◊ K and g = 1.4 for compression
qin = h3 – h2 = Cp (T3 – T2) process and Cp = 1.11 kJ/kg ◊ K and g = 1.333 for
= 1.005 ¥ (1400 – 649) = 754.75 kJ/kg expansion process.

Gas Turbine The temperature T4s, after isentropic Solution


expansion
Given A gas turbine unit with
T3 1400 rp = 6 hT = 0.82
T4s = g -1
= 1.4 -1
= 725 K
hC = 0.80 T3 = 610°C = 883 K
g (10) 1.4
( rp) T1 = 15°C = 288 K ma = 16 kg/s
The isentropic work done per kg of air by gas turbine For compression
wT = h3 – h4s = Cp (T3 – T4s) Cp = 1.005 kJ/kg ◊ K g = 1.4
= 1.005 ¥ (1400 – 725) = 678.25 kJ/kg For expansion
The turbine efficiency is given as Cpg = 1.11 kJ/kg ◊ K gg = 1.333
Actual work w To find The power output of the gas turbine plant.
hT = = out
Isentropic work wT
Schematic
Actual turbine work
wout = hT wT = 0.8 ¥ 678.25 = 542.6 kJ/kg Combustion
chamber T3 = 610°C
The net work output of the plant 2 3
wnet = wout – win Compressor Turbine Generator
= 542.6 – 350.75 = 191.85 kJ/kg
(i) Thermal efficiency of the cycle
1 4
w 191.85
hth = net = = 0.2542 = 25.42% T1 = 15 C
.
qin 754.75 m = 16 kg/s (a) Schematic
(ii) The back work ratio T
Compression work 350.75 3
bwr = = 883 K
Turbine work 542.75
= 0.646 (64.6%)
(iii) Power output of the plant
4
The density of incoming air, 2
4s
2s
p1 100 kPa
r = = = 1.16 kg/m3
RT1 0.287 ¥ 300 288 K
1
s
The mass-flow rate of air,
(b) T–s diagram
ma = rV = 1.161 ¥ 5 = 5.80 kg/s
Fig. 27.8
Gas Turbine Plant 931

Assumptions The power output of the plant


(i) Compression and expansion are adiabatic. P = ma wnet = 16 ¥ 48.14 = 770.26 kW
(ii) Each device operates in steady flow manner.
Example 27.3 In a gas turbine plant, air enters
(iii) No pressure drop during flow of gas through
the compressor at 15°C and it is compressed through
combustion chamber.
a pressure ratio of 4 with isentropic efficiency of 85%.
Analysis The schematic and T–s diagram for a gas The air–fuel ratio is 80 and the calorific value of fuel is
turbine unit are shown in Fig. 27.8. Analysing each 42000 kJ/kg.
device in separately. The turbine inlet temperature is 1000 K and the
isentropic efficiency of the turbine is 82%. Calculate the
Compressor
overall efficiency and air intake for a power output of
The temperature after isentropic compression,
g -1
260 kW. Take the mass of fuel in account.
1.4 - 1
g
T2s = T1 ( rp) = 288 ¥ (6) 1.4 = 480.5 K Solution
The isentropic compression work per kg of air
Given A gas turbine unit with
wC = h2s – h1 = Cp (T2s – T1)
T1 = 15°C = 288 K hT = 0.82
= 1.005 ¥ (480.5 – 288) = 193.5 kJ/kg
hC = 0.85 T3 = 1000 K
The isentropic efficiency of the compressor is given
ma
by rp = 4 = 80
Isentropic work wC mf
hC = =
Actual work win P = 260 kW CV = 42000 kJ/kg

wC 193.5 To find
or win = = = 241.87 kJ/kg
hC 0.80 (i) Overall efficiency of the plant,
(ii) Mass of air intake.
The actual work input to compressor win can also be
expressed as Assumptions
win = h2 – h1 = Cp (T2 – T1) (i) Compression and expansion are adiabatic.
241.87 (ii) Each device operates in a steady flow manner.
or T2 = + 288 = 528.66 K
1.005 (iii) No pressure drop during flow of gas through
combustion chamber.
Turbine The temperature after isentropic expansion
Schematic
with gg = 1.333
T
3
T3 883 1000 K
T4s = g g -1
= 1.333 -1
= 564.37 K

( rp)
gg (6) 1.333

Isentropic turbine work per kg of air 4


wT = Cpg (T3 – T4s) 2
2s 4s
= 1.11 ¥ (883 – 564.37) = 353.67 kJ/kg
The isentropic efficiency of the turbine is given as 288 K
1 s
Actual work output w
hT = = out T–s diagram
Isentropic turbine work wT
Actual work of turbine, Analysis The T–s diagram for gas turbine unit is shown
wout = 0.82 ¥ 353.67 = 290 kJ/kg in Fig. 27.9. Analysing each device separately.
Net work output per kg of the plant
Compressor
wnet = wout – win
The temperature after isentropic compression
= 290 – 241.87 = 48.14 kJ/kg
932 Thermal Engineering

g -1 1.4 -1 P = ( ma + m f ) ¥ Net work output per kg


g
T2s = T1 ( rp ) = 288 ¥ ( 4) 1.4 = 428 K Ê mf ˆ
= ma Á1 + ¥ wnet
The isentropic efficiency of the compressor is given Ë ma ˜¯
by Ê 1ˆ
Isentropic work T2 s - T1 260 kW = ma Á1 + ˜ ¥ 104
hC = = Ë 80 ¯
Actual work T2 - T1 or ma = 2.47 kg/s
428 - 288
or T2 = 288 + = 452.7 K
0.85 Example 27.4 Calculate the required air–fuel ratio
The actual work input to compressor win can also be in a gas turbine plant, whose turbine and compressor
expressed as efficiencies are 85 and 80%, respectively. Maximum
win = h2 – h1 = Cp (T2 – T1) cycle temperature is 875°C. The working fluid can be
= 1.005 ¥ (452.7 – 288) = 165.52 kJ/kg taken as air (Cp = 1.0 kJ/kg ◊ K, g = 1.4), which enters
the compressor at 1 bar and 27°C. The pressure ratio is
Turbine The temperature T4s after isentropic expansion; 4. The fuel used has calorific value of 42000 kJ/kg. There
T3 1000 is loss of 10% of calorific value in combustion chamber.
T4s = g -1
= 1.4 -1
= 672.95 K
( rp) g ( 4) 1.4 Solution
The isentropic efficiency of the turbine is given as Given A gas turbine plant with
Actual work output T -T p1 = 1 bar K T1 = 27°C = 300 K
hT = = 3 4
Isentropic work output T3 - T4 s rp = 4 hT = 0.85
hC = 0.80 T3 = 875°C = 1148 K
Actual temperature after expansion in turbine,
CV = 42000 kJ/kg Cp = 1.0 kJ/kg ◊ K
T4 = 1000 – 0.82 ¥ (1000 – 672.95)
g = 1.4
= 731.82 K
Turbine work output per kg of air To find Air–fuel ratio of plant.
wout = h3 – h4 = Cp (T3 – T4)
Assumptions
= 1.005 ¥ (1000 – 731.82)
(i) Compression and expansion are adiabatic.
= 269.52 kJ/kg
(ii) Each device operates in steady flow manner.
Net work output per kg of the plant
(iii) No pressure drop during flow of gas through
wnet = wout – win
combustion chamber.
= 269.52 – 165.52 = 104 kJ/kg of air
Heat supplied per kg of air Analysis The T–s diagram for a gas turbine unit is
shown in Fig. 27.10. Analysing each device separately.
mf
Qin = ¥ CV of fuel
ma Compressor
1 The temperature after isentropic compression
= ¥ 42000 = 525 kJ/kg of air g -1 1.4 -1
80 g
T2s = T1 ( rp ) = 300 ¥ ( 4) 1.4 = 445.8 K
(i) Overall efficiency of plant
Net work output per kg of air The isentropic efficiency of the compressor is given
hoverall = ¥ 1000 by
Heat supplied per kg of air
Isentropic work T2 s - T1
104 hC = =
= ¥ 100 = 19.8% Actual work T2 - T1
525
(ii) Mass-flow rate of air 445.8 - 300
or T2 = 300 + = 482.24 K
Net power out ot the plant is given by 0.80
Gas Turbine Plant 933

Schematic Dp2-3 = 0.1 bar P = 1065 kW


Cp = 1.024 kJ/kg ◊ K g = 1.4
T
3 To find
1148 K

(i) Air circulation rate


ba
r (ii) Heat supplied per kg of air, and
4
4 (iii) Thermal efficiency of the cycle
2
2s 4s Assumptions
r
1 ba (i) Compression and expansion are adiabatic.
300 K
1 (ii) Each device operates in steady flow manner.
s
T–s diagram
Analysis The T–s diagram for gas turbine unit is
Fig. 27.10 shown in Fig. 27.11. Analysing each device separately.

The heat supplied in a combustion chamber can be Compressor


analysed as The temperature after isentropic compression
g -1 1.4 -1
Heat supplied by fuel = Heat absorbed by burning fuel
g
T2s = T1 ( rp ) = 293 ¥ (5) 1.4 = 464.06 K
(1 – 0.1) m f CV = ( ma + m f ) Cp (T3 – T2)
The isentropic efficiency of the compressor is given
0.9 ¥ 42000 m f = 1.0 ¥ (1148 – 482.24) ( ma + m f ) by
ma + m f 0.9 ¥ 42000 Isentropic work T2 s - T1
or = = 56.777 hC = =
mf 1.0 ¥ (1148 - 482.24) Actual work T2 - T1
ma 464.06 - 293
or = 56.777 – 1 = 55.777 kg/kg of fuel or T2 = 293 + = 494.24 K
mf 0.85

Example 27.5 In a constant-pressure open-cycle


gas turbine , air enters at 1 bar and 20°C and leaves the
compressor at 5 bar. Using the following data:
Temperature of gases entersing the turbine = 680°C
Pressure loss in combustion chamber = 0.1 bar
Isentropic efficiency of compressor = 85%
Isentropic efficiency of turbine = 80%
Combustion efficiency = 85%
For air; Cp = 1.024 kJ/kg ◊ K, g = 1.4
Find
(a) the quantity of air circulation, if plant develops Fig. 27.11
1065 kW,
The actual work input to compressor win can also be
(b) heat supplied per kg of air circulation, expressed as
(c) thermal efficiency of cycle, win = h2 – h1 = Cp (T2 – T1)
Mass of fuel may be neglected. = 1.024 ¥ (494.24 – 293) = 206.07 kJ/kg
Solution Turbine The pressure at inlet of turbine is p3 =
p2 – 0.1 bar = 4.9 bar
Given A gas turbine plant with
The temperature T4s after isentropic expansion;
p1 = 1 bar T1 = 20°C = 293 K
T3 953
p2 = 5 bar rp =5 T4s = = = 605.19 K
g -1 1.4 -1
hT = 0.80 hC = 0.85 Ê p3 ˆ g Ê4.9 ˆ 1.4
hComb = 0.85 T3 = 680°C = 953 K ÁË p ˜¯ ÁË 1 ˜¯
1
934 Thermal Engineering

The isentropic efficiency of the turbine is given as rp = 8 T3 = 900°C = 1173 K


Actual work output T -T hC = 0.80 hT = 0.85
hT = = 3 4 ma = 20 kg/s CV = 42000 kJ/kg
Isentropic work output T3 - T4 s
hmech = 0.96 hgen = 0.95
Actual temperature after expansion in turbine,
For air Cp = 1.005 kJ/kg ◊ K, g = 1.4
T4 = 953 – 0.8 ¥ (953 – 605.19) = 674.75 K
Turbine work output per kg of air For gas es Cpg = 1.128 kJ/kg ◊ K and gg = 1.34
wout = h3 – h4 = Cp (T3 – T4) To find
= 1.024 ¥ (953 – 674.75) = 284.92 kJ/kg (i) Output of plant, and
Net work output per kg of the plant (ii) Thermal efficiency.
wnet = wout – win
Assumptions
= 284.92 – 206.07 = 78.85 kJ/kg of air
(i) Compression and expansion are adiabatic.
Power developed by plant is given by
(ii) Each device operates in a steady flow manner.
P = ma wnet
(iii) No pressure drop during flow of gas through
(i) The mass flow rate of air, combustion chamber.
1065
ma = = 13.5 kg/s Schematic
78.85
(ii) Actual heat supplied per kg of air,
C p (T3 - T2 ) 1.024 ¥ (953 - 494.24)
qin = =
hComb 0.85
= 552.67 kJ/kg
(iii) Thermal efficiency of cycle,
wnet 78.85
hTh = = = 0.1426 or 14.26%
qin 552.67

Example 27.6 A gas turbine plant consists of two


turbines. One turbine drives the compressor and the other
develops the power output. Both turbines have their own
combustion chambers which are served by air directly
from the compressor. Air enters the compressor at 1 bar
and 15°C and is compressed to 8 bar with an isentropic
efficiency of 80%. Due to heat addition in the combustion
chamber, the inlet temperature of gas to both the turbines
reaches to 900°C. The isentropic efficiency of the turbine
is 85%. The mass-flow rate of air at the compressor is 20 Fig. 27.12
kg/s. The calorific value of fuel is 42000 kJ/kg. Calculate
the output of plant and thermal efficiency, if mechanical
Analysis The schematic and T–s diagram are shown in
efficiency is 96% and the generator efficiency is 95%.
Fig. 27.12. Analysing each device separately.
Take Cp = 1.005 kJ/kg ◊ K and g = 1.4 for air and Cp =
1.128 kJ/kg ◊ K and g = 1.34 for gases. Neglect the mass Compressor
of fuel. The temperature after isentropic compression
g -1 1.4 -1
Solution T2s = T1 ( rp ) g
= 288 ¥ (8) 1.4 = 521.7 K
Given A gas turbine plant with two turbines:
The isentropic efficiency of the compressor is given
p1 = 1 bar T1 = 15°C = 288 K
by
Gas Turbine Plant 935

Isentropic work T2 s - T1 (ii) Thermal efficiency


hC = =
Actual work T2 - T1 Heat supplied to combustion chambers
Qin = ma Cpg (T3 – T2)
521.7 - 288 = 20 ¥ 1.128 ¥ (1173 – 580.12)
or T2 = 288 + = 580.12 K
0.8 = 13339.8 kW
The actual work input to compressor win can also be Thermal efficiency of the plant
expressed as
WT 3350.35
win = Cp (T2 – T1) = 1.005 ¥ (580.12 – 288) hth = =
Qin 13339.8
= 293.58 kJ/kg
= 0.2511 or 25.11%
Turbine The temperature T4s after isentropic expansion
of gases;
T3 1173
T4s = g g -1
= 1.34 -1
= 692.08 K
Ê p3 ˆ gg Ê 8ˆ 1.34
ÁË p ˜¯ ÁË 1 ˜¯
1
The isentropic efficiency of the turbine is given as The network output of a gas turbine plant can be
increased by either decreasing compression work
Actual work output T -T
hT = = 3 4 or by increasing the turbine work or both. The
Isentropic work output T3 - T4 s
thermal efficiency of the cycle can be improved by
Actual temperature after expansion in turbine, saving the heat input for the same network output.
T4 = 1173 – 0.85 ¥ (1173 – 692.08) The improvement is possible by
= 764.21 K
Turbine work output of each turbine per kg of air
1. Regeneration by preheating the air leaving
wT = Cpg (T3 – T4)
the compressor with turbine exhaust gases,
= 1.128 ¥ (1173 – 764.21) = 461.1 kJ/kg
thus saving the heat input.
(i) Power output of plant 2. Reheating of gases after each stage of
Let m kg mass flow through the turbine 1, which
expansion to obtain more work from the
drives the compressor. Thus, turbine.
win = m1wT 3. Intercooling during compression stages to
or 293.58 = 461.1 m1 reduce the work input to the compressor.
293.58
or m1 = = 0.6366 kg/kg of air
461.1
The mass of air-flow rate through the turbine 2 The temperature of exhaust gases leaving the
(neglecting the mass of fuel) turbine of a gas turbine engine is considerably
m2 = ma (1 – m1) = 20 ¥ (1 – m ) higher than the temperature of air delivered by the
= 20 ¥ (1 – 0.6366) compressor. Therefore, high-pressure air leaving
= 7.266 kg/s the compressor can be heated by hot exhaust gases.
Power output of the turbine 2 The device, a counterflow heat exchanger used
WT = m2 wT = 7.266 ¥ 461.1 for transfer of heat from hot gases to air, is known
= 3350.35 kW as regenerator. A simple open-cycle gas turbine
Power output of the plant (generator) with a regenerator is shown in Fig. 27.13.
P = WT hMech hgen As illustrated in Fig. 27.14(b) T–s diagram, the
= 3350.35 ¥ 0.95 ¥ 0.96 compressed air at the state 2 enters the regenerator
= 3055.74 kW (a heat exchanger) in which it is heated to the
936 Thermal Engineering

6 Regenerator Combustion 3
chamber 4
5
2

Compressor Turbine wnet

1
Fresh air

Fig. 27.13 A gas turbine cycle with regenerator

p
transfer in regenerator
5
2 3 qgen = h5 – h2 ...(27.6)
Maximum possible heat transfer in regenerator
qmax = h4 – h2 ...(27.7)
The effectiveness of regenerator
4
qgen h5 - h2
1 6 e= = ...(27.8)
qmax h4 - h2
0 v
(a) If specific heat Cp is assumed constant, then
T 3 T5 - T2
qin e = ...(27.9)
T4 - T2
The thermal efficiency of gas turbine cycle with
5¢ 4
q reg 5 regenerator can be found as
ration
Regene
2 6 eg wnet
=qr hth =
ed
qsav qin
1
qout
0 s
wnet = wT – wC = (h3 – h4) – (h2 – h1)
(b) qin = h3 – h5 = Cp (T3 – T5)
Fig. 27.14
hth, gen =
( h3 - h4 ) - ( h2 - h1 )
gas turbine cycle h3 - h5
state 5. The hot exhaust gases leaving the turbine
=
(T3 - T4 ) - (T2 - T1 ) ...(27.10)
at the state 4, are used to preheat the air entering T3 - T5
the combustion chamber. Hence, a heat transfer
Under ideal conditions T4 = T5 for e = 1, then
qin, from an external source is required only to
increase the air temperature from the state 5 to the ÊT ˆ
state 3. This, in turn, decreases the heat input in T1 Á 2 - 1˜
T2 - T1 Ë T1 ¯
the combustion chamber for same network output, hth, reg = 1 - =1-
T3 - T4 Ê T ˆ
and thus thermal efficiency of gas turbine cycle T3 Á1 - 4 ˜
increases. Ë T3 ¯
A certain temperature difference is necessary to Ï g -1 ¸
transfer heat from one fluid to another. Therefore, Ô Ê p2 ˆ g Ô
- 1Ô
the air cannot be preheated to a temperature equal T1 ÔÔ ÁË p1 ˜¯ Ô
= 1- Ì g - 1 ˝
to the turbine exhaust temperature T4. Assuming a T3 Ô Ô
Ê p ˆ g
well-insulated heat exchanger, without any change Ô1 - Á 1 ˜ Ô
in kinetic and potential energy changes, actual heat ÔÓ Ë p2 ¯ Ô˛
Gas Turbine Plant 937

T
Ï g -1 ¸ 873 K
3
ÔÊ p2 ˆ g Ô g -1
- 1Ô
T1 ÔÔ ÁË p1 ˜¯ Ô Ê p2 ˆ g

in
q
= 1- Ì g -1 ˝¥Á ˜
T3 Ô
Ê p2 ˆ g Ô Ë p1 ¯
4
ÔÁ - 1Ô
ÔÓ Ë p1 ˜¯ Ô˛ 2s
2 5 4s

g –1 g –1
288 K
T1 g Tmin 1
= 1– ¥ ( rp ) =1= ( rp ) g 0 s
T3 Tmax
Fig. 27.15
...(27.11)
1.4 -1
Example 27.7 In a gas turbine plant, the compressor
= 288 ¥ ( 4) 1 = 428 K
takes in air at a temperature of 15°C and compresses
it to four times the initial pressure with an isentropic The isentropic efficiency of the compressor is given
efficiency of 85%. The air is then passed through a heat by
exchanger, heated by the turbine exhaust before reaching Isentropic work T2 s - T1
hC = =
the combustion chamber. The turbine inlet temperature Actual work T2 - T1
is 600°C and its efficiency is 80%. Neglecting all losses 428 - 288
or T2 = 288 + = 452.7 K
except those mentioned, and treating the working fluid 0.85
throughout the cycle to have the properties of air, The actual work input to compressor win can also be
calculate thermal efficiency and work ratio of the cycle expressed as
if (a) heat exchanger is perfect, and (b) heat exchanger
win = h2 – h1 = Cp (T2 – T1)
gives 85% of available heat to air.
= 1.0045 ¥ (452.7 – 288) = 165.44 kJ/kg
Take R = 0.287 kJ/kg ◊ K, g = 1.4, and constant
specific heats throughout. Turbine The temperature T4s after isentropic expansion
T3 873
Solution T4s = g -1
= 1.4 -1
= 587.48 K
g ( 4) 1.4
Given A gas turbine power plant with regeneration ( rp)
T = 15°C = 288 K rp = 4 The isentropic efficiency of the turbine is given as
hC = 0.85 g = 1.4 Actual work output T -T
T3 = 600°C = 873 K hT = 0.8 hT = = 3 4
Isentropic work output T3 - T4 s
R = 0.287 kJ/kg ◊ K
Actual temperature after expansion in turbine,
(i) e = 1.0 (ii) e = 0.85
T4 = 873 – 0.80 ¥ (873 – 587.48) = 644.58 K
To find Turbine work output per kg of air
(i) Thermal efficiency of the cycle, wout = h3 – h4 = Cp (T3 – T4)
(ii) Work ratio. = 1.0045 ¥ (873 – 644.58)
Analysis Specific heat = 229.44 kJ/kg
Net work output per kg of the plant
g R 1.4 ¥ 0.287
Cp = = = 1.0045 kJ/kg ◊ K wnet = wout – win = 229.44 – 165.44
g -1 1.4 - 1
= 64 kJ/kg of air
Analysing each device in steady flow manner.
Heat exchanger The effectiveness of heat exchanger is
Compressor Temperature T2s after isentropic compres- given by
sion
T5 - T2
g -1 e =
g T4 - T2
T2s = T1 ( rp )
938 Thermal Engineering

Case (i) given that e = 1.0, Solution


T5 - 452.7
1.0 = Given A gas turbine power plant with regeneration
644.58 - 452.7
T1 = 10°C = 283 K rp = 4
Thus the temperature T5 of air leaving the heat hC = 0.8 hT = 0.85
exchanger T3 = 700°C = 973 K e = 0.75
T5 = 452.7 + (644.58 – 452.7) = 644.58 Dp = 0.14 bar \ p3 = 3.86 bar
The heat supplied to air
qin = Cp (T3 – T5) = 1.0045 ¥ (873 – 644.58) ba
r
1 4
= 229.44 kJ/kg T 0.
3
(i) Thermal efficiency of the cycle 973 K

wnet 64 r
hth = = = 0.2789 = 27.89% ba r
4 ba
qin 229.44 1
4
(ii) Work ratio, 2s 2 4s

Net work output 64


rw = = 283 K
1
Turbine work 229.44 s
= 0.2789
or 27.89% Fig. 27.16
Case (ii) given that e = 0.85,
T5 - 452.7 To find Thermal efficiency of the cycle.
0.85 =
644.58 - 452.7 Assumptions For air;
Thus, the temperature T5 of air leaving heat exchanger Cp = 1.005 kJ/kg ◊ K, and g = 1.4
or T5 = 452.7 + 0.85 ¥ (644.58 – 452.7)
= 615.8 K Analysis The pressure at state 3 being
The heat supplied to air p3 = p4 – 0.14 bar = 3.86 bar
qin = Cp (T3 – T5) = 1.0045 ¥ (873 – 615.8) Analysing each device in steady flow manner.
= 258.36 kJ/kg Compressor Temperature T2s after isentropic
(i) Thermal efficiency of the cycle compression
wnet 64 g -1 1.4 -1
hth = = = 0.2277 = 22.77% g
qin 258.36 T2s = T1( rp) = 283 ¥ ( 4) 1 = 420.53 K
(ii) The work ratio remains unaffected, thus rw = The isentropic efficiency of the compressor is given
27.89% by
Isentropic work T2 s - T1
Example 27.8 In a gas turbine plant, air at 10°C and hC = =
atmospheric pressure is compressed through a pressure Actual work T2 - T1
ratio of 4. In a heat exchanger and combustion chamber, 420.53 - 283
or T2 = 283 + = 454.92 K
air is heated to 700°C with a pressure drop of 0.14 bar. 0.8
After expansion through the turbine, the air passes The actual work input to compressor win can also be
through a heat exchanger, which cools the air through expressed as
75% of maximum range possible and air is finally win = h2 – h1 = Cp (T2 – T1)
discharged to the atmosphere. The isentropic efficiency = 1.005 ¥ (454.92 – 283) = 172.78 kJ/kg
of compressor and turbine is 0.8 and 0.85 respectively.
Turbine The temperature T4s after isentropic expansion
Calculate the thermal efficiency of the plant.
from a pressure of 3.86 bar to 1 bar
Gas Turbine Plant 939

T3 973 expansion are isentropic. Cp for air and exhaust gases


T4s = = = 661.48 K can be taken as 1 kJ/kg ◊ K and g = 1.4.
g -1 1.4 -1
Ê p3 ˆ g Ê 3.86 ˆ 1.4
ÁË p ˜¯ ÁË 1 ˜¯ Solution
1
The isentropic efficiency of the turbine is given as Given A gas turbine power plant with two stage
Actual work output T -T turbine
hT = = 3 4 T1 = 300 K rp = 4
Isentropic work output T3 - T4 s
ma = 20 kg/s T3 = 1000 K
Actual temperature after expansion in turbine,
Cp = 1.0 kJ/kg ◊ K g = 1.4
T4 = 973 – 0.85 ¥ (973 – 661.48)
wC = wHP
= 708.2 K
Turbine work output per kg of air To find
wout = h3 – h4 = Cp (T3 – T4) (i) Pressure ratio of low-pressure turbine,
= 1.005 ¥ (973 – 708.2) = 266.11 kJ/kg (ii) Temperature of exhaust gases, and
Net work output per kg of the plant (iii) Thermal efficiency of the plant.
wnet = wout – win = 266.11 – 172.78 Analysis Analysing each device in a steady flow
= 93.33 kJ/kg of air manner.
Heat exchanger Compressor Temperature T2 after isentropic compres-
The effectiveness of heat exchanger is given by sion
T5 - T2
e =
T4 - T2
C.C.
2 3 1000 K
Given that e = 0.75,
T5 - 454.92 Compressor HP
\ 0.75 = Turbine
708.2 - 454.92
rp = 4 4
Thus the temperature T5 of air leaving heat exchanger 300
. K
ma = 20 kg/s Cp = 1.0 kJ/kg.K
or T5 = 454.92 + 0.75 ¥ (708.2 – 454.92)
g = 1.4
= 644.88 K LP Turbine

The heat supplied to air


qin = Cp (T3 – T5) = 1.005 ¥ (973 – 644.88) 5
= 329.76 kJ/kg
T
Thermal efficiency of the cycle 3
1000 K
wnet 93.3
hth = = = 0.28.29 or 28.29%
qin 329.76
4
Example 27.9 In a gas turbine plant, the compressor
r
ba

is driven by the high-pressure turbine. The exhaust from


4

5
high pressure turbine enters into a low-pressure turbine,
which runs the load. The air flow rate is 20 kg/s, and 2
minimum and maximum temperatures in the cycle are ar
1b
300 K and 1000 K, respectively. The compressor pressure p=
300 K
ratio is 4. Calculate the pressure ratio of low-pressure 1
s
0
turbine, temperature of exhaust gases from the unit
and thermal efficiency of the plant. Compression and Fig. 27.17
940 Thermal Engineering

g -1 1.4 -1 Example 27.10 Air is drawn in a gas turbine unit


g
T2 = T1( rp) = 300 ¥ ( 4) 1 at 15°C and 1.01 bar and the pressure ratio is 7:1. The
= 445 .8 K compressor is driven by HP turbine and the LP turbine
The work input to compressor drives a separate power shaft. The isentropic efficiency
of the compressor, HP and LP turbines are 82, 85 and
= Work output of high pressure turbine
85%, respectively. If the maximum cycle temperature is
wC = wHP
610°C, calculate
Cp (T2 – T1) = Cp (T3 – T4)
(a) pressure and temperature of gases entering the
1.0 ¥ (445.8 – 300)
power turbine
= 1.0 ¥ (1000 – T4)
(b) net power developed by the unit per kg/s mass-
or T4 = 1000 – (445.8 – 300) = 854.2 K flow rate
(i) Pressure ratio of low-pressure turbine (c) the work ratio
The pressure ratio across the H.P. turbine can be (d) thermal efficiency of the unit
given as
g 1.4 Neglect the mass of fuel and assume the following:
p3 Ê T ˆ g -1 Ê 1000 ˆ 1.4 -1 For compression process, Cpa = 1.005 kJ/kg ◊ K and
= Á 3˜ =Á
p4 Ë T4 ¯ Ë 854.2 ˜¯ ga = 1.4,
= 1.736 For combustion and expansion processes, Cpg =
1.15 kJ/kg ◊ K and gg = 1.33.
p3
The pressure ratio can be expressed as
p4 Solution
p3 p p p p
= 3¥ 5 = 2 ¥ 5 Given A gas turbine power plant with a two-stage
p4 p5 p4 p1 p4
turbine
Ê p5 ˆ p1 = 1.01 bar T1 = 15°C = 288 K
or 1.736 = 4 ¥ Á ˜
Ë p4 ¯ rp = 7
wC = wHP T3 = 610°C = 883 K
p5 1.736
or = = 0.434 hC = 0.82
p4 4
hT, HP = 0.85 hT, LP = 0.85
(ii) Temperature of exhaust gases For air Cpa = 1.0 kJ/kg ◊ K and ga = 1.4
The temperature T5 of exhaust gases coming out For gases Cpg = 1.15 kJ/kg ◊ K and gg = 1.33
from unit
g -1 To find
1.4 - 1
T5 Êp ˆ g 1.4 (i) Pressure and temperature of gases entering low
= Á 5˜ = (0.434) = 0.7878
T4 Ë p4 ¯ pressure turbine,
or T5 = 0.788 ¥ 854.2 = 672.95 K (ii) The power developed by the unit per kg/s of mass
flow,
(iii) Thermal efficiency of the plant
(iii) Work ratio, and
Net power output per kg of air of the plant
(iv) Thermal efficiency of the plant.
wnet = wLp = Cp (T4 – T5)
= 1.0 ¥ (854.2 – 672.95) Analysis Analysing each device in a steady flow
= 181.25 kJ/kg manner.
Heat supplied per kg of air Compressor Temperature T2s after isentropic compres-
qin = Cp (T3 – T2) sion
= 1.0 ¥ (1000 – 445.8) g -1 1.4 -1
= 554.2 kJ/kg g
T2s = T1( rp) = 288 ¥ (7) 1
w 181.25
hth = net = = 0.327 or 32.7% = 502.16 K
qin 554.2
Gas Turbine Plant 941

Schamtic and T–s diagram Isentropict emperature,


T3 - T4 883 - 654.76
C.C. T4s = T3 - = 883 -
2 3 610°C hT , HP 0.85
HP = 614.48 K
Compressor
Turbine
The pressure ratio across H.P. turbine is given by
hC = 0.82 hT = 0.85 4
gg 1.33
15°C, p3 Ê T ˆ g g –1 Ê 883 ˆ 1.33 –1
1.01 bar = Á 3˜ =Á
p4 Ë T4 s ¯ Ë 614.48 ˜¯
LP Turbine
= 4.311
p3 7.07
hT = 0.85 5 Pressure, p4 = = = 1.64 bar
4.311 4.311
T
3 Pressure of gases entering LP turbine is 1.64 bar.
883 K
(ii) Net power developed per kg/s of mass flow
4 p4
r The pressure ratio can be expressed as
ba 4s p5
07
7.
p4 p p p p 7
2 5 = 4 ¥ 3 = 4 ¥ 2 =
5s p5 p3 p5 p3 p1 4.311
2s
bar = 1.623
1.01
288 K
1 s Then temperature T5s after isentropic expansion
Fig. 27.18 g g -1 1.33 -1
T4 Êp ˆ gg Ê 1.623 ˆ 1.33
= Á 4˜ =Á
The isentropic efficiency of the compressor is given T5 s Ë p5 ¯ Ë 1 ˜¯
by
= 1.127
Isentropic work T2 s - T1
hC = = 654.76
Actual work T2 - T1 or T5s = = 580.56 K
1.127
502.16 - 288
or T2 = 288 + = 549.17 K
For LP turbine, isentropic efficiency is given by
0.82
(i) Pressure and temperature entering the power hT, LP = T4 - T5
turbine T4 - T5 s
The actual work input to compressor win can also Actual temperature,
be expressed as T5 = T4 – hT, LP (T4 – T5s)
win = Cpa (T2 – T1) = 654.76 – 0.85 ¥ (654.76 – 580.56)
= 1.005 ¥ (549.17 – 288) = 591.64 K
= 262.47 kJ/kg Net power output per kg/s of mass flow rate
Compression work = H.P. turbine work wnet = wLP = Cpg (T4 – T5)
or win = Cpg (T3 – T4) = 1.15 ¥ (654.76 – 591.64)
262.47 = 1.15 ¥ (883 – T4) = 72.59 kJ/kg
or T4 = 654.76 K (iii) Work ratio
Temperature of gas entering the power turbine is
Net work output
654.76 K. Work ratio =
Total work produced
Using isentropic efficiency of HP turbine
Actual work output T -T 72.59
hT, HP= = 3 4 = = 0.2166
Isentropic work output T3 - T4 s 262.47 + 72.59
942 Thermal Engineering

(iv) Thermal efficiency of the plant air-standard Brayton cycle modified as shown
Heat supplied per kg of air in Fig. 27.19(a) After expansion from the state 3
qin = Cpg (T3 – T2) to state 4 in the first turbine, the air is reheated at
= 1.15 ¥ (883 – 549.17) constant pressure from the state 4 to state 5. The
= 383.9 kJ/kg expansion is then completed in the second turbine
w 72.59 from the state 5 to the state 6. The ideal Brayton
hth = net =
qin 383.9 cycle without reheat is cycle 1–2–3–6¢–1 and
= 0.189 or 18.9.7% with reheat is cycle 1–2–3–4–5–6–1 as shown in
Fig. 27.19(b).
Net work output per kg of air in a cycle
For metallurgical reasons, the temperature of the wnet = Cp (T3 – T4) + Cp (T5 – T6)
combustion products entering the turbine cannot be – Cp (T2 – T1) ...(27.12)
increased to a higher level. By introducing reheat- Heat supplied per kg of air in a cycle
ing during expansion, the turbine work and conse- qin = qcomb + qreheat
quently, the network of the gas turbine cycle can = Cp (T3 – T2) + Cp (T5 – T4) ...(27.13)
be improved without changing compressor work or Thermal efficiency of reheat cycle can be
maximum temperature in the cycle. The expansion expressed as
of combustion products takes place in two or more
wnet
turbines with constant pressure heating before each hreheat =
expansion. This constant pressure heating between qin
two turbine expansions is known as reheating and C p (T3 - T4 ) + C p (T5 - T6 ) - C p (T2 - T1 )
=
this modified cycle is known as reheat cycle. C p (T3 - T2 ) + C p (T5 - T4 )
The basic features of a two-stage gas turbine ...(27.14)
with reheat are brought out by considering an ideal
qcomb qreheat

Combustor Reheater

2 3 4 5

Compressor Turbine I Turbine II


wnet

m = 1kg
6
1
(a) Schematic

T
3 5

6
2

1
s
(b) T–s diagram
Fig. 27.19 Ideal gas turbine with reheat
Gas Turbine Plant 943

It can be analytically proved that the maximum


net work output from the plant would be for
rp1 = rp ...(27.15)
p p
where rp1 = 2 = 5 , intermediate pressure ratio,
and p4 p6

p2 p
rp = = 3 , total pressure ratio.
p1 p6
The total work of a two-stage turbine is greater
than that of a single expansion from state 3 to
Fig. 27.20
state 6¢. Thus, the network of reheat cycle is
gerater than that of a cycle without reheat. Despite Analysis The temperature after isentropic compression
the increase in net work with reheat, the thermal g –1 1.4 – 1
efficiency would not necessarily improve because Êp ˆ g Ê 1000 ˆ 1.4
T2 = T1 Á 2 ˜ = 300 ¥ Á
of an increase in total heat addition to the cycle. Ë p1 ¯ Ë 100 ˜¯
However, the temperature at the exit of the turbine is = 579.2 K
higher with reheat than without reheat, so potential Temperature after isentropic expansion 3–4;
g -1
for regeneration is enhanced.
T3 Ê p ˆ g
= Á 3˜
Example 27.11 Air enters the compressor of an T4 Ë p4 ¯
ideal air-standard Brayton cycle at 100 kPa, 300 K g -1 1.4 -1
Êp ˆ g Ê 300 ˆ 1.4
and is compressed to 1000 kPa. The temperature at or T4 = T3 Á 4 ˜ = 1400 ¥ Á
the inlet to the first turbine is 1400 K. The expansion Ë p3 ¯ Ë 1000 ˜¯
takes place isentropically in two stages with reheat to = 992.5 K
1400 K between the two stages at a constant pressure of Temperature after isentropic expansion 5–6;
300 kPa. A regenerator having an effectiveness of 100% g -1 1.4 -1
is also incorporated in the cycle. Determine the thermal Êp ˆ g Ê 100 ˆ 1.4
T6 = T5 Á 6 ˜ = 1400 ¥ Á
efficiency of the cycle. Ë p5 ¯ Ë 300 ˜¯
= 1022.8 K
Solution
Total turbine work;
Given An ideal air-standard Brayton cycle with reheat wT = Cp (T3 – T4) + Cp (T5 – T6)
and regeneration: = 1.005 ¥ [1400 – 992.5] + 1.005
p1 = 100 kPa T1 = 300 K ¥ [1400 – 1022.8]
p2 = 1000 kPa T3 = 1400 K = 409.53 + 379.08 = 788.6 kJ
T5 = 1400 K p4 = p5 = 300 kPa Compressor work;
ereg = 1.0 wC = Cp (T2 – T1) = 1.005 ¥ [579.2 – 300]
To find Thermal efficiency of the cycle. = 280.6 kJ
Net work of the cycle
Assumptions = wT – wC
(i) Each component in the cycle is analysed as = 788.6 – 280.6 = 508 kJ
control volume at steady state. Total heat supplied in the cycle
(ii) Compressor and turbine processes are isentropic. qin = Cp (T3 – T6) + Cp (T5 – T4)
(iii) No pressure drop for flow through the heat = 1.005 ¥ (1400 – 1022.8) + 1.005
exchanger. ¥ (1400 – 992.5)
(iv) Working fluid is air with its standard properties. = 379.08 + 409.53 = 788.6 kJ
944 Thermal Engineering

The thermal efficiency of the cycle p1 = 1 bar T1 = 30°C = 303 K


Network done/kg rp = 5 p5 = p6 = 2.24 bar
h = p2 = 5 bar T3 = 550°C = 823 K
Total heat supplied/kg
T5 = 823 K hC = 0.8
508
= = 0.644 = 64.4% hT = 0.85 P = 5 MW = 5000 kW
788.6
For air Cpa = 1.0 kJ/kg ◊ K ga = 1.4 and
Example 27.12 The following data refers to a gas For gases Cpg = 1.15 kJ/kg ◊ K gg = 1.33
turbine plant: To find
Power developed = 5 MW (i) Mass flow rate of air, and
Inlet pressure and temperature of air to compressor (ii) Overall efficiency of the cycle.
= 1 bar and 30°C
Pressure ratio of the cycle =5 Assumptions
Isentropic efficiency of the compressor = 80% (i) Compression and expansion are adiabatic.
Isentropic efficiency of turbines = 85% (ii) Each device operates in steady flow manner.
Maximum temperature in the turbines = 550°C (iii) No pressure drop during flow of gas through
Take for air, Cp = 1.0 kJ/kg ◊ K, g = 1.4 and combustion chamber.
for gases, Cp = 1.15 kJ/kg ◊ K, g = 1.33. Analysis Analysing each device in steady flow manner.
If a reheater is used between two turbines at a
Compressors The temperature after isentropic com-
pressure of 2.24 bar, calcualte the following:
pression 1–2:
(a) Mass flow rate of air, g -1 1.4 -1
(b) The overall efficiency, Êp ˆ g Ê 5ˆ 1.4
T2s = T1 Á 2 ˜ = 303 ¥ Á ˜
Neglect the mass of fuel. Ë p1 ¯ Ë 1¯
= 479.9 K
Solution
The isentropic efficiency of the compressor is given
Given A gas turbine plant with reheating and by
regeneration: Isentropic work T2 s - T1
hC = =
Actual work T2 - T1
qcomb qreheat

Combustor Reheater
2 3 550°C 4 5 550°C, 2.24 bar

Compressor HP Turbine LP Turbine


wnet = 5 MW

6
1 bar,
1 T
30°C
r
ba

p2
24
2.

4
823 K 6

5s 5
7
7s
2
2s

p1
303 K
1
s
0

Fig. 27.21
Gas Turbine Plant 945

479.9 - 303 or 5000 = 69.88 ma


or T2 = 303 + = 524.12 K
0.8 or ma = 75.55 kg/s
The actual work input to compressor win can also be (ii) Overall efficiency of plant
expressed as The heat supplied to air
win = Cp (T2 – T1) qin = Cpg (T3 – T2) + Cpg (T6 – T5)
= 1.0 ¥ (524.12 – 303) = 221.12 kJ/kg = 1.15 ¥ (823 – 524.12) + 1.15
HP Turbine The temperature T5s after isentropic ¥ (823 – 696.63)
expansion in the HP turbine = 489 kJ/kg of air
T4 823 Overall efficiency of the cycle
T5s = g g -1
= 1.33 -1 wnet 69.88
Ê p2 ˆ gg Ê 5 ˆ 1.33 hth = = = 0.1428 or 14.28%
ÁË 2.24 ˜¯ qin 489
ÁË p ˜¯
5
= 674.33 K
Using isentropic efficiency of HP turbine;
The net work output of a gas turbine can also be
T4 - T5
hT, HP = increased by reducing the compressor work input.
T4 - T5 s
It can be accomplished by use of multistage com-
Actual temperature after expansion in HP turbine, pression with intercooling, i.e., by an isothermal
T5 = 823 – 0.85 ¥ (823 – 674.33) compression.
= 696.63 K The isothermal cooling of gas during compres-
LP Turbine The temperature T7s after isentropic sion is difficult to achieve in practice. The gas is
expansion in the LP turbine compressed in stages with cooling the gas between
T6 823 stages. The heat exchangers used for cooling of gas
T7s = g g -1
= 1.33 -1
Ê 2.24 ˆ
are known as intercoolers.
Ê p5 ˆ gg 1.33
ÁË p ˜¯
1
ËÁ 1 ¯˜ Figure 27.22 illustrates the effect of intercooling
between two compression stages. Process 1–c
= 673.74 K represents an isentropic compression from the
Using isentropic efficiency of LP turbine; state 1 to state c, where the intermediate pressure
T6 - T7 is pi. The process c–d represents constant
hT, LP =
T6 - T7 s pressure gas cooling in a heat exchanger from
Actual temperature after expansion in LP turbine, temperature Tc to Td. The process d–2 is further
T7 = 823 – 0.85 ¥ (823 – 673.74) isentropic compression to state 2. The work input
= 696.13 K for compression is represented on a p–v diagram
Total turbine work output per kg of air by area a–1–c–d–2–b–a which is less than the
wout = wHP + wLP isentropic compression in a single stage from the
= Cpg (T4 – T5) + Cpg (T6 – T7) state 1 to the state 2s, without intercooling, viz.,
= 1.15 ¥ (823 – 696.74) + 1.15 ¥ (823 – 696.13) enclosed area a–1–2s–b–1. The hatched area d–c–
= 145.2 + 145.9 = 291 kJ/kg 2s–2–d on a p–v diagram represents reduction in
Net work output per kg of the plant the work input due to intercooling between two
wnet = wout – win = 291 – 221.12 stages.
= 69.88 kJ/kg of air If the gas after first-stage compression is
(i) Mass-flow rate of air cooled to initial temperature T1 of air at constant
Power develoved by plant is given by intermediate pressure pi, the cooling is called
P = ma wnet perfect intercooling.
946 Thermal Engineering

2 È g -1 ˘
ÍÊ pi ˆ g ˙
Compressor Compressor = C p T1 ÍÁ ˜ - 1˙
I II Ë p1 ¯
Í ˙
Î ˚
c d
1 È g -1 ˘
m = 1 kg
ÍÊ p2 ˆ g ˙
qR + C p Td ÍÁ ˜ - 1˙ ...(27.17)
Ë pi ¯
(a) Schematic Í ˙
p Î ˚
2 2s
b For perfect intercoling between the stages Td =
Isentropic T1
compression
È g -1 g -1 ˘
d c
ÍÊ pi ˆ g Ê p2 ˆ g ˙
Isothermal \ wC = C p T1 ÍÁ ˜ +Á ˜ - 2˙
compression
Ë p1 ¯ Ë pi ¯
Í ˙
1
Î ˚
a p1
g -1
0 v Using k = ,
(b) p–v diagram g
T ÈÊ p ˆ k Ê p ˆ k ˘
p2
then wC = C p T1 ÍÁ i ˜ + Á 2 ˜ - 2˙
2s
ÍË p1 ¯ Ë pi ¯ ˙
pi Î ˚
Tc = T2
2 ...(27.18)
c p1
For minimum compression work or maximum
T1 = Td output of the plant, applying the condition of
d 1
maxima, i.e., differentiating Eq. (27.18) with
s respect to intermediate pressure pi and equating it
0
(c) T–s diagram to zero,
Fig. 27.22 Two-stage compression with - dwC È Ê 1 ˆk ˘
cooling = Cp T1 Í k Á ˜ ( pi ) k -1 - k ( p2) k ( pi ) - k -1 ˙
dpi Í Ë p1 ¯ ˙
Î ˚
Assuming isentropic steady flow in both =0
compressors with negligible changes in kinetic
and potential energies. For 1 kg of air flow into the or ( pi)k–1 ( pi)k+1 = p1k p2k
system, or ( pi)2k = ( p1 p2)k
Work input to first stage compressor or pi = p1 p2 ...(27.19)
wC1 = Cp (Tc – T1) For minimum compression work, optimum
Work input to second stage compressor intermediate pressure is pi = p1 p2 . The
wC2 = Cp (T2 – Td) optimization of the compression work also leads to
Total work input for compression equal pressure ratio across the each stage, i.e.,
wC = wC1 + wC2 pi p
= 2 ...(27.20)
= Cp (Tc – T1) + Cp (T2 – Td) ...(27.16) p1 pi
ÊT ˆ ÊT ˆ The work input for the compression would be
= C p T1 Á c - 1˜ + C p Td Á 2 - 1˜
Ë T1 ¯ Ë Td ¯ reduced to a minimum, if compression is carried out
in several stages with perfect intercooling between
Gas Turbine Plant 947

stages, i.e., the compression process approaches to (iii) Reduction in work input due to mutlistage
an isothermal compression. The optimization of compression and intercooling, and
compression work leads to equal pressure ratios (iv) Isothermal compression work.
across each stage. Assumptions
(i) Each compressor and intercooler are analysed as
Example 27.13 Air is compressed from 100 kPa,
control volume at steady state.
300 K to 1000 kPa in a two-stage compressor with
intercooling between stages. The air is compressed to 300 (ii) There is no pressure drop for flow through the
kPa and is cooled back to 300 K in an intercooler before intercooler.
entering the second stage compressor. Each compression (iii) Working substance in compressor is air as an
stage is isentropic. For steady state operation and ideal gas with standard properties
negligible changes in kinetic and potential energy from Analysis
inlet to exit, determine (a) temperature at exit of second
(i) Temperature T2, at exit of second stage compres-
compressor, (b) the total compressor work input per unit
sor;
of mass flow, (c) reduction in work input by intercooling
and two-stage compression, (d) isothermal compression g -1
T2 Ê p ˆ g
work. = Á 2˜
Td Ë pd ¯
Solution 1.4 -1
Ê 1000 ˆ 1.4
or T2 = 300 ¥ Á
Given Air is compressed at steady state in a two-stage Ë 300 ˜¯
compressor with an intercooler
= 423 K = 150°C
p1 = 100 kPa T1 = 300 K
(ii) Total work input for compression per unit mass
pi = 300 kPa Td = 300 K flow. For steady state operation,
p2 = 1000 kPa wC = hc – h1 + h2 – hd
To find = Cp (Tc – T1) + Cp (T2 – Td)
(i) T2, temperature at exit of the second compressor,
(ii) Total compressor work,
Schematic with given data

p2 = 1000 kPa
2

Compressor Compressor wc
I II
Intercooler
c d
1 Td = 300 K
p1 = 100 kPa
T1 = 300 k
qR
(a) Schematic
p
2 2s
p2 = 1000 kPa

d c p1 = 300 kPa
T
1 =
30
0
K
1 p = 100 kPa
1
0 v
(b) p–v diagram
Fig. 27.23
948 Thermal Engineering

g -1 1.4 -1 To find To prove for minimum compression work.


Êp ˆ g Ê 300 ˆ 1.4
where Tc = T1 Á i ˜ = 300 ¥ Á
Ë 100 ˜¯
Schematic
Ë p1 ¯
p
= 410.6 K
2 p2 specified
Then wC = 1.005 ¥ (410.6 – 300)
+ 1.005 ¥ (423 – 300)

s=
C
= 111.17 + 123.61 c p variable
d i
= 234.76 kJ/kg

s=
T
(iii) Reduction in work input due to intercooler and

C
C
two stage compression; 1
p1 specified
Temperature T2s after single stage compression to
0 v
pressure p2 = 1000 kPa
g -1 Fig. 27.24
T2 s Êp ˆ g
= Á 2˜ Analysis Total compression work per kg for a two-
T1 Ë p1 ¯
stage compressor from pressure p1 to p2;
1.4 -1
wC = hc – h1 + h2 – hd
Ê 1000 ˆ 1.4
or T2s = 300 ¥ Á = 579.2 K
Ë 100 ¯˜
Since Cp is constant in cold air condition,
wC = Cp (Tc – T1) + Cp (T2 – Td)
Single-stage compression work For perfect intercooling Td = T1.
ws = h2s – h1 = Cp (T2s – T1) wC = Cp (Tc + T2 – 2T1)
= 1.005 ¥ (579.2 – 300)
ÈT T ˘
= 280.6 kJ/kg = Cp T1 Í c + 2 - 2˙
Reduction in work input for compression Î T1 T1 ˚
ws – wC = 280.6 – 234.76 = 45.84 kJ/kg For isentropic compressions
g -1 g -1
(iv) Isothermal work input for compression;
Tc Êp ˆ g T2 T2 Ê p2 ˆ g
Êp ˆ Êp ˆ = Á i˜ and = =
wiso = p1v1 ln Á 2 ˜ = RT1 ln Á 2 ˜ T1 Ë p1 ¯ Td T1 ÁË pi ˜¯
Ë p1 ¯ Ë p1 ¯
g -1
Ê 1000 ˆ Using k =
= (0.287 ¥ 300) ¥ ln Á
Ë 100 ˜¯ g
ÈÊ p ˆ k Ê p ˆ k ˘
= 198.25 kJ/kg
we get wC = Cp T1 ÍÁ i ˜ + Á 2 ˜ - 2˙
ÍÎË p1 ¯ Ë pi ¯ ˙˚
Example 27.14 For a two-stage compressor with
For specified values of T1, T2, p1, p2 and constant Cp,
perfect intercooling operating at steady state, prove
the compressor work would be minimum, when
that the minimum total work input is required when
the pressure ratio is same for each stage. Use cold air- dwC
=0
standard analysis, assuming that compression process is dpi
isentropic, no pressure drop through the intercooler and Ï ÈÊ k k ˘ ¸Ô
d Ô pi ˆ Ê p2 ˆ
temperature to each compressor stage is same. Kinetic or ÌC pT1 ÍÁ ˜ + Á ˜ - 2˙ ˝ = 0
dpi Ô ÍÎË p1 ¯ Ë pi ¯ ˙˚ Ô
and potential energy effects are negligble. Ó ˛
Ï Ê 1ˆ
k ¸
Solution Ô Ô
or C p T1 Ìk ( pi ) k -1 Á ˜ - k ( p2 ) k ( pi ) - k -1 ˝ = 0
ÔÓ Ë p1 ¯ Ô˛
Given Two-stage isentropic compression with perfect
intercooling operates at steady state. k
Ê p2 ˆ
k
Ê pi ˆ 1 1
pi
= 2
p or ÁË p ˜¯ ¥ p = ÁË p ˜¯ ¥ p
1 i i i
p1 pi
Gas Turbine Plant 949

g -1 g -1 The isentropic efficiency of the compressor is given


Ê pi ˆ g Êp ˆ g by
or ÁË p ˜¯ = Á 2˜
1 Ë pi ¯ Isentropic work T2 s - T1
hC = =
pi p Actual work T2 - T1
or = 2
p1 pi Ê g -1 ˆ
T2 s - T1 2C p
It is the desired relation for minimum compression \ w C = 2 Cp = Tmin ÁË rp 2g - 1˜¯
hC hC
work input.
The temperature T6s after isentropic expansion in the
Example 27.15 In a gas turbine plant, compression turbine
is carried out in two stages with perfect intercooling T5 Tmax
T6s = g -1
= g -1
and expansion in a one-stage turbine. If maximum
temperature Tmax and minimum temperature Tmin in the g g
( rp ) ( rp )
cycle remains constant, show that for maximum specific Using isentropic efficiency of the turbine
power output of the plant, the optimum overall pressure
ratio is given by T5 - T6
hT =
2g T5 - T6 s
Ê Tmax ˆ 3(g -1)
ropt = Á hT hC Turbine work output per kg of air
Ë Tmin ˜¯
wT = Cp (T5 – T6)
Solution Ê 1 ˆ
Temperature T2s after first stage isentropic compression = C phT (T5 - T6 s ) = C phT Tmax Á1 - g -1 ˜
g -1 Á ˜
Ë rp g ¯
g
T2s = T1( rp1 )
Net work done per kg of air
For perfect intercooling, wnet = wT – wC
1
Ê g -1 ˆ
rp1 = ( rp ) 2 Ê 1 ˆ 2C pTmin Á 2g
= C phT Tmax 1 -
Á g -1 ˜
- Ë rp - 1˜¯
hC
g -1 Á ˜
2g Ë rp g ¯
\ T2s = Tmin ( rp )
For maximum work output
The actual work input to compressor win in two stages dwnet
can also be expressed as =0
drp
wC = 2 Cp (T2 – T1)
Ê Ê g -1ˆ ˆ
Ê g - 1ˆ Á - ÁË g ˜¯ -1˜
- C phT Tmax Á - Ë rp ¯
Ë g ¯˜

Ê g -1 - 1 ˆ
2C pTmin Ê g - 1ˆ
- Á 2g ˜¯ = 0
h ÁË 2g ˜¯ Ë rp
C

Ê - Ê g -1ˆ - 1 ˆ Ê g -1 ˆ
Á ÁË g ˜¯ ˜ Tmin Á 2g - 1˜
or hT Tmax Ë rp ¯- Ë rp ¯ =0
hC
3(g -1)
T 2g
or hT hC max = rp =0
Tmin
Fig. 27.25 T–s diagram
950 Thermal Engineering

Hence optimum pressure ratio is Solution


2g
È T ˘ 3( g - 1) Given A gas turbine power plant with two-stage
ropt = ÍhT hC max ˙
Î Tmin ˚ compression and regeneration
p1 = 1 bar T1 = 300 K
Example 27.16 The air supplied to a gas turbine rp = 6 hC = 0.8
plant is 10 kg/s. The pressure ratio is 6 and pressure hT = 0.85 T6 = 1073 K
at the inlet of compressor is 1 bar. The compressor is e = 0.7 ma = 10 kg/s
two-stage and is provided with perfect intercooling. The
Cp = 1.005 kJ/kg ◊ K
inlet temperature is 300 K and maximum temperature is
limited to 1073 K. Take the following data: To find Thermal efficiency of the cycle.
Isentropic efficiency of compressor at each stage = Analysis Analysing each device in a steady flow
80% manner.
Isentropic efficiency of turbine = 85%
A regenerator is included in a plant whose effectiveness Compressors For perfect intercooling, the pressure
is 0.7. Neglecting the mass of fuel, determine the thermal ratio of a stage
efficiency of the plant. rp1 = p1 p3 = 1 ¥ 6 = 2.45
Take Cp for air as 1.005 kJ/kg ◊ K.

Schematic
Regenerator
8

5
Intercooler
e = 0.7
3 7

4 4 bar Combustion 1073 K


chamber
2 6

Compressor Turbine Load


HP turbine
hdrive = 0.90
Wnet

hC = 0.8 hT = 0.85
1 1 bar,
300 K
(a)

T
6
1073 K

r
ba
=6
p4
5
4 2 p2 7
4s 2s
7s
8
b ar
300 K =1
3 1 p1

0 s
(b) T–s diagram
Fig. 27.26
Gas Turbine Plant 951

Temperature T2s after isentropic compression Turbine work output per kg of air
g -1 1.4 -1 wout = Cp (T6 – T7)
T2s = T1( rp1 ) g
= 300 ¥ ( 2.45) 1.4 = 1.005 ¥ (1073 – 707.57)
= 367.25 kJ/kg
= 387 .51 K
Net work output per kg of the plant
The isentropic efficiency of LP compressor is given
by wnet = wout – win = 367.25 – 219.87
= 143.38 kJ/kg of air
Isentropic work T2 s - T1
hC, LP = =
Actual work T2 - T1 Heat exchanger
387.51 - 300 The effectiveness of a heat exchanger is given by
or T2 = 300 + = 409.4 K
0.8 T5 - T4
e =
The actual work input to LP compressor T7 - T4
win, LP = Cp (T2 – T1) Given that e = 0.7,
= 1.005 ¥ (409.4 – 300) = 109.93 kJ/kg
T5 - 409.4
For perfect intercooling, \ 0.7 =
707.57 - 409.4
p2 p4
T3 = T1 ; rp1 = = and T4 = T2 Thus, the temperature T5 of air leaving a heat
p1 p3
exchanger
\ win, LP = win, HP
or T5 = 409.4 + 0.7 ¥ (707.57 – 409.4)
Therefore, the total work input of two stages
= 618.11 K
win @ 2 win, LP = 219.87 kJ/kg
The heat supplied to air
Turbine The temperature T7s after isentropic expansion qin = Cp (T6 – T5) = 1.005 ¥ (1073 – 618.11)
from a pressure of 6 bar to 1 bar = 457.15 kJ/kg
T6 1073 Thermal efficiency of the cycle
T7s = g -1
= 1.4 -1
= 643 K
Ê p4 ˆ g Ê 6 ˆ 1.4 hth =
wnet 143.38
= = 0.3136 or 31.36%
ÁË p ˜¯ ÁË 1 ˜¯ qin 457.15
1
The isentropic efficiency of the turbine is given as
Actual work output T -T
hT = = 6 7
Isentropic work output T6 - T7 s
Actual temperature after expansion in turbine, When reheat and regeneration are used together, the
T7 = 1073 – 0.85 ¥ (1073 – 643) thermal efficiency can be increased significantly.
= 707.57 K Figure 27.27 shows a schematic atrrangement of a

Fig. 27.27
952 Thermal Engineering

gas turbine plant with reheat and regeneration, and Example 27.17 In a gas turbine plant of 6 MW
Fig. 27.28 shows the corresponding T–s diagram. capacity, air enters the compressor at 100 kPa, 300 K
In this plant, the heat supply is reduced by the and is compressed to a pressure of 600 kPa in one stage.
amount of heat recovered in the regenerator. The temperature at the inlet to first turbine is 1000 K.
The expansion takes place in two stages with reheat to
T p2 1000 K between the two stages . The isentropic efficiency
4 6 of the compressor is 80% and that of both turbines is
Tmax
85%. A regenerator having an effectiveness of 0.72 is
5
also incorporated in the cycle to heat the compressed air
3 before entering into combustion chamber. The calorific
7
ve
d value of fuel is 18500 kJ/kg. Determine the following:
2 sa
=q 8 (a) A/F ratio entering the first turbine,
q reg
p1 (b) Thermal efficiency of the cycle,
T1
1 (c) Air supply to plant,
s
0 (d) Fuel consumption of plant per hour.
Fig. 27.28 Take for air Cp = 1.0 kJ/kg ◊ K, g = 1.4 and
cycle for gases, Cp = 1.15 kJ/kg ◊ K, g = 1.34

It can be analytically proved that the maximum Solution


net work output from the plant would be for
Given A gas turbine plant with reheating and
rp1 = rp ...(27.21) regeneration
p1 = 100 kPa T1 = 300 K
p p p2 = 600 kPa T4 = 1000 K
where rp1 = 2 = 5 , intermediate pressure ratio,
p5 p1 T6 = 1000 K hC = 0.8
and
hT, HP = hT, LP = 0.85 e = 0.72
p2 CV = 18500 kJ/kg
rp = , total pressure ratio. and P = 6 MW = 6000 kW
p1
For air Cpa = 1.0 kJ/kg ◊ K ga = 1.4 and
For a cycle with isentropic compression and
For gases Cpg = 1.15 kJ/kg ◊ K gg = 1.34
isentropic expansion, ideal regeneration and best
division of pressure between stages for reheat, the To find
net work done and heat supplied are calculated as; (i) A/F ratio entering the first turbine,
wnet = 2 Cp (T4 – T5) – Cp (T2 – T1) (ii) Thermal efficiency of the cycle,
...(27.22) (iii) Air supply to plant, and
Heat supplied per kg of air in a cycle (iv) Fuel consumption of plant per hour.
qin = qcomb + qreheat Analysis Analysing each device in a steady flow
= Cp (T4 – T3) + Cp (T6 – T5) ...(27.23) manner.
Thermal efficiency of reheat cycle can be Compressors The temperature after isentropic com-
expressed as pression 1–2:
wnet g -1 1.4 -1
hreheat = Êp ˆ g Ê 600 ˆ 1.4
qin T2s = T1 Á 2 ˜ = 300 ¥ Á
Ë p1 ¯ Ë 100 ˜¯
2C p (T4 - T5 ) - C p (T2 - T1 )
= ...(27.24) = 500.55 K
C p (T4 - T3 ) + C p (T6 - T5 )
Gas Turbine Plant 953

Schematic and T–s diagram


Regenerator
8

e = 0.72 3 Reheater
7
2 600 kPa Combustion 1000 K 5 1000 K
6
chamber 4

HP LP
Compressor Load
turbine turbine
hC = 0.8
hHP = 0.85 hLP = 0.85
1 100 kPa
300 K

T
p2

4 6
1000 K

7
3 5s 5 7
7s
2
2s 8
p1
300 K
1
s
0

Fig. 27.29
The isentropic efficiency of the compressor is given Using isentropic efficiency of the turbine
by T4 - T5
Isentropic work T2 s - T1 hT, HP =
hC = = T4 - T5 s
Actual work T2 - T1
Actual temperature after expansion in HP turbine,
500.55 - 300
or T2 = 300 + = 550.68 K T5 = 1000 – 0.85 ¥ (1000 – 796.7)
0.8
= 827.2 K
The actual work input to compressor win can also be
Since hT, HP = hT, LP
expressed as
\ T5 = T7 = 827.2 K
win = Cp (T2 – T1)
= 1.0 ¥ (550.68 – 300) = 250.68 kJ/kg Heat exchanger
The effectiveness of a heat exchanger is given by
Turbines For perfect pressure division, the intermediate
pressure between stages T3 - T2
e =
T7 - T2
p5 = p1 p2 = 100 ¥ 600 = 245 kPa
Given that e = 0.72,
The temperature T5s after isentropic expansion in the
HP turbine T3 - 550.68
\ 0.72 =
827.2 - 550.68
T4 1000
T5s = g g -1
= 1.34 -1
= 796.7 K Thus the temperature T5 of air leaving heat exchanger
Ê p2 ˆ gg Ê 600 ˆ 1.34 or T3 = 550.68 + 0.72 ¥ (827.2 – 550.68)
ÁË p ˜¯ ÁË 245 ˜¯
3 = 749.7 K
954 Thermal Engineering

(i) A/F ratio supplied to HP turbine (iv) Fuel consumption of plant per hour
Let mf1 = mass of fuel supplied per kg of air to m f = ma (mf1 + mf2) ¥ 3600
main combustion chamber, then energy balance = 38.94 ¥ (0.0137 + 0.011) ¥ 3600
reveals that = 3462.62 kg/h
mf1 CV = (1 + mf1) Cpg (T4 – T3)
or mf1 ¥ 18500 = (1 + mf1) ¥ 1.15 Example 27.18 In a gas turbine plant, the air from
¥ (1000 – 749.7) the compressor passes through a heat exchanger, heated
18500 mf1 = 250.3 + 250.3 mf1 by exhaust gases coming from a low-pressure turbine. The
air then enters the high-pressure combustion chamber.
It gives mf1 = 0.0137 kg/kg of air
The high-pressure turbine drives the compressor only.
1 The exhaust gases coming out of the high-pressure turbine
Air fuel ratio A/F = = 72.9
0.0137 is heated into a low-pressure combustion chamber and
(ii) Thermal efficiency of a cycle then enters the low-pressure turbine, which is coupled to
Considering mf2 is additional fuel added in a external load. The following data refers to the plant:
reheater then Pressure ratio in the compressor, 4:1
mf2 CV = (1 + mf1 + mf2) Cpg (T6 – T5) Isentropic efficiency of the compressor, 0.85
or mf2 ¥ 18500 = (1 + mf1 + mf2) ¥ 1.15 Isentropic efficiency of high-pressure turbine, 0.84
¥ (1000 – 827.2) Isentropic efficiency of low-pressure turbine, 0.8
18500 mf2 = 198.72 + 198.72 Effectiveness of heat exchanger, 0.75
¥ 0.0137 + 198.72 mf2 Mechanical efficiency of the drive to the compressor,
It gives mf2 = 0.011 kg/kg of air 0.90
Total turbine work output per kg of air Temperature of gases entering HP turbine, 660°C
wout = wHP + wLP Temperature of gases entering LP turbine, 625°C
= (1 + mf1) Cpg (T4 – T5) Atmospheric pressure and temperature are 1 bar and
17°C.
+ (1 + mf1 + mf2) Cpg (T6 – T7)
Assume specific heat of gas and air as 1.005 kJ/kg ◊ K
= (1 + 0.0137) ¥ 1.15
and g = 1.4.
¥ (1000 – 827.2) + (1 + 0.0137
Calculate the pressure of gases entering the low-
+ 0.011) ¥ 1.15 ¥ (1000 – 827.2)
pressure turbine and overall efficiency of the plant.
= 201.14 + 203.62 = 404.76 kJ/kg
Net work output per kg of the plant Solution
wnet = wout – win = 404.76 – 250.68
Given A gas turbine plant with reheat and regeneration
= 154.08 kJ/kg of air
p1 = 1 bar T1 = 17°C = 290 K
The heat supplied to air
p2 = 4 kPa T4 = 660°C = 933 K
qin = (mf1 + mf2) CV
T6 = 625°C = 898 K e = 0.75
= (0.0137 + 0.011) ¥ 18500
hC = 0.85 hHP = 0.84
= 456.95 kJ/kg of air
hLP = 0.8 hdrive = 0.9
Thermal efficiency of the cycle
Cp = 1.005 kJ/kg g = 1.4
w 154.08
hth = net =
qin 456.95 To find
= 0.337 or 33.7% (i) Pressure of gas entering LP turbine, and
(iii) Air supplied to the plant (ii) Thermal efficiency of the plant.
Power develoved by plant is given by Assumptions
P = ma wnet (i) Each component in the cycle is analysed as
or 6000 = 154.08 ma control volume at steady state.
or ma = 38.94 kg/s
Gas Turbine Plant 955

Schematic with given data


Regenerator
8

e = 0.75 3 Reheater
7
o
660 C
2 4 bar Combustion 5 625 C
o
6
chamber 4

HP LP
Compressor Load
turbine turbine
hdrive = 0.90
hC = 0.85 Wnet
hHP = 0.84 hLP = 0.8
1 1 bar,
15oC
(a) Schematic

T p2

4
660oC
6 o
625 C

3 5s 5
7
2 7s
2s 8
p1
290 K
1 s
0
(b) T-s diagram
Fig. 27.30
(ii) No pressure drop for flow through the heat win = h2 – h1 = Cp (T2 – T1)
exchangers. = 1.005 ¥ (455.81 – 290)
(iii) Kinetic and potential energy effects are negligible. = 166.64 kJ/kg
Analysis The temperature after isentropic compression Turbines
1–2: Work input to compressor
g -1 1.4 -1
= hdrive ¥ Work done by HP turbine
Êp ˆ g Ê 4 ˆ 1.4
T2s = T1 Á 2 ˜ = 290 ¥ Á ˜ win = hdrive ¥ Cp (T4 – T5)
Ë p1 ¯ Ë 1¯
or 166.64 = 0.9 ¥ 1.005 ¥ (933 – T5)
= 430.94 K
166.94
The isentropic efficiency of the compressor is given or T5 = 933 - = 748.43 K
by 0.9 ¥ 1.005

Isentropic work T2 s - T1 For HP turbine, the isentropic efficiency is given as


hC = =
Actual work T2 - T1 Actual work output T -T
hHP = = 4 5
430.94 - 290 Isentropic work output T4 - T5 s
or T2 = 290 + = 455.81 K
0.85
933 - 748.43
The actual work input to compressor win can also be or 0.84 =
933 - T5 s
expressed as
or T5s = 736.65 K
956 Thermal Engineering

(i) Pressure of gas entering LP turbine ¥ (898 – 748.43)


The pressure ratio of HP turbine can be expressed = 432.72 kJ/kg
as (ii) Thermal efficiency of the cycle
g 1.4 wnet 106.49
p4 Ê T ˆ g -1 Ê 933 ˆ 1.4 -1 hth = =
= Á 4˜ = Á qin 432.72
p5 Ë T5 s ¯ Ë 736.65 ¯˜
= 0.2460 or 24.6%
= 2.286
4 bar
or p5 = = 1.75 bar
2.286
For LP turbine, the temperature ratio can be
expressed as As discussed earlier, the turbine work increases by
g -1 1.4 - 1 reheating between turbine stages and compression
T6 Êp ˆ g Ê 1.75 ˆ 1.4 work reduces by intercooling between compressor
= Á 6˜ =Á = 1.173
T7 s Ë p7 ¯ Ë 1 ˜¯ stages. Further, exhaust gases leaving the turbine
at relatively high temperature thus the potential for
898 K
or T7s = = 765.55 K regeneration is also enhanced. Accordingly, when
1.173
reheating and intercooling are used together with
The isentropic efficiency of LP turbine is given as regeneration, a substancial improvement in the
Actual work output T -T performance can be realised. Fig. 27.31 shows a
hT, LP = = 6 7
Isentropic work output T6 - T7 s schematic of physical arrangement incorporating
Actual temperature, T7 after expansion in turbine, intercooling, reheating and regeneration.
T7 = 898 – 0.80 ¥ (898 – 765.55) The air enters the first stage compressor at state
= 792.04 K 1, it is compressed isentropically to an intermediate
Net work output per kg of air pressure p2, then it enters the intercooler, where
= Output of LP turbine it cools to state 3 (T3 = T1). Then air is again
wnet = h6 – h7 = Cp (T6 – T7) compressed isentropically to state p4 in second
= 1.005 ¥ (898 – 792.04) stage compressor. At this state, the air enters the
= 106.49 kJ/kg regenerator, where it is heated at constant pressure
to the state 5. In an ideal case, the air leaving
Heat exchanger the regenerator at a temperature equal to turbine
The effectiveness of a heat exchanger is given by exhaust temperature T9. Then heat is added to air
T3 - T2 in combustion chamber and it leaves at state 6
e =
T7 - T2 to enter the first stage turbine, where it expands
Given that e = 0.75, isentropically to state 7, and the gases enter the
reheater, and is heated at constant pressure to state
T3 - 455.81
0.75 = 8 and then enter the second stage turbine to expand
792.04 - 455.81
isentropically to the state 9. Finally, exhausted
Thus the temperature T3 of air leaving heat gases at T9 enter the regenerator to preheat the
exchanger
compressed air at constant pressure, and leaves the
T3 = 455.81 + 0.75 ¥ (792.04 – 455.81)
regenerator at state 10.
= 708 K
Fig. 27.32(a) and (b) show p-v diagram and T–s
The heat supplied to air
diagrams respectively for an ideal gas turbine cycle
qin = Cp (T4 – T3) + Cp (T6 – T5)
with intercooling, reheating and regeneration, as
= 1.005 ¥ (898 – 708) + 1.005
arranged in Fig. 27.31.
Gas Turbine Plant 957

Fig. 27.31 A gas-turbine engine with two-stage compression with intercooling, two stage expansion with reheating,
and regeneration.

It is shown with the help of Examples 27.13, p2 p p p


and 27.14 that the work input to the two stages = 4 and 6 = 8
p1 p3 p7 p9
compression is minimum, when equal pressure
ratio is maintained across each stage. Similarly, it In analysis of an actual gas turbine cycles, the
can also be proved that the turbine work would be irreversibilities are present within the compressor,
maximum, if same pressure is maintained between turbine, and the regenerator as well as pressure drop
two turbine stages. Thus, for optimum operation in heat exchangers. The T–s diagram for an actual
gas turbine cycle with intercooler, reheating and
p regeneration is shown in Fig. 27.33. In this figure
5 6
4 pressure drop in heat exchanger is not considered.
7 If the number of compression stages with
8
3 2 intercoolers and number of expansion stages with
reheaters are increased, then the compression and
9
expansion will occur almost isothermally and ideal
1 10
0 v gas turbine cycle will approach the Ericsson cycle
(a) p–v diagram for an ideal gas turbine cycle with as shown in Fig 27.34, and thermal efficiency of
intercooling, reheating and regeneration the cycle will reach theoretically to a maximum
T 6 8
qin T 6
8
5 9
7 7
qreg 5 9

2 qreg = qsaved 4
4
10
2 10
qout
3 1 3
0 s 1
(b) T–s diagram for an ideal gas turbine cycle with 0 s
intercooling, reheating and regeneration
Fig. 27.33 Actual regenerative gas turbine cycle with
Fig. 27.32 intercooling and reheating
958 Thermal Engineering

value (the Carnot efficiency). However, the cost of (v) No pressure drop in heat exchanges piping system
multiple intercooler, reheaters are likely to exceed etc.
the cost of fuel saving, thus use of more than two or (vi) Perfect intercooling.
three stages cannot be justified economically. T–s diagram
T
T

6 8
1300 K
nst. qreheat
co qcomb
=
p
t. 5
ns 9
co 7
=
p

0 4 2
s 10
qout
Fig. 27.34 A gas turbine cycle with large number 300 K
3 qcooling 1
compression and expansion stages with s
intercooling, reheating and regeneration
approaches the Ericsson cycle Fig. 27.35
two stage compression with intercooling,
two stage expansion with reheating, and
Example 27.19 An ideal gas turbine cycle with two
regeneration
stages of compression and two stage of expansion has an
overall pressure ratio of 8. Air enters the each stage of
compressor at 300 K and each stage of turbine at 1300 K. Analysis For perfect intercooling and given overall
Determine the back work ratio and thermal efficiency of pressure ratio
the gas turbine cycle. p p
rp = 2 = 4 = 8 = 2.83
Assuming an ideal generator with 100 per cent p1 p3
effectiveness. p6 p8
and = = 2.83
p7 p9
Solution
For an ideal case,
Given An ideal gas turbine cycle with two stage of At inlet T1 = T3 \ h1 = h3
compression, expansion: and T6 = T8 \ h6 = h8
p2 At exit T2 = T4 \ h2 = h4
=8
p1 and T5 = T7 = T9 \ h5 = h7 = h9
T1 = T3 = Tmin = 300 K Under these conditions, the work input of each
compressor and work output of each turbine would be
T6 = T8 = Tmax = 1300 K
the same.
To find The temperature after first-stage and second-stage
(i) Back work ratio, compression.
(ii) Thermal efficiency of the cycle. g -1
g
T4 = T2 = T1( rp ) = 300 ¥ (2.83)0.286
Assumptions
= 403.95 K
(i) Each component in the cycle as steady flow
model. Temperature after first-stage and second-stage
expansion,
(ii) Constant specific heat. g -1
(iii) Working substance as an ideal gas. T8 T g
= 6 = ( rp )
(iv) Compression and expansion are isentropic. T9 T7
Gas Turbine Plant 959

1300 Heat supplied


or T9 = T7 = 1.4 -1
= 965.6 K
qin = qcomb + qreheat = (h6 – h5) + (h8 – h7)
( 2.83) 1.4 = Cp (T6 – T5) + Cp (T8 – T7)
Total compression work = 2 Cp (T8 – T7)
fi wC = 2 (h2 – h1) = 2 Cp (T2 – T1) = 2 ¥ 1.005 ¥ (1300 – 965.6)
= 2 ¥ 1.005 ¥ (403.95 – 300) = 672.14 kJ/kg
= 208.94 kJ/kg (i) Back work ratio
Total expansion work
wC 208.94
= 2wT = 2 Cp (T6 – T7) bwr = = = 0.31 = 31%
wT 672.12
= 2 ¥ 1.005 ¥ (1300 – 965.6)
= 672.12 kJ/kg (ii) Thermal efficiency
wnet = wT – wC = 672.12 – 208.94 wnet 463.18
hth = =
= 463.18 kJ/kg qin 672.14
= 0.6891 or 68.91%

Summary
Actual work output h -h
engine. A simple gas turbine can be modelled and hT = = 3 4
Isentropic work output h3 - h4 s
as closed cycle, which operates on the Brayton
cycle. where states 1 and 3 are inlet states, 2 and 4 are
actual exit states and 2s and 4s are isentropic exit
states from compressor and turbine, respectively.
given as
Compressor work
bwr = heating of compressed air leaving the compressor
Turbine work
Net work with help of hot exhaust gases leaving the turbine
and work ratio rw = in a counterflow heat exchanger. The effectiveness
Turbine work
of the regenerator is expressed as
on ideal Brayton cycle can be obtained when the qsaved h -h
e = = 5 2
pressure ratio qreg,max h4 - h2
g
Ê T ˆ 2 (g -1)
rp = Á 3 ˜ can also be increased by multistage compression
ËT ¯1
with intercooling, regeneration and multistage
expansion with reheating.
deviate from isentropic ones, and their isentropic
efficiencies are expressed as The gas turbine engines are widely used to propel
the aircraft, because they are light, compact and
Isentropic work input h2 s - h1 have a high power-to-weight ratio
hC = =
Actual work input h2 - h1
960 Thermal Engineering

Glossary
Gas turbine An internal combustion rotary engine Reheating Heating of gases after one stage expansion
using combustion gases as working fluid in turbine by burning of additional fuel
Regenerator A counterflow heat exhanger, which Intercooling Process of cooling of compressed air
transfers heat from hot gases to air between stages of compression
Regeneration Process of heat transfer from hot gases Back work The work input to compressor is called
to air back work

Review Questions
1. How is a gas turbine plant modeled as air standard 7. What are the different methods used to improve
Brayton cycle? Write the assumptions clearly. efficiency of a gas turbine plant? Expain any one
2. What are the deviations in actual gas turbine method with a neat sketch.
cycle as compared with air standard cycle? 8. Describe with a neat sketch, the working of a
3. Define isentropic efficiency of a compressor and constant-pressure combustion gas turbine cycle.
turbine. 9. Sketch the neat diagram of regenerative gas
4. Define isothermal efficiency of the compressor turbine plant and deduce an expression for its
and prove that the isothermal work input to a thermal efficiency.
compressor is always minimum. 10. What are advantages of cooling of compressed air
5. What are the applications of the gas turbine between stages? State.
plants? 11. Explain the working of reheat gas turbine plant
6. Write the merits and demerits of gas turbine plant with the help of a T–s diagram.
over internal combustion engines.

Problems
1. In a gas turbine power plant, operating on Joule 3. Air enters a gas turbine plant at 95 kPa, 5°C.
cycle, air is compressed from 1 bar and 15°C The compression is adiabatic with an efficiency
through a pressure ratio of 6. It is then heated to of 70% and pressure ratio of 5. The regenerative
727°C in the combustion chamber and expanded effectiveness is 60%. The turbine inlet conditions
back to a pressure of 1 bar. Calculate the net work are 475 kPa, 850°C. The expansion in the turbine
done, cycle efficiency and work ratio. Assume is also adiabatic with an efficiency of 70%. The
isentropic efficiencies of turbine and compressor power output of the plant is 1500 kW. Calculate
are 90 and 85%, respectively. (a) mass flow rate of air through the plant.
[134.8 kJ/kg, 27.6%, 0.372] (b) the irreversibility (kJ/kg) or the turbine
2. Air enters the compressor of a simple gas turbine expansion
plant at 100 kPa, 20°C, at a rate of 2.2 kg/s. The [(a) 22.9 kg/s, (b) 44.4 kJ/kg]
compressor efficiency is 60%. The discharge 4. A gas turbine takes in air at 27°C and 1 bar.The
pressure of the compressor is 450 kPa. Calculate pressure ratio is 4. The maximum temperature
the amount of heat (kJ/kg) that must be added to of the cycle is 560°C. The efficiency of the
provide a turbine inlet temperature of 650°C. compressor and turbine is 0.83 and 0.85,
[402 kJ/kg]
Gas Turbine Plant 961

respectively. Find the overall efficiency, if the turbine are 82 and 85%, respectively. Calculate
regenerator effectiveness is 0.75. [21.24%] the power input in kW to an electric generator
5. A gas turbine unit receives air at 1 bar and 300 K geared to the turbine, with transmission efficiency
and compresses it adiabatically to 6.2 bar. The of 95%. The air enters the compressor at 15°C at
compressor efficiency is 88%. The fuel has a a rate of 15 kg/s. Take Cp = 1.005 kJ/kg ◊ K and
heating value of 44,186 kJ/kg and the fuel-air g = 1.4 for compression process and Cp = 1.11
ratio is 0.017 kg of fuel per kg of air. The turbine kJ/kg ◊ K and g = 1.333 for the expansion process.
internal efficiency is 90%. Calculate the work of [875 kW]
turbine and compressor per kg of air and thermal 9. A gas turbine plant has an overall pressure ratio
efficiency of cycle. of 5 and a maximum temperature of 550°C. The
Take for air Cp = 1.005 kJ/kg ◊ K and g = 1.4, and turbine drives the compressor and an electric
For product of combustion Cp = 1.147 kJ/kg ◊ K alternator, with transmission efficiency of 97%.
and g = 1.333. The ambient temperature is 20°C and isentropic
[43.92 kJ/kg, 234.5 kJ/kg, 33.21%] efficiency of the compressor and turbine are 80
6. A gas turbine is supplied with a gas at 5 bar and 83%, respectively. Calculate the power input
and 1000 K and expands isentropically to 1 bar. to alternator for an air flow rate of 15 kg/s. Also,
The mean specific heat at constant pressure calculate the thermal efficiency and work ratio.
and constant volume are 1.0425 and 0.7662 kJ/ [655 kW, 12%, 0.168]
kg ◊ K, respectively. Calculate the specific power 10. A gas turbine plant has a heat exchanger with
developed in kJ/kg of gas and exhaust gas 72% effectiveness. The turbine operates between
temperature. [362 kW/kg, 653 K] pressures of 1.01 and 4.04 bar and the ambient
7. Calculate the efficiency of a gas turbine plant temperature is 20°C. Isentropic efficiencies of
fitted with a heat exchanger of 75% effectiveness. the compressor and turbine are 80 and 85%,
The pressure ratio is 4:1 and the compression is respectively. The pressure drop on each side
carried out in two stages of equal pressure ratio of the heat exchanger is 0.05 bar and in the
with intercooling back to initial temperature combustion chamber, 0.14 bar. Calorific value of
of 290 K. The maximum temperature is 925 K. the fuel is 41800 kJ/kg. Calculate the percentage
The turbine isentropic efficiency is 88% and the increase in efficiency of the plant due to use of
isentropic efficiency of each compressor is 85%. heat exchanger in comparison to a simple plant.
For air, g = 1.4 and Cp = 1.005 kJ/kg ◊ K. Air–fuel ratio used in a simple cycle is 90 and the
[32.8%] turbine entry temperature remains same in both
8. A gas turbine has a pressure ratio of 6 and a cases.
maximum cycle temperature of 600°C. The Take Cp = 1.024 kJ/kg ◊ K and g = 1.4.
isentropic efficiency of the compressor and [5.3%]

Objective Questions
1. A gas turbine plant works on (c) constant temperature
(a) Otto cycle (b) Diesel cycle (d) constant entropy
(c) Dual cycle (d) Brayton cycle 3. A simple gas turbine unit consists of the devices
2. In an ideal Brayton cycle, the heat is added at in the following order
(a) constant volume (a) Air compressor, gas turbine, combustion
(b) constant pressure chamber and generator
962 Thermal Engineering

(b) Air compressor, combustion chamber, (a) to heat the compressed air from the
turbine and generator compressor
(c) combustion chamber, gas compressor, (b) to heat the gas before inlet to gas turbine
turbine and generator (c) to exchange the heat from hot gases from
(d) Gas turbine, generator, compressor and combustion chamber to exhaust gases of the
combustion chamber turbine
4. In a gas turbine plant, the intercooler is used in (d) to heat the compressed air in between the
between stages
(a) air compressor and regenerator 7. The compressor isentropic efficiency is defined
(b) air compressor and combustion chamber an
(c) combustion chamber and turbine Actual work Actual work
(a) (b)
(d) LP compressor and HP compressor Isothermal work Isentropic work
5. The function of an inter cooler in a gas turbine
plant is Isothermal work Isentropic work
(c) (d)
(a) to cool the exhaust gas from the turbine Actual work Actual work
(b) to cool compressed air from compressor 8. Turbine isentropic efficiency is defined as
(c) to cool atmospheric air before inlet to
Actual work Actual work
compressor (a) (b)
Isothermal work Isentropic work
(d) to cool the compressed air in between the
stages Isothermal work Isentropic work
(c) (d)
6. The function of regenerator in a gas turbine plant Actual work Actual work
is

8. (b) 7. (d) 6. (a) 5. (d) 4. (d) 3. (b) 2. (b) 1. (d)


Answers
Jet and Rocket Propulsions 963

28
Jet and Rocket Propulsions

Introduction
Jet and rocket engines are space vehicles. They are propelled by the reaction of a backward streaming jet
of fluid. The jet engines are used in aircraft, while rocket engines are used in satellites. Satellites revolve
around the earth at an altitude where the drag force is absent. Therefore, such satellites remain in orbit
forever without any expenditure of energy.
The principle, construction, working and performance of jet engines are explained in the first part of
the chapter. The operation of ram jet and pulse jet engines is integrated in brief. Rocket Propulsion and
Propellants are also discussed at the end of this chapter.

JET PROPULSION the aircraft in accordance with Newton’s second


and third laws of motion.
Jet engines are usually used as aircraft engines for Whenever, the momentum is applied to a mass of
jet aircraft. They are also used for cruise missiles fluid, a reaction occurs and it gives propulsive force
and unmanned air vehicles. Jet engines have also in the opposite direction.
been used to propel high-speed cars, particularly In a jet propulsion engine, the propulsive power
drag racers, with the all-time record held by a is not produced by the gas turbine; instead it drives
rocket car. only the compressor and auxiliary equipment.
Jet engine designs are frequently modified to Thus, the net work output of a jet propulsion cycle
turn them into gas turbine engines which are used is zero. The exhaust gases that exist in the turbine
in a wide variety of industrial applications. These at relatively higher pressure are subsequently
include electrical power generation, powering accelerated in the nozzle and then are discharged
water, natural gas, or oil pumps, and providing to the surroundings to produce the thrust to propel
propulsion for ships and locomotives. the aircraft as shown in Fig. 28.1. A propeller
discharges the gases at a very high velocity in a
Jet Propulsion large mass of atmospheric air. Since the large mass
A jet propulsion engine is a reaction engine that of atmospheric air cannot be displaced by these
discharges a fast-moving jet of fluid to generate gases, thus a reactive force is produced, which
thrust in the opposite direction of the jet to propell propels the aircraft forward.
964 Thermal Engineering

28.3 RAMMING EFFECT PROPULSION


These are classified as SYSTEMS
1. Non-air-breathing jet engines called rockets.
2. Air-breathing jet engines operating with
A ramjet engine, is a form of jet engine using the
(a) Reciprocating IC engine
engine’s forward motion to compress incoming air,
(b) Ramming effect engines without a rotary compressor as shown in Fig. 28.3.
(c) Gas turbine engines Therefore, ramjet requires forward motion through
A rocket engine carries its own oxidizer for the air to produce thrust.
combustion of fuel and is therefore, independent
Fuel injection
of atmospheric air and altitude. Its specific fuel Inlet
Nozzle
(M>1) Flame holder (M = 1)
consumption is the maximum among all types of
propulsive engines.
An air-breathing engine requires oxygen from
the surroundings for combustion of the fuel. All
aircraft engines are of air-breathing type. Air-
breathing engines with reciprocating IC engines
Compression Combustion Exhaust
work on Otto cycle and those with gas turbines chamber (M > 1)
(M < 1)
work on simple open-cycle Brayton cycle.

28.2 IC ENGINE-DRIVEN PROPULSIVE A ramjet engine


SYSTEM consists of supersonic and subsonic diffusers, a
The engine shown in Fig. 28.2 is called engine- combustion chamber and a discharge nozzle. The
propelled driven aircraft or simply as engine prop. atmospheric air first enters the supersonic diffuser
It is an air cooled, multi cylinder, radial engine. The and then subsonic diffuser, in which the kinetic
net power of the engine is used as shaft power to energy of air entering is converted into static
drive the propeller. With this engine, the aircraft pressure, which is called ram pressure. The rammed
can be used for short range flight with a speed air then enters into the combustion chamber, where
limit of 700 km/h. It is most widely used to power fuel is injected and combustion takes place with
propulsion of helicopters. the help of fuel burners. The air temperature is
raised by continuous combustion of fuel. The
Jet and Rocket Propulsions 965

hot combustion gases then expand in the exhaust jet engine develops the thrust by a high velocity of
nozzles with a velocity exceeding the velocity of jet of exhaust gases without the use of a compressor
air entering. Thus, due to change of momentum of or turbine.
working fluid, a thrust is developed in the direction Ramjets are frequently confused with pulse-jet,
of flight. which use an intermittent combustion, but ramjets
The ramjet engine cannot operate under static use a continuous combustion process, and are a
conditions, as there will be no pressure rise in the quite distinct type of jet engine.
diffuser and it is not self-operating at zero flight A typical pulse jet engine shown in Fig. 28.4,
velocity. The cycle pressure ratio of a ramjet engine comprises an air intake diffuser fitted with a one-
depends on its flight velocity. Higher the flight way flap or reed valve, a combustion chamber, and
velocity, larger the ram pressure and consequently, an acoustically resonant exhaust nozzle. The valves
larger the thrust. are operated due to a pressure difference across
Ramjets require considerable forward speed to them. Fuel in the form of a gas or liquid aerosol is
operate well, and as a class, work most efficiently either mixed with the air in the intake or injected
at speeds around Mach 3, and this type of jet can into the combustion chamber.
operate up to speeds of Mach 5.
Ramjets can be particularly useful in applications
requiring a small and simple engine for high speed
use; such as missiles. They have also been used
successfully, as tip jets on helicopter rotors.
The features of the ramjet engine are as
follows:
1. Its fuel consumption is too high at low and
moderate speeds. Fig. 28.4
2. Its fuel consumption decreases with flight
speed and approaches reasonable value, The incoming air is compressed by ram effect
when the flight Mach number is between in the diffuser section and then passes through
2 and 4. Thus, it is suitable for propelling the passages which are opened and closed by
supersonic missiles. non-return flap valves. The fuel is then injected
3. It has no moving part and hence is light in into the combustion chamber. The combustion is
weight. then initiated by a spark plug. Once the engine is
4. It is simple in construction and is adaptable operating normally, the spark plug is turned off
to mass production at a relatively low cost. and it is ignited by residual heat from the previous
cycle.
5. It can operate with any type of liquid fuels
and even with solid nuclear fuels. As combustion takes place, the pressure and
temperature of combustion products exceed the
6. It cannot be started on its own. It has to be
ram pressure, thus non-return flap valves get
accelerated to a certain flight velocity by
closed. Consequently, the pressurized hot gases
some launching device.
exit through the exhaust nozzle with a high velocity
Pulse Jet Engine and produce forward thrust on the unit. With the
escape of gases to the atmosphere, the pressure
A pulse jet engine (or pulsejet) is a very simple falls in the combustion chamber, the ram air forces
form of internal-combustion engine based jet the flap valves to open and fresh air enters in the
engine where combustion occurs in pulses. A pulse combustion chamber for the next cycle.
966 Thermal Engineering

Starting the engine usually requires forced no moving parts. However, scramjets have weight
air and an ignition method such as a spark plug and complexity issues that must be considered. A
for the fuel–air mix. Once running, the engine scramjet has very poor thrust to weight ratio (~2).
only requires an input of fuel to maintain a self- It has extreme aerodynamic complexity, airframe
sustaining combustion cycle. difficulties and testing difficulties.
The advantages and limitations of a pulse jet
engine are listed below.
SYSTEMS
Advantages
1. It is very simple in construction and cheaper These engines are equipped with a gas turbine to
in comparison to turbojet engines. drive the compressor and other auxiliary equipment
2. It is free from moving parts like a compressor, on the aircraft. These are
turbine and propeller. Hence it is light and 1. Turbojet engines
can carry higher payload. 2. Turboprop engines
3. It is capable to produce sufficient thrust at 3. Turbofan engines
low speeds also.
4. It can be operated as a pilotless aircraft and
is highly suitable for bombers and target
missiles.
Limitations The turbojet engine is propelled by the thrust
1. It can be used for a short flight life of 30 to produced due to acceleration of hot combustion
60 minutes. gases through the exhaust nozzle. Therefore, at
2. It produces very high noise. higher speed, the thrust developed is more and the
3. Due to intermittend pulse, it produces severe turbojet engine gives higher propulsive efficiency.
vibrations.
4. It has high rate of fuel consumption and very
low thermal efficiency of 2 to 3% only. The cross-sectional view of a turbojet engine is
5. The operating altitude is limited due to shown in Fig. 28.5(a), and the basic components
atmospheric air density considerations. of a turbojet engine is shown in Fig. 28.5(b). It
consists of three main sections—the diffuser,
the gas generator, and the nozzle. The diffuser
decelerates the incoming air relative to the engine
A scramjet is a supersonic combustion ramjet.
and a part of the kinetic energy of the air stream
It differs from a ramjet in which supersonic
is converted into pressure by ramming effect. The
combustion takes place. At higher speeds, it is
gas generator section consists of a compressor,
necessary to combust supersonically to maximize
combustor and a turbine, with same function
the efficiency of the combustion process. It operates
as in a stationary gas turbine plant. In a turbojet
at Mach numbers between 12 and 24.
engine, the power produced by the turbine is just
Like a ramjet, a scramjet essentially consists
sufficient to drive the compressor, fuel pump
of a restricted tube through which inlet air is
and other auxiliary equipment. The net power
compressed by the high speed of the vehicle, a
output of propulsive cycle is zero. The hot gases
combustion chamber where fuel is burned, and a
leaving the turbine, relatively at higher pressure
nozzle through which the exhaust jet leaves at higher
are accelerated in a nozzle to a high velocity. The
speed than the inlet air. Also like a ramjet, there are
discharge of high velocity gases to surroundings,
Jet and Rocket Propulsions 967

Applications
Turbojet engines are most suitable for aircrafts
travelling above 800 km/h. They are used in pilot
passengers, cargo long distance aircrafts, military
aircrafts, guided missiles, etc.

Merits
1. Its construction is much simpler as compared
to a multicylinder reciprocating IC engine
for the same power output.
2. The engine runs smoothly without vibrations.
3. The engine speed is higher than a reciprocat-
ing engine.
4. Torque obtained is smooth and uninterrupted.
5. Engine weight to power-output ratio is lower
as compared to a reciprocating engine.
6. These are suitable for longer flights at higher
altitude and speed.
7. Fuel can burn over a large range of mixture
strength.
8. Less maintenance is needed.
Demerits
1. Propulsive efficiency and thrust are lower at
lower speed.
2. It becomes inefficient below a speed of
550 km/h.
3. It produces more noise than a reciprocating
produces forward thrust (propulsive force) on the engine.
aircraft to propel it in opposite direction, because 4. Its capital cost is high.
the mass of atmospheric gases is large to displace 5. It requires longer runway for its take off and
in comparison to engine mass. landing.
Figure 28.5(c) shows the T–s diagram for a 6. Thrust-specific fuel consumption is high.
turbojet engine. Process 1–2 represents isentropic 7. It is not economical for short-distance flights.
pressure rise in the diffuser; Process 2–3—pressure 8. Sudden decrease of acceleration and decel-
rise in compressors Process 3–4 heat addition at eration is difficult.
constant pressure in the combustion chamber;
Process 4 –5—isentropic expansion in the turbine
and Process 5 –6—isentropic expansion in exhaust
nozzle. In an actual turbojet engine, the working
substance is not re-circulated through the process A turboprop engine is also called propjet, and it
6–1. is an intermediate device between a jet engine
968 Thermal Engineering

and a propeller driven by a reciprocating engine. Currently, turboprop engines are used on small
The propeller is coupled to the turbine through a subsonic aircraft, such as on small commuter
reduction gear that converts the high RPM, and low aircraft, where their greater reliability as compared
torque output to low RPM, and high torque. to reciprocating engines offsets their higher initial
cost.

In its simplest form, a turboprop shown in


Fig. 28.6, consists of an intake, compressor, com-
bustion chamber, turbine and a propelling nozzle. Advantages
Air is drawn into the intake and compressed by the 1. It develops higher thrust at low speeds and
compressor. Fuel is then added to the compressed hence take off rolling is short, requiring
air in the combustion chamber, where the fuel–air shorter runway.
mixture burns. The hot combustion gases expand 2. Thrust-specific fuel consumption is low.
through the turbine. Some of the power generated 3. Propulsive efficiency is high.
by the turbine is used to drive the compressor. The 4. Thrust reversal is easily achieved by varying
rest is transmitted through the reduction gearing to the blade angle, and aircraft speed can be
the propeller. Further expansion of the gases occurs drastically decreased.
in the propelling nozzle, where the gases exhaust 5. It is useful for short-range flights with a
to atmospheric pressure. The propelling nozzle speed of 600 km/h.
provides approximately 20 per cent thrust by ve-
locity difference of gases leaving and air entering Limitations
the aircraft, while the rest of thrust is provided by 1. Turboprop engines can be used for low
propeller. speeds, low altitude and short range of
flights.
2. The engine is heavier and more complicated.
It has higher weight per unit thrust.
3. At higher speeds, the propulsive efficiency
decreases drastically.
4. With the use of a heavier propeller,
compressor and turbine, the payload capacity
is low.

A turbofan (shown in Fig. 28.7) is a type of aircraft


engine consisting of a ducted fan which is powered
Turboprops are very efficient at modest flight by a gas turbine. A part of the airstream from the
speeds of 600 km/h because the jet velocity of the ducted fan passes through the gas turbine core,
propeller (and exhaust) is relatively low. Due to the providing oxygen to burn fuel to create power, and
high price of turboprop engines, they are mostly remaining of the air flow is by-passed to the engine
used where high-performance, short-takeoff and core, and is accelerated by the fan blades in the
landing capability and efficiency at modest flight same manner as a propeller. The combination of
speeds are required. thrust produced from the fan and the exhaust from
Jet and Rocket Propulsions 969

the core is a more efficient process than other jet m f is the rate of flow of fuel entering the
engine designs. It results into a comparatively low engine
specific fuel consumption. Vjet is the jet velocity of the gases leaving
Turbofans have a net exhaust speed that is much the nozzle
lower than a turbojet. This makes them much more Va is the velocity of air entering the
efficient at subsonic speeds than turbojets, and engine
somewhat more efficient at supersonic speeds up
( ma + m f ) Vjet represents the nozzle gross thrust
to Mach 1.6, but they have also been found to be
ma Va represents the ram drag force of the
efficient when used with continuous afterburner at
flight
Mach 3 and above.
Since the air–fuel ratio used in the aircraft is
very high, the mass flow rate of exhaust gases is
almost same as of air entering the engine. If the
mass of fuel to the gross thrust is ignored, the net
thrust is
F = ma (Vjet – Va) ...(28.2)
The speed of the jet must exceed the true air
speed of the aircraft, if there is to be a net forward
thrust on the air frame. For an aircraft cruising
at steady speed, the thrust is used to overcome
skin friction. The air at higher altitudes is thinner
and it offers a smaller drag force on the aircraft.
Therefore, commercial aircraft flying at the higher
altitudes to save fuel.
The specific thrust is defined as
All commercial jet aircraft are turbofans. They ratio of thrust deveoped to mass-flow rate of air.
are used mainly because they are highly efficient
Thrust F
and relatively quiet in operation. Turbofans are also Specific thrust = = ...(28.3)
Mass-flow rate ma
used in many military jet aircraft.
The specific impulse is defined
28.8 TERMINOLOGY USED WITH as the ratio of thrust developed to the weight of air,
which passes through the engine;
Thrust Thrust F
Isp = = ...(28.4)
Weight of propellant ma g
The thrust developed in a turbojet engine is the
unbalance force, which is caused by the difference The power developed from thrust
in the momentum of flow velocity of air entreing of the engine is called the thrust power W p , which
the engine and high velocity hot gases leaving the is the product of propulsive force and flight velocity
exhaust nozzle, and is given by Newton’s second Va. That is
law of motion, i.e.,
Wth = F Va
F = ( ma + m f ) Vjet – ma Va ...(28.1) = ma (Vjet – Va) Va ...(28.5)
where ma is the rate of flow of air through the
It is the gross thrust power. It is
engine
defined as change in momentum of mass-flow rate
970 Thermal Engineering

F F It is defined
on the basis of thrust and it is one of the important
parameters of an aircraft engine. It is measured in
kg per kg of air-N thrust. It is given by
v, m/s
wp = FV Fuel flow rate
Thrust sfc =
Thrust produced
mf
= ...(28.11)
of gases. It is the difference between rate of kinetic ma ¥ ( Vjet - Va )
energy of air entering and gas leaving That is,
Ê Vjet
2
- Va2 ˆ
W prop = DKE = maÁ ˜ Watts ...(28.6)
Ë 2 ¯ A turbojet engine consists of a diffuser, a com-
It is ratio of flight velocity pressor, a combustion chamber, a turbine and a jet
to jet velocity. It is designated as s and is given as nozzle. All of these devices operate in a steady flow
V manner. The thermodynamic cycle on T-s diagram
s= a ...(28.7) for a turbojet engine is shown in Fig. 28.9.
VJet
It is defined as the ratio of
thrust power to propulsive power.
Thrust power
hprop =
Propulsive power
ma ¥ ( Vjet - Va ) ¥ Va
=
2
ma ¥ ( Vjet - Va2 )/ 2
2 ( Vjet - Va ) ¥ Va
=
2
( Vjet - Va2 )
2Va
or hprop = ...(28.8)
( Vjet + Va )

The thermal efficiency of a


turbojet engine is defined as Air enters the diffuser from the surrounding
Propulsive power W prop atmosphere at temperature T1 and pressure p1 at
hth = = ...(28.9)
Heat supply rate Qin flight velocity Va. The air is slowed down in the
diffuser and kinetic energy of air is converted into
where Qin = m f CV;
static pressure. Actual diffusion process 1–2 and
m f is the rate of mass of fuel burning and CV is isentropic diffusion process 1–2s, are shown in
calorific value of fuel. Fig. 28.10 (a) on a T–s diagram.
The overall efficiency is The stagnation enthalpy after isentropic
defined as a product of thermal efficiency and diffusion process is expressed as
propulsive efficiency and is given as Va2
hoverall = hth ¥ hprop ...(28.10) h2s = h1 + (28.12)
2
Jet and Rocket Propulsions 971

T
03
T03
03s The diffused air at pressure p2 and temperature
2 3 T2 enters the compressor and is compressed to
T2 3s
pressure p3.
2s
Isentropic compression work
02 wC = h3 – h2
T02
T1 = Cp(T3s – T2)
1 2
s s Actual work input to compressor
(a) Diffusion process (b) Compression process win = h3 – h2
T T
= Cp(T3 – T2) ...(28.17)
The isentropic efficiency of the compressor is
4 05
given by
T04 T5
04 5 Isentropic work T3s - T2
hC = =
Actual work T3 - T2
05 The actual compression work input win can also
05s
be expressed as
5s
5
6s
6
Isentropic work C p (T3s - T2)
s s win = = ...(28.18)
(c) Expansion in turbine (d) Expansion in nozzle hC hC
The pressures and temperatures at two states are
related as
The stagnation temperature T2s after isentropic g -1 g -1

diffusion is given by
T3s Êp ˆ g Êp ˆ g
= Á 3s ˜ =Á 3˜ ...(28.19)
V2
T2 Ë p2 ¯ Ë p2 ¯
T2s = T1 + a (28.13)
2C p
Pressure after isentropic diffusion is given by Let m be mass of combustion gases generated in
g
ÊT ˆ g -1
the combustion chamber;
p2 = p1 Á 2 s ˜ (28.14)
Ë T1 ¯ m = Mass rate of air + Mass Rate of fuel
For small pressure rise in subsonic flow, diffuser = ma + m f
efficiency is given by The heat supplied in combustion chamber
Enthalpy rise in isentropic diffusion Qcomb = ( ma + m f ) h4 - ma h3
hdiffuser =
Enthalpy rise in actuaal diffusion The combustion efficiency is defined as
h2 s - h1 Actual rise in enthalpy of gases
= ...(28.15) hComb =
h2 - h1 Energy sypplied by fuel
For constant specific heat of air, ( ma + m f ) h4 - ma h3
= ...(28.20)
Isentropic temp. rise m f CV
hdiffuser =
Actual temp. rise
T -T
= 2s 1 ...(28.16)
T2 - T1 The hot combustion gases generated in the
combustion chamber expand partially in the turbine
972 Thermal Engineering

from pressure p4 and temperature T4 to pressure p5 The jet velocity can be obtained as
and temperature T5.
Vjet = 2( h5 - h6 ) = 2C p (T5 - T6 ) ...(28.23)
Isentropic expansion work in the turbine
wT = (h4 – h5s) = Cp(T4 – T5s) If the gases are leaving the turbine and entering
Actual work output of turbine; the nozzle with certain velocity, then total energy
wout = h4 – h5 input to nozzle
= Cp(T4 – T5) ...(28.21) Ein = Isentropic enthalpy drop + KE carried over
the turbine
The isentropic efficiency of the turbine is given
Then isentropic efficiency of the nozzle jet can
as
be defined as
Actual turbine work
hT = Final KE of jet
Isentropic turbine work hJet =
Isentropic enthalpy drop + KE
h4 - h5 carried over turbiine
=
h4 - h5 s 2
VJet
For constant specific heat, the isentropic effi-
= 2 ...(28.27)
ciency of the turbine
V2
T4 - T5 C pg (T5 - T6 s ) + duct
hT = ...(28.22) 2
T4 - T5 s
The pressures and temperatures at two states are THRUST AUGMENTATION IN
related as
g -1
T4 Êp ˆ g Several modification to turbojet engines have
= Á 4˜ ...(28.23) been made to cater to the special performance
T5 s Ë p5 ¯
requirement of various aircraft. One of them is
Further, for a turbojet engine thrust augmentation of turbojet engine for short
Turbine work = Compressor work duration for better take-off performance, higher
rate of climb and increased performance at higher
Jet Nozzle altitude. The thrust of a turbojet engine is given by
After partial expansion of combustion gases in F = ( ma + m f ) Vjet – ma Va
the turbine, the gases enter the nozzle and expand In the above equation;
adiabatically from the state 5 to the state 6.
1. Vjet is the function of maximum temperature
h5 = h6 in the cycle, as maximum temperature in the
The nozzle efficiency is given by cycle increases, the exit velocity Vjet also
Actual enthalpy drop h -h increases, and hence thrust increases.
hJet = = 5 6
Isenthalpic enthalpy drop h5 - h6 s 2. The thrust can also be increased by increasing
T5 - T6 the mass-flow rate of air ma .
= ...(28.24)
T5 - T6 s Thus, the thrust of a turbojet engine can be
2
Vjet increased by
where h5 = h6 +
2 1. Installing reheater or after burner before
2 gases entering the exhaust nozzle
Vjet
and T5 = T6 + ...(28.25) 2. Water methanol injection system
2C p
3. Air bleed system
Jet and Rocket Propulsions 973

Combustion
Compressor chamber Turbine Fuel spray bars
Flame holder

Air

In

Diffuser Gas generator Afterburner duct Adjustable


nozzle

flow rate of gases. Heat absorbed by water is


An afterburner, or reheat jet pipe is a device added compensated by the burning methanol, thus the
to the rear of the jet engine. It provides a means working temperature remains same.
of spraying fuel directly into the hot exhaust When water is injected into the combustion
gases, where it ignites and boosts available thrust chamber, the fresh air flow rate is reduced by an
significantly. Its purpose is to provide a temporary amount corresponding to water injected. A decrease
increase in thrust, both for supersonic flight and for in air flow in the compressor results into higher
take off. pressure ratio for same speed of turbine.
Afterburners are used mostly on military aircraft. The water injection causes a lower mass-
On military aircraft, the extra thrust is needed for flow rate of air and higher pressure ratio in the
combat situations. This is achieved by injecting compressor and a higher total mass flow through
additional fuel into the jet pipe downstream after the turbine. Thus, more thrust is produced. Thrust
the turbine. augmentation of about 30% is possible by water
The advantage of afterburning is the significantly injection.
increased thrust; the disadvantage is its very high
fuel consumption and inefficiency, though this is S
often regarded as acceptable for the short periods Since excess air is already present in a gas turbine
during which it is generally used. plant, thus a small quantity of compressed air is
bled to an auxiliary combustion chamber by by-
passing the turbine. In the auxiliary combustion
In some turbojet engines, a mixture of water and
methanol is injected in the combustion chamber.
The evaporation of water increases the mass-
974 Thermal Engineering

chamber, the bled air is heated by burning of an F 19328.15


(iii) Specific thrust = = = 50 N/kg
additional fuel. The high-temperature gases are ma 386.56
then discharged to an additional exhaust jet to bring F Va 19328.15 ¥ 150
thrust augmentation. A shut-off valve is used to (iv) Thrust power = =
1000 1000
bring the engine in normal position. = 2899.22 kW
The diameter of the propeller of an F 19328.15
aircraft is 2.5 m. It flies at a speed of 540 km/h at an (v) Specific impulse = =
ma g 386.56 ¥ 9.81
elevation of 8000 m, where air density is 0.525 kg/m3. = 5.09 s
The flight to jet speed ratio is 0.75. Calculate: (a) the
air-flow rate through the propeller, (b) thrust produced, For the combustion of 420 kg of
(c) specific thrust, (d) specific impulse, (e) thrust power. petrol, a flying missile has a range of 240 km; an average
velocity of 576 km/h and a propulsive force of 2700 N.
The maximum temperature rise in the combustion
Given A propeller aircraft: chamber is 815°C. The diameter of the discharge nozzle
Va = 540 km/h r = 0.525 kg/m3 is 30 cm. The altitude of the flight is 610 m, where the
atmospheric pressure is 0.944 bar. The calorific value of
V
s = a = 0.75 d = 2.5 m the fuel is 42000 kJ/kg. Cp for exhaust gases can be taken
Vjet
as 1.16 kJ/kg ◊ K. Calculate (a) air-fuel ratio, (b) exhaust
z = 8000 m
gas temperature and their velocity relative to missile,
To find (c) propulsive efficiency, and (d) overall efficiency of the
(i) Air flow rate through the propeller, unit.
(ii) Thrust produced,
(iii) Specific thrust,
(iv) Specific impulse, and Given A flying missile
(v) Thrust power. Va = 576 km/h mf = 420 kg
Distance = 240 km F = 2700 N
Assumption Air enters the propeller with flight veloc-
DTcomb = 815°C = 815 K dN = 0.3 m
ity.
p = 0.944 bar = 94.4 kPa z = 610 m
Analysis The flight velocity is CV = 42000 kJ/kg Cp = 1.16 kJ/kg ◊ K
(540 ¥ 1000 m)
Va = = 150 m/s To find
(3600 s)
(i) Air–fuel ratio,
Exit jet velocity;
(ii) Exhaust gas temperature,
Va 150 m
Vjet = = = 200 m/s (iii) Velocity of exhaust gases,
s 0.75
(iv) Propulsive efficiency, and
(i) Air-flow rate through the propeller
(v) Overall efficiency of unit.
The air-flow rate through the propeller
ma = density ¥ cross-sectional area of Assumption Specific gas constant for exhaust gases as
propeller ¥ flight velocity 0.287 kJkg ◊ K.

Êp ˆ Analysis The flight velocity is


= 0.525 ¥ Á ¥ 2.52 ˜ ¥ 150
Ë4 ¯ (576 ¥ 1000 m)
Va = = 160 m/s
= 386.56 kg/s (3600 s)
(ii) Thrust produced Distance range ( 240 ¥ 1000 m)
F = ma (Vjet – Va ) Flight duration, Dt = =
Flight velocity (160 m/s)
= 386.56 ¥ (200 – 150)
= 1500 s or 25 min.
= 19328.15 N
Jet and Rocket Propulsions 975

(i) Air–fuel ratio = 2700 ¥ 160


Fuel consumption rate; = 432000 W or 432 kW
Mass of fuel ( 420 kg) Thermal efficiency
mf = =
Duration of flight (1500 s) W prop 432
hth = =
= 0.28 kg/s Qin 11760
Let ma be mass-flow rate of air with fuel. Then = 0.0367 or 3.67%
energy balance reveals that Overall efficiency;
( ma + m f ) C p ( DT )comb hoverall = hth ¥ hProp = 0.0367 ¥ 0.605
= m f CV = 0.0222 or 2.22%
( ma + 0.28) ¥ 1.16 ¥ 815
A jet propelled engine having two jets
= 0.28 ¥ 42000
and working on turbojet has a velocity of 210 m/s, when
or ma = 12.44 kg/s
flying at an altitude of 12000 m. The density of air at
ma (12.44 kg / s) this altitude is 0.172 kg/m3.The resistance or drag of the
A/F = =
mf (0.28 kg / s) plane is 6670.8 N and propulsive efficiency of the jet is
= 44.42 50%. The overall efficiency of the unit is 18%. Calorific
(ii) Velocity of exhaust gases value of the fuel is 4.895 ¥ 104 kJ/kg. Calculate
Propulsive force is given by (a) Absolute velocity of jet,
F = ( ma + m f ) Vjet – ma Va (b) Quantity of air compressed per minute,
(c) Diameter of jet,
or 2700 = (12.44 + 0.28) VJet – 12.44 ¥ 160
(d) Net power output of the plant,
It gives VJet = 368.74 m/s
(e) Thrust specific fuel consumption,
(iii) Temperature of exhaust gases (f) Air–fuel ratio.
Applying continuity equation for flow of exhaust
gases
AVjet
ma + m f = Given A jet propelled engine of a turbojet
v
It gives specific volume of exhaust gases Va = 210 m/s Njet = 2 Nos
FD = 6670.8 N r = 0.172 kg/m3
AVjet (p /4) ¥ (0.32 ) ¥ 368.74
v = = z = 12000 m hprop = 0.5
ma + m f 12.44 + 0.28
hOverall = 0.18 CV = 48950 kJ/kg
= 2.04 m3/kg
To find
The charecteristic gas equation gives
(i) Absolute velocity of jet,
pv = RT
(ii) Quantity of air compressed per minute,
or 94.4 ¥ 2.04 = 0.287T
(iii) Diameter of jet,
It gives T = 671 K or 398°C (iv) Net power output of the plant,
(iv) Propulsive efficiency (v) Thrust specific fuel consumption, and
It is given by (vi) Air fuel ratio.
2 Va 2 ¥ 160
hprop = = Analysis
Va + Vjet 160 + 368.74
(i) Absolute velocity of jet
= 0.605 or 60.5%
The propulsive efficiency is given by
(v) Overall efficiency:
The rate of heat supply to missile 2Va
hprop =
Va + Vjet
Qin = m f CV = 0.28 ¥ 42000
= 11760 kW 2 ¥ 210
or 0.5 =
The propulsive power 210 + Vjet
W prop = Propulsive force ¥ flight velocity or Vjet = 630 m/s
976 Thermal Engineering

Absolute velocity of jet (vi) Thrust–specific fuel consumption


= Vjet – Va = 630 – 210 Fuel-flow rate
Thrust sfc =
= 420 m/s Thrust produced
(ii) Quantity of air compressed per minute 0.318
=
The thrust force can be approximated as 6670.8
F = ma (Vjet – Va) = 4.767 ¥ 10–5 kg/N thrust/s
It is equal to drag force, thus
A turbojet engine flying at a speed of
6670.8 = ma ¥ 420 990 km/h consumes air at a rate of 54.5 kg/s. Calculate
or ma = 15.88 kg/s (a) Exit velocity of jet, when enthalpy changes for
The volume of air compressed per minute the nozzle is 200 kJ/kg and velocity coefficient is
ma 15.88 0.97,
Va = ¥ 60 = ¥ 60
r 0.172 (b) Fuel-flow rate in kg/s, when air-fuel ratio is 75 : 1
= 5539.5 m3/min (c) Thrust-specific fuel consumption
(iii) Diameter of jet (d) Thermal efficiency of the plant, when combustion
efficiency is 93% and calorific value of fuel is
The mass-flow rate per jet
45000 kJ/kg
ma 15.88
m1 = = = 7.94 kg/s (e) Propulsive power
N jet 2 (f) Propulsive efficiency
Applying continuity equation to obtain the jet (g) Overall efficiency
diameter
Êp ˆ
m1 = r Á d 2jet ˜ Vjet
Ë4 ¯ Given A turbojet engine
Êp ˆ Va = 990 km/h ma = 54.5 kg/s
or 7.94 = 0.172 ¥ Á d 2jet ˜ ¥ 630
Ë4 ¯ DhN = 200 kJ/kg k = 0.97
or djet = 0.305 m or 30.5 cm A/F = 75 : 1 CV = 45000 kJ/kg
(iv) Net power output of the unit hcomb = 0.93
Thrust po wer = Thrust ¥ Flight velocity To find
= 6670.8 ¥ 210 (i) Exit velocity of jet,
= 1400868 W or 1400.868 kW (ii) Fuel flow rate in kg/s,
Propulsive Power (iii) Thrust specific fuel consumption,
Thrust power 1400.868 (iv) Thermal efficiency of the plant,
W prop = = (v) Propulsive power,
Propulsive efficiency 0.5
= 2801.74 kW (vi) Propulsive efficiency, and
(vii) Overall efficiency.
(v) Air–fuel ratio
Overall efficiency is given by Analysis
Power output Propulsive power
hoverall = = Flight velocity
Heat supply rate m f ¥ CV
990 ¥ 1000
Va = = 275 m/s
2801.74 kW 3600
or 0.18 =
m f ¥ 48950 (i) Exit velocity of jet
It gives m f = 0.318 kg/s VJet = k 2DhN
m 15.88 = 0.97 ¥ 2 ¥ ( 200 ¥ 1000 J/kg)
A/F = a =
mf 0.318 = 613.48 m/s
= 49.94 kg of air/kg of fuel (ii) Fuel-flow rate
Given that ma = 54.5 kg/s and A/F = 75
Jet and Rocket Propulsions 977

Further, the Air-fuel ratio is given by Air enters a turbojet engine at 80 kPa,
Mass flow rate of air m 240 K and an inlet velocity of 280 m/s. The pressure ratio
A/F = = a
Mass flow rate of fuel m f across the compressor is 8. The turbine inlet temperature
54.5 kg/s is 1200 K and the pressure at the nozzle exist is 80 kPa.
or mf = = 0.7267 kg/s
75 The work developed by the turbine equals the compressor
(iii) Thrust specific fuel consumption work input. The diffuser, compressor, turbine and nozzle
Thrust produced processes are isentropic, and there is no pressure drop
= ma (Vjet – Va) for flow through the combustor. For operation at steady
= 54.5 ¥ (613.48 – 275) state, determine the velocity at the nozzle exit and the
pressure at each principal state. Neglect kinetic energy at
= 18447.16 N
the exit of all components except the nozzle and neglect
Fuel-flow rate potential energy throughout.
Thrust sfc =
Thrust produced
0.7267
=
18447.16 Given An ideal turbojet engine operates at steady state
= 3.94 × 10–5 kg/N thrust/s with
(iv) Propulsive power p1 = 80 kPa T1 = 240 K
Ê Vjet
2
- Va2 ˆ V1 = 280 m/s rp =8
W prop = ma Á ˜ ( kW ) p6 = p1 = 80 kPa T4 = 1200 K
ÁË 2000 ˜¯
WT = Win
Ê 613.482 - 2752 ˆ
= 54.5 ¥ Á ˜ To find
Ë 2000 ¯ Velocity at the nozzle exit and pressure at each principal
= 8194.96 kW states
Thrust power; Assumptions
Wth = Thrust ¥ Va = 18447.16 ¥ 275 (i) Each component is analysed as a control volume
= 5072969 W = 5072.97 kW at steady state.
(v) Propulsive efficiency (ii) The working fluid is air modelled as an ideal gas
Thrust power 5072.97 with Cp = 1005 J/kg ◊ K, g = 1.4
hprop = =
Propulsive Power 8194.96 Analysis Analysing each device separately.
= 0.619 or 61.9% Diffuser
It can also be obtained by using Temperature T2, after diffusion process,
2Va
hprop = V12 280 2
( Vjet + Va) T2 = T 1 + = 240 + = 279 K
2C p 2 ¥ 1005
(vi) Thermal efficiency
g
Heat supply rate; 3.5
Ê T ˆ g –1 Ê 279 ˆ
p2 = p1 Á 2 ˜ = 80 ¥ Á
Qin = m f CV = 0.7267 ¥ 45000 Ë 1¯
T Ë 240 ˜¯
= 32701.5 kW
= 135.5 kPa
W prop 8194.96
hth = = p3 = rp ¥ p2 = 8 ¥ 135.5 = 1084 kPa
Qin 32701.5 Compressor
= 0.2505 or 25.05% g –1
1.4 -1
(vii) Overall efficiency Êp ˆ g
T3 = T2 Á 3 ˜ = 279 ¥ (8) 1.4
hoverall = hth ¥ hprop Ë p2 ¯
= 0.2505 ¥ 0.619 = 505.4 K
= 0.1551 or 15.51% p4 = p3 = 1084 kPa
978 Thermal Engineering

Schematic with given data

T4 = 1200 K (given) A turbojet engine aircraft flies with


q2–3 = 0 a velocity of 260 m/s at an altitude where the air is at
Compression work 35 kPa and – 40°C. The compressor has a pressure ratio
win = Cp (T3 – T2) + 0 + 0 of 10 and the temperature of the gases at the turbine inlet
or w2–3 = 1005 ¥ (505.4 – 279) = 227.53 kJ/kg is 1095°C. The air enters the compressor at a rate of
8.5 kg/s. Using the cold air-standard assumptions,
Turbine: Given, compressor work is equal to turbine
determine (a) temperature and the pressure of the gases
work,
at the turbine exit, (b) velocity of the gases at the nozzle
WT = Win
exit, and (c) propulsive efficiency of the cycle.
or Win = Cp (T4 – T5)
or 227.53 = 1.005 ¥ (1200 – T5)
227.54
or T5 = 1200 – = 973.6 K Given An air standard jet propulsive cycle with
1.005
g p1 = 35 kPa Va = 260 m/s
3.5
Ê T ˆ g –1 Ê 973.6 ˆ T1 = – 40°C = 233 K rp = p3/p2 = 10
p5 = p4 Á 5 ˜ = 1084 ¥ Á
Ë T4 ¯ Ë 1200 ˜¯ T4 = 1095°C = 1368 K m = 8.5 kg/s
= 521.46 kPa
Nozzle: Expansion through nozzle
Jet velocity; Vjet = 2C p (T5 - T6 )
g –1 1.4 -1
Êp ˆ g Ê 80 ˆ 1.4
where T6 = T5 Á 6 ˜ = 973.6 ¥ Á
Ë p5 ¯ Ë 521.46 ˜¯
= 569.86 K
Thus Vjet = 2 ¥ 1005 (973.6 - 569.86)
= 900.8 m/s
Jet and Rocket Propulsions 979

To find Process 4–5 Isentropic expansion in the turbine;


(i) Temperature and pressure of the gases at the w4–5 = w2–3
turbine exit, h4 – h5= h3 – h2
(ii) Velocity of gases at the nozzle exit, and Cp (T4 – T5)= Cp (T3 – T2)
(iii) Propulsive efficiency. or T5= T4 – T3 + T2
= 1368 – 514.78 + 266.63 = 1120 K
Assumptions
and pressure at turbine exit
(i) Each component in the cycle is analysed as a g 1– 4
control volume at steady state. Ê T ˆ g –1 Ê T ˆ 1.4 –1
p5 = p4 Á 5 ˜ = p3 Á 5 ˜
(ii) Diffuser, compressor, turbine and nozzle opera- Ë T4 ¯ Ë T4 ¯
tions are isentropic. 3.5
Ê 1120 ˆ
(iii) The combustion process is replaced by heat = 561 ¥ Á = 278.43 kPa
Ë 1368 ˜¯
addition.
(iv) There is no pressure drop in combustion chamber. (ii) Process 5–6 Isentropic expansion of gases in
(v) Network output of the turbojet engine is zero i.e., the nozzle
turbine work is equal to compressor work. g –1
Êp ˆ g
(vi) Potential energy changes are negligble. T6 = T 5 Á 6 ˜
(vii) Except at inlet and exit the kinetic energy effects Ë p5 ¯
are ignored. 1.4 – 1
Ê 35 ˆ 1.4
(viii) The working substance is air as an ideal gas with = 1120 ¥ Á
Cp = 1.005 kJ/kg ◊ K and g = 1.4. Ë 278.43 ˜¯
= 619.3
Analysis (iii) Velocity of Gases at nozzle exit
(i) Temperature and pressure at the turbine exit Vjet = 2 C p (T5 – T6 )
Process 1–2 Raming effect: Isentropic compression
=2 ¥ 1005 ¥ (1120 – 619.3)
V2
T2 = T 1 + a = 1003.2 m/s
2 Cp
(iv) Propulsive efficiency
260 2 m ( Vjet – Va ) Va
= 233 + Propulsive power
2 ¥ 1005 hp = =
Heat supply rate mC p (T4 – T3 )
= 233 + 33.63 = 266.63
(1003.2 – 260) ¥ 260
and pressure p2 after isentropic compression = = 0.225 or 22.5%
1005 ¥ (1368 – 514.78)
g
Ê T2 ˆ g –1
p2 = p1 Á ˜ A turbojet aircraft is flying at 800 km/h
Ë T1 ¯
1.4 at 10700 m altitude where pressure and temperature of
Ê 266.63 ˆ 1.4 -1 the atmosphere are 24 kPa and – 50°C, respectively. The
= 35 ¥ Á
Ë 233 ˜¯ pressure ratio in the compressor is 10 and the maximum
cycle temperature is 820°C. Calculate the thrust devel-
= 56.1 kPa
oped and specific fuel consumption, using the following
Process 2–3 Isentropic compression in a information; entry duct efficiency 0.9; isentropic efficien-
compressor; cy of the compressor 0.9; Stagnation pressure drop in
p3 = rp p2 = 10 ¥ 56.1 = 561 kPa the combustion chamber 0.14 bar; calorific value of the
g –1 fuel 43,300 kJ/kg; combustion efficiency 98%; isentropic
Êp ˆ g efficiency of the turbine 0.92; mechanical efficiency of
T3 = T2 Á 3 ˜
Ë p2 ¯ the drive 0.98, jet pipe efficiency 0.92; nozzle outlet area
1.4 -1 0.08 m2; Cp and g for compression process 1.005 kJ/kg ◊ K
= 266.63 ¥ (10) 1.4 and 1.4; Cp and gp for combustion and expansion precess
= 514.78 K 1.15 kJ/kg ◊ K and 1.333, respectively. Assume that the
nozzle is convergent.
980 Thermal Engineering

Now diffuser efficiency


T –T
Given A turbojet engine operates on a jet propulsion hdiffuser = 2 s 1
T2 – T1
cycle
Va = 800 km/h = 222.22 m/s, z = 10700 m 247.56 – 223
T2 = + 233 = 250.3 K
p1 = 24 kPa T1 = – 50°C = 223 K 0.9
g
p3 3.5
rp = = 10 T4 = 820° C = 1093 K Ê T ˆ g –1 Ê 247.56 ˆ
p2 p2 = p1 Á 2 s ˜ = 24 kPa ¥ Á
Ë T1 ¯ Ë 223 ˜¯
hdiffuser = 0.9 hC = 0.9,
p4 – p3 = 0.14 bar CV = 43,300 kJ/kg = 29.67 kPa
hcomb = 0.98 hT = 0.92 For isentropic compression 2–3
g –1 g -1
hmech = 0.98 hnozzle = 0.92
T3s Êp ˆ g g
A6 = 0.08 m2 = Á 3˜ = ( rp )
T2 Ë p2 ¯
For compression and ramming
1.4 -1
Cpa = 1.005 kJ/kg ◊ K g = 1.4
= (10) 1.4 = 1.93
For combustion and expansion
Cpg = 1.15 kJ/kg ◊ K g = 1.333 Temperature after isentropic compression is
T3s = 1.93 ¥ 250.3 = 483.25 K
To find
Using compressor efficiency
(i) Thrust developed, and h –h T –T
(ii) Specific fuel consumption. hC = 3s 2 = 3s 2
h3 – h2 T3 – T2
Schematic with given data 483.25 - 250.3
or T3 = 250.3 + = 509.13 K
0.9
Pressure after compression;
p3 = rp p2 = 10 ¥ 29.67 = 296.7 kPa
Pressure after combustion
p4 = p3 – Dp = 296.7 – 0.14 ¥ 100 = 282.7 kPa
Compressor work,
wcomp = h3 – h2 = Cp (T3 – T2)
= 1.005 ¥ (509.13 – 250.3) = 260.12 kJ/kg
The actual turbine work required, just to drive the
compressor and to overcome the mechanical losses
wcomp 260.12
wT = = = 265.43 kJ/kg
hmech 0.98
Further wT = Cpg (T4 – T5)
Assumptions 265.43
or T5 = 1093 – = 862.18 K
(i) Each component in the cycle is analysed as a 1.15
control volume at steady state. Using isentropic efficiency of the turbine
(ii) Kinetic energy is negligible except at diffuser h –h T –T
hT = 4 5 = 4 5 = 0.92
inlet and nozzle exit. h4 – h5 s T4 – T5 s
(iii) Negligible potential energy change in the 1093 – 862.18
systems. or T5s = 1093 – = 842.1 K
0.92
Analysis Stagnation temperature after ramming The pressure after turbine expansion
g 1.333
Va2 ( 222.22) 2 Ê T ˆ g –1
T2s = T1 + = 223 + Ê 842.1ˆ 1.333 –1
2 ¥ 1005 p5 = p4 Á 5 s ˜ = 282.7 ¥ Á
2 CP
Ë T4 ¯ Ë 1093 ˜¯
= 247.56 K
= 99.5 kPa
Jet and Rocket Propulsions 981

For flow through nozzle (refer compressible fluid (i) The momentum thrust
flow) F = m (Vexit – Va)
For chocked flow, the critical pressure ratio is given = 10.62 ¥ (535.3 – 222.22)
by Eq. (10.26) = 3335.1 N
g 1.333
p* Ê 2 ˆ g –1 Ê 2 ˆ 1.333 –1 Since the pressure at the nozzle exit is greater
= Á = Á than the atmospheric pressure, thus the pressure
p5 Ë g + 1˜¯ Ë 1.333 + 1˜¯
thrust will also act
= 0.54 Pressure thrust = ( p6 – p1)A
and p* = 99.5 ¥ 0.54 = 53.7 kPa = (53.7 – 24) ¥ 103 ¥ 0.08
The atmospheric pressure is 24 kPa, it means mass = 2376 N
flow rate through the nozzle is maximum and velocity of Total thrust (propulsive force)
fluid at the nozzle exit would be sonic velocity.
= 3335.1 + 2376
Temperature at the nozzle exit: Eq. (10.25)
= 5711.1 N
T6 s 2 2
= = = 0.857 (ii) Ideal Heat supplied in the combustion chamber
T5 g + 1 1.333 + 1 = m Cpg (T4 – T3)
and T6s = 0.857 ¥ 862.18 = 739.1 K = 10.62 ¥ 1.15 ¥ (1093 – 509.13)
Jet pipe or nozzle efficiency = 7130.8 kJ/s
h –h T –T
hnozzle = 5 6 = 5 6 Actual heat supplied
h5 – h6 s T5 – T6 s
Ideal heat supply
T6 = T5 – hnozzle ¥ (T5 – T6s) Qact =
hcomb
= 862.18 – 0.92 ¥ (862.18 – 739.1)
= 749 K 7130.8
= = 7276.3 kW
g 0.98
Ê T6 s ˆ g –1 It is also expressed as
p6 = p5 Á
Ë T5 ˜¯ Qact = m f ¥ CV
1.333
or 7276.3 = m f ¥ 43300
Ê 739.1 ˆ 0.333
= 99.5 ¥ Á
Ë 862.18 ˜¯
or m f = 0.168 kg/s
Specific fuel consumption rate
= 53.7 kPa
mf 0.168
Thrust sfc = =
Specific gas constant Total thrust 5711.1
C p (g - 1) 1.15 ¥ (1.333 - 1) = 0.029 kg/kNs
R = =
g 1.333
= 0.287 kJ/kg K Remark:
The specific volume at state 6 (i) The equations used for critical flow through the
nozzle are taken from compressible fluid flow
RT6 0.287 ¥ 749
v6 = = = 4.02 m3/kg given in Chapter 10.
p6 53.5
(ii) Since air fuel ratio used in jet propulsion
The velocity (sonic) at the nozzle exit cycle is very low (= 0.029 kg/kNs), therefore,
Vexit = a = g RT6 approximation of mass flow rate of air equal to
mass flow rate of gases in the turbine and nozzle
= 1.333 ¥ 0.287 ¥ 103 ¥ 749 is proper.
= 535.3 m/s
The mass flow rate of air through the nozzle A turboprop aircraft is flying at a
A ◊ Vexit 0.08 ¥ 535.3 speed of 720 km/h at an altitude where the temperature
m = = = 10.62 kg/s is –18°C. Determine specific power output and thermal
v6 4.02
982 Thermal Engineering

efficiency. Given specifications are Intake duct: Diffuser


Compressor pressure ratio = 9 Temperature T2s after isentropic diffusion process,
Maximum cycle temperature = 800°C Va2 200 2
T2s = T1 + = 255 + = 274.9 K
Intake duct efficiency = 90% 2C p 2 ¥ 1005
Isentropic efficiency of compressor = 86% The pressure ratio across the diffuser
Isentropic efficiency of turbine = 90% g 1.4
Mechanical Efficiency = 92% p2 Ê T ˆ g -1 Ê 274.9 ˆ 1.4 -1
= Á 2s ˜ =Á = 1.3
Neglect the pressure loss in the combustion chamber. p1 Ë T1 ¯ Ë 255 ˜¯
Assume that the exhaust gases leave the aircraft at For subsonic flow, the diffuser efficiency is given by
720 km/h relative to aircraft. For compression, Cp = Isentropic temp. rise T2 s - T1
hdiffuser = =
1.005 kJ/kg ◊ K, g = 1.4; and for expansion, Cp = 1.15 kJ/ Actual temp. rise T2 - T1
kg ◊ K, g = 1.35. 274.9 - 255
or T2 = 255 + = 277.11 K
0.9
Compressor
Given A turboprop engine Temperature T3s after isentropic compression
Va = 720 km/h T1 = –18°C = 255 K g -1 1.4 -1
p3 Êp ˆ g Ê 9ˆ 1.4
=9 T4 = 800°C = 1073 K T3s = T2 Á 3 ˜ = 277.11 ¥ Á ˜
p2 Ë p2 ¯ Ë 1¯
hdiffuser = 0.9 hC = 0.86 = 519.15 K
hT = 0.9 hmech = 0.92 The isentropic efficiency of the compressor is given
For air; Cpa = 1.005 kJ/kg ◊ K ga = 1.4 by
For gas; Cpg = 1.15 kJ/kg ◊ K gg = 1.35 Isentropic work T3s - T2
hC = =
Actual work T3 - T2
To find
519.15 - 277.11
(i) Specific power output, and or T3 = 277.11 + = 558.55 K
0.86
(ii) Thermal efficiency of the plant. The actual work input to compressor win is expressed
as
win = Cp (T3 – T2)
= 1.005 ¥ (558.55 – 277.11)
= 282.85 kJ/kg
Combustion chamber
The heat supplied to air
qin = Cpg (T4 – T3)
= 1.15 ¥ (1073 – 558.55)
= 591.61 kJ/kg of air
Turbine
The pressure ratio across the turbine is given by
p4 p p p
= 3 = 3 ¥ 2 = 9 ¥ 1.3 = 11.70
p5 p1 p2 p1
Analysis The flight velocity The temperature T5s after isentropic expansion in the
720 ¥ 1000 turbine;
Va = = 200 m/s T4 1073
3600 T5s = = = 567 K
g -1 1.35 -1
Thus the flow of air is subsonic. g
Ê p4 ˆ gg Ê 11.70 ˆ 1.35
Analysing each device separately; ÁË p ˜¯ ÁË 1 ˜¯
5
Jet and Rocket Propulsions 983

Using isentropic efficiency of turbine; To find


T4 - T5 (i) Thrust developed,
hT =
T4 - T5 s (ii) Pressure and temperature at inlet of jet nozzle,
Actual temperature after expansion in turbine, (iii) Temperature, velocity and Mach number at
nozzle exit, and
T5 = 1073 – 0.9 ¥ (1073 – 567) = 617.6 K
(iv) Specific fuel consumption.
(i) Specific work output
Turbine work output per kg of air T–s dagram
wout = Cpg (T4 – T5)
= 1.15 ¥ (1073 – 617.6)
= 523.7 kJ/kg
Net work output per kg of the plant
wnet = wout – win = 523.7 – 282.85
= 240.85 kJ/kg of air
It is the specific work output of plant, which is
supplied to propeller of turboprop.
(ii) Thermal efficiency
Thermal efficiency of the cycle
w 240.85
hth = net =
qin 591.61
= 0.4071 or 40.67%
Assumptions
A simple turbojet unit operates with a (i) Each component in the cycle operates at steady
turbine inlet temperature of 1040°C. The following data state.
refers to the design conditions: (ii) Kinetic energy effects are considered at nozzle
Compressor pressure ratio = 7.5 only.
Compressor efficiency = 84% (iii) Pressure at nozzle exit is atmopheric pressure.
Turbine efficiency = 84% (iv) Assuming flight at ground level, i.e., Va = 0.
Nozzle efficiency = 98% (v) Specific heat of air at constant pressure as
Pressure drop in combustion chamber = 0.2 bar 1.005 kJ/kg ◊ K and g = 1.4.
Mass flow rate = 25 kg/s Analysis Analysing each device of the cycle separately
Atmospheric pressure and temperature = 1 bar, 27°C in steady state.
Neglect the mass of the fuel and mechanical losses.
Compressor Temperature T2s after isentropic
Calculate the design thrust, pressure and temperature
compression
at the inlet of the jet nozzle, temperature, velocity and
g -1
Mach number at the nozzle exit.
g
T2s = T1( rp )
1.4 -1
Ê 7.5 ˆ 1.4
Given A simple turbojet unit = 300 ¥ Á = 533.5 K
Ë 1 ˜¯
p1 = 1 bar T1 = 27°C = 300 K
p2 The isentropic efficiency of the compressor is given
= 7.5 T3 = 1040°C = 1313 K
p1 by
hC = 0.84 hT = 0.84 Isentropic work T2 s - T1
hC = =
hNozzle = 0.98 ma = 25 kg/s Actual work T2 - T1
mf ª 0 p3 = p2 – 0.2 bar 533.5 - 300
or T2 = 300 + = 577.98 K
0.84
984 Thermal Engineering

The actual work input to compressor win Mach number at nozzle exit
win = Cp (T2 – T1) Sonic velocity,
= 1.005 ¥ (577.98 – 300) = 279.37 kJ/kg
a = g RT = 1.4 ¥ 287 ¥ 789.3
Turbine For a turbojet engine, = 563.1 m/s
wout = win Mach number,
or win = Cp (T3 – T4) Vjet 702.75
or 279.37 = 1.005 ¥ (1313 – T4) M = = = 1.25
a 563.1
or T4 = 1035 K
Thrust developed
It is the temperature at nozzle entry.
F = ma (Vjet – Va) = 25 ¥ (702.75 – 0)
The isentropic efficiency of the turbine is given as
= 17568.75 N
Actual work output T -T
hT = = 3 4
Isentropic work output T3 - T4 s A turbojet engine develops a thrust
1313 - 1035 power of 750 kW, when flying at an altitude of 9200 m
or 0.84 =
1313 - T4 s at a velocity of 220 m/s. The following data refers to the
or T4s = 982.05 K, design conditions:
Pressure p3 before turbine entry Compressor pressure ratio = 5
p3 = p2 – 0.2 bar = 7.5 – 0.2 = 7.3 bar Compressor efficiency = 85%
The pressure p4 after isentropic expansion in the Turbine efficiency = 85%
turbine can be expressed as Nozzle efficiency = 90%
g 1.4 Inlet pressure and temperature = 0.306 bar, –45.5°C
p3 Ê T ˆ g -1 Ê 1313 ˆ 1.4 -1 Temperature of gas leaving the
= Á 3˜ =Á = 2.763
p4 Ë T4 s ¯ Ë 982.05 ˜¯ combustion chamber = 670°C
Pressure p4 at nozzle entry Calorific value of fuel = 42500 kJ/kg
p3 7.3 bar Velocity in ducts = 200 m/s
p4 = = = 2.64 bar
2.763 2.763 For air; Cp = 1.005 kJ/kg ◊ K g = 1.4
Isentropic expansion through nozzle For combustion gases Cp = 1.087 kJ/kg ◊ K g = 1.33
g -1 1.4 -1 Calculate
T4 Êp ˆ g Ê 2.64 ˆ 1.4 (a) air–fuel ratio
= Á 4˜ =Á = 1.32
T5 s Ë p5 ¯ Ë 1 ˜¯ (b) overall thermal efficiency of unit
1035 (c) rate of air consumption
or T5s = = 784.3 K (d) power developed by the turbine
1.32
(e) outlet area of the jet
The isentropic efficiency of the nozzle is given as
(f) specific fuel consumption
Actual enthalpy drop T -T
hNozzle = = 4 5
Isentropic enthalpy drop T4 - T5 s
Actual temperature after expansion in nozzle,
Given A turbojet unit
T5 = 1035 – 0.98 ¥ (1035 – 784.3)
p1 = 0.306 bar Va = 220 m/s
= 789.3 K p2
It is the temperature at nozzle exit. T1 = – 45.5°C = 227.5 K =5
p1
Exit jet velocity from nozzle T3 = 670°C = 943 K hC = 0.85
Vjet = 2C p (T4 - T5 ) hT = 0.85 hNozzle = 0.9
F Va = 750 kW CV = 42500 kJ/kg
= 2 ¥ 1005 ¥ (1035 - 789.3)
Vduct = 200 m/s D z = 9200 m
= 702.75 m/s
Jet and Rocket Propulsions 985

For air; Cpa = 1.005 kJ/kg ◊ K ga = 1.4 360.3 - 227.5


or T2 = 227.5 + = 383.75 K
For gas; Cpg = 1.087 kJ/kg ◊ K gg = 1.33 0.85
The actual work input to compressor win
To find
win = Cp (T2 – T1)
(i) Air–fuel ratio,
= 1.005 ¥ (383.75 – 227.5) = 157.03 kJ/kg
(ii) Overall thermal efficiency of unit,
Turbine For a turbojet engine;
(iii) Rate of air consumption,
win = wT = Cpg (T3 – T4)
(iv) Power developed by the turbine,
or 157.03 = 1.087 ¥ (943 – T4)
(v) Outlet area of the jet, and
The actual temperature T4 after turbine expansion
(vi) Specific fuel consumption.
T4 = 798.53 K
Assumptions The isentropic efficiency of the turbine is given as
(i) Each component in the cycle operates at steady Actual work output T -T
hT = = 3 4
state. Isentropic work output T3 - T4 s
(ii) Kinetic energy effect is considered in fluid flow 943 - 798.53
through nozzle only. or 0.85 =
943 - T4 s
(iii) Pressure at nozzle exit is atmopheric pressure. or T4s = 773.04 K,
(iv) No pressure drop in combustion chamber. The pressure p4 after isentropic expansion in the
(v) Specific gas constant for exhaust gases as R = turbine can be expressed as
0.287 kJ/kg ◊ K. gg 1.33
p3 Ê T ˆ g g -1 Ê 943 ˆ 1.33 -1
= Á 3˜ =Á = 2.227
p4 Ë T4 s ¯ Ë 773.04 ˜¯
p
Pressure ratio 4 across nozzle is given by
p5
p4 p4 p3 p4 p2 1
= ¥ = ¥ = ¥5
p5 p3 p1 p3 p1 2.227
= 2.244
Isentropic expansion through nozzle
g g -1 1.33 -1
T4 Êp ˆ gg Ê 2.244 ˆ 1.33
= Á 4˜ =Á = 1.222
T5 s Ë p5 ¯ Ë 1 ˜¯
798.53
or T5s = = 653.38 K
1.222
Analysis Analysing each device of the cycle separately
in steady state. The isentropic efficiency of the nozzle jet is given by
Eq. (28.24);
Compressor Temperature T2s after isentropic compres- Final K.E. of jet
sion hNozzle =
g -1 Isentropic enthalpy drop + K.E.
( )g
T2s = T1 rp carried oveer turbine
1.4 -1 2
VJet
Ê 5ˆ 1.4
= 227.5 ¥ Á ˜ = 360.3 K = 2
Ë 1¯ 2
Vduct
C pg (T4 - T5 s) +
The isentropic efficiency of the compressor is given 2
by; 2
VJet
Isentropic work T2 s - T1 or 0.9 =
hC = = 2 ¥ 1087 ¥ (798.53 - 653.38) + 200 2
Actual work T2 - T1
or Vjet = 565.68 m/s
986 Thermal Engineering

The energy balance in nozzle is given by (v) Outlet area of the duct
Change in enthalpy = Change in KE Density of exhaust gas
2 2 p5 (0.306 ¥ 100 kPa )
VJet - Vduct r5 = =
Cp (T4 – T5) = RT5 0.287 ¥ 669.73
2
565.682 - 200 2 = 0.159 kg/m3
1087 ¥ (798.53 – T5) = Nozzle discharge area can be obtained from
2
Actual temperature after expansion in nozzle, continuity equation
T5 = 669.73 K ma (1 + mf ) = r5Ajet Vjet
Combustion chamber 9.838 ¥ (1 + 0.0145)
or Ajet = = 0.11 m2
Heat supplied per kg of air is given by 0.159 ¥ 565.68
qin = m f CV = (1 + m f ) Cpg (T3 – T2) (vi) Specific fuel consumption
or mf ¥ 42500 = (1 + mf) ¥ 1.087 ¥ (943 – 383.75) Thrust Power 750 ¥ 1000 W
F = =
42500 mf = 607.9 + 607.9 m f Flight Velocity 220 m/s
or mf = 0.0145 kg/kg of air = 3409.1 N
m 1 Mass-flow rate of fuel
(i) Air–fuel ratio = a = = 68.91
mf 0.0145 m f = ma ¥ ma = 9.838 ¥ 0.0145
(ii) Overall thermal efficiency of unit = 0.1426 kg/s
mf 0.1426
È( ma + m f ) Vjet - ma Va ˘ Va Thrust sfc = = ¥ 3600
hoverall = Î ˚ F 3409.1
m f CV = 0.1506 kg/kN-h
ÈÊ mf ˆ ˘
ÍÁ1 + ˜¯ Vjet - Va ˙ Va
ÍË m ˙˚
= Î
a
mf
¥ CV
ma
È(1 + 0.0145) ¥ 565.68 - 220 ˘˚ ¥ 220 A rocket or rocket vehicle (shown in Fig. 28.20) is
= Î a missile, which obtains thrust by the reaction from
0.0145 ¥ ( 42500 ¥ 1000 J/kg)
= 0.1263 or 12.63%
(iii) Mass-flow rate of air
Thrust power is given as
Thrustpo wer = Thrust ¥ Flight Velocity
= [( ma + m f ) Vjet – ma Va] Va
750 ¥ 1000 W
= [(1 + 0.0145) ¥ 565.68 –
220] ma ¥ 220
or ma = 9.838 kg/s
(iv) Power developed by the turbine
Turbine output
= ma (1 + mf ) Cpg (T3 – T4)
= 9.838 ¥ (1 + 0.0145) ¥ 1.087
¥ (943 – 798.53)
= 1567.4 kW
Jet and Rocket Propulsions 987

the ejection of fast-moving exhaust fluid from a 5. It needs lots of propellant and has very low
rocket engine. They are not suitable for low-speed specific impulse; typically 100–450 seconds.
use. As compared to other propulsion systems, 6. It offers extreme thermal stresses of
they are very light in weight and powerful, capable combustion chamber.
of generating large accelerations and of attaining 7. Carrying oxidiser on-board makes the rocket
extremely high speeds with reasonable efficiency. a very risky vehicle.
8. It is extremely noisy.

A rocket is a non-air-breathing engine and it has


a few moving parts. It consists of a combustion A rocket vehicle can be classified as following:
chamber and exhaust nozzle. It carries fuel and
T P Used
oxidiser (such as liquid oxygen) on the board of
the craft. The fuel and oxidiser react chemically (a) Solid propellent rocket
in the combustion chamber and then high-pressure (b) Liquid propellent rocket
combustion gases act as rocket propellant. (c) Hybrid propellent rockets
These gases expand through the nozzles, and are
F Used
accelerated to extremely high speed to exert a large
reactive thrust on the rocket (since every action has (a) Chemical fuel rocket
an equal and opposite reaction). (b) Nuclear fuel rocket
(c) Solar rocket
Applications (d) Electrical rocket
Rockets are mostly used to take the space vehicle N Stages
outside the earth’s atmosphere, to launch artificial (a) Single-stage rocket (with one motor)
satellites in the space, human space flight and for (b) Multi stage rocket (with more than one
exploration of other planets. Rockets are also used motor)
for fireworks and weaponry ejection seats.
Chemical rockets store a large amount of ange
energy in an easily released form, and can be (a) Short-range rocket
very dangerous. However, careful design, testing, (b) Medium-range rocket
construction, and use minimizes risks. (c) Long-range rocket
A
Features
(a) Weather-forcasting rockets
1. It is a self-contained, non-air-breathing
(b) Military rockets
system.
(c) Space explorer rockets
2. Rockets are highly efficient at very high
speed (> Mach 5.0 or so). (d) Booster rockets used in multistage rockets to
3. It develops high thrust per unit area. Thrust- elevate the main rocket
to-weight ratio is over 100. (e) Retro rockets used to reduce the speed of
4. It has simple air inlet and high compression the main rocket and are fired in opposite
ratio. direction of the main rocket
988 Thermal Engineering

nozzle. The solid fuel and oxidiser are mixed in a


single propellant and is packed inside the shell as
shown in Fig. 28.21. A liner is provided between
Sr. No. Jet Engine Rocket engine
shell and propellant in order to protect the shell
1. It is an air-breathing It is a non-air- against high temperature.
engine. breathing engine.

Propellent
2. Oxygen supply It carries oxidiser

oxidizer)
(Fuel +
Liner
depends on on board, thus

A
Ignitor
atmospheric oxygen supply is not

Exhaust gases
conditions. It carries affected by outside

Nozzle
only fuel. atmosphere.
3. Intake air supplies Oxidiser supplies
the oxygen required the oxygen for

Combustion
for combustion. combustion. Shell

chamber
A
3. It cannot be operated It can be operated in Exhaust Jet

in vacuum. vacuum efficiently.


Space tavel is
possible.
4. Thrust produced Production of thrust An ignitor, which is a detonator or fuse containing
depends on altitude does not depend on a highly reactive explosive material is used to start
and flight speed. altitude.
the burning of the propellant. The burning occurs
Thrust decreases
with rapid decomposition of propellant and heat
with altitude.
release. The generated combustion products are
5. Skin friction It offers no discharged through the nozzle with very high
increases with flight surface drag. No
velocity. Thus, a reactive thrust is developed and
speed. gravitational effect.
the rocket is propelled in forward direction.
Rate of climb
increases with Once the heat is produced, it sustains the
altitude. combustion without any ignitor. The burning rate
6. Flight velocity is Flight speed is
depends upon the pressure of the chamber and
aways less than jet unlimited, even incrseases with an increase in pressure.
velocity. greater than jet
velocity. Applications
7. It has reasonable Low efficiency, The solid propellant rockets are used in the
thermal efficiency except at extremely following applications:
and reasonable flight high speed.
duration. 1. Assisted take off of missiles and projectiles
2. Small range rocket missiles
PROPELLANT
LIQUID

Rockets which use liquid fuels along with oxidizers


The rockets, which use solid fuels and oxidisers
are called liquid propellant rockets. A liquid
are known as solid propellant rockets. A solid
propellant rocket consists of a fuel tank, an oxidizer
propellant rocket consists of a star-shaped
tank, fuel pump, oxidizer pump, injector, steam
combustion chamber, propellant and an expansion
Jet and Rocket Propulsions 989

turbine, preheater, a combustion chamber, various


control valves and an expanding nozzle as shown in
Fig. 28.22. The fuel and oxidizer are stored at very
low temperature.

The fuel and oxidizer are pumped separately


into the mixing chamber through valves, where
the mixture is preheated to a suitable temperature.
Then the preheated fuel oxidizer mixture is injected
into the combustion chmber, where the mixture is
ignited by an electric torch. The pumps are driven by
a steam turbine. The steam is produced by mixing
a very high concentrated hydrogen paraoxide with
potassium permanganate.
The high-pressure and high-temperature com- injected on the fuel in the combustion chamber. The
bustion products are discharged through the nozzle oxygen reacts with the fuel and combustion starts
with a high supersonic exit velocity. The high vel- autimatically. The high-temperature combustion
city exhaust propels the rocket in the forward direc- products escape with a very high velocity through
tion. the nozzle, thus propelling the rocket in forward
direction.

A hybrid rocket is a rocket with a rocket engine


which uses propellants in two different states of Nuclear rocket uses nuclear fuel. The nuclear
matter—one solid and the other, either gas or liquid. fuel undergoes a fusion reaction for generation
In most of the cases, solid fuel along with liquid of energy. The principal components of a nuclear
oxidizer is used. Fig. 28.23 shows the schematic propellant system are shown in Fig. 28.24. It
arrangement of hybrid propellant rocket. consists of a propellant tank, pump, thrust chamber,
The solid fuel is packed in the combustion nozzle and a pressure shell. The thrust chamber
chamber. The liquid oxidizer is stored in a separate consists of nuclear reactor, a source of energy.
tank. Liquid oxidizer is pumped through a valve and The nuclear reactor consists of fuel element uranium
990 Thermal Engineering

Demerits
1. Nuclear reaction releases the heat energy
at a very high rate. Thus it requires a heavy
radiation shield.
2. A nuclear reactor emits large radiaton rays,
which are harmful to material and persons.

The propellant is a combination of fuel and oxidiser


(such as liquid oxygen) on the board of a craft.
There are
1. Solid propellent rockets
2. Liquid propellent rockets
3. Hybrid propellent rockets

Solid Propellants
Two types of solid propellants are used in rockets.
These are the following.

235, a moderator to slow down the fast-moving These are mixture of


neutrons, a reflector for conserving the neutrons and organic substances like nitroglycereine, cellulose,
control rods to control the reaction rate. nitrates, etc. The compounds act as a plastic and
give a fuel of colloidal charecteristic. Additives
The propellant is the working fluid. It is liquid
are added to impart stability of combustion and
hydrogen. A hydrogen pump driven by a gas
freedom from ageing.
turbine, transports the propellant from the tank to
the thrust chamber. The hot gases from the reactor Characteristics of a homogeneous propellant are
expand in the turbine to generate the power. as follows:
The heat generated in the reactor is transferred 1. These are plastic in nature.
to hydrogen, and subsequently it is expanded in the 2. They contain very high viscosity.
nozzle to a high ejection velocity, and consequently, 3. They appear smooth and waxy.
the thrust is produced to propel the rocket in the 4. These can be made in required shape by
forward direction. casting and extrusion.
The merits and demerits of nuclear propellant
rocket are as following. These are also called
composite propellants. For hetrogeneous solid
Merits propellants, nitrates, potassium and perchlorates
1. Nuclear rocket produces high specific are used as oxidisers and polymers, polyvinyl
impulse. Compared to other type of rockets, chloride (PVC) are used as fuel along with some
a nuclear rocket produces maximum specific additives to regulate the burning rate.
impulse. Characteristics of a heterogeneous propellants
2. Hydrogen is a light-weight gas, thus the self- are as follows:
weight of the rocket is relatively less.
1. They are difficult to cast.
Jet and Rocket Propulsions 991

2. They have high oxidiser content. 4. It produces low specific impulse and cannot
3. The exhaust has high density. be reused.
4. Mechanical properties of a propellant 5. It erodes the nozzle since it cannot be cooled.
depends on the nature of the binder.
Liquid Propellants

1. The chemical reaction between fuel and Liquid propellants are carried under the pressure as
oxidser during combustion must release a cryogenic liquids. These are classified as
large amount of heat energy, giving higher (a) Monopropellants, and
combustion temperature and specific (b) Bi-propellants
impulse.
When fuel and oxidizer are
2. The physical and chemical properties should
present in a single chemical compound or solution,
not change during combustion.
then the propellant is called monopropellant. The
3. The propellant must have lower molecular commonly used monopropellants are acetylene,
weight, higher density. hydrazine, ethylene oxide, and hydrogen peroxide.
4. It should be non-corrosive, non-poisonous The monopropellants are suitable for auxiliary and
and hazardous. turbopump plants in a rocket engine. These rocket
5. The propellant should be chemically inert. engines are smaller and simple in construction.
6. The product of combustion should be
When the fuel and oxidizer
smokeless and colourless.
are different from each other in its chemical
7. It should be easily available.
composition then the propellant is called a bi-
propellant. The fuel and oxidiser are mixed in a
combustion chamber. This propellant is used for
long-range and long-duration flights.
1. These are simple in design and easy to Commonly used bi-propellants (liquid fuel and
manufacture. oxidizer combination) are liquid hydrogen, + N2O3,
2. They do not require mechanical feed system. demethyl hydrazine + hydrogen peroxide, gasolene
+ nitric acid, ethanol + liquid O2, etc.
3. Density of fuel is high, the design is compact.
4. Due to absence of moving and sliding parts,
the vibrations are almost absent. L
5. Maintenance problem is less.
6. They are suitable for short-range applica- 1. More flexible and greater control over the
tions. propellant supplied and thus control on
thrust developed.
1. Once combustion begins, it cannot be 2. Propellants are not stored in a combustion
stopped or controlled. chamber.
2. Since it contains oxidiser with it, the storage 3. It gives high specific impulse.
and transportation require utmost care. 4. It is much easier to stop the operation to
3. Malfuctioning or accident leads to an avoid a catastrophe.
abandon of the project. 5. Auxiliary power plant can easily be operated.
992 Thermal Engineering

6. Liquid propellant rockets are more eco- less plumbing, fewer valves, and simpler
nomical for long-range space and military operations.
operations. 2. Hybrid fuels are in the solid phase, generally
have higher density than those in the liquid
1. Liquid propellant rockets consist of a large phase.
number of parts and thus the design of liquid 3. High-energy metal additives, such as
propellant rockets is more complicated. aluminum, magnesium, lithium or beryllium
2. The liquid propellants are stored at very low can be easily included in the fuel grain for
temperature with very heavy insulation. increasing specific impulse (Isp) .
3. Feed pump and insulated pressurised tanks
increase the total weight of the rocket.
4. Due to low density of liquid propellants 1. Higher theoretical specific impulse (Isp) and
as compared to solid propellants, the store the rocket can carry high payload.
space required is more. 2. Less explosion hazard—Propellant grain has
more tolerance for processing errors such as
A good liquid propellant should have the following cracks.
characteristics. 3. Easy controllable—start/stop/restart and
throttling are all achievable by regulating the
1. It should have very high calorific value. supply of oxidizer.
2. It should have very high density, so that it
4. Safe and non-toxic oxidizers such as liquid
can be stored in a small space.
oxygen and nitrous oxide can be used.
3. It should be easy to store and handle.
5. The propellant can be recharged and reused.
4. It should be non-corrosive and stable.
5. Its combustion should be smooth and
uniform.
1. Oxidizer-to-fuel ratio shift (O/F shift) with
6. It should have minimum change in its
a constant oxidizer flow-rate, the ratio of
viscosity with temperature change.
fuel production rate to oxidizer flow rate
will change as a grain regresses. This leads
to off-peak operation from a chemical
Beryllium, hydride, lithium, polythene are the performance point of view.
fuels used for hybrid propellant rockets and chloro
2. Low regression-rate (rate at which the solid
triflourine, nitrogen tetraoxide are oxidisers. The
phase recedes) fuels often drives multiport
hybrid propellants have the following advantages
fuel grains. Multiport fuel grains have poor
and disadvantages.
volumetric efficiency and, often, structural
Hybrid rocket engines exhibit some advantages
deficiencies.
over liquid-fuel rockets and solid rockets. A brief
summary of some of these is given below: 3. Hybrid propellant erode the nozzle at the
faster rate.
4. If chlorine trifluorine is used as oxidant, it
Liquid
harms the ozone layer in the stratosphere.
1. These are mechanically simpler, require
only a single liquid propellant resulting in
Jet and Rocket Propulsions 993

28.17 ANALYSIS OF

Thrust
PROPULSION

The thrust developed in a rocket engine is


sum of momentum thrust and pressure thrust.
F = Momentum thrust + Pressure thrust
The momentum thrust is caused due to Altitude
difference in flow velocity of propellant entering
the combustion chamber and high velocity hot
gases leaving the exhaust nozzle and is given by
The specific thrust is given as ratio
Newton’s second law of motion, i.e.,
of thrust deveoped to mass-flow rate of propellant.
Fmom = m(Vjet – Vi )
Thrust F
where m = mO2 + m f (mass rate of fuel and Specific thrust = = ...(28.26)
Mass flow rate m
oxidizer). Since oxygen and fuel are stored within
rocket itself, thus its entry velocity relative to the The specific impulse is defined
aircraft is zero (Vi = 0). Thus, the momentum thrust as the ratio of thrust developed to weight of
is propellant, which passes through the rocket engine;
Fmom = m Vjet Vjet
Thrust
Isp = = ...(28.27)
Weight of propellant g

It is the product of specific impulse


and weight flow rate of propellant used. It is given
by
Itotal = Isp ( m g )
VJet
= ¥ mg = m VJet = F ...(28.28)
g
The ambient pressure acts over the entire
It is equal to thrust developed.
surface of the rocket. Let pe be the exit pressure
from nozzle, that is higher than the pressure acting Thrust P The power developed from thrust of
on frontal area. Then the pressure thrust acting in the engine is called the thrust power Wth, which is
forward direction the product of propulsive force and aircraft velocity
FPr = ( pe – pa) Ae Va. That is,
The total thrust produced in a rocket engine Wth = F Va = m Vjet Va ...(28.29)
F = m Vjet + ( pe – pa) Ae ...(28.25) Theoretical
In rocket propulsion, the thrust is independent power developed by the rocket engine is the sum of
of flight velocity. The nozzle exit pressure remains thrust power and power loss in exhaust gases
constant. Therefore, the thrust increases with
altitude on account of decrease in ambient pressure Propulsive Power
pa up to certain altitude. Then the ambient pressure = Thrust power + Power loss in exhaust
variation becomes negligible and thrust becomes
constant as shown in Fig. 28.26.
994 Thermal Engineering

Power lost due to energy carried by exhaust


1
=m ( Vjet - Va ) 2
2
\ Propulsive Power
1
= m Vjet Va + m (VJet – Va)2
2
1 2
= m Vjet Va + m (V Jet + V a2 – 2VJet Va)
2
1 2
= m ( VJet + Va2 ) ...(28.30)
2
It is defined as the ratio of total
thrust developed F to pressure force in combustion The thermal efficiency of a
chamber. rocket engine is defined as
Total Thrust F Propulsive
CF = = ...(28.31) hth =
Pressure Force p0 A* Heat supply rate
where 1 2
m ( VJet + Va2 ) 2
VJet + Va2
p0 = pressure in combustion chamber, = 2 = ...(28.33)
and A* = area of throat of nozzle. mCV 2CV

It is defined as the ratio of It is expressed


thrust power to propulsive power. as weight of propellant used to produce a thrust of
Thrust power 1 Newton. It is denoted as spc and mathematically
hprop = given as;
Propulsive power
Ê V ˆ Weight of propellant
spc =
2Á a ˜ Thrust produced
ma ¥ Vjet ¥ Va Ë Vjet ¯
= = mg 1
1 2 Ê V2 ˆ = = ...(28.34)
ma ¥ ( Vjet + Va2 ) 1 + Á a2 ˜ F I sc
2 Ë VJet ¯
The effective jet velocity from a rocket
2s
or hprop = ...(28.32) engine is 3000 m/s. The forward velocity is 1500 m/s and
1 + s2 propellant consumption is 80 kg/s. Calculate the thrust,
Va thrust power and propulsive efficency.
where s = speed ratio =
Vjet
The variation of propulsive efficiency with
Given A rocket engine
speed ratio is shown in Fig. 28.27. The propulsive
VJet = 3000 m/s
efficiency reaches maxmimum, when speed ratio is
Va = 1500 m/s
unity and for other cases: For
m = 80 kg/s
s = 0; hprop = 0,
s < 1; hprop increases with increase in speed To find
ratio. (i) Thrust developed,
s > 1; hprop decreases with increase in speed (ii) Thrust power, and
ratio from 1. (iii) Propulsive efficiency.
Jet and Rocket Propulsions 995

Analysis (ii) Actual jet velocity


(i) The thrust produced Since nozzle exit pressure pe is greater than
F = m Vjet = 80 ¥ 3000 = 240,000 N ambient pressure pa. Thus pressure thrust will
= 240 kN also act on nozzle
(ii) Thrust power Pressure thrust,
Thrust power = F Va = 240 ¥ 1500 p 2
Fpr = Ae ( pe - pa ) = d ( pe - pa )
= 360,000 kW or 360 MW 4
p
(iii) Propulsive efficiency = ¥ (0.6) 2 ¥ (1.1 - 0.6) ¥ 1 03
4
2s
hprop = = 141.37 kN
1 + s2
Momentum thrust,
Va 1500
where s = = = 0.5 Fmom = F – Fpr = 380 – 141.37
VJet 3000
= 238.63 kN = 238.63 ¥ 103 N
2 ¥ 0.5
\ hprop = = 0.8 or 80% Actual jet velocity;
1 + 0.52
Fmom 238.63 ¥ 103
The following data refer to a Vexit = =
m 200
rocket engine. Propellant flow rate is 200 kg/s. The = 1193.15 m/s
thrust chamber pressure is 37 bar and its temperature (iii) Specific impulse
is 3000 K. The nozzle exit diameter is 0.6 m, nozzle
F 380 ¥ 103
exit pressure is 1.1 bar, ambient pressure is 0.60 bar, Isc = = = 193.68 s
and thrust produced is 380 kN. Calculate effective jet mg 200 ¥ 9.81
velocity, actual jet velocity, specific impulse and specific (iv) Specific propellant consumption
propellant consumption. 1 1
spc = = = 5.16 s–1
I sc 193.68

Given A rocket engine A rocket engine refer to following


m = 200 kg/s p2 = 37 bar data:
T2 = 3000 K d = 0.6 m Jet velocity = 1600 m/s
F = 380 kN pe = 1.1 bar Flight to jet speed ratio = 0.7
pa = 0.6 bar Oxidizer flow rate = 4 kg/s
Fuel flow rate = 1 kg/s
To find Heat of reaction per kg of exhaust gase = 2500 kJ/kg
(i) Effective jet velocity, Calculate the thrust, specific impulse, propulsive
(ii) Actual jet velocity, efficiency, thermal and overall efficiency of rocket engine.
(iii) Specific impulse, and
(iv) Specific propellant consumption.
Analysis Given A rocket engine
(i) Effective jet velocity VJet = 1600 m/s s = 0.7
When combustion gases expands in the nozzle mO2 = 4 kg/s m f = 1 kg/s
up to ambient pressure, without pressure thrust, DhR = 2500 kJ/kg
resulting into effective jet velocity. To find
F = m Vjet
(i) Thrust developed,
or 380 ¥ 103 = 200 ¥ Vjet
(ii) Specific impulse,
or Vjet = 1900 m/s
(iii) Propulsive efficiency,
996 Thermal Engineering

(iv) Thermal efficiency, and (iv) Thermal efficiency


(v) Overall efficiency. Propulsive power developed
hth =
Analysis Flight velocity Heat Supplied
Va = s Vjet = 0.7 ¥ 1600 = 1120 m/s 1 2
Propulsive power = m ( VJet + Va2)
(i) Thrust produced 2
1
F = m Vjet = ( mO2 + m f ) Vjet = ¥ 5 ¥ (1600 2 + 1120 2 )
= (4 + 1) ¥ 1600 = 8000 N 2
= 8.0 kN = 9536000 W = 9536 kW
(ii) Specific impulse Heat supplied = m DhR = 5 ¥ 2500 = 12500 kW
F 8000
Isc = = 9536
( mO2 + m f ) g 5 ¥ 9.81 \ hth = = 0.7628 or 76.28%
12500
= 163.1 s (v) Overall efficiency
(iii) Propulsive efficiency hoverall = hprop ¥ hth= 0.9396 ¥ 0.7628
2s = 0.6885 or 68.85%
hprop =
1 + s2
2 ¥ 0.7
= = 0.9396 or 93.96%
1 + 0.72

jet propulsion engine is a reaction engine velocity difference of gases leaving and air
that discharges a fast-moving jet of fluid which entering the aircraft, while the rest of the thrust is
generates thrust in the opposite direction of a provided by the propeller.
jet to propell the aircraft in accordance with turbofan engine is the combination of turbojet
Newton’s second and third laws of motion. and turboprob engines. It consists of a ducted
ramjet engine uses the engine’s forward fan which is powered by a gas turbine. The
motion to compress incoming air without a combination of thrust produced from the fan and
rotary compressor. Fuel burns to heat the air and the exhaust from the core engine is more efficient
hot gases expand through the nozzle to produce than other jet engine designs.
thrust.
pulsejet engine combustion occurs in pulses. exit is given by
A pulsejet engine develops the thrust by a high = (Vjet – Va) m/s
velocity of jet of exhaust gases without use of Thrust = ma (Vjet – Va) N
compressor or turbine. Ê Vjet
2
- Va2 ˆ
scramjet is a supersonic combustion ramjet. It Propulsive power = ma Á ˜
ÁË 2 ˜¯
operates with supersonic speed at Mach numbers,
12 to 24. 2Va
hprop =
turbojet engine is propelled by the thrust Vjet - Va
produced due to acceleration of hot combustion
gases through the exhaust nozzle. Propulsive work
hth =
turboprop engine, the propelling nozzle Heat supplied
provides approximately 20 per cent thrust by hoverall = hth ¥ hprop
Jet and Rocket Propulsions 997

rocket engine, the thrust produced is the Theoretical power developed by the rocket engine
sum of momentum thrust and pressure thrust. is the sum of propulsive power and power loss in
F = m Vjet + ( pe – pa) Ae exhaust gases
where Fmom = m Vjet PropulsivePo wer
m = mO2 + m f = Thrust power + Power loss
FPr = ( pe – pa) Ae in exhaust
1 2
Specific thrust =
Thrust
=
F \ Engine Power = m ( VJet + Va2)
Mass-flow rate m 2
Propulsive efficiency of a rocket engine is given
Specific impulse;
by
Thrust Vjet 2s
Isp = = hprop =
Weight of propellant g 1 + s2
Va
where s = speed ratio =
Vjet

Jet propulsion Propelling of a vehicle due to thrust Turbojet engine An engine in which thrust is produced
produced by fast-moving gases at the rear end due to acceleration of hot combustion gases through the
Ramjet engine An engine which uses engine’s forward exhaust nozzle
motion to compress the incoming air Turboprop engine A turbojet engine in which a
Pulse jet engine An internal combustion jet engine in propeller is coupled to the turbojet engine
which combustion occurs in pulses

1. What are the principles of jet and rocket 9. What is the difference between a propeller engine
propulsion? and jet engine.
2. Explain the working of a ramjet engine with 10. Why are propeller engines not commonly used
the help of a sketch. What are its advantages, nowdays in aircrafts?
disadvantage and applications? 11. Define the following terms as applied to jet
3. Explain the working of pulsejet engine with the propulsion: specific thrust, specific impulse,
help of a sketch. thrust-specific fuel consumption, thrust power
4. Explain the working of a turboprop engine with and propulsive power.
the help of a sketch. 12. Why is thrust augmentation necessary? What are
5. Explain the working of a turbofan engine with the the methods for thrust augmentation in a turbojet
help of a sketch. engine?
6. Explain the working of a turbojet engine with 13. Why does a ramjet engine not require a
the help of a sketch. What are its advantages, compressor or turbine?
disadvantages and applications? 14. Prove that the propulsion efficiency of a rocket
7. State the difference between jet propulsion and motor is obtained as
rocket propulsion. 2s
hprop =
8. State the difference between air breathing and 1 + s2
Va
non-air breathing propulsion systems. where s = speed ratio = .
Vjet
998 Thermal Engineering

15. Explain briefly with a sketch the working 19. What are desirable properties of a liquid
principle of a rocket. propellant for a rocket engine?
16. List out the applications of rockets. 20. What are the advantages of using nuclear thermal
17. What is a propellant? How are propellants rocket for space propulsion?
classified?
18. What are composite and homogeneous solid
propellant? How do they work? State their merits
and demerits.

1. The diameter of an aircraft is 4 m. The speed ratio efficiency of the compressor is 82%. The
is 0.8 at a flight speed of 450 kmph. If the ambient maximum temperature of the cycle is 1023 K.
conditions of air at flight altitude are 0.54 bar The isentropic efficiency of the turbine is 88%.
and 256 K. Calculate (a) propulsive efficiency, The turbine is used to drive the compressor. A
(b) thrust, (c) thrust power. nozzle having isentropic efficiency 90% is used
[(a) 0.888, (b) 40.5 N, (c) 5073 N] to expand the gases up to a pressure of 100 kPa.
2. A turbojet is flying with a speed of 850 kmph at Calculate the power developed by the turbine to
an altitude, where air density is 0.17 kg/m3. The run the compressor per kg of air. Neglect the mass
propulsive and overall efficiencies are 55% and of fuel used.
17% respectively. If the drag on aircraft is 6100 N, Also calculate air–fuel ratio, if calorific value is
calculate the exit velocity of jet , diameter of jet 42000 kJ/kg, pressure of gases leaving the turbine
and propulsive power. and thrust developed.
[622.5 m/s, 29.24 cm, 1.44 MW] [171.54 kJ/kg, 74, 172 kPa,466 N]
3. An aircraft flying at 241 m/s at an altitude, at 6. A twin jet aircraft travels at 200 m/s at an
which pressure is 0.46 bar and temperature is altitude where, the density of atmospheric air is
–30°C. Calorific value of fuel is 41820 kJ/kg. 0.1712 kg/m3. The aircraft has to overcome a total
The diffuser isentropic efficiency is 100%. The drag force of 6376 N. If propulsion efficiency is
isentropic efficiency of compressor is 80%. 58%, find the diameter of jet. [40.8 cm]
Compressor pressure ratio is 5. Calculate the 7. A turbojet has a speed of 750 km/h, while flying
air–fuel ratio, if maximum temperature must not at an altitude of 10,000 m, where density of air is
exceed 874°C. Assume Cp = 1.005 kJ/kg ◊ K. 0.173 kg/m3. The propulsive efficiency of the jet
[61.4] is 50% and overall efficiency of the unit is 16%.
4. In a jet propulsion unit, the compressor pressure The drag on the plane is 6200 N. Calorific value
ratio is 3.5. The temperature rise during of the fuel is 48000 kJ/kg. Calculate
compression is 1.2 times that of isentropic (a) Absolute velocity of jet,
compression. The maximum temperature of the (b) Quantity of air compressed per minute,
cycle is 753 K. The gases are expanded in the (c) Diameter of jet,
nozzle to 283 K and 1.01325 bar. Calculate the (d) Net power output of the unit in kW,
power required to drive the compressor per kg (e) Air–fuel artio.
of air, thrust developed, air–fuel ratio, if calorific [(a) 417.3 m/s, (b) 5160 m3/min, (c) 41.5 cm,
value of fuel is 43000 kJ/kg.
(d) 2500 kW, (e) 46]
[146.73 kJ/kg, 402 N, 132]
8. A turbojet flies with a speed of 800 km/h at an
5. In a jet propulsion engine, the air is compressed ambient pressure of 60 kPa. The properties of
from 100 kPa, 288K to 400 kPa. The isentropic
Jet and Rocket Propulsions 999

gas entering the nozzle are 300 kPa and 200°C. 11. The following data are applicable to a jet unit
The mass-flow rate is 20 kg/s. assuming air as flight:
working fluid. Find the thrust developed, thrust Speed of airplane = 950 km/h
power and propulsive efficiency. Take g = 1.4 and Stagnation pressure and temperature
R = 0.287 kJ/kg ◊ K. at turbine exit = 180 kN/m2 and 800 K
[7.39 kN, 1643.5 kW, 54.6%] Atmospheric pressure = 60 kN/m2
9. The diameter of jet of a turbojet is 2.5 m and Nozzle efficiency = 98%
it flies at 500 km/h at an elevation of 8 km for Mass flow rate of air = 25 kg/s
flight to jet speed 0.75. The density of air is
Fuel air ratio = 0.018
0.525 kg/m3. Calculate (a) mass flow rate of air,
Lower calorific value
(b) thrust, (c) specific thrust, (d) specific impulse,
of fuel = 40000 kJ/kg
and (e) thrust power.
Calculate
[(a) 477.2 kg/s, (b) 22.1 kN, (c) 3.07 kW,
(a) gross and net thrust,
(d) 46.3 N/kg/s, (e) 4.72 s]
(b) jet equivalent velocity, and
10. A turbojet engine flies with a speed of 880 km/h.
Mass-flow rate of air is 50 kg/s. The isentropic (c) propulsive, thermal and overall efficiencies.
change in enthalpy for a nozzle is 188 kJ/kg and 12. A rocket flies with a velocity of 2800 m/s with an
its velocity as constant as 0.96. The fuel–air ratio effective jet velocity of 1400 m/s and propellant
is 1.2%. Calorific value of the fuel is 44 MJ/kg. flow rate of 5 kg/s. If the heat of reaction of
Determine (a) thermal efficiency, (b) fuel flow propellant is 6500 kJ/kg, calculate propulsive
rate, (c) propulsive efficiency, and (d) overall power, propulsive efficiency, engine output,
efficiency. thermal efficiency and overall efficiency.
[(a) 27.5%, (b) 0.6 kg/s, (c) 58.7%, [19.6 MW, 80%, 24.5 MW, 75.4%, 60.3%]
(d)16.13%]

Objective uestions
1. A jet engine has (c) Injecting methanol into compressor
(a) no propeller (b) propeller at front (d) All of the above
(c) propeller at back (d) propeller on top 5. A turboprop is preferred to a turbojet engine
2. The efficiency of a jet engine is higher at because
(a) low ps eed (b) high ps eed (a) it has high propulsive efficiency at high
(c) low altitude (d) high altitude speed
3. The thrust is calculated as (b) it can fly at supersonic speed
(a) Vjet – Va (c) it can fly at high altitude
(b) ma (Vjet – Va) (d) it has high power for take-off
2Va 6. In a jet propulsion unit, the product of combustion
(c) after passing through the gas turbine are
Vjet + Va
discharged into
(d) ma (Vjet – Va) Va
(a) atmosphere
4. The thrust of a jet engine can be increased by
(b) Vacuum
(a) Injecting water into the compressor
(c) discharge nozzle
(b) reheating gas after turbine
(d) back to compressor for next cycle
1000 Thermal Engineering

7. In a turbojet engine used in military aircraft, (b) compressed atmospheric air


the exhaust gas from the turbine is reheated by (c) oxidizer on board
burning of additional fuel to increase the (d) none of the above
(a) thrust and range of aircraft 13. A rocket engine can be propelled to space,
(b) efficiency of the engine because
(c) both (a) and (b) (a) it generates very high thrust
(d) none of the above (b) it has high propulsive efficiency
8. In an ideal Turbojet engine after heat addition (c) it can work on several fuels
to compressed air, the working substance is (d) it is not air breathing engine
expanded in 14. A rocket engine operates only in
(a) exit nozzle at constant pressure (a) ratified atmosphere (b) vacuum
(b) exit nozzle at isentropic manner (c) atmosphere (d) all of the above
(c) turbine blades at constant temperature 15. When a rocket is used as weapon with a warhead
(d) turbine blades in isentropic manner as payload, then it is called a
9. A jet engine uses oxygen for combustion of its (a) jet
fuel from (b) turbojet
(a) surrounding air (b) oxidizer (c) missile
(c) propellent (d) none of the above (d) turbo propeller unit
10. In a turbojet engine, the diffuser is fitted at 16. The propellent for a rocket engine is a
(a) nose (b) after compressor (a) jet carried by the rocket only
(c) after turbine (d) before urbine
t (b) Oxidizer carried by the rocket only
11. The propulsive efficiency is defined as (c) fuel and oxidizer carried by the rocket only
Vjet – Va2 2Va (d) none of the above
(a) (b)
2 Va Vjet + Va 17. Thrust in a rocket is generated by a jet of
Vjet + Va (a) air taken from the atmosphere
2Va
(c) (d) (b) exhaust gases coming out at high velocity at
Vjet – Va 2 Va
the rear
12. A rocket engine receives oxygen for combustion (c) compressed air
of fuel from (d) none of the above
(a) surrounding air

17. (b)
16. (c) 15. (c) 14. (d) 13. (d) 12. (c) 11. (b) 10. (a) 9. (a)
8. (d) 7. (a) 6. (c) 5. (d) 4. (d) 3. (b) 2. (b) 1. (a)
Answers
Air-conditioning 1001

29
Air-conditioning

Introduction
Air-conditioning is the process of treating air in an internal environment to achieve and maintain required
standards of temperature, humidity, cleanliness and motion of air, regardless of surrounding conditions.
However, in popular usage, the term air conditioning is often improperly referred for providing a cool
environment. The conditioning of air is controlled by
1. Temperature Air temperature is controlled by heating or cooling of air.
2. Humidity The humidity is controlled by adding or removing the water vapour to or from the air.
3. Purity The quality of air is controlled by filtration, removal of undesirable contaminants from air.
4. Motion Air velocity in the conditioned space is controlled by appropriate air distribution equipment.

(iii) transportation of goods and human beings,


(iv) operation theaters, ICU and hospitals,
(v) offices, libraries, threaters and auditoriums.
Most of the air-conditioning systems are used to
(b) Industrial Applications
(a) promote human health and comfort,
It is used in
(b) improve human efficiency and working con-
ditions, (i) laboratories where precision measurements
(c) keep materials and the products in the most are carried out
natural state, while in shortage or under (ii) printing and textile industries
production or manufacturing processes. (iii) pharmaceuticals companies during process-
Air-conditioning is nowadays becoming more ing of medicines
and more popular and is widely used in the follow- (iv) production and preservation of photographic
ing fields. materials
(a) Comfort Air-conditioning
(v) preservation of archeological and important
documents
It is used in
(vi) electronic data-processing units
(i) residential buildings, (vii) control rooms
(ii) consumer stores, hotels and restaurants,
1002 Thermal Engineering

The body convects heat to air immediately around


it self. The warm air moves away naturally and cooler
When the heat loss from the body is within certain air comes in which in turn receives heat. In radiation,
limits, it results in a feeling of comfort. If the rate of the body transmits heat through space directly to
heat loss is too large, as in a cold ambient, one feels the nearby walls and windows. A body can warm
cold. If the rate of heat dissipation is too low, one up in front of fire, even though the air between the
feels hot. The human body requires an environment fire and the body is very cold. Similarly, in a warm
for comfort which is neither very hot, nor very cold, room, one may feel chilly, if the ceiling or the wall
neither very humid nor very dry. surfaces are at a considerable lower temperature.
Human comfort also depends on both psycho- This is due to direct heat transfer between the body
logical and physiolocal conditions of the human and its surroundings.
mind. Therefore, when designing air-conditioning The human body is also cooled by evaporation of
systems for human beings, it is essential to under- water on the skin (perspiration) which absorbs heat
stand the thermodynamic aspect of a human body. from the body and cools it. But this perspiration
The factors which affect human comfort conditions does not help when humidity of air is close to 100%.
are the following: The heat loss from a human body mainly depend
on
1. Air (dry bulb) temperature
2. Air humidity
The human body can be viewed as a heat engine 3. Air motion
with thermal efficiency about 20%. The source of 4. Air purity
the heat engine is the energy input as food, and its
5. Air stratification
sink is the surrounding environment. The human
engine generates heat when the combustion of food 1. Temperature It is necessary to reject the heat at
starts by taking in oxygen from inhaled air. The the same rate at which it is generated in the body in
process of combustion in a human body is called order to keep the body temperature constant. Thus,
metabolism. The human body converts about 20% the environment temperature plays very important
of heat input into useful work and the remaining role in feeling of comfort. Most people feel a sense
heat generated must be rejected to the environment of comfort when the environment temperature is
to regulate the body temperature. Otherwise, the ac- 22°C to 25°C.
cumulation of heat in the body leads to discomfort.
2. Air Humidity Relative humidity affects the
The rate of heat generation in the body depends
heat-transfer rate through evaporation. High
on individual health and the level of activity. On
humidity slows down the heat dissipation rate
an average, an adult generates 87 W when sleeping,
and low humidity speeds it up. Most people feel
115 W when doing office work or resting, 230 W
comfort with a relative humidity of 40 to 60%.
when bowling and 440 W when doing heavy
physical work. The corresponding numbers in a 3. Air Motion Even if the temperature and humid-
female adult are about 15 per cent less. ity are within comfort range, a certain air move-
ment is essential for effective heat transfer from the
body. A high air motion increases the heat loss from
the body by convection and evaporation. It removes
The processes by which a body loses heat to the warm moist air that gathers around the body and
the surroundings are convection, radiation and replaces it with fresh air. Most people feel comfort
evaporation. with an air speed of 5 to 8 m/min.
Air-conditioning 1003

4. Air Purity People cannot feel comfort when


breathing in contaminated air, even if other pa-
rameters are satisfactory. Therefore, proper filtra-
tion, cleanness and purification of air is necessary Sl. Climate Range of Equivalent
No. effective room
to keep it free from dust, dirt and other impurities.
temperature conditions
Other factors that affect human comfort are odour,
noise and radiation effects. 1. Hot and dry 21.1 – 26.7°C 23.9°C and
50% RH
When heat is rejected from a 2. Hot and humid 22.0 – 25.6°C 24.2°C and
human body, the surrounding air gets heated. When 50% RH
air is heated, the density of air gets reduced and 3. Raining season 20.5°C – 24.0°C 23.0°C and
it rises to the ceiling of conditioned space. Hence 50% RH
within a confined space, there may be considerable 4. Winter 19.7 – 22.2°C 22.0°C and
temperature variation between the floor and ceiling. 45% RH
The movement of air from floor to ceiling is referred
as air stratification. 1. Climate and Seasonal Variation The people
living in a warmer region may feel comfort at
higher effective temperature than those living in a
colder region.
The effective temperature is defined as the index 2. Clothing People wearing light clothing need
which correlates the combined effects of air higher optimum effective temperature than a person
temperature, relative humidity and air velocity on wearing heavy clothing.
the human body. The numerical value of effective
temperature is equal to the temperature of saturated 3. Age and Sex Women of all ages require higher
air (f = 100%) moving with a velocity of 5 to 8 effective temperature than men. Children also need
m/min, which produces the same sensation of higher effective temperature than adults.
warmth and coldness as produced under given 4. Duration of Stay For shorter stay, higher
conditions. effective temperature is needed than for long stay.
Extensive tests have been conducted by the
American Society of Heating, Refrigeration and 5. Kind of Activity For heavy activities like
Air-Conditioning Engineers (ASHRAE) on differ- dancing or labour work in a factory, a low effective
ent kinds of people to test the level of comfort feel- temperature is required.
ing by varying the environmental temperature, rela- 6. Density of Occupation In densely occupied
tive humidity and air movement. The values were space like auditorium, the heat is radiated person
obtained on the feelings of a mass percentage of to person, thus required a slight lower effective
people and the results are presented in the form of temperature.
a comfort chart. The recommended effective tem-
perature and relative humidity with air velocity of
5 to 8 m/min, for Indian conditions are presented in
The comfort chart shown in Fig. 29.1, is a
Table 29.1.
graphical representation of a range of effective
temperature for comfort feeling for summer and
winter seasons. It shows dry-bulb temperature on
the horizontal axis, and wet-bulb temperature as
The factors which affect the optimum effective the ordinate. Relative humidity lines are replotted
temperature for comfort are the following:
1004 Thermal Engineering

from a psychrometric chart. The comfort chart 1. Continuous Supply of Oxygen and Removal of
3
reveals several combinations of wet and dry-bulb Carbon Dioxide A normal person inhales 0.65 m
temperatures with different relative humidity, of oxygen per hour, while producing 0.2 m3 of CO2
which will produce the same effect of comfort. under normal conditions. The presence of excessive
By looking at a comfort chart, one can decide the CO2 yields to discomfort and even unconsciousness,
effective temperature, relative humidity and other if it exceeds 10%. Hence, fresh air supply in a
physical aspects of the climate. conditioned space should be maintained.

2. Removal of Heat Continuously All the occu-


pants and equipment release heat continuously in a
conditioned space. This heat must be removed from
A person can feel comfort in an air-conditioned the conditioned space at the same rate at which it is
space if the following conditions are satisfied.
0.07–0.13 m/s 100% 90% 80% 70%
34
es 60%
lin
98% 94% on
i
34% 70% r at
tu
94% 50% Sa 50%
86% 27°
28
75% 40%
60%
24° es
34% lin
ty
idi
m 30%
hu
e
C tiv
Rela
22
20%
Effective G
Wet bulb temperature

temperatures 19°

D B 10%
Central 27°
16°
16 zone
H
F
13° 24°

22°
A

10
E
50%
65%
le” s

17° 75%
ab on
ort ers

16° 84%
mf f p

48%
“co ent o

91%
4
65%
rc

13° 81% 97%


Pe

10 16 22 28 34 40
Dry-tulb temperature (°C)
Air-conditioning 1005

generated in order to maintaine the temperature of


the conditioned space as constant.

3. Removal of Moisture Content of Air Human


beings also dissipate some water vapour from
their bodies as a result of evaporation. These wa-
ter vapours increase the humidity level in the
conditioned space. High humidity in air reduces
comfort level. Therefore, the relative humidity of
the conditioned space should be kept constant by
removing the moisture, which is added by the oc-
cupants.

The air circuit involves two air-flow ducts as


1. A supply duct for circulation of treated air
An air-conditioning system may provide heating, inside the house
cooling or both. An air-conditioning system controls 2. A return duct for flow of air through a
the conditions of a confined space to produce a condenser outside the house
comforting effect over occupants. The size and
The fan forces the conditioned air into the supply
complexity may range from a window unit for a
duct, which is connected to an opening in the room.
small room to a large system for a building complex,
The duct directs the air to the room through its
but the basic principle is same. The air-conditioning
outlets. The air enters the room and creates either a
systems can be classified into various types based on
heating or cooling effect as required.
the following heads.
After use, the air is drawn by a blower from the
1. According to Season room through the return duct and passes over the
(a) Summer air-conditioning system self-cleaning filter and secondary filter (electrostatic
(b) Winter air-conditioning system or fabric type) to remove the dust particles, and
(c) Year-round air-conditioning then passes over the evaporator (or heating coil).
2. According to Purpose
The hot air comes in contact of the cooled fins
and tubes of the evaporator, gets cooled, and the
(a) Comfort air-conditioning system refrigerant in the evaporator vaporizes. The clean
(b) Industrial air-conditioning system cooled air is then again sent to the air-conditioned
3. According to Arrangement of Equipment space through the supply ducts for the next cycle.
(a) Unitary air-conditioning system
(b) Central air-conditioning system
The refrigeration cycle circulates the refrigerant
between the inside and outside units.The refriger-
ant vapour coming out of the evaporator enters the
An air-conditioning cycle as shown in Fig. 29.2, compressor, where it is compressed and delivered
consists of fans, filters, a refrigerating plant, to the condenser. Another blower forces the atmo-
ducting and control units. The working of an air- spheric air over the condenser; thus the refrigerant
conditioning cycle can be divided into air circuit is cooled and condensed. The hot air is exhaust-
and refrigeration circuit separately. ed outside and the condensed refrigerant passes
through the expansion device to the evaporator.
1006 Thermal Engineering

condenses the water vapour into liquid water and


is collected in the swap. The air is then passed over
heating coil for mild heating before entering into
In summer, the comfort conditions are achieved conditioned space by using a circulating fan.
by cooling and dehumidification of air. The dry The conditioned air provides required comfort
bulb temperature of air entering the conditioner is conditions within confined space. A portion of air
quite high and thus it is essentially cooled by using is exhausted to the atmosphere and the remaining
cooling coils. The hot air also has high moisture air is used for recirculation with fresh air for the
contents; therefore, the removal of moisture is next cycle.
achieved by a dehumidifier. The processes of cooling, dehumidification
A summer conditioning system is shown in and heating are shown on the psychrometic plot in
Fig. 29.3. The fresh air from the surrounding Fig. 29.3(b).
atmosphere is mixed with re-circulated air in
suitable proportion and then the mixture is passed
through the filter to remove dust, dirt and other
unwanted materials from air to enter the system. The
air is then cooled sensibly with the help of a cooling In winter, comfort conditions are achieved by
coil. This cooled air is passed through a perforated heating and humidification of air. The dry-bulb
membrane to remove the moisture from air. The temperature of air is quite low, and thus it is
restricted passage of the perforated membrane essentially heated by using heating coils. The
cold air is usually dry and has very low moisture
content; therefore, moisure is added to improve
relative humidity.
A winter air-conditioning system is shown
in Fig. 29.4. The fresh air from the surrounding
atmosphere is mixed with re-circulated air in
suitable proportion and then the mixture is passed
through the filters to remove dust, dirt and other
unwanted materials from air about to enter the
system. The air is then passed over a preheater,
which heats the air sensibly to remove residual
moisture from the fresh air.
The preheated air is then passed through the
humidifier in which the required moisture is added
to improve relative humidity of air. During flow of
air through the humidifier, the air temperature gets
reduced; therefore, the air is again heated to suitable
temperature with the help of re-heater.
The air is then passed to the air-conditioned space
through the ducting system to provide required
comfort conditions within the confined space. A
portion of the air is exhausted to the atmosphere
using a suitable ventilator or exhaust fan and the
remaining air is used for recirculation with fresh air
for the next cycle.
Air-conditioning 1007

Filter Preheater Reheater

Duct

Fresh
air
Damper
Humidification
Recirculated air coil

Air
conditioned
space

Exhaust fan

(a) Circulating
fan

Fig. 29.5 Year-round air-conditioning system


Re-heating w
y
2 For summer air-conditioning, the humidifier and
H
1 x
heating coil are made inactive. The air is passed
Preheating
over a cooling coil. The temperature of the cooling
DBT coil is slightly kept higher than a summer air-
(b)
conditioning system in order to avoid condensation
Fig. 29.4 Winter air-conditioning system along with of water particles.
its processes on a psychrometric plot
During the winter season, the heating coil and
The processes of preheating, humidification and humidifier are made active. The heating coil heats
reheating are shown on the psychrometic plot in the air and the humidifier adds the humidity to air,
Fig. 29.3(b). if it is dry. Thus, the year-round air-conditioning
system can be used in any kind of ambient condition.
29.8 YEAR-ROUND AIR CONDITIONING Types of Air-Conditioners
SYSTEM
Air-conditioning systems may be classified as
In the tropical countries, the air-conditioning system (a) Unitary systems
design is primarily concerned with year-round (b) Central station systems
cooling and humidification/dehumidification.
Thus, the year-round air-conditioning system UNITARY SYSTEM
is the combination of both summer and winter air-
The unitary systems are usually factory-assembled
conditioning systems. The essential components of
units including internal wiring, controls and piping
a year round air-conditioning system are shown in
and are available in wide ranges capacities. These
Fig. 29.5. The air-conditioner has a cooling coil,
units are installed within or near the conditioned
humidifier and heating coil.
space. These systems offer the advantage of ease of
The fresh air taken from the surrounding
selection, low initial cost, ease of installation and
atmosphere is mixed with re-circulated air in
removal. These systems are available as
suitable proportion and then the mixture is passed
through the air filters to remove dust, dirt and other 1. Self-contained or single-package unit
unwanted materials from air about to enter the (window air-conditioner)
system.
1008 Thermal Engineering

2. Split system or split-package unit compressed refrigerant. In order to draw air through
3. Roof-top air-conditioner the filter and force it over the evaporator coil to cool
the atmospheric air, another fan is provided in the
indoor portion. For driving the two fans, either the
same motor or separate motors can be used for
It is a simple type of air-conditioner unit made
driving two fans.
as an enclosed assembly as shown in Fig. 29.6.
It is designed as a unit for mounting in a window A window air-conditioner works on the principle
or through a wall. The function of a window- of vapour-compression refrigeration system.
mounted air-conditioner is to provide comfort to
the occupants in room by circulating clean, cool
air. Their capacity is such that one unit is adequate A split air-conditioner is also known as a remote-
to condition one room. Roughly 0.08 to 0.1 TR mounted air-conditioner. It is basically an air-
capacity is required for cooling of 1 m2 room area. conditioning system built in two distinct units:
It does not require ducts for the free delivery of indoor unit and outdoor unit. The two units are
conditioned air to a space. The unit is divided into connected by refrigerant piping lines.
two parts: The indoor unit consists of a fan and cooling
(a) Indoor part coil. It is located in the space to be conditioned. It
(b) Outdoor part is a well-designed single casing, well insulated on
the inside housing the evaporator coil, twin blower
The indoor part includes a filter, evaporator, system with a motor, capillary tubes for refrigerant
a motor-driven fan or a blower and an expansion expansion, electronic controls and condensate drain
device. The outdoor portion includes a hermetical provision.
sealed motor-driven compressor unit, a condenser The outdoor unit consists of a compressor,
and a fan. condenser coil and propeller fan with motor. The
A fan is used to force the outside air over outdoor unit is connected to an indoor unit by
the condenser coil to remove the heat from the extended suction and liquid pipelines.
Air-conditioning 1009

Split air-conditioners are nowadays becoming


popular because
1. They require less space in the room
A central air-conditioning system provides the
2. They require no wall opening necessary coolant from one central location to different parts
3. They have better air circulations inside, due of a building. In this system, all the components
to flexibility of proper location are grouped together in one central room and
4. The condenser, compressor and fan package conditioned air is distributed from the central room
can be located at any convenient place, to the required places through insulated ducts as
where its noise is less objectionable and shown in Fig. 29.7. The central air-conditioner
more accessible for maintainance. system is generally used for a refrigerating load
5. They have flexibility of multiple evaporators above 25 tonns and 2500 m3/min of conditioned air.
in different rooms using single condenser– The central air-conditioning systems provide
compressor unit. fully controlled heating, cooling and ventilation.
They are widely used in theaters, stores, restaurants
and other public buildings. The system is complex
The rooftop unit is designed to be located on the and generally installed when the building is
roof of the building. The refrigeration, cooling and constructed.
air-handling equipment are available in sections The central air-conditioning system has the
that are assembled together. The compressor and following advantages:
condenser are located remotely. 1. Automatic central control point
The rooftop units are used with duct work and 2. Better air distribution
air outlets. These units do not use valuable building 3. Easier maintenance, single cooling and heat-
space and they are relatively low in cost. ing equipments are installed in one location.

Evaporator
Cool air circulated through house

Refrigerant
Refrigerant House wall
vapor

Furnace

Fan

Warm air Condenser

Outside air

Air filter or Blower


cleaner
Compressor
1010 Thermal Engineering

In an air–water system, both air and water are


distributed to each air–conditioned space to create
The central station air-conditioning systems are a cooling effect. In this system, the temperature
classified according to cooling medium used of same space can be better regulated by varying
to transport thermal energy to or from the air- the temperature of air or water (or both) during all
conditioned space. The main types of cooling seasons of the year. It requires less quantity of air
medium used in a central air-conditioning system circulation as compared to an all-air system. The
are reduced quantity of air can be supplied at high
1. All-air system velocity to the conditioned space.
2. All-water system
3. Air–water system

Air-conditioning units are rated in terms of effective


cooling capacity, which should be properly
In an all-air system, the sensible and latent cooling
expressed in kilowatt units. But manufacturers of
requirement of various parts of a building and air-conditioners still rate them in terms of tonnes
equipments is provided by the conditioned cold of refrigeration, which is equivalent to 3.5 kW.
air supplied by the central air-handling unit. The Window air-conditioners are available in 1 tonne,
circulated air is then returned back to the air- 1.5 tonne, 2 tonne capacity.
handling unit, where it is further treated for the
The indicative TR load required for air-condi-
next cycle. The schematic arrangement of an all-air
tioning is presented below:
system is shown in Fig. 29.8.
2

-
pancy) = 0.06 TR/m2

0.04 TR/m2.

The desired conditions inside a conditioned room


can economically be maintained by properly
designing heating and cooling systems. The
29.11.2 design of heating, ventilation and air-conditioning
In an all-water system, both sensible and latent (HVAC) systems need a close examination of
space cooling are achieved by chilled water thermal charecteristic of the building and the study
obtained from central refrigeration system through of rate of heat transfer from occupants, equipment,
the cooling coils in terminal units located in various infiltration. The heating load calculations require
spaces in the building. Water has high latent heat, consideration of the following parameters:
and thus it has the capacity to transport large
thermal energy with lowest mass-flow rate. This
characteristic of water lowers the size of piping The sources of internal heat gain are lighting
work. It offers lowest initial and operating cost. system, equipment and occupants within the space.
Air-conditioning 1011

(a) Heat Load from Lighting System and Equipments Table 29.3 gives the value of overall heat transfer
Qelectrical = Sum of ratings (watts) of equipments coefficient for some common cross-sections.
and lighting lamps
(a) Heat Gain from Human Body
The human body dissipates heat by convection, Cross section U (W/m2.K)
sweating and evaporation to surroundings. Common brick wall plaster on both 1.7
Qbody = Nos. of occupants ¥ heat gain rate per sides
person Concrete 10 cm thick with plaster 2.8
Table 29.2 shows the heat loads from occupants Thermocole 5 cm thick with 1.12
as functions of their activity. plywood on both sides
Sensible heat load from occupants Single glass 5.5
= Heat gain per person from Table 29.2 ¥ No. of Double glass with 6 mm air gap 3.4
persons inside the room ¥ percentage sensible heat
gain from Table 29.2
Infiltration is defined as the uncontrolled entry
= Heat gain per person from Table 29.2 ¥ No. of of outside air directly into the conditioned space
persons inside room ¥ (1– percentage sensible heat due to wind and buoyancy effects arising out of
gain from Table 29.2) temperature difference between inside and outside
environments.
Ventilation is defined as the fresh air intentionally
mixed with recalculated air to meet the oxygen
Activity Heat gain in W Sensible heat requirement of the occupants.
per person gain %
The entry of infiltration and fresh air into air-
Sleeping 70 75 conditioned space change the temperature and
Seated, quiet 100 60 humidity level of indoor air. Heat gain by room air
Standing 150 50 due to entry of infiltration air can be calculated as
Walking 305 35 Qin, sensible = ma (h3 – h2) ...(29.2)
Office work 150 55 and Qin, latent = ma (h1 – h3) ...(29.3)
Teaching work 175 50
where ma = mass-flow rate of infiltrated air (kg/s)
Industrial work 300–600 35
h3 = Enthalpy at intersection of horizontal
and vertical lines from room and
outside air as shown in Fig. 29.9
Heat loss or gain by thermal transmission through a
building wall can be calculated as
QWall = UA(DT)Wall = UA(To – Ti ) ...(29.1)
where
U
=Overall heat transfer coefficient,
W/m2.K
A = Surface area, m2
(DT)Wall = Temperature difference between
outside To and inside Ti , °C
1012 Thermal Engineering

h2 = Enthalpy of room air, kJ/kg Psychrometric plot with given data


h1 = Enthalpy of outside air, kJ/kg
Ventilation also imposes a considerable heat (or
cooling) load and the choice of ventilation rates
should be considered carefully.

Direct solar heat gain through glass windows can be


calculated as wall heat-transfer rate by replacing T1
by equivalent outside air temeprature Te, increased
by few degrees to account for direct solar radiation.
QSolar = UA(Te – Ti ) ...(29.4)
Analysis The outside and room conditions are
represented on a psychrometric plot with given data.
The point 1 is marked at coordinates 35°C DBT and 55%
The sensible heat factor is the ratio between the RH corresponding to outside conditions and the point 2
total sensible heat to total heat. The total heat is the corresponds to room conditions, 20°C DBT and 60% RH.
sum of total sensible heat and total latent heat. It is The point 3 represents the intersection point of horizontal
designated as SHF and expressed as and vertical lines.

Total Sensible Heat State 1: 35°C DBT and 55% RH


SHF =
Total Sensible Heat + Total Latent Heat h1 = 80.8 kJ/kg, v1 = 0.9 m3/kg
...(29.5) State 2: 20°C DBT and 60% RH
h2 = 42.2 kJ/kg
Example 29.1 An air conditioner is installed
in an auditorium hall. Air supply rate is 5 m3/s. The Intersection state 3 h3 = 57 kJ/kg
auditorium is maintained at 20°C DBT and 60% RH. The The mass-flow rate of air
atmospheric air at 35°C DBT and 55% RH is treated by
V (5 m3/s)
the air-conditioner. Calculate the sensible heat and latent ma = = = 5.555 kg/s
vs 1 (0.9 m3/kg)
heat removal rate. Also, calculate the sensible heat factor.
Sensible heat removal rate
Solution QSH = ma (h3 – h2)
Given An air-conditioner for an auditorium hall = 5.555 ¥ (57 – 42.2) = 82.22 kW
Latent heat removal rate
Outdoor conditions
QLH = ma (h1 – h3)
T1 = 35°C f1 = 55% V = 5 m3/s = 5.555 ¥ (80.8 – 57) = 132.2 kW
Room conditions Total heat removal rate,
T2 = 20°C f2 = 60% = QSH + QLH = 82.22 + 132.2
To find = 214.43 kW
(i) Seansible heat and latent heat heat removal rate, Sensible heat factor
(ii) Sensible heat factor. Sensible heat 82.22
SHF = = = 0.383
Total Heat 214.43
Air-conditioning 1013

Example 29.2 Air at the rate of 300 kg/min is Sensible heat removal rate,
supplied into an air-conditioned space maintained at QSH 2 = ma (h3 – h2) = 5 ¥ (57 – 42.2) = 74 kW
20°C DBT and 60% RH. The atmospheric air is taken
at 34°C DBT and 28°C WBT. Calculate the sensible heat QLH 2 = ma (h1 – h3) = 5 ¥ (90 – 57) = 165 kW
factor if 30 kW of sensible heat and 10 kW latent heat are Total sensible heat rate,
already present in the room. QSH = QSH1 + QSH 2 = 30 + 74 = 104 kW
Total latent heat rate,
Solution
QLH = QLH1 + QLH 2 = 10 + 165 = 175 kW
Given An air-conditioner for an auditorium hall Total heat rate,
Outdoor conditions Q = QSH + QLH = 104 + 175 = 279 kW
T1 = 34°C Twb1 = 28°C Sensible heat factor,
ms = 300 kg/min = 5 kg/s
Sensible heat 104
Room conditions SHF = = = 0.3727
Total Heat 279
T2 = 20°C f2 = 60%
Residual heats; Example 29.3 An air-conditioner plant is required
to supply 50 m3 of air per minute at a DBT of 22°C
QSH1 = 30 kJ/s QLH1 = 10 kJ/s
and 50% RH. The atmospheric condition is 32°C with
To find Sensible heat factor. 65% RH. Calculate the mass of moisture removed and
capacity of cooling coil if the required effect is obtained
Psychrometric plot with given data
by dehumidification and sensible cooling process. Also,
calculate the sensible heat factor.

Solution
Given An air-conditioner plant
Outdoor conditions
T1 = Tdb1 = 32°C f1 = 65%
Room conditions
T2 = Tdb2 = 22°C f2 = 50%
V = 50 m3/min
To find
(i) Mass of moisture removed,
(ii) Capacity of cooling coil, and
Analysis The outside and room conditions are (iii) Sensible heat factor.
represented on the psychrometric plot with given data.
The point 1 is marked at coordinates: 34°C DBT and Psychrometric plot with given data
28°C WBT corresponding to outside conditions, and the
point 2 corresponds to room conditions, 20°C DBT and
60% RH. The point 3 represents the intersection point of
horizontal and vertical lines.
State 1: 34°C DBT and 28 WBT
h1 = 90 kJ/kg
State 2: 20°C DBT and 60% RH
h2 = 42.2 kJ/kg
Intersection state 3
h3 = 57 kJ/kg
1014 Thermal Engineering

Analysis The outside and room conditions are The required conditions are achieved first by heating
represented on a psychrometric plot with given data. and then by adiabatic humidifying. Find (a) heating
The point 1 represents outside conditions and the capacity of coil in kW and surface temperature, if the by-
point 2 represents room conditions. The point 3 represents pass factor of the coil is 0.32, and (b) capacity of the
the intersection point of horizontal and vertical lines. humidifier.
State 1: 32°C DBT and 65% RH Solution
h1 = 82.5 kJ/kg
w1 = 0.0196 kg/kg of air Given An air-conditioner plant
v1 = 0.845 m3/kg Room conditions
State 2: 22°C DBT and 50% RH T3 = 20°C f3 = 60%
h2 = 43 kJ/kg Office capacity = 50 persons
w2 = 0.0084 kg/kg of air Outdoorc onditions
Intersection state 3 T1 = 10°C Twb1 = 8°C
h3 = 53 kJ/kg BPF = 0.32
(i) Mass of moisture removed Air circulation = 0.3 m3/min/person
The mass-flow rate of air To find
3 (i) Heating capacity of coil in kW,
V (50 m /min)
ma = =
v1 (0.845 m3/kg) (ii) Surface temperature of heating coil, and
= 59.17 kg/min (iii) Capacity of humidifier.
Mass of moisture removed Psychrometric plot with given data
= ma (w2 – w1)
= 59.17 ¥ (0.0196 – 0.0084)
= 0.662 kg/min or 39.76 kg/h
(ii) Capacity of cooling coil
The heat transfer
Q = ma (h1 – h2)
= 59.17 ¥ (82.5 – 43)
= 2337.21 kJ/min
Q ( kJ/min) 2337.21
Coilc apacity = =
( 210 kJ/min/Ton ) 210
= 11.13 Ton
(iii) Sensible heat factor Analysis Mark the point 1 corresponding to outdoor
conditions 10°C DBT and 8°C WBT on a psychrometric
Sensible heat h3 - h2 53 - 43 chart and draw a horizontal line. Now mark the point 3
SHF = = =
Total heat h1 - h2 82.5 - 43 corresponding to indoor conditions to 20°C DBT and f
= 0.2523 = 60% and draw a line along the constant WBT line till
it intersects horizontal line at the point 2. Process 1–2
Example 29.4 An air-conditioner plant is to be represents sensible heating and process 2–3 represents
designed for a small office for winter conditions: adiabatic humidification. From psychrometric chart,
Outdoor conditions 10°C DBT and 8°C WBT
State 1: Atmospheric air 10°C DBT and 8°C WBT
Indoor conditions 20°C DBT and 60%
h1 = 24.8 kJ/kg w1 = 0.0058 kg/kg of air
Amount of free air circulation 0.3 m3/min/person
vs1 = 0.81 m3/kg
Seating capacity of office = 50 persons
Air-conditioning 1015

State 2: Intermediate state Twt2 = Twt3 = 27.5°C Outdoor conditions


h2 = 42.2 kJ/kg w2 = 0.0058 kg/kg of air T1 = 30°C f1 = 70%
3
Air circulation = 0.5 m /min/person
State 3: Indoor conditions 20°C DBT and f = 60%
Room conditions
h3 = 42.2 kJ/kg, w 3 = 0.0088 kg/kg of air
T4 = 23°C f4 = 55%
Volume flow rate into office;
Office capacity = 100 persons
V = 50 persons ¥ 0.3 m3/min/person
Temperature of coil, Tc = 35°C
= 15 m3/min
The mass flow rate into office; To find
3 (i) Capacity of cooling coil,
V (15 m /min)
ma = = = 18.52 kg/min (ii) Capacity of heating coil,
vs 1 (0.81 m3/kg)
(iii) Amount of water removed by dehumidifier,
(i) Capacity of heating coils
(iv) By-pass factor.
Heat-transfer rate from the heating coil;
Psychrometric plot with given data
Q = ma (h2 – h1)
= 18.52 ¥ (42.2 – 24.8)
= 322.25 kW
(ii) Surface temperature of heating coil
The bypass factor is given by Eq. (15.33)
Tc - Texit T -T
BPF = = c 2
Tc - Tinlet Tc - T1
T - 27.5
0.32 = c
Tc - 10
fi Tc = 35.73°C
(ii) Capacity of humidifier
mw = ma (w3 – w2)
= 18.52 ¥ (0.0088 – 0.0058) Analysis Mark the point 1 corresponding to outdoor
= 0.0555 kg/s or 3.33 kg/h conditions of 30°C DBT and 70% RH on psychrometric
chart and draw a horizontal line. Now mark point 4
Example 29.5 A restaurant having a capacity of corresponding to indoor conditions to 23°C DBT and f
100 seats is to be air-conditioned, when outdoor = 55%. Draw sensible cooling process as line 1–2 and
conditions are 30°C DBT and 70% RH. Desired inside curve 2–3 as dehumidification and line 3–4 as heating
conditions are 23°C DBT and 55% RH. The quantity of process. From the psychrometric chart:
outdoor air supplied is 0.5 m3/min/person.
The desired conditions are achieved by cooling, State 1: Outdoor conditions 30°C DBT and 70% RH
dehumidifying and then heating. Calculate h1 = 78.5 kJ/kg w1 = 0.0188 kg/kg of air
(a) Capacity of cooling coil, vs1 = 0.885 m3/kg
(b) Capacity of heating coil, State 4: Indoor conditions 23°C DBT and f = 55%
(c) Amount of water removed by dehumidifier, h4 = 47.6 kJ/kg w4 = 0.0095 kg/kg of air
(d) By pass factor of heating coil, if its surface
temperature is 35°C. State 2: Intermediate states before and after humidifica-
tion
Solution h2 = 72.5 kJ/kg h3 = 37.8 kJ/kg
T2 = 24°C T3 = 13.5°C
Given An air-conditioner for a restaurant
1016 Thermal Engineering

Volume flow rate into office; Indoor conditions


V = 100 persons ¥ 0.5 m3/min/person T4 = 21°C f4 = 60%
= 50 m3/min Theater capacity = 1000 persons
The mass flow rate into office; Intermedite state, f3 = 80%
V (50 m3/min) To find
ma = = = 56.5 kg/min
vs 1 (0.885 m3/kg) (i) Capacity of heating coils, and
(i) Capacity of cooling coils (ii) Amount of water added in humidifier.
Heat transfer rate to cooling coil; Psychrometric plot with given data
Q1 = ma (h1 – h3)
= 56.5 ¥ (78.5 – 37.8)
= 2300 kJ/min
Q1 ( kJ/min) 2300
Coil c apacity = =
( 210 kJ/min/Ton ) 210
= 10.95 TR
(ii) Capacity of heating coil
Heat transfer rate from heating coil;
Q1 = ma (h4 – h3)
= 56.5 ¥ (47.6 – 37.8)
= 554 kJ/min or 9.23 kW
(iii) Amount of water removed by dehumdifier
mw = ma (w1 – w4) Analysis Mark the point 1 corresponding to outdoor
= 56.5 ¥ (0.0188 – 0.0095) conditions of 11°C DBT and 55% RH on the psychromtric
= 0.525 kg/min or 31.5 kg/h chart and draw a horizontal line 1–2. Now mark the
point 4 corresponding to indoor conditions to 21°C DBT
(iv) By-pass factor of heating coil and f = 60%. Draw a horizontal line 3–4 till it cuts 80%
Tc - Texit 35 - 23 RH WBT represents
BPF = = = 0.558 humidification. Process 1–2 and process 3–4 represent
Tc - Tinlet 35 - 13.5
heating processes. From psychrometric chart:
Example 29.6 A theater of 1000-person capacity is State 1: Outdoor conditions 11°C DBT and 55% RH
to be air conditioned for winter conditions: h1 = 22.5.5 kJ/kg w1 = 0.0044 kg/kg of air
Outdoor conditions: 11°C DBT and 55% RH vs1 = 0.81 m3/kg
Indoor conditions: 21°C DBT and 60% RH
State 4: Indoor conditions 21°C DBT and f = 60%
Amount of free air supplied per person = 0.5 m3/min
h4 = 44 kJ/kg w4 = 0.0090 kg/kg of air
The required copndition is achieved by heating,
humidifying and then again heating. If the air coming State 2: Intermediate states before and after humidifica-
out of the humifier has 80% RH, calculate the heating tion
capacity of both heating coils and the mass of water h2 = h3 = 39.2 kJ/kg T2 = 27.8°C
added in the humidifier. T3 = 16.2°C
Volume flow rate into office;
Solution
V = 1000 persons ¥ 0.5 m3/min/person
Given An air-conditioner for a theater = 500 m3/min
Outdoor conditions The mass flow rate into office;
T1 = 11°C f1 = 55% V (500 m3/min)
Air circulation = 0.5 m3/min/person ma = = = 10.29 kg/s
vs 1 (0.81 m3/kg) ¥ (60 s/min)
Air-conditioning 1017

(i) Capacity of heating coils discharged as high-temperature, high-pressure


Capacity (heat-transfer rate from) of first heating refrigerant vapour into the condenser, where it is
coil; desurperheated and condensed and sometimes
Q1 = ma (h2 – h1) undercooled also. The high-pressure liquid
= 10.29 ¥ (39.2 – 22.5) = 171.81 kW refrigerant coming out of the condenser then passes
Capacity (heat-transfer rate from) of second through the drier to the capillary tube. The long
heating coil; capillary tube lowers the pressure of the liquid
Q2 = ma (h4 – h3) refrigerant as well as temperature due to resistance
= 10.29 ¥ (44 – 39.2) = 49.4 kW of small diameter of the tube. The low-temperature
(ii) Capacity of humdifier: refrigerant then enters the cooling coils wrapped
mw = ma (w4 – w1) around the water-storage tank. The refrigerant
= 10.29 ¥ (0.0090 – 0.0044) evaporates by extracting the heat of water through
= 0.0473 kg/s or 170.4 kg/h the storage tank. Thus, water cools and the liquid
refrigerant converts into refrigerant gas, which
again enters the compressor for the next cycle.

Water coolers use refrigeration cycle for cooling


The drinking water enters the water tank surrounded
of water. They are widely used in schools, offices
by evaporating coils. The liquid refrigerant extracts
restaurants, factories and stores. They have two
its latent heat of evaporation from water, and the
different circuits:
water cools. The cooled water can be used through
(i) Refrigertion circuit a water tap. The waste water from the basin
(ii) Water circuit comes out through the drain line. The float valve
maintains a constant level of water in the tank, and
a thermostatic valve regulates the temperature of
It is exactly a vapour compression cycle as shown water in the tank.

enters the compressor and it is compressed and


Ice manufacturing is one of the major applications
of refrigeration. Ice is the cheapest means for
short-time preservation of food. An ice plant uses
a vapour-compression refrigeration cycle as shown
in Fig. 29.17. The evaporator is made in the form
of a tank. The cooling coils are mounted inside the
inner wall tank. The tank is filled with secondary
refrigerant. The secondary refrigerant used is brine
solution: salt in water, which lowers the freezing
temperature of water. With sodium chloride (NaCl),
the freezing temperature of water may decrease to
–14°C. With calcium chloride CaCl2, the freezing
temperature of water may decrease to –21°C.
The GI ice cans of any convenient shape and
size are placed in the brine solution.
1018 Thermal Engineering

during the peak cooling months of the year.


Small size air coolers can be installed in a
window, blowing cooled air directly into a room.

The brine solution, a secondary refrigerant, acts from where the air can travel through a duct work
as a heat carrier. It absorbs the heat from water in to individual rooms.
the ice cans and transfers it to cooling coils of the
evaporator.
Since the brine solution is maintained at a
temperature which is lower than 0°C, thus water They consist of a blower, a water circulation pump,
freezes in the ice cans and ice is produced. filter pads and a water sump. The blower draws in
outside air and forces it through the wetted filter
pads. As hot dry air moves through the filter pads,
water evaporates, cooling and humidifying the air.
Air coolers, as shown in Fig. 29.18, are also called The cool air is then supplied to the room. Since
desert coolers. They work on the principle of water is continuously lost through evaporation, thus
evaporative cooling. Evaporative cooling is one of the make-up water is required from time to time,
the most ancient and energy-efficient methods for when the water level gets lowered in the sump.
cooling a house. The air coolers use evaporation of Under normal conditions, a desert cooler can use 3
water for cooling of air. The sensible heat of hot to 15 buckets of water a day.
atmospheric air is transferred continuously to water Small water-distribution lines are provided at
in the form of latent heat for its evaporation, when the top of the pads. The water falls down drop by
the air makes physical contact with falling water drop on the pads. The pads soak water. Some of
on the filter pads. Since at lower temperatures, the the water evaporates from the pads as air passes
latent heat of water is much higher, a large quantity through them. The remaining water from the pads
of air can be cooled by evaporation of a small is collected into the sump at the bottom of the
quantity of water. cooler. A small recirculation water pump sends the
Air coolers are considered environmentally safe collected water back to the top of the pads.
since the process typically uses no ozone-depleting The pads can be made of wood shavings. Grass—
chemicals, and demands only one-fourth as much wood from aspen trees is a traditional choice—or
energy in comparison to an air-conditioning system other materials that absorb and hold moisture while
Air-conditioning 1019

resisting mild dew. Aspen wood pads, also called


excelsiors. The wood pads are generally replaced
after every season or two.
These coolers usually have 2 or 3 fan speeds, and
they actually cool more efficiently at lower speeds.

A fan draws hot atmospheric air through the


moist pads, where evaporation of water drops the
temperature of air by approximately 20 degrees.
The effective cooling of incoming air in coolers
depends on the moisture content of air, air velocity,
surface area of wetted pads and time of contact
between air and water.
1. The cooling capacity of evaporative coolers
is best when the air is drawn slowly over
the wetted pads, which allows maximum
evaporation of water. Coolers with larger
total porous areas will provide better cooling
of air because the air can flow more slowly
over the porous pads.
2. Evaporative coolers can perform better when
the outside air is dry, desert. With increase
in moisture content of air, the capacity of
evaporative coolers to cool the air effectively
decreases.
3. Performance of all evaporative coolers
increases as atmospheric temperature rises
and the air becomes dry.

Desert coolers are classified according to


Cooler with Fan in Vertical Plane The arrangement
(i) Position of fans of a fan and water pump is shown in Fig. 29.19(b).
(a) Fan in vertical plane It uses separate motors for the pump and-fan, so
(b) Fan in horizontal plane they can be run independently. It is provided with
(c) Drum-type fan three vertical pads of cellulose fiber. The water
(ii) Types of pads used pump delivers water from the cooler sump to the
(a) Fibre-pad coolers top of the pad. As water runs down the porous
(b) Rigid-Sheet-pad coolers pads, it is absorbed by the pads and they become
(iii) Independent ducted system wet. When the fan draws atmospheric air through
the porous pads on three sides, water evaporates
by absorbing its latent heat from the incoming air.
1020 Thermal Engineering

Thus air moving into the room becomes cooler than motor. Therefore, the fan and pump cannot be run
the outdoor air. independently. It consists of four vertical fibre pads
with water supply arrangement and water falls drop
Cooler with Fan in Horizontal Plane The by drop on the pads. The fan draws air through the
arrangement of components are shown in pads and is first moved in the vertical direction and
Fig. 29.20(a). The pump and fan are mounted then through the spiral distributors in horizontal
on the same vertical shaft and run by a common planes.
Cool air It has certain unique features. Therefore, it
ensures maximum cooling in minimum time.
Fan 1. It consists of four pads, and thus there is
Upper more exposure area for air. Therefore, the
circular tank
cooler has more cooling capacity for the
Fan same volumetric capacity.
motor
2. Spiral directional air flow provides better air
Common distribution in the room.
shaft
Overflow
3. Noise level is not objectionable.
pipe
Support Cooler Without Water Pump The components of
Modified beam a cooler are shown in Fig. 29.21. It consists of a
pump
drum-type evaporator with rubberized coir, which
Filter is used for getting cooling effect instead of wetted
Lower
tank pads. The evaporator dips in the water during its
rotation and becomes wet. It uses an exhaust
fan instead of conventional fan which provides
maximum air thrust.
(a) Box-type (cooler inner arrangement)
Hot air in Drum type cooler
Spiral air
distributor Electric
control

Flywheel
Exhaust Cool air out
Grill
fan

To water Outer
Fan G.I. Sheet
tank motor
Water tray
Khus pad
(4 Nos.)

Water
pump
Fibre-Pad Cooler The fibre pads are khus straws
or shredded wood fibres packed in a plastic or
Supporting metallic net. These pads are 1 to 2 inches thick. The
Bottom water tank frame
quality and cost vary considerably. The thinnest
(b) Front cut-section view of box-type desent cooler pads are usually least expensive and more effective.
Generally two thin pads are used to improve
Air-conditioning 1021

saturation effectiveness. These pads are replaced piping to the upper header from where water is
with new ones every 1–2 years. distributed to pads.

Rigid-Sheet Pad Coolers The rigid-sheet pad is a


stack of corrugated sheet material that allows air to
move through it at higher velocities instead of fibre
pads. These pads are generally 8 or 12 inches thick. An air-conditioner works on the refrigerating
Independent Ducted System It is an evaporative principle, where the air is cooled, when it passes
cooler which blows cooled air through a single over chilled tubes through which the refrigerant
diffuser in the hall ceiling or into a distributed duct is evaporating. An evaporative cooler is based on
system in the ceiling space. It uses four vertical the principle that when water comes in contact of
filter pads and separate motors for fan and water hot moving air, the hot air is cooled immediately
circulation pump. The fan is driven by a belt drive. through evaporation of water. Although there is no
It also consists of a float valve in the path of water substitute for an air-conditioner’s cooling comfort
supply which ensures a constant level of water in in hot and humid climates. In dry climates, like in
the sump. The pump transfers water through the Rajasthan, Gujarat, MP and most of central India, a
desert cooler is a cheaper and comfortable option.
1022 Thermal Engineering

Summary
Air-conditioning is the process of treating air in indicative TR load required for air-conditioning
an internal environment to achieve and maintain is presented below:
required standards of temperature, humidity, 2

cleanliness and motion of air, regardless of


surrounding conditions. occupancy) = 0.06 TR/m2

and radiation and it generates heat energy as 0.04 TR/m2.


metabolism. The human body converts about
20% of heat input into useful work and the by cooling and dehumidification of air. In winter,
remaining of heat generated must be rejected to the comfort conditions are achieved by heating
the environment to regulate the body temperature. and humidification of air and the year-round air
effective temperature is the index which conditioning system is the combination of both
correlates the combined effects of air temperature, summer and winter air-conditioning systems.
relative humidity and air velocity on the human
body. A person can feel comfort in air-conditioned of water.
space, if he/she gets An ice plant uses vapour-compression refrigera-
1. continuous supply of oxygen and removal tion cycle and it produces ice. The brine solution,
of carbon dioxide, a second refrigerant, acts as a heat carrier in the
2. removal of heat continuously, and ice plant.
3. removal of Moisture content of air.
An air-conditioning system may provide heating, cooling. The sensible heat of hot atmospheric air
cooling or both. An air-conditioning system is transferred continuously to water in the form
controls the conditions of a confined space to of latent heat for its evaporation, when air makes
produce a comfort effect over occupants. The physical contact with falling water on filter pads.

Glossary
Air-conditioning Process of maintaining air at con- DPT Temperature at which vapour starts to condense
trolled temperature, humidity, purity and motion Humidification Addition of water vapour in air
Air cooler Device which cools the air with the help of Ice plant Machine used for producing ice
evaporation of water Psychrometry Study of moist air
Air refrigeration A refrigeration system in which air is Refrigerant It is the working substance used in the re-
the working medium frigeration system
Air–conditioner Machine which creates air-condition- Refrigeration effect Cooling effect
ing
Refrigeration Maintaining a space at low temperature
DBT Temperature indicated by an ordinary thermom- than its surroundings
eter
Relative humidity Ratio of partial pressure of vapour to
Dehumidiciation Reduction of water vapour from total pressure of mixture
moist air
Air-conditioning 1023

Sensible heating/cooling Heat transfer with only Water cooler Machine which cools the water using re-
change in DBT frigeration system
Specific humidity Mass of water vapour per kg of dry WBT Temperature indicated by thermometer, when its
air bulb is covered by wet cotton wick

Review Questions
1. What is sensible heating or cooling? 13. Explain a central air-conditioning unit.
2. How does humidity affect human comfort? 14. What are sources of heating loads in a retaurant?
3. State the applications of air conditioning. List them.
4. State the human comfort conditions. What are the 15. How does the human body maintain its
factors which affect the human comfort? temperature? Why does the human body need air-
5. What is an effective temperature? conditioning?
6. What is a comfort chart? What are its applications? 16. Explain the construction and working of a split
7. What are the requirements of comfort air air-conditioning system.
conditioning? 17. What are infilatration and ventilation? How do
8. Classify the air-conditioning systems. they contribute to heating load?
9. Explain summer air-conditioning systems with 18. Explain the working of an ice plant with the help
the help of a schematic. of a neat sketch.
10. Explain winter air-conditioning systems with the 19. What do you mean by refrigeration and air
help of a schematic. conditioning. Draw a schematic layout of an air-
11. Explain year-round air conditioning systems with conditioner and explain its working.
the help of a schematic. 20. Draw a schematic of desert cooler and explain its
12. What is a unitary air-conditioning system? construction and working.
Explain a window air-conditioner.

Problems
1. The indoor conditions of a building are 21°C Fresh air circulation 0.3 m3/min/person,
and 40% RH, when outdoor conditions are 28°C If the required condition is achieved first by adia-
and 50%. The sensible heat gain to a room is batic humidifying and then by cooling, calculate
12 kW and the latent heat gain is 3 kW. There (a) Capacity of cooling coil and its surface
is no recirculation and fresh air is cooled and temperature, if by-pass factor is 0.25
dehumidified and then heated before entering (b) Capacity of humidifier, and its efficiency
the room. The cooling coil by-pass factor is 0.2
[118.8 kW, 15.3°C, 72 kg/h, 29%]
and the volume-flow rate of fresh air is 5 m3/s.
3. An office of 25 people in Mumbai is to be
Calculate the temperature of air leaving the coil,
air-conditioned when outdoor conditions are
capacity of cooling coil and heat supplied in the
29°C DBT and 73% RH. The required comfort
heater. [6.5°C, 220 kW, 65 kW]
conditions are 21°C DBT and 59% RH with
2. A cinema hall of 1500 seating capacity is to be
0.5 m3/min/person fresh air supply. Air is first
air-conditioned with following data:
cooled, dehumidified and then heated. Calculate
Outdoor conditions 40°C DBT and 20°C WBT, the cooling coil capacity, heating coil capacity
Indoor conditions 20°C, and 60% and capacity of the dehumidifier.
[3.2 TR, 2.1 kW, 9.2 kg/h]
1024 Thermal Engineering

4. The atmospheric air at 30°C DBT with 75% RH capacity of coil and capacity of humidifier. Also,
enters a cooling coil at the rate of 200 m3/min. The calcaulate the surface temperature of heating coil,
dew point temperature of cooling coil is 14°C and if by-pass factor is 0.28.
by-pass factor is 0.25. Calculate the temperature [5.5 kW, 3.33 kg/h, 34.3°C]
of air leaving the cooling coil, capacity of cooling 6. A living room is to be maintained at 35°C DBT
coil, amount of water removed and sensible heat and 22.5 WBT during a winter season. The
factor. atmospheric conditions are 10°C with 90%
[18°C, 41.47 TR, 124.86 kg/h, 0.3766] RH. If the humidified air coming out of the air
5. A small office of 50 seating capacity is to be air- washer is at 90% RH, calculate the temperature
conditioned at 20°C DBT, 60% RH. The outdoor of air entering the air washer and efficiency of air
conditions are 10°C and 8°C WBT. The rate of washer, RH of air leaving the washer and quantity
fresh air circulation is 0.3 m3/min/person. The of moisture added.
required conditions are achieved by heating, [17.5°C, 92.7%, 35%, 0.0054 kg/kg of air]
adiabatic humidifying. Calculate the heating

Objective Questions
1. During cooling and humidification process in air- 6. Capacity of cooling coil decreases with
conditioning (a) decrease in mass-flow rate of air
(a) DBT decrease (b) RH decrease (b) decrease in by-pass factor
(c) DBT increases (d) None of above (c) decrease in entry temperature of air
2. Air at 20°C is heated to 25°C using a heater with (d) all of the above
a surface temperature of 30°C. The by-pass factor 7. Air-conditioning means control of
is (a) DBT
(a) 0.15 (b) 0.5 (b) RH
(c) 0.25 (d) 0.2 (c) velocity and purity of air
3. The by-pass factor of cooling coil is 0.3 when its (d) all of the above
coil surface temperature is 5°C and air enters at 8. In summer air-conditioning, the air is
40°C. The exit air temperature is
(a) cooled
(a) 10°C (b) 12.5°C
(b) cooled and dehumidified
(c) 15.5°C (d) 20°C
(c) heated and humidified
4. Infiltration air in an air-conditioned space
(d) cooled and humidified
(a) reduces heat load 9. In winter air-condioning, the air is
(b) leaks through the gaps (a) heated
(c) increases heat load (b) heated and dehumidified
(d) both (b) and (c) (c) heated and humidified
5. Sensible heat factor is the ratio of total sensible (d) cooled and humidified
heat to
10. In summer air-conditioning, RH of conditioned
(a) total latent heat space is generally kept
(b) total heat (a) 40% (b) 50%
(c) capacity of cooling coil (c) 60% (d) 100%
(d) capacity of heating coil
10. (c) 9. (c)
8. (b) 7. (d) 6. (a) 5. (b) 4. (d) 3. (c) 2. (b) 1. (a)
Answers
Elements of Heat Transfer 1025

30
Elements of Heat Transfer

Introduction
Heat transfer is a branch of thermal science which deals with analysis of rate of heat transfer and temperature
distribution taking place in a system as well the nature of heat transfer. The design of boilers, condensers,
evaporators, heaters, refrigerators and heat exchangers, requires considerations of the amount of heat to
be transmitted as well as the rate at which heat is to be transferred. A heat transfer analysis must also be
accounted in the design of electronic components, electric machines, transformers, and bearings to avoid
the overheating and damage of equipments.

The energy transfer by radiation is fastest and it can


also travel in vacuum.
When temperature gradient exists in a medium
which may be a solid, fluid or gas, then there is
an energy transfer from high temperature region
to low temperature region. This energy transfer
as heat is called heat conduction. In contrast, The rate of heat conduction through a medium
heat convection refers to heat transfer that will depends on its geometry, thickness and material of
occur between a surface and the adjacent moving the medium as well as temperature difference. The
medium, liquid or gas, when they are at different Fourier law states that the rate of heat conduction
temperatures. It involves the combined effects of per unit area (heat flux) is directly proportional to
conduction and fluid motion. If there is no fluid temperature gradient.
motion, then the heat is transferred between a solid Q dT
and its adjacent fluid by pure conduction. The μ
A dx
third mode of heat transfer is thermal radiation.
All surfaces at finite temperature emit energy in Q dT
or q = = –k ...(30.1)
the form of electromagnetic waves or (photons) as A dx
result of the changes in electron configuration of
the atoms or molecules. This mode of heat transfer Q = – kA dT
does not require the presence of a material medium. dx
1026 Thermal Engineering

where q = heat flux (W/m2) Table 30.1


Q = rate of heat transfer, W
Material Thermal Conductivity
A = area normal to direction of heat flow,
k (W/m.K)
m2
Metals
dT
= temperature gradient °C/m, slope of Copper (pure) 399
dx
temperature curve on T–x diagram Aluminium (pure) 227
k =constant of proportionality, called Iron (Pure) 73
thermal conductivity of material, W/m Carbon Steel, 1% C 43
°C or W/m ◊ K Non-Metallic Solids
The minus sign is inserted to make natural heat Window glass 0.72
flow a positive quantity. According to the second Glass wool 0.038
law of thermodynamics, heat always flows in the Asbestos 0.149
direction of decreasing temperature. Thus the Cork 0.045
temperature gradient dT/dx becomes negative.
Fluids
Water 0.556
Ethylene glycol 0.249
The thermal conductivity is a property of a material Ammonia 0.54
and is defined as the ability of the material to Air 0.024
conduct heat through it. It can also be defined as Steam 0.0206
the rate of heat transfer through a unit thickness Carbon dioxide 0.0146
of material per unit area per unit temperature
difference. The thermal conductivity of a material temperature region to a low-temperature region.
is a measure of how fast heat will flow in that If the fluid motion is artificially induced by a
material. A large value of thermal conductivity pump, fan or a blower that forces the fluid over a
indicates that the material is a good heat conductor surface to flow, the heat transfer is said to be forced
and a low value indicates that the material is a poor convection. If the fluid motion is set up by buoyancy
heat conductor or an insulator. effects resulting from density difference caused by
temperature difference in the fluid, the heat transfer
The thermal conductivity is measured in
is said to be by free (or natural) convection.
watts per meter per degree Celsius or Watt metre
per kelvin, when heat flow rate is expressed in y
watts. The thermal conductivity of some typical
substances is given Table 30.1.
Fluid temperature
profile
30.4
T

The convection heat transfer comprises of two h


mechanisms. The first is the transfer of energy due x
to random molecular motion (diffusion), and the
second is the energy transfer by bulk motion of the Plate at Ts
fluid. The molecules of fluid are moving collectively
or as aggregates and thus carry energy from a high-
Elements of Heat Transfer 1027

The Newton’s law of cooling is the governing where, e = a radiative property of the surface is
equation of convection heat transfer. It states that called the emissivity
the rate of heat transfer is directly proportional to The net rate of radiation heat exchange between
temperature difference between a surface and fluid a real surface and its surrounding is
or mathematically Q
= s e (Ts4 – T 4) …(30.5)
Q A
(W/m2) μ (Ts – T ) (°C)
A where, T = surrounding temperature K
Q Ts = surface temperature, K
or = h (Ts – T ) ...(30.2)
A The three other radiation laws, Planck’s law,
where, Ts = surface temperature, °C Wein’s law and Kirchhoff’s law, are also used in
T = fluid temperature, °C radiation heat transfer.
h = constant of proportionality, is called
the heat transfer coefficient.
The heat transfer coefficient is measured in
W/m2 K or W/m2 °C. The value of the heat transfer
coefficient depends on the properties of fluid as
well as fluid flow conditions. Consider a plane wall of thickness L as shown in
Fig. 30.2. Its left face at x = 0, is at a temperature T1
and right face at x = L is at temperature T2. The wall
has a constant thermal conductivity k.
Fourier equation for elemental strip of plane
When energy propagates in the form of
wall
electromagnetic waves from a high-temperature
Q dT
region to a low-temperature region, the form of = –k
energy transfer is referred as thermal radiation. A dx
Stefan–Boltzmann law governs the radiation Rearranging, we get;
heat transfer. It states that the rate of radiation Q dx = – k dT
heat transfer per unit area from a black surface is A
directly proportional to fourth power of the absolute
T
temperature of the surface and is given by
Q
μ (T 4)
A
T(x)
Q
or = s Ts4 ...(30.3)
A T1 dT

where Ts = absolute temperature of surface K


.
s =constant of proportionality, called Q
Stefan Boltzmann Constant and has T2
value of 5.67 ¥ 10–8 W/m2 K4
The heat flux emitted by a real surface is less
x
than that of black surface and is given by dx
L
Q
= s e (Ts4) ...(30.4)
A
1028 Thermal Engineering

Integrating, within limits, r2

Q L T2

A Ú 0
dx = – k ÚT1
dT
r1
Q
or L = – k (T2 – T1) r T1
A
The total heat flow rate Q , through an area A
T2
normal to direction of heat flow, dr

ÊT –T ˆ
Q = kA Á 1 2 ˜ ...(30.6)
Ë L ¯
Applying Fourier law of heat conduction to
elemental strip of thickness dr of hollow sphere,
Q dT
Consider a hollow cylinder as shown in Fig. 30.3. = –k
A dr
r2 Here, A = 4p r2, area of elemental strip
Q r2 dr T2
Then Ú
4p r1 r 2
= – k
T1 ÚdT

r1 Q È1 1 ˘
Í – ˙ = – k (T2 – T1)
r
T1 4p Î r1 r2 ˚
L
4p r1 r2 k (T1 - T2 )
dr or Q = ...(30.8)
T2 r2 - r1

For an elemental strip of cylinder, Rewriting the Eq. (30.6) in the form,
Q dT T1 – T2 DT
= –k Q = = …(30.9)
A dr L Rslab
where A = 2prL, area of elemental strip kA
Rearranging, we get Comparing this equation with Ohm’s law for
Q dr electrical network,
◊ = – k dT
2p L r DV Potential difference
current, I = =
Integrating both sides, R Resistance
Q r2 dr T2 There is analogy between these two equations
2p L Ú
r1 r
= –k ÚT1
dT and
DT = Thermal potential difference, °C,
Q Êr ˆ
◊ ln Á 2 ˜ = – k (T2 – T1) L
2p L Ë r1 ¯ = Thermal resistance (Rslab), °C/W,
kA
2p Lk (T1 - T2 ) Q = Heat flow rate or heat current in W.
or Q = ...(30.7)
Êr ˆ Its equivalent thermal circuit is shown in
ln Á 2 ˜
Ë r1 ¯ Fig. 30.5.
The inverse of thermal resistance is called
kA
thermal conductance K = ( W/K )
Consider a hollow sphere as shown in Fig. 30.4. L
Elements of Heat Transfer 1029

ments. The steady-state heat-transfer rate through


the wall, when its two surfaces are maintained at
constant temperatures T1 and T2, can be expressed
as

Further, the thermal resistance may also be kA (T1 - T2 )


Q1 = ...(30.11)
associated with convection heat transfer at a surface L
The left face and right face involve convection
as shown in Fig. 30.6. Equation (30.2) is rewritten
heat transfer due to temperature difference between
as
the surfaces and their surroundings. From Eq.
Q = hA (Ts – T )
(30.2);
T –T T –T
or Q = s = s ...(30.10) The convection heat transfer at the left face,
1 Rconv
Q2 = h1 A(T 1 – T1) …(30.12)
hA
The convection heat transfer at the right face,
1
where Rconv = = Convective resistance, Q3 = h2 A(T2 – T 2) …(30.13)
hA
In steady-state condition,
Ts – T = Thermal potential difference at
the surface. Q = Q1 = Q2 = Q3
Hence Q = h1A (T 1
– T1)
k A (T1 T2 )
h =
L
Solid = h2 A (T2 – T 2) …(30.14)
Fluid The equivalent thermal resistances for a plane
T wall with convection on both sides as shown in
. Fig. 30.7 may be analysed by considering each
Q
Ts T
element in circuit separately.
1 The concept of thermal circuit may also be
Rconv =
hA introduced for a composite wall. Such a wall may
involve any number of series and parallel thermal
resistances due to a layer of different materials.

T
1
T1
The three modes of heat transfer have been ex-
h1 h 2, T
plained separately earlier. In actual situations, it is T2 2

very rare that only one mechanism is involved in L


T
2
transfer of heat energy, instead a combination of
mechanisms are involved. The thermal circuit rep- x

resentation provides a useful tool for the analysis of (a) Schematic


such heat transfer problems. T1 T2
T T
1 2
Q 1 L 1
h 1A kA h 2A

Consider a plane wall of thickness L, exposed on (b) Equivalent thermal resistances


both sides to two different temperatures environ-
1030 Thermal Engineering

T 1 – T1 T1 – T2 T –T T –T
Q = = = 2 3 = 3 4
Consider a composite wall with three layers and 1 LA LB LC
convection heat transfer on both boundary surfaces h1 A kA A kB A kC A
as shown in Fig. (30.8). T4 – T 2
= …(30.16)
1
T
1
h2 A
Fluid 2

h1 h2
The electrical analogy can also be applied for a
LA LB LC
hollow cylinder by rearranging Eq. (30.7)
T
Fluid 1 2
T –T DT
Q = 1 2 = ...(30.17)
(a) Schematic ln ( r2 /r1 ) Rcy1
T T1 T2 T3 T4 T 2p L k
1 2
ln ( r2 /r1 )
Q¢ where, Rcyl =
Rconv
1
RA RB RC Rconv
2
2p L k
(b) Thermal resistance network where Rcyl = thermal resistance to heat flow
through a hollow cylinder.
r1 = inner radius
Here, heat transfer rate can be expressed as r2 = outer radius
( DT )overall Q1
Q = T1 T2
S Rth
where, (DT )overall = T 1 – T 2 Rcyl

S Rth = Rconv1 + RA + RB + RC + Rconv2


1
Rconv1 =
h1 A
L L Now, consider steady-state heat conduction
RA = A , RB = B through a hollow cylinder that is subjected to
kA A kB A
convection heat transfer on both sides to fluids
LC
RC = , Rconv2 = 1 at temperatures T 1 and T 2 with heat transfer
kC A h2 A coefficients h1 and h2, respectively as shown in
1 L L L 1 Fig. 30.10. The thermal resistance network in this
Thus + A + B + C +
S Rth = case consists of one conduction resistance and two
h1 A k A A k B A kC A h2 A
convection resistances, and the rate of heat transfer
A = Area normal to heat transfer
can be expressed as
T 1 –T 2
Then, Q = T –T 2
1 L L L 1 Q = 1
+ A + B + C + S Rth
h1 A k A A k B A kC A h2 A
...(30.15) where SRth = Rconv1 + Rcyl + Rconv2
Alternatively, the heat-transfer rate associated 1 ln ( r2 /r1 ) 1
with each layer in the composite wall can also be = + +
( 2p r1 L) h1 2p Lk ( 2p r2 L) h2
expressed as
…(30.18)
Elements of Heat Transfer 1031

Total resistance
.
SRth = Rconv1 + RA + RB + RC + Rconv2
Q
1 ln ( r2 /r1 ) ln ( r3 /r2 )
or SRth = + +
2p r1 Lh1 2p Lk A 2p Lk B
ln ( r4 /r3 ) 1
T2 + + …(30.19)
h1, T 1
T1 2p LkC 2p r4 Lh2
r1 T ,h2
2
r2 Rconv Rcyl Rconv2
1

Similar to a plane wall or hollow cylinder, the use


of electrical analogy can also be extended to hollow
Thermal resistances network for a hollow
spheres. Rewriting Eq. (30.8) in the form;
cylinder exposed to convection on both sides
T1 – T2 DT
Q = = …(30.20)
r2 – r1 Rsph
Consider composite system of three hollow cylin- 4p r1 r2 k
ders (thermal conductivity kA, kB, kC respectively) r –r
where Rsph = 2 1 thermal resistance of hollow
as show in Fig. 30.11, with heat convection on in- 4p r1r2 k
ner and outer surfaces. Recalling the treatment giv- sphere.
en to composite wall, the expression for heat flow r1 = inner radius of sphere, and
can be written as r2 = outer radius of sphere
( DT )overall
Q = Now consider steady-state heat flow through
S Rth
a hollow sphere that is subjected to convection
T 1 –T 2 heat transfer on inner and outer sides to fluids
=
1 ln ( r2 /r1 ) ln ( r3 /r2 ) ln ( r4 /r3 ) 1 at temperatures T 1 and T 2, with heat transfer
+ + + +
h1 A1 2p Lk A 2p Lk B 2p LkC h2 A2 coefficients h1 and h2, respectively. The thermal
where, A1 = 2p r1L and A2 = 2p r4 L resistance network for such a case, consists of
one conduction resistance and two convection
C
resistances as shown in Fig. 30.12, and the rate of

B Q¢
T
2
A
r2 r3
r1
r4
h2
h1 h1
T T
1 1 T
r1 2
(a) Schematic Rconv Rsph Rconv2
1
r2
T1 T2 T3 T4 T
T 2 h 2, T
Q 1
Rconv RA RB RC Rconv
1 2

(b) Thermal network


Thermal resistance network for a hollow
Thermal resistance network for a composite
sphere subjected to convection on inner and
cylinder subjected to convection on both
outer surfaces
sides.
1032 Thermal Engineering

heat transfer may be expressed as r3 – r2 1


T –T 2 R2 = , Rconv2 = ...(30.23)
Q = 1 4p r2 r3 k2 4p r32 h2
S Rth
and S Rth = Rconv1 + R1 + R2 + Rconv2
where SRth = Rconv1 + Rsph + Rconv2
It is total thermal resistance between the
1 r2 – r1 1
= + + temperatures T 1 and T 2.
4p r12 h1 4p r1r2 k 4p r22 h2
…(30.21) 30.8 OVERALL HEAT-TRANSFER
COEFFICIENT
30.7.6 Heat Conduction through
Composite Spheres An overall heat-transfer coefficient can be evaluated
The radial heat flow rate Q through multilayer as
sphere shown in figure (30.13) can be obtained by ( DT )overall
Q = UA (DT )overall =
using thermal resistance concept to each later of S Rth
sphere, 1
Therefore, U =
T 1 – T1 T1 – T2 T –T A S Rth
Q = = = 2 3
Rco nv1 R1 R2 For plane wall, from Eq. (30.15)
T3 – T 1
= 2
…(30.22) U = …(30.24)
Rconv2 1 LA LB LC 1
+ + + +
h1 k A k B kC h2
Since all thermal resistances are in series as
shown in Fig. (30.13b). Therefore, For a hollow cylinder the heat-transfer rate is
expressed as
T 1 –T 2
( DT )overall
Q = = Q = U1 A1 (DT )overall = U2 A2 (DT )overall
Rconv1 + R1 + R2 + Rconv2 S Rth
where A1 = 2p r1L and A2 = 2p r2 L.
where various resistances are
Using Eq. (30.19) for composite cylinder
1 r2 – r1 exposed to convection on both sides;
Rconv1 = , R1 =
4p r12 h1 4p r1r2 k1 1
U1 =
2p r1 L S Rth
.
Q 1
or U1 =
È 1 ln ( r2 /r1 ) ln ( r3 /r2 )
r3
T
1 h1
h2 2p r1 L Í + +
2p
Î 1 1r Lh 2p Lk A 2p Lk B
T
r2 r1 2
1
ln ( r4 /r3 ) 1 ˘
+ +
2p LkC 2p r4 Lh ˙˚
(a) Composite sphere 1
U1 =
T T1 T2 T3 T 1 r1 ln ( r2 /r1 ) r1 ln ( r3 /r2 )
1 2
Q + +
Rconv R1
h kA kB
1
R2 Rconv
2
1 ...(30.25)
(b) Equivalent thermal resistences r1 ln ( r4 /r3 ) r1
+ +
Fig. 30.13 kC r4 h2
Elements of Heat Transfer 1033

where U1 = overall heat-transfer coefficient based Now, the heat flow for a hollow cylinder can be
on inner surface area A1 (= 2p r1L). It may also be written as
defined in terms of any of the intermediate area A k (T - T )
providing Q = m 1 2 ...(30.28)
r2 - r1
U1 A1 = U2 A2 = U3 A3 = U4 A4 This approach can be used to transform a cylinder
The specific forms of U2, U3 and U4 may be into an equivalent slab of thickness (r2 – r1).
evaluated from Eq. (30.19). .
Q
The expression of overall heat transfer
coefficient for hollow sphere can also be obtained T1 T2
r1 .
in similar manner. Q

r2
LOG MEAN AREA Area Am
of slab
Sometimes, it is convenient to use an expression r2 – r1
for the heat flow through a hollow cylinder of Fig. 30.14 Concept of log mean area
the same form as that of a plane wall. Then the
hollow cylinder can be replaced by a plane wall of Example 30.1 The wall of a furnace is constructed
thickness (r2 – r1) and area by log mean area Am. from 15 cm thick fire brick having constant thermal
Rewriting Eq. (30.17) conductivity of 1.7 W/m.K. The two sides of the wall are
maintained at 1400 K and 1150 K, respectively. What is
ln ( r2 /r1 ) the rate of heat loss through the wall which is 50 cm ¥
Rcyl =
2p Lk 3 m on a side?
Rearranging this equation as
Solution
È 2p r2 L ˘
ln Í
2p r1L ˙˚
Given The wall of a furnace
= 2 1 ¥ Î
(r – r ) T1 = 1400 K
Rcyl
( r2 – r1 ) 2p Lk A = 50 cm ¥ 3 m = 0.5 ¥ 3 = 1.5 m2
( r2 – r1) ln ( A2 / A1) k = 1.7 W/mK
= T2 = 1150 K
( A2 - A1) k
L = 15 cm = 0.15 m
( r2 – r1 )
or Rcyl = ...(30.26) To find Heat loss through the wall.
Am k
A2 – A1 Assumptions
where, Am = ...(30.27) (i) Steady state conditions.
ln ( A2 / A1 )
where, A2 = 2p r 2L = area of outer surface of
cylinder
A1 = 2pr1L = area of inner surface of T1

cylinder
Q
Am = Logarithmic mean area or log mean
area T2
1.7 W/mK
r2 – r1 = Thickness of cylinder
Rcyl = Thermal resistance 15 cm

Fig. 30.15
1034 Thermal Engineering

(ii) One dimensional heat conduction through the through the wall is given by
wall.
k0 ( T1 – T2 ) Ï b c 2 2 ¸
(iii) Constant properties. q= Ì1 + [T1 + T2 ] + ÈÎT1 + T1T2 + T2 ˘˚ ˝
L Ó 2 3 ˛
Analysis According to Fourier law of heat conduction.
kA (T1 - T2 ) Solution
Q =
L Given The relation for variable thermal conductivity as
Using numerical values
k = k0 (1 + bT + cT 2)
2
(1.7 W/m.K) ¥ (1.5 m ) ¥ (1400 K - 1150 K)
Q = Assumptions
0.15 m
(i) k0 is constant.
= 4250 W
(ii) Steady-state conditions.
Example 30.2 Hot air at 150°C flows over a flat plate (iii) One-dimensional heat conduction.
maintained at 50°C. The forced convection heat transfer
coefficient is 75 W/m2 .°C. Calculate the heat gain rate by
the plate through and area of 2 m2.
T1 2
k = k0(1 + bt + cT )
Solution
Given Flow of hot air over a plate
T2
T = 150°C Ts = 50°C
h = 75 W/m2 .°C
A = 2 m2
0 x
T = 150°C L
2
h = 75 W/m ◊ k Ts = 50°C

Analysis According to Fourier law of heat conduction,


To find Heat transfer rate by air to plate. Q dT
=q= –k
Assumptions A dx
(i) Steady state conditions. dT
or q = – k0 (1 + bT + cT 2)
(ii) Constant properties. dx
(iii) Heat is transferred by forced convection only. or qdx = – k0 (1+ bT + cT 2) dT
Analysis According to Newton’s law of cooling. Integrating both sides,
Q = hAs (T – Ts)
L T2
= (75 W/m2. K) ¥ (2 m2) ¥ (150 – 50) (°C)
3
= 15 ¥ 10 W = 15 kW
q
Ú
0
dx = – k0 ÚT1
(1 + bT + cT 2 ) dT

T
Example 30.3 The thermal conductivity of plane È T2 T3 ˘
2

or q (L – 0) = – k0 ÍT + b +c ˙
wall varies as: ÍÎ 2 3 ˙˚
T1
k = k0 (1 + bT + cT 2)
k0 Ï b 2 2 c 3 3 ¸
If the wall thickness is L and surface temperatures or q =– Ì(T2 - T1 ) + (T2 - T1 ) + (T2 - T1 ) ˝
L Ó 2 3 ˛
are maintained at T1 and T2. Show that the heat flux q
Elements of Heat Transfer 1035

Glass sheets
k0 (T1 - T2 ) Ï b
or q = Ì1 + [T1 + T2 ]
L Ó 2
Air
c ¸
+ ÈT22 + T1 T2 + T22 ˘ ˝ Air Air
3Î ˚˛ k1 k2 k1

Example 30.4 A thermopane window consists of h 1, T 1


h 2, T 2

two 5 mm thick glass (k = 0.78 W/m K ) sheets separated


by 10 mm stagnant air gap (k = 0.025 W/m K ). The
convection heat transfer coefficient for inner and outside 5 mm 10 mm 5 mm
air are 10 W/m2 K and 50 W/m2 K respectively.
T T
2 .
(a) Determine the rate of heat loss per m2 of the glass 1
Q
surface for a temperature difference of 60°C Rconv R1 R2 R3 Rconv
1 2
between the inside and outside air.
(b) Compare the result with the heat loss, if the
window has only a single sheet of glass of
thickness 5 mm instead of thermopane. L2 0.01
R2 = = = 0.40 K/W
(c) Compare the result with the heat flow, if window k2 A 0.025 ¥ 1
has made of sheet of glass 10 mm thick only. 1 1
Rconv2 = = = 0.020 K/W
h2 A 50 ¥ 1
Solution
(i) The heat-flow rate through the glass air window
Given A thermopane glass window, All resistances are in series, thus the total
L1 = L3 = 0.5 cm = 0.005 m resistance
L2 = 10 mm = 0.01 m S Rth = Rconv1 + R1 + R2 + R3 + Rconv2
k1 = k3 = 0.78 W/m K = 0.1 + 0.00641 + 0.40 + 0.00641
k2 = 0.025 W/m K + 0.020
DT = T 1 – T 2 = 60°C = 0.53282 m2 K/W
h1 = 10 W/m2 K T 1 –T 2
Q =
h2 = 50 W/m2 K S Rth
60
To find = = 112.60 W/m2
0.53282
(i) Heat flow rate through the glass per m2.
(ii) If the window has a single sheet of glass of 5 mm
(ii) Heat flow rate when window has only a glass
thick, the total thermal resistance;
sheet of 5 mm thickness.
S Rth = Rconv1 + R1 + Rconv2
(iii) When window has only a glass sheet of 10 mm
= 0.1 + 0.00641 + 0.02
thickness.
= 0.12641 K/W
Assumptions 60
(i) One dimensional steady-state heat flow. Q = = 474.65 W/m2
0.12641
(ii) Constant properties. The heat loss is about four times that of previous
Analysis The various specific thermal resistances (for case.
1 m2) are shown and calculated as, (iii) If the window has a glass sheet of 10 mm thick,
then total resistance are;
1 1
Rconv1 = = = 0.10 K/W S Rth = Rconv1 + 2R2 + Rconv2
h1 A 10 ¥ 1
= 0.1 + 2 ¥ 0.00641 + 0.02
L1 0.005
R 1 = R3 = = = 0.00641 K/W = 0.13282 K/W
k1 A 0.78 ¥ 1
1036 Thermal Engineering

The heat loss per m2;


60
Q = = 451.74 W/m2
0.13282

Example 30.5 An exterior wall of a house consists


of a 10.16 cm layer of common brick having thermal
conductivity 0.7 W/mK. It is followed by a 3.8 cm layer of
gypsum plaster with thermal conductivity of 0.48 W/mK.
What thickness of loosely packed rockwool insulation
(k = 0.065 W/m K) should be added to reduce the heat
loss through the wall by 80%?

Solution
Given
using numerical values.
L1 = 10.16 cm = 0.1016 m k1 = 0.7 W/mK DT
0.8196 DT =
L2 = 3.8 cm = 0.038 m k2 = 0.48 W/mK 0.1016 0.038 L3
+ +
k3 = 0.065 W/mK 0.7 0.48 0.065
L3
or 0.224 + = 1.121
To find Thickness of rockwool insulation 0.065
Assumptions or L3 = 0.0583 m = 5.83 cm, thickness of
rockwool insulation
(i) Steady-state heat conduction.
(ii) Heat conduction in one direction only. Example 30.6 A spherical thin-walled metallic
(iii) Constant properties. container is used to store liquid nitrogen at 77 K. The
(iv) No contact thermal resistance at the interfaces. container has a diameter of 0.5 m and is covered with an
Analysis Considering DT is the temperature difference evacuated reflective insulation system composed of silica
across the composite wall, then heat flow per unit area powder (k = 0.0017 W/m K). The insulation is 25 mm
or heat flux. thick and its outer surface is exposed to ambient air at
300 K. The convective heat transfer coefficient is known
Q1 DT to be 20 W/m2 .K. The Latent heat of vapourisation and
= q1 =
A L1 L2 density of liquid nitrogen are 2 ¥ 105 J/kg and 804 kg/
+
k1 k2 m3, respectively.
DT (a) What is the rate of heat transfer to the liquid
= = 4.458DT nitrogen?
0.1016 0.038
+ (b) What is the rate of liquid boil off?
0.7 0.48
After addition of insulation, heat loss is reduced by Solution
80%, therefore, permissible heat flux will only be 20%
of q1 Given A spherical metallic container filled with liquid
nitrogen
q1
or q2 = 0.2 ¥ = 0.8196 DT d = 0.5 m or r1 = 0.25 m hfg = 2 ¥ 105 J/kg
A
r2 = 0.25 m + 25 mm = 0.275 m k = 0.0017 W/mK
where q2 = heat flux with rock wool insulation and it can ho = 20 W/m2.K r = 804 kg/m3
be expressed as T 1 = 77 K T 2 = 300 K
DT
q2 = To find
L1 L2 L3
+ +
k1 k3 k3 (i) The heat transfer rate to the nitrogen, and
(ii) The mass rate of nitrogen boil off.
Elements of Heat Transfer 1037

Assumptions or on volumetric basis


(i) Steady-state conditions. m 0.235
V = =
(ii) One-dimensional heat transfer in radial direction r 804
only.
= 2.923 ¥ 10–4 lit/h ª 7 lit/day
(iii) Negligible resistance between container wall and
liquid nitrogen.
(iv) Constant properties. 30.10 PRINCIPLE OF HEAT
(v) Negligible radiation heat loss. CONVECTION
Analysis The thermal circuit involves a conduction and
In the preceeding articles, we have discussed the
convection resistance in series, therefore, total resistance,
heat transfer by conduction and have considered
( r2 - r1 ) 1 the convection only to the extent of boundary
S Rth = Rsph + Rconv = +
4p k r1r2 4p r22 ho conditions. As discussed earlier, the convection
0.275 - 0.25 1 is the mode of heat transfer, in which energy is
= +
4p ¥ 0.0017 ¥ 0.25 ¥ 0.275 4p (0.275) 2 ¥ 20 transferred between a surface and a moving fluid.
= 17.074 K/W
The convection heat transfer comprises of two
The rate of heat transfer to liquid nitrogen, mechanisms. First is transfer of energy due to
random molecular motion (diffusion), and second
T –T 300 – 77
Q = 2 1
= = 13.06 W is the energy transfer by bulk or macroscopic
S Rth 17.074 motion of the fluid. The molecules of fluid are
The heat loss to nitrogen will cause evaporation of moving collectively or as aggregates and thus carry
nitrogen energy from a high-termperature region to a low-
temperature region. Hence, the heat-transfer rate
Thus, Q = m hfg
increases in presence of temperature gradient. The
Q 13.06 convection heat transfer is due to superposition of
or m = = 5
= 6.53 ¥ 10–5 kg/sec
h fg 2 ¥ 10 energy transfer by random motion of the molecules
= 0.235 kg/h and by the bulk motion of the fluid.
Convection heat transfer may be classified
.
m.hfg according to the nature of fluid flow. If the fluid
Thin walled motion is artificially induced by a pump, fan or a
blower, that forces the fluid over a surface to flow,
r1 = 0.25 m
T = 300 K r2 = 0.275 m
the heat transfer is said to be forced convection.
2
ho = 20 W/m2.K If the fluid motion is set up by buoyancy
Liquid nitrogen at effects resulting from density difference caused by
T = 77 K
1 temperature difference in the fluid, the heat transfer
Q Insulation is said to be by free (or natural ) convection. For
(a) Schematic of hollow sphere filled with nitrogen an example, a hot plate vertically suspended in
stagnant cool air, causes a motion in the air layer
T T
1 2 adjacent to the plate surface because of temperature
Rsph Rconv Q difference in air, which gives rise to density gradient
(b) Thermal resistance network which in turn set-up the air motion.
Fig. 30.20
1038 Thermal Engineering

30.11 CONVECTION BOUNDARY ary layer as shown in Fig. 30.22. The flow region
LAYERS over a surface in which the temperature variation
in the direction normal to surface is significant is
the thermal boundary layer. It is denoted by dth and
When fluid flows over a surface, the fluid particles is characterized by temperature gradients and heat
adjacent to the surface get zero velocity. These transfer. The thickness of thermal boundary layer
particles, then act to retard the motion of particles dth at any location along the surface is defined as
in the adjoining fluid layer, which acts to retard the the distance from the surface at which the tempera-
motion of particles in the next layer and so on, until ture difference (T – T ) equals 0.99 (Ts – T )
a distance y = d from the surface reaches, where y
these effects become negligible. The region of
flow over the surface, bounded by d in which the
effects of the viscous shearing forces caused by the u

fluid viscosity are felt is called boundary layer or T


Thermal
boundary
velocity boundary layer or hydrodynamic boundary layer
layer. The thickness of boundary layer d is generally dth
defined as a distance from the surface at which the Ts < T

local velocity u = 0.99 of free stream velocity u . x


The retardation of fluid motion in the boundary T(x, y) = Ts + 0.99 (Ts – T )
layer is due to the shear (viscous) stress acting in
opposite direction. With increasing the distance y Thermal
over a cold surface
from the surface, shear stress decreases, the local
velocity u increases until it approaches u . With
increasing the distance from the leading edge, the
effect of viscosity penetrates further into the free The analysis of convection problems require the
stream and boundary layer thickness grows (d knowledge of the type of boundary layer developed,
increases with x) as shown in Fig. 30.21. whether it is laminar or turbulent. The skin friction
and convection coefficient depend strongly on such
conditions.
If the fluid flowing on a surface has a different tem- The developed boundary layer may consist of
perature than that of the surface, the thermal layer laminar boundary, transition region and turbulent
boundary developed is similar to velocity bound- boundary layer.

Laminar boundary Transition


Turbulent boundary
layer region
layer
Turbulent
u layer

u (x, y)
Boundary
layer u
y Thickness d (x)
Buffer layer
Viscous sublayer

xc d (x)
x u x Boundary
Recr =
v Layer Thickness
Elements of Heat Transfer 1039

The velocity boundary layer d (x) is characterized The transition from laminar to turbulent boundary
by the presence of velocity gradients and shear layer occurs, when
stresses. The thermal boundary layer dth (x) is u d
ReD = > 2300 …(30.31)
characterized by temperature gradients and heat n
transfer.
The characteristic of fluid flow is governed by a
dimensionless quantity called the Reynolds number. It is defined as the ratio of the buoyancy forces to the
viscous forces acting in the fluid. It is used in free
30.12 PHYSICAL SIGNIFICANCE OF THE convection and its role is same as that of Reynolds
CONVECTION DIMENSIONLESS number in forced convection. The Grashof number
PARAMETERS characterises the type of boundary layer developed
in natural convection heat transfer. It is denoted by
Gr and expressed as
It is the ratio of inertia forces to viscous forces in Buoyancy forces g DrV
Gr = =
the velocity boundary layer. It is used in forced Viscous forces rn 2
convection and approximated as g bDTL3c
= ...(30.32)
Inertia forces n2
Re =
Viscous forces where, g = Acceleration due to gravity, m/s2
r u Lc m Lc b = Coefficient of volumetric expansion =
= = ...(30.29) 1/(Tmf + 273)
m n
DT = Temperature difference between sur-
where, Lc = Characteristic length of the geometry, face and fluid, °C or K
= distance from leading edge, in the flow direction n = Kinetic viscosity of fluid, m2/s
for a flat plate and d for a cylinder or sphere (m) Lc = Significant length of the body, m
u = free stream velocity (m/s) = Height, L for vertical plates and
r = fluid density (kg/m3) cylinders
m = dynamic viscosity (Ns/m2) = Diameter, d for horizontal cylinder
n = kinetic viscosity (m2/s) or viscous and sphere
diffusivity Surface area A
= = s
The Reynolds number characterizes the type of Perimeter P
flow, whether it is laminar of turbulent flow. 30.12.4
It is defined as the ratio of convection heat flux to
conduction heat flux in fluid boundary layer or
It is the value of Reynolds number, where boundary
layer changes from laminar to turbulent nature. It convection heat flux
Nu =
is denoted by Recr. The value of critical Reynolds conduction heat flux
number is different for different geometries. For hDT hLc
= =
flow over a flat plate, transition from laminar to k f DT /Lc kf
turbulent boundary layer occurs roughly when where, Lc = Characteristic length or length in
critical Reynolds number is direction of flow, m
Recr ≥ 105 …(30.30) h = The heat transfer coefficient, W/m2.K
In fluid flow through tubes, the Reynolds kf = Thermal conductivity of the fluid,
number is also used to characterize the fluid flow. W/m.K
1040 Thermal Engineering

The low value of Nu, indicates the more Rayleigh Number


conduction in the fluid as compared to convection in
It is the product of Grashof number and Prandtl
the boundary layer and large value of Nu, indicates
number in natural convection boundary layer. It is
large convection in the fluid.
denoted by Ra and used to characterize the type of
boundary layer in natural convection.
30.12.5 Prandtl Number
Ra = GrPr
It is defined as the ratio of the momentum diffusivity
For laminar boundary layer 104 £ Ra £ 108
n to the thermal diffusivity a
For turbulent boundary layer Ra ≥ 108
n m rC p mCp
...(30.36)
or Pr = = ¥ = ...(30.33)
a r kf kf The properties of the fluid are used at mean film
The Prandtl number provides a measure of temperature
relative effectiveness of momentum and energy Ts + T
transfer in the velocity and thermal boundary layer, Tf =
2
respectively. In other words, it compares the relative
thickness of velocity and thermal boundary layers. DIMENSIONAL ANALYSIS
For gases Pr @ 1
For liquid metal Pr >> 1 Dimensional analysis differs from conventional
methods of approach in which certain equations
Further, the thickness of two boundaries can be
are solved for a resulting equation. Instead, it
expressed as
combines several variables affecting a phenomenon
d th
ª Pr n in a dimensionless group, such as Nusselt number,
d which facilitates the interpretation and extends its
where, dth = thickness of thermal boundary layer application to experimental data.
d = thickness of velocity boundary layer The dimensional analysis does not provide
Pr = Prandtl number. any information about the nature of phenomenon,
where n is the exponent. hence success or failure of the method depends on
proper selection of affecting variables.
30.12.6 Stanton Number
It is the ratio of the Nusselt number to product Buckingham p Theorem
of Reynolds number and Prandtl number and is According to this rule, the required number of
expressed as independent groups that can be formed by combin-
Nux h ing physical variables, related to a phenomenon is
Stx = =
Rex Pr r C pu equal to the total number of these physical quan-
hDT tities, minus the number of primary dimension m,
or Stx = required to express the dimensional formulae for n
r C p u DT
physical quantities or independent groups are n–m.
Heat flux to the fluid
= …(30.34) The independent dimensionless groups can be
Heat transfer capacity of fluid expressed as
f (p1, p2, p3, …) = 0
Peclet Number
In a particular problem involving seven physical
It is the product of Reynolds number and Prandtl quantities which can be expressed in four primary
number. It is denoted by Pe and expressed as dimensions then
Pe = R e Pr …(30.35)
Elements of Heat Transfer 1041

Dimensionless p terms = n – m = 7 – 4 = 3 exponents are applied on the core group variable,


Hence f (p1, p2, p3) = 0 which are to be determined,
or the form p1 = f (p2, p3). (i) Expressing the Variable in their Primary
Dimensions for p 1,
30.13.2 Dimensional Analysis for the
p1 = M0 L0 T 0 t0
Forced Convection
= La (ML–3)b (ML–1t–1)c (MLt–3 T –1)d (Lt–1)
The force convection phenomenon can be Separating the exponents for
influenced by the variables given in the table below: M: 0 = b + c + d
L: 0 = a – 3b – c + d + 1
Table 30.2
T: 0 = – d
Sr. Parameters Symbol and Primary t: 0 = – c – 3d – 1
No. Unit Dimensions Solving these simultaneous equations, we get
1. Tube Diameter D, m L d = 0, c = –1
(Characteristic b = 1, a = 1
length) Hence, the dimensionless group formed is,
2. Fluid density r, kg/m3 ML –3 Dru
p1 = = Re (Reynolds number)
3. Fluid viscosity m, kg/m s ML–1 t –1 m
4. Fluid velocity u , m/s Lt –1
(ii) Expressing the Primary Dimensions for
5. Fluid thermal kf , W/m ◊ K MLt –3 T –1 Variables of p2,
conductivity
p2 = M0 L0 T 0 t0
6. Heat transfer h, W/m2 ◊ K Mt –3 T –1 = Le (ML–3) f (ML–1t –1)g (ML t –3T –1) h
coefficient
(L2 t–2 T –1)
7. Fluid specific heat Cp, J/kg ◊ K L2 t –2 T –1
Separating the exponents for
These seven variables are expressed in four pri- M: 0 = f + g + h
mary dimensions (M, L, T, t). Therefore, according L: 0 = e – 3f – g + h + 2
to Buckingham p theorem, the independent dimen- T: 0 = –h – 1
sionless groups formed are t: 0 = –g – 3h – 2
= No. of variable affecting the phenom- Solving these simultaneous equations, we get
enon – No. of primary dimensions used h = –1, g = 1
or 7 – 4 = 3( i.e., p1, p2, p3) f = 0, e = 0
Writing these three groups as, Hence, the dimensionless group formed is,
p1 = Da, rb, mc, kfd, u mCp
p2 = = Pr (Prandtl Number)
p2 = De, r f, mg, kfh, Cp kf
p3 = Di, r j, mk, kfl, h (iii) Expressing the Primary Dimensions for
where, D, r, m, k form a core group is called Variables of p3,
repeating variables and u , Cp and h are as p3 = M0 L0 T 0 t0
selected variables. The repeating variables are = Li (ML–3) j (ML–1 t–1)k (ML t –3 T –1)l
chosen arbitrarily such that they together involve (Mt –3 T –1)
all fundamental dimensions and they themselves Separating the exponents for
do not form a dimensionless parameter. Since the M: 0 = j + k + l + 1
groups p1, p2, p3 are dimensionless, hence certain
1042 Thermal Engineering

L: 0 = i – 3j – k + l Table 30.4 convection heat transfer


T: 0 = – l – 1
t: 0 = – k – 3l – 3
Correlation Geometry Remark
Solving these simultaneous equations, we get Condition
l = –1, k = 0 Flat plate, Thickness of
5.0 x
j = 0, i = 1 d= Laminar velocity of
Rex
Hence, the dimensionless group formed is, boundary layer
hD 0.646 Flat plate, Local coefficient of
p3 = = Nu (Nusselt number) Cf x =
kf Rex Laminar friction
Hence for forced convection,
Nux = 0.332 Flat Plate, Local Nusselt
Nu = f (Re, Pr) ...(30.37)
Re 1/2
x Pr
1/3
Laminar number

30.14 SUMMARY OF DIMENSIONLESS d th = d Pr –1/3 Flat Plate, Thickness of


PARAMETERS AND THEIR Laminar thermal boundary
layer
CORRELATIONS
1.328 Flat plate, Average coefficient
Cf = Laminar of friction
Dimensional groups of heat transfer ReL

Groups Definition Interpretation NuL = Flat plate, Average Nusselt


0.664 ReL1/2 Pr1/3 Laminar number
Coefficient of ts Dimensionless
friction (Cf) 2 surface shear 0.059 Flat plate, Local coefficient of
r u /2 Cf x = Turbulent friction
stress Re1x/5
Friction factor Dr Dimensionless
(f) pressure drop 0.37 x Flat plate, Thickness of
( L/D ) ru 2 /2 d= Turbulent velocity of
Re1x/5
Grashof number Ratio of boundary layer
g b(Ts - T ) L3c
(GrL) buoyance forces Nux = 0.296 Flat plate, Local Nusselt
n2 to viscous forces Re x4/5 Pr1/3 Turbulent number
Nusselt number hLc Ratio of
Nud = CRedm Cylinder Average Nusselt
(NuL) convection
kf Pr1/3 number, Table 30.5
heat flux to
conduction heat Nud = 2 + Sphere Average Nusselt
flux (0.4 Red1/2 + number
0.06 Red2/3)
Prandtl number mCp Ratio of
Pr 0.4 (m /m s)
(Pr) kf momentum
diffusivity
to thermal Table 30.5
diffusivity
Reynolds u Lc Ratio of inertia Red C m
number (ReL) n forces to viscous
4 – 40 0.911 0.330
forces
40 – 4000 0.683 0.466
4000 – 40000 0.193 0.681
40000 – 400000 0.027 0.805
Elements of Heat Transfer 1043

Example 30.8 Air at 27°C and 1 atm flows over The average value of heat transfer coefficient
a heated plate with a velocity of 2 m/s. The plate is h1 = 2hx1 = 2 ¥ 6.15 = 12.3 W/m2. K
at uniform temperature of 60°C. Calculate the heat The heat transfer rate upto x = 0.2 m
transferred (a) first 0.2 m of the plate and (b) first 0.4 m Q1 = h1 As (DT )
of the plate.
Q1
= (12.3 W/m2.K) ¥ (0.2 m)
Solution L
¥ (60 – 27) (K)
Given The flow over a heated flat plate = 81.18 W/m
T = 27°C p = 1 atm (ii) The heat transfer from 0.4 m
u = 2 m/s Ts = 60°C
Reynolds no.
x1 = 0.2 m x2 = 0.4 m
u x2 2 ¥ 0.4
To find Rex2 = = = 46082
n 17.36 ¥ 10 6
(i) Heat transfer from first 0.2 m, and The local value of heat transfer coefficient
(ii) Heat transfer from first 0.4 m.
Nux2 = 0.332 Rex21/2 Pr 1/3
Assumptions 0.02749
or hx2 = ¥ 0.332
(i) No heat radiation exchange, 0.4
(ii) The unit depth in z direction, ¥ (46082)1/2 (0.7)1/3
(iii) Air and surface temperatures are different, taking = 4.35 W/m2. K
the properties at mean film temperature. Average heat transfer coefficient
Properties of air The mean film temperature h2 = 2hx2 = 2 ¥ 4.35 = 8.7 W/m2 ◊ K
Ts + T 60 + 27 The heat transfer
Tf = = = 43.5°C
2 2 Q2
= (8.7 W/m2. K) ¥ (0.4 m)
The properties of air at 43.5°C (from properties of air L
¥ (60 – 27) (K)
in appendix, Table A-8)
= 114.8 W/m
n = 17.36 ¥ 10 –6 m2/s. kf = 0.02749 W/m ◊ K
Pr = 0.7 Cp = 1.006 kJ/kg ◊ K
Analysis The Reynolds number at x = 0.2 m
u x The dimensionless parameters such as Reynolds
Rex1 =
n number, Prandtl number and Nusselt number can
( 2 m/s) ¥ (0.2 m) also be used in forced convection through ducts.
= = 23041
(17.36 ¥ 10 - 6 m 2 /s) For flow through tubes the characteristic length Lc
is set to the diameter of the tube d.
(i) The heat transfer from first 0.2 m
The local value of heat transfer coefficient can be In majority of fluid flow through tubes, the
calculated as nature of the flow is turbulent and the correlation
used for approximation of friction factor and
hx1 x
Nux1 = = 0.332 Rex11/2 Pr 1/3 Nusselt number as
kf
f = 0.316 Re–1/4 for Re < 2 ¥ 104
0.332 ¥ (0.02749 W/m ◊ K) = 0.184 Re– 0.2 for 2 ¥ 104 < Re < 3 ¥ 105
or hx1 =
(0.2 m) Nu = 0.023 Re0.8 Prn
¥ (23041)1/2 (0.7)1/3 where n = 0.4 for heating
= 6.15 W/m2. K = 0.3 for cooling
1044 Thermal Engineering

hd Cp = 1025 J/kg ◊ K kf = 0.0386 W/m.K


Nu = m = 2.57 ¥ 10–5 kg/m-s Pr = 0.681
kf
um d Analysis The density of air at 200 kPa 200°C is
Re =
n p 200 kPa
r = =
m Cp n RT (0.287 kJ/kg ◊ K) (473 K)
Pr = =
kf a = 1.473 kg/m3
For flow through non-circular ducts, the tube The Reynolds number is
diameter d is replaced by the hydraulic diameter of r um D
ReD =
a non-circular duct, defined as m
4 Ac 1.473 ¥ 10 ¥ 0.025
Dh = = 5
= 14332
P 2.57 ¥ 10
4 ¥ Cross-section area of duct Since Red > 2300, hence the flow is turbulent.
= .
wetted perimeter Using correlation
hD
Example 30.9 Air at 200 kPa and 200°C is heated Nud = = 0.023 Red0.8 Pr0.4
kf
as it flows through a tube with a diameter of 25 mm at a
velocity of 10 m/s. Calculate the heat-transfer rate per = 0.023 ¥ (14332)0.8 (0.681)0.4 = 41.69
unit length of the tube, if a constant heat-flux condition kf 0.0386
is maintained at the wall and the wall temperature is or h = Nud = ¥ 41.69
D 0.025
20°C above the air temperature, all along the length of
the tube. How much would the bulk temperature increase = 64.37 W/m2. K.
over 3 m length of the tube? (i) The heat transfer rate per metre length
Q
Solution = h (p D) (DT )
L
Given Uniform heating of the tube = 64.37 ¥ (p ¥ 0.025) ¥ (20)
= 101.1 W/m
q W/m2 (ii) Bulk temperature rise
Making the energy balance over 3 m length of the
200 kPa
D = 25 mm

u = 10 m/s tube;
200°C DT = 20°C Heat supply rate
= Enthalpy rise rate of the fluid
L=3m Q
¥ L = m Cp (DTb)
L
when m is mass flow rate of the air and it can be
To find calculated by continuity equation;
(i) Heat transfer rate per unit length of the tube, p D2
m = r um Ac = r um
(ii) Bulk temperature rise over 3 m length of the tube. 4
p ¥ (0.025) 2
Assumptions = (1.473) ¥ (10)
4
(i) Steady-state heat transfer conditions.
(ii) Fully developed flow through a tube. = 7.329 ¥ 10–3 kg/s
(iii) Conduction and radiation effects are negligible. Using mass-flow rate in the energy balance
Properties of the air. The properties of air at tempera- 7.329 ¥ 10–3 ¥ (1025) ¥ (DTb) = 3 ¥ 101.1
ture of 200°C from Table A-8 DTb = 40.37°C
Elements of Heat Transfer 1045

FREE CONVECTION the heat transfer rates from the plate for the cases when
the vertical height is (a) 0.2 m, and (b) 0.4 m.
When the fluid is heated, the density gradients are
Solution
developed, and results into buoyancy force, which
induces free convection. Such a situation is referred Given
as natural or free convection. The buoyancy effect L1 = 0.2 m L2 = 0.4 m
is developed due to the presence of fluid density Ts = 80°C T = 24°C
gradients and body force (gravitational force).
In free convection, the fluid motions set up To find Comparison of heat transfer rates when the
vertical height is (i) 0.2 m, and (ii) 0.4 m
by buoyancy forces are much smaller than those
associated with forced convection. Therefore, Properties of fluid The mean film temperature;
the heat transfer rate in natural convection is also Ts + T 80 + 24
smaller. Tf = = = 52°C = 325 K
2 2
There are many situations, where the heat is
The properties of air are:
transferred by free convection to the surrounding
n = 1.822 ¥ 10–5 m2/s, Pr = 0.703
air. Heat transfer from a heater to heat a room, heat
kair = 0.02814 W/m ◊ K,
transfer from pipes, transmission line, condenser
b = 1/325 = 3.077 ¥ 10–3 K–1
coil of a refrigerator, electric transformer, electric
motors and electronic equipments are some typical Analysis The Grashof number with appropriate signifi-
examples. cant length Lc of plate:

The Grashof no.,


30.17 EMPIRICAL RELATIONS FOR FREE
CONVECTION g b DT L3c
Gr =
v2
The empirical relations are presented in following
(9.81) ¥ (3.077 ¥ 10 – 3 ) ¥ (80 – 24) L3c
functional form for variety of applications, =
(1.822 ¥ 10 – 5 ) 2
h Lc
Nuf = = C (Grf ◊ Prf )m = 5.092 ¥ 109 L c3
kf
The Reyleigh no.,
where Lc is significant length depending on the Ra = Gr Pr = (5.092 ¥ 109 L c3) ¥ (0.703)
position of flow. = 3.579 ¥ 109 ¥ Lc3
C and m are constants and subscript f indicates (i) When 0.2 m side is vertical:
that the properties (except for inclined plate) in the Lc = L1 = 0.2 m
dimensionless group are evaluated at mean film RaL1 = 3.579 ¥ 109 ¥ (0.2)3
temperature, = 28.637 ¥ 106
Ts + T Thus, the flow is laminar, and relation from
Tf = Table 30.6;
2
NuL1 = 0.59 (RaL1)1/4
The product of Grashof number and Prandtl
= 0.59 ¥ (28.637 ¥ 106)1/4
number is called Rayleigh number or
= 43.176
Ra = Gr Pr
The average heat transfer coefficient
Example 30.10 Consider a rectangular plate of size kair 0.02814
h1 = NuL1 = 43.176 ¥
0.2 m ¥ 0.4 m is maintained at a uniform temperature of L1 0.2
80°C. It is placed in atmospheric air at 24°C. Compare = 6.075 W/m2. K
1046 Thermal Engineering

Table 30.6

Sr. Geometry Significant Types of Flow Range of Gr Pr Correlation Average, Nu =


No. Length Lc
1. (a) Vertical Planes and Cylinders Height Laminar 104 £ Ra £ 108 0.59 (Ra)1/4
Turbulent 108 £ Ra £ 1012 0.13 (Ra)1/3
(b) Vertical Plates Height No restriction Ï0.825 ¸
Ô 1/6 Ô
Entire range Ì 0 . 387 Ra ˝
Ô + 9/16 8/27 Ô
or RaL [1 + ( 0 . 492 / Pr) ]
Ó ˛
2. Horizontal Diameter Laminar 104 £ Ra £ 108 0.53 (Ra)1/4
Cylinder Turbulent 108 £ Ra £ 1012 0.13 (Ra)1/3
3. The horizontal Plates
(i) Heated surface facing down Laminar 104 £ Ra £ 107 0.54 Ra1/4
or Cold surface facing up As/P Turbulent 107 £ Ra £ 1011 0.14 Ra1/3
(ii) Heated surface facing up or Laminar 105 £ Ra £ 1010 0.27 Ra1/4
Cold surface facing down
4. Spheres Diameter Laminar 1 £ Ra £ 105 2 + 0.43 Ra1/4
Turbulent 105 £ Ra £ 108 2 + 0.50 Ra1/4
5. Inclined hot surfaces 105 £ Ra £ 1011 0.56 (Ra . cos q)1/4
(i) Facing downward and q < 88°
(ii) Facing upward Height 105 £ Ra £ 1011
and – 15° > q > 0.415 {Ra1/3 – Ra c1/3}
75° + 0.56 (Rac cos q)1/4

The heat transfer rate:


Q1 = h1 As (Ts – T )
T = 24°C
Ts = 80°C = 6.075 ¥ 0.2 ¥ 0.4 ¥ (80 – 24)
0.2 m
= 27.216 W
0.4 m (ii) For the different vertical orientation of the plate
(a) 0.2 m side of the plate is vertical. of L2 = 0.4 m. The relevant Rayleigh no.
RaL2 = 3.579 ¥ 109 ¥ (0.4)3
= 229.0 ¥ 106
The boundary layer is laminar, hence using
relation
L2 = 0.4 m

Ts = 80°C T = 24°C NuL2 = 0.59 (RaL2)1/4


= 0.59 ¥ (229.0 ¥ 106)1/4 = 72.58
kair 0.02814
and h2 = NuL2 = 72.58 ¥
L2 0.4
0.2 m
= 5.10 W/m2. K
(b) 0.4 m side of the plates is vertical.
The heat transfer rate:
Q2 = h2 As (Ts – T )
= 5.10 ¥ 0.2 ¥ 0.4 ¥ (80 – 24)
= 22.848 W
Elements of Heat Transfer 1047

The percentage decrease in heat transfer magnitude of emitted energy by a body at a given
(Q1 - Q2 ) ( 27.216 - 22.848) temperature under ideal conditions.
= = 0.16 Both concept are used in study of thermal
Q1 27.216
radiation.
The heat transfer rate is 16% lower when the vertical
side is 0.4 m instead of 0.2 m.

RADIATION HEAT TRANSFER It is an ideal surface having the following properties:


1. A black body absorbs all incident radiation
Thermal radiation refers to the heat energy emitted from all directions at all wavelengths.
by the bodies because of their temperatures.
2. At any temperature and wavelength, no body
All bodies at a temperature above absolute zero
can emit energy more than a black body.
temperature emit thermal radiation. For example,
3. Although the radiation emitted by a black
the energy emitted by sun travels through space and
body depends upon wavelength and temper-
reaches the earth surface. The energy transfer by
ature, it is independent of direction.
radiation does not require any medium between hot
and cold surfaces. In fact, the radiation heat transfer 4. A black body neither reflects nor transmits
is more effective in vacuum. any amount of incident radiation.
Consider a radiation beam entering the cavity of
THEORIES OF RADIATION an enclosure as shown in Fig. 30.25. It experiences
many reflections within the enclosure and almost
The actual mechanism of radiation propagation is entire beam is absorbed by the cavity and the black
not fully understood till date. Still two theories are body behaviour is experienced.
in use.

Maxwell’s Theory
According to Maxwell’s electromagnetic theory,
the energy is transferred from a hot body to cold
body in the form of the electromagnetic waves.
All electromagnetic waves travel with the speed
of light. This concept is useful in study for the
prediction of the radiation properties of the surfaces
and materials.

Max Planck’s Theory


According to max Planck’s concept, the propagation Fig. 30.25 Characteristic of a black body
of thermal radiation takes place in the form of
discrete quanta called photons, each quantum
having an energy of Black Body Spectral Emissive Power
E = hn Max Planck’s law is based on quantum theory, and
where h is Planck’s constant and n is the frequency he has explained that the radiation energy emitted
of photons. This theory is used to predict the by a black body into vacuum is related as
1048 Thermal Engineering

C1 Emissive Power of a Black Body


Ebl (T ) = ...(30.38)
l 5 .{exp [C2 /( lT )] - 1} The total or hemispherical emissive power is the
where, C1 and C2 are constants, and their magnitude amount of radiation energy emitted by a black body
C1 = 3.743 ¥ 108 W. mm4/m2 at a temperature per unit area per unit time over
C2 = 1.4387 mm.K entire spectrum of wavelength. It is measured in
T = Absolute temperature, K W/m2.
l = Wavelength, mm The total emissive power of a black surface is
calculated as
Ebl = Spectral blackbody emissive power,
W/m2 mm Eb =
Ú 0
Ebl d l (30.39)
109
Visible radiation
Spectral Emissive Power, Ebl,W/m .mm

108
Emissivity
7 lmax T = 2898.6 mm.K
2

10
106 It is the ratio of radiation heat flux emitted by a real
5800 K
105 surface at a temperature T, over all wavelengths
K
n
ra olar
io

00

103
at

into hemispherical space, to that which would have


20
di
S

102 been emitted by a black body at same temperature.


00
10

10
Mathematically
1
K
0
30

0.1 E E
–2 e = = ...(30.40)
Ú
10
K

Eb
Ebl d l
0

–3
10

10
–4 0
10 For real surfaces, the emissive power
0.1 100
0.2 0.4 0.6 1 2 3 4 6 10 20 40 60
Wave length, l, mm E = e Eb = e s T 4 ...(30.41)
Fig. 30.26
30.21 SURFACE ABSORPTION,
REFLECTION AND
TRANSMISSION
When spectral emissive power Ebl(T) is plotted
Monochromatic Irradiation
against wavelength l against various temperature
of a black body, it is observed that as temperature It is also called spectral irradiation Gl (W/m2. mm)
increases, rate of emission increases, and peak of and is defined as the radiant energy incident on a
spectral distribution shifts to shorter wavelength as surface per unit wavelength about wavelength l
shown in Fig. 30.26. from all directions and expressed as
dG
Wein’s Displacement Law Gl = ...(30.42)
dl
Wein found that the product of wavelength and The total irradiation G (W/m2) is the total
absolute temperature corresponds to the locus of all radiation energy incident per unit area per unit time
peaks is always constant and his analytical formula over the entire wavelength from all directions. It
is may be evaluated as
(l T )max = 2897.6 mm K
This formula is valid over entire spectrum of
wavelength for a black body.
G =
Ú
0
Gl d l ...(30.43)
Elements of Heat Transfer 1049

Monochromatic Properties
In most situations, the spectral irradiation in form of When radiation beam is incident on a non-black
a beam incidents on a body as shown in Fig. 30.27, surface, a fraction is reflected by the surface. The
may be absorbed, reflected and transmitted. reflectivity r of a surface is defined as the fraction
of radiation energy incident on a surface from all
1. Monochromatic absorptivity al is the
directions over all wavelengths, that is reflected. It
fraction of monochromatic irradiation
is expressed as;
absorbed.
Gr
2. Monochromatic reflectivity rl is the fraction r = ...(30.46)
of monochromatic irradiation reflected. G
3. Monochromatic transmissivity tl is the frac- If the surface is perfectly smooth and the angle of
tion of monochromatic energy transmitted. incident and reflected rays is equal, the reflection is
With this consideration al + rl + tl = 1 ...(30.44) called specular reflection as shown in Fig. 30.28(a).
If the surface has some roughness, the incident
Incidence radiation is scattered in all directions after reflection.
radiation
G Reflected Such a reflection is called diffuse reflection as
rG
shown in Fig. 30.28(b).
The reflection from real surfaces is neither
specular nor diffuse but combination of diffuse and
Semi specular behaviour as shown in Fig. 30.28(c).
Absorbed
transparent
aG
medium

Transmitted
tG

Fig. 30.27

Absorptivity
A black body absorbs all incident radiation, hence
its absorptivity is considered unity. But real sur-
faces do not absorb all energy incident on it. The
total or average or hemispherical absorptivity a is
defined as fraction of radiation energy incident on
the surface from all directions, over entire wave-
length spectrum, that is absorbed by the surface.
Mathematically,
Fig. 30.28
Ga
a= ...(30.45)
G
where,
Ga = Energy absorbed by the surface, W/m2 Transmissivity
G = Irradiation, W/m2 When a radiation beam is incident on a semi-
transparent surface, a part is reflected, a part is
1050 Thermal Engineering

absorbed and the remaining is transmitted. Hence, E l1 El 2 El 3 El b


transmissivity is the fraction of incident energy = = = =C ...(30.51)
a l1 a l 2 a l 3 a lb
transmitted through the surface. Mathematically, Since alb =1
Gt E l1
t= ...(30.47) = al1
G El b
and for average properties:
a+r+t =1 ...(30.48) or El1 = al1 ...(30.52)
Similarly, for other bodies, it can be shown
White Body A body is called white body when it
that at thermal equilibrium, the energy emitted
reflects almost all radiation incident upon it. For a
by a surface must be equal to energy absorbed by
white body,
the surface. Hence, spectral emissivity is equal to
r ∫1 spectral absorptivity at thermal equilibrium. This
Opaque Body For an opaque surface, there is no law is applicable when the radiation properties
transmission; thus the reflectivity and absorptivity are independent of wavelength (for graybodies)
are or when incident and emitted radiation have the
a+r =1 ...(30.49a) spectral distribution.
and al + rl = 1 ...(30.49b) Example 30.11 A black surface is positioned in
Gray Surface The radiation properties of a real a vacuum container so that it absorbs incident solar
surface such as absorptivity, emissivity, etc., depend radiant energy at the rate of 950 W/m2. If the surface
conducts no heat to its surroundings, determine its
on the wavelength of radiation. The calculation of
equilibrium temperature.
radiation heat transfer for all wavelength becomes
very tedious. To overcome such difficulties, an Solution
uniform emissivity is assumed over the entire
Given q = 950 W/m2
wavelength spectrum. Thus, a gray surface is
defined as a surface for which the monochromatic To find To calculate the equilibrium temperature.
emissivity el is independent of the wavelength. Assumptions
(i) Stefan Boltzmann constant s = 5.67 ¥ 10 –8
W/m2.K4.
(ii) Not heat loss by conduction and convection.
It is defined as total radiation energy, which leaves
a surface due to emission and reflections per unit Analysis The radiant heat flux for a black surface can
time per unit area. It is denoted by J and measured be expressed as
in W/m2 q = s ◊ T4

J = eEb + rG ...(30.50) Hence, T = (q/s)1/4 = [950/5.67 ¥ 10–8)]1/4 = 360 K


J = Radiation emitted by the surface per The equilibrium temperature of black surfaces will
unit area + Radiation reflected by the be, T = 87°C
surface per unit area.
Example 30.12 A black body at 30°C is heated to
100°C. Calculated the increase in its emissive power.

Solution
It states that at thermal equilibrium, the ratio of the
spectral emissive power to spectral absorptivity for Given A black body emission
all bodies is constant or T1 = 30 + 273 = 303 K
T2 = 100 + 273 = 373 K
Elements of Heat Transfer 1051

To find The increase in emissive power. Analysis According to Planck’s law, Eq. (30.38)
Analysis The radiant heat flux or emissive power for a C1
Ebl (T ) =
black surface can be expressed as Ï ÊC ˆ ¸
l 5 Ìexp Á 2 ˜ -1˝
Eb = s T 4 Ó Ë lT ¯ ˛
Where, s = Stefan Boltzmann’s constant, as 5.67 ¥
Where, C1 = 3.743 ¥ 108 W mm4/m2,
10–8 W/m2 ◊ K4.
C2 = 1.4387 ¥ 104 mm.K
Hence the increase in emissive power can be
calculated as Using numerical values,
Eb2 – Eb1 = s (T24 – T14) 3.743 ¥ 108
Ebl (T ) =
or Eb2 – Eb1 = (5.67 ¥ 10–8 W/m2. K4) ÏÔ Ê 1.4387 ¥ 10 4 ˆ ¸Ô
{(373 K)4 – (303 K)4} ( 4)5 Ìexp Á ˜ - 1˝
2 ÔÓ Ë 4 ¥ 600 ¯ Ô˛
= 619.62 W/m
= 913.25 W/(m2. mm)
Example 30.13 The surface temperature of a
central heating radiator is 60°C. What is the net black Example 30.15 A black body emits energy at 2000 K.
body radiation heat transfer between radiator and Determine the wavelength, at which the black-body
surroundings at 20°C? spectral emissive power would be maximum.

Solution Solution
Given Central heating radiator with Given A black-body emission
Ts = 60°C = 333 K T = 2000 K
T = 20°C = 293 K
To find Wavelength corresponds to maximum Ebl .
To find Radiation heat transfer.
Analysis Using Wien’s displacement law, Eq. (30.49)
Assumptions
(i) The Stefan Boltzmann constant (lT)max = 2897.6 mm.K
s = 5.67 ¥ 10–8 W/m2.K4 2897.6
or l = = 1.448 mm
2
(ii) 1 m surface area of radiator 2000
Example 30.16 A large spherical enclosure (black
Analysis The black-body radiation heat transfer per
body) maintains its inner surface at 1000 K. It has a
unit area is expressed as
hole of 0.4 cm diameter. Calculate the rate of emission of
Q radiation energy through this opening.
q = = s (Ts4 – T 4)
A
= 5.67 ¥ 10–8 ¥ (3334 – 2934) Solution
= 5.67 ¥ 10–8 ¥ 4.9263 ¥ 109 Given Radiation through an opening of large spherical
= 279.32 W/m2 enclosure.
T = 1000 K D = 0.4 cm
Example 30.14 A black body at 600 K emits radiation
at a wavelength of 4 mm. Calculate its spectral emissive To find Rate of emission of radiation energy through
power. hole
Analysis The black-body emissive power
Solution
Eb(T ) = s T 4
Given Black-body radiation = (5.67 ¥ 10–8) ¥ (1000)4 = 56700 W/m2
T = 600 K l = 4 mm p 2 p
Hole area A = D = ¥ (0.4)2 = 0.125 cm2
To find Spectral emissive power. 4 4
= 0.125 ¥ 10– 4 m2
1052 Thermal Engineering

Rate of emission of radiation energy through hole Qrad


opening = s e (p D) (T s4 – T 4)
L
Q = A Eb(T )
= 5.67 ¥ 10–8 ¥ 0.8 ¥ (p ¥ 0.05)
= 0.125 ¥ 10–4 ¥ 56700 = 0.7125 W
(5004 – 2984)
Example 30.17 An uninsulated steam pipe is passed = 389.13 W/m
through a room in which air and walls are at 25°C. Total heat loss from pipe surface per unit length,
The outer diameter of the pipe is 50 mm and surface Ê Qˆ Qconv Qrad
temperature and emissivity are 500 K and 0.8 respectively. ÁË L ˜¯ = L + L = 476 + 389.13
If the free convection heat-transfer coefficient is = 865.13 W/m
15 W/m2.K. What is the rate of heat loss from the surface
per unit length of pipe?
Table 30.7
Solution
Given Flow of air over a hot cylinder Terms Definition
T = Tw = 25°C = 298 K Absorption The process of converting the
Surface temp. T = 500 K radiation intercepted by the matter
Dia. of pipe D = 50 mm = 0.05 m to internal thermal energy
Emissivity of pipe surface e = 0.8 Absorptivity Fraction of the incident radiation
Convection coefficient h = 15 W/m2.K absorbed by the matter
Black body Ideal body which absorbs all
0.05 m L
incident radiation and emits
maximum energy
Emission The process of radiation production
h = 15 W/m2 .K
e = 0.8 by the matter at a finite temperature
T = 25°C
Emissive power The rate of radiant energy emitted
by a surface in all direction per unit
area of the surface, E (W/m2)
To find Heat loss per unit length of pipe (Q/L). Emissivity The ratio of the emissive power of a
surface to the emissive power of the
Assumptions black body at the same temperature
(i) Steady state conditions.
Gray surface A surface for which the spectral
(ii) Heat loss by radiation and convection heat absorptivity and emissivity are
transfer. independent of the wavelength over
(iii) Constant properties. the spectral region of the surface
(iv) Stefan Boltzmann constant, s = 5.67 ¥ 10–8 irradiation and emission
W/m2 ◊ K4.
Kirchhoff’s The emission is equal to absorption
Analysis Law for all bodies at thermal equilibrium
(i) Heat loss from the pipe by convection is given by Planck’s aw
l It is associated with spectral
Qconv = h As (Ts – T ) = h (pDL) (Ts – T ) distribution of emission from a
Qconv black body
or = 15 ¥ p ¥ 0.05 ¥ (500 – 298)
L Radiation shield It is made of low emissivity
= 476 W/m material and is used to reduce the
(ii) Heat loss per unit length of pipe by radiation is net radiation transfer between the
given by two surfaces
Contd.
Elements of Heat Transfer 1053

Contd. heat-transfer process, flow arrangement and type of


Reflection The process of reflection of the construction.
radiation energy incident on a (A) According to Heat Transfer Process
surface
Reflectivity The fraction of incident radiation
In this type of heat exchanger,
(i) Direct-contact Type
energy reflected by the matter the two immiscible fluids at different temperatures
come in direct contact. For heat exchange between
Semi- It is a medium in which radiation
two fluids, one fluid is sprayed through the other.
transparent absorption is the volumetric process
Cooling towers, jet condensers, and scrubbers are
Spectral It refers to a single wavelength
the best examples.
(monochromatic) radiation. The
quantity is denoted by subscript l (ii) Direct Transfer-type Heat Exchanger In this type
Specular It refers to the surface for which the of heat exchanger, the cold and hot fluids flow
angle of reflected radiation is equal simultaneously through the device and the heat is
to the angle of incident radiation transferred through the wall separating them. These
Stefan– The emissive power of the black types of heat exchangers are most commonly used
Boltzmann law body is directly proportional in almost all fields of engineering. These are further
to fourth power of the absolute classified as
temperature
Thermal It is the electromagnetic energy
(a) Regenerators These are also called storage
radiation emitted by a matter at a finite type heat exchangers, in which hot and cold fluid
temperature in the spectral region flow alternatively on the same surface. The hot
approximately from 0.1 to 100 mm fluid gives heat to the surface and cold fluid extracts
Transmission It is precess of the thermal radiation heat from it. In many applications a rotating disc-
passing through the matter type matrix is used, a continuous flow of both
the hot and cold fluids are maintained. These are
Transmissivity It is the fraction of radiation energy
transmitted by the matter
preheaters for steam power plants, blast furnaces,
oxygen producers, etc.
HEAT EXCHANGERS (b) Recuperators These are also called transfer-
type heat exchangers. In these heat exchangers, the
The device used for the heat exchange between hot and cold fluids are separated by a plane wall
the two fluids that are at different temperatures or tube surface, hence heat is indirectly transferred
is called a heat exchanger. For example, a car from hot fluid to cold fluid by convection and
radiator, a refrigerator, steam condenser, air cooler, conduction.
water cooler, etc. The heat exchangers are also
(B) According to Constructional Features
used in space heating and air-conditioning, waste
heat recovery and chemical precessing. Therefore, (i) Tubular Heat Exchanger These are also called
different types of heat exchangers are needed for tube-in-tube or concentric tube or double-pipe heat
different applications. exchanger. These are widely used in many sizes
and different flow arrangements and types.

(ii) Shell and Tube-type Heat Exchanger These


Heat exchangers are designed in many sizes, types, are also called surface condensers and are most
configurations and flow arrangements and used for commonly used for heating, cooling, condensation
many purposes. These are classified according to or evaporation applications. It consists of a shell
1054 Thermal Engineering

and number of tubes housed in it. These have large Cold out

surface area in small volume. A typical shell and


tube-type heat exchanger in shown in Fig. 30.30.
Hot Hot
in out
Tube Shell
outlet inlet Baffles Front-end
header
Cold in
(a) Parallel flow arrangement

Rear-end Cold out


header Shell
Tubes Shell Tube
outlet inlet

Hot Hot
in out

( ) When a high-operating
Cold in
pressure or an enchanced heat-transfer rate is
(b) Counter flow arrangement
required, the extended surfaces are used on one
side of the heat exchanger. These heat exchangers Cold in
are used for liquid-to-gas heat exchange. Fins are
always used on gas side. The tube fins are used
Hot Hot
in gas turbines, automobiles, aeroplanes, heat in out
pumps, refrigeration, electronics, cryogenics, air-
conditioning, systems, etc.
Cold out
Compact Heat Exchanger These are a special (c) Cross flow arrangement
class of heat exchanger in which the heat transfer with both fluid unmixed
area per unit volume is greater than 700 m2/m3.
These heat exchangers have dense arrays of finned
tubes or plates, when at least one of the fluid used
is gas. For example, automobile radiators have an Hot
Cold
fluid
fluid
area density in order of 1100 m2/m3. Inside
tubes
(C) According to Flow Arrangement
(d) Cross flow arrangement one fluid
() The hot and cold fluids enter at mixed and one fluid unmixed
same end of the heat exchanger, flow through
in same direction and leave at the same end.
Fig. 30.31(a).

( ) The hot and cold fluids enter at arrangement, the fluid flow may be mixed or
the opposite ends of heat exchanger, flow through unmixed. If both the fluids flow through individual
in opposite direction and leave at opposite ends. channels and are not free to move in transverse
Fig. 30.31(b). direction, the arrangement is called unmixed,
Fig. 30.31(c). If any fluid flows on the surface and
( ) The two fluids flow at right angles is free to move in the transverse direction, then this
to each other. In the cross flow heat exchanger fluid stream is said to be mixed.
Elements of Heat Transfer 1055

HEAT EXCHANGER ANALYSIS For Fig. 30.33,


DT1 = Thi – Tco DT2 = Tho – Tci
In the
thermal analysis of a heat exchanger, the total
heat transfer rate between the hot and cold fluids
can be calculated by using inlet and outlet fluids
temperature, overall heat transfer coefficients and
surface area as
Q = UA (DT )lm ...(30.53)
where, U = Overall heat transfer coefficient
A = Surface area for heat exchange
(DT )lm = Appropriate mean value of tempera-
ture difference
DT1 – DT2 DT2 – DT1 Fig. 30.33
= = ...(30.54) heat exchanger
È DT ˘ È DT ˘
ln Í 1 ˙ ln Í 2 ˙
DT
Î 2˚ Î DT1 ˚
The (DT )lm is called log mean temperature It is defined as the ratio of actual heat transfer rate
difference. Qact by a heat exchanger to maximum possible heat
For Fig. 30.32, transfer rate Qmax . It is denoted by e and expressed
DT1 = Thi – Tci (Temperature difference at as
the inlet (x = 0)) mc C pc (Tco Tci )
Q
DT2 = Tho – Tco (Temperature difference at e = act =
Qmax Qmax
the outlet (x = L))
For Fig. 30.33 mh ◊ C ph (Thi Tho)
= ...(30.56)
DT1 = Thi – Tco Temperature difference at x = 0. Qmax
DT2 = Tho – Tci Temperature difference at x = L. In any heat exchanger, the objective is either
While; Q = UA(DT )lm maximization of heating or cooling rate, i.e., to
= Uo Ao(DT )lm gain the maximum temperature difference and
= UiAi(DT )lm ...(30.55) hence, the maximum heat transfer rate Qmax can
be achieved in counter flow heat exchanger of
infinite length. In such a heat exchanger, one fluid
having minimum value of heat capacity ( mCp)
can experience maximum possible temperature
difference (Thi – Tci), because the energy balance
requires that the heat given by one fluid should be
equal to heat gain by other.
Therefore,
Qmax = ( mCp) min (DT )max
= ( mCp) min (Thi – Tci) ...(30.57)
Fig. 30.32
heat exchanger
1056 Thermal Engineering

NTU Analysis The outlet temperature of water can be


obtained by energy balance.
It is number of transfer units and is dimensionless mh Cph (Thi – Tho) = mc Cpc (Tco – Tci)
parameter, which is expressed as 2 ¥ 2.035 (160 – 60) = 2 ¥ 4.187 (Tco – 25)
UA UA 2.035 ¥ 100
NTU = = or Tco = + 25 = 73.6°C
Cmin ( mCp) min 4.187
Heat capacity of exchanger W/K Since Tco > Tho, hence the heat exchanger is in
=
Heat capacity of flow W/K counterflow arrangement and its LMTD,
...(30.58) DT1 – DT2
(DT )lm =
where U and Cmin are constant quantities for given È DT ˘
ln Í 1 ˙
flow conditions, hence Î DT2 ˚
NTU μ A
where, DT1 = Tho – Tci = 60 – 25 = 35°C
Therefore, NTU is the measure of the physical
DT2 = Thi – Tco = 160 – 73.6 = 86.4°C
size (heat transfer area) of the heat exchanger.
Higher the value of NTU, larger the physical size. 35 – 86.4
and (DT )lm = = 56.88°C
For any heat exchanger, the effectiveness is the È 35 ˘
ln Í ˙
function of NTU and heat capacity ratio. Î 86.4 ˚
Ê Cmin ˆ The heat transfer rate,
e = f Á NTU , ...(30.59)
Ë Cmax ˜¯ Q = mhCph (Thi – Tho)
= UA(DT)lm
where Cmax = ( mCp )max among the two fluids mhC ph (Thi - Tho )
and L =
Example 30.18 A thin-walled concentric tube heat U (p D ) ( DT )lm
exchanger is used to cool engine oil from 160°C to 60°C (2 kg/s) (2.035 ¥ 103 J/kg K) (160 – 60) ( C)
=
and water, which is available at 25°C acts as a coolant. (250 W/m 2 K) (p ¥ 0.5 m) (56.88°C)
The oil and water flow rates are each 2 kg/s each and the =18.22 m
diameter of the inner tube is 0.5 m and the corresponding
value of overall heat transfer coefficient is 250 W/m2 K.
Example 30.19 Water at 225 kg/h is to be heated
How long must the heat exchanger be to accomplish the
from 35°C to 95°C by means of concentric tube heat
desired cooling?
exchanger. Oil at 225 kg/h and 210°C with a specific heat
Take, Cp of water = 4.187 kJ/kg K
of 2095 J/kg K is to be used as hot fluid. If the overall
Cp of engine oil = 2.035 kJ/kg K.
heat-transfer coefficient based on the outer diameter of
inner tube is 550 W/m2.K, determine the length of the
Solution
heat exchanger, if the outer diameter is 100 mm.
Given A concentric tube heat exchanger with
Thi = 160°C Tho = 60°C Solution
Tci = 25°C Cpc = 4.187 kJ/kg K Given A concentric tube heat exchanger
Cph = 2.035 kJ/kg K U = 250 W/m2 K Thi = 210°C mc = 225 kg/h
mh = mc = 2 kg/s D = 0.5 m Tci = 35°C Tco = 95°C
To find Length of the heat exchanger. mh = 225 kg/h Cpc = 4.18 kJ/kg K
Cph = 2095 J/kg K U = 550 W/m2 K
Assumption
D = 100 mm = 0.1 m
(i) No heat loss to the surroundings.
(ii) No scaling on heat transfer surface. To find Length of the heat exchanger.
Elements of Heat Transfer 1057

Assumptions DT1 – DT2


(DT )lm =
(i) No heat loss to the surroundings. Ê DT ˆ
ln Á 1 ˜
(ii) No scaling on heat transfer surfaces. Ë DT2 ¯
(iii) The specific heat of the water as 4180 J/kg K. where, DT1 = DTho – DTci = 90.28 – 35 = 55.28°C
Analysis The heat transfer rate through the heat DT2 = DThi – DTco = 210 – 95 = 115°C
exchanger can be calculated as 55.28 – 115
and (DT )lm = = 81.52°C
Q = mcCpc (Tco – Tci) È 55.28 ˘
ln Í ˙
= (225/3600) (kg/s) ¥ (4180 J/kg K) Î 115 ˚
¥ (95 – 35) (K) Then Q = UA (DT )lm = U (p dL) (DT )lm
= 15675 W
The outlet temperature of oil, Q
or L =
Q = mhCph (Thi – Tho) U (p D ) ( DT )lm
Q 15675 15675W
or Tho = Thi – = 210 – =
mc C pc ( 225/3600) ( 2095) 2
(550 W/m K) p (0.1m) (8.1.52 C)
= 90.28°C
= 1.11 m
The outlet temperature of hot fluid is less than cold
fluid outlet temperature, hence used heat exchanger is in
counterflow arrangement.

Summary
transfer demonstrates
which deals with analysis of rate and nature of ( DT )overall
Q =
heat transfer as well as temperature distribution S Rth
in the system.
1
medium then heat is transferred by conduction. U =
A S Rth
Heat convection takes place between heated
surface and adjacent moving fluid. The
thermal radiation takes place in the form of all incident radiation and emits maximum energy
electromagnetic waves between radiating bodies. at a given temperature. The Stefan–Boltzmann
Fourier law of heat conduction is given by law is the fundamental law of radiation
Q dT Q = s A (T 14 – T 24)
= –k
A dx where s = Stefan–Boltzmann constant, W/m2.K4
where k is thermal conductivity, a property of heat exchanger is a device which exchanges
material, measured in W/m ◊ K. heat energy between two fluids at two different
Newton’s law of cooling is the governing equation temperatures. The log mean temperature differ-
for heat convection and it is given by ence is given by
Q = hAs (Ts – T ) DT1 – DT2
(DT )lm =
where h is the heat transfer coefficient a property Ê DT ˆ
of ambient conditions. ln Á 1 ˜
Ë DT2 ¯
1058 Thermal Engineering

where DT1 = Temperature difference at left end NTU) is given by


of heat exchanger, and Heat capacity rate of heat exchanger
DT2 = Temperature difference at right end NTU =
Heat capacity rate of fluid
of heat exchanger. UA
heat exchanger is given as =
( mC p ) min
Actual heat transfer rate
e=
Maximum possible heat transfer rate

Glossary
Heat flux Heat transfer rate per unit area Thermal resistance Opposes the heat flow through the
Conduction Heat transfer due to existance of material medium
temperature gradient in material medium Heat transfer coefficient Property of ambient condi-
Free convection Heat transfer due to density difference tions
induced by temperature difference in fluids Black body An imaginary ideal body for radiation
Forced convection Heat transfer due to velocity Emisive power Radiation heat transfer per unit area
difference induced artificially
Emissivity Property of a radiating surface
Radiation Heat transfer due to electromagnetic waves
LMTD Log mean temperature difference
from surfaces
Thermal conductivity Ability of material to conduct NTU Number of transfer unit
the heat Effectiveness Ratio of actual heat transfer rate to
Thermal potential Temperature difference; respon- maximum possible heat transfer rate
suible for heat transfer

Problems
1. Determine the heat flow across a plane wall of 4. Consider a furnace wall [k = 1 W/(m°C)] with the
10 cm thickness with a thermal conductivity of inside surface at 1000°C and the outside surface
8.5 W/m.K, when the surface temperatures are at 400°C. If the heat flow through the wall should
steady and at 200°C and 50°C. The wall area is not exceed 2000 W/m2, what is the minimum wall
2 m2. Also find the temperature gradient in flow thickness L? [30 cm]
direction. [25500 W, 1500°C/m] 5. A metal pipe of 10-cm OD is covered with a 2-cm
2. Determine the heat transfer rate by convection thick insulation [k = 0.07 W/(m°C)]. The heat
loss from the pipe is 100 W per meter of length
over a surface of 1 m2 if the surface at 100°C
when the pipe surface is at 100°C. What is the
is exposed to a fluid at 40°C with convection
temperature of the outer surface of the insulation?
coefficient of 25 W/m2.K. [1500 W]
[23.5°C]
3. A surface at 200°C is exposed to surroundings
6. A 6-cm-OD, 2-cm-thick copper hollow sphere
at 60°C and convects and radiates heat to the
[k = 386 W/(m.°C)] is uniformly heated at the
surroundings. Calculate the heat transfer rate
inner surface at a rate of 150 W/m2. The outer
from surface to surroundings, if the convection surface is cooled with air at 20°C with a heat-
coefficient is 80 W/m2.K. Consider the black transfer coefficient of 10 W/(m2.°C). Calculate
bodies for radiation heat transfer. Take s = 5.67 ¥ the temperature of the outer surface.
10–8 W/m2 K4. [14.24 kW/m2] [21.7°C]
Elements of Heat Transfer 1059

7. The wall of a building consists of 10 cm of a viscosity of 0.86 ¥ 10–5 kg/m.s and a density of
brick [k = 0.69 W/(m.°C)], 1.25 cm of Celotex 1.12 kg/m3.
[k = 0.048 W/(m.°C)], 8 cm of glass wool [k = [8.1 mm, 0.0158 N]
0.038 W/(m.°C)], and 1.25 cm of asbestos cement 13. Air flows along a thin flat plate 1 m wide and
board [k = 0.74 W/(m.°C)]. If the outside surface 1.5 m long, at a velocity of 1 m/s. The free stream
of the brick is at 5°C and the inside surface of the temperature is 4°C. Calculate the amount of heat
cement board is at 20°C, calculate the heat flow that must be supplied to plate in order to maintain
rate per square meter of wall surface. it at a uniform temperature of 50°C.
[– 5.94 W/m2] [441.5 W]
8. Consider a brass tube [k = 115 W/(m.°C)], with 14. Water at mean temperature of 60°C flows inside
an outside radius of 4 cm and a thickness of 0.5. a 2.5 cm ID, 10 m long tube with a velocity of
The inside surface of the tube is kept at uniform 6 m/s. The tube wall is maintained at a uniform
temperature, and the outside surface is covered temperature of 100°C by condensing steam.
with two layers of insulation each 1 cm thick, Determine the heat transfer rate to water. Assume
with thermal conductivities of 0.1 W/(m.°C) and an inlet temperature of 30°C. [670 W]
0.05 W/(m.°C) respectively. Calculate the overall 15. Estimate the coefficient of free convection for
heat-transfer coefficient based on the outside a wire, 2 mm in diameter immersed in water at
surface area of the outer insulation. 20°C, if the wire surface is maintained at 300°C.
[2.83 W/m2.°C] [3366 W/m2.K]
9. A double glazed window is made of 2 glass panes 16. A flat electrical heater of 0.5 m ¥ 0.5 m is placed
of 6 mm thickness each with an air gap of 6 mm vertically in still air at 20°C. The heat generated
between them. Assuming that the layer is stagnant is 1200 W/m2. Determine the value of natural
and only conduction is involved, determine the convection coefficient and average temperature
thermal resistance and the overall heat-transfer of the plate. [33.02 W/m2.K, 56.5°C]
coefficient. The inside is exposed to convection 17. A vertical pipe of 10 cm diameter and 3 m length
with h = 1.5 W/m2.K. Compare the values with at a surface temperature of 100°C, is in a room
that of a single glass of 12 mm thickness. The where the air is at 20°C. What is the rate of heat
conductivity of the glass = 1.4 W/m.K and that loss per unit length of the pipe?
for air is 0.025 W/m.K.
[119.7 W/m]
10. Atmospheric air at 27°C flows along a flat plate
18. The heat-transfer rate per unit length due to free
with a velocity of 8 m/s. The critical Reynold
convection from a horizontal tube is 200 W/m,
number at which transition from laminar to
when its surface is maintained at 70°C in the
turbulent takes place is 5 ¥ 105. Determine the
ambient air at 20°C. Estimate the heat-transfer
distance from the leading edge of the plate at
rate per unit length, when the tube surface is
which the transition occurs.
maintained at 145°C. Neglect the heat-transfer
11. Air at 24°C flows along a 4-m long flat plate rate by radiation and any influence of temperature
with a velocity of 5 m/s. The plate is maintained on thermophysical properties of air.
at 130°C. Calculate the heat transfer coefficient
[625 W/m]
over the entire length of the plate and the heat-
19. Calculate the heat flux emitted due to thermal
transfer rate per metre width of the plate.
radiation from a black surface at 6000°C. At what
[9.73 W/m2.K, 4120 W/m]
wavelength is the monochromatic emissive power
12. Air flows along a thin plate with a velocity of maximum and what is the maximum value?
2.5 m/s. The plate is 1 m long and 1 m wide.
[87,798 kW/m2; 0.462 mm; 1.25 ¥ 1014 W/m2]
Estimate the boundary layer thickness at the
20. Estimate the rate at which the sun emits the
trailing edge of the plate and the force necessary
radiant energy. What fraction of this energy is
to hold the plate in the stream of air. The air has
1060 Thermal Engineering

absorbed by the earth and in what amount? If fouling factor is 9.98 ¥ 105 m2.K/W. Calculate the
effective temperature of the sun is 5700 K and overall coefficient of heat transfer.
surface of the sun is treated black. The diameter [U = 851.5 W/m2.K]
of the sun is 1.39 ¥ 106 km. The diameter of the 22. Water at the rate of 4080 kg/h is heated from 35°C
earth is 1.27 ¥ 104 km and the distance between to 75°C by an oil having a specific heat of 1900
sun and earth is 1.5 ¥ 108 km. J/kg/K. The exchanger is of a counterflow double-
[3.81 ¥ 1026 W; 4.48 ¥ 10–10; 1.71 ¥ 1017 W] pipe design. The oil enters at 110°C and leaves at
21. A double pipe heat exchanger is constructed of 75°C. Determine the area of the heat exchanger
0.287 cm thick steel tubing with 2.09 cm inner necessary to handle this load if the overall heat
tube and 2.66 cm outer tube. The inside and the transfer coefficient is 320 W/m2 K.
outside coefficients of heat transfer are 1135 W/ [A = 15.82 m2]
m2.K and 5677 W/m2.K, respectively, and the

Objective Questions
1. Heat transfer takes place from a high-temperature (d) rate of heat flow through 1 m2 area for
body to a low-temperature body according to temperature gradient of 1°C/m
(a) zeroth law of thermodynamics 5. Heat is conducted through a pure metal due to
(b) first law of thermodynamics (a) vibration of lattice structure
(c) second law of thermodynamics (b) flow of free electrons
(d) third law of thermodynamics (c) density difference
2. Conduction heat transfer takes place in the (d) none of the above
medium due to 6. Newton’s law of cooling is expressed as
(a) temperature difference Ê dT ˆ Ê dT ˆ
(a) Q = – k A Á (b) Q = – h A Á
(b) temperature gradient Ë dx ˜¯ Ë dx ˜¯
(c) thermal conductivity
(c) Q = h A2 (Ts – T ) (d) Q = h A(Ts – T )
(d) surface area
7. The value of heat transfer coefficient depends on
3. Which one of the following represents Fourier
(a) velocity of fluid and temperature difference
equation?
(b) thermal conductivity and dynamic viscosity
k Ê dT ˆ k Ê dT ˆ (c) geometry of surface and its ambient
(a) Q = (b) Q = –
A ÁË dx ˜¯ A ÁË dx ˜¯ (d) all of the above
Ê dT ˆ Ê dT ˆ 8. The overall heat-transfer coefficient with respect
(c) Q = – k A Á (d) Q = k A Á
Ë dx ˜¯ Ë dx ˜¯ to surface 1 is expressed as
1 Q
4. Thermal conductivity of a medium may be (a) U1 = (b) U1 =
A1 S Rth A1( DT )overall
defined as
(a) amount of heat flow through 1 m2 area for U 2 A2
(c) U1 = (d) all of the above
temperature gradient of 1 K A1
(b) rate of heat transfer through 1 m2 area for 9. The Reynolds number for a fluid flow is defined
temperature gradient of 1°C as
(c) amount of heat flow through 1 m2 area for Bouyancy force
temperature gradient of 1°C/m (a)
Viscous force
Elements of Heat Transfer 1061

Inertia force (a) Parallel flow heat exchanger


(b) (b) Counter flow heat exchanger
Viscous force
(c) Cross flow heat exchanger
Convection heat transfer
(c) (d) Shell and tube type heat exchanger
Conduction heat transfer
12. Which one of the following is an expression for
Momentum diffusivity LMTD for a counterflow heat exchanger?
(d)
Thermal diffusivity
DT1 – DT2 DT1 – DT2
10. A gray body has one of the following properties: (a) (b)
Ê DT ˆ Ê DT ˆ
(a) It reflects all of the energy falling on it. ln Á 2 ˜ ln Á 1 ˜
Ë DT1 ¯ Ë DT2 ¯
(b) It transmits all of the energy falling on it.
(c) It has constant emissivity. Ê DT ˆ
ln Á 1 ˜
(d) It absorbs all of the energy falling on it. Ë DT2 ¯ DT1 – DT2
11. Which one of the following heat exchanger (c) (d)
DT1 – DT2 Ê DT ˆ
is most efficient for a given surface area and log10 Á 2 ˜
Ë DT1 ¯
temperature difference?

12. (b) 11. (b) 10. (c) 9. (b)


8. (d) 7. (d) 6. (d) 5. (b) 4. (d) 3. (c) 2. (b) 1. (c)
Answers
1062 Thermal Engineering

A
Appendix
Table A.1

Chemical M R r Tc pc pc vc
Formula (kg./kmol) kJ/kg-K kg/m3 (K) (bar) Zc =
Substance RTc
Acetylene C2H2 26.04 0.3193 –1.05 309 62.8 0.274
Air (equivalent) – 28.97 0.287 1.169 133 37.7 0.284
Ammonia NH3 17.03 0.4882 0.694 406 112.8 0.242
Argon Ar 39.94 0.2081 –1.613 151 48.6 0.290
Benzene C6H6 78.11 0.1064 — 563 49.3 0.274
Butane C4H10 58.12 0.1430 2.407 425 38.0 0.274
Carbon C 12.01 — — — — —
Carbon dioxide CO2 44.01 0.1889 1.775 304 73.9 0.276
Carbon monoxide CO 28.01 0.2968 1.13 133 35.0 0.294
Copper Cu 63.54 — — — — —
Ethane C2H6 30.07 0.2765 1.222 305 48.8 0.285
Ethyl alcohol C2H5OH 46.07 0.1805 1.883 516 63.8 0.249
Ethylene C2H4 28.05 0.2964 1.138 283 51.2 0.270
Helium He 4.003 2.0771 0.1615 5.2 2.3 0.300
Hydrogen H2 2.016 4.1243 0.0813 33.2 13.0 0.304
Methane CH4 16.04 0.5183 0.648 191 46.4 0.290
Methyl alcohol CH3OH 32.04 0.2595 1.31 513 79.5 0.220
Nitrogen N2 28.01 0.2968 1.13 126 33.9 0.291
Octane C8H18 114.22 0.07279 0.092 569 24.9 0.258
Oxygen O2 32.00 0.2598 1.292 154 50.5 0.290
Propane C3H8 44.09 0.1886 1.808 370 42.7 0.276
Refrigerant 12 CCl2F2 120.92 0.06876 4.98 385 41.2 0.278
Refrigerant 22 CHClF2 86.48 0.09616 3.54 369 49.8 0.267
Refrigerant 134a CF3CH2F 102.03 0.08149 4.20 374 40.7 0.260
Sulfur dioxide SO2 64.06 0.1298 2.618 431 78.7 0.268
Water H2O 18.02 0.4613 1000 647.3 220.9 0.233
Appendix A 1063

Table A.2

Substance r Cp Substance r Cp
kg/m3 kJ/kg-K kg/m 3
kJ/kg-K
Asphalt 2120 0.92 Ammonia 604 4.84
Brick, common 1800 0.84 Benzene 879 1.72
Carbon, dimond 3250 0.51 Butane 556 2.47
Carbon, graphite 2000-2500 0.61 CCL4 1584 0.83
Coal 1200-1500 1.26 CC2 680 2.9
Concrete 2200 0.88 Ethanol 783 2.46
Glass, plate 2500 0.80 Gasoline 750 2.08
Glass, wool 200 0.66 Glycerine 1260 2.42
Granite 2750 0.89 Kerosene 815 2.0
Ice (0 C) 917 2.04 Methanol 787 2.55
Paper 700 1.2 n-Octane 692 2.23
Plexiglas 1180 1.44 Oil engine 885 1.9
Polystyrene 920 2.3 Oil light 910 1.8
Polyvinyl chloride 1380 0.96 Propane 510 2.54
Rubber, soft 1100 1.67 R-12 1310 0.97
Salt, rock 2100-2500 0.92 R-22 1190 1.26
Sand, dry 1500 0.8 R-134a 1206 1.43
Silicon 2330 0.70 Water 997 4.18
Snow, firm 560 2.1 Liquid Metals
Wood, hard (oak) 720 1.26 Bismuth, Bi 10040 0.14
Wood, soft (pine) 510 1.38 Lead, Pb 10660 0.16
Wool 100 1.72 mercury, Hg 13580 0.14
Metals Potassium, K 828 0.81
Aluminum 2700 0.90 Sodium, Na 929 1.38
Copper, commercial 8300 0.42 Tin, sn 6950 0.24
Brass, 60-40 8400 0.38 Zinc, Zn 6570 0.50
Gold 19300 0.13 Nak (56/44) 887 1.13
Iron, cast 7272 0.42
Iron. 304 St Steel 7820 0.46
Lead 11340 0.13
Megnesium, 2% Mn 1778 1.00
nickel. 10% Cr 8666 0.44
Silver, 99.9% Ag 10524 0.24
Sodium 971 1.21
Tim 7304 0.22
Tungsten 19300 0.13
Zinc 7144 0.39
Table A.4

Substance Formula molar mass, Enthalpy of Gibbs Absolute Heating Values


M (kg/kmol) Function of Entropy, Higher, Lower,
Formation, h f
Formation, s (kJ/kmol-K) HCV (kJ/kg) LCV (kJ/kg)
(kJ/kmol)
g f (kJ/kmol)

Carbon C(s) 12.01 0 0 5.74 32,770 32,770


Hydrogen H2(g) 2.016 0 0 130.57 141,780 119,950
1064 Thermal Engineering

Nitrogen N2(g) 28.01 0 0 191.50 – –


Oxygen O2(g) 32.00 0 0 205.03 – –
Carbon monoxide CO(g) 28.01 –110,530 –137,150 197.54 – –
Carbon dioxide CO2(g) 44.01 –393,520 –394,380 213.69 – –
Water H2O(g) 18.02 –241,820 228,590 188.72 – –
Water H2O(1) 18.02 –285,830 –237,180 69.95 – –
Hydrogen peroxide H2O2(g) 34.02 –136,310 –105,600 232.63 – –
Ammonia NH3(g) 17.03 –46,190 –16,590 192.33 – –
Oxygen O(g) 16.00 249,170 231,770 160.95 – –
Hydrogen H(g) 1.008 218,000 203,290 114.61 – –
Nitrogen N(g) 14.01 472,680 455,510 153.19 – –
Hydroxyl OH(g) 17.01 39,460 34,280 183.75 – –
Methane CH4(g) 16.04 –74,850 –50,790 186.16 – –
Acetylene C2H2(g) 26.04 226,730 209,170 200.85 55,510 50,020
Ethylene C2H2(g) 28.05 52,280 68,120 219.83 49,910 48,220
Ethane C2H6(g) 30.07 –84,680 –32,890 229.49 50,300 47,160
Propylene C3H6(g) 42.08 20,410 62,720 266.94 51,870 47,480
Propane C3H8(g) 44.09 –103,850 –23,490 269.91 48,920 45,780
Butane C4H10(g) 58.12 –126,150 –15,710 310.03 50,350 46,360
Pentane C5H12(g) 72.15 –146,440 –8,200 348.40 49,500 45,720
Octane C8H18(g) 114.22 –208,450 17,320 463.67 49,010 45,350
Octane C8H18(1) 114.22 249,910 6,610 360.79 48,260 44,790
Benzene C6H6(g) 78.11 82,930 129,660 269.20 47,900 44,430
Methyl alcohol CH3OH(g) 32.04 –200,890 –162,140 239.70 42,270 40,580
Methyl alcohol CH3OH(1) 32.04 – 238,810 –166,290 126.80 23,850 21,110
Ethyl alcohol C2H5OH(g) 46.07 – 235,310 – 168,570 282.59 22,670 19,920
Ethyl alcohol C2H5OH(1) 46.07 –277,690 174,890 160.70 29,670 26,800
Appendix A 1065

Table A.5

1. van der Waals and Redlich–Kwong: Constants for pressure in bar, specific volume in m3/kmol, and temperature

van der Waals Redlich–Kwong


Substance a b a b

Ê m3 ˆ
2
m3 Ê m3 ˆ
2
m3
bar Á ˜ kmol bar Á ˜ K½ kmol
Ë kmol ¯ Ë kmol ¯

Air 1.368 0.0367 15.989 0.02541


Butane (C4H10) 13.86 0.1162 289.55 0.08060
Carbon dioxide (CO2) 3.647 0.0428 64.43 0.02963
Carbon monoxide (CO) 1.474 0.0395 17.22 0.02737
Methane (CH4) 2.293 0.0428 32.11 0.02965
Nitrogen (N2) 1.366 0.0386 15.53 0.02677
Oxygen (O2) 1.369 0.0317 17.22 0.02197
Propane (C3H8) 9.349 0.0901 182.23 0.06242
Refrigerant 12 10.49 0.0971 208.59 0.06731
Sulfur dioxide (SO2) 6.883 0.0569 144.80 0.03945
Water (H2O) 5.531 0.0305 142.59 0.02111
Source: Calculated from critical data.

2. Benedict–Webb–Rubin: Constants for pressure in bar, specific volume in m3/kmol, and temperature in K

Substance a A b B c C a g (gamma)
5 6 –3
C4H10 1.9073 10.218 0.039998 0.12436 3.206 ¥ 10 1.006 ¥ 10 1.101 ¥ 10 0.0340
CO2 0.1386 2.7737 0.007210 0.04991 1.512 ¥ 104 1.404 ¥ 105 8.47 ¥ 10-5 0.00539
CO 0.0371 1.3590 0.002632 0.05454 1.054 ¥ 103 8.676 ¥ 103 1.350 ¥ 10-4 0.0060
CH4 0.0501 1.8796 0.003380 0.04260 2.579 ¥ 103 2.287 ¥ 104 1.244 ¥ 10-4 0.0060
N2 0.0254 1.0676 0.002328 0.04074 7.381 ¥ 102 8.166 ¥ 103 1.272 ¥ 10-4 0.0053
1066 Thermal Engineering

C
Cp
= a + bT + gT 2 + dT 3 + eT 4
Ru
T is in K, equations valid from 300 to 1000 K

Gas a b ¥ 103 g ¥ 106 d ¥ 109 e ¥ 1012


CO 3.710 –1.619 3.692 –2.032 0.240
CO2 2.401 8.735 –6.607 2.002 0
H2 3.057 2.677 –5.810 5.521 –1.812
H 2O 4.070 –1.108 4.152 –2.964 0.807
O2 3.626 –1.878 7.055 –6.764 2.156
N2 3.675 –1.208 2.324 –0.632 0.226
Air 3.653 –1.337 3.294 –1.913 0.2763
SO2 3.267 5.324 0.684 5.281 2.559
CH4 3.826 –3.979 24.558 –22.733 6.963
C 2H 2 1.410 19.057 –24.501 16.391 – 4.135
C 2H 4 1.426 11.383 7.989 –16.254 6.749
Monatomic
gasesa 2.5 0 0 0 0
Appendix A 1067

Ideal Gas ◊

Temp. Cp Cv k Cp Cv k Cp Cv k Temp.
K Air Nitrogen, N2 Oxygen, O2 K
250 1.003 0.716 1.401 1.039 0.742 1.400 0.913 0.653 1.398 250
300 1.005 0.718 1.400 1.039 0.743 1.400 0.918 0.658 1.395 300
350 1.008 0.721 1.398 1.041 0.744 1.399 0.928 0.668 1.389 350
400 1.013 0.726 1.395 1.044 0.747 1.397 0.941 0.681 1.382 400
450 1.020 0.733 1.391 1.049 0.752 1.395 0.956 0.696 1.373 450
500 1.029 0.742 1.387 1.056 0.759 1.391 0.972 0.712 1.365 500
550 1.040 0.753 1.381 1.065 0.768 1.387 0.988 0.728 1.358 550
600 1.051 0.764 1.376 1.075 0.778 1.382 1.003 0.743 1.350 600
650 1.063 0.776 1.370 1.086 0.789 1.376 1.017 0.758 1.343 650
700 1.075 0.788 1.364 1.098 0.801 1.371 1.031 0.771 1.337 700
750 1.087 0.800 1.359 1.110 0.813 1.365 1.043 0.783 1.332 750
800 1.099 0.812 1.354 1.121 0.825 1.360 1.054 0.794 1.327 800
900 1.121 0.834 1.344 1.145 0.849 1.349 1.074 0.814 1.319 900
1000 1.142 0.855 1.336 1.167 0.870 1.341 1.090 0.830 1.313 1000
Temp. Carbon Carbon Hydrogen, H2 Temp.
K Dioxide, CO2 Monoxide, CO K
250 0.791 0.602 1.314 1.039 0.743 1.400 14.051 9.927 1.416 250
300 0.846 0.657 1.288 1.040 0.744 1.399 14.307 10.183 1.405 300
350 0.895 0.706 1.268 1.043 0.746 1.398 14.427 10.302 1.400 350
400 0.939 0.750 1.252 1.047 0.751 1.395 14.476 10.352 1.398 400
450 0.978 0.790 1.239 1.054 0.757 1.392 14.501 10.377 1.398 450
500 1.014 0.825 1.229 1.063 0.767 1.387 14.513 10.389 1.397 500
550 1.046 0.857 1.220 1.075 0.778 1.382 14.530 10.405 1.396 550
600 1.075 0.886 1.213 1.087 0.790 1.376 14.546 10.422 1.396 600
650 1.102 0.913 1.207 1.100 0.803 1.370 14.571 10.447 1.395 650
700 1.126 0.937 1.202 1.113 0.816 1.364 14.604 10.480 1.394 700
750 1.148 0.959 1.197 1.126 0.829 1.358 14.645 10.521 1.392 750
800 1.169 0.980 1.193 1.139 0.842 1.353 14.695 10.570 1.390 800
900 1.204 1.015 1.186 1.163 0.866 1.343 14.822 10.698 1.385 900
1000 1.234 1.045 1.181 1.185 0.888 1.335 14.983 10.859 1.380 1000
Air

T (K), h and u(kJ/kg), s∞ (kJ/kg.K)


when Ds = 01 when Ds = 0
T h u s∞ pr vr T h u s∞ pr vr
200 199.97 142.56 1.29559 0.3363 1707.0 450 451.80 322.62 2.11161 5.775 223.6
210 209.97 149.69 1.34444 0.3987 1512.0 460 462.02 329.97 2.13407 6.245 221.4
220 219.97 156.82 1.39105 0.4690 1346.0 470 472.24 337.32 2.15604 6.742 200.1
230 230.02 164.00 1.43557 0.5477 1205.0 480 482.49 344.70 2.17760 7.268 189.5
1068 Thermal Engineering

240 240.02 171.13 1.47824 0.6355 1084.0 490 492.74 352.08 2.19876 7.824 179.7
250 250.05 178.28 1.51917 0.7329 979.0 500 503.02 359.49 2.21952 8.411 170.6
260 260.09 185.45 1.55848 0.8405 887.8 510 513.32 366.92 2.23993 9.031 162.1
270 270.11 192.60 1.59634 0.9590 808.0 520 523.63 374.36 2.25997 9.684 154.1
280 280.13 199.75 1.63279 1.0889 738.0 530 533.98 381.84 2.27967 10.37 146.7
285 285.14 203.33 1.65055 1.1584 706.1 540 544.35 389.34 2.29906 11.10 139.7
290 290.16 206.91 1.66802 1.2311 676.1 550 554.74 396.86 2.31809 11.86 133.1
295 295.17 210.49 1.68515 1.3068 647.9 560 565.17 404.42 2.33685 12.66 127.0
300 300.19 214.07 1.70203 1.3860 621.2 570 575.59 411.97 2.35531 13.50 121.2
305 305.22 217.67 1.71865 1.4686 596.0 580 586.04 419.55 2.37348 14.38 115.7
310 310.24 221.25 1.73498 1.5546 572.3 590 596.52 427.15 2.39140 15.31 110.6
315 315.27 224.85 1.75106 1.6442 549.8 600 607.02 434.78 2.40902 16.28 105.8
320 320.29 228.42 1.76690 1.7375 528.6 610 617.53 442.42 2.42644 17.30 101.2
325 325.31 232.02 1.78249 1.8345 508.4 620 628.07 450.09 2.44356 18.36 96.92
330 330.34 235.61 1.79783 1.9352 489.4 630 638.63 457.78 2.46048 19.84 92.84
340 340.42 242.82 1.82790 2.149 454.1 640 649.22 465.50 2.47716 20.64 88.99
350 350.49 250.02 1.85708 2.379 422.2 650 659.84 473.25 2.49364 21.86 85.34
360 360.58 257.24 1.88543 2.626 393.4 660 670.47 481.01 2.50985 23.13 81.89
370 370.67 264.46 1.91313 2.892 367.2 670 681.14 488.81 2.52589 24.46 78.61
380 380.77 271.69 1.94001 3.176 343.4 680 691.82 496.62 2.54175 25.85 75.50
390 390.88 278.93 1.96633 3.481 321.5 690 702.52 504.45 2.55731 27.29 72.56
400 400.98 286.16 1.99194 3.806 301.6 700 713.27 512.33 2.57277 28.80 69.76
410 411.12 293.43 2.01699 4.153 283.3 710 724.04 520.23 2.58810 30.38 67.07
420 421.26 300.69 2.04142 4.522 266.6 720 734.82 528.14 2.60319 32.02 64.53
430 431.43 307.99 2.06533 4.915 251.1 730 745.62 536.07 2.61803 33.72 62.13
440 441.61 315.30 2.08870 5.332 236.8 740 756.44 544.02 2.63280 35.50 59.82
Contd.
Table A.8

T (K), h and u(kJ/kg), s∞ (kJ/kg ◊ K)


when Ds = 0 when Ds = 0
T h u s° pr vr T h u s° pr vr
750 767.29 551.99 2.64737 37.35 57.63 1300 1395.97 1022.82 3.27345 330.9 11.275
760 778.18 560.01 2.66176 39.27 55.54 1320 1419.76 1040.88 3.29160 352.5 10.747
770 789.11 568.07 2.67595 41.31 53.39 1340 1443.60 1058.94 3.30959 375.3 10.247
780 800.03 576.12 2.69013 43.35 51.64 1360 1467.49 1077.10 3.32724 399.1 9.780
790 810.99 584.21 2.70400 45.55 49.86 1380 1491.44 1095.26 3.34474 424.2 9.337
800 821.95 592.30 2.71787 47.75 48.08 1400 1515.42 1113.52 3.36200 450.5 8.919
820 843.98 608.59 2.74504 52.59 44.84 1420 1539.44 1131.77 3.37901 478.0 8.526
840 866.08 624.95 2.77170 57.60 41.85 1440 1563.51 1150.13 3.39586 506.9 8.153
860 888.27 641.40 2.79783 63.09 39.12 1460 1587.63 1168.49 3.41247 537.1 7.801
880 910.56 657.95 2.82344 68.98 36.61 1480 1611.79 1186.95 3.42892 568.8 7.468
900 932.93 674.58 2.84856 75.29 34.31 1500 1635.97 1205.41 3.44516 601.9 7.152
920 955.38 691.28 2.87324 82.05 32.18 1520 1660.23 1223.87 3.46120 636.5 6.854
940 977.92 708.08 2.89748 89.28 30.22 1540 1684.51 1242.43 3.477.12 672.8 6.569
960 1000.55 725.02 2.92128 97.00 28.40 1560 1708.82 1260.99 3.49276 710.5 6.301
980 1023.25 741.98 2.94468 105.2 26.73 1580 1733.17 1279.65 3.50829 750.0 6.046
1000 1046.04 758.94 2.96770 114.0 25.17 1600 1757.57 1298.30 3.52364 791.2 5.804
1020 1068.89 776.10 2.99034 123.4 23.72 1620 1782.00 1316.96 3.53879 834.1 5.574
1040 1091.85 793.36 3.01260 133.3 22.39 1640 1806.46 1335.72 3.55381 878.9 5.355
1060 1114.86 810.62 3.03449 143.9 21.14 1660 1830.96 1354.48 3.56867 925.6 5.147
1080 1137.89 827.88 3.05608 155.2 19.98 1680 1855.50 1373.24 3.58335 974.2 4.949
1100 1161.07 845.33 3.07732 167.1 18.896 1700 1880.1 1392.7 3.5979 1025 4.761
1120 1184.28 862.79 3.09825 179.7 17.886 1750 1941.6 1439.8 3.6336 1161 4.328
1140 1207.57 880.35 3.11883 193.1 16.946 1800 2003.3 1487.2 3.6684 1310 3.944
1160 1230.92 897.91 3.13916 207.2 16.064 1850 2065.3 1534.9 3.7023 1475 3.601
1180 1254.34 915.57 3.15916 222.2 15.241 1900 2127.4 1582.6 3.7354 1655 3.295
1200 1277.79 933.33 3.17888 238.0 14.470 1950 2189.7 1630.6 3.7677 1852 3.022
1220 1301.31 951.09 3.19834 254.7 13.747 2000 2252.1 1678.7 3.7994 2068 2.776
1240 1324.93 968.95 3.21751 272.3 13.069 2050 2314.6 1726.8 3.8303 2303 2.555
Appendix A

1260 1348.55 986.90 3.23638 290.8 12.435 2100 2377.4 1775.3 3.8605 2559 2.356
1280 1372.24 1004.76 3.25510 310.4 11.835 2150 2440.3 1823.8 3.8901 2837 2.175
2200 2503.2 1872.4 3.9191 3138 2.012
1069

2250 2566.4 1921.3 3.9474 3464 1.864


Table A.9

T(K), h and u (kJ/mol), s (kJ/kmol ◊ K)


Carbon Dioxide, CO2 Carbon Monoxide, CO Water Vapour, H2O Oxygen, O2 Nitrogen, N2
( ht° = – 393,520 ( ht° = – 110,530 ( ht° = – 241,820 ( ht° = 0 kJ/kmol) ( ht° = 0 kJ/kmol)
kJ/kmol) kJ/kmol) kJ/kmol)
T h u s h u s h u s h u s h u s T
0 0 0 0 0 0 0 0 0 0 0 0 0 0 0 0 0
220 6,601 4,772 202.966 6,391 4,562 188.683 7,295 5,466 178.576 6,404 4,575 196.171 6,391 4,562 182.638 220
1070 Thermal Engineering

230 6,938 5,026 204.464 6,683 4,771 189.980 7,628 5,715 180.054 6,694 4,782 197.461 6,683 4,770 183.938 230
240 7,280 5,285 205.920 6,975 4,979 191.221 7,961 5,965 181.471 6,984 4,989 198.696 6,975 4,979 185.180 240
250 7,627 5,548 207.337 7,266 5,188 192.411 8,294 6,215 182.831 7,275 5,197 199.885 7,266 5,188 186.370 250
260 7,979 5,817 208.717 7,558 5,396 193.554 8,627 6,466 184.139 7,566 5,405 201.027 7,558 5,396 187.514 260
270 8,335 6,091 210.062 7,849 5,604 194.654 8,961 6,716 185.399 7,858 5,613 202.128 7,849 5,604 188.614 270
280 8,697 6,369 211.376 8,140 5,812 195.173 9,296 6,968 186.616 8,150 5,822 203.191 8,141 5,813 189.673 280
290 9,063 6,651 212.660 8,432 6,020 196.735 9,631 7,219 187.791 8,443 6,032 204.218 8,432 6,021 190.695 290
298 9,364 6,885 213.685 8,669 6,190 197.543 9,904 7,425 188.720 8,682 6,203 205.033 8,669 6,190 191.502 298
300 9,431 6,939 213.915 8,723 6,229 197.723 9,966 7,472 188.923 8,736 6,242 205.213 8,723 6,229 191.682 300
310 9,807 7,230 215.146 9,014 6,437 198.678 10,302 7,725 190.030 9,030 6,453 206.177 9,014 6,437 192.638 310
320 10,186 7,526 216.351 9,306 6,645 199.603 10,639 7,978 191.098 9,325 6,664 207.112 9,306 6,645 193.562 320
330 10,570 7,826 217.534 9,597 6,854 200.500 10,976 8,232 192.136 9,620 6,877 208.020 9,597 6,853 194.459 330
340 10,959 8,131 218.694 9,889 7,062 201.371 11,314 8,487 193.144 9,916 7,090 208.904 9,888 7,061 195.328 340
350 11,351 8,439 219.831 10,181 7,271 202.217 11,652 8,742 194.125 10,213 7,303 209.765 10,180 7,270 196.173 350
360 11,748 8,752 220.948 10,473 7,480 203.040 11,992 8,998 195.081 10,511 7,518 210.604 10,471 7,478 196.995 360
370 12,148 9,068 222.044 10,765 7,689 203.842 12,331 9,255 196.012 10,809 7,733 211.423 10,763 7,687 197.794 370
380 12,552 9,392 223.122 11,058 7,899 204.622 12,672 9,513 196.020 11,109 7,949 212.222 11,055 7,895 198.572 380
390 12,960 9,718 224.182 11,351 8,108 205.383 13,014 9,771 197.807 11,409 8,166 213.002 11,347 8,104 199.331 390
400 13,372 10,046 225.225 11,644 8,319 206.125 13,356 10,030 198.673 11,711 8,384 213.765 11,640 8,314 200.071 400
410 13,787 10,378 226.250 11,938 8,529 206.850 13,699 10,290 199.521 12,012 8,603 214.510 11,932 8,523 200.794 410
420 14,206 10,714 227.258 12,232 8,740 207.549 14,043 10,551 200.350 12,314 8,822 215.241 12,225 8,733 201.499 420
430 14,628 11,053 228.252 12,526 8,951 208.252 14,389 10,813 200.160 12,618 9,043 215.955 12,518 8,943 202.189 430
440 15,054 11,393 229.230 12,821 9,163 208.929 14,734 11,075 201.955 12,908 9,264 216.656 12,811 9,153 202.863 440
450 15,483 11,742 230.194 13,116 9,375 209.593 15,080 11,339 202.734 13,228 9,487 217.342 13,105 9,363 203.523 450
460 15,916 12,091 231.144 13,412 9,587 210.243 15,428 11,603 203.497 13,535 9,710 218.016 13,399 9,574 204.170 460

Contd.
Table A.9

T(K), h and u (kJ/mol), s (kJ/kmol ◊ K)


Carbon Dioxide, CO2 Carbon Monoxide, CO Water Vapour, H2O Oxygen, O2 Nitrogen, N2
( ht° = – 393,520 ( ht° = – 110,530 ( ht° = – 241,820 ( ht° = 0 kJ/kmol) ( ht° = 0 kJ/kmol)
kJ/kmol) kJ/kmol) kJ/kmol)
T h u s h u s h u s h u s h u s T

470 16,351 12,444 232.080 13,708 9,800 210.880 15,777 11,869 204.247 13,842 9,935 218.716 13,693 9,786 204.803 470
480 16,791 12,800 233.004 14,005 10,014 211.504 16,126 12,135 204.982 14,151 10,160 219.326 13,988 9,997 205.424 480
490 17,232 13,158 233.916 14,302 10,228 212.117 16,477 12,403 205.705 14,460 10,386 219.963 14,285 10,210 206.033 490
500 17,678 13,521 234.814 14,600 10,443 212.719 16,828 12,671 206.413 14,770 10,614 220.589 14,581 10,423 206.630 500
510 18,126 13,885 235.700 14,898 10,658 213.310 17,181 12,940 207.112 15,082 10,842 221.206 14,876 10,635 207.216 510
520 18,576 14,253 236.575 15,197 10,874 213.890 17,534 13,211 207.799 15,395 11,071 221.812 15,172 10,848 207.792 520
530 19,029 14,622 237.439 15,497 11,090 214.460 17,889 13,482 208.475 15,708 11,301 222.409 15,469 11,062 208.358 530
540 19,485 14,996 238.292 15,797 11,307 215.020 18,245 13,755 209.139 16,022 11,533 222.997 15,766 11,277 208.914 540
550 19,945 15,372 239.135 16,097 11,524 215.572 18,601 14,028 209.795 16,338 11,765 223.576 16,064 11,492 209.461 550
560 20,407 15,751 239.962 16,399 11,743 216.115 18,959 14,303 210.440 16,654 11,998 224.146 16,363 11,707 209.999 560
570 20,870 16,131 240.789 16,701 11,961 216.649 19,318 14,579 211.075 16,971 12,232 224.708 16,662 11,923 210.528 570
580 21,337 16,515 241.602 17,003 12,181 217.175 19,678 14,856 211.702 17,290 12,467 225.262 16,962 12,139 211.049 580
590 21,807 16,902 242.405 17,307 12,401 217.693 20,093 15,134 212.320 17,609 12,703 225.808 17,262 12,356 211.562 590
600 22,280 17,291 243.199 17,611 12,622 218.204 20,402 15,413 212.920 17,929 12,940 226.346 17,563 12,574 212.066 600
610 22,754 17,683 243.983 17,915 12,843 218.708 20,765 15,693 213.529 18,250 13,178 226.877 17,864 12,792 212.564 610
620 23,231 18,076 244.758 18,221 13,066 219.205 21,130 15,975 214.122 18,572 13,417 227.400 18,166 13,011 213.055 620
630 23,709 18,471 245.524 18,527 13,289 219.695 21,495 16,257 214.707 18,895 13,657 227.918 18,468 13,230 213.541 630
640 24,190 18,869 246.282 18,833 13,512 220.179 21,862 16,541 215.285 19,219 13,898 228.429 18,772 13,450 214.018 640
650 24,674 19,270 247.032 19,141 13,736 220.656 22,230 16,826 215.856 19,544 14,140 228.932 19,075 13,671 214.489 650
660 25,160 19,672 247.773 19,449 13,962 221.127 22,600 17,112 216.419 19,870 14,383 229.430 19,380 13,892 214.954 660
670 25,648 20,078 248.507 19,758 14,187 221.592 22,970 17,399 216.970 20,197 14,626 229.920 19,685 14,114 215.413 670
680 26,138 20,484 249.233 20,068 14,414 222.052 23,342 17,683 217.527 20,524 14,871 230.405 19,991 14,337 215.866 680
690 26,631 20,894 249.952 20,378 14,641 222.505 23,714 17,978 218.071 20,854 15,116 230.885 20,297 14,560 216.314 690
700 27,125 21,305 250.663 20,690 14,870 222.953 24,088 18,268 218.610 21,184 15,364 231.358 20,604 14,784 216.756 700
710 27,622 21,719 251.368 21,002 15,099 223.396 24,464 18,561 219.142 21,514 15,611 231.827 20,912 15,008 217.192 710
720 28,121 22,134 252.065 21,315 15,328 223.833 24,840 18,854 219.668 21,845 15,859 232.291 21,220 15,234 217.624 720
Appendix A

730 28,622 22,552 252.755 21,628 15,558 224.265 25,218 19,148 220.189 22,177 16,107 232.748 21,529 15,460 218.059 730
740 29,124 22,972 253.439 21,943 15,789 224.692 25,597 19,444 220.707 22,510 16,357 233.201 21,839 15,686 218.472 740
1071

Contd.
Table A.9

T(K), h and u (kJ/mol), s (kJ/kmol ◊ K)


Carbon Dioxide, CO2 Carbon Monoxide, CO Water Vapour, H2O Oxygen, O2 Nitrogen, N2
( ht° = – 393,520 ( ht° = – 110,530 ( ht° = – 241,820 ( ht° = 0 kJ/kmol) ( ht° = 0 kJ/kmol)
kJ/kmol) kJ/kmol) kJ/kmol)
T h u s h u s h u s h u s h u s T

750 29,629 23,393 254.117 22,258 16,022 225.115 25,977 19,741 221.215 22,844 16,607 233.649 22,149 15,913 218.889 750
760 30,135 23,817 254.787 22,573 16,255 225.533 26,358 20,039 221.720 23,178 16,859 234.091 22,460 16,141 219.301 760
1072 Thermal Engineering

770 30,644 24,242 255.452 22,890 16,488 225.947 26,741 20,339 222.221 23,513 17,111 234.528 22,772 16,370 219.709 770
780 31,154 24,669 256.110 23,208 16,723 226.357 27,125 20,639 222.717 23,850 17,364 234.960 23,085 16,599 220.113 780
790 31,665 25,097 256.762 23,526 16,957 226.762 27,510 20,941 223.207 24,186 17,618 235.387 23,398 16,830 220.512 790
800 32,179 25,527 257.408 23,844 17,193 227.162 27,896 21,245 223.693 24,523 17,872 235.810 23,714 17,061 220.907 800
810 32,694 25,959 258.048 24,164 17,429 227.559 28,284 21,549 224.174 24,861 18,126 236.230 24,027 17,292 221.298 810
820 33,212 26,394 258.682 24,483 17,665 227.952 28,672 21,855 224.651 25,199 18,382 236.644 24,342 17,524 221.684 820
830 33,730 26,829 259.311 24,803 17,902 228.339 29,062 22,162 225.123 25,537 18,637 237.055 24,658 17,757 222.067 830
840 34,251 27,267 259.934 25,124 18,140 228.724 29,454 22,470 225.592 25,877 18,893 237.462 24,974 17,990 222.447 840
850 34,773 27,706 260.551 25,446 18,379 229,106 29,846 22,779 226.057 26,218 19,150 237.864 25,292 18,224 222.822 850
860 35,296 28,125 261.164 25,768 18,617 229,482 30,240 23,090 226.517 26,599 19,666 238.264 25,610 18,459 223.194 860
870 35,821 28,588 261.770 26,091 18,858 229.856 30,635 23,402 226.973 26,899 19,666 238.660 25,928 18,695 223.562 870
880 36,347 29,031 262.371 26,415 19,099 230.227 31,032 23,715 227.426 27,242 19,925 239.051 26,248 18,931 223.927 880
890 36,876 29,476 262.968 26,740 19,341 230.593 31,429 24,029 227.875 27,584 20,185 239.439 26,568 19,168 224.288 890
900 37,405 29,922 263.559 27,066 19,583 230.957 31,828 24,345 228.321 27,928 20,445 239.823 26,890 19,407 224.647 900
910 37,935 30,369 264.146 27,392 19,826 231.317 32,228 24,662 228.763 28,272 20,706 240.203 27,210 19,644 225.002 910
920 38,467 30,818 264.728 27,719 20,070 231.674 32,629 24,980 229.202 28,616 20,967 240.580 27,532 19,883 225.353 920
930 39,000 31,268 265.304 28,046 20,314 232.028 33,032 25,300 229.637 28,960 21,228 240.953 27,854 20,122 225.701 930
940 39,535 31,719 265.877 28,375 20,559 232.397 33,436 25,621 230.070 29,306 21,491 241.323 28,178 20,362 226.047 940
950 40,070 32,171 266.444 28,703 20,805 232.727 33,841 25,943 230.499 29,652 21,754 241.689 28,501 20,603 226.389 950
960 40,607 32,625 267.007 29,033 21,051 233.072 34,247 26,265 230.924 29,999 22,017 242.052 28,826 20,844 226.728 960
970 41,145 33,081 267.566 29,362 21,298 233.413 34,653 26,588 231.347 30,345 22,280 242.411 29,151 21,086 227.064 970
980 41,685 33,537 268.119 29,693 21,545 233.752 35,061 26,913 231.767 30,692 22,544 242.768 29,476 21,328 227.398 980
990 42,226 33,995 268.670 30,024 21,793 234.088 35,472 27,240 232.184 31,041 22,809 243.120 29,803 21,571 227.728 990
1000 42,769 34,455 269.215 30,355 22,041 234.421 35,882 27,568 232.597 31,389 23,075 243.471 30,129 21,815 228.057 1000
1020 43,859 35,378 270.293 31,020 22,540 235.079 36,709 28,228 233.415 32,088 23,607 244.164 30,784 22,304 228.706 1020
1040 44,953 36,306 271.354 31,688 23,041 235.728 37,542 28,895 234.223 32,789 24,142 244.844 31,442 22,795 229.344 1040
Contd.
Table A.9

T(K), h and u (kJ/mol), s (kJ/kmol ◊ K)


Carbon Dioxide, CO2 Carbon Monoxide, CO Water Vapour, H2O Oxygen, O2 Nitrogen, N2
( ht° = – 393,520 ( ht° = – 110,530 ( ht° = – 241,820 ( ht° = 0 kJ/kmol) ( ht° = 0 kJ/kmol)
kJ/kmol) kJ/kmol) kJ/kmol)
T h u s h u s h u s h u s h u s T

1060 46,051 37,238 272.400 32,357 23,544 236.364 38,380 29,567 235.020 33,490 24,677 245.513 32,101 23,288 229.973 1060
1080 47,153 38,174 273.430 33,029 24,049 236.992 39,223 30,243 235.806 34,194 25,214 246.171 32,762 23,782 230.591 1080
1100 48,258 39,112 274.445 33,702 24,557 237.609 40,071 30,925 236.584 34,899 25,753 246.818 33,426 24,280 231.199 1100
1120 49,369 40,057 275.444 34,377 25,065 238.217 40,923 31,611 237.352 35,606 26,294 247.454 34,092 24,780 231.799 1120
1140 50,484 41,006 276.430 35,054 25,575 238.817 41,780 32,301 238.110 36,314 26,836 248.081 34,760 25,282 232.391 1140
1160 51,602 41,957 277.403 35,733 26,088 239.407 42,642 32,997 238.859 37,023 27,379 248.698 35,430 25,786 232.973 1160
1180 52,724 42,913 278.362 36,406 26,602 239.989 43,509 33,698 239.600 37,734 27,923 249.307 36,104 26,291 233.549 1180
1200 53,848 43,871 279.307 37,095 27,118 240.663 44,380 34,403 240.333 38,447 28,469 249.906 36,777 26,799 234.115 1200
1220 54,977 44,834 280.238 37,780 27,637 241.128 45,256 35,112 241.057 39,162 29,018 250.497 37,452 27,308 234.673 1220
1240 56,108 45,799 281.158 38,466 28,426 241.686 46,137 35,827 241.773 39,877 29,568 251.079 38,129 27,819 235.223 1240
1260 57,244 46,768 282.066 39,154 28,678 242.236 47,022 36,546 242.482 40,594 30,118 251.653 38,807 28,331 235.766 1260
1280 58,381 47,739 282.962 39,884 29,201 242.780 47,912 37,270 243.183 41,312 30,670 252.219 39,488 28,845 236.302 1280
1300 59,522 48,713 283.847 40,534 29,725 243.316 48,807 38,000 243.877 42,033 31,224 252.776 40,170 29,361 236.831 1300
1320 60,666 49,691 284.722 41,266 30,251 243.844 49,707 38,732 244.564 42,753 31,778 253.325 40,853 29,878 237.353 1320
1340 61,813 50,672 285.586 41,919 30,778 244.366 50,612 39,470 245.243 43,475 32,334 353.868 41,539 30,398 237.867 1340
1360 62,963 51,656 286.439 42,613 31,306 244.880 51,521 40,213 245.915 44,198 32,891 254.404 42,227 30,919 238.376 1360
1380 64,116 52,643 287.283 43,309 31,836 245.388 52,434 40,960 246.582 44,923 33,449 254.932 42,915 31,441 238.878 1380
1400 65,271 53,631 288.106 44,007 32,367 245.889 53,351 41,711 247.241 45,648 34,008 255.454 43,605 31,964 239.375 1400
1420 66,427 54,621 288.934 44,707 32,900 246.385 54,273 42,466 247.895 46,374 34,567 255.968 44,295 32,489 239.865 1420
1440 67,586 55,614 289.743 45,408 33,434 246.876 55,198 43,226 248.543 47,102 35,129 256.475 44,988 33,014 240.350 1440
1460 68,748 56,609 290.542 46,110 33,971 247.360 56,128 43,989 249.185 47,831 35,692 256.978 45,682 33,543 240.827 1460
1480 69,911 57,606 291.333 46,813 34,508 247.839 57,062 44,756 249.820 48,561 36,256 257.474 46,377 34,071 241.301 1480
1500 71,078 58,606 292.114 47,517 35,046 248.312 57,999 45,528 250.450 49,292 36,821 257.965 47,073 34,601 241.768 1500
1520 72,246 59,609 292.888 48,222 35,584 248.778 58,942 46,304 251.074 50,024 37,387 258.450 47,771 35,133 242.228 1520
1540 73,417 60,613 292.654 48,928 36,124 249.240 59,888 47,084 251.693 50,756 37,952 258.928 48,470 35,665 242.685 1540
1560 74,590 61,620 294.411 49,635 36,665 249.695 60,838 47,868 252.305 51,490 38,520 259.402 49,168 36,197 243.137 1560
Appendix A

1580 76,767 62,630 295.161 50,344 37,207 250.147 61,792 48,655 252.912 52,224 39,088 259.870 49,869 36,732 243.585 1580
Contd.
1073
Table A.9

T(K), h and u (kJ/mol), s (kJ/kmol ◊ K)


Carbon Dioxide, CO2 Carbon Monoxide, CO Water Vapour, H2O Oxygen, O2 Nitrogen, N2
( ht° = – 393,520 ( ht° = – 110,530 ( ht° = – 241,820 ( ht° = 0 kJ/kmol) ( ht° = 0 kJ/kmol)
kJ/kmol) kJ/kmol) kJ/kmol)
T h u s h u s h u s h u s h u s T
1600 76,944 63,741 295.901 51,053 37,750 250.592 62,748 49,445 253.513 52,961 39,658 260.333 50,571 37,268 244.028 1600
1620 78,123 64,653 296.632 51,763 38,293 251.033 63,709 50,240 254.111 53,696 40,227 260.791 51,275 37,806 244.464 1620
1074 Thermal Engineering

1640 79,303 65,668 297.356 52,472 38,837 251.470 64,675 51,039 254.703 54,434 40,799 261.242 51,980 38,344 244.896 1640
1660 80,486 66,592 298.072 53,184 39,382 251.901 65,643 51,841 255.290 55,172 41,370 261.690 52,686 38,884 245.324 1660
1680 81,670 67,702 298.781 53,895 39,927 252.329 66,614 52,646 255.873 55,912 41,944 262.132 53,393 39,424 245.747 1680
1700 82,856 68,721 299.482 54,609 40,474 252.751 67,589 53,455 256.450 56,652 42,517 262.571 54,099 39,965 246.166 1700
1720 84,043 69,742 300.177 55,323 41,023 253.169 68,567 54,267 257.022 57,394 43,093 263.005 54,807 40,507 246.580 1720
1740 85,231 70,764 300.863 56,039 41,572 253.582 69,550 55,083 257.589 58,136 43.669 263.435 55,516 41,049 246.990 1740
1760 86,420 71,787 301.543 56,756 42,123 253.991 70,535 55,902 258.151 58,800 44,247 263.861 56,227 41,594 247.396 1760
1780 87,612 72,812 302.271 57,473 42,673 254.398 71,523 56,723 258.708 59,624 44,825 264.283 56,938 42,139 247.798 1780
1800 88,806 73,840 302.884 58,191 43,225 254.797 72,513 57,547 259.262 60,371 45,405 264.701 57,651 42,685 248.195 1800
1820 90,000 74,868 303.544 58,910 43,778 225.194 73,507 58,375 259.811 61,118 45,986 265.113 58,363 43,231 248.589 1820
1840 91,196 75,897 304.198 59,629 44,331 255.587 74,506 59,207 260.357 61,866 46,568 265.521 59,075 43,777 248.979 1840
1860 92,394 76,929 304.845 60,351 44,886 255.976 75,506 60,042 260.898 62,616 47,151 265.925 59,790 44,324 249.365 1860
1880 93,593 77,962 305.487 61,072 45,441 256.361 76,511 60,880 261.436 63,365 47,734 266.326 60,504 44,873 249.748 1880
1900 94,793 78,996 306.122 61,794 45,997 256.743 77,517 61,720 261.969 64,116 48,319 266.722 61,220 45,423 250.128 1900
1920 95,995 80,031 306.751 62,516 46,552 257.122 78,527 62,564 262.497 64,868 48,904 267.115 61,936 45,973 250.502 1920
1940 97,197 81,067 307.374 63,238 47,108 257.497 79,540 63,411 263.022 65,620 49,490 267.505 62,654 46,524 250.874 1940
1960 98,401 82,105 307.992 63,961 47,665 257.868 80,555 64,259 263.542 66,374 50,078 267.891 63,318 47,075 251.242 1860
1980 99,606 83,144 308.604 64,684 48,221 258.236 81,573 65,111 264.059 67,127 50,665 268.275 64,090 47,627 251.607 1980
2000 100,804 84,185 309.210 65,408 48,780 258.600 82,593 65,965 264.571 67,881 51,253 268.655 64,810 48,181 251.969 2000
2050 103,835 86,791 310.701 67,224 50,179 259.494 85,156 68,111 265.838 69,772 52,727 269.588 66,612 49,567 252.858 2050
2100 106,864 89,404 312.160 69,044 51,584 260.370 87,735 70,275 267.081 71,668 54,208 270.504 68,417 50,957 253.726 2100
2150 109,898 92,023 313.589 70,864 52,988 261.226 90,330 72,454 268.301 73,573 55,697 271.399 70,226 52,351 254.578 2150
2200 112,939 94,648 314.988 72,688 54,396 262.065 92,040 74,649 269.500 75,484 57,192 272.278 72,040 53,749 255.412 2200
2250 115,984 97,277 316.356 74,516 55,809 262.887 95,562 76,855 270.679 77,397 58,690 273.136 73,856 55,149 256.227 2250
2300 119,035 99,912 317.695 76,345 57,222 263.692 98,199 79,076 271.839 79,316 60,193 273.981 75,676 56,553 257.027 2300
2350 122,091 102,552 319.011 78,178 58,640 264.480 100,846 81,308 272.978 81,243 61,704 274.809 77,496 57,958 257.810 2350
2400 125,152 105,197 320.302 80,015 60,060 265.253 103,508 83,553 274.098 83,174 63,219 275.625 79,320 59,366 258.580 2400
2450 128,219 107,849 321.566 81,852 61,482 266.012 106,183 85,811 275.201 85,112 64,742 276.424 81,149 60,779 259.332 2450
Contd.
Table A.9

T(K), h and u (kJ/mol), s (kJ/kmol ◊ K)


Carbon Dioxide, CO2 Carbon Monoxide, CO Water Vapour, H2O Oxygen, O2 Nitrogen, N2
( ht° = – 393,520 ( ht° = – 110,530 ( ht° = – 241,820 ( ht° = 0 kJ/kmol) ( ht° = 0 kJ/kmol)
kJ/kmol) kJ/kmol) kJ/kmol)
T h u s h u s h u s h u s h u s T

2500 131,290 110,504 322.808 83,692 62,906 266.755 108,868 88,082 276.286 87,057 66,271 277.207 82,981 62,195 260.073 2500
2550 134,368 113,166 324.026 85,537 64,335 267.485 111,565 90,364 277.354 89,004 67,802 277.979 84,814 63,613 260.799 2550
2600 137,449 115,832 325.222 87,383 65,766 268.202 114,273 92,656 278.407 90,956 69,339 278.738 86,650 65,033 261.512 2600
2650 140,533 118,500 326.396 89,230 67,197 268.905 116,991 94,958 279.441 92,916 70,883 279.485 88,488 66,455 262.213 2650
2700 143,620 121,172 327.549 91,077 68,628 269.596 119,717 97,269 280.462 94,881 72,433 280.219 90,328 67,880 262.902 2700
2750 146,713 123,849 328.684 92,930 70,066 270.285 122,453 99,588 281.464 96,852 73,987 280.942 92,171 69,306 263.577 2750
2800 149,808 126,528 329.800 94,784 71,504 270.943 125,198 101,917 282.453 98,826 75,546 281.654 94,014 70,734 264.241 2800
2850 152,908 129,212 330.896 96,639 72,945 271.602 127,952 104,256 283.429 100,80 77,112 282.357 95,859 72,163 264.895 2850
2900 156,009 131,898 331.975 98,495 74,383 272.249 130,717 106,605 284.390 102,793 78,682 283.048 97,705 73,593 265.538 2900
2950 159,117 134,589 333.037 100,352 75,825 272.884 133,486 106,959 285.338 104,785 80,258 283.728 99,556 75,028 266.170 2950
3000 162,226 137,283 334.084 102,210 77,267 273.508 136,264 111,321 286.273 106,780 81,837 284.399 101,407 76,464 266.793 3000
3050 165,341 139,982 335.114 104,073 78,715 274.123 139,051 113,692 287.194 108,778 83,419 285.060 103,260 77,902 267.404 3050
3100 168,456 142,681 336.126 105,939 80,164 274.730 141,846 116,072 288.102 110,784 85,009 285.713 105,115 79,341 268.007 3100
3150 171,576 145,385 337.124 107,802 81,612 275.326 144,648 118,458 288.999 112,795 86,601 286.355 106,972 80,782 268.601 3150
3200 174,695 148,089 338.109 109,667 83,061 275.914 147,457 120,851 289.884 114,809 88,203 286.989 108,830 82,224 269.186 3200
3250 177,822 150,801 339.069 111,534 84,513 276.494 150,272 123,250 290.756 116,827 89,804 287.614 110,690 83,668 269.763 3250
Appendix A
1075
1076 Thermal Engineering

Compressible-Flow Functions
g

M M* A/A* p/p0 r/r0 T/T0


0.0 0.00000 1.00000 1.00000 1.00000
0.1 0.10944 5.82183 0.99303 0.99502 0.99800
0.2 0.21822 2.96352 0.97250 0.98028 0.99206
0.3 0.32572 2.03506 0.93947 0.95638 0.98232
0.4 0.43133 1.59014 0.89561 0.92427 0.96899
0.5 0.53452 1.33984 0.84302 0.88517 0.95238
0.6 0.63481 1.18820 0.78400 0.84045 0.93284
0.7 0.73179 1.09437 0.72093 0.79158 0.91075
0.8 0.82514 1.03823 0.65602 0.73999 0.88652
0.9 0.91460 1.00886 0.59126 0.68704 0.68059
1.0 1.0000 1.00000 0.52828 0.63394 0.83333
1.1 1.0812 1.00793 0.46835 0.58170 0.80515
1.2 1.1583 1.03044 0.41238 0.53114 0.77640
1.3 1.2311 1.06630 0.36091 0.48290 0.77640
1.4 1.2999 1.11493 0.31424 0.43742 0.71839
1.5 1.3646 1.17617 0.27240 0.39498 0.68966
1.6 1.4254 1.25023 0.23527 0.35573 0.66138
1.7 1.4825 1.33761 0.20259 0.31969 0.63371
1.8 1.360 1.43898 0.17404 0.28682 0.60680
1.9 1.5861 1.55526 0.14924 0.25699 0.58072
2.0 1.6330 1.68750 0.12780 0.23005 0.86059
2.1 1.6769 1.83694 0.10935 0.20580 0.53135
2.2 1.7179 2.00497 0.93522E-01 0.18405 0.50813
2.3 1.7563 2.19313 0.79973E-01 0.16458 0.48591
2.4 1.7922 2.40310 0.68399E-01 0.14720 0.46468
2.5 1.8257 2.63672 0.58528E-01 0.13169 0.44444
2.6 1.8571 2.89598 0.50115E-01 0.11787 0.42517
2.7 1.8865 3.18301 0.42950E-01 0.10557 0.40683
2.8 1.9140 3.50012 0.36848E-01 0.94626E-01 0.38941
2.9 1.9398 3.84977 0.31651E-01 0.84889E-01 0.37286
3.0 1.9640 4.23457 0.27224E-01 0.76226E-01 0.35714
3.5 2.0642 6.78962 0.13111E-01 0.45233E-01 0.28986
4.0 2.3181 10.7188 0.65861E-02 0.27662E-01 0.23810
4.5 2.1936 16.5622 0.34553E-02 0.17449E-01 0.19802
5.0 2.2361 25.0000 0.18900E-02 0.11340E-01 0.16667
6.0 2.2953 53.1798 0.63336E-02 0.51936E-02 0.12195
7.0 2.3333 104.143 0.24156E-03 0.26088E-02 0.09259
8.0 2.3591 190.109 0.10243E-03 0.14135E-03 0.07246
9.0 2.3772 237.189 0.47386E-04 0.81504E-03 0.05814
10.0 2.3905 535.938 0.23563E-04 0.49482E-03 0.04762
2.4495 0.0 0.0 0.0
Appendix A 1077

Table A.11 Normal Shock Functions


g

Mx My py/px ry/rx Ty/T p0y/p0x p0y/px


1.00 1.00000 1.0000 1.0000 1.0000 1.00000 1.8928
1.05 0.95313 1.1196 1.0840 1.0328 0.99985 2.0083
1.10 0.91177 1.2450 1.1691 1.0642 0.99893 2.1328
1.15 0.87502 1.3763 1.2550 1.0966 0.99669 2.2661
1.20 0.84217 1.5133 1.3416 1.1280 0.99280 2.4075
1.25 0.81264 1.6563 1.4286 1.1594 0.98706 2.5568
1.30 0.78596 1.8050 1.5157 1.1909 0.97937 2.7136
1.35 0.76175 1.9596 1.6028 1.2226 0.96974 2.8778
1.40 0.73971 2.1200 1.6897 1.2547 0.95819 3.0492
1.45 0.71956 2.2863 1.7761 1.2872 0.94484 3.2278
1.50 0.70109 2.4583 1.8621 1.3202 0.92979 3.4133
1.55 0.68610 2.6362 1.9473 1.3538 0.91319 3.6057
1.60 0.66844 2.8200 2.0317 1.3880 0.89520 3.8050
1.65 0.65396 3.0096 2.1152 1.4228 0.87598 4.0110
1.70 0.64054 3.2050 2.1977 1.4583 0.85572 4.2238
1.75 0.62809 3.4063 2.2791 1.4946 0.83457 4.4433
1.80 0.61650 3.6133 2.3592 1.5316 0.81268 4.6695
1.85 0.60570 3.8263 2.4381 1.5693 0.79023 4.9023
1.90 0.59562 4.0450 2.5157 1.6079 0.76736 5.1418
1.95 0.58618 4.2696 2.5919 1.6473 0.74420 5.3878
2.00 0.57735 4.5000 2.6667 1.6875 0.72087 5.6404
2.05 0.56906 4.7362 2.7400 1.7285 0.69751 5.8996
2.10 0.56128 4.9783 2.8119 1.7705 0.67420 6.1654
2.15 0.55395 5.2263 2.8823 1.8132 0.65105 6.4377
2.20 0.54706 5.4800 2.9512 1.8569 0.62814 6.7165
2.25 0.54055 5.7396 3.0186 1.9014 0.60553 7.0018
2.30 0.53441 6.0050 3.0845 1.9468 0.58329 7.2937
2.35 0.52861 6.2762 3.1490 1.9931 0.56148 7.5920
2.40 0.52312 6.5533 3.2119 2.0403 0.54014 7.8969
2.45 0.51792 6.8363 3.2733 2.0885 0.51931 8.2083
2.50 0.51299 7.1250 3.3333 2.1375 0.49901 8.5261
2.55 0.50831 7.4196 3.3919 2.1875 0.47928 8.8505
2.60 0.50387 7.7200 3.4490 2.2383 0.46012 9.1813
2.70 0.49563 8.3383 3.5590 2.3429 0.42359 9.8624
2.80 0.48817 8.9800 3.6636 2.4512 0.38946 10.569
2.90 0.48138 9.6450 3.7629 2.5632 0.35773 11.302
3.00 0.47519 10.333 3.8571 2.6790 0.32834 12.061
4.00 0.43496 18.500 4.5714 4.0469 0.13876 21.068
5.00 0.41523 29.000 5.0000 5.8000 0.06172 32.653
10.00 0.38758 116.50 5.7143 20.387 0.00304 129.22
1078 Thermal Engineering

Table A.12

Specific Volume Internal Energy Enthalpy Entropy


m3/kg kJ/kg kJ/kg kJ/kg.K
Sat. Sat. Sat. Sat. Sat. Sat. Evap. Sat. Sat. Sat.
Temp. Presss. Liquid Vapour Liquid Vapour Liquid Vapour Liquid Vapour Temp.
°C bar vf ¥ 103 vg uf ug hf hfg hg sf sg °C
0.01 0.00611 1.0002 206.136 0.00 2375.3 0.01 2501.3 2501.4 0.0000 9.1562 0.01
4 0.00813 1.0001 157.232 16.77 2380.9 16.78 2491.9 2508.7 0.0610 9.0514 4
5 0.00872 1.0001 147.120 20.97 2382.3 20.98 2489.6 2510.6 0.0761 9.0257 5
6 0.00935 1.0001 137.734 25.19 2383.6 25.20 2487.2 2512.4 0.0912 9.0003 6
8 0.01072 1.0002 120.917 33.59 2386.4 33.60 2482.5 2516.1 0.1212 8.9501 8
10 0.01228 1.0004 106.379 42.00 2389.2 42.01 2477.7 2519.8 0.1510 8.9008 10
11 0.01312 1.0004 99.857 46.20 2390.5 46.20 2475.4 2521.6 0.1658 8.8765 11
12 0.01402 1.0005 93.784 50.41 2391.9 50.41 2473.0 2523.4 0.1806 8.8524 12
13 0.01497 1.0007 88.124 54.60 2393.3 54.60 2470.7 2525.3 0.1953 8.8285 13
14 0.01598 1.0008 82.848 58.79 2394.7 58.80 2468.3 2527.1 0.2099 8.8048 14
15 0.01705 1.0009 77.926 62.99 2396.1 62.99 2465.9 2528.9 0.2245 8.7814 15
16 0.01818 1.0011 73.333 67.18 2397.4 67.19 2463.6 2530.8 0.2390 8.7582 18
17 0.01938 1.0012 69.044 71.38 2398.8 71.38 2461.2 2532.6 0.2535 8.7351 17
18 0.02064 1.0014 65.038 75.57 2400.2 75.58 2458.8 2534.4 0.2679 8.7123 18
19 0.02198 1.0016 61.293 79.76 2401.6 79.77 2456.5 2536.2 0.2823 8.6897 19
20 0.02339 1.0018 57.791 83.95 2402.9 83.96 2454.1 2538.1 0.2966 8.6672 20
21 0.02487 1.0020 54.514 88.14 2404.3 88.14 2451.8 2539.9 0.3109 8.6450 21
22 0.02645 1.0022 51.447 92.32 2405.7 92.33 2449.4 2541.7 0.3251 8.6229 22
23 0.02810 1.0024 48.574 96.51 2407.0 96.52 2447.0 2543.5 0.3393 8.6011 23
24 0.02985 1.0027 45.883 100.70 2408.4 100.70 2444.7 2545.4 0.3534 8.5794 24
25 0.03169 1.0029 43.360 104.88 2409.8 104.89 2442.3 2547.2 0.3674 8.5580 25
26 0.03363 1.0032 40.994 109.06 2411.1 109.07 2439.9 2549.0 0.3814 8.5367 26
27 0.03567 1.0035 38.774 113.25 2412.5 113.25 2437.6 2550.8 0.3954 8.5156 27
28 0.03782 1.0037 36.690 117.42 2413.9 117.43 2435.2 2552.6 0.4093 8.4946 28
29 0.04008 1.0040 34.733 121.60 2415.2 121.61 2432.8 2554.5 0.4231 8.4739 29
30 0.04246 1.0043 32.894 125.78 2416.6 125.79 2430.5 2556.3 0.4369 8.4533 30
31 0.04496 1.0046 31.165 129.96 2418.0 129.97 2428.1 2558.1 0.4507 8.4329 31
32 0.04759 1.0050 29.540 134.14 2419.3 134.15 2425.7 2559.9 0.4644 8.4127 32
33 0.05034 1.0053 28.011 138.32 2420.7 138.33 2423.4 2561.7 0.4781 8.3927 33
34 0.05324 1.0056 26.571 142.50 2422.0 142.50 2421.0 2563.5 0.4917 8.3728 34
35 0.05628 1.0060 25.216 146.67 2423.4 146.68 2418.6 2565.3 0.5053 8.3531 35
36 0.05947 1.0063 23.940 150.85 2424.7 150.86 2416.2 2567.1 0.5188 8.3336 36
38 0.06632 1.0071 21.602 159.20 2427.4 159.21 2411.5 2570.7 0.5458 8.2950 38
40 0.07384 1.0078 19.523 167.56 2430.1 167.57 2406.7 2574.3 0.5725 8.2570 40
45 0.09593 1.0099 15.258 188.44 2436.8 188.45 2394.8 2583.2 0.6387 8.1648 45
50 0.1235 1.0121 12.032 209.32 2443.5 209.33 2382.7 2592.1 0.7038 8.0763 50
Contd.
Appendix A 1079

Table A.12

Specific Volume Internal Energy Enthalpy Entropy


m3/kg kJ/kg kJ/kg kJ/kg.K
Sat. Sat. Sat. Sat. Sat. Sat. Evap. Sat. Sat. Sat.
Temp. Presss. Liquid Vapour Liquid Vapour Liquid Vapour Liquid Vapour Temp.
°C bar vf ¥ 103 vg uf ug hf hfg hg sf sg °C
55 0.1576 1.0146 9.568 230.21 2450.1 230.23 2370.7 2600.9 0.7679 7.9913 55
60 0.1994 1.0172 7.671 251.11 2456.6 251.13 2358.5 2609.6 0.8312 7.9096 60
65 0.2503 1.0199 6.197 272.02 2463.1 272.06 2346.2 2618.3 0.8935 7.8310 65
70 0.3119 1.0228 5.042 292.95 2469.6 292.98 2333.8 2626.8 0.9549 7.7553 70
75 0.3858 1.0259 4.131 313.90 2475.9 313.93 2321.4 2635.3 1.0155 7.6824 75
80 0.4739 1.0291 3.407 334.86 2482.2 334.91 2308.8 2643.7 1.0753 7.6122 80
85 0.5783 1.0325 2.828 355.84 2488.4 355.90 2296.0 2651.9 1.1343 7.5445 85
90 0.7014 1.0360 2.361 376.85 2494.5 376.92 2283.2 2660.1 1.1925 7.4791 90
95 0.8455 1.0397 1.982 397.88 2500.6 397.96 2270.2 2668.1 1.2500 7.4159 95
100 1.014 1.0435 1.673 418.94 2506.5 419.04 2257.0 2676.1 1.3069 7.3549 100
110 1.433 1.0516 1.210 461.14 2518.1 461.30 2230.2 2691.5 1.4185 7.2387 110
120 1.985 1.0603 0.8919 503.50 2529.3 503.71 2202.6 2706.3 1.5276 7.1296 120
130 2.701 1.0697 0.6685 546.02 2539.9 546.31 2174.2 2720.5 1.6344 7.0269 130
140 3.6153 1.0797 0.5089 588.74 2550.0 589.13 2144.7 2733.9 1.7391 6.9299 140
150 4.758 1.0905 0.3928 631.68 2559.5 632.20 2114.3 2746.5 1.8418 6.8379 150
160 6.178 1.1020 0.3071 674.86 2568.4 675.55 2082.6 2758.1 1.9427 6.7502 160
170 7.917 1.1143 0.2428 718.33 2576.5 719.21 2049.5 2768.7 2.0419 6.6663 170
180 10.02 1.1274 0.1941 762.09 2583.7 763.22 2015.0 2778.2 2.1396 6.5857 180
190 12.54 1.1414 0.1565 806.19 2590.0 807.62 1978.8 2786.4 2.2359 6.5079 190
200 15.54 1.1565 0.1274 850.65 2595.3 852.45 1940.7 2793.2 2.3309 6.4323 200
210 19.06 1.1726 0.1044 895.53 2599.5 897.76 1900.7 2798.5 2.4248 6.3585 210
220 23.18 1.1900 0.08619 940.87 2602.4 943.62 1858.5 2802.1 2.5178 6.2861 220
230 27.95 1.2088 0.07158 986.74 2603.9 990.12 1813.8 2804.0 2.6099 6.2146 230
240 33.44 1.2291 0.05976 1033.2 2604.0 1037.3 1766.5 2803.8 2.7015 6.1437 240
250 39.73 1.2512 0.05013 1080.4 2602.4 1085.4 1716.2 2801.5 2.7927 6.0730 250
260 46.88 1.2755 0.04221 1128.4 2599.0 1134.4 1662.5 2796.6 2.8838 6.0019 260
270 54.99 1.3023 0.03564 1177.4 2593.7 1184.5 1605.2 2789.7 2.9751 5.9301 270
280 64.12 1.3321 0.03017 1227.5 2586.1 1236.0 1543.6 2779.6 3.0668 5.8571 280
290 74.36 1.3656 0.02557 1278.9 2576.0 1289.1 1477.1 2766.2 3.1594 5.7821 290
300 85.81 1.4036 0.02167 1332.0 2563.0 1344.0 1404.9 2749.0 3.2534 5.7045 300
320 112.7 1.4988 0.01549 1444.6 2525.5 1461.5 1238.6 2700.1 3.4480 5.5362 320
340 145.9 1.6379 0.01080 1570.3 2464.6 1594.2 1027.9 2622.0 3.6594 5.3357 340
360 186.5 1.8925 0.006945 1725.2 2351.5 1760.5 720.5 2481.0 3.9147 5.0526 360
374.14 220.9 3.155 0.003155 2029.6 2029.6 2099.3 0 2099.3 4.4298 4.4298 374.14
1080 Thermal Engineering

Table A.13 Saturated Water

Specific Volume Internal Energy Enthalpy Entropy


m3/kg kJ/kg kJ/kg kJ/kg ◊ K
Sat. Sat. Sat. Sat. Sat. Sat. Evap. Sat. Sat. Sat.
Press. Temp. Liquid Vapour Liquid Vapour Liquid Vapour Liquid Vapour Press.
bar °C vf ¥ 103 vg uf ug hf hfg hg sf sg bar
0.04 28.96 1.0040 34.800 121.45 2415.2 121.46 2432.9 2554.4 0.4226 8.4746 0.04
0.06 36.16 1.0064 23.739 151.53 2425.0 151.53 2415.9 2567.4 0.5210 8.3304 0.06
0.08 41.51 1.0084 18.103 173.87 2432.2 173.88 2403.1 2577.0 0.5926 8.2287 0.18
0.10 45.81 1.0102 14.674 181.82 2437.9 191.83 2392.8 2584.7 0.6493 8.1502 0.10
0.20 60.06 1.0172 7.649 251.38 2456.7 251.40 2358.3 2609.7 0.8320 7.9085 0.20
0.30 69.10 1.0223 5.229 289.20 2468.4 289.23 2336.1 2625.3 0.9439 7.7686 0.30
0.40 75.87 1.0265 3.993 317.53 2477.0 317.58 2319.2 2636.8 1.0259 7.6700 0.40
0.50 81.33 1.0300 3.240 340.44 2483.9 340.49 2305.4 2645.9 1.0910 7.5939 0.50
0.60 85.94 1.0331 2.732 359.79 2489.6 359.86 2293.6 2653.5 1.1453 7.5320 0.60
0.70 89.95 1.0360 2.365 376.63 2494.5 376.70 2283.3 2660.0 1.1919 7.4797 0.70
0.80 93.50 1.0380 2.087 391.58 2498.8 391.66 2274.1 2665.8 1.2329 7.4346 0.80
0.90 96.71 1.0410 1.869 405.06 2502.6 405.15 2265.7 2670.9 1.2695 7.3949 0.90
1.00 99.63 1.0432 1.694 417.36 2506.1 417.46 2258.0 2675.5 1.3026 7.3594 1.00
1.50 111.4 1.0528 1.159 466.94 2519.7 467.11 2226.5 2693.6 1.4336 7.2233 1.50
2.00 120.2 1.0605 0.8857 504.49 2529.5 504.70 2201.9 2706.7 1.5301 7.1271 2.00
2.50 127.4 1.0672 0.7187 535.10 2537.2 535.37 2181.5 2716.9 1.6072 7.0527 2.50
3.00 133.6 1.0732 0.6058 561.15 2543.6 561.47 2163.8 2725.3 1.6718 6.9919 3.00
3.50 138.9 1.0786 0.5243 583.95 2546.9 584.33 2148.1 2732.4 1.7275 6.9405 3.50
4.00 143.6 1.0836 0.4625 604.31 2553.6 604.74 2133.8 2738.6 1.7766 6.8959 4.00
4.50 147.9 1.0882 0.4140 622.25 2557.6 623.25 2120.7 2743.9 1.8207 6.8565 4.50
5.00 151.9 1.0926 0.3749 639.68 2561.2 640.23 2108.5 2748.7 1.8607 6.8212 5.00
6.00 158.9 1.1006 0.3157 669.90 2567.4 670.56 2086.3 2756.8 1.9312 6.7600 6.00
7.00 165.0 1.1080 0.2729 696.44 2572.5 697.22 2066.3 2763.5 1.9922 6.7080 7.00
8.00 170.4 1.1148 0.2404 720.22 2576.8 721.11 2048.0 2769.1 2.0462 6.6628 8.00
9.00 175.4 1.1212 0.2150 741.83 2580.5 742.83 2031.1 2773.9 2.0946 6.6226 9.00
10.0 179.9 1.1273 0.1944 761.68 2583.6 762.81 2015.3 2778.1 2.1387 6.5863 10.0
15.0 198.3 1.1539 0.1318 843.16 2594.5 844.84 1947.3 2792.2 2.3150 6.4448 15.0
20.0 212.4 1.1767 0.09963 906.44 2600.3 908.79 1890.7 2799.5 2.4474 6.3409 20.0
25.0 224.0 1.1983 0.07998 959.11 2603.1 962.11 1841.0 2803.1 2.5547 6.2575 25.0
30.0 233.9 1.2165 0.06668 1004.8 2604.1 1008.4 1795.7 2804.2 2.6457 6.1869 30.0
35.0 242.6 1.2347 0.05707 1045.4 2603.7 1049.8 1753.7 2803.4 2.7253 6.1253 35.0
40.0 250.4 1.2522 0.04978 1082.3 2602.3 1087.3 1714.1 2801.4 2.7964 6.0701 40.0
45.0 257.5 1.2692 0.04406 1116.2 2600.1 1121.9 1676.4 2798.3 2.8610 6.0199 45.0
50.0 264.0 1.2859 0.03944 1147.8 2597.1 1154.2 1640.1 2794.3 2.9202 5.9734 50.0

Contd.
Appendix A 1081

Table A.13

Specific Volume Internal Energy Enthalpy Entropy


m3/kg kJ/kg kJ/kg kJ/kg ◊ K
Sat. Sat. Sat. Sat. Sat. Sat. Evap. Sat. Sat. Sat.
Press. Temp. Liquid Vapour Liquid Vapour Liquid Vapour Liquid Vapour Press.
bar °C vf ¥ 103 vg uf ug hf hfg hg sf sg bar

60.0 275.6 1.3187 0.03244 1205.4 2589.7 1213.4 1571.0 2784.3 3.0267 5.8892 60.0
70.0 285.9 1.3513 0.02737 1257.6 2580.5 1267.0 1505.1 2772.1 3.1211 5.8133 70.0
80.0 295.1 1.3842 0.02352 1305.6 2569.8 1316.6 1441.3 2758.0 3.2068 5.7432 80.0
90.0 303.4 1.4178 0.02048 1350.5 2557.8 1363.3 1378.9 2742.1 3.2858 5.6772 90.0
100.0 311.1 1.4524 0.01803 1393.0 2544.4 1407.6 1317.1 2724.7 3.3596 5.6141 100.0
110.0 318.2 1.4886 0.01599 1433.7 2529.8 1450.1 1255.5 2705.6 3.4295 5.5527 110.0
120.0 324.8 1.5267 0.01426 1473.0 2513.7 1491.3 1193.6 2684.9 3.4962 5.4924 120.0
130.0 330.9 1.5671 0.01278 1511.0 2496.1 1531.5 1130.7 2662.2 3.5606 5.4323 130.0
140.0 336.8 1.6107 0.01149 1548.6 2476.8 1571.1 1066.5 2637.6 3.6232 5.3717 140.0
150.0 342.2 1.6581 0.01034 1585.6 2455.5 1610.5 1000.0 2610.5 3.6848 5.3098 150.0
160.0 347.4 1.7107 0.009306 1622.7 2431.7 1650.1 930.6 2580.6 3.7481 5.2455 160.0
170.0 352.4 1.7702 0.008364 1660.2 2405.0 1690.3 856.9 2547.2 3.8079 5.1777 170.0
180.0 357.1 1.8397 0.007489 1698.9 2374.3 1732.0 777.1 2509.1 3.8715 5.1044 180.0
190.0 361.5 1.9243 0.006657 1739.9 2338.1 1776.5 688.0 2464.5 3.9388 5.0228 190.0
200.0 365.8 2.036 0.005834 1785.6 2293.0 1826.3 583.4 2409.7 4.0139 4.9269 200.0
220.9 374.1 3.155 0.003155 2029.6 2029.6 2099.3 0 2099.3 4.4298 4.4298 220.9
1082 Thermal Engineering

Table A.14 Superheated Water Vapor

T v u h s v u h s
°C m3/kg kJ/kg kJ/kg kJ/kg ◊ K m3/kg kJ/kg kJ/kg kJ/kg ◊ K
p = 0.06 bar = 0.006 MPa p = 0.35 bar = 0.035 MPa
(Tsat = 36.16°C) (Tsat = 72.69°C)
Sat. 23.739 2425.0 2567.4 8.3304 4.526 2473.0 2631.4 7.7158
80 27.132 2487.3 2650.1 8.5804 4.625 2483.7 2645.6 7.7564
120 30.219 2544.7 2726.0 8.7840 5.163 2542.4 2723.1 7.9644
160 33.302 2602.7 2802.5 8.9693 5.696 2601.2 2800.6 8.1519
200 36.383 2661.4 2879.7 9.1398 6.228 2660.4 2878.4 8.3237
240 39.462 2721.0 2957.8 9.2982 6.758 2720.3 2956.8 8.4828
280 42.540 2781.5 3036.8 9.4464 7.287 2780.9 3036.0 8.6314
320 45.618 2843.0 3116.7 9.5859 7.815 2842.5 3116.1 8.7712
360 48.696 2905.5 3197.7 9.7180 8.344 2905.1 3197.1 8.9034
400 51.774 2969.0 3279.6 9.8435 8.872 2968.6 3279.2 9.0291
440 54.851 3033.5 3362.6 9.9633 9.400 3033.2 3362.2 9.1490
500 59.467 3132.3 3489.1 10.1336 10.192 3132.1 3488.8 9.3194

p = 0.70 bar = 0.07 MPa p = 1.0 bar = 0.10 MPa


(Tsat = 89.95°C) (Tsat = 99.63°C)
Sat. 2.365 2494.5 2660.0 7.4797 1.694 2506.1 2675.5 7.3594
100 2.434 2509.7 2680.0 7.5341 1.696 2506.7 2676.2 7.3614
120 2.571 2539.7 2719.6 7.6375 1.793 2537.3 2716.6 7.4668
160 2.841 2599.4 2798.2 7.8279 1.984 2597.8 2796.2 7.6597
200 3.108 2659.1 2876.7 8.0012 2.172 2658.1 2875.3 7.8343
240 3.374 2719.3 2955.5 8.1611 2.359 2718.5 2954.5 7.9949
280 3.640 2780.2 3035.0 8.3162 2.546 2779.6 3034.2 8.1445
320 3.905 2842.0 3115.3 8.4504 2.732 2841.5 3114.6 8.2849
360 4.170 2904.6 3196.5 8.5828 2.917 2904.2 3195.9 8.4175
400 4.434 2968.2 3278.6 8.7086 3.103 2967.9 3278.2 8.5435
440 4.698 3032.9 3361.8 8.8286 3.288 3032.6 3361.4 8.6636
500 5.095 3131.8 3488.5 8.9991 3.565 3131.6 3488.1 8.8342

p = 1.5 bar = 0.15 MPa p = 3.0 bar = 0.30 MPa


(Tsat = 111.37°C) (Tsat = 133.55°C)
Sat. 1.159 2519.7 2693.6 7.2233 0.606 2543.6 2725.3 6.9919
120 1.188 2533.3 2711.4 7.2693
160 1.317 2595.2 2792.8 7.4665 0.651 2587.1 2782.3 7.1276
200 1.444 2656.2 2872.9 7.6433 0.716 2650.7 2865.5 7.3115
240 1.570 2717.2 2952.7 7.8052 0.781 2713.1 2947.3 7.4774
280 1.695 2778.6 3032.8 7.9555 0.844 2775.4 3028.6 7.6299
320 1.819 2840.6 3113.5 8.0964 0.907 2838.1 3110.1 7.7722
360 1.943 2903.5 3195.0 8.2293 0.969 2901.4 3192.2 7.9061
400 2.067 2967.3 3277.4 8.3555 1.032 2965.6 3275.0 8.0330
440 2.191 3032.1 3360.7 8.4757 1.094 3030.6 3358.7 8.1538
500 2.376 3131.2 3487.6 8.6466 1.187 3130.0 3486.0 8.3251
600 2.685 3301.7 3704.3 8.9101 1.341 3300.8 3703.2 8.5892
Appendix A 1083

Table A.14

T v u h s v u h s
3 3
°C m /kg kJ/kg kJ/kg kJ/kg ◊ K m /kg kJ/kg kJ/kg kJ/kg ◊ K
p = 5.0 bar = 0.50 MPa p = 7.0 bar = 0.70 MPa
(Tsat = 151.86°C) (Tsat = 164.97°C)
Sat. 0.3749 2561.2 2748.7 6.8213 0.2729 2572.5 2763.5 6.7080
180 0.4045 2609.7 2812.0 6.9656 0.2847 2599.8 2799.1 6.7880
200 0.4249 2642.9 2855.4 7.0592 0.2999 2634.8 2844.8 6.8865
240 0.4646 2707.6 2939.9 7.2307 0.3292 2701.8 2932.2 7.0641
280 0.5034 2771.2 3022.9 7.3865 0.3574 2766.9 3017.1 7.2233
320 0.5416 2834.7 3105.6 7.5308 0.3852 2831.3 3100.9 7.3697
360 0.5796 2898.7 3188.4 7.6660 0.4126 2895.8 3184.7 7.5063
400 0.6173 2963.2 3271.9 7.7938 0.4397 2960.9 3268.7 7.6350
440 0.6548 3028.6 3356.0 7.9152 0.4667 3026.6 3353.3 7.7571
500 0.7109 3128.4 3483.9 8.0873 0.5070 3126.8 3481.7 7.9299
600 0.8041 3299.6 3701.7 8.3522 0.5738 3298.5 3700.2 8.1956
700 0.8969 3477.5 3925.9 8.5952 0.6403 3476.6 3924.8 8.4391

p = 10.0 bar = 1.0 MPa p = 15.0 bar = 1.5 MPa


(Tsat = 179.91°C) (Tsat = 198.32°C)
Sat. 0.1944 2583.6 2778.1 6.5865 0.1318 2594.5 2792.2 6.4448
200 0.2060 2621.9 2827.9 6.6940 0.1325 2598.1 2796.8 6.4546
240 0.2275 2692.9 2920.4 6.8817 0.1483 2676.9 2899.3 6.6628
280 0.2480 2760.2 3008.2 7.0465 0.1627 2748.6 2992.7 6.8381
320 0.2678 2826.1 3093.9 7.1962 0.1765 2817.1 3081.9 6.9938
360 0.2873 2891.6 3178.9 7.3349 0.1899 2884.4 3169.2 7.1363
400 0.3066 2957.3 3263.9 7.4651 0.2030 2951.3 3255.8 7.2690
440 0.3257 3023.6 3349.3 7.5883 0.2160 3018.5 3342.5 7.3940
500 0.3541 3124.4 3478.5 7.7622 0.2352 3120.3 3473.1 7.5698
540 0.3729 3192.6 3565.6 7.8720 0.2478 3189.1 3560.9 7.6805
600 0.4011 3296.8 3697.9 8.0290 0.2668 3293.9 3694.0 7.8385
640 0.4198 3367.4 3787.2 8.1290 0.2793 3364.8 3783.8 7.9391

p = 20.0 bar = 2.0 MPa p = 30.0 bar = 3.0 MPa


(Tsat = 212.42°C) (Tsat = 233.90°C)
Sat. 0.0996 2600.3 2799.5 6.3409 0.0667 2604.1 2804.2 6.1869
240 0.1085 2659.6 2876.5 6.4952 0.0682 2619.7 2824.3 6.2265
280 0.1200 2736.4 2976.4 6.6828 0.0771 2709.9 2941.3 6.4462
320 0.1308 2807.9 3069.3

You might also like